You are on page 1of 1299

Item: 1 of 100 - ,• Mark -<J [:::> "'I ~ · ~

QIO: 1845 ~ P~v•ous N@xt Labl lues No tes Calculato r

. 1
A 6-year-old boy is brought to the pediatrician by his mother, who is very concerned about his health . Physical examinat ion shows an
•2
extensive erythematous reticula r skin rash on the face, t runk, and extremit ies, as seen in the image. He also exhibits swelling around the
·3 wrists that causes him pain on movement at t he j oint. He is in no acute distress, but his mother is very anxious.
• 4

·5
·6
·7
·8
•9
• 10

· 11
• 12
• 13
• 14 Which of the following is the best next st ep in t reatment for this illness?
• 15 :
. 16 A . No treatment necessary; it is a self-limiting disease
. 17 B. Prescribe a corticosteroid cream
• 18
C. Prescribe an oral cort icosteroid
• 19

• 20
D . Prescribe the appropriate antibiotic cream
. 21
. 22
. 23
• 24
• 25
• 26
. 27

8
Lode.
s
S uspe-nd
8
End Bloc:k
Item: 1 of 100 ~ ,• Mark <::J [:::> ""I ~· ~'j
QID: 1845 J.. Previous Next LAbfaiUI~S Notes Calculator

1
•2 The correct answer is A. 72°/o chose this .
•3 This description is classic for f ifth disease, which is caused by parvovirus Bl9. It is a pediat ric illness common in children 3- 12 years old. The
•4 rash is called eryt hema infectiosum and develops after feve r has resolved as a bri ght , blanchable erythema on t he cheeks ("slapped cheeks")
with perioral pallor. A more diffuse rash appears on the t run k and extremit ies and may wax and wane with temperature changes over 3
. 5
weeks. However, t he disease itself is self- limit ing, req uiri ng no t reatment. I n adults, the disease more f requently causes joint pain. I t can be
.6 dangerous for patients with sickle cell anemia because the virus can infect erythroid precursors and inhibit bone marrow production of RBCs,
• 7 result ing in profound anemia.
Fifth disease Parvovirus 619 Sickle-cell disease Erythema Anemia Blanch (medical) Red blood cell Pallor Parvovirus Virus Bone marrow Fever Rash Bone Mouth Arthralgia
•8
•9 B is not correct. 9% chose this .
• 10 The rash seen in f ifth disease will dissipate on its own after a few weeks and will not respond to cort icosteroid cream .
Corticosteroid Fifth disease Rash
· 11
• 12 C is not correct. 15% chos e this .
• 13 Although t he rash of eryt hema infectiosum is a cent rally dist ribut ed maculopapular erupt ion (like the rashes that accom pany ri cket tsial
illnesses, drug -induced erupt ions, and St ill's diseases), it is self-lim it ing, requiring no t reatment. There is a role for t he use of oral
• 14
corticosteroids in the treat ment of Still's disease, which is a manifest ation of j uvenile rheumat oid arthrit is t hat presents with high spiking
• 15 fever, rash wit hout it ching, and art hrit is. This patient has no fever, making St ill's disease unlikely.
Fifth disease Juvenile idiopathic arthritis Rheumatoid arthritis Erythema Corticosteroid Arthritis Itch Rash Maculopapular rash Rickettsia
. 16
. 17 D is not correct. 4°/o chose this .
• 18 Erythema infectiosum is caused by infect ion wit h parvovirus Bl 9. Vira l illnesses do not respon d to antibiotics .
Fifth disease Parvovirus 819 Erythema Parvovirus Antibiotics Virus Infection
• 19
• 20
. 21 Bottom Line:
. 22 Fifth disease (eryt hema infectiosum) is a self-lim ited condition caused by parvovirus Bl 9.
• 23 Fifth disease Parvovirus 619 Erythema Parvovirus

• 24
• 25
• 26 ljl:fil·11•i for year:( 20 1 7 .,.
FIRST AID FA CTS
. 27

8
Lock
s
Suspend
0
End Block
Item: 1 of 100 ~ ,• Mark <::J [:::> ""I ~· ~'j
QID: 1845 J.. Previous Next LAb faiUI~S Notes Calculator

1 • •
FA17 p 160.1
•2
DNA viruses
•3
VIRAL FAMILY ENVELOPE ONA STRUCTURE MEDICAL IMPORTANCE
•4
. 5
Herpesviruses Yes OS and linear See Herpesviruses entry
. 6 Poxvirus Yes OS and linear Smallpox eradicated world wide by use of the live-
• 7 (largest 0 1 A virus) attenuated vaccine
•8 Cowpox ("mi lkmaid blisters")
•9 Molluscum contagiosum -Aesh-colored papule with
• 10 central umbilication
· 11
Hepadnavirus Yes Partially OS and circular HBV:
• 12
Acule or chron ic hepatitis
• 13
lot a retrovirus but has reverse transcriptase
• 14
• 15 Adenovirus 0 OS and linear Febrile pharyngitis rJ-sore throat
. 16 Acute hemorrhagic cystitis
. 17 Pneumonia
• 18 Conjunctivitis-"pink eye"
• 19
• 20
. 21
. 22
• 23
• 24
Papillomavirus No OS and circular HPV-warts (serotypes 1, 2, 6, 11), Cl , cervical
• 25
cancer (most commonly 16, 18)
• 26
. 27 • Polyomavirus No OS and circular JC virus-progressive multifoca lleukocncephalopathy •

8
Lock
s
Suspend
0
End Block
Item: 1 of 100 ~ ,• Mark <::J [:::> ""I ~· ~'j
QID: 1845 J.. Previous Next LAbfaiUI~S Notes Calculator

1 • •
FA17 p 179.1
•2
Red rashes of childhood
•3
AGENT ASSOCIATED SYNDROME/DISEASE CLINICAL PRESENTATION
•4
. 5
Coxsackievirus type A Hand-foot-mouth disease Oval-shaped vesicles on palms and soles rJ;
. 6 vesicles and ulcers in oral mucosa
• 7 Human herpesvirus 6 Roseola (exanthem subitum) Asymptomatic rose-colored macules appear
•8 on body after se,·eral days of high fever; can
•9 present with febrile seizures; usually affects
• 10 infan ts
· 11
Measles virus Measles (rubeola) ConAuent rash beginning at head and
• 12
• 13
moving down; preceded by cough, coryza,
• 14
conjuncti,·itis, and blue-white (Koplik) spots
• 15
on buccal mucosa
. 16 Parvovirus B19 Erythema infectiosum (fifth disease) "Slapped cheek" rash on face llJ (can cause
. 17 hydrops feta lis in pregnant women)
• 18
Rubella virus Rubella (German measles) Pink macules and papules begin at head
• 19
and move down, remain discrete ..... fine
• 20
desquamating truncal rash; postauricular
. 21
. 22
lymphadenopathy
• 23 Streptococcus pyogenes Scarlet fever Erythematous, sandpaper-like rash with feve r
• 24 and sore throat
• 25
Varicella-Zoster virus Chickenpox Vesicular rash begins on trunk; spreads to face
• 26
. 27 •
m and extremities with lesions of different •

8
Lock
s
Suspend
0
End Block
Item: 1 of 100 - ,• Mark --<] [::> ""'I ~· 1!';:'1
QIO: 1845 ~ Prev1o u s Next Labf a lues Notes Calculator

1
Measles virus Measles (rubeola) ConAuent rash beginning at head and
•2
moving down; preceded by cough, coryza,
·3
•4
conjuncti,·itis, and blue-white (Koplik) spots
·5
on buccal mucosa
·6 Parvovirus 819 Erythema infectiosum (fifth disease) "Slapped cheek" rash on face I] (can cause
·7 hydrops fetal is in pregnant women)
·8
Rubella virus Rubella (German measles) Pink macules and papules begin at head
·9
and mo,·e down, remain discrete - fine
· 10
desquamating truncal rash; postauricular
· 11
• 12
lymphadenopathy
• 13 Streptococcus pyogenes Scarlet fe,·er Erythematous, sandpaper-like rash with fe\'er
• 14 and sore throat
. 15
Varicella-Zoster virus Chickenpox Vesicular rash begins on trunk; spreads to face
. 16
[!] and extremities with lesions of different
. 17
st·ages
• 18
• 19
• 20
. 21
. 22
• 23
• 24
• 25
• 26
. 27 •
8
L.odt
s
Su~pl'nd
~
End Block
Item: 2 of 100 - ,• Mark -<J [:::> "'I ~ · ~
QIO: 349 6 ~ P~v•ous N@xt Labl lues No tes Calcula t o r

A A
1
A 60-year-old woman presents with shaking chills, high fevers, chest pain, and shortness of breath. X-ray of the chest reveals consolidation ~~AI
•2
in her right upper lobe. A quellung t est performed on a sputum specimen results in a positive reaction with capsular swelling. The patient
·3 reports being hospitalized as a teenager after taking penicillin and was told not to take it again. Her physician prescribes an appropriate
• 4 treatment.
·5
·6
Which of the following describes the mechanism of action of the treatment most likely given t o t his patient?
·7 :
A. Inhibit formation of initiation complex and cause misreading of messenger RNA
•9 B. Inhibit protein synthesis by binding t o t he 235 ribosomal RNA of the 505 ribosomal subunit
• 10
C. Inhibit tetrahydrofolic acid production
· 11
D. Inhibits DNA-dependent RNA polymerase
• 12
• 13 E. Inhibits synthesis of mycolic acid
• 14

• 15
. 16
. 17
• 18
• 19

• 20
. 21
. 22
. 23
• 24
• 25
• 26
. 27

8
Lode.
s
Suspe-nd
8
End Bloc:k
Item: 2 of 100 - ,• Mark -<J [:::> "'I ~ · ~
QIO: 349 6 ~ P~v•ous N@xt Labl lues No tes Calcula t o r

A A
1 The correct an swer is B. 57% chose this.
2 This patient presents with classic symptoms of a lobar pneumonia, like that evidenced in this image.
·3 Streptococcus pneumoniae is the most likely organism given the posit ive quellung test for this organism.
• 4
The treatment of choice for document ed penicillin-sensitive pneumococcus is a 13-lactam. However, in this
instance the patient has a document ed penicillin allergy, so doxycycline or macrolides are the treatment of
·5 choice for the outpatient management of pneumonia. Macrolides act by binding to the 505 ribosomal
·6 subunit to inhibit protein synthesis.
Do 'fCYC ne ~ ert ~~,.oc,.~~,.u~ p11eumomoe Pen1ci 1n Lobar pneumonia Mac:rol1de Pneurnon1a Streptococcus r4'1._,,,,.i..,_ 5 Protetn :synthes1s Proten 50S
·7
Proten b•osynthes•s Allergy Organ1sm Pen•c•ll•n allergy Ribosome

•9
• 10
Image courtesy of James
· 11
Heilman, MD
• 12
• 13 A is not correct. 13% chose this .
• 14 Gentamicin and other aminoglycosides act by inhibiting formation of the initiation complex. Aminoglycosides are used for severe gram-
• 15
negative infections. They have some activity against gram-positive organisms, but are less potent and more toxic than other antibiotics .
Gentam1c1n Gram·negat1ve bactena Gram-pos•bve bacteria Aminoglycoside Antibiotics
. 16
. 17
C is not correct. 10°/o chose this .
• 18
Trimethoprim (TMP) inhibi ts dihyd rofo lat e reduct ase, and sulfamethoxazole (SMX) inhibits dihydropteroate synthetase. Toget her,
t rimethoprim-sulfamet hoxazole (TMP-SMX) works synergist ically to inhibit downstream production of t et rahyd rofo lic acid. The combination is
• 19 best used for urinary tract infect ions, for Shigella infection, and fo r Pneumocystis jirovecii pneumonia prophy laxis, but it has poor
• 20 Streptococcus pneumoniae activit y.
Tetrahydrofol•c acid Streptococcus pneumomae Dihydrofolate reductase Folic acid Trimethoprim Pneumocystis j1rovecu Tnmethoprun/sulfamethoxazole Oihydropteroate synthase Sh1gella Sulfamethoxazole
. 21
Pneumonia Urinary tract 1nfecbon 01hydrofohc acid Streptococcus 9(nergy Pneumocyst1s pneumonia Prevent1ve healthcare Infect1on
. 22
. 23 D is n ot correct. 8°/o chose this .
• 24 Rifampin acts by inhibiting RNA polymerase. It is most often used for latent tuberculosis infections and as part of t he four-drug regimen for
active tuberculosis .
• 25
Rtf~mp1c1n Tuberculo ., wtent tuberculos1s RNA polymerase RNA
• 26
. 27
E is not correct. 12°/o chose this .

8
Lode.
s
Suspe-nd
8
End Bloc:k
Item: 2 of 100 - ,• Mark -<J [:::> "'I ~ · ~
QIO: 349 6 ~ P~v•ous N@xt Labl lu es No tes Calcula t o r

1
D is not correct. 8°/o chose this. •
2 Rifampin acts by inhibiti ng RNA polymerase. It is most often used for latent tuberculosis infections and as part of t he four-drug regimen for
·3 active tuberculosis.
Rtf'-mptctn Tubtrculos•• Latent tuberculosis RNA polymerase RNA
• 4

·5 E is not correct. 12°/o chose this .


·6 Isoniazid (INH) is used to treat Mycobacterium tuberculosis infections. It acts by inhibiting mycobacterial InhA, thus preventing the formation
·7
of mycolic acid, a component of the mycobacterial cell wall.
lsonrriZtd M.ro ,. d M fCOboctenum tuberculosts Tuber-culosis Cefl wal Mycobacterium

•9
• 10 Bottom line:
· 11 The most common cause of lobar pneumonia, and t he only one to result in capsular swelling or a positive quellung test, is Streptococcus
• 12
pneumoniae .
Streptococcu! pneuJ'I"ttntae lobar pneumonia Pneumonia Streptococcus
• 13
• 14

• 15
l@l;fii·11•J for year: 2017 •
FI RST AID FACTS
. 16
. 17
• 18 FA17 p 183.1

• 19 Antimicrobial therapy
• 20
. 21
GYRASE
. 22 FOLIC ACID SYNTHESIS
AND REDUCTION

~am_PI_"-,-----"
. 23 (DNAmethylalton) Fluoroquinolones
Metronidazole
• 24 Gprofloxacin
Levofloxaon. etc
• 25 Sulfonamides
Sullamethoxazole BACTERIAL CELL Ouinolone
• 26 Nalidixic acid
Sulfisoxazole PABA
. 27 c•.,, ~,. ....,.~ \

8
Lode.
s
Suspe-nd
8
End Bloc:k
Item: 2 of 100
QID· 349 6
- ,•
.1
M ric
a
-<J [:::>
A
"'I ~·
'
~
· P~v1ous N@xt LabValu es No tes Calcula t o r

1
FA1 7 p183.1

2
·3
Antimicrobial therapy
• 4

·5 GYRASE
FOLIC ACID SYNTHESIS
·6 AND REDUCTION
·7 (DNAmtthyl,tionl Metronidazole Auoroquinolones
Cipfofloxacin
·8 Levofloxaon. etc
Sulfonamides
•9 Sulfamethoxazole BACTERIAl CEll Ouinolone
• 10 Sulfisoxazole PABA Nalidixic acid
Sulfad.azme
· 11
• 12
Tnmethopnm
• 13
• 14

• 15
pc.,,1 ... ~ra
PROTEIN SYNTHESIS
. 16
. 17 50S SUBUNIT
mRNA
• 18 Chloramphenicol
• 19 Ct1ndamyc1n
CELL WALL SYNTHESIS
• 20 Linezolid
THF
. 21 r TIDOG CAN SY~THESIS Macro! ides
Azithromycm
. 22 Celt Clarithromycin
Gtycopeptides membrane
. 23 Vancomyc1n Erythromycin
• 24
Bacltrac1n Celt Wall Streptogramins
Ouinupristln
• 25 Dalfoprrslln
-- - oo- -AN----- ·
• 26
_l_li_n-s- -An
IPenicillinase·se_n_sitl__v_e_oe_n_io ____Ca_rbaoene~
- -o-m_onai
- tiLoseud 30SSUBUNIT
. 27

8
Lode.
s
S uspe-nd
8
End Bloc:k
Item: 2 of 100 - ,• Mark -<J [:::> "'I ~ · ~
QIO: 349 6 ~ P~v•ous N@xt Labl lues No tes Calcula t o r

A A
1 FA17 p 189.2
2 Macrolides Azithromycin, clarithromycin, erythromycin.
·3
• 4
MECHANISM Inhibit protein synthesis by blocking translocation ("m ac ro~lid es"); bind to the 23S rR A of the
·5
50S ribosomal subunit. Bacteriostatic.
·6 CliNICAl USE Atypical pneumonias (.\ Iycoplasma, Chlamydia, Legionella), STls (Chlamydia), gram (±) cocci
·7 (streptococcal infections in patients allergic to penicillin), and B pertussis.
·8
ADVERSE EFFECTS :\IACRO: Gastrointestinal Motility issues, \ rrhr thmia caused br prolonged QT inten·al, acute
•9
C holestatic hepatitis, Rash, eOsinophilia. Increases serum concentration of theophylline, oral
• 10
anticoagulants. Clarithromycin and erythromycin inhibit crtochrome P-450.
· 11
• 12 MECHANISM OF RESISTANCE Methylation of 23S ri 1\ -binding site prevents binding of drug.
• 13
• 14

• 15 FA17 p 175.2
. 16 Common causes of pneumonia
. 17 NEONATES(< 4 WK) CHILDREN (4 WK-18 YR) ADULTS (18- 40 YR) ADULTS (40-65 YR) ElDERlY
• 18
C roup 13 streptococci Viruses (RSV) Mycoplasma S pneu111oniae S pneum.oniae
• 19
E coli Mycoplasma C pneumoniae I I in(luenzae l n Auenza virus
• 20
C trachomatis S pneumoniae Anaerobes Anaerobes
. 21
(infants-3 yr) Viruses (eg, in Auenza) Viruses H influenzae
. 22
. 23
C pneumoniae Mycoplasma Cram 8 rods
• 24
(school-aged
• 25
children)
• 26
S pneumoniae
. 27 Runts \la\' Cough
8
Lode.
s
Suspe-nd
8
End Bloc:k
Item: 2 of 100 - ,• Mark -<J [:::> "'I ~ · ~
QIO: 349 6 ~ P~v•ous N@xt Labl lues No tes Calcula t o r

A
1 A

Croup 13 slrcplococci Viruses (RSV) Mycoplasma S pnewnoniae S pneumoniae


2
E coli i\.Jycoplasma C pneumoniae II inf1uenzae In Auenza \'irus
· 3
C lrachomatis S pneumoniae Anaerobes Anaerobes
•4
(infants-3 yr) Viruses (eg, inAuenza) Viruses H in/1ue11zae
· 5
· 6
C pneumoniae Mycoplasma C ram G rods
· 7
(school-aged
·8 children)
•9 S pneumoniae
• 10 Runts \lay Cough
· 11 Chunky Sputum
• 12 Special g roups
• 13
Alcoholic Klebsiella, anaerobes usua lly due to aspiration (eg. Peptostreptococcus, Fusobacterium, Prevotella,
• 14
Bacteroides)
• 15
. 16 IV drug users S pnewnoniae, S aureus
. 17 Aspiration Anaerobes
• 18
Atypical Mycoplasma, Legionella, Chlamydia
• 19
• 20 Cystic fibrosis Pseudomonas, S aureus, S pnewnoniae, Burkholderia cepacia
. 21 lmmunocompromised S aureus, enteric gram 8 rods, fu ngi, viruses, P jirovecii (with II IV)
. 22
. 23
Nosocomial (hospital S aureus, Pseudomonas, other enteric gram 8 rods
• 24
acquired)
• 25 Postviral S pneumoniae, S aureus, l-1 inf1uenzae
• 26
. 27 •
8
Lode.
s
Suspe-nd
8
End Bloc:k
Item: 3 of 100 - ,• Mark -<J [:::> "'I ~ · ~
QIO: 4 9 18 ~ P~v•ous N@xt Labl lues No tes Calcula t o r

1
A 40-year-old woman has been hospitalized fo r t he t reatment of aspiration pneumonia with clindamycin . On day 4 of hospitalizat ion, her
2
breathing has improved and her X- ray of the chest is normal, but she develops crampy abdominal pain and mu lt iple loose stools in a 24-
·3 hour period. Colonoscopy reveals a patchwork patt ern of mucous and mucosal erosions.
• 4

·5 The agent responsib le for t he patient's current illness produces virulence factors that induce which cellular event in t he intestinal epithelial cell?
·6 :
·7 A. Apoptosis
B. Differentiation
•9
C. Exocytosis
• 10

· 11 D. Proliferation
• 12 E. Swelling
• 13
• 14

• 15
. 16
. 17
• 18
• 19

• 20
. 21
. 22
. 23
• 24
• 25
• 26
. 27

8
Lode.
s
Suspe-nd
8
End Bloc:k
Item: 3 of 100 ~ ,• Mark <::J [:::> ""I ~· ~'j
QID: 4918 J.. Previous Next LAbfaiUI~S Notes Calculator

1
2 The correct a nswer is A. 5 4 °/o chose t h is.
3
The pat ient has pseudomembranous colit is, an infection caused by Clostridium difficile and associated wit h antibiotic use (particularl y t he use
of clindamycin). C. difficile causes a characteristic erosion of t he colonic mucosa . It does so by producing virulence fact ors (toxin A, which
•4 binds to t he colonic brush border, and t oxin B, which causes depolymeri zation of t he act in cyt oskeleton) that induce apopt osis in colonic
. 5 epit helial cells .
Clostridium difficile colitis Clinda,.,cin Clostridium difficile (bacteria) Actin Apoptosis Cytoskeleton Antibiotics Brush border Epithelium Virulence Colitis Toxin Virulence factor Clostridium Large intestine
.6
Mucous membrane Infection Colon (anatomy)
• 7
•8 B is not correct. 6% chose t his .
•9 Although t he colonic crypts do cont ain int estinal st em cells that may different iate int o many different cell types, Clostridium difficile does not
• 10
induce proliferat ion of epit helial cells .
Clostridium difficile colitis Clostridium difficile (bacteria) Epithelium Large intestine Intestinal gland Stem cell Clostridium Colon (anato,.,)
· 11
• 12 C is not co rrect. 13% chose this .
The toxin produced by Clostridium difficile does not induce exocytosis.
• 13
Clostridium difficile colitis Clostridium difficile (bacteria) Toxin Clostridium
• 14

• 15
D is not correct. 11% c hose this .
Proliferat ion of inflammatory cells may occur as a result of infect ion wit h Clostridium difficile, but t he bact eri al t oxins do not induce t he
. 16
epit helial cells t o proliferate .
. 17 Clostridium difficile colitis Clostridium difficile (bacteria) Epithelium Clostridium Infection Inflammation

• 18
E is not correct. 1 6°/o chose this .
• 19
Cellular swelling is t he primary indicator of reve rsible cell damage. It is a result of failure of t he sodium -potassium pum p, which is not the
• 20 mechanism by which Clostridium difficile toxin causes illness .
Clostridium difficile colitis Clostridium difficile (bacteria) Toxin Clostridium
. 21
. 22
• 23 Bottom Line :
• 24
Clostridium difficile causes a charact erist ic erosion of epit helial cells in t he colonic mucosa by producing factors (t oxins A and B) t hat induce
• 25 apoptosis .
Apoptosis Clostridium difficile colitis Clostridium difficile (bacteria) Epithelium Clostridium Mucous membrane Large intestine
• 26
. 27

8
Lock
s
Suspend
0
End Block
Item: 3 of 100 - ,• Mark --<] [::> ""'I ~· 1!';:'1
QIO: 4918 ~ Prev1o u s Next Labf a lues Note s Calculator

1 • •
FA17 p 134.2
2
3 Clostridia (with Gram EB, spore-forming, obligate anaerobic rods.
•4 exotoxins)
· 5 Ctetani Produces tetanospasmin, an exotoxin causing Tetanus is tetanic paralysis.
·6 tetanus. Tetanus toxin (and botulinulll toxin)
· 7 are proteases that clea,·e SJ ARE proteins for
·8
neurotransmitters. Blocks release of inhibitory
·9
neurotransmitters, GABA and glrcine, from
· 10
Renshaw cells in spinal cord.
· 11
Causes sp astic paralysis, trismus (lockjaw), risus
• 12
sardonicus (raised eyebrows and open gri n),
• 13
• 14
opisthotonos (spasms of spinal extensors).
. 15
Pre' enl with tetanus vaccine. Treat" ith
. 16
antitoxin +1- vaccine booster, diazepam (for
. 17 muscle spasms), and wound debridement.
• 18 Cbotulinum Produces a heat-labile toxin that inhibits Symptoms of botulism (the 4 D 's}: D iplopia,
• 19 ACh release at the neuromuscular june!ion, O )rsarthria, D ysphagia, D yspnea .
• 20 causi ng botulism. In adults, disease is caused Botulinum is from bad bottles of food , ju ice, and
. 21 by ingestion of preformed toxin. In babies, honey (causes a descending flaccid paralysis).
. 22
ingestion of spores (eg, in honey) leads to Local botox injections used to treat focal
• 23
disease (flopp~ baby synd rome). Treat with dyston ia, achalasia, and muscle spasms. Also
• 24
antitoxin . used for cosmetic reduction offacial wrinkles.
• 25

-
• 26 C perfringens Produces a toxin (lecithinase, a pho)pholipase) Perfringens perforates a gangrenous leg.
. 27 - that can cause nwonecrosis (eas eanf!renc f.l) •
8
L.odt
s
Su~pl'nd
~
End Block
Item: 3 of 100 - ,• Mark --<] [::> ""'I ~· 1!';:'1
QIO: 4918 ~ Prev1o u s Next Labf a lues Note s Calculator

1
Cbotulinum Produces a heat-labile toxin that inhibits Symptoms of botulism (the 4 D's): Diplopia,
2
ACh release at the neuromuscular junction, Dysarthria, Dysphagia, Dyspnea.
3
causing botulism. In adults, disease is caused Botulinum is from bad bottles of food, juice, and
•4

·5
by ingestion of preformed toxin. In babies, honey (causes a descending Aaccid paralysis).
·6
ingestion of spores (eg, in honey) leads to I .ocal botox injections used to treat focal
·7 disease (Aopp~ baby syndrome). Treat" ith dystonia, achalasia, and muscle spasms. Also
·8 antitoxin. used for cosmetic reduction of fa cial wrinkles.
·9 Cperfringens Produces a toxin (lecithinase, a phospholipase) Perfringens perforates a gangrenous leg.
· 10 that can cause myonecrosis (gas gangrene )
· 11 and hemolvsis.

• 12
Spores can sun·i,·e in undercookcd food;
• 13
when ingested, bacteria release heat-labile
• 14
enterotoxin ..... food poisoning.
. 15
. 16
. 17
• 18
• 19
C difficile Produces 2 toxins. Toxin A, an enterotoxin, Oi{{icile causes diarrhea. Treatment:
• 20
binds to brush border of gut and alters Ouid metronidazole or oral vancomycin. For
. 21
secretion. Toxin B, a cytotoxin, disrupts recurrent cases, consider repeating prior
. 22 cytoskeleton via actin depolymerizalion. Both regimen, fidaxomicin, or fecal microbiota
• 23 toxins lead to diarrhea ..... pseudomembranous transplant.
• 24 zo
colitis (l). Often to antibiotic usc, especially
• 25 clindamycin or ampicillin; associated with PPI
• 26 use. Diagnosed by detecting one or both toxins
. 27 ;., .... tnnl h.,- .,.,t-; non rl.ot-.o,..t-; n n nr P r Q •
8
L.odt
s
Su~pl'nd
~
End Block
Item: 4 of 100 - ,• Mark -<J [:::> "'I ~ · ~
QIO: 20 19 ~ P~v•ous N@xt Labl lues No tes Calcula t o r

1
After 11 days in the intensive care unit following an automobile accident, an 18-year-old woman develops increased urinary urgency,
2
burning du ring urination, and lower abdominal pain. When her uri ne is cultured, a red pigment is observed in the Petri dish.
3
• 4 What is the most likely organism causing these symptoms?
·5
:
·6 A. Escherichia coli
·7
B. Klebsiella pneumoniae

•9 C. Proteus mirabilis
• 10 0 . Serratia marcescens
· 11
E. Staphylococcus saprophyticus
• 12
• 13
• 14

• 15
. 16
. 17
• 18
• 19
• 20
. 21
. 22
. 23
• 24
• 25
• 26
. 27

8
Lode.
s
Suspe-nd
8
End Bloc:k
Item: 4 of 100 ~ ,• Mark <::J [:::> ""I ~· ~'j
QID: 2019 J.. Previous Next LAb faiUI~S Notes Calculator

1 The correct a nswer is 0. 56°/o chose t his.


2 Serratia is a common cause of nosocomial uri nary t ract infections, along wit h E. coli, Proteus, Klebsiella and Pseudomonas. Some strains
3 produce a red pigment. Nosocomial infections generally are more resistant t han communit y acquired infect ions. Serratia is a gram-negat ive,
facultatively anaerobic bacillus.
4
Escherichia coli Gram-negative bacteria Hospital-acquired infection Pseudomonas Facultative anaerobic organism Klebsiella Serratia Bacillus Urinary tract infection Anaerobic organism Proteus (bacterium)
. 5
Bacillus (shape) Urinary system Proteus Pigment
.6
• 7
A is not correct . 14% chos e this.
Escherichia coli is t he nu mber-one cause of uri nary t ract infections in all ambulat ory patients, accounting for about 80% of pat ients wit hout
•8
cat heters, urologic abnormalities, or calculi. E. coli is not associated wit h red pigmentat ion .
•9 Escherichia coli Urinary tract infection Urinary system Catheter Urology

• 10
B is not correct. 15% chose this .
· 11 Klebsiella species are common et iologic agents of urinary t ract infection t hat predispose to st one format ion through t he production of
• 12 extracellu lar slime and polysacchari de, and are frequently found in pat ients with calculi. Urease-posit ive bact eria such as Klebsiella and
• 13 Proteus species can form ammonium magnesium phosphate (st ruvite) st ones that can be a nidus for fut ure recurrent infections. Klebsiella is
not associat ed with red pigmentation .
• 14
Urinary tract infection Struvite Polysaccharide Klebsiella Urinary system Magnesium Bacteria Phosphate Ammonium Infection
• 15
C is not correct. 8 °/o chose this .
. 16
Proteus mirabilis predisposes to st one format ion ( because of its urease product ion) and is found more frequent ly in patients with calculi.
. 17
Proteus is a gram-negat ive, facult at ively anaerobic, urease -posit ive bacillus. Proteus is not associat ed wit h red pigmentation.
• 18 Urease Proteus mirabilis Gram-negative bacteria Proteus (bacterium) Facultative anaerobic organism Bacillus Anaerobic organism Proteus Bacillus (shape)

• 19
E is not correct. 7°/o chose this .
• 20 Staphylococcus saprophyticus is t he second most common cause of urinary t ract infection in young ambulat ory women ( 10%- 15%). Lab
. 21 identif ication of S. saprophyticus is import ant, because although t hese organisms are coagulase-negat ive, t hey are uri nary pat hogens. They
. 22
are charact erized by resistance to novobiocin, t he absence of hemolysin and coagulase, and a lack of pigment (colorless colonies will be seen) .
Urinary tract infection Novobiocin Coagulase Staphylococcus saprophyticus Hemolysin Staphylococcus Urinary system Infection Pathogen
• 23
• 24
• 25 Bottom Line :
• 26 One of the most common causes of nosocomial uri nary t ract infect ions is Serratia marcescens; some strains produce a red pigment t hat can
. 27
be det ected when urine is cult ured .

8
Lock
s
Suspend
0
End Block
Item: 4 of 100 - ,• Mark -<J [:::> "'I ~ · ~
QIO: 20 19 ~ P~v•ous N@xt Labl lues No tes Calcula t o r

A A
1
FA17 p 177.1
2
3 Urinary tract Cystitis presents with dysmia, frequency, urgency, suprapubic pain, and VIBCs (but not WBC
4 infections casts) in urine. Primarily caused by ascension of microbes from urethra to bladder. Males-
·5 infants with congenital defects, vesicoureteral reAux. Elderly- enlarged prostate. Ascension to
·6 kidney results in pyelonephrit is, " hich presents " ith ber, chills, Aank pain, costmertebral angle
·7 tenderness, hematuria, and WBC casts.
Ten times more common in \\ Omen (shorter urethras colonized by fecal Aora). Other predisposing
•9 factors: obstruction, kidney surger), catheterization, GU malformation, diabetes, pregnancy.
• 10
· 11
FA17 p 177.2
• 12
• 13
UTI bugs
• 14 SPECIES FEATURES COMMENTS
• 15 Escherichia coli Leading cause of UTI. Colonies show green Diagnostic markers:
. 16 metallic sheen on EMB agar. EF> Leukocyte esterase = evidence of WBC
. 17
Staphylococcus 2nd leading cause of UTI in sexually aet·ive activity.
• 18
saprophyticus women. EF> itrite test = reduction of minary nitrates
• 19 by bacterial species (eg, E coli).
• 20 Klebsiella pneumoniae 3rd leading cause of UTI. Large mucoid capsule
Ef> Urease test = urease-producing bugs (eg,
. 21 and viscous colonies .
S saprophyticus, Proteus, Klebsiella).
. 22 Serratia marcescens Some strains produce a red pigment; often
. 23 nosocomial and drug resistant.
• 24
Enterococcus Often nosocomial and drug resistant.
• 25
• 26 Proteus mirabilis tlotility causes "swarming" on agar; produces
. 27 urP:>~P · :>~~nl' i :> tPtl w ith d ru\'iiP dnn P~

8
Lode.
s
Suspe-nd
8
End Bloc:k
Item: S of 100 - ,• Mark -<J [:::> "'I ~ · ~
QIO: 2 185 ~ P~v•ous N@xt Labl lues No tes Calcula t o r

A A
1
A 20-year-old man present s to the physician with a nontender indurated mass over his mandible. He has had t his mass for 4 months after ~~AI
2
undergoing oral surgery and decided to come to the physician because the mass started t o ooze a thick yellow exudate. Yellow granules
3 are seen on microscopic examination of the discharge and an antibiotic is prescribed.
4
·5 Which of the following best describes the mechanism of action of the antibiotic most likely prescribed?
·6 :
·7 A. Binds ergosterol, forming pores in the membrane
·8 B. Block bacterial cell wall synthesis by inhibiting transpeptidase crosslinking
•9
C. Block bacterial nucleotide synthesis
• 10
· 11 0 . Block bacterial protein synthesis
' 12 E. Inhibits ergosterol synthesis
' 13
' 14
' 15
. 16
. 17
• 18

' 19
' 20
. 21
. 22
' 23
' 24
' 25
' 26
. 27

8
Lode.
s
Suspe-nd
8
End Bloc:k
Item: S of 100 ~ ,• Mark <::J [:::> ""I ~· ~'j
QID: 2185 J.. Previous Next LAb faiUI~S Notes Calculator

1 The correct answer is B. 66% chos e this.


2 The infect ion caused by Actinomyces israe/ii ty pically presents as a chron ic, slowly progress ing mass t hat eventually evolves into a dra ining
3 sinus tract. Characteristic sulfur granules are seen in t he t hick yellow exudat e. Penicillin G is t he f irst-line t reat ment .
Actinomyces israelii Penicillin Actinomyces Benzylpenicillin Exudate Infection Sulfur
4
5 A is not correct. 6°/o chose this.
.6 This describes the mechanism for amphotericin B, which is used t o t reat systemic mycoses. While Actinomyces fo rm long, branching filament s
t hat resem ble fung i, t hey are bacteria.
• 7
Amphotericin B Fungus Mycosis Actinomyces Bacteria
•8
•9
C is not correct. 9 °/o chose this .
Sulfonamides act by inhibit ing dihydropteroat e synt het ase, prevent ing nucleotide synt hesis. They are first -line medicat ions for Nocardia
. 10 infection but not fo r Actinomyces.
· 11 Nocardia Actino,.,ces Sulfonamide (medicine) Sulfonamide Dihydropteroate synthase
. 12
D is not correct. 14% chose this .
• 13
Man y antibiot ics act by blocking protein synt hesis including macrolides, aminoglycosides, and tetracyclines. However, none of these are first -
• 14 line treat ments fo r Actinomyces infect ion, which best explains this patient's presentat ion .
Aminoglycoside Protein Antibiotics Actinomyces Tetracycline antibiotics Macrolide Protein biosynthesis Protein synthesis
• 15
. 16 E is not correct. 5°/o chose this .
. 17 This describes the mechanism for azoles such as f lucona zole and ketoconazole, which are used t o t reat fungal infections. While Actinomyces
• 18 form long, branching filaments t hat resem ble fungi, t hey are bact eri a .
Ketoconazole Fluconazole Fungus Mycosis Azole Bacteria Actinomyces
• 19
. 20
. 21 Bottom Line:
. 22 A. israelii is a gram-posit ive rod t hat can cause oral and facial abscesses t hat re lease yellow sulfur granules in the pus. First -l ine t reatment is
• 23
penicillin G, which acts by blocking bact eri al cell wall synthesis .
Penicillin Benzylpenicillin Gram-positive bacteria Cell wall Abscess Pus Sulfur Actinomyces israelii Therapy
• 24

• 25
• 26
141;fil·1i•J for year:[2017 •
. 27 .,. FIRS T AID FACTS

8
Lock
s
Suspend
0
End Block
Item: S of 100 ~ ,• Mark <::J [:::> ""I ~· ~'j
QID: 2185 J.. Previous Next LAb faiUI~S Notes Calculator

1 • •
FA17 p 135.3
2
3
Nocardiavs Both are gram ® and form long, branching fila ments resembling fu ngi.
4
Actinomyces Nocardia Actinomyces
5
fi.1 Aerobe Anaerobe
. 6
• 7 Acid fast (weak) fi.1 Not acid fast [l]
•8 Found in soil ormal oral, reproductive, and C l Aora
•9
Causes pulmonary infections in Causes oral/facial abscesses that drain through
• 10
immunocompromised (can mimic T B but sinus tracts; often associated with dental caries/
· 11
• 12
with 8 PPO); cutaneous infections after extraction; form s yellow "sulfur granules"; ca n
• 13
trauma in immunocompetent; can spread to also cause PID with IUDs
• 14 c s
• 15 Treat with sulfonamides (TMP-SMX) Treat with penicillin
. 16
Treatment is a SNAP: Sulfonam ides-Nocardia; Actinomyces-Penicillin
. 17
• 18
• 19
• 20
. 21
. 22
FA17 p 182.2
• 23
• 24 Bug hints (if all else CHARACTERISTIC ORGANISM
• 25 fails) Asplcnic patient (due to surgical splenectomy Encapsulated microbes, especially SHiN
• 26 or autosplenectomy, eg, chronic sickle cell (S pneumoniae >> II influen:zae type B >
• ...J : _____ , _ : e: J: _\ •
. 27 1\. T · · · - · · : ••

8
Lock
s
Suspend
0
End Block
Item: 5 of 100 - ,• Ma rk --<] [::> ""'I ~· 1!';:'1
QIO: 2 185 ~ Prev1o u s Next Labf a lues Note s Calculator

1 •
2 FA17 p 182.2
3
Bug hints (if all else CHARACTERISTIC ORGANISM
4
fails) Asplcnic patient (due to surgical splenectomy Encapsulated microbes, especially S ll i~
5
·6
or autosplenectomy, eg, chronic sickle cell (S pneumoniae >> H influen;:ae type B >
· 7
disease) ;'\ meningitidis)
·8 Branching rods in oral infection, sulfur granules Actinomyces israelii
·9
C hronic granulomatous disease Catalase (±) microbes, espeeiall) S aureus
· 10
· 11
"Currant jellr" sputum Klebsiella
• 12 Dog or cat bite Pasteurella multocida
• 13
F'acial nen·e palsy (typically bilateral) Borrelia burgdor{eri (Lyme disease)
• 14
. 15 F'ungal infection in diabetic or Mucor or Rhizopus spp.
. 16 immunocompromised patient
. 17 Health care provider HBV (from needlestick)
• 18
eutropenic patients Candida albicans (system ic), Aspergillus
• 19
• 20 Orga n transplant recipient CMV
. 21 PAS (±) Tropheryma whipplei (Whipple disease)
. 22
• 23
Pediatric infection llaemophilus influenzae (including epiglottitis)
• 24 Pneumonia in cystic fibrosis, burn infection Pseudomonas aeruginosa
• 25 Pus, empyema, abscess S aureus
• 26
. 27
Rash on hands and feet Coxsackie A virus, Treponema pallidum, •
8
L.odt
s
Su~pl'nd
~
End Block
Item: 5 of 100 - ,• Ma rk --<] [::> ""'I ~· 1!';:'1
QIO: 2 185 ~ Prev1o u s Next Labf a lues Note s Calculator

1 • PAS <±l Tropheryma whipplei (Whipple disease)


2
Pediatric infection 1Taemophilus in{luenzae (including epiglottitis)
3
4 Pneumonia in cystic fibrosis, burn infection Pseudomonas aeruginosa
5 Pus, empyema, abscess S aureus
·6
Rash on hands and feet Coxsackie A virus, Treponema pallidum,
· 7
·8
Hicketlsia ricketlsii
·9 Sepsis/meningitis in newborn Croup B strep
· 10 Surgical \\ Otmd S aureus
· 11
• 12
Traumatic open wound Clostridium perfringens
• 13
• 14
FA17 p 183.2
. 15
Penicillin G, V Penicill in C (IV and ll\ I form), penicillin (ora I). Prototype P-lactam antibiotics.
. 16
. 17 MECHANISM 0 -Aia-D-Aia structural analog. Bind penicillin-binding proteins (transpeptidases).
• 18 Block transpeptidase cross-linking of peptidoglycan in cell wall.
• 19 Act ivatc autolytic enzymes .
• 20 CLI NICAL USE Mostly used for gram <±l organisms (S pneumoniae, S pyogenes, Actinomyces). Also used for gram 8
. 21 cocci (mainly I meningitidis) and ~piroch e t cs (namely T pallidum). Bactericidal for gram <±l cocci,
. 22
gram <±l rods, gram 8 cocci, and spirochetes. Penicill inase sensitive.
• 23
• 24
ADVERSE EFFECTS Hypersensitivity reactions, direct Coombs <±l hemolytic anemia.
• 25 RESISTANCE Penicillinase in bacteria (a type of P-lactamase) cleaves P-lactam ring.
• 26
. 27

8
L.odt
s
Su~pl'nd
~
End Block
Item: 6 of 100 - ,• Mark -<J [:::> "'I ~ · ~
QIO: 4 951 ~ P~v•ous N@xt Labl lues No tes Calcula t o r

A A
1
A 74-year-old woman presents to her primary care physician because of pressure in the lower part of t he abdomen. She lives in a nursing ~~AI
2
home and has a long hist ory of diabetes t hat has been well controlled on medication. Her heart rate is 72/ min, blood pressure is 124/82
3 mm Hg, and temperature is 37.1 oc (98.7° F) . Urinalysis result s are significant for 7-8 WBCs/hpf, 0-1 RBC/hpf, and protein <0. 15 g/24 hr.
4 Results of a stool guaiac test are negative.
5
·6
Without any additional tests, what is the most likely diagnosis?
·7 :

·8
A. Constipation
•9 B. Diverticulosis
• 10
C. Membranous nephropathy
· 11
D. Pyelonephritis
' 12
' 13 E. Urinary tract infection
' 14
' 15
. 16
. 17
• 18

' 19
' 20
. 21
. 22
' 23
' 24
' 25
' 26
. 27

8
Lode.
s
Suspe-nd
8
End Bloc:k
Item: 6 of 100 ~ ,• Mark <::J [:::> ""I ~· ~'j
QID: 4951 J.. Previous Next LAbfaiUI~S Notes Calculator

1
2 The correct a nswer is E. 67°/o chose this .
3 This patient is most likely suffering from urinary tract infect ion (UTI). UTis can cause infect ion in t he lower uri nary t ract (uret hrit is and
cystit is) or in the upper uri nary t ract (pyelonephrit is). Despite well-cont rolled glucose levels, patient s with diabetes are at an increased risk for
4
UTis with atypical present at ion due to autonomic neuropathy. They may present wit h bacteria in t he uri ne (bacteriuri a) but none of t he typical
5 symptoms of UTI, such as increased frequency and burn ing on urination . This makes diagnosis challenging because bact eriu ria does not need
6 t o be t reat ed even in diabetics. Furthermore, diabetic pat ients are more prone to having severe UTis than are patient s without diabet es. Risk
factors for UTis include female sex, pregnancy, sexual intercourse, diabetes, indwelling cathet ers, urinary t ract obst ruct ion such as an
• 7
anatomic abnormalit y or kidney st ones, hospit alization, or residence in a nu rsing home. Women are more prone t o UTis due to their shorter
•8 urethra and shorter dist ance bet ween t he rectum and the opening of t he uret hra. Escherichia coli remains t he most common causat ive agent
•9 of UTis .
• 10
· 11 St erile pyuri a (whit e blood cells in t he urine but absence of infection) is also a possibilit y and could be ruled out with a urine culture. Although
• 12 UTis req uire prom pt ant ibiot ic t rea t ment, antibiot ics are not recommended for sterile pyuria .
• 13 Urinary tract infection Pyuria Pyelonephritis Escherichia coli Urethra Bacteriuria Antibiotics Kidney Urine Urinary system Diabetes mellitus Kidney stone White blood cell Urination Urine culture

• 14 Urinary retention Glucose Bacteria Catheter Sexual intercourse Peripheral neuropathy Pregnancy Infertility Infection Rectum

• 15 A is not correct . 11% chose this .


. 16 Constipation is a common occurrence in t he elderl y populat ion and may manifest wit h vague lower abdominal discomfort, but it would not
. 17 result in WBCs being found on urinalysis .
Urinalysis Constipation
• 18
• 19 B is not correct. 7% chose t his .
• 20 Diverticulosis refers to the presence of m ultiple divert icula in t he lower gastrointest inal (GI ) t ract , is common in t he elderl y, and manifests
with recurrent abdominal pain in the left lower quadrant and a posit ive stool guaiac test or massive lower GI hemorrhage .
. 21
Diverticulosis Digestion Diverticulum Human gastrointestinal tract Gastrointestinal tract Quadrant (abdomen) Bleeding Abdominal pain Stool guaiac test
. 22
• 23
C is not co rrect. 5 °/o chose this .
Membranous nephropathy is one of the most common cause of nephrotic synd rome in adults (along wit h foca l segment al glomeruloscl erosis)
• 24
and would ma nifest wit h prot einuri a on uri nalysis. RBC casts are often found in t he uri ne as well. It may be caused by infections ( hepatit is B
• 25 or C, syphilis, schist osomiasis, or malari a), drugs (gold salts, penicillamine, or captopril), autoimmune diseases (lupus), and malignancy.
• 26 Captopril Schistosomiasis Nephrotic syndrome Penicillamine Proteinuria Focal segmental glomerulosclerosis Membranous glomerulonephritis Syphilis Hepatitis 8 Urinalysis Malaria Autoimmune disease

. 27 Kidnev disease Altoimmunitv Svstemic luous ervthematosus Urinarv cast Heoatitis Malionancv Urine Luous ervthematosus Cancer Red blood cell

8
Lock
s
Suspend
0
End Block
Item: 6 of 100 ~ ,• Mark <::J [:::> ""I ~· ~'j
QID: 4951 J.. Previous Next LAbfaiUI~S Notes Calculator

1 B is not correct. 7% ch ose t his .


2 Diverticulosis refers to the presence of m ultiple divert icula in t he lower gastrointest inal (GI ) t ract , is common in t he elderl y, and manifests
3 with recurrent abdominal pain in the left lower quadrant and a posit ive stool guaiac test or massive lower GI hemorrhage.
Diverticulosis Digestion Diverticulum Human gastrointestinal tract Gastrointestinal tract Quadrant (abdomen) Bleeding Abdominal pain Stool guaiac test
4
5 C is not co r r ect. 5 °/o chose this .
6
Membranous nephropathy is one of the most common cause of nephrotic synd rome in adults (along wit h foca l segment al glomeruloscl erosis)
and would manifest wit h prot einuri a on uri nalysis. RBC casts are often found in t he uri ne as well. It may be caused by infections (hepatit is B
• 7
or C, syphilis, schist osomiasis, or malari a), drugs (gold salts, penicillamine, or captopril), autoimmune diseases (lupus), and malignancy.
•8 Captopril Schistosomiasis Nephrotic syndrome Penicillamine Proteinuria Focal segmental glomerulosclerosis Membranous glomerulonephritis Syphilis Hepatitis 8 Urinalysis Malaria Autoimmune disease

•9 Kidney disease Altoimmunity Systemic lupus erythematosus Urinary cast Hepatitis Malignancy Urine Lupus erythematosus Cancer Red blood cell

. 10 D is n ot cor rect. 10% c h ose this.


· 11 Pyelonephri t is typically manifest s with flank pain, increased frequency of urination, and persistent fever. Physical examinat ion t ypically revea ls
. 12 t enderness at t he costovertebral angle, and WBC casts are ty pically found in t he uri ne. The onset of symptoms often occurs approximately 1
• 13 week after t he onset of a lower UTI.
Costovertebral angle tenderness Pyelonephritis Abdominal pain Urine Fever Physical examination Costovertebral angle
• 14

• 15
. 16 Bottom Line:
. 17 Pat ients wit h diabet es are at an increased risk for asymptomatic urina ry t ract infect ions, which manifest with bacteria in the uri ne but none
• 18
of t he ty pical symptoms.
Diabetes mellitus Urinary system Urine Urinary tract infection Asymptomatic Bacteria
• 19
. 20
. 21
141;fil·1i•J f or yea r:[ 2017 •
FIRST AID FA CTS
. 22
• 23
• 24 FA 17 p 562.2

• 25 Casts in urine Presence of casts indicates that hematuria/pyuria is of glomerular or renal tubular origin.
• 26 Bladder cancer, kidney stones - hematuria, no casts .
. 27 .. , .
8
Lock
s
Suspend
0
End Block
Item: 6 of 100 - ,• Mark -<J [:::> "'I ~ · ~
QIO: 4 951 ~ P~v•ous N@xt Labl lu es No tes Calcula t o r

A A
1
2 l@l;fil·!l•l for y ea r: 2017 •
fiRS T AID FACTS
3

4
FA17 p 562.2
5
6 Cast s in urine Presence of casts indicates that hematuria/pyuria is of glomerular or renal tubular origin.
·7 Bladder cancer, kidney stones - hematuria, no casts.
Acute cystitis - pyuria, no casts.
•9
RBC casts Glomerulonephritis, malignant hypertension.
• 10
· 11
WBC casts Tubulointerstitial inAammation, acute prelonephritis, transplant rejection.
• 12 Fatty casts ("oval fat lcphrotic syndrome. Associated with '' Maltese cross" sign.
• 13 bodies")
• 14
Granular ("muddy Acute tubular necrosis.
• 15
brown") casts
. 16
. 17 Waxy casts [!] End-stage renal disease/chronic renal fa ilure.
• 18 Hyaline casts 01 Nonspecific, can be a normal find ing, orten seen in eonccntrateclurine samples.
• 19
• 20
. 21
. 22
. 23
• 24
• 25
• 26
. 27

8
Lode.
s
Suspe-nd
8
End Bloc:k
Item: 6 of 100 ~ ,• Mark <::J [:::> ""I ~· ~'j
QID: 4951 J.. Previous Next LAb faiUI~S Notes Calculator

1 • FA17 p 177.2 •
2
UTI bugs
3
SPECIES FEATURES COMMENTS
4
5
Escherichia coli Leading cause of UTI. Colonies show green Diagnostic markers:
6
metallic sheen on EMB agar. (f) Leukocyte esterase = evidence ofWBC
• 7 Staphylococcus 2nd leading cause of UT I in sexually active activity.
(f) litrite test = reduction of urinary nitrates
•8 saprophyticus women.
•9 by bacterial species (eg, E coli).
Klebsiella pneumoniae 3rd leading cause of UTI. Large mucoid capsule
• 10 Ef> Urease test = urease-producing bugs (eg,
and viscous colonies.
· 11 S sa(Jrophyticus, Proteus, Klebsiella).
• 12 Serratia marcescens Some strains produce a red pigment; often
• 13 nosocomial and drug resistant.
• 14 Enterococcus Often nosocomial and drug resistant.
• 15
Proteus mirabilis Motility causes "swarming" on agar; produces
. 16
. 17
urease; associated with struvite stones .
• 18 Pseudomonas Blue-green pigment and fruity odor; usually
• 19 aeruginosa nosocomial and drug resistant.
• 20
. 21
FA17 p 140.1
. 22
• 23 Escherichia coli Gram 8 rod. E coli virulence factors: fimbriae - cystitis and pyelonephritis (P-pili); K capsule-
• 24 pneumonia, neonatal meningitis; LPS endotoxin-septic shock.
• 25
STRAIN TOXINAND MECHANISM PRESENTATION
• 26
. 27 • EIEC Microbe invades intestinal mucosa and causes Invasive; dysentery. Clinical manifestations •

8
Lock
s
Suspend
0
End Block
Item: 6 of 100
QIO: 4 951

2
- ,•
~
Mark

,. , .. , -<J
P~v•ous
[:::>
N@xt
... .
Labl
"'I
lues
~ ·
No tes
.. .
~
Calcula t o r

3
FA17 p 140.1
4
5
Escher ich ia coli Cram 8 rod. E coli virulence factors: fimbriae-cystitis and pyelonephritis (P-pili); K capsule-
6 pneumonia, neonatal meningitis; LPS endotoxin-septic shock.
· 7 STRAIN TOXIN AND MECHANISM PRESENTA liON
·8
EIEC Microbe invades intestina I mucosa and causes ln\'asi, e; d~senter). Clin ical manifestations
•9
necrosis and inflammation. similar to Shigella .
• 10
· 11 ETEC Produces heat-labile and heat-stable Tra,cJers' diarrhea (watery).
• 12 enteroToxins. No inflammation or invasion .
• 13 EPEC o toxin produced. Adheres to apical surface, Diarrhea, usually in children (Pediatrics).
• 14
fl attens \'illi, prevents absorption.
• 15
. 16
EHEC 0157:H7 is most common serotype in US. Orten Dysentery (toxin alone causes necrosis and
. 17
transmilted via unclercooked meat, raw leafy inAammat ion).
• 18 vegetables. Does not ferment sorbitol (vs other E coli).
• 19 Shiga-like toxin causes hemolytic-uremic ll cmorrlwgic, Hamburgers, llemolytic-uremic
• 20 syndrome: triad of anemia, thrombocytopenia, syndrome.
. 21 and acute renal failure due to microthrombi
. 22 forming on damaged endothelium
. 23 - mechanical hemolysis (with schistocytcs on
• 24 peripheral blood smear), platelet consumption,
• 25 and l rena I blood Aow.
• 26
. 27 •
8
Lode.
s
Suspe-nd
8
End Bloc:k
Item: 7 of 100 - ,• Mark -<J [:::> "'I ~ · ~
QIO: 2043 ~ P~v•ous N@xt Labl lues No tes Calcula t o r

1
An 8-year-old boy presents t o his pediatrician complaining of a fever and a sore throat for 2 days. Physical exam inat ion revea ls the
2
presence of purulent tonsillar exudat es and swollen, tender anterior cervical nodes. Culture of the tonsillar exudat e reveals gram-positive
3 cocci.
4
5 Whi ch of the following are characteristic of t he organism most likely causing this boy's infection?
6 :
·7 A. Catalase-negative, a-hemolytic, optochin-resistant
·8 B. Catalase-negative, a-hemolytic, optochin-sensitive
•9
C. Catalase-negative, 13-hemolytic, bacitracin- resistant
• 10
· 11 0 . Catalase-negative, 13-hemolytic, bacitracin-sensitive
• 12 E. Catalase-positive, coagulase -negative, novobiocin-sensitive
• 13
• 14

• 15
. 16
. 17
• 18
• 19
• 20
. 21
. 22
. 23
• 24
• 25
• 26
. 27

8
Lode.
s
Suspe-nd
8
End Bloc:k
Item: 7 of 100 ~ ,• Mark <::J [:::> ""I ~· ~'j
QID: 2043 J.. Previous Next LAbfaiUI~S Notes Calculator

1
2 The correct answer is 0. 59°/o chose this.
3 This patient present s with t he classic symptoms (fever, sore t hroat , ant eri or cervical lymphadenopathy, lack of cough) for streptococcal
4 pharyng it is (st re p t hroat) . This is caused by t he cat alase- negative, ~ - hemo l yti c, bacit racin-sensit ive Streptococcus pyogenes. Diagnosis is
confirmed with a rapid st rep t est or with t hroat swab cultu re, and t reatment is with penicillin.
5
Streptococcal pharyngitis Streptococcus pyogenes Rapid strep test Penicillin Pharyngitis Lymphadenopathy Streptococcus Cervical lymphadenopathy Sore throat Cough Catalase Fever
6
7
A is not correct. 7°/o chose this.
Cat alase- negative, a-hemolytic, optochin-resistant are charact eristics of t he v irid ans group of streptococci, which includes Streptococcus
•8
mutans and Streptococcus sanguis . S. mutans is part of t he normal flora of the oropharynx and is associated wit h t he format ion of dental
•9 caries (cavit ies). S. sanguis is associat ed with subacut e bact erial endocardit is.
• 10 Streptococcus mutans Streptococcus sanguinis Endocarditis Dental caries Pharynx Streptococcus Subacute bacterial endocarditis Infective endocarditis Human microbiota Catalase

· 11 B is not correct. 17% chose this.


• 12 Cat alase- negative, a-hemolytic, and opt ochin-sensitive describe Streptococcus pneumoniae. This organism causes ot it is media and sinusit is in
• 13 children. It also commonly causes pneumonia in adults and t he elderly. Moreover, it is t he number one cause of meningit is in adults and t he
elderly, alt hough rat es of S. pneumoniae meningitis have decreased wit h t he advent of the pneumococcal vaccine .
• 14
Streptococcus pneumoniae Otitis media Sinusitis Meningitis Pneumonia Streptococcus Pneumococcal vaccine Otitis vaccine Catalase Organism
• 15
. 16
C is not correct. 12% chose this .
Streptococcus aga/actiae (aka Group B Streptococcus) is a cat alase- negative, ~ - hemo l ytic, bacit racin-resistant bacteri um. It is a significant
. 17
cause of serious bact eri al infect ion (can cause pneumonia, meningit is, and sepsis) in neonat es .
• 18 Streptococcus agalactiae Meningitis Sepsis Pneumonia Streptococcus Pathogenic bacteria Bacteria Infant Catalase

• 19
E is not correct. 5°/o chose this .
• 20
Cat alase- posit ive, coagulase- negative, novobiocin -sensit ive describes Staphylococcus epidermidis. I nfect ion wit h S. epidermidis is associated
. 21 with skin penet ration by implant ed prosthetic devices such as prosthet ic heart valves, intravenous lines, and int raperitoneal cat het ers .
Staphylococcus epidermidis Intravenous therapy Catheter Staphylococcus Heart valve Prosthesis Infection
. 22
• 23
• 24 Bottom Line:
• 25 Streptococcus pyogenes is a Group A ~ - hemo lyti c, catalase-negat ive, bacitracin-sensit ive organism that causes pharyngit is.
• 26 Streptococcus pyogenes Pharyngitis Streptococcus Organism

. 27

8
Lock
s
Suspend
0
End Block
Item: 7 of 100 - ,• Mark -<J [:::> "'I ~ · ~
QIO: 2043 ~ P~v1ous N@xt Labl lu es No tes Calcula t o r

1 •
2
3
l@l;fil·!ltl for year: 2017 •
FIRST AID f ACTS

4
5 FA17 p 132.4
6
Streptococcus Cram ® cocci in chains · . Croup strep J• J\ ES (major criteria for acute rheumatic
7
pyogenes (group A cause: fe, er):
·8
streptococci) • Progenic-pharyngitis, cellulitis, impetigo Joints-polyarthritis
•9
• 10
("honey-crusted" lesions), erysipelas • -carditis
· 11
• Toxigenic- scarlet fever, toxic shock- like :\odules (subcutaneous)
• 12 srndrome, necrotizing fasciitis Erythema marginatum
• 13 • Immunologic-rheumatic fever, Srdenham chorea
'
• 14 glomerulonephritis Pharyngitis can result in rheumatic ''phever"
• 15 Bacitracin sensiti,·e, ~-h emolytic, pyrrolidonyl and glomeruloneph ritis.
. 16 arylamidase (PYR) ®. Hyaluronic acid capsule Impetigo usually precedes glomerulonephritis.
. 17
c inhibits phagocytosis. Antibodies to I protein Scarlet fever-blanchi ng, sandpaper-like body
• 18 enhance host defenses against S (J)'Ogenes but rash, strawberry tongue, and circumoral
• 19 can give rise to rheumatic fever. pallor in the setting of group A streptococca1
• 20 ASO titer or anti-DNasc 8 antibodies indicate pharyngitis (erythrogenic toxin ®).
. 21
recent S pyogenes infection .
. 22
. 23
• 24 FA17 p 130.1
• 25
Gram-positive lab algorithm
• 26
. 27 r -.. -~ t................ , ..........,

8
Lode.
s
Suspe-nd
8
End Bloc:k
Item: 7 of 100
QID' 2043
- ,•
.1 Ma
ric <J [:::> "'I
A

·-
~
P~v•ous N @xt LabValu es N o tes Calcula t o r

A
1 A

FA17p1 30.1
2
3 Gram-positive lab algorithm
4
Gram(±) (purple/blue)
5
6
7
Branching
·8 Bacilti Cow filaments
•9
• 10 I 1
· 11 AefobK Anaerobic Anaerobic/facultabve Aerobtc Anaerobic
• 12
Ustem
• 13
Bacillus aostridium Nocardlil Actinomyces
Corynebacterium (weakly acid fast) (not actd fast)
• 14 Propionibacterium
• 15 Catalase
. 16
1 e I
® )
. 17 Streptococcus Staphylococcus
• 18
T . T
• 19 Hemolys1s Coagulase
• 20 _l
. 21 (Partial (Complete
. 22 a hemolysis.
green)
hemolysis•
clear)
(No hemolySis,
grows in bile)
Saureus
. 23
Novobiocin
• 24 sensitivity
Optochin sensitivity Bacitracin sensitivity Growth in 6.5% NaCl
• 25 and bile solubility

~ ~
• 26
. 27 •
8
Loclt
s
Suspe-nd
8
End Bloc:k
2
Novobiocin
Bacitracin sensitivity Growth in 6.5% NaCl sensitivity
3 Optochin sensitivity
and bile solubility
4
5
6
GroupS Group A S s.prophyticus S epidennidis
7 sagalactiae S~nes
·8
•9
• 10 Vindans streptococci GroupO
· 11 (no capsule) S pneumoniae Nonenterococcus (enterococcus)
Smutans (encapsulated) 5 boVIS Efaecwm
• 12 S mitiS Efaecalls
• 13
• 14
Important tests are 1n bold. Important pathogens are in bold italics.
Note: Enterococcus IS either a- or y-hemolytic.
• 15
. 16
. 17 FA17 p 131 .2
• 18
~hemolytic bacteria Gram EB cocci. Complete lysis of RBCs ..... clear area surround ing colony on blood agar fl. Include
• 19
the fo llowing organisms:
• 20
. 21
Staphylococcus aureus (catalase and coagulase EB)
. 22
Streptococcus p)'ogenes-group A strep (catalase 8 and bacitracin sensitive)
. 23 Streptococcus agalactiae-group B strep {catalase 8 and bacitracin resistant)
• 24
• 25
• 26
. 27 •
8
Lode.
s
Suspe-nd
8
End Bloc:k
Item: 8 of 100 - ,• Mark -<J [:::> "'I ~ · ~
QIO: 3228 ~ P~v•ous N@xt Labl lues No tes Calcula t o r

1
A biotechnology firm is developing a new small prot ein drug designed to prevent the spread of a sexually transmitted infect ion. Scientists
2
want to block the infect ious step of the bacteria's reproductive cycle. A Giemsa-stained smear of a patient's uret hral discharge shows
3 cytoplasmic inclusions similar to those shown in the image.
~--~~~-------------------,~-----,
4
5
6
7
·8
•9
• 10
· 11
• 12
• 13
• 14

• 15
. 16
. 17
Du ring which stage of this pathogen's life cycle is it most infect ious?
• 18
:
• 19
A. Cytoplasmic inclusion body
• 20
. 21
B. Extracellular elementary body
. 22 C. Intracellular elementary body
. 23
D. Multiplication of reticu late bodies
• 24
• 25
E. Reticulate body
• 26
. 27

8
Lode.
s
Suspe-nd
8
End Bloc:k
Item: 8 of 100 ~ ,• Mark <::J [:::> ""I ~· ~'j
QID: 3228 J.. Previous Next LAbfaiUI~S Notes Calculator

1
2 The correct answer is B. 52% chos e this.
3 Chlamydia trachomatis infection causes ureth rit is, cervicit is, and pelvic inflammatory disease (PID) in women, as well as conj unctiv itis and
4 reactive arthrit is (Reit er's synd rome), alt hough it is f req uently asymptomatic. Cytoplasmic inclusions can be seen on Giemsa- or fluorescent
ant ibody-stained urethral or cervical smea r~ but diagnosis can also be made from a uri ne sample using nucl eic acid amplification techniq ues.
5
Treatment of Chlamydia infection requires a course of eit her doxycycline or azithromycin. The ext racellu lar element ary body is the infect ious
6 form of Chlamydia as it can attach t o host cells and ent er t hem.
7 Pelvic inflammatory disease Chlamydia trachomatis Azithromycin Reactive arthritis Doxycycline Urethritis Cervicitis Chlamydia infection Conjunctivitis Nucleic acid Asymptomatic Urine Chlamydia (genus)

8 Arthritis Polymerase chain reaction Pap test Infection Cytoplasm Cervix Giernsa stain Urinalysis Inflammation

•9 A is not correct. 7°/o chose this .


• 10 The cytoplasmic inclusions are collections of elementary bodies in t he host cell before t heir release. Because t hey are intracellular, t hey are
· 11 not infectious.
Inclusion bodies Cytoplasm Intracellular Chemical element
• 12
• 13 C is not correct. 16% chos e this .
• 14 The int racellu lar element ary body does not have access t o host cells, and t herefo re is not infect ious .
Intracellular
• 15
. 16 D is not correct. 11% chose this .
. 17 The m ult iplicat ion of ret iculat e bodies is an important st ep in t he reproduct ion of Chlamydia . Because t his process is intracellular, however, it
is not an infectious st ep in the life cycle of this bact eri um .
• 18 Chlamydia infection Chlamydia (genus) Bacteria Intracellular
• 19
E is not correct. 14°/o chose this .
• 20
The ret iculate body is solely int race llular. This is t he st age in which t he bacteria replicat e within the cell.
. 21 Intracellular Bacteria
. 22
• 23
Bottom Line:
• 24
Chlamydia trachomatis, which can cause uret hrit is, cervicit is, and pelvic inflammatory disease in women, is most infectious as an elementary
• 25
body t hat can enter host cells .
• 26 Pelvic inflammatory disease Chlamydia trachomatis Chlamydia infection Urethritis Cervicitis Pelvis Inflammation
. 27

8
Lock
s
Suspend
0
End Block
Item: 8 of 100 - ,• Mark --<] [::> ""'I ~· 1!';:'1
QIO: 3 2 28 ~ Prev1o u s Next Labf a lu es Note s Calculator

1
2
3
141:l'il!11•1
FIRST AIO FACTS
for yea r : 2017 •

5 FA17 p 146.1
6
Chlamydiae Chlamydiae cannot make their own ATP. They Chlamys = cloak (intracellular).
7
are obligate intracellular organisms that cause C psittaci-has an avian resenoir (parrots),
8
·9
mucosal infections. 2 forms: causes atypical pneumonia.
· 10
Llementa ry body (sma II, dense) Lab diagnosis: PCR, nucleic acid amplification
· 11
is "Enfectious" and E nters cell ,·ia test. Cytoplasmic inclusions (reticulate bodies)
• 12 Endocytosis; transforms into reticulate body. seen on Ciemsa or Auorescent antibody-
• 13 Reticulate body Replicates in cell by fission; stained smear.
• 14 Reorganizes into elementary bodies. The chlamydia! cell wall lacks classic
. 15 Chlamydia trachomatis causes reactive arthritis peptidoglycan (due to reduced muramic acid),
. 16 (Reiter synd rome), follicular conjunctivitis fJ, rendering ~-lactam antibiotics ineffective.
. 17 nongonococcal urethritis, and PID.
• 18 Chlamydophila pneumoniae and Chlamydophi/a
• 19 psittaci cause atypical pneumon ia; transmitted
• 20 by aerosol.
. 21
Treatment: azithromycin (fa,·ored because one-
. 22
1ime treatment) or doxycycl ine (+ ceft riaxone
• 23
for possible concomitant gonorrhea).
• 24
• 25
• 26 FA17 p 146.2
. 27 Chlomvdio trochomotis S@rotvD@S •
8
L.odt
s
Su~pl'nd
~
End Block
2 Reorganizes into elementary bodies. T he ch lamydia ! cell wall lacks classic
3 Chlamydia trachomatis causes reacti,·e arthritis peptidoglycan (due to reduced muramic acid),
4 (Reiter syndrome), follicular conjunctivitis lfJ, rendering P-lactam antibiotics ineffective.
5 nongonococcal urethritis, and PID.
6 Chlamydophila pneumoniae and Chlamydophila
7 psittaci cause atypical pneumonia; transmitted
8 by aerosol.
•9 Treatment: azithromycin (fa,·ored because one-
• 10
time treatment) or dox)cycline (+ ceftri<l\One
· 11
for possible concomitant gonorrhea).
• 12
• 13
• 14 FA17 p 146.2
• 15 Chlamydia trachoma tis serotypes
. 16 Types A, 8, and C Chronic infection, cause blindness due to ABC = Africa, Blindness, C hronic infection.
. 17 follicular conjunctivitis in Africa .
• 18
• 19
Types D- K Urethritis/PTD, ectopic pregnancy, neonatal 0-K = everything else.
• 20
pneu monia (staccato cough) with eosinophi lia, eonata l disease can be acquired during
. 21
neonatal conjunctivitis (1-2 weeks after birth). passage through in fected birth canal.
. 22 Types l 1, l 2, and L3 Lymphogranuloma venereum-small, painless
. 23 ulcers on genitals - swollen, painful inguinal
• 24 lymph nodes that ulcerate (buboes). Treat with
• 25 doxycycline.
• 26
. 27 •
8
Lode.
s
Suspe-nd
8
End Bloc:k
Item: 9 of 100 - ,• Mark -<J [:::> "'I ~ · ~
QIO: 3295 ~ P~v•ous N@xt Labl lues No tes Calcula t o r

1
A young girl living in ru ral New Mexico is brought t o her pediat ri cian with complaints of feve r, cou gh, and fat igue for the past 2 weeks. The
2
physician notices that the pati ent is having intermittent bouts of many coughs in a single breath, followed by a deep inspiration. The
3 parents report that this pattern of coughing had started in the past 2 days. The physician info rms t hem t hat thei r daughter will most likely
4 recover with only supportive care. However, she want s to confirm his diagnosis. A swab is obt ained from t he posterior nasopharynx is sent for
culture for a specific organism.
5
6
Which of the following culture media will be used?
7
:
8
A. Bordet-Gengou medium
·9
• 10 B. Charcoal yeast extract with iron and cysteine
· 11 C. Chocolate agar with factors V and X
• 12
D. Leffler's medium
• 13
• 14
E. Thayer-Martin medium
0
15
. 16
. 17
• 18
0 19
0
20
. 21
. 22
0
23
0 24
0
25
0
26
. 27

8
Lode.
s
Suspe-nd
8
End Bloc:k
Item: 9 of 100 ~ ,• Mark <::J [:::> ""I ~· ~'j
QID: 3295 J.. Previous Next LAb faiUI~S Notes Calculator

1
2 The correct a nswer is A. 56°/o chose t h is .
3 This patient is present ing with a classic case of whooping cough caused by Bordetel/a pertussis. The init ial phase is characterized by f lu- like
4 symptoms for t he f irst 1- 2 weeks. Duri ng t his t ime, erythromycin is an effect ive t reat ment . The second phase, th e paroxysmal stage, is
marked by bouts of m ultiple coughs in a single brea th followed by a deep inspira t ion (the cl assic whooping cough) . Treat ment duri ng this
5
phase does not change the disease course, so only supportive care is indicat ed, and the infect ion ought t o pass in otherwise healt hy
6 ind ividuals. I n the Unit ed Stat es, t he dipht heri a/tetanus/pertussis (DTaP) vaccine is supposed to be given to all infants and prot ects t hem
7 against diphtheria, tetanus, and pertussis. I nfants who are not vaccinat ed are at risk for infect ion . B. pertussis can only be cultured on
Bordet-Gengou med ium .
8
Erythromycin Bordetella pertussis Diphtheria Pertussis Tetanus Bordet-Gengou agar Bordetella Cough Vaccine Influenza-like illness Infection
9
• 10
B is not correct. 10% chose this .
Charcoal yeast extract when buffered with increased levels of iron and cyst eine is used t o cultu re Legionel/a pneumophila. Legionel/a is a
· 11
common cause of both community-acquired pneumonia and hospi tal -acquired pneumonia, especially in t he elderl y populat ion. Legionnaires'
• 12 disease is characterized by acutely ill pat ients wit h pneumonia, high fever, nausea, vomit ing , and diarrhea .
• 13 Legionella pneumophila Legionnaires' disease Legionella Pneumonia Yeast extract Cysteine Diarrhea Yeast Hospital-acquired pneumonia Community-acquired pneumonia Nausea

• 14 Buffered charcoal yeast extract agar Vomiting Fever Charcoal Iron

• 15 C is not correct. 13% chos e this .


. 16 Chocolate agar wit h factor V and X is used t o cult ure Haemophilus influenzae. H. influenzae commonly coloni zes the nasopharyngeal mucosa .
. 17 H. influenzae type B (H ib) is a human pat hogen t hat causes pneumonia, epiglott itis, bact eremia, and sept ic arthri tis. With immunizat ion
recommendat ions, Hib is now a rare cause of disease. Nontypable H. influenzae causes less severe disease, such as acut e otit is media and
• 18
acute sinusit is .
• 19 Epiglottitis Haemophilus influenzae Bacteremia Otitis media Chocolate agar Septic arthritis Pneumonia Sinusitis Pathogen Immunization Otitis Pharynx Agar Mucous membrane Arthritis Nasopharynx

• 20 Haemophilus

. 21
D is not correct. 13% c hose this .
. 22
Loffler's medium is needed to culture Corynebacterium diphtheriae . Respiratory dipht heri a is caused by t oxin product ion by pathogenic
• 23 bacteri a. Sympt oms include sore throat , malaise, cervical lymphadenopathy, and low-grade fever. Clinical signs charact eri st ically include a
• 24 grayish-whit e exudat e on the oropharyngeal m ucosa. I n add it ion, t here is a pseudomembrane t hat fo rms on the m ucosa that bleeds wit h
scraping .
• 25
Corynebacterium diphtheriae Diphtheria Lymphadenopathy Pharynx Exudate Sore throat Malaise Pathogenic bacteria Bacteria Pathogen Corynebacterium Mucous membrane Toxin Fever
• 26
. 27
E is not correct. 8°/o chos e this .
8
Lock
s
Suspend
0
End Block
Item: 9 of 100 ~ ,• Mark <::J [:::> ""I ~· ~'j
QID: 3295 J.. Previous Next LAbfaiUI~S Notes Calculator

1
E is not correct. 8 °/o chose thi s.
2
Thayer-Mart in medium is used t o cult ure Neisseria gonorrhoeae . Gonorrhea is a sexually transmit ted infection t hat is very often
3 asymptomat ic. When symptomatic, female patients com plain of vaginal discharge and dysuria. Unt reated infection can result in pelvic
4 inflammatory disease, which can lead t o scarring of t he reproductive t ract , leading t o infertility.
Pelvic inflammatory disease Sexually transmitted infection Dysuria Gonorrhea Neisseria gonorrhoeae Vaginal discharge Asymptomatic Neisseria Infertility ThCf)'er-Martin agar Reproductive system Pelvis
5
Infection Inflammation
6
7
8 Bottom Line:
9 Bordete/la pertussis infect ion is indicated by whooping cough and is cult ured on Bordet -Gengou med ium .
Bordetella pertussis Pertussis Bordet-Gengou agar Bordetella Cough Infection
. 10
· 11
. 12
• 13
l@l;fil·1i•l for year:[ 2017
FIRST AID FACTS .
•j .

• 14

• 15 FA17 p 123.1
. 16 Special culture requirements
. 17 BUG MEDIA USED FOR ISOLATION MEDIACONTENTS/OTHER
• 18
H influenzae Chocolate agar Factors V ( AD+) and X (hematin)
• 19
. 20 N gonorrhoeae, Thayer-Martin agar Selectively favors growth of eisseria by
. 21 N meningitidis in hibiting growth of gram Etl organisms
. 22 with Vancomycin, gram 8 organisms except
• 23 Neisseria with Trimethoprim and Colistin,
• 24 and fungi with :\'ystatin
• 25 Very Typically C ultures :-.!eisseria
• 26
Bpertussis Bordet-Cengou agar (Bordet for Bordetella) Potato extract
. 27
8
Lock
s
Suspend
0
End Block
Item: 9 of 100 ~ ,• Mark <::J [:::> ""I ~· ~'j
QID: 3295 J.. Previous Next LAbfaiUI~S Notes Calculator

1 • •
FA17 p 123.1
2
Special culture requirements
3
BUG MEDIA USED FOR ISOLATION MEDIACONTENTS/OTHER
4
5 H influenzae Chocolate agar Factors V ( AD+) and X (hematin)
6 N gonorrhoeae, Thayer-Martin agar Selectively favors growth of eisseria by
7 N meningitidis inhibiting growth of gram Etl organisms
8
with Vancomycin, gram 8 organisms except
9
Neisseria with Trimethoprim and Colistin,
• 10
and fungi with :\'ystatin
· 11
Very Typically Cultures :-.!eisseria
• 12
• 13 Bpertussis Bordet-Cengou agar (Bordet for Bordetella) Potato extract
• 14 Regan-Lowe medium Charcoal, blood, and antibiotic
• 15 Cdiphtheriae Tellurite agar, LofAer med ium
. 16
M tuberculosis Lowenstein-Jensen agar
. 17
• 18 M pneumoniae Eaton agar Requires cholesterol
• 19 Lactose-fermenting MacConkey agar Fermentation produces acid, causing colonies to
• 20 enterics turn pink
. 21
. 22
E coli Eosin-methylene blue (EMB) agar Colonies with green metallic sheen
• 23 Legionella Charcoal yeast extract agar buffered with
• 24 cysteine and iron
• 25
Fungi Sabouraud agar "Sab's a f un guy1"
.
• 26
• •
. 27

8
Lock
s
Suspend
0
End Block
Item: 9 of 100 - ,• Mark -<J [:::> "'I ~ · ~
QIO: 3295 ~ P~v•ous N@xt Labl lu es No tes Calcula t o r

A A
1
FA17 p 137.2
2
3 Gram-negative lab algorithm
4
5
Gram e (ptnk)
6
7
DiplocoCCI Coccobacim Comma-shaped rods
8
9
• 10
· 11 Aerobic HMmOphilus innwnne Oxidase '±'
(reqUires factors Vand X)
• 12 Pasteurella
Maltose utilization
• 13 Brucella
• 14
Elorrktella perlussis Grows in 42•c Grows in alkaline media Produces urease
Francisella tularensis
0
15
Ngonorrh~H Campyloblcterjejuni Vibrio cholerile Helicoblcter pylori
. 16 N meningitidis
Moraxelu
. 17
• 18
0 19
0
20
Bacilli
. 21
. 22 Lactose fermentation
0
23
0 24
0
25 Oxidase Fast Slow
0
26
. 27

8
Lode.
s
Suspe-nd
8
End Bloc:k
Item: 9 of 100 - ,• Mark -<J [:::> "'I ~ · ~
QIO: 3295 ~ P~v•ous N@xt Labl lues No tes Calcula t o r

A A
1
2 H1emophilus influtnz.e
Aerob•c Oxidase(±)
3 (reqwes fadors V and X)
Pasteurella
4 Maltose utilization
Brucella
5 Bordtttlu ptrlussis Grows in 42•c Grows in alkaline media Produces urease
6 Frandsella tularensis
7 CiJmpylobiJcttr jtjuni Vibrio choltriJt
N mtningitidis
8
9
• 10
· 11
• 12
• 13
• 14 lactose fermentation
0
15
-e---'--~(j
. 16
. 17 Oxidase Fast ] S_lo_w---.,
)
• 18
Kltbsifll•
0 19 Citrobacter
Ecoli
0
20 H2S production Pseudomon~s
Enterobacter
Serratia
. 21 on TSI agar
. 22
0
23
0 24
SiJtmonttu
0
25 Prottus
0
26
. 27
Important tests are in bold. Important PiJthogens are in bold it1lics.

8
Lode.
s
Suspe-nd
8
End Bloc:k
Item: 9 of 100 ~ ,• Mark <::J [:::> ""I ~· ~'j
QID: 3295 J.. Previous Next faiUI~S
LAb Notes Calculator

1 • •
FA17 p 128.1
2
Bugs with exotoxins
3
BACTERIA TOXIN MECHANISM MANIFESTATI ON
4
5 Inhibit protein synthesis
6 Corynebacterium Diphtheria toxin3 Pharyngitis with pseudomembranes in th roat
7 diphtheriae and severe lymphadenopathy (bull neck)
Inactivate elongation factor
8
Pseudomonas Exotoxin N (EF-2) Host cell death
9
• 10
aeruginosa
· 11 Shigella spp. Shiga toxin (ST)3 GI mucosal damage .... dysentery; ST also
• 12 Inactivate 60S ribosome by enhances cytokinc release, causi ng hemolytic-
• 13 removing adenine from uremic syndrome (HUS)
• 14 rR A
Enterohemorrhagic Shiga-like toxin SLT en hances cytokine release, causing HUS
• 15
E coli (EHEC) (SLT)a (prototypically in EHEC serotype 0157:H7).
. 16
Unlike Shigella, EHEC does not invade host
. 17
• 18
cells
• 19 Increase fluid secretion
• 20
Enterotoxigenic Heat-labile Overactivates adenylate Watery diarrhea: "labile in the Air (Aclenylate
. 21
E coli (ETEC) toxin (LT)3 cyclase (t cAMP) .... t CI- cyclase), stable on the Ground (Guanylate
. 22
secretion in gut and H7 0 cyclase)"
• 23
efAux
• 24
I Ieat-stable Overactivates guanylate
• 25
toxin (ST ) cyclase (t cGMP)
• 26
. 27 •
- l resorption of NaCI •

8
Lock
s
Suspend
0
End Block
Item: 9 of 100 - ,• Mark --<] [::> ""'I ~· 1!';:'1
QIO: 3 295 ~ Prev1o u s Next Labf a lu es Notes Calculator

protem receptor), a set P


2 Clostridium Botulinum toxin 3 of proteins required for Flaccid paralysis, Aoppy baby; toxin prevents
3
botulinum neurotransmitter release release of stimulatory (ACh) signals at
4
via vesicular fusion neuromuscular junctions - Aaccid paralysis
5
6
a An AB toxi n (aka, two-component toxin [or three for anthrax]) with B enabling binding and triggering uptake (endocytosis)
7 of I he act i'e A component. T he A components are usually ADP ribosylt ransferases; others ha\'e enzymatic activities as I isted
8 in chart.
9
Lyse cell membranes
· 10
· 11 Clostridium Alpha toxin Phospholipase (lecithinase) Degradation of phospholipids - myonecrosis
• 12 perfringens that degrades tissue and ("gas gangrene") and hemolysis ("double zone"
• 13 cell membranes of hemolysis on blood agar)
• 14
Streptococcus Streptolysin 0 Protein that degrades cell I .yses RBCs; contributes to ~-h em olys is;
. 15
pyogenes membrane host antibodies against toxin (ASO) used to
. 16
diagnose rheumatic fe,·er (do not confuse
. 17
with immune complexes of poststreptococcal
• 18
glomerulonephritis)
• 19
• 20 Superantigens causing shock
. 21 Staphylococcus Toxic shock Binds to IIIC II and TC R Toxic shock syndrome: fever, rash, shock; other
. 22 aureus syndrome toxin toxins cause scalded skin syndrome (exfoliative
outside of antigen binding
• 23 to>.in) and food poisoning (heat-stable
(TSST-1) site to cause O\'e rwhelming
• 24
release of IL-1, IL-2, enterotoxin)
• 25
Streptococcus Exotoxin A IF, -y, and T N F-a Toxic shock-like svndrome: fe\'er, rash, shock;
• 26
. 27 pyogenes - shock scarlet b ·er •
8
L.odt
s
Su~pl'nd
~
End Block
Item: 10 of - ,• Mark -<J [:::> "'I ~ · ~
100 ~ P~v1ous N @xt Labl lues N o tes Calcula to r

1
A 3-week-old infant is brought to t he clinic because of vomiting, fever, refusing feed ings, and cry ing more t han usual. The physician
2 suspects bacterial meningit is. A cult ure of t he pat ient's spinal fluid shows a gram-negat ive rod.
3

4 What other property could identi fy t he most likely causative bacteria?


5
:
6 A. Aerobe
7
B. Brown colonies growing on MacConkey agar
8
9 C. Lactose fermentation
• 10 0 . Oxidase positive
· 11
E. Urease positive
• 12
• 13
• 14
0
15
. 16
. 17
• 18
0 19
0
20
. 21
. 22
0
23
0 24
0
25
0
26
. 27

8
Lode.
s
Suspe-nd
8
End Bloc:k
Item: 10 of - ,• Mark -<J [:::> "'I ~ · ~
100 ~ P~v1ous N @xt Labl lu es N o tes Calcula t o r
A A
1
The correct answer is C. 6 3°/o chose this.
2
I n children 0-6 months old, group B streptococci, Escherichia coli, and Listeria are the most common
3 bacteria causing bacterial meningitis. Of these three, only E. coli is a gram- negative rod, and it also
4 ferments lactose quickly. Listeria is a gram- positive rod. Group B streptococci are gram-positive cocci and
the most common causative agent, usually because of colonization of the mother's vaginal tract. In general,
5
meningeal symptoms in a young infant are nonspecific and require a high degree of suspicion.
6
The image shows lactose-fermenting (pink) vs. nonlactose-fermenting (brown) colonies on MacConkey agar.
7
Esct,er-d'ua col Q- •egatl e bi:Ktena MacConkey agar Meningitis Meningism Grarn-poS' b e b.actena St-rer•o ot.. .,s Lister• a L.ectose
8
Strt!ptococcus agalact1ae Coccus Bactena Agar Bacterial merungitis
9
10
· 11 Image courtesy of Wikimedia
Commons
• 12
• 13 A is not correct. 7°/o chose this.
• 14 Escherichia coli is a facultative anaerobe, as are Listeria and group B streptococci. True obligate aerobes include Nocardia and Mycobacterium
0
15 tuberculosis.
Facultat1ve anaerobiC organ1sm Mycobactenum tuberculosis Escherichia coli Nocardia Tuberculosis AnaerobiC organ1sm Streptococcus Listeria Streptococcus agalactiae Mycobactenum Aerob1c organ1sm
. 16
Cellular resp~ratlon Obligate aerobe
. 17
• 18 B is not correct. 11 °/o chose thi s .
0 19 Any lactose-fermenting bacteria, such as Escherichia coli, will form pink colonies on MacConkey agar. Bact eri a t hat is not fermenting lactose
will appear clear or brownish on t he agar.
0
20 Escherichia col1 MacConkey agar Lactose Bacteria Agar Fermentation Fermentation in food processing Ethanol fermentation
. 21
D is not correct. 1 1 °/o chose this .
. 22
An oxidase test would be used to distinguish between gram-negative rods that are lactose nonferment ers. Oxidase negat ive organisms include
23
Shigella, Salmonella, and Proteus species, whereas oxidase-positive gram- negative rods include Pseudomonas and Helicobacter pylori (see
0

0 24 flow chart below from First Aid for a complete descri ption). Because Escherichia coli is a lactose fermenter, it will grow pink colonies on
0
25 MacConkey agar.
Ox1dl'<e t .. t Elich •ch•a colo Hehcob..cter pylon Gram-negative bactena Shogella MacConkey agar Pseudomon.,. Proteu• (bacter1um) Lactose Salmonella Oxidase Agar Industnal fermentation
0
26
. 27 E is not correct. 8°/o chose this .
8
Lode.
s
Suspe-nd
8
End Bloc:k
Item: 10 of ~ ,• Mark <:::1 t::> ""I ~· ~'j
100 J.. Previous Next LAb faiUI~S Notes Calculator

1 E is not correct. 8 °/o chos e this.


2 Escherichia coli is a gram-negat ive rod t hat is not urease posit ive . Klebsiella is a gram-negat ive, urease-positive rod that ferments lactose .
3
However, Klebsiella is associat ed with pneumonia and urinary t ract infections, not meningit is. Proteus and Ureaplasma urealyticum are other
important urease -posit ive bact eri a.
4 Escherichia coli Urease Gram-negative bacteria Ureaplasma urealyticum Meningitis Pneumonia Lactose Klebsiella Ureaplasma Proteus (bacterium) Urinary tract infection Bacteria Urinary system
5
6
Bottom Line:
7
8
The most common causes of meningitis in newborns are group B streptococci, Escherichia coli, and Listeria, of which E. coli is a gram-
negative lactose fermenter.
9 Escherichia coli Meningitis Gram-negative bacteria Lactose Streptococcus Listeria Streptococcus agalactiae I ndustrial fermentation

10
· 11
• 12 i@l;fil·1i•J for year:[2017 • J
FIRST AID FACTS
• 13
• 14
FA17 p 176.1
• 15
Common causes of meningitis
. 16
. 17
NEWBORN (0- 6MOl CHILDREN (6 M0- 6 YR) 6- 60YR 60YR+

• 18 Group B streptococci S pneumoniae S pneumoniae S pneumoniae


• 19 E coli N meningitic/is N meningitic/is (111 in teens) Gram 8 rods
• 20 Listeria H influenzae type B Enterovi ruses Listeria
. 21 Enterovi ruses HSV
. 22
Give ceftriaxone and vancomycin empirically (add ampicill in if Listeria is suspected).
• 23
Viral causes of meningitis: enteroviruses (especially coxsackievirus), HSV-2 (HSV-1 = encephalitis), HIV, West ile virus (also
• 24
causes encephalitis), VZV.
• 25
In HIV: Cryptococcus spp.
• 26
. 27
ote: Incidence of H influenzae meningitis has l greatly due to conjugate H inflt~enzae vaccinations. Today, cases are usually
8
Lock
s
Suspend
0
End Block
Item: 10 of - ,• Mark -<J [:::> "'I ~ · ~
100 ~ P~v1ous N@xt Labl lu es No tes Calcula t o r

A A
1
FA17 p 137.2
2
3 Gram-negative lab algorithm
4
5
Gram e (ptnk)
6
7
DiplocoCCI Coccobacim Comma-shaped rods
8
9
10
· 11 Aerobic HMmOphilus innwnne Oxidase '±'
(reqUires factors Vand X)
• 12 Pasteurella
Maltose utilization
• 13 Brucella
• 14
Elorrktella perlussis Grows in 42•c Grows in alkaline media Produces urease
Francisella tularensis
0
15
Ngonorrh~H Campyloblcterjejuni Vibrio cholerile Helicoblcter pylori
. 16 N meningitidis
Moraxelu
. 17
• 18
0 19
0
20
Bacilli
. 21
. 22 Lactose fermentation
0
23
0 24
0
25 Oxidase Fast Slow
0
26
. 27

8
Lode.
s
Suspe-nd
8
End Bloc:k
Item: 10 of - ,• Mark -<] 1:> ""'I ~· 1!';:'1
100 ~ Prev1o u s Next Labf a lu es Note s Calculator

1
FA17 p 123.1
2
Special culture requirements
3
BUG MEDIA USED FOR ISOLATION MEDIA CONTENTS/OTHER
4

5
H influenzae Chocolate agar Factors V ( 1A D +) and X (hematin)
6 N gonorrhoeae, T hayer- 1artin agar Select ivel) favors growth of Neisseria by
7 N meningitidis inhibiting growth of gram$ organisms
8 ,, ith Vancomycin, gram 8 organisms except
1
9 eisseria '' ith Trimethoprim and Colistin,
10 and fungi with :\ystatin
· 11
\'err Typically Cultures :-..·eisseria
• 12
• 13
Bpertussis Bordet-Gengou agar (Bordet for Bordetella) Potato extract
• 14 Regan-Lowe medium C harcoa I, blood, and antibiotic
. 15 C diphtheriae Tellurite agar, LOfAer med ium
. 16
M tuberculosis Lowenstein-Jensen agar
. 17
• 18 M pneumoniae Eaton agar Requires cholesterol
• 19 Lactose-fermenting .MacConkey agar Fermentation produces acid, causing colonies to
• 20 enterics tum pink
. 21
Ecoli Eosin-methylene blue (EM B) agar Colonies with green meta II ic sheen
. 22
• 23 Legionella Charcoal yeast extract agar buffered with
• 24 cysteine and iron
• 25 Fungi Sabouraud agar "Sab's a fun guy!"
• 26
. 27 •
8
L.odt
s
Su~pl'nd
~
End Block
Item: 11 of - ,• Mark -<J [:::> "'I ~ · ~
100 ~ P~v1ous N @xt Labl lues N o tes Calculato r
6
1
A 30-year-old man comes to th e emergency department wit h a fever of 39° C (102.2° F} and a sore t hroat. As shown in the image, physical
2 examination reveals an inflamed oropharynx, enlarged tonsils with exudates, and petechiae on his soft palat e. His WBC count is
3 15,000/mml.
4
5
6
7
8
9
10
· 11
' 12
' 13
' 14
' 15
. 16
. 17
• 18

' 19
Which of the fol lowing statements about t he causative organism is correct?
' 20
:
. 21
A. Sulfonamides are t he dru g class of choice for t his infection
. 22
' 23
B. The causative organism is a facult at ive intracellular bacterium
' 24 C. The causative organism is a gram- positi ve, catalase-negative, a- hemolytic coccus
' 25
D. The causative organism is a nonencapsulated bacterium
' 26
. 27 E. The major virulence factor of the causative organism is M protein

8
Lode.
s
S uspe-nd
8
End Bloc:k
Item: 11 of ~ ,• Mark <:::1 t::> ""I ~· ~'j
100 J.. Previous Next LAb faiUI~S Notes Calculator

1
2
The correct a nswer is E. 6 1 °/o chose this .
An inflamed oropharynx, lymphadenopathy, and white exudates on enlarged tonsils are indicat ive of "strep throat, " caused by Streptococcus
3
pyogenes. Not e the inflammation of t he oropha rynx and the pet echiae, or small red spots, on t he soft palat e in t he v ignette image. M protein
4 is the maj or virulence fact or of S. pyogenes. It inhibits t he act ivat ion of t he complement syst em and prot ects t he organism f rom
5 phagocytosis. B lymphocytes are able to recog nize t his protein and produce antibod ies t hat opson ize t he bact erium and dest roy it . These
ant ibod ies against M prot ein are able to aid in t he opsonizat ion and destruction of the m icroorganism by macrophages and neut rophils. It is
6
believed t hat cross- reactivity of t hese ant i-M protein ant ibod ies with heart muscle is t he basis of rheumat ic feve r.
7 Microorganism Opsonin Streptococcus pyogenes Rheumatic fever Complement system Phagocytosis Streptococcal pharyngitis Virulence factor Lymphadenopathy Pharynx Petechia Antibody Streptococcus

8 Macrophage Neutrophil B cell M protein (Streptococcus) Protein Virulence Inflammation Lymphocyte Tonsil Cross-reactivity Bacteria Tonsillitis Organism Soft palate Fever Cardiac muscle Muscle

9
A is not correct . 4°/o chose t his .
10
The t reat ment of choice includes penicillin or eryt hromycin fo r penicillin-sensit ive ind ividua ls. Sulfonamides and t et racycl ine should not be
11 used for group A 13-hemolytic pharyngit is. Sulfonam ides are useful in t he t reatment of uri nary t ract infect ions and Pneumocystis pneumon ia
• 12 and toxoplasmosis prophy laxis .
Erythromycin Toxoplasmosis Tetracycline Penicillin Pharyngitis Pneumonia Sulfonamide (medicine) Urinary tract infection Urinary system Preventive healthcare Pneumocystis pneumonia
• 13
• 14 B is not correct. 5% chose t his .
• 15 Mycobacterium, Brucella, Listeria, Salmonella, Legionella, Francisella, and Yersinia are facu ltat ive int race llular organ isms .
Legionella Brucella Listeria Mycobacterium Salmonella Yersinia Francisella Intracellular
. 16
. 17 C is not correct. 2 1% chos e this .
• 18 Streptococcus pyogenes, also known as group A streptococcus (GAS) based on it s Lancefield grou ping, is a gram- posit ive, cat alase- negative,
13-hemolyt ic coccus. S. pyogenes (GAS) is the causat ive organism of "st rep throat. " However, Streptococcus pneumoniae is an a-hemoloytic
• 19
coccus, which is not associated wit h strep throat. It is associat ed with a variet y of other infect ions such as pneumonia, otit is med ia,
• 20 men ingit is, et c .
Streptococcus pyogenes Streptococcus pneumoniae Otitis media Streptococcal pharyngitis Lancefield grouping Gram-positive bacteria Meningitis Streptococcus Pneumonia Coccus Otitis Catalase Organism
. 21
. 22 D is not correct. 9°/o chose this .
• 23 Streptococcus pyogenes is an encapsulat ed organ ism, primarily f rom hyaluronic acid. Ot her organ isms wit h capsules include Streptococcus
• 24 pneumoniae, Salmonella, and Klebsiella .
Streptococcus pyogenes Streptococcus pneumoniae Hyaluronic acid Streptococcus Salmonella Klebsiella Organism
• 25
• 26
. 27 Bottom Line :
8
Lock Suspend
s 0
End Block
Item: 11 of - ,• Mark -<J [:::> "'I ~ · ~
100 ~ P~v1ous N @xt Labl lues N o tes Calculato r

1 •
Bottom Li ne:
2
3
An inflamed oropharynx, lym phadenopathy, and enlarged tonsils with exudates are indicative of strep throat, caused by Streptococcus
pyogenes. This organism's main virulence factor is the M protein, which protects it from the host's immune system.
4 Streptococcus pyogenes V1rulence factor Streptococcal pharyngitis Lymphadenopathy Pharynx Streptococcus Immune ;..ystem Protein Virulence M protein (Streptococcus) Tonsil Tonsllht1s Organ1sm

5
6
7 l@l ;f.ii·11•J for year : 2017 •
FI RST A ID FACTS
8
9
FA17 p 132.4
10
11 Streptococcus Gram E£) cocci in chains · . Group strep J¥ l\ ES (major criteria for acute rheumatic
' 12 pyogenes (group A cause: fc,er):
' 13 streptococci) • Pyogenic-pharyngitis, cellulitis, impetigo Joints-polyarthritis
' 14 (" honey-crusted" lesions), erysipelas • -cmditis
' 15 • Toxigenic- scarlet fever, toxic shock- like ~od ules (subcutaneous)
. 16 syndrome, necrotizing fasciitis Erythema marginatum
. 17
• Immunologic- rheumatic fever, Sydenham chorea
• 18
glomcruloneph ritis Pharyngitis can result in rheumatic "phcver"
' 19
Bacitracin sensitive, ~-hemolytic, pyrrolidonyl and glomerulonephritis.
' 20
arylamidase (PYR) <±>. Hya luronic acid caps1tlc Impetigo usually precedes glomeruloncphril is.
. 21
inhibits phagocytosis. An tibodies toM protein Scarlet fever-blanching, sandpaper-like body
. 22
enhance host defenses against S /))'Ogelles but n1sh, stra" berry tongue, and ci rcumora l
' 23
can give rise to rheumatic fe,·er. pallor in the setting of group A streptococcal
' 24
ASO titer or anti-D ase B antibodies indicate pharyngitis (erythrogenic toxin$).
' 25
' 26
recent S pyogenes infection.
. 27

8
Lode.
s
S uspe-nd
8
End Bloc:k
Item: 11 of
100
- ,•
.1 Ma
ric <J [:::> "'I
A

·-
~
P~v•ous N @xt LabValues N o tes Calculato r

1 FA17 p 130.1 •
2
Gram-positive Jab algorithm
3

4
Gram<±> (purple/blue)
5
6
7
Branching
Bacllti Cocci
8 filaments
9
10
Aerobic Anaerobic Anaerobic/facultabve Aerobic Anaerobic
11

' 12
List em Nocardia Actinomyces
' 13 Bacillus Qostridium
(weakly add fast) (not acid fast)
' 14 Corynebacterium Propion1bactenum
' 15 Catalase
_l
. 16
I ---<8>-- -
. 17 Streptococcus Staphylococcus
• 18 T.
Hemolys1s Coagulase
' 19
' 20
j_
(Partial (Complete (
. 21
. 22
a hemolysis.
green)
hemolysis,
clear)
y (No hemolysis,
grows in bile)
Saureus
' 23 Novobiocin
' 24 Optochin sensitivity Bacitracin sensitivity Growth in 6.5% NaCl sensitivity
' 25 and bile solubility
' 26
. 27

8
Loclt
s
S uspe-nd
8
End Bloc:k
Item: 11 of - ,• Mark -<J [:::> "'I ~ · ~
100 ~ P~v1ous N @xt Labl lues N o tes Calculato r
A A
1
Aerobic Anaerobic Anaerobic/facultative Aerobic Anaerobic
2
3 Usteria
Nocardia Adinomyces
4 Bacillus Clostridium
(weakly ac1d fasO (not acid fast)
Corynebacterium Propion•bactenum
5
6
Catala.se
7
---1er---
8 Streptococcus Staphylococcus
9
Hemolysis Coagula.se
10
11
(Partial (Complete
' 12 (No hemolysis.
a, hemolysis. hemolysis,
grows in bile)
' 13 green) clear) Saureus
' 14 Novobiocin
' 15 Oplochin sensitivity Bacitracin sensitivity Growth in 6.5% NaCI
sensitivity
. 16 and bile solubility
. 17
• 18
Group B Group A S saprophyticus Sepidermidis
' 19
S agalactiae Spyogenes
' 20
. 21
. 22
Vindans streptococci Group D
' 23 (no capsule) S pneumoniae Nonenterococcus (enterococcus)
Smutans (encapsulated) SboVIS Efaecium
' 24
S mitis Efaecalis
' 25
' 26 Important tests are 1n bold. Important pathogens are in bold italics.
Note: Enterococcus IS either a· or y-hemolytic.
. 27 •
8
Lode.
s
S uspe-nd
8
End Bloc:k
Item: 12 of - ,• Mark -<J
P~v1ous
[:::> "'I ~ ·· ~
100 ~ N @xt Labl lues N o tes Calculato r

1
A 35-year-old man comes t o th e emergency department after stepping on a rusty nail in his backyard . He has not received any
2
immunizations since he was 12 years old, but he states th at he "had all the regu lar childhood shots."
3

4 If left untreated, which of t he following is t he patient most likely to develop?


5
:
6 A. Crepitus
7
B. Flaccid paralysis
8
9 C. Tenesmus
10 0 . Trismus
11
E. Watery diarrhea
• 12
• 13
• 14

• 15
. 16
. 17
• 18
• 19
• 20
. 21
. 22
. 23
• 24
• 25
• 26
. 27

8
Lode.
s
S uspe-nd
8
End Bloc:k
Item: 12 of ~ ,• Mark <::J [:::> ""I ~· ~'j
100 J.. Previous Next LAb faiUI~S Notes Calculator

1
2 The correct answer is 0. 57°/o chose this.
3 Assuming that t his man received an init ia l course of immunizat ions against t et anus, he must receive a t et anus boost er shot following t his
4
punct ure wound (tetanus booster shots are good fo r only 10 years) . Tetanus is a disease caused by an exotoxin (t et anospasmin) re leased by
Clostridium tetani. This t oxin causes sustained cont raction of skelet al muscl es (tetany). When t his occurs in t he muscles of th e j aw, a
5 condit ion called t rismus (or lockjaw) results.
6 Clostridium tetani Trismus Tetanus Exotoxin Booster dose Toxin Clostridium Skeletal muscle Penetrating trauma Immunization

7
A is not correct. 6°/o chose this.
8 Crepitus is a grinding or crunching noise t hat can be associat ed with Clostridium perfringens infect ion (also known as "gas gangrene"). The
9 organism creates pockets of gas in skin, muscle, and subcutaneous t issues t hat can be released when the skin is palpated. Trau mat ic gas
gangrene is associated wit h large, penetrating wounds t hat compromise vascular suppl y (eg, gun shot wounds); it would be unlikely t o occur
10
following t his pat ient's inj ury.
11 Clostridium perfringens Gas gangrene Crepitus Gangrene Subcutaneous tissue Clostridium Subcutaneous injection Palpation Blood vessel Organism Infection Ballistic trauma

12
B is not correct. 8% chose this .
• 13
Flaccid paralysis is characterist ic of infect ion wit h Clostridium botulinum . Botulinu m t oxin prevents the re lease of acetylcholine f rom motor end
• 14 plat es, causing a f laccid paralysis.
• 15 Clostridium botulinum Botulinum toxin Flaccid paralysis Acetylcholine Paralysis Toxin Clostridium Infection

. 16 C is not correct. 26% chos e this.


. 17 Tenesmus is t he sensat ion of t he need to empty t he bowel accompanied by pain and crampi ng, although there is little stool to pass. I t is
• 18 usually associated wit h inf lammatory (eit her infectious or noninfectious) diseases of t he bowel.
Rectal tenesmus Gastrointestinal tract
• 19
• 20 E is not correct. 3°/o chose this .
. 21 Pseudomembranous colit is is caused by Clostridium difficile, usually aft er a pat ient has been treat ed with broad-spectrum antibiotics for
several days. This organism causes diarrhea, abdominal crampi ng, and fever via t wo exotoxins : toxin A and t oxin B. Treatment of this illness
. 22
includes discont inuat ion of t he original antibiot ic and administration of oral vancomycin or oral or int ravenous met ronidazole .
• 23 Clostridium difficile colitis Metronidazole vancomycin Antibiotics Diarrhea Clostridium difficile (bacteria) Intravenous therapy Colitis Clostridium Toxin Exotoxin Abdominal pain Organism Fever

• 24 Broad-spectrum antibiotic

• 25
• 26
Bottom Line:
. 27

8
Lock
s
Suspend
0
End Block
Item: 12 of - ,• Mark -<J
P~v1ous
[:::> "'I ~ ·· ~
100 ~ N @xt Labl lues N o tes Calculato r

1 •
2 Bottom Line:
3 After childhood immunization for t etanus, a booster shot should be administered every 10 years to prevent Clostridium tetani-related trismus
and muscle spasms following trauma.
4
Clostnd1um tet "' Tn5mu:.. Tetanu~ Immumzation Spasm Clostridium Booster dose Muscle
5
6
7 l@l ;fii·11•J for year: 2017 •
FIRST AIO FACTS
8
9
FA17 p 134.2
10
11 Clostridia (with Gram<±>, spore-forming, obligate anaerobic rods.
12 exotoxins)
• 13
Ctetani Produces tetanospasmin, an exotoxin causing Tetanus is tetanic paralysis.
• 14
tetanus. Tetanus toxin (and botu linum toxin)
• 15
are proteases that clea,·e S ARE proteins for
. 16
. 17
neurotransmitters. Blocks release of inhibitory
• 18
neurotransmitters, GABA and glycine, from
• 19
Renshaw cells in spinal cord.
• 20 Causes spastic paralysis, trismus (lockjaw), risus
. 21 sardonicus (raised eyebrows and open grin),
. 22 opisthotonos (spasms of spinal extensors).
. 23 Pre, enl with tetanus vaccine. Treat \\ ith
• 24 antitoxin +1- ,·accine booster, diazepam (for
• 25 muscle spasms), and wound debridement.
• 26
Cbotulinum Produces a heat-labile toxin that inhibits Symptoms of botulism (the 4 D's): Diplopia,
. 27

8
Lode.
s
S uspe-nd
8
End Bloc:k
Item: 12 of - ,• Mark -<] 1:> ""'I ~· 1!';:'1
100 ~ Prev1o u s Next Labf a lu es Notes Calculator

1 • •
Cbotulinum Produces a heat-labile toxin that inhibits Symptoms of botulism (the 4 D's): Diplopia,
2
ACh release at the neuromuscular junction, Dysarthria, Dysphagia, Dyspnea.
3
4
causing botulism. In adults, disease is caused Botulinum is from bad bottles of food, juice, and
5
by ingestion of preformed toxin. In babies, honey (causes a descending flaccid paralysis).
6
ingestion of spores (eg, in honey) leads to Local botox injections used to treat focal
7 disease (flopp~ baby syndrome). Treat" ith dystonia, achalasia, and muscle spasms. Also
8 antitoxin. used for cosmetic reduction of fa cial wrinkles.
9 Cperfringens Produces a toxin (lecithinase, a phospholipase) Perfringens perforates a gangrenous leg.
10 that can cause myonecrosis (gas gangrene )
11 and hemolvsis.
12 -
Spores can sun·i,·e in undercooked food;
• 13
when ingested, bacteria release heat-labile
• 14
enterotoxin - food poisoning.
. 15
. 16
. 17
• 18
• 19
C difflcile Produces 2 toxins. Toxin A, an ent·erotoxin, /)i{{icile causes diarrhea. Treatment:
• 20
binds to brush border of gul and alters Ouid metronidazole or oral vancomycin. For
. 21 secretion. Toxin B, a cytotoxin, disrupts recurrent cases, consider repeating prior
. 22 cytoskeleton via actin depolymerizalion. Both regimen, fidaxomicin, or fecal microbiota
• 23 toxins lead to diarrhea - pseudomembranous transplant.
• 24 zo
colitis [lJ. Often to antibiotic usc, especially
• 25 clindamycin or ampicillin; associated with PPI
• 26 use. Diagnosed br detecting one or both toxins
. 27 ;.,. ....." "I h .. - .... .,.._: ......,. .... ,..~ ..,.._..,.,..._; ......... """ or o •
8
L.odt
s
Su~pl'nd
~
End Block
Item: 13 of - ,• Mark -<J [:::> "'I ~ · ~
100 ~ P~v1ous N @xt Labl lues N o tes Calculato r

1
A 32-year-old man presents to t he emergency department because of shortness of breath and palpitations. He and his fam ily recently
2
immigrated from Peru. X-ray of the chest demonstrates pulmonary congestion. Echocardiography revea ls bivent ricular dilatation with
3 massive cardiac enlargement. Endomyocardial biopsy demonstrates the presence of intracellular protozoan parasites.
4
5 Which vector is most likely responsible for transmitting this patient's disease?
6 :
7 A. Anopheles mosquito
8 B. Ixodes tick
9
C. Phlebotomus sandflies
10
11 0 . Triatoma (reduviid) bug
12 E. Tsetse flies
. 13
• 14

• 15
. 16
. 17
• 18
• 19
• 20
. 21
. 22
. 23
• 24
• 25
• 26
. 27

8
Lode.
s
S uspe-nd
8
End Bloc:k
Item: 13 of - ,• Mark -<J [:::> "'I ~ · ~
100 ~ P~v1ous N @xt Labl lues N o tes Calculato r
A A
1
2 The correct answ er is D. 65°/o chose this.
3 The patient has myocarditis due to Trypanosoma cruzi. This infection, known as Chagas disease, is endemic
4 in vast areas of South America and is transmitted by Triatoma (reduviid) bugs, also known as "kissing
bugs." Resevoirs of the T. cruzi include wild animals such as rodents, raccoons, dogs, and humans. T. cruzi
5
is an intracellular protozoan that localizes mainly in the heart and nerve cells of the myenteric plexus,
6 leading to myocarditis and dysmotility of hollow organs such the esophagus, colon, and ureter. Cardiac
7 involvement manifests with ventricular dilatation and congestive heart failure secondary to myocyte
necrosis and fibrosis. Intracellular parasites can be visualized in tissue sections, as shown in the image.
8
Chagas disease is also a cause of acquired achalasia. This is due to loss of innervation to the lower
9 esophageal sphincter, which prevents relaxation of the sphincter. This, in turn, leads to dilation of the distal
10 third of the esophagus as food is unable to enter the stomach. A similar pathologic mechanism accounts for
megacolon and megaureter in Chagas disease.
11
Chaq d•~e e 1'1' fP no»oMa cruz• M ocyte M ocarditis Esophagus Myenteric plexus Achalas•a Megacolon H~ fa Jure Congesbve heart fa•lure Endemism
12
Colon (anatomy) Trypanosoma South Amenca Protozoa Neuron Fibrosis Sphincter Heart Necroses Infection Paras1t1sm Stomach Reduviidae Cardia
13

' 14 Image courtesy of CDC


' 15
. 16 A is not correct. 8°/o chose this .
. 17
Malaria caused by Plasmodium falciparum is t ransmitted through Anopheles mosquitoes. Malaria can manifest wi th feve r~ headache, anemia,
and splenomegaly. Cardiomyopathy, as seen in this patient, is not a sign of malaria .
• 18 Plasmodium falciparum Splenomegaly Malana Anemia Anopheles Plasmodium Cardiomyopathy Headache Mosqueto Fever

' 19
B is not correct. 9 % chose this.
' 20
Ba besiosis and Lyme disease are both t ransm itt ed through Ixodes t icks. Though Lyme disease may lead to cardit is and AV block, this patient's
. 21 symptoms are more likely caused by Chagas disease .
. 22 Babesiosis Chagas disease Lyme desease Ixodes Atrioventricular block Carditis

' 23 C is not correct . 8°/o chose this.


' 24 Leishmaniasis is transmitted through Phlebotomus sandflies. The visceral form of this cond ition (kala-azar) presents as spiking fevers,
' 25 hepatosplenomegaly, and pancytopenia while t he cutaneous form is associated with skin ulceration.
Pancytopen11 Phle otomus Hep<ttosplenomegaly Leoshmamasis Sandfly Phlebotommae Ulcer (der~ology)
' 26
. 27 E is not correct . 10°/o chose this .
8
Lode.
s
S uspe-nd
8
End Bloc:k
Item: 13 of ~ ,• Mark <::J [:::> ""I ~· ~'j
100 J.. Previous Next LAbfaiUI~S Notes Calculator

1 E is not correct. 10°/o chos e this.


2 Af rica n t ry panosomiasis caused by T1ypanosoma brucei is t ransmit ted by tsetse f lies genus Glossina. This cond ition can cause
3 lympha denopathy, recurring fevers due t o ant igenic variation, somnolence, and even coma .
African trypanosomiasis Trypanosoma brucei Tsetse fly Lymphadenopathy Antigenic variation Trypanosomiasis Trypanosoma Genus Coma
4
5
6 Bottom Line:
7 Chagas disease, which manifests with myocarditis, achalasia, megacolon, and megauret er, is t ransmitted by t he Triatoma (reduviid) bug.
Chagas disease Myocarditis Megacolon Achalasia Triatoma Reduviidae
8
9
10
11
141;fil·1i•J for year:[2017
FIRST AID FA CTS .
•j .

12
13 FA17p154.1
• 14 Protozoa- others
• 15 ORGANISM DISEASE TRANSMISSION DIAGNOSIS TREATM ENT
. 16
Visceral infections
. 17
• 18 Trypanosoma Chagas disease- dilated Reduviid bug Trypomastigote in Benznidazole
• 19 cruzi cardiomyopathy with ("kissing bug") blood smear fJ or nifurtimox;
• 20 apical atrophy, megacolon, feces, deposited C n JZing in my
. 21 megaesophagus; predominantly in a painless bite Benz, with a fur
. 22 in South America (much Iike a kiss) coat on
• 23 Unilateral periorbital swelling
• 24 (Romai'i a sign) characteristic of
• 25 acute stage
• 26
. 27 •
Leishmania Visceral leishmaniasis Sand fly Macrophages Amphotericin B, •

8
Lock
s
Suspend
0
End Block
Item: 13 of - ,• Mark -<J [:::> "'I ~ · ~
100 ~ P~v1ous N @xt Labl lues N o tes Calculato r

1 A
Trypanosoma Chagas disease- dilated Reduviid bug Trypomastigote in Benznidazole A

2 cruzi cardiomyopathy with ("kissing bug") blood smear fl or nifu rtimox;


3
apical atrophy, megacolon, feces, deposited C ruzing in my
4
megaesophagus; predominantly in a painless bite Benz, with a fur
5
in South America (much like a li\\) coat on
6
Unilateral periorbital swelling
7
(Romana sign) characteristic of
8
acute stage
9
10 Leishmania Visceral leishmaniasis SandA) ~ lacrophages Amphotericin B,
11 donovani (kala-azar)- spiking fevers, containing sodium
12 hepatosplenomegaly, amastigotes : stibogluconate
13 pancytopenia
' 14 Cutaneous leishmaniasis-skin
' 15
ulcers
. 16
. 17
• 18

' 19 Sexually transmitted infections


' 20
Trichomonas Vaginitis- foul-smelling, green ish Sexual (cannot exist Trophozoitcs Metronidazole for
. 21
. 22
vagina/is discharge; itching and burning; outside human m
(motile) on wet patient and partner
do not confuse with Cardnerella bccallSe it cannot mount; "strawberry (prophylaxis)
' 23
vagina/is, a gram-variable form cysts) cerv1.x"
' 24
' 25
bacterium associated with
' 26
bacterial vaginosis
. 27 •
8
Lode.
s
S uspe-nd
8
End Bloc:k
Item: 13 of - ,• Mark -<] 1:> ""'I ~· 1!';:'1
100 ~ Prev1o u s Next Labf a lu es Note s Calculator

1 • •
2 FA17 p 153 01
3 Protozoa- hematologic infections
4 ORGANISM DISEASE TRANSMISSION DIAGNOSIS TREATMENT
5 Plasmodium Mala ria-fever, headache, anemia, Anopheles mosquito Blood smear: Chloroquine (for
6
P vivax/ovale splenomegaly trophozoite ring sensiti,·e species),
7
P falciparum P ''imx!ol'ale-48-hr cycle (tertian; form" ithin '' hich blocks
8
Pmalariae includes fever on first day and RBC t'J, schizont Plasmodium heme
9
third day, thus fevers are actuall) containing polymerase; if
10
48 hr apart); dormant form merozoites; red resistant. use
11

12
(hypnozoite) in liver granules (Schi.iffner meAoquine or
13
P falciparum-se,·ere; irregular stippling) III ato,·aquone/
• 14 fever patterns; parasitized RBCs throughout RBC proguanil
. 15 occlude capillaries in brain cytoplasm seen If life-threatening,
0 16 (cerebral malaria), kidneys, lungs wi th P vivax/ovale use intravenous
. 17 P malariae-72-hr cycle (quartan) quinidine or
• 18 artesunale (lest fo r
• 19 G6PD deficiency)
• 20 For P vil'(IXIovale,
02 1 add primaquine for
022 hypnozoite (lest for
• 23 G6PD deficiency)
• 24
Babesia Babesiosis- b·er and hemolvtic Ixodes tick (sa me as Blood smear: ring Atovaquone
• 25 '
• 26
anemia; predominantly in Borrelia burgdor{eri form , ".\ laltese + azithromycin
0 27 northeastern United States; of L} me disease; cross" •oPCR •
8
L.odt
s
Su~pl'nd
~
End Block
Item: 13 of - ,• Mark -<] 1:> ""'I ~· 1!';:'1
100 ~ Prev1o u s Next Labf a lu es Note s Calculator

1 • FA17 p 15201

2
Protozoa- CNS infections
3
ORGANISM DISEASE TRANSMISSION DIAGNOSIS TREATMENT
4
5
Toxoplasma Congenital toxoplasmosis = Cysts in meat (most Serology, biopsy Sulfadiazine +
6
gondii classic triad of chorioretinitis, common); OOC} sis (tachyzoite)rn prrimcthamine
7 hydrocephalus, and intracranial in cat feces; crosses
8 calcifications; reacti,·ation in placenta (pregnant
9 AIDS ... brain abscesses usually women should
10 seen as multiple ring-enhancing a' oid cats)
11 lesions on MRI rJ
12
Naegleria fowleri Rapidly fata l meningoencephalitis Swimming in Amoebas in spinal Amphotericin B has
13
fre~>hwater lakes Auid 9 been effecti,oe for a
• 14
(think Nalgcnc few Sllr\'i\'OrS
. 15
bottle fi lied
0 16
0 17 with frc\h wat er
• 18
containing
• 19
Naegleria); enters
• 20
via cribriform plate
02 1 Trypanosoma African sleeping sickness- ' lsetse ny, a pa inful Trypomast igote in Suramin for blood-
022 brucei enlarged lymph nodes, recurring bite blood smear m borne disease or
• 23 fever (due to antigenic variation), mclarsoprol for
• 24 C t S penetration
somnolence, coma
• 25 ("I \urc am
Two subspecies: Trypanosoma
• 26
brucei rhodesiense, Trypanosoma mellow when •
0 27
8
L.odt
s
Su~pl'nd
~
End Block
Item: 13 of - ,• Mark -<] 1:> ""'I ~· 1!';:'1
100 ~ Prev1o u s Next Labf a lu es Note s Calculator

1 •
2
bottle fi lied
3 with frc\h water
4 containing
5 T\aeglcria); enters
6 via cribriform plate
7 Trypanosoma African sleeping sickness- Tsetse n). a painful Tr) pomast igote in Suramin for blood-
8
9
brucei enlarged lymph nodes, recurring bite blood smear m borne disease or
ber (due to antigenic variation), mclarsoprol for
10
somnolence, coma c~s penetration
11
Two subspecies: Tr)'panosoma ("I \urc am
12
brucei rhodesiense, Trypanosoma mellow when
13
• 14
brucei gambiense I'm sleeping";
. 15
remember
. 16
melatonin helps
. 17 with sleep)
• 18
• 19
• 20
. 21
. 22
• 23
• 24
• 25
• 26
a •
. 27

8
L.odt
s
Su~pl'nd
~
End Block
Item: 14of - ,• Mark -<J
P~v1ous
[:::> "'I ~ ·· ~
100 ~ N @xt Labl lues N o tes Calculato r

1
A 35-year-old woman is brought to the emergency department because of diffuse muscle contractions. On examination she is unable to
2
open her mouth. Her husband repo rts that last week she accidentally stuck her finger with a rusty nail.
3

4 The organism that causes these symptoms does so t hrough what mechanism?
5
:
6 A. The organism produces a cyt otoxin that damages colonic mucosa
7
B. The organism produces a heat- labile toxin that stimulates adenylate cyclase
8
9 C. The organism produces an exotoxin that blocks glycine release at spinal synapses
10 D. The toxin produced is a superantigen that binds to MHC II protein and T lymphocyte receptors
11
E. The toxin blocks release of acetylcholine at spinal synapses
12
13

' 14
' 15
. 16
. 17
• 18

' 19
' 20
. 21
. 22
' 23
' 24
' 25
' 26
. 27

8
Lode.
s
S uspe-nd
8
End Bloc:k
Item: 14 of ~ ,• Mark <::J [:::> ""I ~· ~'j
100 J.. Previous Next LAb faiUI~S Notes Calculator

1
2 The correct answer is C. 70°/o chose this.
3 St ro ng muscl e cont ractions and t rism us (contraction of the j aw m uscles) are symptoms of t et anus. Tetanus is caused by Clostridium tetani
4
spores t hat are found in the soil. They usually ent er the body t hrough a punct ure wound. The disease is caused by t he exotoxin produced by
t he bacteriu m . The exotoxin blocks glycine and GABA release, preventing the inhibi tory signal from reaching motor neurons downst ream, thus
5 predisposing motor neurons t o tonic contraction, or tetanus.
6 Clostridium tetani Trismus Tetanus Glycine Gamma-Aminobutyric acid Exotoxin Clostridium Bacteria Muscle Motor neuron

7
A is not correct. 2°/o chose this.
8 Clostridium difficile causes pseudomembranous colit is by producing a cytot oxin (the A-B t oxin) that kills ent erocytes and causes
9 pseudomembranous colitis, not lockjaw.
Clostridium difficile colitis Cytotoxicity Clostridium difficile (bacteria) Tetanus Clostridium Toxin Colitis Enterocyte
10
11 B is not correct. 7% chose this.
12 Escherichia coli produces a heat -labile toxin t hat stimulates adenylat e cyclase by adenosine diphosphate ribosylation of G proteins, which t hen
causes watery diarrhea .
13
Adenosine diphosphate Adenylyl cyclase Diarrhea Adenosine Pyrophosphate G protein Toxin
14
• 15
D is not correct. 2°/o chose this .
Staphylococcus aureus causes disease by producing a t oxin t hat acts as a superant igen and binds to MHC II protein and T lymphocyte
. 16
receptors. This leads to toxic shock charact erized by feve r, rash, and shock .
. 17 T cell Superantigen Staphylococcus aureus Lymphocyte MHC class II Protein Staphylococcus Toxin Toxic shock syndrome Major histocompatibility complex Rash Fever
• 18
E is not correct. 19°/o chose this .
• 19
Clostridium botulinum causes botulism by producing a t oxin t hat blocks t he release of acet ylcholine at spinal synapses and can cause
• 20 ant icholinergic symptoms. Cranial nerves are often t he f irst to be affect ed, but t he paralysis is f laccid rat her t han cont ract ile. Furthermore, t he
. 21 method of t ransmission is often through improperly canned goods and honey ( in babies) rat her t han rusty nails .
Anticholinergic Clostridium botulinum Botulism Acetylcholine Cranial nerves Botulinum toxin Honey Toxin Paralysis Synapse
. 22
• 23
• 24 Bottom Line:
• 25 Clostridium tetani is a spore-forming, gram- posit ive bacillus t hat is t ra nsmitt ed t hrough cont aminat ed soil/feces, whose ent rance into t he
• 26 body is oft en facilitated by puncture wounds from sharp obj ects like nails. It produces an exotoxin that blocks glycine and GABA f rom
inhibit ing muscular cont ract ions, result ing in t onic contraction (tetanus) .
. 27

8
Lock
s
Suspend
0
End Block
Item: 14 of - ,• Mark --<) [::> ""'I ~· 1!';:'1
100 ~ Prev1o u s Next Labf a lu es Notes Calculator

1 • •
FA17 p 134.2
2
3
Clostridia (with G ram EB, spore-forming, obligate anaerobic rods.
4
exotoxins)
5 Ctetani Produces tetanospasmin, an exotoxin causing Tetanus is tet anic paralysis.
6 tetanus. Tetanus toxin (and botulinum toxin)
7 are proteases that clea,·e SNARE proteins for
8 neurotransmitters. Blocks release of inhibitory
9
neurotransmitters, GABA and glrcine, from
10
Renshaw cells in spinal cord.
11
Causes sp astic paralysis, trismus (lockjaw), risus
12
sardonicus (raised eyebrows and open grin),
13
opisthotonos (spasms of spinal extensors).
14
. 15
Pre, en! with tetanus vaccine. Treat'' ith
. 16
antitoxin +1- vaccine booster, diazepam (for
. 17
muscle spasms), and wou nd debridement.
• 18 Cbotulinum Produces a heat-labile toxin that inhibits Symptoms of botulism (the 4 D 's): D iplopia,
• 19 ACh release at the neuromuscular june!ion, O )rsarthria, D ysphagia, D yspnea .
• 20 causi ng botulism. In adults, disease is caused Botulinum is from bad bottles of food , ju ice, and
. 21 by ingestion of preformed toxin. In babies, honey (causes a descending flaccid paralysis) .
. 22 Local botox injections used to treat foca l
ingestion of spores (eg, in honey) leads to
• 23
disease (flopp~ baby synd rome). Treat with dystonia, achalasia, and muscle spasms. Also
• 24
antitoxin . used for cosmetic reduction offacial wrinkles.
• 25
• 26
. 27
C perfringens
.. Produces a toxin (lecithinase, a phospholipase)
that can cause m\·onecrosis (gas gangrene )
Perfringens perforates a gangrenous leg.

8
L.odt
s
Su~pl'nd
~
End Block
Item: 14 of - ,• Mark --<) [::> ""'I ~· 1!';:'1
100 ~ Prev1o u s Next Labf a lu es Notes Calculator

1 • Cbotulinum Produces a heat-labile toxin that inhibits Symptoms of botulism (the 4 D's): Diplopia, •
2 AC h release at the neuromuscular junction, Dysarthria, Dysphagia, Dyspnea.
3 causing botulism. In adults, disease is caused Botulinum is from bad bottles of food, juice, and
4
by ingestion of preformed toxin. In babies, honey {causes a descending flaccid paralysis).
5
ingestion of spores {eg, in honey) leads to Local botox injections used to treat focal
6
disease (Aopp~ baby syndrome). Treat " ith dystonia, achalasia, and muscle spasms. Also
7
antitoxin. used for cosmetic reduction of fa cial wrinkles.
8
9 Cperfringens Produces a toxin (lecithinase, a phospholipase) Perfringens perforates a gangrenous leg.
10 that can cause myonecrosis (gas gangrene )
11 and hemolysis.
12 Spores can sun·i,·e in undercookcd food;
13 when ingested, bacteria release heat-labile
14 enterotoxin ..... food poisoning.
. 15
. 16
. 17
• 18 /)i{{icile causes diarrhea. Treatment:
C difficile Produces 2 toxins. Toxin A, an enterotoxin,
• 19
binds to brush border of gut and alters Auid metronidazole or oral vancomycin. For
• 20
secretion. Toxin B, a cytotoxin, disrupts recurrent cases, consider repeat ing prior
. 21
cytoskeleton via actin depolymerization. Both regimen, fidaxomicin, or fecal microbiota
. 22
toxins lead to diarrhea ..... pseudomembranous transplant.
• 23
• 24
zo
colitis (lJ. Often to antibiotic usc, especially
• 25
clindamycin or ampicillin; associated with PPI
• 26
use. Diagnosed by detecting one or both toxins
. 27 in stool b,· antigen detection or PCR. •
8
L.odt
s
Su~pl'nd
~
End Block
Item: lS of - ,• Mark -<J [:::> "'I ~ · ~
100 ~ P~v1ous N @xt Labl lues N o tes Calculato r

1
A 12-year-old boy wit h sickl e cell disease presents to th e emergency department with complaints of severe pain in t he area of his right
2
humerus. His temperat ure is 37. 1oc (98 .8° F), blood pressure is 100/60 mm Hg, pulse is 89/min, and respi ratory rate is 22/min. Physical
3 examination shows no other abnormalit ies. A radiogra ph is obtained that shows lytic changes and periosteal elevation in t he middle and
4 distal humeral shaft.
5
6
What is the most likely pathogen responsibl e for t his patient's condition?
7 :
A. Staphylococcus aureus
8
9 B. Candida albicans
10 C. Mycobacterium tuberculosis
11
D. Neisseria gonorrhoeae
12
13 E. Pasteurella multocida
14 F. Pseudomonas aeruginosa
. 15
G. Salmonella
. 16
. 17 H . Staphylococcus epidermidis
• 18
• 19
• 20
. 21
. 22
. 23
• 24
• 25
• 26
. 27

8
Lode.
s
S uspe-nd
8
End Bloc:k
Item: lS of ~ ,• Mark <::J [:::> ""I ~· ~'j
100 J.. Previous Next LAb faiUI~S Notes Calculator

1
The correct a nswer is G. 66°/o chos e this .
2
Salmonella is the most common cause of osteomyelit is in pat ients wit h sickle cell disease. Sickle cell disease refers to any combinat ion of Hb
3 t hat includes HbS, whereas sickle cell anemia classically refers t o homozygous HbS, and sickle cell t rait refers to heterozygous HbS. The
4 biochemical abnormality responsible for t his condit ion occurs when glutamic acid, t he 6th amino acid on the 13-globin peptide chain, is
replaced by valine (an E6V missense m utation). Therefore patients creat e HbS inst ead of HbA. At low oxygen levels, HbS agg regat es and
5
distorts re d blood cells into a sickle shape. I n t he United States, 36 in 500 African Americans carry the sickle cell t rait and 1 in 365 African
6 Americans have sickle cell disease, ma king t his ethnic group at highest risk for sickle cell disease. Historically, hetero zygot es fo r HbS are
7 believed t o carry a survival advantage with protect ion from malari a. Pat ients wit h sickle cell anemia often suffer from aplast ic cri sis, splenic
sequestrat ion cri sis, acut e chest syndrome, and osteomyelit is. Aplastic anemia often arises from infection wit h parvovirus B19 (fift h disease) .
8
Children with t his disease are also very suscept ible to infect ion from encapsulated bacteria, as t heir spleen is often nonfunctional after t he age
9 of 4 years. Other common pathogens which cause ost eomyelit is in sickle cell pat ients include Escherichia coli and Staphylococcus auerus.
Sickle-cell disease Missense mutation Osteomyelitis Escherichia coli Parvovirus 819 Sickle cell trait Aplastic anemia Homozygous Amino acid Glutamic acid Valine Malaria Heterozygous Salmonella
10
Red blood cell Zygosity Fifth disease Spleen Anemia Mutation Reticulocytopenia Polysaccharide encapsulated bacteria Acute chest syndrome Peptide Bacteria Parvovirus Pathogen Oxygen United States
11

12 Staphylococcus Sickle

13 A is not correct . 17% chose this.


14 Staphylococcus au reus is t he most common cause of osteomyelit is in t he general populaton but only t he second most common cause of
15 ost eomyelit is in sickle cell pat ients, account ing for a quarter of cases. Salmonella is t he most common cause of ost eomyelit is in sickle cell
patients. S. aureus is a gram- posit ive coccus oft en responsible for su perficial skin infections and also causing joint infect ions after orthopedic
. 16
procedures. S. aureus is often carried in the nares or peri neum of indiviuda ls, somet imes lead ing to autoinoculat ion. The organism can be
. 17 sensit ive to met hicillin or resistant t o met hicillin, making laborat ory sensit ivit ies often essent ial to it s successful t reatment .
• 18 Osteomyelitis Staphylococcus aureus Gram-positive bacteria Staphylococcus Perineum Salmonella Nostril Meticillin Coccus

• 19 B is not correct. 3% chose t his .


• 20 Candida albicans should be suspected in int rave nous drug users who present wit h ost eomyelit is. Candida is t he most common fungal
. 21 pathogen affect ing humans. It is a fungus that can fo rm pseudohyphae and t rue hyphae. One of the most common manifest at ions of Candida
infection is as thrush, a creamy white oral plaque, in immunocom promised pat ient s. I t is also responsible fo r vulvovaginit is in fema les, often
. 22
descri bed as having a "cotta ge-cheese" consistency. Osteomyelitis due to Candida is rare in t he general population; therefore, drug use should
• 23 be suspected if a pat ient present s with ost eomyelitis due t o Candida .
• 24 Candida albicans Osteo,.,elitis Hypha Immunodeficiency Intravenous therapy Candida (fungus) Pathogen Vaginitis Fungus Drug injection

• 25 C is not correct. 2 °/o chose this .


• 26 Mycobacterium tuberculosis usually causes osteomyelit is f rom hemat ogenous spread in immunocompromised pat ients or in areas of endemic
. 27 infection . Add it ionally, t uberculosis usually involves t he vertebral bodies (Pott disease), not the humerus. I nfect ion occurs by inhalinq

8
Lock
s
Suspend
0
End Block
Item: lS of ~ ,• Mark <::J [:::> ""I ~· ~'j
100 J.. Previous Next LAb faiUI~S Notes Calculator
• • w - .. w • • • • • • • • • • • • • • • • • .. • • •

1
2 C is not correct. 2 °/o chose this .
3
Mycobacterium tuberculosis usually causes osteomyelit is f rom hemat ogenous spread in immunocompromised patients or in areas of endemic
infection . Add it ionally, t uberculosis usually involves t he vertebral bodies (Pott disease), not the humerus. I nfect ion occurs by inhaling
4 aerosolized pa1t icl es of M. tuberculosis from an infect ed person. As this organism is slow growing, it can remain ine1t for many years before
5 causing sympt oms. Pott disease, also known as tuberculous spondylit is, is ra re in developed count ries, where ant ituberculous drugs are
6
readily available. Patient s present with a combination of osteomyelit is and arthrit is involving more than one vertebra. They often have a
hist ory of unt reated tuberculosis infect ion and complain of back pain for several weeks.
7 Osteomyelitis Mycobacterium tuberculosis Pott disease Vertebra Tuberculosis Humerus Mycobacterium Immunodeficiency Arthritis Tuberculosis management Spondylitis Bacteremia Infection Endemism
8 Back pain Developed country
9
D is not correct. 2°/o chose t his.
10
Neisseria gonorrhoeae is a rare cause of ost eomyelit is in sexually act ive people, but is more likely t o result in septic art hrit is. This gram -
11 negative, aerobic diplococcus can lead t o pelvic inflammat ory disease in females if left unt reated, leading to infertility and tubal scarring .
12 Other serious consequences include Fitz-Hugh and Curtis syndrome, in which the infect ion spreads from the reproductive organs t o t he liver
13
capsule, causing acut e perihepatit is.
Pelvic inflammatory disease Osteomyelitis Neisseria gonorrhoeae Septic arthritis Gram-negative bacteria Diplococcus Neisseria Liver Aerobic organism Arthritis Infertility Perihepatitis Inflammation
14
15 E is not correct. 3 °/o chose this .
. 16
Pasteurella multocida is a cause of osteomyelit is in persons with a hist ory of cat and dog bit es or scratches. This gram- negative, non-spore-
forming bact erial species commonly reside in domestic pet species, t ransferring to humans with an anima l bite or scratch. Locally, t he
. 17 infection causes cellu lit is or abscesses t o develop in t he area of injury. If unt reated, it can become blood-born e, and infect ed persons can
• 18 develop ost eomyelit is or septic arthri t is. These com plicat ions are more likely in cat bites, as cat bi tes tend to be smaller and more frequent ly
• 19 go unnot iced .
Osteomyelitis Pasteurella multocida Septic arthritis Gram-negative bacteria Cellulitis Pasteurella Abscess Arthritis
• 20
. 21 F is not correct . 6°/o chose this .
. 22
I nfection by Pseudomonas aeruginosa, like Candida spp., should be suspected in int ravenous drug users who present with osteomyelitis. This
gram-negat ive rod classically produces a blue-green pigment wit h a sweet odor. Pseudomonas is an opportunistic pat hogen seen in
• 23 intravenous drug users, immunocompromised persons, and in burn wound sepsis. Ost eomyelit is due t o Pseudomonas can be blood -borne,
• 24 f rom a systemic infection, or it may be cont iguous, spread f rom penetrating wound t rauma . Persons wit h a history of puncture wounds of t he
• 25
feet are at risk for Pseudomonas infect ion .
Osteomyelitis Pseudomonas aeruginosa Gram-negative bacteria Sepsis Immunodeficiency Pseudomonas Opportunistic infection Pathogen Intravenous therapy Candida (fungus) Pigment Systemic disease
• 26
Drug injection Infection
. 27

8
Lock Suspend
s 0
End Block
Item: lS of ~ ,• Mark <::J [:::> ""I ~· ~'j
100 J.. Previous Next LAb faiUI~S Notes Calculator

1 H is not correct . 1 °/o chose this.


2 Staphylococcus epidermidis, along wit h Staphylococcus aureus, can cause osteomyelit is in pat ients who have undergone prosthet ic
3 replacements. S. epidermidis is gram-posit ive and coagulase-negat ive, whereas S. aureus is gram- posit ive and coagulase- posit ive. S.
epidermidis is found on laborat ory cultures due to skin contamination, where it may reside. However, it is a major cause of infection in the
4
immunocomprom ised and in pat ients wit h indwelling catheters. I ts ability to fo rm a biof ilm on prost hetic devices makes it a possible cause of
5 infection in those wit h joint replacement s.
6 Biofilm Osteo,.,elitis Staphylococcus epidermidis Staphylococcus aureus Gram- positive bacteria Immunodeficiency Staphylococcus Catheter Prosthesis Infection

7
8 Bottom Line :
9
I n patient s with sickle cell anemia, consider both Salmonella and Staphylococcus aureus as causes for osteomyelit is. The lyt ic bone changes
10 and periost ea l elevation signifies an infect ion of bone and t herefore osteomyelit is.
Osteomyelitis Sickle- cell disease Staphylococcus aureus Anemia Staphylococcus Salmonella Lysis Lytic cycle Bone
11

12
13
14
l@);fil ~1hl for yea r:l 2o17 y
FIRST AID FACTS

15
. 16 FA17 p 176.4
. 17
Osteomyelitis RISK FACTOR ASSOCIATED INFECTION
• 18
• 19 Assume if no other information is available S aureus (most common overall)
• 20 Sexually active Neisseria gonorrhoeae (rare), septic arthritis more
. 21 common
. 22
Sickle cell disease Scdmonella and S aureus
• 23
• 24 Prosthetic joint replacement S aureus and S epidennidis
• 25 Vertebral involvement S aureus, Mycobacterium tuberculosis (Pott
• 26 disease)
. 27

8
Lock
s
Suspend
0
End Block
Item: lS of - ,• Mark -<J [:::> "'I ~ · ~
100 ~ P~v1ous N @xt Labl lues N o tes Calculato r
A A
1
2
3 l@l : fil ~11•1 fo r yea r: 2017 •
FI RST AIO FACTS
4
5
FA17 p 176.4
6
7
Osteomyelitis RISK FACTOR ASSOCIATED INFECTION

8 Assume if no other information is a,·ailable S aureus (most common o,·erall)


9 Sexually active I eisseria gonorrhoeae (rare), septic arthritis more
10 common
11

12
Sickle cell disease Salmonella and S aureus
13 Prosthetic joint replacement S aureus and S epidermidis
14
Vertebral involvement S aureus, J\lycobacterium tuberculosis (Pott
15
disease)
. 16
. 17
Cat and clog bites Pasteurella multocida
• 18 IV drug abuse Pseudomonas, Candida, S aureus are most
• 19 common
• 20
Elevated C-reactive protein (CRP) and erythrocyte sedimentation rate common but nonspecific.
. 21
M Rl is best for detecl'ing acute infection and detailing anatomic involvement f.i.l. Radiographs are
. 22
insensiti\·e early but can be useful in chronic OSleOill)'e litis (il.
. 23
• 24
• 25 FA1 7 p400.1
• 26 Intrinsic hemolytic anemia
. 27

8
Lode.
s
S uspe-nd
8
End Bloc:k
Item: lS of - ,• Mark -<J [:::> "'I ~ · ~
100 ~ P~v1ous N@xt Labl lues No tes Calculato r

A
1 A

FA17 p400.1
2
Intrinsic hemolytic anemia
3
DESCRIPTION f iNDINGS
4
5
Hereditary Extravascular hemolysis due to defect in Splenomegaly, aplastic crisis {parvo,·irus 1319
6 spherocytosis proteins interacting with RBC membrane infection).
7 skeleton and plasma membrane (eg, ankyrin, Labs: osmotic fragility test®. i\'onnal to
8 band 3, protein 4.2, spectrin). \ lostly ~ \ ICV with abundance of cells.
9 autosomal dominant inheritance. Treatment: splenectomy.
10 Results in sma ll, round RBCs with less surface
11 area and no central pallor {T i\ ICIIC)
12 ..... premature removal by spleen.
13
G6PD deficiency Most common enzymatic disorder of RBCs. Back pain, hemoglobinuria a few days after
14
Causes extravascular and intravascular oxida nt \trc'>s.
15
hemolysis. X-linked recessive. Labs: blood smear shows RBCs with Heinz
. 16
. 17
Defect in C6PD ..... ~ glutathi on e ..... T RBC bodies and bite cel ls.
• 18
susceptibility to oxidant stress. Hemolytic "Strcl>s makes me eat bites of fava beans with
• 19 anemia following oxidant stress (eg, sul fa I leinz ketchup."
• 20 drugs, antimalarials, infections, fava beam).
. 21 Pyruvate kinase Autosomal recessive pyruvate kinase defect llemolytic anemia in a newborn.
. 22 deficiency ..... ~ ATP ..... rigid RBCs ..... extravascular
. 23
hemolysis. Increases levels of 2,3-BPC
• 24
..... ~ hemoglobin affinity for 0, .
• 25
• 26
Paroxysmal nocturnal T complement-mediated intra,·aseular RBC Associated with aplastic anemia.
. 27 • hemoglobinuria lrsis (impaired S\·nthesis of CPI anchor Triad: Coombs 8 hemoh·tic anemia,
8
Lode.
s
S uspe-nd
8
End Bloc:k
Item: lS of - ,• Mark -<J [:::> "'I ~ · ~
100 ~ P~v1ous N @xt Labl lues N o tes Calculato r
A A
1
Paroxysmal nocturnal t complement-mediated intravascular RI3C Associated with aplastic anemia.
2
hemoglobinuria lysis (impaired synthesis of C PI anchor Triad: Coombs 8 hemolytic anemia,
3
for decay-accelerating factor that protects pancytopenia, and venous thrombosis.
4
RBC membrane from complement). Labs: C0))/59 8 RBCs on Aow cytometry.
5
Acquired mutation in a hematopoietic Treatment: eculizumab (terminal complement
6
7
stem cell. t incidence of acute leukemias. inhibitor).
8
Patients may report red or pin k uri ne (from
9 hemoglobinuria).
10 Sickle cell anemia ll bS point mutation causes a single amino Complications in sickle cell disease:
11 acid replacement in~ chain (substitution plastic crisis (due to pan·o,·irus 819).
12 of glutamic acid with valine). Causes Autosplenectomy (!lowell-jolly bodies)
13
extravascular and intra,·ascular hemolysis. - t risk of infection by encapsulated
14
Pathogenesis: low 0 2, high altitude, or acidosis organisms (eg, S pneumoniae).
15
precipitates sickling (deoxygenated I lbS Splenic infarct/sequestration crisis.
. 16 • polymerizes) - anemia, vaso-occlusive disease. Salmonellct osteomyelitis.
. 17
ewborns are initially asymptomatic because of Painful crises {vaso-occlusive): dactylitis
• 18
• 19
- t HbF and ! HbS. (pai nful swelling of hands/feet), priapism,
• 20
Heterozygotes (sickle cell trait) also have acute chest syndrome, avascular necrosis,
. 21
resistance to malaria. stroke.
. 22 8% of African Americans carr)' an HbS allele. Renal papi llary necrosis (l Po2 in papilla)
. 23 Sickle cells are crescent-shaped RBCs fJ. and microhematuria (medullary infarcts).
• 24 "Crew cut" on skull x-rav due to marrow Diagnosis: hemoglobin electrophoresis.
'
• 25 expansion from t erythropoiesis (a l~o seen in Treatment: hydroxyurea (t llbF), hydration.
• 26 thalassemias).
. 27

8
Lode.
s
S uspe-nd
8
End Bloc:k
Item: 16 of - ,• Mark -<J
P~v1ous
[:::> "'I ~ ·· ~
100 ~ N @xt Labl lues N o tes Calcula to r

A A
1
A previously healthy 6-mont h-old girl is brought to the pediatrician with a cough that has been increasing in severity for several days. Her ~~AI
2
parents worry that she is having occasional difficulty breathing. On examination, temperature is 37 .4° C (99 .4° F) and oxygen saturation is
3 92%. Auscultation of the lungs revea ls inspiratory crackles and expiratory wheezing. The pediatrician suspects the child is suffering from
4 bronchiolitis, and this diagnosis is confirmed through serologic analysis of a nasopharyngeal swab.
5
6
Which of the following describes the etiologic agent of the girl's condition?
7 :
A. It is a non-enveloped RNA virus with a double-stranded, segmented genome
8
9 B. It is a prokaryotic organism lacking a cell wall
10
C. It is a prokaryotic organism that is also the most common cause of otitis media in this age group
11
D. It is an enveloped RNA virus with a single-stranded, nonsegmented genome
12
13 E. It is an enveloped RNA virus with a single-stranded, segmented genome
14

15
. 16
. 17
• 18
• 19
• 20
. 21
. 22
. 23
• 24
• 25
• 26
. 27

8
Lode.
s
Suspe-nd
8
End Bloc:k
Item: 16 of - ,• Mark -<J
P~v1ous
[:::> "'I ~ ·· ~
100 ~ N @xt Labl lues N o tes Calcula to r
A A
1
2
3
The correct answer is D. 59°/o c hose this.
4
The most common cause of bronchiolit is and pneumonia in children is respiratory syncytial virus (RSV) . This paramyxovirus has a single-
5 stranded, nonsegmented RNA genome and is surrounded by a lipid envelope. Clinically, RSV can present as either an upper respiratory tract
6 or a lower respiratory tract infection. Almost all children have had at least one bout of RSV illness by t he t ime t hey are 2 years old. When it
infects the lower respiratory tract, symptoms may include cough, tachypnea, low-grade fever, and a mildly reduced oxygen saturation.
7
Crackles and expiratory wheezing can be heard on auscultation. A presumptive clinical diagnosis of RSV can now be verified through a rapid
8 serological test for viral antigens.
• fecuon Par-cr··'")))"''"dae Human respiratory syncytial VIrus Tachypnea Pneumon•a """::."'"' tcltlon Resp•ratory tract Bronchiolitis Cough Respiratory tract •nfecbon Anbgen
9
Upper te\PI o b :t ();- Jen .aGC.uratJon medicine) Upid Genome Crackles Serology Wheeze 9fncybum Oxyven saturation Fever Oxygen RNA Virus
10
11 A is not correct. 9°/o chose this.
12 The only classes of human viral pathogens with a double-stranded, segmented, RNA genome are the reoviruses (such as Colorado tick fever)
13 and rotavirus (infectious diarrhea). They do not cause childhood pneumonia or bronchiolitis.
Colorado bck fever Rotav~rus Pneumoma 01arrhea Gastroenteritis Bronchiolitis Tick Genome Pathogen Reov1ndat. Few!r RNA V1rus
14

15 B is not correct. 5°/o chose this.


16 This choice describes Mycoplasma, the most common cause of pneumonia in teenagers and young adults. Mycoplasma pneumonia is often
referred to as either "walking pneumonia" (because pat ients do not fee l extremely ill) or "atypical pneumonia" (because they experience
. 17
nonproductive cough and streaky infilt rates on x-ray of th e chest).
• 18 Pneumonia Mycoplasma Cough X-ray

• 19
C is not correct. 11% chose this .
• 20 Streptococcus pneumonia is the most common cause of ot itis media in children and the most common cause of pneumonia in older adults and
. 21 th e elderly. However, it is not the etiologic agent of bronchiolit is in children .
Pneumonia Ot1t1s med1a Streptococcus Streptococcus pneumoniae Bronchiolitis Ot1t1s Pathogen Et1ology
. 22
. 23 E is not correct . 17°/o chose this .
• 24 This choice describes the influenza virus, an orthomyxovirus with a lipid envelope and a single-stranded, segmented RNA genome. The
• 25
respiratory symptoms of influenza can be similar to RSV, but it is a less common cause of disease .
Orthomyxov1 1d~tt I nflue""tza V1rus Genome Human respiratory syncytial virus Lipid RNA
• 26
. 27

8
Lode.
s
Suspe-nd
8
End Bloc:k
Item: 16 of ~ ,• Mark <::J [:::> ""I ~· ~'j
100 J.. Previous Next faiUI~S
LAb Notes Calculator

1
2 Bottom Line:
3 RSV paramyxovirus is the most common cause of bronchiolit is and pneumonia in children. It is a single-st randed, enveloped RNA v irus wit h a
4 nonsegment ed genome.
Para,.,xoviridae Pneumonia Bronchiolitis RNA virus Genome Human respiratory syncytial virus Virus RNA
5
6
7
8
l@l;fil·1i•l for year:[ 2017
FIRST AID FACTS .
•j .

9
10 FA17 p 175.2
11 Common causes of pneumonia
12 NEONATES I< 4WK) CH ILDREN(4WK-18 YR) ADULTS(18- 40 YR) ADULTS (40-65YR) ELDERLY
13
Group B streptococci Viruses (RSV) Mycoplasma S pneumoniae S pneumoniae
14
E coli Mrcoplasma C pnewnoniae 1I influenzae In Auenza virus
15
C trachomatis S pnewnoniae Anaerobes Anaerobes
16
(infants-3 yr) Viruses (cg, inAuenza) Viruses H influenzae
. 17
• 18
C pnewnoniae Mycoplasma Gram 8 rods
• 19
(school-aged
. 20 children)
. 21 S pneumoniae
. 22 Runts May Cough
• 23 C hunky Spul um
• 24 Special groups
• 25
Alcoholic Klebsiella, anaerobes usually due to aspiration (eg, Peptostreptococcus, Fusobacterium, Prevotelfa,
• 26
Bacteroides)
. 27
8
Lock
s
Suspend
0
End Block
Item: 16 of - ,• Mark -<J
P~v1ous
[:::> "'I ~ ·· ~
100 ~ N @xt Labl lues N o tes Calcula to r
A
1 FA17 p 163.1
A

2
RNA viruses
3
VIRAL FAMILY ENVELOPE RNA STRUCTURE CAPSID SYMMETRY MEDICAL IMPORTANCE
4
5
Reoviruses I o OS linear Icosahedral Coltivirus3 - Colorado tick fever
6
10-1 2 segments (double) Rotavirus-cause of fatal diarrhea in children
7 Picornaviruses 0 SS @ linear Icosahedral Poliovirus-polio-Salk/Sabin vaccines- IPV/OPV
8 l•.cho\'irus-aseptic meningitis
9 Rhino\'irus-"common cold"
10 Co, sacJ..ievirus-aseptic meningitis; herpangina
11 (mouth blisters, fever); hand, foot, and mouth
12 disease; mrocarditis; pericarditis
13 II V -acute "ira I hepatitis
14 PERC II
15
Hepevirus 0 SS ® linear Icosahedral HEV
16
. 17 Caliciviruses 0 SS ® linear Icosahedral Norovirus-vira l gastroenteritis
• 18 HCV
Flaviviruses Yes SS ® linear Icosa hedral
• 19
Yellow fever1
• 20 Oenguea
. 21
Sl. I.ouis encephalitisa
. 22
West 'ile virus3 {meningoencephalitis)
. 23
Zika virus
• 24
• 25
Togaviruses Yes SS $ linear Icosahedral Rubella
• 26
Western and Eastern equine encephalitisU
. 27 • Chikungunra ,·irus
8
Lode.
s
Suspe-nd
8
End Bloc:k
Item: 17 of - ,• Mark -<J
P~v1ous
[:::> "'I ~ ·· ~
100 ~ N @xt Labl lues N o tes Calcula to r

1
Oncogenic viruses act through a variety of mechanisms. Some introduce oncogenes directly into host cells, while others force cells to
2
repeatedly undergo cycles of proliferat ion t hat event ually become unregulated. Still others int roduce oncogenic pot ential by manipulating
3 chromosomal structure through delet ions or translocat ions.
4
5 Which of the following viruses causes neoplasia by inactivating tumor suppressor genes such as p53 and Rb?
6 42883S :
7 A. Epstein-Barr virus
8 B. Hepatitis C virus
9
C. Human immunodeficiency virus
10
11 0 . Human papillomavirus
12 E. Human T-cell lymphotropic virus type 1
13
14

15
16
. 17
• 18
• 19
• 20
. 21
. 22
. 23
• 24
• 25
• 26
. 27

8
Lode.
s
S uspe-nd
8
End Bloc:k
Item: 17 of ~ ,• Mark <::J [:::> ""I ~· ~'j
100 J.. Previous Next LAbfaiUI~S Notes Calculator

1
2
The correct answer is 0. 69°/o chose this.
3
Human papillomavirus (H PV) causes ca rcinoma (usua lly cervical) by inactivating t umor suppressor genes such as p53 and Rb through t he
4 act ions of vira l proteins E6 and E7, respectively.
Human papillomavirus P53 Tumor suppressor gene Neoplasm Papillomaviridae Carcinoma Retinoblastoma protein Virus Cervix Protein Cervical cancer
5
6 A is not correct. 13% chos e this.
7 Epstein-Ba rr v irus ( EBV) is associated wit h Burkit t 's lym phoma (a B-lymphocyte lym phoma) and nasopharyngeal carcinoma . The t(S; 14)
8
t ranslocation is consistent ly associat ed with Burkitt's lymphoma, but t he t ranslocation alone is not responsible fo r the neoplasm and is not
found in nasopharyngea l carcinomas. The other factors that determine oncogenesis of EBV rema in uncl ear.
9 Burkitt's lymphoma Epstein-Barr virus Nasopharynx cancer 8 cell Carcinogenesis Neoplasm Lymphoma Virus Carcinoma Pharynx Chromosomal translocation Nasopharynx
10
B is not correct. 5% chose this.
11
Bot h hepat it is C and hepat it is B virus infections are associated wit h an increased ri sk of developing hepatocellular carcinoma . The liver has a
12 high regenerat ive potent ial, but if t his process is overused, the chance of an oncogenic mutat ion occurring duri ng the regenerat ion of cells
13 increases.
Hepatocellular carcinoma Hepatitis C Hepatitis 8 Hepatitis Hepatitis 8 virus Virus Carcinogenesis Liver Oncogene Mutation Carcinoma
14
15 C is not correct. 5 °/o chose this .
16 HIV as a direct oncogenic agent is being intensely researched, but it is already known t hat immune suppression and dysregulation caused by
HIV infect ion give rise t o lymphomas and Kaposi's sarcoma.
17
Kaposi's sarcoma HIV Carcinogenesis Immunosuppression Oncogene Sarcoma Lymphoma
• 18
• 19 E is not correct. 8°/o chos e this .
Human T-cell lymphotropic v irus causes adult T-cell leukem ia, and although t he mechanism of oncogenesis rema ins u nclea r~ there is some
• 20
evidence that integ rat ion int o t he host genome at locations near cellular growt h genes may play a role .
. 21 Adult T-cell leukemia/lymphoma Carcinogenesis Leukemia Virus T cell Genome T-cell leukemia Cell growth
. 22
• 23
Bottom Line:
• 24
HPV inactivat es p53 and Rb using E6 and E7 proteins.
• 25
PS3 Human papillomCPJirus Retinoblastoma protein Protein
• 26
. 27

8
Lock
s
Suspend
0
End Block
Item: 17 of ~ ,• Mark <::J [:::> ""I ~· ~'j
100 J.. Previous Next faiUI~S
LAb Notes Calculator

1
2
141;fil·1i•J
FIRS T AID FACTS
fo r yea r:[ 201 7
.
•j .
3
4
5 FA17 p 609.2
6 Cervical pathology
7 Dysplasia and Disordered epithelial growth; begins at basal layer of squamocolumnar junction (transformation
8
carcinoma in situ zone) and extends outward. Classified asCI 1, CIN 2, or CI 3 (severe, irreversible dysplasia or
9
carcinoma in situ), depending on extent of dysplasia. Associated with HPV-16 and 1-JPV-18, which
10
produce both the E6 gene product (inhibits p53 suppressor gene) and E7 gene product (inhibits
11

12
a
RB suppressor gene); koi locrtes are pathognomonic of HPV infection. May progress slowly to
invasive carcinoma if left untreated. Typically asymptomatic (detected with Pap smear) or presents
13
14
as abnorma l vaginal bleeding (often postcoita l).
15
Risk factors: multiple sexual partners (#1), smoking, starting sexual intercourse at young age, HIV
16
infection.
17
• 18 Invasive carcinoma Often squamous cell carcinoma. Pap smear ca n detect cervical dysplasia before it progresses to
• 19 invasive carcinoma. Diagnose via colposcopy and biopsy. Lateral invasion can block ureters
• 20 ... renal fa ilure.
. 21
. 22
FA17 p 160.2
• 23
• 24
Herpesviruses Enveloped, OS, and linear viruses
• 25 VIRUS ROUTE OF TRANSMISSION CLI NICAL SIGNIFICANCE NOTES
• 26
Herpes Respiratory Gingivostomatitis, keratoconjunctivitis a, Most common cause of sporadic
. 27
8
Lock
s
Suspend
0
End Block
Item: 17 of - ,• Mark -<J
P~v1ous
[:::> "'I ~ ·· ~
100 ~ N @xt Labl lues N o tes Calcula to r
A
1 FA17 p 160.2
A

2
3
Herpesviruses Enveloped, OS, and linear viruses
4 VIRUS ROUTE OFTRANSMISSION CLIN ICAl SIGNI FICANCE NOTES
5 Herpes Respiratory Gingivostomatitis, keratoconjunctivitis \1ost common cause of sporadic
6
simplex secretions, saliva herpes labia lis , herpetic "h itlo" on finger, encephalitis, can present as altered
7
virus-1 temporal lobe encephalitis, esophagitis, menta I status, seizures, and/or
8
en·thema mult iforme. aphasia.
9
10
Herpes Sexual contact, Herpes genital is ~. neonatal herpes. Latent in sacral ganglia. Viral
11
simplex perinatal meningitis more common with
12 virus-2 liS -2 than with HSV-1.
13 Varicella- Respiratory Varicella-zoster (chickenpox , shingles 0 ), Latent in dorsal root or trigeminal
14 Zoster virus secretions encephalitis, pneumonia. ganglia; CN V1 branch
15 (HHV-3) Most common complication of shingles is post- involvement can cause herpes
16 herpetic neuralgia. zoster ophthalmicus.
17
• 18 Epstein-Barr Respiratory Mononucleosis- fever, hepatosplenomegaly, Infects B cells through C02 I.
• 19 virus (HHV-4) secret ions, pharyngitis, and lymphadenopathy (especially Atypical lymphocytes on peripheral
• 20 saliva·, aka posterior cervical nodes (J). Avoid contact sports blood smear [!I- not infected B
. 21 "kissing disease," until resolution due to risk of splenic rupture. cells but reactive cytotoxic T cells.
. 22 (common in Associated with lymphomas (eg, endemic Et> Monospot test-heterophile
. 23 teens, young Burkitt lymphoma), nasopharyngeal antibodies detected by agglutination
• 24 adults) carcinoma (especially Asian adults), of sheep or horse RBCs.
• 25 lymphoproliferative disease in transplant Use of amoxicillin in mononucleosis
• 26 patients. can cause characteristic
. 27 • ..,,..,,..,,ln ...,...,...,.,,), r .....,~h

8
Lode.
s
S uspe-nd
8
End Bloc:k
Item: 17 of - ,• Mark --<) [::> ""'I ~· 1!';:'1
~
100
1 •
Prev1o u s Next Labf a lu es
. .Notes
.. .
Calculator
. .
Congenital CM
2
3 Human Saliva Roseola infantum (exanthem subitum): high Roseola: fe,·er first, Rosie (checks)
4 herpes- fevers for several days that can cause seizures, later.
5 viruses 6 followed by diffuse macular rash HHV-7-less common cause of
6 and7 roseola.
7
Human Sexual contact Kaposi sarcoma (neoplasm of endothelial cells). Can also affect C ltract and lungs.
8
herpesvirus Seen in HIV/AIDS and transplant patients.
9
8 Dark/violaceous plaques or nodules
10
representing \'ascular proliferations.
11

12
13
14

15
16
17
• 18
• 19
• 20
. 21
. 22
• 23
• 24
• 25
• 26
. 27 •
8
L.odt
s
Su~pl'nd
~
End Block
Item: 17 of - ,• Mark --<) [::> ""'I ~· 1!';:'1
100 ~ Prev1o u s Next Labf a lu es Notes Calculator

1 • •
FA 17 p 160o1
2
DNA viruses
3
VIRAL FAMILY ENVELOPE DNA STRUCTURE MEDICAL IMPORTANCE
4
5
Herpesviruses Yes DS and linear See Hcrpesviruses entry
6 Poxvirus Yes OS and linear Smallpox cn1dicated world wide by use of the li, e-
7 (largest DNA virus) attenuated vaccine
8 Co" pox ("milkmaid blisters")
9 Molluscum contagiosum- flesh-colored papule with
10 cent ral umbilication
11
Hepadnavirus Yes Partially OS and circular II BV:
12
13
Acute or chron ic hepatitis
14
Not a retrovirus but has re,·erse transcriptase
15 Adenovirus I 0 OS and linear Febrile pharyngitis r.l-sore throat
16 Acute hemorrhagic cystitis
17 Pneumonia
• 18 Conju nctivitis-"pink eye"
• 19
• 20
0
21
0
22
• 23
• 24
Papillomavirus 0 DS and circular II P - warts (serotrpes I, 2, 6, II}, Cli , cervical
• 25
cancer (most commonly 16, 18)
• 26
0
27 Polvomavirus I 0 OS and circular IC 'irus-oro2rcssive multifocalleukocnceoha looatlw •
8
L.odt
s
Su~pl'nd
~
End Block
Item: 18 of - ,• Mark -<J [:::> "'I ~ · ~
100 ~ P~v1ous N @xt Labl lues N o tes Calcula to r

A A
1
A 24-year-old medical student presents to t he clinic wit h a sore throat accompanied by lymphadenopat hy. A rapid strep t est is positive fo r ~~AI
2
streptococcus-induced pharyngitis. The physician decides to give the patient penicillin, but t he pati ent indicates t hat t he last t ime that he
3 was given penicillin, he experienced moderate airway edema and difficulty breathing t hat requi red hospitalizat ion.
4
5 Which of the following is the best choice fo r t reat ing t his patient?
6 :
7 A. Amoxicillin
8 B. Cephalexin
9
C. Erythromycin
10
11 0 . Metronidazole
12 E. Vancomycin
13
14

15
16
17
• 18
• 19
• 20
. 21
. 22
. 23
• 24
• 25
• 26
. 27

8
Lode.
s
S uspe-nd
8
End Bloc:k
Item: 18 of ~ ,• Mark <::J [:::> ""I ~· ~'j
100 J.. Previous Next LAb faiUI~S Notes Calculator

1
2
3
The correct a nswer is C. 5 4 °/o chose this .
I n a penicillin-allergic patient (such as t he patient descri bed), eryt hromycin is considered the next best choice. Erythromycin is a macrolide
4
ant ibiot ic t hat has activity against gram-posit ive organisms (especially Pneumococcus, Streptococcus, Staphylococcus, and Corynebacterium),
5 Enterococci, and gram- negative organisms.
6 Erythromycin Macrolide Gram-negative bacteria Antibiotics Gram-positive bacteria Streptococcus pneumoniae Enterococcus Streptococcus Staphylococcus Corynebacterium

7 A is not correct . 10% chos e this .


8 Amoxicillin is a penicillin-based drug. Given that t he patient descri bes an anaphylactoid react ion to penicillin, t reating his pharyngit is wit h
9 amoxicillin is not an option.
Amoxicillin Penicillin Anaphylaxis Pharyngitis
10
11 B is not correct. 21% chose this .
12 Cephalexin is a cephalosporin, a class of antibiot ics relat ed to the penicillins. Alt hough not every patient who is allergic to penicillins is allergic
t o cephalosporins, t here is an increased rate of cross- reactivity t o cephalosporins in penicillin -allergic patients. Given this cross-allergy,
13
cephalosporins would not be a good choice for t reat ment of t he pat ient's pharyngit is. This ref lect s t he current standard of t reatment fo r all
14 patients with penicillin or cephalosporin allergies.
Cephalosporin Cefalexin Penicillin Pharyngitis Allergy Antibiotics Cross-reactivity
15
16 D is not correct. 4 °/o chose this.
17 Met ronidazole is an ant ibiot ic t hat fo rms toxic metabolites in t he bact eri al cell. It is typically used t o t reat protozoal infections and anaerobic
18 infections from Clostridium or Bacteroides species. I t is not used to t reat streptococcal infect ions.
Metronidazole Antibiotics Bacteroides Anaerobic organism Streptococcus Clostridium Protozoa Metabolite Toxicity Anaerobic infection
• 19
• 20 E is not correct. 11°/o chose this .
. 21 Vancomycin inhibit s cell wall formation by binding to D-ala D-ala portion of cell wall precursors. It is used to t reat serious infections with
gram-posit ive, mult idrug- resistant organisms such as Staphylococcus aureus and Clostridium difficile . Because the organism causing strep
. 22
t hroat (Streptococcus pyogenes) is susceptible to penicillin, using a drug reserved for more virulent pat hogens would be unreasonable.
• 23 Furthermore, vancomycin is administered intravenously and would req uire int ravascular access. This option would lead to unnecessary
• 24 procedures and inconvenience in a st able pat ient who will benefit f rom out pat ient follow-up and oral antibiotics .
Streptococcus pyogenes Vancomycin Penicillin Cell wall Streptococcal pharyngitis Staphylococcus aureus Gram-positive bacteria Clostridium difficile colitis Clostridium difficile (bacteria) Streptococcus
• 25
Staphylococcus Antibiotics Intravenous therapy Clostridium Organism Pathogen Multiple drug resistance Blood vessel
• 26
. 27

8
Lock
s
Suspend
0
End Block
Item: 18 of ~ ,• Mark <::J [:::> ""I ~· ~'j
100 J.. Previous Next faiUI~S
LAb Notes Calculator

1
Bottom Line:
2
Streptococcus infect ions are best treat ed with penicillin; however, if t he patient is allergic to penicillin, t hen erythromycin is t he next -best
3
choice.
4 Erythromycin Penicillin Streptococcus

5
6
7 i@l;fil·1i•J for year:[ 2017 • J
FIRST AID FACTS
8
9
FA17 p 189.2
10
Macrolides Azithromycin, clarithromycin, erythromycin.
11

12 MECHANISM In hibit protei n synthesis by blocking translocation ("macros!ides"); bind to I he 23S rR A of the
13 50S ribosomal subunit. Bacteriostatic.
14 CLINICAL USE Atypical pneumonias (Mycoplasma, Chlamydia, Legionella), STis (Chlamydia), gram Et> cocci
15
(streptococcal infections in patients allergic to penicillin), and B pertussis.
16
17 ADVERSE EFFECTS MACRO: Gastrointestinal Motility issues, Arrhythmia caused by prolonged QT interval, acute
18
C holestatic hepatitis, Rash, eOsinophil ia. Increases serum concentration of theophylline, oral
• 19 anticoagulants. Clarithromycin and erythromycin inhibit cytochrome P-450.
. 20 MECHANISM OF RESISTANCE Methylation of 23S rR 'A-binding site prevents binding of drug.
. 21
. 22
• 23 FA17 p 183.2
• 24 Penicillin G, V Penici llin G (IV and IM form), penicillin V (oral). Prototype ~-lactam antibiotics .
• 25
• 26
MECHANISM D-Ala-0-Ala structural analog. Bind penicill in-binding proteins (transpeptidases).
. 27
Block transpeptidase cross-linking of peptidoglycan in cell wall.
8
Lock
s
Suspend
0
End Block
Item: 18 of - ,• Mark -<] 1:> ""'I ~· 1!';:'1
100 ~ Prev1o u s Next Labf a lu es Note s Calculator

1 FA17 p 183.2
2
Penicillin G, V Penicil lin C (IV ancll i\ 1 form), penicillin (oral). Prototype ~-lacta m a ntibiotics.
3
4
MECHANISM 0 -Aia-D-Aia structural analog. Bind penicillin-binding proteins (transpeptidases).
5
Block transpeptidasc cross-linl..ing of peptidoglycan in cell wall.
6 Activate autolytic enzymes.
7 CliNICAL USE 1ostlr used for gram @ organisms (S pneumoniae, S pyogenes, Actinomyces). Also used for gram 8
8 cocci (mainly N meningitidis) and spirochetes (namely T pallidum). Bactericidal for gram @cocci,
9
gram @ rods, gram e cocci, and spirochetes. Penicillinase sensiti,·e.
10
11
ADVERSE EFFECTS Hypersensitivity reactions, direct Coombs @ hemolr tic anemia.
12 RESISTANCE Penicillinase in bacteria (a t) pe of ~-lact amase) cleaves ~-I actam ring.
13
14
FA17 p 184.1
15
16 Penicillinase-sensitive Amoxicillin, ampicillin; aminopcnicill ins.
17 penicillins
18
MECHANISM Same as penicillin . Wider spectrum; AM inoPenicillins are Al\ IPed-up penicillin.
• 19
penicill inase sensitive. Also combine wit h AmO xicillin has greater O ral bioavailabi lity
• 20
clavulanic acid to protect against destruction than ampicillin.
. 21
by ~-lactama se.
. 22
0 23 CLINICAL USE Extended-spectrum penicilli n- I I influen;:ae, CO\·cragc: ampicillin/amoxicillin IlliELPSS
0 24 H pylori, E coli, Listeria monocytogenes, kill enterococci.
0
25 Proteus mirabilis. Salmonella, Shigella,
0
26 enterococci.
. 27 •
8
L.odt
s
Su~pl'nd
~
End Block
Item: 19 of - ,• Mark -<J
P~v1ous
[:::> "'I ~ ·· ~
100 ~ N @xt Labl lues N o tes Calculato r

A A
1
An 8-year-old boy cl im bs over a fence into his neighbor's backyard t o retri eve a soccer ball. While searching for t he ball, he is attacked and ~~AI
2
bitten by t he neighbor's dog. He is rescued by a nearby neighbor. Ten hours l ate r~ the bite wound is puru lent and inflamed. The purulent
3 exudate is cultured and reveals gram-negat ive coccobacilli.
4
5 Which of the fol lowing microorganisms is the most likely cause?
6 :
7 A. Bartonella henselae
8 B. Brucella canis
9
C. Eikenella corrodens
10
11 0 . Francisella tularensis
12 E. Pasteurella multocida
13
14

15
16
17
18
• 19
• 20
. 21
. 22
. 23
• 24
• 25
• 26
. 27

8
Lode.
s
S uspe-nd
8
End Bloc:k
Item: 19 of ~ ,• Mark <::J [:::> ""I ~· ~'j
100 J.. Previous Next LAb faiUI~S Notes Calculator

1
2
The correct answer is E. 61 °/o chose this.
3
The pat ient was bitt en by a dog carry ing Pasteurella multocida, a gram-negat ive coccobacillus (alt hough t here is some variat ion in
4 morphology) that is part of the norma l oral flora in cats and dogs. It is an aggressive and ra pidly spreading infection t hat can cause skin
5 abscesses, as noted in t his pat ient. Despite the fact t hat this is a gram-negat ive organism, ~ -l actam ant ibiotics, such amoxicillin-clavulanat e,
are highly effective at t reat ing this infect ion and this t reatment should be started immed iat ely. The distinct coccobaccilus morphology of this
6
infection, along with t he cl inical hist ory, help separate Pasteurella f rom ot her answer choices.
7 Pasteurella multocida Amoxicillin/ clavulanic acid Gram-negative bacteria Coccobacillus Antibiotics Pasteurella Abscess Dog Infection

8
A is not correct. 10% chose this.
9 Bartonella hense/ae is a gram-negat ive rod t hat is usually associat ed with cat scrat ches. Chronic lymphadenit is would most commonly be seen
10 in young children .
Bartonella henselae Gram-negative bacteria Bartonella Lymphadenopathy Cat
11

12 B is not correct. 16% chose thi s.


13 Brucella canis, which is a gram-negat ive rod t hat is normally found in dogs, can cause fever, malaise, and hepatosplenomegaly in hu mans.
14 Those exposed to blood of infect ed animals are at ri sk, typically vet erinarians. I mmunocompromised persons should not handle dogs known to
be infect ed wit h B. canis.
15 Gram-negative bacteria Brucella Immunodeficiency Hepatosplenomegaly Malaise Fever
16
C is not correct. 6 °/o chose this .
17
Eikenella corrodens is a gram-negat ive facu lt ative anaerobic bacillus found in human bit es. It may cause infection in persons with insulin-
18 dependent diabetes or drug users who lick t heir needles. It is one of the HACEK organisms, which can cause culture-negative endocardit is.
19 The HACEK organisms are a group of gram -negative bacteria that includes Haemophilus spp., Actinobacillus actinomycetemcomitans,
• 20
Cardiobacterium hominis, Eikene/la corrodens, and Kinge/la kingae .
Aggregatibacter actinomycetemcomitans Eikenella corrodens Endocarditis Kingella kingae Gram-negative bacteria HACEK endocarditis Anaerobic organism Facultative anaerobic organism
. 21
Cardiobacterium hominis Actinobacillus Bacillus Bacteria Diabetes mellitus
. 22
• 23 D is not correct. 7°/o chose this •
• 24
Francisel/a tularensis is a gram-negat ive rod t hat is commonly found in rabbits, and it is t ransm itted t o humans via cont act with infect ed
animal t issues .
• 25 Francisella tularensis Gram-negative bacteria Francisella
• 26
. 27

8
Lock
s
Suspend
0
End Block
Item: 19 of ~ ,• Mark <::J [:::> ""I ~· ~'j
100 J.. Previous Next faiUI~S
LAb Notes Calculator

1
Bottom Line:
2
3
The most common organism causing cellulit is on the boards after a cat or dog bi te is Pasturella multocida . St aphylococcal species are also
common. Treat ments fo r cellulitis caused by dog bite include amoxicillin, or a combinat ion of amoxicillin and clavulanat e (Augmentin).
4 Amoxicillin Amoxicillin/ clavulanic acid Cellulitis Clavulanic acid Staphylococcus
5
6
7 l@l;fil·1i•l f or yea r:[ 2017
FIRST AID FACTS .
•j .
8
9
FA17 p 144.1
10
11 Zoonotic bacteria Zoonosis: infectious disease transmitted between animals and humans.
12
SPECI ES DISEASE TRANSMISSION AND SOURCE
13
14
Anaplasma spp. Anaplasmosis Ixodes ticks (live on deer and mice)
15 Bartonella spp. Cat scratch disease, bacilla ry angiomatosis Cat scratch
16
Borrelia burgdorferi Lyme disease Ixodes ticks (live on deer and mice)
17
18
Borrelia recurrentis Relapsing fever Louse (recurrent due to variable surface
19 antigens)
. 20 Brucella spp. Brucellosis/undulant fever Unpasteurized dairy
. 21
Campy/obacter Bloody diarrhea Feces from infected pets/an imals; contaminated
. 22
meats/foods/hands
• 23
• 24 Chlamydophila psittaci Psittacosis Parrots, other birds
• 25 Coxiella burnetii Q fever Aerosols of cattle/sheep amn iotic Auid
• 26
Ehrlichia chaffeensis Ehrlich iosis Amblyomma (Lone Star tick)
. 27
8
Lock
s
Suspend
0
End Block
Item: 19 of - ,• Mark -<] 1:> ""'I ~· 1!';:'1
100 ~ Prev1o u s Next Labf a lu es Notes Calculator

1 •
2 FA17 p 182.2
3 Bug hints (if all else CHARACTERISTIC ORGANISM
4
fails) Asplenic patient (due to surgical splenectomy Encapsulated microbes, especially Sl l i~
5
or autosplenectomy, eg, chronic sickle cell (S pneumoniae >> H influen;:ae trpe B >
6
disease) 1\ meningitidis)
7
8 Branching rods in oral infection, sulfur granules Actinomyces israelii
9 Chronic granulomatous disease Catalase (±) microbes, especiall) S aureus
10
11
"Currant jelly" sputum Klebsiella
12 Dog or cat bite Pasteurella multocida
13
Facial nen·e palsy (typically bilateral) Borrelia burgdorferi (Lyme disease)
14

15
Fungal infection in diabetic or Mucor or Rhizopus spp.
16
immunocompromised patient
17 Health care provider HBV (from needlestick)
18
eutropenic patients Candida albicans (system ic), Aspergillus
19
• 20 Organ transplant recipient CMV
. 21 PAS (±) Tropheryma whipplei (Whipple disease)
. 22
Pediatric infection llaemophilus influenzae (including epiglottitis)
0 23
0 24 Pneumonia in cystic fibrosis, burn infection Pseudomonas aeruginosa
0
25 Pus, empyema, abscess S aureus
0
26
Rash on hands and feet Coxsackie A virus, Treponema pallidum, •
. 27

8
L.odt
s
Su~pl'nd
~
End Block
Item: 20 of - ,• Mark -<J [:::> "'I ~ · ~
100 ~ P~v1ous N @xt Labl lues N o tes Calcula to r

A A
1
A 26-year-old Basqlle immigrant who has exposllre t o dogs and sheep presents to the clinic wit h a 3- month hist ory of right upper quadrant ~~AI
2
abdominal pain. The patient denies any hist ory of fever, vomit ing, or diarrhea in the last 6 mont hs. An ultrasound of t he right upper
3 quadrant reveals a well-circumscribed hypoechoic lesion with eggshell calcifications on the left lobe of the liver, measuring 6 em in
4 diameter. Laboratory find ings are as follows :
5
WBC count: 9000/mml
6 Segmented neutrophils: 56%
7 Band forms: 4%
Eosinophils: 10%
8
Basophils: 0.6%
9 Lymphocytes: 26%
10 Monocytes: 3%
11

12
Which of the following best describes the most likely offending agent?
13
14
A. Aerobic gram-negative bacteria
15 B. Nonmotile gram-negative bacteria
16
C. Protozoan
17
18
D. Tapeworm
19 E. Trophozoite
• 20
. 21
. 22
. 23
• 24
• 25
• 26
. 27

8
Lode.
s
S uspe-nd
8
End Bloc:k
Item: 20 of - ,• Mark -<] 1:> ""'I ~· 1!';:'1
100 ~ Prev1o u s Next Labf a lu es Notes Calculator

1 •
Th e correct a nswer is D. 50°/o chose this.
2
This patient is infected with Echinococcus granu/osus, manifesting with a classic hydat id cyst lesion of t he
3
liver. Hydatid cysts contain larvae of the tapeworm E. granulosus, whose eggs are carried from the
4 intestinal tract to the liver via the portal circulation. Echinococcus is a dog tapeworm that incidentally
5 infects humans and causes walled-off granulomatous lesions that result in anaphylactoid reactions when
they erupt. Treatment is surgical excision. Note that the patient's lab values show eosinophilia, which is
6
characteristic but not frequently seen.
7
A history involving exposure to dogs or sheep, which are hosts in the life cycle of the tapeworm,
8 hypereosinophilia, and right upper quadrant abdominal pain are all suggestive findings that point toward
9 infection with Echinococcus. Ultrasound of the liver is fairly sensitive and often shows a hypoechoic lesion of
10
the liver, which may have a calcified rim (ie, the eggshell calcification sign). Serology can provide a
definitive diagnosis, and cyst aspiration should only used as a last resort for diagnostic purposes due to the
11 risk of anaphylaxis. The image is of an axial spiral CT showing a hydatid cyst.
12
Cestoda Echmococco$'• Echtnococcus granulosus Anaphylaxis Eosinophilia Serology Cyst CT .cca"' MediC ult. aliound Abdormnal pain Medical imaging Image copyright © 2008
13 Duttaroy et a/; licensee BioMed
Granuloma Ultrasound l.Jver Quadrant (abdomen) Echinococcus Gastrointestinal tract Calcification Human gastrointestinal tract Sheep Lesion
14
Central Ltd.

15 A i s no t correct. 6°/o chose this .


16 Aerobic gram-negative bacteria such as Pseudomonas species do not cause mass lesions. Also of note, an infection by a gram-negative
17 bacterium would not cause the eosinophilia seen in the patient's lab data.
Gram·negat1ve bacteria Bacteria Eosinoph1ha Aerobic organism Pseudomonas Spec1es
18
19 B i s not correct. 6 % chose this .
20 Nonmotile gram-negative bacteria such as Shigella species do not cause mass lesions. Al so of note, an infect ion by a gram-negative bacterium
. 21
would not cause t he eosinophilia seen in t he pat ient's lab data .
Sh1gella Gram·negatlve bacteria Bactena Flagellum Motility Eosinophilia Species
. 22
0 23
C is n ot correct . 23°/o chose this .
Protozoans such as Entamoeba histolytica are definitely on the differential for hepatic abscesses in an immigrant from an endemic area such
0 24
as Mexico or parts of Central and South America. However, patients with ambeic liver abscess usually present more acutely with a 1- to 2-
0
25 week history of acute right upper quadrant pain and fever. Less than one-third of infected pat ients have concurrent diarrhea, but some may
0
26 report a history of bloody diarrhea weeks ago. Moreover, most patients are expected to have leukocytosis without eosinophilia. For imaging,
ultrasound is also fairly sensitive, but lesions are most commonly found in the right lobe of the liver. Serology usually establishes the •
. 27

8
L.odt
s
Su~pl'nd
~
End Block
Item: 20 of ~ ,• Mark <:::1 t::> ""I ~· ~'j
100 J.. Previous Next LAb faiUI~S Notes Calculator

1
B is not correct. 6% chose t his .
2
Nonmotile gram- negative bacteria such as Shigella species do not cause mass lesions. Also of note, an infect ion by a gram -negative bacterium
3 would not cause t he eosinophilia seen in t he pat ient's lab data .
Shigella Gram-negative bacteria Bacteria Flagellum Motility Eosinophilia Species
4
5 C is not correct. 2 3% chos e this .
6 Protozoans such as Entamoeba histolytica are def initely on t he different ial fo r hepat ic abscesses in an immigrant f rom an endemic area such
7
as Mexico or parts of Cent ral and South America . However, patients with ambeic liver abscess usually present more acut ely with a 1- to 2-
week history of acut e right upper quadrant pain and feve r. Less than one-third of infected pat ients have concurrent diarrhea, bu t some may
8 report a history of bloody diarrhea weeks ago . Moreover, most patients are expect ed t o have leukocytosis wit hout eosinophilia. For imaging,
9 ult rasound is also fa irly sensit ive, but lesions are most commonly found in t he ri ght lobe of t he liver. Serology usually establishes t he
10 diagnosis, but caut ion should be exercised as immigrant s from high -ri sk areas may already have ant ibodies due to a prior infection. Aspiration
is usually not warranted, but if perfo rmed may show brown f luid likened t o "anchovy past e. "
11 Entamoeba histolytica Leukocytosis Abscess Liver abscess Eosinophilia Serology Quadrant (abdomen) Medical ultrasound Diarrhea South America Protozoa Liver Antibody Ultrasound Fever Infection
12
E is not correct. 1 5°/o chose this .
13
I nfection wit h trophozoites such as Trichomonas species manifest with dysuri a and genit al discharge.Trophozoit es are generally a life cycle
14 form of protozoa, not helminths. Since Echinococcus is a tape worm, a helminth, it has no such fo rm .
15 Dysuria Protozoa Cestoda Helminths Echinococcus Trichomonas Infection Worm Species Biological life cycle

16
17
Bottom Line :
18
Right upper quadrant pain with a liver mass, along with a laboratory f inding of eosinophilia, indicate an Echinococcus granu/osus hydat id
19 cyst.
20 Echinococcosis Echinococcus granulosus Quadrant (abdomen) Cyst Echinococcus Liver Eosinophilia

. 21
. 22
• 23 141;fil·1i•J for yea r:[2017 •
FIRST AID FA CTS
• 24

• 25
FA17p 154.1
• 26
Protozoa- others
. 27

8
Lock
s
Suspend
0
End Block
Item: 20 of - ,• Mark -<J [:::> "'I ~ · ~
100 ~ P~v1ous N @xt Labl lues N o tes Calcula to r
A A
1 FA17 p 154.1
2 Protozoa- others
3
ORGANISM DISEASE TRANSMISSION DIAGNOSIS TREATMENT
4
Visceral infections
5
6 Trypanosoma Chagas disease-dilated Redu\'i id bug Trypomastigote in Ben.tnidazole
7 cruzi cardiomyopathy with (" kissing bug") blood smear a or nifurtimox;
8 apical atrophy, megacolon, feces, deposited C ru.r.ing in 111)
9 megaesophagus; predominant!)' in a painless bite Benz, with a fur
10 in South America (much like a ki \\) coat on
11 Unilateral periorbital swelling
12 (Romaiia sign) characte ristic of
13
acute stage
14

15 Leishmania Visceral leishmaniasis SanciAy t\ lacrophages Amphotericin B,


16 donovani (kala -azar) - spiking fevers, containing sodium
17 hepatosplenomegaly, amastigotes : stibogluconate
18 pancytopenia
19 Cutaneous leishmaniasis-skin
20 ulcers
. 21
. 22
. 23
• 24 Sexually transmitted infections
• 25 Trichomonas Vaginitis-foul-smelling, green ish Sexual (cannot exist Trophozoites lvlctronidazolc for
• 26
. 27
vagina/is
... -
discharge; itching and burning;
.. outside human (motile) on wet

m
44 •
patient and partner
• •
8
Lode.
s
S uspe-nd
8
End Bloc:k
Item: 20 of ~ ,• Mark <:::1 t::> ""I ~· ~'j
100 J.. Previous Next LAbfaiUI~S Notes Calculator

1 • •
FA17 p 151 .1
2
3 Protozoa- gastrointestinal infections
4 ORGANISM DISEASE TRANSMISSION DIAGNOSIS TREATMENT
5 Giardia Iamblia Giardiasis-bloating, flatulence, Cysts in water Multinucleated Metronidazole
6 fou l-smelling, fa tty diarrhea trophozoites fJ or
7
8
(often seen in campers/hikers)- rn
cysts in stoo1 J

think fat-rich Ghirardelli antigen detection


9
chocolates for fatty stools of
10
Giardia
11

12 Entamoeba Amebiasis- bloody diarrhea Cysts in water Serology and/or Metronidazole;


13 histolytica (dysentery), liver abscess trophozoites (with paromomycm or
14 ("anchovy paste" exudate), engulfed RBCs [i iodoguinol for
15 RUQ pain; histolog)' shows in t·he cytoplasm) asymptomatic cyst
16 Aask-shaped ulcer or cysts with up to passers
17 4 nuclei in stool [!];
18 Entamoeba Eats
19 Erythrocytes,
20 antigen detection
. 21
. 22
• 23
Cryptosporidium Se,·ere diarrhea in AIDS Oocysts in water Oocysts on acid-fast Pre\·ention (by
• 24
Mild disease (watery diarrhea) in stain 0 , antigen filtering city
• 25 immunocompetent hosts detection water supplies);
• 26 nitazoxan ide in
• immunocomoetent •
. 27

8
Lock
s
Suspend
0
End Block
Item: 20 of - ,• Mark -<] 1:> ""'I ~· 1!';:'1
100 ~ Prev1o u s Next Labf a lu es Notes Calculator

1 FA17 p 156.1
2 Cestodes (tapeworms)
3 TRANSMISSION
ORGANISM DISEASE TREATMENT
4
Taenia solium Intestinal tapeworm Ingestion of larvae encysted in Praziquantcl
5
undercooked pork
6
7 Cysticercosis, Ingestion of eggs in food Praziquantel; albendazole for
8 neurocysticercosis contaminated'' ith human neurocysticercosis
9 feces
10
Diphyllobothrium itamin B12 deficiency Ingestion of lan·ae in raw Praziquantel
11
Ia tum (tapeworm competes for B12 freshwater fish
12
in intestine) - megaloblastic
13
anem1a
14

15 Echinococcus Hydatid cysts ("eggshell Ingestion of eggs in food Albendazole


16 granulosus calcification" [!]) in contaminated with dog feces
17 liver 0 ; cyst rupture ca n Sheep are an intermediate host
18 cause anaphylaxis
19
20
. 21
. 22
0 23
0 24
0
25
0
26
. 27 •
8
L.odt
s
Su~pl'nd
~
End Block
Item: 21 of - ,• Mark -<J
P~v1ous
[:::> "'I ~ ·· ~
100 ~ N @xt Labl lues N o tes Calculato r

A A
1
A previously healthy 5-year-old boy is brought t o t he pediat rician with a 3-day history of sore throat , conjuncti vit is, rh initis, and cough. His ~~AI
2
mot her explains that more than 10 child ren at his summer camp have similar symptoms, parti cularly conjunct ivitis. No cultures are
3 ordered, and the mother is assured t hat her son's illness will go away on its own. One week lat er, t he mother reports that her son is
4 healthy and back at school.
5
6
Which of the following is the most likely cau sative agent in this child's illness?
7 :
A. Adenovirus
8
9 B. Coxsackie A virus
10 C. Cytomegalovirus
11
D. Herpes simplex virus type 1
12
13 E. Rotavirus
14

15
16
17
18
19
20
. 21
. 22
. 23
• 24
• 25
• 26
. 27

8
Lode.
s
S uspe-nd
8
End Bloc:k
Item: 21 of ~ ,• Mark <::J [:::> ""I ~· ~'j
100 J.. Previous Next LAb faiUI~S Notes Calculator

1
The correct a nswer is A. 82°/o chose t h is.
2
This patient is exhibiting feat ures of pharyngoconj unctival feve r caused by infection wit h adenovirus. Pharyngoconj unct ival fever symptoms
3 include benign fo llicular conjunct ivitis with a febrile pharyngit is and cervical adenit is. These out breaks commonly occur in kids in summer
4 cam ps and are typically associated wit h swimming pools or lakes. Epidemic keratoconjunct ivit is ("pink eye" ) is anot her syndrome t hat can
5
occur in children infected wit h adenovirus and causes a more severe disease charact erized by bilateral conj unct ivit is with preauricular
adenopat hy and subsequent painful corneal opacit y development . This syndrome is most commonly self- lim ited but can t ake up to 4 weeks t o
6 fully resolve. Epidemic kerat oconj unctivit is may spread in hospitals and medical off ices v ia cont aminated eye drops or unwashed hands of
7 medical staff. other adenovirus infections include acut e diarrheal illnesses, acut e hemorrhag ic cyst it is, meningitis, encephalit is, myocardit is,
8
and disseminat ed infect ion. Adenovirus is also the fo urt h most common cause of childhood respi ratory t ract infections aft er respi ratory
syncyt ial virus, parainf luenza, and rh inovirus. It is a naked, icosahedral, double-stranded linear DNA that resu lts in a self- limit ed illness that
9 requires no t reatment.
10 Rhinovirus Hospital-acquired infection Keratoconjunctivitis Myocarditis Conjunctivitis Pharyngitis Adenoviridae Meningitis Human respiratory syncytial virus Lymphadenopathy Urinary tract infection

11 Respiratory tract Encephalitis Hemorrhagic cystitis Virus Fever Cervical lymphadenopathy Human parainfluenza viruses Respiratory tract infection Infection DNA Epidemic

12 B is not correct. 7% chose t his .


13 Coxsackie A virus causes cold symptoms and rashes. It is also the causat ive agent of herpangina and hand, foot, and mout h ( HFM) disease.
14 Children with HFM present with fever and vesicles on t he buccal mucosa and tongue, along with peripherally distributed cut aneous lesions on
15 hands, feet , and buttocks.
Hand, foot and mouth disease Herpangina Coxsackie A virus Virus Oral mucosa Mucous membrane Fever Vesicle (biology and chemistry)
16
17 C is not correct. 3 °/o chose this .
Cytomegalovirus (CMV) can react ivate and cause a variety of illnesses in t he immunocom promised but is usually asympt omat ic in healt hy
18
individuals. I n sympt omat ic patients, CMV can manifest with symptoms resem bling mononucleosis, wit h prolonged fevers, and, in less t han a
19 t hird of cases, dermat olog ic manifestations such as a rash.
20 Cytomegalovirus Infectious mononucleosis Immunodeficiency Asymptomatic Rash Human cytomegalovirus Dermatology

21 D is not correct. 1 °/o chose t his.


. 22 Herpes simplex virus t ype 1 (HSV l ) causes gingivostomatit is, herpetic kerat it is of t he eye, and encephalit is. Children with oral HSVl can
• 23 present with fever, pharyngit is, and painful vesicular lesions. HSV kerat itis has an acut e onset and manifests wit h pain, visual blurring, and
• 24
discharge .
Herpes simplex virus Pharyngitis Keratitis Encephalitis Herpes simplex Virus Fever Vesicle (biology and chemistry)
• 25
• 26
E is not correct. 7°/o chos e this .
Rotavirus is the most common cause of diarrhea in infants younger t han 3 years old. Typically, children wit h rot avirus present wit h vomit ing,
. 27

8
Lock
s
Suspend
0
End Block
Item: 21 of ~ ,• Mark <::J [:::> ""I ~· ~'j
100 J.. Previous Next LAb faiUI~S Notes Calculator

1
E is not correct. 7 °/o chos e this .
2
Rotavirus is the most common cause of diarrhea in infants younger t han 3 years old. Typically, children wit h rot avirus present wit h vomit ing,
3 nonbloody diarrhea, and fever.
Rotavirus Diarrhea Vomiting Fever
4
5
6 Bottom Line:
7 Adenovirus is a double-st randed, linear DNA virus t hat causes a highly contageous, self-lim it ed pharyngoconj unctival feve r characterized by
8 benign follicu lar conj unctivit is wit h a febrile pharyngitis and cerv ical adenit is.
DNA virus Adenoviridae Conjunctivitis Pharyngitis Virus Fever DNA
9
10
11

12
l@l;fil·1i•l for year:[2017
FIRST AID FACTS .
•j .

13
14 FA17 p 160.1
15 DNA viruses
16 VIRAL FAMILY ENVELOPE DNA STRUCTURE MEDICAL IMPORTANCE
17
Herpesviruses Yes OS and linear See Herpesviruses entry
18
19 Poxvirus Yes OS and linear Smallpox eradicated world wide by use of the live-
20 (largest 0 A virus) attenuated vaccine
21 Cowpox ("mi lkmaid blisters")
. 22 Molluscum contagiosum -Aesh-colored papule with
• 23 centra I umbiIication
• 24
Hepadnavirus Yes Partially OS and circular HBV:
• 25
• cute or chronic hepatitis
• 26
• I ot a retrovirus but has re,·erse transcriptase
. 27

8
Lock
s
Suspend
0
End Block
Item: 21 of - ,• Mark -<] 1:> ""'I ~· 1!';:'1
100 ~ Prev1o u s Next Labf a lu es Notes Calculator

1 • FA 17 p 160o1 •
2 DNA viruses
3
VIRAL FAMILY ENVELOPE DNA STRUCTURE MEDICAL IMPORTANCE
4
Herpesviruses Yes OS and linear See Hcrpesviruses entry
5
6 Poxvirus Yes OS and linear Smallpox eradicated world wide by use of the li,·e-
7 (largest Oi'\A virus) attenuated vaccine
8 Co" pox ("milkmaid blisters")
9 Molluscum contagiosum- flesh-colored papule with
10 cent ral umbilication
11
Hepadnavirus Yes Partially OS and circular I IBV:
12
Acute or chron ic hepatitis
13
lot a retrovirus but has re,·erse transcriptase
14

15 Adenovirus I 0 OS and linear Febrile pharyngitis r.J-sore throat


16 Acute hemorrhagic cystitis
17 Pneumonia
18 Conju nctivitis-"pink eye"
19
20
21
0
22
0 23
24 Papillomavirus II P - warts (scrotrpes 1, 2, 6, 11), Cll , cervical
0
No OS and circular
0
25 cancer (most commonly 16, 18)
0
26
Polyomavirus 0 OS and circular JC 'irus-progressivc multifocalleukocncephalopathy •
0
27
8
L.odt
s
Su~pl'nd
~
End Block
Item: 21 of - ,• Mark -<J
P~v1ous
[:::> "'I ~ ·· ~
100 ~ N @xt Labl lues N o tes Calculato r
A A
1
BK vims- tramplant patients, commonly targets kidney
2
JC: Junky Cerebrum; BK: Bad Kidney
3

4
Parvovirus No SS and linear Bl 9 virus-aplastic crises in sickle cell disease,
5
(smallest 0 1 virus) "slapped cheek" rash in children (erythema
6 infcctiosum, or fifth disease)
7 RBC destruction in fetus leads to hydrops fetalis
8 and death, in adults leads to pure RBC aplasia and
9 rheumatoid arthritis-like symptoms
10
11
FA17 p 159.5
12
13 DNA virus Some general rules-all 0 A \'iruscs:
14 cha racte rist ics
GENERAL RULE COMMENTS
15
Are IHIAPPPPr " iruses llepadna, llerpes, Adeno, Pox, Parvo,
16
17
Papilloma, Polyoma.
18 Arc double stranded Except pan·o (single stranded).
19
I lave li near genomes Except papilloma and polyoma (circular,
20
supercoiled) and hepadna (circular,
21
incomplete).
. 22
. 23 Are icosahedral Except pox (complex).
• 24 Replicate in the nucleus Except pox (carries own Dr A-dependent R1 A
• 25 polymerase).
• 26
. 27

8
Lode.
s
S uspe-nd
8
End Bloc:k
Item: 22 of - ,• Mark -<J [:::> "'I ~ · ~
100 ~ P~v1ous N@xt Labl lues Notes Calculator

1
A 32-year-old man with ast hma is admitt ed to t he hospit al in status asthmaticus and is given a continuous albuterol nebulizer. Four hours
2
into his hospital course he experiences acute respi rat ory fa ilure; he is intubated and transferred to the inten sive care unit. Aft er 96 hours
3 of mechanical ventilation, the physician notes he requires more ventilatory support and has developed a fever and purulent
4 tracheobronchial secretions. X-ray of t he chest revea ls a right lower lobe infiltrate. Tracheobronchial aspiration y ields a sample that is sent for
analysis.
5
6
Which of the following is most likely to be the cause of this patient's symptoms?
7
:
8
A. Legionella pneumophila
9
10
B. Candida albicans
11 C. Clostridium difficile
12
D. Pseudomonas aeruginosa
13
14
E. Staphylococcus epidermidis
15
16
17
18
19
20
21
. 22
. 23
• 24
• 25
• 26
. 27

8
Lode.
s
Suspe-nd
8
End Bloc:k
Item: 22 of ~ ,• Mark <::J [:::> ""I ~· ~'j
100 J.. Previous Next LAbfaiUI~S Notes Calculator

1
2
The correct a nswer is 0. 65°/o chose t his.
3 This patient has likely developed vent ilat or-associat ed pneumonia (VAP) a common complication among patient s requiri ng ventilation. In
4 order for VAP t o occur~ at least 48 hours of mechanical vent ilation is req uired . Bot h chronic lung disease and convalescence in the int ensive
5
care unit are risk fact ors fo r the development of this infect ion, which is most commonly caused by t he gram-negat ive bacillus Pseudomonas
aerugmosa.
6 Pseudomonas aeruginosa Gram-negative bacteria Ventilator-associated pneumonia Pneumonia Mechanical ventilation Chronic obstructive pulmonary disease Pseudomonas Intensive care unit
7 Intensive care medicine Respiratory disease Bacillus Lung Bronchopulmonary dysplasia
8
A is not correct . 16% chos e this .
9
Legionel/a pneumophila is an aerobic, gram -negative bacilli t hat is ubiq uitous in natural and man-made wat er environments, including hospital
10 air condit ion ing and hot water distri but ion systems. It is t he causat ive organ ism of Pontiac fever, a flu-like illness, and Leg ionnaires' disease,
11 an acut e, severe pneumonia. It is relat ively common cause of healthcare -associat ed pneumonia.
Legionella pneumophila Legionella Gram-negative bacteria Pontiac fever Legionnaires' disease Pneumonia Hospital-acquired pneumonia Bacilli Aerobic organism Fever
12
13 B is not correct. 7% chose t his .
14 Candida albicans is a ubiquit ous fungus and t he most common fungal pathogen affecting humans. It is a cause of many different kinds of
infections, including vulvovaginit is, pneumonia, and endophthalmitis (infection of the v it reous and ant eri or chamber of the eye). I nfect ion with
15
C. albicans is associated wit h parent eral hyperalimentat ion.
16 Candida albicans Pneumonia Candida (fungus) Endophthalmitis Pathogen Fungus Infection Vaginitis
17
C is not correct. 3 °/o chose this .
18
Clostridium difficile is a gram-posit ive, anaerobic, spore-fo rm ing bacillus t hat frequent ly causes antibiot ic associat ed diarrhea and abdomina l
19 cramping and occurs in approximat ely 20% of hospitalized patients. Approximat ely 10% of cases of C. difficile diarrhea can be complicat ed by
20 pseudomembranous colit is.
Clostridium difficile colitis Gram-positive bacteria Antibiotics Diarrhea Colitis Antibiotic-associated diarrhea Anaerobic organism Clostridium Clostridium difficile (bacteria) Bacillus Abdominal pain
21
Bacillus (shape)
22
• 23 E is not correct. 9°/o chose this .
• 24 Staphylococcus epidermidis is a coagulase-negat ive, gram -posit ive organism that is a component of norma l skin f lora. It is a f requent cause of
infections in indwelling prost hetic devices, int ravenous lines, and Foley cathet ers, as well as being a blood culture cont aminant .
• 25
Skin flora Staphylococcus epidermidis Gram-positive bacteria Blood culture Staphylococcus Intravenous therapy Catheter Prosthesis Organism Flora
• 26
. 27

8
Lock
s
Suspend
0
End Block
Item: 22 of ~ ,• Mark <::J [:::> ""I ~· ~'j
100 J.. Previous Next LAb faiUI~S Notes Calculator

1
2
Bottom Line:
3
Pseudomonas aeruginosa is t he organism most commonly associat ed with vent ilat or-associat ed pneumonia; however, ot her gram - negative
4
and gram-posit ive organisms cause ventilator-associat ed pneumon ia.
5 Pseudomonas aeruginosa Pneumonia Ventilator-associated pneumonia Pseudomonas Organism

6
7
8 141;fil·1i•J for year:[ 2017
FIRST AID FA CTS .
•j .
9
10
FA17 p 181 .2
11
Nosocomial infections E coli (UTI) and S aureus (wound in fection) are the two most common causes.
12
RISK FACTOR PATHOGEN UNIQUESIGNS/SYMPTOMS
13
14 Antibiotic use Clostridiu m difficile Watery diarrhea, leukocytosis
15 Aspiration (2° to Polymicrobial, gram 8 bacteria, often Right lower lobe infiltrate or right upper/
16 altered mental status, anaerobes midd le lobe (patient recumbent); purulent
17 old age) malodorous sputum
18
Decubitus ulcers, S au reus (including M RSA), gram 8 anaerobes Erythema, tenderness, induration, drainage
19
20
surgical wounds, (Bacteroides, Prevotella, Fusobacterium) from surgica l wound sites
21 drains
22 Intravascular catheters S aureus (including MRSA), S epidermic/is (long Erythema, induration, tenderness, drainage
• 23 term), Enterobacter from access sites
• 24
Mechanical ventilation, Late onset: P aeruginosa, Klebsiella, ew infiltrate on CXR, t sputum production;
• 25
endotracheal Acinetobacter, S aureus sweet odor (Pseudomonas)
• 26
intubation
. 27
8
Lock
s
Suspend
0
End Block
Item: 22 of - ,• Mark -<J [:::> "'I ~ · ~
100 ~ P~v1ous N@xt Labl lues Notes Calculator

1 A A

FA17 p 139.3
2
3 Pseudomonas Aerobic, motile, gram 8 rod. 1 on-lactose Treatments include "CA.\lPF IRE" drugs:
4 oeruginoso fermenting, oxidase$. Produces p)ocyanin Carbapencm~
5 (blue-green pigment fJ}; has a grape-!ike odor. \rninoglycosides
6 Produces endotoxin (fever, shock), c-:oloxin A \lonobactams
7 (inacti\·ates EF-2}, phospholipase C (degrades Polymyxins (eg, polymyxin B, colistin)
8 cell membranes), and pyocyanin (generales F luoroquinolones (eg, ciproAoxacin,
9 reacti\·e oxygen species). levofloxacin)
10 Th IRd- and fourth-generation
PSEUD0.\10:\.\S is associated'' ith:
11
P neumon ia, pyocpnin cephalosporins (eg, cefta zidime, cefepime)
12
Sepsis Jo::, tcnded-spectru m penicillins (eg,
13
Ecthyma gangrenosum piperacillin, ticarcillin)
14
UTls Acrug inosa-acrobic.
15
D iabetes, drug use Mucoid polysaccharide capsule may contribute
16
• Osteomyelitis (eg, puncture wounds) to chronic pneumonia in cystic fibrosis patients
17
18
• ~ Iucoid polysaccharide capsule due to biofilm formation.
19
• O titis externa (swimmer's car) Can cause wound infection in burn victims.
20 • Nosocomial infections (catheters, Cornea l u lcers/keratitis in contact lens wearers/
21 equipment) minor eye trauma.
22 Exotoxin A Freque ntly found in water -+ hot tub fo lliculitis.
0
23 Skin infections (hot tub folliculitis) Ecthyma gangrenosum- rapidly prog ressi\ e,
0 24 necrotic cutaneous lesion I}) caused by
0
25 Pseudomonas bacteremia. Typically seen in
0
26 immunocompromised patients.
0
27 •
8
Lode.
s
Suspe-nd
8
End Bloc:k
Item: 22 of - ,• Mark -<] 1:> ""'I ~· 1!';:'1
100 ~ Prev1o u s Next Labf a lu es Notes Calculator

1 • •
FA17 p 134.2
2
3 Clostridia (with Gram EB, spore-forming, obligate anaerobic rods.
4 exotoxins)
5 Ctetani Produces tetanospasmin, an exotoxin causing Tetanus is tetanic paralysis.
6 tetanus. Tetanus toxin (and botulinulll toxin)
7 are proteases that clea,·e SJ ARE proteins for
8
neurotransmitters. Blocks release of inhibitory
9
neurotransmitters, GABA and glrcine, from
10
Renshaw cells in spinal cord.
11
Causes spastic paralysis, trismus (lockjaw), risus
12
sardonicus (raised eyebrows and open gri n),
13
14
opisthotonos (spasms of spinal extensors).
15
Pre' enl with tetanus vaccine. Treat" ith
16
antitoxin +1- vaccine booster, diazepam (for
17
muscle spasms), and wou nd debridement.
18 Cbotulinum Produces a heat-labile toxin that inhibits Symptoms of botulism (the 4 D 's}: D iplopia,
19 ACh release at the neuromuscular june!ion, O)rsarthria, Dysphagia, D yspnea.
20 causi ng botulism. In adults, disease is caused Botulinum is from bad bottles of food , juice, and
21 by ingestion of preformed toxin. In babies, honey (causes a descending flaccid paralysis).
22
ingestion of spores (eg, in honey) leads to Local botox injections used to treat foca l
23
0
disease (flopp~ baby synd rome). Treat with dyston ia, achalasia, and muscle spasms. Also
0 24
antitoxin. used for cosmetic reduction offacial wrinkles.
0
25

-
0
26 C perfringens Produces a toxin (lecithinase, a pho)pholipase) Perfringens perforates a gangrenous leg.
. 27 - that can cause nwonecrosis (eas eanf!renc f.l) •
8
L.odt
s
Su~pl'nd
~
End Block
Item: 22 of - ,• Mark -<] 1:> ""'I ~· 1!';:'1
100 ~ Prev1o u s Next Labf a lu es Notes Calculator

1 • Cbotulinum Produces a heat-labile toxin that inhibits Symptoms of botulism (the 4 D's): Diplopia, •
2 ACh release at the neuromuscular junction, Dysarthria, Dysphagia, Dyspnea.
3 causing botulism. In adults, disease is caused Botulinum is from bad bottles of food, juice, and
4
by ingestion of preformed toxin. In babies, honey (causes a descending flaccid paralysis).
5
ingestion of spores (eg, in honey) leads to Local botox injections used to treat focal
6
disease (flopp~ baby syndrome). Treat" ith dystonia, achalasia, and muscle spasms. Also
7
antitoxin. used for cosmetic reduction of fa cial wrinkles.
8
9 Cperfringens Produces a toxin (lecithinase, a phospholipase) Perfringens perforates a gangrenous leg.
10 that can cause myonecrosis (gas gangrene )
11 and hemolysis.
12 Spores can sun·i,·e in undercooked food;
13 when ingested, bacteria release heat-labile
14 enterotoxin - food poisoning.
15
16
17
18 Produces 2 toxins. Toxin A, an enterotoxin, /)i{{icile causes diarrhea. Treatment:
C difflcile
19
binds to brush border of gut and alters Auid metronidazole or oral vancomycin. For
20
secretion. Toxin 13, a cytotoxin, disrupts recurrent cases, consider repeating prior
21
cytoskeleton via actin depolymerizalion. Both regimen, fidaxomicin, or fecal microbiota
22
toxins lead to diarrhea - pseudomembranous transplant.
23
0

0 24
zo
colitis (lJ. Often to antibiotic usc, especially
0
25
clindamycin or ampicillin; associated with PPI
0
26
use. Diagnosed br detecting one or both toxins
. 27 in stool b,· antigen detection or PCR. •
8
L.odt
s
Su~pl'nd
~
End Block
Item: 23 of - ,• Mark -<J [:::> "'I ~ · ~
100 ~ P~v1ous N @xt Labl lues N o tes Calculato r

1
A 5-year-old child with a history of recurrent respirat ory infections presents to the emergency depart ment with a 2-day hist ory of fever
2
(38 .3° C [101 ° FJ), sore throat, nasal congestion, and violent coughing. The physician notes that the child has a loud, "barking" cough. On
3 auscultation, there is an inspiratory st ridor. Between coughing spells, the child is noted to have int ercostal ret ractions.
4
5 What is the RNA genome structure of the most common virus causing this child's illness?
6 :
7 A. Double-stranded, linear
8 B. Single-stranded, negative-sense, circular
9
C. Single-stranded, negative-sense, linear, nonsegmented
10
11 D . Single-stranded, negative-sense, linear, segmented
12 E. Single-stranded, positive-sense, linear, nonsegmented
13
14

15
16
17
18
19
20
21
22
. 23
• 24
• 25
• 26
. 27

8
Lode.
s
S uspe-nd
8
End Bloc:k
Item: 23 of ~ ,• Mark <::J [:::> ""I ~· ~'j
100 J.. Previous Next LAb faiUI~S Notes Calculator

1
The correct a nswer is C. 66°/o chose this .
2 This child's symptoms, especially t he " bark ing " cough and t he f inding of inspirat ory st rid or, are consistent wit h a diagnosis of croup, which is
3 caused by the parainfluenza virus of t he Paramyxoviridae fami ly. Paramyxoviruses have an envelope and a helical capsid . They are single-
4
stranded, negative-sense, l i nea r~ nonsegmented RNA viruses. Other illnesses caused by paramyxoviruses include resp iratory syncytial virus,
measles, and m umps. A mnemonic for circula r RNA viruses: D hey Are Balls (Deltavirus, Arenavirus, Bunyavirus) . All other RNA viruses apart
5 f rom t hese three are linear.
6 Bunyaviridae Hepatitis 0 Paramyxoviridae Arenavirus Measles Capsid Human respiratory syncytial virus Mumps Croup Sense (molecular biology) Negative-sense Human parainfluenza viruses RNA virus

7 Circular RNA Virus Cough Stridor RNA Helix

8
A is not correct . 4 °/o chose this .
9 Reoviruses are double-stranded, linear, segmented RNA viruses. They have an icosahedral capsid and lack an envelope. Bot h Coltivirus (t he
10 cause of Colorado t ick feve r) and Rotavirus (t he most common cause of diarrhea in children worl dwide) are f rom t he Reoviri dae fami ly.
Colorado tick fever Reoviridae Capsid Coltivirus Diarrhea RNA virus Rotavirus Virus Tick RNA Fever Regular icosahedron
11

12 B is not correct. 5% chose t his .


13 Delta v irus (hepatit is D virus) is a single-stranded, negative-sense, circular virus. It has an envelope and a helical capsid. Hepat it is D virus
infection is dependent on concomit ant hepat it is B virus infection. When present, it worsens the signs and symptoms of hepatitis B virus
14
infection .
15 Hepatitis 0 Capsid Hepatitis 8 Hepatitis Hepatitis 8 virus Sense (molecular biology) Negative-sense Virus Helix Herpes 8 virus

16
D is not correct. 16% c hose this .
17
Orthomyxoviruses are sing le-stranded, negat ive -sense, linear, segmented RNA viruses. They possess an envelope and helical capsid .
18 I nf luenza virus, a common cause of "f lu," is t he most prominent member of t his viral fa mily.
19 Capsid Ortholll(Xoviridae Sense (molecular biology) Negative-sense Influenza Virus RNA virus RNA Helix

20 E is not correct. 9°/o chos e this .


21 Caliciviruses are single-st randed, positive-sense, l i nea r~ nonsegmented RNA viruses. They are icosahedra l in shape and lack an envelope.
22 Norwalk virus (a cause of gast roent eri tis) is one of th e best known caliciviruses.
Norwalk virus Gastroenteritis Caliciviridae Regular icosahedron Sense (molecular biology) Positive-sense Virus RNA virus RNA Norwalk, Ohio Icosahedron
23
• 24

• 25 Bottom Line :
• 26 "Barking" cough is found in croup, which is caused by parainf luenza virus (a single-stranded, negative-sense, l i nea r~ nonsegmented RNA
. 27 virus) .
8
Lock
s
Suspend
0
End Block
Item: 23 of - ,• Mark -<J [:::> "'I ~ · ~
100 ~ P~v1ous N @xt Labl lues N o tes Calculato r

1 A
FA17 p 165.3
A

2
3
Paramyxoviruses Paramyxo\'iruses cause disease in children. They include those that cause parainfluenza (croup:
4
seal-like barking cough). mumps, measles, RS , and human metapncumo,·irus, which causes
5
respiratory tract infection (bronchiolitis, pneumonia) in infants. All contain surface F (fusion)
6 protein, which causes respiratory epithelial cells to fuse and form multinucleated cells.
7 Pali\'izumab (monoclonal antibod) against F protein) prevents pneumonia caused by RSV
8 infection in premature infants.
9 Pali, izumab for Paramyxovirus (R V) Prophyla, is in Premies.
10
11
FA17 p 163.1
12
RNA viruses
13
VIRAL FAMILY ENVELOPE RNA STRUCTURE CAPSID SYMMETRY MEDICAL IMPORTANCE
14
15
Reoviruses 0 OS linear Icosahedral Coltivirus3 - Colorado tick fever
16 10- 12 segments (double) Rotavirus-causc of fatal diarrhea in children
17 Picornaviruses l 0 SS ® linear Icosa hedral Poliovirus-polio-Salk/Sabin vaccines-!PV/OPV
18 Echovirus-aseptic meningitis
19 Rhi novirus-"common cold"
20 Coxsackicvirus-aseptic meningitis; herpangina
21 (mouth blisters, b ·er); hand, foot, and mouth
22 disease; myocarditis; pericarditis
23 II V -acute ,-iral hepatitis
' 24 PERCII
' 25
Hepevirus I o SS ® linear Icosahedral HEV
' 26
. 27 • C ;:~ lirivi ruc;pc; i n <;:<;: t+\ lin P'> r l rn~::~hNir::~ l n rnvi rn ~ -vi r::1 I o::~drnPntPriti ~

8
Lode.
s
S uspe-nd
8
End Bloc:k
Item: 24 of - ,• Mark -<J [:::> "'I ~ · ~
100 ~ P~v1ous N @xt Labl lues N o tes Calculato r

A A
1
A 23-year-old woman returns f rom a week swimming at th e beach and presents to her physician with a complaint of pain in her ri ght ear. ~~AI
2
Her temperature is 38.7°C ( 101.7°F), pulse is 98/min, and blood pressure is 128/85 mm Hg. Physical examination reveals an area of
3 redness extending from her right ear toward her rig ht mandible that is warm to the touch. There is tenderness on manipulation of the right
4 pinna but not of the left. The right tympanic membrane is not visualized because of pain, but the left tympanic membrane is normal. The
patient's oropharynx is without exudates or erythema.
5
6
What is the most likely pathogen causing t hese symptoms?
7
:
8
A. Haemophilus influenzae
9
10
B. Moraxella catarrhalis
11 C. Pseudomonas aeruginosa
12
D. Staphylococcus aureus
13
14
E. Streptococcus pneumoniae
15
16
17
18
19
20
21
22
23
. 24
• 25
• 26
. 27

8
Lode.
s
S uspe-nd
8
End Bloc:k
Item: 24 of ~ ,• Mark <::J [:::> ""I ~· ~'j
100 J.. Previous Next LAbfaiUI~S Notes Calculator

1
2
The correct answer is C. 56°/o chose this.
3 The pat ient has ot it is ext erna, which often occurs after swimming. Ot it is ext erna typically present s with ear pain, prurit is, discharge and pain
4 on manipulation of t he pinna. Periauricular cellu lit is and feve r are signs of more serious infection. As opposed t o ot it is med ia, t here is t ypically
5
no evidence of m iddle ear fluid. Pseudomonas aeruginosa is t he most common cause of ot it is ext erna (39% of cases), fo llowed by
Staphylococcus epidermidis (around 9% of cases) .
6 Pseudomonas aeruginosa Staphylococcus epidermidis Otitis media Otitis externa Cellulitis Auricle (anatomy) Itch Middle ear Staphylococcus Pseudomonas Otitis Ear pain Fever
7
A is not correct. 10% chose this.
8
This organism causes ot it is interna, not ot it is extern a. Additionally, as the seroty pes t hat cause ot it is are not Haemophilus influenzae type B,
9 t he HiB vaccine offers lit tle protection against ot it is cause by t his organism.
10 Haemophilus influenzae Otitis externa Labyrinthitis Otitis vaccine Organism

11 B is not correct. 10% chose this.


12 Moraxella catarrhalis is a common cause of otit is interna. Most strains produce 13-lact amase.
13 Moraxella catarrhalis Labyrinthitis Moraxella Otitis

14 D is not correct. 10% chose this.


15 Although Staphylococcus aureus can indeed cause ot it is ext erna, it is a much less commonly involved than P. aeruginosa (only 8% of cases).
16 Although S. aureus could indeed be responsible fo r t he pa t ient's cellulit is, breakdown of t he skin barri er is typically required for its
development and is not observed in this scenario.
17
Staphylococcus aureus Otitis externa Cellulitis Pseudomonas aeruginosa Staphylococcus Otitis Epidermis
18
19
E is not correct. 14°/o chose this.
Streptococcus pneumoniae is a gram- posit ive organism t hat is t he most common cause of ot it is media, not ot it is externa . Although it can
20
indeed cause otit is externa, S. pneumonias is a much less common causal organism than P. aeruginosa (only observed in around 9% of
21 cases).
22 Streptococcus pneumoniae Otitis externa Gram-positive bacteria Pseudomonas aeruginosa Streptococcus Otitis Organism

23
24 Bottom Line:
• 25
Pseudomonas aeruginosa is t he most common cause of ot it is ext erna, especially after swimming .
• 26 Pseudomonas aeruginosa Otitis externa Pseudomonas Otitis

. 27

8
Lock
s
Suspend
0
End Block
Item: 24 of - ,• Mark -<J [:::> "'I ~ · ~
100 ~ P~v1ous N @xt Labl lues N o tes Calculato r

1 A
FA17 p 139.3
A

2
3
Pseudomonas Aerobic, motile, gram 8 rod. 1 on-lactose Treatments include "CAMPFIRE" drugs:
4
aeruginosa fermenting, oxidase$. Produces p)ocyanin C arbapencms
5 (blue-green pigment fJ); has a grape-) ike odor. \ minoglycosides
6 Produces endotoxin (fever, shock), exotoxin A \Jonobactams
7 (inactivates EF-2), phospholipase C (degrades Polymyxins (eg, pol~myxin B, colistin)
8 cell membranes), and pyocyanin (generates F luoroquinolones (eg, ciprofloxacin ,
9 reactive oxygen species). le,·o Aoxacin)
10 PSEUD0~10:'\A S is associated with: Th IRd- and fourth-generation
11 Pneumonia, pyocyanin cephalosporins (eg, ceftazidime, cefcpime)
12 Sepsis E xtended-spectrum penicillins (eg,
13 Ecthyma gangrenosum piperacillin, ticarcillin)
14 Aeruginosa-aerobic.
UT ls
15
Diabetes, drug use Mucoid polysaccharide capsule m<l)' contribute
16
• Osteomrelitis (eg, puncture wounds) to chronic pneumonia in cystic fibrosis patients
17
• \lucoid polysaccharide capsule due to biofilm formation.
18
• O titis externa (swimmer's ear) Can cause wound infection in burn victims.
19
• Nosocom ial infections (catheters, Cornea l u lcers/keratitis in contact lens wearers/
20
equipment) minor eye trauma.
21
22
Exotoxin A Frequently found in water ..... hot tub fo lliculitis.
23 Skin infections (hot tub folliculitis) Ecthyma gangrenosum- rapidly progressi,·e,
24 necrotic cutaneous lesion 1J caused by
• 25 Pseudomonas bacteremia. Typically seen in
• 26 immunocompromised patients.
. 27 •
8
Lode.
s
S uspe-nd
8
End Bloc:k
Item: 25 of - ,• Mark -<J [:::> "'I ~ · ~
100 ~ P~v1ous N @xt Labl lues N o tes Calculato r

A A
1
A 27-year-old homeless man presents to t he clinic because of a 5-day history of pain and swell ing in his right upper arm . MRI of the area ~~AI
2
reveals diffuse soft tissue and bone inflammation . Results of bone biopsy are shown in t he image. Blood culture speciation is pending, but
3 preliminary results have grown out a gram-negative, oxidase-positive rod that does not ferment lactose.
4
5
6
7
8
9
10
11

12
13
14

15
16
17
18
19 Which of the fol lowing would a complet e hist ory of this patient most likely reveal?
20 :

21
A. Corticosteroid use
22 B. Intravenous dru g use
23
C. Miliary tuberculosis
24
. 25
D. Multiple sexual partners
• 26 E. Sickle cell anemia
. 27

8
Lode.
s
S uspe-nd
8
End Bloc:k
Item: 2S of ~ ,• Mark <::J [:::> ""I ~· ~'j
100 J.. Previous Next LAbfaiUI~S Notes Calculator

1
2
3
The correct answer is B. 58% chos e this.
4
This patient has osteomyelit is, commonly caused by Pseudomonas aeruginosa (gram-negat ive aerobic rod t hat produces a green pigment) in
5 intravenous drug users. The image shows a bone abscess wit h polymorphonuclear leukocyt es and foci of degraded bone collagen. Treatment
6 would require several weeks of ant ipseudomonal ant ibiot ics such as ticarcillin or piperacillin.
Ticarcillin Piperacillin Osteomyelitis Pseudomonas aeruginosa Gram-negative bacteria Collagen Abscess White blood cell Intravenous therapy Pseudomonas Antibiotics Aerobic organism Pigment
7
8 A is not correct. 7°/o chose this.
9 Chronic cort icosteroid use can lead to immunosuppression and ost eoporosis, but does not predispose one to pseudomonal osteomyelit is.
Corticosteroid Osteoporosis Osteomyelitis Immunosuppression Pseudomonas
10
11 C is not correct. 4 °/o chose this .
12 Pat ients wit h miliary Mycobacterium tuberculosis infection are at ri sk for vertebral ost eomyelit is, known as Pott disease. I t is uncommon for
t ubercular ost eomyelitis t o affect the humerus.
13 Osteomyelitis Mycobacterium tuberculosis Pott disease Humerus Tuberculosis Mycobacterium Vertebral osteomyelitis Infection
14
D is not correct. 8°/o chose this.
15
Mult iple sexual partners would put t he patient at increased ri sk of Neisseria gonorrhoeae infect ion, which can cause osteomyelitis. More
16 commonly, t he patient would have urethri tis, epidydim it is, or perhaps Fitz- Hugh-Curtis syndrome (for fema le pat ients), which is the fo rmation
17 of postinfection adhesions on the liver capsule, leading t o chronic upper rig ht quadrant pain.
Osteomyelitis Neisseria gonorrhoeae Urethritis Neisseria Fitz-Hugh-Curtis syndrome Liver Infection
18
19 E is not correct. 23°/o chose this.
20 Pat ients wit h sickle cell anemia are at risk for ost eomyelitis due t o Salmonella or Staphylococcus, but not to pseudomonal osteomyelit is.
Osteomyelitis Sickle-cell disease Anemia Salmonella Staphylococcus Pseudomonas Sickle
21
22
23 Bottom Line:
24 I nt ravenous drug use is a major risk factor fo r osteomyelit is caused by Pseudomonas .
25 Osteomyelitis Intravenous therapy Drug injection Pseudomonas Risk factor

• 26
. 27

8
Lock
s
Suspend
0
End Block
Item: 2S of ~ ,• Mark <::J [:::> ""I ~· ~'j
100 J.. Previous Next LAb faiUI~S Notes Calculator

1
2
3 i@l;fil·1i•J for year:[ 201 7 • J
FIRST AID FACTS
4
5
FA17 p 176.4
6
7
Osteomyelitis RISK FACTOR ASSOCIATED INFECTION

8 Assume if no other information is available S aureus (most common overall)


9 Sexually active eisseria gonorrhoeae (rare), septic arthritis more
10 common
11

12
Sickle cell disease Salmonella and S aureus
13 Prosthetic joint replacement S aureus and S epidennidis
14
Vertebra I involvement S aureus, Mycobacterium tuberculosis (Pott
15
disease)
16
17
Cat and dog bites Pasteurella multocida
18 IV drug abuse Pseudomo11as, Ca11dida, S aureus are most
19 common
20
Elevated C-reactive protein (CRP) and erythrocyte sedimentation rate common but nonspecific.
21
M Rl is best for detecting acute infection and detail ing anatomic involvement fl. Radiographs are
22
23
insensitive early but can be useful in chronic osteomyelitis rn.
24
25 FA17 p 139.3
• 26
Pseudomonas Aerobic, motile, gram 8 rod. Non-lactose Treatments include "CAMPF IRE" drugs:
. 27
8
Lock
s
Suspend
0
End Block
Item: 25 of - ,• Mark -<J [:::> "'I ~ · ~
100 ~ P~v1ous N @xt Labl lues N o tes Calculato r

1 A
FA17 p 139.3
A

2
3
Pseudomonas e
Aerobic, motile, gram rod. I on-lactose Treatments include "CAMPFIRE" drugs:
4
aeruginosa fermenting, oxidase$. Produces pyocyanin Carbapencms
5 (blue-green pigment fJ); has a grape-) ike odor. \minoglycosides
6 Produces endotoxin (fe\'cr, shock), exotoxin A \Jonobactams
7 (inactivates EF-2), phospholipase C (degrades Polymyxins (eg, polymyxin B, colistin)
8 cell membranes), and pyocyanin (generates F luoroquinolones (eg, ciprofloxacin,
9 reactive oxygen species). le,·ofloxacin)
10 PSEUDO~IO:\AS is associated with: Th IRd- and fourth-generation
11 Pneumonia, pyocyanin cephalosporins (eg, ceftazidime, cefcpime)
12 Sepsis Extended-spectrum penicillins (eg,
13 piperacillin, licarcillin)
Ecthyma gangrenosum
14
UTls Acrugi nosa-aerobic.
15
Diabetes, drug use Mucoid polysaccharide capsule m<lY contribute
16
• Osteomrelitis (eg, puncture wounds) to chronic pneumonia in cystic fibrosis patients
17
• \lucoid polysaccharide capsule due to biofilm formation.
18
• Otitis externa (swimmer's ear) Can cause wound infection in burn victims.
19
• Nosocom ial infections (catheters, Cornea l ulcers/keratitis in contact lens wearers/
20
equipment) minor eye trauma.
21
22 Exotoxin A Frequently found in water ..... hot tub fo lliculitis.
23 Skin infections (hot tub folliculitis) Ecthyma gangrenosum- rapidly progressi,·e,
24 necrotic cutaneous lesion [lJ caused by
25 Pseudomonas bacteremia. Typically seen in
• 26 immunocompromised patients.
. 27 •
8
Lode.
s
S uspe-nd
8
End Bloc:k
Item: 25 of - ,• Mark -<] 1:> ""'I ~- 1!';:'1
100 ~ Prev1o u s Next Labf a lu es Notes Calculator

1 FA1 7 p 137.2
2
3
Gram-negative lab algorithm

Gram 8 (ptnk)
4

5
6
7
Diplococci Coccobacilli Comma-shaped rods
8
9
10
Aerobic HMmOphilus infl~m~ZM Oxidase ~±~
11 (requires factQ(S Vand XJ
12 Maltose utilization Pasteurella
Brucella
13
Bordetella pertussis Grows in 42"C Grows in alkaline media Produces urease
14 Francisella tularens1s
15 N gonorrhoe~e Campyloblderjejuni Vibrio cholerae Helicoblder pylori
N meningitidis
16 Moraxella
17
18
19
20 Bacilli
21
22 lactose fermentation
23
24 r~--~e~----L---~~
25 Oxidase Fast Slow
• 26
. 27 •
8
L.odt
s
Su~pl'nd
~
End Block
Item: 25 of - ,• Mark -<J [:::> "'I ~ · ~
100 ~ P~v1ous N @xt Labl lues N o tes Calculato r
A A
1
2 Aerob•c H1emophilus influenzH Oxidase (±)
3
(requires factors Vand X)
Pasteurella
4
Maltose utilization
Brucella
5 Bordetelll pertussis Grows in 42•c Grows in alkaline media Produces urease
Franasella tularens•s
6
C1mpyloblcterjejuni Vibrio cholerae Helicoblcter pylori
7 N meningitic/is
8
9
10
11

12
13
Lactose fermentation
14

15
r~--~e~----~--~~
16
17
Oxidase Fast] _ Slow

18
Klebs~/1
19 Eco/1
Cttrobacter
H2S production Pseudomonas
Enterobacter
Serratia
20
21 on TSI agar
22
23
24 Sllmonelll
Shi~ll
25 Yersinil Proteus
• 26
Important tests are in bold. Important pathogens are in bold italics.
. 27

8
Lode.
s
S uspe-nd
8
End Bloc:k
Item: 26 of - ,• Mark -<J [:::> "'I ~ · ~
100 ~ P~v1ous N @xt Labl lues N o tes Calcula to r

1
A 10-year boy is brought t o the ER by his mot her due to several days of diarrhea, irritability, fat igue, and pallor. The diarrhea is visibly
2
bloody. The mother denies anyone else in the fami ly with similar complaints. However, she does mention t hat they took a family trip to
3 Mexico where her son ate a lot of food f rom t he local street vendors.
4
5 Which tests, in addition to a complete blood cell count, are most important to perform on this woman 's son?
6 :
7 A. Cardiac enzymes
8 B. Liver enzymes
9
C. Pulmonary function tests
10
11 0 . Renal function tests
12 E. Urine culture
13
14

15
16
17
18
19
20
21
22
23
24
25
. 26
. 27

8
Lode.
s
S uspe-nd
8
End Bloc:k
Item: 26 of ~ ,• Mark <::J [:::> ""I ~· ~'j
100 J.. Previous Next LAb faiUI~S Notes Calculator

1
The correct a nswer is 0. 63°/o chose t his.
2
Enterohemorrhagic Escherichia coli (EHEC) picks up t he Shiga- like toxin via specia lized ( lysogenic) t ransduct ion. EHEC disease usually results
3 in self-lim it ing, bloody diarrhea last ing 5-10 days, but may cause hemolytic uremic syndrome (HUS), especially in young and elderly patient s
4 infected wit h t he 0 157 : H7 seroty pe of E. coli. HUS is characterized by t he t ria d of anemia, t hrom bocytopenia, and acut e re nal failu re. HUS
com plicat es up t o 9% of EHEC infect ions and usually begins 5- 10 days after the onset of diarrhea . HUS is the most common cause of acute
5
renal fa ilure in children in t he United States, and about half of patients will require dia lysis.
6
Common rout es of infect ion of 0 157 : H7 serot ype of E. coli include swimming, person-to -person, animal contact , food born e, and drinking
7 water. Although contaminat ed ground beef was the most common cause of food borne outbreaks, produce-associat ed out breaks are also
8 common (eg, lettuce, sprouts, cabbage, apple cider, apple j uice). These have been at t ribut ed t o fecal cont amination of produce in the fields,
9
f rom wild an ima ls, or from fertilization conta ining human or animal feca l matter.
Escherichia coli Thrombocytopenia Hemolytic-uremic syndrome Shiga-like toxin Acute kidney injury Diarrhea Serotype Feces Verotoxin-producing Escherichia coli Apple juice Toxin Hemolysis Dialysis
10
Transduction (genetics) Anemia Enterohemorrhagic
11

12 A is not correct . 4 °/o chose this.


13 Cardiac enzymes would be useful if a myocardial infa rction or myocardit is is suspected, but not in t his case. The most common cause of
14 infectious myocardit is is a coxsackie B v iral infect ion .
Myocarditis Myocardial infarction Coxsackie 8 virus Coxsackievirus Viral disease Cardiac marker Enzyme Infection Infarction
15
16 B is not correct. 20 % chose this .
17 Li ver enzyme assays such as t hose for AST and ALT would not be useful for a pat ient with suspect ed EHEC, but suspicion of a hepatit is v irus,
such as hepatit is A, would req uire liver enzyme tests.
18 Enzyme Hepatitis A Liver function tests Liver Hepatitis Aspartate transaminase Virus Verotoxin-producing Escherichia coli
19
C is not correct. 3 °/o chose this .
20
The presence of bloody diarrhea suggest s an oral route of infect ion in t he son; even if pulmonary symptoms were present, blood and sputu m
21 t ests to detect infection would be t he f irst tests to consider. There would be no need to carry out any pulmonary test ing in this case.
22 Diarrhea Sputum Dysentery

23 E is not correct. 10°/o chose this.


24 The son has likely been infect ed by way of the oral route, result ing in bloody diarrhea . A urine cultu re would not be t he most helpful test in
25 t his sit uation . Uri nary t ract infect ions are much less common in males than in females and t he son is not complaining of any symptoms
particularly indicat ive of a uri nary t ract infect ion.
26
Urinary tract infection Diarrhea Urinary system Urine Urine culture Dysentery
. 27

8
Lock
s
Suspend
0
End Block
Item: 26 of ~ ,• Mark <::J [:::> ""I ~· ~'j
100 J.. Previous Next LAbfaiUI~S Notes Calculator

1
2 Bottom Line:
3 E. coliis a common cause of urinary t ract infection, but can cause a host of other diseases, including bloody diarrhea and HUS, when it
4
obtains Shiga-like toxin from a lysogenic phage.
Urinary tract infection Escherichia coli Diarrhea Shiga-like toxin Toxin Bacteriophage Urinary system Infection
5
6
7
l@l;fil·1i•l f or yea r:[ 2017
FIRST AID FACTS .
•j .
8
9
10 FA17 p 177.1
11 Urinary tract Cystitis presents with dysuria, frequency, urgency, suprapubic pain, and WBCs (but not WBC
12 infections casts) in urine. Primarily caused by ascension of microbes from urethra to bladder. Males-
13
infants with congenital defects, vesicoureteral reAux. Elderly- enlarged prostate. Ascension to
14
kidney results in pyelonephritis, which presents with fever, chi lls, flank pain, costovertebra l angle
15
tenderness, hematuria, and WBC casts.
16
Ten times more common in women (shorter urethras colonized by fecal flora). Other predisposing
17
factors: obstruction, kidney surgery, catheterization, GU malformation, diabetes, pregnancy.
18
19
20 FA17 p 175.1
21 Bugs causing diarrhea
22
Bloody diarrhea
23
24 Campylobacter Comma- or S-shaped organisms; growth at 42°C
25 E histolytica Protozoan; amebic dysentery; liver abscess
26
Enterohemorrhagic 0157:1-17; can cause !IUS; makes Shiga-like toxin
. 27

8
Lock
s
Suspend
0
End Block
Item: 26 of ~ ,• Mark <::J [:::> ""I ~· ~'j
100 J.. Previous Next LAbfaiUI~S Notes Calculator

1 • •
FA17 p 175.1
2
Bugs causing diarrhea
3
4 Bloody diarrhea
5 Campylobacter Comma- or S-shaped organisms; growth at 42°C
6
E histolytica Protozoan; amebic dysentery; liver abscess
7
8 Enterohemorrhagic 0157:H7; can cause IIUS; makes Shiga-like toxin
9 Ecoli
10 Enteroinvasive Ecoli Invades colonic mucosa
11

12
Salmonella (non- Lactose 8; Aagellar motility; has animal reservoir, especially poultry and eggs
13
typhoidal)
14 Shigella Lactose 8; very low 10 ;0; produces Shiga toxin (human reservoir only); bacillary dysentery
15
Yenterocolitica Day care outbreaks, pseudoappendicitis
16
17 Watery diarrhea
18 Cdifficile Pseudomembranous colitis; associated with antibiotics and PPis; occasionally bloody diarrhea
19
C perfringens Also causes gas gangrene
20
21 Enterotoxigenic E coli Tra,·elers' diarrhea; produces heat-labile (LT) and heat-stable (ST) toxins
22 Protozoa Giardia, Cryptosporidium
23
Vcholerae Comma-shaped organisms; rice-water diarrhea; often from infected seafood
24
25 Viruses Rota,·irus, norovirus, adenovirus
26
• •
. 27

8
Lock
s
Suspend
0
End Block
Item: 26 of ~ ,• Mark <::J [:::> ""I ~· ~'j
faiUI~S
100
1
J..
- .. .
Previous
- . Next LAb
. Notes
... Calculator
., . .. ... .
2 Protozoa Giardia, Cryptosporidium
3 Vcholerae Comma-shaped organisms; rice-water diarrhea; often from infected seafood
4
5
Viruses Rota,·irus, norovirus, adenovirus
6
7 FA17 p 177.2
8
UTI bugs
9
SPECIES FEATURES COMMENTS
10
11
Escherichia coli Leading cause of UTI. Colonies show green Diagnostic markers:
12
metallic sheen on EMB agar. (f) Leukocyte esterase= evidence ofWBC
13 Staphylococcus 2nd leading cause of UTI in sexua lly active activity.
14 saprophyticus women. Ef> litrite test = reduction of urinary nitrates
15 by bacterial species (eg, E coli).
Klebsiella pneumoniae 3rd leading cause of UTI. Large mucoid capsule
16 Ef> Urease test = urease-producing bugs (eg,
and viscous colonies.
17 S sa(Jrophyticus, Proteus, Klebsiella).
18 Serratia marcescens Some strains produce a red pigment; often
19 nosocomial and drug resistant.
20 Enterococcus Often nosocomial and drug resistant.
21
Proteus mirabilis Motility causes "swarming" on agar; produces
22
23
urease; associated with struvite stones.
24 Pseudomonas Blue-green pigment and fruity odor; usually
25 aeruginosa nosocomial and drug resistant.
26
. 27 •
8
Lock
s
Suspend
0
End Block
Item: 27 of - ,• Mark -<J
P~v1ous
[:::> "'I ~ ·· ~
100 ~ N @xt Labl lues N o tes Calcula to r

24 A A

A 2-year-old child is brought to his pediat rician by his mother, who states that he has had wat ery diarrhea and abdominal pain for 36 ~~AI
25
hours. The patient attends daycare, and recen tly many children have had similar symptoms. Physical exam is negative except th at the
26 child appears dehydrated with increased capillary refill t ime. The child has not received a complet e set of vaccinations and has v itals within
. 27 normal limits except for mild hypot ension and tachycard ia.
• 28
• 29 Which of the following best describes t he most likely pathogen in this case?
• 30 :
• 31 A. A double stranded, enveloped, linear DNA virus
. 32 B. A double-stranded, non-enveloped, linear-segmented RNA virus
. 33
C. A single-stranded enveloped RNA virus with negative polarity
• 34
• 35 D. A single-stranded enveloped RNA virus wit h positive polarity
' 36
. 37
. 38

' 39
• 40

' 41
• 42
• 43
• 44

' 45
' 46
' 47
. 48
. 49
. 50

8
Lode.
s
S uspe-nd
8
End Bloc:k
Item: 27 of ~ ,• Mark <::J [:::> ""I ~· ~'j
100 J.. Previous Next LAb faiUI~S Notes Calculator

24
25
26
The correct a nswer is B. 57% chose t his .
27
Rotavirus is a common cause of viral gastroenterit is and fat al diarrhea in children bet ween t he ages of 6 months and 2 years. It is part of t he
• 28 reovirus fami ly and is a nonenveloped, double-stranded, linear-segment ed RNA virus. It is worth not ing t hat in more developed count ries,
• 29 norovirus is becoming a more common cause of viral gastroenterit is due to rout ine vaccinat ion against rot avirus.
Rotavirus Norovirus Gastroenteritis Viral envelope Rotavirus vaccine Diarrhea RNA virus Viral gastroenteritis Virus Reoviridae RNA vaccination Developed country
• 30
• 31 A is not correct . 8°/o chose this .
. 32 Poxviruses are double stranded, enveloped DNA viruses. These include smallpox, cowpox, monkeypox, and molluscum contagiosum .
Cowpox Monkeypox Smallpox Molluscum contagiosum Virus Poxviridae DNA DNA virus
. 33
• 34 C is not co rrect. 1 5% chose this .
• 35 Rh abdoviruses are single-stranded enveloped viruses with negative polarity. The neurot ropic Rabies virus is perhaps one of t he most not orious
members of t his family .
• 36
Rabies virus Rhabdoviridae Neurotropic virus Rabies Virus Nervous system
. 37
. 38
D is not correct. 20% c hose this .
Retroviruses are single-st randed, enveloped RNA viruses wit h posit ive polarity. They have reverse t ranscri ptase and include HIV (which causes
• 39
AIDS) and hu man T-lym phocyt e lymphoma virus (which causes T- lymphocyte leukemia) .
• 40 Reverse transcriptase Leukemia T cell Retrovirus HIV Virus HIV/AIDS Lymphoma RNA virus RNA Chemical polarity

• 41
• 42
Bottom Line :
. 43
Rotavirus, a double-stranded, linear-segment ed RNA virus, is a common cause of viral gast roent erit is in children 6 months t o 2 years of age .
• 44 Rotavirus Gastroenteritis Viral gastroenteritis RNA virus Virus RNA
• 45
• 46
• 47 i@l;fil·1i•J for yea r:[2017 • J
FIRST AID FACTS
. 48
. 49
• 50
FA17 p 163.1

8
Lock
s
Suspend
0
End Block
Item: 27 of - ,• Mark -<J
P~v1ous
[:::> "'I ~ ·· ~
100 ~ N @xt Labl lues N o tes Calcula to r

24 A A

25 FA17 p 163.1
26 RNA viruses
27 YIRAlfAMILY ENVELOPE RNA STRUCTURE MEDICAL IMPORTANCE
CAPSID SYMMETRY
• 28
Reoviruses 0 OS linear Icosahedral Coltivirusa- Colorado tick fever
• 29
10-12 segments (double) Rotavirus-cause of fatal diarrhea in children
• 30
• 31 Picornaviruses lo SS $ linear Icosahedral Poliovirus-polio-Salk/Sabin ,·accines-IPV/OPV
. 32 l'.chovirus-aseptic meningitis
. 33 Rhinovirus-"common cold"
• 34 Co\ Sclcl..ievirus-aseptic meningitis; herpangina
• 35 (mouth blisters, b ·er); hand, foot, and mouth
• 36 disease; myocarditis; pericarditis
. 37 II V -acute ,·iral hepatitis
. 38 PERC II
• 39
Hepevirus No SS EB linear Icosa hedral H EV
• 40
Caliciviruses No Icosahedral 1orovirus- viral gastroenteritis
• 41 SS EB linear
• 42 Flaviviruses Yes SS EB linear Icosahedral IICV
• 43 Yellow fever'1
• 44 Dengue·'
. 45 St. Louis encephalitis<t
• 46 \ est 'ile virusa (men ingoencephalitis)
• 47 Zika virus
. 48
Togaviruses Yes SS EB linear Icosahedral Rubella
. 49
Western and Eastern equine encephalitis<t
. 50

8
Lode.
s
S uspe-nd
8
End Bloc:k
Item: 27 of ~ ,• Mark <::J [:::> ""I ~· ~'j
100 J.. Previous Next LAbfaiUI~S Notes Calculator

24 • •
25
FA17 p 175.1
26
Bugs causing diarrhea
27
• 28
Bloody diarrhea
• 29 Campylobacter Comma- or S-shaped organisms; growth at 42°C
• 30
E histolytica Protozoan; amebic dysentery; liver abscess
• 31
. 32
Enterohemorrhagic 0157:1-17; can cause IIUS; makes Shiga-like toxin
. 33 Ecoli
• 34 Enteroinvasive Ecoli Invades colonic mucosa
• 35
Salmonella (non- Lactose 8; Aagellar motility; has animal reservoir, especially poultry and eggs
• 36
typhoidal)
. 37
. 38 Shigella Lactose 8; very low 10 ;0; produces Shiga toxin (human reservoir only); bacillary dysentery
• 39
Yenterocolitica Day care outbreaks, pseudoappendicitis
• 40
• 41
Watery diarrhea
• 42 Cdifficile Pseudomembranous colitis; associated with antibiotics and PPis; occasionally bloody diarrhea
. 43
C perfringens Also causes gas gangrene
• 44
• 45
Enterotoxigenic E coli Tra,·elers' diarrhea; produces heat-labile (LT) and heat-stable (ST) toxins
• 46 Protozoa Giardia, Cryptosporidium
• 47
Vcholerae Comma-shaped organisms; rice-water diarrhea; often from infected seafood
. 48
. 49
Viruses Rota,·irus, norovirus, adenovirus
• 50 • •

8
Lock
s
Suspend
0
End Block
Item: 28 of - ,• Mark -<J [:::> "'I ~ · ~
100 ~ P~v1ous N @xt Labl lues N o tes Calcula to r

24
A 41-year-old woman complains of gnawing, burning pain in her abdomen. The pain is especially pronounced after meals. The patient
25
denies any chest pain, shortness of breath, constipat ion, or diarrhea. Over the past 2 months, the patient claims t o have unintentionally
26 lost 25 lb (11.4 kg). Endoscopic examinat ion is performed, t he results of which are shown in the image.
27
. 28
• 29
• 30
• 31
. 32
. 33
• 34
• 35

• 36
. 37
. 38
• 39
• 40
• 41
• 42
• 43
Which of the following is the most likely cause of t hese findings?
• 44
:
. 45
A. A gram-negative, fl agellated, helical organism
• 46
• 47
B. A long history of acetaminophen intake
. 48 C. Atrial fibrillation
. 49
D. Crohn disease
. 50

8
Lode.
s
S uspe-nd
8
End Bloc:k
Item: 28 of - ,• Mark -<J [:::> "'I ~ · ~
100 ~ P~v1ous N @xt Labl lues N o tes Calcula to r
I :.~ "I I ... I • -

25
26
27
. 28
• 29
• 30
• 31
. 32
. 33
• 34
• 35

• 36
. 37
. 38
• 39
• 40
• 41 Which of the fol lowing is the most likely cause of t hese find ings?
• 42
:
• 43 A. A gram-negative, flagellated, helical organism
• 44
B. A long history of acetaminophen int ake
. 45
• 46 C. Atrial fibrillation
• 47 D. Crohn disease
. 48
E. Linitis plastica
. 49
. 50

8
Lode.
s
S uspe-nd
8
End Bloc:k
Item: 28 of ~ ,• Mark <::J [:::> ""I ~· ~'j
100 J.. Previous Next LAb faiUI~S Notes Calculator

24
25 The correct a ns wer is A. 75°/o chose t h is.
26 Gnawing epigastric pain that is exacerbat ed by meals is suggestive of gastri c ulcer. I n up t o 70% of cases, Helicobacter pylori infection is
27 associat ed with gast ric ulcer. Oft en pat ients will lose weight when they have chronic gast ric ulcer disease because of pain associated wit h food
28
consumption. The image shows t he ty pical lesion : an antral ulcer with regular, rounded edges and a smooth base. Despit e t his classic
appearance, biopsy of t hese lesions is essent ial to exclude gastric adenocarcinoma, which may mimic the appearance of H. pylori-induced
• 29 ulcers. Uncomplicat ed cases of gastric ulcer may be t reated medically by t herapy direct ed at neut ralizing gastri c acid (wit h an H2 antagonist or
• 30 proton pump inhibitor [ PPI]) and eradicat ing H. pylori infect ion using appropriate ant ibiotics as part of a t rip le-therapy regimen. The t riple-
• 31
t herapy regimen f requently consists of a PPI, cl arithromycin, and amoxicillin or metronidazole, although specif ic agents may vary. Bismut h can
be added if quadrup le therapy is needed .
. 32 Proton-pump inhibitor Metronidazole Clarithromycin Helicobacter pylori Amoxicillin Peptic ulcer Gastric acid Bismuth Stomach cancer Adenocarcinoma Abdominal pain Biopsy Stomach Epigastrium Ulcer
. 33 Proton Antibiotics Lesion Pyloric antrum Enzyme inhibitor Helicobacter Infection
• 34
B is not correct. 12% chose this .
• 35
Ce1tain drugs can cause t he development of ulcers. Nonsteroidal ant i- inflammatory drugs (NSAIDs), such as aspiri n, inhibit t he format ion of
• 36 prost agland ins needed to maintain a protect ive mucous layer. When local prost aglandin levels are reduced, the wall of t he stomach is no
. 37 longer protected from gast ric acid and it ulcerates. Acetaminophen, however, does not have ant i- inflammatory properties and is not classified
. 38
as an NSAID. It is therefore not known t o cause gastric ulcers, but high doses of acet aminophen are known t o be hepat otoxic.
Hepatotoxicity Prostaglandin Aspirin Gastric acid Paracetamol Peptic ulcer Anti-inflammatory Nonsteroidal anti-inflammatory drug Ulcer (dermatology) Ulcer Stomach Equine gastric ulcer syndrome
• 39
• 40 C is not co rrect. 2 °/o chose this .
• 41
Pat ients wit h at rial fibrillation who are not anticoagulated are prone to developing thrombi in t heir left at ria, where they may embolize and
cause strokes as well as limb or mesenteric ischemia. An embolus t hat lodges in an artery supplying t he mesentery usually results in an
• 42 ischemic sect ion of bowel. This is ext remely painful to t he patient and, if left unt reated, fatal. However, cardiac emboli are not implicat ed in
. 43 t he genesis of gast ric ulcers .
Atrial fibrillation Mesenteric ischemia Embolism Atrium (heart) Embolization Thrombus Mesentery Embolus Ischemia Peptic ulcer Anticoagulant Gastrointestinal tract Ulcer
• 44
• 45 D is not correct. 4 °/o chose t his .
• 46 Crohn disease present s with weight loss, abdominal pain, and rect al bleeding. However, Crohn ulcerat ion can be located anywhere f rom buccal
• 47
mucosa t o t he rectum, and an isolat ed stomach ulcer like this is uncommon .
Hematochezia Lower gastrointestinal bleeding Peptic ulcer Crohn's disease Rectum Rectal bleeding Oral mucosa Abdominal pain Weight loss Ulcer (dermatology) Mucous membrane Mouth ulcer Ulcer
• 48
Stomach
. 49
• 50
E is not correct. 7 °/o chos e this .
8
Lock
s
Suspend
0
End Block
Item: 28 of ~ ,• Mark <::J [:::> ""I ~· ~'j
100 J.. Previous Next LAbfaiUI~S Notes Calculator

24 .. Stomach

25 E is not correct. 7°/o chose this .


26 Linit is plastica ("leat her bottle stomach") is a consequence, not a cause, of gast ric cancer. The type of cancer giving rise to the leat hery
27 appearance is diffuse stomach cancer, which is dist inct from gastric adenocarcinoma . Only t he lat ter is associated wit h ulceration.
28
Nonetheless, a malignant process must be ruled out wit h biopsy.
Linitis plastica Adenocarcinoma Stomach cancer Biopsy Malignancy Peptic ulcer Ulcer Mouth ulcer Ulcer (dermatology) Cancer
• 29
• 30
• 31 Bottom Line :
. 32 Gnawing epigastric pain that is exacerbat ed by meals is suggestive of gastri c ulcer. Most gastric ulcers are caused by He/icobacter pylori.
Helicobacter pylori Peptic ulcer Abdominal pain Epigastrium Ulcer Ulcer (dermatology) Helicobacter Equine gastric ulcer syndrome
. 33
• 34
• 35
141;fil·1i•J for yea r:[2o17 •
• 36 FIRST AID FA CTS

. 37
. 38 FA17 p 142.1
• 39
Helicobacter pylori Curved, flagellated (motile), gram 8 rod fJ that is triple EB: catalase EB, oxidase EB, and urease EB
• 40
(can use urea breath test or feca l antigen test for diagnosis). Urease produces ammonia, creating
• 41
an alkaline environment, which helps H pylori survive in acidic mucosa. Colonizes mainly
• 42
. 43
antrum of stomach; causes gastritis and peptic ulcers (especially duodenal). Risk factor for peptic
• 44
ulcer disease, gastric adenocarcinoma, and MALT lymphoma .
• 45
Most common initial treatment is triple therapy: Amoxicillin (metronidazole if penicillin allergy)
• 46 + Clarithromycin + Proton pump inhibitor; Antibiotics C ure Pylori .
• 47
• 48
. 49
• 50 FA 17p 363.1

8
Lock
s
Suspend
0
End Block
Item: 28 of - ,• Mark -<] 1:> ""'I ~· 1!';:'1
100 ~ Prev1o u s Next Labf a lu es Notes Calculator

24 • FA17p363.1

25
Peptic ulcer disease
26
27
Gastric ulcer Duodenal ulcer
28 PAIN Can be Greater with meals-weight loss Decreases with meals- weight gain
• 29
HPYLORI INFECTION - 70% - 90%
. 30
. 31 MECHANISM l mucosal protection against gastric acid l mucosal protection or t gastric acid secretion
. 32 OTHER CAUSES 1SAIDs Zollinger-EIIison syndrome
. 33
RISK OF CARCINOMA t Generally benign
. 34
• 35 OTHER Biopsy margi ns to rule oulmalignanc) llypertrophy of Bnmner glands
• 36
. 37
FA17 p 362.1
. 38
Gastritis
. 39
• 40 Acute gastritis F.rosions can be caused by: F.specia lly common among alcoholics and
• 41 • 1 SA IDs- l PGE ..... l gastric mucosa
2 patients taking daily NSA IOs (eg, patients with
• 42 protection rheu matoid arthritis).
. 43 • Burns (Curling ulcer)- hypovolemia
• 44 ..... mucosal ischemia Burned by the C urling iron .
• 45 • Brain injury (Cushing ulcer)- t vaga I
• 46 stimulation ..... t ACh ..... t JJ+ product ion l"'ars C u.,hion the brain.
• 47
'
. 48
Chronic gastritis Mucosal inAammation, often leading to atrophy
. 49
(hypochlorhydria ..... hypergastrinemia) and •
• 50

8
L.odt
s
Su~pl'nd
~
End Block
Item: 28 of - ,• Mark -<J [:::> "'I ~ · ~
100 ~ P~v1ous N @xt Labl lues N o tes Calcula to r
I- · ' I -.: I ; ~ I I ~

25 RISKOF CARCINOMA t Generally benign


26
OTHER Biopsy margins to rule out malignancy llypertrophy of Brunner glands
27
28
• 29 FA17 p 362.1
• 30 Gastritis
• 31
Acute gastritis Erosions can be caused by: Especially common among alcoholics and
. 32
NSAIDs- l PGE 2 - l gastric mucosa patients taking daily I': SAl Ds (eg, patients with
. 33
• 34
protection rheumatoid arthritis).
• 35
Burns (Curling ulcer)- h) povolemia
• 36
- mucosal ischemia Burned by the Curling iron.
. 37 Brain injury (Cushing ulcer)- t \'aga I
. 38 stimulation - t ACh - t JJ+ production ]wars Cmhion the brain .
'
• 39
• 40 Chronic gastritis Mucosal inAammation, often leading to atrophy
• 41
(hypochlorhydria - hypergastrinemia) and
• 42
intestinal C-ecil metaplasia (t risk of ga~tric
. 43
cancers).
• 44
. 45 H pylori Most common. t risk of peptic ulcer disease, Affects antrum first and spreads to body of
• 46 M LT lymphoma . stomach.
• 47 Autoimmune Autoantibodies to parietal cells and intrinsic Affects body/fundus of stomach.
. 48 fa ctor. t risk of pernicious anemia .
. 49
. 50 •
8
Lode.
s
S uspe-nd
8
End Bloc:k
Item: 29 of - ,• Mark -<] 1:> ""'I ~· 1!';:'1
100 ~ Prev1o u s Next Labf a lu es Notes Calculator

24 •
25
An unconscious 40-year-old man is brought t o t he emergency department by his wife. She st ates that her husband complained of severe r~AJ
muscle aches, fever, chills, and headache of sudden onset 4 days earlier. Last night, he developed a dry cough . She also notes th at he
26 recently went to clean out their cabin in rura l Colorado. On physical exam, the patient is tachypneic, tachycardic, and febri le. A chest
27 rad iograph shows bilateral interst itial pulmonary edema.
28
• 29 What infectious agent was the pat ient most likely exposed to?
. 30 :
. 31 A. Alphavirus
. 32 B. Flavivirus
. 33
C. Hantavirus
. 34
• 35 D. Plasmodium falciparum
• 36 E. West Nile virus
. 37
. 38
. 39
• 40
• 41
• 42
. 43
• 44
• 45
• 46
• 47
. 48
. 49
• 50 •

8
L.odt
s
Su~pl'nd
~
End Block
Item: 29 of ~ ,• Mark <::J [:::> ""I ~· ~'j
100 J.. Previous Next LAb faiUI~S Notes Calculator

24
25
The correct a nswer is C. 69°/o chose this .
26 The Hantavirus is a genus in t he Bunyaviri dae fam ily t hat causes t he Hantavirus pulmonary syndrome, which starts wit h fevers, headaches,
27 severe myalgias, GI upset , dizziness, and chills. Pat ients init ially have no respi ratory symptoms. As t he disease progresses, howeve r~ t he virus
28
causes capillary leak in the lungs, which causes pulmona ry edema. Duri ng this phase of the disease, patients will develop cough and severe
respiratory dist ress. Treat ment is supportive, and many pat ients will require mechanical vent ilat ion for respirat ory support. The deer mouse is
29 t he reservoir for t he virus, and exposure to its droppi ngs and urine account fo r human infect ion, so this patient may have encount ered the
• 30 virus while cleaning .
Hantavirus Pulmonary edema Mechanical ventilation Bunyaviridae Genus Dyspnea Edema Capillary Chills Dizziness Virus Cough Urine Lung Infection
• 31
. 32 A is not correct . 7°/o chose this .
. 33 Alphavirus is a mem ber of the Togaviri dae fa m ily t hat uses t he mosquito as a vect or. Three kinds of Alphavirus cause three different fo rms of
• 34
encephalit is: Western equine encephalit is, Eastern equine encephalit is, and Venezuelan equine encephalit is .
Togaviridae Alphavirus Encephalitis Eastern equine encephalitis virus Western equine encephalitis virus Mosquito Venezuelan equine encephalitis virus Vector (epidemiology) Equus (genus)
• 35
• 36
B is not correct. 11 % chose this .
. 37
Flavivirus is a group of viruses which include dengue virus, yellow fever, West Nile Virus, Japanese Encephalit is, St . Louis Encephalitis. These
viruses are often refe rred to as arbov iruses as t hey are t ransmitted by an art hopod (a vet or). Dengue and yellow feve r viruses ut ilize the
. 38 Aedes mosquit o as a vect or. Affect ed pat ient s will t ypically present with acut e fever, gastrointest inal symptoms, and headache.
• 39 Flavivirus Saint Louis encephalitis Yellow fever West Nile virus Japanese encephalitis Dengue virus Mosquito Aedes Encephalitis Dengue fever Virus Vector (epidemiology) Arbovirus Headache Fever Nile

• 40
D is not correct. 4 °/o chose t his.
• 41 Plasmodium falciparum (P falciparum) is t he protozoan responsible for malaria. It is t ransmitted by t he Anopheles mosquito. Patients will
• 42 t ypically present with paroxysms of feve r, shaking chills, and sweating. Diagnosis of malari a is made by a peripheral blood smear which shows
t ro phozoit e ring within RBC. Treatment of malaria include chloroquine (for sensit ive species), mef loquine or atovaquone/proguanil (for
. 43
resistant ones), and artesunate or quinidine (for life -t hreat ening illness) . Other causes of ma laria include P vivax, P ovate, and P malariae .
• 44 Trophozoite Mefloquine Chloroquine Plasmodium falciparum Quinidine Malaria Artesunate Mosquito Anopheles Blood film Plasmodium Atovaquone/proguanil Protozoa Fever Red blood cell
• 45
E is not correct. 9°/o chose this .
• 46
The West Nile virus is a Flavivirus t hat causes encephalit is, gast roint estinal distress, and feve r. It is t ransmitted via t he Culex mosquito .
• 47 West Nile virus Flavivirus Encephalitis Mosquito Virus Culex Nile Human gastrointestinal tract Fever

• 48
. 49
Bottom Line :
• 50

8
Lock
s
Suspend
0
End Block
Item: 29 of - ,• Mark -<J [:::> "'I ~ · ~
100 ~ P~v1ous N @xt Labl lu es N o tes Calcula t o r

8
Lode.
s
S uspe-nd
8
End Bloc:k
Item: 29 of - ,• Mark -<J [:::> "'I ~ · ~
100 ~ P~v1ous N @xt Labl lues N o tes Calcula to r

24 A A
Togaviruses Yes SS ® linear Icosahedral Rubella
25
Western and Eastern equine encephalilisa
26
Chikungunya ,·irus
27
28 Retroviruses Yes SS ®linear Icosahedral I lave reverse transcriptase
29 2 copies (HTLV}, IITLV-T-cellleukemia
• 30 complex IIIV-AIDS
• 31 and conical
. 32 (I II )
. 33 Coronaviruses Yes SS ®linear Helical "Common cold," SARS, \IERS
• 34
• 35
Orthomyxoviruses Yes sse linear llelical Influenza virus
• 36
8 segments
. 37 Paramyxoviruses Yes ss e linear Helical PaRa k• xovirus:
. 38 onsegmenled Parainfluenza-croup
• 39 RSV-bronchiolitis in babies; Rx- ribavirin
• 40 \1easles, Mumps
• 41
• 42
Rhabdoviruses Yes ss e linear Helica l Rabies
• 43 Filoviruses Yes ss e linear Helica l Ebola/Ma rburg hemorrhagic fever-often fatal!
• 44 Arenaviruses Yes ss ®a nd e l lelica l LCM - lymphocytic choriomeningit is \'irus
. 45
circular Lassa fever encephalitis-spread by rodents
• 46
2 segments
• 47
. 48
Bunyaviruses Yes ss e circular Helical Calirornia encephalitis 3
. 49 3 segments SandAy/Rirt alley revers3
. 50
Crimean-Congo hemorrhagic b·e~
8
Lode.
s
S uspe-nd
8
End Bloc:k
Item: 30 of - ,• Mark -<] 1:> ""'I ~· 1!';:'1
100 ~ Prev1o u s Next Labf a lu es Note s Calculator

24 •
25
A 2-year-old girl presents to the emergency room because of the acute onset of fevers to 40°C (104°F) . On physical examination, she r~AJ
refuses to walk and does not move very much . In addition a purpuric rash is noted on her legs. Blood for cult ures is drawn and empiric
26 antibiotic treatment is begun. Twenty-four hours later, a Gram stain of the culture growth is performed, and the results are shown in the
27 image.
28

29 •
• r
. 30
. 31 •
. 32

' • •
. 33
. 34 •
• 35
• I
• 36
. 37

..
• ••
. 38 I

. 39
Image COUitesy of CDC I Dr. Brodsky
• 40
• 41
This child's infection is associated wit h a defi ciency of which of the fo llowing complement molecules?
• 42
:
. 43
A. Cl esterase
• 44
• 45
B. C2
• 46 C. C4
• 47 D. CS-C9
. 48
E. Decay-accelerating factor
. 49
• 50 •
8
L.odt
s
Su~pl'nd
~
End Block
Item: 30 of ~ ,• Mark <:::1 t::> ""I ~· ~'j
100 J.. Previous Next LAb faiUI~S Notes Calculator

24
The correct answer is 0. 76°/o chose this.
25
This acut ely ill child present s with signs and symptoms suggestive of meningococcemia, given her acute, pyrexia, pet echial rash, and blood
26
cult ure f indings. I n add it ion, t he bact eri a shown in t he image are gram- negative diplococci, consistent wit h Neisseria meningitidis. Pat ients
27 with meningococcemia are less likely to exhibit classic signs of meningit is. A def iciency in complement molecules CS- C9 can lead speci fically
28 t o Neisseria bacteremia. Prot eins CS-C9 are part of t he membrane at tack complex and are involved in cytolysis.
Meningococcemia Neisseria meningitidis Bacteremia Gram-negative bacteria Fever Blood culture Meningitis Meningococcal disease Petechia Diplococcus Purpura Neisseria Bacteria Rash
29
Complement membrane attack complex
30
• 31 A is not correct. 9°/o chose this .
. 32 A deficiency in Cl esterase inhibitor is one of t he causes of hereditary angioedema .
Hereditary angioedema Angioedema Esterase Cl-inhibitor
. 33
• 34 B is not correct. 5% chose this .
• 35 C2 def iciency does not lead t o any specif ic disease, although as part of t he classic pat hway (along with Cl , C3, and C4), it will decrease the
ability of t he immune syst em to neut ralize viruses .
• 36 Complement component 2 Immune system Virus
. 37
C is not correct. 6 °/o chose this .
. 38
C4 def iciency does not lead t o any specif ic disease, although as part of t he classic pat hway (along with C2, C3, and Cl), it will decrease the
• 39 ability of t he immune syst em to neut ralize viruses.
• 40 Complement component 4 Immune system Virus C4 carbon fixation C-4 (explosive)

• 41
E is not correct. 4°/o chose this.
• 42 A deficiency in decay-accelerating fact or leads to paroxysmal nocturna l hemoglobinuri a. The normal role of t his molecule, along wit h
. 43 membrane inhibit or of reactive lysis (MIRL), is t o prevent com plement from attacking and lysing t he RBCs. I ts lack t herefore leads to red
• 44
blood cell lysis and hematuria .
Red blood cell Paroxysmal nocturnal hemoglobinuria Decay-accelerating factor Hematuria Lysis Hemoglobinuria
• 45
• 46
• 47 Bottom Line:
• 48 Com plement proteins CS- C9 are components of t he membrane attack complex involved in cytolysis. A def iciency of t hese proteins leaves t he
. 49
patient part icularly suscept ible to infect ion wit h gram -negative bacteria such as Neisseria .
Gram-negative bacteria Cytolysis Complement membrane attack complex Complement system Bacteria Cell membrane Protein Neisseria Biological membrane
• 50

8
Lock
s
Suspend
0
End Block
Item: 30 of ~ ,• Mark <:::1 t::> ""I ~· ~'j
100 J.. Previous Next LAbfaiUI~S Notes Calculator

24
FA17 p 103.2
25
Complement disorders
26
27 Complement protein deficiencies
28 C3 deficiency Increases risk of severe, recurrent pyogenic sinus and respiratory tract infections; t susceptibility to
29 type Ill hypersensitivity reactions.
30
• 31
CS- C9 deficiencies Termi nal complement deficiency increases susceptibil ity to recurrent Neisseria bacteremia.
. 32 Complement regulatory protein deficiencies
. 33
Cl esterase inhibitor Causes hereditary angioedema due to unregulated activation of ka ll ikrein - t bradykinin.
• 34
deficiency Characterized by l C4 levels. ACE in hibitors are contraindicated.
• 35
• 36
CDSS deficiency Also called decay-accelerating factor (OAF) deficiency. Causes complement-mediated lysis of RBCs
. 37 and paroxysmal nocturnal hemoglobinuria .
. 38
• 39 FA17 p 103.1
• 40
• 41
Complement System of hepatically synthesized plasma proteins that play a role in innate immunity and
• 42
inAammation. Membrane attack complex (MAC) defends against gram 8 bacteria.
. 43 ACTIVATION Classic pathway- lgG or IgM mediated. GM makes classic cars.
• 44 Alterna tive pathway- microbe surface
• 45 molecules.
• 46 Lectin palhway- mannose or other sugars on
• 47 microbe surface.
• 48
. 49
FUNCTIONS C3b- opsonization. C3b binds bacteria.
• 50
C3a, C4a, C5a- anaphylaxis.
8
Lock
s
Suspend
0
End Block
Item: 30 of - ,• Mark -<J [:::> "'I ~ · ~
100 ~ P~v1ous N @xt Labl lues N o tes Calcula to r

24 A A
Opsonins-C3b and lgC are the two 1° Opsonin (Creek) = to prepare for eating.
25
opsonins in bacterial defen se; enhance
26
phagocytosis. C3b also helps clear immune
27
complexes.
28
29 Inhibitors- decay-accelerating factor ( OM~ aka
30 CD55) and Cl esterase inhibitor help pre, ent
• 31 complement activation on self cells (eg, RBCs).
. 32

rrl~
. 33
• 34
C3
• 35
Altttnalive GbBb
C3 --~ C3b
' 36 (C3 convertasel
Spont.JneOus and
. 37 microbial surfaces
Ampt1fies generation o1 C3b
. 38 C3a
' 39 CSa C6-C9
• 40

' 41
lectin
CHike
complex C3b cs --+-'+ Csb I t ~~ r.:l'Arl ----->o..
u::!m.1 ____,....
lysiS.
cytotoxicity
Microbial surfaces (C5b·9l
• 42 (eg, mannose) C4a

• 43 C4 ~ C4b C3a

~
• 44
C4b2b C4b2b3b
' 45 Classic
Cl ,! o I ((3 convertase) {CS convertasel
' 46
' 47
Antlgen·ant1body
complexes (
C2 db C3
. 48 ~
. 49 '
C2a •Historic.llly, the larger fragment of C2 was
called C2a but 1S now referred to as C2b
' 50

8
Lode.
s
Suspe-nd
8
End Bloc:k
Item: 31 of - ,• Mark -<J
P~v1ous
[:::> "'I ~ ·· ~
100 ~ N @xt Labl lues N o tes Calculato r

24 A A

A 19-year-old man comes t o t he emergency department because of blood in his sput um. The pat ient ment ions he has had weight loss and ~~AI
25
night sweats. On examination, t he pat ient has a fever and bronchial breath sounds with crepit ant ra les. Laborat ory test s show
26 lymphocytosis and an increased erythrocyte sedimentation rate. X-ray of the chest shows a calcified lung lesion and hilar
27 lymphadenopathy.
28
29 Which of the following stains is used to ident ify t he most likely infectious organism?
30 :
• 31 A. Congo red
. 32 B. Giemsa
. 33
C. India ink
• 34
• 35 D. Periodic acid-Schiff
• 36 E. Ziehi-Neelsen
. 37
. 38
• 39
• 40
• 41
• 42
• 43
• 44
. 45
• 46
• 47
. 48
. 49
. 50

8
Lode.
s
S uspe-nd
8
End Bloc:k
Item: 31 of - ,• Mark -<] 1:> ""'I ~· 1!';:'1
100 ~ Prev1o u s Next Labf a lu es Notes Calculator

24 •
25 The correct ans w e r is E. 58°/o chose this.
26 This patient most likely has Mycobacterium tuberculosis infection. M. tuberculosis is an aerobic, acid-fast
27 bacillus, and the Ziehi-Neelsen stain is used to reveal acid-fast bacteria (shown in the image).
Characteristics favoring a diagnosis of tuberculosis include pulmonary symptoms, night sweats, weight loss,
28
and chest X-ray findings. Primary tuberculosis is known to result in Ghon complexes, which show up as lung \
29 lesions, often calcified, in combination with hilar adenopathy. other pathologies that can manifest with
30 hilar/mediastinal nodes are lymphoma and sarcoidosis, making the Ziehi-Neelsen stain important in
diagnosing M. tuberculosis. The aerobic nature of M. tuberculosis is reflected by the organism's preferential
31
involvement of the upper lobes of the lung .
. 32 "...co~-"Cte-" 1m tub«culos•s Sarcoidosis Chest radiograph Lymphadenopadly AC•d·fast auberc:u os•s ll'.,cobactenum L~homa Bacteria
. 33 Noght sw...U We.ght loss Bactllus Organosm Aeroboc organosm X-ray Infection Lung Root of the lung
Image courtesy of CDC/ Or.
. 34
George P. Kubica
• 35
• 36 A is not correct . 6 °/o chose this .
. 37 Congo red is used to visualize amyloid, showing apple-green birefringence in polarized light. It is used t o visualize amyloidosis associated with
multiple myeloma, tuberculosis, rheumatoid arthritis, and chronic conditions .
. 38 Multiple myeloma Rheuml'rto•d arthnt1s B1refnngence Congo red Tuberculosis Amylo1dos•s Amylo•d Polanz 1t1on (waves) ArthritiS
. 39
B is not correct. 9 % chose this.
• 40
Giemsa stain is used to revea l Borrelia, Plasmodium, t rypanosomes, and Chlamydia organisms .
• 41 Giemsa sta1n 'Ttypanosoma Chlamydia (genus) Plasmodium Trypanosomatida Borrelia Chlamyd1a Infection
• 42
C is not correct. 8 °/o chose this .
. 43
India ink is the stain of choice for C!yptococcus neoformans. Mucicarmine can also be used t o stain t he t hick polysaccharide capsule red .
• 44 Polysaccharide Cryptococcus neoformans India ink I ndia Mucicarmine stain
• 45
D is not correct. 19°/o chose this .
• 46
Periodic acid-Schiff stains glycogen and mucopolysaccharides. It is used to diagnose Whipple disease, caused by Tropheryma whipplei.
• 47 Whtpple's dtse .se Per~od1c ac1d·Schtff Glycogen Tropheryma whipplei
. 48
. 49
Botto m Line : •
• 50

8
L.odt
s
Su~pl'nd
~
End Block
Item: 31 of ~ ,• Mark <::J [:::> ""I ~· ~'j
100 J.. Previous Next LAbfaiUI~S Notes Calculator

24
Bottom Line:
25
26
Tuberculosis can manifest wit h fever, night sweats, weight loss, and blood in t he sput um . Ghon complexes can often be seen on X- ray of the
chest. I nit ia l detect ion of tuberculosis is wit h the acid-fast st ain, but definit ive diagnosis of M. tuberculosis requires cultu re or molecular
27 detect ion met hods.
28 Tuberculosis Sputum Chest radiograph Acid-fast Mycobacterium tuberculosis Night sweats Weight loss X-ray Fever

29
30
31 141;fil·1i•J for year:[ 2017
FIRST AID FA CTS .
•j .
. 32
. 33
FA17 p 136.1
• 34
• 35
Primary and secondary tuberculosis
• 36 _, Mycobacterium PPD ® if current infection or past exposure.
.-r- ''''i'
' '• tuberculosis
. 37
Hilarnodes PPD 8 if no infection and in sarcoidosis or
. 38 HI V infection (especially with low CD4+ cell
+
• 39
Ghon
complex Ghon focus ---...... count).
• 40 (usually mid/ lnterferon-y release assay (IGRA) has fewer false
• 41 lower lobes)
Primary tuberculosis positives from BCG vaccination.
• 42
> 90X <lOX Caseating granulomas fJ with central necrosis
. 43
HeaUng by fibrosis Progressive primary tuberculosis (upper left) and Langhans giant cells (arrow)
• 44 Calcification (AIDS. malnutrition)
(tubefculin (!})
I are characteristic of 2° tuberculosis.
• 45

! l
• 46
• 47
Reactivation
2' tuberculosis
j Progressive
lung dise~

• 48 Fib<ocaseous
. 49 cavitary lesion
(usually uppet
• 50 ,._,_._.,--- Meninges

8
Lock
s
Suspend
0
End Block
Item: 31 of - ,• Mark -<] 1:> ""'I ~· 1!';:'1
100 ~ Prev1o u s Next Labf a lu es Notes Calculator

24 • •
FA17 p 122.1
25
Stains
26
27 Gram stain First-line lab test in bacterial identification. Bacteria with thick peptidoglycan layer retain crystal
28 violet dye (gram<±>); bacteria with thin peptidoglycan layer turn red or pink (gram 8 ) with
29 counterstain.
30 These bugs do not Gram stain well ('I hese Little \ licrobes t\lay Unfortunately Lack Real Color
31 But Are Everp\ here).
. 32
Treponema. Leptospira Too thin to be visualized.
. 33
. 34 .\lycobacteria Cell wall has high lipid content.
• 35 :\1ycoplasma, Ureaplasma No cell wall.
• 36
Legionella, Rickettsia, Chlamydia, Bartonella, Primarily intracellular; also, Chlamydia lack
. 37
,\ naplasma, Ehrlichia classic peptidoglycan because of l muramic
. 38
acid .
. 39
• 40 Giemsa stain Chlamydia, Borrelic1, Rickettsia, C ertain Bugs Really Try my Patience.
• 41 Trypanosomes fl. Plasmodium
• 42
Periodic acid- Schiff Stains glycogen, mucopolysaccha rides; used PaSs the sugar.
. 43
stain to diagnose Whipple disease (Tropherynw
• 44
whipplei [l])
• 45
• 46 Ziehi-Neelsen stain Acid-fast bacteria (cg, Mrcobacteria C urrent standard of care is auram inc-
• 47 (carbolfuchsin) 1 ocardia; stains mycolic acid in cell wall); rhodamine stain for screening (ine:>..'pensi,·e,
. 48 protozoa (eg, Cryptosporidium oocysts) more scnsiti,·e but less specific).
. 49 India ink stain Cryptococcus neoformans • ; mucicarmine
• 50 -- ·~ _................. ..........I ·- ...... . :_ IL.. ! .... I - .-I . ...............J ...... -:..1 ... •
8
L.odt
s
Su~pl'nd
~
End Block
Item: 31 of - ,• Mark -<] 1:> ""'I ~· 1!';:'1
100 ~ Prev1o u s Next Labf a lu es Notes Calculator

24 •
25 Giemsa stain Chlamydia, Borrelia, Rickettsia, C ertain Bugs Really Try my Patience.
26 Trypanosomes rJ, Plasmodium
27
Periodic acid- Schiff Stains glycogen, mucopolysaccharides; used PaSs the sugar.
28
stain to diagnose Whipple disease (Tropher)•llw
29
30
whipplei IE])
31 Ziehi-Neelsen stain cid-fast bacteria (eg, Mycobacteria Current standard of care is auraminc-
(carbol fuchsin) 1ocardia; rhodamine stain for screening (inexpcnsi,·e,
. 32 stains mycolic acid in cell wall);
. 33 protozoa (eg, Cryptosporidium oocysts) more sensitive but less specific).
. 34
India ink stain Crrptococcus neoformans • ; mucicarmine
• 35
can also be used to stain thick pol) saccharide
• 36
. 37
capsule red
. 38 Silver stain Fungi (eg, Coccidioides 11 P11eumocystis
. 39 jirovecii), Legionel/a, Helicobacter pylori
• 40
Fluorescent antibody Used to identify many bacteria a nd viruses. Example is FTA-ABS for syphilis.
• 41
stain
• 42
. 43
• 44
• 45
• 46
• 47
. 48
. 49

• 50 •
8
L.odt
s
Su~pl'nd
~
End Block
Item: 31 of - ,• Mark -<J
P~v1ous
[:::> "'I ~ ·· ~
100 ~ N @xt Labl lues N o tes Calculato r

24
fJ
A A

25
26
27
28
29
30
31
. 32
. 33
FA17 p 193.1
• 34
Isoniazid
• 35
• 36 MECHANISM l synthesis of mycolic acids. Bacterial catalase-
. 37 peroxidase (encoded by KatG) needed to
. 38 convert INI I to active metabolite.
• 39
CLI NICAL USE J'vfycobacteriwn tuberculosis. The only agent Different I 1H half-lives in fast vs slow
• 40
used as solo prophylaxis against TB. Also used acetyla tors.
• 41
as monotherapy fo r latent TB .
• 42
• 43
ADVERSE EFFECTS Hepatotoxicity, P-450 inhibition, drug-induced IN IIl njures Neurons and ll epatocyles.
• 44 SLE, anion gap metabolic acidosis, vitamin
. 45 B6 defi ciency (peripheral neuropathy,
• 46 sideroblastic anemia). Administer with
• 47 pyridoxine (B6).
. 48
MECHANISM OF RESISTANCE Mutations leading to underexpression of KatC.
. 49
. 50

8
Lode.
s
S uspe-nd
8
End Bloc:k
Item: 32 of - ,• Mark -<J [:::> "'I ~ · ~
100 ~ P~v1ous N@xt Labl lues Notes Calculator

24 A A

A middle-aged man hosts a barbecue in his neighborh ood to welcome several fam ilies that have j ust moved into the community. He ra ises ~~AI
25
pigs and decides to spit roast a whole animal to eat at th e event. After the party, several of his neighbors develop diarrhea and abdominal
26 pain. A week later, they began experiencing myalgias and have petechial conjunctival hemorrhages and severe periorbital erythema and
27 swelling . Several went to the hospital, where peripheral blood smears revea led increased eosinophil counts.
28
29 What organism most likely caused t hese symptoms?
30
31 A. Bacillus cereus
• 32 B. Clostridium botulinum
. 33
C. Giardia Iamblia
• 34
• 35 D. Trichinella spiralis
• 36 E. Yersinia enterocolitica
. 37
. 38
• 39
• 40
• 41
• 42
• 43
• 44
. 45
• 46
• 47
. 48
. 49
. 50

8
Lode.
s
Suspe-nd
8
End Bloc:k
Item: 32 of ~ ,• Mark <::J [:::> ""I ~· ~'j
100 J.. Previous Next faiUI~S
LAb Notes Calculator

24
25
The correct a nswer is 0. 7 4 °/o chose t his.
26
Undercooked pork, fox, cat , and bear meats are usually responsible fo r human disease of Trichinella spiralis . Trichinella larvae are released in
27 t he gast roint estinal tract after ingestion and t hen penetrate t he intest inal mucosa. They can develop into adult worms wit hin 30 hours. I n
28 about 1 week, fe male worms produce larvae after mat ing; t hese larvae seek out st riated m uscle in which to encyst. Entry of larvae int o t he
muscle causes severe myalgias, among ot her symptoms. The eosinophilia found on the peri pheral blood smear suggests the presence of
29
helm inth infection (prot ozoal infect ions, in contrast , do not usually cause eosinophilia.)
30 Helminths Trichinella spiralis Striated muscle tissue Gastrointestinal tract Blood film Fox Cyst Protozoa Pork Helminthiasis Intestinal mucosa Eosinophilia Mucous membrane Microbial cyst

31 Human gastrointestinal tract Trichinella Cat Infection

32
A is not correct . 5°/o chose this .
. 33
Bacillus cereus can cause similar gast roint estinal sympt oms through a preformed toxin when food is imprope rl y cooked ("fri ed ri ce
• 34 syndrome") . 8 cereus would not cause sympt oms in t he eyes. Furt hermore, t his agent would not be associat ed with eosinophilia .
• 35 Bacillus cereus Bacillus Toxin Gastrointestinal tract Rice Human gastrointestinal tract Eosinophilia

• 36 B is not correct. 3% chose t his .


. 37 Clostridium botulinum can cause sim ilar gastrointest inal symptoms, but it would not cause sympt oms in t he eyes. Suspect C botulinum
. 38 infection in cases of f laccid paralysis, particularly in infants who have eat en honey. Furt hermore, t his agent would not be associat ed wit h
eosinophilia .
• 39
Clostridium botulinum Flaccid paralysis Clostridium Paralysis Gastrointestinal tract Botulinum toxin Honey
• 40
C is not correct. 6°/o chose this .
• 41
Giardia Iambiia is transmit ted by the ingest ion of contaminat ed water with t he cysts of t his prot ozoa. Hikers and climbers who drink f rom
• 42
streams are commonly affected. Clinical disease includes int estinal sympt oms such as bloat ing and watery diarrh ea. G Iambiia is not invasive
. 43 and protozoa infect ions do not generally present with eosinophilia .
Giardia Iambiia Protozoa Giardia Diarrhea Microbial cyst Cyst Bloating Eosinophilia
• 44
• 45 E is not correct. 1 2°/o chose this .
• 46 Yersinia enterocolitica can cause similar gast roint estinal sympt oms. The bact eria does invade the int estinal m ucosa but does not generally
• 47 cause eosinophilia. This agent is usually t ransmit ted t hrough pet feces, contaminat ed milk, and pork .
Yersinia enterocolitica Feces Yersinia Bacteria Mucous membrane Pork Gastrointestinal tract Eosinophilia Milk Intestinal mucosa
• 48
. 49
• 50 Bottom Line :
8
Lock
s
Suspend
0
End Block
Item: 32 of ~ ,• Mark <::J [:::> ""I ~· ~'j
100 J.. Previous Next LAb faiUI~S Notes Calculator

24 • •
25 Bottom Line:
26 Trichinella spiralis is a nematode that can infect hu mans th rough ingestion of larvae in undercooked meat.
Nematode Trichinella spiralis Trichinella Meat
27
28
29
30 l@);fil ~1hl for year:l 2o17
FIRST AID FACTS
•I
31
32 FA17 p 155.1
. 33 Nematodes (roundworms)
• 34
ORGANISM DISEASE TRANSMISSION TREATMENT
• 35
Intestinal
• 36
. 37 Enterobius vermicularis Causes <mal pruritus (diagnosed by seeing Fecal-oral Pyrantel pamoate or
. 38 (pinworm) a
egg via the tape test) bend azoles (because
• 39 worms are bendy)
• 40
Ascaris lumbricoides May cause obstruction at ileocecal Fecal-ora I; knobby-coated, Bendazoles
• 41
(giant roundworm) valve, biliary obstruction, intestinal oval eggs seen in feces
• 42
perforation, m igrates from nose/mouth under microscope EJ
. 43
• 44 Strongyloides Causes vomiting, diarrhea, epigastric Larvae in soil penetrate skin; Ivermectin or
• 45 stercora/is pain (mar m imic peptic ulcer) rhabditiform lan·ae seen in bendazoles
• 46 (threadworm) feces under m icroscope
• 47
Ancylostoma Cause anemia by sucking blood from Larvae penetrate skin Bendazoles or pyrantel
. 48
duodenale, Necator intestinal wall pamoate
. 49
• 50 •
americanus Cutaneous larva migrans- pruritic,
• r .. . r

8
Lock
s
Suspend
0
End Block
Item: 32 of - ,• Mark -<J [:::> "'I ~ · ~
100 ~ P~v1ous N@xt Labl lues Notes Calculator

24 A A

Tissue
25
26 Toxocara canis Visceral larva migrans- nematodes Fecal-oral Bendazoles
27 migrate to blood through intestinal wall
28 - inflammation and damage. Often
29 affects heart (m}ocarditis), Ji,er, e)eS
30 (,·isual impairment, blindness), and
31 CNS (seizures, coma)
32
Onchocerca volvulus Skin changes, loss of elastic fibers, and Female blackAy lvermectin (ivermectin
. 33
ri\'er blindness (black flies, black skin for rh cr blindness)
• 34
nodules, "black sight"); allergic reaction
• 35
to microfilaria possible
' 36
. 37 Loa loa Swelling in skin, worm in conjuncli,·a Deer Ay, horse Ay, mango Ay Diethylcarbamazine
. 38 Female mosquito
Wuchereria bancrofti Lymphatic fi lariasis (elephantiasis)- Diethylcarbamazine
' 39 worms invade lymph nodes
• 40
- infl ammation - lymphedema l!t
' 41
symptom onset after 9 mo- l yr
• 42
• 43
• 44

' 45
' 46
' 47
. 48
. 49

' 50

8
Lode.
s
Suspe-nd
8
End Bloc:k
26
27
FA17 p 156.2
28
29
Trematodes (flukes)
30 ORGANISM DISEASE TRANSMISSION TREATMENT
31 Schistosoma Li,·er and spleen enlargement Snails arc host; cercariae Praziquantel
32 (S mansoni, egg with penetrate skin of humans
. 33 lateral spine fJ). fibrosis,
. 34 inAammation, portal
• 35 h) per tension
• 36
Chronic infection with
. 37
0 0 S haematobium (egg" ith
. 38 0 •
terminal spine I]]) can lead
. 39
to squamous cell carcinoma
• 40
of the bladder (painless
• 41
• 42
hematuria) and pulmonary
. 43
hypertension
• 44 Clonorchis sinensis Bi liary tract inAammation Undcrcookcd fish Praziquantel
• 45 ..... pigmented gallstones
• 46 Associated with
• 47 chola ngioca rcinoma
. 48
. 49

• 50 •

8
L.odt
s
Su~pl'nd
~
End Block
Item: 33 of - ,• Mark -<J [:::> "'I ~ · ~
100 ~ P~v1ous N @xt Labl lues N o tes Calculato r

24
A 50-year-old man who recent ly returned from visit ing fam ily in northern New Mexico comes t o th e physician with exquisit ely tender and
25
enlarged lymph nodes. He also complains of fever, chills, and general weakness. On physical examination, the physician notes a painful
26 ulcer surrounded by dark, hemorrhagic purpura on the right arm in the area where, according to the patient , a fl ea had bitt en him 5 days
27 ago. After being admitted to t he hospit al, t he pat ient soon develops abnormal coagulation t imes and is quickly st arted on a regimen of
streptomycin and tetracycline.
28
29
Which of the following organisms is most likely responsible for this patient's symptoms?
30
:
31
A. Babesia microti
32
• 33 B. Bacillus anthracis
• 34 C. Leishmania donovani
• 35
D. Trichinella spiralis
• 36
. 37 E. Yersinia pestis
. 38
• 39
• 40
• 41
• 42
• 43
• 44
. 45
• 46
• 47
. 48
. 49
. 50

8
Lode.
s
S uspe-nd
8
End Bloc:k
Item: 33 of ~ ,• Mark <::J [:::> ""I ~· ~'j
100 J.. Previous Next LAbfaiUI~S Notes Calculator

24
25
26 The correct answer is E. 51 °/o chose this.
27
Yersinia pestis is t he organism responsible for t he plague, also known as t he Black Death . The bacterium can be spread to humans by f leas
f rom rodents, especially prairie dogs in the Unit ed Stat es. The disease develops after 2-8 days of incubation and is characterized by t he
28 presence of exquisitely tender lymph nodes called buboes. Unlike in the case of anthra x, the skin ulcers seen in Y. pestis infect ion are painful.
29 Furthermore, prolonged infection and spread of Y. pestis can lead t o dissem inat ed intravascular coagulation.
Disseminated intravascular coagulation Black Death Yersinia pestis Anthrax Bubonic plague Plague (disease) Bubo Yersinia Bacteria Lymph node Coagulation Organism Prairie dog Lymph Rodent
30
31 A is not correct. 14% chose this.
32 Babesia microti is t ra nsmit ted t o humans t hrough t he bi te of a t ick. It causes a sickness similar t o malaria wit h symptoms of feve r and
anemia.
33
Babesia Malaria Anemia Tick Theileria microti Fever
• 34
• 35 B is not correct. 12% chose this .
Bacillus anthracis can cause cut aneous ant hrax, which is characterized by a painless ulcer with a black scab.
• 36
Bacillus anthracis Anthrax Bacillus Wound healing Ulcer Cutaneous anthrax Peptic ulcer
. 37
. 38
C is not correct. 15% chose this .
Leishmania donovani is t ransmitted t hrough t he bi te of a sandf ly and causes visceral leishmaniasis. This disease is characterized by abdominal
• 39
pain and dist ent ion, anorexia, weight loss, and feve r.
• 40 Leishmania donovani Leishmaniasis Visceral leishmaniasis Leishmania Anorexia (symptom) Sandfly Anorexia nervosa Abdominal pain Fever

• 41
D is not correct. 7°/o chose this .
• 42
A person infect ed with Trichinella spiralis presents wit h feve r, periorbi tal and facia l edema, mya lgia, and eosinophilia .
. 43 Trichinella spiralis Myalgia Eosinophilia Edema Trichinella Fever Periorbita

• 44
• 45
Bottom Line:
• 46
Y. pestis is t ransm it ted by f leas f rom a rodent reservoir and causes painful lymphadenopat hy, sepsis, and disseminat ed intravascular
• 47
coagulation.
• 48 Disseminated intravascular coagulation Rodent Sepsis Lymphadenopathy Yersinia pestis Coagulation

. 49
• 50

8
Lock
s
Suspend
0
End Block
Item: 33 of ~ ,• Mark <::J [:::> ""I ~· ~'j
100 J.. Previous Next faiUI~S
LAb Notes Calculator

24 • •
FA1 7 p 144.1
25
26 Zoonotic bacteria Zoonosis: infectious disease transmitted between animals and humans.
27
SPECIES DISEASE TRANSMISSION AND SOURCE
28
29
Anaplasma spp. Anaplasmosis Ixodes ticks (live on deer and mice)
30 Bartonella spp. Cat scratch disease, baci llary angiomatosis Cat scra tch
31
Borrelia burgdorferi Lyme cl isease Ixodes ticks (live on deer and mice)
32
33
Borrelia recurrentis Relapsing fever Louse (recurrent due to variable surface
• 34 antigens)
• 35 Brucella spp. Brucellosis/undulant fever Unpasteurized dairy
• 36
Campylobacter Bloody diarrhea Feces from infected pets/animals; contaminated
. 37
meats/foods/hands
. 38
• 39 Chlamydophila psittaci Psittacosis Parrots, other birds
• 40 Coxiella burnetii Q fever Aerosols of cattle/sheep am niotic Auid
• 41
Ehrlichia chaffeensis Ehrlich iosis Amblyomma (Lone Star tick)
• 42
. 43 Francisella tularensis Tularemia T icks, rabbits, deer Aies
• 44 Leptospira spp. Leptospirosis Animal urine in \\·ater; recreational water use
• 45
Mycobacterium leprae Leprosy l lumans with lepromatous leprosy; armadillo
• 46
• 47
(rare)
. 48 Pasteurella multocida Cellulitis, osteomyelitis Animal bite, cats, dogs
. 49
Rickettsia prowazekii Epidemic typhus Human to human vi<l human body louse
• 50 • •

8
Lock
s
Suspend
0
End Block
Item: 33 of - ,• Mark -<] 1:> ""'I ~· 1!';:'1
100 ~ Prev1ous Next Labfa lues Notes Calculator

24 • •
Borrelia burgdorferi Lyme disease Ixodes ticks (live on deer and mice)
25
26 Borrelia recurrentis Relapsing fever Louse (recurrent due to variable surface
27 antigens)
28
Brucella spp. Brucellosis/unduJant b ·er Unpasteurized dairy
29
30
Campylobacter Bloody diarrhea Feces from infected pets/animals; cont~uni nat cd
31
meats/foods/hands
32 Chlamydophila psittaci Psittacosis Parrots, other birds
33
Coxiella burnetii Qb·er crosols of cattle/sheep amniotic Auid
. 34
• 35 Ehrlichia chaffeensis Ehrlichiosis Amblromma (Lone Star tick)
• 36 Francisella tularensis Tularemia Ticks, rabbits, deer Aies
. 37
Leptospira spp. Leptospirosis Animal urine in water; recreational water use
. 38
. 39 Mycobacterium leprae Leprosy ll umans '' ith lepromatous leprosy; armadillo
• 40 (rare)
• 41 Pasteurella multocida Cellulitis, osteomyelitis Animal bite, cats, dogs
• 42
Rickettsia prowazekii F.pidemic typhus I Iuman t·o human via human body louse
. 43
• 44 Rickettsia rickettsii Rocky Mountain spotted fever Dermacentor (dog tick)
• 45 Rickettsia typhi Endemic typhus Fleas
• 46
Salmonella spp . Diarrhea (which may be bloody), vomiting, Reptiles and poultry
• 47
. 48
(exceptS typhi) fever, abdominal cramps
. 49 Yersinia pestis Plague Fleas (rats and prairie dogs are reservoirs)
• 50 •
8
L.odt
s
Su~pl'nd
~
End Block
Item: 34 of - ,• Mark -<] 1:> ""'I ~· 1!';:'1
100 ~ Prev1o u s Next Labf a lu es Note s Calculator

24 •
25
An 8-year-old boy is brought t o the pediat ri cian wit h a fever of 39.4°C (103° F), as well as cough, chills, and dyspnea. His mother reports r~AJ
that he has had numerous respi ratory infections over the past 3 years and also has a history of foul -smelling, fatty stools. On physical
26 examination, the patient looks small fo r his age. The patient's work-up reveals colonization by a non-lactose-fermenting, oxidase-positive
27 organism that produces pyocyanin .
28
29 What is the mechanism of action of the t oxin produced by this organism?
30 :
31 A. ADP ribosylates and inhibits elongation factor 2
32 B. Binds to 60S ribosomal subunit, inhibiting protein synthesis
33
C. Blocks release of neurotransmitters
. 34
• 35
D. Directly binds to and activates MHC-11 and T-lymphocyte receptors
• 36 E. Increases cAMP levels
. 37
. 38
. 39
• 40
• 41
• 42
. 43
• 44
• 45
• 46
• 47
. 48
. 49
• 50 •

8
L.odt
s
Su~pl'nd
~
End Block
Item: 34 of ~ ,• Mark <:::1 t::> ""I ~· ~'j
100 J.. Previous Next LAbfaiUI~S Notes Calculator

24
25 The correct a nswer is A. 67°/o chose t h is.
26 This patient has clinica l and physical find ings associated wit h a t ypica l bact eri al pneumonia caused by Pseudomonas aeruginosa, an aerobic
27
gram-negat ive, oxidase- posit ive bact eri a t hat produces pyocyan in, which gives it its blue-green color. A major virulence facto r of
Pseudomonas is exot oxin A, which ADP ri bosylates and inhibit s elongation fact or 2 in the host cell, thereby inhibi t ing prot ein synthesis.
28 (Diphtheria toxin acts in a sim ilar fashion.) This pat ient also has signs and sym ptoms of cystic fibrosis: foul-smelling, fat ty stools and freq uent
29 pneumonia. Pat ients may also present with clubbing of t he digits (a sign of chronic hypoxia). Pulmonary infection with alginate- producing P.
30 aeruginosa is the leading cause of death in pat ients wit h cystic fibrosis.
Cystic fibrosis Pseudomonas aeruginosa Pyocyanin Virulence factor Hypoxia (environmental) Gram-negative bacteria Exotoxin Bacterial pneumonia Hypoxia (medical) Diphtheria toxin Protein Diphtheria
31
Virulence Pneumonia Pseudomonas exotoxin Aerobic organism Pseudomonas Bacteria EEF2 Adenosine diphosphate Toxin Protein synthesis Protein biosynthesis Infection Elongation factor
32
33 B is not correct. 11 % chose this .
34 The Shiga toxin of Shigella dysenteriae and related species is notorious for ha lt ing prot ein synthesis. Composed of two subunits, A and B, t he
Shiga t oxin binds t o cell membrane glycolipids and get s internalized, where the A subunit heads to the ribosome and inhibits prot ein
• 35
synt hesis. The Shiga t oxin is potent and unfortunat ely is often dead ly, especially in the developing world. Symptoms of infection with Shigella
• 36 resemble more t hat of a colit is (rat her t han t he clinical pict ure present ed here) and can lead to the development of hemolyt ic urem ic
. 37 syndrome .
Shigella dysenteriae Shiga toxin Ribosome Shigella Hemolytic-uremic syndrome Cell membrane Protein Toxin Protein synthesis Glycolipid Colitis Hemolysis Protein biosynthesis Species
. 38
• 39 C is not co rrect. 4 °/o chose this .
• 40
Exotoxins t hat block the release of neurotransm itters are produced by Clostridium tetani and Clostridium botulinum, wh ich block the re lease
of inhibit ory neurot ransm it ters and acet ylcholine, respect ively. Pat ients wit h botulism often experi ence nausea, vomit ing, and fat igue followed
• 41 by a gradual and progressive para lysis, which beg ins wit h the eyes and face and moves down toward the chest, where it can cause potentially
• 42 fatal resp iratory distress. Pat ients wit h tetanus characterist ically present wit h lock-j aw fo llowed by progressive muscl e rigidit y and spasms .
Clostridium botulinum Clostridium tetani Botulism Tetanus Acetylcholine Neurotransmitter Exotoxin Clostridium Nausea Botulinum toxin Vomiting Fatigue (medical) Paralysis Dyspnea Muscle Hypertonia
. 43
• 44 D is not correct. 7°/o chose t his .
• 45 A toxin t hat directly binds to and activates MHC-II and T-lymphocyte receptors is a characterist ic product of superant igens such as TSST-1 ,
• 46
made by Staphylococcus aureus and resu lt ing in toxic shock syndrome (TSS). A patient wit h a Staphylococcus aureus infection would most
likely present with skin and soft t issue infection (abscess, f uruncl e, cellulit is) . I n t he event of a S. aureus infection causing TSS, the pat ient
• 47 would present wit h sudden onset of m ult isyst em disease, and sympt oms would include fever, chills, hypot ension, and rash, and could also
• 48 include vomit ing, myalgias, re nal dysfunct ion, hepat ic abnormalit ies, and thrombocytopen ia. The given clinica l picture does not fit a
. 49 description of TSS present at ion .
Skin and skin structure infection Boil Toxic shock syndrome Cellulitis Thrombocytopenia Abscess Hypotension Staphylococcus aureus Superantigen Staphylococcus T cell Toxin MHC class II Vomiting Liver
• 50

8
Lock
s
Suspend
0
End Block
Item: 34 of ~ ,• Mark <:::1 t::> ""I ~· ~'j
100 J.. Previous Next LAbfaiUI~S Notes Calculator

24
E is not correct. 11°/o chose this.
25
Pseudomonas aeruginosa does not produce a toxin t hat increases the cAMP level. The exotoxins that do are produced by Vibrio cholerae,
26 Escherichia coli, Bordete/la pertussis, and Bacillus anthracis.
Vibrio cholerae Pseudomonas aeruginosa Bordetella pertussis Bacillus anthracis Escherichia coli Bordetella Cyclic adenosine monophosphate Vibrio Pseudomonas Bacillus Pertussis Exotoxin Toxin
27
28
29 Bottom Line :
30
Pseudomonas aeruginosa, a common cause of repeated pneumonia in patients with cystic f ibrosis, is known to produce exot oxin A, which
31 inhibits ribosomal elongation factor 2 in t he host cell, t hereby shutt ing down protein synt hesis. Pseudomonas exotoxin A has the ident ical
32 mechanism of action t o Diphtheria toxin.
Cystic fibrosis Pseudomonas aeruginosa Exotoxin Pneumonia Pseudomonas Protein Diphtheria Pseudomonas exotoxin Protein biosynthesis Protein synthesis EEF2 Toxin Elongation factor Mechanism of action
33
34
• 35
l@ljl'il·1i•J for yea r:[2017 • J
• 36 FIRST AID FA CTS

. 37
. 38 FA17 p 139.3
• 39 Pseudomonas Treatments include "CAMPFIRE" drugs:
Aerobic, motile, gram 8 rod. Non-lactose
• 40
aeruginosa fermenting, oxidase$. Produces pyocyanin • Carbapencms
• 41
(blue-green pigment IJ); has a grape-like odor. • Aminoglycosides
• 42
Produces endotoxin (fever, shock), exotoxin A • lonobactams
. 43
(inactivates EF-2), phospholipase C (degrades • Polymyxins (eg, polymyxin B, colistin)
• 44
cell membranes), and pyocyanin (generates • Fluoroquinolones (eg, ciproAoxacin,
• 45
• 46
reactive oxygen species). levoAoxacin)
• 47 PSEUDO~ONAS is associated with: • Th lRd- and fourth-generation
• 48 • Pneumon ia, pyocyanin cephalosporins (eg, ceftazidime, cefepime)
. 49 • Sepsis • Extended-spectrum penicilli ns (eg,
• 50 • Ecthyma gangrenosum piperacillin, ticarcillin)
8
Lock
s
Suspend
0
End Block
Item: 34 of ~ ,• Mark <:::1 t::> ""I ~· ~'j
100 J.. Previous Next faiUI~S
LAb Notes Calculator

24 • •
FA17 p 128 .1
25
Bugs with exotoxins
26
BACTERIA TOXIN MECHANISM MANIFESTATI ON
27
28 Inhibit protein synthesis
29 Corynebacterium Diphtheria toxin3 Pharyngitis with pseudomembranes in th roat
30
diphtheriae Inactivate elongation facto r and severe lymphadenopathy (bull neck)
31
Pseudomonas Exotoxin N (EF-2) Host cell death
32
33
aeruginosa
34 Shigella spp. Shiga toxin (ST)3 GI mucosal damage .... dysentery; ST also
• 35 Inactivate 60S ribosome by enhances cytokinc release, causi ng hemolytic-
• 36 removing adenine from uremic syndrome (HUS)
. 37 rR A
Enterohemorrhagic Sh iga-like toxin SLT en hances cytokine release, causing HUS
. 38
E coli (EHEC) (SLT)a (prototypically in EHEC serotype 0157:H7).
• 39
• 40
Unlike Shigella, EHEC does not invade host
• 41
cells
• 42 Increase flu id secretion
. 43
Enterotoxigenic Heat-labile Overactivates adenylate Watery diarrhea: "labile in the Air (Aclenylate
• 44
E coli (ETEC) toxin (LT)3 cyclase (t cAMP) .... t CI- cyclase), stable on the Ground (Guanylate
• 45
secretion in gut and H7 0 cyclase)"
• 46
efAux
• 47
I Ieat-stable Overactivates guanylate
. 48
. 49
toxin (ST) cyclase (t cGMP)
• 50 •
- l resorption of NaCI •

8
Lock
s
Suspend
0
End Block
Item: 34 of
100

25
- ,•
~
Mark

Clostridium
-<]
Prev1o u s
1:>
Next
""'I
Labf a lu es

Botulinum toxin 3

Note s

p
1!';:'1
Calculator

of proteins required for


~ .
Flaccid paralysis, Aoppy baby; toxin prevents
26 botulinum neurotransmitter release release of stimulatory (ACh) signals at
27 via vesicular fusion neuromuscular junctions - Aaccid paralysis
28
a An AB toxin (aka, two-component toxin [or three for anthrax]} with B enabling binding and triggering uptake (endocytosis)
29
30
of I he act i'e A component. The A components are usuallr ADP ribosylt ransfcrases; others ha\'e enzymatic activities as listed
31
in chart.
32
Lyse cell membranes
33
34
Clostridium Alpha toxin Phospholipase (lecithinase) Degradation of phospholipids - myonecrosis
• 35 perfringens that degrades tissue and ("gas gangrene") and hemolysis ("double zone"
• 36 cell membranes of hemolysis on blood agar)
. 37 Streptococcus Streptolysin 0 Protein that degrades cell l.yses RBCs; contributes to ~-h emolysis;
. 38 pyogenes membrane host antibodies against toxin (ASO) used to
. 39 diagnose rheumatic fe,·er (do not confuse
• 40
with immune complexes of postslreptococcal
• 41
glomeruloneph ril is)
• 42
. 43
Superantigens causing shock
• 44 Staphylococcus Toxic shock Binds to IIIC II and TC R Toxic shock syndrome: fever, rash, shock; other
• 45 aureus syndrome toxin outside of antigen binding toxins cause scalded skin syndrome (exfoliative
• 46 (TSST-1) site to cause overwhelming to>.in) and food poisoning (heat-stable
• 47 cnterotoxi n)
release of IL-1, IL-2,
. 48
Streptococcus Exotoxin A IF.N-y, and TN F-a Toxic shock-like svndrome: fe\'er, rash, shock;
. 49
pyogenes - shock scarlet b ·er •
• 50

8
L.odt
s
Su~pl'nd
~
End Block
Item: 35 of - ,• Mark -<J [:::> "'I ~ · ~
100 ~ P~v1ous N @xt Labl lues N o tes Calculato r

24
A 36-year-old nurse comes to t he physician with a runny nose, headache, sneezing, sore t hroat, and a complaint of post nasal drip. He
25
denies any history of fever, myalgia, or dysphonia. The pati ent says that every year he has similar symptoms during t he same season. He
26 has felt this way for 3 days and notes t hat the doctor he works for has complained of similar sympt oms. The patient also states t hat he
27 forgot to get his f lu shot this year.
28
29 Which of the following is the most common cause of this patient's illness?
30 :

31 A. Adenovirus
32 B. Coronavirus
33
C. Orthomyxovirus
34
. 35 D. Parainfluenza virus
• 36 E. Rhinovirus
. 37
. 38
• 39
• 40
• 41
• 42
• 43
• 44
. 45
• 46
• 47
. 48
. 49
. 50

8
Lode.
s
S uspe-nd
8
End Bloc:k
Item: 3S of ~ ,• Mark <::J [:::> ""I ~· ~'j
100 J.. Previous Next LAbfaiUI~S Notes Calculator

24
25 The correct ans wer is E. 54°/o chos e this .
26 The rhinovirus is t he most f req uent cause of t he common cold . Coronaviruses are t he second most common cause . Viral upper respi rat ory
t ract infections most often occur during winter and are a common afflict ion. They are self- limit ing and req uire no t reatment.
27 Rhinovirus Common cold Upper respiratory tract Respiratory tract Coronavirus Upper respiratory tract infection Coronaviridae Respiratory tract infection
28
A is not correct . 9 °/o chos e this.
29
Adenovirus is a common cause of upper respi ratory t ract infection and conjunct ivit is in child ren. It can man ifest very similarly t o group A st re p
30 infection, with exudat ive tonsilit is and cervical adenopat hy, and can also manifest wit h malaise, headache, myalgias, and abdominal pain.
31 Tonsillitis Upper respiratory tract infection Adenoviridae Conjunctivitis Lymphadenopathy Malaise Headache Respiratory tract Upper respiratory tract Abdominal pain Respiratory tract infection Exudate

32 Infection Group A streptococcal infection

33
B is not corre ct. 7% chos e this .
34 The coron avirus is t he second most common cause of the common cold . They are often charact erized by upper respi rat ory t ra ct symptoms of
35 nasal congest ion and rh inorrhea. They have also been implicated in acut e vira l ot it is media.
Rhinorrhea Otitis media Common cold Nasal congestion Coronavirus Upper respiratory tract Respiratory tract Otitis Virus
• 36
. 37 C is not co rrect. 22% chos e this .
. 38 Orthomyxovirus causes t he f lu. Pat ients wit h the flu generally present with feve r (often a hig h feve r), headache, muscl e aches, dry coug h,
• 39
rh inorrhea, and extreme fat igue. The symptoms of f lu t end t o last for a more prolonged duration t han this patient is present ing with and tend
t o manifest with high feve r.
• 40 Orthomyxoviridae Rhinorrhea Myalgia Headache Fatigue (medical) Cough Influenza Fever
• 41
D is not correct. 8°/o chos e this .
• 42
Parainf luenza v irus is a common cause of vira l respiratory illness in children. Most adults present wth nonspecific symptoms such as feve r,
. 43 rh inorrhea, cough, and/or sore throat, although most immunocompetent adu lt s are asympt omat ic when infected wit h para inf luenza v irus.
• 44 Rhinorrhea Human parainfluenza viruses Asymptomatic Sore throat Cough Virus Respiratory disease Fever

• 45
• 46
Bottom Line:
• 47
The rhi novirus, a sing le-stranded linear RNA virus of t he fa m ily Picornaviridae, is the most common cause of the common cold .
• 48 Rhinovirus Picornavirus Common cold Virus RNA virus RNA
. 49
• 50

8
Lock
s
Suspend
0
End Block
Item: 3S of ~ ,• Mark <::J [:::> ""I ~· ~'j
100 J.. Previous Next LAb faiUI~S Notes Calculator

24 • •
FA1 7 p 164.4
25
26 Rhinovirus A picornavirus. 1 onenveloped R A ,·irus. Rhino has a ru nny nose.
27 Cause of common cold; > 100 serologic
28 types. Acid labile- destroyed by stomach acid;
29 therefore, does not infect the G l tract (unlike
30 the other picornaviruses).
31
32
FA1 7 p 163.1
33
RNA viruses
34
35 VIRAL FAMILY ENVELOPE RNA STRUCTURE CAPSID SYMMETRY MEDICAL IMPORTANCE
• 36 Reoviruses 0 OS linear Icosahed ral Coltivirus" - Colorado tick fever
. 37 10-12 segments (double) Rotavirus-cause offatal diarrhea in children
. 38
Picornaviruses No SS ® linear Icosahed ral Poliovirus-polio-Salk/Sabin vaccines-IPV/OPV
• 39
Echovirus- aseptic meningitis
• 40
Rhi novirus-"common cold"
• 41
Coxsackievirus-aseptic meningitis; herpangina
• 42
(mouth blisters, fever); hand, foot, and mouth
. 43
disease; myoca rditis; pericard itis
• 44
IIAV-acute viral hepatitis
• 45
P E RCI I
• 46
• 47 Hepevirus No SS ® linear Icosahed ral HEV
. 48 Caliciviruses No Icosahedral 'orovirus-vira l gastroenteritis
SS ® linear
. 49
Flaviviruses Yes SS ® linear Icosahedral HCV •
• 50 •
8
Lock
s
Suspend
0
End Block
Item: 35 of - ,• Mark -<J [:::> "'I ~ · ~
100 ~ P~v1ous N @xt Labl lues N o tes Calculato r

24 A A

25
Retroviruses Yes SS ® linear Icosahedral ! lave re,·erse transcriptase
26
2 copies (HTLV), HTLV-T-cellleukemia
27
complex II IV-AIDS
28
and conical
29
(HIV)
30 Coronaviruses Yes SS ® linear Helical "Common cold," SARS, \fERS
31
Orthomyxoviruses Yes sse linear Helical lnAuenza virus
32
8 segments
33
34 Paramyxoviruses Yes sse linear Helical PaRaMno,·irus:
'
35 1 onsegmented ParainAucnza-croup
' 36 RSV-bronchiolitis in babies; Rx- ribavirin
. 37 \lcaslcs, \lumps
. 38 Rabies
Rhabdoviruses Yes sse linear Helical
' 39
• 40
Filoviruses Yes sse linear Helical Ebola/ 1arburg hemorrhagic fever-often fatal!
' 41 Arenaviruses Yes SS®and e f felica l I.CMV- lymphocytic choriomeningitis virus
• 42 circular Lassa fever encephalitis-spread by rodents
. 43 2 segments
• 44
Bunyaviruses Yes ss e circular Helical Ca Iiforn ia encephal itis 3
' 45
3 segments S<l11d ny/Rift Valley fevers 3
' 46
Crimean-Congo hemorrhagic fe,·e~
' 47
. 48
llantavirus- hemorrhagic fe\'er, pneumonia
. 49 Delta virus Yes ss e circular Uncertain HDV is a "defective'' ,-irus that requires the
' 50 • presence of II BV to rep) icate
8
Lode.
s
S uspe-nd
8
End Bloc:k
Item: 36 of - ,• Mark -<J [:::> "'I ~ · ~
100 ~ P~v1ous N @xt Labl lues N o tes Calcula to r

24 A A

A 24-year-old woman complains of increased frequency of urinati on and a burning sensati on when she urinates. Her symptoms began the ~~AI
25
previous day, and she denies feve r, chil ls, flank pain, and any new sex partners. Urinalysis is posit ive for leukocyte est erase and nitrites.
26 She is placed on an antibiot ic classically used to t reat her infect ion; this antibiotic can result in anemia wit h hypersegment ed neutrophils
27 and enlarged red blood cells.
28
29 What is the mechanism of action of the drug prescribed to treat this patient's infection?
30 :
31 A. Blocks cell wall synthesis
32 B. Blocks mRNA synthesis
33
C. Blocks nucleotide synthesis
34
35
D. Blocks peptidoglycan synthesis
' 36 E. Blocks protein synthesis by binding t o t he 505 ri bosomal subunit
. 37
F. Forms toxic metabolit es in the cell wall
. 38

' 39
• 40

' 41
• 42
• 43
• 44

' 45
' 46
' 47
. 48
. 49

' 50

8
Lode.
s
S uspe-nd
8
End Bloc:k
Item: 36 of ~ ,• Mark <::J [:::> ""I ~· ~'j
100 J.. Previous Next LAb faiUI~S Notes Calculator

24 The correct answer is C. 53°/o chose this.


25 About 80% of urina ry t ract infections (UTis) are caused by Escherichia coli, a gram-negat ive bacillus. I n young women, t he second most
26 common cause of UTis (approximat ely 10% -15%) is Staphylococcus saprophyticus, a gram- posit ive, coagulase-negative, and novobiocin-
resistant organism. Trimethoprim -sulfamet hoxazole (TMP-SMX) is t he f irst-line therapy fo r uncomplicat ed UTis and acts by blocking the
27
synt hesis of folate, which is essent ial for nucleot ide synthesis. Tri methopri m inhibits bacterial dihydrofolate reductase, and sulfamet hoxazole
28 inhibits dihydropteroat e synt hase. Megaloblast ic anemia is charact erised by enlarged red cells and hypersegment ed neut rophils on a
29 peripheral blood smear. This illust rates a pot ent ial t oxicity to TMP-SMX.
Megaloblastic anemia Escherichia coli Dihydrofolate reductase Trimethoprim/sulfamethoxazole Trimethoprim Gram-negative bacteria Staphylococcus saprophyticus Dihydropteroate synthase Folic acid
30
Gram-positive bacteria Anemia Neutrophil Sulfamethoxazole Urinary tract infection Staphylococcus Nucleotide Urinary system Blood film Bacillus Dihydrofolic acid Red blood cell
31
32 A is not correct. 9°/o chose this.
33 Penicillin blocks t ranspeptidase cross -linking of cell walls, but does not cover gram- negat ive rods. Ampicillin has the same mechanism of
act ion of penicillin, but covers a wider spect rum of bacteria, including Escherichia coli. Howeve r~ for uncomplicat ed urinary t ract infect ions,
34
TMP-SMX remains t he fi rst - line t herapy.
35 Escherichia coli Ampicillin Penicillin Gram-negative bacteria Trimethoprim/sulfamethoxazole Urinary tract infection Bacteria Urinary system Peptidyl transferase 00-transpeptidase Mechanism of action

36
B is not correct. 9% chose this .
. 37
Rifamp in blocks mRNA synt hesis by inhibi t ing DNA-dependent RNA polymerase, and it is t ypically used to t reat tuberculosis.
. 38 Rifampicin Tuberculosis Messenger RNA RNA polymerase RNA

• 39
D is not correct. 9°/o chose this .
• 40
Vancomycin blocks peptidoglycan synthesis by binding D-ala-D-ala in cell wa ll precursors. The use of vancomycin is t ypically rest rict ed t o
• 41 mult idrug- resistant gram- posit ive bacteria, such as met hicillin-resist ant Staphylococcus aureus (M RSA) .
• 42 Peptidoglycan Vancomycin Staphylococcus aureus Cell wall Gram-positive bacteria Methicillin-resistant Staphylococcus aureus Staphylococcus Bacteria Multiple drug resistance 0-Aia-0-Aia dipeptidase

. 43 E is not correct. 14°/o chose this .


• 44 Erythromycin and other macrolides inhibit protein synt hesis by binding t o t he 50S subunit of t he bacteria l ribosome. Howeve r~ these
• 45 ant ibiot ics are not recommen ded for the treat ment of UTis, because t hey do not work against Escherichia coli.
Erythromycin Escherichia coli Ribosome Protein biosynthesis Macrolide Protein Protein synthesis Antibiotics 50S Urinary tract infection
• 46
• 47 F is not correct. 6°/o chose this .
• 48 Met ron idazole is a bact ericidal agent t hat works by creat ing t oxic metabolites in t he bact eri al cell wall. It covers anaerobes and prot ozoa, such
as Giardia, Entamoeba, Trichomonas, and Gardnere/la vagina/is .
. 49
Metronidazole Protozoa Cell wall Gardnerella vaginalis Anaerobic organism Bactericide Trichomonas Giardia Entamoeba
• 50

8
Lock
s
Suspend
0
End Block
Item: 36 of ~ ,• Mark <::J [:::> ""I ~· ~'j
100 J.. Previous Next LAbfaiUI~S Notes Calculator

24
25
Bottom Line:
26 TMP-SMX is the first -line t herapy for uncomplicated urinary t ract infect ions and acts by blocking the synthesis of folat e, which is essential fo r
nucleotide synt hesis.
27 Folic acid Trimethoprim/sulfamethoxazole Nucleotide Urinary system Urinary tract infection First-line therapy
28
29
30 l@ljl'il·1i•J for year:[ 2017 • J
FIRST AID FA CTS
31
32
FA17 p 177.2
33
34
UTI bugs
35 SPECIES FEATURES COMMENTS
36 Escherichia coli Leading cause of UTI. Colonies show green Diagnostic markers:
. 37 metallic sheen on EMB agar. (f) Leukocyte esterase= evidence of WBC
. 38
Staphylococcus 2nd leading cause of UTI in sexua lly active activity.
• 39
saprophyticus women. Ef> litrite test = reduction of urinary nitrates
• 40 by bacterial species (eg, E coli).
• 41 Klebsiella pneumoniae 3rd leading cause of UTI. Large mucoid capsule
Ef> Urease test = urease-producing bugs (eg,
• 42 and viscous colon ics .
S sa(Jrophyticus, Proteus, Klebsiella).
. 43 Serratia marcescens Some strains produce a reel pigment; often
• 44 nosocomial and drug resistant.
• 45
• 46
Enterococcus Often nosocomial and drug resistant.
• 47 Proteus mirabilis Motility causes "swarming" on agar; produces
• 48 urease; associated with struvite stones .
. 49
Pseudomonas Blue-green pigment and fruity odor; usually
• 50 - · ---- • 1 1 1

Lock
8 s
Suspend
0
End Block
Item: 36 of - ,• Mark --<] [::> ""'I ~· 1!';:'1
100 ~ Prev1o u s Next Labf a lu es Notes Calculator

g I

25
Enterococcus Often nosocomial and drug resistant.
26
27
Proteus mirabilis Motility causes '·swa rming" on agar; produces
28
urease; associated with strU\·ite stones.
29 Pseudomonas Blue-green pigment and fruity odor; usuall)
30 aeruginosa nosocomial and drug resistant.
31
32
FA17 p 190.3
33
34
Trimethoprim
35 MECHANISM Inhibits bacterial dihydrofolate reductase.
36 Bacteriostatic.
. 37
CLINICAL USE Used in combination with sulfonamidcs
. 38
(trimethoprim-sulfamethoxazole [TMP-
. 39
• 40
S IX]), causing sequential block of folate
• 41
synthesis. Combination used for UTis,
• 42 Shigella, Salmonella, P11eumocystis jirovecii
. 43 pneumonia treatment and prophylaxis,
• 44 toxoplasmosis prophylaxis.
• 45 ADVERSE EFFECTS Megaloblastic anemia, leukopenia,
• 46 granulocytopenia. (Mar alb·iatc with
• 47
supplemental folinic acid). T~ IP '1reats
. 48
!\Iarrow Poorh-.

. 49

• 50 •
8
L.odt
s
Su~pl'nd
~
End Block
Item: 36 of - ,• Mark -<J [:::> "'I ~ . ~
100 ~ P~v1ous N@xt Labl lu es No tes Calcula t o r

24
FA17p183o1 •
25
26
Antimicrobial therapy
27
28 GYRASE
FOLIC ACID SYNTHESIS
29 AND REDUCTION
30 (DNAmtthyt.ltlonl Metronidazole Auoroquinolones
Ciprotloxacin
31 Levofloxacin. etc
Sulfonamides
32 Sulfamethoxazole BACTERIAl CEll Ouinolone
33 Sulfisoxazole Nalidixic acid
34
Suit adlclzme
35
Tnmethopnm
36
. 37 RN
0
38 Pl-1 n
DHF PROTEIN SYNTHESIS
0 39
• 40 50S SUBUNIT
mRNA
• 41 Chloramphenicol
• 42 Cl1ndamyc1n
CELL WALL SYNTHESIS
0
43 Linezolid
THF
• 44 r 'TIDOG YCAN SYNTHESIS Macrolides
Azithromycm
. 45 Celt Clarithromycin
Glycopeptides membrane
• 46 Vancomyc1n Erythromycin
• 47 BacitraCin Celt Wall Streptogramins
Ouinupnstm
0
48 Dalfopnslln
DO AN ~ft-. . ' '
. 49
0 50 I Penicillinase·sensitive oenicimns Antipseudomonal Carbaoene~ 30SSUBUNIT

8
Lode.
s
S uspe-nd
8
End Bloc:k
25 Sulfonamides
Sulfamethoxazole BACTERIAL CELL Ouinolone
26
Sulhsoxazole PABA Nalidixic acid
27 SulfadiaZine
28
29 Tnmethopnm
30
31
32 PROTEIN SYNTHESIS
33
50S SUBUN""
34
Chloramphenicol
35
Cllndamycm
36 CUL WAU SYNTHESIS
unezolid
. 37 THF
r 100" -AN S •!JHESIS Macrolides
. 38 Az1thromycm
Cett Clarithromycin
0 39 Glycopeptides membrane
Vancomycin Erythromycin
40 BacitraCin
0
Celtwau Streptogramins
0
41 Ouinupristin
Oalfopnstin
0
42 " DJIOOG YCAN c OSS·LINKING
. 43 30SSUBUNIT
Penicillinase-sensitive penicillins Antipseudomonal Carbapenems
0 44 Penicillin G. V Ticarcillin lmipenem Aminoglycosides Glycyclines
0
45 Ampic1lhn Piperacillin Meropenem Gentamidn Tigecycline
Amoxicllhn Cephalosporins (1-Vl E.rtapenem Neomycin
46 Tetracyclines
0

Penicillinase-resistant penicillins lst-Cefazolin. etc Ooripenem Am1kacin Tetracycline


0
47 Oxac.1lhn 2nd-Cefoxitin. etc Monobactams Tobramydn Doxycycline
. 48 Nafc•lhn 3rd-Ceftriaxone. etc Aztreonam Streptomycin Minocycline
D•cloxacllbn 4th-Cerep•me
. 49 Slh-Ceftarotine J
0 50 •
8
Lode.
s
S uspe-nd
8
End Bloc:k
Item: 37 of - ,• Mark -<J
P~v1ous
[:::> "'I ~ ·· ~
100 ~ N @xt Labl lues N o tes Calcula to r

24
An epidemic of a diarrhea has broken ou t in a city hospital. Colonoscopy of one of the affected pat ients reveals colonic inflammation with
25
exudates and necrosis of th e mucosal surface. Assays for toxin A and toxin B are posit ive.
26
27
Which of the following is the m icrobiology laboratory likely t o isolate from the affected patient s?
28
:
29 A. A gram-negative facultative int racellu lar organism
30
B. A gram-negative lactose fermenter
31
32 C. A gram-negative lactose nonfermenter
33 0. A gram-positive aerobe
34
E. A gram-positive anaerobe
35

36
. 37
. 38
0 39
• 40
0
41
• 42
• 43
• 44
0
45
0 46
0
47
. 48
. 49
0 50

8
Lode.
s
S uspe-nd
8
End Bloc:k
Item: 37 of ~ ,• Mark <::J [:::> ""I ~· ~'j
100 J.. Previous Next LAb faiUI~S Notes Calculator

24
25 The correct a nswer is E. 5 1 °/o chose this .
26 The description of colonic inflammation wit h exudates and necrosis of the m ucosal surface describes the pseudomembranous colit is of
Clostridium difficile, of which there have been several out breaks. C. difficile is a gram -posit ive anaerobe spore-fo rmer t hat produces toxin A
27
(which causes diarrhea) and toxin B (which is cytot oxic). AB t oxin causes cell apopt osis. Lat er st ages of C. difficile infection can result in
28 necrosis as described in t he vignett e. Strains t hat produce an increased amount of these toxins have led to increased morbidity and even
29 death associated wit h C. difficile colit is.
Clostridium difficile colitis Clostridium difficile (bacteria) Gram-positive bacteria Apoptosis Diarrhea Necrosis Anaerobic organism Colitis Cytotoxicity Toxin Inflammation Clostridium Morbidity
30
Colon (anatomy) Large intestine Mucous membrane
31
32 A is not correct . 9°/o chose t his.
33 Salmonella is an exam ple of a gram- negative facultat ive int racellu lar organ ism and could produce diarrhea, but only Clostridium difficile
produces t oxin A and toxin B. It ent ers via the int estinal cell's surface membrane using a type III injector system t o inject prot eins into the
34
cyt oplasm and rearrange t he cytoskelet on of the host cell to engulf the bacteria. Salmonella blocks lysosomes within the cell and Salmonella
35 divides wit hin its vacuole and event ually engulfs t he ent ire cell.
36 vacuole Gram-negative bacteria Cytoplasm Cytoskeleton Salmonella Clostridium difficile colitis Diarrhea Clostridium difficile (bacteria) Toxin Intracellular Clostridium Bacteria Organism Lysosome

37 Biological membrane Intracellular parasite

. 38 B is not correct. 14% chose this .


• 39 Gram -negative lactose fermenters t hat can cause diarrhea include Escherichia coli, but not Clostridium difficile. The heat - labile ent erotoxin
• 40 (ETEC) act ivat es adenylate cyclase via ADP ribosylation of Gs, which activat es a second messenger pat hway which increases cAMP and causes
secret ory diarrhea . Heat -st able ent erotoxin (ETEC) act ivates guanylat e cyclase act ivates t he same second messenger pathway and causes
• 41
secret ory diarrhea .
• 42 Heat-labile enterotoxin Gram-negative bacteria Escherichia coli Heat-stable enterotoxin Adenylyl cyclase Diarrhea Clostridium difficile (bacteria) Lactose Cyclic adenosine monophosphate
. 43 Clostridium difficile colitis Guanylyl cyclase Enterotoxin Adenosine diphosphate Second messenger system
• 44
C is not co rrect. 16% chose this .
• 45
Gram -negative lactose nonferment ers t hat can cause diarrhea include Shigella and Salmonella, but not Clostridium difficile. Shigella acts via
• 46 t he shiga toxin and binds t o t he 605 ribosome, which inhibit s protein synt hesis.
• 47 Gram-negative bacteria Ribosome Shiga toxin Lactose Shigella Clostridium difficile colitis Diarrhea Clostridium difficile (bacteria) Protein Salmonella Protein synthesis Clostridium Toxin Protein biosynthesis

• 48 D is not correct. 10% c hose this .


. 49 Clostridium difficile is an obligate anaerobe; the ot her gram -posit ive organisms are aerobic or facult at ive anaerobes .
Obligate anaerobe Gram-positive bacteria Anaerobic organism Facultative anaerobic organism Clostridium difficile colitis Clostridium difficile (bacteria) Clostridium Aerobic organism Facultative
• 50

8
Lock
s
Suspend
0
End Block
Item: 37 of - ,• Mark -<J
P~v1ous
[:::> "'I ~ ·· ~
100 ~ N @xt Labl lues N o tes Calcula to r

24 •
25 Botto m Line:
26 Clostridium difficile is a gram-positive anaerobic organism that produces toxin A and toxin B, causing diarrhea and necrosis of the colonic
27
mucosa, respectively. It is a source of increasing morbidity, and appropriate preventive measures, most not ably hand washing and judicious
use of antibiotics, must be employed.
28 An.Jterob1c oro~" ~m Gra.n~pos1bve bactena Clostridium difficile colitis Diarrhea Clostridium drffictle (bacte 11) Closb 1d1um Anbb1otics Toxm Organ1sm Necrosis Morbidity Mucous membrane Hand ..-ashtno
29 Large •nte5bne

30
31
32 l@l :fil·11•1 for year: 2017 •
FI -ST A IO FACTS
33
34
FA17 p 134.2
35
Clostridia (with Cram EEl, spore-forming, obligate anaerobic rods.
36
exotoxins)
37
0
38 Ctetani Produces tetanospasmin, an exotoxin causing Tetanus is tetanic paralysis.
0 39 tetanus. Tetanus toxin (and botulinum toxin)
• 40 are proteases that clea,·e SJ ARE proteins for
• 41 neurotransmitters. Blocks release of in hibitory
• 42 neurotransmitters, C ABA and glycine, from
0
43 Renshaw cells in spinal cord .
• 44
Causes spastic paralysis, trismus (lockjaw), ris11s
. 45
sardonicus (raised eyebrows and open grin),
• 46
opisthotonos (spasms of spinal extensors).
• 47
Pre' enl with tetanus vaccine. Treat "ith
0
48
antitoxin +/- ,·accine booster, diazepam (for
0
49
0 50
muscle spasms), and wound debridement.
8
Lode.
s
S uspe-nd
8
End Bloc:k
Item: 37 of - ,• Mark -<J
P~v1ous
[:::> "'I ~ ·· ~
100 ~ N @xt Labl lues N o tes Calcula to r

24 A A

C botulinum Produces a heat-labile toxin that inhibits Symptoms of botulism (the 4 D's): Diplopia,
25
ACh release at the neuromuscular junction, Dysarthria, Dysphagia, Dyspnea.
26
27
causing botulism. In adu lts, disease is caused Botulinum is from bad bottles of food, j1lice, and
28
by ingestion of preformed toxin. In babies, honey (causes a descending flaccid paralysis).
29
ingestion of spores (eg, in honey) leads to Local botox injections used to treat focal
30 disease (Aopp) baby S) ndrome). Treat "ith dystonia, achalasia, and muscle spasms. Also
31 antitoxin. used for cosmetic reduction of facial wrinkles.
32 C perfringens Produces a toxin (lecithinase, a phospholipase) Per{ringens perforates a gangrenous leg.

.... that can cause mronecrosis (gas gangrene )


33

34
.,.. and hemoh·sis

.
35 I • Spores can surYi,·e in undercooked food;
'
......
36
I when ingested, bacteria release heat-labile
37
• ' enterotoxin - food poisoning.
0
38
0 39
• 40 ·M·

• 41
• 42
c difflcile Produces 2 toxins. Toxin A, an enterotoxin, Oif{icile causes diarrhea. Treatment:
0
43
binds to brush border of gut and alters nuid metronidazole or oral vancomycin. For
• 44 secretion. Toxin B, a cytotoxin, disrupts recurrent cases, consider repeating prior
. 45 cytoskeleton via actin depolymerizalion. Both regimen, fidaxomicin, or fecal microbiota
• 46 toxins lead to diarrhea - pseudomembranous transplant.
• 47 zo
colitis B Often to antibiotic use, especia lly
0
48 clindamycin or ampicillin ; associated with PPI
0
49 use. Diagnosed by detecting one or both toxins
0 50 in dnnt h~· :o n~i oPn riPtPr ~inn n r Pr R
8
Lode.
s
S uspe-nd
8
End Bloc:k
Item: 37 of ~ ,• Mark <::J [:::> ""I ~· ~'j
100 J.. Previous Next faiUI~S
LAb Notes Calculator

24 • •
FA17p 127.1
25
26 Spore-forming Some bacteria can form spores fJ at the end Bacillus anthracis Anthrax
27 bacteria of the stationary phase when nutrients are Bacillus cereus Food poison ing
28 limited. Clostridium botulinum Botulism
fJ
29 Spores are highly resistant to heat and Clostridium difficile Pseudomembranous
30 chemicals. Have dipicolinic acid in their core. colitis
31 Ha,·e no metabolic activity. Must autoclave to Clostridium per{ringens Gas gangrene
32 potentially kill spores (as is done to surgical Clostridium tetani Tetanus
33 equipment) by stct~ming at I21°C for 15
34 minutes.
35
1,000 "1:1
36
37 FA17 p 175.1
. 38 Bugs causing diarrhea
• 39
• 40
Bloody diarrhea
• 41 Campylobacter Comma- or S-shaped organisms; growth at 42°C
• 42 E histolytica Protozoan; amebic dysentery; liver abscess
. 43
• 44
Enterohemorrhagic 0157:1-17; can cause llUS; makes Shiga-like toxin
• 45 Ecoli
• 46 Enteroinvasive Ecoli Invades colonic mucosa
• 47
Salmonella (non- Lactose 8; Aagellar motility; has animal reservoir, especially poultry and eggs
. 48
typhoidal)
. 49
• 50 •
Shigella Lactose 8; very low 10,11; produces Shiga toxin (human reservoir only); bacillary dysentery •

8
Lock
s
Suspend
0
End Block
Item: 37 of ~ ,• Mark <::J [:::> ""I ~· ~'j
100 J.. Previous Next LAbfaiUI~S Notes Calculator

24 • •
25
FA17 p 175.1
26 Bugs causing diarrhea
27 Bloody diarrhea
28
Campylobacter Comma- or S-shaped organisms; growth at 42°C
29
30 E histolytica Protozoan; amebic dysentery; liver abscess
31 Enterohemorrhagic 0157:H7; can cause !IUS; makes Shiga-like toxin
32 Ecoli
33
Enteroinvasive Ecoli Invades colonic mucosa
34
35 Salmonella (non- Lactose 8; Aagellar motility; has animal reservoir, especially poultry and eggs
36 typhoidal)
37 Shigella Lactose 8; very low 10 ;0; produces Shiga toxin (human reservoir only); bacillary dysentery
. 38
• 39
Yenterocolitica Day care outbreaks, pseudoappendicitis
• 40 Watery diarrhea
• 41
Cdifficile Pseudomembranous colitis; associated with antibiotics and PPis; occasionally bloody diarrhea
• 42
. 43
C perfringens Also causes gas gangrene
• 44 Enterotoxigenic E coli Tra,·elers' diarrhea; produces heat-labile (LT) and heat-stable (ST) toxi ns
• 45
Protozoa Giardia, Cryptosporidium
• 46
• 47 Vcholerae Comma-shaped organisms; rice-water diarrhea; often from infected seafood
. 48 Viruses Rota,·irus, norovirus, adenovirus
. 49
• 50 • •

8
Lock
s
Suspend
0
End Block
Item: 38 of - ,• Mark -<J [:::> "'I ~ · ~
100 ~ P~v1ous N @xt Labl lues N o tes Calcula to r

24 A A

A 19-year-old student presents wit h complaint s of a low-grade fever and general fatigue for approximat ely 9 days. His temperature is 38° C ~~AI
25
( 100.4° F), blood pressure is 108/65 mm Hg, pulse is 87/min, and respiratory rate is 20/min. Physical examinat ion reveals cervical
26 lymphadenopathy. Results of a cervical chain lymph node biopsy are shown in the image.
27
28
29
30
31
32
33
34
35
36
37
. 38
0 39
• 40
0
41
What pathogen is respons ible fo r t he pat ient's il lness?
• 42
• 43
:
A. Cytomegalovirus
• 44
0
45 B. Epstein-Barr virus
0 46 C. Human herpesvirus 1
0
47
D. Human herpesvirus 8
. 48
. 49 E. Varicella-zoster virus
0 50

8
Lode.
s
S uspe-nd
8
End Bloc:k
Item: 38 of - ,• Mark -<J [:::> "'I ~ · ~
100 ~ P~v1ous N @xt Labl lues N o tes Calcula to r

24 A A

25
The correct an sw er is A . 48°/o chose this.
26 The patient presents with symptoms of an infectious mononucleosis- like syndrome. These symptoms
27 include malaise, low-grade fever, and cervical lymphadenopathy. Although mononucleosis is due to Epst ein-
28 Barr virus, frequent causes of mononucleosis-like syndromes include cytomegalovirus (CMV), acute
toxoplasmosis, and acute HIV infect ion. In this case, the image shows the characteristic "owl's eye"
29
inclusions of CMV. CMV's "owl 's eye" inclusions are cellular nuclei that are about half the size of the cell and
30 may have a small surrounding halo. These are not to be confused with the inclusions seen in Reed-
31 Sternberg cells of Hodgkin lymphoma (shown here), though RS cells often have two nuclei, instead of just
one. Though there are many similarities between CMV and EBV mononucleosis syndrome, there are some
32
differences that should be discussed. In contrast to EBV, CMV mononucleosis causes a milder, "typhoidal"
33 picture, where systemic symptoms and fever predominate and there is generally less significant
34 lymphadenopathy and mild or absent pharyngitis. In addition, splenomegaly and hepatomegaly are much
less common in CMV mononucleosis. Typically, patients are diagnosed by IgM antibodies to CMV, rather than lymph node biopsy.
35 Spl ~Of'""t!!al
InfectiOUS mononu~ leo$1$ Lr ph node Toxoplasmosis Epstein-Barr virus Cytomegalovirus HepMol'l"'\eeaf LYJ phadenopath Pharyngitis Immunoglobulin M HIV Mala se S.ops Anbbody
36 Lymphoma Voru• Sogn. and symptoms of HIV/A!DS Cervocallymphadenopathy Lymph Infection Fever Cell nucleus
37
B is n ot correct . 44°/o chose thi s.
38
Epstein- Barr virus causes mononucleosis in young adults, but does not cause numerous giant inclusion cells. EBV is described as having
39
0
atypical lymphocytes on histology and a posit ive monospot test.
• 40 Epste1n-Barr v~rus InfectiOUS mononucleosis Histology Heterophile antibody test Virus Lymphocyte

0
41
C is n ot correct. 4 °/o chose thi s .
• 42
Human herpesvirus type 1 is t ransmitt ed through respirat ory secretions and saliva and can result in gingivost omatit is, keratoconjun ctivit is,
. 43 temporal lobe encephalit is, and herpes labialis. It is not a known cause of mononucleosis .
Herpes lab1al•s Infectious mononucleosis Temporal lobe Encephalitis Herpes simplex Herpesviridae Sal•va
• 44
0
45 D i s not correct . 2 °/o chose this .
0 46 Human herpesvirus type 8 is transmit ted through sexual contact and can resu lt in Kaposi 's sarcoma in pat ient s co-infected with HIV. It is not a
0
47
known cause of mononucleosis.
k'!lpos•'s e:'lrcoma lnfect•ou~ mononucleosis Herpesviridae Sarcoma Sexually transm1tted 1nfect1on
. 48
. 49 E is n ot correct . 2 °/o chose thi s .
. 50
Varicella-zoster is a type of herpesvirus that causes chickenpox, shingles, encephalitis, and pneumonia. It does not result in mononucleosis.

8
Lode.
s
S uspe-nd
8
End Bloc:k
Item: 38 of ~ ,• Mark <::J [:::> ""I ~· ~'j
100 J.. Previous Next LAbfaiUI~S Notes Calculator

24
25
Bottom Line:
26
CMV causes a mononucleosis-t ype syndrome t hat is Monospot negat ive. Microscopically, CMV shows characterist ic "owl's eye" inclusion
27 bodies.
28 Cytomegalovirus Inclusion bodies Human cytomegalovirus

29
30
31 i@l;fil·1i•J for year:[ 2017 • J
FIRST AID FACTS
32
33
FA17 p 160.2
34
35
Herpesv iruse s Enveloped, OS, and linear viruses
36 VIRUS ROUTE OF TRANSMISSION CLI NICAL SIGNIFICANCE NOTES
37
Herpes Respiratory Gingivostomatitis, keratoconjuncti vitis a, Most common cause of sporadic
38
simplex secretions, saliva herpes labial is 11], herpetic whitlow on finger, encephalitis, can present as altered
• 39
virus-1 temporal lobe encephalitis, esophagitis, mental status, seizures, and/or
• 40
erythema multiforme. aphasia .
• 41

• 42 Herpes Sexual contact, llerpes genital i s ~. neonatal herpes. Latent in sacral ganglia. ira!
. 43 simplex peri natal meningitis more com mon with
• 44 virus-2 HS -2 than with II SV-1.
• 45 Varicella- Respiratory Varicella-zoster (chickenpox m,
shingles 1)}, Latent in dorsal root or trigem inal
• 46 Zoster virus secretions ganglia; C V 1 bra nch
encephalitis, pneumonia .
• 47
(HHV-3) Most common complication of shingles is post- involvement can cause herpes
• 48
herpetic ne uralgia. zoster ophthalmicus .
. 49
• 50 Epstein-Barr Respiratory Mononucleosis- fever, hepatosplenomegaly, Infects B cells through CD2l.
8
Lock
s
Suspend
0
End Block
Item: 38 of - ,• Mark -<J [:::> "'I ~ · ~
100 ~ P~v1ous N @xt Labl lues N o tes Calcula to r

24 A A

25
Epstein-Barr Respiratory Mononucleosis-fever, hepatosplenomegaly, Infects B cells through CD21.
26
virus (HHV-4) secretions, pharyngitis, and lymphadenopathy (especially Atypical lymphoertes on peripheral
27 sa li,·a· aka posterior cervical nodes [l). Avoid contact sports blood smear [!J-not infected B
'
28 "kissing disease," until resolution due to risk of splenic rupture. cells but reactive cytotoxic T cells.
29 (common in Associated with lymphomas (eg, endemic Et) lonospot test-heterophile
30 teens, young Burkitt l~mph oma), n asopha~ ngeal antibodies detected by agglutination
31 adults) carcinoma (especially sian adults), of sheep or horse RBCs.
32 lymphoproliferati\'e disease in transplant Use of amoxicillin in mononucleosis
33 patients. can cause characteristic
34 maculopapular rash.
35
Cytomegalo- Congenital Mononucleosis (8 Monospot) in Infected cells have characteristic
36
virus (HHV-5) transfusion, immunocompetent patients; infection in "owl eye" inclusions CJ.
37
sexual contact, immunocompromised, especially pneumonia Latent in mononuclear cells.
38
saliva, urine, in transplant patients; esophagitis; AIDS
0 39
transplant retinitis ('\ ightomega lovirus"): hemorrhage,
• 40
cotton-wool exudates, vision loss.
0
41
• 42
Congenital CM
. 43 Human Saliva Roseola infantum (exanthem subitum): high Roseola: fe,·er fi rst, Rosie (cheeks)
• 44 herpes- fevers for several days that can cause seizu res, later.
0
45 viruses 6 foll owed by diffuse macular rash HHV-7-less common cause of
0 46 and 7 roseola.
0
47
Human exual contact Kaposi sarcoma (neoplasm of endothcl ia I cells). Can also affect C I tract and lungs.
. 48
herpesvirus Seen in HIV/AIDS and transplant patients.
. 49
8 Dark/violaceous plaques or nodules
. 50

8
Lode.
s
S uspe-nd
8
End Bloc:k
.
Item: 38 of - ,• Mark -<] 1:> ""'I ~· 1!';:'1
100 ~ Prev1o u s Next Labf a lu es Notes Calculator
; -
25 Human Saliva Roseola infantum (exanthem subitum): high Roseola: b·er first. Rosie (cheeks)
26 herpes- fevers for several days that can cause seizures, later.
27 viruses 6 followed by diffuse macular rash HHV-7- less common cause of
28 and 7 roseola.
29
Human Sexua I contact Kaposi sarcoma (neoplasm of endothelial cells). Can also affect CI tract and lungs.
30
herpesvirus Seen in HI V/AIDS and transplant patients.
31
32
8 Dark/violaceous plaques or nodules
33
representing ,·ascular proIiferat ions.
34
35
36
37
38
. 39
• 40
• 41
• 42
. 43
• 44
• 45
• 46
• 47
. 48
. 49
• 50 •

8
L.odt
s
Su~pl'nd
~
End Block
Item: 39 of - ,• Mark -<] 1:> ""'I ~· 1!';:'1
100 ~ Prev1o u s Next Labf a lu es Notes Calculator

24 •
A 19-year-old college student presents to t he st udent health clinic with a 1-week history of feve r~ headache, and painfu l exudative r~AJ
25
pharyngitis. Physical examination shows significant lymphadenopathy of the cervical nodes, palatal petechiae, and hepatosplenomegaly.
26 Laboratory studies show a WBC count of 15,000/mm 3 with 55% lymphocytes. To determine t he cause of the student's illness, a sample of
27 his blood is added to a solution of horse erythrocytes and found to cause agglutination, yielding a positive test and subsequent diagnosis.
28
29 Which of the following is the most likely cause of t his patient's symptoms?
30 :
31 A. Cytomegalovirus
32 B. Epstein-Barr virus
33
C. HIV infection
34
35
D. Streptococcus pneumoniae
36 E. Toxoplasma gondii
37
38
• 39
• 40
• 41
• 42
. 43
• 44
• 45
• 46
• 47
. 48
. 49
• 50 •

8
L.odt
s
Su~pl'nd
~
End Block
Item: 39 of ~ ,• Mark <::J [:::> ""I ~· ~'j
100 J.. Previous Next LAb faiUI~S Notes Calculator

24
25 The correct a nswer is B. 8 1% chose t his .
26 Epstein-Barr virus ( EBV) causes infect ious mononucleosis and is a member of t he Herpesviri dae fam ily. Mononucleosis typically presents with
27 high feve r~ an elevated WBC count , painful pharyngit is, and enlarged lymph nodes. There are some clinical differences bet ween EBV
mononucleosis and t hat due t o cytomegalovirus (CMV) : exudat ive pharyngit is and cervical lymphadenopathy are commonly seen wit h EBV,
28
but not wit h CMV. The maj ori ty of cases of EBV mononucleosis occur in teenagers and young adults, whereas CMV-induced disease occurs
29 most commonly in adults aged 30- 60 years. A posit ive het erophile antibody test (in which het erophile ant ibodies react t o ant igens from
30 phylogenetically unrelat ed species, causing agglutinat ion) is specific for EBV infection.
Infectious mononucleosis Epstein-Barr virus Herpesviridae Cytomegalovirus Heterophile antibody test Pharyngitis Lymphadenopathy Heterophile Virus Antigen Exudate Antibody Lymph node Fever Cervix
31
Infection Cervical lymphadenopathy
32
33 A is not correct . 9°/o chose this.
34 Cytomegalovirus (CMV) causes a mononucleosis syndrome t hat is very similar to t he one caused by EBV; however, one rarely sees an
exudat ive pharyngit is or cervical lympha denopat hy wit h CMV. CMV can also cause pneumonia, and if t ransmitted congenitally, birth defects.
35
After infection wit h CMV, most healt hy individuals are asymptomatic, but CMV can reactivat e in t he immunocompromised and cause a variety
36 of illnesses in various organ syst ems.
37 Cytomegalovirus Infectious mononucleosis Pharyngitis Lymphadenopathy Immunodeficiency Pneumonia Asymptomatic Exudate Epstein-Barr virus Cervical lymphadenopathy Congenital disorder

38 Human cytomegalovirus

39 C is not co rrect. 2 °/o chose this .


• 40 An acute HIV prodrome would manifest in a similar fashio n t o EBV mononucleosis and can result in either an elevated or a depressed WBC
• 41 count. I t would not , h oweve r~ cause a posit ive het erophile test.
Infectious mononucleosis Heterophile antibody test Heterophile Prodrome HIV Epstein-Barr virus
• 42
. 43 D is not correct. 6°/o chose t his .
• 44 Streptococcus pneumoniae is a gram- posit ive diplococcus and the most common cause of lobar pneumonia worl dwide. It is unlikely t o be
causing t he const ellation of symptoms seen in our pat ient, which are more indicat ive of an infectious mononucleosis. 5. pneumoniae infect ion
• 45
would not resu lt in a posit ive heterophile antibody t est .
• 46 Infectious mononucleosis Streptococcus pneumoniae Heterophile antibody test Pneumonia Lobar pneumonia Streptococcus Gram-positive bacteria Antibody Heterophile

• 47
E is not correct. 2°/o chose this .
• 48
Toxoplasma gondii is the prot ozoa t hat causes t oxoplasmosis, and it is t ransm it ted by ingest ion of undercooked meat or food contam inated by
. 49 cat feces. I mmunocompet ent pat ients are usually asymptomatic. Immunocompromised pat ient s, however, may present wit h foca l neurologic
• 50 deficits due to a brain lesion (often ring -enhancing) . I nfection wit h Toxoplasma does not ty pically present as a mononucleosis syndrome and

8
Lock
s
Suspend
0
End Block
Item: 39 of ~ ,• Mark <::J [:::> ""I ~· ~'j
100 J.. Previous Next LAb faiUI~S Notes Calculator

24
25 E is not correct. 2°/o chose this .
26
Toxoplasma gondii is the prot ozoa t hat causes t oxoplasmosis, and it is t ra nsmit ted by ingest ion of undercooked meat or food contam inated by
cat feces. I mmunocompet ent pat ients are usually asymptomatic. I mmunocompromised pat ient s, however, may present wit h foca l neurologic
27 deficits due to a brain lesion (often ri ng-enhancing) . I nfection wit h Toxoplasma does not ty pically present as a mononucleosis syndrome and
28 would not resu lt in a posit ive heterophile antibody t est .
Toxoplasma gondii Protozoa Toxoplasmosis Infectious mononucleosis Immunodeficiency Heterophile Heterophile antibody test Antibody Feces Asymptomatic Immunocompetence Lesion Cat Brain
29
30
31 Bottom Line :
32 Reactive lymphocytosis is most commonly caused by EBV, alt hough it can also be caused by CMV, toxoplasmosis, and acute HIV. Het erophile
33 IgM ant ibodies are diagnost ic of EBV. I n a peripheral blood smea r~ at ypical lymphocytes are seen, having abundan t cytoplasm and azurophilic
34
granules.
Toxoplasmosis Cytoplasm Lymphocytosis Immunoglobulin M Lymphocyte Antibody Blood film Epstein-Barr virus Azurophilic granule Cytomegalovirus
35
36
37
l@ljl'il·1i•J for yea r:[2017 • J
FIRST AID FACTS
38
39
• 40 FA17 p 160.2
• 41 Herpesviruses Enveloped, OS, and linear viruses
• 42
VIRUS ROUTE OF TRANSMISSION CLI NICAL SIGNIFICANCE NOTES
. 43
• 44 Herpes Respiratory Gingivostomatitis, keratoconjunctivitis fJ, Most common cause of sporadic
• 45 simplex secretions, saliva herpes labialis [l), herpetic whitlow on finger, encephalitis, can present as altered
• 46 virus-1 temporal lobe encephalitis, esophagitis, mental status, seizures, and/or
• 47 erythema mu lti forme. aphasia .
• 48
Herpes Sexual contact, 1-Ierpes genital i s ~, neonatal herpes. Latent in sacral ganglia. Vi ral
. 49
simplex perinatal meningitis more common with
• 50 --!---- - I 1 C"' \ I ~ ol 'o1 I I C"" \ T 1

8
Lock
s
Suspend
0
End Block
Item: 39 of ~ ,• Mark <::J [:::> ""I ~· ~'j
100 J.. Previous Next LAb faiUI~S Notes Calculator

24 • •
FA17 p 160.1
25
DNA viruses
26
VIRAL FAMILY ENVELOPE DNA STRUCTURE MEDICAL IMPORTANCE
27
28
Herpesviruses Yes OS and linear See Herpesviruses entry
29 Poxvirus Yes OS and linear Smallpox eradicated world wide by use of the live-
30 (largest 0 A virus) attenuated vaccine
31 Cowpox ("milkmaid blisters")
32 Molluscum contagiosum -Aesh-colored papule with
33 central umbilication
34
Hepadnavirus Yes Partially OS and circular HBV:
35
• Acute or chron ic hepatitis
36
37
• ot a retrovirus but has re,·erse transcriptase
38 Adenovirus 0 OS and linear Pebrile pharyngitis r.J- sore throat
39 Acute hemorrhagic cystitis
• 40 Pneumonia
• 41 Conju ncti,·itis-"pink eye"
• 42
. 43
• 44
• 45
• 46
• 47
Papillomavirus 0 OS and circular 1--1 PV- warts (serotypes I, 2, 6, ll), Cl , cervica l
. 48
cancer (most commonly 16, 18)
. 49
• 50 • Polyomavirus 0 OS and circular JC virus-progressive multifoca ll eu koc n ceph a l opt~ t h y •

8
Lock
s
Suspend
0
End Block
Item: 39 of - ,• Mark -<] 1:> ""'I ~· 1!';:'1
100 ~ Prev1o u s Next Labf a lu es Notes Calculator

24 • Acute or chronic hepatitis


25 1 ot· a retrovirus but has re,·erse transcriptasc
26
Adenovirus I 0 OS and linear Febrile pharyngitis rJ-sore throat
27
Acute hemorrhagic cystitis
28
29
Pneumon ia
30
Conjuncti\'itis-"pin k ere"
31
32
33
34
35
36 Papillomavirus 0 0 and circular II PV-warts (scrotypes I, 2, 6, II), C l1 , cer\'ical
37 cancer (most commonly 16, 18)
38
Polyomavirus No OS and circular JC vi rus-progressive multifocalleukoenccphalopat hy
39
(P 1L) in HIV
• 40
BK virus-tr;111splant patients, commonly targets kidney
• 41
JC: Junky Cerebrum; BK: Bad Kidney
• 42
. 43 Parvovlrus 0 SS and linear 1319 virus-<lplastic crises in sickle cell disease,
• 44 (smallest 0 A virus) "slapped cheek" rash in children (erythema
• 45 infectiosum, or fifth disease)
• 46 RBC destruction in fetus leads to hydrops fetal is
• 47 and death, in adults leads to pure RBC aplasia and
. 48 rheumatoid arthritis-like symptoms
. 49
• 50 •

8
L.odt
s
Su~pl'nd
~
End Block
Item: 40 of - ,• Mark -<J [:::> "'I ~ · ~
100 ~ P~v1ous N @xt Labl lues N o tes Calcula to r

24 A A

A 47-year-old pet store owner presents to his physician complaining of headache, malaise, and an increasi ng cough over t he past 10 days. ~~AI
25
The cough is nonproduct ive. His temperat ure is 38.3°C (10 1° F). X-ray of the chest revea ls bilateral patchy interstitial infilt rat es in the lung.
26 The patient is prescribed doxycycline for 3 weeks, and his physician reports the infection to t he state public health service. A
27 bronchoalveolar lavage is pe1formed t o confi rm diagnosis.
28
29 What microscopic features are most likely seen on bronchoalveolar lavage?
30 :
31 A. Acid fast-staining bacilli
32 B. Clustered, gram-positive cocci
33
C. Flying saucer-shaped, silver-staining organisms
34
35
D. Intracellular organisms within an inclusion body
36 E. Lancet-shaped, gram-positive diplococci
37
38
39
• 40
• 41
• 42
• 43
• 44
. 45
• 46
• 47
. 48
. 49
. 50

8
Lode.
s
Suspe-nd
8
End Bloc:k
Item: 40 of ~ ,• Mark <:::1 t::> ""I ~· ~'j
100 J.. Previous Next LAb faiUI~S Notes Calculator

24
25
26 The correct a nswer is 0. 55°/o chose t his.
27 This patient has psittacosis, an atypical pneumonia spread by Chlamydophila psittaci. Histology will show an obligat e intracellular
28
organism wit hin membrane-conf ined inclusions (inclusion bodies) . Approximately 80% of patients wit h t his disease have had a recent
exposure t o a bird. Being a pet shop ow n e r~ t his pat ient is undoubtedly exposed t o parrots and other birds. The presentation is one of malaise,
29 a high fever, and progressive nonproduct ive cough. X- ray of the chest usually reveals a diffuse, bilat eral pat chy inf ilt rate. Psitt acosis is a
30 disease t hat is reportable to public health aut horit ies in most states. Treat ment is a 10-day course of doxycycline or tetracycline.
Psittacosis Doxycycline Atypical pneumonia Tetracycline Chlamydophila psittaci Pneumonia Histology Chest radiograph Intracellular Organism X-ray Fever Malaise Bird Cough
31
32 A is not correct . 14% chos e this .
33 Tuberculosis, an acid-fast bacillus that invades pulmonary macrophages, often presents wit h granulomas in the hilar lymph nodes and lower
lobes. These densit ies are called Ghon com plexes. Signs and symptoms are night sweats, shortness of breat h, weight loss, cough, and
34
hemoptysis. Treat ment includes isoniazid, rifam pin, et hambu tol, and pyrazinamide .
35 Hemoptysis Isoniazid Pyrazinamide Ethambutol Acid-fast Rifampicin Tuberculosis Granuloma Lymph node Night sweats Bacillus (shape) Macrophage Weight loss Bacillus Dyspnea Cough Lymph

36
B is not correct. 6% chose t his .
37
Staphylococcus aureus is a gram-posit ive coccus t hat grows in clusters, which t ypically manifests as bronchopneumonia wit h bilat eral
38 infilt rat es on chest x-ray. Howeve r~ primary S aureus infection may occasionally manifest as lobar pneumonia wit h unilateral consolidation . It
39 is most commonly linked t o hospit al-associated pneumonia. Signs and symptoms are fatigue, myalgia, feve r~ and a product ive cough.
40
Treatment with ceftriaxone, penicillin, or a f luoroquinolone is usually successful.
Quinolone Pneumonia Ceftriaxone Penicillin Staphylococcus aureus Myalgia Gram-positive bacteria Lobar pneumonia Chest radiograph Staphylococcus Cough Hospital-acquired infection X-ray Coccus
• 41
Fatigue (medical) Fever Infection
• 42
. 43
C is not co rrect. 19% chos e this .
Pneumocystis jirovecii oft en causes bilateral, widespread infection in the lungs of immunocompromised patients. Previously m isclassified as a
• 44
protozoan, it is a parasit ic f ungus t hat m icroscopically resembles f ried eggs or flying saucers. Patients who are not immunosuppressed are
• 45 usually not suscept ible to t his form of pneumonia. It is successfully treat ed with t ri met hoprim/sulfamet hoxazole .
• 46 Trimethoprim/sulfamethoxazole Pneumocystis jirovecii Immunodeficiency Pneumonia Immunosuppression Lung Parasitism Protozoa Pneumocystis pneumonia Fungus Infection

• 47 E is not correct. 6°/o chos e this .


• 48 Streptococcus pneumoniae, a gram -posit ive, lancet -shaped diplococcus, is t he leading cause of community-acquired lobar pneumonia. This
. 49 pneumonia usually manifests with myalgias, fever, and a cough product ive of brown sputum . Treatment involves a course of penicillin,
cephalosporin, or f luoroquinolone .
• 50

8
Lock
s
Suspend
0
End Block
Item: 40 of ~ ,• Mark <:::1 t::> ""I ~· ~'j
100 J.. Previous Next LAbfaiUI~S Notes Calculator

24
25 Bottom Line:
26 Any pat ient com plaining of feve r and dry cough who has had a recent exposure to birds should be suspected of having psittacosis. Treat ment
entails an ext ended t reat ment wit h either doxycycline or t etracycli ne.
27 Psittacosis Doxycycline Tetracycline Cough Fever
28
29
30
i@l;fil·1i•J f or yea r:[ 2017 • J
FIRST AID FACTS
31
32
33
FA17 p 645.1
34 Pneumonia
35 TYPE TYPICAL ORGANISMS CHARACTERISTICS
36 Lobar S pneumoniae most frequently, also Legionella, Intra-alveolar exudate -+ consolidation r.il; may
37 Klebsiella rn
iJwolve entire lobe or lung.
38
Bronchopneumonia S pneumoniae, S aureus, H infiuenzae, Acute inflammatory infiltrates B1 from
39
Klebsiella bronchioles into ad jacent alveoli; patchy
40
• 41
distribution involving;;:: 1 lobe (!].
• 42 Interstitial (atypical) Mycoplasma, Chlamydophila pneumoniae, Diffuse patchy inAammation localized to
. 43 pneumonia Chlamydia psittaci, Legionella, viruses (RSV, interstitial areas at alveolar walls; diffuse
• 44 C tfV, inAuenza, adenovirus) distribution involving;;:: 1 lobe 1]. Generally
• 45 follows a more indolent course ("walking"
• 46 pneumonia).
• 47
Cryptogenic Formerly known as bronchiolitis obliterans
• 48
. 49
organizing organizing pneumonia (BOOP). Noninfectious
• 50
pneumonia pneumonia characterized by inflammation of
8
Lock
s
Suspend
0
End Block
Item: 40 of ~ ,• Mark <:::1 t::> ""I ~· ~'j
100 J.. Previous Next faiUI~S
LAb Notes Calculator

24
25
26
27
28
29
30
31
32
33
34 FA17 p 144.1
35
Zoonotic bacteria Zoonosis: infectious disease transmitted between animals and humans.
36
37 SPECI ES DISEASE TRANSMISSION AND SOURCE
38 Anaplasma spp. Anaplasmosis Ixodes ticks (live on deer and mice)
39
Bartonella spp. Cat scratch disease, bacilla ry angiomatosis Cat scratch
40
• 41 Borrelia burgdorferi Lyme disease Ixodes ticks (live on deer and mice)
• 42 Borrelia recurrentis Relapsing fever Louse (recurrent due to variable surface
. 43 antigens)
• 44
Brucella spp. Brucellosis/undulant fever Unpasteurized dairy
• 45
• 46 Campy/obacter Bloody diarrhea Feces from infected pets/animals; contaminated
• 47 meats/foods/hands
• 48
Chlamydophila psittaci Psittacosis Parrots, other birds
. 49
• 50
Coxiella burnetii Q fever Aerosols of cattle/sheep amniotic Auid
8
Lock
s
Suspend
0
End Block
Item: 40 of ~ ,• Mark <:::1 t::> ""I ~· ~'j
100 J.. Previous Next LAbfaiUI~S Notes Calculator

24
FA17 p 175.2
25
Common causes of pneumonia
26
NEONATES(< 4WK) CH ILDREN (4WK-18 YR) ADULTS(18- 40YR) ADULTS (40-65 YR) ELDERLY
27
28 Group B streptococci Viruses (RSV) Mycoplasma S pneumoniae S pneumoniae
29 E coli M)1coplasma C pnewnoniae H influenzae 1nAuenza virus
30 C trachomatis S pneumoniae Anaerobes Anaerobes
31 (infants-3 yr) Viruses (eg, influenza) Viruses H influenzae
32 C pnewnoniae Mycoplasma Cram 8 rods
33 (school-aged
34 ch ildren)
35 S pneumoniae
36 Runts May Cough
37
Chunky Sputum
38
Special groups
39
40 Alcoholic Klebsiella, anaerobes usually due to aspiration (eg, Peptostreptococcus, Fusobacterium, Prevotella,
• 41 Bacteroides)
• 42 IV drug users S pneumoniae, S aureus
. 43
Aspiration Anaerobes
• 44
• 45 Atypical Mrcoplasma, Legionella, Chlamydia
• 46 Cystic fibrosis Pseudomonas, S aureus, S pneumoniae, Burkholderia cepacia
• 47
lmmunocompromised S aureus, enteric gram 8 rods, fungi, viruses, P jirovecii (with II IV)
• 48
. 49 Nosocomial (hospital S aureus, Pseudomonas, other enteric gram 8 rods
• 50 acauiredl
8
Lock
s
Suspend
0
End Block
Item: 41 of - ,• Mark -<J
P~v1ous
[:::> "'I ~ ·· ~
100 ~ N @xt Labl lues N o tes Calculato r

24 A A 75-year-old homeless alcoholic man presents t o t he emergency departm ent complaining of shortness of breath, high fever~ shaking
25 chills, and a productive cough. His t emperat ure is 39.6°C (103.2°F), pulse is 110/min, respi ratory rate is 27/min, and blood pressure is
26
130/85 mm Hg. X-ray of the chest shows a lobar consolidation. Gram staining of his sput um cult ure resembles th at shown in t he image.
The pathology lab determines that t he organism is oxi dase- negative.
27
28
29
30
31
32
33

-
34
35
36
37

--
I

-.,
38
39 ...
40
- /-'.
I - ...,
...

• 41 \ - t
Jf ....
I.,..
• 42
·~
• 43
• 44
Image cowtesy of Bob J. Galindo '
. 45
Which of the fol lowing ant ibiot ics would be appropriat e for t his pat ient's illness?
• 46
:
• 47
A. Amphotericin B
. 48
. 49
B. Aztreonam
. 50 C. Clavulanic acid
8
Lode.
s
S uspe-nd
8
End Bloc:k
Item: 41 of - ,• Mark --<) [::> ""'I ~· 1!';:'1
~
-
100 Prev1o u s Next Labf a lu es Notes Calculator

,-
24 •
25
26
27
I
28
29
30
31
32
33
34
-
35
36
37
38
...
39
40 Image coUttesy of Bob J. Galindo

• 41
• 42 Which of t he following ant ibiot ics would be appropriate for t his patient's illness?
. 43 :
• 44 A . Am phot ericin B
• 45 B. Aztreonam
• 46
C. Clavulanic acid
• 47
. 48 D. Isoniazid
. 49 E. Vancomycin
• 50 •
8
L.odt
s
Su~pl'nd
~
End Block
Item: 41 of ~ ,• Mark <::J [:::> ""I ~· ~'j
100 J.. Previous Next LAbfaiUI~S Notes Calculator

24
25 The correct a nswer is B. 48% chose t his .
26 The man suffers from community -acquired pneumonia, as evidenced by high feve r, shaking chills, product ive cough, lobar consolidat ion on X-
ray of the chest, and t he Gram stain f ind ings. Alt hough t he most common cause of community-acquired pneumonia is Streptococcus
27
pneumoniae, the Gram st ain revea ls gram -negative rods. I n cases of pneumonia in malnouri shed, debilitated, or alcoholic pat ients, t he most
28 likely gram -negat ive infect ion is Klebsiella pneumoniae. Gram- negative rods respond well to azt reonam, a monobactam antibiotic.
29 Suscept ibility test ing will det ermine the appropriate ant ibiot ic in different hospit al settings; however, you will need to choose the best answer
based on limited informat ion on board exams. Keep in m ind that ant imicrobial drug resistance is a serious emerging problem and t hat
30
ant ibiot ics like aztreonam should only be ut ilized when the case absolut ely requires it.
31 Gram staining Aztreonam Klebsiella pneumoniae Gram-negative bacteria Streptococcus pneumoniae Community-acquired pneumonia Antibiotics Chest radiograph Pneumonia Monobactam Klebsiella

32 Streptococcus Pulmonary consolidation Antimicrobial Cough X-ray Chills Drug resistance Malnutrition Fever Alcoholism

33
A is not correct . 7°/o chose this.
34
Amphot eri cin B is an ant ifungal agent, with no activity against t he gram-negat ive rods that are infecting t his pat ient. Amphot ericin B has a
35 nasty side effect re pertoire, including flushi ng, diarrhea, dizziness, nausea, vomiting, shortness of breath, and muscle and joint aches. It is
36 generally given only fo r progressive and invasive fungal diseases.
Amphotericin B Gram-negative bacteria Antifungal Diarrhea Nausea Adverse effect Dyspnea Vomiting Fungicide Fungus Dizziness Side effect
37
38 C is not co rrect. 13% chose this .
39 Clavulanic acid is a 13- lactamase inhibitor used to prevent degradat ion of 13- lact am ant ibiot ics when t reat ing infections wit h bacteria that
produce 13-lactamases. It has no ant ibiotic activity as a monot herapy.
40 Clavulanic acid Antibiotics Bacteria
41
D is not correct. 11% c hose this .
• 42
Isoniazid is an antit uberculosis drug and has m inimal act ivity against gram- negat ive organisms. Mycobacterium tuberculosis causes
. 43 t uberculosis, which does not commonly manifest with high feve rs and shaking chills of such acut e onset. Additionally, M. t uberculosis does not
• 44 stain well wit h Gram st ain, and t hus, a special acid-fast stain is needed to visualize t he bact erium .
Gram staining Isoniazid Mycobacterium tuberculosis Gram-negative bacteria Tuberculosis Acid-fast Mycobacterium Chills Tuberculosis management Bacteria
• 45
• 46 E is not correct. 2 1 °/o chose this .
• 47 Vancomycin has activity against gram -posit ive bact eria only. This patient is infect ed wit h a gram -negative organism. Vancomycin can be used
• 48
for t reatment of Clostridium difficile infections that develop after a course of an t ibiotics. It can also be used in t reatment of bact eremia,
sepsis, and other acquired infect ions of gram -posit ive bacteria .
. 49 vancomycin Bacteremia Gram-positive bacteria Gram-negative bacteria Sepsis Clostridium difficile colitis Bacteria Clostridium difficile (bacteria) Antibiotics Clostridium Organism
• 50

8
Lock
s
Suspend
0
End Block
Item: 41 of ~ ,• Mark <::J [:::> ""I ~· ~'j
100 J.. Previous Next LAb faiUI~S Notes Calculator

24
25
Bottom Line:
26 Acute bacterial pneumonia caused by Klebsiella pneumoniae, a gram -negative rod, ofte n affects t he malnouri shed and debilitated, such as
chronic alcoholics. Azt reonam would be an appropriat e t reatment.
27 Aztreonam Klebsiella pneumoniae Pneumonia Gram- negative bacteria Bacterial pneumonia Klebsiella Alcoholism Malnutrition
28
29
30 l@ljl'il·1i•J for year:[ 2o17 • ]
FIRST AID FA CTS . .
31
32
FA1 7 p 140.2
33
34 Klebsiella Gram 8 rod; intestinal flora that causes lobar 5 Ns of KlebsiellA:

..... .-·..,..... pneumonia in alcoholics and diabetics when Aspiration pneumonia


35
36 A··...
. ... . ·.., /llil-' aspirated. Very mucoid colonies fJ caused by Abscess in lungs and liver
37 abundant polysaccharide capsu les. Dark red Alcoholics
38 ·-· '~ "currant jelly" sputum (blood/mucus). Di-A-betics
39
40
•• •• Also cause of nosocomial UTis. "Curr-A-nt jelly" sputum

41
• 42
• • • fl#. *
. 43
• 44
• 45
FA1 7 p 132.2
• 46 Streptococcus Gram E£), lancet-shaped diplococci fl. Pneumococcus is associated with "rusty"
• 47 pneumoniae Encapsulated. IgA protease. Optoch in sputum, sepsis in patients with sickle cell
• 48
fJ ,, sensitive. Most common cause of: disease, and asplenic patients.
. 49
• 50 , .....
• Meningitis
,........ • . . . 1. , . l •1 1
o virulence without capsule.

8
Lock
s
Suspend
0
End Block
Item: 41 of - ,• Mark -<J
P~v1ous
[:::> "'I ~ ·· ~
100 ~ N @xt Labl lues N o tes Calculato r

24 A A

25
FA17 p 132.2
26
27 Streptococcus Gram E!), lancet-shaped diplococci · . Pneumococcus is associated with "rusty"
28 pneumoniae Encapsulated. lgA protease. O ptochin sputum, sepsis in patients with sickle cell
29 sensitive. Most common cause of: disease, and asplenic patients.
rJ .,
30 \ leni ngitis 1 o virulence without capsule.
,
31 • Otitis media (in children)
32 Bacterial pneumonia
33 • Sinusitis
34
.,..
35
36
37
• . .
.,. '- ......"""ll
38
FA17 p 124.2
39
40 Encapsulated bacteria Examples are Pseudomonas aeruginosa, Please S III~E my SKiS.
41 Streptococcus pneumoniae fJ, Ilaemophilus Arc opsoni zed, and then cleared by spleen.
• 42 I Influenzae type B, Neisseria meningitic/is, Asplcnics have l opsonizing abi lity and thus
• 43 Escherichia coli, Salmonella, Klebsiella t risk for severe infections. Give S pnewnoniae,
• 44 pneumoniae, and group B Strcp. Their 11 influenzae, meningitidis vaccines.
. 45 capsules serve as an antiphagocytic virulence
• 46 factor.
• 47 Capsular polysaccharide+ protein con jugate
. 48
sen ·es as an antigen in vaccines.
. 49
. 50

8
Lode.
s
S uspe-nd
8
End Bloc:k
Item: 42 of - ,• Mark -<J
P~v1ous
[:::> "'I ~ ·· ~
100 ~ N @xt Labl lues N o tes Calculato r

24
A 17-year-old boy complains of fever and painfu l, swollen cheeks. He has recently emigrated from Mexico, and one of his relat ives had a
25
similar illness a few weeks ago. He finds it hard t o t alk, eat , and swallow. He denies headache or neck pai n.
26
27 Which of the following findings might also be present on examination?
28
:
29 A. Hepatomegaly
30
B. Orchitis
31
32 C. Papilledema
33 0. Peripheral edema
34
E. Splenomegaly
35
36
37
38
39
40
41
• 42
• 43
• 44
. 45
• 46
• 47
. 48
. 49
. 50

8
Lode.
s
S uspe-nd
8
End Bloc:k
Item: 42 of ~ ,• Mark <::J [:::> ""I ~· ~'j
100 J.. Previous Next faiUI~S
LAb Notes Calculator

24
The correct a nswer is B. 7 5% chose t his .
25
Mumps is an infectious disease that can cause swollen cheeks. Although not often seen in t he Un it ed St at es because of the
26 measles/mumps/rubella vaccine, mumps occasionally manifests in t hose who have not been vaccinated. Mumps is caused by an RNA
27 paramyxovirus that replicates in t he upper respirato ry t ract and causes parotidit is and, f requent ly, orch it is. Pancreat itis and mening it is also
28
can be present. Respirat ory droplets and direct cont act are its means of t ransmiss ion . Treatment is usua lly supported wit h pain medication.
Param(xoviridae Orchitis Mumps Pancreatitis Meningitis Infection vaccine Respiratory tract Analgesic RNA Pharmaceutical drug
29
30 A is not correct . 7°/o chose this.
Some infectious causes of hepatomegaly include infectious mononucleosis, acute v iral hepatit is, and cytomegalovirus. Mumps is not
31
commonly associated wit h hepat omegaly.
32 Infectious mononucleosis Cytomegalovirus Hepatomegaly Mumps Viral hepatitis Hepatitis Virus

33
C is not correct. 5 °/o chose this .
34
Papilledema often is a marker of increased int racrania l pressure and can be seen in encephalit is and meningit is. Alt hough m umps can manifest
35 as meningit is, given t his pat ient 's denial of headache or neck pain, it is unlikely t hat mum ps would be t he cause of increased int racranial
36 pressure t hat would result in papilledema .
Papilledema Intracranial pressure Meningitis Mumps Encephalitis Headache Neck pain
37
38 D is not correct. 5°/o chose t his.
39 Peri pheral edema has numerous causes. I nfectious causes include filariasis and cellulitis. Mumps is not known t o be associated wit h peri pheral
edema.
40
Edema Mumps Cellulitis Filariasis Peripheral edema
41
E is not correct. 8°/o chose this .
42
I nfectious causes of splenomegaly include infect ious mononucleosis, acute viral hepatit is, and cyt omegalovirus .
. 43
Infectious mononucleosis Cytomegalovirus Splenomegaly Viral hepatitis Hepatitis Virus
• 44
• 45
• 46
Bottom Line :
• 47 Clinical manifest at ions of m umps include parot idit is, orch it is, and meningit is. Because of rout ine measles/mumps/rubella vaccination, mumps
is re lat ively uncommon in t he United States .
• 48 Orchitis Mumps Meningitis vaccination United States
. 49
• 50

8
Lock
s
Suspend
0
End Block
Item: 42 of ~ ,• Mark <::J [:::> ""I ~· ~'j
100 J.. Previous Next LAb faiUI~S Notes Calculator

24 • •
FA17 p 166.3
25
26 Mumps virus A paramyxovirus that causes mumps, Mumps makes your parotid glands and testes as
27 uncommon due to effectiveness of MMR big as POl\1-Poms.
28 vaccme.
29 Symptoms: Parotitis r:J. O rchitis (inflammation
30 of testes), aseptic M eningitis, and Pancreatitis.
31 Can cause sterility (especially after puberty).
32
33
34
35
36
FA17 p 163.1
37
RNA viruses
38
VIRAL FAMILY ENVELOPE RNA STRUCTURE CAPSID SYMMETRY MEDICAL IMPORTANCE
39
40 Reoviruses 0 OS linear Icosahed ral Coltivirus" - Colorado tick fever
41 10- 12 segments (double) Rotavirus-cause of fata l diarrhea in children
42 Picornaviruses No SS ® linear Icosahed ral Poliovirus-polio-Salk/Sabin vaccines-IPV/OPV
. 43 Echovirus- aseptic men ingitis
• 44 Rhi novirus-"common cold"
• 45 Coxsackievirus-aseptic mening itis; herpang ina
• 46 (mouth blisters, fever); hand, foot, and mouth
• 47 disease; myoca rditis; pericard it is
. 48 IIAV-acute viral hepatitis
. 49 P E RC ! I
• 50 • •

8
Lock
s
Suspend
0
End Block
Item: 42 of - ,• Mark -<J
P~v1ous
[:::> "'I ~ ·· ~
100 ~ N @xt Labl lues N o tes Calculato r

24 A A

25
Hepevirus 0 SS ® linear lcosa heel ra I HEV
26 Caliciviruses 0 SS ® linear Icosahedral 1'\orovirus- viral gastroenteritis
27
Flaviviruses Yes SS ® linear Icosahedral IICV
28
Yellow fe,·er1
29
Denguea
30
St. Louis encephalitis3
31
\ est lile virus3 (meningoencephalitis)
32
Zika virus
33
34 Togaviruses Yes SS ® linear Icosahedral Rubella
35 Western and Eastern equine encephalitis3
36 Chikungunya \'irus
37 Retroviruses Yes SS ® linear Icosahedral H<l\'C re,·erse transcriptase
38
2 copies (HTLV), I IT LV-T-cellleukemia
39 complex HI V- AIDS
40 and conical
41 (HlV)
42
• 43
Coronaviruses Yes SS ® linear Helical "Common cold," SARS, MERS
• 44 Orthomyxoviruses Yes ss e linear ll elical Innucma virus
. 45 8 segments
• 46
• 47
Paramyxoviruses Yes ss e linear llelical Pa Ra ~h·xovirus:
'
1 onsegmented ParainAuenza- croup
. 48
RSV- bronchiolitis in babies; Rx-ribavirin
. 49
\leasles, .\lumps
. 50

8
Lode.
s
S uspe-nd
8
End Bloc:k
Item: 43 of - ,• Mark -<J [:::> "'I ~ · ~
100 ~ P~v1ous N @xt Labl lues N o tes Calculato r

24
An 19-year-old college freshman comes t o t he physician with a 12-hour history of a t emperature of 39°C ( 102 .2° F). He also complains of
25
headache and neck stiffness. Physical exam reveals nuchal ri gidity. A smear of his cerebrospinal fl uid revea ls gram-negat ive diplococci,
26 cultures grow on Thayer-Martin media. Medical history is significant for absence of vaccinations as a child .
27
28 What symptoms can develop as a severe complication of t his infection?
29 :
30 A. Fever, new murmur, small erythematous lesions on the palms, and splinter hemorrhages on the nail bed
31
B. Migratory polyarthritis, carditis, erythema marginatum
32
C. Refractory hypotension, widespread purpura
33
34 0 . Simultaneous ataxia, urinary incontinence, and cognitive decline
35
E. Symmetric ascending muscle weakness beginning in the distal lower extremities
36
37
38
39
40
41
42
• 43
• 44
. 45
• 46
• 47
. 48
. 49
. 50

8
Lode.
s
S uspe-nd
8
End Bloc:k
Item: 43 of - ,• Mark -<J [:::> "'I ~ · ~
100 ~ P~v1ous N @xt Labl lues N o tes Calculato r

24 A A
The correct answ er is C. 66°/o chose this.
25
The causative agent in t his scenari o, based on t he Gram stain, is Neisseria meningitidis. Dissem inat ed
26 infection by this bacterium can result in hypotension, t achycardia, a rap idly enlarging petechial or purpuric
27 rash (like that shown in th e image), disseminated intravascu lar coagulation, and coma. Waterhouse-
28
Friderichsen syndrome is a possible complication of meningococcemia. In this disorder, bilat era l hemorrhage
into the adrenal glands causes acute adrenal insufficiency.
29 Wi"'ter.,ou~~F· de c ~e-'1 5Yl drome Gram sta1n1ng Disseminated intravascular coagulatlon Netssena merung1bdts Mer11ngococcem~a Adrenal 1nsufficiency

30 Hypotenston Merungococcal d1sease Tachycard•a Purpura Petechia Bleeding Adrenal gland Coagulation Nessena Rash Coma 8actena Infection

31 Blood vessel
32
33
34 I mage copyright © J Global
I nfect Dis
35
36 A is not correct . 8°/o chose thi s.
37 Fever, a new murmur, Janeway lesions, and nail-bed hemorrhages are all signs of bacterial endocarditis. Acute endocarditis is caused by
38 Staphylococcus aureus and subacute infection can be caused by Streptococcus viridans .
Vindan -streptococci Janeway leston Endocard1bs Staphylococcus aureus Infect1ve endocardttis Streptococcu! Staphylococcus Bleedmg Heart murmur Fever
39
40 B is no t correct. 10°/o chose thi s.
41 Rh eumatic fe ver is characterized by fever, migrat ory polyarth rit is, and carditis. It may fo llow group A streptococca l pharyng it is.
Streptococcal pharyng1t1s Rheumatic fever Pharyngitis Polyarthritis Carditis Streptococcus Fever Rheumatism
42
43 D is not correct . 12°/o chose this.
• 44 I n normal pressu re hydrocephalus (a form of communicating or nonobstructive hydrocephalus), t here is increased subarachnoid space
volume, but no increase in CSF pressure. This leads to increased volume of t he ventricles, which can distort t he f ibers of t he corona rad iata
. 45
and lead to t he classic clinical t riad of ataxia, uri nary incont inence and cogniti ve dysfunct ion. This constellation of symptoms best describes
• 46 normal pressure hydrocephalus, and not t he likely sequelae of Neisseria meningitidis infect ion .
• 47 Normal pr~sure hydrocephalus Subarachnoid space Hydrocephalus Urinary incontinence Ne1s~eria men1ng1t1d1~ Atax1a Ventncular system Infection Sequela

. 48 E is not correct . 4 °/o chose thi s .


. 49 Guillain-Barre syndrome is characterized by ra pidly progressing ascending paralysis. It is t hought t o follow a variety of infectious diseases,
. 50 such as cytomegalovirus, Epst ein-Barr viru s, HIV, and gastroenteritis caused by Campylobacter jejuni.

8
Lode.
s
S uspe-nd
8
End Bloc:k
Item: 4 3 of ~ ,• Mark <::J [:::> ""I ~· ~'j
faiUI~S
100
24
J.. Previous Next LAb Notes Calculator
-
Infect Dis
.. ~ ... -.

25
26 A is not correct. 8°/o chose this.
27 Fever, a new murmur, Janeway lesions, and nail-bed hemorrhages are all signs of bacterial endocarditis. Acut e endocardit is is caused by
28
Staphylococcus aureus and subacute infection can be caused by Streptococcus viridans .
Viridans streptococci Janeway lesion Endocarditis Staphylococcus aureus Infective endocarditis Streptococcus Staphylococcus Bleeding Heart murmur Fever
29
30 B is not correct. 10% chose this .
Rh eumatic feve r is characterized by fever, migrat ory polyarthrit is, and cardit is. It may fo llow group A streptococcal pharyngit is.
31
Streptococcal pharyngitis Rheumatic fever Pharyngitis Polyarthritis Carditis Streptococcus Fever Rheumatism
32
33
D is not correct. 12% chose this .
I n normal pressure hydrocephalus (a fo rm of communicating or nonobst ruct ive hydrocephalus), there is increased subarachnoid space
34
volume, but no increase in CSF pressure. This leads to increased volume of t he ventricles, which can dist ort t he f ibers of t he corona radiata
35 and lead t o t he classic clinical t riad of ataxia, urinary incont inence and cognit ive dysf unct ion. This const ellation of symptoms best descri bes
36 normal pressure hydrocephalus, and not the likely sequelae of Neisseria meningitidis infect ion .
Normal pressure hydrocephalus Subarachnoid space Hydrocephalus Urinary incontinence Neisseria meningitidis Ataxia Ventricular system Infection Sequela
37
38 E is not correct. 4 °/o chos e this.
39 Guillain-Barre syndrome is characterized by rapidly prog ressing ascending paralysis. It is t hought to fo llow a variet y of infectious diseases,
such as cytomegalovirus, Epstein-Barr virus, HIV, and gast roent erit is caused by Campylobacter jejuni.
40
Campylobacter jejuni Epstein-Barr virus Cytomegalovirus Gastroenteritis Campylobacter HIV Paralysis Virus Infection
41
42
43 Bottom Line:
• 44 Neisseria meningitidis is a gra m -negat ive diplococcus t hat is a major cause of meningit is and sepsis. It can resu lt in disseminat ed
intravascular coagulation, shock, and adrenal fa ilure (Waterhouse-Friderichsen syndrome) .
• 45
Waterhouse-Friderichsen syndrome Disseminated intravascular coagulation Neisseria meningitidis Gram-negative bacteria Meningitis Sepsis Diplococcus Neisseria Adrenal insufficiency Coagulation Blood vessel
• 46
• 47
• 48 l@ljl'il·1i•J for year:[2017 • J
FIRS T AID FACTS
. 49
• 50

8
Lock
s
Suspend
0
End Block
Item: 44 of - ,• Mark -<] 1:> ""'I ~· 1!';:'1
100 ~ Prev1o u s Next Labf a lu es Notes Calculator

24 •
25
A 45-year-old homeless man is brought to the emergency room by ambulance after being found int oxicat ed in a nearby park. Physical r~AJ
examination reveals poor dentition and several punctate areas on the buccal mucosa that are leaking a thick, yellow exudate. The patient
26 also has a hard, tender, redd ish lump on his mandible.
27
28 Which of the following is the most likely causat ive organism?
29 :
30 A. Actinomyces israe/ii
31
B. Nocardia asteroides
32
C. Pseudomonas aeruginosa
33
34 0. Serratia marcescens
35 E. Staphylococcus aureus
36
37
38
39
40
41
42
43
• 44
• 45
• 46
• 47
. 48
. 49
• 50 •

8
L.odt
s
Su~pl'nd
~
End Block
Item: 44 of ~ ,• Mark <:::1 t::> ""I ~· ~'j
100 J.. Previous Next LAbfaiUI~S Notes Calculator

24
25
26 The correct ans wer is A. 76°/o chose this.
27
The sinus t racts on the buccal m ucosa, the yellow exudate, and t he mandibular lesion suggest t hat t his pat ient has an oral abscess caused by
t he gram-posit ive organ ism Actinomyces israelii. A. israelii is an anaerobic bacillus that forms long branching filaments t hat resemble fung i
28 but are much thinner by compa rison. This organ ism is part of the normal f lora of t he mout h, colon, and vagina and t ends to cause infection in
29 patients with dent al carri es, extractions, or ging iv it is/ging ival t rauma. Men wit h poor oral hygiene are at highest risk. The charact eri st ic
30 feat ure of t his organism is t he yellow clumps, known as sulfur granules. Treatment of A. israelii infect ion is int ravenous penicillin for 2-6
weeks, followed by oral therapy wit h penicillin or amoxicillin for 6- 12 months.
31 Amoxicillin Penicillin Fungus Actinomyces Oral hygiene Gram-positive bacteria Oral mucosa Abscess Anaerobic organism Exudate vagina Actinomyces israelii Sulfur Mucous membrane Lesion
32 Human microbiota Bacillus Intravenous therapy Infection Bacillus (shape)
33
B is not correct. 6% chose this.
34
Nocardia asteroides is a gram- posit ive, weakly acid-fast organism t hat grows in a f ilament ous patt ern similar to some fungi. It is often
35 confused wit h Actinomyces. However, N. asteroides mainly causes pulmonary infection in immunocompromised patients.
36 Nocardia asteroides Nocardia Gram-positive bacteria Actinomyces Fungus Immunodeficiency Acid-fast Organism Hypha Infection

37
C is not correct. 5 °/o chose this .
38 Pseudomonas aeruginosa is an oxidase- posit ive, gram-negat ive bacillus t hat causes skin infection in burn vict ims and pneumon ia in t hose wit h
39 cystic fibrosis. This bacteri um produces a blue-green pigment.
Cystic fibrosis Pseudomonas aeruginosa Gram-negative bacteria Pneumonia Pseudomonas Bacteria Bacillus (shape) Bacillus Pigment Skin infection Fibrosis
40
41 D is not correct. 4°/o chose this .
42 Serratia marcescens is a member of the Enterobacter fam ily, wh ich is not able for t he production of a bright red pigment. It is a common
cause of urinary t ract infections, wound infections, or pneumonia. While it could infect t he type of wound descri bed in this quest ion, it would
43
not produce yellow granu les.
44 Pneumonia Urinary tract infection Enterobacter Serratia Urinary system Pigment
• 45
E is not correct. 10°/o chos e this .
• 46
Staphylococcus aureus is a catalase-posit ive, coagulase- posit ive, gram-posit ive coccus that is the most common cause of skin and soft t issue
• 47 infections. Alt hough these bact eria produce gold-colored colonies when cultured, t hey do not fo rm the characteristic sulfur granules of
• 48 act inomycot ic infections.
Staphylococcus aureus Gram-positive bacteria Staphylococcus Coccus Bacteria Sulfur Soft tissue
. 49
• 50

8
Lock
s
Suspend
0
End Block
Item: 44 of - ,• Mark -<] 1:> ""'I ~· 1!';:'1
100 ~ Prev1o u s Next Labf a lu es Notes Calculator

24
Bottom Line:
25
26
Branching rods on culture from a patient wit h an ora l infection are likely Actinomyces israelii.
Actinomyces 1sraelu Act1nomyc6
27
28
29
Ulit'il;1l•J for year: 2017 •
30 FI~ST AIO FACTS

31
32 FA17 p 135.3
33 Nocardiavs Both are gram$ and form long, branching filaments resembling fungi.
34
Actinomyces Nocardia Actinomyces
35
36 Aerobe Anaerobe
37
Acid fast (weak) rJ l ot acid fast [l]
38
39
Found in soil lormal oral, rcproducti,·e, and C l Aora
40 Causes pulmonary infections in Causes oral/facial abscesses that drain through
41 immunocompromised (can mimic TB but sinus tracts; often associated with dental caries/
42 with 8 PPO); cutaneous infections after extraction; forms yellow "sulfur granules"; ca n
43 trauma in immunocompetent; can spread to also cause PID with lUDs
44 CNS
• 45
• 46
Treat with sulfonamidcs (TMP-SMX) Treat with penicillin
• 47 Treatment is a S"'iAP: Sulfonamidcs-Nocardia; \ eli nomyces- Pen ici IIi n
. 48
. 49
• 50 •
8
L.odt
s
Su~pl'nd
~
End Block
Item: 44 of ~ ,• Mark <:::1 t::> ""I ~· ~'j
100 J.. Previous Next faiUI~S
LAb Notes Calculator

24 • •
FA17 p 131 .3
25
26 Staphylococcus aureus Gram EE>, ~-hemolytic, catalase EE>, coagulase TSST-1 is a superantigen that binds to MHC
27 (E) cocci in clusters fJ. Protein A (virulence JJ and T-ccl l receptor, resulting in polyclonal
28 • factor) binds Fc-lgG, inhibiting complement T-cell activation.
29 activation and phagocytosis. Common ly Staphylococca l toxic shock syndrome
30 colonizes the nares, axilla, and groin. (TSS) presents as fever, vomiting, rash,
31 Causes: desquamation, shock, end-orga n failure. TSS
32 .' Inflammatory disease- skin infections, results in t AST, t ALT, t bilirubin. Associated
Jli~ organ abscesses, pneumonia (often after
33 with prolonged use of vaginal tampons or nasal
34 influenza virus in fection), endocarditis, packing.
35
septic arth ritis, and osteomyelitis. Compa re with Streptococcus pyogenes TSS (a
36
• Toxin-mediated disease-toxic shock toxic shock-like syndrome associated with
37
syndrome (TSS1 ~ 1 ), scalded skin syndrome painfu l skin infection).
38
(exfoliative toxin), rapid-onset food S aureus food poisoning due to ingestion of
39
poisoning (enterotoxins). preformed toxin - short incubation period
40
• MRSA (methicillin-resistantS aureus) (2-6 hr) followed by nonbloody diarrhea
41
42
infection-important cause of serious and emesis. Enterotoxin is heat stable - not
43
nosocomial and community-acquired destroyed by cooking.
44 infections; resistant to methicillin and Bad staph (aureus) ma ke coagulase and toxins.
• 45 nafcill in because of altered penici llin- Forms fibrin clot around self - abscess .
• 46 binding protein .
• 47
. 48 FA17 p 183.2
. 49 Penicillin G, V Penicillin G (IV and IM form), penicillin V (oral). Prototype ~-lactam antibiotics.
• 50 • •

8
Lock
s
Suspend
0
End Block
Item: 44 of - ,• Mark -<] 1:> ""'I ~· 1!';:'1
100 ~ Prev1o u s Next Labf a lu es Notes Calculator

24
septic arthritis, and osteomyelilis. Compa re with Streptococcus pyogenes TSS (a
25
• Toxin-mediated disease-toxic shock toxic shock-like syndrome associated with
26
syndrome (TSST-1), scalded ski n syndrome painful skin infection).
27
28
(exfoliati,·e toxjn), rapid-onset food S aureus food poisoning due to ingestion of
29
poisoning (enterotoxins). preformed toxin - short incubation period
30
:\IRSA {methiciJiin-resistant S aureus) (2-6 hr) followed by nonbloody diarrhea
31 infection- important cause of seriou~ and emesis. Enterotoxin is heat stable - not
32 nosocomial and community-acquired destroyed by cooking.
33 infections; resistant to methicillin <md Bad staph (aureus) make coagulase and toxins.
34 nafcillin because of altered penicillin- Forms fi brin clot around self - abscess.
35 binding protein.
36
37
FA17 p 183.2
38
Penicillin G, V Penicill in C (I and I~ I form), penicillin (ora 1). Prototype ~-lactam antibiotics.
39
40 MECHANISM 0 -Ala-0 -Aia structural analog. Bind penicillin-binding proteins (transpeptidases).
41 Block transpeptidasc cross-linking of peptidoglyca n in cell wall.
42 Activate autolytic enzymes.
43
44
CLI NICAL USE s
Mostly used for gram <t> orgcmiSI11S (S p11€liii/OI1iae, pyogenes, Actinomyces). Also used for gra m e
cocci (mainly r meningitidis) and spirochetes (namely T pallidum). Bactericidal for gram <t> cocci,
• 45
gram <t> rods, gram 8 cocci, and spirochetes. Penicill inase sensitive.
• 46
• 47 ADVERSE EFFECTS Hypersensitivity reactions, direct Coombs <t> hemolytic anemia.
. 48 RESISTANCE Penicillinase in bacteria (a I ) pe of ~-l ac t amase) cleaves ~-la c t a m ring.
. 49
• 50 •

8
L.odt
s
Su~pl'nd
~
End Block
Item: 4S of - ,• Mark -<J [:::> "'I ~ · ~
100 ~ P~v1ous N @xt Labl lues N o tes Calculato r

24
25
A

A 35-year-old woman comes t o th e emergency department ( ED) with shortness of breat h and producti ve cou gh. Her vit als are temperature ,..
102.3° F, heart rate 105/min, blood pressure 110/60 mm Hg, and resp iratory rate of 28/min. Chest X-ray shows a left lower lobe A
AI A

26 consolidation, consistent wit h a lower lobe pneumonia. She is discharged home and placed on broad-spectrum ant ibiotics on an outpatient
27 basis. After completing her ant ibiot ic t reatment she comes back to the ED because she is experiencing a white vaginal discharge associated with
a "fishy odor." A wet mount shows vaginal squamous cells coated with coccobacilli. Vaginal pH is 5.0.
28
29
Which of the following most likely led to the pat ient's current symptoms?
30
:
31
A. Acquisition of protozoan infection
32
33 B. Loss of dimorphic budding yeast
34 C. Loss of gram-positive cocci in clusters
35
D. Loss of gram-positive facult ative anaerobes
36
37 E. Loss of obligate anaerobes
38
39
40
41
42
43
44
• 45

' 46
' 47
. 48
. 49
. 50

8
Lode.
s
S uspe-nd
8
End Bloc:k
Item: 4S of ~ ,• Mark <::J [:::> ""I ~· ~'j
100 J.. Previous Next LAbfaiUI~S Notes Calculator

24
25
26 The correct a nswer is 0. 55°/o chose t his.
27 Lact obacilli make up a maj orit y of t he normal f lora of the vagina. The com posit ion of the normal f lora vari es f rom premenarchal, childbearing,
and menopausal stages. Under t he influence of est rogen, lactobacilli make up the maj orit y of the norma l f lora of the vagina during
28
childbeari ng years. A key feature of the normal vaginal env ironment during t he childbearing years is a low pH (3.5- 4 .2), which inhibits growth
29 of ot her, possibly pat hogen ic, organisms. This pH is likely maintained by t he lactobacilli, and when numbers of these bact eri a are red uced
30 duri ng the course of ant ibiot ic t reatment (wit h, for example, amoxicillin), t he vaginal pH may increase re lat ively (4.0 - 4 .5), making condit ions
favorable for Gardnere/la vagina/is to grow. Features of a G. vagina/is infection (bacterial vaginosis) are as descri bed, diagnosis is made by a
31
wet mount microscopy which shows >20% clue cells.
32 Gardnerella vaginalis Amoxicillin Bacterial vaginosis Estrogen Lactobacillus vagina Antibiotics PH Human microbiota Bacteria Clue cell Microscopy Menopause Pathogen Microscope slide Infection

33
A is not correct . 7°/o chose this.
34 Trichomonas vagina/is is a pat hogenic protozoan and is never considered normal flora. It is responsible for causing symptoms of frot hy, foul-
35 smelling, t hin discharge. Add it ional symptoms include itching and burn ing on urinat ion . Mot ile parasites are seen on wet mount.
Trichomonas vaginalis Protozoa Motility Itch Parasitism Trichomonas Human microbiota Urination Pathogen
36
37 B is not correct. 6% chose t his .
38 This is a description of a Candida yeast infect ion. Vulvovaginal candidiasis (VVC) may manifest wit h vaginal it ching and a t hick, white,
39 cottage-cheese discharge. Diagnosis is made by 10% pot assium hydroxide preparat ion on which pseudohyphae are seen. Vaginal pH is not
expected t o increase above normal levels with VVC. It is important to note that, unlike bact erial vaginosis, t he loss of lactobacilli from t he
40
vagina is not associat ed with VVC .
41 Potassium hydroxide Bacterial vaginosis Candidiasis PH lactobacillus Hypha Candida (fungus) vagina Itch Yeast Potassium

42
C is not correct. 13% chos e this .
43 St aphy lococci do not represent a majori ty of t he f lora in the vagina but may be found around the external vag inal opening, as staphylococci
44 are normal skin f lora. Their loss would likely not influence the suscept ibility to bacterial vaginosis.
Skin flora Staphylococcus Bacterial vaginosis vagina Flora
45
• 46 E is not correct. 19°/o chose this .
• 47 Bacteroides fragilis is an anaerobe bacteria. It colonizes some parts of human body, such as t he colon and the vagina. The loss of Bacteroides
• 48
f rom t he vag ina, however, does not increase t he ri sk of bacterial vag inosis. M oreove r~ t he pat ient was most likely st art ed on amoxicillin, which
is not effective against Bacteroides .
. 49 Amoxicillin Bacteroides fragilis Bacteroides Bacterial vaginosis Anaerobic organism vagina Bacteria Colon (anato,.,)
• 50

8
Lock Suspend
s 0
End Block
Item: 4S of ~ ,• Mark <::J [:::> ""I ~· ~'j
100 J.. Previous Next faiUI~S
LAb Notes Calculator

24
25 Bottom Line:
26 Lactobacillus predominat es in normal vaginal flora.
List of microbiota species of the lower reproductive tract of women Vaginal flora Lactobacillus Flora
27
28
29
30
141;fil·1i•J f or yea r:[ 2017
FIRST AID FA CTS .
•j .

31
32 FA17 p 174.2
33
Normal flora: lOCATION MICROORGAN ISM
34
dominant Skin S epidermidis
35
36 Nose S epidermidis; colon ized by S aureus
37
Oropha rynx Viridans group streptococci
38
39
Dental plaque S mutans
40 Colon B fragilis > E coli
41
Vagina Lactobacillus, colonized byE coli and group
42
B strep
43
44 eonates delivered by C-section have no Aora but are rapidly colonized after birth.
45
• 46
FA17 p 144.2
• 47
• 48
Gardnerella vagina/is A pleomorphic, gram-variable rod im·olved in I don' t have a clue why I smell fish in the vagina
. 49 bacterial vaginosis. Presents as a gray vaginal garden!
• 50 discharge with a fishy smell; nonpainful (Ys Amine whiff test- mixing discharge with 10%
8
Lock
s
Suspend
0
End Block
Item: 45 of - ,• Mark --<) [::> ""'I ~· 1!';:'1
100 ~ Prev1o u s Next Labf a lu es Note s Calculator

24
FA17 p 144.2
25
26 Gardnerella vagina/is A pleomorphic, gram-\·ariable rod im·olved in I don't have a clue why I smell fish in the ' agina
27 bacterial vaginosis. Presents as a gray v<lginal garden!
28 discharge with a fishy smell; nonpainful (\'S Amine whiff test- mixing discharge with 10%
29 vaginitis). Associated with sexmll activity, but KOH enhances fishy odor.
30 not sexuallr transmitted. Bacterial \'3ginosis
31 is also characterized by overgrowth of
32 certain anaerobic bacteria in \·agina. Clue
33 cells (vaginal epithelial cells covered\\ ith
34
Gardnerella) have stippled appearance along
35
outer margin (arrow in ).
36
Treatment: metronidazole or clindamycin.
37
38
39 FA17 p 177.3
40 Common vaginal infections
41 Bacterial vaginosis Candida vulvovaginitis
Trichomonas vaginitis
42
43
SIGNSAN DSYM PTOMS o inflammation lnAalllmation ("strawberry Inflammation
44
Thin, white discharge r.J with . ")
CCrVIX T hick, white, "cottage cheese"
45 fishy odor Frothy, yel low-green, fou l- discharge 9
• 46 ~me l ling discharge
• 47 LAB FINDINGS Clue cells Motile trichomonads Pseudohyphae
. 48 pH normal (4.0-4.5)
pH > 4.5 pll > 4.5
. 49
TREATMENT Metronidazole lelronidazole -azoles •
• 50

8
L.odt
s
Su~pl'nd
~
End Block
Item: 45 of - ,• Mark --<) [::> ""'I ~· 1!';:'1
100 ~ Prev1o u s Next Labf a lu es Note s Calculator

24
25
FA1 7 p 177.3
26 Common vaginal infections
27 Bacterial vaginosis Trichomonas vaginitis Candida vulvovaginitis
28
SIGNS AND SYMPTOMS o inflammation In Aammat ion ("strawbcrry Inflammation
29
T hin, white discharge fJ with cervix") Thick, white, "cottage cheese"
30
31
fishy odor Frothy,) cllo\\-grecn, foul- discharge m
~mc llin g discharge
32
33 LAB FINDINGS Clue cells 1\ lotile triehomonads Pseudohyphae
34 pH > 4.5 pi I > 4.5 pH normal (4.0-4.5)
35
TREATMENT letron idazole letronidazole -azoles
36
Treat sexual partner(s)
37
38
39
40
41
42
43
44
45
• 46
• 47
. 48
\ t
. 49
• 50 •

8
L.odt
s
Su~pl'nd
~
End Block
Item: 46 of - ,• Mark -<J [:::> "'I ~ · ~
100 ~ P~v1ous N @xt Labl lues N o tes Calcula to r

24
A pregnant woman in Nairobi, Kenya, develops nausea, vomit ing, and watery diarrhea associated with abdom inal pain. On physical
25
examination, she has icteric sclera and jaundice. Stool testing is negative for ova and parasites, and the st ool culture is negative for
26 pathogenic bacteria. She has a low-grade fever and stable vital signs but dies within 2 weeks of developing sympt oms.
27
28 This patient's condition is associated with what histologic finding?
29
30 A. Hepatic tissue necrosis
31
B. Hepatocyte congestion
32
C. Irregular mass lesion
33
34 0 . Micronodular cirrhosis
35
E. Well -circumscribed granuloma
36
37
38
39
40
41
42
43
44
45
' 46
' 47
. 48
. 49
. 50

8
Lode.
s
Suspe-nd
8
End Bloc:k
Item: 46 of ~ ,• Mark <::J [:::> ""I ~· ~'j
100 J.. Previous Next LAbfaiUI~S Notes Calculator

24
25
The correct a nswer is A. 60°/o chose t h is.
26
This patient has acut e hepat it is E, which is cl assica lly associat ed with fat alit ies and fulm inant hepat it is in pregnant women. Acut e hepatit is E
27 infection consists of two phases: a prodromal phase and an ict eric phase. I n t he prod romal phase, t ypical symptoms include fever, myalgias,
28 nausea, and vomit ing. The ict eri c phase manifests with symptoms of cholestasis, such as jaundice, dark uri ne, pa le stools, and pruri tus. This
patient 's sympt oms were consist ent wit h the prodroma l phase. Hepat it is E is transmit ted v ia the feca l-oral rout e, and infect ion rat es are high
29
in sub-Saharan Africa. Acut e hepat it is infections may be associated wit h foca l hepat ocyte damage, which results in pat chy necrosis of liver
30 cells. Altern at ively, fu lminant hepat it is can show massive necrosis of hepatocyt es in all zones (possible peri portal sparing) wit h ret iculin
31 collapse.
Hepatocyte Jaundice Hepatitis E Fecal-oral route Cholestasis Itch Hepatitis Necrosis Prodrome Acute liver failure Fulminant Liver Nausea Sub-Saharan Africa Vomiting Urine Fever Reticular fiber Infection
32
Acute hepatitis
33
34 B is not correct. 17% chose this .
35 Hepatocyte congest ion suggests ri ght -sided heart failure and nut meg liver.
Hepatocyte Congestive hepatopathy Nutmeg Liver Heart failure
36
37 C is not correct. 6°/o chose this .
38
An irregular mass lesion would be typical fo r hepat ocellular carcinoma.
Hepatocellular carcinoma Neoplasm Carcinoma Lesion
39
40 D is not correct. 12% c hose this .
41 Micronodular cirrhosis is associated wit h chronic liver disease and is a result of disorders such as alcoholism, Wilson disease, and
hemochromat osis. Note t hat postinfectious or drug- induced hepat it is usua lly result in macronodular (nodules > 3 mm) cirrhosis.
42 Iron overload Cirrhosis Wilson's disease Liver Hepatitis Liver disease Alcoholism Chronic liver disease
43
E is not correct. 5°/o chos e this .
44
A well-circumscri bed granu loma suggests Echinococcus infect ion.
45 Granuloma Echinococcus Infection
46
• 47
Bottom Line :
• 48
. 49
Hepatit is E can be fat al, part icula rly to pregnant women, due t o cirrhosis, seen as pat chy necrosis on biopsy.
Hepatitis E Cirrhosis Biopsy Hepatitis Necrosis
• 50

8
Lock
s
Suspend
0
End Block
Item: 46 of ~ ,• Mark <::J [:::> ""I ~· ~'j
100 J.. Previous Next LAb faiUI~S Notes Calculator

24 • •
25 FA17 p 168.1
26 Hepatitis Signs and symptoms of all hepatitis viruses: episodes of fever, jaundice, t ALT and AST. Naked viruses (HAY
27 viruses and I lEV) lack an envelope and are not destroyed by the gut: the \"Owcls hit your bowels.
28
HBV D 'A polrmerase has D A- and RNA-dependent activities. Upon entry into nucleus, the polymerase
29
completes the partial dsD 'A. Host R1 A polymerase transcribes m RNA from \·iral D 'A to ma ke vira l
30
proteins. The DNA polymerase then re\·erse transcribes viral R A to D1 A, which is the genome of the
31
progeny v1rus.
32
33
HCV lacks 3'-5' exonuclease activity -+ no proofreading ability -+ variation in antigenic structures of HCV
34
envelope proteins. Host antibody production lags behind production of new mutant strains of HCV.
35 Viru s HAV HBV HCV HDV HEV
36 FAMILY RNA picornavirus DNA hepadnavirus R 'A Aavivirus R A deltavirus R1 A hepcvirus
37 TRANSMISSION Feca l-oral (shellfish, Parenteral (Blood), Primarily blood Parenteral, sexual, Fecal-oral,
38
travelers, day care) sexual (Baby- (IVDU, post- perinatal especially
39
making), perinatal transfusion) waterborne
40
41
(Birthing)
42 INCUBATION Short (weeks) Long (months) Long Superinfection Short
43 (HOY after
44 ! !BY) =short
45 Coinfection (HOY
46 with HBV) = long
• 47
CLINICAL COURSE Asymptomatic Initially like serum May progress to Similar to HBV Fulminant hepatitis
. 48
. 49
(usually), Acute sickness (fever, C irrhosis or in Expectant
• 50
arth ralgias, rash); C arcinoma (pregnant) women •

8
Lock
s
Suspend
0
End Block
Item: 46 of - ,• Mark --<) [::> ""'I ~· 1!';:'1
100 ~ Prev1o u s Next Labf a lu es Notes Calculator

24 • LIVER BIOPSY 1-1 epa tocyte Granular Lymphoid Similar to HBV •


Patchy necrosis
25
swelling, eosinophilic aggregates with
26
monocyte "ground glass" foca l areas of
27
infiltration, appearance; macro,esicttlar
28
29
Councilman cytotoxic T cells steatosis
30
bodies mediate damage
31 NOTES No carrier state Carrier state Carrier state ,·ery Dcfecti,·e \irus, E nteric. Epidemic,
32 ("Alone") common common Depends on 1-1 B no carrier state
33 HBsAg
34
35
36 FA17 p 170.1
37 Hepatitis serologic markers
38
Anti-HAV (lgM) lgM antibody to IIAV; best test to detect acute hepatitis A.
39
40 Anti-HAV (lgG) IgG antibody indicates prior HAV infection and/or prior vaccination; protects against reinfection.
41 HBsAg Antigen found on surface of IIBV; indicates hepatit is B infection.
42
Anti-HBs Antibody to HBsAg; indicates iu 1munity to hepatitis B due to vaccination or recovery from
43
44
infection.
45 HBcAg Antigen associated with core of II B .
46 Anti-HBc Antibody to HBcAg; lg.YI = acute/recent infection; lgG = prior exposure or chronic infection. lgM
47
0
anti-IIBe may be the sole <f) marker of in fection during window period.
. 48
. 49
HBeAg Secreted by infected hepatocyte into ci rculation. Not part of mature HBV ,·irion. Indicates acti,·e
0 50
viral replication and therefore high transmissibility and poorer prognosis. •
8
L.odt
s
Su~pl'nd
~
End Block
Item: 46 of - ,• Mark --<) [::> ""'I ~- 1!';:'1
100 ~ Prev1o u s Next Labf a lu es Notes Calculator

24 • •
Import ant Incubation Prodrome. Convalescence
25 Early Late
d iagnostic penod acute disea~
26 tests
HBsAg Anb· Antt·HBs Surface antigen
HBsAg
27 (anb·HBcl HBc !antt·HBcl HBsAg \
DNA
28 Anti· H Bc polymerase
DNA
29 Relat ive polymerase
30
con centration
o f reactants
HBV particles DNA ~
Envelope
31 anttgen (-)
32 HBsAg HBeAg- - - . (+)
33 '""'
Wtndow pertod Anti· HBs
34

¥~
Coreanttgen _ /
HBeAg Anti·H Be HBcAg
35
Level o f
36 d etect ion
37
Mo n ths after 0 1 2 3 4 5 6 7 8
38 exposure Sympto ms
39
SGPT !ALT)
40
41
42
HBsAg Anti-HBs HBeAg Anti-HBe Anti-HBc
43 Acute HBV ,/ ,/ lg 1
44 ,/
Window lgM
45
Chronic HBV (high infectivity) ,/ ,/ lgC
46
0 47 Chronic HBV (low infectivity) ,/ ,/ lgC
. 48 ,/ ,/
Recovery lgC
. 49
Immunized ,/ •
0 50

8
L.odt
s
Su~pl'nd
~
End Block
Item: 46 of ~ ,• Mark <::J [:::> ""I ~· ~'j
100 J.. Previous Next LAb faiUI~S Notes Calculator

24 • •
FA17 p 160.1
25
DNA viruses
26
VIRAL FAMILY ENVELOPE ONA STRUCTURE MEDICAL IMPORTANCE
27
28 Herpesviruses Yes OS and linear See Herpesviruses entry
29 Poxvirus Yes OS and linear Smallpox eradicated world wide by use of the live-
30 (largest 0 1 A virus) attenuated vaccine
31 Cowpox ("milkmaid blisters")
32 Molluscum contagiosum -Aesh-colored papule with
33 central umbilication
34
Hepadnavirus Yes Partially OS and circular HBV:
35
Acule or chronic hepatitis
36
37
lot a retrovirus but has reverse transcriptase
38 Adenovirus 0 OS and linear Febrile pharyngitis rJ-sore throat
39 Acute hemorrhagic cystitis
40 Pneumonia
41 Conjunctivitis- "pink eye"
42
43
44
45
46
• 47
Papillomavirus No OS and circular HPV-warts (serotypes 1, 2, 6, 11), Cl , cervical
. 48
cancer (most commonly 16, 18)
. 49
• 50 • Polyomavirus No OS and circular JC virus-progressive multifoca lleukocncephalopathy •

8
Lock
s
Suspend
0
End Block
Item: 47 of - ,• Mark -<J
P~v1ous
[:::> "'I ~ ·· ~
100 ~ N @xt Labl lues N o tes Calcula to r

24
A 5-month-old girl is brought t o t he pediatrician wit h rhinorrh ea, fever, cough, sneezing, and breat hing difficulty. She is using accessory
25
muscles to breathe and has m ild cyanosis. Auscult at ion revea ls bilateral wheezing and rhonchi. The pediatrician suspects t hat her
26 symptoms are due to a v iral agent.
27
28 The pathogenesis of the most likely cause of this patient 's symptoms is
29 :
30 A. Exotoxin that permanently inactivates G; (adenosine diphosphate ribosylation), leading to increased cAMP
31
B. IgE attaches mast cell/basophils, leading to degranulation
32
C. Polyribosylribitol phosphate capsule that inhibits phagocytosis
33
34 0 . Release of oxygen radicals that causes epithelial necrosis within the lung
35
E. Surface fusion protein that causes cell membranes of nearby cells to merge
36
37
38
39
40
41
42
43
44
45
46
. 47
. 48
. 49
. 50

8
Lode.
s
S uspe-nd
8
End Bloc:k
Item: 47 of ~ ,• Mark <::J [:::> ""I ~· ~'j
100 J.. Previous Next LAbfaiUI~S Notes Calculator

24
25 The correct a ns wer is E. 58°/o chose this .
26 This is a case of bronchiolit is, most commonly caused by resp irato ry syncytial virus (RSV) . Transmission is t hrough close contact wit h an
27 infected individual. Bronchiolit is is t ypically caused by RSV in infants and young children <5 years of age, and by Mycoplasma pneumoniae in
28
children >5 years of age. I n t he RSV, t he viral surface contains fusion prot eins (F prot ein), which cause membranes of nearby cells to merge,
forming the characteristic syncytia. Fusion prot eins are also associat ed with parainfluenza, which leads to viral cytolysis and the format ion of
29 multinucleat ed giant cells. RSV infect ion can be diagnosed by direct immunofluorescence of t he viral ant igen .
30 Syncytium Human respiratory syncytial virus Immunofluorescence Bronchiolitis Antigen Direct fluorescent antibody Protein Human parainfluenza viruses Virus Giant cell Infection

31 A is not correct . 10 % chos e this .


32 This describes the pat hogenesis of Bordetella pertussis infection, which is mainly found in infants and young children. It is not commonly seen
33 in the Unit ed States because of t he nearly universal use of t he pert ussis vaccines. Symptoms can include loud inspiratory "whoop " after
violent expirat ory coughs. Even t hough B. pertussis infection can occur in infant s, our pat ient does not exhibit sympt oms of whooping cough.
34
Also, B. pertussis infect ion is associat ed with a bacteria l, not a viral origin. Respirat ory syncytial virus (RSV) is t herefore a more likely cause.
35 Bordetella pertussis Pertussis Human respiratory syncytial virus Bordetella Pathogenesis Virus Cough Infection

36
B is not correct. 10% chose this .
37
This describes the pat hogenesis of allergic rh init is, which is caused by allergens (eg, pollens) t hat induce a local hypersensitivity react ion.
38 Suscept ible individ uals produce IgE against foreign mat erial, such as pollen. This IgE attaches to mast cells wit hin t he nasal cavity. When re-
39 exposure t o t he same pollen occurs, the pollen can bind to t he IgE on mast cells, causing acute and/or prolonged deg ran ulation, leading to
t he release of several immune reg ulators. However, t his process is not an infectious-relat ed disease .
40
Mast cell Allergic rhinitis Immunoglobulin E Degranulation Pollen Rhinitis Nasal cCPJity Allergen Pathogenesis Hypersensitivity Allergy
41
42 C is not co rrect. 13% chos e this .
43 This describes the pat hogenesis of epig lott itis, caused by t he pathogen Haemophilus influenzae (usually t ype B), which causes fever, increased
44
WBC counts, inspirat ory st ridor, and respi ratory failure in 2- to 3-year-olds. This does not fit in with our patient 's prof ile, who is only 5 mont hs
old and presents wit h sympt oms of vira l rat her t han bacterial respiratory t ract infection.
45 Stridor Epiglottitis Haemophilus influenzae Pathogenesis Respiratory failure Respiratory tract Respiratory tract infection Fever Pathogen Infection
46
D is not correct. 9°/o chose this.
47
This describes the pat hogenesis of Mycoplasma pneumoniae, which is relat ed to bronchit is or bronchiolit is in children >5 years old. This does
• 48 not f it our particular patient profile.
. 49 Mycoplasma pneumoniae Bronchitis Bronchiolitis Pathogenesis Mycoplasma

• 50

8
Lock
s
Suspend
0
End Block
Item: 47 of ~ ,• Mark <::J [:::> ""I ~· ~'j
100 J.. Previous Next LAbfaiUI~S Notes Calculator

24 • •
25 Bottom Line:
26 The most common cause of bronchiolitis fo r infants and young children <5 yea rs of age is RSV, a member of the fam ily Paramyxoviridae. Its
27 pathogenesis involves t he surface protein F, which causes nearby cells to merge and induces t he cha racteristic syncytia l a ppearance in
pathology.
28 Param(xoviridae Protein Bronchiolitis Syncytium Human respiratory syncytial virus Pathogenesis
29
30
31 ljl:fil·11•i for year:[ 20 1 7
FIRST AID FA CTS
•I
32
33
FA17 p 163.1
34
35
RNA viruses
36 VIRAL FAMILY ENVELOPE RNA STRUCTURE CAPSID SYMMETRY MEDICAL IMPORTANCE
37 Reoviruses 0 O S linear Icosahed ral Coltivirus" - Colorado tick fever
38 10- 12 segm ents (double) Rotavirus- cause offatal diarrhea in child ren
39
Picornaviruses No SS ® linear Icosahed ral Poliovirus- polio-Salk/Sabin vaccines- IPV/OPV
40
Echovirus- aseptic men ingitis
41
Rhi novirus-"common cold"
42
C oxsackievirus- aseptic mening itis; herpangina
43
(mouth blisters, fever); hand, foot, and mo uth
44
disease; myoca rditis; pericard itis
45
IIAV-aeute viral hepatitis
46
P E RCI I
47
. 48 Hepevirus No SS ® linear Icosahed ral HEV
. 49 Caliciviruses No Icosahedral 'orovirus-vira l gastroenteritis
SS ® linear
• 50 • •

8
Lock
s
Suspend
0
End Block
Item: 47 of - ,• Mark -<J
P~v1ous
[:::> "'I ~ ·· ~
100 ~ N @xt Labl lues N o tes Calcula to r

24 A A

Caliciviruses No SS ® linear Icosahedral lorovirus-viral gastroenteritis


25
26 Flaviviruses Yes SS ® linear Icosahedral HCV
27 Yellow fever1
28 Denguea
29 I. Louis encephalitis3
30 \Vest Nile viru~ {meningoencephalitis)
31 Zika 'i rus
32
Togaviruses Yes SS ® linear Icosahedral Rubella
33
Western and Eastern equine eneephalitis3
34
Chikungunra ,-irus
35
36
Retroviruses Yes SS ® linear Icosa hedraI Have reverse transcriptase
37 2 copies (HTLV), HTLV- T-cellleukemia
38 complex IIIV-AIDS
39 and conica l
40 (IIIV)
41 Coronaviruses Yes SS ® linear Helica l "Common cold," SARS, ivrF.RS
42
43
Orthomyxoviruses Yes ss e linear r.lelica l lnAuenza virus
44
8 segments
45 Paramyxoviruses Yes ss e linear IIclical PaRal\lp ovirus:
46 1 onsegmented ParainAuenza-croup
47 RSV- bronchiolitis in babies; Rx- ribavirin
. 48 \ 1easles, ~ t umps
. 49
. 50
Rhabdoviruses Yes ss e linear Helical Rabies

8
Lode.
s
S uspe-nd
8
End Bloc:k
Item: 47 of - ,• Mark -<J
P~v1ous
[:::> "'I ~ ·· ~
100 ~ N @xt Labl lues N o tes Calcula to r

24 A A

25 Rhabdoviruses Yes SS 8 1inear Helica l Rabies


26 Filoviruses Yes SS 8 linear Helica l Ebola/]1. larburg hemorrhagic fe,·er-often fatal!
27
28
Arenaviruses Yes SS (if) and 8 ll elical LCMV-Iymphocytic choriomeni ngitis' irus
29
circular Lassa b ·er encephalitis-spread by rodents
30 2 segments
31 Bunyaviruses Yes SS 8 circular Helical California encephalitis3
32 3 segments SandAy/Rift Va lley fe,·ers3
33 Crimean-Congo hemorrhagic b·e~
34 II a ntm irus-hemorrhagic fever, pneumon ia
35
Delta virus Yes SS 8 circular Uncertain H DV is a "defecti,·e" ,·irus that requires the
36
presence of I IBV to repIicate
37
38 SS, single-stranded; OS, double-stranded; <if>, positive sense; 8 , negative sense; a= arbO\·irus, arthropod borne (mosquitoes, ticks).
39
40
FA17 p 165.3
41
42 Paramyxoviruses Paramyxoviruses cause disease in children. They include those that cause parain fluenza (croup:
43 seal-like barking cough), mumps, measles, RSV, and human mctapncumo,·irus, which causes
44 respi ratory tract infection (bronchiolitis, pneumonia) in infants. All contain surface F (fusion)
45 protein, wh ich causes respiratory epithelial cells to fuse and form multinucleated cells.
46 Palivizumab (monoclonal antibod) against F protein) prevents pneumonia caused by RSV
47 infection in premature infants.
. 48
Palivizumab for Paramyxovirus (RS ) Proph)'la\ is in Premies.
. 49
. 50

8
Lode.
s
S uspe-nd
8
End Bloc:k
Item: 48 of - ,• Mark --<) [::> ""'I ~· 1!';:'1
100 ~ Prev1o u s Next Labf a lu es Note s Calculator

24 •
25
A med ical student presents to an infectious disease specialist complaining of abdominal distention and tenderness. The patient reports no r~AJ
recent changes in normal bowel habits. Physical examination shows hepatosplenomegaly. Bowel sounds are normal. On questioning, the
26 patient reports that he has traveled to Bra zil 1- 2 times per year for the past several years to study tribal medical practices. He frequently
27 went swimming in the Amazon river to wash himself.
28
29 Which of the following conditions led to this patient's abdominal distention?
30 :

31 A . Appendicitis
32 B. Bowel obstruction
33
C. Enterocolitis
34
35
D. Portal hypertension
36 E. Ruptured viscus
37
38
39
40
41
42
43
44
45
46
47
. 48
. 49
• 50 •

8
L.odt
s
Su~pl'nd
~
End Block
Item: 48 of - ,• Mark -<J [:::> "'I ~ · ~
100 ~ P~v1ous N @xt Labl lues N o tes Calcula to r

24 A A

25 The correct answer is D. 60°/o chose this.


26 Schistosomiasis is a parasit ic disease with hepatic involvement. Schistosoma mansoni (see image)
27 cercariae, which are commonly found in fresh waters of South America, penetrate the host's skin, invade
28
the peripheral vasculature, and event ually settle in the portal or pelvic venous vasculature. Due to
migration of parasites to the intest inal vasculature, patients may develop diarrhea and possibly dysentery
29 several weeks after infection. Patients may also develop more systemic signs of infection, such as fever.
30 Parasites provoke local fibrosis and granuloma formation. Therefore, during chronic infection, the presence
31
of S. mansoni in the portal circulation may lead to cirrhosis and portal hypertension. Sequalae of portal
hypertension, including hepatosplenomegaly, ascites, and esophageal varices, may also be observed. The
32 abdominal distension seen in this patient may be due to ascites.
33 Esop~Hie~ •ce .-• ~t" 01 a IT"''GIIlson• Portal hypertension Ascites Hepatosplenomega l>v'\enter, C•rrhos1s ~t, £to.o ••a.s•s Paras•bc d•sease D'arrhea

Granuloma Trematode hfe cycle stages Hypertens•on Schistosoma Fibros•s Parasitism C1rculatory system fever Liver Abdommal d•stens•on
Image courtesy of CDC
34
35 Portal \olerlOUS system varices Infection

36 A is not correct. 3°/o chose this.


37 Appendicitis commonly manifests with right lower quadrant abdominal pain, fever, nausea, vomiting, and leukocytosis, but not ascites.
38 Ascit~ Leukocytosis AppendiCitis Quadrant (abdomen) Nausea Abdominal pam Vom•tmg Fever

39 B is not correct. 11°/o chose this.


40 Bowel obstruction generally man ifests wit h nausea, vomiting, and decreased or absent bowel sounds. This pat ient has none of th ese signs or
41 symptoms.
Bowel obstruct1on Nausea Vom1bng Stomach rumble Gastrointestinal tract
42
43 C is not correct. 22% chose this.
44 Enterocolitis would not manifest with signs and sym ptoms of ascit es. The usual symptom is diarrhea . The pat ient 's current symp toms do not
inciLJde changes in bowel habits.
45
Asc1tes Diarrhea EnterocolitiS S(mptom Gastrointestinal tract
46
E is not correct. 4 °/o chose this.
47
Ruptured viscera may manifest wit h signs of peritonitis, such as rebound tenderness. Rupture may result from ischemic bowel disease or
48
obstruction. This patient's history, physical exam ination, and imaging studies are inconsistent wit h this cause.
. 49 Penton.t.e: Blumberg ~•on Organ (anatomy) Ischemia Physical examination Gastrointesbnal tr~

. 50

8
Lode.
s
S uspe-nd
8
End Bloc:k
Item: 48 of ~ ,• Mark <::J [:::> ""I ~· ~'j
100 J.. Previous Next LAbfaiUI~S Notes Calculator

24 • •
25 Bottom Line:

26 Schistosoma mansoni, a parasit e that lives in fresh wat er in many parts of t he world, can cause portal hypertension by inhabiting t he venous
27
vasculat ure of t he liver. Treat ment is a single oral dose of praziq uant el.
Praziquantel Schistosoma mansoni Portal hypertension Liver Schistosoma Hypertension Parasitism Circulatory system Fresh water
28
29
30
31
i@l;fil·1i•J for year:[ 2017
FIRST AID FACTS
•I
32
33 FA17p154.1
34 Protozoa- others
35 ORGANISM DISEASE TRANSMISSION DIAGNOSIS TREATM ENT
36 Visceral infections
37
Trypanosoma Chagas disease-dilated Reduviid bug Trypomastigote in Benznidazole
38
cruzi cardiomyopathy with ("kissing bug") blood smear fJ or nifurtimox;
39
40
apical atrophy, megacolon, feces, deposited C n JZing in my
41
megaesophagus; predominantly in a painless bite Benz, with a fur
42
in South America (much Iike a kiss) coat on
43 Unilateral periorbital swelling
44 (Romai'ia sign) characteristic of
45 acute stage
46
Leishmania Visceral leishmaniasis Sand fly Macrophages Amphotericin B,
47
donovani (kala-azar) -spiking fevers, contain ing sodium
48
hepatosplenomegaly, a mastigotes stibogluconate
. 49
• 50
pancytopenia •

8
Lock
s
Suspend
0
End Block
Item: 48 of ~ ,• Mark <::J [:::> ""I ~· ~'j
100 J.. Previous Next faiUI~S
LAb Notes Calculator

24 • • •
25
26
=
27
28 FA17 p 156.2
29 Trematodes (flukes)
30 ORGANISM DISEASE TRANSMISSION TREATMENT
31 Schistosoma Liver and spleen enlargement Snails are host; cercariae Praziquantel
32 (S mansoni, egg with penetrate skin of humans
33
lateral spine r:J), fibrosis,
34
inflammation, portal
35
hypertension
36
Chronic infection with
37
[l) 0 ... 0 S haematobium (egg with
38 0
39
t> ":l term inal spine [l)) can lead
40
to squamous cell carcinoma
41 of the bladder (pa inless
42 hematuria) and pulmonary
43 hypertension
44 Clonorchis sinensis Biliary tract inflammati on Undercooked fi sh Praziquantel
45 ..... pigmented gallstones
46 Associated with
47
chola ngioea rei noma
48
. 49
• 50 • •

8
Lock
s
Suspend
0
End Block
Item: 49 of - ,• Mark --<) [::> ""'I ~· 1!';:'1
100 ~ Prev1o u s Next Labf a lu es Notes Calculator

24 •
25
A 45-year-old woman wi th AIDS is brought to the emergency department with a temperature of 38.8°C ( 101.8° F), malaise, and altered r~AJ
mental status, but no other history is available. Lumbar punctu re shows:
26
Opening pressure: 230 mm H2 0
27 WBC count: 3500/mm 3
28 Protein: 120 mg/dL
29 Glucose: 35 mg/dL
30 A bronchoalveolar lavage specimen stained positively with India ink.
31
32 Treatment with which of the following medications should be started?
33
:
34 A. Acyclovir
35
B. Ceftriaxone
36
37
C. Flucytosine
38 D. Nystatin
39
E. Ribavirin
40
41
F. Vidarabine
42
43
44
45
46
47
48
. 49
• 50 •

8
L.odt
s
Su~pl'nd
~
End Block
Item: 49 of - ,• Mark -<J [:::> "'I ~ · ~
100 ~ P~v1ous N @xt Labl lues N o tes Calculato r

24 A A

25
The correct an sw er is c. 65°/o chose this.
26
Cerebrospinal fluid find ings in fungal meningitis and encephalitis include high opening pressure, increased
WBCs with lymphocytic/monocytic predominance, high protein, and low glucose. Stiff neck, photophobia,
27 and vomiting are only seen in about one-fourth of patients; immunocompromised patients do not tend t o
28 present with classic symptoms. India ink staining showing encapsulated yeast is suggestive of Cryptococcus
29 (shown in the image with mucicarmine stain). It is typically never used alone due to resistance.
Amphotericin B should be started as induction t herapy along with flucytosine. If there is clinical
30
improvement, these can be discontinued and fluconazole can be initiated and continued for 8 weeks.
31 Flucytosine works by blocking fungal precursors from converting into pyrimidines.
Muctc¥ ne " n ,... 'C)I \)tenc n 8 Fluconazole RuC'ft:osme India mk Meningitis Stam•no Photophob•a J,..,.,..,..u·t~ le .. •e-•c Fungitl merung•bs Encephalitis
32
Prote.n fungus Yeast Glucose New England Compounding Center- meningitis outbreak \A:uT'IItmg
33 Image courtesy of CDC/ Dr.
34 Edwin P. Ewing, Jr.
35
A is not correct. 5% chose thi s.
36
Empiric treatment for herpes simplex virus- 1 infect ion with acyclovir, which halts viral replication, should be initiated as soon as
37
meningitis/encephalitis is suspected without an obvious et iology because it is associated with a signif icant decrease in morbidity and mortality.
38 Aciclovlr Herpes- ~1mple)C Et1ology Emp1nc therapy Morbidity Virus Infection

39
B is n o t correct. 11 °/o chose thi s.
40 Empiric coverage of bacterial meningit is consist s of vancomycin and a third-generation cephalosporin, such as ceft riaxone. Given the
41 appearance of t he Ind ia ink-stained slide, it is unlikely t hat the patient's illness is being caused by bacteri a. As a third-generat ion
42 cephalosporin, it has broad act ivity against gram- posit ive and gram-negative bacteria. It is t he t reatment of choice fo r Neisseria gonorrhoeae.
Cephalosporin Ceftriaxone vancomycin Neisseria gonorrhoeae Meningitis Gram-negative bacteria Gram-positive bacteria Bactenal meningitis Bacteria Neisseria
43
44 D is n ot correct . 12°/o chose this.
45
Nystatin is chemically sim ilar to amphotericin B; however~ it is used only for topical infections such as oral thrush because it has systemic
toxicity. Its mechanism of action involves binding t o ergosterol to disrupt funga l membranes. It can also be used t opically for diaper rash or
46 vaginal candidiasis.
47 NystM1n Amphotenc1n B Ergosterol Irntant d1aper dermatitis Candidiasis vaginal yeast 1nfect1on Oral cand1dUt!.il• Mechan1sm of action Fungus Rash Diaper Toxicity

48 E is n ot correct . 3°/o chose thi s.


49 Ribavirin is used in chronic hepatit is C and respiratory syncytial virus. It is not used in HIV patients with Cryptococcus. It works by inhibiting
. 50 svnthesis of auanine nucleotides. This medication is a teratoaen: thus women of child-bearina aae must be on two forms of birth control while
8
Lode.
s
S uspe-nd
8
End Bloc:k
Item: 49 of ~ ,• Mark <::J [:::> ""I ~· ~'j
100 J.. Previous Next LAb faiUI~S Notes Calculator

24
25
E is not correct. 3°/o chos e this .
Ribavirin is used in chro nic hepatitis C and respirat ory syncytial virus. It is not used in HI V pat ients wit h Clyptococcus . It works by inhibit ing
26
synt hesis of guanine nucleot ides. This med icat ion is a t erat ogen; thus women of child-bearing age must be on two forms of birth control while
27 t hey are taking this medicat ion . Add it ionally, t his medication can cause severe hemolyt ic anemia, so pat ients must be monitored closely fo r
28 changes in blood cell values.
Teratology Ribavirin Hepatitis C Guanine Hemolytic anemia Hepatitis Anemia HIV syncytium Nucleotide Virus Human respiratory syncytial virus Pharmaceutical drug Birth control Hemolysis Cryptococcus
29
30 F is not correct . 4 °/o chos e this.
31 Vidarabine is used fo r t reat ment of varicella zost er virus in HIV pa t ients, decreasing lesion formation and severi ty of disease. Adverse effects
can be severe, and include leukopenia and t hrom bocyt openia.
32
Vidarabine Varicella zoster virus Leukopenia Thrombocytopenia HIV Virus Chickenpox Lesion
33
34
35
Bottom Line :
36 Cryptococcus is an opportunist ic infe ction t hat st ains with I ndia in k. It is t reat ed with amphotericin Band f lucytosine.
Opportunistic infection Amphotericin 8 Flucytosine India ink Cryptococcus India Infection
37
38
39
l@);fil ~1hl for yea r:l 2o17 y

40 FIRST AID FACTS

41
42 FA17 p 195.1
43 Amphotericin B
44 Binds ergosterol (unique to fungi); forms Amphotericin "tears" holes in the rungal
MECHANISM
45
membrane pores that allow leakage of membrane by forming pores.
46
electrolytes.
47
48
CLINICAL USE Serious, systemic mycoses. Cryptococcus
49 (amphotericin B with/without Aucytosine
• 50 for cryptococcal meningitis), Blastomyces,
8
Lock
s
Suspend
0
End Block
Item: 49 of - ,• Mark --<) [::> ""'I ~· 1!';:'1
100 ~ Prev1o u s Next Labf a lu es Notes Calculator

24
FA17 p 195.1
25
Amphotericin B
26
27 MECHANISM Binds ergosterol (unique to fungi); forms mphotericin "tears" holes in the fungal
28 membrane pores that allow leakage of membrane by forming pores.
29 electrolytes.
30
CliNICAl USE Serious, systemic mrcoscs. Cryptococcus
31
(amphotericin B with/without Aucytosine
32
for cryptococcalmeningitis), Blastomyces,
33
Coccidioides, Histoplasma, Candida,
34
lucor. Intrathecally for fungal meningil is.
35
36
Supplement K+ and ~ lg2+ because of altered
37
renal tubule permeability.
38 ADVERSE EFFECTS Fe,·er/chills ("shake and bake"), hypotension,
39 nephrotoxicity, arrhythmias, anemia, 1V
40 phlebitis ("amphotcrriblc"). Hydration
41 ! nephrotoxicity. Liposomal amphotericin
42 ! toxicity.
43
44
FA17 p 195.3
45
Flucytosine
46
47 MECHANISM Inhibits D 'A and RNA biosrnthcsis by conversion to 5-Auorouracil br cytosine deaminasc.
48 CliNICAl USE Systemic fungal infections (especially meningitis caused by Cryptococcus) in combination with
49 amphotericin B.
• 50 •
8
L.odt
s
Su~pl'nd
~
End Block
Item: 49 of - ,• Mark --<) [::> ""'I ~· 1!';:'1
100 ~ Prev1o u s Next Labf a lu es Notes Calculator

24 •
25 FA17 p 149.1
26 Opportunistic fungal infections
27
Candida albicans alba = white. Dimorphic; forms pseudoh) phac and budding yeasts at 20°C · , germ tubes at
28
3JOC I]l.
29
Systemic or superficial fun gal infection. Causes oral and esophageal thrush in
30
31
immunocompromised (neonates, steroids, di<Jbetes, AI OS), vulvovaginitis (diabetes, use of
32
antibiotics), diaper rash, endocarditis (I drug users), disseminated candidiasis (especiallr in
33
neutropenic patients), chronic mucocutaneous candidiasis.
34 Treatment: oral Auconazole/topical a7ole for vaginal; nystatin, Auconazole, or caspofungin for oral/
35 esophageal; Auconazole, caspofungin, or amphotericin B for systemic.
36 Aspergillus Septate hyphae that branch at 45° \ cute \ nglc . Produces conidia in radiating chains at end of
37 fumigatus conidiophore O.
38 Causes invasive aspergillosis in immunocompromised, patients with chronic granulomatous disease.
39
Can cause aspergillomas in pre-existing lung cavities, especially after TB infection.
40
Some species of Aspergillus produce Anatoxins (associated with hepatocellular carcinoma).
41
42 Allergic bronchopulmonary aspergillosis (ABPA): hypersensitivity response associated with
43 asthma and cystic fibrosis; may cause bronchiectasis and eosinophilia.
44
Cryptococcus 5-IO J.lm with narrow buckling. lleavily encapsulated yeast. lot dimorphic.
45
neoformans Found in soil, pigeon droppings. Acquired through inhalation with hematogenous dissemination
46
to meninges. Culture on Sabouraud agar. I lighlighted with India ink (clear halo 0) and
47
mucicarmine (red inner capsule tal. Latex agglutination test detects polysaccharide capsular
48
49
antigen and is more specific.
• 50
Causes cry ptococcosis, cryptococcal meningitis, cryptococcal encephalitis ("soap bubble" lesions •
8
L.odt
s
Su~pl'nd
~
End Block
Item: 49 of - ,• Mark --<) [::> ""'I ~· 1!';:'1
100 ~ Prev1o u s

24
Next Labf a lu es Notes Calculator
.
25 Mucor and Rh izopus Irregular, broad, nonseplale hyphae branching al "ide angles
26 spp. Mucormycosis. Causes disease mostly in keloaeidolic diabetic and/or neutropenic patients (cg,
27 leukemia). Fungi proliferate in blood vessel walls, penetrate cribriform plate, and enter brain.
28 Rhinocerebral, frontal lobe abscess; cavernous sinus thrombosis. Headache, facial pain, black
29 necrotic eschar on face; mav he~ve cmnial nerve involvement.
'
30 Treatment: surgical debridement, amphotericin B.
31
32
33
34
35
36
37
38
39
40
D
41
42
43
44 •
45
46
47
48
a
49
• 50 •

8
L.odt
s
Su~pl'nd
~
End Block
Item: SO of - ,• Mark -<J [:::> "'I ~ · ~
100 ~ P~v1ous N @xt Labl lues N o t es Calcula to r
24
A 55-year-old woman present s to t he clinic because of a painless mass at the angle of the mandible 1 week after undergoing a dental ~~AI
25
procedure. On physical examination t he physician not es that the mass is draining pus that, on microscopic examination, shows yellowish
26 granules. A stained sample from the abscess shows branching gram-positive rods.
27
28 Which antibiotic would be most effective in t reating this infection?
29
:
30 A. Metronidazole
31
B. Nystatin
32
33 C. Oxacillin
34 0 . Penicillin
35
E. Sulfonamide
36
37
38
39
40
41
42
43
44
45
46
47
48
49
. 50 •
8
Lode.
s
Suspe-nd
8
End Bloc:k
Item: SO of ~ ,• Mark <:::1 t::> ""I ~· ~'j
100 J.. Previous Next LAb faiUI~S Notes Calculator

24
25 The correct a nswer is 0. 73°/o chose t his.
26 This infect ion is caused by Actinomyces israelii, which is a m icroaerophilic organism that can cause oral and facial abscesses. It is part of t he
normal flora of t he mouth and gast rointest inal t ract of humans, and infect ion is init iated usually by t rauma . One import ant characteristic is its
27
"sulfur granules," which can be seen under t he microscope and may drain t hrough sinus t racts in skin. Treatment for t his infection is penicillin.
28 Penicillin Microaerophile Actinomyces Human gastrointestinal tract Gastrointestinal tract Actinomyces israelii Abscess Sulfur Human microbiota Infection Organism Microscope

29
A is not correct . 7°/o chos e this.
30
Met ronidazole is a versat ile drug that can t reat anaerobic infect ions, Clostridium difficile, and parasites, including Giardia and amoebas. It
31 would not be effect ive in t reating Actinomyces infect ions, however.
Metronidazole Actinomyces Clostridium difficile colitis Clostridium difficile (bacteria) Giardia Anaerobic organism Clostridium Parasitism Anaerobic infection
32
33 B is not correct. 4 % chose t his .
34 The long, branched filaments of Actinomyces can resemble fungi, such as Candida albicans. The ora l lesions caused by Candida (known as
35 t hrush) usually occur on the m ucous membranes such as the throat and tongue in immunocompromised pat ients and appear white. It would
be an unlikely cause of an oral abscess in a pat ient with a healt hy immune syst em. Treat ment fo r oral t hrush is nystatin .
36 Candida albicans Nystatin Fungus Actino,.,ces Immunodeficiency Oral candidiasis Candida (fungus) Candidiasis Immune system Abscess Mucous membrane Thrush (bird)
37
C is not correct. 8 °/o chos e this .
38
Oxacillin would be an excellent choice fo r t reat ing cellulit is caused by met hicillin-sensit ive Staphylococcus aureus. However, the presentation
39 of t his infection and its m icroscopic features do not indicat e cellulit is and t hus t his drug would not be indicated for this infect ion.
40 Oxacillin Staphylococcus aureus Cellulitis Staphylococcus Infection

41
E is not correct. 8 °/o chos e this .
42 Sulfonamides can t reat many t ypes of infect ions, including Nocardia, a fi lamentous, beaded, aerobic, partially acid-fast , gram-posit ive
43 bact eriu m . Unlike Actinomyces, Nocardia is not considered part of t he normal f lora. Nocardia causes pulmonary infections in
44 immunocomprom ised pat ients.
Gram-positive bacteria Actinomyces Nocardia Immunodeficiency Acid-fast Sulfonamide (medicine) Aerobic organism Bacteria Human microbiota Sulfonamide
45
46
47 Bottom Line :
48 The pat hognomonic f indings of Actinomyces infect ion are sulfur granules, which can be appreciated under t he m icroscope. Treat ment is
49
penicillin.
Penicillin Pathognomonic Actino,.,ces Microscope Infection
50

8
Lock
s
Suspend
0
End Block
Item: 50 of - ,• Mark -<] 1:> ""'I ~· 1!';:'1
100 ~ Prev1o u s Next Labf a lu es Note s Calculator
24
25 FA17 p 135.3
26
Nocardiavs Both are gram Et> and form long, branching filaments resembling fungi.
27
Actinomyces Nocardia Actinomyces
28
29 Aerobe naerobe
30
Acid fast (weak) fJ I Ol acid fast IE)
31
32 Found in soil 1 orma I oral, reproducti,·e, and C I Aora
33 Causes pulmonary infections in Causes oral/facial abscesses that drain through
34
immunocompromised (can mimic TB but sinus tracts; often associated with dental caries/
35
with 8 PPD}; cutaneous infections after extraction; forms yellow "sulfur granules"; can
36
trauma in immunocompetent; can spread to also cause PID "ith IUOs
37
Ct-:S
38
39 Treat with sulfonamides (T lP-SMX) Treat with penicillin
40 Treatment is a SNAP: Sulfonamidcs-Nocardia; \ctinomyces-Pcnicillin
41
42
43
44
45
46
FA17 p 131.4
47
48
Staphylococcus Cram Et>, catalase Et>, coagulase 8, urease Et> cocci in clusters. Novobiocin sensitive. Does not
49 epidermidis ferment mannitol (vs S aureus).
50 'ormal Aora of skin; contaminates blood cultures. •
8
L.odt
s
Su~pl'nd
~
End Block
Item: 50 of - ,• Mark -<] 1:> ""'I ~· 1!';:'1
100 ~ Prev1o u s Next Labf a lu es Note s Calculator
24
25
FA17 p 131.4
26
27 Staphylococcus Cram®, catalase ®, coagulase 8, urease ® cocci in clusters. Novobiocin sensitive. Does not
28 epidermidis ferment mannitol (vs S cHtreus).
29 'ormal Aora of skin; contaminates blood cultures.
30 Infects prosthetic de,·ices (eg, hip implant, heart valve) and IV catheters by producing adherent
31 biofilms.
32
33 FA17 p 134.2
34
Clostridia (with Cram®, spore-forming, obligate anaerobic rods.
35
exotoxins)
36
37 Ctetani Produces tetanospasmin, an exotoxin causing Tetanus is tetanic paralysis.
38 tetanus. Tetanus toxin (and botulinum toxin)
39 are proteases that clea,·e SJ ARE proteins for
40 neurotransmitters. Blocks release of in hibitory
41 neurotransmitters, CABA and glycine, from
42 Renshaw cells in spinal cord.
43 Causes spastic paralysis, trismus (lockjaw), risus
44
sardon icus (raised eyebrows and open grin),
45
opisthotonos (spasms of spinal extensors).
46
Pre,·enl with tetanus vaccine. Treat" ith
47
antitoxin +1- ,·accine booster. diazepam (for
48
muscle spasms), and \\Ound debridement.
49
50 ( botulinum Produces a heat-labile toxin that inhibits Symptoms of botulism (the 4 D's): Diplopia. •
8
L.odt
s
Su~pl'nd
~
End Block
Item: 50 of - ,• Mark -<] 1:> ""'I ~· 1!';:'1
100 ~ Prev1o u s Next Labf a lu es Note s Calculator
24
25
Cbotulinum Produces a heat-labile toxin that inhibits Symptoms of botulism (the 4 D's): Diplopia,
26
ACh release at the neuromuscular junction, Dysarthria, Dysphagia, Dyspnea.
27 causing botulism. In adults, disease is caused Botulinum is from bad bottles of food, ju ice, and
28 by ingestion of preformed toxin. In babies, honey (causes a descending Aaccid paralysis).
29 ingestion of spores (eg, in honey) leads to I .ocal botox injections used to treat focal
30 disease (Aopp~ baby syndrome). Treat " ith dystonia, achalasia, and muscle spasms. Also
31 antitoxin. used for cosmetic reduction of facial wrinkles.
32
Cperfringens Produces a toxin (lecithinase, a phospholipase) Perfringens perforates a gangrenous leg.
33
that can cause myonecrosis (gas gangrene )
34
and hemolvsis

.
35
Spores can sun·i,·e in undercookcd food;
36
when ingested, bacteria release heat-labile
37
38
enterotoxin ..... food poisoning.
39
40
41
42 C difficile Produces 2 toxins. Toxin A, an enterotoxin, Oi{{icile causes diarrhea. Treatment:
43 binds to brush border of gut and alters Ouid metronidazole or oral vancomycin. For
44 secretion. Toxin B, a cytotoxin, disrupts recurrent cases, consider repeating prior
45 cytoskeleton via actin depolymerizalion. Both regimen, fidaxomicin, or fecal microbiota
46 toxins lead to diarrhea ..... pseudomembranous transplant.
47 zo
colitis (l). Often to antibiotic usc, especially
48 clindamycin or ampicillin; associated with PPI
49 use. Diagnosed by detecting one or both toxins
50 ;., .... t nnl h.,- .,.,t-; non rl.ot-.o,..t-;nn nr PrQ •
8
L.odt
s
Su~pl'nd
~
End Block
Item: Sl of - ,• Mark -<J
P~v1ous
[:::> "'I ~ ·· ~
100 ~ N @xt Labl lues N o tes Calculato r
34
A 22-year-old woman is brought t o t he emergency depart ment via ambulance because of right lower quadra nt pain. Suspecting ~~AI
35
appendicit is, physicians order a routine CT scan. To t heir surprise, a mass unrelated to the appendix is discovered within the pelvic cavity.
36 An obstetrician is consult ed t o perform an emergent surgical procedure.
37
38 Previous infection by which of the following agents most likely put this patient at higher risk for developing t his complication?
39
:
40 A. Chlamydia trachomatis
41
B. Escherichia coli
42
43 C. Herpes simplex virus
44 0 . Human papillomavirus
45
E. Streptococcus agalactiae
46
47
48
49
50
• 51
• 52
. 53
. 54
• 55
' 56
' 57
' 58
. 59
' 60 •
8
Lode.
s
S uspe-nd
8
End Bloc:k
Item: Sl of ~ ,• Mark <::J [:::> ""I ~· ~'j
100 J.. Previous Next LAb faiUI~S Notes Calculator

34
35 The correct answer is A. 73°/o chose this.
36 Chlamydia, t he most common sexually t ransm it ted bact eri a, can cause sca rring of t he fallopian tubes. These lesions can cause a recent ly
37 fertilized egg t o adhere t o t he fallopian t ube wall or cerv ix, or im properly m igrate to the peri toneal or pelvic cavity, resulting in ectopic
pregnancy. An ect opic pregnancy can be complicat ed by rupture and can develop into a life-t hreaten ing condit ion. Risk fact ors include a
38
hist ory of pelvic inflammatory disease, endometriosis, post operative adhesions, and chronic salpingit is.
39 Pelvic inflammatory disease Salpingitis Fallopian tube Endometriosis Ectopic pregnancy Chlamydia infection Pelvic cavity Cervix Peritoneum Adhesion (medicine) Pelvis Bacteria Pregnancy Inflammation

40 Zygote
41
B is not correct. 6 % chose this.
42
Escherichia coli is part of t he norma l flo ra of t he vagina, along wit h Group B streptococcus. It is also one of the most common causes of
43 uri nary t ract infection (UTI) in ambulatory young women, along wit h Staphylococcus saprophyticus. UTis are not associated wit h pelvic
44 inflammatory disease or ectopic pregnancy.
Pelvic inflammatory disease Urinary tract infection Ectopic pregnancy Escherichia coli Streptococcus agalactiae Staphylococcus saprophyticus Streptococcus Staphylococcus Vagina Urinary system Pregnancy
45
Human microbiota Inflammation Pelvis
46
47 C is not correct. 4 °/o chose this .
48 Herpes simplex virus (HSV) type 2 is a sexually t ransm itted, enveloped DNA herpesvirus. Genit al herpes presents with painful penile, vulva r,
or cervical ulcers. HSV is also one of the ToRCHeS (To xoplasmosis, Rubella, Cytomegalovirus, Herpesvirus/H I V, Syphilis) organisms t hat can
49
cross the placent a and negatively affect the fetus. However, HSV is not associat ed with pelv ic inf lammat ory disease or ect opic pregnancy.
50 Pelvic inflammatory disease Ectopic pregnancy Placenta Herpes simplex virus Herpes simplex Genital herpes Virus Fetus Pregnancy DNA Pelvis Inflammation Cervix Cervical cancer Herpesviridae Sex organ

51 Ulcer (dermatology)
• 52
D is not correct. 15% chose thi s •
. 53
Human papillomavirus is associat ed with cervical and ana l carcinomas. It is not associat ed with pelvic inflammatory disease or ectopic
. 54 pregnancy.
• 55 Pelvic inflammatory disease Ectopic pregnancy Human papillomavirus Pregnancy Cervix Cervical cancer Inflammation

• 56 E is not correct. 2 °/o chose this.


• 57 Streptococcus agalactiae is a Group B st rept ococcus t hat is part of t he normal flora in t he vagina and would not increase the risk of developing
• 58 pelv ic inf lammat ory disease or ect opic pregnancy.
Pelvic inflammatory disease Ectopic pregnancy Streptococcus agalactiae Streptococcus vagina Pregnancy Human microbiota Inflammation
. 59
• 60 •
8
Lock
s
Suspend
0
End Block
Item: Sl of - ,• Mark -<J
P~v1ous
[:::> "'I ~ ·· ~
100 ~ N @xt Labl lues N o tes Calculato r
34 •
35 Botto m Li ne:
36 I nfection by Chlamydia can result in scarri ng, which can lead to ectopic pregnancy.
EctopiC pregn.ncy Chlam,od1a 1nfecbon Chlamydia (genus) Pregnancy lnfect1on
37
38
39
40
l@l ;fii ·11•J for year : 2017 •
FIRST A ID FACTS

41
42 FA17 p 181 .1
43
Pelvic inflammatory Top bugs-Chlamydia trachomatis (subacute, Salpingitis is a risk factor for ectopic pregnancy,
44

45
disease often undiagnosed), reisseria gonorrhoeae inrertility, chronic pelvic pain, and adhesions.
46
(acute). C trachomatis-most common Can lead to Fitz-Hugh-Curtis syndrome -
47
bacterial STI in the United Stales. Cervical inrection of the liver capsule and'\·iolin string"
48 motion tenderness (chandelier sign), purulent adhesions of peritoneum to )i,·er rn.
49 cervica l discharge t'J. PI D may include
50 salpingitis, endometritis, hydrosalpinx, and
51 lubo-ova ria n abscess.
• 52
. 53
. 54
• 55

' 56
' 57
' 58
. 59

' 60

8
Lode.
s
S uspe-nd
8
End Bloc:k
Item: 51 of - ,• Mark -<] 1:> ""'I ~· 1!';:'1
100 ~ Prev1o u s Next Labf a lu es Note s Calculator
34
FA17 p 146. 2
35
36
Chlamydia trachomatis serotypes
37 Types A, 8, a nd C Chronic infection, cause blindness due to \BC = Africa, Blindness, C hronic infection.
38 follicular conjunctivitis in Africa.
39
Types D-K Urethritis/PID, ectopic pregnane}', neonatal D-K = everything else.
40
pneumonia (staccato cough) with eosinophilia, leonatal disease can be acquired during
41
neonatal conjunctivitis ( 1-2 "eeks after birth). passage through infected birth canal.
42
43 Types Ll , l2, and L3 Lymphogranuloma venereum -small, painless
44 ulcers on genitals ..... swollen, painful inguinal
45 lymph nodes that ulcerate (buboes). Treat with
46 doxyc}'cline.
47
48
FA17 p606.1
49
Pregnancy complications
50
51 Placental abruption Premature separation (partial or complete) of
• 52 (abruptio placentae) placenta from uterine wall before deliver)'
. 53 of infant. Risk factors: trauma (eg, motor
. 54 vehicle accident), smoking, hypertension,
• 55 preeclampsia, cocaine abuse.
• 56 Presentation: abrupt, painful bleeding
. 57 (concealed or apparent) in third trimester;
• 58 possible DIC, maternal shock, fetal distress.
. 59
Life threatening for mother and fetus .
• 60 •
8
L.odt
s
Su~pl'nd
~
End Block
Item: Sl of - ,• Mark -<J [:::> .il ~ ·· ~
100 ~ P~v1ous N @xt LabValues N o tes Calculato r
34 A A

35 Placenta accreta/ Defective decidual layer -+ abnormal


36 increta/percreta attachment and separation after delivery.
37 Risk factors: prior C-section, inAammation,
38 placenta previa. Three types distinguishable
39 by the depth of penetration:
40 Placenta accreta-placenta attache'> to
41 myometrium without penetrating it; most
42
common type.
43 percreta
Placenta increta- placenta penetrates into
44
myometrium.
45
Placenta percreta -placenta penetrates
46
("perforates") through 11l)'Ometrium and into
47
uterine serosa (invades entire uterine wall);
48
49
can result in placental attachment to rectum
50
or bladder.
51 Presentation: often detected on ultrasound prior
• 52 to delivery. t o separation of placenta after
. 53 delivery -+ postpa rtum bleeding (can cause
. 54 Sheehan syndrome).
• 55 Placenta previa Allachment of placenta to lower uterine
' 56 segment over (or< 2 em from) internal
' 57
cen·ical os. Risk factors: multiparil), prior
' 58
C-section. Associated with painless third-
. 59
trimester bleeding.
' 60

8
Lode.
s
S uspe-nd
8
End Bloc:k
Item: 51 of - ,• Mark -<] 1:> ""'I ~· 1!';:'1
100 ~ Prev1o u s Next Labf a lu es Note s Calculator
34
Placenta previa Attachment of placenta to lo,,er uterine
35
segment over (or< 2 em from) internal
36
cen·ical os. Risk factors: multiparity, prior
37
C-section. Associated with painless third-
38
39
trimester bleeding.
40
41
42
43
44

45
46
47 Partial placenta Complete placenta
48
previa previa Ia
49
Vasa previa Fetal vessels run over, or in close pro,imity
50
to, cervical os. May result in ,·essel rupture,
51
exsanguination, fetal death. Presents with
• 52
triad of membrane rupture, painless vaginal
. 53
. 54
bleeding, fct<d bradycardia (< 110 bce~ls/min).
• 55
Emergency C -section usually indicated .
• 56
Frequently associated with \'elamcnlous
. 57 umbilical cord insertion (cord inserts in
• 58 chorioamniotic membrane rather I han
. 59 placenta .... fetal vessels tra,·el to placenta
• 60 unprotected by Wharton jell}). •
8
L.odt
s
Su~pl'nd
~
End Block
Item: 51 of - ,• Mark -<] 1:> ""'I ~· 1!';:'1
100 ~ Prev1o u s Next Labf a lu es Note s Calculator
34
35 Vasa previa Fetal vessels run over, or in close pro,imity
36 to, cervical os. May result in \'essel rupture,
37 exsanguination, fetal death. Presents with
38 triad of membrane rupture. painless ,·aginal
39 bleeding, fetal bradycardia (< 110 beats/min).
40 Emergency C-section usually indicated.
41
Frequently associated with \'elamentous
42
umbilical cord insertion (cord inserts in
43
chorioamniotic membrane rather than
44
placenta -+ fetal \'esse1s tra,·el to placenta
45
unprotected by Wharton jelly).
46
47 Postpartum Due to 4 T's: Tone (uterine atony; most
48 hemorrhage common), Trauma (lacerations, incisions,
49 uterine rupture}, T hrombin (coagulopathy},
50 T issue (retained products of conception).
51
Ectopic pregnancy Most often in ampulla offallopi<l n lube Pain +/- bleeding.
• 52
(rJ shows 10-mm embryo in oviduct at 7 Risk factors:
. 53
weeks of gestation}. Suspect with history of Prior ectopic pregnancy
. 54
amenorrhea, lower-than-expected rise in hCC History of infertility
• 55
• 56
based on dates, and sudden lower abdomina l Salpingitis (PID)
. 57
pain; confirm with ultrasound. Often Ruptured appendix
• 58
clinically mistaken for appendicitis. Prior tuba I surgery
. 59
• 60 •
8
L.odt
s
Su~pl'nd
~
End Block
Item: 52 of - ,• Mark -<J [:::> "'I ~ · ~
100 ~ P~v1ous N @xt Labl lues N o tes Calculato r
34
A 5-year-old boy is brought to the physician because he is let hargic and has vom ited t wice in 1 day. The pat ient' s temperature is 39.6°C ~~AI
35
(103.3° F). On physical examination, t he pat ien t has an extremely tender neck and a maculopapular rash on his t runk, wit h pet echiae. A
36 sample of cerebrospinal flu id is t aken, and Gram staining shows gram-negative diplococci.
37
38 Which of the following is/are the most likely sequelae of this infection?
39
:
40 A. Ascending peripheral nerve paralysis
41
B. Paroxysmal hypertension, palpitations, anxiety, and drenching sweats
42
43 C. Right-sided lower facial droop
44 0 . Sepsis with adrenocortical insufficiency
45
E. Sudden onset of fever, anemia, and a new heart murmur
46
47
48
49
50
51
• 52
. 53
. 54
• 55
' 56
' 57
' 58
. 59
' 60 •
8
Lode.
s
S uspe-nd
8
End Bloc:k
Item: S2 of - ,• Mark -<J [:::> "'I ~ · ~
100 ~ P~v1ous N @xt Labl lues N o tes Calculato r
34 A A

35
The correct answer is D. 68°/o chose this.
36
The boy's symptoms and laboratory find ings are consistent with meningitis and septicemia caused by
37 Neisseria meningitidis. This gram- negative diplococcus is the second most common cause of meningit is in
38 children aged 6 months to 6 years and is the leading cause of meningitis in older adolescents and young
39 adults. The bacterium grows best on Thayer-Martin VCN medium, which contains antibiotics that kill
competing bacteria and fungi. The cerebrospinal fluid sample would most likely show a WBC count typically
40 > 1000/mm 3 with mostly neutrophils, glucose <40 mg/dl, and protein 100-500 mg/dl, in keeping with the
41 bacterial origin. Once in the bloodstream, the bacteria can cause a characteristic petechial rash (like that
42 shown in the image) due to endotoxin, and it predisposes an individual to Waterhouse-Friderichsen
syndrome (sepsis with adrenal insufficiency).
43 W?'terhOil,e--fiJd~ i.i s 1dro te l.Jpopol <saccharide Netssetia meningitidis Cerebrospmai flUid AdrenalJnsuft•c1ency Gr..,._.negabve bacteria Seps•s
44 Fungus Men•no•bs Petech•a Neutrophil Purpura Protein Antibiotics Netssetia otplococcus Glucose Bactena Rash
45 Image copyright © 2009 Kerneis
et at.; licensee Cases Network
46
Ltd.
47
48 A is not correct. 6 °/o chose this.
49 Ascending peripheral nerve paralysis describes Guillain-Barre syndrome, which could result after infection wit h Campylobacter jejuni. Guillain -
50
Barre is believed to be an autoimmune phenomenon in which peripheral nerves are demyelinated, result ing in an ascending paralysis. The
most dangerous complicat ion is respirat ory failure if t he diaphragm is involved .
51 Campylobacter jejun1 Thorac1c d1aphragm Penpheral nervous system Paralysis Respiratory fa1lure Auto1mmune d1sease Demyelinating d1sease Autoimmunity Nerve Myelin Campylobacter Infection

52
B is n ot correct. 10°/o chose this .
. 53
Paroxysmal hypertension, palpitations, anxiety, and drenching sweats describe th e sig ns and symptoms com monly associated wit h
. 54 pheochromocytoma, a tumor of the adrenal medulla t hat leads to the overproduction of ca techolam ines. Pheochromocytomas are not
• 55 associated with Neisseria meningitidis infect ion .
Pheochromocytoma Adrenal medulla Hypertension Palpitations Catecholamine Hypertensive emergency Neoplasm Anx1ety Medulla oblongata Infection
' 56
' 57 C is not correct. 7°/o chose this.
' 58 Right-sided lower facial droop describes an upper motor neuron deficit that can be found when a pat ient develops a st roke.
Neuron Upp~ moto1 neu on Motor neuron Stroke
. 59

' 60 E is not correct. 9 °/o chose t his.

8
Lode.
s
S uspe-nd
8
End Bloc:k
Item: S2 of ~ ,• Mark <::J [:::> ""I ~· ~'j
100 J.. Previous Next LAbfaiUI~S Notes Calculator

34
E is not correct. 9 °/o chos e this .
35
Sudden onset of feve r, anemia, and a new heart murm ur describe acut e bact eri al endocardit is, which is most often caused by Staphylococcus
36 aureus. other signs and sympt oms of bacterial endocarditis that are oft en t ested include Roth spots, Osier nodes, and splinter hemorrhages.
37 Roth's spot Osler's node Heart murmur Endocarditis Anemia Staphylococcus Infective endocarditis Bleeding Fever

38
39 Bottom Line :
40
Neisseria meningitidis is a gram-negat ive diplococcus t hat is a common cause of bacterial meningit is; it is often associated wit h a petechial
41 rash in children and young adults. I n severe cases it can cause Waterhouse -Friderichsen syndrome, a condit ion charact eri zed by sepsis and
42 adrenal insufficiency.
Waterhouse-Friderichsen syndrome Neisseria meningitidis Adrenal insufficiency Gram-negative bacteria Meningitis Sepsis Petechia Diplococcus Neisseria Purpura Bacterial meningitis Rash
43
44
45
l@ljl'il·1i•J for yea r:[2017 • J
46 FIRST AID FA CTS

47
48 FA17 p 176.1
49 Common causes of meningitis
50 NEWBORN (0- 6MOl CHILDREN (6 M0- 6 YR) 6- 60YR 60YR+
51 Group B streptococci S pneumoniae S pneumoniae S pnewnoniae
52
E coli N meningitidis N meningiticlis (#1 in teens) Gram 8 rods
. 53
Listeria H infiuenzae type B Enterovi ruses Listeria
. 54
Enteroviruses HSV
• 55
• 56 Give ceftriaxone and vancomycin empirically (add ampicillin if Listeria is suspected).
• 57 Viral causes of meningitis: enteroviruses (especially coxsackievirus), llSV-2 {llSV-1 = encephalitis), l iiV, West ile virus (also
• 58 causes encephalitis), VZV.
. 59 In HIV: Cryptococcus spp.
• 60 • ote: Incidence of H influenzae meningitis has l greatly due to conjugate H influenzae vaccinations. Today, cases are usually
8
Lock
s
Suspend
0
End Block
Item: S2 of - ,• Mark -<J [:::> "'I ~ · ~
100 ~ P~v1ous N @xt Labl lues N o tes Calculato r
34 A A
FA17 p 138.2
35
36 Neisseria Gram 8 diplococci. Metabolize glucose \ leninGococci ferment :\ laltose and Glucose.
37 and produce lgA proteases. Contain Gonococci ferment Glucose.
38 Iipool igosaccharides (LOS) with strong
39 endotoxin acti\'ity. gonorrhoeae is oft en
40 intracellular (within neutrophils)
41
42 Gonococci Meningococci
43 o polysaccharide capsule Pol} saccharide capsule
44
1 o maltose metabolized Ylaltose fermentation
45
46 1 o vaccine due to antigenic \'ariation of pilus Vaccine (type B vaccine not widely a\'ailablc)
47 proteins
48 Sexually or peri natally transmitted Transmitted via respiratory and oral secretions
49
50
Causes gonorrhea, septic arthritis, neonatal Causes meningococcemia with petechial
51
conjuncti,·itis (2-5 days after birth), pelvic hemorrhages and gangrene of toes rn.
52
inflammatory disease (PID), and f'itz- llugh- meni ngitis, Waterhouse-Friderichscn
. 53 Curtis syndrome syndrome (adrenal insufficiency, fever, DIC,
. 54 shock)
• 55 Condoms l sexual transm ission, erythromycin Rifampin, ciproAoxacin, or ceftriaxone
' 56 eye ointment prevents neonatal blindness prophylaxis in close contacts
' 57
Treatment: ceftriaxone + (azithromycin Treatment: ceftria.xone or penicillin C
' 58
. 59
or doxycycline) for possible ch lamydia!
' 60
coinfection
8
Lode.
s
S uspe-nd
8
End Bloc:k
Item: S2 of - ,• Mark -<J [:::> "'I ~ · ~
100 ~ P~v1ous N@xt Labl lues No tes Calculato r

34 A A
FA17 p 324.1
35
36 Adrenal insufficiency Inability of adrenal glands to generate enough Diagnosis im·olves measurement of serum
37 glucocorticoids +/- mineralocorticoids for the electrolytes, morning/random serum cortisol
38 body's needs. Symptoms include weakness, and ACTH (low cortisol, high ACTH in 1°
39 fatigue, orthostatic hypotension, muscle adrenal insufficiency; low cortisol, lo" ACTI I
40 aches, weight loss, Cl disturbances, sugar and/ in 2°/3° adrenal insufficiency due to pituitary/
41 or salt cra\'ings. Treatment: glucocorticoid/ hypothalamic disease), and response to CTII
42 mineralocorticoid replacement. stimulation test.
43 Alternative!~. can use mel) rapone stimulation
44
test: metyrapone blocks last step of cortisol
45
synthesis ( 11 -deoxycort isol ..... cor! isol). 1 orma I
46
response is l cortisol and compensatory
47
t ACTII and 11-deoxycortisol. In 1° adrenal
48
insufficiency, ACTH is f but 11-cleoxycortisol
49
remains l after test. In 2°/3° adrenal
50
51
insufficiency, both ACTH and 11-deoxycortisol
52
remain l after test.
. 53 Primary adrenal Deficiency of aldosterone and cortisol Primary Pigments the skin/mucosa.
. 54 insufficiency product ion due to loss of gland f1m ct ion Associated with autoimmu ne polyglandular
• 55 ..... hypotension (hyponatremic volume syndromes.
' 56 contraction), hyperkalemia, metabolic acidosis, Waterhouse-Friderichsen syndrome-acute
' 57 skin and mucosal hyperpigmcntation rJ (due , due to adrenal
1° adrenal insufficienc\·
' 58
to t MSII, a byproduct of ACTII production hemorrhage as~ociated with septicemia
. 59
from proopiomelanocortin [POMC)). (usually Neisseria meningitidis), DIC,
' 60 - ... ___ ..._ _ _ _ _ _t_] __ -- '- I - '- - I
- . .1 - "- - -·= - - L -- L

8
Lode.
s
S uspe-nd
8
End Bloc:k
Item: 52 of - ,• Mark -<] 1:> ""'I ~· 1!';:'1
100 ~ Prev1o u s Next Labf a lu es Note s Calculator
34
35
Primary adrenal Deficiency of aldosterone and cortisol Primary Pigments the skin/mucosa.
36 insufficiency production due to loss of gland runction Assoc i<~ted with autoimmune polyglandul<1r
37 - hypotension (hyponatremic volume syndromes.
38 contraction), hyperkalemia, metabolic acidosis, Waterhouse-Friderichsen syndrome- acute
39 skin and mucosal hyperpigmentation (due 1° adrenal insufficienc,·

due to adrenal
40 to f MSII, a byproduct of CT II production hemorrhage associated with septicemia
41 from proopiomelanocortin [ PO~ lC)). (usually Neisseria meningitidis), DIC.
42 Acute-sudden omet (cg, due to massi' c cndotoxic shock.
43 hemorrhage). ~far present with shock in
44 acute adrenal crisis.
45
Chronic-aka Addison disease. Due to
46
adrenal atrophy or destruction b) disease
47
(autoimmune destruction most common in
48
the Western \\ Oriel; TB most common in the
49
developing " ·oriel).
50
51 Secondary adrenal Seen with ! pituitary ACTII production. Secondary Spares the skin/mucosa.
52 insufficiency o skin/mucosal hyperpigmentation, no
. 53 hyperkalemia (aldosterone synthesis preserved
. 54 due to intact renin-angiotensin-aldosterone
• 55 axis).
• 56
Tertiary adrenal Seen in patients with chronic exogenous Tertiary from Treatment.
. 57
insufficiency steroid use, precipitated by abrupt withdrawal.
• 58
Aldosterone synthesis unaffected .
. 59
• 60 •
8
L.odt
s
Su~pl'nd
~
End Block
Item: 53 of - ,• Mark -<J [:::> "'I ~ · ~
100 ~ P~v1ous N @xt Labl lues N o tes Calculato r
34
A 33-year-old woman was diagnosed wit h Pneumocystis pneumonia. In addit ion to drugs to t reat her pneu monia, she began taking ~~AI
35
zidovudine, lamivudine, and rit onavir. Alt hough she init ially responds to treatment, 1 year lat er, her physician considers adding enfuvirtide
36 to her regimen.
37
38 What is the mechanism of action of enfuvirtide?
39
:
40 A. Blocking incorporation of the viral genome into the host cell genome
41
B. Blocking translation of the viral messenger RNAs
42
43 C. Interfering with posttranslational processing of viral proteins
44 0 . Preventing entry of viral particles into the target host cell
45
E. Preventing synthesis of DNA from viral RNA
46
47
48
49
50
51
52
. 53
. 54
• 55
' 56
' 57
' 58
. 59
' 60 •
8
Lode.
s
S uspe-nd
8
End Bloc:k
Item: S3 of - ,• Mark -<J [:::> "'I ~ · ~
100 ~ P~v1ous N@xt Labl lues No tes Calculato r

34 A
The correct answ er is D. 60°/o chose this. A

35
Pneumocystis pneumonia is an AIDS -defin ing illness, and the patient is being treated with highly act ive antiretrovi ral th erapy (HAART). As
36 shown in the diagram, HIV enters the host cell by binding its viral envelope glycoprotein (gp120) to t he host cell's CD4 glycoprotein. This
37 interaction induces a conformational change in gp120. The altered gp120 then binds the host cell's coreceptor, CXCR4 (on T-lymphocytes) or
CCRS (on T-lymphocytes and macrophages). Another viral envelope glycoprotein (gp41) is brought closer to the host cell, and it then
38
mediates the fus ion of the viral envelope with t he host cell's lipid membrane. Entry inhibitors may act either by blocking the interaction
39 between gp120 and CD4 or by interfering wit h gp41-mediated fusion. Enfuvirtide is an example of t he latter.
CD4 M,..,l'g..,..., r l"il 'AIDS CCRS Gp41 EnfuwtJde Envelope glycoprote•n GP120 l..ral ..,.,elope Glycoprot n E11b h•b•tor CXCR4 T cell Macrophage HIV Pneumon•a l.Jpod Conformatlonal change
40
Co·receptor ~ral entry LJp•d bdaye- Pneumocystis pneumonia Virus
41
42
43
44

45
46
47
48
Holt Cell
49
50
'lh'tl ONA It
51 lrii~IQO.. Iht
~· 11'<1 it'!!t(rltol
Into lht hOII DNA
52
53
. 54
• 55 Matu~ Virion
'
-- ~w•I RHA

(t
' 56
' 57
' 58
. 59
' 60 •
8
Lode.
s
S uspe-nd
8
End Bloc:k
Item : S3 o f - ,• Ma rk -<J [:::> "'I ~ · ~
100 ~ P~v1ous N@xt Labl lues Notes Calcu lator
-- - - ----
34 A A

35
36
37
38
39
40
41
HostC.II
42
43
44

45
I
, --·
,....ltld.,,.._
•-.ocwtld ..... .,.

-. ... no .. o""'
...
46
I
47
'
48 Mature V1rion
---H.w w .. RNA

:e:
49
50
51
52 Tht 'lln1s m-,tur~>\
by ptote11te
r~lng lndMd~l
53 HIV p t Q1(1ns.

. 54
• 55
' 56 Image courtesy of NIAID
' 57
A i s not correct. 1 5°/o chose thi s.
' 58
Integ rase is a viral enzyme that integrates viral DNA into the host's own genome. This allows for the transcription and t ranslation of the viral
. 59
genetic information. Integrase inhibitors are a newer class of antiretroviral drug that inhibit the act ion of integrase. An example of this is
' 60 • raltearavir.
8
Lode.
s
Suspe-nd
8
End Bloc:k
Item: S3 of ~ ,• Mark <::J [:::> ""I ~· ~'j
100 J.. Previous Next LAb faiUI~S Notes Calculator

34
B is not correct. 8% chose t his .
35
An add it ional t arget current ly under invest igat ion as an HIV t reatment is t he viral prot ein tat (encoded by t he tat gene) . One feat ure of the
36 HIV life cycle not mentioned above is that when t he viral DNA is init ially transcri bed, t he mRNA t ranscript undergoes interna l nucleotide base-
37 pairi ng. This hairpi n structure prevent s ful l t ranscription of the viral DNA. The protein tat is one of t he few mRNA products that st ill get
38
properly t ranscri bed/translat ed, and it t hus functions to destabilize t he mRNA hairpin, allowing ful l t ranscription . Overall, it is a posit ive-
feedback mechanism for HIV act ivation. A drug t hat could block t he f unction of tat would prevent synthesis of t he majority of viral proteins.
39 Gene HIV Protein Nucleotide Messenger RNA DNA Transcription (genetics) Base pair Stem- loop Viral protein Virus Biological life cycle Small hairpin RNA
40
C is not co rrect. 10% chose this .
41
This is t he mechanism of act ion of protease inhibit ors such as saquinavir. Protease is an HIV-encoded enzyme responsible for cleaving the
42 init ial pol and gag polypeptides int o t heir final product s. (Not e: The env gene product also undergoes post t ranslat ional cl eavage. A host -
43 encoded enzyme med iat es this cleavage, howeve r~ not HIV prot ease.)
Saquinavir Enzyme Gene Protease Protease inhibitor (pharmacology) HIV Peptide Protease inhibitor (biology) HIV- 1 protease Mechanism of action
44
45 E is not correct. 7°/o chose this .
46 Because t he original genet ic information from t he HIV virus is in t he form of RNA, reverse t ranscriptase is needed to convert the sing le-
stranded RNA into dou ble-stranded DNA. There are two classes of reverse transcri ptase inhibi tors. Nucleoside analogue reverse t ranscriptase
47
inhibitors, such as zidovudine, are incorporated int o t he DNA strand and prevent elongat ion . On the ot her hand, nonnucleoside reve rse
48 t ra nscript ase inhibit ors, such as nevirapine, bind t he enzyme directly to block the catalytic site.
49 Zidovudine Nevirapine Reverse transcriptase Reverse- transcriptase inhibitor Enzyme HIV Nucleoside analogue Virus RNA Nucleoside DNA Active site

50
51
Bottom Line :
52
Highly active antiretrovira l t hera py (HAART) targets HI V through m ult iple mechanisms. Two common entry inhibit ors are enf uvirt ide and
53 maraviroc. Ma ravi roc binds CCR-5 on T cells and monocytes, blocking its interaction wit h gp120 . Enf uvirtide binds vira l gp41 and inhibits
. 54 gp41-mediated fusion . Other drugs t hat are used include integrase inh ibitors, nucleoside analog reverse t ranscript ase inhibit ors,
nonnucleoside reve rse t ranscriptase inh ibitors, protease inhibit ors, and inhibit ors of messenger RNA t ra nslation .
• 55
Messenger RNA Enfuvirtide Management of HIV/AIDS Reverse transcriptase Gp41 Envelope glycoprotein GP120 Jntegrase Reverse- transcriptase inhibitor Maraviroc Entry inhibitor Nucleoside analogue
• 56 Protease inhibitor (pharmacology) HIV Integrase inhibitor Nucleoside T cell Protease Monocyte RNA Translation (biology) Virus Protease inhibitor (biology)
• 57
• 58
. 59 l@);fil ~1hl for yea r:l 2o17 y
FIRST AID FACTS
• 60

8
Lock
s
Suspend
0
End Block
Item: S3 of - ,• Mark -<J [:::> "'I ~ · ~
100 ~ P~v1ous N @xt Labl lues N o tes Calculato r
34 •
35 l@l:fil,11•1 for year: 2017 ...
FIRST AID FACTS
36
37 FA17 p 171 .1
38
HIV
39
40 Envelope proteins Diploid genome (2 molecules orR ' ).
iiCQUred through budd109 Irom
41
hos1 cell plasma membrane The 3 structural genes (protein coded for):
42 p17: Matnll prolein env (gpl20 and gp41):
43 gpl20 Formed from cleavage of gpl60 to form
44 Docking I.Jpod e~Miopt emelope glycoproteins.
glycoprotM ...........
45 gpl 20-attachment to host CD4+ T cell.
gp41
46 Transmembrane p24 gp41 - fu sion and entry.
47
glycoprotein / CipSid protein
gag (p24 and p17) -capsid and matrix
48 Reverse proteins, respectively.
49 transcriptast
pol- reverse transcriptase, aspartate protease,
50
integrase.
51
Reverse transcriptase synthesizes dsD A rrom
52
53
genomic R 'A; dsD A integrates into host
. 54 genome .
• 55 Virus binds CD4 as well as a coreeeptor, either
' 56 CCR5 on macrophages (early infection) or
' 57 CXCR4 on T cells (late infection).
' 58 llomozygous CCR5 mutation = immunity.
. 59 Heterozygous CCR5 mutation = slower course.
' 60

8
Lode.
s
S uspe-nd
8
End Bloc:k
Item: 53 of - ,• Mark -<] 1:> ""'I ~· 1!';:'1
100 ~ Prev1o u s Next Labf a lu es Note s Calculator
34
FA17 p 172.1
35
36 Time course of untreated HIV infection
37 Window period +I- Acute HIV infection Skm and mucous Systemic immuno-
38 Viral dissemination membrane deficiency/AIDS·
Seeding of lymphoid organs Clinical latency infections defining ~lnesses
39
40
41
1200 Anti·envdope antibody (gpl20) t lO'
42
43 uoo
44

45
1000
46
900
47 ~

E
48 E
.._ 800 :I:
49 .!:!!
Q)
<
50
u
+
00
700
~
n
51 0 0
u.... 600 '0
0 ~·
52 .._
+ 3
53
v 500 ,-
0
u
. 54
HIVRNA
400 10'
• 55
• 56 300
. 57
• 58 200
. 59
100
• 60 o.:-·-· •
8
L.odt
s
Su~pl'nd
~
End Block
Item: S3 of ~ ,• Mark <::J [:::> ""I ~· ~'j
100 J.. Previous Next LAb faiUI~S Notes Calculator

34 • •
FA17 p 199.1
35
36
HIVtherapy Highly active antiretroviral therapy (llAART): often in itiated at the time of HIV diagnosis.
37 Sl·rongesl indica! ion for patients presenting with AIDS-defi ning ill ness, low C D4+ cell counts
38 (< 500 cells/mm3), or high viral load. Regimen consists of 3 drugs to prevent resistance:
39 2 NRT is and preferably an intcgrase inh ibitor.
40
DRUG MECHANISM TOXICITY
41
NRTis
42
43 Abacavir (ABC) Competiti,·ely inhibit nucleotide binding to Bone marrow suppression (can be re,·ersed with
44 Didanosine (ddl) reverse transcriptase and terminate the D1 A granulocyte colony-stimulating factor [G-CSF]
45 Emtricitabine (FTC) chain (lack a 3' 0 1-1 group). Tenofo,·ir is a and erythropoietin), peripheral neuropathy,
46 Lamivudine (3TC) nucleoT ide; the others are nucleosides. All lactic acidosis (nucleosides), anemia (ZD ),
47 Stavudine (d4T) need to be phosphorylated to be active. pancreatitis (d idanosine).
48 Tenofovir (TDF) ZDV can be used for general prophylaxis Abacavir contraindicated if patient has
49
Zidovudine (ZDV, and duri ng pregnancy to l risk of feta l HLA-8"' 5701 mutation due to t risk of
50
formerly AZT) transmission. hypersensitivity.
51
Have you dined (vudine) with my nuclear
52
(nucleosides) family?
53
. 54 NNRTis
• 55 Delavirdine Bind to reverse transcriptase at site different Rash and hepatotoxicity are common to all
• 56 Efavirenz from NRT is. Do not require phosphorylation l ' RT \s. Vivid dreams and C1 S symptoms
• 57 Nevirapine to be active or compete with nucleotides. are common with efaviren z. Delavirdi ne and
• 58
efavirenz are contraindicated in pregnancy.
. 59
• 60
Protease inhibitors •

8
Lock
s
Suspend
0
End Block
Item: S3 of ~ ,• Mark <::J [:::> ""I ~· ~'j
100 J.. Previous Next LAbfaiUI~S Notes Calculator

34 • •
NNRTis
35
36 Delavirdine Bind to reverse transcriptase at site different Rash and hepatotoxicity are common to all
37 Efavirenz from RTis. Do not requ ire phosphorylation ' RT ls. Vivid dreams and C r S symptoms
38 Nevirapine to be active or compete with nucleotides. are common with efavirenz. Delavirdi ne and
39 efavirenz are contraindicated in pregnancy.
40
Protease inhibitors
41
42 Atazanavir Assembly of virions depends on H IV-I prot·easc Hyperglycemia, Cl intolerance (nausea,
43 Darunavir (pol gene), which cleaves the polypeptide diarrhea), lipodystrophy (Cushing-like
44 Fosamprenavir products of HIV mR 'A into their fun ctional syndrome).
45 lndinavir parts. Thus, protease inh ibitors prevent ephropathy, hematuria, thrombocytopenia
46 Lopinavir maturation of new viruses. (indinavir).
47 Ritonavir Ritonavir can "boost" other drug concentrations Rifampin (potent CYP/ UCT inducer) reduces
48 Saquinavir by inh ibiting cytochrome P-450. protease inhibitor concentrations; usc rifabutin
49 avir (never) tease a protease. instead.
50
lntegrase inhibitors
51
52 Raltegravir Inh ibits HIV genome integration into host cell t creatine kinase.
53 Elvitegravir chromosome by reversibly inhibiting HJV
. 54 Dolutegravir integrase.
• 55
Fusion inhibitors
• 56
• 57
Enfuvirtide Binds gp41 , inhibiting viral entry. Skin reaction at injection sites.
• 58
Enfuvirtide inhibits fusion .
. 59 Maraviroc Binds CCR-5 on surface ofT cclls/monocytes, Maraviroc inhibits clocking.
• 60 • inhibiti nv interaction with vnl20. •

8
Lock
s
Suspend
0
End Block
Item: S4 of - ,• Mark -<J [:::> "'I ~ · ~
100 ~ P~v1ous N @xt Labl lues N o tes Calculato r
34
A 43-year-old woman with t wo grown childre n presents wit h a 1- to 2-week history of fat igue, cold intolerance, const ipation, and neck ~~AI
35
pain. About 1 month ago she had a short, flu -like syndrome f rom which she completely recovered. On examinat ion her skin is cool, her
36 deep tendon reflexes show delayed relaxat ion phases, and her t hyroid gland is extremely tender t o light palpat ion with no masses.
37 Laboratory studies are significant fo r an elevat ed thyroid-stimulating hormone level and erythrocyt e sedimenta tion rat e.
38
39 Which of the following is the most likely diagnosis?
40 :
41 A. De Quervain thyroiditis
42 B. Graves disease
43
C. Hashimoto thyroiditis
44

45 D. Postpartum thyroiditis
46 E. Vitamin 8 12 deficiency
47
48
49
50
51
52
53
. 54
• 55
' 56
' 57
' 58
. 59
' 60 •
8
Lode.
s
S uspe-nd
8
End Bloc:k
Item: S4 of ~ ,• Mark <:::1 t::> ""I ~· ~'j
100 J.. Previous Next LAbfaiUI~S Notes Calculator

34
35
The correct answer is A. 69°/o chose this.
36 De Quervain thyroiditis is a common cause of t ransient hypothyroidism seen following viral illnesses. A hypert hyroid stage caused by
37 unreg ulated T4 and T 3 re lease generally precedes t he t ransie nt hypothyroid st age. Also known as subacute granulomat ous thyroidit is, t his
38
condit ion is characterized by an exquisitely t ender t hyroid gland, elevated erythrocyt e sed imentat ion rat e, and ot her manifestat ions of
hypothyroidism (eg, fat igue, cold intolerance, cool skin, decreased deep t endon ref lexes) seen here. No t reat ment is necessary because
39 patients quickly ret urn to a eut hyroid st at e. The cause of t his disease is unknown, with numerous viral agents such as m umps, Epst ein -Barr
40 virus, coxsackie virus, f lu, and adenovirus being implicat ed.
De Quervain's thyroiditis Hypothyroidism Epstein-Barr virus Erythrocyte sedimentation rate Mumps Adenoviridae Red blood cell Thyroid Coxsackievirus Hyperthyroidism Thyroiditis Granuloma
41
Fatigue (medical) Virus
42
43 B is not correct. 6% chose this.
44 Graves disease is an aut oimmune form of hyperthyroid ism caused by production of aut oantibodies that act ivate the thyroid-stim ulating
45
hormone receptor. Hyperthyroid ism is charact eri zed by heat intolerance, weight loss, palpitations, exopht halmos, and increased deep tendon
reflexes.
46 Graves' disease Thyroid-stimulating hormone Exophthalmos Hyperthyroidism Heat intolerance Palpitations Autoimmunity Weight loss Autoimmune disease Hormone Autoantibody Tendon
47
C is not correct. 20% chos e this.
48
Hashimoto thyroidit is is another cause of hypothyro idism; however, this autoimmune condit ion t ypically has a slow course and ma nifests wit h
49 a nontender t hyro id. Ant im icrosomal and ant it hyroglobulin antibodies are usually seen, and t reat ment is t hyroid hormone replacement.
50 Hashimoto's thyroiditis Hypothyroidism Thyroid hormone Thyroid Antibody Autoimmune disease Thyroiditis Hormone Autoimmunity

51 D is not correct. 3°/o chose this .


52 Postpartum t hyroidit is typically occurs < 1 year after pregnancy and is associated wit h painless hypothyro idism . Like de Quervain thyroidit is,
53 post partum thyroidit is is also usually self- limit ing, and no t reat ment is req uired .
De Quervain's thyroiditis Postpartum thyroiditis Hypothyroidism Thyroiditis Postpartum period Pregnancy
54
• 55 E is not correct. 2 °/o chos e this .
• 56 Vitamin B12 deficiency causes a macrocytic anemia and, event ually, a peri pheral neuropathy associated wit h decreased deep t endon reflexes .
However, vit amin B 12 def iciency does not cause elevat ed t hyroid-st imulat ing hormone levels or t hyroid gland tenderness .
• 57
Thyroid-stimulating hormone Peripheral neuropathy 8 vitamins Anemia Thyroid Vitamin 812 Vitamin Macrocytic anemia Hormone Gland
• 58
. 59
• 60
Bottom Line:

8
Lock
s
Suspend
0
End Block
Item: S4 of ~ ,• Mark <:::1 t::> ""I ~· ~'j
100 J.. Previous Next LAb faiUI~S Notes Calculator

34
35 Bottom Line:
36 De Quervain thyroiditis is a transient hypothyroidism seen after viral illnesses. It is charact eri zed by an exquisit ely tender thyroid gland,
37 elevated erythrocyt e sed iment at ion rat e, and ot her man ifest at ions of hypot hyroidism.
De Quervain's thyroiditis Hypothyroidism Erythrocyte sedimentation rate Red blood cell Thyroid Thyroiditis Gland Virus
38
39
40
41
i@l;fil·1i•J f or yea r:[ 2017 • J
FIRST AID FACTS

42
43 FA17 p 328.1
44 Hypothyroidism
45
Hashimoto thyroiditis Most common cause of hypothyroidism in iodine-sufficient regions; an autoi mmune disorder with
46
antithyroid peroxidase (antimicrosomal) and antithyroglobulin antibodies. Associated with t risk
47
of non-Hodgkin lymphoma (typically of B-cell origin).
48
49
May be hyperthyroid early in course due to thyrotoxicosis during folli cular rupture.
50
m.
Histologic findings: Hi.irthle cells, lymphoid aggregates with germ inal centers
51
Findings: moderately enlarged, nontender thyroid.
52 Congenital Severe fetal hypothyroidism due to maternal hypothyroidism, thyroid agenesis, thyroid dysgenesis
53 hypothyroidism (most common cause in US), iodine deficiency, dyshormonogenetic goiter.
54 (cret inism) Findings: Pot-bellied, Pale, Puffy-faced child with P rotruding umbilicus, P rotuberant tongue, and
• 55 Poor brain development: the 6 P's llJ [i.
• 56
Subacute Self-limited disease often following a Au-like illness (eg, viral infection).
• 57
• 58
granulomatous May be hyperthyroid early in course, follo\\"ed by hypothyroidism.
. 59
thyroiditis (de Histology: granulomatous inflammation.
• 60
Quervain) Findings: t ESR, jaw pain, very tender thyroid. (de Quervain is associated with pain.)
8
Lock
s
Suspend
0
End Block
Item: S4 of - ,• Mark -<] 1:> ""'I ~· 1!';:'1
100 ~ Prev1o u s Next Labf a lu es Notes Calculator
34
35
Riedel thyroiditis Thyroid replaced by fibrous tissue with inAammatory infiltrate 1!]. Fibrosis may extend to local
36
structures (eg, trachea, esophagus), mimic~ing anaplastic carcinoma. Vl are hypothyroid.
37 Considered a manifestation of lgG4 -related systemic disease (eg, autoimmune pancreatitis,
38 retroperitoneal fibrosis, noninfectious aortitis).
39 Findings: fixed, hard (rock-like). painless goiter.
40 Other causes Iodine deficiency 1], goitrogens (eg, amiodarone, lithium), Wolff-Chaikoff effect (thyroid gland
41 downreguJation in response to t iodide).
42
43
44

45
46
47
48
49
50
51
52
53
54
• 55
• 56
. 57
• 58
. 59
• 60 •
8
L.odt
s
Su~pl'nd
~
End Block
Item: S4 of - ,• Mark -<] 1:> ""'I ~· 1!';:'1
100 ~ Prev1o u s Next Labf a lu es Notes Calculator
34
FA17p329.1
35
Hyperthyroidism
36
37 Graves disease Most common cause of hyperthyroidism. Thyroid-stimulating immunoglobulin (lgC; type II
38 hypersensitivity) stimulates TSll receptors on thyroid (hyperthyroidism, diffuse goiter) and
39 dermal fibroblasts (pretibial myxedema). Infiltration of rctroorbital space by activated T-cclls
40 -+ f cytokines (eg, T NF-a, IF '-y) - f fibroblast secretion of hydrophilic CACs - f osmotic
41 muscle swelling, muscle inAammation, and ad ipocyte count - exophthalmos fJ. Often presents
42 during stress (eg, pregnancy). Associated "ith H LA-DR3 and HLA-B8.
43 Tall, crowded follicular epithelial cells; scalloped colloid
44

45
Toxic multinodular Focal patches of hrperfunctioning follicular cells distended with colloid 9 working independently
46
goiter ofTS II (due to TSII receptor mutations in 60% of cases). f release ofT3 and T~. I lot nodules arc
47
rarely malignant.
48 Thyroid storm Uncommon but serious complication that occurs when hyperthr roidism is incompletely treated/
49 untreated and then significantly worsens in the setting of acute stress such as infection, trauma,
50 surgery. Presents with agit·ation, delirium, fever, diarrhea, coma, and tachyarrhythm ia (cause
51 of death). May see t LFTs. Treat with the 4 P's: ~ -b l ocke rs (eg, P ropranolol), Propylthiouracil,
52 corticosteroids (cg, Prednisolone), Pota s~ium iodide (Lugol iodine).
53
54
Jod-Basedow Thyrotoxicosis if a patient with iodine defi ciency and partially autonomous thyroid tissue (eg,
• 55
phenomenon autonomous nodule) is made iodine replete. Opposite ofWolff-Chaikoff effect.
• 56
. 57
• 58
. 59
• 60 •
8
L.odt
s
Su~pl'nd
~
End Block
Item: S4 of - ,• Mark -<] 1:> ""'I ~· 1!';:'1
100 ~ Prev1o u s Next Lab falu es Notes Calculator
34
35 FA17 p 398.1
36 Macrocytic (MCV > 100 fl) anemia
37
DESCRIPTIOH FIHDINGS
38
Megaloblastic anemia Impaired D. A synthesis ..... maturation of RBC macrocytosis, hypersegmented
39
40
nucleus of precursor cells in bone marro\\ neutrophils fJ, glossitis.
41
delayed relati\'e to maturation of C) toplasm.
42
43
44

45
46 t
47
a
48 Folate deficiency Causes: malnutrition (eg, alcoholics), t homocysteine, normal methylmalonic acid.
49 malabsorption, drugs (eg, methotrexate, No neurologic symptoms (vs B12 deficiency).
50 trimethoprim, phenytoin), t requirement (eg,
51 hemolytic anemia, pregnancy).
52
Vitamin 812 Causes: insufficient intake (cg, vegan ism), t homocysteine, t methylmalonic acid.
53
54
(cobalamin) malabsorption (cg, Crohn disease), pernicious Neurologic symptoms: reversible dementia,
• 55
deficiency anemia, Diphyllobothrium tatum (fish subacute combined degeneration (due to
• 56
tapeworm), gastrectomy. involvement of B12 in fatty acid pathways and
. 57 myelin synthesis): spinocerebellar tract, lateral
• 58 corticospinal tract, dorsal column dysfun ction.
. 59 llistorically diagnosed with the Schilling test,
• 60 a 4-stagc test that determines if the cause is •
8
L.odt
s
Su~pl'nd
~
End Block
Item: S4 of - ,• Mark -<] 1:> ""'I ~· 1!';:'1
100 ~ Prev1o u s Next Labf a lu es Notes Calculator
34
myelin synthesis): spinocerebellar tract, lateral
35
corticospinal tract, dorsal column drsfunction.
36
Historically diagnosed with the Schilling test,
37
a 4-stagc test that determines if the cause is
38
39
dietary insufficiency vs malabsorption.
40
nemia 2° to insufficient intake llla}' take Se\entl
41
years to develop due to liver's ability to store B12
42 (as opposed to folate deficiencr).
43 Orotic aciduria Inability to com·ert orotic acid to U \I P Orotic acid in urine.
44 (de no,·o pyrimidine synthesis pathway) Treatment: uridine monophosphate to bypass
45 because of defect in UMP smthase.

mutated enzyme.
46
Autosomal recessive. Presents in children as
47
failure to thri,·e, developmental delay, and
48
megaloblastic anemia refractory to folate
49
and B12 . No hyperammonemia (vs orn ithine
50
transcarbamylase deficienC)'- t orotic acid
51
with hyperammonemia).
52
53 Nonmegaloblastic Macrocytic anemia in which D A synthesis is RBC macrocytosis without hypersegmcnt ed
54 anemia unimpaired. ncutrophils.
• 55 Causes: alcoholism, li,·er disease.
• 56
Oiamond-Biackfan Rapid-onset anemia within 1st year of life due to t % llbF (but l total Hb).
. 57
anemia intrinsic defect in erythroid progenitor cells. Short stature, craniofacial abnormalities, and
• 58
upper extremity malformations (triphalangcal
. 59
• 60
thumbs) in up to 50% of cases. •
8
L.odt
s
Su~pl'nd
~
End Block
Item: SS of - ,• Mark -<J [:::> "'I ~ · ~
100 ~ P~v1ous N @xt Labl lues N o tes Calcula to r
34
A 45-year-old woman present s to t he emergency depart ment because of dim inished pulses, sunken eyes, and poor skin turgor. Her ~~AI
35
husband says they returned f rom Bangladesh 3 days ago and that she has been having severe wat ery diarrhea since that t ime. Microscopic
36 examination demonstrates motile, comm a-shaped organisms t hat do not ferment lactose and are oxidase positive.
37
38 Which of the following describes the toxin that is causing this patient's diarrhea?
39
:
40 A. A heat-stable toxin that activates guanylyl cyclase to increase cGMP
41
B. A toxin that activates adenyl cyclase by ADP ribosylation to increase cAMP
42
43 C. A toxin that disables G; to increase cAMP
44 0 . An exotoxin that inhibits translation via ADP ribosylation of elongation factor 2
45
E. Edema factor that acts as a bacterial adenyl cyclase to increase cAMP
46
47
48
49
50
51
52
53
54
• 55
• 56
. 57
• 58
. 59
. 60 •
8
Lode.
s
S uspe-nd
8
End Bloc:k
Item: SS of ~ ,• Mark <::J [:::> ""I ~· ~'j
100 J.. Previous Next LAb faiUI~S Notes Calculator

34
The correct a nswer is B. 62% chose t his .
35
The pat ient has sym ptoms indicat ive of severe dehydrat ion. A classic sign of cholera is "rice -wat er" diarrhea, which can lead t o severe
36 dehydration. The Vibrio cholerae toxin ADP-ribosylat es t he G5 -a subu nit. This G5 -a subunit activates adeny ly l cycla se, but cannot hydrolyze
37 t he bound GTP t o GOP. Therefore, t he G5 -a subunit cont inuously activates adeny ly l cycla se, which increases cAMP levels and overactivates
38
protein kinase A (PKA) . PKA phosphorylat es the CFTR chlori de channel, which increases chlori ne excret ion and causes t he subsequent H2 0
eff lux and charact eristic "ri ce-water" diarrhea . The organism is non-lactose ferment ing and is oxidase posit ive.
39 Vibrio cholerae Protein kinase A Chloride channel Adenylyl cyclase Cholera ADP-ribosylation Cyclic adenosine monophosphate Protein Chlorine Cystic fibrosis transmembrane conductance regulator Diarrhea
40 Dehydration Hydrolysis Organism Fermentation Toxin Phosphorylation Efflux (microbiology) Guanosine triphosphate Kinase Excretion
41
A is not correct . 9°/o chose this.
42
Enterot oxigenic Escherichia coli ( ETEC) is a common cause of t raveler's diarrhea and manifests wit h sym pt oms sim ilar t o t hose of cholera;
43 however, E. coli is lact ose ferment ing. I ts heat -stable toxin stimulates guanyly l cycl ase, which in turn act ivat es ent erocyt e cyclic GMP. It also
44 leads t o t he st im ulat ion of chlori de secret ion and inhibit ion of sodium chlori de absorpt ion, result ing in free wat er secretion into t he intest ina l
45 lumen and watery diarrhoea . This heat -st able t oxin is plasm id-regulat ed .
46 ETEC's heat-labile toxin acts by st imulat ing adenylat e cycla se and increasing int racellular cyclic AMP, which results in secretion of chlori de
f rom intest ina l crypt cells and inh ibit ion of absorption of sodium chloride at the villus t ips. This heat -labile toxin is st ructurally similar to
47
cholera toxin.
48
Enterocyte Traveler's diarrhea Sodium chloride Escherichia coli Intestinal gland Cholera Adenylyl cyclase Cyclic adenosine monophosphate Diarrhea Lactose Cholera toxin Human gastrointestinal tract
49
Gastrointestinal tract Sodium Lumen (anatomy) Fermentation Intracellular Enterotoxin Guanylyl cyclase Chloride Toxin Secretion
50
51
C is not correct. 16% chose this .
Pertu ssis toxin also act s as a cAMP inducer bu t does so by irreversibly disabling G;; this toxin is made by Bordet el/a pertussis, which causes
52
whooping cough.
53 Bordetella pertussis Pertussis toxin Pertussis Bordetella Cyclic adenosine monophosphate Cough Toxin
54
D is not correct. 4 °/o chose t his.
55
The Co1ynebacterium diphtheriae exotoxin inh ibits prot ein synthesis by ADP ri bosylation of elongat ion factor 2, leading to inh ibit ion of
• 56 t ranslat ion. This can lead to pseudomem branous pha ryngit is. Toxin production can be det ected by a posit ive Elek t est.
• 57 Corynebacterium diphtheriae Exotoxin Pharyngitis Protein biosynthesis Protein Elongation factor Corynebacterium Toxin EEF2

• 58 E is not correct. 9°/o chose this .


. 59 Edema fa ctor is produced by Bacillus anthracis and is t oxic when present wit h lethal factor. Although B. anthracis can infect all regions of t he
• 60 gast roint estinal t ract and cause diarrhea, t he organism descri bed in t his quest ion is Vibrio cholerae. •

8
Lock
s
Suspend
0
End Block
Item: SS of ~ ,• Mark <::J [:::> ""I ~· ~'j
100 J.. Previous Next faiUI~S
LAb Notes Calculator

34
35
Bottom Line:
36 Vibrio cholerae causes a secret ory diarrhea by irreve rsibl y act ivating G5 •
Vibrio cholerae Diarrhea Vibrio
37
38
39
l@ljl'il·1i•J for year:[ 2017 • J
40 FIRST AID FA CTS

41
42 FA17 p 175.1
43 Bugs causing diarrhea
44 Bloody diarrhea
45
Campylobacter Comma- or S-shaped organisms; growth at 42°C
46
47 E histolytica Protozoan; amebic dysentery; liver abscess
48 Enterohemorrhagic 0157:1-17; can cause llUS; makes Shiga-like toxin
49
Ecoli
50
51
Enteroinvasive Ecoli Invades colonic mucosa
52 Salmonella (non- Lactose 8; Aagellar motility; has animal reservoir, especially poultry and eggs
53 typhoidal)
54
Shigella Lactose 8; very low 10 ;0; produces Shiga toxin (human reservoir only); bacillary dysentery
55
• 56 Yenterocolitica Day care outbreaks, pseudoappendicitis
• 57 Watery diarrhea
• 58
Cdifficile Pseudomembranous colitis; associated with antibiotics and PPis; occasionally bloody diarrhea
. 59
• 60 C perfringens Also causes gas gangrene
8
Lock
s
Suspend
0
End Block
Item: SS of ~ ,• Mark <::J [:::> ""I ~· ~'j
100 J.. Previous Next faiUI~S
LAb Notes Calculator

34 • •
35 FA17 p 128 .1
36 Bugs with exotoxins
37 BACTERIA TOXIN MECHANISM MANIFESTATI ON
38 Inhibit protein synthesis
39
Corynebacterium Diphtheria toxin3 Pharyngitis with pseudomembranes in th roat
40
41
diphtheriae Inactivate elongation facto r and severe lymphadenopathy (bull neck)
42 Pseudomonas Exotoxin N (EF-2) Host cell death
43 aeruginosa
44
Shigella spp. Shiga toxin (ST)3 GI mucosal damage .... dysentery; ST also
45
Inactivate 60S ribosome by enhances cytokinc release, causing hemolytic-
46
removing adenine from uremic syndrome (HUS)
47
Enterohemorrhagic Shiga-like toxin rR A SLT en hances cytokine release, causing HUS
48
49 E coli (EHEC) (SLT)a (prototypically in EHEC serotype 0157:H7).
50 Unlike Shigella, EHEC does not invade host
51 cells
52
Increase flu id secretion
53
54
Enterotoxigenic Heat-labile Overactivates adenylate Watery diarrhea: "labile in the Air (Aclenylate
55
E coli (ETEC) toxin (LT)3 cyclase (t cAMP) .... t CI- cyclase), stable on the Ground (Guanylate
• 56 secretion in gut and H7 0 cyclase)"
• 57 efAux
• 58 I Ieat-stable Overactivates guanylate
. 59 toxin (ST) cyclase (t cGMP)
• 60 • - l resorption of NaCI •

8
Lock
s
Suspend
0
End Block
Item: 55 of - ,• Mark -<] 1:> ""'I ~· 1!';:'1
100 ~ Prev1o u s Next Labf a lu es Note s Calculator
34 • •
35
Bacillus anthracis Edema toxin" Mimics the <1denylate Li kely responsible for characteristic edematous
36
cyclase enzyme (t cAMP) borders of black eschar in cutaneous anthrax
37 Vibrio cholerae C holera toxin 3 Overactivates adenylale Voluminous ··rice-water'' diarrhea
38 cyclase (t cA\IP) by
39 permanently activating C 5
40 ..... t Cl- secretion in gut
41 and H 20 efAux
42
Inhibit phagocytic ability
43
44 Bordetella pertussis Pertussis toxin 3 o,·eracti\<Jtes adenylate Whooping cough -child coughs on expiration
45 c~ clase (t c \1 P) by and "whoops" on inspiration (toxin may
46 disabling Ci, impairing not actually be a cause of cough; can cause
47 phagoc) tosis to permit "I00-day cough" in adults)
48 sun·ival of microbe
49
Inhibit release of neurotransmitter
50
51 Clostridium tetani Telanospasmi n3 Both arc protcascs that Spastic paralysis, risus sardonicus, and "lockjaw";
52 cleave Sr ARE (soluble toxin prevents release of inhibitory (CA BA and
53 1 SF' attachment glycine) neurotransmitters from Renshaw cells
54 protein receptor), a set in spinal cord
55
Clostridium Botulinum toxin 3 of proteins required for fi' laccid paralysis, Aoppy baby; toxin prevents
• 56
botulinum neurotransmitter release release of stimulatory (ACh) signals at
. 57
via vesicular fusion neuromuscular junctions ..... Aaccid paralysis
• 58
. 59
3
An B toxin (aka, two-component toxin [or three for anthrax]) with B enabling binding and triggering uptake (endocytosis)
• 60 of the aeli\e A component. T he A components are usually ADP ribosyhransferases; others have enzymatic activities as listed •
8
L.odt
s
Su~pl'nd
~
End Block
Item: 56 of - ,• Mark -<J [:::> "'I ~ · ~
100 ~ P~v1ous N @xt Labl lues N o tes Calcula to r
6

39 A 6-year-old male is brought to t he emergency department by his parents because of a 4-hour hist ory of abnormal behavior, fever, and
40
weakness. This morning since awakening t he pati ent has been unusually irritable. He also had t rouble rising from his seat at the breakfast
table, resulting in him fal ling t o th e f loor. His parent s report t hat this is the first episode of fall ing, but t hey have not iced t hat the patient
41 "seems to be getting weaker" over t he past couple of weeks. The patient and his fami ly returned from a trip to Pakistan 30 days prior. His
42 parents note that he only had a mild fever approximately 8 days prior. His past medical history is notable fo r vaccine noncompliance. The
43
patient's vital signs are temperature 39°C (102.2°F), heart rate 90/min, and blood pressure 128/93 mm Hg. On neurologic exam, the patient
exhibits 3/5 weakness of left hip flexion and knee extension. Sensation is intact throughout. When the physician passively flexes the patient's
44 right hip to 90 degrees, complete knee extension cannot be attained. Lumbar puncture shows elevated protein and pleocytosis. Nerve
45 conduction study and electromyography reveal decreased action potential amplitude and decreased motor unit recruitment.
46
47 What is the pathophysiology of the patient's disease?
48 :
49 A . Autoimmune central demyelinat ion
50 B. Autoimmune peri pheral demyelinat ion
51
C. Neuron loss in th e anterior horns
52
53 D. Neuron loss in th e lat eral funiculi
54 E. Neuron loss in t he posteri or horns
55
• 56
• 57
• 58
. 59
• 60
. 61
• 62
• 63
. 64

8
Lode.
s
Suspe-nd
8
End Bloc:k
Item: S6 of - ,• Mark -<J [:::> "'I ~ · ~
100 ~ P~v1ous N@xt Labl lu es No tes Calcula t o r

A A

39 The correct a ns w e r is C. 60°/o chose this.


40 r-:---::--~----,;-;-i=Jifi"":!tZ~~;r--::~:o::J~:-:1
Poliovirus initially Infects the Peyer pat ches of t he int est ine and the motor neurons. It l'lA
is passed by the feca l-oral rout e and can manifest with a spectrum of severity. It has
41
three main disease manifestations. (1) The first is manifestation of infection could be
42 a mild illness in which a febri le viral illness develops. This is the most common form
43 and simply resolves. This is quite common in infants in less-developed nations, where
sanitation is poor. (2) The second manifest ation is aseptic meningitis. In this case •
44
fever and meningismus can develop as the virus infects the meninges. Recovery is ,
45 usually complete in 1 week. (3) The third is the most feared presentation (the one
46 described in this vignette) - paralytic poliomyelitis. In these cases, a mild febrile
illness resolves after initial infection with the virus. Subsequently, 5-10 days later the
47
fever recurs, followed by meningismus, and then flaccid asymmetric paralysis. The
©2010 Shan eta/; licensee BioMed
48 flaccid paralysis is a result of destruction of lower motor neurons in the anterior horn Images copyright
Central Ltd.
49 of the spinal cord. In general, there can be a wide range of paralyses, but more
severe forms take place in older patients.
50
51 Replication of poliovirus in motor neurons of the anterior horn of the spinal cord results in cell destruct ion, which leads to the neurologic
sequelae of poliomyelitis. I mage A shows normal anterior horn cells (neurons, large purple/dark pink polygonal cells in a background of lighter
52 pink glial material). I mage B shows neurons that have been dest royed by the virus and are now dying, evidenced by pyknotic changes
53 whereby the cytoplasm of the neuron is shrunken and becomes a darker pink/eosinophilic color and t here is edema/Vacuolization around the
54 neurons that have been destroyed; addit ionally, the axonal proj ect ions are shrunken/destroyed, impairing neuronal fu nct ion.
55 Anterior grey column Poliomyelitis Memnges Aseptic meningitis Fecal-oral route Flaccid paralysis Cytoplasm Men1ng1t1S Pohov1rus Neuron Spinal cord Gastrointestinal tract Motor neuron ParalySIS

56 Meningism Axon Fever V1rus Neurology Neuroglia I nfection Sanitation Lower motor neuron

• 57 A is not correct . 9 °/o chose this .


• 58 Multiple sclerosis is an autoimmune disease involving demyelination of the central nervous syst em . CSF findings show elevated protein and
. 59 MRI is diagnostic. The mean age of onset is f rom 28 t o 31 years and occurs more often in females .
Multiple sclerosis .Auto1mmune d1sease Central nervous system Demyelinating disease Prote1n Nervous system Auto1mmun1ty Magnet1c resonance imaging
• 60
. 61 B is not correct. 2 2 °/o chose this .
• 62 Autoimmune peripheral demyeli nation is the underl ying mechanism of Guillain- Barre syndrome (GBS) . GBS is most commonly seen in
patients with ascending paralysis and weakness of bilat eral lower limbs following a gastrointest inal bacterial or viral infection. Campylobacter
• 63
is implicated in the pathogensis of GBS. Rapidly ascending weakness is characteristic of GBS, whereas polio often manifests with slowly
. 64
• progressive, asymmetrical paralysis .

8
Lode.
s
Suspe-nd
8
End Bloc:k
Item: S6 of ~ ,• Mark <::J [:::> ""I ~· ~'j
100 J.. Previous Next faiUI~S
LAb Notes Calculator

39 D is not correct. 4 °/o chose this.


40 Because t his is a lower motor neuron disease, reflexes are lost in the affected limbs, and t he limbs atrophy. Loss of neurons in the lat eral
41
funiculi (corticospinal t racts) would produce an upper motor neuron synd rome, manifesting wit h a posit ive Babinski sign and hyperreflexive
deep t endon reflexes.
42 Plantar reflex Upper motor neuron syndrome Lower motor neuron Upper motor neuron Motor neuron disease Neuron Motor neuron Lower motor neuron lesion Pyramidal tracts Atrophy Tendon
43
E is not correct. 5 °/o chos e this .
44
The neu ron loss t hat occurs affects t he motor neu rons in t he anterior horns, not t he posterior horns.
45 Neuron Motor neuron Lateral ventricles

46
47
Bottom Line:
48
Unvaccinat ed ch ildren are at risk for infection by the polio virus, which can invade t he int estine and the mot or neurons and may resu lt in
49
neurologic sequelae such as respirat ory m uscle paralysis.
50 Poliomyelitis Poliovirus Gastrointestinal tract Paralysis Motor neuron Virus Sequela Neuron Neurology Muscle Infection

51
52
53 i@l;fil·1i•J fo r yea r:[2017 • J
FIRST AID FACTS
54
55
FA17 p 500.1
56
Spinal cord lesions
• 57
AREA AFFECTED DISEASE CHARACTERISTICS
• 58
. 59 Poliomyelitis and Werdnig-Hoffmann Congenital degeneration of anterior horns of spinal
• 60 disease cord. L If lesions only. "Floppy baby" with marked
• 61
• 62
• 63
•• hypotonia and tongue fasciculations. Infa ntile type
has median age of death of 7 months. Autosomal
recessive inheritance.
. 64 Poliomyelitis .... asymmetric weakness.

8
Lock
s
Suspend
0
End Block
Item: 56 of - ,• Mark --<] [::> ""'I ~· 1!';:'1
100 ~ Prev1o u s Next Labf a lu es Note s Calculator


39 Amyotrophic lateral sclerosis Combined UMN and LM deficits with no sensory
or bowel/bladder deficits (clue to loss of cortical and
40
41
42
43
• •• • spinal cord motor neurons, respectively).
Can be caused br defect in superoxicle clismulase I.
Commonlr presents with as~-mmetric limb weakness
44 (hands/feel), fasciculations, eventual atrophy. Fatal.
45 Commonlr known as Lou Gehrig disease.
46 Treatment: riluzole.
47 Complete occlusion of anterior Spares dorsal columns and Lissauer tract; mid-
48 spinal artery I horacic ASA territory is watershed area, as artery
49 of damkiewicz supplies ASA below- T8. Presents
50 "ith U I deficit below the lesion (corticospinal
51 tract), LM I deficit at the level of the lesion (anterior
52 horn), and loss of pain and temperature sensation
53 below the lesion (spinothalamic tract).
54
55 Tabes dorsalis Caused by 3° syphilis. Results from degeneration
56 (demyel ination) of dorsal columns and roots
• 57 - progressive sensor)' ataxia (impaired
• 58 proprioception - poor coordination).
. 59 Associated wit h Charcot joints, shooting pain, Argyll
• 60 Robertson pupils.
• 61 E,am will demonstrate absence of DTRs and
• 62 (!3 Romberg sign .
• 63 Syringomyelia S) rinx expands and damages anterior white
. 64 •
c-nmmi~<;nrP nf sninnth::~l:tmic- tr::~c-t (2ncl-nrclt>r
8
L.odt
s
Su~pl'nd
~
End Block
Item: S6 of - ,• Mark -<J [:::> "'I ~ · ~
100 ~ P~v1ous N @xt Labl lues N o tes Calcula to r
A A

39 Tabes dorsalis Caused by 3° syphi lis. Results from degeneration


40 (demyelination) of dorsal columns and roots
41 - progressive sensory ataxia (impaired
42 proprioception - poor coordination).
43 Associated with Charcot joints, shooting pain, Argyll
44 Robertson pupils.
45 E\am "ill demonstrate absence of DTRs and
46 G:> Romberg sign.
47
48
Syringomyelia S) rinx expands and damages anterior white
commissure of spinothalamic tract (2nd-order
49
50
51
52
• neurons) - bilateral loss of pain and temperature
sensation in cape-like distribution; seen with Chiari
I malformation; can expand and affect other tracts.
53 Vitamin B12 deficiency Subacute combined degeneration (SCD)-
dcmyclination of Spinocerebellar tracts, lateral
I
54
55
56
• 57
• 58
' Cauda equina syndrome
Corticospinal tracts, and Dorsal columns. Ataxic
gait, paresthesia, impaired position/vibrat ion sense.

Unilateral symptoms including radicular pai n, absent


. 59 knee and ankle reAex, loss of bladder and anal
• 60 sphincter control. Can cause saddle anesthesia .
. 61 Treatment: emergent surgery and steroids.
• 62 Due to compression of spinal roots from L2 and
• 63 below, often caused by intravertebral disk herniation
. 64 or tumors.
8
Lode.
s
Suspe-nd
8
End Bloc:k
Item: S6 of - ,• Mark -<J [:::> "'I ~ · ~
100 ~ P~v1ous N@xt Labl lu es No tes Calcula t o r

A A

39 FA17 p 501.1
40 Poliomyelitis Caused by polio\·irus (fecal-oral transmission). Replicates in oropharynx and small intestine before
41 spreading via bloodstream to C S. Infection causes destruction of cells in anterior horn of spinal
42
cord (UIN death).
43
Signs of L\1N lesion: asymmetric weakness, hypotonia, flaccid paralysis, fasciculations,
44
hyporeAexia, muscle atrophy. Respiratory muscle im·oh-ement leads to respiratory fa ilure. Signs of
45
infection: malaise, headache, fe\'er, nausea, etc.
46
CSF shows t WBCs (lymphoc) tic pleocytosis) and slight t of protein (with no change in CSF
47
glucose). Virus reco\·ered from stool or throat.
48
49
50 FA17 p 163.1
51 RNA viruses
52 VIRAL FAMILY ENVELOPE RNA STRUCTURE CAPSID SYMMETRY MEDICAL IMPORTANCE
53
Reoviruses 0 OS linear Icosahedral Coltivirusa -Colorado tick fever
54
10-12 segments (double) Rotavirus-cause of fatal diarrhea in children
55
56
Picornaviruses No SS ® linear Icosahedral Poliovirus-polio-Salk/Sabin vaccines- !PV/OPV
• 57
Echovirus-aseptic meningitis
• 58 Rhinovirus-"common cold"
. 59 Coxsaekievin1s- aseptic meningitis; herpangina
• 60 (mouth blisters, fever); hand, foot, and mouth
. 61 disease; myocarditis; peri card it is
• 62 II V-acute viral hepatitis
• 63 PERCII
. 64 Hepevirus I 0 SS ® linear Icosahedral HEV
8
Lode.
s
Suspe-nd
8
End Bloc:k
Item: 57 of - ,• Mark -<J [:::> "'I ~ · ~
100 ~ P~v1ous N@xt Labl lues Notes Calculator
6

39 A 56-year-old man present s with sharp substernal chest pain radiating to his back and arms. The patient is seated and leaning forward. He ~~AI
40
states that the pain is less severe in t his posit ion and worsens when he lies down and takes a deep breath . He recently recovered from a
week of runny nose, cough, and fever. On physical examination the sound in the aud io clip is heard at the left lower sternal border. An ECG
41 shows diffuse ST-segment elevat ion.
42
OPEN MEDIA
43
44
~ - - - - - 0000 10001 .. -
45
46
47 Which of the following describes the microorganism that is the most likely cause of this condition?
48 :
49 A . Catalase-positive ( + ), coagulase-positive ( +) cocci
50 B. Double-stranded, linear, enveloped, icosahedral DNA virus
51
C. Double-stranded, segmented RNA virus
52
53 D. Positive, single-stranded, helical RNA viru s
54 E. Small, naked, sing le-stranded RNA virus
55
56
• 57
• 58
. 59
• 60
. 61
• 62
• 63
. 64

8
Lode.
s
Suspe-nd
8
End Bloc:k
Item: 57 of - ,• Mark -<J [:::> "'I ~ · ~
100 ~ P~v1ous N @xt Labl lues N o tes Calcula to r
A A

39
40 Th e correct an swer is E. 4 9°/o chose thi s.
41 The patient presents with classic signs and sympt oms of pericarditis, including precordial chest pain. This pain is relieved when leaning
42 forward and worsens with inspiration. On auscultation a pericardia I friction rub is heard, which accounts fo r the scratchy, leathery sound heard
during both systole and diastole. Classic ECG findings include diffuse ST-segment elevation and depression of the PR segment (as illustrated in
43
the ECG). Pericarditis is frequently preceded by a viral upper respiratory infection. Although many viruses may cause pericarditis, coxsackie B
44 is one of the most common causes of inflammation of the pericardia! membrane. Coxsackievirus is a picornavirus, the smallest of the RNA
45 viruses. They are positive, single-stranded, naked, icosahedral RNA viruses. Additionally, Coxsackievirus is a common cause of viral
myocarditis.
46 P•com ' "' ~ t•~ COASGCku~vm.o.:. Systole Auscultation Pericardial friction rub PencardJbS Diastole Chat p-.n Co:uc.cl..•e 8 -.uus Upper- resp1ratory tract infection Electrocard•ography Inflatnn'lrlt•on
47 RNA Virus RNA V1rus Per•card•um Precord•um
48
49
50
51
52
53
54
55
56
57
• 58
. 59 I mage courtesy of James Heilman, MD
• 60 A i s not correct. 13°/o ch ose thi s .
. 61 Staphylococcus au reus is a gram-positive ( + ), catalase- posit ive ( + ), and coagulase-positive ( +) bacterium. Infection with S. aureus may lead
• 62 to acute bacterial endocardit is from seeding secondary t o bacteremia. The bacterium rarely causes pericardit is .
B«terem1a Stil!lphylococcu~ c~~ureu~ Endocarditis Pericarditis Gram-positive bacteria Bacteria Infective endoc,.rdtb St•phylococcus
• 63
. 64 B is not co rrect. 10°/o chose this .

8
Lode.
s
Suspe-nd
8
End Bloc:k
Item: S7 of ~ ,• Mark <::J [:::> ""I ~· ~'j
100 J.. Previous Next LAbfaiUI~S Notes Calculator

39 A is not correct . 13% chos e this .


40 Staphylococcus au reus is a gram-posit ive (+ ),cat alase-posit ive ( + ), and coagulase-posit ive ( +) bact erium . I nfection wit h S. au reus may lead
t o acut e bact eri al endocardit is from seeding secondary to bacteremia. The bacterium rarely causes pericardit is.
41 Bacteremia Staphylococcus aureus Endocarditis Pericarditis Gram-positive bacteria Bacteria Infective endocarditis Staphylococcus
42
B is not correct. 10 % chose t his .
43
This describes the st ructure of Herpesviridae. Herpesvirus is characteri zed by mult inucleated giant syncytial cells wit h int ranuclear inclusion
44 bodies. Members of t his family include cytomegalovirus and Epst ein-Barr virus. Both cause a mononucleosis syndrome in young adults
45 similarl y charact eri zed by feve r and pharyngit is. Both cause pericardit is only rarely.
Epstein-Barr virus Herpesviridae Cytomegalovirus Infectious mononucleosis Pharyngitis Pericarditis Syncytium Virus Inclusion bodies Fever
46
47 C is not correct. 9 °/o chos e t his .
48 Reoviri dae is t he only double-st randed RNA virus. Members of t his family include rotavirus, which is responsible fo r dia rrhea in children, and
49
colt ivirus, which causes Colorado t ick feve r.
Coltivirus Colorado tick fever Rotavirus Reoviridae RNA virus Virus Diarrhea Tick Double-stranded RNA viruses Fever RNA
50
51
D is not correct. 19 % c hose this .
This answer choice descri bes t he structure of Coronaviridae . Coronavirus (CoV) is a common virus t hat causes a self- limit ed cold- like
52
syndrome. SARS-CoV, h oweve r~ has been ident if ied as the cause of severe acute respirat ory syndrome.
53 Severe acute respiratory syndrome Coronaviridae SARS coronCPJirus Virus Coronavirus

54
55
Bottom Line :
56
Peri cardit is can cause precordial chest pain, f ri ction rub, diffuse ST-segment elevations in all leads on ECG, as well as pulsus paradoxus and
57
distant heart sounds secondary to perica rdia! effusion . I t is often associated wit h infect ions, including coxsackievirus, an RNA v irus in t he
• 58 Picornaviri dae fam ily.
. 59 Pericardia! effusion Picornavirus Coxsackievirus Pulsus paradoxus Pericarditis Chest pain Heart sounds Electrocardiography Virus Precordium RNA virus RNA Pericardia! friction rub Pericardium

• 60
• 61
• 62 141;fil·1i•J for yea r:[2017 •
FIRST AID FA CTS

• 63
. 64 FA17 p 163.1
8
Lock
s
Suspend
0
End Block
Item: S7 of - ,• Mark -<J [:::> "'I ~ · ~
100 ~ P~v1ous N @xt Labl lues N o tes Calcula to r
A A
FA17 p 163.1
39
40
RNA viruses
41 VIRAL FAMILY ENVELOPE RNA STRUCTURE CAPSID SYMMETRY MEDICAL IMPORTANCE
42 Reoviruses I o OS linear Icosahedral Coltivirus3 - Colorado tick fever
43 10-1 2 segments (double) Rotavirus-cause of fata l diarrhea in children
44
Picornaviruses 0 SS @ linear Icosahedral Poliovirus-polio-Salk/Sabin vaccines- IPV/OPV
45
l•.cho\'irus-aseptic meningitis
46
Rhino\'irus-"common cold"
47
Co, sacJ..ievirus- aseptic meningitis; herpangina
48
(mouth blisters, fever); hand, foot, and mouth
49
disease; mrocarditis; pericarditis
50
II V -acute "ira I hepatitis
51
PERC II
52
53 Hepevirus 0 SS ® linear Icosahedral HEV
54 Caliciviruses 0 SS ® linear Icosa hedral Norovirus-viral gastroenteritis
55
56
Flaviviruses Yes SS ® linear Icosa hedral HCV
57
Yellow fever1
• 58
Oenguea
. 59 Sl. I.ouis encephalitisa
• 60
West 'ile virus3 {meningoencephalitis)
. 61 Zika virus
• 62 Togaviruses Yes SS $ linear Icosahedral Rubella
• 63 Western and Eastern equine encephalitisU
. 64
• Chikungunra ,-irus
8
Lode.
s
Suspe-nd
8
End Bloc:k
Item: 57 of - ,• Mark --<) [::> ""'I ~· 1!';:'1
100 ~ Prev1o u s Next Labf a lu es
. Note s Calculator
. - - ...

39 Delta virus Yes ss e circular Uncertain HDV is a "defective" ,-irus that requires the
40 presence of H BV to replicate
41
42
SS, single-stranded; OS, double-stranded; EE>, positi,·e sense; 8 , negative sense; 3 =arbmi ms, arthropod borne (mosquitoes, ticks).
43
44 FA17 p 164.3
45
46
Picornavirus Includes Poliovirus, Echovirus, Rhinovirus, Pi coR"\A\'irus = small R:\A ,·irus.
47
Coxsackievirus, and I lA . Rt is translated PERC II on a "peal.:" (pico).
48 into 1 large polypeptide that is clea\ ed b)
49 proteases into functional viral proteins.
50 Can cause aseptic (,·ira)) meningitis (except
51 rhinovirus and llAV). II arc cntcroviruses
52 except rhinovirus.
53
54
FA17 p 300.2
55
56 Acute pericarditis InAam mation of the pericardium [fJ, red arrows). Commonly presents with sharp pain, aggravated
57 by inspiration, and relieved by sil ting up and leaning forward. Often complicated by pericardia!
• 58 effusion [between yellow arrows in f.J]. Presents with fri ction rub. ECC changes include
. 59 widespread ST-segment ele\'ation and/or PR depression .
• 60 Causes include idiopathic (most common; presumed viral), confirmed infection (cg,
• 61 Coxsackie,·irus), neoplasia, autoimmune (cg, SLE, rheumatoid arthritis), uremia. cardiovascular
• 62 (acute STEM I or Dressler S) ndromc), radiation thcrap} .
• 63
. 64 •
8
L.odt
s
Su~pl'nd
~
End Block
Item: SB of - ,• Mark -<J [:::> "'I ~ · ~
100 ~ P~v1ous N @xt Labl lues N o tes Calcula to r

39 A 72-year-old man from rural Pennsylvania presents t o t he emergency department with a large rash on his back and buttocks, fever, ~~AI
40
headache, and malaise. The patient has several dogs t hat sleep on his bed with him at night. He recent ly found several dead t icks in his
carpet, but he does not recal l seeing any t ick bites on himself. He is given a prescription for doxycycline, and his sympt oms resolve after 2
41 weeks. A micrograph of one of t he patient's pretreat ment blood samples, shown in the image, shows berry - like cyt oplasmic inclusions.
42
43
44

45
46
47
48
49
50
51
52
53
54
Image courtesy of CDC
55
56 This patient most likely cont racted which of t he following?
57
:
• 58 A. Bubonic plague
. 59
B. Ehrlichlosis
• 60
. 61 C. Lyme disease
• 62 D. Rocky Mountain spotted fever
• 63
E. Typhus fever
. 64

8
Lode.
s
Suspe-nd
8
End Bloc:k
Item: sa of ~ ,• Mark <::J [:::> ""I ~· ~'I
100 J.. Previous Next LAbfaiUI~S Notes Calculator

39
40 The correct a nswer is B. 4 1% chose t his .
41 This patient has cont ract ed ehrlichiosis (specif ically human monocyt ic ehrlichiosis), which is caused by an obligate int racellular gra m -negat ive
42 species of ri cket tsiales bacteria, Ehrlichia chaffeensis. As wit h most rickettsial diseases, it is carried by a t ick vector (in this case, the Lone
St ar t ick, Amblyomma americanum ). Ehrlichia fo rms charact eristic berry-like cytoplasmic inclusions in macrophages or neut rophi ls (see arrow
43
in image) . The most common sym ptoms in ehrl ichiosis include feve rs, headaches, malaise, and myalgias. Rash is ra re but if present, usually
44 occurs on the t runk, legs, arms, or face and may be macular, maculopapular, and/or petechial. Ehrl ichiosis is readily t reatable wit h
45 doxycycline.
Rickettsiales Doxycycline Ehrlichia chaffeensis Amblyomma americanum Ehrlichiosis Rickettsia Human monocytotropic ehrlichiosis Gram-negative bacteria Tick Macrophage Neutrophil Maculopapular rash
46
Vector (epidemiology) Bacteria Monocyte Amblyomma Ehrlichia Obligate intracellular parasite Malaise Petechia Intracellular Rash
47
48 A is not correct . 5°/o chose this.
49 Bubonic plague is caused by t he bact erium Yersinia pestis, which is t ransmitted by flea bites or aerosol. It causes charact eri st ic lymph node
enlargement and ulcerat ion at the site of the f lea bit e.
50
Lymph node Bubonic plague Yersinia pestis Plague (disease) Bacteria Aerosol Lymphadenopathy Flea Lymph Peptic ulcer Yersinia
51
C is not co rrect. 2 5% chose this .
52
Lyme disease is a t ick-borne illness that is caused by Borrelia burgdotferi. It produces a charact eristic target lesion known as eryt hema
53
chronicum m igrans. Later, t he disease spreads t o involve t he cent ra l nervous syst em, the heart, and the joints.
54 Lyme disease Borrelia burgdorferi Central nervous system Erythema Erythema chronicum migrans Tick-borne disease Borrelia Lesion Nervous system

55
D is not correct. 21% c hose this.
56
Rocky mount ain spotted fever is caused by infection wit h Rickettsia rickettsii. It usually causes a charact eri st ic rash that involves the soles and
57 palms and usually is t ra nsmitted t hrough t icks endemic to Nort h and South Ameri ca, especially t he East Coast in t he United States.
Rickettsia rickettsii Rocky Mountain spotted fever Rickettsia South America Spotted fever Infection Fever United States
58
. 59 E is not correct. 8°/o chose this .
• 60 Typhus feve r is caused by members of t he genus Rickettsia and is associat ed with t ransmission through t he feces of fleas and lice. I t is usually
• 61 associat ed with cramped, unsanit ary conditions or exposure to flea-carrying rats .
Typhus Rickettsia Louse Genus Feces Fever
• 62
• 63
. 64 Bottom Line :

8
Lock
s
Suspend
0
End Block
Item: sa of ~ ,• Mark <::J [:::> ""I ~· ~'I
100 J.. Previous Next LAbfaiUI~S Notes Calculator

39 Bottom Line:
40 Ehrlichiosis is caused by Ehrlichia chaffeensis, which is carri ed by a t ick (from dogs or deer in Nort h and South Ameri ca) . I t is associat ed wit h
41 fevers, headaches, malaise, and myalgias. Rash is rare bu t, if present , usually occurs on the t runk, legs, arms, or face. Blood may also show
42 berry-like cytoplasmic inclusions in macrophages or neutrophils.
Ehrlichiosis Ehrlichia chaffeensis Macrophage Neutrophil Tick South America
43
44
45
i@l;fil·1i•J f or yea r:[ 2017 • J
46 FIRST AID FACTS

47
48 FA17 p 145.1
49
Rickettsial diseases Treatment: doxycycline (caution during pregnancy; alternative is chloramphenicol).
50
and vector-borne
51
illnesses
52
RASH COMMON
53
54 Rocky Mountain Rickettsia rickettsii, vector is tick. Despite its Classic triad- headache, fever, rash (vasculitis).
55 spotted fever name, disease occurs primarily in the South Palms and soles rash is seen in Coxsackievirus
56 Atlantic states, especially orth Carolina. A infection (hand, foot, and mouth disease),
57 Rash typically starts at wrists · and ankles and Rocky Mountain spotted fever, and 2° Syphilis
58 then spreads to trunk, palms, and soles. (you drive CARS using your palms and soles).
. 59
Typhus Endemic (Aeas)- R typhi. Rickettsii on the wRists, Typhus on the Trunk.
• 60
Epidemic (human body lousc)- R prowazekii.
• 61
Rash starts centrally and spreads out, sparing
• 62
palms and soles.
• 63
RASH RARE
. 64

8
Lock
s
Suspend
0
End Block
Item: sa of ~ ,• Mark <::J [:::> ""I ~· ~'I
100 J.. Previous Next faiUI~S
LAb Notes Calculator

39
40
41
42
43
44
45
46
47
48
49
50
51 FA17 p 144.1
52
53
Zoonotic bacteria Zoonosis: infectious disease transmitted between animals and humans.
54 SPECIES DISEASE TRANSMISSION AND SOURCE
55
Anaplasma spp. Anaplasmosis Ixodes ticks (live on deer and mice)
56
57
Bartonella spp. Cat scratch disease, bacilla ry angiomatosis Cat scra tch
58 Borrelia burgdorferi Lyme cl iseasc Ixodes ticks (live on deer and mice)
. 59
Borrelia recurrentis Relapsing fever Louse (recurrent due to variable surface
• 60
antigens)
• 61
• 62 Brucella spp. Brucellosis/undulant fever Unpasteurized dairy
• 63 Campylobacter Bloody diarrhea Feces from infected pets/animals; contaminated
. 64 meats/foods/hands
8
Lock
s
Suspend
0
End Block
Item: 59 of - ,• Mark -<J [:::> "'I ~ · ~
100 ~ P~v1ous N @xt Labl lues N o tes Calcula to r
6

39 A mother brings her 1-month -old infant to t he pediat rician . She says the baby is crying more t han usual, is vomit ing, and does not want to ~~AI
40
eat. Physical examination reveals a bulging fontanel. Lumbar puncture shows:
41 Opening pressure: 240 mm H2 0 (normal : 100- 200 mm H2 0)
42 WBC count: 1200/mm 3 , 90% neutrophils
43
Protein: 200 mg/dL
Glucose: 30 mg/dl
44

45
Which of the following organisms is most likely responsible for this infant's presentation?
46
:
47
A . Enterovirus
48
B. Herpes simplex virus
49
50 C. Listeria monocytogenes
51 D. Neisseria meningitidis
52
E. Streptococcus pneumoniae
53
54
55
56
57
58
. 59
• 60
. 61
• 62
• 63
. 64

8
Lode.
s
Suspe-nd
8
End Bloc:k
Item: 59 of ~ ,• Mark <::J [:::> ""I ~· ~'j
100 J.. Previous Next LAb faiUI~S Notes Calculator


39 The correct answer is C. 67°/o chos e this.
40 I ncreased irrit ability, feeding difficulty, and ot her general nonspecific signs along wit h a bulging fontanel characterize meningit is in infants. The
cerebrospinal fl uid f indings point t oward a bacteria l meningit is ( increased prot ein and decreased glucose). I n infants 0- 3 months old, t he most
41
common organisms causing mening it is are group B st rept ococci, Escherichia coli, and Listeria monocytogenes. For this reaso n, empiric
42 ant ibiot ic t herapy for men ingit is in t he young infant must include a drug such as ampicillin wit h good coverage against Listeria.
Empiric therapy Listeria monocytogenes Escherichia coli Ampicillin Cerebrospinal fluid Fontanelle Meningitis Antibiotics Streptococcus Bacterial meningitis Streptococcus agalactiae Protein Glucose
43
Irritability Listeria
44
45 A is not correct. 4°/o chos e this.
46 Enteroviruses can cause meningit is in neonates, but t he CSF profile would look different from t he one detailed here. The glucose level would
47 likely be higher, t he WBCs seen would be lymphocyt es rather than neut rophils, and t he WBC count would likely be lower. This infant 's CSF
profile st rongly suggest s bact eri al meningit is.
48 Meningitis Neutrophil Enterovirus Infant Lymphocyte Glucose Cerebrospinal fluid Bacterial meningitis
49
B is not correct. 3% chose this.
50
Herpes simplex virus has also been shown t o cause meningit is in infants. I n viral mening it is, t he CSF findings would show fa r fewe r leukocytes
51 ( 11-500/mm 3 ), protein levels bet ween 50 and 200 mg/dL, and normal glucose levels; a Gram st ain would not show any organisms.
52 Gram staining Viral meningitis Herpes simplex virus Meningitis Herpes simplex Protein White blood cell Glucose Virus

53 D is not correct. 10% chose this .


54 Neisseria meningitidis also causes bacterial men ingitis but is commonly seen in older ch ildren. It is also a gram-negat ive diplococcus.
55 Neisseria meningitidis Meningitis Gram-negative bacteria Diplococcus Bacterial meningitis Neisseria

56 E is not correct. 16°/o chose this.


57 Streptococcus pneumoniae is a common cause of bacterial meningit is in older age groups; however, it is seen less frequently in young infants
58 age 0- 3 months. I n t his age group, t he most common causes of men ingit is are grou p B Strep (eg, S. agalactiae) and E. coli.
Escherichia coli Pneumonia Meningitis Bacterial meningitis Streptococcus Streptococcus pneumoniae Streptococcus agalactiae
59
• 60
• 61 Bottom Line:
• 62 Neonata l bact eri al meningit is is characteri zed by an elevated WBC count (neut rophil predominant ), increased prot ein levels, and decreased
• 63 glucose levels in cerebrospinal flu id . It is most often caused by group B streptococci, E. coli, and Listeria .
Neutrophil Cerebrospinal fluid Meningitis Escherichia coli Bacterial meningitis Streptococcus Protein Streptococcus agalactiae Listeria Glucose Infant
. 64

8
Lock
s
Suspend
0
End Block
Item: 59 of ~ ,• Mark <::J [:::> ""I ~· ~'j
100 J.. Previous Next LAbfaiUI~S Notes Calculator

39 • i@l;fil·1i•J fo r yea r:[2017 • J


FIRST AID FACTS
40
41
FA1 7 p 176.1
42
Common causes of meningitis
43
NEWBORN (0- 6MOl CHILDREN (6 M0- 6 YR) 6- 60YR 60YR+
44
45
Group B streptococci S pneumoniae S pneumoniae S pneumoniae
46
E coli N meningitic/is N meningitic/is (111 in teens) Gram 8 rods
47 Listeria H influenzae type B Enterovi ruses Listeria
48 Enteroviruses HSV
49 Give ceftri axone and vancomycin empirically (add ampicill in if Listeria is suspected).
50 Viral causes of meningitis: enteroviruses (especially coxsackievirus), HSV-2 (HSV-1 = encephalitis), HIV, West ile virus (also
51
causes encephalitis), VZV.
52
In HIV: Cryptococcus spp.
53
ote: Incidence of H influenzae meningitis has l greatly due to conjugate H inflt~ enzae vaccinations. Today, cases are usually
54
seen in unimmunized children.
55
56
57 FA17 p 176.2
58 CSF findings in meningitis
59 OPENING PRESSURE CELL TYPE PROTEIN GLUCOSE
• 60
Bact erial t t PM s t l
• 61
• 62 Fungai/TB t t lymphocytes t l
• 63 Viral Norma 1/t t lymphocytes Norma 1/t Normal
. 64

8
Lock
s
Suspend
0
End Block
Item: 60 of - ,• Mark -<J [:::> "'I ~ · ~
100 ~ P~v1ous N @xt Labl lues N o t es Calcula to r
6

39 A mother brings her 4-year-old son into t he pediatrician because he has been acting inappropriately at his day care. His teacher said that ~~AI
40
he has refused to stop scrat ching his anus and that it has created a distraction in class on numerous occasions. The pediatrician places a
piece of cellophane tape on t he child's anus and then observes under a microscope any eggs that adhere to the tape.
41
42
What medicine should the pediatrician prescribe for the child?
43
:
44
A. Benznidazole
45
B. Doxycycline
46
47 C. Ivermectin
48 0 . Mebendazole
49
E. Niridazole
50
51
52
53
54
55
56
57
58
59
. 60
. 61
• 62
• 63
. 64

8
Lode.
s
Suspe-nd
8
End Bloc:k
Item: 60 of ~ ,• Mark <:::1 t::> ""I ~· ~'j
100 J.. Previous Next LAb faiUI~S Notes Calculator

39
40 The correct answer is 0. 65°/o chose this.
41 This worm is Enterobius vermicularis ( pinworm) which is t ransmit ted in food cont aminat ed wit h eggs. It causes int estinal infect ion and anal
42 pruri t us. The disease is diagnosed by placing t ape on the anus and observ ing any t ransferred eggs under a microscope. The appropriate
medicat ions for t reat ment include albendazole, mebendazole, or pyrantel pamoat e.
43
Pruritus ani Albendazole Pinworm Mebendazole Pyrantel pamoate Itch Anus Worm Microscope Intestinal parasite Pyrantel Infection Gastroenteritis
44
A is not correct. 12% chos e this.
45
Benznidazole and nifurt imox are t reatment options rese rved for Chagas' disease. Chagas' disease is caused by Trypanosoma cruzi and is
46
common ly fo und in Sout h and Cent ra l America.
47 Nifurtimox Trypanosoma cruzi Chagas disease Benznidazole Central America Trypanosoma

48 B is not correct. 5% chose this.


49
Doxycycline is a ribosomal ant ibiot ic t hat has potent act ion against a number of m icrobial organisms including Chlamydia and Rickettsia . It is
50 not appropriat e fo r t he t reatment of worm infections.
Doxycycline Antibiotics Rickettsia Chlamydia infection Chlamydia (genus) Ribosome
51
52 C is not correct. 15% chos e this.
53 Although ivermectin shows good eff icacy for t he t reatment of pinworm infections, it is not the best t reatment fo r pinworms. Its main use is for
54
t he t reatment of Onchocerca volvulus, which causes river blindness.
Pinworm Ivermectin Onchocerca volvulus Onchocerciasis Volvulus Onchocerca
55
56 E is not correct. 3 °/o chos e this.
Niridazo le is used fo r the t reat ment of Dracunculus medinensis, which causes skin ulcerat ion and inf lammat ion .
57
Ulcer (dermatology) Inflammation
58
59
60 Bottom Line:
• 61 Enterobius vermicularis, or pin worm, causes anal pruritu s. It can be diagnosed wit h t he "Scotch Tape" t est and is t reated wit h albendazole,
• 62
mebendazole, or pyrantel pamoat e.
Pruritus ani Albendazole Scotch Tape Mebendazole Pyrantel pamoate Pinworm Itch Pyrantel Worm
• 63
. 64 Nematodes (roundworms) FA17 p 155.1 Protozoa-others FA17 p 154.1

8
Lock
s
Suspend
0
End Block
Item: 1 of 100 - ,• Mark -<J [:::> "'I ~ · ~
QIO: 5111 ~ P~v•ous N@xt Labl lues No tes Calcula t o r

A
. 1 A

A 35-year-old white man presents to the clinic wit h acute chest pain. The chest pain began 3 days ago, and is exacerbated by deep ~~AI
•2
inspiration and a persistent dry cough of the same duration. Associated symptoms include fatigue, malaise, and occasional chills. He has no
·3 significant past medical hist ory. The patient is currently sexually active, and reports 35 pack-years of tobacco use, social dri nking, and
• 4 occasional marijuana and heroin use. His temperature is 38.9° ( 102° F), pulse is 94/min, respiratory rate is 20/min, and blood pressure is
130/86 mm Hg. Physical examination reveals a diaphoretic man with a slight build, tachypnea, track marks on the arms, and jugular venous
·5 distention synchronous with pulse. Auscultation of the heart reveals a systolic murmur.
·6
·7 Which of the following culture characteristics is representative of the most likely cause for this patient's symptoms?
·8
:
•9 A . Catalase-negative, clear zone surrounding cocci on blood agar
• 10
B. Catalase-positive, coagulase -negative cocci
· 11
• 12 C. Catalase-positive, coagulase -positive cocci
• 13 D . Green zone surrounding cocci on blood agar with optochin resistance
• 14
E. Green zone surrounding cocci on blood agar with optochin sensitivity
• 15
. 16 F. No discoloration surrounding cocci on blood agar
. 17
• 18
• 19

• 20
. 21
. 22
. 23
• 24
• 25
• 26
. 27

8
Lode.
s
S uspe-nd
8
End Bloc:k
Item: 1 of 100 ~ ,• Mark <::J [:::> ""I ~· ~'j
QID: 5111 J.. Previous Next LAb faiUI~S Notes Calculator

1
•2
•3 The correct a nswer is C. 71 °/o chose this .
•4
. 5
.6
This patient most likely has infective endocardit is of t he t ricuspid valve caused by Staphylococcus aureus
bact eremia int roduced through intravenous drug use . The major clues to the diagnosis include int ravenous
drug use, t he acut e nature of his presentat ion, and a syst olic murmur secondary t o t ricuspid regurgitation.
..' • .. •
.. • " + •

Over 50% of cases of infect ive endocardit is secondary to intravenous drug use will only affect t he t ri cuspid
• 7 valve. Tricuspid valve veget ations may also seed septic emboli from the right side of the heart, resulting in .. •' t
.... -• f...
•8 primary manifest at ions in the lung, wit h t he presence of cough, pleurit ic chest pain, diffuse pulmonary
infilt rat es, and pyopneumot horax. S. aureus is a catalase -posit ive, coagulase-posit ive, 13-hemolytic, gram- .. .. .... - .. .
•9
• 10
· 11
posit ive anaerobe (shown in t he image); it is also clust er-forming and facult at ive . I t produces a golden -
yellow pigment in culture .
Infective endocarditis Bacteremia Endocarditis Tricuspid insufficiency Staphylococcus aureus Tricuspid valve Gram-positive bacteria Pleurisy
"-
• • \.,

..... •
·- - ...
tO ~o~ m
..
Intravenous therapy Anaerobic organism Embolism Staphylococcus Systole Chest pain Septic embolism Cough Lung Heart murmur Drug injection j
• 12
Image cowtesy of Y Tam be
• 13
• 14 A is not correct . 5°/o chose this .
• 15 Lancef ield group A st re ptococci, or Streptococcus pyogenes, is a catalase- negat ive, 13-hemolytic, gram -positive cocci that is associated wit h
. 16 pharyng it is, skin infect ions, scarlet feve r, and toxic shock syndrome. It can also cause rheumat ic fever and acut e postst rept ococcal
glomerulonephrit is through an ant ibody-mediat ed immune react ion . The rh eumatic feve r follows the pharyngit is in pediat ric patients, and
. 17
damages t he heart valve, t hus predisposing t o subsequent endocardit is, such as wit h viridans streptococci. This pat ient does not display
• 18 t ypical symptoms of acut e rheumatic fever, and is out of the typical age range of pediatri c patients (4- 14 years old) . Streptococcus agalactiae
• 19 is the ot her catalase- negative, 13- hemolyt ic, gram-posit ive cocci, and it causes neonatal meningit is, pneumonia, and sepsis .
Scarlet fever Streptococcus pyogenes Rheumatic fever Toxic shock syndrome Streptococcus agalactiae Gram-positive bacteria Coccus Pharyngitis Endocarditis Group A streptococcal infection
• 20
Viridans streptococci Meningitis Pneumonia Glomerulonephritis Streptococcus Sepsis Heart valve Immune system Infant Fever Catalase
. 21
. 22 B is not correct. 10% chose this .
• 23 Staphylococcus epidermidis and S. saprophyticus are bot h catalase -posit ive, coagulase- negat ive gram-posit ive facu ltative anaerobes .
(Coagulase conve1ts f ibrinogen to fibrin, causing blood clot t ing--t esting fo r the presence of coagulase is a common way of diffe rent iat ing
• 24
between Staph speciies.) S. epidermidis colonizes the skin and is a frequent contaminant in blood cultures. It can also lead to nosocomial
• 25 infections of indwelling prost het ic devices (including prost hetic hea1t valves), but it does not usually cause infect ion of native heart valves .
Staphylococcus epidermidis Coagulase Fibrin Gram-positive bacteria Facultative anaerobic organism Staphylococcus saprophyticus Staphylococcus Hospital-acquired infection Anaerobic organism Fibrinogen
• 26
. 27 Prosthesis Blood culture Heart valve Infection

8
Lock Suspend
s 0
End Block
Item: 1 of 100 ~ ,• Mark <::J [:::> ""I ~· ~'j
QID: 5111 J.. Previous Next LAbfaiUI~S Notes Calculator

1 Prosthesis Blood culture Heart valve Infection


•2
D is not correct. 7°/o chose this .
•3
Viridans streptococci are cat alase- negative, a-hemolyt ic gram-posit ive organisms t hat are classically associat ed with dental infections,
•4 subacut e endocardit is, and abscesses. Unlike the acute presentation of Staphylococcus aureus endocardit is associated wit h high fevers and
. 5 rapid valve dest ruct ion, viridans streptococci cause subacut e endocarditis characterized by low-grade feve rs and slow development of valve
.6 dest ruct ion, and most commonly affect a previously damaged hea1t valve (eg, a congenit al lesion or rheumat ic fever). The mit ral valve is
most commonly affected, followed by the aortic valve.
• 7 Rheumatic fever Staphylococcus aureus Viridans streptococci Gram-positive bacteria Endocarditis Streptococcus Heart valve Staphylococcus Subacute bacterial endocarditis Aortic valve Mitral valve Abscess
•8 Lesion Catalase Fever Congenital disorder
•9
E is not correct. 5°/o chos e this .
• 10
Streptococcus pneumoniae is a catalase -negat ive, a- hemolyt ic, gram- positive diplococcal bacterium . It is also ident if iable and differentiat ed
· 11 by being optochin-sensit ive (in contrast to the a- hemolytic v iridans st reptococci), and produces a positive Quellung reaction (capsule swelling
• 12 in the presence of ant iserum antibodies) . Although S. pneumoniae is t he most common cause of pneumonia, and the pat ient demonst ra tes
• 13
certain featu res suggestive of pneumonia, t he presence of a heart murmur and jugular venous dist ent ion is incongruous with t his diagnosis.
Streptococcus pneumoniae Gram-positive bacteria Heart murmur Streptococcus Pneumonia Antibody Bacteria Jugular venous pressure Quellung reaction Catalase Jugular vein
• 14

• 15 F is not correct . 2°/o chose th is .


. 16
The Lancefield group D st re ptococci include t he enterococci (eg, Enterococcus faecalis and E. faecium), and the nonenterococci (eg,
peptostreptococci) . This grou p of organisms is cat alase- negative and can bey-hemolyt ic. They are t ypically known fo r bilia ry and urinary tract
. 17 infections, but t hey can also result in subacute bacterial endocarditis. Howeve 1~ this patient has ri sk fact ors and sym ptoms for an acut e
• 18 endocarditis .
Enterococcus faecalis Enterococcus Endocarditis Streptococcus Infective endocarditis Urinary tract infection Urinary system Subacute bacterial endocarditis Catalase
• 19
• 20
. 21 Bottom Line :
. 22 Staphylococcus aureus can cause an acut e infect ive endocardit is of nat ive heart valves, result ing in high feve rs, chills, and an acut e heart
• 23 murmur. It must always be suspected in int ravenous drug users and may not present wit h the cl assic symptoms ( Roth spot s, Osier nodes,
• 24
and Janeway lesions) because of isolat ed t ri cuspid valve infection .
Roth's spot Osler's node Infective endocarditis Tricuspid valve Janeway lesion Endocarditis Staphylococcus aureus Heart murmur Staphylococcus Heart valve Intravenous therapy Drug injection Infection
• 25 Tricuspid atresia
• 26
. 27

8
Lock
s
Suspend
0
End Block
Item: 1 of 100 - ,• Mark --<] [::> ""'I ~· 1!';:'1
QIO: 5111 ~ Prev1o u s Next Labf a lues Notes Calculator

1
•2
141:fi1·1i•J
FIRST AID FACTS
for year: 201 7 •

·3
•4 FA17 p 131 .3
·5
Staphylococcus au reus Gram (f), ~-hemolytic, catalase (f), coagulase TS T-1 is a superantigen that binds to MHC
·6
(f) cocci in clusters Ll Protein (\·irulence II and T-cell receptor, resulti ng in polyclonal
·7
·8
factor) binds Fc-lgG, inhibiting complement T-cell acti\'ation.
·9
acti,·ation and phagocytosis. Commonlr Staphylococcal toxic shock syndrome
· 10
colonizes the nares, axilla, and groin. (T S) presents as fever, \'Omiting, rash,
· 11 Causes: desquamation, shock, end-organ failure. TSS
• 12 lnAammatory disease-skin infections, results in t ST, t ALT. t bilirubin. ssociated
• 13 organ abscesses, pneumonia (often after with prolonged use of vaginal tampons or nasal
• 14 influenza virus infection), endocarditis, packing.
. 15 septic arthritis, and osteomrelit is. Compare with Streptococcus pyogenes TSS (a
. 16 Toxin-mediated disease-toxic shock toxic shock- like syndrome associated with
. 17 syndrome (TSS1~1 }, scalded skin syndrome painfu l skin infection).
• 18 (exfoliative toxin), rapid-onset food S aureus food poisoning due to ingestion of
• 19 poisoning (cntcrotoxins). preformed toxin _. short incubation period
• 20 11RSA (2-6 hr} followed by nonbloody diarrhea
(methicill in-resistantS aureus}
. 21
infection- important cause of serious and emesis. Enterotoxin is heat stable - not
. 22
nosocomial and community-acquired destroyed by cooking.
• 23
infections; resistant to methicillin and Bad staph (aureus} make coagulase and toxins .
• 24
nafcillin because of altered penicillin- Forms fibrin clot around self _. abscess .
• 25
binding protein .
• 26
. 27 •
8
L.odt
s
Su~pl'nd
~
End Block
Item: 1 of 100 - ,• Mark --<] [::> ""'I ~· 1!';:'1
QIO: 5111 ~ Prev1o u s Next Labf a lues Notes Calculator

1 •
•2
FA17 p 299.2
·3 Bacterial endocarditis Fever (most common symptom), new murmur, Mitral valve is most frequently invoked.
•4 Roth spots (round white spots on retina Tricuspid valve endocarditis is associated "ith
· 5 surrounded by hemorrhage · ), Osler nodes I drug abuse (don' t '' tri" drugs). Associated
·6 (tender raised lesions on linger or toe pads [] with S aureus, PseudomoiiCIS, and Candide~.
· 7
due to immune complex deposition), Janeway Culture 8; most likely Coxiella bumetii,
·8
lesions (small, painless, erythematous lesions BciTioneiiCI spp., II CEK (l laemophilus,
·9
· 10
on palm or sole) m,
glomerulonephritis, Aggregatibacter (formerly Actinobacillus),
septic arterial or pulmonary emboli, splinter Cardiobe~cterium, Eikenella, Kingella)
· 11
• 12
m
hemorrhages on nail bed. ~lulti ple blood • Bacteria FR0:\1 JANE • :
• 13
cultures necessary for diagnosis. Fe,er
• 14
Acute-S aureus (high virulence). Roth spots
. 15 Large ,·egetations on previously normal Osler nodes
. 16 valves 0 . Rapid onset. ~ 1urmur
. 17 Subacute-viridans streptococci (low Janeway lesions
• 18 virulence). Smaller vegetations on Anemia
• 19 congenitally abnorma l or diseased valves. Nail-bed hemorrhage
• 20 Sequela of dental procedures. Gradual Emboli
. 21 onset.
. 22 S bovis (gallolyticus) is present in colon cancer,
• 23 S epidermidis on prosthetic valves .
• 24 Endocarditis may• also be nonbacterial
• 25 zo
(marantic/thrombotic) to malignancy,
• 26
hypercoagulable state, or lupus. •
. 27
8
L.odt
s
Su~pl'nd
~
End Block
Item: 1 of 100 - ,• Mark -<J [:::> "'I ~ · ~
QIO: 5111 ~ P~v•ous N@xt Labl lues No tes Calcula t o r

1 A
on palm or sole) [!1, glomerulonephritis, Aggregatibacter (formerly Actinobacillus), A

•2
septic arterial or pulmonary emboli, splinter Cardiobacterium, Eikenella , Kingella)
·3
hemorrhages [!] on nail bed. Multiple blood • Bacteria FR0:\1 JANE • :
• 4
cultures necessary for diagnosis. Fe,cr
·5
• Acute -S aureus (high virulence). Roth spots
·6
Large ,·egetations on previously normal Osler nodes
·7
,·alves 1). Rapid onset. \lurmur
•9
Subacute-\'iridans streptococci (lo" Janeway lesions
• 10
\'irulence). Smaller "egetations on \ nemia
· 11
congenitally abnormal or diseased vahes. 1\ail-bed hemorrhage
• 12 Sequela of dental procedures. Gradual Emboli
• 13 onset.
• 14 S bol'is (gallolyticus) is present in colon cancer,
• 15 S epidermidis on prosthetic \ctlvcs.
. 16 Endocarditis mar also be non bacterial
. 17 (marantic/thrombotic) 2° to ma lignancy,
• 18 hypercoagulable state, or lupus.
• 19

• 20
. 21
. 22
. 23
• 24
• 25
• 26 t
. 27

8
Lode.
s
S uspe-nd
8
End Bloc:k
Item: 2 of 100 - ,• Mark -<J [:::> "'I ~ · ~
QIO: 2915 ~ P~v•ous N@xt Labl lues No tes Calcula t o r

A A
1
A 56-year-old immigrant from Chi na presents to t he cl inic because of weight loss, upper abdominal pain, early sat iet y, and a palpable mass ~~AI
•2
in t he upper abdomen. Serum levels of a-fetoprotein are markedly elevated, and ultrasound of the liver reveals a lesion with poorly defined
·3 marg ins and coarse, irregu lar internal echogenicity. He is diagnosed with hepatocellular carcinoma. A family history reveals th at his father
• 4 died of liver cancer when t he pat ient was 14 years old.
·5
·6
Which of the following is most likely responsible for t his patient's disease?
·7 :
A. Hepatitis A virus
•9 B. Hepatitis B virus
• 10
C. Hepatitis C virus
· 11
D . Hereditary Hemochromatosis
• 12
• 13 E. Wilson Disease
• 14

• 15
. 16
. 17
• 18
• 19

• 20
. 21
. 22
. 23
• 24
• 25
• 26
. 27

8
Lode.
s
Suspe-nd
8
End Bloc:k
Item: 2 of 100 ~ ,• Mark <::J [:::> ""I ~· ~'j
QID: 2915 J.. Previous Next LAb faiUI~S Notes Calculator

1
2
•3 The correct a nswer is B. 43 % chos e t his .
•4 Globa lly, hepat it is B virus (HBV) remains t he most common risk factor fo r the development of hepatocellular ca rcinoma (HCC) . Although most
. 5
adults (90%) have full resolut ion follow ing an acute infection, vert ical t ransmission f rom infected mother to child often results in a chronic
carri er st at e which increases t he risk for HCC. When fathers are chronically infected, int ra -household t ransmission leads to infect ion in
.6 ch ildren wh ich may become ch ron ic. HBV-associated HCC may occur in younger adults (20-40 yea rs old) without cirrhosis since HBV is
• 7 oncogenic. More commonly, HCC is a consequence of long-st anding infection with cont inued inflammat ion and cell turnover. HBV can also be
•8
t ransm itt ed sexually or via blood. Hepat it is B is an endemic in China. Treat ment of HCC is current ly unsatisfactory. Surgical resection (non-
cirrhotic liver) and liver t ransplantat ion are the best management options.
•9 Hepatocellular carcinoma Hepatitis 8 Cirrhosis Liver transplantation Hepatitis Hepatitis 8 virus Virus Vertically transmitted infection Endemism Liver Oncogene Risk factor Carcinogenesis
• 10 Sexually transmitted infection Inflammation Carcinoma Organ transplantation Infection Cell cycle Segmental resection

· 11
A is not correct . 3°/o chos e this .
• 12
Hepatit is A v irus ( HAV) is not associated wit h HCC. The v irus is usua lly acquired t hrough the feca l-oral rout e or int ra -household spread. HAV
• 13 usua lly results in acute hepat it is, ranging in severit y from asymptomatic to fu lminant. without causing a chronic ca rrier state, in cont rast to
• 14 hepatitis Band C viruses. HAV infect ion can be t reat ed conservat ively, and a vaccine is current ly available fo r those t ravelling to endemic
reg ions .
• 15
Fecal-oral route Hepatitis A Hepatitis 8 Hepatitis Endemism Hepatitis A virus Virus Asymptomatic Fulminant Vaccine
. 16
. 17
C is not correct. 23 % chos e this .
Hepatit is C ( HCV) v irus is f requently associated wit h HCC in developed countries but is not t ransmit ted f rom father t o child (mother t o child
• 18
t ransm ission can occur, more frequent ly at high viral loads) . HCV-associat ed HCC commonly occurs in older adults (>60 years old) and
• 19 cirrhosis is present in most cases. HCV is parenterally t ransm itt ed usually through needle shari ng or via contam inat ion of mu lt i-u se drug vials .
• 20 Approximat ely 80 -90% of patients develop chronic hepatitis fo llowing HCV exposure, and of those who develop cirrhosis, up t o 15% develop
HCC. Previous first-line t reat ments included interferon -alpha (I FNa) and ribavirin but these have given way to the new curative t reat ment s
. 21
with protease and polymerase inhibi tors .
. 22 Ribavirin Hepatitis C Cirrhosis Hepatitis Interferon Protease Hepatitis C virus Developed country Virus Polymerase Route of administration Needle sharing Chronic hepatitis

• 23
D is not correct. 20% c hose this .
• 24
Hereditary hemochromatosis has a high rat e of hepatocellular carcinoma, up t o 10% of patients. However, t he commonest genetic defect
• 25 (t ype 1 "classical" hemoch romat osis, HFE) is ra re or non -exist ent in Asia. I n add it ion, classical hemochromat osis is inherit ed in an autosoma l
• 26 recessive manner and would not lead to fat her-to -son t ransmission .
Hepatocellular carcinoma Iron overload Autosomal recessive Recessive Dominance (genetics) Genetic disorder Autosome HFE (gene) Carcinoma
. 27

8
Lock
s
Suspend
0
End Block
Item: 2 of 100 ~ ,• Mark <::J [:::> ""I ~· ~'j
QID: 2915 J.. Previous Next LAb faiUI~S Notes Calculator

1 E is not correct. 11°/o chos e this.


2 Wilson disease is an autosomal recessive disorder of copper t ransport . Hepatocellular carcinoma is relatively rare, and parent -to -child
•3 t ransmission would only occur if one parent had 2 abnormal alleles (most pat ients are compound heterozygot es and t he ot her heterozygot e .
Hepatocellular carcinoma Heterozygote Wilson's disease Autosomal recessive Compound heterozygosity Recessive Autosome Heterozygous Dominance (genetics) Zygosity Allele Carcinoma Copper
•4
. 5
.6 Bottom Line:
• 7 HBV infection is the most common cause of HCC because it is endemic in much of Asia and Af rica . I n the absence of neonatal immunization,
•8 HBV is spread vertically f rom infected mot hers wit h consequent chronic infect ion in neonates and infants t hat are unable to mount an
immune response. Vertical t ransmission is t he most common mode of t ransmission world wide .
•9
Endemism Vertically transmitted infection Immune system Hepatitis 8 virus Africa Infant Infection Immunization Asia
• 10
· 11
• 12
i@l;fil·1i•J for year:[2017 • J
FIRST AID FACTS
• 13
• 14

• 15
FA17 p 168.1
. 16 Hepatitis Signs and symptoms of all hepatitis viruses: episodes of fever, jaundice, t ALT and AST. Naked viruses (HAY
. 17 viruses and I lEV) lack an envelope and are not destroyed by the gut: the \"Owcls hit your bowels.
• 18
HBV D 'A polrmerase has D A- and RNA-dependent activities. Upon entry into nucleus, the polymerase
• 19
completes the partial dsD 'A. Host R1 A polymerase transcribes mRNA from \·iral D 'A to ma ke vira l
• 20
proteins. The DNA polymerase then re\·erse transcribes viral R A to D1 A, which is the genome of the
. 21
progeny v1rus.
. 22
HCV lacks 3'-5' exonuclease activity -+ no proofreading ability -+ variation in antigenic structures of HCV
• 23
envelope proteins. Host antibody production lags behind production of new mutant strains of HCV.
• 24
• 25
Virus H~ H~ HCV H~ HEV
• 26 FAMILY RNA picornavirus DNA hepadnavirus R 'A Aavivirus R A deltavirus R1 A hepcvirus
•l I
. 27 Yt'l .ll lU"'t. .O O,.. . O,..U 1 I I U ("" I ~. t

8
Lock
s
Suspend
0
End Block
Item: 2 of 100 ~ ,• Mark <::J [:::> ""I ~· ~'j
QID: 2915 J.. Previous Next LAb faiUI~S Notes Calculator

1 • •
Virus HAV HBV HCV HDV HEV
2
FAMILY RNA picornavirus DNA hepadnavirus RNA Aavivirus Rl A deltavirus RNA hepcvirus
•3
•4 TRANSMISSION Feca l-oral (shellfish, Parenteral (Blood), Primarily blood Parenteral, sexual, Fecal-oral,
. 5 travelers, day care) sexual (Baby- (IYDU, post- perinatal especially
. 6 making), perinatal transfusion) waterborne
• 7 (Birthing)
•8
INCUBATION Short (weeks) Long (months) Long Superinfection Short
•9
(HOY after
• 10
!!BY) = short
· 11
• 12
Coinfection (HOY
• 13
with HBV) = long
• 14 CLINICAL COURSE Asymptomatic Initially li ke serum l\llay progress to Similar to HBY Fulminant hepatitis
• 15 (usually), Acute sickness (fever, Cirrhosis or in Expectant
. 16 arth ralgias, rash); Carcinoma {pregnant) women
. 17 may progress to
• 18
carcmoma
• 19
PROGNOSIS Good Adults ..... mostly Nlajority develop Superinfection High mortality in
• 20
. 21
full resolution; stable, C hronic ..... worse prognosts pregnant women
. 22
neonates ..... worse hepatitis C
• 23 prognosis
• 24 HCC RISK lo Yes Yes Yes 'o
• 25
LIVER BIOPSY Hepatocyte Granular Lymphoid Similar to HBY Patchy necrosis
• 26
swelling, eosinophilic aggregates with •
. 27 •
8
Lock
s
Suspend
0
End Block
Item: 2 of 100 - ,• Mark -<J [:::> "'I ~ · ~
QIO: 2915 ~ P~v•ous N@xt Labl lues No tes Calcula t o r

1 A
swelli ng, eosinophilic aggregates with A

2 monocyte "grou nd glass" foca l areas of


·3 infiltration, appearance; macro,esicula r
• 4
Councilman cytotoxic T cells steatosis
·5
bodies mediate damage
·6
·7
NOTES 1 o carrier state Carrier state Carrier state ,·cry Defecti,·e ,·irus, Enteric, Epidemic,
(".\ lone") common common Depends on HB no carrier state
•9 HBsAg
• 10

· 11
FA17 p 375.1
• 12
• 13 Hepatocellular Most common 1° malignant tumor of)i,·er
• 14 carcinoma/hepatoma in adults rJ. Associated with II BV (+/-
• 15 cirrhosis) and all other causes of cirrhosis
. 16 (including HCV, alcohol ic and nonalcoholic
. 17 fatty liver disease, autoimmune disease,
• 18
hemochromatosis, a 1-antil rypsin deficiency)
• 19
and specific carcinogens (eg, aflatoxin
• 20
from Aspergillus). May lead to Budd-Chi<Hi
. 21
syndrome.
. 22
Findings: jaundice, tender hepatomegaly,
. 23
ascites, polycythemia, anorexia. Spreads
• 24
hematogenously.
• 25
• 26
Diagnosis: t a-fetoprotein; ultrasound or
. 27
contrast CT/ lRI , biopsy.
8
Lode.
s
Suspe-nd
8
End Bloc:k
Item: 3 of 100 - ,• Mark -<J [:::> "'I ~ · ~
QIO: 4 930 ~ P~v•ous N@xt Labl lues No tes Calcula t o r

A A
1
A 19-year-old soldier returns from a tour of duty in Iraq complaining of fever, myalgia, diarrhea, and abdominal pains. She was healthy ~~AI
2
prior to deployment. On further questioning she reports frequent contact with the water supplies while there because her patrol
·3 respons ibilities involved safeguarding a major river. She has a temperature of 38.4°C ( 101.1 ° F) and generalized lymphadenopathy. Her
• 4 liver edge is palpated 2.5 em below the right costal margin, and her spleen tip is easily felt 4 em below her left costal margin. Her complete
blood cell count shows:
·5
·6 Hemoglobin: 12.2 g/dl
·7 Hematocrit: 35.4%
WBCs: 8400/mml
Neutrophlls: 53%
•9 Lymphocytes: 25%
• 10 Monocytes: 5%
Eosinophils: 17%
· 11
• 12 A parasitic infection is diagnosed .
• 13
• 14 Which parasite is respons ible for this patient's acute symptoms?
• 15 :
. 16 A. Babesia microti
. 17
B. Dracunculus medinensis (Guinea worm)
• 18
C. Schistosoma mansoni
• 19

• 20 D. Toxoplasma gondii
. 21 E. Trichinella spiralis
. 22
. 23
• 24
• 25
• 26
. 27

8
Lode.
s
Suspe-nd
8
End Bloc:k
Item: 3 of 100 ~ ,• Mark <::J [:::> ""I ~· ~'j
QID: 4930 J.. Previous Next LAbfaiUI~S Notes Calculator

1
2 The correct a nswer is C. 59°/o chose this .
3 This patient has acut e Kat ayama fever due to schistosoma! infect ion acquired in I raq. Schistosomiasis caused by Schistosomiasis mansoni is
•4 endemic to sub-Saharan Africa, the Midd le East , Sout h America, and t he Caribbean. Key t o recognizing that t his pat ient has S. mansoni
infection is the patient's dramatic hepatosplenomegaly and peripheral eosinophilia. Eosinophilia is due t o Th2-t ype inf lammat ion against this
. 5
extracell ular parasit e. Aft er male and female worms mate in t he host circulat ion, many egg forms are produced . Females produce hu ndreds of
.6 eggs per day, which lodge in vari ous peri pheral sit es (around which granulomas fo rm) or can be excreted in t he feces . The onset of egg laying
• 7 is the t rigger fo r Kat ayama feve r. Schistosomiasis can be t reat ed with praziquantel.
Praziquantel Schistosomiasis Endemism Hepatosplenomegaly Eosinophilia South America Sub-Saharan Africa Schistosoma Schistosoma mansoni Granuloma Parasitism Feces Fever Middle East Infection
•8
Inflammation Caribbean Africa
•9
• 10 A is not correct . 12% chose this .
· 11 Babesia microti, a blood parasit e, is one cause of babesiosis, a malaria-like illness. Although babesiosis can featu re a feve r as seen in t his
patient, babesiosis ra rely involves t he liver. Furthermore, babesiosis, like malaria, is caused by parasites that mult iply in red blood cells,
• 12
causing a hemolytic anemia when re plicat ed parasit es are released. This patient 's hemog lobin and hematocrit are within normal lim its.
• 13 Babesiosis Babesia Hematocrit Hemoglobin Hemolytic anemia Malaria Anemia Parasitism Red blood cell Theileria microti Hemolysis Liver Fever

• 14
B is not correct. 12% chose this .
• 15 Guinea worm disease is cont ract ed by drinking stagnant wat er cont aining Guinea worm larvae; however, t his parasite causes skin and soft-
. 16 t issue infections, rat her t han infecting the liver and intest ines .
Dracunculiasis Dracunculus medinensis Worm Guinea Liver Parasitism Gastrointestinal tract
. 17
• 18 D is not correct. 6°/o chose t his .
• 19 Although t oxoplasmosis can affect t he liver, t his pat ient 's WBC count indicat es t hat she is not immunocompromised. I n an acut e infection,
• 20
t oxoplasmosis can cause a mild flu -like illness, but is unlikely to cause as serious an illness as seen here in an ot herwise healt hy patient .
Likewise, alt hough many individuals remain colonized with Toxoplasma following an init ial infection, long-t erm complications are ra re in
. 21 individuals who are otherwise immunocompet ent.
. 22 Toxoplasmosis Immunodeficiency Immunocompetence Toxoplasma gondii Liver Infection Influenza-like illness

• 23
E is not correct. 11°/o chose this .
• 24 Rather than causing hepatosplenomegaly, Trichinella infect ion would cause m uscl e inflammation and periorbi tal edema . Trichinella infect ion
• 25 comes from eat ing undercooked meat.
Hepatosplenomegaly Edema Inflammation Periorbital puffiness Trichinella
• 26
. 27

8
Lock
s
Suspend
0
End Block
Item: 3 of 100 - ,• Mark --<] [::> ""'I ~· 1!';:'1
QIO: 4930 ~ Prev1o u s Next Labf a lu es Note s Calculator

1
Bottom Line:
2
Schistosomiasis is a parasit ic f luke infecti on t hat can cause Katayama fever, characterized by feve r~ hepat osplenomegaly, and peripheral
3 eosinophilia. Consider Katayama fever in ot herwise healthy pat ients with recent travel history to the Midd le East, sub-Saharan Africa, and
•4 South America, especially if t he individual was exposed to sources of fresh water.
Sch•stosomta.st HepllltO$plenomegaly Eosinophilia Parasitism Sub-Saharan Africa South Amer1ca Fevet Tret ~oda Mtddle East Infection Africa
·5
·6
·7
·8
141;fjl;!io) for year: 2017 •
FI~ST AIO FACTS

·9
· 10 FA17 p 156.2
· 11 Trematodes (flukes)
• 12
ORGANISM DISEASE TRANSMISSION TREATMENT
• 13
Schistosoma Li,·er and spleen enlargement Snails are host; cercariae Praziquantel
• 14
(S mansoni, egg with penetrate skin of humans
. 15
. 16
lateral spine fJ), fibrosis,
. 17
inflammation, porta I
• 18
hypertension
• 19 C hronic infection with
S haematobium (egg with
• 20
. 21
rn()
o"~" o
0.
terminal spine []) can lead
. 22 to squamous cell carcinoma
• 23 of I he bladder (painless
• 24 hematuria) and pulmonary
• 25 hypertension
• 26
Clonorchis sinensis Biliary tract inflammation Undercooked fish Praziquantel •
. 27
8
L.odt
s
Su~pl'nd
~
End Block
Item: 3 of 100 ~ ,• Mark <::J [:::> ""I ~· ~'j
QID: 4930 J.. Previous Next LAbfaiUI~S Notes Calculator

1 • •
FA17p154.1
2
Protozoa- others
3
•4
ORGANISM DISEASE TRANSMISSION DIAGNOSIS TREATM ENT
. 5 Visceral infections
. 6
Trypanosoma Chagas disease- dilated Reduviid bug Trypomastigote in Benznidazole
• 7
cruzi cardiomyopathy with ("kissing bug") blood smear fJ or nifurtimox;
•8
apical atrophy, megacolon, feces, deposited C n JZing in my
•9
megaesophagus; predominantly in a painless bite Benz, with a fur
• 10
in South America (much Iike a kiss) coat on
· 11
Unilateral periorbital swelling
• 12
• 13
(Romai'ia sign) characteristic of
• 14
acute stage
• 15 Leishmania Visceral leishmaniasis Sand fly Macrophages Amphotericin B,
. 16 donovani (kala-azar) - spiking fevers, contain ing sodium
. 17 hepatosplenomegaly, a mastigotes : stibogluconate
• 18
pancytopenia
• 19
Cutaneous leishmaniasis-skin
• 20
ulcers
. 21
. 22
• 23
• 24 Sexually transmitted infections
• 25
Trichomonas Vaginitis-foul-smelling, greenish Sexua l (cannot exist Trophozoites Metronidazole for
• 26

vagina/is discharge; itching and burning; outside human (motile) [!jl on wet patient and partner •
. 27

8
Lock
s
Suspend
0
End Block
Item: 3 of 100 ~ ,• Mark <::J [:::> ""I ~· ~'j
QID: 4930 J.. Previous Next LAbfaiUI~S Notes Calculator

1 • •
Sexually transmit ted infections
2
3 Trichomonas Vaginitis-foul-smelling, greenish Sexua l (cannot exist Trophozoites Metronidazole for
•4 vagina/is discharge; itching and burning; outside human (motile) [!jl on wet patient and partner
. 5 do not confuse with Gardnerella because it cannot mou nt; "strawI)erry (prophylaxis)
. n
. 6 vagina/is, a gram-variable form cysts) cerv1x
• 7 bacterium associated with
•8 bacterial vaginosis
•9
• 10
· 11
• 12
• 13 FA17 p 157.2
• 14
Parasite hints ASSOCIATIONS ORGANISM
• 15
. 16
Bi liary tract d isease, chola ngioca rcinoma Clonorchis sinensis
. 17 Brain cysts, seizures Taenia solium (neurocysticercosis)
• 18
Hematuria, squamo us cell bladder cancer Schistosoma haematobium
• 19
• 20 Liver (hydatid) crsts Echinococcus granulosus
. 21 Microcytic anemia Ancylostoma, Necator
. 22
Myalgias, periorbita I edema Trichinella spiralis
• 23
• 24 Perianal pruritus Enterobius
• 25 Portal hypertension Schistosoma mansoni, Schistosoma japonicum
• 26
Vita min B12 deficiency Diphyllobothrium latwn •
. 27 •
8
Lock
s
Suspend
0
End Block
Item: 4 of 100 - ,• Mark -<J [:::> "'I ~ · ~
QIO: 2026 ~ P~v•ous N@xt Labl lues No tes Calcula t o r

1
A 19-year-old man presents to his family physician complaining of rash on the palms of his hands fo r about 1 week. Physical examination
2
reveals lesions on his palms and soles bilat erally, like those shown in the images. Genital examinat ion reveals the presence of a wart- like
3 lesion on the lateral scrotum.
• 4

·5
·6
·7 •

•9
• 10
· 11
• 12
• 13
• 14 Images courtesy CDC/ Robert Sumpter
• 15
. 16 What is the most specific t est with which to diagnose this man's infection?
. 17 :
• 18 A. Fluorescent treponema! antibody absorption t est
• 19 B. Heterophile ant ibody t est
• 20
C. Rapid plasma reag in test
. 21
. 22 D. Venereal Disease Research Laborat ory test
. 23 E. Weii - Felix reaction
• 24
• 25
• 26
. 27

8
Lode.
s
S uspe-nd
8
End Bloc:k
Item: 4 of 100 ~ ,• Mark <::J [:::> ""I ~· ~'j
QID: 2026 J.. Previous Next LAb faiUI~S Notes Calculator

1
2 The correct answer is A. 61 °/o chose this.
3 This patient is manifest ing signs of secondary syphilis (t he bact eremic stage), which occurs about 6 weeks after t he painless chancre of
4
primary syphilis has healed. This stage is syst emic, characterized by widespread rash, generalized lymphadenopathy, and mult iorgan
involvement. The wart-like lesion on t he scrot um is condylomata lata. Alt hough darkf ield microscopy of a specimen from an act ive lesion may
. 5
yield t he diagnosis, t he f luorescent t reponema! ant ibody absorpt ion (FTA-ABS) test is the most specific serologic test for syphilis (caused by
.6 Treponema pa/lidum ). I t is import ant to note, howeve r~ t hat most patients will undergo Venereal Disease Research Laborat ory (VORL) or rapid
• 7 plasma reag in ( RPR) t esting first and t he resu lt will be confirmed with FTA-ABS, as it is t he most specific t est. Compared wit h the VORL t est,
t he FTA-ABS also tu rns posit ive earlier in t he disease and remains posit ive longer.
•8 Condylomata lata Treponema pallidum Rapid plasma reagin Syphilis Dark field microscopy Lymphadenopathy Chancre Fluorescent treponema! antibody absorption test Serology Antibody Secondary syphilis
•9 Microscopy Bacteremia Genital wart Lesion Venereal Disease Research Laboratory test Sexually transmitted infection Exanthem Primary syphilis
• 10
B is not correct. 5% chose this.
· 11
The heterophile ant ibody test forms the basis fo r the monospot test used t o diagnose Epst ein-Barr virus (EBV) mononucl eosis. This test is
• 12 specif ic for het erophile ant ibodies produced by t he immune syst em against EBV.
• 13 Epstein-Barr virus Heterophile antibody test Infectious mononucleosis Heterophile Antibody Immune system Virus

• 14
C is not correct. 8 °/o chose this .
• 15 The rapid plasma reagin test is t ypically done f irst in t he work up for syphilis. However, it is a nonspecif ic serologic t est fo r syphilis that often
. 16 produces false- positive results. This quest ion st em asked for t he most specific t est fo r diagnosing syphilis.
Rapid plasma reagin Syphilis Serology Blood plasma Type I and type II errors
. 17
• 18 D is not correct. 21% chose this .
• 19 The VORL t est is a nonspecif ic screening t est fo r syphilis. The t est det ects nonspeci fi c antibodies that react with beef cardiolipin. False-posit ive
results occur with some viral infect ions, some drugs, rheumat ic fever, syst emic lupus erythematosus, and leprosy.
• 20
Systemic lupus erythematosus Rheumatic fever Syphilis Leprosy Antibody Cardiolipin Lupus erythematosus Fever Screening (medicine) Type I and type II errors Venereal Disease Research Laboratory test
. 21
. 22
E is not correct. 5°/o chos e this .
The Weii - Felix react ion is a cl assic test for ri ckettsial infection such as Rocky Mountain spotted feve r ( RM SF), which is caused by Rickettsia
• 23
ricketsii. This patient 's rash closely resem bles t hat seen wit h RMSF; h oweve r~ the presence of con dylomat a lat a makes RMSF less likely.
• 24 Rocky Mountain spotted fever Condylomata lata Rickettsia Spotted fever Genital wart Infection Fever

• 25
• 26
Bottom Line:
. 27

8
Lock
s
Suspend
0
End Block
Item: 4 of 100 ~ ,• Mark <::J [:::> ""I ~· ~'j
QID: 2026 J.. Previous Next LAb faiUI~S Notes Calculator

1
2 Bottom Line:
3 The most specific t est fo r syphilis is t he FTA-ABS t est; t he VORL and RPR t ests each yield higher rat es of fa lse-posit ive f ind ings.
Syphilis Fluorescent treponema! antibody absorption test Venereal Disease Research Laboratory test Type I and type II errors
4
. 5
.6
• 7
i@l;fil·1i•J for year:[ 2017 • J
FIRST AID FACTS

•8
•9 FA1 7 p 143.2
. 10
VORL false positives VORL detects nonspeci fic antibody that False-positive results on VORL with:
· 11
. 12
reacts with beef ca rdiolipin. Quantitative, Viral infection (eg, EBV, hepatitis)
• 13
inexpensive, and widely available test for Drugs
• 14
syph ilis (sensitive but not specific). Rheumatic fever
• 15 Lupus and leprosy
. 16
. 17 FA1 7 p 143.1
• 18
Syphilis Caused by spirochete Treponema pallidwn .
• 19
. 20
Primary syphilis Localized disease presenting with painless chancre f.J. If available, use dark-field microscopy to
. 21
visualize treponemes in fluid from chancre Ill VORL ED in ~ 80%.
. 22 Secondary syphilis Disseminated disease with constitutional symptoms, macu lopapu lar rash 1!1 (including palms [!]
• 23 and soles), condylomata lata 0 (smooth, moist, painless, wart-like white lesions on genitals),
• 24 lymphadenopathy, patchy hair loss; also con firmable with dark-fi eld microscopy.
• 25 Serologic testing: VDRL/RPR (nonspecific), confirm diagnosis with specific test (eg, FTA-ABS).
• 26 Secondary syphilis = Systemic. Latent syphilis(® serology without symptoms) may foll ow.
. 27
8
Lock
s
Suspend
0
End Block
Item: 4 of 100 - ,• Mark -<J [:::> "'I ~ · ~
QIO: 2026 ~ P~v•ous N@xt Labl lues No tes Calcula t o r

Gummas 0 (chronic granulomas), aortitis (vasa ,·asorum destruction), neurosyphilis (tabes dorsalis,
A A
1
Tertiary syphilis
2
"general paresis"), Argyll Robertson pupil (cons! riels with accommodation but is not reactive to
3
light; also called "prostitute's pupil" since it accommodates but does not react).
4
Signs: broad-based ataxia, Et> Romberg, C harcot joint, stroke without hypertension.
·5
For neurosyphilis: test spinal fluid with DRL, FTA-ABS, and PCR.
·6
·7 Congenital syphilis Presents '' ith facial abnormalities such as rhagades (linear scars at angle of mouth, black arro\\
in [!!), snuffles (nasal d ischarge, red arro" in ~.saddle nose, notched (H utchinson) teeth Cl.
•9 mulberry molars, and short maxilla; saber shins; C1 Vlll deafness.
• 10 To pre,·ent. treat mother earl y in pregnancy, as placental transmission typically occurs after first
· 11 trimester.
• 12
• 13
• 14

• 15
. 16
. 17
• 18
• 19
• 20
. 21
. 22
. 23
• 24
• 25
• 26
. 27

8
Lode.
s
S uspe-nd
8
End Bloc:k
Item: S of 100 - ,• Mark -<J [:::> "'I ~ · ~
QIO: 2 195 ~ P~v•ous N@xt Labl lues No tes Calcula t o r

A A
1
A 47-year-old man with chronic asthma who has been t reated with high-dose steroids for many years beg ins experiencing a productive ~~AI
2
cough, weight loss, and night sweats. Imaging studies revea l abscesses in the lungs and brain. Culture shows gram-positive fi laments that
3 are weakly acid fast.
4
·5 What is the mechanism of action of the drug of choice for treating this infection?
·6 :
·7 A. Binds ergosterol and membrane pores in the outer membrane of the organism
·8 B. Decreases synthesis of mycolic acids
•9
C. Inhibition of dihydropteroate synthetase by PABA antimetabolites
• 10
· 11 0 . Inhibits formation of the initiation complex and misreading of mRNA by binding to the 305 ribosomal subunit
' 12 E. Inhibits peptidoglycan cross-linking to block bacterial cell wall synthesis
' 13
' 14
' 15
. 16
. 17
• 18

' 19
' 20
. 21
. 22
' 23
' 24
' 25
' 26
. 27

8
Lode.
s
Suspe-nd
8
End Bloc:k
Item: S of 100 ~ ,• Mark <::J [:::> ""I ~· ~'j
QID: 219 5 J.. Previous Next LAbfaiUI~S Notes Calculator

1
2
The correct a nswer is C. 4 2 °/o chos e this .
3
Nocardia asteroides is a f ilament ous gram -posit ive organism that is weakly acid-fast. I nfect ion with t his organ ism is typically seen in
4 immunocomprom ised indiv iduals (eg, someone who has taken steroids for long periods). Inhaled by an immunocompromised person,
s Nocardia produces lung abscesses and cavitat ions. The bacteria may erode blood vessels and disseminat e humorally, leading to abscesses in
.6
t he brain and elsewhere. Nocardia infect ion is commonly m isdiagnosed as t uberculosis because of its acid-fast nat ure and its sim ilar disease
process. The drugs of choice for t reat ing Nocardia infect ions are t he sulfo namides, which act by blocking nucleot ide synt hesis by inhibit ing
• 7 dihyd ropteroate synthetase.
•8 Nocardia asteroides Nocardia Gram-positive bacteria Tuberculosis Sulfonamide (medicine) Immunodeficiency Acid-fast Organism Bacteria Lung Abscess Nucleotide Hypha Infection

•9 A is not correct . 11% chos e this.


• 10 This describes am phot eri cin B, which is used to t reat syst em ic f ungal infect ions. Alt hough Nocardia is filamentous, it is stil l a bacterium and is
· 11 not t reated wit h am phot eri cin B.
Amphotericin 8 Nocardia Mycosis Fungus Bacteria Hypha
• 12
• 13 B is not correct. 23% chose this .
• 14 This describes the mechanism of isoniazid, which is used to t reat Mycobacterium tuberculosis infect ion. The symptoms of t uberculosis are
similar to those of t he disease caused by N. asteroides, and bot h M. tuberculosis and Nocardia are acid fast ; however, Nocardia species are
• 1S
gram-posit ive filamentous bacteria, whereas M. tuberculosis does not Gram stain .
. 16 Gram staining Isoniazid Mycobacterium tuberculosis Tuberculosis Nocardia Gram-positive bacteria Mycobacterium Acid-fast Bacteria Infection

. 17
D is not correct. 1 2% c hose this .
• 18
This describes the mechanism of aminoglycosides, which are generally used to t reat severe gram -negative rod infect ions.
• 19 Gram-negative bacteria Aminoglycoside

• 20
E is not correct. 1 2°/o chose t his .
. 21 This describes the mechanism of penicillin, which is used to t reat infection wit h Actinomyces israelii, a filamentous organism . However, unlike
. 22 Nocardia, A. israelii is not weakly acid fast. It is part of t he normal flora of the mouth and gastrointest ina l t ract but can cause abscesses and
• 23 invasive infections, most commonly in the head/neck, oropharynx, and abdomen. A. israelii easily t rave rses t issue planes, so it is known for
causing draining sinus t racts. It is classically associated wit h the finding of "sulfur granules" on m icroscopy .
• 24 Penicillin Actinomyces Nocardia Pharynx Gastrointestinal tract Actinomyces israelii Acid-fast Sulfur Abscess Hypha Infection Human microbiota Human gastrointestinal tract Organism Microscopy
• 2S
• 26
. 27
Bottom Line :
8
Lock Suspend
s 0
End Block
Item: S of 100 - ,• Mark -<J [:::> "'I ~ · ~
QIO: 2 195 ~ P~v•ous N@xt Labl lu es No tes Calcula t o r

A A
1
Botto m Li n e:
2
Nocardia asteroides is a fi lamentous, gram- posit ive, weakly acid-fast organism that can cause lung abscesses in immunocompromised
3 patients. Infection is treated with sulfonamides, which act by inhibiting dihydropteroate synthet ase.
4 Noc~trd11 ~tero1des Nocard1a Gram-pos1bve bacteria Immunodeficiency Sulfonamide (medicine) Acld-fMt Absceh Organ1sm Lung

5
·6
·7 iiJi ifii·11•J f or
FIRST AID FAC'"'S
y ear: 2017 •

·8
•9 FA17 p 135.3
• 10
Nocardiavs Both are gram E!) and fonn long, branching filaments resembling fungi.
· 11
Actinomyces
' 12 Nocardia Actinomyces
' 13
Aerobe naerobe
' 14
' 15
Acid fast (weak) fJ 1 ot acid fast [l]
. 16 Found in soil ormal oral, rcproducti,·e, and C l Aora
. 17
Causes pulmonary in fectio ns in Causes oral/facial abscesses that drain through
• 18
immunocompromised (can mim ic TB bul sinus tracts; often associated with denta l caries/
' 19
with 8 PPO); cutaneous infections after extraction; forms yellow "sulfur granules"; ca n
' 20
trauma in immunocompclenl; can spread to also cause PID with IUDs
. 21
. 22
c 1$

' 23 Treat with sulfonamides (TMP-SMX) Treat with pcniciJlin


' 24 Treatment is a S~ \ P: Sulfonamidcs-Nocardia; \ctinomyces-Penicillin
' 25
' 26
. 27

8
Lode.
s
Suspe-nd
8
End Bloc:k
Item: S of 100 - ,• Mark -<J [:::> "'I ~ · ~
QIO: 2 195 ~ P~v•ous N@xt Labl lues No tes Calcula t o r

A A
1
FA17 p 136.2
2
3 Mycobacteria Mycobacterium tuberculosis (TB, often resistant TB symptoms include fever, night S\\'eats,
4 to multiple drugs). weight loss, cough (nonproductive or
5 M avium-intracellulare (causes disseminated, producti\'e), hemoptysis.
·6 non-TB disease in AIDS; often resistant to Cord factor creates a "serpentine cord"
·7 multiple drugs). Proph)laxis ''ith a.lithrom}cin appearance in 'irulent 1\1 tuberculosis
·8 ''hen CD4+ count< )0 cells/mm 1. strains; acti\'ates macrophages (promoting
•9 M scro{ulaceum (cervical lymphadenitis in granuloma formation) and induces release of
• 10 children). Tl'\F-u . Sulfatides (surface glycolipids) inhibit
· 11 M marinum (hand infection in aquarium phagolysosomal fusion.
' 12 handlers).
' 13
All mycobacteria are acid-fast organisms (pink
' 14
rods; arrows in rJ).
' 15
. 16
. 17 FA17 p 190.1
• 18
Sulfonamides Sulfamethoxazole (S lX), sulfisoxazole, PABA+ Pteridine
' 19
' 20
. 21 MECHANISM
sulfacl iazine.
Inhibit dihydropteroate synthase, thus inhibiting
Drhyaropterc.o,c
synthilse
JI-G- Sulfonamides,
dapsone
. 22 folate synthesis. Bacteriostatic (bactcricida I
Dihydropteroic acid
' 23 when combined with trimethoprim).
' 24
CLINICAl USE Cram <:B, gram 8, 1 ocardia. SMX for simple
' 25
UTI.
' 26
. 27 ADVERSE EFFECTS l lvocrsensitivih· reactions. hcmolvsis if G6PD
8
Lode.
s
Suspe-nd
8
End Bloc:k
Item: S of 100 - ,• Mark -<J [:::> "'I ~ · ~
QIO: 2 195 ~ P~v•ous N@xt Labl lues No tes Calcula t o r

A A
1
All mycobacteria are acid-fast organisms (pink
2
3
rods; arrows in t'J).
4
5
FA17 p 190.1
· 6
Sulfonamides Sulfamethoxazole (S:\ lX), sulfisoxazole, PABA + Pteridine
· 7
·8
•9 MECHANISM
sulfad iazine.
Inhibit dihydropteroate synthase, thus inhibiting
Dhyol '"'w "
cvntivl<e
l-0- Sulfonamides,
dapsone
• 10 folate synthesis. Bacteriostatic (bactericidal +
Dihydropteroic aod
· 11 when combined with trimethoprim).
' 12
CliNICAl USE Gram G:>, gram 8 , 'ocardia. \1 for simple
' 13
' 14
UT I.
' 15 ADVERSE EFFECTS llypersensitivity reactions, hemolysis if C6PD
. 16 deficient, nephrotoxicity (tubulointerstitial
Dihydrofolic acid
neph ritis), photosensitivity, Stevens-johnson
·- jl-0-
. 17
• 18 syndrome, kernicterus in infants, displace D ,1drofol Trimethoprim,
' 19 other drugs from albumi n (eg, warfarin). reduct"a pyrimethamine
' 20
MECHANISMOF RESISTANCE Altered enzyme (bacterial dihrdroptcroate Tetrahydrofolic acid
. 21
. 22
synthase), t uptake, or t PABA synthesis.
/!"'-.
Purines Thymidine Methionine
' 23
' 24
' 25
l
DNA. RNA
l
DNA
l Protein
' 26
. 27 •
8
Lode.
s
Suspe-nd
8
End Bloc:k
Item: 6 of 100 - ,• Mark -<J [:::> "'I ~ · ~
QIO: 1729 ~ P~v•ous N@xt Labl lues No t es Calcula t o r

A A
1
A 40-year-old woman presents with complaints of burning during uri nation. Urinalysis shows t he presence of leukocytes and a subsequent ~~AI
2
Gram stain shows numerous gram-posit ive cocci. The organism is cultured in the laboratory for 48 hours whi le t he patient is t reat ed with
3 trimethoprim-sulfamethoxazole. Two days later, cult ures grow out pan-susceptible gram-positi ve cocci in sheep's blood agar with no visible
4 discoloration in the media.
5
·6
Which of the following is the first-line t reatment for the probable cause of this patient's infection?
·7 :

·8
A. Ampicillin
•9 B. Doxycycline
• 10
C. Gentamicin
· 11
D. Piperacillin
' 12
' 13 E. Quinupristin/dalfopristin
' 14
' 15
. 16
. 17
• 18

' 19
' 20
. 21
. 22
' 23
' 24
' 25
' 26
. 27

8
Lode.
s
Suspe-nd
8
End Bloc:k
Item: 6 of 100 ~ ,• Mark <::J [:::> ""I ~· ~'j
QID: 1729 J.. Previous Next LAbfaiUI~S Notes Calculator

1
2
3 The correct a nswer is A. 6 1 °/o chose t h is.
4 These lab result s are consistent with Staphylococcus epidermidis, Staphylococcus saprophyticus, or Enterococcus. However, S. epidemidis is a
common skin commensal with low pat hogenic potent ial, and it is frequent ly cultu red as a contaminant. S. saprophyticus is part of t he normal
5
female genit al t ract fl ora and is ra rely isolat ed in UTis ot her t han t hose occuring in sexually act ive adolescent females. Enterococcus is most
6 likely the pathogen in t his case. Enterococcus is a common cause of nosocomial urinary t ract infection and subacut e endocardit is.
• 7 Enterococcus is a catalase-negat ive bact erium t hat grows in chains. Most cases of Enterococcus infect ions are associat ed with
•8
inst rument ation, such as a urinary cat het er, but not all. Ampicillin is the standard first -line t reatment for suscept ible enterococcal infect ions,
as in t his case, and gentamicin may be added in cases of endocardit is. Many strains of Enterococcus are resist ant to am picillin; add it ionally,
•9 even ampicillin-susceptible strains may produce ~ -l actamases when occurring in large quant it ies (as in t he case of a valve vegetation) .
• 10 Vancomycin can be used for coverage of ampicillin -resistant strains. For vancomycin -resistant st ra ins, linezolid is commonly used. It is an
oxazolidi none ant ibiotic t hat acts by binding t o t he 50S subunit of t he bacterial ri bosome and prevent ing formation of the init iat ion complex.
· 11
Linezolid can also be used for t reatment of MR5A infect ions .
• 12 Urinary tract infection Linezolid Gentamicin Ampicillin Commensalism Staphylococcus epidermidis Ribosome Vancomycin 2-0xazolidone Staphylococcus saprophyticus Antibiotics Endocarditis Enterococcus
• 13 Methicillin-resistant Staphylococcus aureus Staphylococcus Hospital-acquired infection Catheter Urinary system Pathogen Bacteria 50S Urinary catheterization Infection Catalase
• 14 Subacute bacterial endocarditis
• 15
B is not correct. 14% chose this .
. 16
Doxycycline is a bacteriostat ic ant ibiot ic t hat binds t o t he 305 ri bosomal subunit to prevent attachment of aminoacyl-t RNA. It is effective
. 17 against atypical pneumonias and Borrelia burgdolferi, Rickettsia, and Chlamydia infections.
• 18 Doxycycline Borrelia burgdorferi Bacteriostatic agent Antibiotics Rickettsia 305 Chlamydia infection Chlamydia (genus) Borrelia Aminoacyl-tRNA Ribosome

• 19 C is not co rrect. 10% chose this .


• 20 Gentamicin is an aminoglycoside ant ibiotic. It in hibits the 305 subunit by inhibit ing the formation of the initiat ion complex. It is not an
. 21 effect ive monot hera py for ent erococci, but is often used in combi nation wit h ampicillin to t reat ent erococcal endocardit is .
Aminoglycoside Gentamicin Ampicillin Enterococcus Endocarditis Antibiotics 305
. 22
• 23 D is not correct. 9°/o chose t his .
• 24 Piperacillin is an ext ended-spect rum penicillin agent t hat inhibi ts bact eri al cell wall synt hesis. I n addit ion to gram -posit ive organisms, it is
particularly act ive against Pseudomonas aeruginosa and t he Ent erobact eri aceae. It is freq uent ly used with a ~ -l actamase inhibit or such as
• 25
t azobacta m .
• 26 Piperacillin Enterobacteriaceae Pseudomonas aeruginosa Tazobactam Penicillin Gram-positive bacteria Cell wall Pseudomonas Extended-spectrum penicillin

. 27

8
Lock
s
Suspend
0
End Block
Item: 6 of 100 - ,• Mark -<J [:::> "'I ~ · ~
QIO: 1729 ~ P~v•ous N@xt Labl lues No t es Calcula t o r

1 •
E is not correct. 6°/o c hose thi s.
2
Quinuprist in/dalfopristin (Synercid) is a new therapy that may be effective for vancomycin-resistant Enterococcus endocarditis. It is not first-
3 line treatment for Enterococcus infections.
4 Qutnupnsttn/dltlfopn5tln Endocarditis Enterococcus

5
6 Bottom Line :
·7
Ampicillin is standard treatment for susceptible enterococcal infections.
Ar'lptc ~ Ef'ttroc O\.~~.u~
•9
• 10
· 11 141ifil·1i•l fo r year:
fiRST AID fAC TS
2017 •
• 12
• 13
FA17 p 133.3
• 14

• 15 Enterococci Gram$ cocci. Enterococci (E {aecalis and Enterococci, hardier than nonenterococcal
. 16 E {aecium) are normal colonic Oom that arc group D, can grow in 6.5% NaCI and bile (lab
. 17 penicill in G resistant and cause UTI, biliary test).
• 18 tract infections, and subacute enclocard it is Entero = intestine, faecalis = feces, strepto =
• 19 (following Gl!GU procedures). Catalase 8 , twisted (chains), coccus= berry.
• 20 PYR $, ,·ariable hemolysis.
. 21 VRE (vancomycin-resistant enterococci) are an
. 22
important cause of nosocomial infection .
. 23
• 24
• 25
FA1 7 p 184.1
• 26 Penicillinase-sensitive Amoxicillin, ampicillin; aminopcnicillins.
. 27 - - - ! - ! 11: - -

8
Lode.
s
Suspe-nd
8
End Bloc:k
Item: 6 of 100 - ,• Mark -<J [:::> "'I ~ · ~
QIO: 1729 ~ P~v•ous N@xt Labl lues No t es Calcula t o r

A
1 A

FA17p184.1
2
3 Penicillinase-sensitive Amoxicillin, ampicillin; aminopenicillins.
4 penicillins
5 MECHANISM Same as penicillin. Wider spectrum; \ \ tinoPenicillins are AXIPed-up penicillin.
6
penicillinase sensiti\'e. lso combine with mO \icillin has greater O ral bioavailabilit)
· 7
cla"ulanic acid to protect against destruction than ampicillin.
·8
br ~-la c ta m ase.
•9
• 10 Cll NICAl USE Extended-spectrum penicillin - [ I inf/uen::ae, Co,·erage: ampicillin/amoxicillin I II IELPSS
· 11 II pylori, E coli, Listeria monocytogenes, kill enterococci.
• 12 Proteus mirabilis, Salmonella, Shigella,
• 13 enterococci.
• 14
ADVERSE EFFECTS Hypersensitivity reactions; rash;
• 15
pseudomembranous colitis.
. 16
. 17 MECHANISM OF RESISTANCE Penicillinase in bacteria (a type of ~-lac ta mase)
• 18 cleaves ~-lacta m ring.
• 19
• 20
FA17 p 184.3
. 21
. 22 Antipseudomonal Piperacillin, ticarcill in.
. 23 penicillins
• 24 MECHANISM Same as penicillin. Extended spectrum .
• 25
CLINICAl USE Pseudomonas spp. and gram 8 rods; susceptible to penicillinase; use with ~-lactamase inhibitors.
• 26
. 27 • ADVERSE EFFECTS llvner!'ensitivitv re<~cl ion!'.
8
Lode.
s
Suspe-nd
8
End Bloc:k
Item: 7 of 100 - ,• Mark -<J [:::> "'I ~ · ~
QIO: 2050 ~ P~v•ous N@xt Labl lues No tes Calcula t o r

1
A While on rounds in the ICU, you present t he case of a 67-year-old male who received a kidney t ransplant 4 mont hs ago, now presenting
2 with a fever of 38.9° C (102°F) and a productive cough. Fearing that pneumonia is developing, t he resident suggest s taking a sputum
3 sample. Later in the day you receive resu lts f rom t he pathology lab. An acid-fast stain of this patient's sputum is shown in the image
below.
4
5
6
·7
·8
•9
• 10
· 11
• 12
• 13
• 14

• 15
. 16
. 17
• 18 Given these resu lts in conj unction wit h t he pat ient's clinical picture, the attending asks you to name the medicat ion t hat will best eradicate this
• 19 patient's infection. You respond by stat ing t hat he should prescri be:
• 20 :
. 21 A. Fluconazole
. 22 B. Gentamicin
. 23
C. Metronidazole
• 24
• 25 D. Penicillin
• 26 E. Trlmethoprim-sulfamethoxazole
. 27

8
Lode.
s
Suspe-nd
8
End Bloc:k
Item: 7 of 100 ~ ,• Mark <::J [:::> ""I ~· ~'j
QID: 2050 J.. Previous Next LAb faiUI~S Notes Calculator

1
2
3 The correct a nswer is E. 5 1 °/o chose this .
4 The cause of t his pat ient's pulmonary symptoms is most likely Nocardia asteroides. The image shows long, branching f ilaments. Organisms
5 with long, branching f ilaments are fu ngi and the bacteria Nocardia asteroides and Actinomyces israelii. Fungi would not necessarily be evident
on acid-fast st ain. I n addit ion, t he f ilaments of fungi t end t o be thicker than t hose shown in t his stain. Therefore, bacteria are more likely. The
6
clinical scenario is important to help distinguish between N. asteroides and A. israe/ii. Due to his rece nt kidney t ransplant, this man is very
7 likely to be immunocompromised and is at ri sk of developing pulmonary infect ion from N. asteroides. A. israe/ii, on t he other hand, causes
•8 oral and facial abscesses. Although N. asteroides is weakly acid fast , A. israelii is not . The in itia l t reat ment of choice for Nocardia pneumonia
is a sulfa- based ant ibiotic, such as sulfamet hoxazole, given int ravenously in high doses. Following this, imipenem/cilastatin is given fo r at
•9
least 4 weeks .
• 10 Nocardia asteroides Nocardia Actinomyces israelii Actinomyces Fungus Sulfonamide (medicine) Immunodeficiency Antibiotics Pneumonia Acid-fast Bacteria Kidney Sulfamethoxazole Abscess

· 11 Intravenous therapy
• 12
A is not correct . 20% chose this .
• 13
Fluconazole is an antif ungal medication . Although Nocardia asteroides can resemb le the hyphae of growing fu ngi, the infect ion will not be
• 14 resolved wit h t his medication .
Antifungal Fluconazole Nocardia asteroides Fungus Nocardia Hypha Fungicide
• 15
. 16 B is not correct. 9 % chose t his .
. 17 Gentamicin is an aminoglycoside ant ibiotic that is useful for t he t reatment of infect ions wit h gram- negative bacilli. Alt hough it is not used by
• 18 itself to t reat Nocardia asteroides infect ions, it can be added t o a sulfa -based ant ibiot ic as an adjunct ive therapy fo r fulminant nocardiosis .
Aminoglycoside Gentamicin Nocardia asteroides Antibiotics Gram-negative bacteria Nocardia Sulfonamide (medicine) Nocardiosis Bacilli Bacillus (shape)
• 19
• 20 C is not co rrect. 9 °/o chose this .
. 21
Met ronidazole is used fo r the t reat ment of giardiasis, amebiasis, bacterial vaginitis, t ri chomoniasis, and Clostridium difficile colit is.
Metronidazole Giardiasis Trichomoniasis Amoebiasis Clostridium difficile colitis Colitis Clostridium difficile (bacteria) Vaginitis Clostridium
. 22
• 23 D is not correct. 11% c hose this .
• 24
Penicillin is ineffective against Nocardia asteroides. However, it is t he t reatment of choice fo r Actinomyces species infections, which can appear
similarl y on histology ( long branching f ilaments) . However, Actinomyces typically cause oral or facial abscesses and not pneumonia .
• 25 Nocardia asteroides Penicillin Nocardia Actinomyces Histology Pneumonia Abscess
• 26
. 27

8
Lock
s
Suspend
0
End Block
Item: 7 of 100 - ,• Mark -<J [:::> "'I ~ · ~
QIO: 2050 ~ P~v•ous N@xt Labl lues No tes Calcula t o r

1 •
Bottom Li ne:
2
3
Nocardia asteroides causes pulmonary infection in immunocompromised hosts and is t reated wit h a sulfa- based ant ibiotic, whereas
Actinomyces israelii causes oral/facial abscesses and is t reated with penicillin-based antibiotics.
4 Noc~trdu1 1Ktero1de11 Nocardia Sulfonamide (medicine) Actinolll(ces israelii Actinomyces ImmunodefiCiency Antlb10t1CS Abscess Infection

5
6
7 141 ifii·11•J for ye a r :
FI RST AID FAC"'S
2017 •
·8
•9
FA17 p 190.1
• 10
· 11 Sulfonamides Sulfamethoxazole (S:\IX), sulfisoxazole, PABA t Pteridine
• 12
• 13 MECHANISM
sui fad iazine .
Inhibit clihyclropteroate synthase, thus inhibiting
0 hyo IJ!ll!:tuo
<vntM~e
!1--0- Sulfonamides,
dapsone
• 14
folate synthesis. Bacteriostatic (bactericidal
• 15
Oihydropteroic acid
when combined with trimethoprim).
. 16
. 17
CLINICAl USE Cram Ef>, gram 8 , Nocardia. SMX for simple
• 18 UTI.
• 19 ADVERSE EFFECTS l.lypersensitivity reactions, hemolysis if C6PD
• 20 deficient, nephrotoxicity (tubulointerstitial
. 21 nephritis), photoscnsil ivi ty, Stevens-Johnson Oihydrofotic acid
. 22
. 23
syndrome, kernicterus in infants, displace
other drugs from albumin (eg, warfarin).
0 1-.rofo
·~ucr •a
-jl--0- Trimethoprim•
pyrimethamine
• 24
• 25
MECHANISM OF RESISTANCE Altered enzyme (bacterial dihydroptcroate Tetrahydrofolic acid
synthase), l uptake, or t PABA synthesis.
• 26
. 27 Purines
/!"" Thvmidine Methionine

8
Lode.
s
Suspe-nd
8
End Bloc:k
Item: 7 of 100 - ,• Mark -<J [:::> "'I ~ · ~
QIO: 2050 ~ P~v•ous N@xt Labl lu es No tes Calcula t o r

A
1 A

FA17 p 135.3
2
3 Nocardiavs Both are gram® and form long, branching filaments resembling fungi.
Actinomyces
4 Nocardia Actinomyces
5
Aerobe Anaerobe
6
7 Acid fast ("·eak) rJ 1 ot acid fast [l]
·8 Found in soil Normal oral, reproducti,·e, and C I Aora
•9
Causes pulmonary infections in Causes oral/facial abscesses that drain through
• 10
immunocompromised (can mimic TB but sinus tracts; often associated \\'ith dental caries/
· 11
• 12 D with 8 PPD); cutaneous infections after extraction; forms yellow "sulfur granules"; can
• 13
trauma in immunocompetent; can spread to also cause PID with IUDs
• 14 Ct\S
• 15 Treat with sulfonamides (T.MP-SM ) Treat with penicillin
. 16
Treatment is a SNAP: Sulfonamicles-Nocardia; \ctinomyces- Penicil lin
. 17
• 18
• 19
• 20
. 21
. 22
FA17 p 123.2
. 23
• 24 Aerobes Use an Oz-dependent system to generate ATP. Nagging Pests l\l ust Breathe.
• 25 Examples include :-.locardia, Pseudomonas
• 26 aeruginosa, and \1)'coBacterium tuberculosis.
. 27 • 0 ,.. .... ,..&.: ••, &. :,.. ...... ,...( ' I I •• J.. .......,..•• / ,..1\:A /,..,. ,..(, _ ,..

8
Lode.
s
Suspe-nd
8
End Bloc:k
Item: 8 of 100 - ,• Mark -<J [:::> "'I ~ · ~
QIO: 1024 ~ P~v•ous N@xt Labl lues No tes Calcula t o r

6
1
A moth er brings her 12-year-old daught er to an out patient clinic. The child complains of aching pain localized to the joints of t he
2 extremit ies. The mother recalls her daughter was sick with a sore throat about a month ago, but recovered complet ely with out medical
3 attention. The girl is admitted t o t he hospit al for fu rt her examinat ion and testing. A t issue biopsy is taken, and t he abnorma l resul ts are
seen in the image.
4
5
6
7
·8
•9
• 10
· 11
• 12
• 13
• 14

• 15
. 16
. 17
• 18 Based on the patient 's diagnosis, which of t he follow ing murm urs is most likely to develop in adulthood?
• 19
:
• 20
. 21
A. A fri ction ru b heard th roughout th e precordium
. 22
. 23 B. A harsh crescendo-decrescendo early syst olic murm ur heard at the right upper st ernal border with radiation t o t he carot ids
• 24 C. A mid-diastolic murmur heard best at the apex
• 25
D. A midsystolic click heard best at the apex
• 26
. 27 E. An 54 qallop heard at t he apex
8
Lode.
s
Suspe-nd
8
End Bloc:k
Item: 8 of 100 - ,• Mark -<J [:::> "'I ~ · ~
QIO: 1024 ~ P~v•ous N@xt Labl lues No tes Calcula t o r

A A
1
The correct answ er is c. 52°/o chose this.
2
The image in the vignette reveals Aschoff nodules, which are pathognomonic for rheumatic fever.
3
Rheumatic fever is caused by group A streptococci. Years after a bout of rheumatic fever, rheumat ic heart
4 disease can develop due to calcificat ion of warty vegetations on fibrotic healing, most commonly on the
5 mitral valve. As a result, it most commonly causes mitral stenosis (shown in this image), which in mild
disease causes a mid-diastolic opening snap followed by a low-pitched murmur heard best at the apex. In
6
the United States, the incidence of this mit ral stenosis secondary to rheumatic fever is low, given the
7 widespread use of antibiotics. However, rheumatic mitral stenosis is prevalent in immigrant populations.
8 RJ\e.• c f~ret ,. rra. • 1e .tenos•~ Pathognomonic Mitral valve Gr-oup A streptococcal •nfectJon Steno St-' tococcus C.-d•ovascular disease Fever-

•9 Anbb•obcs Calc•f•cat:Jon f•brostS Heart murmur

• 10 Image courtesy of CDC/ Dr.


· 11 Edwin P. Ewing, Jr.
• 12
A is not correct. 7 % chose this .
• 13
A friction rub heard throughout the precordium would correspond to pericarditis, an inflammation of the pericardia! sac that leads to friction
• 14 with an expanding and contracting myocardium . It is not a long-term complication of rheumatic fever. Rather, a friction rub may be associated
• 15 with malignancy, uremia, active infections such as viral or tuberculous, and other rare causes .
Urem1a Rheumatic fever Pencard1bs Pencard1um Cardiac muscle Precordium Maltgnancy Inflammation Pericard1al frict1on rub Tuberculosis Fever Cancer
. 16
. 17 B is not correct. 11 °/o chose this .
• 18 Aortic stenosis is represe nted by a harsh crescendo-decrescendo early systolic murmur heard best at t he rig ht upper sternal border with
• 19
radiation to the carot ids. Rheumatic heart disease can lead t o calcification of the aortic valve, which leads t o aortic st enosis, but fo r unclear
reasons, t his occu rs much less f req uently t han on t he mit ral valve. An aortic stenosis murm ur in a middle-aged woman should raise t he
• 20 suspicion for a calcified bicuspid aort ic valve (remember t he "f ish mouth" ) .
. 21 Aortic stenosis Rheumat•c fever B1cusp1d aort1c valve Aortic valve Mitral valve Systole Stenosis Calcification Sternum Cardiovascular d1sease Carotid artery Common carotid artery Heart murmur

. 22 D is not correct. 25°/o chose this.


. 23 A systolic click heard best at the apex corresponds to mitral valve prolapse, which is a relatively common and benign finding in middle-aged
• 24 women. It occurs when an abnormally t hickened mitral valve leaflet displaces into the left atrium during vent ricular systole. It may lead to
mitral regurgitation and ultimately valve surgery may be required, but typically it is followed by a cardiologist with serial exams and
• 25
echocardiography. Mitral valve prolapse may rarely be caused by acute rheumatic fever, but subsequent ret ract ion of valve tissue makes mitral
• 26 valve prolapse an unlikely sequelae of chronic rheumatic fever, such as is seen in this patient.
. 27 M1tnl "' e orolii'O e teumdtlc fever Svstole EchocardioaraDhv Mitral nsuffic1encv M1tral v~ve tr1um (her-+) C rd1ctc: cvcle Reaura1tat:Jon (Circulation) cardioloav Prolaose SeQuela Fever Ventr1cle (hebrt)

8
Lode.
s
Suspe-nd
8
End Bloc:k
Item: 8 of 100 ~ ,• Mark <::J [:::> ""I ~· ~'j
QID: 1024 J.. Previous Next LAb faiUI~S Notes Calculator

1
E is not correct. 5 °/o chos e this .
2 An 54, which is heard best at t he apex and is associat ed with concentric left ventri cular hypert rophy, is indicat ive of eit her chronic extensive
3 afterload (ie, f rom uncont rolled hypertension) or long-standing aortic stenosis. It also can be caused by diast olic heart failure because t he left
4
vent ricle is not as compliant as it should be. The exam finding is due to turbulent blood flow caused by blood fil ling a stiff ventricle. An 5 4 is
not associat ed with rheumatic heart disease.
5 Left ventricular hypertrophy Rheumatic fever Aortic stenosis Afterload Ventricle (heart) Hypertension Ventricular hypertrophy Diastole Stenosis Hypertrophy Diastolic heart failure Heart failure
6 Cardiovascular disease Heart
7
8
•9
Bottom Line :
• 10
Rheumatic heart disease, uncommon in t he Unit ed St at es, occurs after pharyngeal infect ion with group A streptococci. High-pressure valves
are affected f irst; hence mit ral, and then aort ic, and final ly t ri cuspid valves are involved . Aschoff bodies are seen on m icroscopy.
· 11 Rheumatic fever Group A streptococcal infection Streptococcus Cardiovascular disease Aschoff body Pharynx Tricuspid valve Rheumatism I nfection

• 12
• 13
• 14 i@l;fil·1i•J for yea r:[2017 • J
FIRST AID FACTS
• 15
. 16
FA17 p 279.1
. 17
Heart murmurs
• 18
• 19 Systolic
• 20
Aortic stenosis Crescendo-decrescendo systolic ejection murmur (ejection click may be present).
. 21
LV >> aortic pressure during systole. Loudest at heart base; radiates to carotids.
. 22 Sl 52
"Pulsus parvus et tardus" -pulses are weak with a delayed peak. Can lead to
• 23
• 24 ~I Syncope, Angina, and Dyspnea on exertion (SAO). Most commonly clue to age-
related calcification in older patients (> 60 years old) or in younger patients with
• 25
early-onset calcification of bicuspid aortic v<llve.
• 26
. 27 Mitral/tricusoid reauraitation Holosvstolic. hivh-nitchecl "hlowinv mu rmu r."
8
Lock
s
Suspend
0
End Block
Item: 8 of 100 ~ ,• Mark <::J [:::> ""I ~· ~'j
QID: 1024 J.. Previous Next faiUI~S
LAb Notes Calculator

1
FA17 p 279.1
2
Heart murmurs
3

4
Systolic
5 Aortic stenosis Crescendo-decrescendo systolic ejection murmur (ejection click may be present).
6 LV >> aortic pressure during systole. Loudest at heart base; radiates to carotids.
51 52
7 "Pulsus parvus et tardus"-pulses are weak with a delayed peak. Can lead to
8
•9
~I Syncope, Angina, and Dyspnea on exertion (SAO). Most commonly clue to age-
related calcification in older patients (> 60 years old) or in younger patients with
• 10
early-onset calcification of bicuspid aortic v<llve.
· 11
• 12
Mitral/tricuspid regurgitation Holosystolic, high-pitched "blowing murmur."
• 13 Mitral- loudest at apex and rad iates toward axilla. MR is often due to ischemic heart
51 52
disease (post-MI), fVP, LV dilatation.
~
• 14

• 15 Tricuspid- loudest at tricuspid area. T R common ly caused by RV di latation .


. 16 Rheumatic fever and infective endocarditis can cause either MR or T R.
. 17 Mitral valve prolapse Late systolic crescendo murmur with midsystolic click (MC; due to sudden tensing
• 18
of chordae tend incae). Most frequen t valvular lesion. Best heard over apex. Loudest
• 19 51 MC 52
just before S2. Usually benign. Can predispose to infective endocarditis. Can be
• 20
. 21
I LMWJI caused by myxomatous degeneration (1° or zoto connective tissue disease such as
Ylarfan or Ehlers-Danlos syndrome), rheumatic fever, chordae rupture.
. 22
• 23 Ventricular septal defect llolosystolic, harsh-sounding murmur. Loudest at tricuspid area.
• 24
51 52
• 25
• 26
. 27
~
8
Lock
s
Suspend
0
End Block
Item: 8 of 100 - ,• Mark -<J [:::> "'I ~ · ~
QIO: l 024 ~ P~v1ous N@xt Labl lues No tes Calcula t o r

1
FA17 p 132.4

2
3
Streptococcus Gram <31 cocci in chains rJ. Group A strep J¥1\ ES (major criteria for acute rheumatic
4
pyogenes (group A cause: re, er):
5 streptococci) • Pyogenic - pharyngitis, cellulitis, impetigo Joints-polyarthritis
6 ("honey-crusted " lesions), erysipelas ¥ -carditis
7 • Toxigenic-scarlet fever, toxic shock- like :\odules (subcutaneous)
8 syndrome, necrotizing fasciit is Erythema marginatum
•9 • Immunologic- rheumatic fever, Sydenham chorea
• 10 glomerulonephritis Pharrngitis can result in rheumatic ''phever"
· 11 Bacitracin sensiti,·e, ~-hemolytic, p)Trol idonyl and glomerulonep h ritis.
• 12 arylamidase (PYR) $. Hyaluronic acid capsule Impetigo usuall) precedes glomerulonephritis.
• 13 c inhibits phagoc}1osis. Antibodies to M protein Scarlet fever-blanching, sandpaper-like body
• 14
enhance host defenses aga inst S pyogenes but rash, strawberry tongue, a nd circu moral
• 15
can g ive rise to rheumatic fe,·er. pallor in the setting of group streptococca l
. 16
ASO tiler or anli-0 asc Bantibodies indicate pharyngitis (erythrogenic toxin $).
. 17
recent S pyogenes infection .
• 18
• 19
• 20 FA17 p 300. 1
. 21
Rheumatic fever A consequence of pharyngeal infection with J• J\ES (major criteria):
. 22
. 23
g roup A ~-h e molytic streptococci. Late Joint (migratory polyarthritis)
• 24
sequelae include rheumatic heart disease, • (carditis)
• 25 which affects heart vak es- m itral > aortic>> Nodules in skin (subcutaneous)
• 26 t ricuspid (high-pressure va l, es a ffected most). Erythema marginatum
. 27 Earh· lesion is mitral ,·alve reJmmitation: s ,·denham chorea
8
Lode.
s
Suspe-nd
8
End Bloc:k
Item: 9 of 100 - ,• Mark -<J [:::> "'I ~ · ~
QIO: 2047 ~ P~v•ous N@xt Labl lues No tes Calcula t o r

1 A A 6-year-old girl presents to her family physician complaining of a lesion on her chin for 5 days. Physical examination reveals a lesion on
2 the jaw line with a thick adherent crust and several enlarged cervical lymph nodes. Gram stain of t he cu lt ure is shown.
3

4
5
6
7
8
·9
• 10

,.,
· 11
' 12

,.
' 13
I
' 14
10J1m
' 15
. 16 Image courtesy of Wikimedia Commons
. 17
• 18 Which of fo llowing is the most likely sequela of this infect ion?
' 19 :
' 20 A. Dental caries
. 21 B. Hematu ria
. 22
C. Jaundice
' 23
' 24 D. Meningitis
' 25 E. Otitis media
' 26
F. Scalded skin syndrome
. 27

8
Lode.
s
Suspe-nd
8
End Bloc:k
Item: 9 of 100 - ,• Mark -<J [:::> .il ~ · ~
QIO: 2047 ~ P~v•ous N@xt LabValues No tes Calcula t o r

A A
1
2
(. )
3

4
"f'J
5
6
7
8 /
·9

• 10
· 11
• 12
• 13
I
,. 10pm
• 14
Image courtesy of Wikimedia Commons
0
15
. 16 Which of fol lowing is the most likely sequela of this infection?
. 17
:
• 18 A . Dental caries
0 19
B. Hematu ria
0
20
. 21 C. Jaundice
. 22 D. Meningitis
0
23
E. Otitis media
0 24
F. Scalded skin syndrome
0
25
0
26 G. Valvular heart disease
. 27

8
Lode.
s
Suspe-nd
8
End Bloc:k
Item: 9 of 100 - ,• Mark -<J [:::> "'I ~ · ~
QIO: 2047 ~ P~v•ous N@xt Labl lues No tes Calcula t o r

A A
1
2
The correct answer is B. 3 4 % chose this.
3
This patient likely has nonbullous impetigo (shown in the image) caused by the 13-hemolytic, catalase-
negative, bacitracin-sensitive, gram- positive coccus Streptococcus pyogenes. Poststreptococcal
4 glomerulonephritis (PSGN) is a complication of S. pyogenes infection and commonly manifests with gross
5 hematuria. The latent period from impet igo t o PSGN is approximately 21 days. Renal function often ret urns
to normal in 1-2 weeks, and hematuria resolves within 6 months.
6
7 PSGN can occur following infection with S. pyogenes strains that cause either impetigo or pharyngitis.
However, rheumatic fever (and the Jones criteria symptoms that go along with it) only occurs following
8
infection with the strains responsible for pharyngitis. This is an important distinction to make.
9
StreptoCOC"" I r .... le-'le5 Rheu c fever Hematuria Impetigo Pharyngitis Gram-posibve bactena Go.,._ lor • [1 Streptococcus Incubation period
• 10
Coccus Catalase Fever Infection Kidney
Image copyright © 2015 by the
· 11 authors; licensee MOP!, Basel,
Switzerland
• 12
• 13 A is not correct. 11% chose this .
• 14 Dental carries are a complication of infection with viridans group streptococci (Streptococcus mutans, for example) .
Streptococcu mutans Streptococcus Infection
0
15
. 16 C is not correct. 5°/o chose this .
. 17 Jaundice Is typically a clinical manifestat ion of hepatitis .
Jaundice Hepatitis
• 18
0 19 D is not correct. 8 °/o chose this.
0
20 I n this patient's age group, t he most com mon causes of meningitis are Streptococcus pneumoniae, Neisseria meningitidis, and Haemophilus
. 21
influenzae .
Ne1sseria men1ng1tld1S Streptococcus pneumomae Haemophilus influenzae Mening1t1s Streptococcus Neisseria Haemoph1lus
. 22
0
23
E is not correct . 9°/o chose thi s.
0 24 Common causes of otit is media include Streptococcus pneumoniae (catalase-negative, a-hemolytic, optochin-sensit ive), Haemophilus
influenzae (gram-negative pleomorphic coccobacillus), and Moraxella catarrhalis (gram-negative diplococcus).
0
25 Mor.'lXella c.- rrh~l ii ..>treptococcus pneumomae Haemophilus influenzae Otitis media Coccob~1llu! Grillm·"'egMwe betetena Diplococcus Streptococcus Moraxella Obtis Pleomorphism (m~crob1ology) Catalase
0
26
F is not correct . 1 3°/o chose this.
. 27

8
Lode.
s
Suspe-nd
8
End Bloc:k
Item: 9 of 100 ~ ,• Mark <::J [:::> ""I ~· ~'j
QID: 2047 J.. Previous Next faiUI~S
LAb Notes Calculator

1
F is not correct . 13% chose this.
2
Scalded skin synd rome is a toxin-mediat ed disease caused by Staphylococcus aureus infection.
3 Staphylococcus aureus Staphylococcus Staphylococcal scalded skin syndrome Infection

4
G is not correct. 20°/o chose this.
5
Valvular heart disease, especially involving the m it ral valve, is an im p01tant f inding in rheumat ic fever. I nfection wit h Streptococcus pyogenes
6 strains t hat cause pharyngitis can lead t o rheumatic feve r, bu t st rains of S. pyogenes that cause impetigo do not lead to rheumat ic fever. I n
7 cont ract, poststreptococcal glomerulonephrit is (PSGN) can occur weeks after either pharyngit is or impet igo.
Acute proliferative glomerulonephritis Streptococcus pyogenes Rheumatic fever valvular heart disease Impetigo Pharyngitis Mitral valve Glomerulonephritis Streptococcus Cardiovascular disease Infection
8
Fever
9
• 10
· 11 Bottom Line:
• 12 Think about PSGN when acute -onset hemat uria is preceded by impet igo or pharyng it is caused by a group A ~ -h emolyti c st re pt ococcus .
Hematuria Impetigo Pharyngitis Streptococcus
• 13
• 14

• 15
. 16
l@);fil ~1hl for year:l 2o17 y
FIRST AID FACTS

. 17
• 18 FA17 p 130.1
• 19
Gram-positive lab algorithm
• 20
. 21
Gram@ {purple/blue)
. 22
• 23
• 24
Branching
Bacilli Cocci
• 25 filaments
• 26
. 27
(
8
Lock
s
Suspend
0
End Block
Item: 9 of 100 - ,• Mark -<J [:::> "'I ~ · ~
QIO: 2047 ~ P~v•ous N@xt Labl lues No tes Calcula t o r

1
Aerobic Anaerobic Anaerobic/facultative Aerobic Anaerobic
2
3
Usteria
Nocardia Actinomyces
4 Bacillus Clostridium
Corynebacterium Prop10n1bactenum (weakly acid fasU lnot acid fast)
5
6 Catala.se
7 -~e>---
8 Streptococcus Staphylococcus
9
Hemolysis Coagula.se
• 10
· 11
• 12 !Partial (Complete
(No hemolysis,
• 13
0. hemolysis. hemolysis.
grows in bile)
green) clear) Saureus
• 14 Novobiocin
15 sensitivity
Optochin sensitivity Bacitracin sensitivity Growth in 6.5% NaCI
0

. 16 and bile solubility


. 17
• 18 ---0----- ~
0 19
Group B Group A S saprophyticus S epidermidis
S agalactiae S pyogenes
0
20
. 21
. 22
Vindans streptococci Group D
0
23 (no capsule) S pneumoniae (enterococcus)
Nonenterococcus
0 24 Smutans (encapsulated) 5OOVIS Efaecium
Smitis Efaecalis
0
25
0
26 lmpcl(tant tests are tn bold. Important pathogens are in bold italics.
. 27 • Note: Enterococcus ts either a- or y-hemolytic.

8
Lode.
s
Suspe-nd
8
End Bloc:k
Item: 9 of 100 - ,• Mark -<J [:::> "'I ~ · ~
~ P~v1ous
QIO: 2047

1
2
-
Vindans streptococci
N@xt Labl lues No tes Calcula t o r

1 - 1
Group D

(no capsule} Spneumoniae Nonenterococcus (enterococcus}


3 Smutans (encapsulated} S bows Efaecium
4 Smitis Efaecalis
5 Important tests are 1n bold. Important pathogens are in bold italics.
6 Note: Enterococcus IS either a· or y-hemolytic.
7
8
FA17 p 132.4
9
• 10 Streptococcus Gram EE> cocci in chains · . Group A strep J• l\ ES (major criteria for acute rheumatic
· 11 pyogenes (group A cause: fe, er):
• 12 streptococci) • Pyogenic-pharyngitis, cellulit is, impetigo Joints- polyarthritis
• 13 • -carditis
("honey-crusted" lesions), erysipelas
• 14 '\odules (subcutaneous)
Toxigenic- scarlet fever, toxic shock- like
15 Erythema marginatum
syndrome, necrotizing fasciitis
0

. 16
Immunologic- rheumatic fever, Sydenham chorea
. 17
glomerulonephritis Pharyngitis can result in rheumatic "phever"
• 18
Bacitracin sensitive, ~ -hem ol ytic, pyrrolidonyl and glomeru lonephritis.
0 19
arylamidase (PYR) $ .Hyaluronic acid capsule Impetigo usually precedes glomerulonephritis.
0
20 c inhibits phagocytosis. An tibodies lo I protein Scarlet fever- blanchi ng, sandpaper-like body
. 21
. 22
enhance host defen ses against S pyogenes bul rash, strawberry tongue, and circumoral
0
23 can gi,·e rise to rheumatic fe,·er. pallor in the setting of group . streptococcal
0 24 ASO titer or anti-0 'ase B antibodies indicate pharyngitis (erythrogenic toxin EE>).
0
25 recent S pyogenes infection.
0
26
. 27 •
8
Lode.
s
Suspe-nd
8
End Bloc:k
Item: 10 of - ,• Mark -<J [:::> "'I ~ · ~
100 ~ P~v1ous N @xt Labl lues N o tes Calcula to r

1
A A 55-year-old man with well-controlled diabet es presents t o his internist because of feve r, nasal discharge, and headache. Because over-
2 the-counter cold remed ies have been ineffecti ve, his internist decides to treat him by evacuating his maxillary and ethmoid sinuses. The
3 sinus contents are sent for pathologic examinati on, and hist ology reveals the broad, wide-branching, nonsept ate hyphae shown in the
image.
4
5
6
7
8
9
• 10
· 11
' 12
' 13
' 14
' 15
. 16
. 17
• 18

' 19 Which of the fol lowing is the most worrisome complication of this infect ion?
' 20 :
. 21 A. Meningit is
. 22 B. Otitis media
' 23
C. Pneumonia
' 24
' 25 D. Sepsis
' 26 E. Skin abscess
. 27

8
Lode.
s
Suspe-nd
8
End Bloc:k
Item: 10 of ~ ,• Mark <:::1 t::> ""I ~· ~'j
100 J.. Previous Next faiUI~S
LAb Notes Calculator

1
2
The correct answer is A. 57°/o chose this.
Diabet ics are more susceptible to m ucormycosis t han are members of t he general population. It is thought that t he high glucose cont ent in
3
t heir nasal secretions is associat ed wit h increased vulnerability. With Mucor having been ident if ied in t he patient 's nasal sinuses, t he main
4 medical concern here is t he possible int rod uct ion of t he microorganism into t he meninges, which can cause life-t hreaten ing meningitis. Even if
5 t he patient does not have meningitis, the infect ion must still be treat ed emergently to prevent damage to facial st ruct ures and nerves.
Microorganism Meninges Mucormycosis Meningitis Paranasal sinuses Diabetes mellitus Glucose Mucus Mucor
6
7 B is not correct. 5% chose this.
8 Mucor can cause otit is media, but t his would not be of emergent risk to the patient. Ot itis media is oft en caused by pneumococcus.
Otitis media Streptococcus pneumoniae Otitis
9
10 C is not correct. 12% chose this.
· 11 Mucor can cause pneumonia, but t his would not be of emergent risk to the patient. Pneumonia can be caused by a a number of organisms,
epecially pneumococcus .
• 12
Pneumonia Streptococcus pneumoniae
• 13
• 14
D is not correct. 14% chose this .
Although Mucor can cause sepsis, it is unlikely in a pat ient without an immunodef iciency.
• 15 Sepsis Immunodeficiency
. 16
E is not correct. 12°/o chose this .
. 17
Mucor can cause skin abscesses, but this would not be of emergent risk t o t he pat ient. Skin abscesses are often cauesd by Staphlyococcus
• 18
and Streptococcus.
• 19 Streptococcus Abscess Mucor

• 20
. 21
Bottom Line:
. 22
Mucor infect ion can cause life-threatening meningit is in pat ients wit h diabetes .
• 23 Meningitis Diabetes mellitus Mucor Infection
• 24

• 25
• 26 141;fil·1i•J for year:[2017 •
FIRST AID FA CTS
. 27

8
Lock
s
Suspend
0
End Block
Item: 10 of - ,• Mark -<] 1:> ""'I ~· 1!';:'1
100 ~ Prev1o u s Next Labf a lu es Note s Calculator

1 •
FA17 p 149.1
2
Opportunistic fungal infections
3
4 Candida albicans alba = white. Dimorphic; forms pseudoh) phae ;mel budding yeasts at 20°C · , germ t·ubes at
5 37°C il.
6 Systemic or superficial fun gal infection. Causes ora l ~ and esophageal thrush in
7 immunocompromised (neonates, steroids, diabetes, AI OS), vulvovaginitis (diabetes, use of
8 antibiotics), diaper rash, endocarditis (I drug users), disseminated candidiasis (especiallr in
9 neutropenic patients), chronic mucocutaneous candidiasis.
10 Treatment: oral fluconazole/topical a1olc for ' 'aginal ; nystatin, fluconazole, or caspofungin for oral/
· 11 esophageal; fluconazole, caspofungin, or amphotericin B for systemic.
• 12
Aspergillus Septate hyphae that branch at 45° \ cute \ ngle . Produces conidia in radiating chains at end of
• 13
• 14
fumigatus conidiophore D.
. 15
Causes im·asive aspergillosis in immunocompromised, patients " ith chronic granulomatous disease.
. 16
Can cause aspergillomas in pre-existing lung cavities, especially after TB infection .
. 17 Some species of Aspergillus produce Anatoxins (associated with hepatocellular carcinoma).
• 18 Allergic bronchopulmonary aspergillosis (ABPA}: hypersensitivity response associated with
• 19
asthma and cystic fibrosis; may c<~u se bronchiectasis and eosinophilia .
• 20
. 21
Cryptococcus 5- 10 jlm with narrow buclcling. lleavily encapsulated yeast. ot dimorphic.
. 22 neoformans l•'ound in soil, pigeon droppings. Acquired through inhalation with hematogenous dissemination
• 23 to meninges. Culture on Sabouraud agar. llighlighted with India ink (clear halo 1]1) and
• 24 mucicarmine (red inner capsule ). Latex agglutination test detects polysaccharide capsular
• 25 antigen and is more specific.
• 26 Causes cryptococcosis, cryptococcalmeningitis, cryptococcal encephalitis ("soap bubble" lesions
. 27 in hr~ ln\ nrim~ ril v in j.,,,,.nnl'nmnrl"' t..,...i('orl •
8
L.odt
s
Su~pl'nd
~
End Block
Item: 10 of - ,• Mark -<] 1:> ""'I ~· 1!';:'1
100 ~ Prev1o u s Next Labf a lu es Note s Calculator

1 Treatment: amphotericin B + Aucytosine followed by fluconazole for cryptococcal meningitis.


2
Mucor and Rhizopus Irregular, broad, nonseplale hyphae branching at" ide angles
3
spp. Mucormycosis. Causes disease mostly in keloaeidolic diabetic and/or neutropenic patients (eg,
4
leukemia). Fungi proliferate in blood vessel walls, penetrate cribri form plate, and enter brain.
5
6
Rhinocerebral, frontal lobe abscess; c<wernous sinus thrombosis. lleadache, facial pain, black
7
necrotic eschar on fa ce; ma\' have cranial nerve invoh ement.
'
8
Treatment: surgical debridement, amphotericin B.
9
10
· 11
• 12
• 13
• 14
. 15
. 16
. 17
• 18
• 19
• 20
. 21

. 22
• 23
• 24
• 25
• 26 a
. 27 •
8
L.odt
s
Su~pl'nd
~
End Block
Item: 10 of ~ ,• Mark <:::1 t::> ""I ~· ~'j
100 J.. Previous Next LAbfaiUI~S Notes Calculator

1 * * * *
2
3 FA17 p 176.2
4 CSF findings in meningitis
5 OPENING PRESSURE CELLTYPE PROTEIN GLUCOSE
6
Bact erial t t P II s t l
7
8 Fungai/TB t t lymphocytes t l
9 Viral orma 1/t t lymphocytes Norma lit Normal
10
· 11
• 12 FA17 p 176.1
• 13 Common causes of meningitis
• 14 NEWBORN (0- 6MOl CHILDREN (6 M0- 6 YR) 6- 60YR 60YR+
• 15 Group B streptococci S pneumoniae S pneumoniae S pneumoniae
. 16 E coli N meningitidis N meningiticlis (#1 in teens) Gram 8 rods
. 17
Listeria H infiuenzae type B Enterovi ruses Listeria
• 18
Enteroviruses HSV
• 19
• 20
Give ceftriaxone and vancomycin empirically (add ampicillin if Listeria is suspected).
. 21
Viral causes of meningitis: enteroviruses (especially coxsackievirus), llSV-2 {llSV-1 = encephalitis), IIIV, West ile virus (also
. 22 causes encephalitis), VZV.
• 23 In HIV: Cryptococcus spp.
• 24 Note: Incidence of H infiuenzae meningitis has l greatly due to conjugate H infiuenzae vaccinations. Today, cases are usually
• 25 seen in unimmunized children .
• 26
. 27 •
8
Lock
s
Suspend
0
End Block
Item: 11 of - ,• Mark -<J [:::> "'I ~ · ~
100 ~ P~v1ous N @xt Labl lues N o tes Calculato r

A A
1
After scaling the wall of a maximum security penitent iary, a 24-year-old convict steps on barbed wire with his rig ht foot . After hiding from ~~AI
2
the police for 8 days he starts having muscle spasms and difficul ty swallowing. Within the next 2 days, his sister notices an odd grimace on
3 his face. She urges him to go to the hospital to treat his foot wound, but he refuses, fearing the police will apprehend him. Within the week
4 he develops total body muscle spasms followed by a tense, back-arching posit ion . He dies several days later.
5
6
What is the mechanism of toxicity of the organism responsible for this patient's pathology?
7 :

8
9 A . Adenosine diphosphate ribosylation of a G protein, increasing chloride secretion
10
B. Adenosine diphosphate ribosylation of an elongation factor, disrupting protein synthesis
· 11
' 12
C. Binding to the major histocompatibility complex type II receptor and T-lymphocyte receptor, causing cytokine synthesis
' 13 D . Blocking the release of acetylcholine
' 14
E. Blocking the release of GABA and glycine
' 15
F. Lysis of RBCs
. 16
. 17 G. Stimulation of guanylate cyclase
• 18

' 19
' 20
. 21
. 22
' 23
' 24
' 25
' 26
. 27

8
Lode.
s
S uspe-nd
8
End Bloc:k
Item: 11 of ~ ,• Mark <:::1 t::> ""I ~· ~'j
100 J.. Previous Next LAbfaiUI~S Notes Calculator

1
2 The correct answer is E. 77°/o chose this.
3 The penet rat ing wound f rom t he barbed wire pu ts t his pat ient at risk fo r infection with Clostridium tetani, t he symptoms of which are caused
by t he t et anus t oxin . This toxin blocks t he release of the inhibitory neurotransmitters (GABA and glycine) f rom Renshaw cells in the spinal
4
cord and resu lt s in " lockj aw" and similar sym ptoms. Had the patient been vaccinated fo r tetanus, he would not have had t hese symptoms.
5 Tetanus vaccination is required approximately every 10 years to ensure adequate blood levels of prot ective ant ibod ies. The patient is
6 demonst rating risus sardonicus, a m uscle spasm- induced grimace, which bodes a poor prognosis. The patient t hen exhibit ed opisthotonos, a
severe hyperext ension of t he head, neck, and back.
7
Risus sardonicus Clostridium tetani Opisthotonus Glycine Gamma-Aminobutyric acid Tetanospasmin Tetanus Renshaw cell Spinal cord Antibody Clostridium Neurotransmitter Prognosis Toxin vaccination
8
Infection vaccine Muscle
9
10
A is not correct . 3°/o chose this.
The Vibrio cholerae toxin adenosine diphosphate- ri bosylates a G stim ulatory prot ein in t he intest ines, increasing adenylat e cyclase act ivity and
11
causing pumping of wat er and chlori de ions into t he gut lumen. Its most characterist ic symptom is voluminous " ri ce wat er" diarrhea. The
• 12 heat-labile toxin of Escherichia coli has the same mechanism of action .
• 13 Vibrio cholerae Escherichia coli Adenylyl cyclase Toxin Diarrhea Protein Gastrointestinal tract Lumen (anatomy) Adenosine symptom Vibrio Mechanism of action

• 14 B is not correct. 3% chose this.


• 15 The Co1ynebacterium diphtheriae toxin inactivates the elongation fact or- 2 by adenosine diphosphate ri bosylat ion, disrupting prot ein synt hesis .
. 16 It causes pharyng it is and a "pseudomem brane" in the oropharynx. The syst emic effects of dipht heria t oxin are most commonly seen in cardiac
dysfunction, which can present wit h tachycardia, hypot ension, and heart fa ilure .
. 17
Corynebacterium diphtheriae Diphtheria EEF2 Diphtheria toxin Pharyngitis Adenosine diphosphate Hypotension Protein Pharynx Tachycardia Protein biosynthesis Corynebacterium Toxin Heart failure
• 18
Protein synthesis
• 19
• 20
C is not correct. 2 °/o chose this .
Superantigens such as the Staphylococcus aureus t oxic shock synd rome toxin and the Streptococcus pyogenes erythrogenic toxin (A, B, and
. 21
C) binds t o t he major histocom patibilit y complex t ype II receptor and t he T-lymphocyt e receptor, resu lt ing in cytokine re lease and somet imes
. 22 t oxic shock syndrome (TSS) . This pat ient does not demonst rate signs and sympt oms of TSS, as neurolog ic involvement is not associat ed with
• 23 t his disease. Fu rthe rmore, TSS is most commonly associat ed wit h superficial staphylococcal infect ions and t am pon use, neit her of which is
present in this patient's hist ory .
• 24
Erythrogenic toxin Streptococcus pyogenes Toxic shock syndrome Major histocompatibility complex Cytokine Tampon Staphylococcus aureus Superantigen Toxin Streptococcus T cell Staphylococcus
• 25
D is not correct. 11% chose this .
• 26
The Clostridium botulinum toxin inhibits t he release of acetylcholine, result ing in anticholinergic sym ptoms and even cent ral nervous system
. 27

8
Lock
s
Suspend
0
End Block
Item: 11 of ~ ,• Mark <:::1 t::> ""I ~· ~'j
100 J.. Previous Next LAb faiUI~S Notes Calculator

1 D is not correct. 11% chose this .


2 The Clostridium botulinum toxin inhibits t he release of acetylcholine, result ing in anticholinergic sym ptoms and even central nervous system
3 paralysis. C. botulinum is most often fo und in canned food and honey and can result in "f loppy baby" syndrome when consumed by young
children.
4 Anticholinergic Clostridium botulinum Central nervous system Acetylcholine Toxin Botulinum toxin Nervous system Paralysis Honey Clostridium
5
F is not correct. 1 °/o chose this.
6
The st reptolysin 0 toxin of Streptococcus pyogenes is a hemolysin t hat inserts int o t he cell mem brane, form ing pores t hat lead t o leakage and
7 cell lysis. Signs and sympt oms of this disease include pharyngit is, laryngit is, and mala ise.
8 Streptococcus pyogenes Hemolysin Pharyngitis Streptococcus Cell membrane Laryngitis Malaise Lysis Toxin Streptolysin

9
G is not correct. 3% chos e this.
10 The heat -st able toxin of Escherichia coli st imulat es guanylate cyclase, whereas t he heat-labile cholera -like t oxin stim ulates adeny lat e cycl ase.
11 Escherichia coli Guanylyl cyclase Adenylyl cyclase Toxin

• 12
• 13
Bottom Line:
• 14
Tetanus toxin blocks glycine and GABA release from inhibitory interneurons, which results in severe muscle spasms and ult imat ely deat h if
• 15 untreat ed .
. 16 Glycine Tetanus Gamma-Aminobutyric acid Tetanospasmin Interneuron Spasm Toxin Muscle

. 17
• 18
• 19 ljl:fil·11•i for year:( 20 1 7 .,.
FIRS T AID FACTS
• 20
. 21 FA17 p 127.2
. 22
Main features of exotoxins and endotoxins
• 23
• 24
Exotoxins Endotoxin
• 25 SOURCE Certain species of gram ® and gram 8 bacteria Outer cell membrane of most gram 8 bacteria
• 26
SECRETEO FROM CELL Yes ro
. 27

8
Lock
s
Suspend
0
End Block
Item: 11 of - ,• Mark -<] 1:> ""'I ~· 1!';:'1
100 ~ Prev1o u s Next Labf a lu es Notes Calculator

1 • •
2
FA1 7 p 12702
3 Main features of exotoxins and endotoxins
4 Exotoxins Endotoxin
5
SOURCE Certain species of gram EF> and gram 0 bacteria Outer cell membrane of most gram 0 bacteria
6
7 SECRETED FROM CEll Yes 0
8 CHEMISTRY Polypeptide I.ipid A component of LPS (structural part of
9 bacteria; released when lysed)
10
11
LOCATION OF GENES Plasmid or bacteriophage Bacterial chromosome
• 12 ADVERSE EFFECTS High (fatal dose on the order of l Jig) Low (fatal dose on the order of hundreds of
• 13 micrograms)
• 14
CliNICAl EFFECTS Various effects (see following pages) F'e,oer, shock (hypotension), OIC
. 15
0 16 MODE OF AC TI ON Various modes (see following pages) Induces T ' F, IL-l, and IL-6
0 17 ANTIGENICITY Induces high-titer antibodies called antitoxins Poorly antigen ic
• 18
VACCINES Toxoids used as vaccines o toxoids formed and no vacci ne avai lable
• 19
• 20
HEATSTABILITY Destroyed rapid!)' at 60°C (except Stable at 100°C for 1 hr
02 1 staphylococcal enterotoxin and E coli heat-
022 stable toxin)
• 23 TYPICAl DISEASES Tetanus, botulism, diphtheria Meningococcemia; sepsis by gram 0 rods
• 24
• 25
FA17 p 12801
• 26
0 27 Bugs with exotoxins •
8
L.odt
s
Su~pl'nd
~
End Block
Item: 11 of ~ ,• Mark <:::1 t::> ""I ~· ~'j
100 J.. Previous Next faiUI~S
LAb Notes Calculator

1 • •
FA17 p 128.1
2
3
Bugs with exotox ins
4 BACTERIA TOXIN MECHANISM MANIFESTATION
5 Inhibit protein synthesis
6
Corynebacterium Diphtheria toxin3 Pharyngitis with pseudomembranes in th roat
7
diphtheriae Inactivate elongation factor and severe lymphadenopathy (bull neck)
8
Pseudomonas Exotoxin A3 (EF-2) Host cell death
9
10 aeruginosa
11 Shigella spp. Shiga toxin (ST)3 GI mucosal damage -+ dysentery; ST also
• 12 enhances cytokine release, causing hcmolrtic-
Inactivate 60S ribosome by
• 13
removing adenine from uremic syndrome (HUS)
• 14
Enterohemorrhagic Shiga-!ike toxin rRNA SLT en hances cytokine release, causing HUS
• 15
. 16
E coli (EHEC) (SLT)a (prototypically in EHEC serotype 0157:H7).
. 17 Unlike Shigella, F.HEC does not invade host
• 18 cells
• 19 Increase fluid secretion
• 20
Enterot oxigenic Heat-labile Overactivates adenylate Watery diarrhea: "labile in the Air (Adenylate
. 21
E coli (ETEC) toxin (LT)3 cyclase (t cAMP) - t CJ- cyclase), stable on the Ground (Guanylate
. 22
• 23
secretion in gut and H20 cyclase)"
• 24
efflux
• 25
Heat-stable Overactivates guanylate
• 26
toxin (ST) cyclase (t cGMP)
. 27 • - l resorotion of aCI •

8
Lock
s
Suspend
0
End Block
Item: 11 of - ,• Mark -<] 1:> ""'I ~· 1!';:'1
100 ~ Prev1o u s Next Labf a lu es Notes Calculator

1 Edema toxin 3 Mimics the adem·latc


Bacillus anthracis , Li kelr responsible for characteristic edematous
2
cyclase enzyme (t cA l\ IP) borders of black eschar in cutaneous anthrax
3
4
Vibrio cholerae Cholera toxina Overacti,·ates aden)date Voluminous "rice-water" diarrhea
5
c~ clase (t cA\I P) by

6 permanently activating C,
7 - t Cl- secretion in gut
8 and 1-l 0 efAu x
9 Inhibit phagocytic ability
10
Bordetella pertussis Pertussis toxin'1 Q, eracti\·ates adenylate Whooping cough-child coughs on e:'l.piration
11
• 12
cyclase (t cA\I P) by and ··whoops" on inspiration (toxin may
• 13
disabling C;. impairing not actually be a cause of cough; can cause
• 14 phagocytosis to permit "100-day cough" in adults)
. 15 survival of microbe
. 16 Inhibit release of neurotransmitter
. 17
Clostridium tetani Tetanospasmi na Both are proteases that Spastic paralysis, risus sardonicus, and "lockjaw";
• 18
cleaveS ARE (soluble toxin prevents release of inhibitory (CABA and
• 19
SF attachment glycine) neurotransmitters from Renshaw cells
• 20
. 21
protein receptor), <1 set in spinal cord
. 22 Clostridium Botulinum toxin3 of proteins required for Flaccid paralysis, floppy baby; toxin prevents
• 23 botulinum neurotransmitter release release of stimulatory (ACh) signals at
• 24 via vesicular fusion neuromuscular junctions - flaccid paralysis
• 25
:l n B lo\in (ala, two-component toxin [or three for anthrax]} with B enabling binding and triggering uptake (endOC)'tosis)
• 26
of the acti' e A component. T he A components are usually ADP ribosyltransferases; others ha,·e enzymatic activities as listed •
. 27

8
L.odt
s
Su~pl'nd
~
End Block
Item: 11 of - ,• Mark -<J [:::> "'I ~ . ~
100 ~ P~v1ous N @xt Labl lues N o tes Calculato r

1 •
2 FA17p 129.1
3 E'l\ DOTOXINS:
Endotoxin LPS found in outer membrane of gram 8
4
bacteria (both cocci and rods). Composed of Edema
5
0 antigen + core polysaccharide + lipid A (the :\ it ric oxide
6
toxic component). DIC/Death
7
Released upon cell lysis or by li\'ing cel ls b~ Outer membrane
8
blebs detaching from outer surface membrane 'J 1\F-a
9
(\ 'S exotoxin, which is activel\' secreted). 0 -antigen +core polysaccharide+ lipid A
10
11
Three main effects: macrophage actin1tion eXtremelv •
heat stable
' 12
(T LR4), complement acti,·ation, and tissue IL-l and IL-6
' 13 factor activation. l\eutrophil chemotaxis
' 14 Shock
' 15
. 16
. 17 IL-l.IL-6 - Fever
• 18

' 19 TNF-a - - Fever and hypotension


' 20
. 21 Nitric OXIde - Hypotension
. 22
' 23 Histam1ne release.
C3a
Hypotension and edema
' 24
' 25
' 26
EndotoXIn
(bptd Acomponent)
'"""'"'"' '""""'" { CSa Neutrophil chemolaxls

. 27

8
Lode.
s
S uspe-nd
8
End Bloc:k
Item: 12 of - ,• Mark -<J
P~v1ous
[:::> "'I ~ ·· ~
100 ~ N @xt Labl lues N o tes Calculato r

A A
1
A 24-year-old student presents wit h rig ht upper quadrant pain, nausea, and occasional vomit ing. She says she recent ly ret urned from a ~~AI
2
camping trip to Australia, where she intera cted wit h many animals, including dogs, wallabies, and kangaroos. Physical examination revea ls
3 an enlarged liver, and ultrasound imaging shows a 2-cm liver cyst.
4
5 Which of the following is the most appropri at e management?
6 :
7 A. Albendazole
8 B. Diethylcarbamazine
9
C. Observation
10
11 D. Praziquantel
• 12 E. Surgical drainage
• 13
• 14

• 15
. 16
. 17
• 18
• 19
• 20
. 21
. 22
. 23
• 24
• 25
• 26
. 27

8
Lode.
s
S uspe-nd
8
End Bloc:k
Item: 12 of ~ ,• Mark <::J [:::> ""I ~· ~'j
100 J.. Previous Next LAbfaiUI~S Notes Calculator

1 The correct answer is A. 43°/o chose this.


2 Echinococcus granulosus is a parasit ic t apeworm transmit ted by the feces of animals such as dogs, wallabies, and kangaroos. I nfection can
3 result in liver cysts. The contents of the cysts may cause anaphylact ic shock if exposed t o t he immune system . First -l ine t reatment is
albendazole. Albendazole act s by inhibit ing tubule polymeri zation in the tapeworm. I t is quite a toxic drug, commonly causing abnormal liver
4 function t ests and (rarely) causing a variet y of cytopenias and renal fa ilure. Cyst s t hat are less t han 5 em may be t reated wit h albendazole
5 alone.
Echinococcus granulosus Albendazole Cestoda Anaphylaxis Liver function tests Parasitism Feces Immune system Liver Echinococcus Cyst Infection
6
7 B is not correct. 8% chose this.
8 Loa loa is t ransmit ted by the deer fly and causes swelling in t he skin. It is t rea ted wit h diethylcarbamazine. The worm can sometimes be seen
9
crawling in t he conjunct iva.
Diethylcarbamazine Conjunctiva Loa loa Deer fly Loa
10
11 C is not correct. 7 °/o chose this.
12
Observation is not warra nted in this patient , since effect ive medicat ion (albendazole) is available and serious anaphylact ic react ions can occur
if t he tapeworm remains act ive .
• 13 Albendazole Cestoda Anaphylaxis Pharmaceutical drug
• 14
D is not correct. 25% chose this .
• 15
Schistosoma is t ransmitted by snails and causes granulomas, fibrosis, and inflammation of the spleen and liver. It may also lead to bladder
. 16 carcinoma . It is t reat ed with praziquantel.
. 17 Praziquantel Spleen Granuloma Liver Bladder cancer Fibrosis Schistosoma Urinary bladder Inflammation Carcinoma

• 18 E is not correct. 17°/o chose this.


• 19 Surgical drainage of the cyst is not indicat ed in t his pat ient. Rupture of the cyst can precipit at e an anaphylactic react ion, often result ing in
• 20 death. Percut aneous drainage can be considered for large cysts (>5 em), but albendazole should be t ri ed first .
Albendazole Anaphylaxis Cyst
. 21
. 22
• 23 Bottom Line:
• 24 Echinococcus granulosus is found in t he feces of dogs and sheep, as well as wallabies and kangaroos, and should be suspect ed in shepherds
• 25 or in individuals wit h cl ose contact wit h t he animals. Patients have cysts in t he liver and right upper quadrant pain. Treat with albendazole,
and be aware of anaphylaxis if the echinococcal antigen is re leased from the cysts .
• 26
Albendazole Echinococcus granulosus Anaphylaxis Feces Echinococcus Antigen Liver Sheep Quadrant (abdomen) Cyst
. 27

8
Lock
s
Suspend
0
End Block
Item: 12 of ~ ,• Mark <::J [:::> ""I ~· ~'j
100 J.. Previous Next faiUI~S
LAb Notes Calculator

1 • FA17 p 156.1 •
2
Cestodes (tapeworms)
3
ORGANISM DISEASE TRANSMISSION TREATMENT
4
Taenia solium · Intestinal tapeworm Ingestion of larvae encysted in Praziquantel
5
6
undcrcookcd pork
7 Cysticercosis, Ingestion of eggs in food Praziquantel; albendazole for
8 neurocysticercosis rn contaminated with human neu rocysticercosis
9 feces
10
Diphyllobothrium Vitamin B12 deficiency Ingestion of larvae in raw Praziquantel
11
Jatum (tapeworm competes for 8 12 fres hwater fi sh
12
• 13
in intestine) - megaloblastic
• 14
anemia
• 15 Echinococcus Hydatid cysts ("eggshell Ingestion of eggs in food Albcndazole
. 16 granulosus calcification" I!]) in contaminated with dog feces
. 17 Iiver ll cyst rupture can Sheep are an intermediate host
• 18 cause anaphylaxis
• 19
• 20
. 21
. 22
• 23
• 24
• 25
• 26
• •
. 27

8
Lock
s
Suspend
0
End Block
Item: 13 of - ,• Mark -<J [:::> "'I ~ · ~
100 ~ P~v1ous N @xt Labl lues N o tes Calculato r

A A
1
A 35-year-old man who is a long-term intravenous drug abuser presents to the emergency department with a cough and fever. X- ray of the ~~AI
2
chest reveals faint bilat eral int erstitial infiltrates. Histologic analysis of induced sputum revea ls trophozoite fo rms of Pneumocystis jirovecii.
3

4 Which of the fol lowing correct ly states the function of t he surface protein of the virus that predisposed this patient t o his present infection?
5
:
6 A. Bind to CD3 receptors
7
B. Bind to CD4 receptors
8
9 C. Bind to CDS receptors
10 D. Kill cytotoxic T lymphocytes
11
E. Kill helper T lymphocytes
12
. 13
• 14

• 15
. 16
. 17
• 18
• 19
• 20
. 21
. 22
. 23
• 24
• 25
• 26
. 27

8
Lode.
s
S uspe-nd
8
End Bloc:k
Item: 13 of - ,• Mark -<J [:::> "'I ~ · ~
100 ~ P~v1ous N @xt Labl lues N o tes Calculato r
A A
1
The correct an sw er is B. 72% chose this.
2 The causal agent of AIDS is the HIV v irus. The HIV surface protein gp120 binds to CD4 receptors on T helper
3 lymphocytes. After a conformational shift, gp120 then binds to a chemokine coreceptor, either CCRS on
4
macrophages or CXCR4 on T cells. As a result HIV is engulfed into the cell.
5 Pneumonia caused by Pneumocystis jirovecii is an AIDS-defining illness, typically occurring when a pat ient 's
CD4+ T- lymphocyte count decreases to <200/!JL. Chest X-ray will typically show a bilateral, diffuse, reticular
6
pattern with interstitial markings.
7
P"" ">OC' " ' ro ~ C <CR4 CD4 Chemo ne CCRS Envelope gly<:oprotem GP120 HIV T cell Pneumonoe Lymphocyte MICrophage Protem HIV/AIDS T helper cell
8
V.rus Co-receptor Chest rachograph X-ra,t Pneumocystis pneumonia Receptor (biochemastry) AIOS.defin•no ch01cat cond•bon AJiostenc regulation
9
10
11

12
13

' 14 A is n ot correct. 3°/o chose this.


' 15 The surface proteins of HIV bind to CD4 recept ors, not CD3 receptors. However, it is true t hat CD3 receptors are found on both helper T
. 16
lymphocytes and cytotoxic T lymphocyt es .
CD4 T cell CytotoxiC T cell Lymphocyte CDJ (Immunology) HIV Cytotoxicity Protem Receptor (b1ochem1stry)
. 17
• 18 C is not correct. 6 °/o chose this .
' 19
The surface proteins of HIV bind to CD4 receptors, not CDS receptors. CD4 receptors are fou nd on helper T cells, whereas CDS receptors are
found on cytotoxic T cells. HIV select ively ta rget s CD4+ helper T cells by binding to t he CD4 rece pt or.
' 20 CD4 CDS CytotoxiC T cell HIV T helper cell T cell Cytotoxicity Protein Receptor (biochem1stry)
. 21
D is n ot correct . 4 °/o ch ose this .
. 22
The surface proteins of HIV are not direct ly involved in cellular kill ing. The gp120 vira l surface protein plays a role in binding to host cell
' 23 CD4. Another viral surface protein, gp41, is exposed after gp120 also binds to coreceptors CCRS or CXCR4. gp4 1 aids fusion of the viral
' 24 envelope to the host cell membrane.
CXCR4 Cell membr ne CCRS Gp41 CD4 Envelope glycoprotein GP120 Voral envelope HIV Proteon Vorus
' 25
' 26 E is not correct. 15°/o chose this .
. 27 The function of the surface oroteins of HIV is to allow entrv into the heloer T cells. Althouah the T heloer cell will ultimatelv die from infection .
8
Lode.
s
S uspe-nd
8
End Bloc:k
2
3
A is not correct . 3°/o chose this.
The surfa ce prot eins of HIV bind to CD4 receptors, not CD3 receptors. However, it is t rue th at CD3 receptors are found on both helper T
4
lymphocyt es and cytot oxic T lymphocyt es.
5 CD4 T cell Cytotoxic T cell Lymphocyte CD3 (immunology) HIV Cytotoxicity Protein Receptor (biochemistry)

6
C is not correct. 6 °/o chose this.
7
The surfa ce prot eins of HIV bind to CD4 receptors, not CD8 receptors. CD4 recept ors are fo und on helper T cells, whereas CD8 rece pt ors are
8 found on cytotoxic T cells. HIV select ively target s CD4+ helper T cells by binding to t he CD4 recept or.
CD4 CDS Cytotoxic T cell HIV T helper cell T cell Cytotoxicity Protein Receptor (biochemistry)
9
10 D is not correct. 4°/o chose this.
11 The surfa ce prot eins of HIV are not dire ctly involved in cellular killing. The gp120 vira l surface prot ein plays a role in binding t o host cell
12 CD4. Another vira l surface protein, gp41, is exposed after gp120 also binds to coreceptors CCRS or CXCR4 . gp41 aids fusion of the vira l
envelope to the host cell membrane.
13 CXCR4 Cell membrane CCRS Gp41 CD4 Envelope glycoprotein GP120 Viral envelope HIV Protein Virus
• 14
E is not correct. 15°/o chose this .
• 15
The function of th e surface proteins of HIV is to allow ent ry int o t he helper T cells. Alth ough the T helper cell will ult imat ely die from infection,
. 16
its death is not due t o t he surface proteins but rat her t o th e uncontrolled replicat ion of t he virus.
. 17 T helper cell T cell HIV Virus Protein

• 18
• 19
Bottom Line:
• 20
Pneumocystis jirovecii pneumonia is an AIDS -defining illness t hat occurs when CD4+ count s drop below 200/!J L. The HIV surface proteins
. 21 gp120 and gp4 1 cause binding and f usion, respectively, to allow ent ry int o CD4+ helper T cells.
. 22 Pneumocystis jirovecii Envelope glycoprotein GP120 Gp41 HIV T helper cell Pneumonia T cell AIDS-defining clinical condition Pneumocystis pneumonia CD4 Cytotoxic T cell

• 23
• 24

• 25 l@ljl'il·1i•J for year:[2017 • J


FIRST AID FA CTS
• 26
. 27

8
Lock
s
Suspend
0
End Block
Item: 13 of - ,• Mark -<J [:::> "'I ~ · ~
100 ~ P~v1ous N @xt Labl lues N o tes Calculato r

1 •
2 Ulifil·!l•l for year:
F I RST AI D FA CTS
2017 •
3

4
FA17 p 171 .1
5
HIV
6
7 Envtlope pro(ttns Diploid genome (2 molecules of R 'A).
acqwed through buddlllCJ from
8
hosl cell plasma membrane The 3 structural genes (protein coded for):
9
p17: Matrlll ptotem env (gpl 20 and gp41):
10 gpl20 Formed from clea\·age of gpl60 to form
11 Dockrlg cmelope glycoproteins.
glycoproteon ""'
12
gp41 gp1 20-attachment to host C D4+ T cell.
13 Transmemblane gp41 - fu sion and entry.
' 14 glyc()jlfOieln
gag (p24 and p17)-capsid and matrix
' 15
Reverse proteins, respectively.
. 16
. 17
RNA""" transcnptase
pol- re\'erse transcriptase, aspartate protease,
• 18
integrase.
' 19
Reverse transeriptase synthesizes dsD1 A from
' 20
genomic R 'A; dsD A integrates into host
. 21 genome .
. 22 Virus binds CD4 as well as a corcccptor, either
' 23 CC R5 on macrophages (early infection) or
' 24 CXCR4 on T cells (late infection).
' 25 llomozygous CCR5 mutation =immunity.
' 26 Heterozygous CCR5 mutation = slower course.
. 27

8
Lode.
s
S uspe-nd
8
End Bloc:k
Item: 13 of - ,• Mark -<J [:::> "'I ~ · ~
100 ~ P~v1ous N @xt Labl lues N o tes Calculato r
A
1 A

FA17p106.1
2
Cell surface proteins MIIC I present on all nucleated cells (ie, not mature RBCs).
3
4 T cells TCR (binds antigen-/\ I HC complex)
5 CD3 (associated with TC R for signal
6 transduction)
7 C D28 (binds B7 on APC)
8 CXCR4/CCR5 (co-receptors for I IIV)
9
Helper T cells C D4, C D40L
10
11
Cytotoxic T cells C DS
12 CXC R4/CCR5
13 Regulatory T cells C D4, CD25
' 14
B cells lg (binds antigen)
' 15
. 16
CD19, CD20, C02 I (receptor for F:BV), CD40 You can drink Beer at the Bar when rou're 21:
. 17
MHC II, B7 B cells, Epstein-Barr virus, CD21.
• 18 Macrophages CD14 (receptor for PAi\IPs, eg, LPS), C D40
' 19 CCR5
' 20 MHC II, B7 (CDS0/86)
. 21 Fe and C3b receptors (enhanced phagocytosis)
. 22
NK cells CD56 (suggesti\'e marker for NK)
' 23
' 24 Hematop oietic stem CD34
' 25 cells
' 26
. 27 •
8
Lode.
s
S uspe-nd
8
End Bloc:k
Item: 13 of - ,• Mark -<] 1:> ""'I ~· 1!';:'1
100 ~ Prev1o u s Next Labf a lu es Note s Calculator

( gg s )
2
Hematopoietic stem CD34
3
cells
4

5
6 FA17 p 150.1
7 Pneumocystis jirovecii Causes Pneumocystis pneumonia (PCP), a di ffuse interstitial pneumonia · . Yeast-like
8
fungus (originally classified as protozoan). Inhaled. \lost infections are as~ mptomatic.
9
Immunosuppression (eg, AIDS) predisposes to disease. Diffuse, bilateral ground-glass opacities on
10
CXR/CT [l). Diagnosed b) lung biops)' or lavage. Disc-shaped yeast seen on methenamine sih er
11
stain oflung tissue 9 .
12
Treatment/prophylaxis: TMP-SMX, pentamidine, dapsone (prophylaxis only), ato,·aquone. Start
13
• 14
prophylaxis when C04+ count drops to< 200 eells/mm3 in HIV patients.
. 15
. 16
. 17
• 18
• 19
• 20
. 21
. 22
• 23
• 24
• 25
• 26
. 27

8
L.odt
s
Su~pl'nd
~
End Block
Item: 14of - ,• Mark -<J
P~v1ous
[:::> "'I ~ ·· ~
100 ~ N @xt Labl lues N o tes Calculato r

1
A 32-year-old man present s to his doctor wit h painful uri nation and a clear, watery urethral discharge. The discharge is stained, and the
2 result is shown in the image below.
3

4
5
6
7
8
9
10
11

12
13

' 14
' 15
. 16
. 17
Which of the fol lowing is t he t reat ment of choice fo r this infection?
• 18 :

' 19
A. Azithromycin
' 20 B. Ceftriaxone
. 21
C. Fluconazole
. 22
D. Penicillin
' 23
' 24 E. Vancomycin
' 25
' 26
. 27

8
Lode.
s
S uspe-nd
8
End Bloc:k
Item: 14 of ~ ,• Mark <::J [:::> ""I ~· ~'j
100 J.. Previous Next LAb faiUI~S Notes Calculator

1
The correct a nswer is A. 55°/o chose t h is.
2 These symptoms are typical of uret hrit is. The most common causes of urethrit is in males are Chlamydia trachomatis and Neisseria
3 gonorrhoeae. The image shows int racellu lar inclusions (dark brown) consist ent wit h infection by C. trachomatis. Alt hough infection from these
4
t wo organisms may be diff icult t o diffe rent iat e, C. trachomatis infect ion induces a predominant ly immunolog ic react ion, wit h on ly a few
polymorphonuclear leukocytes (PMNs), whereas N. gonorrhoeae induces predominantly nonimmunolog ic inf lammat ion, with a PMN-ri ch
5 infilt rat e. The ant ibiot ic of choice for t reat ing chlamydia urethri t is is azi thromycin (macrolide) or doxycycline (tetracycline).
6 Macrolide Chlalll(dia trachomatis Azithromycin Doxycycline Neisseria gonorrhoeae Chlamydia infection Urethritis Antibiotics Immune system Chlam(dia (genus) Neutrophil White blood cell Inflammation

7 Granulocyte Neisseria Intracellular

8
B is not correct. 25% chose this .
9 Ceftriaxone is an effect ive t reatment fo r gonorrhea, but t he cephalospori n class of ant ibiotics is relat ively ineffective aga inst Chlamydia
10 trachoma tis .
Cephalosporin ChlaiTP(dia trachomatis Ceftriaxone Gonorrhea Chlal1l(dia infection Chlamydia (genus) Antibiotics
11

12 C is not correct. 8 °/o chose this .


13 Fluconazole inhibits funga l st eroid synthesis. It is used in the t reat ment of fungal infect ions, such as Candida albicans.
Candida albicans Fluconazole Candida (fungus) Steroid Fungus Mycosis
14
• 15 D is not correct. 8°/o chose t his .
. 16 Penicillin has been shown to suppress chlamydia! mult iplication. However, it does not eradicat e t he organ ism and thus is not the best
t reatment for t his t ype of infect ion. Penicillin is t he t reatment of choice fo r syphilis .
. 17
Penicillin Syphilis Chlal1l(dia infection Organism
• 18
• 19
E is not correct. 4 °/o chose this .
Vancomycin has not been shown to be effect ive in t he t reatment of chlamydia! infection. It is used to t reat drug-resistant Staphylococcus
• 20
aureus and Clostridium difficile .
. 21 vancomycin Staphylococcus aureus Clostridium difficile colitis Clostridium difficile (bacteria) Staphylococcus Chla,.,dia infection Clostridium Antimicrobial resistance Drug resistance

. 22
• 23
Bottom Line :
• 24
The two most common causes of uret hrit is in men are Chlamydia trachomatis and Neisseria gonorrhoeae . I n N. gonorrhoeae, on histology,
• 25
you will see many PMNs. C. trachomatis has t wo forms during its life cycle: the elementary body (ext race llular) and t he reticulat e body
• 26 (int race llular), and on histology, t here will be few PMNs .
Chlamydia trachomatis Neisseria gonorrhoeae Urethritis Histology Chlamydia infection Chlamydia (genus) Neutrophil Granulocyte Neisseria
. 27

8
Lock
s
Suspend
0
End Block
Item: 14 of - ,• Mark --<) [::> ""'I ~· 1!';:'1
100 ~ Prev1o u s Next Labf a lu es Notes Calculator

1
2
141il;ii!11•J
FIRST AIO FACTS
for yea r : 2017 •
3
4

5 FA17 p 146.1
6 Chlamydiae Chlamydiae cannot make their own TP. The) Chlamys =cloak (intracellular).
7 C psittaci-has an avian reservoir (parrots),
are obligate intracellular organisms that cause
8
mucosal infections. 2 forms: causes al) pica I pneumonia.
9
Elementary body (small, dense) Lab diagnosis: PCR, nucleic acid amplification
10
is "Enfectious" and h nlcrs cell 'ia test. Cytoplasmic inclusions (reticulate bodies)
11
Endocytosis; transforms into reticulate bod)'. seen on Ciemsa or fluorescent antibody-
12
Reticulate body Replicates in cell by fission; stained smear.
13
14
Reorganizes into clementar)' bodies. The chlamydia! cell wall lacks classic
. 15
Chlamydia trachoma tis causes reacti' c arthritis peptidoglycan (due to reduced muramic acid),
. 16 (Reiter syndrome), follicular conjunctivitis fJ. rendering P-lactam antibiotics ineffective.
. 17 nongonococcal urethritis, and PID.
• 18 Chlamydophila pneumoniae and Chlamydophila
• 19 fJsittaci cause atypica I pneumon ia; Ira n~mit ted
• 20 by aerosol.
. 21 Treatment : azit hromyein (favored because one-
. 22 time treatment) or doxycycline (+ ceft riaxone
• 23 for possible concomitant gonorrhea).
• 24
• 25
FA17 p 180.1
• 26
Sexually transmitted infections •
. 27

8
L.odt
s
Su~pl'nd
~
End Block
Item: 14 of - ,• Mark --<) [::> ""'I ~· 1!';:'1
100 ~ Prev1o u s Next Labf a lu es Notes Calculator

1 FA17 p 180.1
2 Sexually transmitted infections
3 DISEASE CLINICAL FEATURES ORGANISM
4
AIDS Opportunistic infections, Kaposi sarcom<l, Ill
5
lr mphoma
6
7 Chancroid Painful genital ulcer with exudate, inguinal Haemophilus ducreyi (it's so painful, you "do
8 adenopathr CT\ " )

9 Chlamydia Urethritis, cervicitis, epididymitis, Chlamydia trachomalis (0 - K)


10
conjunctivitis, react i\e arthritis, PI D
11

12
Condylomata Genital warts, koilocytes HP -6 and -11
13
acuminata
14 Genital herpes Painful penile, n.lvar, or cervical vesicles and HS -2, less commonly HSV-1
. 15 ulcers; can cause systemic symptoms such as
. 16 fe,·er, headache, myalgia
. 17
Gonorrhea Urethritis, cervicitis, PID, prostal it is, eisseria gonorrhoeae
• 18
epididymitis, arthritis, creamy purulent
• 19
• 20
discharge
. 21 Granuloma inguinale Painless, beefy red ulcer that bleeds readily on Klebsiella (Calymmatobacterium) granulomatis;
. 22 (Donovanosls) contact fJ cytoplasmic Donovan bodies (bipolar staining)
• 23 Not common in US seen on m1croscopy
• 24
• 25
• 26
. 27 •
8
L.odt
s
Su~pl'nd
~
End Block
Item: 14 of - ,• Mark --<) [::> ""'I ~· 1!';:'1
100 ~ Prev1o u s Next Labf a lu es Notes Calculator

1 • Gonorrhea Urethritis, cervicitis, PID, prostatit is, Neisseria gonorrhoeae •


2
epididymitis, arthritis, creamy purulent
3
discharge
4
5
Granuloma inguinale Painless, beefy red ulcer that bleeds readil) on Klebsiella (Calymmatobacterium) granulomatis;
6 (Oonovanosis) contact rJ C) toplasmic Donovan bodies (bipolar staining)

7 1 ot common in US seen on m1croscopy


8
9
10
11

12
13
14 a
. 15 Hepatitis B HB
Jaundice
. 16
. 17
Lymphogranuloma Infection of lymphatics; painless genital ulcers, C /rachomatis (Ll-L3)
• 18
venereum painful lymphadenopathy (ie, buboes)
• 19 Primary syphilis Painless chancre Treponema pallidum
• 20
Secondary syphilis Fever, lymphadenopathy, skin rashes,
. 21
condylomata lata
. 22
• 23 Tertiary syphilis Gummas, tabes dorsalis, general paresis, aortitis,
• 24 Argyll Robertson pupil
• 25 Trichomoniasis Vaginitis, strawberry cer"ix, motile in wet prep Trichomonas vagirwlis
• 26
. 27

8
L.odt
s
Su~pl'nd
~
End Block
Item: lS of - ,• Mark -<J [:::> "'I ~ · ~
100 ~ P~v1ous N @xt Labl lues N o tes Calculato r

1
A 19-year-old woman is admitted t o a Colorado hospital immediately after returning from an extended hiking t rip in the mountains. She
2
presents with malaise and a diffuse red rash over most of her body. Her temperature is 39. 1oc (102 .4° F), pulse is 112 beats/min,
3 respirations are 22 breaths/min, and blood pressure is 7 1/53 mm Hg upon stand ing. The patient reports t hat this is the fourth day of her
4 menstrual cycle, and she thinks she last changed her t ampon about 24-48 hours ago. She denies alcohol use, drug use, and recent sick
contacts.
5
6
Which of the following is the most likely causative agent in this patient's condition?
7
:
8
A. Clostridium perfringens enterotoxin
9
10
B. Disseminated Staphylococcus aureus infection
11 C. Group A streptococcal exotoxin
12
D. Rickettsia rickettsii
13
14
E. Staphylococcus aureus exotoxin
. 15 F. Treponema pallidum
. 16
. 17
• 18
• 19
• 20
. 21
. 22
. 23
• 24
• 25
• 26
. 27

8
Lode.
s
S uspe-nd
8
End Bloc:k
Item: lS of - ,• Mark -<J [:::> "'I ~ · ~
100 ~ P~v1ous N @xt Labl lues N o tes Calculato r
A A
1
Th e correct an swer is E. 65°/o ch ose thi s.
2
This patient likely presents wit h st aphy lococca l toxic sh ock syndro m e (TSS). A classic vignette fo r this
3
syndrome is a menstruating woman who has kept a t ampon inside the vagina for an extended period of t ime.
4 Staphylococcus au reus exotoxin, t oxic shock syndrome toxin 1 (TSST- 1 ), is an example of a superantigen . TSST-
5 1 cross- links MHC II molecules with certain T-lymphocyte receptor subsets, leading to excessive T-lymphocyt e
activation. Anywhere from 2%-20% of all T lymphocytes can be activated. This leads to supraphysiologic
6
production of cytokines, including interleukin- 1 (IL- l) and tumor necrosis factor-a by Thl cells but also I L-6, IL-
7 12, and interferon-y. These cytokines are most responsible for the clinical symptoms of TSS. Patients often
8 exhibit a diffuse red rash on all body surfaces, with desquamation (like that shown in the image)
characteristically occurring on palmar and plantar surfaces.
9 hoc s 1dro Desquamation Staph lococcus aureus Exoto in MHC class II Interleuk.n· l famuy lnterteulon 6 Jnterleulun 12 T helper ceU
SuperMtu)en "'!"o 1e

10
Lymphocyte V09ona T cell Cytoklne Tampon Necrosos Menstruation Menstrual cycle Thl cell Neoplasm Tox10 Staphylococcus Rash Ma)or hostocompatlbility complex
11
Tox1c shock syndrome toxm
12
13
14

15
. 16
Image copyright © 2007
Raghavendra eta/;
. 17 licensee BioMed Central
• 18 Ltd.
• 19
A is n ot correct . 2 °/o ch ose this .
• 20
Clostridium pedringens is t he causat ive agent of gas gangrene. Skin findings of an affect ed patient can include blist eri ng, pallor (not
. 21 erythema) or other color changes, and su bcutaneous emphysema (air under the skin which produces crepit us upon palpation). The main risk
. 22 factors for gas gangrene are penetrating inj uries or surgery.
Clostridium perfringens Ga:. gangrene Subcutaneous emphysema Crepitus Gangrene Chron1c obstruct1ve pulmonary d1sease Erythema Subcutaneous tissue Clostridium Palpat1on Penetrat1ng trauma
. 23
Subcutaneous IM)ecbon
• 24
• 25 B is not co rrect . 18% chose this .
• 26 The clinical vignette is most likely indicative of t oxic shock syndrome, a result of a circulating staphylococcal toxin and not the bacterium Itself.
. 27
In fact, blood cultures are most often negative. In this patient's case, vaginal cultures may be positive for Staphylococcus aureus .

8
Lode.
s
S uspe-nd
8
End Bloc:k
Item: lS of ~ ,• Mark <::J [:::> ""I ~· ~'j
100 J.. Previous Next LAbfaiUI~S Notes Calculator

1
B is not correct. 18 % chose this .
2
The clinical vignette is most likely indicative of toxic shock syndrome, a result of a circulat ing staphylococcal toxin and not the bacterium it self.
3 I n fact, blood cultures are most often negat ive. I n this patient's case, vag inal cultures may be posit ive for Staphylococcus aureus.
Toxic shock syndrome Staphylococcus aureus Staphylococcus Bacteria Toxin Toxicity Blood culture
4
5 C is not co rrect. 7 °/o chose this .
6 Group A strep (GAS) can also cause toxic shock syndrome (TSS), but not in associat ion with tampon use. Rat her, TSS fo llows a severe GAS
7 infection, such as necrot izi ng fasciitis, in an at- risk individual. Risk factors include diabet es, surgery, alcoholism, vari cella infect ions, and other
immunocompromised states. A healthy young perso n wit h prolonged tampon use points t oward staphylococcal TSS .
8
Necrotizing fasciitis Tampon Toxic shock syndrome Immunodeficiency Group A streptococcal infection Diabetes mellitus Staphylococcus Alcoholism Chickenpox Infection Toxicity
9
D is not correct. 8°/o chose t his .
10
Rickettsia rickettsii is the causat ive agent of Rocky Mountain spotted feve r (RMSF). Th ough an eryt hemat ous rash is a characterist ic sympt om,
11 it classically st arts distally on the wrists and ankles befo re progressing t o include t he t runk. I n t he absence of a known t ick bit e or ot her
12 diagnostic clues of Rocky Mountain spott ed fever, toxic shock syndrome is a better answer due t o t he pat ient's repo rted history of extended
13 t ampon use. The sett ing of this cl inical scenario in Colorado is a dist ract or, as RMSF is much more common in t he southeast ern United States
t han in the Rocky Mountains.
14 Rocky Mountain spotted fever Toxic shock syndrome Rickettsia rickettsii Rickettsia Erythema Fever Tick Spotted fever Rocky Mountains Rash Colorado United States
15
F is not correct . 0°/o chose this .
. 16
Rash coveri ng the palms and soles is one key f inding in secondary syphilis. However, not hing else in the pat ient 's presentation or history
. 17 suggests syphilis, and it would be unlikely to cause such a severe disseminated reaction in t he secondary phase.
• 18 Secondary syphilis 9(philis

• 19
. 20
Bottom Line :
. 21
One form of toxic shock syndrome (fever, malaise, diffuse rash, aut onomic dysfunctions) is caused by Staphylococcus aureus toxin, often as
. 22 a result of keeping a t am pon in t he body t oo long.
• 23 Tampon Toxic shock syndrome Staphylococcus aureus Malaise Staphylococcus Toxin Fever Toxicity Rash

• 24

• 25
• 26 141;fil·1i•J
FIRST AID FACTS
for yea r:[2017 •
. 27
8
Lock
s
Suspend
0
End Block
Item: 15 of - ,• Mark -<] 1:> ""'I ~· 1!';:'1
100 ~ Prev1o u s Next Labf a lu es Note s Calculator

1
l@l :fi1·1hi for yea r: 201 7 •
2 ,IRST AID , ACTS

3
4 FA17 p 131 .3
5
Staphylococcus au reus Gram (f), ~-hemolytic, catalase (f), coagulase TSST-1 is a superantigen that binds to MHC
6
(f) cocci in clusters fJ. Protein A (\·irulcnce ll and T-cell receptor, resulting in polyclonal
7
factor) binds Fc-lgG, inhibiting complement T-cell acti\'ation.
8
acti,·ation and phagocytosis. Commonlr Staphylococcal toxic shock syndrome
9
10
colonizes the nares, axilla, and groin. (TS ) presents as fever, \'Omiting, rash,
Causes: desquamation, shock, end-organ failure. TSS
11

12 .,., ... . Inflammatory disease-skin infections, results in t AST, t ALT. t bilirubin. ssociated
13
14
Jill"\ organ abscesses, pneumonia {often after
influenza virus infection}, endocarditis,
with prolonged use of ,·aginal tampons or nasal
packing.
15 septic arthritis, and osteomyelitis. Compare with Streptococcus pyogenes TSS {a
. 16 Toxin-mediated disease-toxic shock toxic shock- like syndrome associated with
. 17 syndrome (TSST-1}, scalded skin syndrome painfu l skin infection).
• 18 {exfoliative toxin), rapid-onset food S aureus food poisoning due to ingestion of
• 19 poisoning (cntcrotoxins). preformed toxin ..... short incubation period
• 20 1RSA (methicillin-resistantS aureus} (2-6 hr} followed by nonbloody diarrhea
. 21
infection- important cause of serious and emesis. Enterotoxin is heat stable - not
. 22
nosocomial and community-acquired destroyed by cooking.
• 23
infections; resistant to methicillin and Bad staph (aureus) make coagulase and toxins .
• 24
nafcillin because of altered penicillin- Forms fibrin clot around self ..... abscess .
• 25
binding protein .
• 26
. 27 •
8
L.odt
s
Su~pl'nd
~
End Block
Item: 15 of - ,• Mark -<] 1:> ""'I ~· 1!';:'1
100 ~ Prev1o u s Next Labf a lu es Note s Calculator

1 • •
bind ing protein.
2
3
4 FA17 p 127.2
5 Main features of exotoxins and endotoxins
6 Exotoxins Endotoxi n
7
8
SOURCE Certain species of gram $and gram 0 bacteria Outer cell membrane of most gram 0 bacteria
9 SECRETED FROM CEll Yes 0
10 Lipid A component ofLPS (structural part of
CHEMISTRY Polypeptide
11
bacteria; released when lysed)
12
13
lOCAIION OF GENES Plasmid or bacteriophage Bacterial chromosome
14 ADVERSE EFFECTS High (fatal dose on the order of 1 pg) Low (fatal dose on the order of hundreds of
15 micrograms)
. 16
CliNICAl EFFECTS Various effects (see following pages) F'cver, shock (hypotension), OIC
. 17
• 18 MODE OF AC TI ON Various modes (see following pages) Induces T ' F, lL-1, and IL-6
• 19 ANTIGENICITY Induces high-titer antibodies called antitoxins Poorly antigenic
• 20
VACCINES Toxoids used as vaccines o toxoids fo rmed and no vaccine available
. 21
. 22 HEAT STABiliTY Destroyed rapidly at 60°C (except Stable al 100°C for 1 hr
• 23 staphylococcal enterotoxin and E coli heal-
• 24 stable toxin)
• 25
• 26
TYPICAl DISEASES Tetanus, botulism. diphtheria Ylcningococccmia; sepsis by gram e rods
. 27

8
L.odt
s
Su~pl'nd
~
End Block
Item: 16 of - ,• Mark -<J
P~v1ous
[:::> "'I ~ ·· ~
100 ~ N @xt Labl lues N o tes Calcula to r

A A
1
A 33-year-old man who is HIV posit ive presents t o t he physician with a 3-week "burning" sensation in his mout h. On physical examination, ~~AI
2
his oral mucosa is diffusely covered with t hick, white patches. The patches can be gently rubbed off t o reveal an eryt hematous and raw
3 surface, which may bleed. Laboratory t est s revea l a CD4+ cell count of 200/mm 3 (normal: 440- 1600/mm 3 ) and a CDS+ cell count of
4 300/mm 3 (normal: 180-850/mm 3 ).
5
6
What is the mechanism of action of the antimicrobial that will most likely treat this patient's condition?
7
A. Binds ergosterol to form pores in membranes
8
9 B. Binds tubulin
10 C. Inhibits DNA polymerase by mimicking a nucleoside
11
D. Inhibits DNA polymerase by mimicking pyrophosphate
12
13 E. Inhibits folate synthesis
14

15
. 16
. 17
• 18
• 19
• 20
. 21
. 22
. 23
• 24
• 25
• 26
. 27

8
Lode.
s
Suspe-nd
8
End Bloc:k
Item: 16 of - ,• Mark -<J
P~v1ous
[:::> "'I ~ ·· ~
100 ~ N@xt Labl lu es No tes Calcula t o r

A A
1
2 The correct answ er is A . 66°/o chose this.
3 Candida stomatitis (often called t hrush) and esophagit is classically manifest on physical exam with sore
throat and dysphagia, with friable whit e plaques and erythematous buccal mucosa, like that shown in the
4
image. Candida albicans is an opportunist ic fungal pathogen most commonly found in the oropharynx of
5 immunosuppressed patients. It also can be present as diaper rash in infants or as a diffuse mucocut aneous
6 fungal infection in severely immunosuppressed individuals. Nystatin "swish and swallow" is often used to
7
treat oral candidiasis; amphotericin B or fluconazole can be used for serious systemic infection. Nystatin
kills fungi by forming pores in the surface membrane.
8 C.wtcf -1a b ... .,. N .tlGtl ,.. .pllotencm 8 Dysphag•a Ruconazole <Xal candidiasiS Cand•d•as•s Fungus Phatynx hnrnun05uppress•on Candida \fungus)
9 MycosiS lrrotant doaper dermatJbs O.al mucosa Erythema stomatitis Esophagotis Mucous membrane Systemoc dosease Pathogen Sore throat Rash
10 Thrush (btrd) lnfectoon
Image courtesy of CDC
11

12 B is not correct . 9 °/o chose this.


13 Griseofulvin is an antifungal agent t hat binds tubulin, interfering with microtubule function. It effectively treats fungal infections of the skin
14 (tinea capitis, corporis, pedis, and the like). It is not used t o t reat candidiasis.
Gnseofulv1n Mtcrotubule Cand1dtas1s Tubulin Antifungal Fungicide Fungus Mycos1s
15
16 C is not correct. 11 °/o chose this.
. 17 Herpes simplex virus (HSV) st omatit is and esophagit is manifest with vesicular lesions and punched-out m ucosal erosions characterized by
intranuclear inclusion bodies on light m icroscopy. Acyclovir, a nucleoside analog, can be used to t reat HSV by inhibit ing DNA repl ication .
• 18
Aciclovir Herpes simplex v1rus DNA replication Herpes simplex Virus Nucleoside analogue Stomatitis Esophag1t1s Nucleoside Inclus•on bodies DNA Optical microscope vesicle (biology and chemistry)
• 19 Microscopy Mucous membrane
• 20
D is not correct. 7 °/o chose this .
. 21
Cytomegalovirus (CMV) esophag itis has a present at ion similar to that of herpes simplex virus esophag itis, with punched-out mucosal lesions
. 22 and "owl's-eye" inclusion bod ies on light m icroscopy. It would not produce the physical exam findings described in this vignette, however. CMV
. 23 is typically treated by ganciclovir; ganciclovir-resistant strains can be treated with foscarnet, a pyrophosphate analog .
Cytomegalovirus Ganc1clov1r Foscarnet Herpes Simplex virus Esophagitis Herpes Simplex InciU!'tlon bod1es VlrU$ Pyrophosphate Human cytomegalovirus
• 24
• 25 E is not correct . 7 °/o chose this.
• 26 Pneumocystis jirovecii (formerly carinii) infection classically causes a mixed alveolar and interstitial pneumonia in patients with CD4 counts
. 27
<400/mm 3 . Prophylaxis for this condition should begin at a CD4 count of <200/mm 3 . It is associated with hypoxia, elevated lactate

8
Lode.
s
Suspe-nd
8
End Bloc:k
Item: 16 of ~ ,• Mark <::J [:::> ""I ~· ~'j
100 J.. Previous Next faiUI~S
LAb Notes Calculator

1 E is not correct. 7 °/o chos e this.


2 Pneumocystis jirovecii (formerly carinii) infection classically causes a m ixed alveolar and int erst it ia l pneumonia in patients wit h CD4 counts
3 <400/mm 3 . Prophylaxis for t his con dit ion should begin at a CD4 count of <200/mm 3 . It is associated wit h hypoxia, elevat ed lactate
dehydrogenase, and systemic symptoms such as fever and chills. Tri methopri m-sulfamethoxazole is considered f irst-l ine therapy fo r
4 Pneumocystis pneumonia, and it im pairs DNA replication by inhibit ing folate synthesis.
5 Trimethoprim/sulfamethoxazole Hypoxia (medical) Pneumocystis jirovecii Lactate dehydrogenase CD4 Pneumonia Folic acid DNA replication Lactic acid Fever Pneumocystis pneumonia DNA

6 Preventive healthcare Infection Pulmonary alveolus

7
8
Bottom Line:
9
Candida (present ing wit h white oral plaques) is common in the oral cavit ies of immune-comprom ised patients and is t reat ed with nystat in
10 "swish and swallow."
11 Nystatin Candida (fungus) Immunodeficiency

12
13
14 l@ljl'il·1i•J for year:[2017 • J
FIRST AID FA CTS
15
16
FA17 p 149.1
. 17
Opportunistic fungal infections
• 18
• 19
Candida albicans alba = white. Dimorphic; Forms pseudohyphae and budding yeasts at 20°C r.J, germ tubes at
• 20 37°C rn.
. 21 Systemic or superfi ciaI funga I in Feel ion. Causes ora I [!I and esophageal thrush in
. 22 immunocompromised (neonates, steroids, diabetes, AIDS), vulvovaginitis (diabetes, use of
• 23 antibiotics), diaper rash, endocarditis (IV drug users), disseminated candidiasis (especially in
• 24 neutropenic patients), chronic mucocutaneous candidiasis .
• 25 Treatment: oral Auconazolc/topical azoic for vaginal; nystatin, flu conazole, or caspofungin for oral/
• 26 esophageal; fluconazole, caspofungin, or amphotericin B for systemic.
. 27

8
Lock
s
Suspend
0
End Block
Item: 16 of - ,• Mark --<) [::> ""'I ~· 1!';:'1
100 ~ Prev1o u s Next Labf a lu es Notes Calculator

1
Aspergillus Septate h)'phae that branch at 45° Acute Angle rn. Produces conidia in radiating chains at end of
2
fumigatus conid iophore 1).
3
Causes invasive aspergillosis in immunocompromised, patients with chronic granulomatous disease.
4
Can cause aspergillomas in pre-existing lung cavities, espec iall~· after T B infection.
5
6
Some species of Aspergillus produce \ fl ato-.ins {associated with hepatocellular carcinoma).
7 Allergic bronchopulmonary aspergillosis {ABP ): hr persensitivity response associated with
8 asthma and cystic fibrosis; ma} cause bronchiectasis and eosinophilia.
9
Cryptococcus 5- 10 J..l m with narrow budding. Ilea' ily encapsulated )Cast. ~ot dimorphic.
10
11
neoformans Found in soil, pigeon droppings. Acquired through inhalation with hematogenous dissemination
12
to meninges. Culture on Sabouraud agar. ll ighlighted with India ink (clear halo ) and
13
mucicarmine (red inner capsule ). Latex agglutination test detects polysaccharide capsular
14 antigen and is more specific.
15 Causes cryptococcosis, cryptococcalmcningitis, cryptococcal encephalitis ("soap bubble" lesions
16 in brain), primarily in immunocompromised.
. 17 Treatment: amphotericin B + flucytosine followed by fluconazole for cryptococcal meningitis.
• 18
Mucor and Rhizopus Irregular, broad, nonseptate hyphae bra nching at wide angles ITJ.
• 19
spp. Mucormycosis. Causes disease mostly in ketoacidotic diabetic and/or neutropenic patients (cg,
• 20
leukemia). Fungi proliferate in blood vessel walls, penetrate cribriform plate, and enter bra in.
. 21
Rhinocerebral, fronta l lobe abscess; cavernous sinus thrombosis. llcaclache, facia] pain, black
. 22
necrotic eschar on face; mar have cranial nerve invok cment.
• 23
• 24
Treatment: surgical debridement, amphotericin B.
• 25
• 26
. 27 •
8
L.odt
s
Su~pl'nd
~
End Block
Item: 16 of - ,• Mark --<) [::> ""'I ~· 1!';:'1
100 ~ Prev1o u s Next Labf a lu es Notes Calculator

1 •
2
FA17 p 194.4
3 Antifungal therapy
4
FUNGAL CELL
5 LANOSTEROL SYNTHESIS
6
7
8 '---------~
h
( Terbmafine
~
' Squalene
Echinocandins
Anidulafungin
Caspofungin
9 X ualene epoxidase Micafungin
10 Squalene epoxide
11 CELL MEMBRANE INTEGRITY
12
13
/
Lanosterol r-;:lyenes
14 Amphoteridn B
~-a-demethylase Nystatin

....
Ergosterol
15 Azoles
Clotrimazole
16
. 17
Fluconazole
llraconazole
' ~,1
• 18 Ketoconazole
• 19
• 20
. 21
Miconazole
Voriconazote
-----· [
'--·
Flucytosine
---
)

. 22
FA17 p 195.2
• 23
• 24
Nystatin
• 25 MECHANISM Same as amphotericin B. Topical use only as too toxic for systemic use.
• 26
CliNICAL USE "Swish and swallow" for ontl cand idia~is (thrush); topical for diaper rash or vaginal candidia~is.
. 27 •
8
L.odt
s
Su~pl'nd
~
End Block
Item: 17 of - ,• Mark -<J
P~v1ous
[:::> "'I ~ ·· ~
100 ~ N @xt Labl lues N o tes Calcula to r

1
A 35-year-old man presents to the clinic with progressive odynophagia, anorexia, and weight loss. His history includes use of intravenous
2
heroin. On physical examinat ion, the pat ient has submandibular lymphadenopathy and white plaques lining the tongue and throat as
3 shown in the image. The plaques are easily scraped off with a depressor.
4
5
6
7
8
9
10
11

12
13
14

15
16
. 17
The pathology of this patient's ora l and esophagea l lesions is most likely to reveal which of the fo llowing?
• 18
• 19 :
A. Adenocarcinoma
• 20
. 21 B. Germ tube -negat ive fungus
. 22 C. Germ tube -posit ive fungus
. 23
D. Herpes simplex virus
• 24
• 25 E. Squamous cell carcinoma
• 26
. 27

8
Lode.
s
S uspe-nd
8
End Bloc:k
Item: 17 of - ,• Mark -<J
P~v1ous
[:::> "'I ~ ·· ~
100 ~ N@xt Labl lu es No tes Calcula t o r

A A
1
2 The correct answ er is c. 7 4 °/o chose this.
3 The patient's weight loss and drug use are consistent with human immunodeficiency virus (HIV) infect ion.
The oral and esophageal lesions described are thrush in an HIV-positive patient, caused by Candida
4
albicans, a germ tube-positive fungus. Germ tubes are the Initial form of hyphae that extend from the
5 fungal spore. A standard test used to detect Candida involves incubating the specimen In a serum for 3
6 hours at 37°C (98.6° F) and then examining t he spores for signs of germ tubes. The micrograph shows
germ tubes.
7 Cl"'doda b • ..,. <>re nl. lmmunodeficoency Candoda (fungus) VIrus Hypha Mocrograph Fungus .leoght loss Hlvt.UDS Infection
8
9
10pm
10
11 Image courtesy of Wikimedia
12
13 A is not correct. 3°/o chose t his.
14 Adenocarcinoma of the esophagus is typically present In patients with a long history of gastroesophageal
reflux disease and Barrett esophagus.
15 Barrett'~ ~ophaous G troesophageal reflux dtsease Esophagus Adenocarcinoma Reflux
16
17
• 18
• 19
• 20
. 21
I mage courtesy of Dr. Michael
. 22 Bonert
. 23
• 24
B is not co rrect . 1 5 °/o chose this .
Germ tube- negative fungi include hist oplasmosis, blastomycosis (shown in the image), and
• 25
coccidioidomycosis, none of which present s as oral thrush .
• 26 Cocctdtotdomyco Bl6iito nycost5 Htstoplasmosts Fungus Oral candidiasts Candidiasis

. 27

8
Lode.
s
S uspe-nd
8
End Bloc:k
Item: 17 of - ,• Mark -<J
P~v1ous
[:::> "'I ~ ·· ~
100 ~ N @xt Labl lues N o tes Calcula to r
A A
1
2 B is not correct . 15°/o chose this.
3
Germ tube- negative fungi include hist oplasmosis, blast omycosis (shown in the image), and
coccidioidomycosis, none of which present s as oral t hrush .
4 CoccidiOidomycosis Bla5tomycos1s H1stoplasmos1s Fungus Oral candidiasis Candidiasis
5
6
7
8
9
10
11 Image courtesy of Dr. Yale
12 Rosen
13
D is not correct . 4 °/o chose this.
14
Herpes simplex virus esophagitis is characterized by well-circumscribed, erythematous, punched-out
15 lesions. The image shows the cytologic changes of herpes esophagitis.
16 Esophag1t1~ Herpb esophag1t1s Herpes s1mplex Erythema Cytopathology Virus

17
• 18
• 19
• 20
. 21
. 22 Image courtesy of Dr. Michael
. 23 Bonert
• 24
E is not correct . 4 °/o chose this.
• 25 Squamous cell carcinoma (shown in t his image) is typically found in older black men with a history of
• 26 smoking or lye ingestion .
Squ;.mou~~ce I c c •oma C¥cmoma
. 27

8
Lode.
s
S uspe-nd
8
End Bloc:k
Item: 17 of - ,• Mark -<J
P~v1ous
[:::> "'I ~ ·· ~
100 ~ N @xt Labl lues N o tes Calcula to r

1 A
D is not correct. 4 °/o chose this. A

2 Herpes simplex virus esophagit is is characterized by well-circumscribed, erythematous, punched-out


3 lesions. The image shows t he cytologic changes of herpes esophagitis.
Esoph~1t1s Htrpes esophag1t1s Herpes s1mplex Erythema Cytopathology Virus
4
5
6
7
8
9
Image courtesy of Dr. Michael
10
Bonert
11

12 E i s n ot correct. 4 % chose this.


13 Squamous cell carcinoma (shown in this image) is typically found in older black men with a history of
smoking or lye ingestion.
14
Squamou~·cell carc~noma Carc1noma
15
16
17
• 18
• 19
• 20
. 21
Image courtesy of Wikimedia
. 22 Commons
. 23
• 24
• 25 Bottom Line:
• 26 Candida albicans is a germ tube-positive fungus t hat causes white plaques (thrush) in immunocompromised individuals .
C11nd1dl b•c""s I nnu odefit~ency Cand•da (fungus) Fungus
. 27

8
Lode.
s
S uspe-nd
8
End Bloc:k
Item: 17 of - ,• Mark --<) [::> ""'I ~· 1!';:'1
100 ~ Prev1o u s Next Labf a lu es Notes Calculator

1 •
FA17 p 149.1
2
Opportunistic fungal infections
3
4
Candida albicans alba = white. Dimorphic; forms pseudoh) phac and budding yeasts at 20°C · , germ tubes at
5 3JCC I]l.
6 Systemic or superficial fun gal infection. Causes oral B and esophageal thrush in
7 immunocompromised (neonates, steroids, diabetes, AI OS), vulvovaginitis (diabetes, use of
8 antibiotics), diaper rash, endocarditis (I drug users), disseminated candidiasis (espcciallr in
9 neutropenic patients), chronic mucocutaneous candidiasis.
10 Treatment: oral fluconazole/topical a1ole for ''aginal ; nystatin, fluconazole, or caspofungin for oral/
11 esophageal; fluconazole, caspofungin, or amphotericin B for systemic.
12
Aspergillus Septate hyphae that branch at 45° \ cute \ ngle . Produces conidia in radiating chains at end of
13
fumigatus conidiophore O.
14
Causes invasive aspergillosis in immunocompromised, patients " ith chronic granulomatous disease.
15
16
Can cause aspergillomas in pre-existing lung cavities, especially after TB infection.
17
Some species of Aspergillus produce \ natoxins (associated with hepatocellular carcinoma).
• 18 Allergic bronchopulmonary aspergillosis (ABPA): hypersensitivit)' response associated with
• 19
asthma and cystic fibrosis; may cause bronchiectasis and eosinophilia.
• 20
. 21
Cryptococcus 5- 10 jlm with narrow buckling. lleavily encapsulated yeast. lot dimorphic.
. 22
neoformans Found in soil, pigeon droppings. Acquired through inhalation with hematogenous dissem ination
• 23 to meninges. Culture on Sabouraud agar. llighlighted with India ink (clear halo 1]1) and
• 24 mucicarmine (red inner capsule ). Latex agglutination test detects polysaccharide capsular
• 25 antigen and is more specific.
• 26 Causes cryptococcosis, cryptococcalmcningitis, cryptococcal encephalitis ("soap bubble" lesions
. 27 in hr!lln\ nrim!lril v in imnln nnl'nmnrru,n icPrl •
8
L.odt
s
Su~pl'nd
~
End Block
Item: 17 of - ,• Mark --<) [::> ""'I ~· 1!';:'1
100 ~ Prev1o u s

1
Next Labf a lu es Notes Calculator
.
2 Mucor and Rh izopus Irregular, broad, nonseplale hyphae branching al "ide angles
3 spp. Mucormycosis. Causes disease mostly in keloaeidolic diabetic and/or neutropenic patients (cg,
4 leukemia). Fungi proliferate in blood vessel walls, penetrate cribriform plate, and enter bra in.
5 Rh inocerebral, frontal lobe abscess; cavernous sinus thrombosis. Headache, facial pain, black
6 necrotic eschar on face; ma\' h e~ve cmnial nerve involvement.
'
7 Treatment: surgical debridement, amphotericin B.
8
9
10
11

12
13
14

15
16
17
D
• 18
• 19
• 20
. 21

. 22
• 23
• 24
• 25
a
• 26
. 27

8
L.odt
s
Su~pl'nd
~
End Block
Item: 18 of - ,• Mark -<J [:::> "'I ~· ~
100 ~ P~v1ous N @xt Labl lues N o tes Calcula to r
6
1
An 8-year-old boy with an unknown immunizat ion history presents to the emergency department wit h a fever of 39°C ( 102.2°F) . He has
2 had a "runny nose" for several days and has eaten very little for the past 2 days. He has a moderately red throat with a white to yellow-
3 gray tonsillar coating as shown in t he image, a clear chest, and bilateral enla cervical lymph nodes.
4
5
6
7
8
9
10
11
12
13
14

15
16
17
• 18
• 19
What is t he mechanism of t he t oxin causing t he child's illness?
• 20
. 21 :
A. Adenosine diphosphat e ri bosylat ion of adenylate cyclase
. 22
. 23 B. Adenosine diphosphat e ribosylation of elongat ion factor 2
• 24 C. Glycosidase cleavage of ribosomal RNA
• 25
D. Guanylate cyclase stimulation
• 26
. 27 E. Overactivates adenvlate cvclase to increase cyclic adenosine monoohosohate
8
Lode.
s
S uspe-nd
8
End Bloc:k
Item: 18 of - ,• Mark -<J [:::> "'I ~ · ~
100 ~ P~v1ous N @xt Labl lues N o tes Calcula to r
A A
1
The correct answ er is B. 87% chose this.
2
This patient most likely has diphtheri a. Symptoms of dipht heria start usually 2-5 days after exposure and \
.....,.... ,.
3 come on gradually, beginning wit h a sore t hroat and fever. A classic sign is a gray or white patch that ' < -
4 develops in the throat as seen in the vignette image. Diphtheria toxin produced by Co1ynebacterium
diphtheria (shown in this image) causes adenosine diphosphate ribosylation of elongation factor 2. This _J I

5
inhibits protein synthesis in animal cells, resulting in cell death. -,_,- ...,'
--'· '
-
. ....
6
7
8
Dtphthena Otpht+u 11 tOJ '" Protem Elongabon factor Protein synthesis Toxin Protem b1osynth~ s Corynebac:tenum Fever Adenos1ne Sore throat Apoptosis . J•

, ''
, ,... -'
'"'
'
- /
,

.,
9 I
10
11 Image courtesy of CDC/Graham
--
Heid
12
13 A is not correct. 5°/o chose this.
14 Cholera toxin causes ADP ribosylation of adenylate cyclase (G 5 regulatory protein). This results in increased levels of intracellular cAMP that
15 promote secretion of fluid and electrolytes in intestinal epithelium, leading to diarrhea.
Cholera tox1n Adenylyl cyclase Cholera Cyclic adenosine monophosphate Protem Diarrhea Adenosine d1phosph4te Ep1thellum Toxm Electrolyte Intracellular Regulation of gene express1on Secret1on
16
17 C is not corr ect. 2 °/o chose thi s.
18 Shiga toxin causes glycosidase cleavage of ri bosomal RNA. This results in inactivation of t he mammalian 60S ri bosomal subunit and leads to
• 19
inhibition of protein syn thesis and deat h of t he susceptible cell.
Shiga toxin Glycos1de hydrolase R1bosomal RNA Protein Ribosome Eukaryotic large ribosomal subun1t (60S) Prote1n synthesis Tox1n RNA Protein biosynthesis
• 20
. 21
D is not co rrect. 2 °/o chose this .
Escherichia coli stable toxin st imulat es guanylate cyclase and also leads to the stimulation of chloride secretion and inhibit ion of sodium
. 22
chloride absorption. The end resu lt is secret ion of free water int o the intestinal lumen, which manifest s clinically as watery diarrh ea .
. 23 Sodium chloride E~chenchia coh Guanylyl cyclase Lumen (anatomy) Diarrhea Human gastro1ntest1nal tract Glt~tromtest1nal tract Sod1um Tox1n Secretion Absorption (chem1stry) Chloride

• 24
E is not correct. 4 °/o chose this.
• 25
The heat-labile toxin produced by Enterotoxigenic E coli (ETEC) stimulates adenylate cyclase to increase cAMP. The increase in cAMP results In
• 26 increased chloride secretion in the gut and water efflux .
. 27 Adenytyt cycl e Cyc!•c ade o~me monophosphate Escherichia coli Toxin Efflux mcrob•olooy) Sec::rebon Lab•lltv Gc.tto•nte5bnal tract

8
Lode.
s
S uspe-nd
8
End Bloc:k
Item: 18 of - ,• Mark -<] 1:> ""'I ~· 1!';:'1
100 ~
1
. yy y y
Prev1o u s
.
Next
p p
Labf a lu es Note s Calculator
. gy . ty

2
3 Bottom Line:
4 Diphtheria, which presents with tonsil infection and a red sore throat covered with a grayish membrane, results from a bacterial toxin that
5 causes adenosine diphosphate ribosylat ion of elongation factor 2.
O.phthena Adeno4i•ne dtphosphMe Tonsil Adenosine Toxin Exotoxin Elongation factor Infection
6
7
8
141;filo 1iol for year: 2017 •
9 FIRST A ID FACTS

10
11 FA17 p 135.1
12
Corynebacterium Gram Et> rod; transmitted via respiratory Coryne = club shaped.
13
diphtheriae droplets. Causes diphtheria' ia exotoxin Black colonies on cystine-Lellurite agar.
14
encoded by P-prophage. Potent exotoxin ABC DEFG:
15
inhibits protein synthesis via ADP-ribosylation ADP-ribosylation
16
17
of EF-2. P-prophage
18
Symptoms include pseudomembranous Corynebacterium
• 19 pharyngitis (grayish-while metnbra ne r.J) Diphtheriae
• 20 with lymphadenopathy, myocarditis, and Elongation Factor 2
. 21 arrhythmias . Granules
. 22 Lab diagnosis based on gram Et> rods with
0 23 metachromatic (blue and red) granules and
0 24 Et> Elek test for toxin.
0
25 Toxoid vaccine prevents diphtheria.
0
26
. 27

8
L.odt
s
Su~pl'nd
~
End Block
Item: 19 of - ,• Mark -<J
P~v1ous
[:::> "'I ~ ·· ~
100 ~ N @xt Labl lues N o tes Calculato r

A A
1
A 22-year-old man visits his college medical cent er complaining of flu- like symptoms that have persisted fo r about a week. He reports low- ~~AI
2
grade fevers, night sweats, a painful sore throat, headaches, and increasing fatigue. He used to exercise 5 days a week but is no longer
3 able to because of "lack of energy." Physical examination reveals enlarged cervical lymph nodes and splenomegaly.
4
5 What is the structure of the virus that has infected this patient?
6 :
7 A. Linear, double-stranded, DNA virus with an envelope
8 B. Linear, double-stranded, DNA virus without an envelope
9
C. Linear, double-stranded, RNA virus without an envelope
10
11 D . Partially circular, double-stranded, DNA virus with an envelope
12 E. Segmented, linear, single-stranded, negat ive-polarity, RNA virus with an envelope
13
14

15
16
17
18
• 19
• 20
. 21
. 22
. 23
• 24
• 25
• 26
. 27

8
Lode.
s
S uspe-nd
8
End Bloc:k
Item: 19 of ~ ,• Mark <::J [:::> ""I ~· ~'j
100 J.. Previous Next LAb faiUI~S Notes Calculator

1
2
The correct answer is A. 60°/o chose this.
The quest ion st em descri bes a cl assic case of mononucleosis, an infect ion caused by the Epst ein-Barr virus (EBV) t hat often presents wit h f lu-
3
like symptoms, increasing fat igue, lymphadenopat hy, and splenomegaly. EBV is a mem ber of the herpesvirus fa mily and is characterized by
4 linear, double-stranded, enveloped DNA. Herpesviruses are unique in t hat they are t he only viruses t hat obtain envelope via budding f rom t he
5 nuclear membrane.
Epstein-Barr virus Splenomegaly Infectious mononucleosis Lymphadenopathy Herpesviridae Virus Influenza-like illness Fatigue (medical) DNA Cell membrane Infection
6
7 B is not correct. 20% chose this .
8 Linear, double-stranded DNA wit hout an envelope describes the adenoviruses. Adenovirus is well known for conjunct ivit is, or " pink eye." I t can
also cause respirat ory t ract infections, hemorrhagic cystit is, and gast roent erit is.
9
Hemorrhagic cystitis Adenoviridae Gastroenteritis Conjunctivitis Urinary tract infection DNA Respiratory tract Respiratory tract infection Bleeding
10
C is not correct. 8 °/o chose this .
11
The only double-stranded RNA viruses known t o infect huma ns are t he reoviruses. These pat hogens include Coltivirus, which causes Colorado
12
t ick feve r, and Rotavirus, which causes gastroenterit is. Rotavirus infection is the principal cause of fatal diarrhea in children t hroughout t he
13 worl d. Colorado tick fever is t ransmitted v ia t icks carried by rodents and is fo und most commonly in hikers.
14 Colorado tick fever Rotavirus Coltivirus Gastroenteritis RNA virus Diarrhea Double-stranded RNA viruses Virus Reoviridae Pathogen Tick Fever RNA Babesiosis

15 D is not correct. 6°/o chose this.


16 Partially circular, double-stranded, enveloped DNA descri bes t he hepadnaviruses. The main infectious pat hogen in t his family is hepatitis B
17 virus. Its main modes of t ra nsmission are via blood and sexual cont act. This pat ient does not present with signs of hepat it is.
Hepatitis 8 Hepatitis Hepatitis 8 virus Pathogen Hepadnaviridae Virus DNA
18
19 E is not correct. 6°/o chos e this.
• 20 The ort homyxoviruses are characterized by segment ed, single-stranded, enveloped RNA with negative polarity. The principal disease caused
by t hese v iruses is inf luenza. Although t his pat ient does have flu-like symptoms, lymphadenopat hy and splenomegaly are not characterist ic of
. 21
t he f lu .
. 22 Splenomegaly Lymphadenopathy Orthomyxoviridae Influenza Virus Influenza-like illness RNA

• 23
• 24
Bottom Line:
• 25
Epstein-Barr virus ( EBV), which causes mononucleosis, is a linear, double-st randed, enveloped DNA virus .
• 26 DNA virus Epstein-Barr virus Infectious mononucleosis Virus DNA
. 27

8
Lock
s
Suspend
0
End Block
Item: 19 of ~ ,• Mark <::J [:::> ""I ~· ~'j
100 J.. Previous Next LAb faiUI~S Notes Calculator

1 • •
FA17 p 160.1
2
DNA viruses
3
VIRAL FAMILY ENVELOPE ONA STRUCTURE MEDICAL IMPORTANCE
4
5
Herpesviruses Yes OS and linear See Herpesviruses entry
6 Poxvirus Yes OS and linear Smallpox eradicated world wide by use of the live-
7 (largest 0 1 A virus) attenuated vaccine
8 Cowpox ("mi lkmaid blisters")
9 Molluscum contagiosum -Aesh-colored papule with
10 central umbilication
11
Hepadnavirus Yes Partially OS and circular HBV:
12
Acule or chron ic hepatitis
13
lot a retrovirus but has reverse transcriptase
14
15 Adenovirus 0 OS and linear Febrile pharyngitis rJ-sore throat
16 Acute hemorrhagic cystitis
17 Pneumonia
18 Conjunctivitis-"pink eye"
19
• 20
. 21
. 22
• 23
• 24
Papillomavirus No OS and circular HPV-warts (serotypes 1, 2, 6, 11), Cl , cervical
• 25
cancer (most commonly 16, 18)
• 26
. 27 • Polyomavirus No OS and circular JC virus-progressive multifoca lleukocncephalopathy •

8
Lock
s
Suspend
0
End Block
Item: 19 of ~ ,• Mark <::J [:::> ""I ~· ~'j
100 J.. Previous Next LAbfaiUI~S Notes Calculator

1 • •
FA17 p 159.5
2
3 DNA virus Some general rules-all DNA viruses:
4 characteristics
GENERAL RULE COMMENTS
5
Are HHAPPPPy viruses Hepadna, Herpes, Adeno, Pox, Parvo,
6
Papilloma, Polyoma.
7
8 Are double stranded Except pan·o (single stranded).
9 Have linear genomes Except papilloma and polyoma (circular,
10 supercoiled) and hepadna (circular,
11
incomplete).
12
13
Are icosahedral F.xcept pox (complex).
14 Replicate in the nucleus Except pox (carries own Dr A-dependent RNA
15 polymerase).
16
17
FA17 p 163.1
18
RNA viruses
19
VIRALFAMILY ENVELOPE RNA STRUCTURE CAPSID SYMMETRY MEDICAL IMPORTANCE
• 20
. 21 Reoviruses No OS linear Icosahedral Coltivirusa- Colorado tick fever
. 22 10- 12 segments (double) Rotavirus-cause of fatal diarrhea in children
• 23 Picornaviruses 0 SS ® linear Icosahedral Poliovirus-polio-Salk/Sabin vaccines- IPV/OPV
• 24 Echovirus- aseptic meningitis
• 25 Rhi novirus-"common cold"
• 26 Coxsackievirus-aseptic meningitis; herpangina
• / _ __ __ .._)_ LJ :_ .._ ___ r_ __ __\ L - - .J C ._.._ - · - .J - - • • • t..L •
. 27

8
Lock
s
Suspend
0
End Block
Item: 19 of - ,• Mark -<J
P~v1ous
[:::> "'I ~ ·· ~
100 ~ N @xt Labl lues N o tes Calculato r
A
1 A

Retroviruses Yes SS ® linear Icosahedral Have re,·erse transcriptase


2
2 copies (IITLV), IITLV-T-cellleukemia
3
complex HIV-AIDS
4
and conical
5
(HIV)
6
7 Coronaviruses Yes SS ®linear Helical "Common cold," SARS, ~fERS
8 Orthomyxoviruses Yes ss e linear llelical lnAuen1.a virus
9
8 segments
10
11
Paramyxoviruses Yes ss e linear llelical PaRa h• xovirus:
12
1 onsegmented Parainfluenza-croup
13
RSV-bronchiolitis in babies; Rx-riba,·irin
14
\leasles, \.lumps
15 Rhabdoviruses Yes sse linear Helical Rabies
16
17
Filoviruses Yes ss e linear llelica l Ebol a/~ larburg hemorrhagic fever-often fatal!
18 Arenaviruses Yes ss ®a nd e Helica l LCM V- lymphocytic choriomeningitis vi rus
19 circular Lassa fever encephalitis-spread by rodents
• 20 2 segments
. 21
. 22
Bunyaviruses Yes ss e circular Helica l California encephalitis3
3 segments Sand Ay/Rift aile)' fevers 3
. 23
Crimean-Congo hemorrhagic fever
• 24
llanla\'irus-hemorrhagic fever, pneumonia
• 25
• 26
Delta virus Yes sse circular Uncertain HOVis a "defective" ,-irus that requires the
. 27 • presence of HBV to rep! icatc
8
Lode.
s
S uspe-nd
8
End Bloc:k
Item: 20 of - ,• Mark -<J [:::> "'I ~· ~
100 ~ P~v1ous N @xt Labl lues N o tes Calcula to r

1 A A 75-year-old man presents with a few weeks of fever and chills. He has hypertension and takes med icines daily t o lower his blood ~AI A

2 pressure and cholesterol, although he cannot remember t heir names. He also has a history of mit ra l valve replacement and is on
3
anticoagulation therapy. His wife comments t hat he underwent a dental procedure a few weeks ago. The pat ient admits t hat he was
supposed to take antibiotics, but did not complete th e ful l course. On physical exam, the physician notes painful, red, raised lesions on his
4 fingers, similar to that shown in the image.
5
6
7
8
9
10
11
12
13
14

15
16
17
18
West J Emerg Med. Feb 2012; 13(1) : 92-93. doi: 10.5811/ westjem.2011.6.6806
19
• 20
Which of the fol lowing is most likely causing this man's condit ion?
. 21
:
. 22
A. Haemophilus influenzae
. 23

' 24 B. Staphylococcus aureus


' 25 C. Staphylococcus epidermidis
' 26
D. Streptococcus pyogenes
. 27

8
Lode.
s
S uspe-nd
8
End Bloc:k
Item: 20 of - ,• Mark -<J [:::> "'I ~· ~
100 ~ P~v1ous N @xt Labl lues N otes Calculator

1 g py I I p I g p I

2
supposed to take ant ibiot ics, but did not complet e t he full course. On physical exam, the physician notes painful, red, raised lesions on his
fingers, similar to that shown in the image.
3

4
5
6
7
8
9
10
11
12
13
14

15
16
17 I mage courtesy ofTsagartos, eta/. West J Emerg Med. Feb 2012; 13(1) : 92- 93. doi: 10. 5811/ westjem. 2011 .6.6806

18
19
Which of the following is most likely causing t his man 's condi tion?
• 20 :
. 21 A. Haemophilus influenzae
. 22 B. Staphylococcus aureus
. 23
C. Staphylococcus epidermidis
' 24
' 25
D. Streptococcus pyogenes
' 26 E. Viridans streptococci
. 27

8
Lode.
s
Suspe-nd
8
End Bloc:k
Item: 20 of ~ ,• Mark <:::1 t::> ""I ~· ~'j
100 J.. Previous Next LAb faiUI~S Notes Calculator

1
The correct a nswer is E. 76°/o chose this .
2
Viridans streptococci are normal flora of the oral cavit y and can cause subacute bacterial endocarditis, one of t he symptoms of which is Osier
3 nodes, as shown in the image. Prophy laxis against viri dans st rept ococci should be init iated before dent al procedures in patients wit h art if icial
4 heart valves and/or ot her risk facto rs for bact eri al endocardit is.
Osler's node Endocarditis Viridans streptococci Streptococcus Artificial heart valve Subacute bacterial endocarditis Infective endocarditis Heart valve Acute (medicine) Mouth Artificial heart
5
Preventive healthcare Human microbiota
6
7 A is not correct . 1 °/o chose t his.
8 Haemophilus influenzae does not commonly cause endocardit is and is not part of normal oral flora, and t hus prophylaxis for it is not needed
before oral procedures in t hose wit h artificial heart valves.
9 Haemophilus influenzae Endocarditis Haemophilus Artificial heart Heart valve Preventive healthcare
10
B is not correct. 5% chose t his .
11
Although Staphylococcus aureus may cause endocardit is, prophylaxis against S. aureus is not usually needed before dent al procedures in
12 patients with artificial heart valves.
13 Staphylococcus aureus Endocarditis Staphylococcus Heart valve Artificial heart valve Artificial heart Preventive healthcare

14
C is not correct. 1 2% chos e this .
15 Staphylococcus epidermidis can cause endocardit is on artif icial heart valves. I n such cases it generally gains access to the bloodst ream
16 t hrough int ravenous or arterial lines in hospit al. It is not part of t he normal oral flora, and t hus prophylaxis for it is not given before oral
17
procedures in t hose wit h artif icial heart valves.
Staphylococcus epidermidis Endocarditis Staphylococcus Intravenous therapy Artificial heart Artificial heart valve Heart valve Preventive healthcare Circulatory system
18
19 D is not correct. 6°/o chose this.
Streptococcus pyogenes does not commonly cause endocardit is; however, it does cause pancardit is in patient s with a history of rheumatic
20
heart disease .
. 21 Streptococcus pyogenes Carditis Rheumatic fever Endocarditis Streptococcus Cardiovascular disease
. 22
• 23
Bottom Line :
• 24
Pat ients wit h cert ain conditions t hat cause turb ulent cardiac blood f low, such as a prost het ic heart valve, history of endocardit is, and
• 25
unrepaired congenital cyanot ic heart disease should receive antibiot ic prophylaxis before undergoing dental procedures. Bact eri a from the
• 26 oral cavit y such as viridans streptococci may enter the bloodst ream and cause bacterial endocardit is .
. 27 Artificial heart valve Endocarditis Antibiotics Viridans streotococci Heart valve Cvanosis Infective endocarditis Streotococcus Bacteria Conoenital disorder Mouth Prosthesis Cvanotic heart defect

8
Lock
s
Suspend
0
End Block
Item: 20 of - ,• Mark -<J [:::> "'I ~ · ~
100 ~ P~v1ous N @xt Labl lues N o tes Calcula to r
A
1 A

FA17 p 132.3
2
3 Viridans group Cram El1, a-hemolytic cocci. They are normal Sanguinis = blood. Think, "there is lots of
4 streptococci Aora of I he oropha rym: that cause dental blood in the heart" (endocarditis). S sanguinis
5 caries (Streptococcus mutans and S mitis) makes dcxtrans, which bind to fibrin-platelet
6 and subacute bacterial endocarditis at aggregates on damaged heart valves.
7 damaged heart valves (S sanguinis). Resistant Viridans group strep li,·e in the mouth because
8 to optochin, differentiating them from they are not afraid of-the-chin (op-to-chin
9 S pneumoniae, which is a-hemolytic but is resistant).
10 optochin sensiti,·e.
11

12
FA17 p 194.1
13
14 Antimicrobial CLINICAL SCENARIO MEDICATION
15 prophylaxis High risk for endocarditis and undergoing A111oxicillin
16
surgical or dental procedures
17
18
Exposure to gonorrhea Ceft riaxone
19 History of recurrent UTis TMP-SMX
20
Exposure to meningococcal infection Ceftriaxone, ciprofloxacin, or rifampin
. 21
. 22 Pregnant " ·oman carrying group B strep Intrapartum penicillin Cor ampicillin
. 23 Prevention of gonococcal conjunctivitis in Erythromycin ointment on eyes
• 24 newborn
• 25
Pre,·ention of postsurgical infection due to Cefazolin
• 26
S aureus
. 27 •
8
Lode.
s
S uspe-nd
8
End Bloc:k
Item: 20 of - ,• Mark -<J [:::> "'I ~ · ~
100 ~ P~v1ous N @xt Labl lues N o tes Calcula to r
A A
1
FA17 p 299.2
2
3 Bacterial endocarditis F'e \·er (most common S)111plom), new murmur, Mitral valve is most frequently involved.
4 Roth spots (round white spots on ret ina Tricuspid \'alve endocarditis is associated with
5 surrounded by hemorrhage ), Osler nodes l drug abuse (don't ''tri" drugs). Associated
6 (lender raised lesions on flnger or toe pads \\ ith S aureus, Pseudomonas, and Candida.
7
due to immune complex deposition), Janeway Culture 8 ; most likely Coxiella bumetii,
8
lesions (small, painless, err thematous lesions Bartonella spp., II CEK (l laemophilus,
9
on palm or sole) [i, glomerulonephritis, Aggregatibacter (formerly Actinobacillus),
10
septic arterial or pulmonary emboli, splinter Cardiobacterium. Eikenella, Kingella)
11

12
m
hemorrhages on nail bed. .\ lultiple blood 'I Bacteria FR0:\1 JA~E '1:
cultures necessary for diagnosis. Fe\·er
13
14
Acute-$ aureus (high virulence). Roth spots
15
Large vegetations on pre\•iousl)' normal Osler nodes
16
va lves 0 . Rapid onset. i\turmur
17 • Subacute-viridans streptococci (low Janeway lesions
18 viru lence). Smaller vegetations on Anemia
19 congenitally abnormal or diseased valves. Nail-bed hemorrhage
20 Sequela of denta l procedures. Gradual Emboli
. 21 onset.
. 22 S bovis (gallolyticus) is present in colon ca ncer,
. 23 S epidermidis on prosthetic valves.
• 24 Endocarditis mar also be nonbaclerial
• 25 {marantic/thrombotic) zoto malignancy,
• 26
hypercoagulable state, or lupus.
. 27

8
Lode.
s
S uspe-nd
8
End Bloc:k
Item: 21 of - ,• Mark -<J
P~v1ous
[:::> "'I ~ ·· ~
100 ~ N @xt Labl lues N o tes Calculato r

1
A 25-year-old woman with no significant medical history complains of palpitations and chest pain on exertion . Physical examination is
2
notable for mild fever and a new systolic murmur. Cardiac enzymes are elevated. If a cardiac biopsy were t o be performed at this t ime, it
3 would show diffuse myocardial degeneration and necrosis with an inflammatory infiltrate.
4
5 Which pathogen is most likely responsib le for these find ings?
6 :
7 A. Coxsackievirus
8 B. Hepatitis B virus
9
C. Rubella virus
10
11 0 . Staphylococcus aureus
12 E. Staphylococcus epidermidis
13
14

15
16
17
18
19
20
. 21
. 22
. 23
• 24
• 25
• 26
. 27

8
Lode.
s
S uspe-nd
8
End Bloc:k
Item: 21 of - ,• Mark -<] 1:> ""'I ~· 1!';:'1
100 ~ Prev1o u s Next Labf a lu es Notes Calculator

1 •
The correct answer is A. 66°/o chose this.
2 Palpitations, chest pain, and mitral regurgit ation in an otherwise healthy adult are symptoms of vent ricular ,
3 dysfunction typical of acute myocarditis. Elevated cardiac enzymes and inflammatory infiltrate wit h necrosis ' •
>
I
4 on biopsy (shown in the image) are signs of acute myocardial damage. Coxsackievirus B is a single-
stranded RNA virus with a linear structure and a nonenveloped, icosahedral capsid that is a member of the
5 enterovirus group of the Picornaviridae family. Some of the most common viral causes of myocarditis
6 include human herpesvirus-6, parvovirus Bl9, and the enteroviruses, most notably, coxsackievirus B. Other
7 infectious causes of myocarditis include Trypanosoma cruzi, Borrelia burgdorferi, and Corynebacterium
diphtheriae. I
8 us HoJman herpesV1rus 6 Trypanosoma cruz, Corynebacterium d1phthenae Borrd•a a.,... ,1dorfen ParwVIrus 819 Coxsackie--nrus
Entero us P•cor"
9 Myoc:ard•bs M•tral•nsufftc•encv V.ral envelope CoxsacJOe 8 virus Cctps•d RNA VIrus Chest pam &opsy ParvoVItuS Palp1tabons Necros•s VIrus RNA
10
Regul..- ocosahedron
11
Image courtesy of Wikimedia
Commons
12
13 B is not correct. 2°/o chose this.
14 Hepatitis B, a double-stranded DNA virus wit h a circu lar structure and an enveloped, icosahedral capsid, is not a frequent cause of viral
15
myocarditis. It has both acute and chronic manifestations that include symptoms such as fever, malaise, headache, anorexia, vomiting, dark
urine, and jaundice.
16 DNA v1rus Myocard•t•s Jaund1ce Hepat1t1s B Caps1d Hepatitis Malaise Headache Unne Anorex•a nervosa Anorex•a (symptom) Vom1t1ng DNA Regular icosahedron Icosahedron V1rus Fever
17
C is not correct. 2°/o chose this.
18
Rubella, a sing le-stranded RNA v irus wit h a linear structure and an enveloped icosahedra l capsid, is t eratogenic after it crosses t he placenta.
19 Rubella can cause congenital cardiac complications, including patent ductus arteriosus and pulmonary stenosis, as well as catara cts,
20 microcephaly, and deafness. It is not , however, a cause of acute viral myocarditis in adu lts.
Patent ductus arteriosus Myocard1tis Microcephaly Placenta Rubella Teratology Caps1d Pulmomc stenosis RNA v~rus Stenosis RNA Ductus arteriosus Cataract Hearing loss Congemtal d1sorder Virus
21
. 22 D is not correct. 23°/o chose this .
0 23 Staphylococcus aureus is a gram-posit ive, coagulase-positive, catalase-positive coccus that is a very common cause of endocarditis.
0 24
Endocarditis typically manifests with high fevers, chills, myalgias, Janeway lesions, and a new murmur. Myocardial biopsy would not
demonstrate inflammatory infiltration or necrosis.
0
25 staphylococcus aureu Endocard1t1,.. GrafTI-positive bacteria Janeway lesion Staphylococcus Nec:rosts Btopsy Coccus Heart murmur
0
26
E i s not correct. 7% chose this. •
. 27

8
L.odt
s
Su~pl'nd
~
End Block
Item: 21 of ~ ,• Mark <::J [:::> ""I ~· ~'j
100 J.. Previous Next LAbfaiUI~S Notes Calculator

1
E is not correct. 7 °/o chose this.
2
Staphylococcus epidermidis is a gram -posit ive, catalase -posit ive, coagulase-negat ive coccus t hat causes endocarditis, wh ich is inflammation
3 of t he interior lining of heart and it s va lves, as opposed to myocarditis, or inf lammat ion of t he cardiac muscl e. S. epidermidis is a common
4 species of skin flora that tends t o infect prost het ic heart va lves and nat ive heart valves in int ravenous drug users.
Skin flora Staphylococcus epidermidis Endocarditis Gram- positive bacteria Myocarditis Staphylococcus I ntravenous therapy Cardiac muscle Heart valve I nflammation Coccus Prosthesis Muscle
5
Drug injection
6

I
7
8 Bottom Line:
9 I nfectious diseases can damage t he endocardium, myocardiu m, and pericardiu m of the heart. Myocardit is damages the ca rdiac myocytes,
10 leading t o signs of vent ricula r dysfunct ion, elevat ed cardiac enzymes, inflammatory infilt rat ion, and myocardial necrosis. There are many
causes of t he underly ing inf lammat ion of t he myocardium; among the most common v iral infect ions associat ed wit h myocardit is is
11
coxsackievirus B.
12 Pericardium Endocardium Coxsackievirus Myocarditis Cardiac muscle I nflammation I nfection Necrosis Myocyte Cardiac muscle cell Cardiac marker Ventricle (heart) Virus

13
14
15 l@);fil ~1hl for year:l 2o17 y
FIRST AID FACTS
16
17
FA17 p 163.1
18
RNA viruses
19
VIRAL FAMILY ENVELOPE RNA STRUCTURE CAPSID SYMMETRY MEDICAL IMPORTANCE
20
21 Reoviruses 0 OS linear Icosahed ral C oltivirus"- Colorado tick fever
. 22 10- 12 segments (double) Rotavirus-cause of fa tal diarrhea in children
• 23 Picornaviruses No SS ® linear Icosahed ral Poliovirus-polio-Salk/Sabin vaccines- IPV/OPV
• 24 Echovirus-aseptic meningitis
• 25 Rhi novirus-"common cold"
• 26 Coxsackievirus-aseptic mening itis; herpang ina
. 27 I , I 1 I · , r \ 1 1 r .1

8
Lock
s
Suspend
0
End Block
Item: 21 of - ,• Mark -<] 1:> ""'I ~· 1!';:'1
100 ~ Prev1o u s Next Labf a lu es Notes Calculator

1 • •
FA17 p 297.1
2
Cardiomyopathies
3
4 Dilated Most common cardiomyopathy (90% of cases). Systolic dysfun ction ensues.
5 cardiomyopathy Often idiopathic or familial. Other etiologies Eccentric hypertrophy rJ (sarcomcrcs added in
6 include chronic Alcohol abuse, wet Beriberi, series).
7 Coxsackie B \'iral nwocarditis, chronic ABCCCD.
'
8 Cocaine use, C hagas disease, Doxorubicin Takotsubo cardiomyopathy: "broken heart
9 toxicity, hemochromatosis, sarcoidosis, syndrome"-ventricular apical ballooning
10 peripartum cardiomyopathy. likely clue to increased sympathetic stimulation
11 Findings: HF, S3, systolic regurgitant murmur, (stressful situations).
12 dilated heart on echocardiogram, balloon
13 appearance of hea rt on C XR.
14
Treatment: 1 a+ restriction, ACE inhibitors,
15
~-blockers, diuretics, digoxin, lC D, heart
16
17
18
19
Hypertrophic
cardiomyopathy
transplant.
60-70% of cases are familit~l , autosomal
dominant (most common ly due to mutations
Diastolic d)•sfunction ensues.
larked ventricular concentric hypertrophy
I
20 in genes encoding sarcomeric proteins, such as (sarcomercs added in parallel) rn,often septal
21 myosin bind ing protein C and ~ -myos in heavy predominance. Myofibrillar disarray and
0
22 chain). Can be associated with Friedrcich fibrosis.
0 23 ataxia. Causes syncope during exercise and Hypertrophic obstructi,·e cardiomyopathy
0 24 mar lead to sudden death in young athletes (subsel)-asymmetric septal h) pertroph)'
0
25 due to ,·entricular arrlwthmia. and systolic anterior motion of mitral valve
'
0
26 Findings: S4, systolic murmur. Ma) sec mitral - outAO\\ obstruction - dyspnea, possible
0
27 .. I •
8
L.odt
s
Su~pl'nd
~
End Block
Item: 21 of ~ ,• Mark <::J [:::> ""I ~· ~'j
100 J.. Previous Next LAb faiUI~S Notes Calculator

1 • •
FA17 p 205.1
2
3 Necrosis Enzymatic degradation and protein denaturation of cell due to exogenous injury -+ intracellular
4 components leak. lnAam matory process (unlike apoptosis).
5 TYPE SEENIN DUE TO HISTOLOGY
6 Coagulative lschem ia/in farcts in Ischemia or in fa ret ion; Cell outlines preserved but nuclei disappear;
7 most tissues (except proteins denature, then t cytoplasmic bi nding of eosin dyes fJ
8
brain) enzymatic degradation
9
10
Liquefactive Bacterial abscesses, Neutrophils release Early: cellular debris and macrophages
11
brain infarcts lysosomal enzymes Late: cystic spaces and cavitation (brain)
12 that digest the leutrophils and cell debris seen with
13 tissue Ill; enzymatic bacterial infection
14 degradation first, then
15 proteins denature
16 Caseous TB, systemic fun gi Macrophages wall F'ragmented cells and debris su rrounded by
17 (eg, Histoplasma off the infecting lymphocytes and macrophages
18 . .
capsulatum), Nocardia m1croorgan 1sm
19
-+ granular debris [!II
20
21
. 22
• 23
• 24
Fat Enzymatic: acute
pancreatitis
(saponification of
peripancreatic fat)
Damaged cells release
lipase to break
down triglycerides,
Iiberating fatty acids
Outlines of dead fat cells without peripheral
nuclei; saponification of fat (combined with
Ca 2+) appears dark blue on H&E sta in [!] I
• 25 Nonenzymatic: to bind ca lcium
• 26 traumatic (eg, injury to - saponification
. 27 • ........,.. . . ...... : ...,.... ..,.\ •

8
Lock
s
Suspend
0
End Block
Item: 21 of - ,• Mark -<] 1:> ""'I ~· 1!';:'1
100 ~ Prev1o u s Next Labf a lu es Notes Calculator

1 • vessels (eg, polyarteritis combine with


2 nodosa), preeclampsia, fibrin - vessel wall
3 malignant damage (type Il l
4
hypertension hypersensitivit)
5
reaction)
6
7
Gangrenous Distal extremity and Dry: ischemia Coagulative
8
C I tract, after chronic Wet: superinfection Liquefactive superimposed on coagulati,·e
9 ischemia
10
11

12
13
14

15
16
17
18
19
20
21
. 22
0 23
0 24
0
25
0
26
. 27 •
8
L.odt
s
Su~pl'nd
~
End Block
Item: 22 of - ,• Mark -<J [:::> "'I ~ · ~
100 ~ P~v1ous N@xt Labl lues Notes Calculator

1
Exchange of genet ic material between bacteria can occur through t he F factor plasmid. F+ bacteria may donat e genetic material to F-
2 partners through direct cont act via a sex pilus.
3

4 Which of the fol lowing processes best descri bes this mode of genetic exchange?
5
:
6 A. Conjugation
7
B. Hybridization
8
9 C. Transduction
10 0 . Transformation
11
E. Translocation
12
13
14

15
16
17
18
19
20
21
. 22
. 23
• 24
• 25
• 26
. 27

8
Lode.
s
Suspe-nd
8
End Bloc:k
Item: 22 of ~ ,• Mark <::J [:::> ""I ~· ~'j
100 J.. Previous Next LAbfaiUI~S Notes Calculator

1
2
3 The correct answer is A. 77°/o chose this.
4 Conj ugat ion refers to the process by wh ich DNA is t ransferred f rom one bact erium to another. It involves prokaryotic cells and the t ransfer of
5 ch romosoma l or plasm id DNA. It is the on ly mechanism list ed t hat involves direct cell-to -cell cont act.
Prokaryote Plasmid DNA Bacteria Conjugated system DNA supercoil Chromosome
6
7 B is not correct. 2% chose this.
8 Hybridizat ion does not describe the exchange of genetic mat eri al bet ween cells. It refers to the act of m ixing diffe rent species or variet ies of
animals or plants.
9 Hybrid (biology) Species
10
C is not correct. 12% chose this.
11
I n t ransduct ion, DNA is t ransferred by a virus f rom one cell t o anot her. It does not involve cell-to -cell cont act and takes place in prokaryotic
12 cells.
13 Prokaryote Virus DNA Signal transduction Transduction (genetics)

14
D is not correct. 7°/o chose this.
15 Transformat ion is the process by wh ich purif ied DNA is taken up by a cell. It occurs in prokaryotic and eukaryotic cells and does not involve
16 cell -to-cell cont act.
Eukaryote Prokaryote DNA
17
18 E is not correct. 2°/o chose this.
19 Translocation descri bes t he t ransfe r of part of a ch romosome to a new posit ion on t he same or on a diffe rent ch romosome wit h result ant
20
rearrangement of the genes. It is not a mechanism by which bact eri a exchange DNA.
Chromosomal translocation Chromosome Bacteria DNA Protein targeting
21
22
• 23 Bottom Line:
• 24 Bacteri al DNA is t ransferred direct ly f rom one bacterium to anot her by conj ugation .
DNA Bacteria Conjugated system
• 25
• 26
. 27

8
Lock
s
Suspend
0
End Block
Item: 22 of - ,• Mark -<J [:::> "'I ~ · ~
100 ~ P~v1ous N@x t Labl lues Notes Calculato r
A
1 A

FA17 p 126.1
2
Bacterial genetics
3
4
Transformation Competent bacteria are able to bind and import Degraded
uncombined \ & -Reciptent DNA
short pieces of environmental naked bacterial
5
6
7
chromosomal D 'A (from bacterial cel l
lysis). The transfer and expression of newly
~
?
~
%-
~"- o
""'
DNA
-·-... Donor DNA

Naked DNA Recipient celt Transformed cell (;]


8 transferred genes is called transformation.
9 A feature of many bacteria, especially S
10 pneumoniae, H influen;:ae type B, and
11 l'\eisseria (SHi:\). Any DNA can be used.
12 Adding deoxyribonuclease to environment
13 will degrade naked DNA in medium ..... no
14
transformation seen.
15
Conjugation
16
17 F+ X F F+ plasmid contains genes required for sex pi lus Single strand
Sex pilus transferred
18 and conjugation. Bacteria without this plasm id
19
20
are termed F-. Sex pilus on F+ bacterium
contacts F- bacterium. A single strand
Plasmid o l 'V' -+ ~~ .. 0 0
@ @
21 of plasmid DNA is transferred across the f' cell F· cell F' cell f· cell f' cell f' cell
22 conjugal bridge ("mating bridge"). '\fo transfer (;]
. 23
of chromosomal D A.
• 24
Hfr x F- F+ plasmid can become incorporated into PIM'oid"""'l'''iles
,.oblruNIDNA
Tronslft ondr~
• 25
I a~m:i~
• 26 bacterial chromosomal D lA, termed high- ,
. 27 • frequenc,· recombination (ll fr) cell. Transfer """""'""-~
8
Lode.
s
S uspe-nd
8
End Bloc:k
Item: 22 of - ,• Mark -<J [:::> "'I ~ · ~
100 ~ P~v1ous N@x t Labl lues Notes Calculato r
A A
1
Transduction
2
3
4
Generalized A "packaging" e\·ent. Lytic phage infects
bacterium, leading to cleavage of bacterial
Lytic
phage
t)
I
Cleavage of
Bacteria bacterial DNA
Bacterial DNA package
in phage caps;d

5 D lA. Parts of bacterial chromosomal Dt A , r"\ X"l:::::->


6 may become packaged in phage capsid. Phage
7
8
9
infects another bacterium, transferring these
genes. ..• ••. -
10
11

Release of new phage Infects other Phage's genes
12 from lysed cell bacteria transferred ~
13
Specialized n "excision" e\·ent. Lysogenic phage infects Viral DNA . Viral DNA .

r
14 LysogeniC Incorporates 1n Phage partiCles
bacterium; viral DNA incorporates into phage I Bacteria bacterial DNA carry bacterial DNA

-
15
bacterial chromosome. When phage D
~ -
16
17 is excised, flanking bacterial genes may be 0 -
18 excised with it. 0 A is packaged inlo phage
capsid and can infect another bacterium.

-
19
20 Genes for the following 5 baclerialloxins are
21 encoded in a lysogenic phage (ABCD'S): Group
22 A strep eryth rogenic toxin, Botulinum toxin,
Release of new phage Infects other Genes different from
. 23 C holera toxin, Diphtheria toxin, Shiga toxin . from lysed cell bacteria donor and reopient Cl
• 24
• 25
Transposition Segment of D 'A (eg, transposon) Ihat can Plasmid
• 26
"jump" (excision and reintegration) from
. 27 one location to another. can transfer eenes
8
Lode.
s
S uspe-nd
8
End Bloc:k
Item: 23 of - ,• Mark -<J [:::> "'I ~ · ~
100 ~ P~v1ous N @xt Labl lues N o tes Calculato r

A A
1
A 12-year-old boy presents t o his pediat rician with a 3-day history of watery, nonbloody, foul -smelling diarrhea. He has had some nausea, ~~AI
2
malaise, and bloating but remained afebrile durin g this peri od. His fam ily recently retu rned from a vacat ion duri ng which t hey spent a day
3 in Tijuana, Mexico. The pediatri cian suspects infection with Giardia Iamblia.
4
5 The physician can diagnose acute giardiasis by seeing which of the following?
6 :
7 A. Cysts in the blood
8 B. Eggs In the stool
9
C. Oocytes in stool with an acid-fast stain
10
11 0 . Trophozoites in the stool
12 E. Trophozoites on blood smear
13
14

15
16
17
18
19
20
21
22
. 23
• 24
• 25
• 26
. 27

8
Lode.
s
S uspe-nd
8
End Bloc:k
Item: 23 of - ,• Mark -<J [:::> "'I ~ · ~
100 ~ P~v1ous N @xt Labl lues N o tes Calculato r
A A
1
The correct answ er is D. 63°/o chose this.
2
Infection with Giardia Iambiia causes an acute nonbloody, watery, foul-smel li ng diarrhea. It is t ransm itted
3 via the fecal-oral route in contaminated food or water. It occurs throughout the world but is mostly related
4 to places with poor sanit ation. I n its active state, diarrheal specimens can be examined for tear- shaped
trophozoites with two nuclei and four pairs of flagella (shown in the image). Cysts may also be seen in stool
5
samples can be used for diagnosis.
6 Qa.rdta l~~ornblta Ft-oe lu n Feedll·oral route Diarrhea Giardia Trophozoites Infection Mtcrob1al cyst Santtltlon C I . ucleus Cyst

7
8
9
10
11

12
13
14

15
16
17
Image courtesy of Jerard M.
Gardner, MD
18
19 A is not correct . 8°/o chose this.
20 Giardia does not produce cysts in the blood st ream, instead cyst s may be found in a stool sample.
G1ard1a Stool test Cyst
21
22 B i s not correct. 1 5°/o chose this.
23 Eggs are not part of the Giardia life cycle. Eggs in the st ool are used for the diagnosis of Ascaris lumbricoides (the giant roundworm).
Asc~ris lumbrtcotde~~ Nematode Giard1a Ascans Biological life cycle Egg
' 24
' 25 C is not correct . 9°/o chose this.
' 26 Oocytes are not part of the Giardia life cycle. Acid-fast positive oocytes in stool are characteristic of Cryptosporidium pa1vum, a pathogen that
. 27
does produce a watery diarrhea .

8
Lode.
s
S uspe-nd
8
End Bloc:k
Item: 23 of ~ ,• Mark <::J [:::> ""I ~· ~'j
100 J.. Previous Next LAb faiUI~S Notes Calculator

1
E is not correct. 5 °/o chos e this.
2
Giardia does not appea r in the blood. Plasmodia spp. t rophozoit es are seen on blood smear f rom patient s with malaria.
3 Malaria Giardia Plasmodium Plasmodium (life cycle) Blood film Apicomplexan life cycle
4
5
Bottom Line:
6
The easiest way to diagnose Giardia is by finding t rophozoit es in t he st ool.
7
Giardia
8
9
10 141;fil·1i•J for year:[2017
FIRST AID FA C T S .
•j.
11

12
13 FA17 p 151 .1
14 Protozoa- gastrointestinal infections
15 ORGAN ISM DISEASE TRANSMISSION DIAGNOSIS TREATMENT
16
Giardia Iamblia Giardiasis- bloating, flatulence, Cysts in water Multinucleated Metronidazole
17
foul-smelling, fatty diarrhea trophozoites fJ or
18
19
(often seen in campers/hikers)- rn
cysts in stoo1 J

think fat-rich Ghirardelli antigen detection


20
21
chocolates for fatty stools of
22 Giardia
23 Entamoeba Amebiasis-bloody diarrhea Cysts in water Serology and/or Metronidazole;
• 24 histolytica (dysentery), liver abscess trophozoites (with paromomycm or
• 25 ("anchovy paste" exudate), engulfed RBCs [i iodoguinol for
• 26 RUQ pain; histolog)' shows in t·he cytoplasm) asymptomatic cyst
. 27 n ' ' ' ' .. '
8
Lock
s
Suspend
0
End Block
Item: 23 of - ,• Mark -<J [:::> "'I ~ · ~
100 ~ P~v1ous N @xt Labl lues N otes Calculator

1 A A

2
Entamoeba Amebiasis- bloody diarrhea C ysts in water Serology and/or Metronidazole;
3
histolytica (dysentery), li,·er abscess trophozoites (with paromomycm or
4 ("ancllO\) ' paste" exudate), engulfed RBCs 1!1 iodoquinol for
5 RUQ pain; histology shows in I he cytoplasm) aS) mptomat ic cyst
6 Aask-shaped ulcer or cysts with up to passers
7 4 nuclei in stool • ;
8 Entamoeba Eats
9 Er)'l h rocytes,
10 antigen detection
11

12
Cryptosporidium Se,·ere diarrhea in AIDS Oocrsts

in water O ocysts on acid-fast Pre,·ention (by
13
Mild disease (watery diarrhea) in stain (), antigen filtering c ity
14
immunocompetent hosts detection water supplies);
15
nitazoxanidc in
16
17
immunocompetent
18 hosts
19 fJ rn rn
20
21
22
23
' 24
' 25
' 26 a a a a
. 27 •
8
Lode.
s
Suspe-nd
8
End Bloc:k
Item: 23 of ~ ,• Mark <::J [:::> ""I ~· ~'j
100 J.. Previous Next LAbfaiUI~S Notes Calculator

1 • •
FA17 p 175.1
2
Bugs causing diarrhea
3
4 Bloody diarrhea
5 Campylobacter Comma- or S-shaped organisms; growth at 42°C
6
7
E histolytica Protozoan; amebic dysentery; liver abscess
8 Enterohemorrhagic 0157:H7; can cause IIUS; makes Shiga-like toxin
9 Ecoli
10
Enteroinvasive Ecoli Invades colonic mucosa
11

12 Salmonella (non- Lactose 8; Aagellar motility; has animal reservoir, especially poultry and eggs
13 typhoidal)
14 Shigella Lactose 8; very low 10 ;0; produces Shiga toxin (human reservoir only); bacillary dysentery
15
Yenterocolitica Day care outbreaks, pseudoappendicitis
16
17 Watery diarrhea
18 Cdifficile Pseudomembranous colitis; associated with antibiotics and PPis; occasionally bloody diarrhea
19
C perfringens Also causes gas gangrene
20
21 Enterotoxigenic E coli Tra,·elers' diarrhea; produces heat-labile (LT) and heat-stable (ST) toxins
22 Protozoa Giardia, Cryptosporidium
23
• 24
Vcholerae Comma-shaped organisms; rice-water diarrhea; often from infected seafood
• 25 Viruses Rota,·irus, norovirus, adenovirus
• 26
• •
. 27

8
Lock
s
Suspend
0
End Block
Item: 24 of - ,• Mark -<J [:::> "'I ~ · ~
100 ~ P~v1ous N @xt Labl lues N o tes Calculato r

1
A 40-year-old man present s to the emergency department via taxi complaining of blurred and double vision . Beg inning thi s morning, the
2
patient's vision was only slightly blurred; he says it was less sharp than normal and he found it harder to focus. As t ime passed,
3 however, he began to experience double vision . He does not report any sick contacts or recent travel. He is self-employed at his own
4 homemade canning company. On examination, the patient's pupils are 8 mm and minimally responsive to direct and consensual stimulation,
bilaterally. Gaze is grossly disconjugate, and the palpebral fissures are narrowed bilaterally. Lingual, labial, and guttural phonation are abnormal
5
on examination. Strength is -4/5 in the bilateral upper extremities and 5/5 in the bilateral lower extremit ies. Sensation is intact to vibratory,
6 fine, and pin-prick sensation throughout.
7
8 Which of the following is the location of the neurotransmitter implicated in this disease?
9 :
10 A . Basal nucleus of Meynert
11 B. Hypothalamus
12
C. Locus ceruleus
13
14 D. Raphe nucleus
15 E. Ventral tegmentum and substantia nigra
16
17
18
19
20
21
22
23
. 24
• 25
• 26
. 27

8
Lode.
s
S uspe-nd
8
End Bloc:k
Item: 24 of - ,• Mark -<J [:::> "'I ~ · ~
100 ~ P~v1ous N @xt Labl lues N o tes Calculato r

1 •
2
The correct a ns w e r is A. 51 °/o chos e this.
3
Adult botulism is characterized by t he development of diplopia and dysphagia. This is followed by the development of general muscle
4 weakness, respiratory muscle failu re, and even death. The organism responsible for this disease is Clostridium botulinum, and it can be found
5 in contaminated homemade canned goods and smoked fish. C. botulinum produces a toxin t hat cl eaves synaptobrevin, preventing re lease of
acetylcholine at the neuromuscular j unction. Acetylcholine is synthesized in the basal nucleus of Meynert in the brain.
6 Clostr1d1um 'lotul1 u n Dplop1a Botul1sm Dysphagia Synaptobrevin Neuromuscular junctlon Acetylchol1ne i-Jucleus b ~~of Meynert Botulmum toxin Muscle Brain Toxin

7
B is not correct. 9°/o chose this .
8
The hypothalamus is not known to be a primary site of neurotransmitter synthesis.
9 H'fPOth~ 's Nf'l utr1t- tter
10
C is not correct. 1 7°/o chose this .
11
The locus ceruleus is the primary synthesis site of norepinephrine, which is not affected by botulinum toxin.
12 Locu coeruleu~ Botulinum to..cm Noreptnephrine locus (genetics) Toxin
13
D is not correct . 13°/o c hose this .
14
Serotonin is synthesized in the Raphe nucleus but is not affected by botulinum toxin.
15 Raphe nucle1 Botul1num tox1n Serotomn Tox1n Cell nucleus

16
E is not correct. 10°/o chose this.
17
Dopamine is largely synthesized in t he vent ral tegmentum and substantia nigra but is not affected by bot ulinum toxin.
18 Botulinum toxin Substantia n1gra Oopamme Ventral tegmental area Anatomical terms of locat1on Ventral Tegmentum M1dbra1n tegmentum Toxin

19
20
Bottom Line :
21
Characterized by diplopia, dysphagia, and muscle paralysis, botulism is caused by a toxin produced by Clostridium botulinum, which blocks
22
the release of acetylcholine. This neurotransmitter is synthesized in the basal nucleus of Meynert.
23 Clostridium botul1num 01plop1a Dysphag1a Botulism Neurotransmitter Nucleus basalis of Meynert Acetylcholine ParalysiS Botulinum toxm Muscle Toxin Basal ganglia

24
• 25
• 26 I@ Iifi 1 ·11-J fo r year :
FI R ST AID FACTS
2017 •
. 27

8
Lode.
s
S uspe-nd
8
End Bloc:k
Item: 24 of - ,• Mark --<) [::> ""'I ~· 1!';:'1
100 ~ Prev1o u s Next Labf a lu es Notes Calculator

1 • •
2 FA17 p 134.2
3 Clostridia (with G ram Ei1, spore-forming, obligate anaerobic rods.
4
exotoxins)
5
6
Ctetani Produces tetanospasmin, an exotoxin causing Tetanus is tet anic paralysis.
7
tetanus. Tetanus toxin (and botulinum toxin)
8 are proteases that clea,·e SJ ARE proteins for
9 neurotransmitters. Blocks release of inhi bitory
10 neurotransmitters, GABA and glrcine, from
11 Renshaw cells in spinal cord.
12 Causes sp astic paralysis, trismus (lockjaw), risus
13 sardonicus (raised eyebrows and open gri n),
14 opisthotonos (spasms of spinal extensors).
15 Pre,·enl with tetanus vaccine. Treat'' ith
16 antitoxin +1- vaccine booster, diazepam (for
17 muscle spasms), and wou nd debridement.
18
( botulinum Produces a heat-labile toxin that inhibits Symptoms of botulism (the 4 D 's): D iplopia,
19
ACh release at the neuromuscular june!ion, D ysarthria, D ysphagia, D yspnea.
20
21
causi ng botulism. In adults, disease is caused Botulinum is from bad bottles of food , ju ice, and
22 by ingestion of preformed toxin. In babies, honey (causes a descending flaccid paralysis).
23 ingestion of spores (eg, in honey) leads to Local botox injections used to treat foca l
24 disease (Aopp) baby synd rome). Treat with dystonia, achalasia, and muscle spasms. Also
0
25 antitoxin. used for cosmetic reduction offa cial wrinkles.
0
26 Cp erfringens Produces CL toxin (lecithinase. a phospholipase) Perfringens perforates a gangrenous leg.
. 27 •
8
L.odt
s
Su~pl'nd
~
End Block
Item: 24 of - ,• Mark --<) [::> ""'I ~· 1!';:'1
100 ~ Prev1o u s Next Labf a lu es Notes Calculator

1 • AC h release at the neuromuscular junction, Dysarthria, Dysphagia, Dyspnea,


2 causing botulism. In adults, disease is caused Botulinum is from bad bottles of food, ju ice, and
3
by ingestion of preformed toxin. In babies, honey (causes a descending Aaccid paralysis).
4
ingestion of spores (eg, in honey) leads to Local botox injections used to treat focal
5
disease (Aopp~ baby syndrome). Treat" it h dyston ia, achalasia, and muscle spasms. Also
6
antitoxin. used for cosmetic reduction offacial wrinkles.
7
8 Cperfringens Produces a toxin (lecithinase. a phospholipase) Perfringens perforates a gangrenous leg.
9 • that can cause myonecrosis (gas gangrene )
10
11
T , '
,o,.. and hemolysis.
Spores can sun·i,·e in undercooked food;
12
13
I
,
,.,... when ingested, bacteria release heat-labile
enterotoxin ..... food poisoning.
• 0
'
14
15
16 ·K·

17 C difflcile Produces 2 toxins. Toxin A, an enterotoxin, Oif{icile causes diarrhea. Treatment:


18
binds to brush border of gut and alters Auid metronidazole or oral vancomycin. For
19
secretion. Toxin B, a cytotoxin, disrupts recurrent cases, consider repeating prior
20
cytoskeleton via actin depolymerization. Both regimen, fidaxomicin, or fecal microbiota
21
toxins lead to diarrhea ..... pseudomembranous transplant.
22
colitis (lJ. Often 2o to antibiotic usc, especially
23
24
clindamycin or ampicillin; associated with PPI
0
25
use. Diagnosed by detecting one or both toxins
0
26
in stool by antigen detection or PCR.
. 27 •
8
L.odt
s
Su~pl'nd
~
End Block
Item: 24 of - ,• Mark -<J [:::> "'I ~ · ~
100 ~ P~v1ous N @xt Labl lues N o tes Calculato r

1 A
binds to brush border of gut and alters fluid metronidazole or oral ,·ancomycin. For A

2 secretion. Toxin B, a cytotoxin, disrupts recurrent cases, consider repeating prior


3 cytoskeleton via actin depolymerization. Both regimen, fidaxomicin, or fecal microbiola
4 toxins lead to diarrhea - pseudomembranous transplant.
5 colitis rn. Often to antibiotic usc, especially
zo
6
clindamycin or ampicillin; associated with PPI
7
use. Diagnosed by detecting one or both toxins
8
in stool by antigen detection or PCR.
9
10
FA17 p 465.3
11

12
Neurotransmitter changes with disease
13 LOCATION OF ANXIETY DEPRESSION SCHIZOPHRENIA ALZHEIMER HUNTINGTON PARKINSON
14 SYNTHESIS DISEASE DISEASE DISEASE

15 Acetylcholine Basal nucleus l l t


16 of Meynert
17
Dopamine Ventral t t l
18
tegmentum,
19
S 'c
20
21 GABA Nucleus
22 accum bens
23 Norepmephrine Locus ceruleus t l
24
Serotonin Raphe nucleus l l
• 25
• 26
. 27 •
8
Lode.
s
S uspe-nd
8
End Bloc:k
Item: 25 of - ,• Mark -<J [:::> "'I ~ · ~
100 ~ P~v1ous N @xt Labl lues N o tes Calculato r

1
A 6-year-old boy is brought to the pediatrician with an extensive itchy rash in various stages of healing . He also complains of fever and
2
lethargy. Physical examination shows a vesicular rash wit h macules, papules, and pustules over his face and t runk. He has had these
3 symptoms for approximately 4 days.
4
5 What is another clinical manifestation attribut ed to this pathogen?
6 :
7 A. Autoimmune hemolytic anemia
8 B. Congenital deafness
9
C. Dermatomal vesicular eruption
10
11 D. Orchitis
12 E. Subacute sclerosing panencephalitis
13
14

15
16
17
18
19
20
21
22
23
24
. 25
• 26
. 27

8
Lode.
s
S uspe-nd
8
End Bloc:k
Item: 25 of - ,• Mark -<J [:::> "'I ~ · ~
100 ~ P~v1ous N @xt Labl lues N o tes Calculato r
A A
1
2 The correct answer is c. 65°/o chose t his .
3 This child has a classic case of chickenpox, which is caused by varicella-zoster virus (VZV) . It is a hig hly
4 contagious DNA herpesvirus that is transmitted by respiratory droplets or direct contact and produces a
self- limited infection. Herpes zoster (shingles) is a sporadic disease that is the consequence of reactivation
5
of latent VZV in the dorsal root ganglia. Shingles is characterized by a unilateral vesicular eruption within a
6 dermatome (like that shown in the image) and is often associated with severe pain. Patients with herpes
7 zoster can transmit their infection to seronegative individuals, who will then develop chickenpox.
~ Of'!'lle ~ 01 C •C e po Sh ngles Dorsal root gang! on Somite Vaficella zostet" Vl'lJ'f '"'leMe-- ~•"'Pie Gi. ;I o Herpes 1ndae VIrus ONA
8
lles•cl• (btoloov and chemstry) Jnfecbon
9
10 Image courtesy of Wikimedia
11 Commons
12
A is not correct. 5% chose t h is.
13
Autoimmune hemolytic anemia (AIHA) arises when I gM (cold agglutinin) or IgG (warm agglutinin) autoantibodies are generated against RBCs.
14 AIHA can be associated with conditions that may also cause rashes such as systemic lupus erythemat osis (malar or discoid rash), infectious
15 EBV mononucleosis (maculopapular rash especially after using amoxicillin), and drug-induced AIHA (eg, a-met hyldopa). The vesicular, pruritic
rash seen in this patient is a classic presentation of chickenpox. AIHA would not be expected in a patient with chickenpox.
16
Infect1ous mononucleos•s Itch System•c lupus erythematosus Chickenpox Amoxicilhn Immunoglobulin M HemolytiC anem•a Maculopapular rash Immunoglobulin G Autoantibody Anem1a Auto1mmune d•sease
17
Auto1mmune hemolytic anem1a Auto•mmun•ty Rash Lupus erythematosus Epstein-Barr virus Hemolysis Cold agglut1n1n d1sease
18
19
B is not correct. 10°/o chose this.
Rubella, or German measles, is a less severe viral exanthem t hat starts on the face and spreads to t he t runk and extremit ies. Up to 33% of
20
adult women experience arthritis in conjunction with rubella infection. Many infections are subclinical, bu t when infection occurs during the
21 prenatal period rubella can cause severe bi1th defects, includ ing congenital deafness. Sim ilarly, congenital deafness and ot her severe birth
22 defects can also be caused by cyt omegalovirus (CMV), which is also a cause of spontaneus abortion.
Cytomegalovirus Me ~ler.. Rubella Exanthem Abort1on Arthritis Hearing loss AsymptomatiC V1rus Congen1tal d1sorder Germany
23
24 D is not correct. 7°/o chose th is.
25 Mumps is caused by an RNA paramyxovirus t hat replicates in the upper respiratory tract and causes parotidit is and, frequently, orchitis.
Pancreatitis and meningitis can also be present.
• 26
P.ar'""fXO 1d~ ()-cl 1b~ Mu np!l> Men1ng1tis Pancreatitis RNA Respiratory tract
. 27

8
Lode.
s
S uspe-nd
8
End Bloc:k
Item: 2S of ~ ,• Mark <::J [:::> ""I ~· ~'j
100 J.. Previous Next LAbfaiUI~S Notes Calculator

1
E is not correct. 13°/o chose this.
2 Measles (rubeola) presents wit h a relatively nonspecific prodrome of feve r~ cough, and runny nose. The rash develops 1-2 days after the
3 appearance of Koplik spots, which are re d oral lesions wit h blue-whit e cent ers. It spreads from head t o t oe over a 3-day period. Rarely, it can
4
cause subacut e sclerosing panencephalit is in which pat ients exhibit psychoneuro logic deteriorat ion 10 or more years after infection wit h the
measles virus. It is a progressive disorder that can manifest with seizures, myocl onus, ataxia, and perso nality changes and event ually resu lts
5 in death.
6 Subacute sclerosing panencephalitis Measles Myoclonus Ataxia Measles virus Prodrome Cough Epileptic seizure Rash Virus Fever Neurodegeneration Rhinorrhea

7
8
Bottom Line:
9
Vari cella -zoster v irus is a double-stranded DNA virus in t he herpesvirus fam ily. It is responsible fo r causing chickenpox and herpes zost er, or
10 shingles. Chickenpox is t ransmitted by respiratory droplet s and produces a self-lim ited infection. Zost er is caused by reactivation of latent
11 VZV in t he dorsal root ganglia.
DNA virus Chickenpox Shingles Dorsal root ganglion varicella zoster virus Herpes simplex Virus Herpesviridae Zoster vaccine Ganglion DNA
12
13
14
15
l@l;fil·1i•l for year:[ 2017
FIRST AID FACTS .
•j .

16
17 FA17 p 179.1
18 Red rashes of childhood
19 AGENT ASSOCIATED SYNDROME/DISEASE CLINICAL PRESENTATION
20
Coxsackievirus type A Hand-foot-mouth disease Oval-shaped vesicles on palms and soles r:.t
21
vesicles and ulcers in oral mucosa
22
23 Human herpesvirus 6 Roseola (exanthem subitum) Asymptomatic rose-colored macules appear
24 on body after se,·eral days of high fever; can
25 present with febrile seizures; usually affects
• 26 infants
. 27 I l 1 "
' ' . '
8
Lock
s
Suspend
0
End Block
Item: 25 of - ,• Mark -<] 1:> ""'I ~· 1!';:'1
100 ~ Prev1o u s Next Labf a lu es Notes Calculator

1 Measles virus Measles (rubeola) ConAuent rash beginning at head and


2 moving down; preceded by cough, coryza,
3
conjunctivitis, and blue-white (Koplik) spots
4
on buccal mucosa
5
6
Parvovirus 819 Errthema infectiosum (fifth disease) "Slapped cheek" rash on face IE] (can cause
7 hydrops fetal is in pregnant women)
8 Rubella virus Rubella (German measles) Pink macules and papules begin at head
9 and mo,·e down, remain discrete - fi ne
10 desquamati ng truncal rash; postauricular
11
lymphadenopathy
12
13
Streptococcus pyogenes Scarlet fe,·er Erythematous, sandpaper-like rash with fever
14
and sore throat
15 Varicella-Zoster virus Chickenpox Vesicular rash begins on trunk; spreads to face
16 m and extremities with lesions of different
17 stages
18
19
20
21
22
23
24
25
• 26
. 27 •
8
L.odt
s
Su~pl'nd
~
End Block
Item: 2S of ~ ,• Mark <::J [:::> ""I ~· ~'j
100 J.. Previous Next LAb faiUI~S Notes Calculator

1 • •
FA17 p 160.1
2
DNA viruses
3
VIRAL FAMILY ENVELOPE DNA STRUCTURE MEDICAL IMPORTANCE
4
5
Herpesviruses Yes OS and linear See Herpesviruses entry
6 Poxvirus Yes OS and linear Smallpox eradicated world wide by use of the live-
7 (largest 0 A virus) attenuated vaccine
8 Cowpox ("mi lkmaid blisters")
9 Molluscum contagiosum -Aesh-colored papule with
10 centra I umbiIication
11
Hepadnavirus Yes Partially OS and circular HBV:
12
13
• cute or chronic hepatitis
• I ot a retrovirus but has re,·erse transcriptase
14
15 Adenovirus 0 OS and linear F'ebrile pharyngitis f.J - sore throat
16 Acute hemorrhagic cystitis
17 Pneumonia
18 Conju ncti,·itis-"pink eye"
19
20
21
22
23
24
Papillomavirus No OS and circular 1--1 PV- warts (serotypes l, 2, 6, ll), Cl1 , cervica l
25
cancer (most com monly 16, 18)
• 26
. 27 • Polyomavirus 0 OS and circular JC virus-progressive multi focal leukoencephalopathy •

8
Lock
s
Suspend
0
End Block
Item: 26 of - ,• Mark -<J [:::> "'I ~· ~
100 ~ P~v1ous N @xt Labl lues N o tes Calcula to r

1
A 36-year-old previously healthy man from southwestern Ohio who works as a poultry fa rmer presents to t he clinic with a 2-week history
2
of coughing up green sputum. He is otherwise asymptomatic, and his physical examination is unremarkable. X-ray of the chest
3 demonstrates a 1-cm calcified coin lesion on the upper lung lobe, along with hilar lymphadenopat hy. A histopathologic image is shown
4 below.
5
6
7
8
9
10
11
12
13
14

15
16 Image courtesy of CDC/ Or. D. T. McCienan
17
18 What is the most likely diagnosis?
19 :
20 A. Blastomycosis
21 B. Coccidioidomycosis
22
C. Cryptococcosis
23
24 D. Histoplasmosis
25 E. Paracoccidioidomycosis
. 26
. 27

8
Lode.
s
S uspe-nd
8
End Bloc:k
Item: 26 of ~ ,• Mark <::J [:::> ""I ~· ~'j
100 J.. Previous Next LAbfaiUI~S Notes Calculator

1
2 The correct a nswer is 0. 58°/o chose t his.
3 Hist oplasmosis is caused by the fungus Histoplasma capsulatum. The infection is endemic in cent ra l, mid-At lant ic, and sout heastern states,
4 particularly in the Ohio and Mississippi River valleys. Because the fungus is common in soils cont aminated wit h bat or bird droppings, people
who work wit h soil or around t hese animals are at increased risk for hist oplasmosis. Immunocom promised people are at pa1t icular risk for
5
disseminat ed infection . The primary infect ion is in the lungs and may be latent , or brief and self- lim ited. Symptoms include fever, cough, and
6 sweats. X-ray shows calcif ied coin lesions and hilar lymphadenopat hy.
7 Histoplasma Mississippi River Endemism Histoplasmosis Lymphadenopathy Immunodeficiency Fungus Bird Fever Ohio Mississippi Infection Bat Cough Lung

8 A is not correct . 15% chose this.


9 Blastomycosis is caused by t he dimorph ic f ungus Blastomyces dermatitidis, which is endemic to t he cent ral Unit ed Stat es f rom t he Great
10 Lakes to Arkansas. It usually affects t he lungs, but may disseminat e.
Blastomycosis Blastomyces dermatitidis Endemism Dimorphic fungus Fungus United States Lung
11

12 B is not correct. 15% chose this .


13 Coccidioidomycosis is a disease caused by t he fungus Coccidioides immitis . Spores of t he organism are inhaled, usually f rom t he soil. These
spores enlarge to sporangia, which burst and spread. The disease is characterized by pulmonary symptoms but is usually self- lim ited. This
14
organism is common in ari d desert states such as Arizona and can affect people aft er brief visits.
15 Coccidioidomycosis Coccidioides immitis Sporangium Spore Fungus Arizona Arid Coccidioides Basidiospore

16
C is not co rrect. 9 °/o chose this .
17
Cryptococcus neoformans is a fungus found worl dwide. The main reservoir is pigeon droppings. The small crypt ococci are inhaled into the
18 lungs, but do not usually cause disease in immunocom petent individuals. I n the immunosuppressed pat ient , Cryptococcus can cause
19 pulmonary and cent ral nervous syst em disease, especially meningit is.
Cryptococcus neoformans Central nervous system Meningitis Immunosuppression Immunocompetence Nervous system Cryptococcus Fungus Lung Columbidae
20
21 E is not correct. 3°/o chose this .
22 Paracoccidioidomycosis, also called South American blastomycosis, is endemic in Sout h America and is caused by Paracoccidioides brasiliensis.
Clinical present at ion includes lung lesions or mucocutaneous lesions. The m ult iple budd ing yeasts classically demonstrat e a ship's "captain's
23 wheel" appearance.
24 Paracoccidioido,.,cosis Blastomycosis Paracoccidioides brasiliensis South America Yeast Lung

25
26
Bottom Line :
. 27

8
Lock
s
Suspend
0
End Block
Item: 26 of ~ ,• Mark <::J [:::> ""I ~· ~'j
100 J.. Previous Next LAbfaiUI~S Notes Calculator

1
2 Bottom Line:
3 Hist oplasmosis is endemic in central, m id-Atlant ic, and southeast ern st at es. It infects pri marily t he lu ngs, and causes feve r, coug h, and
sweats. Working wit h or near bats or birds increases t he ri sk of contract ing t he disease .
4
Endemism Histoplasmosis Cough Fever
5
6
7
l@l;fil·1i•l for year:[2017
FIRST AID FACTS .
•j .
8
9
10 FA17 p 147.1
11 Systemic mycoses All of the following can cause pneumonia and can disseminate.
12 All are caused by dimorphic fungi: cold (20°C) = mold; heat (37°C) = yeast. Only exception is
13 Coccidioides, which is a spherule (not yeast) in tissue.
14
Systemic mycoses can form granu lomas (like TB); ca nnot be transmitted person-to-person (unlike
15
TB).
16
Treatment : Auconazole or itraconazole for loc~•l infection; amphotericin B for systemic infection.
17
18 DISEASE ENDEMIC LOCATION PATHOLOGIC FEATURES UNIQUE SIGNS/SYMPTOMS NOTES
19 Histoplasmosis Mississippi and Ohio Macrophage fill ed Palatal/tongue ulcers, ll isto h ides (within
20 Ri,'er Valleys with Histoplasma splenomegaly macrophages)
21 (smaller than Bird (eg, starlings) or
22 RBC) f.i.1 bat droppings
23 Diagnosis via urine/
24
serum antigen
25
26
. 27 Blas tomvcoc;is P.astPrn ::mrl lnA a mm a tnrv R b ~ tn h11rl ~ h rnarll v
8
Lock
s
Suspend
0
End Block
Item: 26 of - ,• Mark --<) [::> ""'I ~· 1!';:'1
100 ~ Prev1o u s Next Labf a lu es Notes Calculator

1 • F.aslern and •
Blastomycosis Broad-based budding lnOammalory Blasto buds broadly
2
Central US of Blastomyces (same lung disease, can
3
si7e as RBC) disseminate to skin/
4
bone
5
Verrucous skin lesions
6
7
can simulate sec
8
Forms granulomatous
9
nodules
10 Coccidioidomycosis Southwestern US, Spherule (much larger Disseminates to skin/
11 California than RBC) filled bone
12 with endospores of Er) thema nodosum
13 Coccidioides 8 (desert bumps) or
14
multi forme
15 Arthralgias (desert
16
rhcumat ism)
17
Can cause meningitis
18
19 Para- Latin America Budding yeast of Similar to Paracoccidio parasa ils
20 coccidioidomycosis Pamcoccidioicles with Coccicl ioiclomycosis, with the captain's
21 "captam. '., '' I1cc I" males> fema les wheel all the way to
22 formation (much Latin America
23 larger than RBC) 1!]
24
25
26
. 27 •
8
L.odt
s
Su~pl'nd
~
End Block
Item: 27 of - ,• Mark --<) [::> ""'I ~· 1!';:'1
100 ~ Prev1ous Next Labfa lues Notes Calculator

1
2
A 7-year-old girl is brought to the emergency department by her mother with complaints of a rash on her t runk and extremities. Her
mother reports noticing the rash this morning but that 4 days ago her daughter did have a fever, headache, nausea, and a small red rash
3 on her cheeks, all of which subsided. Her past medical history is noncontributory, and her immunizations are up to dat e. Her vital signs are
4 stable, and her temperature is 38.0°C (100.4°F). Physical examination reveals erythematous cheeks and a pruritic maculopapular rash on the
5
trunk and legs. Her complete blood cell count shows leukocytosis with a predominance of lymphocytes.
6
Which of the following organisms is most likely to have caused this patient's rash?
7
8 :
A. Coxsackie virus
9
10 B. Human herpesvirus 6
11
C. Paramyxovirus
12
D. Parvovirus B19
13
14 E. Rubella virus
15
16
17
18
19
20
21
22
23
24

25
26
. 27 •

8
L.odt
s
Su~pl'nd
~
End Block
Item: 27 of - ,• Mark -<J
P~v1ous
[:::> "'I ~ ·· ~
100 ~ N @xt Labl lues N o tes Calcula to r
2 A A

3 The correct answ er is D. 56°/o chose this.


4 Parvovlrus B19 is the causative organism of fifth disease (also called erythema infectiosum), which typically
5 affects children 5-10 years old. This condit ion is characterized by an init ial prodromal phase that includes
find ings such as a low-grade fever, headache, and gastrointestinal symptoms. The virus causes an
6
erythematous rash that looks like both cheeks have just been slapped (like shown in this image). I n
7 addition, a pruritic maculopapular rash starts on t he arms and spreads to the trunk and legs. Parvovirus
8 also causes aplastic anemia in patients wit h sickle cell anemia and hereditary spherocytosis.
liere<'.. ~phero ~" ~ Mfth d .ef.;Se ParvoY1rus 619 Aplastic anemia Sickle-eel d1sease Mo?~Culop~' .,_ A.le a ErvU ema ParvoY1rus Itch Headache
9
Rash Fev.,. Prodrome VIrus
10
11

12
13
14

15
16
17
18
19 Image courtesy of Andrew Kerr
20
A is not cor rect . 8°/o chose this .
21
Coxsackie virus causes fever, oral ulcers, and a maculopapular rash on the hands and feet.
22 Coxsackievirus Maculopapular rash V1rus Ulcer Fever Rash

23
B i s not correct. 1 5°/o chose this .
24 Human herpesvirus 6 causes roseo la infant um, which is characterized by an acute onset of high fever (above 40°C [ 104° F]) and a
25 maculopapular rash t hat appears as t he fever wanes. The rash begins on the trunk and spreads to the face and extremities, and typically lasts
26
<24 hours.
Hum~n her..,~vlru• 6 Moculopapular rash Roseola Herpesviridae Rash Fever
27
• 28 C is not cor rect. 9°/o chose this .

8
Lode.
s
S uspe-nd
8
End Bloc:k
Item: 27 of ~ ,• Mark <::J [:::> ""I ~· ~'j
100 J.. Previous Next LAb faiUI~S Notes Calculator

2
3
C is not correct. 9 °/o chose this .
Paramyxovirus causes measles, wh ich presents wit h a low-grade fever wit h Cough, Coryza, and Conj unctivit is (the "3 C's "), and Koplik spots
4
(small blue-gray or bluish-white spots on t he buccal mucosa). The rash of measles is maculopapu lar and begins on the head and spreads
5 t oward the feet.
6 Param(xoviridae Measles Oral mucosa Maculopapular rash Fever Mucous membrane Rash

7 E is not correct. 12°/o chose this.


8 Rubella virus causes t ender generalized lymphadenopat hy and a t ender macu lopapular rash that spreads f rom t he head t o t he feet (in t he
9 same fashion as the measles rash) and disappears wit hin 5 days (unlike t he rash in measles) .
Maculopapular rash Measles Lymphadenopathy Rubella virus Virus Rubella Rash
10
11

12 Bottom Line:
13 Parvovirus B19 is the causat ive organism of fifth disease (erythema infectiosum), which is cha racterized eryt hematous rash on both cheeks.
Fifth disease Parvovirus 819 Erythema Parvovirus Rash
14
15
16
17 141;fil·1i•J
FIRS T AID FACTS
for year:[ 2017
.
•j .

18
19 FA17 p 160.1
20 DNA viruses
21 VIRAL FAMILY ENVELOPE DNA STRUCTURE MEDICAL IMPORTANCE
22
Herpesviruses Yes OS and linear See Herpesviruses entry
23
24 Poxvirus Yes OS and linear Smallpox eradicated world wide by use of the live-
25 (largest 0 1 A virus) attenuated vaccine
26 Cowpox ("mi lkmaid blisters")
27 Molluscum contagiosum -Aesh-colored papule with
• 28 central umbilication
8
Lock
s
Suspend
0
End Block
Item: 27 of - ,• Mark --<) [::> ""'I ~· 1!';:'1
100 ~ Prev1ous Next Labfa lues Notes Calculator

2
3
Hepadnavirus Yes Partially OS and circular li B :
4 cute or chronic hepatitis
5 Not a retrovirus but has reverse transcriptasc
6 Adenovirus 0 OS and linear Febrile pharyngitis rJ-sore throat
7 Acute hemorrhagic cystitis
8 Pneumon ia
9 Conjunctivitis- ''pink eye"
10
11

12
13
14

15
Papillomavirus I 0 OS and circular II P - warts (serol)·pes 1, 2, 6, 11), C h , cervical
16
17
cancer (most commonly 16, 18)
18 Polyomavirus No OS and circular JC virus-progressive multifocalleukocncephalopathy
19 (P 11.) in !lTV
20 BK virus-transplant patients, commonly targets kidney
21 JC: Junky Cerebrum; BK: Bad Kidney
22
Parvovirus No SS and linear B19 virus-aplastic crises in sickle cell disease,
23
(smallest 0 1 virus) "slapped cheek" rash in children (erythema
24
infcc tiosum, or fifth disease)
25
RBC destruction in fetus leads to hydrops fetalis
26
and death, in adults leads to pure RBC aplasia and
27
• 28
rheumatoid arthritis-like symptoms •
8
L.odt
s
Su~pl'nd
~
End Block
Item: 27 of ~ ,• Mark <::J [:::> ""I ~· ~'j
100 J.. Previous Next LAbfaiUI~S Notes Calculator

2 • •
FA17 p 179.1
3
Red rashes of childhood
4
AGENT ASSOCIATED SYNDROME/DISEASE CLINICAL PRESENTATION
5
6 Coxsackievirus type A Hand-foot-mouth disease Oval-shaped vesicles on palms and soles rJ;
7 vesicles and ulcers in oral mucosa
8 Human herpesvirus 6 Roseola (exanthem subitum) Asymptomatic rose-colored macules appear
9
on body after se,·eral days of high fever; can
10
present with febrile seizures; usually affects
11
infants
12
13 Measles virus Measles (rubeola) ConAuent rash begi nning at head and
14 moving down; preceded by cough, coryza,
15 conjuncti,·itis, and blue-white (Koplik) spots
16 on buccal mucosa
17 Parvovirus B19 Erythema infectiosum (fifth disease) "Slapped cheek" rash on face llJ (can cause
18
hydrops feta lis in pregnant women)
19
20
Rubella virus Rubella (German measles) Pin k macules and papules begin at head
21
and move down, remain discrete ..... fine
22 desquamating truncal rash; postauricular
23 lymphadenopathy
24 Streptococcus pyogenes Scarlet fever Erythematous, sandpaper-like rash with feve r
25 and sore throat
26
Varicella-Zoster virus Chickenpox Vesicular rash begins on trunk; spreads to face
27
• 28 •
m and extremities with lesions of different •

8
Lock
s
Suspend
0
End Block
3
4
desquamating truncal rash; postauricular
5
lymphadenopathy
6
7
Streptococcus pyogenes Scarlet fe,·er Erythematous, sandpaper-like rash with fever
8
and sore throat
9 Varicella-Zoster virus Chickenpox Vesicular rash begins on trunk; spreads to face
10 • and extremities with lesions of different
11 stages
12
13
14

15
16
17
18
19
20
21
22
23
FA17 p 158.4
24

25
DNA viral genomes All D1 A ,·iruses except the Parvoviridae are All arc dsDNA (like our cells), except "part-of-a-
26
dsDI A. ' irm " (pan o\'irus) is ssDI A.
27 All are linear except papilloma-, pol}oma-, and Parvus =small.
• 28 hcpadna,·iruses (circular). •
8
L.odt
s
Su~pl'nd
~
End Block
Item: 28 of - ,• Mark -<J [:::> "'I ~ · ~
100 ~ P~v1ous N @xt Labl lues N o tes Calcula to r
2
A 4-year-old boy is brought to a rura l clinic because of a 1-week history of intense coughing spells that last 1- 2 m inut es, occasionally ~~AI
3
fo llowed by vomiting. On questi oning, t he patient's mot her recalls that he had a cold about 2 weeks ago, bu t that t he cou ghing has become
4 much worse recently. He has not received an y immunizations. His vital signs are within normal limits, and physical examination is
5 unremarkable. Laboratory studies show a WBC count of 22,000/mm 3 with 60% lymphocytes. X- ray of the chest is unremarkable.
6
7 Which of the following is the most likely cause of this patient's symptoms?
8 :
9 A. Bordetella pertussis
10 B. Corynebacterium diphtheriae
11
C. Haemophilus influenzae
12
13 D. Mycobacterium tuberculosis
14 E. Streptococcus pneumoniae
15
16
17
18
19
20
21
22
23
24
25
26
27
. 28 •
8
Lode.
s
S uspe-nd
8
End Bloc:k
Item: 28 of ~ ,• Mark <::J [:::> ""I ~· ~'j
100 J.. Previous Next LAbfaiUI~S Notes Calculator

2
3 The correct a nswer is A. 73°/o chose t h is.
4 This patient present s with a typical picture of pertussis, or whooping cough, caused by the gram -negat ive organism Bordete/la pertussis. A
prod romal phase resembling an upper respi ratory t ract infection is followed by roughly 1-6 weeks of coughing spells int erspersed wit h a loud
5
"whooping " sound caused by inspiration against a narrowed airway. Commonly, t he cough is followed by posttu ssive emesis. A prominent
6 lymphocytosis is often present as well. The prog ression of pertussis can be divided into three phases : (1) the first is the catarrhal phase,
7 which resembles an upper respirat ory infect ion and lasts 1-2 weeks. (2) The second phase is a paroxysmal stage in which the cough gets
8
prog ressively worse, and t his ty pically lasts 2- 3 weeks. (3) Finally, the third phase is the convalescent phase, in which the cough slowly
subsides (this can last more than 4 weeks). The pertussis vaccine is a rout ine childhood immunizat ion, but t his pat ient has not been
9 immunized. Culture using Regan -Lowe charcoal blood agar (more common) or Bordet-Gengou agar is t he gold standard in diagnosis. Chest X-
10 rays are not always revea ling: t hey may be normal or show at electasis, peri bronchial cuff ing, or peri hilar infilt rat es.
Atelectasis Bordetella pertussis Upper respiratory tract infection Pertussis Gram-negative bacteria Vomiting Bordet-Gengou agar Bordetella Lymphocytosis Prodrome Cough Respiratory tract Immunization
11
Respiratory tract infection Chest radiograph Pertussis vaccine vaccine Organism Upper respiratory tract Agar X-ra.t Catarrh Infection
12
13 B is not correct. 6% chose t his .
14 Corynebacterium diphtheriae manifest s as a pharyng it is t hat progresses to the formation of a pharyngeal pseudomembrane. This is caused by
a phage-encoded t oxin, dipht heri a t oxin, t hat f unct ions to inactivate t he eukaryotic host ri bosomal prot ein elongation factor 2 (EF-2) .
15
Corynebacterium diphtheriae Diphtheria Diphtheria toxin Pharyngitis Elongation factor Protein Toxin Corynebacterium Eukaryote EEF2 Ribosome Pharynx
16
17
C is not correct. 12% chose this .
Haemophilus influenzae (nont ypeable strains, meaning that t hey lack a capsule) are associat ed with upper respi ratory infect ion such as acut e
18
and chronic otit is and sinusit is. Typeable st rains such as H. influenzae type b (HiB) cause invasive disease such as epiglot it is, pneumonia, and
19 meningitis in unimmunized ind ividuals.
20 Haemophilus influenzae Sinusitis Meningitis Pneumonia Upper respiratory tract infection Respiratory tract infection Haemophilus Immunization Otitis

21 D is not correct. 3°/o chose this.


22 Pri mary Mycobacterium tuberculosis infect ion does not manifest wit h int ense coughing spells and the "whoop. " It usually st arts with the
23 format ion of the Ghon complex and hilar lymphadenopat hy.
Mycobacterium tuberculosis Tuberculosis Lymphadenopathy Mycobacterium Hilar lymphadenopathy Root of the lung Infection Ghon's complex
24
25 E is not correct. 6°/o chose this .
26 Streptococcus pneumoniae mult iplies in the alveolar spaces. Most patient s have product ive cough wit h blood-tinged sput um . I nfect ion is
generally localized t o t he lower lobes of t he lungs.
27 Streptococcus pneumoniae Sputum Streptococcus Cough Pulmonary alveolus Lung
28 •
8
Lock
s
Suspend
0
End Block
Item: 28 of ~ ,• Mark <::J [:::> ""I ~· ~'j
100 J.. Previous Next LAbfaiUI~S Notes Calculator

2
3
Bottom Line:
4
Bordete/la pertussis is t he most likely causative organism of cough in child ren who have not been immunized . I nfect ion manifests with
5
paroxysmal cough ("whooping cough" ) and int ermittent posttu ssis vomiting.
6 Bordetella pertussis Bordetella Pertussis Cough Vomiting Organism

7
8
9 i@l;fil·1i•J for year:[ 2017 • J
FIRST AID FACTS
10
11
FA17 p 128.1
12
Bugs with exotox ins
13
14
BACTERIA TOXIN MECHANISM MANIFESTATI ON

15 Inhibit protein synthesis


16 Corynebacterium Diphtheria toxin3 Pharyngitis with pseuclomembranes in th roat
17 diphtheriae and severe lymphadenopathy (bull neck)
Inactivate elongation factor
18
Pseudomonas Exotoxin N (EF-2) Host cell death
19
20
aeruginosa
21 Shigella spp. Shiga toxin (ST)3 GI mucosal damage .... dysentery; ST also
22 Inactivate 60S ribosome by enhances cytokinc release, causing hemolytic-
23 removing adenine from uremic syndrome (HUS)
24 rR A
Enterohemorrhagic Shiga-like toxin SLT enhances cytokine release, causing HUS
25
E coli (EHEC) (SLT)a (prototypically in EHEC serotype 0157:H7).
26
Unlike Shigella, EHEC does not invade host
27
28
cells
8
Lock
s
Suspend
0
End Block
Item: 28 of ~ ,• Mark <::J [:::> ""I ~· ~'j
100 J.. Previous Next LAbfaiUI~S Notes Calculator

2 • •
FA17 p 123.1
3
4
Special culture requirements
5 BUG MEDIAUSEDFOR ISOLATI ON MEDIACONTENTS/OTHER
6 H influenzae Chocolate agar Factors V ( AD+) and X (hematin}
7
N gonorrhoeae, Thayer-Marti n agar Selectively favors growth of 1 eisseria by
8
N meningitidis inhibiting growth of gram EEl organisms
9
with Vancomycin, gram 8 orga nisms except
10
Neisseria with Trimethoprim and Colistin,
11
and fun gi with ~ystati n
12
13
Very Typically Cultures Neisseria
14 Bpertussis Bordet-Gengou agar (Bordct for Bordetella) Potato extract
15 Regan-Lowe medium Charcoal, blood, and antibiotic
16
Cdiphtheriae Tellurile agar, LofA er medium
17
18
M tuberculosis Lowenstein-Jensen agar
19 M pneumoniae Eaton agar Requires cholesterol
20
Lactose-fermenting MacConkey agar Fermentation produces acid, causing colonies to
21
enterics turn pink
22
23 E coli Eosin- methylene blue (EMB) agar Colonies with green metallic sheen
24 Legionella Charcoal yeast extract agar buffered with
25 cysteine and iron
26
Fungi Sabouraud agar "Sa b'sa f un guy.,,
27
28 • •

8
Lock
s
Suspend
0
End Block
Item: 29 of - ,• Mark -<J [:::> "'I ~ · ~
100 ~ P~v1ous N @xt Labl lues N o tes Calcula to r
3
A 27-year-old man present s to the physician with fever~ malaise, cough, and wheezing . During the physical examination, t he patient ~~AI
4
intermittently coughs up brownish-colored mucus. The rest of t he physical examination is unremarkable. A chest x-ray reveals a mass
5 within a left lung cavity. The pathology repo rt on mucus specimens indicates infection with an ident ified opportunistic microorganism.
6 Another species in this infectious agent's family can also produce a toxin that can result in hepat ocellular carci noma.
7
8 What is the microscopic morphology of this agent?
9 :
10 A. A cyst with a thick cell wall and intracystic bodies
11 B. A heavily encapsulated yeast that is not dimorphic
12
C. A mold with irregular nonseptate hyphae branching at wide angles
13
14 D. A mold with septate hyphae that branch at a V-shaped 45-degree angle
15 E. Cigar-shaped budding yeast visible in pus
16
17
18
19
20
21
22
23
24
25
26
27
28
. 29 •
8
Lode.
s
S uspe-nd
8
End Bloc:k
Item: 29 of - ,• Mark -<J [:::> "'I ~ · ~
100 ~ P~v1ous N@xt Labl lu es No tes Calcula t o r

3 A A

4
The correct answ er is D. 67°/o chose this.
5
This patient likely has an infection wit h Aspergillus fumigatus, a ubiquitous fungus that can form
6
aspergillomas ("fungus balls," shown in the image) in patients with preexisting lung cavities. Under t he
7 microscope, A fumigatus appears as a nondimorphic mold with septate hyphae that branch at V-shaped 45-
8 degree angles. In patients with significant hemoptysis, treatment is surgical removal of the aspergilloma.
Otherwise, itraconazole may be given. Aspergillus infection via inhalation can lead to aspergillomas.
9
Aspergillus flavus is another member of t he Aspergillus family that is classically found growing on grains. It
10 produces aflatoxins that, when ingested in sufficient quantities, can cause hepatocellular carcinoma.
A'lPf!'Q ' ' 0,.. " opt,~ s Jtraconazole Asperg · Ius flaws Hepatocellular carcmo"'il As:perg o,.. "''~'~' ;• n,. H p 1a Aflato .n Lung Fungus Septum
11
Mold Mtcroscope Carcmoma Infection
12
13
14 Image courtesy of Yale Rosen,
15
MD
16
A is not correct . 8°/o chose thi s.
17 Under microscopy, Pneumocystis jirovecii appears as cup-shaped cysts with intracystic bodies. Infection may man ifest with fever, malaise,
18 dyspnea, and nonproductive cough in HI V-posit ive and immunocompromised patients. Chest x-ray would typically show a ground-glass
19
appearance (diffuse interstitial pneumonia}, not cavitat ions.
Dyspnea Pneumocyst1s JlrOvecll Immunodefic•ency Pneumonia Chest radiograph Pneumocystis pneumonia M•croscopy Mala•se Cough Cyst HIV Interstitial lung disease X-ray Fever Infect1on
20
21
B is not correct. 8 % chose this.
22
Cryptococcus neoformans is a heavily encapsulated nondimorphic yeast th at is found in soil and pigeon droppings. It oft en does not produce
any symptoms in an immunocompet ent host. An immunocompromised individual, however, can present wit h meningoencephalitis.
23 Cryptococcus neoformans Immunodefic•ency Cryptococcus Yeast MeningoencephalitiS Immunocompetence Columbtdae
24
C is not correct . 12% chose thi s.
25
Mucor species are molds with irregular nonsept ate hyphae that branch at wide angles (more than 90 degrees). Mucor infection is particularly
26 worrisome in patients with diabetic ketoacidosis, who may init ially present with allergy-like symptoms. Cavit at ions can be seen in Mucor-
27 related pulmonary disease, but Mucor caviations are much rarer in the stated site.
Dt•betJc ketOIOCidO Mucor Hypha Diabetes mellitus Ketoacidosis Infection
28
29 E is not correct . 5°/o chose thi s.

8
Lode.
s
S uspe-nd
8
End Bloc:k
Item: 29 of ~ ,• Mark <::J [:::> ""I ~· ~'j
100 J.. Previous Next faiUI~S
LAb Notes Calculator

3
E is not correct. 5 °/o chose this.
4
Sporothrix schenckii is a dimorphic f ungus with ciga r-shaped budding yeast t hat are visible in pus. It can cause sporot richosis when
5
introduced, usually by way of a thorn prick, into the skin. Sym ptoms include t he format ion of a loca l pustule or ulcer on the skin with nodules
6 along t he draining lymphatics (ascend ing lymphangit is) . S schenckii rarely causes syst em ic illness.
7 Sporotrichosis Sporothrix schenckii Lymphangitis Dimorphic fungus Lymphatic system Pus Yeast Fungus Ulcer (dermatology)

8
9 Bottom Line:
10 Aspergillus is a mold wit h septat e hyphae that branch at 45-degree ang les. Aspergillus fumigatus infection commonly causes aspergill omas
11 ("fungus balls") . Anot her member in t he Aspergillus fami ly, A flavus, produces t he t oxin aflatoxin, the consumption of which has been
12 associat ed with increased ri sk for hepatocellular carcinoma.
Aspergillus fumigatus Aflatoxin Hepatocellular carcinoma Aspergillus Hypha Toxin Mold Infection Septum Septate Carcinoma
13
14
15
141;fil·1i•J for year:[2017
FIRST AID FA CTS .
•j .
16
17
18 FA17 p 149.1
19 Opportunistic fungal infections
20 Candida albicans alba = white. Dimorphic; Forms pseudohyphae and budding yeasts at zooc fJ, germ tubes at
21
22
37°C rn.
Systemic or superficial fungal in Fection. Causes oral ~ and esophageal thrush in
23
immunocompromised (neonates, steroids, diabetes, AIDS), vulvovaginitis (diabetes, use of
24
antibiotics), diaper rash, endocarditis (IV drug users), disseminated candidiasis (especially in
25
neutropenic patients), chronic mucocutaneous candidiasis.
26
Treatment: oral Auconazolc/topical azoic for vagina l; nystatin, fluconazole, or caspofungin for oral!
27
28
esophageal; fluconazole, caspofungin, or amphotericin B for systemic.
29 Aspergillus Septate hyphae that branch at45° Acute An~ l e [!]. Produces conidia in ra dia tin~ cha ins at end of
8
Lock
s
Suspend
0
End Block
Item: 29 of - ,• Mark -<] 1:> ""'I ~· 1!';:'1
100 ~ Prev1o u s Next Labf a lu es Notes Calculator

3
Aspergillus Septate hyphae that branch at 45° Acute Angle (!l. Produces conidia in radiating chains at end of
4
fumigatus conidiophore D.
5
Causes invasive aspergillosis in immunocompromiscd, patients with chronic granulomatous disease.
6
7
Can cause aspergillomas in pre-existing lung ca' ities, especially after TB infection.
8
Some species of Aspergillus produce \ fl ato,ins {associated with hepatocellular carcinoma).
9 Allergic bronchopulmonary aspergillosis { BP ): hr persensitivity response associated with
10
asthma and cystic fibrosis; ma} cause bronchiectasis and eosinophilia.
11

12
Cryptococcus 5-10 Jlm with narrow budding. Ilea' ilr encapsulated )Cast. i':ot dimorphic.
13
neoformans Found in soil, pigeon droppings. cquircd through inhalation with hematogenous dissemination
14 to meninges. Culture on Sabouraud agar. II ighlightcd "ith India ink (clear ha lo ~) and
15 mucicarmine (red inner capsule ). Latex agglutination test detects polysaccharide capsular
16 antigen and is more specific.
17 Causes cryptococcosis, cryptococcal meningitis, cryptococcal encephalitis ("soap bubble" lesions
18 in brain), primarily in immunocompromiscd.
19 Treatment: amphotericin B + flucytosine followed by fluconazole for cryptococcal meningit is.
20
Mucor and Rhizopus Irregular, broad, nonseptale hyphae branching at wide angles ITJ.
21
spp. Mucormycosis. Causes disease mostly in ketoacidot ic diabetic and/or neutropenic patients (eg,
22
leukemia). Fungi proliferate in blood vessel walls, penetrate cribriform plate, and enter bra in.
23
Rhinocerebral, fronta l lobc abscess; cavernous sinus thrombosis. llcaclache, facial pain, black
24
necrotic eschar on face; mar have cranial nerve invokcment.
25
26
Treatment: surgical debridement, amphotericin B.
27
28
29 •
8
L.odt
s
Su~pl'nd
~
End Block
Item: 29 of - ,• Mark -<] 1:> ""'I ~· 1!';:'1
100 ~ Prev1o u s Next Labf a lu es Notes Calculator

3
FA17 p 150.1
4

5 Pneumocystis jirovecii Causes Pneumocystis pneumonia (PCP), a diffuse interstitial pneumonia f.i.l. Yeast-like
6 fungus (originally classified as p roto;~oan). Inhaled. ~ l ost in fections are asrmptomatic.
7 Immunosuppression (eg, AJ DS) predisposes to disease. Diffuse, bilateral ground-glass opacities on
8 C ' R/CT m. Diagnosed by lung biopsy or lavage. Disc-shaped yeast seen on methenamine siher
9 stain of lung tissue m.
10 Treatment/prophylaxis: TYI P-StVIX, pentamidine, dapsone (prophrlaxis only), atontquone. Start
11 prophylaxis when CDf+ count drops to < 200 cells/mm 3 in HIV patients.
12
13
14

15
16
17
18
19
20
21
22
23
24
25 FA17 p 150.2
26
Sporothrix schenckii Sporotrichosis. Dimorphic, cigar-shaped budding yeast that grows in branching hyphae with

--·
27
rosettes of conidia; lives on 'egetation. When spores are traumatically introduced into the
28
"""'-
~~~ ·· .. ..,., skin, typically by a thorn {"rose gardener's disease"), causes local pustule or ulcer fJ "ith
29 ""'--- . 'If-.. •
8
L.odt
s
Su~pl'nd
~
End Block
Item: 29 of - ,• Mark -<J [:::> "'I ~ · ~
~ P~v1ous
100 N @xt Labl lues
. N otes
~
Calculator
.. .. " . . .
4 prophylaxis when CD4+ count drops to< 200 cells/mm3 in HIV patients.
5
6
7
8
9
10
11

12
13
14
15
16
17
18 FA17 p 150.2
19 Sporothrix schenckii Sporotrichosis. Dimorphic, cigar-shaped buckling yeast that grows in branching hyphae with
20
rosettes of conidia; lives on vegetation. When spores are traumatically introduced into the
21
skin, typically by a thorn ("rose gardener's disease"), causes local pustu le or ulcer r.J with
22
nodules along dra ining lymphatics (ascending lymphangitis). Disseminated disease possible in
23
immunocomprom iscd host.
24
Treatment: itraconazole or potassium iodide.
25
Th ink of a rose gardener who smokes a cigar and pot.
26
27
28
29 •
8
Lode.
s
Suspe-nd
8
End Bloc:k
Item: 30 of - ,• Mark -<J [:::> "'I ~ · ~
100 ~ P~v1ous N @xt Labl lues N o tes Calcula to r
4
A patient has a blood test that is positive for a member of the hepadnavirus fami ly. Several weeks earlier he had a mild fever, myalgias, ~~AI
5
and easy fatigability.
6
7
Which behavior is known to increase t he risk of acquiring this condition?
8
:
9
A. Excess alcohol consumption
10
11
B. Overeating
12 C. Smoking
13
0 . Unprotected sex
14
E. Vaccination
15
16
17
18
19
20
21
22
23
24
25
26
27
28
29
. 30 •
8
Lode.
s
Suspe-nd
8
End Bloc:k
Item: 30 of ~ ,• Mark <:::1 t::> ""I ~· ~'j
100 J.. Previous Next LAb faiUI~S Notes Calculator

4
5 The correct a nswer is 0. 82°/o chose t his.
6 This patient has been infect ed wit h t he hepatit is B v irus, the sole member of th e hepadnavirus family. I t is t ransm it ted t hrough sexual,
hemat ogenous, and materna l/fetal rout es. A blood test posit ive for HBcAb (antibody against hepat it is B core ant igen) but negat ive for HBsAg
7
(hepat it is B surface ant igen) and HBsAb (hepatit is B surface ant ibody) indicates t hat t he pat ient had acut e hepat it is B disease and is currently
8 in the window period . I n ot her words, t he acute disease has resolved and he can eit her go on to a complet e recovery or chron ic
9 infection/ carrier statu s.
Hepadnaviridae HBcAg Hepatitis 8 Hepatitis Antigen Virus Antibody Hepatitis 8 virus Blood test Window period HBsAg Acute (medicine)
10
11 A is not correct . 9°/o chos e this.
12 Excessive alcohol int ake is not a risk fact or for hepatit is B. However, alcohol can induce nonviral hepat it is and cirrh osis of the liver.
Cirrhosis Hepatitis Liver Alcohol Alcoholic beverage Risk factor
13
14 B is not correct. 3% chose t his .
15
Overeat ing is not a ri sk factor for hepatit is B. However, obesity and excess int ake of fatt y foods can cause non -alcoholic fat ty liver disease.
Non-alcoholic fatty liver disease Hepatitis Fatty liver Liver Obesity Liver disease Risk factor
16
17 C is not co rrect. 4 °/o chose this .
Smoking is not a risk factor fo r hepat it is B.
18
Hepatitis Risk factor Tobacco smoking Smoking
19
20 E is not correct. 2 °/o chos e this .
Vaccinat ion would not result in the clinica l scenario described. Unlike the pat ient described, individuals who have been vaccinated aga inst
21 hepatit is Bare posit ive fo r hepatit is B surface antibody, but negat ive for hepatit is B surface ant igen and hepat it is B core ant igen ant ibody.
22 Hepatitis 8 HBcAg Hepatitis Antigen vaccination Antibody Vaccine HBsAg

23
24
Bottom Line :
25
Hepatit is B, t he sole mem ber of the hepadnavirus fam ily, is t ransmitted through sexual, blood, and mat erna l/fetal rout es.
26 Hepadnaviridae Hepatitis B Hepatitis
27
28
29 141;fil·1i•J for yea r:[2017 •
FIRST AID FA CTS
30

8
Lock
s
Suspend
0
End Block
Item: 30 of - ,• Mark -<] 1:> ""'I ~- 1!';:'1
100 ~ Prev1o u s Next Labf a lu es Note s Calculator
4

5
FA17 p 170.1

6
Hepatitis serologic markers
7
Anti-HAV (lgM) Ig I antibody to II V; best test to detect acute hepatitis .
8
9
Anti-HAV (lgG) IgC antibody indicates prior HAV infection and/or prior vaccination; protects against reinfection.
10 HBsAg Antigen found on surface of I JBV; indicates hepatitis B infection.
11
Anti-HBs ntibody to HBsAg; indicates immunity to hepatitis B due to ,-accination or rcco,·cry from
12
infection.
13
14 HBcAg Antigen associated with core of II BV.
15 Anti-HBc ntibody to HBc g; I g~ 1 =acute/recent infection; IgC =prior exposure or chronic infection. l g\11
16 anti-JIBe may be the sole<±> marker of infection during window period.
17
HBeAg Secreted by infected hepatocyte into circulation. 'ot part of mature HBV ,·irion. Indicates active
18
viral replication and therefore high transmissibility and poorer prognosis.
19
20 Anti-HBe Antibody to HBeAg; indicates low transmissibil il).
21
Important Incubation Prodrome. Convalescence
22 diagnostic period acute disease Early Late
23 tests
HBsAg Anti· Ant1·HBs Surface antigen
HBsAg
24 (anb-HBcl HBc (anti-HBcl HBsAg \
DNA
25 Anti -HBc polymerase
DNA
26 Relative polymerase

27
concentration
of reactants HBV parbcles DNA~
Envelope
28 antigen (-)

I
HBsAg HBeAg --........._ (+)
29
....... •
30

8
L.odt
s
Su~pl'nd
~
End Block
Item: 30 of - ,• Mark -<] 1:> ""'I ~- 1!';:'1
100 ~ Prev1o u s Next Labf a lu es Note s Calculator

4
Important Incubation Prodrome. Convalescence
5 Early Late
diagnostic penod acute disease
6 tests
HBsAg Anb· Antt·HBs Surface antigen
HBsAg
7 (anb·HBcl HBc (anti· HBc l HBsAg \
DNA
8 polymEraSe
DNA Anti· HBc
9 Relative polymerase
10
concentration
of reactants
HBV particles DNA ~
Envelope
11
anttgen
12 HBsAg HBeAg- - - .
13
Anii· HBs
14 Core aoogen /
HBeAg Anti·HBe HBcAg . . /
15
Level of
16
detection
17
Months after 0 1 2 4 5 6 7 8
18 exposure Symptoms
19
SGPT !ALTJ
20
21
22
HBsAg Anti- HBs HBeAg Anti-HBe Anti-HBc
23 Acute HBV ./ ./ lg 1
24
Window ./ lgM
25
Chronic HBV (high infectivity) ./ ./ lgC
26
27 Chronic HBV (low infectivity) ./ ./ lgC
28
Recovery ./ ./ lgC
29
30 Immunized ./ •
8
L.odt
s
Su~pl'nd
~
End Block
Item: 30 of ~ ,• Mark <:::1 t::> ""I ~· ~'j
100 J.. Previous Next LAb faiUI~S Notes Calculator

4
FA17 p 168.1
5
6 Hepatitis Signs and symptoms of all hepatitis viruses: episodes of fever, jaundice, t ALT and AST. Naked viruses (HAV
7 viruses and HEV) lack an envelope and are not destroyed by the gut: the vowels hit your bowels.
8 HBV D A polymerase has D A- and RNA-dependent activities. Upon entry into nucleus, the polymerase
9 completes the partial clsD 1A. Host R1 A polymerase transcribes mRNA from ,·iral D A to make ,·ira !
10 proteins. The DNA polymerase then reverse transcribes viral R A to D1 A, which is the genome of the
11 progeny v1rus.
12 HCV lacks 3'-5' exonuclease activity - no proofreading ability - variation in antigenic structures of HCV
13
envelope protei ns. Host antibody production lags behind production of new mutant strains of HCV.
14
Virus H~ H~ HCV H~ HEV
15
16
FAMILY R 'A picornm·irus DNA hepadnavirus R A Aavivirus R A dcltavirus R1 A hepevirus
17 TRANSMISSION Fecal-oral (shellfish, Parenteral (Blood), Primarily blood Parenteral, sexual, Fecal-oral,
18 travelers, day care) sexual (Baby- (IVDU, post- perinatal especially
19 making), perinatal transfusion) waterborne
20 (Birthing)
21
INCUBATION Short (weeks) Long (months) Long Superinfection Short
22
23
(l lDV after
24
HBV) = short
25
Coinfcc tion (HDV
26 with HBV) = long
27 CLINICAL COURSE Asymptomatic Initially li ke serum ~'layprogress to Similar to HBV Fulminant hepatitis
28 (usually), Acute sickness (fever, C irrhosis or in E xpectant
29 arthralgias, rash); Carcinoma (pregnant) women
30

8
Lock
s
Suspend
0
End Block
Item: 30 of - ,• Mark -<J [:::> "'I ~ · ~
100 ~ P~v1ous N @xt Labl lues N o tes Calcula to r
4 A A

INCUBATION Short (weeks) Long (months) Long Superinfection Short


5
6 (llOV after
7 HBV) =short
8 Coinfcction (IJDV
9 with HBV) = long
10 CliNICAL COURSE Asymptomatic Initially like serum r-. lay progress to Similar to HBV Fulminant hepatitis
11
(usually), Acute sickness (fever, C irrhosis or in Expectant
12
arthralgias, rash); Carcinoma (pregnant) "omen
13
may progress to
14
carcmoma
15
16 PROGNOSIS Good dults - mostlr !\ lajorit) de,·elop Superinfection l ligh mortality in
17 full resolution; stable, C hronic - worse prognosis pregnant women
18 neonates - worse hepatitis C
19 prognosis
20
HCC RISK 0 Yes Yes Yes I 0
21
22
LIVER BIOPSY Hepatocyte Granular Lymphoid Similar to HBV Patchy necrosis
23 swel ling, eosinophilic aggregates with
24 monocyte "ground glass" foca 1areas of
25 infiltration, appearance; nu1cro' esicula r
26 Counci lman cytotoxic T cells steatosis
27 bodies mediate damage
28
NOTES No carrier state Carrier state Carrier state ,·err Defecti,·e ,·irus, Enteric, Epidemic,
29
("Alone") common common Depends on HBV no carrier state
30 • .. . ......

8
Lode.
s
Suspe-nd
8
End Bloc:k
Item: 31 of - ,• Mark -<J
P~v1ous
[:::> "'I ~ ·· ~
100 ~ N @xt Labl lues N o tes Calculato r
5
A soldier presents to the infirmary wit h complaint s of several days of malaise and headache. Vit al signs are re markable for a temperature ~~AI
6
of 38.0° C (100.4° F). Physical examination reveals pain wit h flexion and extension at the neck and nonblanching maculopapular lesions on
7
his extremit ies. Lumbar puncture is performed, and cerebrospinal fluid analysis shows high protein, low glucose, 14 RBCs/mm 3 and WBC
8 count of 7543/mm 3 , with 72% segmented neutrophils and 18% lymphocytes.
9
10 Bacterial culture of the cerebral spinal fluid is most likely to show which of the fol lowing?
11 :
12 A. Gram-negative bacilli
13 B. Gram-negative coccobacilli
14
C. Gram-negative diplococci
15
16 D. Gram-positive bacilli
17 E. Nothing, since this is most likely a viral infection
18
19
20
21
22
23
24
25
26
27
28
29
30
. 31 •
8
Lode.
s
S uspe-nd
8
End Bloc:k
Item: 31 of - ,• Mark -<J
P~v1ous
[:::> "'I ~ ·· ~
100 ~ N @xt Labl lues N o tes Calculato r
5 A A

6 The correct answer is c. 70°/o chose this.


7 This patient most likely has bact eri al meningit is. Bacterial meningit is is characterized by increased
polymorphonuclear leukocyt es, increased protein levels, and decreased glucose levels in the cerebrospinal
8
flu id. The most likely causes of bacterial meningitis in young adults are Streptococcus pneumoniae and
9 Neisseria meningitidis. S. pneumoniae is a gram-positive diplococcus, wherease N. meningitidis is a gra m -
10 negative diplococcus that ferments malt ose and glucose. Often a petechial rash ( like that shown in t he
image) will occur when meningococcemia occurs in conjunction with meningitis. N. meningitidis should be
11
cultured on Thayer-Martin media, and antibiotic therapy should be initiated as soon as there is suspicion of
12 a bacterial cause.
13 Streptococcus pneumontae Cer-ebrospinal fluid Meningitis Maltose Grafn.necJabvt bactena Mcrungococcet'Tllla Mentngococcal disease

14 Anbb•ot•cs Gram-pos•bve bactena Dplococcus Petechta Purpura streptococcus Nefssena Wh•te blood cell Protem a.ctenal memng•bs Glucose Granulocyte

15 Rash Th~·Martm aQar

16 Image courtesy of CDC


17
18
A is not correct. 8°/o chose this.
19
Escherichia coli is a gram-negat ive rod (bacillus) and is t he second most common cause of meningitis in neonates.
Escherichta colt Gram~negattve bactena Mentngtbs Bacillus (shape) Bacillus Infant
20
B is not correct. 12°/o chose this.
21
Haemophilus influenzae is a gram-negative coccobacillus. In the past It was a major cause of meningitis in t he young, but wi t h the
22
introduction of an effective vaccine, the frequency of t his t ype o f infection has decreased markedly.
23 Haemophtlus tnfluenzae Coccobactllus Gram-negative bacteria Meningitis vaccine Haemophilus Infection

24
D is not correct. 7°/o chose this.
25
Gram-posit ive bacilli include species of Clostridium, Corynebacterium, Listeria, and Bacillus. Listeria monocytogenes is the third most common
26 cause of meningitis in newborn s.
Ltsteria monocytogenes Gram·pos1bve bactena Meningitis Usteria Clostridium Bac1llus (shape) Bacilli Corynebactenum 6ac1llus Spec1es
27
28 E is not correct . 3°/o chose this.
29 Viral meningitis characteristically resu lts in lymphocytosis, not increased polymorphonuclear leukocyt es. Normal glucose and protein levels are
30 also characteristic of viral meningitis. The most common causes of viral meningitis in the United States are enteroviruses.
Men1nQ1b V1"' """""I gtb5 Whrte blood ceU V1rus Protein Glucose Enterovirus Lymphocyt:ost-r
31 •
8
Lode.
s
S uspe-nd
8
End Bloc:k
Item: 31 of ~ ,• Mark <::J [:::> ""I ~· ~'j
100 J.. Previous Next LAbfaiUI~S Notes Calculator

5
6
7 Bottom Line:
8 The most likely gram-negative diplococcus responsible fo r mening itis is Neisseria meningitidis.
Neisseria meningitidis Gram-negative bacteria Meningitis Diplococcus Neisseria
9
10
11

12
141;fil·1i•J for yea r:[ 2017
FIRST AID FA CTS .
•j .

13
14 FA17 p 176.1
15 Common causes of meningitis
16 NEWBORN (0- 6MOl CHILDREN (6 M0- 6 YR) 6- 60YR 60YR+
17
Group B streptococci S pneumoniae S pneumoniae S pneumoniae
18
E coli N meningitidis N meningiticlis (#1 in teens) Gram 8 rods
19
Listeria H infiuenzae type B Enterovi ruses Listeria
20
Enteroviruses HSV
21
22 Give ceftriaxone and vancomycin empirically (add ampicillin if Listeria is suspected).
23 Viral causes of meningitis: enteroviruses (especially coxsackievirus), llSV-2 {llSV-1 = encephalitis), IIIV, West ile virus (also
24 causes encephalitis), VZV.
25 In HIV: Cryptococcus spp.
26 Note: Incidence of H infiuenzae meningitis has l greatly due to conjugate H infiuenzae vaccinations. Today, cases are usually
27 seen in unimmunized children.
28
29
30
FA17 p 176.2
31 CSF findings in meningitis

8
Lock
s
Suspend
0
End Block
Item: 31 of - ,• Mark -<J
P~v1ous
[:::> "'I ~ ·· ~
100 ~ N @xt Labl lues N o tes Calculato r
5 A A
FA17 p 138.2
6
7 Neisseria Gram 8 diplococci. Metabolize glucose \leninGococci ferment :\laltose and Glucose.
8 and produce lgA proteases. Contain Gonococci ferment Glucose.
9 Iipool igosaccharides (LOS) with strong
10 endotoxin acti\'ity. gonorrhoeae is oft en
11 intracellular (\\'ithin neutrophils)
12
13 Gonococci Meningococci
14 o polysaccharide capsule Polysaccharide capsule
15
l o maltose metabolized Maltose fermentation
16
17 o vaccine due to antigenic \'ariation of pilus Vaccine (type B vaccine not widely a\'ailablc)
18 proteins
19 Sexually or peri natally transmitted Transmitted via respiratory and ora l secretions
20
Causes gonorrhea, septic arthritis, neonatal Causes meningococcemia with petechial
21
22
conjunctivitis (2-5 days after birth), pelvic hemorrhages and gangrene of toes rn.
23
inAammatory disease (PID), and f'itz- llugh- meni ngitis, Waterhouse-Friderichsen
24 Curtis syndrome syndrome (adrenal insufficiency, fever, DIC,
25 shock)
26 Condoms l sexual transm ission, erythromycin Ri fampin, ciproAoxacin, or ceftriaxone
27 eye ointment prevents neonatal blindness prophylaxis in close contacts
28
Treatment: ceftriaxone + (azithromycin Treatment: ceftriaxone or penicillin G
29
30
or doxycycline) for possible chlamydia]
31
coinfection

8
Lode.
s
S uspe-nd
8
End Bloc:k
Item: 32 of - ,• Mark -<J [:::> "'I ~· ~
100 ~ P~v1ous N@xt Labl lues Notes Calculator
6 A A

A 27-year-old woman present s to her primary care physician com plaining of feeling "under the weather" fo r t he past 2 weeks. Her history ~~AI
7
is remarkable only for previously diagnosed m itra l valve prolapse. On examination, her t emperature is 37.8°C ( 100°F). The patient has
8
small erythematous papules on her palms and painf ul, raised lesions on her finger pads. The result s of an ophthalmologic examination are
9 shown in the image.
10
11

12
13
14

15
16
17
18
19
20
21
22
23
24
25
26
27
28 What is the most likely path ogen causing t his woman 's symptoms?
29 :
30 A. Streptococcus agalactiae
31
B. Streptococcus sanguinis
. 32

8
Lode.
s
Suspe-nd
8
End Bloc:k
7
8
9
10
11

12
13
14
15
16
17
18
19
20
21
22
23
24 What is the most likely pathogen causing t his woman's symptoms?
25 :
26 A . Streptococcus agalactiae
27
B. Streptococcus sanguinis
28
29
C. Haemophilus influenzae
30 D. Staphylococcus aureus
31 E. Streptococcus pyogenes
. 32 •
8
L.odt
s
Su~pl'nd
~
End Block
Item: 32 of ~ ,• Mark <::J [:::> ""I ~· ~'j
100 J.. Previous Next LAb faiUI~S Notes Calculator

6
The correct a nswer is B. 38% chose t his .
7
Subacute bacterial endocarditis is charact erized by feve r and signs of microemboli. Signs of m icroemboli include Osier nodes (t ender ra ised
8 lesions on finger and toe pads), Rot h spots (round, whit e spot s on the ret ina surrounded by hemorrhage; see arrow in t he v ignette image),
9 Janeway lesions (small, nontende r~ painless, erythemat ous lesions on the palm or sole), and splinter hemorrh ages (small blood clots t hat run
vertically under t he fingernails) . Anot her sign of endocardit is is a new-onset heart murmur. Viri dans st re pt ococci (eg, S. mutans, S. sanguinis,
10
S. ora/is, and S. mitis) are gram-posit ive cocci t hat grow in chains. They oft en cause subacute bacterial endocardit is, part icularly in the sett ing
11 of m itral valve prolapse. They generally have low v irulence, but can form small veget at ions on congenitally abnormal or damaged valves when
12 seeded into the bloodstream. Complications from endocardit is include emboli and heart fa ilure.
Roth's spot Osler's node Mitral valve prolapse Endocarditis Janeway lesion Retina Subacute bacterial endocarditis Viridans streptococci Heart murmur Mitral valve Streptococcus Thrombosis
13
Gram-positive bacteria Infective endocarditis Septic embolism Coccus Embolism Thrombus Streptococcus mutans Bleeding Heart failure Fever Erythema
14
15 A is not correct . 7°/o chose this.
16 Group B Streptococcus (S. agalactiae) is best known for causing life-th reaten ing illness in newborns. Because pregnant women can be
asymptomat ic carri ers of Grou p B Streptococcus, it is t he leading cause of bloodstream infect ion and meningit is in neonates.
17
Meningitis Streptococcus Sepsis Bacteremia Streptococcus agalactiae Asymptomatic Infant Asymptomatic carrier Circulatory system Infection
18
19
C is not co rrect. 8°/o chose this .
Haemophilus influenzae is a small gra m -negat ive rod t hat causes epiglott it is, meningitis, ot it is media, and pneumonia. Haemophilus, along
20
with Aggregatibacter, Cardiobacterium, Eikenella, and Kingella (HACEK organisms), are commensal bact eria of t he oral cavity t hat are isolat ed
21 in some cases of su bacute endocardit is. Yet even combined, the HACEK organisms are t he cause of endocardit is much less commonly t han
22 viri dans st reptococci alone.
Commensalism Haemophilus influenzae Otitis media Gram-negative bacteria Endocarditis Viridans streptococci Meningitis Pneumonia Streptococcus Kingella kingae Epiglottitis Eikenella corrodens Bacteria
23
HACEK endocarditis Mouth Cardiobacterium hominis Haemophilus Subacute bacterial endocarditis Aggregatibacter
24
25 D is not correct. 22% c hose this.
26 Staphylococcus aureus, a gram-posit ive coccus that grows in clust ers, is responsible fo r bacterial endocarditis characterized by rapidly
prog ressive illness, high fever, and shaking chills. I t is a highly virulent bact eri a and causes large vegetations. I t is a common pat hogen in IV
27
drug users.
28 Staphylococcus aureus Gram-positive bacteria Endocarditis Staphylococcus Pathogen Infective endocarditis Coccus Bacteria Drug injection Chills Fever

29
E is not correct. 25°/o chose this.
30
Streptococcus pyogenes is a Group A Streptococcus that is the causat ive agent in streptococcal pharyngit is. It is not associated wit h bacteria l
31 endocarditis. Children wit h streptococcal pharyngit is can have fever, anterior cervical lymphadenopat hy, and a red pharynx with exudat e. They
32 also have a "sandpaper" rash primari ly on t he t runk. I nfection wit h S. pyogenes can lead to sequelae such as poststreptococcal •

8
Lock
s
Suspend
0
End Block
Item: 32 of ~ ,• Mark <::J [:::> ""I ~· ~'j
100 J.. Previous Next LAb faiUI~S Notes Calculator

6
E is not correct. 25°/o chos e this.
7
Streptococcus pyogenes is a Group A Streptococcus t hat is t he causat ive agent in streptococcal pharyngit is. It is not associated wit h bacterial
8
endocarditis. Child ren wit h streptococcal pharyngit is can have fever, anterior cervical lymphadenopat hy, and a red pharynx with exudat e. They
9 also have a "sandpaper" rash primari ly on t he t runk. I nfection wit h S. pyogenes can lead to sequelae such as poststreptococcal
10 glomerulonephrit is or acute rh eumatic feve r (migratory arthri t is, cardiac damage, subcut aneous nodules, erythema marginat um, Syndeham
chorea) .
11 Streptococcal pharyngitis Erythema marginatum Streptococcus pyogenes Rheumatic fever Pharyngitis Lymphadenopathy Pharynx Endocarditis Erythema Glomerulonephritis Streptococcus Exudate Chorea
12 Rash Sequela Infective endocarditis Arthritis Cervical lymphadenopathy Subcutaneous tissue Fever
13
14
15
Bottom Line :
16
Viridans streptococci are t he most common cause of subacut e bact eri al endocardit is in patients with damaged valves. Clinical signs include
retinal spots (Rot h spots) and Janeway lesions (nonten der lesions on t he hands).
17 Roth's spot Janeway lesion Viridans streptococci Endocarditis Infective endocarditis Streptococcus Subacute bacterial endocarditis
18
19
20 l@);fil ~1hl for yea r:l 2o17 y
FIRST AID FACTS
21
22
FA17 p 299.2
23
24 Bacterial endocarditis Fever (most common symptom), new murmur, Mitral valve is most freque ntly involved.
25 Roth spots (round white spots on retina Tricuspid valve endocarditis is associated with
26 surrounded by hemorrhage rl), Osler nodes IV drug abuse (don't "tri" drugs). Associated
27 (tender raised lesions on Finger or toe pads EJ with S aureus, Pseudomonas, and Candida.
28 clue to immune complex deposition), Janeway Culture 8; most likely Coxiella bumetii,
29
lesions (small, pai nless, erythematous lesions Bartonella spp., IIACEK (llaemophilus,
30
on palm or sole)~. glomerulonephritis, Aggregatibacter (formerly Actinobacillus),
31
septic arterial or pulmonary emboli, splinter Cardiobacterium, Eikenella, Kingella)
32 • •I I I a. d- J, • I 1 I .. n

8
Lock
s
Suspend
0
End Block
Item: 32 of - ,• Mark -<J [:::> "'I ~ · ~
100 ~ P~v1ous N@xt Labl lues Notes Calculator
6 •
S bovis (galfolyticus) is present in colon cancer,
7
8
S epidermidis on prosthetic valves.
9
Endocarditis mar also be nonbactcrial
10 {marantic/thrombotic) 2° to malignancy,
11 hrpercoagulable state, or lupus.
12
13
14
15
16
17
18
19
t
20
21
FA17 p 132.3
22
23 Viridans group Gram®, a-hemolytic cocci. They are normal Sanguinis= blood. Think, "there is lots of
24 streptococci Aora of the oropharynx that cause dental blood in the heart" (endocarditis). S scmgui11is
25 caries (Streptococcus mutans and S mitis) makes clcxtrans, which bind to fibrin-platelet
26 and subacute bacterial endocarditis <II e~ggrcga tes on damaged heart valves.
27 damaged heart valves (S sanguinis). Resistant Viridans group strep live in the mouth because
28 to optochin, differentiating them from they are not afraid of-the-chin {op-to-chin
29 resistant).
S pneumoniae, which is a-hemolytic but is
30
optoch in sensiti\'e.
31
32 •
8
Lode.
s
Suspe-nd
8
End Bloc:k
Item: 33 of - ,• Mark -<J [:::> "'I ~· ~
100 ~ P~v1ous N @xt Labl lues N o tes Calculato r
7 A A

A 25-year-old woman complains of a nonproductive, hacking cough starting 1 week ago. Additionally, she has had headaches and a mild ~~AI
8
fever during this time. Physical examinat ion is not able only for crackles heard in her lungs bilaterally. A chest radiograph is shown below. A
9 sample of her sputum revea ls numerous cytoplasmic inclusions.
10
11

12
13
14

15
16
17
18
19

20
21
22
23
24
25 Image cowtesy of James Heilman, MD

26
27 Which of the fol lowing organi sms is most likely responsible for t his woman 's illness?
28 :

29 A. Chlamydia trachomatis
30 B. Chlamydophila pneumoniae
31
C. Mycoplasma pneumoniae
32
. 33
D. Nocardia asteroides

8
Lode.
s
S uspe-nd
8
End Bloc:k
8
9
10
11

12
13
14
15
16
17
18
19
20
21
22
23 Image couttesy of James Heilman, MD

24
Which of the fol lowing organisms is most likely responsible for t his woman 's illness?
25
26 :
A. Chlamydia trachomatis
27
28 B. Chlamydophila pneumoniae
29 C. Mycoplasma pneumoniae
30
D. Nocardia asteroides
31
32 E. Streptococcus pneumoniae
. 33 •
8
L.odt
s
Su~pl'nd
~
End Block
Item: 33 of - ,• Mark -<J [:::> "'I ~ · ~
100 ~ P~v1ous N @xt Labl lues N o tes Calculato r
7 A A

8
The correct a ns w e r is B. 56% chose this.
9
Chlamydophila pneumoniae is an obligate int racel lular bacterium that causes atypical pneumonia, 3
10 characterized by a dry, hacking cough, sore throat, fever, and headaches. Unlike typical pneumonias, the 1
11 onset of atypical pneumonia is insidious and patients look less acutely ill. X- ray of the chest may show
12 patches or streaks of infiltrate, but may also be normal. Cytoplasmic inclusions (indicated by the number 3
in this image) are observed on sputum samples from patients with C. pneumoniae infection. Also, this type
13
of pneumonia cl assically comes with eosinophilia, so an acute increase in absolute eosinophil count together
14 with the classic X-ray findings of patches or streaks of infiltrate can be a good indicator of chlamydia! origin.
Eo •rtop~u Chi lo1 Ia p 1eu lOn de Atyp1cal pneumonia Pneumonia Sputum Chest rad•ooraph Co• gh ff"c. 10 ja Sore throdt Bacteria lntracellular-
15
F~ X·r..,. Chlamydophlla Chlamydoa onfecbon
16
17 Image courtesy of Wikimedia
Commons
18
19 A is not correct. 7°/o chose this .
20 Chlamydia trachomatis is the most common sexually transmitted disease worldwide, and it can cause urethrit is, cervicitis, and pelvic
21 inflammatory disease as well as conj unctivitis and infant pneumonia. It does not, however, cause atypical pneumonia. All Chlamydia species
are obligate intracellular pathogens t hat appear as cytoplasmic inclusions on a Giemsa stain.
22
Pelvic inflammatory d1sease Sexually transm1tted Infection Chlarrt(dia trachomat1s Urethnt1s ConJunctivitiS CerviCitiS Atyp1cal pneumonia Chlam,'dia infection Chlamydia (genus) Pneumonia Pathogen
23
Inflammation Giemsa sta•n Cytoplasm
24
25
C is not correct . 28% chos e this.
Mycoplasma pneumoniae is anot her cause of aty pical pneumonia, but unlike Chlamydophila pneumoniae, it does not produce cytoplasmic
26
inclusions. Mycoplasma organisms lack cell walls, making them resistant to penicillin and other ~ - lacta m ant ibiot ics. A quick test
27 for Mycoplasma infection is a DNA st ain using indirect Hoescht staining techniques to detect Mycoplasma DNA from cell cultures.
28 Mycoplasma pneumoniae Atyp1cal pneumoma Chlamydophila pneumoniae Penicillin Pneumonia Mycoplasma AntibiOtiCS Chlamydoph1la DNA Cytoplasm

29 D is not correct . 4 °/o chos e this .


30 Nocardia asteroides is a gram-posit ive, acid-fast aerobe that causes pneumonia and lung abscesses, primarily in immunocompromised
31 individuals. Staining does not revea l cytoplasmic inclusions.
Noc.Jtrd1a tero1de!t Noc d1a Aerob1c orgamsm Immunodeficiency Gram-pos1bve bacteria Pneumon1a CeUulat rsp1rat•on Ac1d-fast Abscess Lung
32
33 E is not correct . 5°/o chose this .
8
Lode.
s
S uspe-nd
8
End Bloc:k
Item: 33 of ~ ,• Mark <::J [:::> ""I ~· ~'j
100 J.. Previous Next LAbfaiUI~S Notes Calculator

7
E is not correct. 5 °/o chose this .
8
Streptococcus pneumoniae is an encapsulated, gram-posit ive, lancet-shaped coccus t hat commonly causes pneumonia. Sym ptoms include
9 fever, chills, pleuri t ic chest pain, and cough productive of rust-colored sputum. Chest X-ray may show lobar consolidation in the lungs.
10 Cytoplasmic inclusions are not detect ed in sput um samples.
Streptococcus pneumoniae Chest radiograph Pneumonia Gram-positive bacteria Sputum Pleurisy Streptococcus Coccus Chest pain Cough Pulmonary consolidation X-ray Fever
11

12
13 Bottom Line :
14 Chlamydophila pneumoniae infection causes an aty pica l pneumonia with dry, hacking cough, sore throat , fever, and headache. The
15 organ isms are seen as cyt oplasm ic inclusion bod ies on Giemsa st ain of sputum .
Chlamydophila pneumoniae Atypical pneumonia Giernsa stain Sputum Pneumonia Sore throat Headache Cough Fever Infection Cytoplasm
16
17
18
19
i@l;fil·1i•J for yea r:[2017 • J
FIRST AID FACTS

20
21 FA17 p 146.1
22
Chlamydiae Chlamydiae cannot make their own ATP. T hey Chlamys = cloak (intracellular).
23
are obligate intracellular organisms that cause C psittaci- has an avian reservoir (parrots),
24
mucosal infections. 2 forms: causes atypical pneumonia.
25
Elementary body (small, dense) Lab diagnosis: PC R, nucleic acid amplification
26
27
is "E nfcctious" and Enters cell via test. Cytoplasmic inclusions (reticulate bodies)
28
E ndocytosis; transforms into reticulate body. seen on Gicmsa or Auorescent antibody-
29 Reticulate body Replicates in cell by fission; stained smear.
30 Reorganizes into elementary bodies. The chlamydial cell wall lacks classic
31 Chlamydia trachomatis causes reactive arthritis peptidoglycan {due to reduced muramic acid),
32 (Reiter syndrome), folli cular conjunctivitis tl, rendering ~-lacta m antibiotics ineffective.
33 • nongonococcalurcthritis, and PID.
8
Lock
s
Suspend
0
End Block
Reticulate body Replicates in cell by fission; stained smea r.
8
Reorganizes into elementary bodies. The chlamydia! cell wall lacks classic
9
10
Chlamydia trachomatis causes rcacti'c arthritis peptidoglycan (due to reduced muramic acid),
11
(Reiter syndrome), follicular conjunctivitis fJ, rendering ~-lactam antibiotics ineffective.
12 nongonococcal urethritis, and PID.
13 Chlamydophila pneumoniae and Chlamydophila
14 psittaci cause at) pi cal pneumonia; transmitted
15 by aerosoL
16 Treatment: azithromycin (fa,ored because one-
17 time treatment) or doxycycline (+ ceftria\one
18 for possible concomitant gonorrhea).
19
20
FA17 p 146.2
21
22
Chlamydia trachomatis serotypes
23 Types A, 8, a nd C Chronic infection, cause bli ndness due to AI3C = Africa, Blindness, C hronic infection.
24 follicular conjunctivitis in Africa.
25 Types D- K Urethritis/PID, ectopic pregnancy, neonatal D-K = everything else.
26
pneu monia (staccat·o cough) with eosinophi lia, eonata l disease can be acquired during
27
neonatal con junctivitis (1-2 weeks afterbirth). passage through in fected bi rth canal.
28
29 Types L1, L2, and L3 Lymphogranuloma venereum-small, painless
30 ulcers on genitals .... swollen, painful inguinal
31 lymph nodes that ulcerate (buboes). Treat with
32 doxycycline.
33 •
8
Lode.
s
Suspe-nd
8
End Bloc:k
Item: 34 of - ,• Mark -<J [:::> "'I ~ · ~
100 ~ P~v1ous N @xt Labl lues N o tes Calcula to r
8
A previously healthy 85-year- old woman presents to the emergency department with fever, chills, pleuritic chest pain, and a cough ~~AI
9
productive with rusty-colored sputum for the past 3 days. The patient lives alone in an apartment and denies recent hospitalizations,
10 travel, or sick contacts. Vital signs are notable for temperature 38°C (100.4° F), respiratory rate 20/min, and oxygen saturation 94% on
11 room air (normal : 95%-99%). Bronchial breat h sounds are heard over the right lower lobe along with dullness to percussion, egophony, and
12 increased tactile fremitus in the same area.
13
14
Which of the following is characteristic of the most likely causal organism?

15 :
A. Bacitracin sensitivity
16
17 B. Coagulase positivity
18 C. Lactose fermenting
19
D. Optochin sensitivity
20
21 E. Presence of cold agglutinins
22
23
24
25
26
27
28
29
30
31
32
33
. 34 •
8
Lode.
s
S uspe-nd
8
End Bloc:k
Item: 34 of ~ ,• Mark <:::1 t::> ""I ~· ~'j
100 J.. Previous Next LAbfaiUI~S Notes Calculator

8
9 The correct a nswer is 0. 70°/o chose t his.
10 Sensitivity t o optochin is charact eri st ic of Streptococcus pneumoniae, t he most common cause of communit y-acquired pneumonia. Suspect
pneumococca l pneumonia in a pat ient wit h lobar findings on physica l exam (dullness t o percussion, bronchial breath sounds, and egophony in
11
a foca l area), as well as lobar consolidation on chest radiograph . This pat ient lives in the community and has no comorbid condit ions or recent
12 hospit alizat ions t hat would predispose her t o hospital-acquired organ isms or pulmonary infect ions cont racted by t ravelers; t his makes
13 community-acquired pathogens most likely.
Streptococcus pneumoniae Chest radiograph Community-acquired pneumonia Pneumonia Radiography Comorbidity Streptococcus Pulmonary consolidation Physical examination Pneumococcal pneumonia
14
Bronchus Respiratory sounds Pathogen
15
16 A is not correct . 10 % chos e this .
17 Bacit racin sensit iv ity is ind icative of Streptococcus pyogenes. S. pyogenes is a common cause of pharyngit is, cellulit is, and impet igo but does
not cause lobar pneumonia.
18
Streptococcus pyogenes Bacitracin Impetigo Pharyngitis Cellulitis Pneumonia Streptococcus Lobar pneumonia
19
B is not correct. 8% chose t his .
20
Coagulase posit ivity is indicat ive of Staphylococcus such asS. aureus infection, wh ich typically manifest s as a patchy bronchopneumonia that
21 can evolve into a lobar pneumonia if the infect ion goes untreated . It is often seen in elderly or younger pat ients after a vira l infection. S .
22 aureus also can cause empyema, necrot izi ng pneumon ia, or pulmonary abscesses.
Pneumonia Coagulase Empyema Lobar pneumonia Staphylococcus Staphylococcus aureus Abscess Necrosis Viral disease Virus Necrotizing pneumonia Infection
23
24 C is not correct. 8 °/o chose this .
25 The ability t o ferment lactose is cha racterist ic of Klebsiella, a common cause of pneumonia in alcoholic and diabet ic pat ients. Ot her features of
26
Klebsiella infection include bloody "currant jelly" sput um and cavit at ions on X- ray of the chest .
Pneumonia Sputum Lactose Fermentation in food processing Chest radiograph Klebsiella X-ray Diabetes mellitus Fermentation Infection
27
28 E is not correct. 4 °/o chos e this .
29
The presence of cold agg lut inins in serum is indicative of Mycoplasma pneumoniae, t he most common cause of atypical pneumonia. At ypica l,
or "wa lking," pneumonia has an insid ious onset and is most likely to manifest in younger adults. X- ray of the chest often reveals patchy
30 infi lt rat es rather t han lobar consolidation. Pat ients also can have extrapu lmonary manifestat ions, t he most serious of which include neurologic
31 symptoms (asept ic men ingit is, peri pheral neuropat hy, and ataxia).
Mycoplasma pneumoniae Atypical pneumonia Peripheral neuropathy Meningitis Chest radiograph Ataxia Pneumonia Lung Mycoplasma Pulmonary consolidation X-ray Cold agglutinin disease
32
Agglutination (biology) Aseptic meningitis Serum (blood) Blood plasma Neurology Tuberculosis
33
34 •
8
Lock
s
Suspend
0
End Block
Item: 34 of ~ ,• Mark <:::1 t::> ""I ~· ~'j
100 J.. Previous Next LAbfaiUI~S Notes Calculator

8
9 Bottom Line:
10 Pneumococcal pneumonia is t he most common cause of community-acquired pneumonia .
Pneumonia Community-acquired pneumonia Pneumococcal pneumonia Streptococcus pneumoniae
11

12
13
14
141;fil·1i•J for year:[ 2017
FIRST AID FA CTS .
•j .

15
16 FA17 p 645.1
17 Pneumonia
18 TYPE TYPICAL ORGANISMS CHARACTERISTICS
19
Lobar S pneumoniae most frequently, also Legionella, Intra-alveolar exudate -+ consolidation r.il; may
20
21
Klebsiella rn
iJwolve entire lobe or lung.
22 Bronchopneumonia S pneumoniae, S aureus, H infiuenzae, Acute inflammatory infiltrates B1 from
23 Klebsiella bronchioles into ad jacent alveoli; patchy
24 distribution involving;;:: 1 lobe (!].
25 Interstitial (atypical) Mycoplasma, Chlamydophila pneumoniae, Diffuse patchy inflammation localized to
26 pneumonia Chlamydia psittaci, Legionella, viruses (RSV, interstitial areas at alveolar walls; diffuse
27 C tfV, influenza, adenovirus) distribution involving;;:: 1 lobe 1]. Generally
28
follows a more indolent course ("walking"
29
pneumonia).
30
31
Cryptogenic Formerly known as bronchiolitis obliterans
32 organizing organizing pneumonia (BOOP). Noninfectious
33 pneumonia pneumonia characterized by inflammation of
34 bronchioles and surrounding structure. Etiology
8
Lock
s
Suspend
0
End Block
Item: 34 of ~ ,• Mark <:::1 t::> ""I ~· ~'j
100 J.. Previous Next LAbfaiUI~S Notes Calculator

8
rheumatoid arthritis) or med ication side effects
9
(eg, amiodarone). e sputum and blood cultures,
10
no response to antibiotics.
11

12
13
14
15
16
17
18
19
20
21 FA17 p 175.2
22 Common causes of pneumonia
23
NEONATES(< 4WK) CHILDREN (4WK-18 YR) ADULTS (18-40YR) ADULTS (40-65 YR) ELDERLY
24
Group B streptococci Viruses (RSV) Mycoplasma S pneumoniae S pneumoniae
25
E coli M )1coplasma C pnewnoniae H influenzae 1nAuenza virus
26
27
C trachomatis S pneumoniae Anaerobes Anaerobes
28
(infants-3 yr) Viruses (eg, influenza) Viruses H influenzae
29 C pnewnoniae Mycoplasma C ram 8 rods
30 (school-aged
31 children)
32 S pneumoniae
33 Runts May Cough
34 • C hunkv Soutum
8
Lock
s
Suspend
0
End Block
Item: 34 of - ,• Mark -<J [:::> "'I ~ · ~
100 ~ P~v1ous N @xt Labl lues N o tes Calcula to r
8 A
Special groups A

9
10
Alcoholic Klebsiella, anaerobes usually due to aspiration (eg, Peptostreptococcus, Fusobacterium, Prevotella,
11
Bacteroides)
12 IV drug users S pneumoniae, S aureus
13
Aspiration naerobes
14

15
Atypical i'vlycoplasma, Legioneffa, Chlamydia
16 Cystic fibrosis Pseudomonas, S aureus, S pneumoniae, Burkholderia cepacia
17
lmmunocompromised S aureus, enteric gram 8 rods, fungi , viruses, P iirol•ecii (with Ill\/)
18
19 Nosocomial (hospital S aureus, Pseudomonas, other enteric gram 8 rods
20 acquired)
21 Postviral S pneumoniae, S aureus, 11 influen:we
22
23
24 FA17 p 132.2

25 Streptococcus Gram $, lancet-shaped diplococci rJ. Pneumococcus is associated with "rusty"


26 pneumoniae sputum, sepsis in patients with sickle cell
Encapsulated. IgA protease. Optochin
27
sensitive. Most common cause of: disease, and asplenic patients.
28 rJ " Meningitis o viru lence without capsu le.
29 ,
• Otitis media (in children)
30
Bacterial pneumonia
31
• Sinusitis
32
33
·t' . . .. '

-
'-,
34 .J _ """'
8
Lode.
s
S uspe-nd
8
End Bloc:k
Item: 35 of - ,• Mark -<J [:::> "'I ~ · ~
100 ~ P~v1ous N @xt Labl lues N o tes Calculato r
9
A 36-year-old man comes to the physician because he is experiencing abdominal pain, vomiting, and nonbloody diarrh ea. He last ate ~~AI
10
chicken and rice about 4 hours ago at a Chinese restaurant. He has no other symptoms.
11

12
Which is the appropriate treatment for this patient?
13
:
14
A. Prescribe an antibiotic that induces DNA damage in anaerobes
15
16
B. Prescribe an antibiotic that inhibits 305 ribosome
17 C. Prescribe an antibiotic that inhibits protein translation by the 505 ribosome
18
D . Prescribe an antibiotic that inhibits topoisomerase II
19
E. Prescribe no antibiotic therapy and provide supportive procedures
20
21
22
23
24
25
26
27
28
29
30
31
32
33
34
. 35 •
8
Lode.
s
S uspe-nd
8
End Bloc:k
Item: 3S of ~ ,• Mark <::J [:::> ""I ~· ~'j
100 J.. Previous Next LAbfaiUI~S Notes Calculator

9
The correct answer is E. 57°/o chose this.
10
The most likely cause of food poisoning in this case is t he exot oxin f rom Bacillus cereus . These exot oxins are fast acting, so the sympt oms of
11 food poisoning (nausea, vomiting, diarrhea) are usually rapid in onset (within 4-8 hours of ingestion). Because t he disease is toxin mediated,
12 ant ibiot ics are not effective and only supportive care, including oral rehydration wit h electrolyte and glucose replen ishment, is recommended.
Addit ion of glucose t o an elect ro lyt e-wat er solut ion t riggers act ive cot ransport of sod ium and glucose across t he intest ina l epit helium t hrough
13
SGLT t ransporters, helping rest ore nut ri ents and water lost t hrough vom it ing and diarrhea . Consum ing undercooked chicken could lead t o
14 infection with Salmonella, but sym pt oms of food poisoning would take approximat ely 12- 72 hours to develop, not ably slower t han wit h B.
15 cereus .
Bacillus cereus Exotoxin Diarrhea Electrolyte Foodborne illness Glucose Vomiting Salmonella Nausea Epithelium Bacillus Antibiotics Sodium Toxin Infection Chicken
16
17 A is not correct . 8°/o chos e this.
18 Met ron idazole is used to t reat Helicobacter pylori infection in combinat ion wit h bismuth subsa licylate and amoxicillin. The mechanism of action
of metronidazole induces DNA damage in anaerobes.
19
Metronidazole Amoxicillin Helicobacter pylori Bismuth subsalicylate Bismuth Anaerobic organism Helicobacter DNA Infection
20
B is not correct. 11 % chose this .
21
This describes the mechanism of act ion of t et racyclines, which can be used to t reat Mycoplasma pneumoniae, Rickettsia rickettsii, Chlamydia
22
species, and Borrelia burgdorferi.
23 Mycoplasma pneumoniae Rickettsia rickettsii Borrelia burgdorferi Rickettsia Chlamydia infection Chlamydia (genus) Tetracycline antibiotics Mycoplasma Borrelia

24
C is not correct. 10% chos e this.
25 By binding to the 50S ribosomal subunit, erythromyc in inhibits the prot ein synt hesis and replication of bacteria . Eryt hromycin can be used to
26 t reat Campylobacter jejuni enterocolit is.
Erythromycin Campylobacter jejuni Campylobacter Protein 50S Bacteria Ribosome Protein biosynthesis Protein synthesis
27
28 D is not correct. 14% chose this .
29 Fluoroquinolones exhibit t heir ant ibacteria l effects by inhibit ing topoisomerase II ligase domain. Fluroquinolones can be used to t reat severe
30 Shigella infection, which causes bloody diarrh ea.
Shigella Diarrhea Quinolone Type II topoisomerase Topoisomerase Antibiotics
31
32
33 Bottom Line:
34 Bacillus cereus, oft en found in reheated rice, produces an exot oxin t hat can rapidly cause abdominal pain, vom it ing, and nonbloody diarrhea .
35
I nfection is best t reat ed with support ive care.

8
Lock
s
Suspend
0
End Block
Item: 35 of - ,• Mark -<J [:::> "'I ~ · ~
100 ~ P~v1ous N @xt Labl lues N o tes Calculato r
9 A A

FA17 p 174.3
10
11 Bugs causing food- S aureus and B cereus food poisoning starts quickly and ends quickly.
12 borne illness
MICROORGANISM SOURCE OF INFECTION
13
14
B cereus Reheated rice. "Food poisoning from reheated
15
rice? Be \eriom!"' (B cereus)
16 C botulinum Improperly canned foods (toxins), raw honey
17 {spores)
18
C perfringens Reheated meat
19

20 E coli 0157:117 Undercooked meat


21 L monocytogenes Deli meats, soft cheeses
22
Salmonella Poultry, meat, and eggs
23
24 S aureus !eats, mayonnaise, custard; preformed toxin
25 V parahaemolyticus and V vulnificus-• Contaminated seafood
26
"V vulnificus can also cause wound infect ions from contact with contam inated water or shell fish.
27
28
29 FA17p 134.1
30
Bacillus cereus Gram EB rod. Causes food poisoning. Reheated rice syndrome.
31
Spores survive cooking rice. Keeping rice
32
warm results in germination of spores and
33
enterotoxin formation.
34
35
Emetic type usually seen with rice and pasta.

8
Lode.
s
S uspe-nd
8
End Bloc:k
Item: 36 of - ,• Mark -<J [:::> "'I ~ · ~
100 ~ P~v1ous N @xt Labl lues N o tes Calcula to r

A A 37-year-old man with a history of HIV infection is brought to t he emergency department wit h altered mental st at us. On physical [.AA] A

25 examination, he exhibits hemiparesis, dysphasia, aphasia and ataxia. He is admitted to the hospital and experiences multiple focal seizures
26 during his stay. An MRI shows high signal in the temporal lobes and hippocampus. His CD4 count is 78 cells/IJ L (normal : 500-1500 cells/
27
IJL). Polymerase chain reaction test ing of his cerebrospinal fluid is completed, and a transmission electron microg raph of the pathogen is shown
in the image.
28
29
30
31
32
33
34
35
' 36
. 37

' 38
• 39
• 40

' 41
• 42
• 43
• 44 During this latency period, which v ira l component can be detected in infected neurons?
' 45 :
' 46 A. Double-stranded DNA
' 47
B. Latency-associated proteins
. 48
. 49 C. Single-stranded DNA
. 50 D. Viral nucleocapsids
8
Lode.
s
S uspe-nd
8
End Bloc:k
Item: 36 of - ,• Mark -<J [:::> "'I ~ · ~
100 ~ P~v1ous N @xt Labl lues N o tes Calcula to r

A du nng h1s stay. An MRI shows h1gh s1gnal m the temp01allobes and hippocampus. H1s CD4 count 1s 78 cells/IJ L (normal : 500-1500 cells/ A

25 IJL). Polymerase chain reaction test ing of his cerebrospinal fluid is completed, and a transmission electron micrograph of the pathogen is shown
26 in the image.
27
28
29
30
31
32
33
34
35
' 36
. 37
. 38
• 39
• 40

' 41
• 42
During this latency period, which v iral component can be det ected in infected neurons?
• 43
:
• 44
A. Double-stranded DNA
' 45
' 46
B. Latency-associated proteins
' 47 C. Single-stranded DNA
. 48
D. Viral nucleocapsids
. 49
. 50
E. Viral reverse transcriptase

8
Lode.
s
S uspe-nd
8
End Bloc:k
Item: 36 of ~ ,• Mark <::J [:::> ""I ~· ~'j
100 J.. Previous Next LAbfaiUI~S Notes Calculator

25
26 The correct ans wer is A. 49°/o chose this.
27 Herpes simplex virus t ype 1 (HSV- 1) is a double-s tranded DNA virus which can cause temporal lobe encephalitis in
28 immunocompromised pat ients. This is the likely explanation in this patient wit h HIV, low CD4 count , t em poral lobe changes on MRI , and
neurologic manifestations. I nit ial infection with HSV- 1 occurs at skin and mucous membranes in the oropharynx or genit als. During t his init ial
29
stage, t he virus act ively replicates and produces vesicular skin lesions t hat contain nu merous viral particles. Following t he primary infection,
30 t he HSV- 1 virus may ent er a lat ent peri od, lying dormant in the t rigeminal ganglion. During t he lat ent phase, v iral genetic material is
31 preserved as an episome: circular, double-st randed DNA (shown in t he image) which is not yet integ rat ed into the host chromosome.
Subsequently, t he virus may be reactivat ed by various st imuli including st ress, menstruat ion, and exposure t o ult raviolet light. Upon
32
reactivation, viral prot eins are synthesized and a secondary infection can occur.
33 Herpes simplex virus Trigeminal ganglion DNA virus Episome Pharynx Immunodeficiency Temporal lobe CD4 Encephalitis Herpes simplex Ultraviolet HIV Incubation period Infection Menstruation
34 Mucous membrane Virus Neurology Magnetic resonance imaging Chromosome DNA Ganglion Vesicle (biology and chemistry) Neurological disorder Protein Genome Sex organ
35
B is not correct. 16% chose this .
36
Latency-associat ed mRNAs (single-stranded) are produced by herpes simplex v irus type 1 (HSV- 1) during the latent phase in order t o evade
. 37 t he host's immune response. However, t hese mRNAs are not t ranslat ed into prot ein product s during the latency peri od .
. 38 Herpes simplex virus Protein Herpes simplex Incubation period Immune system Virus

• 39 C is not correct. 11% chos e this .


• 40 HSV- 1 is a double-st randed DNA virus. It produces single-stranded m RNA during lat ency, which can be detect ed wit h real-time polymerase
• 41 chain reactio n (RT-PCR). It does not produce single-st randed DNA, which would be found in the parvovirus.
DNA virus Polymerase chain reaction Real-time polymerase chain reaction DNA Messenger RNA Virus Herpes simplex virus Parvovirus Herpes simplex Polymerase
• 42
. 43 0 is not correct. 13% chose this .
• 44 Viral nucleocapsids are not produced during t he lat ent period .
Virus Viral video
• 45
• 46 E is not correct. 11°/o chose this .
• 47 Viral reverse t ranscriptase is not associat ed with HSV- 1 or HSV- 2, but rather HIV and human T-cell leukemia virus t ype 1.
Human T-lymphotropic virus 1 Reverse transcriptase Leukemia HIV T cell Herpes simplex virus Herpes simplex Human T-lymphotropic virus Virus
• 48
. 49
• 50 Bottom Line:
8
Lock Suspend
s 0
End Block
Item: 36 of ~ ,• Mark <::J [:::> ""I ~· ~'j
100 J.. Previous Next LAb faiUI~S Notes Calculator

25 Bottom Line:
26 After primary infect ion wit h the double-stranded DNA virus HSV-1, t he virus ent ers a latent phase charact eri zed by the creat ion of an
27 episome. React ivat ion of the virus in an immunocompromised host can lead to t emporal lobe encephalit is.
Episome DNA virus Encephalitis Herpes simplex Herpes simplex virus Immunodeficiency Temporal lobe Virus DNA Virus latency Infection
28
29
30
31
141;fil·1i•J
FIRS T AID FACTS
for year:[ 2017
.
•j .

32
33 FA17 p 160.2
34
Herpesviruses Enveloped, OS, and linear viruses
35
36 VIRUS ROUTE OF TRANSMISSION CLI NICAL SIGNIFICANCE NOTES
. 37 Herpes Respiratory Gingivostomatitis, keratoconjunctivitis fJ, Most common cause of sporadic
. 38 simplex secretions, saliva herpes labialis m , herpetic whitlow on finger, encephalitis, can present as altered
• 39 virus-1 temporal lobe encephalitis, esophagitis, mental status, seizures, and/or
• 40 erythema mu lti forme. aphasia .
• 41
Herpes Sexual contact, Herpes genital i s ~. neonatal herpes. Latent in sacral ganglia. Vira l
• 42
simplex perinatal meningitis more common with
. 43
virus-2 HSV-2 than with HSV-1.
• 44
• 45 Varicella- Respiratory Varicella-zoster (chickenpox [!], shingles 0}, Latent in dorsal root or trigeminal
• 46 Zoster virus secretions encephalitis, pneumonia. ganglia; C V1 bra nch
• 47 (HHV-3) Most common complication of shingles is post- involvement can cause herpes
• 48 herpetic neuralgia. zoster ophthalmicus .
. 49
Epstein-Barr Respiratory Mononucleosis- fever, hepatosplenomegaly, Infects B cells through CD21.
• 50 ... 1 1 o1 I " 11 A o I 1 • 1

8
Lock
s
Suspend
0
End Block
Item: 36 of - ,• Mark -<J [:::> "'I ~ · ~
100 ~ P~v1ous N @xt Labl lues N o tes Calcula to r
A A

Epstein-Barr Respiratory Mononucleosis-fever, hepatosplenomegaly, Infects B cells through CD2l.


25
26
virus (HHV-4) secret ions, pharyngitis, and lymphadenopathy (especially typical lymphocytes on peripheral
27
sali\·a; aka posterior cervical nodes 0). Avoid contact sports blood smear [!~-not infected B
28 "kissing disease," until resolution due to risk of splenic rupture. cells but reactive cvtotoxic T cells.
'
29 (common in Associated with lymphomas (eg, endemic Ei1 Monospot test-heterophile
30 teens, young Burkitt lymphoma), nasopharyngcHI antibodies detected by agglutination
31 adults) carcinoma (especially sian adults), of sheep or horse RBCs.
32 lymphoproliferati\'e disease in transplant Use of amoxicillin in mononucleosis
33 patients. can cause characteristic
34 maculopapular rash.
35
Cytomegalo- Congenital Ylononucleosis (8 Vlonospot) in Infected cells have characteristic
36 virus (HHV-5) tnmsfusion, immunocompetent patients; infection in "owl eye" inclusions CJ.
. 37
sexual contact, immunocompromised, especially pneumonia Latent in mononuclear cells.
0
38
sali\·a, urine, in transplant patients; esophagitis; AI OS
• 39
transplant retinitis ("sightomega lovirus"): hemorrhage,
• 40
cotton-wool exudates, vision loss .
• 41
Congenital CM
• 42
0
43 Human Saliva Roseola infantum (exanthem subitum): high Roseola: b ·er first, Rosie (checks)
• 44 herpes- fevers for several days that can cause seizures, later.
• 45 viruses 6 foll owed by diffuse macular rash 0 . III-I V-7 - less common cause of
• 46 and 7 roseola.
• 47
Human Sextml contact Kaposi sarcoma (neoplasm of endothelial cells). Can also affect Cl tract and lungs.
48
herpesvirus Seen in HTV/AIDS and transplant patients.
0

. 49
8 Dark/violaceous plaques or nodules 0
0 50 - - - - - - - - .._:- ~ - • - - -- I • - - - ... I · r ... - ... . _ · ... __
8
Lode.
s
S uspe-nd
8
End Bloc:k
Item: 36 of ~ ,• Mark <::J [:::> ""I ~· ~'j
100 J.. Previous Next LAbfaiUI~S Notes Calculator

25
26
27
28
29
30
31
32
33
34
35
36
. 37
. 38
• 39
• 40
• 41 FA17 p 159.5
• 42
. 43
DNA virus Some general rules-all DNA viruses:
characteristics
• 44 GENERAL RULE COMMENTS
• 45
Are HHAPPPPy viruses Hepadna, Herpes, Adeno, Pox, Parvo,
• 46
Papilloma, Polyoma .
• 47
. 48
Are double stranded Except pan·o (single stranded).
. 49 Have linear genomes Except papilloma and polyoma (circular,
• 50 • suoercoiled) and heoadna (circular.
8
Lock
s
Suspend
0
End Block
Item: 37 of - ,• Mark -<J
P~v1ous
[:::> "'I ~ ·· ~
100 ~ N @xt Labl lues N o tes Calcula to r

I n 1918-1919, an influenza virus was responsible for t he deaths of 20-40 million people worldwide; t his is more people t han were killed ~~AI
25
during World War I, and more t han were killed during 4 years of t he bubonic plague.
26
27
Which of fol lowing characteristics of the infl uenza virus makes this kind of pandemic possible?
28
:
29
A. It is a negative-stranded virus
30
31
B. It is a segmented virus
32 C. It is an orthomyxovirus
33
0. It is an RNA virus
34
35
E. RNA polymerase has no proofreading capability
36
. 37
. 38
• 39
• 40
• 41
• 42
. 43
• 44
• 45
• 46
• 47
. 48
. 49
. 50

8
Lode.
s
S uspe-nd
8
End Bloc:k
Item: 37 of ~ ,• Mark <::J [:::> ""I ~· ~'j
100 J.. Previous Next LAbfaiUI~S Notes Calculator

The correct a nswer 1s B. 65% chose t h1s .


25
Compared with an epidemic, a pandem ic refers t o t he spread of an infect ious disease t hat affects a much larger popu lation and in a much
26 larger reg ion . Influenza pandemics are t he result of genetic/ant igen ic shift, a phenomenon t hat occurs when viruses with segmented genomes
27 (eg, influenza virus) exchange segments, resu lt ing in high-frequency t ransfer of genet ic information, particularly t he hemagg lut inin and
neuraminidase ant igens. The result is a new virus with a combinat ion of surface antigens that has never been exposed to a human immune
28
system anywhere on the planet. Thus, t he entire human populat ion would be susceptible, leading t o a pandem ic.
29 Neuraminidase Hemagglutinin Hemagglutinin (influenza) Viral neuraminidase Influenza Immune system Infection Pandemic Virus Antigen Orthomyxoviridae Epidemic

30
A is not correct . 3°/o chose this.
31 Inf luenza is a negat ive-stranded RNA v irus, but this property is not a feat ure t hat resu lts in pandem ics. Negat ive-stranded v iruses must be
32 t ranscri bed to a posit ive strand using RNA polymerase.
RNA virus RNA Influenza RNA polymerase Virus Pandemic Polymerase
33
34 C is not correct. 10% chose this .
35 Inf luenza v iruses are ort homyxoviruses. This is not a feat ure specif ic fo r genet ic sh ift.
Orthomyxoviridae Antigenic shift Influenza Virus Influenza vaccine
36
37 D is not correct. 5°/o chose t his.
. 38 Inf luenza v iruses are RNA viruses. RNA v iruses can be single stranded (ssRNA) or double stranded (dsRNA). Single-st randed RNA viruses are
• 39
eit her posit ive stranded or negative stranded . This characteristic does not confer an advantage t hat makes pandem ics more likely.
Influenza RNA Virus Pandemic RNA virus OsRNA Orthomyxoviridae
• 40
• 41 E is not correct. 1 7°/o chose this .
• 42
The lack of proofreading act ivity of RNA polymerase leads t o an increased chance fo r m utations duri ng the replicat ion process. However,
mutat ions during replication lead to on ly m inor changes in t he hemagglut inin and neu ram inidase glycoproteins. This phenomenon, known as
. 43 antigenic drift, leads to new strains of t he inf luenza v irus that are partially at tacked by our immune syst em, result ing in m ilder disease in
• 44 adults who have previously acquired antibodies.
Antigenic drift Neuraminidase Hemagglutinin Hemagglutinin (influenza) Viral neuraminidase Immune system Proofreading (biology) Antibody RNA polymerase Influenza Glycoprotein Proofreading RNA
• 45
Orthomyxoviridae Virus Polymerase Mutation
• 46
• 47
. 48 Bottom Line :
. 49 Genet ic (ant igenic) sh ift results f rom re-assort ment of the segments of a v iral genome and causes influenza pandemics. Genetic (antigenic)
• 50 • dri ft causes influenza epidemics and is the resu lt of random m utations. •

8
Lock
s
Suspend
0
End Block
Item: 37 of - ,• Mark -<J
P~v1ous
[:::> "'I ~ ·· ~
100 ~ N @xt Labl lues N o tes Calcula to r


25
FA17 p 165.1
26 Influenza viruses Orthomyxo,·iruses. Em·eloped, 8 ssR 'A Reformulated vaccine ("the Au shot") contains
27 viruses with 8-segment genome. Contain nu
viral strains most likely to appear during the
28 season, due to the virus' rapid genetic change.
hemagglutinin (binds sialic acid and promotes
29
viral entry) and neuraminidase (promotes Killed viral ,·accine is most frequent I) used.
30
progeny \'irion release) antigens. Patients at Li\'e attenuated ,·accine contains temperature·
31
risk ror ratal bacterial superinrection, most sensitive mutantlhat replicates in the nose but
32
commonly S aureus, S pneumoniae, and not in the lung; administered intra nasally.
33
II influen;;ae.
34
35 Genetic Causes pandemics. Rcassortmcnt or \'ira I Sudden shift is more deadlr than gradual drift.
36 shift/antigenic shift genome segments, such as when segments of
37
•• human Au A virus reassort with swine Au
0
38 ••
••• VlfUS.
• 39 • •
••••
• 40 ReassOt'tment !
• 41 ••
• 42 ••
0
43 Genetic drift/ Causes epidem ics. ifinor (antigenic drift)
• 44 changes based on random mutation in
antigenic drift
• 45
•• hemagglutinin or neura minidase genes.
• 46 ••••
Random
mutatiOns- - l
• 47
0
48
• ·~.
0
49 ~ • 0
0 50

8
Lode.
s
S uspe-nd
8
End Bloc:k
Item: 38 of - ,• Mark -<J [:::> "'I ~ · ~
100 ~ P~v1ous N @xt Labl lues N o tes Calcula to r

A 57-year-old woman comes to the physician 6 weeks after returning from a trip to Greece. She has had a fever of 38.8°C (101.8°F) that ~~AI
25
rises during the day and decreases at night. She says that she feels tired and has lost weight. She mentions she enjoyed her vacation and
26
trying the local specialt ies, including fresh goat cheese. Her physical examination is notable for hepatosplenomegaly and generalized
27 lymphadenopathy.
28
29 Which of the following organisms is most likely responsible for this patient's symptoms?
30 :
31 A. Borrelia recurrentis
32 B. Bartonella henselae
33
C. Brucella melitensis
34
35 D. Pasteurella multocida
36 E. Plasmodium malariae
37
0
38
• 39
• 40
• 41
• 42
0
43
• 44
• 45
• 46
• 47
0
48
0
49
50
0

8
Lode.
s
S uspe-nd
8
End Bloc:k
Item: 38 of ~ ,• Mark <::J [:::> ""I ~· ~'j
100 J.. Previous Next LAb faiUI~S Notes Calculator

25 The correct a ns wer is C. 53°/o chos e this .


26 This patient is exhibit ing signs and sym pt oms of undulant feve r~ caused by infection with Brucella species. Brucella melitensis enters the body
27 after t he ingestion of cont aminated m ilk products or direct contact wit h contam inat ed livest ock. It is an int racellular bact erium t hat causes
undulat ing feve r~ weakness, and loss of appet it e. Brucella are small gram-negat ive coccobacilli t hat are largely int racellu lar. Li ke Nocardia,
28
t hey stain wit h a modif ied acid-fast stain.
29 Brucella melitensis Coccobacillus Gram-negative bacteria Brucella Nocardia Brucellosis Acid-fast Fever Bacteria Milk Anorexia (symptom) Infection Intracellular

30
A is not correct . 11% chos e this .
31
Borrelia recurrentis, a spirochet e, is spread between humans by a body louse . I nfection may cause relapsing feve r.
32 Spirochaete Relapsing fever Body louse Borrelia recurrentis Borrelia Louse Fever

33
B is not correct. 9 % chose t his .
34
Bartonella henselae is the causat ive agent of cat scratch disease and may also lead to bacillary angiomat osis. Like Brucella, Ba1tonella
35 infection can lead to lymphadenopat hy and feve r, but exposure to unpast eurized milk in t his case strongly point s Brucella as t he causa l agent.
Bacillary angiomatosis Cat-scratch disease Bartonella henselae Bartonella Lymphadenopathy Brucella Angiomatosis Fever Cat
36
37 D is not correct. 20% c hose this .
38 Pasteurella infection commonly result s from dog or cat bi te wounds and typically causes a cellulit is. High fever, hepatosplenomegaly, and
• 39 exposure t o unpasteuri zed m ilk products are not generally associated wit h infection .
Pasteurella Hepatosplenomegaly Fever Infection Cellulitis Pasteurization
• 40
• 41 E is not correct. 7 °/o chos e this .
• 42
Plasmodium malariae is a prot ozoan t hat causes ma laria. This disease is charact erized by episodes of fever and chills every 72 hours. It is
spread through t he bite of the Anopheles mosquito. While splenomega ly is often present, lym phadenopathy is not. This patient 's history also
. 43 ind icates that he has not been t o an area endem ic for malari a.
• 44 Splenomegaly Plasmodium malariae Endemism Lymphadenopathy Anopheles Malaria Mosquito Protozoa Plasmodium Fever Chills

• 45
• 46
Bottom Line :
• 47
Brucella infect ion is t ransmitted via dairy products and cont aminated livestock, and can result in a fever t hat spikes and disappears in a
• 48 single day.
. 49 Brucella Fever Infection Livestock Dairy

• 50

8
Lock
s
Suspend
0
End Block
Item: 38 of ~ ,• Mark <::J [:::> ""I ~· ~'j
100 J.. Previous Next faiUI~S
LAb Notes Calculator

25 FA17 p 144.1
26 Zoonotic bacteria Zoonosis: infectious disease transmitted between animals and humans.
27
SPECIES DISEASE TRANSMISSION AND SOURCE
28
29 Anaplasma spp. Anaplasmosis Ixodes ticks (live on deer and mice)
30 Bartonella spp. Cat scratch disease, bacilla ry angiomatosis Cat scra tch
31
Borrelia burgdorferi Lyme disease Ixodes ticks (live on deer and mice)
32
33 Borrelia recurrentis Relapsing fever Louse (recurrent due to variable surface
34 antigens)
35
Brucella spp. Brucellosis/undulant fever Unpasteurized dairy
36
37
Campylobacter Bloody diarrhea Feces from infected pets/animals; contaminated
38
meats/foods/hands
• 39 Chlamydophila psittaci Psittacosis Parrots, other birds
• 40
Coxiella burnetii Q fever Aerosols of cattle/sheep am niotic Auid
• 41

• 42 Ehrlichia chaffeensis Ehrlich iosis Amblyomma (Lone Star tick)


. 43 Francisella tularensis Tularemia Ticks, rabbits, deer Aies
• 44
Leptospira spp. Leptospirosis Animal urine in \\·ater; recreational water use
• 45
• 46 Mycobacterium leprae Leprosy ll umans with lepromatous leprosy; armadillo
• 47 (rare)
• 48 Pasteurella multocida Cellulitis, osteomyelitis Animal bite, cats, dogs
. 49
Rickettsia prowazekii Epidemic typhus Human to human vi<l human body louse
• 50 •
8
Lock
s
Suspend
0
End Block
Item: 39 of - ,• Mark -<J [:::> "'I ~ · ~
100 ~ P~v1ous N @xt Labl lues N o tes Calculato r

25 A mother brings her 1-year-old daughter t o t he emergency departent. She says that for the last 2 days her daught er has been fussy and
crying more than usual. She also refuses formu la. The patient has a fever of 39.4°C (102.9°F). Meningitis is suspected, and a lumbar
26
puncture is performed. Analysis of the cerebrospinal flu id shows an opening pressure of 187 mm H2 0, a WBC count of 1256/mm3, a
27 protein level of 210 mg/dL, and a glucose level of 31 mg/dl. The mother says that the "family has no healt h insurance and that the baby has not
28 seen a physician since birth.
29
30
What organism is most likely responsible for t his pat ient's illness?
31 :

32
A. Clostridium botulinum
33 B. Group B streptococci
34
C. Haemophilus influenzae
35
D. Herpes simplex virus
36
37 E. Listeria monocytogenes
38
• 39
• 40
• 41
• 42
. 43
• 44
• 45
• 46
• 47
. 48
. 49
. 50 •
8
Lode.
s
S uspe-nd
8
End Bloc:k
Item: 39 of - ,• Mark -<] 1:> ""'I ~· 1!';:'1
100 ~ Prevto u s Next Labf a lu es Notes Calculator

25 The correct answer is C. 5 2 °/o chose this .


26 The cerebrospinal fluid findings (decreased glucose level, increased WBC count, elevated protein level, and
Most common bacterial c auses of
opening pressure > 180 mm H2 0) indicate meningitis of bacterial origin. One of the most common causes of mening itis by age
27
meningitis in the 1-year-old age group is Haemophilus influenzae, especially in the unimmunized
Age Most Important Bacteria
28 population. The incidence of H. influenzae meningit is has greatly declined with the development of a highly
Group B sorepeococcl
29 effective conjugate vaccine. The other common causes in this age group include Streptococcus pneumoniae, Neonate E:whericllla COli
Neisseria meningitidis, and the enteroviruses. The t able indicates other causes of meningitis by age.
30
Ne er•a P'C..n 1Qibd• Strer'ltOCOCCUS pneumoniae Cerebrospinal fluid Haemoph•lus inftuenzae Men no•t• Strep~ococc ·~ Net~~erla Enterovirus Protein
Lislona .
Ctlildren SlrepiOOOCCu$ pneumon/<te
31
ConJugate VKCme Glucose vaccme Invnunizatlon Haernophilus
32
Youoga<lults Nei$$erKI ''*Ji~

Adul5 Slreptoooccut pneumonia.


33
34
A is not correct. 2°/o chose this.
35
Clostridium botulinum causes botulism. Infants may initially become constipated and then develop generalized muscle weakness ("floppy
36 baby"). However, this child 's symptoms are not consistent with botulism.
37 Clostnd1um botulinum Botulism Clostndium Botulinum toxin Constipation Muscle weakness

38 B is not correct. 27°/o chose this .


39 Group B streptococci are a common cause of meningitis in children younger than 3 months of age. It is much less common in children > 6
• 40 months old. The early-onset form can be cont racted fro m colonization of the mother's vagina or rect um, but antibiot ics during labor
• 41 significantly reduce infection rat es fro m group B st rep .
Men•no•tls Rectum Streptococcus agalactiae Streptococcus Antibiotics Vagina Infection
• 42
. 43 D is not correct . 4 °/o chose this .
Herpes simplex virus has been shown t o cause meningit is in infan ts but it is not a common source . In vira l meningit is, t he cerebrospinal fluid
• 44
f indings would show far fewer WBCs ( 11 - 500/mm 3 ), protein levels between SO and 200 mg/dL, and normal glucose levels; a Gram stain
• 45 would not show any organisms.
• 46 Gram stammo Cerebrospinal fluid Viral meningitiS Herpes simplex virus Memng1tls Herpes s1mplex Protem Glucose Virus

• 47 E i s not correct. 1 5°/o chose t h is.


. 48 Listeria monocytogenes is a common cause of meningit is in newborns and in older adults ( >60 years old ) but not in the 1-year-old age group.
. 49 l.Jstena rnonocvtooener Men1ng1t1s LJstena

• 50 •
8
L.odt
s
Su~pl'nd
~
End Block
Item: 39 of - ,• Mark -<J [:::> "'I ~ · ~
100 ~ P~v1ous N @xt Labl lues N o tes Calculato r
A A

25
Bottom Line:
26 A common cause of bacterial meningit is in unimmunized 1-year-old children is Haemophilus influenzae. Cerebrospinal fluid analysis in such
cases would show decreased glucose levels, increased WBC counts, elevated protein levels, and increased opening pressure.
27 Cerebrosp1n1'tl flu1d Haemoph1lu~ mfluenzae Memng1tis Bacterial meningitis Protem Glucose Immun17~1on Hi'ternoph1lus
28
29
30 14l ifil·1i•J for year: 2017 •
FI RST AID FAC"'S
31
32
FA17 p 138.3
33
34 Haemophilus Small gram 8 (coccobacillary) rod. Aerosol accine contains type b capsular polysaccharide
35 influenzae transmission. 'ontypeable (unencapsulated) (pol} ribos} Iribitol phosphate) conjugated
36 strains are the most common cause of mucosal to diphtheria toxoid or other protein. Given
37 infections (otitis media, conjuncti' itis, between 2 and 18 months of age.
38 bronchitis) as well as invasive infections since Does not cause the fl u (influenza virus does).
39
the vaccine for capsular type b was int roduced .
• 40
Produces lgA protease. Culture on chocolate
• 41
agar, which conta ins fa ctors V ( AD+) and X
• 42
(hematin) for growth; can also be grown with
. 43
• 44
S aureus, wh ich provides factor V through the
• 45
hemolysis of RBCs. HaEJ\10Philus causes
• 46 Epiglottitis (endoscopic appearance in fJ.
• 47 can be "cherry red" in ch ildren; "thumb sign"
. 48 on x-ray III), ~len in giti s, Otitis media, and
. 49 Pneumonia .
. 50 • Treatment: amoxicillin +1- clandanate for
8
Lode.
s
S uspe-nd
8
End Bloc:k
Item: 39 of ~ ,• Mark <::J [:::> ""I ~· ~'j
100 J.. Previous Next LAbfaiUI~S Notes Calculator

25
26
FA17 p 176.1
27
Common causes of meningitis
28
NEWBORN (0- 6MOl CHILDREN (6 M0- 6 YR) 6- 60YR 60YR+
29
30 Group B streptococci S pneumoniae S pneumoniae S pneumoniae
31 E coli N meningitidis N meningiticlis (f/ 1 in teens) Gram 8 rods
32 Listeria H influenzae type B Enterovi ruses Listeria
33 Enteroviruses HSV
34 Give ceftri axone and vancomycin empirically (add ampicill in if Listeria is suspected).
35 Viral causes of meningitis: enteroviruses (especially coxsackievirus), HSV-2 (HSV-1 = encephalitis), HlV, West ile virus (also
36
causes encephalitis), VZV.
37
In HIV: Cryptococcus spp.
38
ote: Incidence of H influenzae meningitis has l greatly due to conjug<ltc H influenzae vaccinations. Today, cases are usually
39
seen in unimmunized children .
• 40
• 41
• 42 FA17 p 176.2
. 43 CSF findings in meningitis
• 44
OPENING PRESSURE CEl l TYPE PROTEIN GLUCOSE
• 45
Bacterial t t P II s t l
• 46
• 47 Fungai/TB t t lymphocytes t l
. 48 Viral orma 1/t t lymphocytes Norma lit Normal
. 49
• 50 •
8
Lock
s
Suspend
0
End Block
Item: 40 of - ,• Mark -<J [:::> "'I ~· ~
100 ~ P~v1ous N @xt Labl lues N o tes Calcula to r

A 45-year-old man who recently emigrated f rom I ndia presents with a low-grade fever, hoarseness, cough, and a sore throat. On physical ~~AI
25
examination, he has swollen cervical lymph nodes. Gram stain of his sputum is obtained, and the specimen is shown in the image.
26
Secretions obtained from the patient's tonsils are analyzed by polymerase chain reaction. The presence of a specific gene encoding an
27 exotoxin is found.
28
29
30 I

f
31
32
' -

33
34 ' J '
35
36 , ..
- t
37 • •
J
38
39
--
Image courtesy of CDC/ Dr. P.B. Smith
'
• 40
• 41
The exotoxin respons ible for his symptoms inhibits protein synt hesis by which of the fol lowing mechanisms?
• 42
. 43 :
A. Activation of adenyl cyclase by adenosine diphosphat e ribosy lation
• 44
• 45 B. Activation of Gs
• 46 C. Adenosine diphosphate ribosylation of elongation factor 2
• 47
D. Inhibition of G;
. 48
. 49 E. Stimulating macrophages t o release tumor necrosis factor-a
. 50 •
8
Lode.
s
Suspe-nd
8
End Bloc:k
Item: 40 of ~ ,• Mark <:::1 t::> ""I ~· ~'j
100 J.. Previous Next LAb faiUI~S Notes Calculator

25
26
The correct a nswer is C. 6 4 °/o chose this .
27
Dipht heria should be considered in pat ients wit h pharyngitis, cervical adenopat hy, and feve r. The Gram-stained sputum specimen shows
28 Corynebacterium diphtheriae, which is a gram-positive, rod-shaped bacterium. This pat ient has classic symptoms of dipht heria, which is
29 caused by C. diphtheriae via an exotoxin t hat is encoded by t he B prophage. The exot oxin inhibits protein synt hesis by adenosine diphosphat e
(ADP) ribosylat ion of elongat ion fact or 2, prevent ing it f rom t ranslocat ing t RNA from A sit e to P site. The mechanism of diphtheria exot oxin is
30
identical to that of Pseudomonas exotoxin A.
31 Prophage Corynebacterium diphtheriae Diphtheria Pharyngitis Lymphadenopathy Gram-positive bacteria Exotoxin Sputum Transfer RNA Protein biosynthesis Gram staining Protein Pseudomonas Bacteria

32 Corynebacterium Elongation factor Bacillus (shape) EEF2 Protein synthesis Ribosome Fever Adenosine

33
A is not correct . 13% chose this .
34
Although Vibrio cholerae t oxin and pertussis toxin are sim ilar to dipht heria t oxin in that all act through adenosine diphosphate (ADP)
35 ri bosylation, both t he Vibrio cholerae t oxin and the pertussis toxin act via ADP ribosylation t o perma nently activate adenyl cyclase, thereby
36 increasing concent rations of cAMP.
Vibrio cholerae Pertussis toxin Diphtheria Diphtheria toxin Adenosine diphosphate Toxin Adenylyl cyclase Pertussis
37
38 B is not correct. 9 % chose t his .
39 Vibrio cho/erae t oxin permanent ly activat es G5 , thereby causing ri ce-water diarrhea . The clinical pict ure of a person wit h a cholera infect ion
includes acut e-onset severe diarrhea, abdominal discomfort, borborygmi, and vomit ing. Fever is uncommon. Most complications are re lat ed t o
40
t he massive loss of volume and electrolytes, and severe disease can be fatal within hours .
• 41 Vibrio cholerae Stomach rumble Cholera Diarrhea Vomiting Vibrio Electrolyte Toxin Fever Infection

• 42
D is not correct. 8°/o chose this .
. 43 Pertu ssis toxin permanent ly disables G;, thereby causing whooping cough . The early phase of pertussis infection involves nonspecific
• 44 symptoms (mild cough, rh inorrhea, malaise), followed by a paroxysmal stage with t he hallmark cough, which usually beg ins duri ng the
• 45 second week of illness .
Pertussis toxin Rhinorrhea Pertussis Cough Toxin Malaise Infection
• 46
• 47 E is not correct. 6°/o chose this .
. 48 Lethal facto r~ which is released by Bacillus anthracis, is a zinc metalloprot ease that inact ivates protein kinase and that stimulates
macrophages to re lease tumor necros is fact or-a .
. 49 Bacillus anthracis Metalloproteinase Macrophage Protein Anthrax lethal factor endopeptidase Zinc Bacillus Necrosis Neoplasm Anthrax toxin
• 50 •
8
Lock
s
Suspend
0
End Block
Item: 40 of - ,• Mark -<J [:::> "'I ~ · ~
100 ~ P~v1ous N @xt Labl lues N o tes Calcula to r


25
Bottom Line:
26
27
Diphtheria is caused by Corynebacterium diphtheriae via an exotoxin that is encoded by the B prophage. The exotoxin inhibits protein
synthesis by ADP ribosy lation of elongation factor 2.
28 Prophage Coryn b ~enum d1phthenae Diphtheria E»:>t oxin Prot ein biosynthesis Protein Corynebacterium Protem ~vnthes,,;. Elongation factor

29
30
31 14lif.il·1i•J
FlltST AIO FAc·s
for year: 2017 •
32
33
FA17 p 135.1
34
35 Corynebacterium Cram Et> rod; transmitted via respiratory Coryne =club shaped.
36 diphtheriae droplets. Causes diphtheria ,·ia exotoxin Black colonies on cystine-tellurite agar.
37 encoded by ~-prophage. Potent exotoxin ABCDEFG:
38 inhibits protein synthesis via ADP-ribosylation ADP-ribosylation
39 of EF-2. ~-pro phage
40 Symptoms include pseudomembranous Corynebacterium
• 41 pharyngitis (grayish-white membrane f'.J) Diphtheriae
• 42 with lymphadenopathy, myocardit is, and Elongation Factor 2
. 43
arrhyth mias . Granu les
• 44
Lab diagnosis based on gram Et> rods with
• 45
metachromatic (blue and red) granules and
• 46
Et> Elek test for toxin .
• 47
. 48
Toxoid vaccine prevents diphtheria.
. 49
. 50 • FA17 p 128.1

8
Lode.
s
Suspe-nd
8
End Bloc:k
Item: 40 of ~ ,• Mark <:::1 t::> ""I ~· ~'j
100 J.. Previous Next faiUI~S
LAb Notes Calculator

FA17 p 128 .1
25
Bugs with exotox ins
26
BACTERIA TOXIN MECHANISM MAN IFESTATION
27
28 Inhibit protein synthesis
29 Corynebacterium Diphtheria toxin3 Pharyngitis with pseudomembranes in th roat
30
diphtheriae Inactivate elongation facto r and severe lymphadenopathy (bull neck)
31
Pseudomonas Exotoxin A3 (EF-2) Host cell death
32
33
aeruginosa
34 Shigella spp. Shiga toxin (ST)3 GI mucosal damage -+ dysentery; ST also
35 Inactivate 60S ribosome by enhances cytokine release, causing hcmolrtic-
36 removing adenine from uremic syndrome (HUS)
37 rRNA
Enterohemorrhagic Shiga-like toxin SLT enhances cytokine release, causing HUS
38
E coli (EHEC) (SLT)a (prototypically in EHEC serotype 0157:H7).
39
40
Unlike Shigella, F.HEC does not invade host
• 41
cells
• 42 Increase fluid secretion
. 43
Enterot oxigenic Heat-labile Overactivates adenylate Watery diarrhea: "labile in the Air (Adenylate
• 44
E coli (ETEC) toxin (LT)3 cyclase (t cAMP) - t CJ- cyclase), stable on the Ground (Guanylate
• 45
secretion in gut and H20 cyclase)"
• 46
efflux
• 47
Heat-stable Overactivates guanylate
. 48
. 49
toxin (ST) cyclase (t cGMP)
• 50
- l resorption of aCI

8
Lock
s
Suspend
0
End Block
Item: 40 of - ,• Mark -<] 1:> ""'I ~· 1!';:'1
100 ~ Prev1o u s Next Labf a lu es Notes Calculator

FA17 p 123.1
25
Special culture requirements
26
BUG MEDIA USED FOR ISOLATION MEDIA CONTENTS/OTHER
27
28 Hinfluenzae Chocolate agar Factors V ( 1AD+) and X (hematin)
29 N gonorrhoeae, T hayer-Martin agar Selectivel) favors growth of 1 eisseria by
30 N meningitidis inhibiting growth of gram$ organisms
31 ,, ith Vancomycin, gram 8 organisms except
32 1eisseria
with Trimethoprim and Colistin,
33
and fungi with ::\ystatin
34
\'cry Typically Cultures :\eisseria
35
36
Bpertussis Bordet-Gengou agar (Bordet for Bordetel/a) Potato extract
37
Regan-Lowe medium Charcoal, blood, and antibiotic
38 C diphtheriae Tellurite agar, LofAer med ium
39
M tuberculosis Lowenstein-Jensen agar
40
• 41 M pneumoniae F.aton agar Requires cholesterol
• 42 Lactose-fermenting MacConkey agar Fermentation produces acid, causing colonies to
. 43 enterics turn pink
• 44
Ecoli Eosin-methylene blue (EM B) agar Colonies with green metallic sheen
• 45
• 46 Legionella Charcoal yeast extract agar buffered with
• 47 cysteine and iron
. 48 "Sab's a fun guy!"
Fungi Sabouraud agar
. 49
• 50 •

8
L.odt
s
Su~pl'nd
~
End Block
Item: 41 of - ,• Mark -<J
P~v1ous
[:::> "'I ~ ·· ~
100 ~ N @xt Labl lues N o tes Calculato r

A 21-year-old woman present s to t he clinic com plaining of a painful ulcer on her bottom li p fo r 3 days. On further questioning she states ~~AI
25
that she has experienced similar lesions in the past. Biopsy of the lesion shows multinucleated giant cells and intranuclear inclusions.
26
27
Which of the following conditions is an additional manifest at ion of the same pathogen affecting t he patient?
28
:
29
A. Impetigo
30
31
B. Keratoconjunctivitis
32 C. Macular rash
33
0 . Mononucleosis
34
35
E. Oral thrush
36 F. Shingles
37
38
39
40
• 41
• 42
. 43
• 44
• 45
• 46
• 47
. 48
. 49
. 50 •
8
Lode.
s
S uspe-nd
8
End Bloc:k
Item: 41 of - ,• Mark -<J
P~v1ous
[:::> "'I ~ ·· ~
100 ~ N @xt Labl lues N o tes Calculato r
A A

25 The correct answ er is B. 6 2% chose this.


26 The recurrent nature of this pat ient's oral lesion and t he biopsy find ings suggest an infecti on wit h
herpes simplex virus type 1 (HSV-1 ). The pat ient is likely suffering from heq >es labialis ("cold
27
sores" on the lips).
28
HSV-1 infection can cause a number of ot her syndromes when an individual is infected for the
29 first time, or upon reactivation of the latent virus. Primary HSV-1 infection in children can
30 manifest as g ingivost omatitis (ulcers on the gingiva and mucosal membranes).
31 Ke rat oconjuncti v itis most commonly arises as a recurrent disease, however, up to 5% of cases
may arise as primary infection. It is characterized by acute onset of pain, blurry vision, and
32 ocular discharge and may lead to corneal blindness. Te mporal lo be encepha lit is is another
33 serious illness that can result from HSV- 1 infection.
34 A key finding here is the presence of multinucleated giant cells and intranuclear inclusions, which
35 are characteristic (but not specific) for HSV. The inset in this image shows the giant cells (CG).
IC, inflammatory cells; BV, blood vessels.
36 Image copyright© 2012 Carolina
Herpe-- labtalts Ter 1poral lobe Herpes Simplex virus Encephalitis Herpes simplex Biopsy Leston Virus Blood ves.sel V1sual 1mpa1rment
37 Andolfatto et at.
Inclu~10n bod1et InflammatiOn
38
39 A is not correct . 3°/o chose thi s.
40 Impetigo is a localized bact eri al infection of the superficial skin th at clinically manifests with honey-colored,
41
crusted skin lesions. It is usually caused by Staphylococcus aureus or Streptococcus pyogenes. Pat hology
would not show mult inucleated giant cells or intranuclear inclusions.
• 42 Streptococcus pyogenes Staphylococcus aureus Impetigo Streptococcus Pathogenic bacteria Staphylococcus Pathology
. 43
• 44
• 45
• 46
• 47
. 48
. 49
. 50 •
8
Lode.
s
S uspe-nd
8
End Bloc:k
Item: 41 of ~ ,• Mark <::J [:::> ""I ~· ~'j
100 J.. Previous Next LAb faiUI~S Notes Calculator

25 C is not correct. 10 % chos e this .


26 HHV-6 and HHV-7 are herpesviruses associated wit h roseola infa ntum. This child hood exanthem manifest s as a period of high feve r followed
by diffuse macular rash . I n this patient , HHV-6 and HHV- 7 are unlikely since neit her virus is known fo r causing oral ulcers.
27
Exanthem Human herpesvirus 6 Herpesviridae Roseola Rash Virus Fever
28
29
D is not correct. 10 % c hose this .
I nfectious mononucleosis is associat ed with Epstein-Barr virus (EBV) infection, while a mononucleosis-l ike infect ion can occur wit h
30
cyt omegalovirus (CMV) infection in adults. Mononucl eosis usually present with feve r, hepat osplenomegaly, pharyngit is, and cerv ical
31 lympha denopathy. Ot her illnesses caused by EBV include Burkitt lymphoma, nasopharyngeal carcinoma, and lymphoprolife rative disease (in
32 t ransplant patients). CMV, on t he ot her hand, can cause pneumonia, esophagit is, and retinit is in immunocomprised patients. Both CMV and
EBV are herpesviruses. Eit her virus can cause the cellular changes given in t his stem . However, neit her one commonly presents wit h recu rrent
33
ulcers on the lips.
34 Infectious mononucleosis Burkitt's lymphoma Epstein-Barr virus Cytomegalovirus Nasopharynx cancer Pharyngitis Lymphadenopathy Hepatosplenomegaly Pneumonia Esophagitis Lymphoma Herpesviridae
35 Lymphoproliferative disorders Virus Infection Pharynx Carcinoma Cervical lymphadenopathy Cervical cancer Fever Nasopharynx Human cytomegalovirus Cervix Ulcer (dermatology)
36
E is not correct. 2 °/o chos e this .
37
Oral t hrush is a sign of infect ion wit h Candida species. Ot her sym ptoms of Candida infect ion in t he upper digest ive t ract could include pain on
38 swallowing (odynophag ia due t o esophagit is, especially in HIV-posit ive patient s) or painful f issures on t he corners of t he mout h (angular
39 cheilit is) . Candia specimen would be posit ive for fungal forms rather than t he cellular changes seen in t his pat ient's biopsy results.
Odynophagia Angular cheilitis Candidiasis Oral candidiasis Esophagitis Candida (fungus) Biopsy Cheilitis Gastrointestinal tract Fungus HIV Infection Thrush (bird) Species
40
41 F is not correct . 13% chos e this .
• 42 Shingles is caused by a reactivation of varicella zoster v irus (VZV) which is also a herpesvirus. Patients present with a painful herpetic-
appeari ng rash, classically in a dermatomal dist ribut ion. VZV may remain latent in dorsal root ganglia after a child hood infect ion wit h chicken
. 43
pox, which is similar to the mechanism of herpes simplex reactivat ion . VZV can show int ranuclear inclusions on pathology but does not cause
• 44 recurrent ulcers to appear on t he lips .
• 45 varicella zoster virus Dorsal root ganglion Chickenpox Herpes simplex Shingles Virus Herpesviridae Rash Dermatome (anatomy) Ganglion Pathology Varicella vaccine

• 46 Kaposi's sarcoma-associated herpesvirus Ulcer

• 47
. 48 Bottom Line :
. 49
I n add it ion to herpes labia lis (cold sores), HSV-1 can cause gingivost omat it is, keratoconjunct ivit is, or t emporal lobe encephalit is. Biopsy of
• 50 • an HSV lesion mav show characteristic m ultinucleated aiant cells or int ranuclear inclusions .
8
Lock
s
Suspend
0
End Block
Item: 41 of ~ ,• Mark <::J [:::> ""I ~· ~'j
100 J.. Previous Next LAb faiUI~S Notes Calculator

FA17 p 160.2
25
26 Herpesviruses Enveloped, OS, and linear viruses
27
VIRUS ROUTE OF TRANSMISSION CLI NICAL SIGNIFICANCE NOTES
28
29
Herpes Respiratory Gingivostomatitis, keratoconjunctivitis fJ, Most common cause of sporadic
30
simplex secretions, saliva herpes labial is [l), herpetic whitlow on finger, encephalitis, can present as altered
31
virus-1 temporal lobe encephalitis, esophagitis, mental status, seizures, and/or
32
erythema mu lti forme. aphasia.
33 Herpes Sexual contact, t-Ierpes genital i s ~, neonatal herpes. Latent in sacral ganglia. Viral
34 simplex perinatal meningitis more common with
35 virus-2 HSV-2 than with HSV-1.
36
Varicella- Respiratory Varicella-zoster (chickenpox 1!], shingles 0), Latent in dorsal root or trigeminal
37
Zoster virus secretions encephalitis, pneumonia. ganglia; Cl V1 bra nch
38
(HHV-3) Most common complication of shingles is post- involvement can cause herpes
39
40
herpetic neuralgia. zoster ophthalmicus.
41 Epstein-Barr Respiratory Mononucleosis- fever, hepatosplenomegaly, Infects B cells through CD21.
• 42 virus (HHV-4) secretions, pharyngitis, and lymphadenopathy (especially Atypical lymphocytes on peripheral
. 43 saliva; aka posterior cervical nodes 0 ). Avoid contact sports blood smear [!!- not infected B
• 44 "kissing disease," until resolution due to risk of splenic rupture. cells but reactive cytotoxic T cells .
• 45 (common in Associated with lymphomas (eg, endemic (±) Monospot test-heterophile
• 46
teens, young Burkitt lymphoma), nasopharyngeal antibodies detected by agglutination
• 47
adults) carcinoma (especially Asian adults), of sheep or horse RBCs.
• 48
lymphoprol iferative disease in transplant Use of amoxicillin in mononucleosis
. 49
patients. can cause characteristic
• 50 ~ .......... 1... ........ ...... 1..... -~ ... 1...

8
Lock
s
Suspend
0
End Block
Item: 41 of - ,• Mark --<) [::> ""'I ~· 1!';:'1
100 ~ Prev1o u s Next Labf a lu es Notes Calculator

cotton-\\ ool exudates, vas1on loss.


25
Congenital CM
26
27
Human Saliva Roseola infantum (exanthem subitum): high Roseola: fe\·er first, Rmic (checks)
28
herpes- fe\·ers for several days thai can cause seizures, later.
29 viruses 6 followed br diffuse maculttr rash HHV-7-less common cause of
30 and 7 roseola.
31 Human Sexual contact Kaposi sarcoma (neoplasm of endothelial cells). Can also affect Cl tract and lungs.
32 herpesvirus Seen in HIV/AIDS and transplant patients.
33 8 Dark/violaceous plaques or nodules
34 representing vascular proliferations.
35
36
37
38
39
40
41
• 42
. 43
• 44
• 45
• 46
• 47
. 48
. 49
• 50 •
8
L.odt
s
Su~pl'nd
~
End Block
Item: 41 of ~ ,• Mark <::J [:::> ""I ~· ~'j
100 J.. Previous Next LAb faiUI~S Notes Calculator

FA17 p 160.1
25
26
DNA viruses
27
VIRAL FAMILY ENVELOPE DNA STRUCTURE MEDICAL IMPORTANCE

28 Herpesviruses Yes OS and linear See Herpesviruses entry


29 Poxvirus Yes OS and linear Smallpox eradicated world wide by use of the live-
30
(largest 0 A virus) attenuated vaccine
31
Cowpox ("milkmaid blisters")
32
Molluscum contagiosum -Aesh-colored papule with
33
centra I umbiIication
34
35 Hepadnavirus Yes Partially OS and circular HBV:
36 • cute or chronic hepatitis
37 • I ot a retrovirus but has re,·erse transcriptase
38 Adenovirus 0 OS and linear F'ebrile pharyngitis f.J - sore throat
39 Acute hemorrhagic cystitis
40 Pneumonia
41 Conju ncti,·itis-"pink eye"
• 42
. 43
• 44
• 45
• 46
• 47
. 48
Papillomavirus No OS and circular 1--1PV- warts (serotypes l, 2, 6, ll), Cl1 , cervica l
. 49 cancer (most com monly 16, 18)
• 50 • Polyomavirus 0 OS and circular JC virus-progressive multi focal leukoencephalopathy •

8
Lock
s
Suspend
0
End Block
Item: 42 of - ,• Mark -<J
P~v1ous
[:::> "'I ~ ·· ~
100 ~ N @xt Labl lues N o tes Calculato r

A 70-year-old woman present s to t he emergency depart ment with fever, chills, swelling, erythema, and decreased range of mot ion in the ~~AI
25
left knee. She had a left total knee replacemen t two weeks ago and had a prosthetic joint implanted. Her surgeon decides t o tap the joint
26 and sends the flu id for Gram st ain and culture.
27
28 Which of the following is most likely t o be observed upon laboratory examination of the joint fluid?
29
:
30 A. Gram-negative, glucose-fermenting cocci
31
B. Gram-negative, lactose-fermenting bacilli
32
33 C. Gram-negative, non-lactose fermenting bacilli
34 0 . Gram-positive, catalase-negative cocci
35
E. Gram-positive, catalase-posit ive cocci
36
37
38
39
40
41
• 42
. 43
• 44
• 45
• 46
• 47
. 48
. 49
. 50 •
8
Lode.
s
S uspe-nd
8
End Bloc:k
Item: 42 of ~ ,• Mark <::J [:::> ""I ~· ~'j
100 J.. Previous Next LAbfaiUI~S Notes Calculator

25
26
The correct answer is E. 70°/o chose this .
27 Gram -posit ive, catalase -posit ive cocci could refer t o eit her Staphylococcus aureus or Staphylococcus epidermidis. Overall, S. aureus is t he
28 most common cause of sept ic arthri tis in adults and children . However, S. epidermidis is also known for causing nosocomia l infect ions of
29 implanted fo reign bodies such as cat heters, joint prost het ics, and prosthetic heart valves. S. aureus and S. epidermidis can be different iated
by coagulase test ing: S. au reus is coagulase-posit ive whileS. epidermidis is coagulase-negative.
30 Staphylococcus epidermidis Gram-positive bacteria Staphylococcus aureus Septic arthritis Coagulase Coccus Hospital-acquired infection Staphylococcus Foreign body Catheter Prosthesis Arthritis
31 Heart valve
32
A is not correct. 3°/o chos e this.
33
Gram -negative, glucose-fe rment ing cocci ident ifies an organism such as Neisseria gonorrhoeae. N. gonorrhoeae is a common cause of septic
34 arthri t is in young, sexually act ive adults. I t is sexually t ransm itt ed and causes a purulent infilt ration of t he synovium . N. gonorrhoeae
35 ferments only glucose, whereas N. meningitidis fe rment s both glucose and maltose.
Gram-negative bacteria Neisseria gonorrhoeae Maltose Septic arthritis Synovial membrane Coccus Neisseria meningitidis Neisseria Glucose Pus Arthritis Organism
36
37 B is not correct. 2% chose this.
38 Gram -negative, lactose-fe rment ing bacilli ident if ies an organism such as Escherichia coli. E. coli is the most common cause of urinary t ract
39
infections. It is not implicat ed as a cause of postsurgica l septic arthri t is.
Gram-negative bacteria Escherichia coli Septic arthritis Urinary tract infection Bacilli Urinary system Arthritis Bacillus (shape) Organism
40
41 C is not correct. 3 °/o chos e this .
42
Gram -negative, non-lactose fe rment ing bacilli ident if ies an organism such as Salmonella. Salmonella species are usually implicated in bloody
diarrhea. When infecting bone, Salmonella causes osteomyelit is rat her t han septic art hrit is, pa1t icularly in asplenic individuals such as adults
. 43 with sickle cell anem ia.
• 44 Osteomyelitis Gram-negative bacteria Septic arthritis Sickle-cell disease Salmonella Anemia Diarrhea Fermentation Asplenia Arthritis Bone

• 45 D is not correct. 22% chose this .


• 46 Gram -posit ive, catalase -negative cocci ident if ies an organism such as Streptococcus. S. pyogenes is implicat ed in bact erial pharyngit is and
• 47 scarlet feve r, among other diseases. I t could cause polyarthra lgias in a pat ient with rheumatic feve r, but it is not a likely cause of postsurgica l
sept ic arthrit is.
. 48
Scarlet fever Rheumatic fever Gram-positive bacteria Septic arthritis Pharyngitis Coccus Streptococcus Streptococcus pyogenes Arthritis Catalase Fever
. 49
• 50 •
8
Lock
s
Suspend
0
End Block
Item: 42 of ~ ,• Mark <::J [:::> ""I ~· ~'j
100 J.. Previous Next LAb faiUI~S Notes Calculator

25 Bottom Line:
26 Staphylococcus aureus and Staphylococcus epidermidis are gram-posit ive, catalase-posit ive cocci t hat are likely t o cause septic arthrit is after
j oint replacement surge ry.
27
Staphylococcus epidermidis Staphylococcus aureus Septic arthritis Gram- positive bacteria Coccus Staphylococcus Arthritis Joint replacement
28
29
30
l@l;fil·1i•l for year:[ 2017
FIRST AID FACTS .
•j .
31
32
33 FA17 p 441 .2

34 Septic arthritis S aureus, Streptococcus, and Neisseria gonorrhoeae are common causes. Affected joint is swollen a,
35 red, and painful. Synovial fluid purulent (WBC > 50,000/mm3).
36 Gonococcal arthritis-STI that presents as either purulent arthritis (eg, knee) or triad of
37
polyarthralgia, tenosynovitis (eg, hand), dermatitis (eg, pustules).
38
39
40
41
42
. 43
• 44
FA17 p 131 .3
• 45
• 46 Staphylococcus aureus Cram EB, P-hemolytic, catalase ®, coagulase TSST-1 is a superantigen that binds to MHC
• 47 E!7 cocci in clusters a. Protein A (virulence II and T-cell receptor, resulting in polyclonal
• 48
• fa ctor) binds Fc-lgC, inh ibiting complement T-cell activation .
. 49 activation and phagoC)tosis. Commonly Staphylococcal toxic shock syndrome
• 50 r.olo nizPs thr. n:lrf's. ::~x i l b . ::~ nri P roin . fTSS) nrP~~Pnts :1s fpvpr vomitinP r:1sh
8
Lock
s
Suspend
0
End Block
26
27 FA17 p 131 .3
28
Staphylococcus au reus Gram (f), ~-hemolytic, catalase Ef>, coagulase TSST-1 is a superantigen that binds to MHC
29
(f) cocci in clusters r.:J. Protein A (\·irulence ll and T-cell receptor, resulting in polyclonal
30
factor) binds Fc-IgG, inhibiting complement T-cell acti\'ation.
31
acti,·ation and phagocytosis. Commonlr Staphylococcal toxic shock syndrome
32
33
colonizes the nares, axilla, and groin. (TS ) presents as fever, \'Omiting, rash,
34
Causes: desquamation, shock, end-organ failure. T S
35
Inflammatory disease-skin infections, results in t AST, t LT, t bilirubin. ssociated
36 organ abscesses, pneumonia (often after with prolonged use of ,·aginal tampons or nasal
37 influenza virus infection}, endocarditis, pad .ing.
38 septic arthritis, and osteomyelitis. Compare with Streptococcus pyogenes TSS (a
39 Toxin-mediated disease-toxic shock toxic shock- like syndrome associated with
40 syndrome (TSST-1}, scalded skin syndrome painfu l skin infection).
41 (exfoliative toxin), rapid-onset food $ aureus food poisoning due to ingestion of
42 poisoning (cnterotoxins). preformed toxin - short incubation period
. 43 1RSA (methicilli n-resistantS aureus} (2-6 hr} followed by nonbloody diarrhea
• 44
infection- important cause of serious and emesis. Enterotoxin is heat stable _. not
• 45
nosocomial and community-acquired destroyed by cooking.
• 46
infections; resistant to methicillin and Bad staph (aureus) make coagulase and toxins .
• 47
nafcillin because of altered penicillin- Forms fibrin clot around self - abscess .
. 48
binding protein .
. 49
• 50 •

8
L.odt
s
Su~pl'nd
~
End Block
Item: 43 of - ,• Mark -<J [:::> "'I ~ · ~
100 ~ P~v1ous N @xt Labl lues N o tes Calculato r

A 12-year-old girl begins to experience bloody diarrhea a few days after eating at a fast food restaurant. Several ot hers are affected, and ~~AI
25
investigators are able to trace it back t o unwashed vegetables used at the restaurant. The pat ient is t reated with plasmapheresis and
26 eventually recovers from acute renal fai lure, anemia, and a low platelet count.
27
28 Which of the following mechanisms is used by the toxin described in this patient?
29
:
30 A. Blocking release of acetylcholine
31
B. Inactivation of elongation factor- 2
32
33 C. Inactivation of ribosomes
34 0 . Increasing the release of acetylcholine
35
E. Superantigen binding to MHC II protein and T-lymphocyte receptor
36
37
38
39
40
41
42
. 43
• 44
• 45
• 46
• 47
. 48
. 49
. 50 •
8
Lode.
s
S uspe-nd
8
End Bloc:k
Item: 43 of ~ ,• Mark <::J [:::> ""I ~· ~'j
100 J.. Previous Next LAb faiUI~S Notes Calculator

25
26 The correct a nswer is C. 56°/o chos e this .
27 The pat ient descri bed in t he stem suffered from hemolyt ic-uremic syndrome (HUS). HUS is most commonly associat ed with infection wit h
28 Shigella and Escherichia coli 0 157 : H7, a st ra in of E. coli t hat produces a Shiga -like toxin . The t oxin leads t o rena l fa ilure, m icroangiopat hic
29
hemolytic anem ia, and disseminated int ravascu lar coagulat ion, as well as thrombocytopen ia and feve r. The toxin includes t wo components: A
and B. The B component binds to the cell membrane and allows entry of the A t oxin. This com ponent inact ivates ri bosomes, t hereby stopping
30 protein synthesis, which leads to cell deat h. Since it is common ly acqu ired via t he digestive t ract, the cells exposed die and slough off, leading
31 t o t he bloody dia rrhea experienced by t he pat ient. Both t hese foodborn e pathogens attach to int estinal epit helium, and t he t oxin (not the
32
bacteri a) invade t he cell. The two pat hogens can be diffe rent iat ed on st ool cultures.
Microangiopathic hemolytic anemia Disseminated intravascular coagulation Hemolytic-uremic syndrome Escherichia coli Thrombocytopenia Shigella Hemolytic anemia Shiga-like toxin Cell membrane Diarrhea
33
Anemia Protein Gastrointestinal tract Hemolysis Pathogen Toxin Coagulation Apoptosis Epithelium Protein biosynthesis Bacteria Protein synthesis Ribosome Fever Microangiopathy Kidney Stool test
34
Blood vessel Infection
35
36 A is not correct . 6°/o chos e this.
The neurotoxin produced by Clostridium botulinum blocks the re lease of acetylcholine at t he neuromuscular j unct ion, causing flaccid paralysis.
37
Clostridium botulinum Neurotoxin Neuromuscular junction Flaccid paralysis Acetylcholine Botulinum toxin Paralysis
38
39
B is not correct. 19 % chose this .
The toxin produced by Corynebacterium diphtheriae inact ivat es elongat ion facto r (EF)-2 by adenosine diphosphat e ri bosylation, but EF-2 is
40
not a G protein. I nact ivat ion of EF-2 inhibi ts protein synthesis, leading t o t he deat h of t he cell.
41 Corynebacterium diphtheriae Adenosine diphosphate Protein biosynthesis Elongation factor Protein Pyrophosphate Protein synthesis Toxin Adenosine Corynebacterium G protein

42
D is not correct. 7°/o chos e this.
43
The venom produced by Lactrodectus, or t he black widow s pi de r~ drast ically increases t he amount of acetylcholine released, wh ich causes
• 44 muscl e cont ract ion unt il all of t he acetylcholine has been used; paralysis can t hen occur as a result .
• 45 Latrodectus Acetylcholine Venom Spider Muscle contraction Paralysis Muscle

• 46 E is not correct. 1 2°/o chos e this.


• 47 Staphylococcus aureus produces an exotoxin that binds major histocompatibilit y complex II and T-lym phocyt e receptor, causing t he massive
. 48 release of cytokines result ing in toxic shock syndrome .
Major histocompatibility complex Toxic shock syndrome Staphylococcus aureus Exotoxin T cell Cytokine Staphylococcus Receptor (biochemistry) Toxicity
. 49
• 50 •
8
Lock
s
Suspend
0
End Block
Item: 43 of ~ ,• Mark <::J [:::> ""I ~· ~'j
100 J.. Previous Next LAbfaiUI~S Notes Calculator

25 Bottom Line:
26 E. coli 0157: H7 (EHEC) and Shigella are two foodborne pathogens t hat can lead t o HUS and possibly even death . They att aches to int estinal
27 epit helial cells, but does not gain entry into the cell. They secret e bact eri al prot ein toxin into t he cell, which inhibits protein synt hesis by
28 bind ing t o ri bosomes. Bloody diarrhea, acute renal fa ilure, and t hrombocytopenia are some of t he more common associat ions.
Escherichia coli Shigella Thrombocytopenia Diarrhea Acute kidney injury Protein synthesis Epithelium Verotoxin-producing Escherichia coli Ribosome Protein Pathogen Toxin Protein biosynthesis Kidney
29
30
31
141;fil·1i•J
FIRS T AID FACTS
for year:[ 2017
.
•j .
32
33
34 FA17 p 128.1
35 Bugs with exotox ins
36 BACTERIA TOXIN MECHANISM MANIFESTATION
37 Inhibit protein synthesis
38
Corynebacterium Diphtheria toxin3 Pharyngitis with pseudomembranes in throat
39
40
diphtheriae Inactivate elongation facto r and severe lymphadenopathy (bull neck)
41 Pseudomonas Exotoxin A3 (EF-2) Host cell death
42 aeruginosa
43
Shigella spp. Shiga toxin (ST)3 GI mucosal damage -+ dysentery; ST also
• 44
Inactivate 60S ribosome by enhances cytokine release, causing hcmolrtic-
• 45
removing adenine from uremic syndrome (HUS)
• 46
• 47 Enterohemorrhagic Shiga-like toxin rRNA SLT en hances cytokine release, causing HUS
. 48 E coli (EHEC) (SLT)a (prototypically in EHEC serotype 0157:H7).
. 49 Unlike Shigella, F.HEC does not invade host
• 50 • cells
8
Lock
s
Suspend
0
End Block
Item: 43 of - ,• Mark --<) [::> ""'I ~· 1!';:'1
100 ~ Prev1o u s Next Labf a lu es Notes Calculator

FA17 p 140.1
25
26 Escherichia coli Gram 8 rod. E coli virulence factors: fimbriae-cystitis and pyelonephritis (P-pili); K capsule-
27 pneumonia, neonatal meningitis; LPS endotoxin-septic shock.
28
STRAIN TOXIN AND MECHANISM PRESENTATION
29
30 EIEC 1icrobe in\'ades intestinal mucosa and causes lnvasi\'e; dysentery. Clinical manifestations
31 necrosis and inAammation. similar to Shigella.
32 ETEC Produces heat-labile and heat-stable Tra\'clcrs' diarrhea (watery).
33
enteroToxins. No inAammation or in\'asion.
34
35
EPEC 1o toxin produced. Adheres to apical surface, Diarrhea, usually in children (Pediatrics).
36
Aattens "illi, pre,·ents absorption.
37 EHEC 0157:1-17 is most common serotype in US. Often Dysentery (toxin alone causes necrosis and
38 transmitted via undercooked meat, raw leafy inAammat ion).
39 vegetables. Does not ferment sorbitol (vs other E coli).
40 Shiga-like toxin causes hemolytic-ure mic ll cmorrhagic, Hamburgers, llemolytic-urcmic
41
syndrome: triad of anemia, thrombocytopenia, syndrome.
42
and acute renal fai lure clue to 111 icroth rombi
43
formi ng on damaged endothelium
• 44
- mechanical hemolysis (with schistocytcs on
• 45
peripheral blood smear), platelet consumption,
• 46
• 47
and l renal blood Aow.
0
48
0
49 FA17 p 175.1
• 50 Buqs causinq diarrhea •
8
L.odt
s
Su~pl'nd
~
End Block
Item: 43 of ~ ,• Mark <::J [:::> ""I ~· ~'j
100 J.. Previous Next LAbfaiUI~S Notes Calculator

25 FA17 p 175.1
26 Bugs causing diarrhea
27 Bloody diarrhea
28
Campylobacter Comma- or S-shaped organisms; growth at 42°C
29
30 E histolytica Protozoan; amebic dysentery; liver abscess
31 Enterohemorrhagic 0157:H7; can cause llUS; makes Shiga-like toxin
32
Ecoli
33
34
Enteroinvasive Ecoli Invades colonic mucosa
35 Salmonella (non- Lactose 8; Aagellar motility; has animal reservoir, especially poultry and eggs
36 typhoidal)
37
Shigella Lactose 8; very low 10 ;0; produces Shiga toxin (human reservoir only); bacillary dysentery
38
39 Yenterocolitica Day care outbreaks, pseudoappendicitis
40 Watery diarrhea
41
Cdifficile Pseudomembranous colitis; associated with antibiotics and PPis; occasionally bloody diarrhea
42
43 C perfringens Also causes gas gangrene
• 44 Enterotoxigenic E coli Tra,·elers' diarrhea; produces heat-labile (LT) and heat-stable (ST) toxi ns
• 45
Protozoa Giardia, Cryptosporidium
• 46
• 47 Vcholerae Comma-shaped organisms; rice-water diarrhea; often from infected seafood
. 48 Viruses Rota,·irus, norovirus, adenovirus
. 49
• 50 •
8
Lock
s
Suspend
0
End Block
Item: 44 of - ,• Mark -<J [:::> "'I ~ · ~
100 ~ P~v1ous N @xt Labl lues N o tes Calculato r

25 Bacterial strain X is resistant to ampicillin and sensit ive to gentamicin. Bacterial strain Y is resistant t o gentam icin and sensitive to
ampicillin. Bacterial strains X and Yare grown in m ixed culture in medium without anti biot ics, and t hen t he culture is plated on medium
26
containing both ampicillin and gentamicin. Bacterial colonies grow on the plates. In a second experiment , separate strains of X and Yare
27 co-cultured in DNAse-contai ning antibiotic-free media then plat ed on culture medium containing ampicillin and gentamicin. No colonies grow on
28 these plates.
29
30 Assuming that bacterial cells are impermeable to DNAse, what process best explains these observat ions?
31 :

32
A. Mutation
33 B. Transcription
34
C. Transduction
35
D. Transformation
36
37 E. Transposition
38
39
40
41
42
43
• 44
• 45
• 46
• 47
. 48
. 49
. 50 •
8
Lode.
s
S uspe-nd
8
End Bloc:k
Item: 44 of - ,• Mark -<] 1:> ""'I ~· 1!';:'1
100 ~ Prev1o u s Next Labf a lu es Notes Calculator

25
The correct a nswer is D. 57°/o chose this.
26
Transformation is gene transfer resulting from the uptake of DNA from the
27 environment. Strain X has an ampicillin resistance gene (AmpR), and strain Y has a
28 gentamicin resistance gene (GenR). During the initial co-culture step of the first
29 experiment (no DNAseL some bacteria die and release genetic material into the
medium. The surviving bacteria tend t o pick up t his genetic material in a process
30 called transformation, illustrated in the diagram. Therefore, some GenR bacteria
31 picked up the AmpR gene and vice versa, and when this co-culture was then plated on
32 ampicillin + gentamicin-containing media, bacteria with both AmpR and GenR genes Image courtesy of Wikimedia Commons
survived. In the second experiment, when bacteria died and spilled their genetic
33 material into the media, their DNA was destroyed by the DNAase, so bacteria did not have a chance to undergo transformation.
34 Gent 1l1Cm Aon n Oeo ynbonudease Gene DNA Bacteria Amp res1stance Genome Honzont ge e tra

35
A i s not correct. 4 °/o chose this .
36 Mutation can explain the results of the first experiment, although highly improbable. However, the effect of DNAse should not have influenced
37 the results if mutation occurred in the first experiment.
Deoxynbonucle e MutM1on
38
39 B is not correct. 3% chose this.
40 Transcription is the fi rst step of gene expression where a particular segment of DNA is copied into RNA by an enzyme RNA polymerase. This is
41
unlikely to have caused a transfer of DNA bet ween bacteria, as typically transcription is used fo r making new prot eins.
Gene Enzyme DNA RNA RNA polymerase Transcription (genetics) Gene expression Bacteria Prote1n
42
43 C is not co r r ect . 26 % chose this.
Transduction is gene transfer f rom a donor to a recipient by way of a bacteriophage. There is no evidence to demonstrate the presence of a
44
phage vector In this experiment. Also, since t he DNA is enclosed within a phage vector, the DNA would not be susceptible to the DNAse within
• 45 th e medium.
• 46 Bacteriophage Gene Deoxyribonuclease DNA Signal transduction Horizontal gene transfer Transduet1on (genet1cs) Vector (ep1dem1ology)

• 47 E i s not correct. 10°/o chose this.


. 48 Transposable elements are segments of DNA t hat are able to move from one location to another. Although transposit ion can be used to confer
. 49 antibiotic resistance, there is no evidence of a specific element conferring resistance in this experiment. Again, there would be no free floating
• 50
DNA found outside the cell that would be susceptible to the DNAse in the medium . •
8
L.odt
s
Su~pl'nd
~
End Block
Item : 44 o f - ,• Ma rk -<J [:::> "'I ~ · ~
100 ~ P~v1ous N @xt Labl lues N otes Calculator


25
Bottom Line:
26
27 Transformation occurs when a cell takes up DNA from a donor cell, which is why when two types of bacteria are grown in mixed culture,
some might survive bactericides that could kill each type individually, unless DNAse is added to destroy DNA spilled int o the medium before
28 cells can absorb it.
29 Deo)f(lbonucl~e ONA Bctctena ec.ctencide

30
31
32 14l;fil·1i•J for year : 2017 •
FIRST AIO fACTS
33
34 FA17 p 126.1
35
Bacterial genetics
36
37
Transformation Competent bacteria are able to bind and import Degraded

·",-.. a
uncombined \ . ; (Rec1ptent DNA
38
short pieces of environmental naked bacterial DNA
DonOI"DNA
39 chromosomal 0 1A (from bacterial cell 9
~
0
40 lysis). The transfer and expression of new!)'
Naked DNA Recipient cell Transformed cell
41 transferred genes is called transformal ion.
42 A feature of many bacteria, especially S
43 tmeumoniae, II inf/uenzae type 13, ami
44 Neisseria (SHi ~). Any 0 A can be used.
• 45 Adding deoxyribonuclease lo environment
• 46 will degrade naked 0 1 A in medium ..... no
• 47 transformation seen .
. 48
Conjugation
. 49
. 50 F X F" f'+ plasmid contains genes required for sex pilus
8
Lode.
s
Suspe-nd
8
End Bloc:k
Item: 44 of - ,• Mark -<J [:::> "'I ~ · ~
100 ~ P~v1ous N @xt Labl lues N o tes Calculato r
A A

Transduction
25
26
27
Generalized A "packaging" event. Lytic phage infects
bacterium, leading to cleavage of bacterial
Lytict
phage )
/
Cleavage of
Bacteria bacterial DNA
Bacterial DNA package
in phage caps~d

28 DNA. Parts of bacterial chromosomal Dt A ,~ ~)


29 may become packaged in phage capsid. Phage
30 infects another bacterium, transferring these

-
31 genes.
32
33
34
0 0)
35
Release of new pha9e Infects other Phage's genes
from lysed cell bacteria transferred ~
36
37
38
Specialized n "excision" event. Lysogenic phage infects
bacterium; viral DNA incorporates into '"""" f
phage
Viral DNA

'"""""'"""
. Viral DNA .
["Bacteria bacterial DNA
Phage partiCles
carry bacterial DNA
39
40
41
bacterial chromosome. When phage D A
is excised, flanking bacterial genes may be
excised with it. D A is packaged into phage
capsid and can infect another bacterium .
~ - 0 - -
-
42
43 Genes for the following 5 bacterial toxins are
44 encoded in a lysogenic phage (ABCD'S): Croup
• 45 A strep eryth rogenic toxin, Bot ul inum toxin,
' 46 Release of new phage Infects other Genes different from
C holera toxin, Diphtheria toxin, Shiga toxin. from lysed cell bacteria donor and reapient Cl
' 47
. 48 Transposition Segment of DNA (eg, transposon) that can Plasmid
. 49 "jump" (excision and reintegration) from
' 50 • one location to another, can transfer genes
8
Lode.
s
S uspe-nd
8
End Bloc:k
Item: 45 of - ,• Mark -<J [:::> "'I ~ · ~
100 ~ P~v1ous N @xt Labl lues N o tes Calculato r

A 6-year-old girl with an allergy t o amoxicillin was brought in by her mother after developing a feve r and rash after attending summer ~~AI
25
camp a few hours away from her home in New Jersey. The rash was initially small, but expanded in t he form of a ring. Laboratory tests
26 reveal an elevated erythrocyte sedimentation rate, but no anemia or leukocytosis.
27
28 If administerd the best choice antibiotic to t reat her infection in adults, what common adverse effect is t his pat ient at risk for?
29
:
30 A. Discoloration of teeth
31
B. Gray baby syndrome
32
33 C. Ototoxicity
34 D. Pseudomembranous colitis
35
E. Seizures
36
37
38
39
40
41
42
43
44
• 45
• 46
• 47
. 48
. 49
. 50 •
8
Lode.
s
S uspe-nd
8
End Bloc:k
Item: 4S of - ,• Mark -<J [:::> "'I ~ · ~
100 ~ P~v1ous N @xt Labl lues N o tes Calculato r
A A

25 The correct answer is A . 61 °/o chose this.


26 Discoloration of teeth in children is an adverse effect of doxycycline, the treatment of choice fo r Lyme
27 disease. Initially, Lyme disease manifest s with a rash similar to the one in the image that surrounds t he bi te
28
site of the Ixodes tick, the organism t hat is responsible for transmitting the Borrelia burgdo1feri spirochete.
There are three stages of Lyme disease: St age 1, known as erythema chronicum migrans (which is
29 described in the stem of the question) manifests with fevers, chills, fatigue, and malaise. This patient is
30 currently in stage 1 of the disease. Stage 2 manifests with neurologic abnormalities and cardiac
31
abnormalities, which occurs 4 - 6 weeks after primary infection. Stage 3 occurs months to years after the
initial infection and manifests as migratory arthritis and synovitis. Doxycycline is the treatment of choice for
32 patients with stage 1 Lyme disease. Doxycycline acts by inhibiting attachment of aminoacyl-transfer RNA in
33 bacteria. Its adverse effects include gastrointestinal distress, discoloration of teeth, inhibited bone growth in
34
children, and photosensitivity in adults. Other toxicities include liver toxicity and nephrotoxicity. Doxycycline
is also contraindicated in pregnancy. Alternative treatments for this patient would be cefuroxime or
35 erythromycin.
36 EMhror'l'"(c•n Sp~rochaete Lyme d1sease Doxycycline Borrelia burgdorferi Cefuroxime Erythema Erythema chromcum m•oran5 Photosensitivity Mala1se

37 Adverse effect Nephrotox•c•ty Rash T1ck Borrelia Hepatotoxicity Liver Fatigue (medical) Bacteria Arthr•t•s Ixodes Preonancy Contra•nd•cabon RNA Organism
Image courtesy of CDC/James
38 Bone Gastrointestinal tract
Gathany
39
40 B is n ot correct. 7 °/o chose this.
41 Gray baby syndrome is an adverse effect of chloramphenicol, which inhibits 50S peptidyl t ransferase act ivit y. Other adverse effects include
anemia and aplastic anemia.
42 Gray baby syndrome Chloramphemcol Aplastic anemia Peptidyl transferase Transferase Anemia
43
C is not correct. 16% chose this.
44
Ototoxicity can be an adverse eff ect of ant ibiotics (eg, aminog lycosides and vancomycin) . Bot h t hese ant ibiot ics are also associated with
45
nephrotoxicity.
' 46 vancomycin Ototox1c1ty Nephrotox•c•tv Am~noglycoside Antibiotics Adverse effect

' 47
D is n ot correct. 8 °/o chose this .
. 48
Pseudomembranous colitis is an adverse effect commonly associated with clindamycin, which is used t o t reat anaerobic infections by inhibiting
. 49 peptide bond formation at the 50S ribosome. Alt hough pseudomembranous colitis can occur with any ant ibiotic, it is not the best answer
. 50 choice for this drug/patient combination .

8
Lode.
s
S uspe-nd
8
End Bloc:k
Item: 4S of ~ ,• Mark <::J [:::> ""I ~· ~'j
100 J.. Previous Next LAbfaiUI~S Notes Calculator

D is not correct. 8°/o chose this.


25
Pseudomembranous colit is is an adverse effect commonly associated wit h clindamycin, which is used to treat anaerobic infections by inhibit ing
26 peptide bond format ion at t he 505 ri bosome. Although pseudomembranous colit is can occur wit h any antibiotic, it is not the best answer
27 choice for t his drug/patient combination.
Clostridium difficile colitis Peptide bond Clindamycin Ribosome Antibiotics 50S Colitis Anaerobic organism Adverse effect Peptide Anaerobic infection Peptidyl transferase
28
29 E is not correct. 8 °/o chos e this.
30 Seizures can be an adverse effect of antibiot ics (eg, met ronidazole) . Metronidazole is used to t reat anaerobic infections by form ing toxic
31
metabolit es that damage DNA in bacterial cells.
Metronidazole Anaerobic organism Antibiotics Epileptic seizure Adverse effect DNA Anaerobic infection
32
33
34 Bottom Line:
35 Lyme disease is transmit ted by the Ixodes t ick. It init ially manifests with a feve r and a rash (eryt hema chronicum migrans), and, if left
untreat ed, it can involve the central nervous system and the joints. Trea t ment of choice is the ant ibiotic doxycycline, which can lead to such
36
adverse effects as discolorat ion of teet h and inhibit ed bone growth in children.
37 Lyme disease Doxycycline Central nervous system Erythema Erythema chronicum migrans Antibiotics Tick Ixodes Rash Bone Fever Nervous system

38
39
40 141;fil·1i•J for year:[2017 •
FIRST AID FA CTS
41
42
FA17 p 142.4
43
44 Lyme disease Caused by Borrelia burgdorferi, which is A Key Lyme pie to the "FACE:
45 transmitted b)' the Ixodes deer tick rJ (also Facial nerve palsy (typically bilateral)
• 46 vector for Anaplasma spp. and protozoa Arthritis
• 47 Babesia). atural reservoir is the mouse (and Cardiac block
. 48 important to tick life cycle). Erythema migrans
. 49 Common in northeastern United States. Treatment: doxycrcline (lst line); amoxicillin
• 50 • S t::.O"P 1- P::. r lv l of'::~ li?Prl· Prvth Prn::. rniPr::.n ~ ::~nrl f'Pfnrovi n1P in nrPPn::.nt worn Pn ::~nrl

8
Lock
s
Suspend
0
End Block
ca nerve pa
25 migratory myalgias/transicnt arthritis.
26
Stage 3-late disseminated: encephalopathies,
27
chronic arthritis.
28
29
30
31 FA17 p 502.2
32
Facial nerve lesions
33
34
Upper mot or neuron Destruction of motor cortex or connection
35 lesion between motor cortex and facial nucleus in
36 pons - contralateral paralr~is of IO\\er muscles
37 of facial expression. Forehead is spared due to
38 its bilateral U~lt innervation. tr«t
39 Lower motor neuron Destruction of facia I nucleus or C 1 VII
40
lesion anywhere along its course - ipsilatera l
41
paralysis of upper and lower muscles of
42
facia I expression fi), hypcracusis, loss of taste
43
sensation to anterior tongue.
44
When idiopathic (most common), facial nerve
45
• 46
palsy is called Bell palsy. May also be caused
• 47
by Lyme disease, herpes simplex, herpes
. 48 zoster (Ramsay Hunt syndrome), sarcoidosis,
. 49 tumors (eg, parotid gland), diabetes mellitus.
• 50 Treatment is corticosteroids, acyclo\ ir. \ lost •
8
L.odt
s
Su~pl'nd
~
End Block
Lower motor neuro n Destruction of fa cia I nucleus or C
25
26
lesion anywhere along its course - ipsilatera l
27
paralysis of upper and lower muscles of _ .. __ U4N

28 facial expression rJ, hypcracusis, loss of taste lt1oon

29 sensation to anterior tongue.


30 When idiopathic (most common), facial nen e
31 palsy is called Bell palsy. .\lay also be caused
32 by Lyme disease, herpes simplex, herpes
33 zoster (Ramsay Hunt syndrome), sarcoidosis,
34 tumors (eg, parotid gland), diabetes mellitu .
35 Treatment is corticosteroids, acyclO\ ir. \lost
36 patients gradually recover function.
37
38
39 FA17 p 142.2
40 Spirochetes Spiral-shaped bacteria · with axial filam ents. B.LJ'.
41 Borrelia is Big.
Includes Borrelia (big size), Leptospira, and
42
A'~ . Treponema. Only Borrelia can be visua lized
43
using aniline dres (Wright or C iemsa stain)
44 •
~ in light microscopy due to size. Treponema is
~,,
45
visualized by dark-field microscopy or eli rect
' 46
fluorescent antibody (DFA) microscopy.
' 47 • •

. 48
. 49
..
. 50 •
8
Lode.
s
Suspe-nd
8
End Bloc:k
Item: 46 of - ,• Mark -<J [:::> "'I ~ · ~
100 ~ P~v1ous N @xt Labl lues N o tes Calcula to r

A child who suffers from an immunosu ppressive disease caused by deficiency In adenosine deaminase is brought to t he pediatrician for his ~~AI
25
vaccinations.
26
27
This patient should be given only which of the following vaccines?
28
:
29
A. Measles
30
31
B. Mumps
32 C. Rubella
33
D. Salk polio
34
35
E. Varicella
36
37
38
39
40
41
42
43
44
45
. 46
• 47
. 48
. 49
. 50 •
8
Lode.
s
Suspe-nd
8
End Bloc:k
Item: 46 of ~ ,• Mark <::J [:::> ""I ~· ~'j
100 J.. Previous Next LAbfaiUI~S Notes Calculator

25
26
The correct an swer is 0. 74°/o chose t his.
27
The Salk polio vaccine is a killed vaccine, mean ing t hat it ind uces hu mora l immunity only, and it
uve vacetnes contratndleatect In lmmunosupprMsed Patients
28 does not have the ability t o replicate. Li ve attenuat ed vaccines induce both humoral and cell - 1nnuenza (lnlranasal)
29 mediated immunity, but are more dangerous for immunosuppressed pat ients because they can Measles, mu mps. rubella (MMR)
cause illness. The ot her vaccine for polio is t he Sabin vaccine, which is given orally and is a live- PoliO (Sabin (oral))
30 Rotavirus
att enuat ed vaccine. The table shows t he vaccines that are cont raindicated in immunosuppressed rypno10 (oral)
31 patients. Smallpox
Varicella
32 Humoral immunity Polio vaccine Poliomyelitis Cell-mediated immunity Attenuated vaccine Immunosuppression Vaccine Contraindication Yellow fever
Zoster
33 Immunity (medical) Humorism

34
A is n ot correct . 5°/o ch ose this.
35 Measles is a live at tenuated vaccine and is therefore cont raindicated in t he immunosu ppressed . Measles, m umps, rubella (MMR) vaccination is
36 cont raindicated in patient s with symptoms of immunodef iciency, includ ing bot h congenital (as in t his pat ient ) and acquired (eg, AIDS) fo rm s.
37 However, MMR is not cont raindicated in HIV-positive indiv iduals who do n ot show symptoms of immunodef iciency. This condition occurs in
t he earl y stages of infect ions when CD4+ T-cell count is still high.
38 Mumps Measles Attenuated vaccine Immunosuppression Rubella T cell T helper cell HIV/AIDS Immunodeficiency Vaccination Vaccine HIV Contraindication Congenital disorder CD4 MMR vaccine
39
B is not correct. 4 % chose this .
40
Mumps is a live attenuated vaccine and is t herefore cont raindicat ed in t he immunosu ppressed . Not e that measles, mumps, and rubella (MMR)
41 vaccinat ion is not cont ra indicat ed in HIV-posit ive pat ient s who do not show symptoms of immunodef iciency (eg, low CD4+ T-cell count ).
42 Measles Mumps Attenuated vaccine Immunodeficiency Immunosuppression T helper cell Rubella T cell vaccination vaccine HIV Contraindication CD4

43
C is not co r r ect. 5°/o chose this .
44 Rubella is a live att enuat ed vaccine and is t herefore cont raindicated in the immunosuppressed. Note t hat measles, mum ps, and rubella (MMR)
45 vaccinat ion is not cont ra indicat ed in HIV-posit ive pat ient s who do not show symptoms of immunodef iciency (eg, low CD4+ T-cell count ).
Measles Mumps Attenuated vaccine Rubella Immunodeficiency Immunosuppression T helper cell T cell vaccination vaccine HIV Contraindication CD4
46
• 47 E is n ot correct. 1 2°/o ch ose this •
. 48 Vari cella is a live at tenuated vaccine and is therefore contraindicat ed in t he immunosu ppressed .
Attenuated vaccine Varicella vaccine Immunosuppression Vaccine Chickenpox Contraindication
. 49
• 50 •
8
Lock
s
Suspend
0
End Block
Item: 46 of - ,• Mark -<J [:::> "'I ~ · ~
100 ~ P~v1ous Labl lu es No tes Calcula t o r
. .. . N@xt
..

25 Varicella is a live attenuated vaccine and is t herefore contraindicated In the immunosuppressed.
Attenuated vacc1ne Vancella vaccme Immunosuppression Vaccine Chickenpox Contraindication
26
27
28 Bottom Line:
29 The Salk inactivated (killed) poliovirus vaccine may be given to an immunocompromised patient. The measles, varicella, mumps, and rubella
30 vaccines are all attenuated live vaccines, and may cause illness in patients with compromised immune systems.
'-te,.-1~ o.t "'lP Jo 10 rus Ion.nunodefic•ency Vacc.ne Varicella vaccine Rubella Chic'<enpo Po 10 "W:cme
31
32
33
141ifil·1i•l
fl llt.ST A IO fA CTS
for year: 2017 •
34
35
36 FA17 p 158.3

37 Viral vaccines
38 Live attenuated \11 IR, Yellow fever, Rotavirus, lnAuenza "Music and LYR ICSS are best en jo)ed Li,c."
39 vaccines (intranasal), Chickenpox (VZV), Sma ll pox, MMR = measles, mumps, rubella; Ji,·e
40 Sabin polio virus. allenu<~l"ed vaccine that can be given to
41 HIV ® patients who do not show signs of
42
immunodcficicncy.
43
44
Killed Rabies, Influenza (injected), Sa lk Polio, and SalK = Killed.
45 HAV vaccines. Killed/inacli,·ated vaccines RIP Always.
46 induce only humoral immunity but are stable.
• 47 Subunit llBV (antigen = llBsAg), I I PV (types 6, II, 16,
. 48 and 18).
. 49
. 50 •
8
Lode.
s
Suspe-nd
8
End Bloc:k
Item: 47 of - ,• Mark -<J
P~v1ous
[:::> "'I ~ ·· ~
100 ~ N @xt Labl lues N o tes Calcula to r

25 A 68-year-old man who smokes and is alcoholic abruptly develops high fever, shakes, a severe headache, and abdom inal and muscle pain. ~~AI
He initially has a dry, insignificant cough, but over t he next few days he develops marked shortness of breat h requiri ng assisted ventilation.
26
X-ray of the chest demonstrates homogenous radiog raphic shadowing involving both the lungs extensively. Culture of bronchoalveolar
27 lavage fluid on buffered charcoal yeast extract demonst rates a bacillary pathogen.
28
29 What is the most likely causative organism?
30
31 A. Chlamydia pneumoniae
32 B. Legionella pneumophila
33
C. Listeria monocytogenes
34
35 D. Staphylococcus aureus
36 E. Streptococcus pneumoniae
37
38
39
40
41
42
43
44
45
46
. 47
. 48
. 49
. 50 •
8
Lode.
s
S uspe-nd
8
End Bloc:k
Item: 47 of ~ ,• Mark <::J [:::> ""I ~· ~'j
100 J.. Previous Next faiUI~S
LAb Notes Calculator

25 The correct a ns wer is B. 8 4 % chos e t his .


26 The pat ient has a severe, potent ia lly fat al, pneumonia wit h prom inent syst em ic sympt oms. Cult ure on buffered charcoa l yeast extract is t he
27 specif ic clue t hat t he organism is Legionel/a pneumophila. The disease is respi ratory legionellosis, also known as leg ionnaires' disease, so
28
named because t he disease was f irst described when it occurred in epidem ic fo rm after an American Leg ion convent ion at a Philadelphia hotel.
Hyponatremia is a common lab abnormality associat ed with legionna ires' disease and is more often present in legionnaires' disease than in
29 other pneumon ias. Furthe rmore, pat ients wit h legionnaires' disease tend to be older and may have risk factors, including cigarette use,
30 alcoholism, diabet es, ch ron ic illness, or immunosuppressive t herapy. A urine ant igen t est is now also ava ilable fo r legionnaires' disease.
Legionnaires' disease Hyponatremia Legionella pneumophila Legionella Pneumonia Diabetes mellitus American Legion Immunosuppression Alcoholism EUStl. Chronic condition Yeast Antigen Yeast extract
31
Buffered charcoal yeast extract agar Epidemic Charcoal
32
33 A is not correct . 4 °/o chos e this .
34 Chlamydia pneumoniae (also known as Chlamydophila pneumoniae) is a common cause of atypical pneumonia . It presents sim ilarly to ot her
35
causes of atypical pneumonia, making it difficult to diagnose clinically. I n addit ion, most labs are not equipped to culture C. pneumoniae, as it
requires specia l cell culture technique. For t hese reasons, most cases of C. pneumoniae must be t reat ed empiri cally. I n cases where
36 Chlamyd ia infect ion is known, doxycycline is t he t reatment of choice .
37 Doxycycline Chlamydophila pneumoniae Ptypical pneumonia Pneumonia Chlamydia infection Chlamydophila Chlamydia (genus) Cell culture Infection

38 C is not co rrect. 8 °/o chose this .


39 Listeria monocytogenes causes list eriosis and is not a not able cause of pneumon ia. Patient s with list eriosis (older adults, pregnant women,
40 newborns and immunocompromised pat ients) typically present wit h feve r and muscle aches preceded by dia rrhea and/or gast rointest inal
symptoms. Pregnant women t ypically present wit h nonspecif ic sym pt oms, such as fatigue and body aches. However, infections duri ng
41
pregnancy can cause st illbirths, m iscarriage, premat ure delivery, or infect ion of newborns.
42 Listeria monocytogenes Listeriosis Immunodeficiency Pneumonia Listeria Diarrhea Preterm birth Myalgia Fatigue (medical) Nonspecific symptoms Miscarriage Pregnancy Gastrointestinal tract Muscle Fever

43 Human gastrointestinal tract Stillbirth

44
D is not correct. 2°/o chose t his.
45
Staphylococcus aureus can cause pneumonia but is easily cultured on routine media. Pat ients wit h S. aureus pneumonia t ypically present with
46 a sh01t prod rome of feve r followed by t he onset of respirat ory issues. A clinical hist ory of alcoholism may keep S. aureus on your diffe rential
47 diagnosis fo r cause of pneumonia, but t he key here is t he cult uri ng process, as noted in the vignett e.
Pneumonia Staphylococcus aureus Staphylococcus Differential diagnosis Alcoholism Medical history Fever Prodrome
. 48
. 49 E is not correct. 2 °/o chos e this .
• 50 Streptococcus pneumoniae can cause pneumon ia but is easily cultured on rout ine media. Pneumococcal disease can occur across the world, •

8
Lock
s
Suspend
0
End Block
Item: 47 of ~ ,• Mark <::J [:::> ""I ~· ~'j
100 J.. Previous Next faiUI~S
LAb Notes Calculator

25
E is not correct. 2°/o chose this.
Streptococcus pneumoniae can cause pneumonia but is easily cultured on rout ine media. Pneumococcal disease can occur across the world,
26
and high-ri sk populations include children younger t han 2 years, adults over the age of 65, and people with weakened immune systems. The
27 clinical pict ure alone would not rule outS. pneumoniae, but t he info rmation about t he cult uring mechanism used points away f rom
28 pneumococcal disease here.
Streptococcus pneumoniae Pneumonia Streptococcus Immunosuppression Immunodeficiency
29
30
31 Bottom Line:
32 Legionel/a pneumophila causes pneumonia in older pat ients with a history of alcoholism, tobacco use, diabetes, chronic illness, or
33 immunosu ppression, and can be cult ured on buffered charcoal yeast extract.
Legionella pneumophila Legionella Pneumonia Immunosuppression Yeast extract Diabetes mellitus Yeast Alcoholism Chronic condition Tobacco Buffered charcoal yeast extract agar
34
35
36
37
141;fil·1i•J for year:[2017
FIRST AID FA C T S .
•j .

38
39 FA17 p 139.2
40
Legion ella Gram 8 rod. Gram stains poorly- use silver Think of a French legionnaire (soldier) with
41
pneumophila stain. Grow on charcoal yeast extract his silver helmet, sitting around a campfire
42
medium with iron and cysteine. Detected by (charcoal) with his iron dagger- he is no sissy
43
presence of antigen in urine. Labs may show (cysteine).
44
hyponatremia. Aerosol transmission from
45
46
environmental " ·ater source habitat (eg, air
47 conditioning systems, hot water tanks). o
. 48 person-to-person transmission .
. 49 Treatment: macrolide or quinolone .
• 50 •
Legionnaires' disease- severe pneumonia
8
Lock
s
Suspend
0
End Block
Item: 47 of - ,• Mark --<) [::> ""'I ~· 1!';:'1
100 ~ Prev1o u s Next Labf a lu es Notes Calculator

25
FA1 7 p 123.1

26 Special culture requirements


27 BUG MEDIA USED FOR ISOlATION MEDIA CONTENTS/OTHER
28 Hinfluenzae Chocolate agar Factors V (NAD+) and X (hematin)
29
N gonorrhoeae, Tha~er-Martin agar elect ivel) favors growth of I eisseria by
30
N meningitidis inhibiting growth of gram$ organisms
31
32
'' ith Vancomycin, gram 8 organisms except
1eisseria
33
with Trimethoprim and Colistin,
34 and fungi with '\ystatin
35 Very Trpically Cultures :'\eisseria
36 Bpertussis Bordet-Gengou agar (Bordct for Bordetella) Pol ato extract
37 Regan-Lowe medium Charcoal, blood, and antibiotic
38
Cdiphtheriae Tcllurite agar, Lofner medium
39
40 M tuberculosis Lowcnstein-Jcnscn agar
41 M pneumoniae Eaton agar Requires cholesterol
42
Lactose-fermenting lacConkey agar l'crmenlalion produces acid, causing colonies to
43
44
enterics turn pink
45 Ecoli Eosin-methylene blue (EM B) agar Colonies with green metallic sheen
46
Legionella Charcoal yeast extract agar buffered with
47
cysteine and iron
. 48
. 49 Fungi Sabouraud agar "S
• a b'sa f un guyI"
.
• 50 •
8
L.odt
s
Su~pl'nd
~
End Block
Item: 47 of ~ ,• Mark <::J [:::> ""I ~· ~'j
100 J.. Previous Next LAbfaiUI~S Notes Calculator

FA17 p 175.2
25
Common causes of pneumonia
26
NEONATES(< 4WK) CHILDREN (4 WK-18 YR) ADULTS (18- 40 YR) ADULTS (40- 65 YR) ELDERLY
27
28 Group B streptococci Viruses (RSV) Mycoplasma S pneumoniae S pneumoniae
29 E coli Mrcoplasma C pnewnoniae 1I influenzae InAuenza virus
30 C trachomatis S pnewnoniae Anaerobes Anaerobes
31 (infants-3 yr) Viruses (cg, in Auenza) Viruses H influenzae
32 C pnewnoniae Mycoplasma Cram 8 rods
33 (school-aged
34 children)
35
S pneumoniae
36
Runts May Cough
37
C hunky Sputum
38
Special groups
39
40 Alcoholic Klebsiella, anaerobes usually due to aspiration (eg, Peptostreptococcus, Fusobacterium, Prevotelfa,
41 Bacteroides)
42 IV drug users S pneumoniae, S aureus
43
Aspiration Anaerobes
44
45 Atypical Mycoplasma, Legionella, Chlamydia
46
Cystic fibrosis Pseudomonas, S aureus, S pneumoniae, Burkholderia cepacia
47
• 48
lmmunocompromised S aureus, enteric gram 8 rods, fungi, viruses, P iirovecii (wiH1 II IV)
. 49 Nosocomial (hospital S aureus, Pseudomonas, other enteric gram 8 rods
• 50 acauiredl
8
Lock
s
Suspend
0
End Block
Item: 48 of - ,• Mark -<J [:::> "'I ~ · ~
100 ~ P~v1ous N @xt Labl lues N o tes Calcula to r

A 30-year-old sexually active woman presents t o her primary care physician with a painf ul genital lesion. She says th e lesion began as a ~~AI
25
red "bump" that subsequently became f illed wit h pus. On exam, you note an ulcerated lesion on t he labia majora and bilat eral inguinal
26
lymphadenopathy. A Tzanck smear from the lesion is negative, and a Venereal Disease Research Labora tory assay is also negative.
27
28 Which of the following medications would be most appropriate for this patient?
29
:
30 A. Acyclovir
31
B. Ceftriaxone
32
33 C. Foscarnet
34 0 . Ribavirin
35
E. Vancomycin
36
37
38
39
40
41
42
43
44
45
46
47
. 48
. 49
. 50 •
8
Lode.
s
S uspe-nd
8
End Bloc:k
Item: 48 of - ,• Mark -<J [:::> "'I ~ · ~
100 ~ P~v1ous N @xt Labl lues N o tes Calcula to r
A A

25
The correct answer is B. 6 4 % chose this.
26
The differential diagnosis of a genital ulcer in a sexually act ive patient should include primary sy philis
(t hough these ulcers are usually painless), genit al herpes, lymphogranuloma venereum (which also
27 manifests with painless ulcers), and chancroid . Because this pati ent's ulcers are painful and t he Tzanck
28 smear (which looks for multinucleated giant cells typical of herpes infection-image shows posit ive Tzanck
test) is negative, as is the VORL t est for syphilis, chancroid becomes most likely. Chancroid is a bacterial

29
infection caused by Haemophilus ducreyi, which manifest s typically as a painful genital ulcer with associated
30 inguinal lymphadenopathy. It is typically trea ted wit h ceftriaxone given as a one-time, 250-mg,
31 intramuscular injection, or with azithromycin as a single 1000- mg dose.
Lr'IP~oo• 'M'II Of"!\1 rt.1ereu1 Ge111tc. ulcer Chancroid AzithroO'I)Cin Ceftriaxone Syph1 s ~hMenor ....., uaernophllu5 ducrf!'(l Gerutal herpes
32
Ulcer (dermatology) Intramuscular onJecbon O.fferenbal doagnosos Herpes somplex Pathogen1c bact1!1'1a Pnmary syph1hs Ute..-
33
I mage courtesy of National
34 I nstitutes of Health
35
36
A is not correct. 19% chose this.
37
Acyclovir is an antiviral agent used to treat herpes infections. It is activated by viral thymidine kinase, whereupon it inhibits t he herpes viral
polymerase. It can be used to treat herpes simplex virus types 1 and 2, varicella-zoster virus, and Epst ein -Barr virus infect ions.
38 Epste1n~Barr v~rus AclciOVIr Thym1dme k1nase Herpes simplex virus varicella zoster v1rus Thym1d1ne Herpe$ $1mple) Ant1v1ral drug K1nase RNA- dependent RNA polymerase Shingles Virus

39
C is not correct. 7°/o chose this.
40
Foscarn et inhibit s vira l DNA polymerase without t he need of act ivation by t hymidine kinase. It is used t o t reat cytomegalovirus ret initis, but it
41 can also be used to t rea t acyclovir- resistant herpes simplex virus.
42 Cytomegalovirus Foscarnet Thym1d1ne k1nase Cytomegalovirus retinitis Herpes simplex virus DNA polymerase Thymid1ne Herpes s1mplex DNA Virus Polymerase Retinitis

43 D is not correct. 4°/o chose this.


44 Riba virin is used to t reat respiratory syncyt ial virus. It f unct ions by inhibiting inosine monophosphate dehydrogenase, thus blocking the
45 synt hesis of guanine nucleot ides.
IMP dehydrogenase R1bav1nn Inosm1c ac1d Human respiratory syncytial virus Guamne Inosine Nucleotide Virus 9)tncyt1um
46
47 E is not correct . 6°/o chose this.
48 Vancomycin is a bacteri cidal ant ibioti c used for mult idrug-resistant gram-positive organisms such as Staphylococcus aureus and Clostridium
. 49
difficile. It functions by binding to mucopeptide precursors, preventing formation of the bact eri al cell wall .
Vit.r'lcomyctn Shtphytococcus aureus Antibiotics Gram-positive bacteria Cell wall Clostndium dtfFicde col1b Clost• dtum d1ff1t1le (bi:Ktena) staphylococcus Bactericide Mulbple drug res1stance Clostt•dtum
. 50 •
8
Lode.
s
S uspe-nd
8
End Bloc:k
Item: 48 of ~ ,• Mark <::J [:::> ""I ~· ~'j
100 J.. Previous Next LAbfaiUI~S Notes Calculator

25
Bottom Line:
26 I n a sexually act ive patient wit h a genit al u lce r~ one should consider the possibilit y of primary syphilis, genit al herpes, or chancroid.
Ceftriaxone is a recommended t reat ment for chancroid, wit h cure rates as high as 98% .
27 Genital ulcer Chancroid Ceftriaxone Syphilis Genital herpes Ulcer (dermatology) Ulcer Herpes simplex Primary syphilis Peptic ulcer
28
29
30 l@ljl'il·1i•J for year:[ 2017 • J
FIRST AID FA CTS
31
32
FA17 p 180.1
33
34
Sexually transmitted infections
35 DISEASE CLINICALFEATURES ORGANISM
36 AIDS Opportunistic infections, Kaposi sarcoma, HIV
37 lymphoma
38
Chancroid Painfu l genita l ulcer with exudate, inguinal Haemophilus ducreyi (it's so painful, you "do
39
adenopathy cry")
40
41 Chlamydia Urethritis, cervicitis, epididymitis, Chlamydia trachomatis (0 - K)
42 conjunctivitis, reactive arthritis, PI D
43 Condylomata Genital warts, koilocytes HPV-6 and -11
44 acuminata
45
Genital herpes Painful penile, vulvar, or cervical vesicles and HSV-2, less commonly HSV-1
46
47
ulcers; can cause system ic symptoms such as
48
b·er, headache, myalgia
. 49 Gonorrhea Urethritis, cervicitis, PI D, prostatitis, eisseria gonorrhoeae
• 50 eoidiclvmitis. arthritis. creamv ourulent
8
Lock
s
Suspend
0
End Block
Item: 48 of - ,• Mark --<) [::> ""'I ~· 1!';:'1
100 ~ Prev1o u s Next Labf a lu es Note s Calculator

25
FA17 p 143.1

26 Syphilis Caused by spirochete Treponema pallidum.


27 Primary syphilis Localized disease presenting " ith painless chancre tl lf available, use dark-fi eld microscopy to
28 visualize treponemes in Auid from chancre [lJ. DRL EEl in ~ 80%.
29
Secondary syphilis Disseminated disease with constitutional S) mptoms, maculopapular rash B (including palms [!]
30
and soles}, condylomata lata D (smooth, moist, painless, wart-like white lesions on genitals},
31
32
lp11phadenopathy, patchy hair loss; also confirmable " ith dark-field microscopy.
33
Serologic testing: DRURPR (nonspecific}, confirm diagnosis with specific test (eg, FTA- BS).
34 Secondary syphilis= Systemic. Latent syphilis (EEl serology without S) mptoms) may folio\\.
35 Tertiary syphilis Gummas (chronic granulomas), aortitis (\'asa ,·asorum destruction), neurosyphilis (tabes dorsalis,
36 "general paresis"), Argyll Robertson pupil (constricts with accommodation but is not reacti,·c to
37 light; also called "prostitute's pupil" since it accommodates but does not react).
38 Signs: broad-based ataxia, EEl Romberg, Charcot joint, stroke without hypertension.
39
F'or neurosyphilis: test spinal Auid with VORL, FTA-ABS, and PCR.
40
41
Congenital syphilis Presents with facial abnormalities such as rhagades (linear scars at angle of mouth, black arrow
42
in [!1), snufAes (nasal discharge, red arrow in [!1), saddle nose, notched (Hutchinson) teeth CJ,
43 mulberry molars, and short maxi lla; saber shins; C VI II deafness.
44 To prevent, treat mother early in pregna ncy, as placental transmission typically occurs after first
45 trimester.
46
47
48
. 49
• 50 •
8
L.odt
s
Su~pl'nd
~
End Block
Item: 48 of - ,• Mark --<) [::> ""'I ~· 1!';:'1
100 ~ Prev1o u s Next Labf a lu es Note s Calculator

FA1 7 p 185.1
25
26
Cephalosporins (generations 1-V)
27 MECHANISM ~-lacta m drugs that inhibit cell wall synthesis Organisms typically not covered by 1st-4th
28 but are less susceptible to penicillinases. generation cephalosporins are LA:\ IE:
29 Bactericida I. Listeria, .\typicals (Chlamydia, Mycoplasma ),
30 ~t RSA, and E nterococci.
31
(LINI(Al USE 1st generation (cefazolin, cephalexin)-gram@ lsi generation- PEcK.
32
33
cocci, Proteus mirabilis. E coli, Klebsiella
34 pneumoniae. Cefazolin used prior to surger) to
35 pre,·ent S aureus wound infections.
36 2nd generation (cefa clor, cefo\ilin, Fake fox fur.
37 cefuroxime)- gram @ cocci, 11 influenzae, 2nd generation- HENS PEcK
38 Enterobacter aerogenes, 'leisseria ~pp., Serratia
39 marcescens, Proteus mirabilis, E coli, Klebsiella
40 pneumoniae.
41 3rd generation (ceftriaxone, cefotaxime, Can cross blood-brain barrier.
42
cefpodoxime, ceftazidime)-scrious gram e Ceftriaxone- meningitis, gonorrhea,
43
infections resistant to other ~-lactams . disseminated Lyme disease.
44
Ceflazid i me- Pseudomonas.
45
46
4th generation (cefepime)- gram organisms, e
47
with f activity against Pseudomonas and gram
48
@organisms.
. 49 5th generation (ceftaroline)-broad gram @ and
• 50 gram e organism coverage; unlike 1st-4th •
8
L.odt
s
Su~pl'nd
~
End Block
Item: 49 of - ,• Mark -<J [:::> "'I ~ · ~
100 ~ P~v1ous N @xt Labl lues N o tes Calculato r

25 A 43-year-old man present s 3 days after returning from a rafting t rip to South America. Duri ng this t ri p he was bit ten by mosquitos. He did ,..
not eat any shellfish or raw vegetables and only consumed bottled water. He denies any sexual intercourse or drug use duri ng this trip. A
AI
26 and pain in his knees and lower back. He says th at his appetite
Now he complains of flu-l ike symptoms as wel l as headache, photophobia,
27 has been very poor. He has been nauseated and has vomited dark blood on two occasions. He is febrile at 39°C (102.2°F) with a pulse of 40
28 bpm. His face is flushed and his conj unctivae are red. On abdominal examination, he is tender in t he epigastrium and his liver edge is palpable 3
em below the costal margin. A complete blood count and liver function tests show:
29
30
WBC count: 2000/mmJ
Neutrophils: 40%
31 Aspartate aminotransferase: 900 U/L
32 Alanine aminotransferase: 350 U/L
33 The patient's condition worsens and he is hospitalized. A liver biopsy is performed. The specimen is shown.
34
35
36
37
38
39
40
41
42
43
44
45
46 Image cowtesy of Wikimedia Commons
47
48 What is this patient's most likely diagnosis?
. 49 :
. 50 A. Dengue fever

8
Lode.
s
S uspe-nd
8
End Bloc:k
Item: 49 of - ,• Mark -<J [:::> "'I ~ · ~
100 ~ P~v1ous N @xt Labl lues N o tes Calculato r

A Neutrophlls: 40% A

25 Aspartate aminotransfera se : 900 U/L


26 Alanine aminotransferase: 350 U/L
27 The patient's condition worsens and he Is hospitalized. A liver biopsy Is performed. The specimen is shown.
28
29
30
31
32
33
34
35
36
37
38
39
40 Image courtesy of Wikimedia Commons
41
42 What is this patient's most likely diagnosis?
43 :
44 A. Dengue fever
45 B. Hepatitis A
46
C. Hepatitis B
47
48 D. Malaria
. 49 E. Yellow fever
. 50

8
Lode.
s
S uspe-nd
8
End Bloc:k
Item: 49 of ~ ,• Mark <::J [:::> ""I ~· ~'j
100 J.. Previous Next LAbfaiUI~S Notes Calculator

25 The correct a nswer is E. 4 9°/o chose this .


26 This pat ient has yellow fever, caused by a flav ivirus, a single-st ra nded RNA virus t ra nsmit ted via a mosquito vect or. I nit ial symptoms are
27 nonspecif ic. One cla ssical present at ion seen in yellow feve r, among ot her illnessess, is t he Faget sign: fever coinciding wit h bradycardia.
28
Leukopenia is present in t he earl y phases of yellow fever infection, followed by elevat ion in t ransami nases and direct bilirubin . I n cont rast t o
other viruses affect ing t he liver, aspart at e aminot ransferase is elevat ed to a greater extent t han alanine aminot ra nsfe rase, due to vira l
29 damage to myocytes (skeleta l muscle) in addit ion t o hepatic cells. Hepat ocellular damage is charact erized by eosinophilic degenerat ion and
30 condensed nuclear chromat in, or Councilman bodies (shown in the black box in t he vignet te image). Specif ically, involvement of zone 2 of t he
31
liver acinus is commonly seen in yellow feve r.
Flavivirus Leukopenia Yellow fever Bilirubin Alanine transaminase Aspartate transaminase Bradycardia Transaminase Faget sign Mosquito Virus Fever RNA virus Liver Vector (epidemiology) RNA
32
Lobules of liver Infection
33
34 A is not correct . 26% chose this .
35
Dengue fever is t ransmitted by mosquitoes, and alt hough it belongs to t he flav ivirus family, it is actually an arbovirus (given its arthopod-
mediated transmission). I nitially, patients are asymptomat ic or may present wit h nonspecific fevers, beginning on day 3 and lasting for up to
36 7 days. Classic dengue disease is accompanied by high feve rs, chills, facial f lushing, and erythemat ous mott ling of t he skin. Pat ient s usually
37 have a hist ory of t raveling abroad. A liver biopsy is not warranted in t his disease, and t he patient described in t he vignette present s with
38
symptoms atypical of dengue fever. Therefore, yellow feve r is t he best answer choice.
Flavivirus Arbovirus Dengue fever Yellow fever Biopsy Mosquito Asymptomatic Liver Erythema Fever
39
40
B is not correct. 7% chose t his .
Hepatitis A is caused by a picornavirus (single-st randed RNA virus) . Acut e vira l hepatit is causes equivalent elevations in alanine
41
aminot ransferase and aspartate aminot ra nsferase, and ballooning degeneration of hepat ocytes. Councilman bodies may be seen. Hepat it is A
42 is t ransmitted v ia t he feca l-oral rout e and risk facto rs include eat ing raw shellfish and drinking water contaminat ed with feca l mat ter.
43 Picornavirus Alanine transaminase Aspartate transaminase Fecal-oral route Hepatitis A Hepatitis Viral hepatitis Ballooning degeneration Hepatocyte RNA virus Virus Feces RNA Shellfish

44 C is not co rrect. 6°/o chose this .


45 Hepadnavirus is a double-st randed DNA virus t hat causes hepatit is B. Acute vira l hepatit is causes equivalent elevations in alanine
46 aminot ransferase and aspartate aminot ra nsferase, and ballooning degeneration of hepat ocytes. Councilman bodies may also be seen.
Hepatitis B is t ransmitted via blood or body fluids and risk fact ors include I VDU, unprotect ed sexual intercourse and needlestick inj uries.
47
DNA virus Alanine transaminase Aspartate transaminase Hepatitis 8 Hepadnaviridae Ballooning degeneration Viral hepatitis Hepatitis Needlestick injury Hepatocyte Sexual intercourse Virus DNA Aspartic acid
48
Alanine
49
• 50
D is not correct. 1 2% c hose this .

8
Lock
s
Suspend
0
End Block
Item: 49 of ~ ,• Mark <::J [:::> ""I ~· ~'j
100 J.. Previous Next faiUI~S
LAb Notes Calculator

Alamne

25
D is n ot cor rect. 1 2% c h ose this.
26 Malaria is caused by t he plasmodium species. It is t ransmitt ed by t he anopheles mosquito. Common cl inical ma nifest at ions are fever,
27 headache, splenomegaly. I n cont rast, hepatic involvement is less common t han that see in dengue and yellow feve r.
Yellow fever Splenomegaly Malaria Plasmodium Anopheles Mosquito Dengue fever Fever Headache Liver Species
28
29
30 Bottom Line:
31 Yellow fever is caused by a flav ivirus and is characterized by headache, photophobia, and musculoskelet al pain.
32 Flavivirus Yellow fever Photophobia Headache Fever

33
34
35 i@l;fil·1i•J f or yea r:[ 2017 • J
FIRST AID FACTS
36
37 FA17 p 164.5
38
39
Yellow fever virus A Aavivirus (also an arbovirus) transmitted by Flavi =yellow, jaund ice.
40 Aedes mosquitoes. Virus has a monkey or
41 human reservoir.
42 Symptoms: high fever, black vomitus, and
43 jaundice. May see Councilman bodies
44 (eosinophilic apoptotic globules) on liver
45 biopsy.
46
47
48 FA 17 p 163.1
49 RNA viruses
• 50 VIRAL FAMILY ENVELOPE RNA STRUCTURE CAPSID SYMMETRY MEDICAL IMPORTANCE

8
Lock
s
Suspend
0
End Block
Item: 49 of ~ ,• Mark <::J [:::> ""I ~· ~'j
100 J.. Previous Next LAb faiUI~S Notes Calculator

FA17 p 163.1
25
RNA viruses
26
VIRAL FAMILY ENVELOPE RNA STRUCTURE CAPSID SYMMETRY MEDICAL IMPORTANCE
27
28 Reoviruses 0 OS linear Icosahedral Coltivirus"- Colorado tick fever
29
10-12 segments (double) Rotavirus-cause offatal diarrhea in children
30 Picornaviruses No SS ®linear Icosahedral Poliovirus-polio-Salk/Sabin vaccines-IPV/OPV
31 Echovirus- aseptic meningitis
32 Rhi novirus-"common cold"
33 Coxsackievirus- aseptic meningitis; herpangina
34 (mouth blisters, fever); hand, foot, and mouth
35 disease; myoca rditis; pericarditis
36 IIAV-acute viral hepatitis
37 PERCI I
38
Hepevirus No SS ®linear Icosahedral HEV
39
40 Caliciviruses No SS ® linear Icosahedral 'orovirus-vira l gastroenteritis
41
Flaviviruses Yes SS ® linear Icosahedral HCV
42
Yellow fever1
43
Dengue3
44
St. Louis encephalitis3
45
West Tile virus3 (meningoencephalitis)
46
Zika virus
47
48 Togaviruses Yes SS ®l inear Icosahedral Rubella
49 Western and Eastern equine encephalitis"
• 50 Chikungunya ,·irus •

8
Lock
s
Suspend
0
End Block
Item: SO of - ,• Mark -<J [:::> "'I ~ · ~
100 ~ P~v1ous N @xt Labl lues N o t es Calcula to r

A 3-year-old girl present s to the pediatrician for her annual well child visit. She is developing well and her mother has no concerns at this ~~AI
25
time. Her temperature is 38°C ( 100.4° F} . Physical examination reveals an enlarged, tender, erythemat ous left axillary lym ph node. Close
26
inspection of the skin revea ls a series of small linear scra tches on the left forearm, with a nearby erythemat ous papule.
27
28 Which of the following organisms is t he most likely cause of these findings?
29
:
30 A. Bartonella henselae
31
B. Borrelia burgdo1feri
32
33 C. Eikenella corrodens
34 0 . Francisella tularensis
35
E. Pasteurella multocida
36
37
38
39
40
41
42
43
44
45
46
47
48
49
. 50 •
8
Lode.
s
Suspe-nd
8
End Bloc:k
Item: SO of - ,• Mark -<J [:::> "'I ~ · ~
100 ~ P~v1ous N @xt Labl lues N o t es Calcula to r
A A

25
The correct a ns w e r is A. 79°/o chos e this .
26
This patient presents with a primary inoculation lesion, reg ional lymphadenopathy (see image), and low-
27 grade fever characteristic of cat scrat ch disease, which is caused by Bartonella henselae. Typically (in 60%
28
29
Clll:-.cr~ch d' -e
of cases) infection occurs when a child is scrat ched by a young cat.
Bartonella hen.elae Lymphadenopathy Bartonella Cat Les1on Fever lnfec:bon

30
31
32
33
34
35
36
37
38
39
40
41 Image copyright © Mazur-
42 Melewska eta/. 2015
43
B is not correct. 4 % chos e this.
44
The spiro chete Borrelia burgdorferi is t he cause of Lyme disease. The spirochete is carried by th e Ixodes t ick, which is most common in the
45 north eastern United States. It initially manifest s with an expanding ring -shaped lesion at the site of the tick bite, known as erythema m igrans.
46 Sp1rochaete Lvme d1se e Borrelia burgdorlen Erythema chronicum migrans Erythema Tick Borrelia Ixodes Les1on Un1ted States T1ck-borne disease

47 C is not correct . 3°/o chose this .


48 Eikenella corrodens is a gram-negative organism that is part of the normal flora of the mouth and nasopharynx. It is associated with infections
49 resulting from human bites.
Gr"-rrrneg~; e b~tcte 1a Hu n¥1 ~n~crobtota Nasopharynx Organism Eikenella corrodens
50 •
8
Lode.
s
Suspe-nd
8
End Bloc:k
Item: SO of ~ ,• Mark <:::1 t::> ""I ~· ~'j
100 J.. Previous Next LAbfaiUI~S Notes Calculator

25
D is not correct. 5°/o chose this.
Francisella tularensis is the cause of tularemia. This disease is carri ed by wild rabbit s and t icks in t he sout heastern Un it ed St at es. It often
26
man ifests wit h lymphadenopat hy and an ulcer at t he sit e of entry as well as wit h feve r.
27 Tularemia Francisella tularensis Lymphadenopathy Fever Ulcer (dermatology) United States

28
E is not correct. 9°/o chose this .
29
Pasteurella multocida is caused by cat bit es and dog bit es. This infection causes a rapid inflammation (often
Important zoonotic bacteria transmitted by direel
30 within hours) and is accom panied by cellulit is and purulent drainage. The scratches on t he girl 's arms and contact
31 her ot her symptoms suggest Bartonella henselae is more likely responsible. Main Mode of
Pasteurella multocida Bartonella henselae Cellulitis Bartonella Pasteurella Inflammation Cat Pus Infection Oiieas.e Bacteria
Transmission
32
Cat scratch Bo:lrtoroo/111
33 Cat scratCh
disease ttenselae

34 Pasteurella
Cefulilis Cat bite
mutoeida
35
36
37
Bottom Line:
38
The clinica l picture of a primary inoculation lesion, regional lymphadenopathy, and low-grade fever are highly suggestive of cat scratch
39 disease, caused by the organism Bartonella henselae.
40 Cat-scratch disease Bartonella henselae Lymphadenopathy Bartonella Lesion Cat Fever Organism

41
42
43 l@ljl'il·1i•J for year:[2017 • J
FIRST AID FA CTS
44
45
FA17 p 144.1
46
47
Zoonotic bacteria Zoonosis: infectious disease transmitted between animals and humans.
48 SPECI ES DISEASE TRANSMISSION AND SOURCE
49
Anaplasma spp. Anaplasmosis Ixodes ticks (live on deer and mice)
50

8
Lock
s
Suspend
0
End Block
Item: SO of ~ ,• Mark <:::1 t::> ""I ~· ~'j
100 J.. Previous Next faiUI~S
LAb Notes Calculator

25
FA17 p 144.1
26 Zoonotic bacteria Zoonosis: infectious disease transmitted between animals and humans.
27
SPECIES DISEASE TRANSMISSION AND SOURCE
28
29 Anaplasma spp. Anaplasmosis Ixodes ticks (live on deer and mice)
30 Bartonella spp. Cat scratch disease, bacilla ry angiomatosis Cat scra tch
31
Borrelia burgdorferi Lyme cl isease Ixodes ticks (live on deer and mice)
32
33 Borrelia recurrentis Relapsing fever Louse (recurrent due to variable surface
34 antigens)
35 Brucella spp. Brucellosis/undulant fever Unpasteurized dairy
36
Campylobacter Bloody diarrhea Feces from infected pets/animals; contaminated
37
38
meats/foods/hands
39 Chlamydophila psittaci Psittacosis Parrots, other birds
40
Coxiella burnetii Q fever Aerosols of cattle/sheep amniotic Auid
41
42
Ehrlichia chaffeensis Ehrlich iosis Amblyomma (Lone Star tick)
43 Francisella tularensis Tularemia Ticks, rabbits, deer Aies
44
Leptospira spp. Leptospirosis Animal urine in \\·ater; recreational water use
45
46
Mycobacterium leprae Leprosy ll umans with lepromatous leprosy; armadillo
47 (rare)
48 Pasteurella multocida Cellulitis, osteomyelitis Animal bite, cats, dogs
49
Rickettsia prowazekii Epidemic typhus Human to human vi<l human body louse
50 • •

8
Lock
s
Suspend
0
End Block
Item: Sl of - ,• Mark -<J
P~v1ous
[:::> "'I ~ ·· ~
100 ~ N @xt Labl lues N o tes Calculato r
6

43 A 10-year-old girl living in New Jersey is brought to t he physician because she has had a fever and headache accompanied by vomiting and ~~AI
44 bloody diarrh ea over t he last few days. She has no history of recent travel or sick contacts but has a pet puppy, which t he mot her says has
45 also had diarrhea for t he past week. A stool cult ure incubat ed at 42° C in a microaerophilic environment produces many organisms like
those in the electron micrograph shown here.
46
47
48
49
50
• 51
• 52
. 53
. 54
• 55
• 56
0
57
• 58
. 59
• 60
0 61
0 62
0
63
. 64
. 65 The organism respons ible for this patient's sickness is associat ed with the possible development of which of the following sympt oms?
0 66 :
0 67 A. Acute renal fa ilure and thrombocytopenia wit h hemolytic anemia
0
68 B. Fever, migratory polyarthritis, and carditis
. •
8
Lode.
s
S uspe-nd
8
End Bloc:k
43
44
45
46
47
48
49
50
• 51
• 52
. 53
. 54
• 55
• 56
. 57
• 58
. 59
• 60 The organism respons ible for t his patient's sickness is associated with the possible development of which of t he fo llowing symptoms?
• 61
:
• 62 A. Acute rena l fai lure and t hrombocytopenia with hemolytic anemia
• 63
B. Fever, migratory polyart hrit is, and cardi t is
. 64
. 65
C. Fever, new murmur, small erythemat ous lesions on the palms, and splinter hemorrhages on the nail bed
• 66 D. Petechial rash and bilateral hemorrhage int o t he adrenal glands
• 67
E. Symmetric ascending muscle weakness beginning in the distal lower extremities
• 68

8
L.odt
s
Su~pl'nd
~
End Block
Item: Sl of ~ ,• Mark <::J [:::> ""I ~· ~'j
100 J.. Previous Next LAbfaiUI~S Notes Calculator


43 The correct a nswer is E. 4 9°/o chose this .
44 Guillain-Barre syndrome is characterized by rapidly prog ressing ascending paralysis. It is an aut oimmune- mediat ed illness t hat can occur
following a vari et y of infectious diseases, but is particularly associat ed with prior infect ion by Campylobacter jejuni. C. jejuni gastroenterit is is
45
charact erized in t his patient by her vomiting and bloody diarrhea, t oget her with t he f inding of spiral-shaped organisms with a single polar
46 f lagellum at each end (shown in t he vignette image) when cultured at 42°C in a microaerophilic environment . Ot her enteric pat hogens wit h
47 t his morphology include bact eria of t he Vibrio genus (eg, V. cholera) . These species, however, are not endemic to the Unit ed St at es and would
not be expect ed in a pat ient without a recent t ravel history, nor do t he symptoms mat ch. C. jejuni is t ransmitted to humans via t he fecal-oral
48
rout e from either domest ic animals or eating undercooked poult ry.
49 Campylobacter jejuni Gastroenteritis Microaerophile Fecal-oral route Endemism Flagellum Cholera Campylobacter Diarrhea Paralysis Vibrio Genus Infection Poultry Vomiting Bacteria Pathogen

50 Gastrointestinal tract Morphology (biology) Human gastrointestinal tract United States

51
A is not correct . 15% chose this.
• 52
Hemolytic-uremic syndrome (HUS) is characterized by acut e renal fa ilure and t hrombocytopenia wit h hemolytic anemia. HUS can be a
. 53 com plicat ion of infection caused by Escherichia coli 0 157 : H7, not Campylobacter jejuni. Li ke C. j ejuni, E. coli 0 157 : H7 is a gram- negative rod
. 54 t hat can cause enterit is. E. coli, however, is charact erized by numerous flagella and aerobic metabolism .
Hemolytic-uremic syndrome Campylobacter jejuni Escherichia coli Flagellum Thrombocytopenia Gram-negative bacteria Campylobacter Hemolytic anemia Enteritis Acute kidney injury Anemia
• 55
Aerobic organism Metabolism Cellular respiration Hemolysis Kidney Infection
• 56
• 57 B is not correct. 14% chose this .
• 58 Rh eumatic feve r is characterized by fever, migrat ory polyarthrit is, and cardit is. It is known t o occur after group A streptococcal pharyng it is .
There is no evidence of Streptococcus pyogenes infection in this patient , as she does not have sympt oms of pharyngit is or culture f indings of
. 59
gram-posit ive cocci in chains.
• 60 Streptococcal pharyngitis Streptococcus pyogenes Rheumatic fever Coccus Pharyngitis Polyarthritis Gram-positive bacteria Streptococcus Carditis

• 61
C is not co rrect. 11% chose this .
• 62
Fever, a new murmur, Janeway lesions, and nail bed hemorrh ages are signs of bacterial endocarditis. Acut e endocardit is is most often caused
• 63 by Staphylococcus aureus, and subacute infect ion are oft en caused by viridans group streptococci. Endocarditis is not a known sequela of C.
. 64 jejuni infection .
Sequela Janeway lesion Staphylococcus aureus Endocarditis Streptococcus Staphylococcus Infective endocarditis Bleeding Nail (anato,.,) Heart murmur Fever Infection
. 65
• 66 D is not correct. 11% c hose this .
• 67 Wat erhouse-Frideri chsen syndrome is charact eri zed by high fever, shock, purpura, and adrenal insufficiency. I t is classically associated wit h
Neisseria meningitides sept icemia. This pat ient's symptoms do not resemble meningit is nor does t he culture mat ch t he gram-negat ive
• 68 diplococci of Neisseria
. •
8
Lock
s
Suspend
0
End Block
Item: Sl of ~ ,• Mark <::J [:::> ""I ~· ~'j
100 J.. Previous Next LAb faiUI~S Notes Calculator

43 Bottom Line:
44
Campy/obacter jejuni infection is characteri zed by bloody diarrh ea wit h gram- negative, spiral-shaped organisms on smear. It has been linked
45 t o t he subsequent development of Guillain -Barre syndrome.
46 Campylobacter jejuni Campylobacter Gram- negative bacteria Diarrhea Infection

47
48
49 i@l;fil·1i•J for year:[ 2017 • J
FIRST AID FACTS
50
51 FA17 p 493.2
• 52
. 53
Acute inflammatory Most common subtype of Guillain-Barre Associated with infections (eg, Campylobacter
. 54
demyelinating syndrome. Autoimmune condition that jejuni, viral) ..... autoimmune att<lck of
• 55 polyradiculopathy destroys Schwa1m cells ..... inflammation and peripheral myelin due to molecular mimicry,
• 56 demyelination of peripheral nerves and motor inoculations, and stress, but no definit ive link
• 57 fibers . Results in symmetric ascending muscle to pathogens.
• 58 weakness/paralysis and depressed tendon Respiratory support is critica l until recovery.
. 59 reflexes beginning in lower extremities. Facial Additional treatment: plasmapheresis, 1V
• 60 paralysis in 50% oF cases. lay see autonomic immu noglobulins. No role for steroids.
• 61 dysregulation (eg, cardiac irregularities,
• 62 hypertension, hypotension) or sensory
• 63 abnormalities. Almost all patients survive; the
. 64 majority recover completely after weeks to
. 65
months.
• 66
f CSF protein with normal cell count
• 67
(albuminocytologic dissociation). f protein
• 68
. may cause papilledema .
8
Lock
s
Suspend
0
End Block
Item: Sl of ~ ,• Mark <::J [:::> ""I ~· ~'j
100 J.. Previous Next faiUI~S
LAb Notes Calculator

• •
43 FA17 p 140.3

Gram 8, comma or s shaped (with polar flagella) a. oxidase ~. gro\\"S at 42°C ("Campylobacter
44
Campylobacter jejuni
45
likes the hot campfire"). Major cause of bloody diarrhea, especially in children. Fecal-oral
46
47
transmission through person-to-person contact or via ingestion of undercooked contaminated
48
poultry or meat, unpasteurized milk. Contact with infected animals (clogs, cats, pigs) is also a risk
49
factor. Common antecedent to Guilla in-Barre syndrome and reactive arthritis.
50
51
• 52
. 53
. 54
• 55 FA 17 p 175.1
• 56 Bugs causing diarrhea
• 57
Bloody diarrhea
• 58
. 59 Campylobacter Comma- or S-shaped organisms; growth at 42°C
• 60 E histolytica Protozoan; amebic dysentery; liver abscess
• 61
Enterohemorrhagic 0157:H7; can cause llUS; makes Shiga-like toxin
• 62
Ecoli
• 63
. 64 Enteroinvasive Ecoli Invades colonic mucosa
. 65
Salmonella (non- Lactose 8; flagellar motility; has animal reservoir, especially poultry and eggs
• 66
typhoidal)
• 67
• 68
Shigella Lactose 8; very low 10 ;0; produces Shiga toxin (human reservoir only); bacillary dysentery

. •
8
Lock
s
Suspend
0
End Block
Item: Sl of ~ ,• Mark <::J [:::> ""I ~· ~'j
100 J.. Previous Next LAbfaiUI~S Notes Calculator


43 FA17 p 175.1
44
Bugs causing diarrhea
45
46
Bloody diarrhea
47 Campylobacter Comma- or S-shaped organisms; growth at 42°C
48
E histolytica Protozoan; amebic dysentery; liver abscess
49
50
Enterohemorrhagic 0157:1-17; can cause llUS; makes Shiga-like toxin
51
Ecoli
• 52 Enteroinvasive Ecoli Invades colonic mucosa
. 53
Salmonella (non- Lactose 8; Aagellar motility; has animal reservoir, especially poultry and eggs
. 54
typhoidal)
• 55
• 56 Shigella Lactose 8; very low 10;0; produces Shiga toxin (human reservoir only); bacillary dysentery
• 57 Yenterocolitica Day care outbreaks, pseudoappendicitis
• 58
Watery diarrhea
. 59
• 60 Cdifficile Pseudomembranous colitis; associated with antibiotics and PPis; occasionally bloody diarrhea
• 61 C perfringens Also causes gas gangrene
• 62
Enterotoxigenic E coli Tra,·elers' diarrhea; produces heat-labile (LT) and heat-stable (ST) toxins
• 63
. 64 Protozoa Giardia, Cryptosporidium
. 65 Vcholerae Comma-shaped organisms; rice-water diarrhea; often from infected seafood
• 66
• 67
Viruses Rota,·irus, norovirus, adenovirus
• 68
. •
8
Lock
s
Suspend
0
End Block
Item: S2 of - ,• Mark -<J [:::> "'I ~ · ~
100 ~ P~v1ous N @xt Labl lues N o tes Calculato r
6

43 An 8-year-old boy is treated with intravenous clindamycin for 7 days as a resu lt of a perforated appendix . Alt hough he does well initially, at ~~AI
44 the end of his antibiotic course, he develops abdominal cramping and a temperature of 39.4°C (103°F) . He also experiences several
45
episodes of watery diarrhea.
46
Which of the following antibiotics should be used to treat this patient?
47
48 :
A. Ampicillin
49
50 B. Ceftriaxone
51 C. Clindamycin
• 52
0 . Metronidazole
. 53
. 54 E. Tetracycline
• 55
• 56
0
57
• 58
. 59
• 60
0 61
0 62
0
63
. 64
. 65
0 66
0 67
0
68
. •
8
Lode.
s
S uspe-nd
8
End Bloc:k
Item: 52 of - ,• Mark -<] 1:> ""'I ~· 1!';:'1
100 ~ Prev1o u s Next Labf a lu es Note s Calculator


43
The correct answer is D. 79°/o chose this.
44
Pseudomembranous colitis is found primarily in hospitalized patients and is caused by rA=--~.,.,...----:-:
u-_-,:-:-,«- - - - - - - =111.,"''• Know
45 Clostridium difficile. This organism causes diarrhea (usually watery, but small Ampldln can cause psMc•,.••Dienot.~~ cohiJt
46
47
amounts of blood may be present), abdominal cramping, and fever through the
production of two exotoxins: toxin A and toxin B. This infection usually manifests after
a patient has been treated with broad-spectrum antibiotics for several days. The
Nei:ssetfa gonow11'loeae c.-
Senous Ql'tii'IWieQillVe nfec.tlons,

Can cause ~I IIDf*IIOUS CCIIhU

-
48 broad-spectrum antibiotic treatment shifts t he balance of normal intestinal flora and
49 allows C. difficile to grow in abundance. Subsequent treatment of pseudomembranous M e - Atlaefotllc l'lfec:tlelns "betow . . O$Aihm lb reacuott W'Ch YM Of
d~·

50
colitis includes discontinuation of the original antibiotic and administration of oral
vancomycin or metronidazole. Of these two drugs, metronidazole is the first-line "'""""" (- . """""'
Can ewse met111c: tMit ., lnOIAft

-
llidlotnonos)

51 agent, whereas vancomycin is second-line treatment and usually reserved for more ~
U;; ' tn1

52 severe C. difficile infections. Vancomycin should be administered orally, not


intravenously, because bactericidal concent rations are not achieved in the colon if
. 53
given intravenously. The table shows some reg ularly prescribed antibiotics and the conditions for which they are used .
. 54 Clostrtdtum d•tttc le cohb Broad-$pectrum anbbiotic Metronidazole Vancomycin Clostnd1um d1ffic le (betctc •a) Anbbtot1~ O.ttr-rhea Gut flora Intravenous therapy Colitis Bactencide Clostndtum Tox•n

• 55 Abdomtnal patn E)l)toxtn Colon (~ntrtomy) Organism Fever

• 56
A i s not correct. 6°/o chose this .
. 57
Ampicillin is an extended-spectrum penicillin used for the treatment of infection with certain gram -posit ive organisms and some gram-
• 58 negative rods. It is one of the anti biot ics commonly implicated as a cause of pseudomembra nous colit is. It is not used t o treat Clostridium
. 59 difficile .
Clostndtum dtfftctle colttls Ampictlhn Pentcillin Gram-negative bacteria Gram-positive bactena Clostridtum dtfftc1le (bacteria) Antibiotics Colitis Clostridium Extended-spectrum pemc1ll1n
• 60
• 61 B is not correct. 7% chose this .
• 62 Ceft ri axone is a third-generat ion cephalosporin. It is used in the t reatm ent of serious gram -negative infections as well as resistant gram-
posit ive infections. It is not effective in the treat ment of Clostridium difficile colitis .
• 63 Cephalosporin Ceftriaxone Gram-negative bacteria Gram-positive bacteria Clostndtum dtffictle coht1s Clostnd1um d1fficile (bacteria) Cohtts Clostridium
. 64
C is not correct . 4 °/o chose thi s .
. 65
Clindamycin is one of the most common antibiot ics implicated as a cause of pseudomembranous colitis. It is usually indicated for anaerobic
• 66 infections.
• 67 Clostnd1um d1ff1 tie coltb Cltndar"f.!cin Anaerobic organism Antibiotics Colitis Arlaerobic infect1on

• 68 E i s not correct. 4 °/o chose this . •


8
L.odt
s
Su~pl'nd
~
End Block
Item: 52 of - ,• Mark -<] 1:> ""'I ~· 1!';:'1
100 ~ Prev1o u s Next Labf a lu es Note s Calculator


43
E i s not corr ect. 4 °/o chose this.
44 Tetracycline is used to treat Chlamydia, Mycoplasma, and rickettsial illnesses. However, tetracycline is ineffective against Clostridium difficile.
45 It also may cause permanent discolorat ion of teeth and slow growth of bones in children.
Tetraeycl1ne Clostndtum dtffictle colttts Chlamtdia infection Chlamydia (genus) Clostndtum dtffic1le {bactena) Mycopl~ma Rtckettsia Clostridium
46
47
48 Bottom Line:
49 Clostridium difficile pseudomembranous colitis can manifest following treatment with broad-spectrum antibiotics, particularly ampicillin and
50 clindamycin. Treat with metronidazole, or vancomycin if the infection is severe.
Clo~tnd•u d• h! colltJ~ ., etron•da:zole Chndarr1"1Cin Arnpici in vancomycin Clostr•d•um d•ffic le (bGCt~~:o •I) C •t•s Cl,...~tl"ldhJm Anbb•obcs Broad-spectrum antibiotic Infection
51
52
. 53
. 54
l@lifii!H•J for year : 2017 •
FIRST AID FACTS

• 55
• 56 FA17 p 134.2
. 57
Clostridia (with Cram<±), spore-forming, obligate anaerobic rods.
• 58
exotoxins)
. 59
• 60 Ctetani Produces tetanospasmin, an exotoxin causing Tetanus is tetanic paralrsis.
• 61 tetanus. Tetanus toxin (and botuli nu11 1toxin)
• 62 are proteases that clea\·c SI ARE proteins for
• 63 neurotransmitters. Blocks release of inhibitory
. 64 neurotransmitters, CABA and glycine, from
. 65 Renshaw cells in spinal cord .
• 66 Causes spastic paralysis, trismus {lockjaw), risus
• 67 sardonicus (raised eyebrows and open grin),
• 68 opisthotonos (spasms of spinal extensors). •
8
L.odt
s
Su~pl'nd
~
End Block
Item: 52 of - ,• Mark -<] 1:> ""'I ~· 1!';:'1
100 ~ Prev1o u s Next Labf a lu es Note s Calculator

• •
43 FA17 p 134.2
44
Clostridia (with Gram Ei1, spore-forming, obligate anaerobic rods.
45
exotoxins)
46
47 Ctetani Produces tetanospasmin, an exotoxin causing Tetanus is tet anic paralysis.
48 tetanus. Tetanus toxin (and botulinum toxin)
49 are proteases that clea,·e SNARE proteins for
50 neurotransmitters. Blocks release of inhibitory
51 neurotransmitters, GABA and glycine, from
52 Renshaw cells in spinal cord.
. 53 Causes spastic paralysis, trismus (lockjaw), risus
. 54 sardonicus (raised eyebrows and open grin),
• 55 opisthotonos (spasms of spinal extensors).
• 56
Pre\ enl with tetanus vaccine. Treat'' ith
. 57
antitoxin +1- vaccine booster, diazepam (for
• 58
muscle spasms), and wound debridement.
. 59
• 60
Cbotulinum Produces a heat-labile toxin that inhibits Symptoms of botulism (the 4 D 's): D iplopia,
• 61
ACh release at the neuromuscular june!ion, Dysarthria, Dysphagia, D yspnea .
• 62 causi ng botulism. In adults, disease is caused Botulinum is from bad bottles of food, ju ice, and
• 63 by ingestion of preformed toxin. In babies, honey (causes a descending flaccid paralysis).
. 64 ingestion of spores (eg, in honey) leads to l .oca I botox injections used to treat foca I
. 65 disease (flopp) baby syndrome). Treat with dystonia, achalasia, and muscle spasms. Also
• 66 antitoxin . used for cosmetic reduction offa cial wrinkles.
• 67
• 68
Cperfringens
- .. Produces a toxin (lecithinase, a phospholipase)
that can cause myonecrosis (gas gangrene )
Perfringens perforates a gangrenous leg.

8
L.odt
s
Su~pl'nd
~
End Block
43
enterotoxin ..... food poisoning.
44
45
46 c
47
48
c difficile Produces 2 toxins. Toxin A, an enteroto-<in , Dif{icile causes diarrhea. Treatment:
binds to brush border of gut and alters Ouid metronidazole or oral \'3ncomycin. For
49
50
secretion. Toxin B, a C) toto-<in, disrupts recurrent cases, consider repeating prior
51
cytoskeleton via actin depolymerization. Both regimen, fidaxomicin, or fecal microbiota
52
toxins lead to diarrhea ..... pseudomembranous transplant.
. 53 colitisrn. Often zoto antibiotic usc, especially
. 54 clindamycin or ampicillin; associated with PPI
• 55 use. Diagnosed by detecting one or both toxins
• 56 in stool by antigen detection or PCR .
0
57
0
58 FA17 p 127.1
. 59
Spore-forming Some bacteria can form spores rJ at the end Bacillus anthracis Anthrax
0 60
0 61
bacteria of the stationary phase when nu lrients are Bacillus cereus Food poisoni ng
0 62
limited. Clostridium botulinum Botulism
0
63
Spores are highly resistant to heat and Clostridium difficile Pseudomembranous
. 64 chemicals. Have dipicolinic acid in their core. colitis
. 65 113\·e no metabolic activity. Must autocJa,·c to Clostridium per{ringens Gas gangrene
0 66 potentially kill spores (as is done to surgical Clostridium tetani Tetanus
0 67 equipment) by steaming at 121°C for 15
0
68 minutes.
. •
8
Lode.
s
S uspe-nd
8
End Bloc:k
Item: 53 of - ,• Mark -<J [:::> "'I ~· ~
100 ~ P~v1ous N @xt Labl lues N o tes Calculato r

43 A 9-year-old girl with an uncomplicated medical history is brought to her pediatrician by her moth er because of a rash . Physical
44 examination reveals the lesions shown in t he image; the woman says the lesions do not itch . No other lesions are present, and review of
45 systems is diffusely negative. The mother also notes that the child recentl started swimm lessons at the local pool.
46
47
48
49
50
51
52
. 53
. 54
• 55

' 56
' 57
• 58
. 59 What fam ily of viruses is respons ible fo r causing t his rash?
• 60
:
' 61 A. Adenovirus
' 62
B. Hepadnavirus
' 63
. 64
C. Herpesvirus
. 65 D. Papillomavirus
' 66 E. Polyomavirus
' 67
F. Poxvirus
' 68
.
8
Lode.
s
S uspe-nd
8
End Bloc:k
Item: S3 of ~ ,• Mark <::J [:::> ""I ~· ~'j
100 J.. Previous Next LAb faiUI~S Notes Calculator


43
44 The correct answer is F. 62°/o chos e this.
45
Molluscum contagiosum is a member of t he poxvirus fami ly t hat causes a localized infect ion consist ing of nonerythematous, pearly, dome-
shaped pa pules on t he skin. Howeve r~ this infect ion frequent ly leads t o localized derma tit is, causing the surrounding skin t o become pink, dry,
46 and prurit ic. As t he papules resolve, the skin may become inflamed and crust over fo r a week or two. This is a common disease of childhood
47 and often spreads via direct skin-to-skin contact or via indirect cont act such as wit h the sharing of bat h towels or other use. An associat ion
48
with swimming pool use has been re ported. The lesions may appear anywhere on t he body except t he palms and soles, and are usually self-
lim it ed in immunocompet ent people. If t he lesions appear on t he gen it alia of sexually active people, it is considered a sexually transmit ted
49
disease. I n immunocompromised pa t ients, particularly t hose with HIV-positive statu s, lesions can be quit e larg e (eg, giant molluscum) and
50 widespread .
Sexually transmitted infection Molluscum contagiosum Poxviridae Immunodeficiency Dermatitis HIV Sex organ Papule Itch Infection Immunocompetence
51
52 A is not correct. 7°/o chos e this.
53 Adenovirus is a common cause of upper respiratory infect ions in children . Pharyngit is and cory za are t he most common clinica l feat ures in
. 54
such cases, and, in many cases, exudative t onsillit is and cervica l lymphadenopathy may be present , making it very difficult to distinguish it
f rom group A st repococcal infect ion .
• 55 Adenoviridae Pharyngitis Tonsillitis Lymphadenopathy Rhinitis Upper respiratory tract infection Exudate Cervical lymphadenopathy Infection
• 56
B is not correct. 4 % chose this .
• 57
Hepadnavirus causes hepat it is B, wit h jaundice being a possible dermat olog ic sequela. Acut e hepatit is B infections are often subclinical, but
• 58 some individua ls may develop chronic hepatitis, which manifests with stigamata of chronic liver disease and may lead to cirrhosis.
. 59 Sequela Jaundice Hepatitis 8 Cirrhosis Hepatitis HepadnCPJiridae Liver Hepatitis 8 virus Liver disease Chronic liver disease Asymptomatic

• 60
C is not correct. 8 °/o chose this .
• 61 The various herpes simplex viruses (HSVs) cause several different diseases t hat resu lt in rash. HSV- 1 and HSV-2 cause gen it al herpes and
• 62 cold sores. Vari cella-zost er virus causes chickenpox, which can reactivat e and resu lt in shingles. The lesions are painful vesicles wit h an
eryt hematous base .
• 63
Chickenpox Varicella zoster virus Herpes labialis Genital herpes Herpes simplex virus Herpes simplex Virus Erythema Rash Vesicle (biology and chemistry) Shingles
. 64
. 65
D is not correct. 12% chose this .
Papillomavirus causes can cause plant ar wa1ts, common warts, flat warts, and butcher's warts, depend ing on t he serot ype of the virus.
• 66
Anogenita l warts are often caused by serotypes 6 and 11. Cert ain serotypes are a risk facto r fo r cancer of t he cervix, namely 16 and 18, and
• 67 other reg ions of t he body, such as head and neck cancers.
• 68 Genital wart Cervix Serotype Cervical cancer Virus Verruca plana Head and neck cancer Cancer Papillomaviridae

. •
8
Lock
s
Suspend
0
End Block
Item: S3 of ~ ,• Mark <::J [:::> ""I ~· ~'j
100 J.. Previous Next LAb faiUI~S Notes Calculator

43 E is not correct. 7°/o chose this .


44 Reactivation of polyomavirus results in progressive mult ifocal leukoencephalopathy in immunosuppressed pat ients.
Progressive multifocal leukoencephalopathy Immunosuppression Leukoencephalopathy Polyomaviridae
45
46
47 Bottom Line :
48 Molluscum conta giosum is caused by a poxvirus. This self-limiting rash is characterize d by pink papules, which may be um bilicated. Itching
49 may be present or absent.
Poxviridae Molluscum contagiosum Papule Rash
50
51
52
53
l@l;fil·1i•l for yea r:[2017
FIRST AID FACTS .
•j .

. 54
• 55 FA17 p 160.1
• 56 DNA viruses
• 57 VIRAL FAMILY ENVELOPE DNA STRUCTURE MEDICAL IMPORTANCE
• 58
Herpesviruses Yes OS and linear See Herpesviruses entry
. 59
• 60 Poxvirus Yes OS and linear Smallpox eradicated world wide by use of the live-
• 61 (largest 0 A virus) attenuated vaccine
• 62 Cowpox ("milkmaid blisters")
• 63 Molluscum contagiosum -Aesh-colored papule with
. 64 centra I umbiIication
. 65
Hepadnavirus Yes Partially OS and circular HBV:
• 66
• cute or chronic hepatitis
• 67
• I ot a retrovirus but has re,·erse transcriptase
• 68
. • AnAnnv ir••c N" n' <>nrl linP !Jr ~PhriiP nh ~r\J noitic: R -cu,rP thrn~t •

8
Lock
s
Suspend
0
End Block
Item: 53 of - ,• Mark -<] 1:> ""'I ~· 1!';:'1
100 ~ Prev1o u s Next Labf a lu es Note s Calculator

• •
43 Adenovirus No DS and linear Febrile pharyngitis r.J-sore throat
44 Acute hemorrhagic cystitis
45 Pneumonia
46 Conjuncti\'itis-"pink ere"
47
48
49
50
51
52
Papillomavirus 0 DS and circular liP - warts (scrotypcs I, 2, 6, II), CIN, cervical
53
. 54
cancer (most commonly 16, 18)
• 55 Polyomavirus I 0 OS and circular JC 'irus-progrcssive multifocalleukocncephalopathy
• 56 (P 1L) in HI
. 57 BK 'irus-tn:111~plant patients, commonly targets kidney
• 58 JC: Junky Cerebrum; BK: Bad Kidney
. 59
Parvovirus 0 SS and linear 131 9 virus-aplastic crises in sickle cell disease,
• 60
(smallest D 'A virus) "slapped cheek" rash in children (erythema
• 61
infcctiosum, or fifth disease)
• 62
RBC destruction in fetus leads to hydrops fetal is
• 63
and death, in adu lts leads to pure RBC aplasia and
. 64
. 65
rheumatoid arthritis-like symptoms
• 66
• 67 FA17 p 451.1
• 68 Skin infections •
8
L.odt
s
Su~pl'nd
~
End Block
Item: S3 of ~ ,• Mark <::J [:::> ""I ~· ~'j
100 J.. Previous Next faiUI~S
LAb Notes Calculator

• •
43
FA17 p451 .1
44
Skin infections
45
46 Bacterial infections
47 Impetigo Very superficial skin infection. Usually from S aureus or S pyogenes. Highly contagious. Honey-
48 colored crusting a.
49
Bullous impetigo IE] has bullae and is usually caused by S aureus.
50
Erysipelas In fection involving upper dermis and superficial lymphatics, usually from S pyogenes. Presents with
51
52
well-defined demarcation between infected and normal skin B!.
53 Cellulitis Acute, painful, spreading infection of deeper dermis and subcutaneous tissues. Usually from
. 54 S pyogenes or S aureus. Often starts with a break in skin from trauma or another infection [!].
• 55
Abscess Collection of pus from a walled-off infection within deeper layers of skin D. Offend ing organism is
• 56
almost always S aureus.
• 57
• 58 Necrotizing fasciitis Deeper tissue injury, usually from anaerobic bacteria or S pyogenes. Pain may be out of proportion
. 59 to exam find ings. Results in crepitus from methane and C02 production. "Flesh-eating bacteria."
• 60 Causes bullae and a purple color to the skin 1).
• 61 Staphylococcal scalded Exotoxin destroys keratinocyte attachments in stratum granulosum only (vs toxic epidermal
• 62
skin syndrome necrolysis, which destroys epidermal-dermal junction). Characterized by fever and generalized
• 63
. 64
m
erythematous rash with sloughing of the upper layers of the epidermis that heals completely.
® 1 ikolsky sign. Seen in newborns and children, adults with renal insufficiency.
. 65
• 66
Viral infections
• 67 Herpes Herpes virus infections (HSVl and HSV2) of skin can occur anywhere from mucosal surfaces to
• 68 normal skin. These include herpes labial is, herpes genita lis, herpetic whitlow CJ (fi nger). •
. •
8
Lock
s
Suspend
0
End Block
Item: S3 of - ,• Mark -<J [:::> "'I ~ · ~
100 ~ P~v1ous N @xt Labl lues N o tes Calculato r
A A
43 Herpes Herpes virus infections (HSVl and HSV2) of skin can occur anywhere from mucosal surfaces to
44 normal skin. These include herpes labialis, herpes genitalis, herpetic whitlow CJ (finger).
45
46
Molluscum Umbilicated papules caused by a powirus. While frequently seen in children, it may be sexually
47
contagiosum transmitted in adults.
48 Varicella zoster virus Causes ,·aricella (chickenpox) and zoster (shingles). Varicella presents with multiple crops of
49 lesions in ,·arious stages from \'esicles to crusts. Zoster is a reacti,·ation of the ,·irus in dermatomal
50 distribution (unless it is disseminated).
51
Hairy leukoplakia Irregular, white, painless plaques on lateral tongue that cannot be scraped off D. EB mediated.
52
Occurs in HI -positi\ e patients, orga n transplant recipients. Contrast with thrush (scrapable) and
53
leukoplakia (precancerous).
. 54
• 55

' 56
' 57
• 58
. 59
• 60

' 61
' 62
' 63
. 64
. 65

' 66
' 67
' 68
.
8
Lode.
s
S uspe-nd
8
End Bloc:k
Item: S4 of - ,• Mark -<] 1:> ""'I ~· 1!';:'1
100 ~ Prev1o u s Next Labf a lu es Notes Calculator


43 A 2-month-old boy was delivered at 30 weeks' gestat ion. His mother brings him to the pediatrician's off ice because he "has been acting r~AJ
44 fussy." On physical examination, the pediatri cian not es that the baby is flaring his nostrils and grunting . He also notes a low-grade fever,
45
some wheezing, and crackles on lung auscultation. An enzyme-linked immunosorbent assay (EUSA) of nasal washings is used to confirm
the diagnosis.
46
47
What organism is causing this infant's symptoms?
48
:
49
A. Chlamydophila pneumoniae
50
51
B. Escherichia coli
52 C. Group B Streptococcus
53 D. Mycoplasma
. 54
E. Respiratory syncytial virus
• 55
• 56
. 57
• 58
. 59
• 60
• 61
• 62
• 63
. 64
. 65
• 66
• 67
• 68

8
L.odt
s
Su~pl'nd
~
End Block
Item: S4 of ~ ,• Mark <:::1 t::> ""I ~· ~'j
100 J.. Previous Next LAbfaiUI~S Notes Calculator

• The correct answer is E. 71 °/o chose this .


43
44
Because t his infant was born premat urely, he is at risk fo r bronchiolit is caused by respirat ory syncytial virus (RSV) . RSV is a sing le-stranded
RNA paramyxovirus. Transmission of RSV is primarily by direct contact wit h respirat ory secretions or fomites, t hough aerosol droplets also
45 have been im plicat ed in its transmission . RSV infection generally manifests like a bad cold (ie, low-grade fever, rhinorrhea, cough, and apnea) .
46 Exam can also reveal tachy pnea, wheezing, and crackles. Wheezing, in particular, can suggest a bronchial or bronchiolar process, as opposed
47
t o pneumonia, which occurs in t he lung parenchyma . Unlike the ot her options, t his is t he only infect ion that can be diagnosed by ELISA of
nasal washings.
48 Param(xoviridae Rhinorrhea Human respiratory syncytial virus Tachypnea EUSA Parenchyma Pneumonia Bronchiolitis Cough Wheeze Virus Aerosol Lung Crackles Syncytium RNA Fomite Fever Apnea
49 Infection Bronchus
50
A is not correct. 7°/o chose this.
51
Chlamydophila pneumoniae (formerl y known as Chlamydia pneumoniae) can cause pneumonia in infants, but it is not diagnosed wit h ELISA of
52 nasal washings.
53 Chlamydophila pneumoniae EUSA Pneumonia Chlamydia infection Chlamydia (genus) Chlamydophila

54
B is not correct. 4 % chose this .
• 55 Escherichia coli is t he second most common cause of neonat al sepsis and mening it is. However, its presence would not be diagnosed by ELISA.
• 56 Escherichia coli Sepsis Meningitis EUSA Neonatal sepsis Infant

• 57 Cis not correct. 14% chose this .


• 58 Group B Streptococcus (GBS) is a common cause of neonatal sepsis and meningit is. GBS also causes pneumonia in infants less t han 4 weeks
. 59 of age; however, it would not be diagnosed with ELISA .
Meningitis Sepsis EUSA Pneumonia Streptococcus Infant Neonatal sepsis
• 60
• 61 D is not correct. 4°/o chose this .
• 62 Mycoplasma can cause atypical "walking" pneumonia along wit h cold hemolyt ic anemias in some pat ients. It is not diagnosed wit h ELISA of
nasal washings, nor is it a common cause of infect ion in infants .
• 63
EUSA Pneumonia Mycoplasma Hemolytic anemia Hemolysis Anemia
. 64
. 65
• 66
Bottom Line:
• 67 Respirato ry syncytial virus (RSV) is a common cause of bronchiolit is (which may manifest wit h nasal flaring, grunt ing, respiratory dist ress,
low-grade feve r, and lung crackles) in premature infants. ELISA of nasal washings can be used t o det ect vira l infections, including RSV,
• 68
rhinovirus, and coronavirus. •
. •
8
Lock
s
Suspend
0
End Block
Item: S4 of ~ ,• Mark <:::1 t::> ""I ~· ~'j
100 J.. Previous Next LAbfaiUI~S Notes Calculator

• •
43 FA17 p 175.2
44 Common causes of pneumonia
45 NEONATES(< 4WK) CHILDREN (4 WK-18 YR) ADULTS (18- 40 YR) ADULTS (40- 65 YR) ELDERLY
46
Group B streptococci Viruses (RSV) Mycoplasma S pneumoniae S pneumoniae
47
E coli Mycoplasma C pneumoniae TI inf/uenzae In Auenza virus
48
C trachomatis S pneumoniae Anaerobes Anaerobes
49
(infants-3 yr) Viruses (eg, inAuenza) Viruses H inf/uenzae
50
C pnewnoniae Mycoplasma Gram 8 rods
51
(school-aged
52
53
children)
54
S pneumoniae
• 55 Runts May Cough
• 56 C hunky Sput um
• 57 Special groups
• 58
Alcoholic Klebsiella, anaerobes usually due to aspiration (eg, Peptostreptococcus, Fusobacterium, Prevotelfa,
. 59
Bacteroides)
• 60
• 61
IV drug users S pneumoniae, S attreus
• 62 Aspiration Anaerobes
• 63
Atypical Mycoplasma, Legionella, Chlamydia
. 64
. 65
Cystic fibrosis Pseudomonas, S aureus, S pneumoniae, Burkholderia cepacia
• 66 lmmunocompromised S aureus, enteric gram 8 rods, fungi, viruses, P iirovecii (wiH1 II IV)
• 67
Nosocomial (hospital S aureus, Pseudomonas, other enteric gram 8 rods
• 68
. • acquired) •

8
Lock
s
Suspend
0
End Block
Item: S4 of ~ ,• Mark <:::1 t::> ""I ~· ~'j
100 J.. Previous Next LAb faiUI~S Notes Calculator

43 FA17 p 645.1
44 Pneumonia
45 TYPE TYPICAL ORGANISMS CHARACTERISTICS
46
Lobar S pneumoniae most frequently, also Legionella, Intra-alveolar exudate -+ consolidation rl; may
47
48
Klebsiella rn
iJwolve enti re lobe or lung.
49 Bronchopneumonia S pneumoniae, S aureus, H influenzae, Acute inflammatory infiltrates [!1 from
50 Klebsiella bronchioles into adjacent alveoli; patchy
51 m.
distribution involving;;:: 1 lobe
52 Interstitial (atypical) Mycoplasma, Chlamydophila pneumoniae, Diffuse patchy inflammation localized to
53
pneumonia Chlamydia psittaci, Legionella, viruses (RSV, interstitial areas at alveolar walls; diffuse
54
C.MV, influenza, adenovirus) distribution involving;;:: 1 lobe D. Generally
• 55
follows a more indolent course ("walking"
• 56
pneumonia) .
• 57
• 58 Cryptogenic Formerly known as bronchiolitis oblitera ns
. 59 organizing organizing pneumonia (BOOP). Noninfectious
• 60 pneumonia pneumonia characterized by inflammation of
• 61 bronchioles and surrounding structure. Etiology
• 62 unknown. Secondary organizing pneumonia
• 63 caused by chronic inflammatory diseases (eg,
. 64 rheumatoid arthritis) or medication side effects
. 65
(eg, amiodarone). e sputum and blood cultures,
• 66
no response to antibiotics.
• 67
..
" ..
~- :-ir~f~
~

• 68
. • • I
.
- ·· -·~-
"c D
•• ....._
,.-. .'- -

8
Lock
s
Suspend
0
End Block
Item: S4 of - ,• Mark -<] 1:> ""'I ~· 1!';:'1
100 ~ Prev1o u s Next
. Labf a lu es Notes Calculator
. .. ... .. ... ..
43 distribution invoking :2: 1 lobe m.
44
45
Interstitial (atypical) Mycoplasma, Chlamydophila pneumoniae, Diffuse patchy inflammation localized to
46
pneumonia Chlamydia psittaci, Legionella, viruses (RSV, interstitial areas at ah-eolar walls; diffuse
47
C~ I V, influenza, adeno,•irus) distribution involving :2: I lobe 0 . Generally
48 follows a more indolent course ("" ·alking"
49 pneumonia).
50 Cryptogenic Formerly known as bronchiolitis obliterans
51 organizing organizing pneumonia (BOOP). 'oninfectious
52 pneumonia pneumonia characterized by inflammation of
53 bronchioles and surrounding structure. Etiology
54
unknown. Secondary organizing pneumonia
• 55
caused by chronic inflammatory diseases (eg,
• 56
rheumatoid arthritis) or medication side effects
. 57
(eg, amiodarone). e sputum and blood cultures,
• 58
. 59
no response to antibiotics.
• 60
• 61
• 62
• 63
. 64
. 65
• 66
• 67
• 68

8
L.odt
s
Su~pl'nd
~
End Block
Item: SS of - ,• Mark -<J [:::> "'I ~ · ~
100 ~ P~v1ous N @xt Labl lues N o tes Calcula to r
6

43 A 4-year-old child is brought to the pediatri cian wit h a parvovirus infection.


44
45 Which of the fol lowing signs or symptoms would likel y be seen with an infection from a virus in t his family?
46
:
47 A. A barking cough
48
B. A vesicular rash that appears in crops
49
50
C. An upper respiratory infection
51 0 . Erythema of the cheeks
52
E. Parotid gland swelling
53
54
. 55
• 56
• 57
• 58
. 59
• 60
• 61
• 62
• 63
. 64
. 65
• 66
• 67
• 68
. •
8
Lode.
s
S uspe-nd
8
End Bloc:k
Item: SS of ~ ,• Mark <::J [:::> ""I ~· ~'j
100 J.. Previous Next LAb faiUI~S Notes Calculator

43 The correct ans wer is 0. 77°/o chose this.


44 Parvovirus B19 is associated wit h reddened cheeks or a "slapped cheek " appearance. Once t he "slapped cheeks " are appa rent, the virem ia is
45 over. The rash is associated wit h I gG antibody formation t o t he v irus. Parvovirus B19 is also associat ed with anemia. It is part icularly
46
dangerous to a fetus of a pregnant woman who becomes infected wit h a parvovirus during her pregnancy. It is a single-st randed DNA virus.
DNA virus Parvovirus 619 Immunoglobulin G Parvovirus Antibody Anemia Viremia Fetus Virus Rash DNA Pregnancy
47
48 A is not correct. 6°/o chos e this.
49
A barking cough is a symptom of croup, which is caused by parainfluenza v irus, a member of the Paramyxoviri dae fam ily. Croup affects young
children and causes swelling of the upper airway.
50 ParaiTP(xoviridae Cough Human parainfluenza viruses Respiratory tract Symptom Virus Croup
51
B is not correct. 5% chose this.
52
Vari cella is an infection in the Herpes family. I t is associat ed with vesicula r rash t hat appea rs in crops that are in vari ous st ages of healing .
53 Chickenpox Varicella vaccine Rash Infection Herpes simplex

54
C is not correct. 7 °/o chose this .
55
An upper respiratory infection (URI) can be caused by respi rat ory syncyt ial virus, a member of t he Paramyxoviridae family. I n adults this URI
• 56 is oft en m ild, but in children it can cause severe bronchit is.
• 57 ParaiTP(xoviridae Human respiratory syncytial virus Bronchitis Upper respiratory tract infection Respiratory tract infection Virus Syncytium Infection

• 58 E is not correct. 5 °/o chos e this .


. 59 The m umps virus is in t he Pa ramyxoviridae family. Symptoms of infection include swollen parot id glands and orchit is .
• 60 Param(xoviridae Orchitis Mumps Mumps virus Virus Parotid gland

• 61
• 62 Bottom Line:
• 63
Parvovirus is a single-stranded DNA v irus that causes erythema infectiosium, or f ifth disease, so named because of it is the fifth of t he
. 64 childhood exanthems. The "slapped cheek" rash is charact eristic in children .
DNA virus Fifth disease Erythema Virus Parvovirus Rash DNA
. 65
• 66
• 67
• 68
l@ljl'il·1i•J for year:[2017 • J
FIRST AID FA CTS
.
8
Lock
s
Suspend
0
End Block
Item: SS of ~ ,• Mark <::J [:::> ""I ~· ~'j
100 J.. Previous Next LAb faiUI~S Notes Calculator

• •
43 FA17 p 160.1
44 DNA viruses
45 VIRAL FAMILY ENVELOPE ONA STRUCTURE MEDICAL IMPORTANCE
46
Herpesviruses Yes OS and linear See Herpesviruses entry
47
48
Poxvirus Yes OS and linear Smallpox eradicated world wide by use of the live-
49
(largest 0 1 A virus) attenuated vaccine
50
Cowpox ("milkmaid blisters")
51 Molluscum contagiosum -Aesh-colored papule with
52 central umbilication
53 Hepadnavirus Yes Partially OS and circular HBV:
54 Acule or chronic hepatitis
55 lot a retrovirus but has reverse transcriptase
• 56
Adenovirus 0 OS and linear Febrile pharyngitis rJ-sore throat
• 57
• 58
Acute hemorrhagic cystitis
. 59
Pneumonia
• 60
Conjunctivitis- "pink eye"
• 61
• 62
• 63
. 64
. 65
• 66 Papillomavirus No OS and circular HPV-warts (serotypes 1, 2, 6, 11), Cl , cervical
• 67 cancer (most commonly 16, 18)
• 68
. • Polyomavirus No OS and circular JC virus-progressive multifoca lleukocncephalopathy •

8
Lock
s
Suspend
0
End Block
Item: 55 of - ,• Mark -<] 1:> ""'I ~· 1!';:'1
100 ~ Prev1o u s Next Labf a lu es Note s Calculator


43 BK virus-transplant patients, commonly targets kidney
44 JC: Junky Cerebrum; BK: Bad Kidney
45
Parvovirus 0 SS and linear Bl9 virus- aplastic crises in sickle cell disease,
46
(smallest D 'A virus) "slapped cheek" rash in children (erythema
47
infectiosum, or fifth disease)
48
RBC destruction in fetus leads to hydrops fetal is
49
and death, in adults leads to pure RBC aplasia and
50
51
rheumatoid arthritis-like symptoms
52
53 FA17 p 159.5
54
55
DNA virus Some general rules-all 0 'A viruses:
• 56 characteristics GENERAl RUlE COMMENTS
. 57
Are HHAPPPPy viruses llepadna, Herpes, Adeno, Pox, Pan·o,
• 58
Papilloma, Polyoma .
. 59
• 60
Arc double stranded Except parvo (single stranded).
• 61 Have linear genomes Except papilloma and polyoma (circular,
• 62 sttpercoiled} and hepadna (circular,
• 63 incomplete).
. 64
Are icosahedral ~·: xccpl pox (complex).
. 65
• 66 Replicate in the nucleus Except pox (carries own D1 A-dependent Rl A
• 67 polymerase).
• 68

8
L.odt
s
Su~pl'nd
~
End Block
Item: S6 of - ,• Mark -<J [:::> "'I ~ · ~
100 ~ P~v1ous N @xt Labl lues N o tes Calcula to r
6

43 A 24-year-old man present s to t he emergency department with fever, chills, night sweats, malaise, and fat igue t hat st art ed 3 days ago. In
44 the past 24 hours, he has become short of breat h. He admit s t o using intravenous drugs regularly. At presentati on, t he pat ient is shaking
45 and appears pale. Physical examination is remarkab le for a temperature of 39.4°C (103°F), hypoxia t o 88% on room air, jugular venous
distention bilaterally, decreased breath sounds at the bases of the lungs, and a grade III/VI systolic murmur heard best at the lower left sternal
46 border.
47
48 Which pathogen is most likely responsib le for this patient's condition?
49
:
50 A. Enterococcus faecalis
51
B. Haemophilus aphrophilus
52
53
C. Staphylococcus aureus
54 D. Streptococcus bovis
55
E. Viridans streptococci
' 56
' 57
• 58
. 59
• 60

' 61
' 62
' 63
. 64
. 65

' 66
' 67
' 68
. •
8
Lode.
s
Suspe-nd
8
End Bloc:k
Item: S6 of ~ ,• Mark <::J [:::> ""I ~· ~'j
100 J.. Previous Next LAbfaiUI~S Notes Calculator


43 The correct ans wer is C. 82°/o chos e this.
44 This is a classic case of acut e bact eri al endocardit is (ABE). Endocardi t is often is characterized by const it utional sympt oms (feve r~ ma laise,
45 ch ills), new-onset cardiac m urmur, and a combinat ion of other signs and symptoms (eg, Janeway's lesions [small nont ender eryt hematous
lesions on t he soles of the feet], Osier's nodes [painf ul, red, raised lesions on t he hands and feet ] , Rot h 's spots [ ret ina l hemorrh ages with
46
whit e or pale cent ers]). Acute and subacut e endoca rdit is can be diffe rent iated based on history, because the acut e case will have the most
47 severe and sudden onset, as in t his pat ient. ABE also is seen most often in cases of int ravenous drug use and indwelling cat heters, and S.
48 aureus is the most common bact eri al pat hogen isolat ed in these cases, because it is part of t he skin f lora and enters t he blood at needle sites.
This patient 's hist ory of intravenous drug abuse, as well as auscu lt at ion of a m urmu r consistent wit h t ricuspid regurgitation, point to a right-
49
sided ABE infect ion. In right-sided endocardit is, one more often sees septic emboli to t he lungs, leading to bilat eral infilt rates. This patient is
50 man ifest ing signs of bilat eral infil t rates wit h hypoxia, decreased breat h sounds, and dullness to percussion . It is import ant to note that many
51 of t he classic signs of endoca rdit is, such as Janeway's lesions, Osier's nodes, and Roth's spots, are seen most ly as a complication of left-sided
endocardit is, in which septic emboli leave the heart and enter t he systemic circulation .
52 Janeway lesion Osler's node Roth's spot Hypoxia (medical) Endocarditis Embolism Infective endocarditis Tricuspid insufficiency Malaise Auscultation Heart murmur Intravenous therapy
53 Subacute bacterial endocarditis Pathogen Tricuspid valve Septic embolism Bleeding Staphylococcus aureus Catheter Erythema Circulatory system Drug injection Fever Lung
54
55
A is not correct. 3°/o chos e this.
Enterococcus faecalis also causes subacute endocardit is. The classic picture is a slow onset of const it utional sympt oms wit h low-grade fever.
56
Enterococcus infect ion is not seen as f requently as Streptococcus viridans, but it is known to colon ize damaged heart valves, especially in
• 57 patients with a history of rheumatic feve r.
• 58 Enterococcus faecalis Rheumatic fever Viridans streptococci Endocarditis Enterococcus Streptococcus Subacute bacterial endocarditis Heart valve Fever

. 59 B is not correct. 1% chose this .


• 60 Haemophilus aphrophilus is part of the HACEK group of fastidious gram- negat ive bacilli (Haemophilus aphrophilus, A ctinobacillus
• 61 actinomycetemcomitans, Cardiobacterium hominis, Eikenella corrodens, and Kingella kingae) t hat cause 5%-10% of cases of bacterial
endocardit is t hat are not related to int ravenous drug use. These organisms are slow growing and diff icult to culture from blood samples,
• 62
making diagnosis more complex. The t ime course of t his pat ient's onset of symptoms is too acute t o be relat ed t o HACEK organisms .
• 63 Aggregatibacter actinomycetemcomitans Eikenella corrodens Gram-negative bacteria Endocarditis Kingella kingae HACEK endocarditis Haemophilus aphrophilus Infective endocarditis Actinobacillus

. 64 Cardiobacterium hominis Bacilli Haemophilus Drug injection Fastidious organism

. 65
D is not correct. 4°/o chose this .
• 66 Streptococcus bovis also causes subacute bacterial endocardit is, wh ich presents with low-grade feve r and insidious onset. I t normally inhabits
• 67 t he lower gast rointest inal t ract , and lesions in the colon, such as t hose t hat occur in colon cancer, allow t he bact eri a access to the
• 68 bloodstream. I t most commonly affects t he aort ic valve .
. • Streptococcus bovis Colorectal cancer Endocarditis Streptococcus Gastrointestinal tract Human oastrointestinal tract Aortic valve Infective endocarditis Bacteria Fever Colon (anatomy) Cancer

8
Lock
s
Suspend
0
End Block
Item: S6 of ~ ,• Mark <::J [:::> ""I ~· ~'j
100 J.. Previous Next LAbfaiUI~S Notes Calculator

43 D is not correct. 4°/o chose this .


44 Streptococcus bovis also causes subacute bacterial endocardit is, wh ich presents with low-grade feve r and insidious onset. I t normally inhabits
45 t he lower gast rointest inal t ract , and lesions in the colon, such as t hose t hat occur in colon cancer, allow t he bact eri a access to the
bloodstream. I t most commonly affects t he aort ic valve.
46 Streptococcus bovis Colorectal cancer Endocarditis Streptococcus Gastrointestinal tract Human gastrointestinal tract Aortic valve Infective endocarditis Bacteria Fever Colon (anatomy) Cancer
47
Acute (medicine) Circulatory system
48
E is not correct. 10°/o chose this.
49
Viridans streptococci are t he most common cause of bact erial endocardit is overall. This group of bacteria is seen most often in subacute cases
50 in which the onset of symptoms usua lly is chronic and low-grade fevers are common . These streptococci commonly colon izes hea1t valves
51 previously damaged by rheumatic feve r, t hus causing left-sided infective endocardit is as opposed to t he right-sided version seen more
52
common ly wit h S. aureus. One common source of infection by viridans streptocicci is dent al procedures, duri ng which norma l flo ra of t he
oropharynx can enter t he bloodst ream .
53 Rheumatic fever Viridans streptococci Pharynx Infective endocarditis Endocarditis Streptococcus Staphylococcus aureus Bacteria Heart valve Human microbiota Fever Acute (medicine) Circulatory system
54
55
56
Bottom Line:
• 57
New-onset m urmu r in drug-abusing patient s is most likely endocardit is due t o S . aureus .
Endocarditis Staphylococcus aureus Heart murmur
• 58
. 59
• 60
i@l;fil·1i•J for year:[2017 • J
• 61 FIRST AID FACTS

• 62
• 63 FA17 p 299.2
. 64 Bacterial endocarditis Fever (most common symptom), new murmur, Mitral valve is most frequently involved.
. 65
Roth spots (round white spots on retina Tricuspid valve endoca rditis is associated with
• 66
surrounded by hemorrhage fl), Osler nodes IV drug abuse (don' t "tri" drugs). Associated
• 67
(tender raised lesions on Fi nger or toe pads I}] with S aureus, Pseudomonas, and Candida.
• 68
. clue to immune complex deposition), Janeway Culture 8; most likely Coxiella burnetii,
8
Lock
s
Suspend
0
End Block
43
tons , pat eryt tous spp.,
44 on palm or sole) m. glomerulonephritis, Aggregatibacter (formerly Actinobacillus),
45 septic arterial or pulmonmr emboli, splinter Cardiobacterium, Eikenella, Kingella)
46 hemorrhages fil on nail bed. Multiple blood • Bacteria FR0:\1 JANE • :
47 cultures necessary for diagnosis. Fever
48 Acute-$ aureus (high virulence). Roth spots
49 Large \'egetations on previously normal Osler nodes
50 \CJlves . Rapid onset. \lurmur
51 Subacute- viridans streptococci (lo" Janeway lesions
52 'irulence). Smaller vegetations on Anemia
53 congenitally abnormal or diseased vahes. 1\ail-bed hemorrhage
54
Sequela of dental procedures. Gradual Emboli
55
onset
56
S bovis (gallolyticus) is present in colon cancer,
. 57
S epidermidis on prosthetic ,·alves.
• 58
. 59
Endocarditis may also be nonbactcri<tl
• 60
(marantic/thrombotic) 2° to malignancy,
• 61
hypcrcoagu lable state, or l11pus.
• 62
• 63
. 64
. 65
• 66
• 67
• 68

8
L.odt
s
Su~pl'nd
~
End Block
43
44
45
46
47
t
48
49
FA17 p 300.1
50
51 Rheumatic fever A consequence of pharyngeal infection with J•~ES (major criteria):
52 group A ~ -hemolytic streptococci. Late Joint (migratory polyarthritis)
53 sequelae include rheumatic heart disease, • (carditis)
54 which affects heart \'ah es-m itral > <~ or I ic >> Nodules in skin (subcutaneous)
55 tricuspid (high-pressure valves affected most). Erythema marginatum
56 S)denham chorea
Early lesion is mitral ,-alve regurgitation;
. 57
late lesion is mitral stenosis. Associated
• 58
with Aschoff bod ies (granulom<l with giant
. 59
·. - - cells [blue arrows in D)), Anitschkow cells
• 60
(enlarged macrophagcs with ovoid, wavy,
• 61
• 62
rod-like nucleus [red arrow in an. t anti-
• 63
strcptoi)'Sin 0 (ASO) titers.
. 64
Immune mediated (type II hypersensitivity);
. 65 not a direct effect of bacteria. nt ibodics
• 66 toM protein cross-react with self antigens
• 67 (molecular mimicry).
• 68 Treatment/prophylaxis: penicillin .

8
L.odt
s
Su~pl'nd
~
End Block
Item: 57 of - ,• Mark -<J [:::> "'I ~ · ~
100 ~ P~v1ous N @xt Labl lues N o tes Calcula to r
6

43 A 40-year-old male presents to t he emergency department via emergency services after his fam ily wit nessed him sufferi ng tonic-clonic
44 movements. The patient was in his usual state of health last night; however, he has complai ned of a decreased sense of taste for the past
45 4 days. This morning, upon waking, he was not feel ing well. He suffered an objective fever of 103.0°F, complained of a headache, and
vomited twice. His fam ily also notes that he was confused but refused to seek medical help. However, after his falling to the ground and
46 exhibiting tonic-clonic movements they contacted EMS. His family denies sick contacts. Travel history is notable for visiting the family's lake
47 house 5 days prior. On examination, his vital signs are 104.1 °F, HR 140/min, BP 145/90 mm Hg, and RR 15/min. He is difficult to arouse. A CSF
48 sample is obtained with an opening pressure of 300 em H2 0, WBC count of > 1000, with polymorphonuclear leukocyte (PMN) predominance,
erythrocyte count of 1000, decreased glucose, and elevated protein. Gram stain and viral PCR are negative. A wet mount of the CSF
49 demonstrates motile organisms.
50
51 Through which structure did the organism gain access to the patient's body?
52
:
53 A . Conjunctival membrane
54
B. Cribriform plate
55
56 C. Extrernal auditory canal
. 57 D . Oral mucosa
• 58
E. Skin abrasion
. 59
• 60
• 61
• 62
• 63
. 64
. 65
• 66
• 67
• 68
. •
8
Lode.
s
Suspe-nd
8
End Bloc:k
Item: S7 of ~ ,• Mark <::J [:::> ""I ~· ~'j
100 J.. Previous Next LAb faiUI~S Notes Calculator


43
44 The correct a nswer is B. 64 % chose t his .
45 I n light of his exposure hist ory and progression of symptoms, t his man was most likely infected wit h Naegleria fowleri, which man ifests wit h a
rapidly progressing meningoencepha lit is that can progress t o coma or deat h wit hin 6 days. Ot her symptoms include nausea, vomiting, and
46 irrational behavior. Transmission occurs through swimming in freshwater lakes. Microscopic ana lysis will reveal amoebas in the spinal flu id.
47 Amphot eri cin B has been effective in some cases of N. fowleri infection, but in most cases t here has been no t reatment fo r this ra pidly
48 prog ressing infect ion. The pat hogen gains access to the CNS through t he criibriform plat e.
Naegleria fowleri Amphotericin B Nausea Pathogen Central nervous system Meningoencephalitis Vomiting Cerebrospinal fluid Amoeba Fresh water Coma
49
50 A is not correct . 7°/o chose this.
51 Pat hogens that can cause conjunct ivit is include v iruses and bacteria. Viral causes include adenovirus and herpesvirus. Bact erial causes include
Staphylococcus and Chlamydia.
52 Adenoviridae Conjunctivitis Chlamydia infection Chlamydia (genus) Staphylococcus Virus Herpesviridae Bacteria Pathogen
53
C is not correct. 9°/o chose this .
54
Pat hogens normally do not ent er the body through the extremal audit ory canal (EAM) as the tympanic membrane provides a nat ura l barri er.
55 However, Pseudomonas is a common pat hogen causing otit is externa . This would manifest as pain reproduced by movement to t he pinna.
56 Eardrum Otitis externa Auricle (anatomy) Pathogen Ear canal Otitis Sound Hearing

57 D is not correct. 11% c hose this .


• 58 I nfection wit h Candida, herpes sim plex v irus, and Coxsackie virus can cause oral lesions.
. 59 Herpes simplex virus Coxsackievirus Herpes simplex Candida (fungus) Infection Virus

• 60 E is not correct. 9°/o chose this .


• 61 Pat hogens that gain access t o t he body t hrough open skin include various viral pathogens such as HIV and HBV as well as bact erial pat hogens
• 62 includ ing Staphylococcus and Streptococcus species .
HIV Streptococcus Staphylococcus Pathogen Virus Hepatitis 8 virus
• 63
. 64
. 65 Bottom Line :
• 66 I nfection wit h Naegleria fowleri, an amoeba found in freshwater lakes, manifest s with a rapidly progressing meningoencephalit is that can
• 67 prog ress t o coma or deat h wit hin 6 days .
Naegleria fowleri Amoeba Naegleria Meningoencephalitis Infection Fresh water
• 68
. •
8
Lock
s
Suspend
0
End Block
Item: 57 of - ,• Mark --<) [::> ""'I ~· 1!';:'1
100 ~ Prev1o u s Next Labf a lu es Note s Calculator

• •
43 FA17 p 15201
44 Protozoa- CNS infections
45
ORGANISM DISEASE TRANSMISSION DIAGNOSIS TREATMENT
46
Toxoplasma Congenital toxoplasmosis = Cysts in meat (most Serology, biopsy Sulfadiazine +
47
48
gondii classic triad of chorioretinitis, common); OOC) sis (tachyzoite)rn pyrimethamine
hydrocephalus, and intracranial in cat feces; crosses
49
50
calcifications; reacti,·ation in placenta (pregnant
51
AIDS .... brain abscesses usually women should
52 seen as multiple ring-enhancing a' oid cats)
53 lesions on MRI rJ
54 Naegleria fowleri Rapidly fatal meningoencephalitis Swimming in Amoebas in spinal Amphotericin B has
55 fre!.hwater lakes Auid [i been effecti,oe for a
56 (think Nalgcnc few sur\'i\'ors
57
bottle fi lied
• 58
with frc\h water
0 59
containing
• 60
Naegleria ); enters
• 61
via cribriform plate
• 62
• 63 Trypanosoma African sleeping sickness- ' lsctse ny, a pa inful Trypomast igote in Suramin for blood-
064 brucei enlarged lymph nodes, recurring bite blood smear m borne disease or
065 fever (due to antigenic variation), mclarsoprol for
• 66 somnolence, coma C t S penetration
• 67 Two subspecies: Trypanosoma ("I '> llTC am
• 68 brucei rhodesiense, Trypanosoma mellow when •
8
L.odt
s
Su~pl'nd
~
End Block
Item: 57 of - ,• Mark --<) [::> ""'I ~· 1!';:'1
100 ~ Prev1o u s Next Labf a lu es Note s Calculator

43
• bottle fi lied
44 with frc\h water
45 containing
46 1\aegleria); enters
47 via cribriform plate
48 Suramin for blood-
Trypanosoma African sleeping sickness- 'lsetse fl), a painful Tr) pomast igote in
49
50
brucei enlarged lymph nodes, recurring bite blood smear m borne disease or
fe,er (due to antigenic vmiation), mclarsoprol for
51
somnolence, coma CNS penetration
52
Two subspecies: Tr)'panosoma ("I \ life am
53
54
brucei rhodesiense, Trypanosoma mellow when
55
brucei gambiense I'm ~ lccpi ng";
56 remember
57 melatonin helps
• 58 with sleep)
. 59
• 60
• 61
• 62
• 63
. 64
. 65
• 66
• 67
a
• 68

8
L.odt
s
Su~pl'nd
~
End Block
Item: 58 of - ,• Mark -<] 1:> ""'I ~· 1!';:'1
100 ~ Prev1o u s Next Labf a lu es Note s Calculator


43 A Salvadoran woman brings her 9-year-old son t o t he ped iat rician. She states her son had been very exuberant and well liked by all of his
44 teachers and had received excellent marks in school. Over th e past 6 months, however, he has become withdra wn and lethargic, and his
45
performance in school has deteriorat ed. Physical examination reveals occasional myoclonic jerking motions and some sensory deficits. The
mother has the child's medical chart with her, and a pat ient note from a clinic in El Salvador descri bes that the patient had a peculiar rash when
46 he was 11 months old that started on his face and then moved to his trunk and arms. The rash was bri ght red, raised, blanched when pressed,
47 and accompanied by fevers and sympt oms of the common cold. The rash resolved a week later in t he same order in which it appeared.
48
49 What is the most likely infectious cause of this child's current neurologic symptoms?
50 :
51 A. Human herpesvirus 6
52 B. Measles virus (rubeola virus)
53
C. Parvovirus B19
54
55 D. Streptococcus pyogenes
56 E. Togavirus
57
• 58
. 59
• 60
• 61
• 62
• 63
. 64
. 65
• 66
• 67
• 68

8
L.odt
s
Su~pl'nd
~
End Block
Item: sa of ~ ,• Mark <::J [:::> ""I ~· ~'I
100 J.. Previous Next LAbfaiUI~S Notes Calculator


43
The correct answer is B. 71% chos e this.
44
The pat ient's medical hist ory is t he key to identifying t he cause of his current symptoms. Measles typically manifests wit h a blanching red rash
45
t hat sta1ts on t he face, t hen moves down t o t he t runk and limbs. The rash is often ra ised and accompanied by feve r. The rash mostly resolves
46 in the same order in which it appeared . Subacut e sclerosing panencephalitis (SSPE) is a ra re complication of measles t hat occurs 7- 9 years
47 after t he init ial infect ion and is init ially charact erized by personality changes, lethargy, difficult y in school, and odd behavior. It can progress to
dement ia, severe myoclonic j erking, and t hen t o event ual flaccid it y and decorticate ri gidity. Death is possible at any stage of SSPE.
48 Subacute sclerosing panencephalitis Measles Dementia Rash Myoclonus Flaccid paralysis Medical history Fever Lethargy
49
50
A is not correct. 7°/o chos e this.
Ninety percent of pediat ric roseola cases occur before t he age of 2 years. It is characterized by a high fever t hat can last fo r 3- 5 days and can
51 be accompanied by conj unctivit is, ot it is media, and malaise . As t he fever resolves, a blanching macular rash develops f irst on the neck and
52 t runk and t hen spreads t o t he face and ext remit ies.
Otitis media Conjunctivitis Fever Rash Malaise Roseola Otitis
53
54 C is not correct. 9 °/o chos e this .
55 Also known as f ifth disease, erythema infect iosum can cause a prodrome of feve r and headache. A rash wit h a "slapped cheek " appearance
56 t hen develops and is accom panied by a1t hralg ias fo r several days. The virus will infect and lyse eryt hroid precursor cells in t he bone marrow.
Therefore, in pat ients wit h increased eryt hropoiet ic demand, such as sickle cell pat ients, this disease can lead to t ransient aplastic anemia.
57 Fifth disease Aplastic anemia Erythema Anemia Prodrome Red blood cell Bone marrow Rash Arthralgia Headache Virus Lysis Bone Fever Sickle-cell disease
58
D is not correct. 4°/o chose this .
. 59
As stated, scarl et fever is one of t he t op pediat ric diseases that will manifest wit h a rash . Scarlet feve r most often manifests wit h a fever, sore
• 60 t hroat , and "strawberry tongue." The rash can involve t he t runk, neck, and limbs, bu t the palms and soles most often are spared . After t he
• 61 rash subsides, t hough, there is desquamation of t he palms and soles. Complications of scarl et fever include toxic shock syndrome and
• 62 necrot izing fasciit is, which occur duri ng the acute infect ion .
Necrotizing fasciitis Scarlet fever Toxic shock syndrome Desquamation Strawberry tongue Sore throat Fever Rash Necrosis Infection Pediatrics Acute (medicine) Sole (foot)
• 63
. 64 E is not correct. 9 °/o chos e this.
. 65 Rubella often manifests wit h a descending rash that st art s on the face. However, feve r precedes the rash and resolves before t he rash
develops. Posterior cervical lymphadenopat hy is a common featu re of ru bella. There are no known neurologic sequelae of rubella t hat
• 66 resemble subacute sclerosing panencephalit is.
• 67 Subacute sclerosing panencephalitis Lymphadenopathy Rubella Cervical lymph nodes Sequela Rash Fever Neurology Cervical lymphadenopathy

• 68
. •
8
Lock
s
Suspend
0
End Block
Item: 58 of - ,• Mark -<] 1:> ""'I ~· 1!';:'1
100 ~ Prev1o u s Next Labf a lu es Note s Calculator


43
44 Bottom Line:
45 Measles virus is capable of causing subacut e sclerosing panencephalitis (SSPE), which resu lts in dementia, myoclonus, and personality
changes, 7-9 years after init ial infect ion.
46
Subacute sclero,;tno panencephalttts M.,aclonus Measles Measles virus Virus Dementia Acute (medtcane) Infection
47
48
49 141;fi1·1hi for year: 2017 •
fiRST A ID f ACTS
50
51
FA17 p 166.2
52
53 Measles (rubeola) A paramyxo,•irus that causes measles. Usual 3 C 's of measles:
54 virus presentation im·olves prodroma l fe,er " ith Cough
55 cough, coryza, and conjuncti,·itis, then Coryza
56 e,·entually Koplik spots (bright red spots " ith Conjunctivitis
57 blue-white center on buccal mucosa rJ), Vitamin A supplementation can reduce
58 followed 1- 2 days later by a maculopapular morbid ity and mortality from measles,
. 59
rash rn that starts at the head/neck and spreads particularly in malnourished children .
• 60
downward. Lymphadenitis with Warth in-
• 61
Finkeldey giant cells (fused lymphocytes) in
• 62
a background of paracortica l hyperplasia.
• 63
. 64
SSPE (subacute sclerosing panencephalitis,
. 65
occurring years later), encephalitis (I:2000),
• 66 and giant cell pneumonia (rarely, in
0 67 immunosuppressed) are possible sequelae.
0
68

8
L.odt
s
Su~pl'nd
~
End Block
Item: SB of - ,• Mark -<J [:::> "'I ~ · ~
100 ~ P~v1ous N @xt Labl lues N o tes Calcula to r
A A
43 FA17 p 163.1
44 RNA viruses
45 VIRAL FAMILY ENVELOPE RNA STRUCTURE CAPSID SYMMETRY MEDICAL IMPORTANCE
46 Reoviruses 0 OS linear Icosahedral Coltivirus3 - Colorado tick fever
47 10-1 2 segments (double) Rotavirus-cause of fata l diarrhea in children
48
49
Picornaviruses lo SS $ linear Icosahedral Poliovirus-polio-Salk/Sabin vaccines- IPV/OPV
50
l•.chovirus-aseptic meningitis
51
Rhino,·irus-"common cold"
52
Co, sacJ..ievirus-aseptic meningitis; herpangina
53
(mouth blisters, fever); hand, foot, and mouth
54
disease; mrocarditis; pericarditis
55
II V -acute viral hepatitis
56 PERC II
57 Hepevirus 0 SS ® linear Icosahedral HEV
58
Caliciviruses 0 SS ® linear Icosa hedral 'orovirus-viral gastroenteritis
. 59
• 60 Flaviviruses Yes SS ® linear Icosahedral HCV
• 61 Yellow fevern
• 62 Oenguea
• 63 St. I.ouis encephalitisa
. 64 West ile virusa {meningoencephalitis)
. 65 Zika virus
• 66
Togaviruses Yes SS E!:) Iinear Icosahedral Rubella
• 67
Western and Eastern equine encephalilis3
• 68
Chikungunra ,·irus
. •
8
Lode.
s
Suspe-nd
8
End Bloc:k
Item: SB of - ,• Mark -<J [:::> "'I ~ · ~
100 ~ P~v1ous N @xt Labl lues N o tes Calcula to r
A A
43 Retroviruses Yes SS ® linear Icosahedral I lave re,·erse transcriptase
44 2 copies (HTLV), HTLV-T-cellleukemia
45 complex II IV-AIDS
46 and conical
47 (HIV)
48
Coronaviruses Yes SS ® linear Helical "Common cold," SARS, \IERS
49
50 Orthomyxoviruses Yes ss e linear l lelical lnAuenza virus
51 8 segments
52 Paramyxoviruses Yes ss e linear Helical PaRaMno,·irus:

53 ParainAucnza-croup
1 onsegmented
54 RSV-bronchiolitis in babies; Rx-ribavirin
55 \lcaslcs, \lumps
56
57
Rhabdoviruses Yes ss e linear Helical Rabies
58 Filoviruses Yes ss e linear Helical Ebolall\larburg hemorrhagic fever- often fatal!
. 59
• 60
Arenaviruses Yes ss ®and e f felica l I.CMV- lymphocytic choriomeningitis \'irus
circular Lassa fever encephalitis-spread by rodents
• 61
• 62
2 segments
• 63 Bunyaviruses Yes sse circular Helical Ca Iiforn ia encephal itis 3
. 64 3 segments Sc111d ny/Ri ft Valley fevers 3
. 65 Crimean-Congo hemorrhagic fe,·e~
• 66 llantavirus- hemorrhagic fe\'er, pneumonia
• 67
Delta virus Yes ss e circular Uncertain HDV is a "defective'' ,-irus that requires the
• 68
presence of IIBV to replicate
. •
8
Lode.
s
Suspe-nd
8
End Bloc:k
Item: sa of ~ ,• Mark <::J [:::> ""I ~· ~'I
100 J.. Previous Next LAb faiUI~S Notes Calculator

• •
43 FA17 p 160.1
44 DNA viruses
45 VIRAL FAMILY ENVELOPE ONA STRUCTURE MEDICAL IMPORTANCE
46
Herpesviruses Yes OS and linear See Herpesviruses entry
47
48 Poxvirus Yes OS and linear Smallpox eradicated world wide by use of the live-
49 (largest 0 1 A virus) attenuated vaccine
50 Cowpox ("milkmaid blisters")
51 Molluscum contagiosum -Aesh-colored papule with
52 central umbilication
53 Hepadnavirus Yes Partially OS and circular HBV:
54 Acule or chronic hepatitis
55 lot a retrovirus but has reverse transcriptase
56
57
Adenovirus 0 OS and linear Febrile pharyngitis rJ-sore throat
58
Acute hemorrhagic cystitis
. 59 Pneumonia
• 60 Conjunctivitis- "pink eye"
• 61
• 62
• 63
. 64
. 65
• 66 Papillomavirus No OS and circular HPV-warts (serotypes 1, 2, 6, 11), Cl , cervical
• 67 cancer (most commonly 16, 18)
• 68
. • Polvomavirus No OS and circular IC virus-oro2rcssivc multifoca lleukocnceohalooathv •

8
Lock
s
Suspend
0
End Block
Item: S9 of - ,• Mark -<J [:::> "'I ~ · ~
100 ~ P~v1ous N @xt Labl lues N o tes Calcula to r
6

43 A 28-year-old man present s to t he emergency department complaining of increasing muscle weakness. He stat es he first not iced that he
44 had some m ild weakness in his feet and legs 4 days ago. He initially attributed the weakness t o "being t ired" bu t became concerned when
45 he began to have difficulty walking. He st ates that his symptoms have progressed over the last few days and that now he says the
weakness has spread to his arms and hands. Further questioning reveals that the patient reports recently experiencing a self-lim ited episode of
46 gastroenteritis.
47
48 Which organism is commonly associated with this patient's neurologic symptoms?
49
:
50 A. a-Hemolytic, encapsulated, gram-positive cocci that produce an IgA protease
51
B. Curved, oxidase-positive, gram- negative bacteria that can be grown at 42° C
52
53
C. Non-lactose-fermenting, oxidase-positive, gram-negative, aerobic bacilli
54 D. Rod-shaped, gram-positive, spore-forming anaerobe that produces a heat-labile toxin
55
E. Spiral-shaped bacteria with axial filaments, visualized using dark-field microscopy
56
57
58
. 59
• 60
• 61
• 62
• 63
. 64
. 65
• 66
• 67
• 68
. •
8
Lode.
s
Suspe-nd
8
End Bloc:k
Item: 59 of ~ ,• Mark <::J [:::> ""I ~· ~'j
100 J.. Previous Next LAb faiUI~S Notes Calculator


43
44 The correct a nswer is B. 72% chose t his .
45 Guillain-Ba rre syndrome (GBS) is a common cause of acute peripheral neuropathy t hat results in progressive weakness over a period of days.
Findings include elevat ed gamma -globulin, decreased nerve conduction velocity indicative of demyelination, and albuminocytologic
46
dissociation ( increased protein, normal WBC) . Two-t hirds of pat ients have preceding viral or gastroint estinal illness. Campylobacterjejuni, a
47 curved, oxidase-posit ive, gram-negative bacterium, has been associat ed with GBS . The pat hogenesis is t houg ht t o involve generat ion of
48 ant igang lioside ant ibodies induced by C. jejuni exposure.
Campylobacter jejuni Demyelinating disease Gram-negative bacteria Peripheral neuropathy Campylobacter Protein Gastroenteritis Antibody Bacteria Virus Pathogenesis Antiganglioside antibodies
49
50 A is not correct . 5°/o chose this.
51 Streptococcus pneumoniae is an a-hemolytic, encapsulat ed, gram-posit ive coccus that produces an I gA prot ease . Pneumococcal pneumonia
can result in bacterem ia, mening itis, osteomyelit is, or sept ic arthrit is, but is not associated wit h the development of GBS syndrome.
52 Osteomyelitis Streptococcus pneumoniae Bacteremia Septic arthritis Meningitis Gram-positive bacteria Pneumonia Streptococcus Protease Coccus Immunoglobulin A Arthritis
53
C is not correct. 8°/o chose this .
54
Pseudomonas aeruginosa is a non-lactose -fe rmenting, oxidase-positive, gram-negative, aerobic bacillus that can cause ot it is externa, uri nary
55 t ract infection, pneumonia, and sepsis in immunocomprom ised hosts.
56 Urinary tract infection Pseudomonas aeruginosa Gram-negative bacteria Sepsis Otitis externa Pneumonia Pseudomonas Immunodeficiency Aerobic organism Bacillus (shape) Urinary system Bacillus Otitis

57
D is not correct. 10% c hose this.
58 Clostridium botulinum is a rod-shaped, gram-posit ive, spore-forming anaerobe that produces a heat - labile t oxin t hat inhibi ts acetylcholine
59 release at t he neuromuscular j unct ion, causing flaccid para lysis.
Clostridium botulinum Neuromuscular junction Flaccid paralysis Gram-positive bacteria Acetylcholine Anaerobic organism Botulinum toxin Toxin Paralysis Clostridium Bacillus (shape)
• 60
• 61 E is not correct. 5°/o chose this .
• 62 Treponema pa/lidum, t he causative agent of syphilis, is a spirochet e, visualized by using dark-f ield m icroscopy. Tertiary syphilis causes
• 63 sensory deficits rat her t han m uscle weakness.
Spirochaete Treponema pallidum Syphilis Treponema Dark field microscopy Microscopy Tertiary syphilis
. 64
. 65
• 66 Bottom Line :
• 67 GBS may be preceded by Campy/obacter jejuni infection and is characterized by an ascending paralysis t hat evolves over days to weeks with
• 68
t he loss of deep-tendon reflexes. Cerebrospinal flu id shows albuminocytologic dissociat ion. Treat ment is with plasmapheresis, int ravenous
immunoqlobulin, and supportive care. •
. •
8
Lock
s
Suspend
0
End Block
Item: 59 of - ,• Mark -<] 1:> ""'I ~· 1!';:'1
100 ~ Prev1o u s Next Labf a lu es Note s Calculator

43 FA17 p 493.2
44
Acute inflammatory Most common subtype of Guillain-Barre ssociated with infections (eg, Campylobacter
45
demyelinating syndrome. Autoimmune condition that ieiuni, viral) ..... autoimmune attack of
46
polyradiculopathy destroys Schwann cells ..... inAammation and peripheral myelin due to molecular mimicry,
47
demyelination of peripheral nerves and motor inoculations, and stress, but no definitive link
48
fibers. Results in symmetric ascending muscle to pathogens.
49
50
weakness/para I~ sis and depressed tendon Respiratory support is critical until recover).
51
reAexes beginning in lower extremities. Facial Additional treatment: plasmapheresis, IV
52 paralysis in )0% of cases. \ Ia) see autonomic immunoglobulins. No role for steroids.
53 dysrcgulation (eg, cardiac irregularities,
54 hypertension, hypotension) or sensory
55 abnormalities. Almost all patients survi,·e; the
56 majority reco,·er completely after weeks to
57 months.
58 t CSF protein with normal cell count
59 (albuminocytologic dissociation). t protein
• 60
ma)' cause papilledema .
• 61
• 62
• 63 FA17 p 140.3
. 64 Campylobacter jejuni Gram 8, comma or S shaped (with polar Aagclla) rJ, oxidase$, grows at 42°C ("Campylobacter
. 65
likes the hot campfire''). Major cause of bloody diarrhea, especially in children. Fecal-oral
' 66 transmission through person-to-person contact or via ingestion of undercooked contaminated
' 67
poultry or meat, unpasteurized milk. Contact with infected animals (dogs, cats, pigs) is also a risk
' 68
factor. Common antecedcntto Guillain-Barre S\ ndromc and reactive arthritis. •
8
L.odt
s
Su~pl'nd
~
End Block
Item: 60 of - ,• Mark -<J [:::> "'I ~ · ~
100 ~ P~v1ous N @xt Labl lues N o t es Calcula to r
6

43 A 47-year-old woman presents to the physician with bloating, flatu lence, and foul-smelling diarrhea . A st ool sample is taken fo r laboratory ~~AI
44 evaluation. The results show teardrop-shaped trophozoites with a ventral sucking disc.
45
46 Which of the fol lowing medications should be prescribed for this patient?
47 :
48 A. Metronidazole
49 B. Nifurtimox
50
C. Quinine
51
52 0 . Sodium stibogluconate
53 E. Sulfadiazine and pyrimethamine
54
55
56
57
58
59
• 60
• 61
• 62
• 63
. 64
. 65
• 66
• 67
• 68
. •
8
Lode.
s
Suspe-nd
8
End Bloc:k
Item: 60 of ~ ,• Mark <:::1 t::> ""I ~· ~'j
100 J.. Previous Next LAbfaiUI~S Notes Calculator


43
44 The correct answer is A. 83°/o chos e this.
45 This patient is infected wit h Giardia Iamblia, which is t reated wit h metronidazole. G. Iamblia infect ion presents wit h bloat ing, f latulence, fou l-
smelling diarrh ea, and light-colored fat ty stools. G. Iamblia is t ransmitt ed via cysts in wat er (feca l-oral t ransmission) . On m icroscopy, one
46 observes t eardrop-shaped t rophozoit es with a ventral sucking disc or cysts.
47 Giardia Iambiia Metronidazole Fecal-oral route Giardia Diarrhea Trophozoites Bloating Cyst Microbial cyst Flatulence Microscopy Infection

48 B is not correct. 4 % chose this.


49 Nifurtimox is used to t reat Trypanosoma cruzi. Chronic T. cruzi infection causes Chagas disease, a cond it ion charact eri zed by ca rdiomegaly
50 and, often, dilation of the int estinal t ract. Microscopic examinat ion reveals flagellat ed t rypomastigot es in t he blood and nonmotile amastigot es
51
in t issue cult ure. T. cruzi is t ransmitted via t he reduviid bug.
Nifurtimox Chagas disease Trypanosoma cruzi Trypanosoma Reduviidae Cardiomegaly Gastrointestinal tract Amastigote Infection Flagellum
52
53 C is not correct. 4 °/o chos e this .
Quinine is used in combinat ion with other medications to t reat ma laria in areas wit h chloroquine resist ance. Mala ria is caused by infect ion wit h
54
t he parasit e Plasmodium fa/ciparum, which is carri ed by t he Anopheles mosquito. Suspect malari a in a patient wit h feve r, ch ills, arthrl agia,
55 myalgia, and severe fatigue who has recent ly t rave led to an endemic area. Quinine is also used (albeit off-label) to t reat babesiosis. Babesia
56 species infect ion manifests wit h a malari a- like synd rome. Babesiosis is t ransmit ted by the Ixodes t ick. On m icroscopic examinat ion, however,
no RBC pigment appea rs, but t he Maltese cross-appeari ng parasite is evident.
57
Babesiosis Chloroquine Plasmodium falciparum Quinine Myalgia Malaria Babesia Endemism Mosquito Anopheles Plasmodium Tick Parasitism Fatigue (medical) Off-label use Fever Ixodes Pigment Infection
58
59 D is not correct. 4°/o chose this.
Sodium stibogluconat e is used to t reat Leishmania donovani infection. L. donovani infect ion manifests with hepatomegaly and splenomegaly,
60
mala ise, anemia, and weight loss. L. donovani is t ransmitted by t he sandfly. Microscopica lly, macrophages containing amast igot es are
• 61 observed.
• 62 Leishmania donovani Hepatomegaly Splenomegaly Leishmania Sodium stibogluconate Anemia Macrophage Sandfly L. donovani Amastigote Malaise Infection

• 63 E is not correct. 5 °/o chos e this.


. 64 Sulfad iazine and pyri met hamine are used t o t reat toxoplasmosis. Toxoplasma gondii infection manifests wit h brain abscesses in HI V- posit ive
. 65 patients and with birth defects if infection occurs duri ng pregnancy (t oxoplasmosis is one of t he TORCHeS organisms) . T. gondii is t ransmitt ed
via cysts in raw meat or cat feces. The definit ive stage (sexual stage) occurs in cats. There are no definit ive prot ocols fo r det ection of
• 66
t oxoplasmosis. Microscopically, cysts are fou nd on hematoxylin and eosin (H&E) staining .
• 67 Toxoplasma gondii Toxoplasmosis Pyrimethamine Eosin Haematoxylin Sulfadiazine Feces Congenital disorder Abscess HIV Brain Pregnancy Microbial cyst Cyst Infection Cat Staining

• 68
. •
8
Lock
s
Suspend
0
End Block
Item: 60 of ~ ,• Mark <:::1 t::> ""I ~· ~'j
100 J.. Previous Next faiUI~S
LAb Notes Calculator

43
Bottom Line:
44
Giardia Iamblia infect ion is characterized microscopically by t eardrop-shaped t rophozoit es with a ventral sucking disc. I t is t reat ed with
45
metronidazole.
46 Giardia Iambiia Metronidazole Giardia Infection

47
48
49 l@ljl'il·1i•J for year:[ 2017 • J
FIRST AID FA C T S
50
51
FA17 p 151 .1
52
53
Protozoa- gastrointestinal infections
54 ORGAN ISM DISEASE TRANSMISSION DIAGNOSIS TREATMENT
55 Giardia Iamblia Giardiasis-bloating, flatu lence, Cysts in water Multinucleated Metronidazole
56 foul-smelling, fatly diarrhea trophozoites t'J or
57 (often seen in campers/hikers)- cysts I]) in stooI,
58 lhink fat-rich Ghirardelli anligen delection
59
chocolates for fatty stools of
60
Giardia
• 61
• 62
Entamoeba Amebiasis- bloody diarrhea Cysts in water Serology and/or Metronidazole;
• 63 histolytica (dysentery), liver abscess trophozoites (with paromomycm or
. 64 ("anchovy paste" exudate), engulfed RBCs [i iodoguinol for
. 65 RUQ pain; histolog)' shows in 1·he cytoplasm) asymptomatic cyst
• 66 Aask-shaped ulcer or cysts with up to passers
• 67 4 nuclei in stool I!];
• 68 Entamoeba Eats
.
8
Lock
s
Suspend
0
End Block
Item: 60 of - ,• Mark -<J [:::> "'I ~ · ~
100 ~ P~v1ous N @xt Labl lues N o t es Calcula to r
A A
43 Cryptosporidium Se,·ere diarrhea in AIDS Oocrsts in water Oocysts on acid-fast Pre,·ention (by
44 Mild disease (watery diarrhea) in stain , antigen filtering city
45 detection water supplies);
immunocompetent hosts
46
nitazoxanide in
47
immunocompetent
48
hosts
49
50
51
52
53
54
55
56
a a .___ _ _ ~
a
57
58
59
FA17 p 191 .3
60 Metronidazole
• 61 MECHANISM Forms toxic free radical metabolites in the
• 62 bacterial cell that damage D A. Bactericida l,
• 63 anti protozoal.
. 64
. 65
CLINICAL USE Treats Giardia, Entamoeba, Trichomonas, GET GAP on the .Metro with metronidazole!
• 66
Cardnerella vaginafis, \ naerobes (Bacteroides, Treats anaerobic infection below the diaphragm
• 67
C difficile). Can be used in place of amoxicillin vs clindamycin (anaerobic infections above
• 68 in H pylori "triple therapy" in case of penicillin diaphragm).
. .... II ..... -,.. . .

8
Lode.
s
Suspe-nd
8
End Bloc:k
Item: 61 of - ,• Mark -<J
P~v1ous
[:::> "'I ~ ·· ~
100 ~ N @xt Labl lues N o tes Calcula to r
6

43 A neonate with purulent umbilical discharge for 1 day presents with fever, irritability, and diffuse flushing. One day lat er she is covered in
44 large, flu id-filled blist ers t hat rupture easily, leaving raw red areas beneath. Blood cultures are taken, whi ch wit hin 24 hours grow an
organism that is subsequent ly Gram stained with the results shown below.
45 .
46 t. - 'II "

47 , \ '*'
• ~--·
48
49
50
· .4
.
• .t ...,:. . .. . -.

....
'
51
52 ~

~ ·q
53
54
..... ...
55
56
-·#
.. t

57 I mage courtesy of CDC/ Or. Richard Fackfam


58
59 The skin sym ptoms observed in t his case are due to t he involvement of which of the following int ercellular st ructures?
60
:
• 61 A. Desmosomes
• 62
B. Gap j unctions
• 63
. 64 C. Hemidesmosomes
. 65 D. Intermediate junctions
• 66
E. Tight junctions
• 67
• 68
. •
8
Lode.
s
Suspe-nd
8
End Bloc:k
Item: 61 of ~ ,• Mark <::J [:::> ""I ~· ~'j
100 J.. Previous Next LAb faiUI~S Notes Calculator


43
The correct answer is A. 57°/o chos e this.
44
The image shows gram-posit ive cocci in clust ers. Staphylococcal sca lded skin syndrome (SSSS) is caused by the re lease of two exotoxins
45 (epidermolytic toxins A and B) from Staphylococcus aureus. Desmosomes (also ca lled "macula adherens" ) are responsible for bind ing
46 epit helial cells t o one another t o fo rm a coherent whole. The exotoxins t hat are released bind to a molecule within the desmosome ca lled
desmog lein 1, t hereby disrupting cell adhesion. In SSSS, t he epiderm is separates at the stratum granulosum due to the binding of exotoxins
47
t o desmosomes in t his layer. Clinically, t his results in bullous lesions and a posit ive Nikolsky's sign .
48 Desmosome Oesmoglein-1 Stratum granulosum Staphylococcal scalded skin syndrome Nikolsky's sign Staphylococcus aureus Epidermis Epithelium Exotoxin Gram-positive bacteria Oesmoglein Cell adhesion
49 Staphylococcus Coccus
50
B is not correct. 7% chose this.
51
Gap j unctions are circu lar intercellular contact areas that permit t he passage of sma ll molecules between adj acent cells, allowing
52 communication to facilitate electrotonic and metabolic function .
53 Gap junction Metabolism Electrotonic potential

54 C is not correct. 23% chos e this.


55 Hemidesmosomes are present on t he basal surface of epi thelial cells adjacent t o t he basement membrane, and serve t o connect epit helial
56 cells t o t he underlying extracellular mat ri x .
Basement membrane Extracellular matrix Hemidesmosome Epithelium Extracellular Basal (phylogenetics)
57
58 D is not correct. 5°/o chose this.
59 I nt ermediate j unctions lie deep t o t ight j unctions, comprised of actin f ilaments fo rm ing a cont inuous band around t he cell, providing structural
support j ust below t ight j unctions.
60 Actin Tight junction Microfilament
61
E is not correct. 8 °/o chos e this .
• 62
Tight j unct ions are located beneat h t he lu minal surface of simple colum nar epit helium (eg, int estinal lining) and seal t he int ercellular space to
• 63 prevent diffus ion between cells.
. 64 Simple columnar epithelium Tight junction Columnar epithelial cell Epithelium Lumen (anatomy) Diffusion

. 65
• 66
Bottom Line:
• 67
I n st aphylococcal scalded skin syndrome, t he epiderm is separates at the st ratum granu losum as a result of binding of epidermolyt ic t oxins A
• 68 and B to desmoglein 1 in t his layer.
. •
8
Lock
s
Suspend
0
End Block
Item: 61 of ~ ,• Mark <::J [:::> ""I ~· ~'j
100 J.. Previous Next faiUI~S
LAb Notes Calculator

• •
43
FA17 p451 .1
44
Skin infections
45
46 Bacterial infections
47 Impetigo Very superficial skin infection. Usually from S aureus or S pyogenes. Highly contagious. Honey-
48 colored crusting a.
49 Bullous impetigo IE] has bullae and is usually caused by S aureus.
50
Erysipelas In fection involving upper dermis and superficial lymphatics, usually from S pyogenes. Presents with
51
52
well-defined demarcation between infected and normal skin B!.
53 Cellulitis Acute, painful, spreading infection of deeper dermis and subcutaneous tissues. Usually from
54 S pyogenes or S aureus. Often starts with a break in skin from trauma or another infection [!].
55
Abscess Collection of pus from a walled-off infection within deeper layers of skin D. Offend ing organism is
56
almost always S aureus.
57
58 Necrotizing fasciitis Deeper tissue injury, usually from anaerobic bacteria or S pyogenes. Pain may be out of proportion
59 to exam find ings. Results in crepitus from methane and C02 production. "Flesh-eating bacteria."
60 Causes bullae and a purple color to the skin 1).
61 Staphylococcal scalded Exotoxin destroys keratinocyte attachments in stratum granulosum only (vs toxic epidermal
• 62 skin syndrome necrolysis, which destroys epidermal-dermal junction). Characterized by fever and generalized
• 63
. 64
m
erythematous rash with sloughing of the upper layers of the epidermis that heals completely.
® 1 ikolsky sign. Seen in newborns and children, adults with renal insufficiency.
. 65
• 66 Viral infections
• 67 Herpes Herpes virus infections (HSVl and HSV2) of skin can occur anywhere from mucosal surfaces to
• 68 normal skin. These include herpes labial is, herpes genita lis, herpetic whitlow CJ (fi nger). •
. •
8
Lock
s
Suspend
0
End Block
Item: 61 of - ,• Mark -<] 1:> ""'I ~· 1!';:'1
100 ~ Prev1o u s Next Labf a lu es Note s Calculator
- -
43 Umbilicated papules caused by a poxvirus. While frequently seen in children, it may be sexually
Molluscum
44
contagiosum transmitted in adults.
45
46
Varicella zoster virus Causes varicel la (chickenpox) and zoster (shingles). Varicella presents with multiple crops of
47
lesions in ,-arious stages from ,-csicles to crusts. Zoster is a reactivation of the virus in dcrmatomal
48 distribution (unless it is dissemim1ted).
49 Hairy leukoplakia Irregular, white, painless plaques on lateral tongue that cannot be scraped off . EB mediated.
50 Occurs in HIV-positi'e patients, organ transplant recipients. Contrast with thrush (scrapable) and
51 leukoplakia (precancerous).
52
53
54
55
56
57
58
59
60
61
• 62
• 63
. 64
. 65

' 66
' 67
' 68

8
L.odt
s
Su~pl'nd
~
End Block
Item: 61 of - ,• Mark -<] 1:> ""'I ~· 1!';:'1
100 ~ Prev1o u s Next Labf a lu es Note s Calculator


43
44 FA17 p 131 .3
45
Staphylococcus aureus Gram EB, ~-hemolytic, catalase EB, coagulase TSST-1 is a superantigen that binds to MHC
46
E!1 cocci in clusters fl. Protein A (,·irulcnce !! and T-cell receptor, resulting in polyclonal
47
factor) binds Fc-lgG, inhibiting complement T-cell activation.
48
49
activation and phagoc) tosis. Commonl) Staphylococcal toxic shock syndrome
50
colonizes the nares, axilla, and groin. (TSS) presents as fever, vomiting, rash,
51 Causes: desquamation, shock, end-organ failure. TSS
52 lnAammatory disease-skin infections, results in t AST, t ALT, t bilirubin. Associated
53 organ abscesses, pneumonia (often after with prolonged use of vaginal tampons or nasal
54 inAuenza virus infection), endocarditis, packing.
55 septic arthritis, and osteOm) elit is. Compare "ith Streptococcus pyogenes TSS (a
56 • Toxin-mediated disease-toxic shock toxic shock-like syndrome associated with
57 syndrome (TSST-1 ), sca lded skin syndrome painful skin infection).
58 (exfoliative toxin), rapid-onset food S au reus food poisoning due to ingestion of
59 poisoning (enterotoxi ns). preformed toxin - short incubation period
60 MRSA (methicillin-resistantS aureus) (2-6 hr) followed by nonbloody diarrhea
61 infection-important cause of serious and emesis. Enterotoxin is heat stable - not
• 62
nosocomial and commun ity-acqui red destroyed by cooking.
• 63
infections; resistant to methicillin and Bad staph (aureus) make coagulase and toxins .
. 64
nafcillin because of altered penicillin- Forms fibrin clot around self - abscess .
. 65
binding protein.
' 66
' 67
' 68 FA17 p 446.2

8
L.odt
s
Su~pl'nd
~
End Block
Item: 61 of - ,• Mark -<] 1:> ""'I ~· 1!';:'1
100 ~ Prev1o u s Next Labf a lu es Note s Calculator


43 FA17 p 446.2
44
45
Skin layers Skin has 3 layers: epidermis, dermis, Californ ians Like Girls in String Bikinis.
46 subcutaneous fat (hypodermis, subcul is).
47 Epidermis layers from surface to base
48 Stratum Corneum (keratin)
49 Stratum Lucidum
50 • Stratum Granulosum
51 Stratum Spinosum (dcsmosomcs)
52 Stratum Basale (stem cell site)
53
54
55
56
57
58
59
60
61
• 62 ,. •
• 63
• • •.. "
. 64 4
• • • ,• •
••
. 65
...
' 66
' 67
• •
,.
" <I

' 68

8
L.odt
s
Su~pl'nd
~
End Block
Item: 62 of - ,• Mark -<J [:::> "'I ~· ~
100 ~ P~v1ous N @xt Labl lues N o tes Calcula to r
6

43 A 15-year-old girl comes t o t he clinic complaining of lower abdominal pain that worsens during coitu s. She also re ports new onset of ~~AI
44 vaginal discharge, proctit is, feve rs, and chills. She uses oral contraception and has had three sexual part ners. Her most recent partner is
45 present and complains of penile discharge. Cell lines are grown with cultures from her as shown in the image.
46
47
48
49
50
51
52
53
54
55
56
57
58
59 What is a possible complicat ion of t he pat ient's current presentation?
60
:
61 A. Cervical adenocarcinoma
• 62
B. Ectopic pregnancy
• 63
. 64 C. Inguinal lymphadenopat hy
. 65 D. Menorrhagia
• 66
E. Recurrent cystitis
• 67
• 68
. •
8
Lode.
s
Suspe-nd
8
End Bloc:k
Item: 62 of ~ ,• Mark <::J [:::> ""I ~· ~'j
100 J.. Previous Next LAbfaiUI~S Notes Calculator


43 The correct answ er is B. 65% chos e this.
44 This girl has signs and symptoms of pelvic inflammato ry disease (PID), which is an ascending infection affecting t he uterus and fa llopian
45 t ubes, wit h or without involvement of the ovaries. Diagnosis is supported by t he following crit eria : Oral t emperat ure> 38 .3C, cervica l
46 discharge/f ri ability, > 15 WBCs on m icroscopy of vaginal secretions, and recent/curre nt infection wit h N. gonorrhoeae or
Chlamydia trachomatis. PID can leave scars and adhesions in t he affected t issues, which may lead to chronic pelvic pain, infe1t ility, ectopic
47 pregnancy, and Fitz-Hugh-Curtis syndrome; these symptoms involve either direct extension of infect ed mat eri al from t he cul-de-sac t hrough
48 t he perit oneum and/or lymphatics or an immunologically med iat ed mechanism leading to perihepat it is. It is f requently caused by C.
49 trachomatis, t he most com mon genital infection in the world. More t han 75% of people are asymptomat ic. Chlamydia is an obligate
int ra cellular parasite t hat exists in two forms: t he elementary body (EB) and the ret iculat e body (RB). The EB is met abolically inact ive, as it
50 lacks t he abili ty to produce it s own adenosine t riphosphat e; it must be endocytosed by host cells. Once inside t he endosome, it becomes t he
51 RB and uses t he host's metabolic energy to grow and replicate. Although t hese bacteria are classif ied as gra m -negat ive, they are ext remely
52 difficult to stain wit h t radit ional techniques, as they have a un ique outer cell wall and are pri marily located wit hin host cells. C. trachomatis
can be det ected through direct fluo rescent ant ibody, enzyme im munoassay, or polymerase chain react ion t est ing of a sample from a cervical
53 swab. The m icrograph in the vignette shows inclusion bod ies (the brown spots) suggestive of chlamydia! infect ions. Overall, the different ial for
54 PID should include ect opic pregnancy, appendicit is, rupt ured ovarian cyst , and sept ic abortion .
OVarian cyst Pelvic inflammatory disease Chlam(dia trachomatis Ectopic pregnancy Adenosine triphosphate Direct fluorescent antibody Polymerase chain reaction Peritoneum Septic abortion
55
Neisseria gonorrhoeae ChlaiTP(dia infection Gram-negative bacteria Enzyme Appendicitis Uterus Cell wall OVary Abortion EUSA Antibody Asymptomatic Fallopian tube Pelvic pain Cervix Lymphatic system
56
Bacteria Inclusion bodies Immunoassay Parasitism Infertility Cyst Chlamydia (genus) Infection Adhesion (medicine) Pregnancy Inflammation vagina
57
58 A is not correct . 7°/o chose this.
59 Cervical adenocarcinoma is not a complicat ion of pelvic inf lammat ory disease. This neoplasm is associated wit h infect ion by human
60 pa pillomavirus types 16, 18, and 3 1.
Pelvic inflammatory disease Human papillomavirus Adenocarcinoma Neoplasm Cervical cancer Cervix Pelvis Inflammation
61
62 C is not correct. 16% chos e this.
• 63 I nguinal lympha denopathy is not a complication of pelv ic inf lammat ory disease. I nguinal lymphadenopat hy is seen in lymphogranuloma
venereum, which is caused by Chlamydia trachomatis serovars Ll, L2, and L3 .
. 64 Pelvic inflammatory disease Lymphogranuloma venereum Chlamydia trachomatis Lymphadenopathy Chlamydia infection Serotype Chlamydia (genus) Inguinal hernia Groin Inflammation Pelvis
. 65
D is not correct. 4°/o chose this .
• 66
Menorrh agia describes heavy and prolonged menst ruat ion. I t can be caused by a number of underl ying disorders, including clott ing
• 67 abnormalit ies and uteri ne fib roids. It is not a complicat ion of pelvic inf lammat ory disease.
• 68 Pelvic inflammatory disease Menorrhagia Uterine fibroid Menstruation Inflammation Coagulation Pelvis

. •
8
Lock
s
Suspend
0
End Block
Item: 62 of ~ ,• Mark <::J [:::> ""I ~· ~'j
100 J.. Previous Next LAbfaiUI~S Notes Calculator

43 E is not correct. 8°/o chose this .


44 Recurrent cystit is, infection of t he bladder, is not a complication of pelvic inflammatory disease. Recurrent cystit is is defined as at least t wo
infections in t he past 6 mont hs or t hree infect ions wit hin t he past year. Factors t hat may pred ispose t o recurrent cystit is include first infection
45
at a young age, diabetes, incomplete bladder emptying, kid ney stones, and mult iple sex partners.
46 Pelvic inflammatory disease Kidney Urinary tract infection Diabetes mellitus Kidney stone Urinary bladder Pelvis Infection Inflammation

47
48
Bottom Line :
49
Pelvic inf lammat ory disease (DID) is an ascending infect ion of t he reproduct ive t ract most common ly caused by Chlamydia trachomatis .
50
Some long-t erm complications associated wit h DID include chronic pelvic pain, infe1t ility, and ect opic pregnancy.
51 Pelvic inflammatory disease Chlamydia trachomatis Ectopic pregnancy Pelvic pain Chlamydia infection Chlam(dia (genus) Pregnancy Infertility Infection Reproductive system Inflammation Pelvis

52
53
54 l@ljl'il·1i•J for yea r:[2o17 • ]
FIRST AID FA CTS . .
55
56
FA17 p 146.1
57
58
Chlamydiae Chlamydiae cannot make their own ATP. T hey Chlamys = cloak (intracellular).
59 are obligate inl racell ular organ isms that cause C fJsittaci- has an avian reservoir (parrots),
60 mucosal infections. 2 forms: causes atypical pneumon ia.
61 Elementary body (small, dense) Lab diagnosis: PC R, nucleic acid ampl ification
62 is "E nfcctious" and Enters cell via test. Cytoplasmic inclusions (reticulate bodies)
• 63 E ndocytosis; transforms into reticulate body. seen on Giemsa or Auorescent antibody-
. 64 Reticulate body Replicates in cell by fission; stained smet~r.
. 65 Reorganizes into elementary bodies. The chlamydia! cell wall lacks classic
• 66 Chlamydia trachomatis causes reactive arthritis peptidoglycan (due to reduced muramic acid),
• 67
(Reiter syndrome), folli cular conjunctivitis f.ZJ, rendering ~-lactam antibiotics ineffective.
• 68
nongonococcal urethritis, and PID.
.
8
Lock
s
Suspend
0
End Block
Item: 62 of • ,• Mark -<] 1:> ""'I ~· 1!';:'1
100 ~ Prev1o u s Next Labf a lu es Note s Calculator

• psittaci cause atrpical pneumon ia; transmitted


43
44 b)' aerosoL
45 Treatment: azit hromycin (fm·orcd because one-
46 time treatment) or doxycycline (+ ceftriaxone
47 for possible concomitant gonorrhea).
48
49
FA17 p 181 .1
50
51 Pelvic inflammatory Top bugs-Chlamydia trachomatis (subacute, Salpingitis is a risk factor for ectopic pregnane)',
52 disease often undiagnosed}, Neisseria gonorrlweae infertility, chronic pekic pain, and adhesions.
53 (acute). C trachomatis-mosl common Can lead to Fitz-Hugh- Curtis syndrome-
54 bacterial STI in the United States. Cervical infection of the Ji,·er capsule and '\·iolin string"
55 motion tenderness (chandelier sign), purulent adhesions of peritoneum to liver
56 cervical discharge (J. PID may include
57 salpingitis, endometritis, hydrosalpinx, and
58 tubo-ovarian abscess.
59
60
61
62
BJ ' 1' ,•• . --~
,, -~

·
I /". f,

v.~..~ ~·~~
• 63
. 64 1
r •
. 65 I . . ...,
' 66
' 67
' 68 ... •
8
L.odt
s
Su~pl'nd
~
End Block
Item: 63 of - ,• Mark -<J [:::> "'I ~ · ~
100 ~ P~v1ous N @xt Labl lues N o tes Calcula to r
6

43 A 12-year-old girl present s to t he emergency department aft er a 1-week history of difficulty remembering t hings and bizarre behavior ~~AI
44 according to her mother. The pat ient's physical exam ination is normal. Spinal fluid analysis shows an elevated WBC count with a
45 lymphocytic predominance, mildly elevat ed protein level, and RBCs. CT of the head shows focal unilat era l pathology in the temporal lobe.
46
Which of the following is the most likely diagnosis?
47
48 :
A. Encephalitis due to cytomegalovirus
49
50 B. Encephalitis due to herpes simplex virus type 1
51 C. Encephalitis due to herpes simplex virus type 2
52
0 . Meningoencephalitis due to HIV
53
54 E. Progressive multifocalleukoencephalopat hy
55
56
57
58
59
60
61
62
• 63
. 64
. 65
• 66
• 67
• 68
. •
8
Lode.
s
S uspe-nd
8
End Bloc:k
Item: 63 of ~ ,• Mark <::J [:::> ""I ~· ~'j
100 J.. Previous Next LAb faiUI~S Notes Calculator


43 The correct a nswer is B. 66% chose t his .
44 The pat ient has encephalit is due to herpes simplex virus (HSV) type 1. The pat ient's presentat ion is typical fo r HSV- 1 encephalit is, wit h
45 symptoms t hat range f rom amnesia and behavioral changes (eg, hypomania) t o Kluver-Bucy syndrome (charact eri zed by loss of anger or fear
46
responses and hypersexua lit y) . Later stages of the disease can include coma and death . WBC can be elevat ed in aspet ic meningit is bu t
elevated numbers of RBCs are also often seen in t he CSF in HSV encephalit is. CT of t he brain demonstrat ing focal unilat eral pat hology in t he
47 t emporal lobe is also a t ypical finding in t his infection. Xan t hroch rom ia is a ch aract eri st ic of HSV encephalit is as well.
48 Hypomania Herpes simplex virus Meningitis Temporal lobe Kluver-Bucy syndrome Hypersexuality Encephalitis Virus Coma Pathology Amnesia Herpes simplex Cerebrospinal fluid Brain Infection

49 A is not correct . 13% chose this .


50 Cytomegalovirus encephalit is is usually a neonat al infection, unless the pat ient has some fo rm of immunosuppression, such as infection with
51 HIV. This infection leads t o perivent ricular necrosis and eventually periventricular calcif icat ions, and has devast at ing global effects on
neurological function.
52
Cytomegalovirus Neonatal infection Immunosuppression Encephalitis Infant Necrosis Neurology Infection
53
54
C is not correct. 11% chose this .
HSV-2 encephalit is could present in a sim ilar fashion to HSV-1 infection; however, infect ion wit h the virus usually resu lts in meningit is or
55
panencephalit is, which is not suggested by t his pat ient's presentat ion or imaging .
56 Meningitis Encephalitis Herpes simplex virus Herpes simplex Virus Infection

57
D is not correct. 2°/o chose t his.
58 Pat ients wit h HIV meningoencephalit is present wit h gradual onset , slowly worsening dement ia. These pat ients would have a posit ive
59 immunoassay for HI V.
Immunoassay Meningoencephalitis HIV Dementia
60
61 E is not correct. 8°/o chose this .
62 Prog ress ive mult ifoca l leukoencephalopat hy is caused by t he JC virus and ty pically affects only the immunosuppressed t hrough reactivat ion of
63 lat ent infect ion wit h the virus. Clinical presentat ion would include altered mental st at us, motor def icits such as hemiparesis or lim b ataxia, and
visual symptoms such as hemianopia .
. 64 Progressive multifocal leukoencephalopathy JC virus Hemiparesis Ataxia Leukoencephalopathy Immunosuppression Virus Altered level of consciousness
. 65
• 66
• 67 Bottom Line :
• 68 Herpes simplex virus t ype 1 (HSV-1 ) encephalit is can lead to am nesia, behavioral ch anges, coma, and deat h. Mildly elevat ed levels of protein
. • and RBCs also are findinas t voical of HSV- 1 enceohalit is .
8
Lock
s
Suspend
0
End Block
Item: 63 of ~ ,• Mark <::J [:::> ""I ~· ~'j
100 J.. Previous Next LAb faiUI~S Notes Calculator

• •
43 FA17 p 160.2
44
Herpesviruses Enveloped, OS, and linear viruses
45
46 VIRUS ROUTE OF TRANSMISSION CLI NICAL SIGNIFICANCE NOTES
47 Herpes Respiratory Gingivostomatitis, keratoconjunctivitis fJ, Most common cause of sporadic
48 simplex secretions, saliva herpes labialis m , herpetic whitlow on finger, encephalitis, can present as altered
49 virus-1 temporal lobe encephalitis, esophagitis, mental status, seizures, and/or
50 erythema mu lti fo rme. aphasia.
51
52
Herpes Sexua I contact, Herpes genital i s ~. neonatal herpes. Latent in sacral ganglia. Vi ral
53
simplex perinatal meningitis more common with
54
virus-2 HSV-2 than with HSV-1.
55 Varicella- Respiratory Varicella-zoster (chickenpox [!], shingles 0}, Latent in dorsal root or trigeminal
56 Zoster virus secretions encephalitis, pneumonia. ganglia; C V1 branch
57 (HHV-3) Most common complication of shingles is post- involvement can cause herpes
58 herpetic neuralgia. zoster ophthalmicus.
59
Epstein-Barr Respiratory Mononucleosis- fever, hepatosplenomegaly, Infects B cells through CD21.
60
61
virus (HHV-4) secretions, pharyngitis, and lymphadenopathy (especially Atypical lymphocytes on peripheral
62
saliva; aka posterior cervical nodes 0 ). Avoid contact sports blood smear [!!-not infected B
63
"kissing disease," until resolution due to risk of splenic rupture. cells but reactive cytotoxic T cells.
. 64 (common in Associated with lymphomas (eg, endem ic EE> Monospot test-heterophile
. 65 teens, young Burkitt lymphoma), nasopharyngeal antibodies detected by agglutination
• 66 adults) carcinoma (especially Asian adults), of sheep or horse RBCs.
• 67 lymphoproliferative disease in transplant Use of amoxicillin in mononucleosis
• 68 patients. can cause characteristic

. •
8
Lock
s
Suspend
0
End Block
Item: 63 of - ,• Mark -<] 1:> ""'I ~· 1!';:'1
100 ~ Prev1o u s Next Labf a lu es Note s Calculator

• cotton-wool exudates, vision loss .


43
44 Congenital CM
45 Roseola: fe,·er first, Ro\ie (checks)
Human Saliva Roseola infantum (exanthem subitum): high
46
herpes- fe,·ers for se\'eral days that· can cause seizures, later.
47
viruses 6 followed br diffuse macular rash HHV-7-less common cause of
48
and7 roseola.
49
50
Human Sexual contact Kaposi sarcoma (neoplasm of endothelial cells). Can also affect Cl tract and lungs.
51 herpesvirus Seen in HIV/AIDS and transplant patients.
52 8 Dark/violaceous plaques or nodules
53 representing \'ascular pro Iiferations.
54
55
56
57
58
59
60
61
62
63
. 64
. 65
' 66
' 67
' 68

8
L.odt
s
Su~pl'nd
~
End Block
Item: 64 of - ,• Mark -<J [:::> "'I ~ · ~
100 ~ P~v1ous N @xt Labl lues N o tes Calculato r
6

43 A 27-year-old man present s to his primary care provider complaining of an extremely painful ulcer on t he shaft of his penis. Upon ~~AI
44 examination, the ulcer has an erythematous base and clearl y demarcated borders. The base of the ulcer bleeds when scraped. The
45 pat ient's inguinal lymph nodes on t he left side are swollen, tender on palpation, and produce a purulent discharge. He reports nine sexual
partners over the course of the past year.
46
47
Which of the following descriptions best f it the pat hogen most likely to be responsible for this man's condition?
48
:
49
A. Double-stranded linear DNA virus
50
51
B. Gram-negative coccobacillus
52 C. Gram-negative diplococcus
53 D. Gram-negative spirochete
54
E. Obligate intracellular bact eria
55
56
57
58
59
60
61
62
63
. 64
. 65
• 66
• 67
• 68
. •
8
Lode.
s
S uspe-nd
8
End Bloc:k
Item: 64 of ~ ,• Mark <:::1 t::> ""I ~· ~'j
100 J.. Previous Next LAb faiUI~S Notes Calculator

43
44 The correct a ns wer is B. 38% chos e t his .
45 This is a classic presentat ion of chancroid, a sexually t ransmitted disease caused by Haemophi/us ducreyi. The genital lesions may begin as
46 small papules t hat rapidly evolve into painful ulcers with an erythematous base that bleeds easily with mechanical manipulation. Painful,
swollen inguinal lymph nodes, often unilateral, develop in about 30% of infected persons. I n 50% of t hese pat ients, t he swollen lym ph nodes
47
will rupture and drain pus. H. ducreyi is a gram-negat ive coccobacillus.
48 Sexually transmitted infection Chancroid Gram-negative bacteria Haemophilus ducreyi Lymphadenopathy Coccobacillus Ulcer (dermatology) Lymph node Erythema Pus Inguinal lymph node Papule Sex organ

49 Haemophilus

50
A is not correct . 16 % chos e this .
51 The herpes simplex viruses (HSV) -1 and HSV-2 are double-st randed linear DNA viruses that cause oral and genital lesions. The genital lesions
52 begin as small vesicles t hat rupture over a 1- to 2-week period to produce shallow, painfu l ulcers t hat on ly rarely bleed with mechanical
53
manipulation. HSV also does not produce purulent inguinal lymph nodes.
Virus Herpes simplex DNA virus Lymph node Pus Herpes simplex virus Vesicle (biology and chemistry) DNA Lymph Ulcer (dermatology)
54
55 C is not correct. 15% chos e this .
56
A gram-negat ive diploccocus t hat causes sexua lly t ransmitted diseases is Neisseria gonorrhoeae . N. gonorrhoeae causes uret hrit is, cervicit is,
and pelvic inflammatory disease. It does not cause genita l ulcers and swollen inguinal lymph nodes.
57 Pelvic inflammatory disease Neisseria gonorrhoeae Urethritis Cervicitis Gram-negative bacteria Sexually transmitted infection Lymph node Lymph Inflammation
58
D is not correct. 14% c hose this .
59
Gram -negative spirochet es descri bes Treponema pa!lidum, the cause of syphilis, a sexua lly t ransmitted disease also associated wit h genit al
60 ulcers. Primary syphilis presents as a solit ary, ra ised, f irm, and painless papule called a chancre. The papule may ulcerate to creat e a crat er
61 with slightly elevated edges. T. pa/lidum also does not produce purulent inguinal lymph nodes.
Papule Sexually transmitted infection Gram-negative bacteria Treponema pallidum Syphilis Primary syphilis Chancre Spirochaete Ulcer (dermatology) Pus Lymph node Ulcer Inguinal lymph node
62
63 E is not correct. 1 7°/o chose t his.
64 Chlamydia trachomatis (serot ypes Ll , L2, and L3) are obligat e intracellular pat hogens responsible fo r lymphogranuloma venereum, a sexually
t ransm itt ed disease . Lymphogranuloma venereum starts with painless genital papules or ulcers t hat often hea l spontaneously. This disease is
. 65
associat ed with tender, purulent inguinal lymph nodes. Unlike chancroid, in which genital lesions and swollen inguinal lymph nodes co- present ,
• 66 t hey do not co- present in lym phogranuloma venereum .
• 67 Lymphogranuloma venereum Sexually transmitted infection Chancroid Chlamydia trachomatis Chlamydia infection Lymph node Serotype Pus Papule Intracellular Pathogen Ulcer Ulcer (dermatology)

• 68 Sex organ

.
8
Lock
s
Suspend
0
End Block
Item: 64 of ~ ,• Mark <:::1 t::> ""I ~· ~'j
100 J.. Previous Next LAbfaiUI~S Notes Calculator

43
44 Bottom Line:
45 Genital ulcers are associat ed with chancroid, syphilis, lym phogranuloma venereum, and herpes simplex infection. Chancroid is caused H.
ducreyi, a gram- negative coccobacillus. Chancroid ulcers are painful wit h an eryt hematous base t hat bleeds wit h mechanical manipulation.
46 Lymphogranuloma venereum Chancroid Syphilis Haemophilus ducreyi Gram-negative bacteria Herpes simplex Coccobacillus Erythema Ulcer (dermatology) Sex organ Infection
47
48
49 141;fil·1i•J f or yea r:[ 2017
FIRST AID FA CTS .
•j .
50
51
FA17 p 180.1
52
53
Sexually transmitted infections
54 DISEASE CLINICALFEATURES ORGANISM
55 AIDS Opportunistic infections, Kaposi sarcoma, HIV
56 lymphoma
57
Chancroid Painfu l genita l ulcer with exudate, inguinal Haemophilus ducreyi (it's so painful, you "do
58
adenopathy cry")
59
60 Chlamydia Urethritis, cervicitis, epididymitis, Chlamydia trachomatis (0 - K)
61 conju nctivitis, reactive arthritis, PID
62 Condylomata Genital warts, koilocytes HPV-6 and -11
63 acuminata
64
. 65
Genital herpes Painful penile, vulvar, or cervical vesicles and 1-ISV-2, less commonly HSV-1
• 66
ulcers; can cause system ic symploms such as
• 67
b·er, headache, myalgia
• 68 Gonorrhea Urethritis, cervicitis, PI D, prostatitis, Neisseria gonorrhoeae
.
8
Lock
s
Suspend
0
End Block
Item: 64 of - ,• Mark -<J [:::> "'I ~ · ~
100 ~ P~v1ous N @xt Labl lues N o tes Calculato r
A A
43 Lymphogranuloma Infection of lymphatics; painless genital ulcers, C trachoma/is (LI-L3)
44 venereum painfullpnphadenopathy (ie, buboes)
45
Primary syphilis Painless chancre Treponema pallidum
46
47 Secondary syphilis Fe,·er, lymphadenopathy, skin rashes,
48 condylomata lata
49 Tertiary syphilis Gummas, tabes dorsalis, general paresis, aortitis,
50 Argyll Robertson pupil
51
Trichomoniasis Vaginitis, strawberry cer\'ix, motile in wet prep Trichomonas ragilwlis
52
53
54 FA17 p 146.1

Chlamys =cloak (intracellular).


55
Chlamydiae Chlamydiae cannot make their own TP. They
56
are obligate intracellular organisms thai cause C psittaci-has an avian reservoir (parrots),
57
mucosal infections. 2 forms: causes atypical pneumonia.
58
59
Elementary body (small, dense) Lab diagnosis: PCR, nucleic acid <~mplification
60
is "E nfectious" and Enters cell via test. Cytoplasmic inclusions (reticulate bodies)
61 Endocytosis; tra nsforms into reticulate body. seen on Cicmsa or Auorcscent antibody-
62 • Reticulate body Replicates in cell by fission; stained smea r.
63 Reorganizes into elementary bodies. T he chlamydia! cell wall lacks classic
64 Chlamydia trachomatis causes reactive arthritis peptidoglycan (due to reduced muramic acid),
. 65 (Reiter synd rome), follicular conjunctivitis rJ, rendering P-lactam antibiotics ineffective.
• 66 nongonococcal urethritis, and PID.
• 67 Chlamydophila pneumoniae and Chlamydophila
• 68 psittaci cause atypical pneumonia; transmitted
.
8
Lode.
s
S uspe-nd
8
End Bloc:k
Item: 64 of ~ ,• Mark <:::1 t::> ""I ~· ~'j
100 J.. Previous Next LAb faiUI~S Notes Calculator

• •
43 FA17 p 160.2
44
Herpesviruses Enveloped, OS, and linear viruses
45
46 VIRUS ROUTE OF TRANSMISSION CLI NICAL SIGNIFICANCE NOTES
47 Herpes Respiratory Gingivostomatitis, keratoconjunctivitis fJ, Most common cause of sporadic
48 simplex secretions, saliva herpes labialis m , herpetic whitlow on finger, encephalitis, can present as altered
49 virus-1 temporal lobe encephalitis, esophagitis, mental status, seizures, and/or
50 erythema mu lti fo rme. aphasia.
51
Herpes Sexua I contact, Herpes genital i s ~. neonatal herpes. Latent in sacral ganglia. Vi ral
52
simplex perinatal meningitis more common with
53
virus-2 HSV-2 than with HSV-1.
54
55 Varicella- Respiratory Varicella-zoster (chickenpox [!], shingles 0}, Latent in dorsal root or trigeminal
56 Zoster virus secretions encephalitis, pneumonia. ganglia; C V1 branch
57 (HHV-3) Most common complication of shingles is post- involvement can cause herpes
58 herpetic neuralgia. zoster ophthalmicus.
59
Epstein-Barr Respiratory Mononucleosis- fever, hepatosplenomegaly, Infects B cells through CD21.
60
virus (HHV-4) secretions, pharyngitis, and lymphadenopathy (especially Atypical lymphocytes on peripheral
61
saliva; aka posterior cervical nodes 0 ). Avoid contact sports blood smear [!!-not infected B
62
"kissing disease," until resolution due to risk of splenic rupture. cells but reactive cytotoxic T cells.
63
64
(common in Associated with lymphomas (eg, endem ic EE> Monospot test-heterophile
. 65
teens, young Burkitt lymphoma), nasopharyngeal antibodies detected by agglutination
• 66
adults) carcinoma (especially Asian adults), of sheep or horse RBCs.
• 67 lymphoproliferative disease in transplant Use of amoxicillin in mononucleosis
• 68 patients. can cause characteristic

. • •.,.., .,,..., ,J .,..,.,,., ,.,, .] ... •• -rn t- h

8
Lock
s
Suspend
0
End Block
Item: 64 of - ,• Mark -<] 1:> ""'I ~· 1!';:'1
100 ~ Prev1o u s Next Labf a lu es Notes Calculator

• - -
43 cotton-wool exudates, vision loss.
44 Congenital CM
45
Human Saliva Roseolct infantum (exanthem subitum): high Roseola: fe,·er first, Ro,ie (cheeks)
46
herpes- fe,·ers for se\·eral days that can cause seizures, later.
47
viruses 6 followed by diffu~c macular rash HHV-7-less common cause of
48
and 7 roseola.
49
50 Human Sexual contact Kaposi sarcoma (neoplasm of endothelial cells). Can also affect Cl tract and lungs.
51 herpesvirus Seen in HIV/AIDS and transplant patients.
52 8 Dark/violaceous plaques or nodules
53 representing vascular proliferations.
54
55
56
57
58
59
60
61
62
63
64
. 65
• 66
0 67
0
68

8
L.odt
s
Su~pl'nd
~
End Block
Item: 65 of - ,• Mark -<] 1:> ""'I ~· 1!';:'1
100 ~ Prev1o u s Next Labf a lu es Note s Calculator


43 A 13-year-old girl recently returned f rom spending a day in t he woods with the Girl Scouts. Upon returning she st arted to complain of a r~AJ
44 headache and chills. After 3 days of these constitutional sympt oms, she developed a rash that began on her palms and soles, but spread
45
inward to her trunk. Her worsening condit ion led her parents to take her to the emergency department, where a blood test reveals
antibodies that react with the Proteus antigen.
46
47
This patient is most likely infected with which of the following?
48
:
49
A. Borrelia burgdorferi
50
51
B. Coxiella burnetti
52 C. Coxsackievirus A
53 D. Rickettsia rickettsii
54
E. Rickettsia typhi
55
56 F. Treponema pallidum
57
58
59
60
61
62
63
64
0
65
• 66
0 67
0
68

8
L.odt
s
Su~pl'nd
~
End Block
Item: 6S of ~ ,• Mark <::J [:::> ""I ~· ~'j
100 J.. Previous Next LAbfaiUI~S Notes Calculator

• The correct answer is 0. 73°/o chose this .


43
44
This patient most likely has Rocky Mount ain spot ted fever, as indicat ed by t he rash on her palms and soles and t he inward, "cent ri pet al"
pattern of spread . other supporting evidence is t he accompanying headache and fever, and a posit ive Weii -Felix reaction, wh ich is a cross-
45 reaction of certain ant irickettsial ant ibod ies wit h t he Proteus ant igen . Rocky Mount ain spot ted feve r is caused by Rickettsia rickettsii and is
46 endemic to t he east coast of t he United States. It is t ransmit ted by the Dermacentor t ick; t hus t he patient probabl y became infect ed during
47 her recent outdoor excursion . Treat ment is with a course of doxycycline or, if necessary, ch loramphenicol.
Rocky Mountain spotted fever Rickettsia rickettsii Doxycycline Chloramphenicol Rickettsia Tick Antibody Fever Headache Dermacentor Spotted fever Antigen Proteus (bacterium) Rash
48
49 A is not correct. 9°/o chose this.
Borrelia burgdorferi causes Lyme disease. The rash of Lyme disease is typically a bull's-eye type rash known as eryt hema chro nicum m igrans,
50
and it has a negative Weii-Felix reaction. Treatment is wit h doxycycline .
51 Lyme disease Doxycycline Borrelia burgdorferi Erythema Erythema chronicum migrans Borrelia Rash

52
B is not correct. 4% chose this.
53
Coxiella burnetti is a ri cket tsial organism . It is highly infect ious, as it is t ransmitt ed by aerosolized endospores. It causes Q feve r~ which is
54 charact erized by high feve r, headache, shaking, malaise, and mya lgia. Q feve r can somet imes cause a life-t hreaten ing endocardit is, especially
55 in pat ients wit h abnorma l heart valves. It is not associated wit h rash or a posit ive Weii-Felix reaction .
Q fever Coxiella burnetii Myalgia Endocarditis Malaise Headache Coxiella (bacterium) Rickettsia Rash Heart valve Endospore Fever Organism
56
57 C is not correct. 7 °/o chose this .
58 Coxsackie A is an RNA virus t hat causes hand, foot , and mouth disease, which can also present wit h a rash on the palms and soles, in addit ion
t o oral and occasionally genita l lesions. However, t he positive Weii-Felix react ion in t his case and recent history of a camping t ri p point to
59
Rickettsia rickettsiae as a more likely causat ive organ ism in t his case.
60 Hand, foot and mouth disease RNA virus Virus Rickettsia RNA Rash

61
E is not correct. 6°/o chos e this.
62
Rickettsia typhi causes endem ic typhus and is t ransm it ted by f leas. This disease is often spread in refugee camps and war zones, where
63 rodents can spread t he fleas that carry t his disease. The rash of typhus is cent rifuga l; it spreads outward, not inward, as in t his pat ient.
Typhus Rickettsia typhi Murine typhus Rickettsia
64
65 F is not correct. 1 °/o chose this.
• 66 Treponema pal/idum is t he spirochet e that causes syphilis, a sexually t ransm itted disease. Alt hough secondary syphilis can also present wit h a
• 67 rash on the palms and soles, this patient has no history of a sexual encounter that would put her at risk fo r t his disease. Social hist ory
suggests a t ick-borne disease .
• 68 Sexually transmitted infection Spirochaete Treponema pallidum Syphilis Tick-borne disease Secondary syphilis Rash Treponema
. •
8
Lock
s
Suspend
0
End Block
Item: 6S of ~ ,• Mark <::J [:::> ""I ~· ~'j
100 J.. Previous Next faiUI~S
LAb Notes Calculator

43 Bottom Line:
44 Pat ients complaining of headache and centripetal rash after an outdoor t ri p should receive a work-up fo r Rocky Mount ain spotted feve r.
45 Rocky Mountain spotted fever Rash Headache Fever Spotted fever Centripetal force

46
47
48 l@l;fil·1i•l f or yea r:[ 2017
FIRST AID FACTS .
•j .
49
50
FA17 p 145.1
51
52
Rickettsial diseases Treatment: doxycycline (caution during pregnancy; alternative is chloramphenicol).
53
and vector-borne
54 illnesses
55 RASH COMMON
56 Rocky Mountain Rickettsia rickettsii, vector is tick. Despite its Classic triad- headache, fever, rash (vasculitis).
57 spotted fever name, disease occurs primarily in the South Palms and soles rash is seen in Coxsackievirus
58
Atlantic states, especially J orth Carolina. A infection (hand, foot, and mouth disease),
59
Rash typically starts at wrists · and ankles and Rocky Mountain spotted fever, and 2° Syphilis
60
then spreads to trunk, palms, and soles. (you drive CARS using your palms and soles).
61
62 Typhus Endemic (Aeas)-R typhi. Rickettsii on the wRists, Typhus on the Trunk.
63 Epidemic (human body lousc)- R prowazekii.
64 Rash starts centrally and spreads out, sparing
65 palms and soles.
• 66 RASHRARE
• 67
Ehrlichiosis Ehrlichia, vector is tick. \1onocytes with MEGA berry-
morulae rn (mulberry-like inclusions) in
• 68
. Monocvtes = Ehrlichiosis
8
Lock
s
Suspend
0
End Block
Item: 6S of ~ ,• Mark <::J [:::> ""I ~· ~'j
100 J.. Previous Next faiUI~S
LAb Notes Calculator


43 Epidemic (human body lousc)- R prowazekii.
44 Rash starts centrally and spreads out, sparing
45 palms and soles.
46 RASHRARE
47
Ehrlichiosis Ehrlichia, vector is tick. \itonocytes with MEGA berry-
48
49
rn
morulae (mulberry-like inclusions) in Monocytes = Ehrlichiosis
50
cytoplasm. Granulocytes = Anaplasmosis
51 Anaplasmosis Anaplasma, vector is tick. GranulOC)ies with
52 morulae 9 in cytoplasm.
53 Qfever Coxiella bumetii, no arthropod vector. Spores Q fever is Queer because it has no rash or vector
54
inhaled as aerosols from cattle/sheep amniotic and its causative organism can survive outside
55
Auid. Presents as pneumonia. Common cause in its endospore form. ot in the Rickettsia
56
e
of culture endocarditis. genus, but closely related .
57 .._..;:::,;-;:;;;:-;;;:::;::;-:;:::::;
58
59
60
61
62
63
64
65
• 66
• 67
• 68
. •
8
Lock Suspend
s 0
End Block
Item: 65 of - ,• Mark -<] 1:> ""'I ~· 1!';:'1
100 ~ Prev1o u s Next Labf a lu es Note s Calculator

43 FA17 p 124.1
44 Intracellular bugs
45
Obligate intracellular Rickettsia, CIIlamydia, COxiella. Relr on host Stay inside (cells) when it is Really CHilly and
46
ATP. COld.
47
48 Facultative Salmonella, ~eisseria, Brucella, ~lrcobacterium, Some Nasty Bugs \ lay Li,·e FacultativeLY.
49 intracellular Listeria, Francisellc1, Legionella, )ersinia pestis.
50
51 FA17 p 142.4
52
Lyme disease Caused by Borrelia burgdorferi, which is A Key Lyme pie to the E \ CE:
53
54
transmitted by the Ixodes deer tick (also Facial ncn·e palsy (typically bilateral)
55
vector for Anaplasma spp. and protozoa Arthritis
56
Babesia). Natural reser\'oir i~ the mouse (and Card iac block
57
important to tick life cycle). Erythema migrans
58 Common in northeastern United States. Treatment: doxycycline (1st line); amoxicillin
59 Stage !-early localized: erythema migrans and ccfuroxime in pregnant women and
60 (typica I "bu lls-eye" con figuration . is children.
61 pathognomonic but· not always present),
62 Au-like symptoms.
63 Stage 2-early disseminated: secondary lesions,
64 carditis, AV block, facial nerve (Bell) palsy,
65 migratory myalgias/transicnt arthritis.
• 66 Stage 3-late disseminated: encephalopathies,
67
0
chronic arthritis.
0
68

8
L.odt
s
Su~pl'nd
~
End Block
Item: 66 of - ,• Mark --<) [::> ""'I ~· 1!';:'1
100 ~ Prev1o u s Next Labf a lu es Notes Calculator


43 A couple presents to the emergency department wit h their 2-month-old son. The parents report that t heir son has never been fussy.
44 However, last night he began to cry incessantly and neith er changing his diapers nor feeding was able to calm him. This morning he had
45
become very lethargic and warm, and his temperature reached 39.4°C (103°F). The physician examines the baby and notices some rig idity
in his nuchal region. The physician becomes ext remely concerned and orders a lumbar puncture, which revea ls an elevated opening pressure,
46 elevated protein level, decreased glucose level, and numerous polymorphonuclear cells.
47
48 What is the most likely cause of this patient's symptoms?
49
:
50 A. Cryptococcus neoformans
51
B. Cytomegalovirus
52
53
C. Enterovirus
54 D. Escherichia coli
55
E. Haemophilus influenzae
56
F. Neisseria meningitidis
57
58
59
60
61
62
63
64
65
• 66
• 67
• 68

8
L.odt
s
Su~pl'nd
~
End Block
Item: 66 of ~ ,• Mark <::J [:::> ""I ~· ~'j
100 J.. Previous Next LAb faiUI~S Notes Calculator


43 The correct a nswer is 0. 5 1°/o chose t his.
44 The concern for t his pat ient is mening it is, wh ich can be caused by bacteria, viruses, or f ungi. The classic t riad is fever, headache, and nuchal
45 rigidity. Ot her symptoms such as rash and photophobia are also commonly present . In neonates, though, it may be difficult to identify these
symptoms. I nference from the history, such as severe cry ing in a normally quiet baby, may indicate headache or muscle st iffness. A lumbar
46 punct ure and cerebrospinal f luid (CSF) analysis is essential in det erm ining t he cause. Classic CSF find ings for bact erial mening it is are elevat ed
47 open ing pressure, elevat ed levels of polymorphonuclear leukocyt es, elevat ed protein levels, and decreased glucose levels, as seen in t his
48 patient. In newborns from 0- 6 months old, the three most common causes of mening it is are all bacterial and include grou p B st re ptococci,
Escherichia coli, and Listeria. Culture and m icrobiology analysis can different iate among these species. However, the mortalit y rat e fo r
49 untreat ed meningit is is high, and patients can quickly det eri orate. Therefore, empiric t reat ment should be init iated pri or t o obtaining t he
50 results of CSF analysis.
Lumbar puncture Escherichia coli Cerebrospinal fluid Photophobia Meningitis Bacterial meningitis Headache Neck stiffness White blood cell Fungus Meningism Streptococcus Protein Virus
51
Streptococcus agalactiae Bacteria Infant Glucose Listeria Mortality rate Microbiology Granulocyte Empiric therapy Rash Muscle Fever
52
53 A is not correct . 3°/o chose this.
54 Cryptococcus mostly causes mening it is in pat ients wit h HIV/AIDS . Cerebrospinal f luid find ings for funga l mening it is are most often an
55 elevated opening pressure, elevated lymphocyt es, elevat ed protein level, and decreased glucose level. The number of polymorphonuclea r
leukocytes is norma l, different iating it f rom bact erial causes.
56 Cerebrospinal fluid Meningitis White blood cell Protein Fungal meningitis New England Compounding Center meningitis outbreak Cryptococcus Glucose Lymphocyte Fungus Granulocyte
57
B is not correct. 4 % chose this .
58
Cytomegalovirus is anot her pathogen t hat causes meningit is in pat ients with HIV/AIDS and immunosuppressed pat ients. Findings on
59 cerebrospinal fluid ana lysis are typical fo r viral men ingit is (norma l or slight ly elevated opening pressure, elevated lym phocytes, norma l protein
60 level, and normal glucose level) .
Cytomegalovirus Cerebrospinal fluid Viral meningitis Meningitis Protein Immunosuppression Pathogen Lymphocyte Glucose HIV/AIDS Virus
61
62 C is not co rrect. 4 °/o chose this .
63 Enterovirus commonly causes vira l meningit is in child ren. Clinically, t he specific v irus causing viral mening it is is not always det erm ined, but
t he family of enterovirus is most common ly seen . Cerebrospinal fluid ana lysis in v iral meningit is revea ls normal or slight ly elevat ed opening
64
pressu re, elevated lymphocytes, normal protein level, and normal glucose level.
65 Enterovirus Cerebrospinal fluid Viral meningitis Meningitis Protein Glucose Virus Lymphocyte
66
E is not correct. 24 °/o chose this .
• 67
Haemophilus influenzae ty pe B used to be a very common cause of bacteria l meningit is in ch ildren between the ages of 6 mont hs and 6 years.
• 68 However, t he incidence has dramatically decreased in t he past 15 yea rs wit h the int roduct ion of H. influenzae type B vaccines .
. •
8
Lock
s
Suspend
0
End Block
Item: 66 of ~ ,• Mark <::J [:::> ""I ~· ~'j
100 J.. Previous Next LAbfaiUI~S Notes Calculator

43
F is not correct. 14% chose this .
44 Neisseria meningitidis is t he most common cause of mening it is is patients 6- 60 years old. It is also seen less f requently in pat ients age 6
45 months to 6 years. It is rarely seen in newborn s. Findings of cerebrospinal f luid analysis would be consist ent wit h bact erial men ingit is.
Neisseria meningitidis Cerebrospinal fluid Meningitis Bacterial meningitis Neisseria
46
47
48 Bottom Line:
49 Group B streptococci, E. coli, and Listeria are t he most common causes of men ingit is in children 0- 6 mont hs old.
50 Escherichia coli Meningitis Streptococcus agalactiae Streptococcus Listeria

51
52
53 141;fil·1i•J for year:[2017
FIRST AID FA CTS .
•j .
54
55
FA17 p 176.1
56
Common causes of meningitis
57
NEWBORN (0- 6MOl CHILDREN (6 M0- 6 YRI 6- 60YR 60YR+
58
59
Group B streptococci S pneumoniae S pneumoniae S pneumoniae
60
E coli N meningitidis N meningiticlis (#1 in teens) Gram 8 rods
61 Listeria H infiuenzae type B Enteroviruses Listeria
62 Enteroviruses HSV
63 Give ceftriaxone and vancomycin empirically (add ampicillin if Listeria is suspected).
64 Viral causes of meningitis: enteroviruses (especially coxsackievirus), llSV-2 {llSV-1 = encephalitis), IIIV, West ile virus (also
65 causes encephalitis), VZV.
66
In HIV: Cryptococcus spp.
• 67
Note: Incidence of H infiuenzae meningitis has l greatly due to conjugate H infiuenzae vaccinations. Today, cases are usually
• 68
. seen in unimmunized children .
8
Lock
s
Suspend
0
End Block
Item: 66 of ~ ,• Mark <::J [:::> ""I ~· ~'j
100 J.. Previous Next LAbfaiUI~S Notes Calculator

• •
43 FA17 p 176.2
44 CSF findings in meningitis
45 OPEN ING PRESSURE CELL TYPE PROTEIN GLUCOSE
46
Bacterial t t PM s t l
47
48 Fungai/TB t t lymphocytes t l
49 Viral ormal/t t lymphocytes ormal/t ormal
50
51
52 FA17 p 140.1
53 Escherichia coli Cram 8 rod. E coli virulence factors: fimbriae-cystitis and pyelonephritis (P-pili); K capsule-
54 pneumonia, neonatal meningitis; LPS endotoxin-septic shock.
55
56 STRAIN TOXINAND MECHANISM PRESENTATION
57 EIEC Microbe invades intestinal mucosa and causes Invasive; dysentery. Clinical manifestations
58 necrosis and inflammation. similar to Shigella.
59
ETEC Produces heat-labile and heat-stable Travelers' diarrhea (watery).
60
enteroToxins. No inflammation or invasion.
61
62 EPEC o toxin produced. Adheres to apical surface, Diarrhea, usually in children (Pediatrics).
63 fl attens villi, prevents absorption.
64 EHEC 0 157: 11 7 is most common serotype in US. Often Dysentery (toxin alone causes necrosis and
65 transm itted via undercooked meat, raw leafy inflammation).
66 vegetables. Does not ferment sorbitol (vs other E coli).
• 67
Shiga-! ike toxin causes hemolytic-uremic Hemorrhagic, Hamburgers, llcmolytic-uremic
• 68
. • syndrome: triad of anemia, thrombocytopenia, syndrome. •

8
Lock
s
Suspend
0
End Block
Item: 67 of - ,• Mark -<J
P~v1ous
[:::> "'I ~ ·· ~
100 ~ N @xt Labl lues N o tes Calcula to r
6

43 A 35-year-old woman comes t o th e physician because of a feve r~ malaise, headache, and a nonproduct ive cough. Aft er obtaining the ~~AI
44 patient's medical history and conducting a physical examination, the physician suspects atypical pneumonia.
45
46 Which of the following tests can support a clinical diagnosis of Mycoplasma pneumoniae infection?
47 :
48 A. c-ANCA in serum
49 B. Cold agglutinin titer in serum
50
C. Enzyme-linked immunosorbent assay to detect p24
51
52 D . Fluorescent treponema! antibody absorption test
53 E. Western blot to detect antibodies t o viral proteins
54
55
56
57
58
59
60
61
62
63
64
65
66
. 67
• 68
. •
8
Lode.
s
Suspe-nd
8
End Bloc:k
Item: 67 of ~ ,• Mark <::J [:::> ""I ~· ~'j
100 J.. Previous Next LAbfaiUI~S Notes Calculator

43
44
45 The correct answer is B. 87% chos e this.
46 Diffe rentiat ing v iral at ypical pneumonia caused by Mycoplasma pneumoniae can be done through detect ion of elevated cold agg lutinin t it ers in
47 t he serum. Cold agg lut inins are ant ibodies t hat cause RBCs to clump t oget her (agglut inate) at cold t emperatures. These are present in about
half of patients with M. pneumoniae infect ion. These cold agg lut inins develop 1-2 weeks after infection. The incidence of cold agglut inins is
48
highest in children and decreases wit h age. However, ot her bact eri al, ri ckettsial, and v iral (in part icular, adenovirus and inf luenza) infect ions
49 can also result in t he product ion of cold agglut inins, though not nearly as commonly as in Mycoplasma . As a result, test ing for cold agglut inins
50 is neit her a sensit ive nor specific, and the test is not widely used in clinical practice. However, t he associat ion between cold agg lut inins and
51
mycoplasma is well-established and freq uent ly tested.
Mycoplasma pneumoniae Atypical pneumonia Mycoplasma Adenoviridae Pneumonia Cold agglutinin disease Agglutination (biology) Antibody Influenza Virus Sensitivity and specificity Serum (blood)
52
Blood plasma Rickettsia
53
54
A is not correct. 2°/o chose this.
c-ANCA, also known as PR3-ANCA or cyt oplasmic ant ineut rophil cytoplasmic ant ibody is commonly found in granu lomatosis with polyangiit is
55
(previously known as Wegener granulomatosis) . This disease is characteri zed by a t ri ad of foca l necrot izi ng vasculitis, necrotizing granulomas
56 in the lung and upper airway, and necrotizing glomerulonephrit is. Affected individuals present wit h sym pt oms of upper respiratory t ract
57 infection (sinusit is, mastoidit is, ot it is media), sadd le nose deformit y, hemoptysis, and hemat uria wit h RBC casts. Treatment is wit h
cyclophosphamide and corticost eroids.
58
Granulomatosis with polyangiitis Hemoptysis Cyclophosphamide Hematuria Otitis media Upper respiratory tract infection Anti-neutrophil cytoplasmic antibody Sinusitis Vasculitis Glomerulonephritis
59
Corticosteroid Saddle nose Granuloma Antibody Respiratory tract Urinary cast Mastoiditis Necrosis Lung Upper respiratory tract Respiratory tract infection systemic vasculitis C-ANCA Cytoplasm
60
61
C is not correct. 6 °/o chose this .
Enzyme -l inked immunosorbent assay (ELISA) is used t o det ect ant ibodies against t he HI V ant igen p24 in serum. ELISA is used in conj unction
62
with t he West ern blot to diagnose HI V infection .
63 Western blot HIV EUSA Antibody Antigen Diagnosis of HIV/AIDS Assay Blood plasma Serum (blood) Infection
64
D is not correct. 2°/o chose this.
65
The fluorescent t reponema! ant ibody absorpt ion t est is used to det ect ant ibodies direct ed specif ically to t he bacteria t hat cause syphilis,
66 Treponema pallidum. This t est is used rout inely to conf irm whether a posit ive screen ing t est for syphilis ref lects t rue infect ion with syphilis.
67 Treponema pallidum Syphilis Antibody Bacteria Treponema Fluorescence Screening (medicine)

• 68 E is not correct. 3 °/o chos e this.


.
8
Lock
s
Suspend
0
End Block
Item: 67 of ~ ,• Mark <::J [:::> ""I ~· ~'j
100 J.. Previous Next LAb faiUI~S Notes Calculator

43
• E is not correct. 3 °/o chos e this .
For t esting purposes, West ern blot is used t o confirm a diagnosis of HIV infect ion by using ant i-HIV ant ibodies in the patient's serum.
44
However, in rece nt pract ice, PCR is more often used as the confirmatory t est fo r HI V. When diagnosing an individual wit h HIV, the first step is
45 an antigen/antibody immunoassay that detects HI V- 1 and HIV-2 antibodies and HIV- 1 p24 antigen . If reactive, t he second st ep is an FDA-
46 approved ant ibody immunoassay t hat different iates HI V- 1 from HIV-2 antibodies.
Western blot Immunoassay P24 antigen HIV Antibody Antigen Polymerase chain reaction Blood plasma Serum (blood) Infection
47
48
49 Bottom Line :
50 Cold agglut inin tit er can different iate Mycoplasma pneumoniae infection from v iral atypical pneumonia.
51 Mycoplasma pneumoniae Atypical pneumonia Pneumonia Mycoplasma Cold agglutinin disease Virus

52
53
54 l@ljl'il·1i•J for yea r:[2017 • J
FIRST AID FA CTS

55
56 FA17 p 146.3
57
Mycoplasma Classic cause of atypical "walking" pneumonia 1o
cell wall. Not seen on Gram stain.
58
59
pneumoniae (insid ious onset, headache, nonproductive Pleomorphic rJ.
60
cough, patchy or diffuse interstitial infiltrate). Bacterial membrane contains sterols for stability.
61
X-ray looks worse than patient. ll igh titer of Mycoplasmal pneumonia is more common in
62 cold agglutinins (IgM), which can agglutinate patients < 30 years old.
63 or lyse RBCs. Grown on Eaton agar. Frequent outbreaks in military recruits and
64 Treatment: macrolides, doxycycl ine, or pnsons.
65 Auoroquinolone (penicillin ineffective since Mycoplasma gets cold without a coat (cell wa ll).
66 Mycoplasma have no cell wall).
67
• 68
.
8
Lock
s
Suspend
0
End Block
Item: 67 of ~ ,• Mark <::J [:::> ""I ~· ~'j
100 J.. Previous Next LAbfaiUI~S Notes Calculator


43
44 FA17 p 645.1
45 Pneumonia
46
TYPE TYPICAL ORGANISMS CHARACTERISTICS
47
Lobar S pneumoniae most frequently, also Legionella, Intra-alveolar exudate -+ consolidation r.il; may
48
49
Klebsiella rn
iJwolve entire lobe or lung.
50 Bronchopneumonia S pneumoniae, S aureus, H infiuenzae, Acute inflammatory infiltrates B1 from
51 Klebsiella bronchioles into ad jacent alveoli; patchy
52 distribution involving;;:: 1 lobe (!].
53
Interstitial (atypical) Mycoplasma, Chlamydophila pneumoniae, Diffuse patchy inAammation localized to
54
pneumonia Chlamydia psittaci, Legionella, viruses (RSV, interstitial areas at alveolar walls; diffuse
55
C tfV, inAuenza, adenovirus) distribution involving;;:: 1 lobe 1]. Generally
56
follows a more indolent course ("walking"
57
pneumonia).
58
59 Cryptogenic Formerly known as bronchiolitis obliterans
60 organizing organizing pneumonia (BOOP). Noninfectious
61 pneumonia pneumonia characterized by in fla mmation of
62 bronchioles and surrounding structure. Etiology
63 unknown. Secondary organizing pneumonia
64 caused by chronic inAammatory diseases (eg,
65 rheumatoid arthritis) or medication side effects
66
(eg, amiodarone). e sputum and blood cultures,
67
• 68
. • F.i~~====~~-=~~~----~~~----~~
i;-~!~~
~=~~-~r..~
8
......
no response to antibiotics.
.a........ l~~~
s 0
Lock Suspend End Block
Item: 67 of ~ ,• Mark <::J [:::> ""I ~· ~'j
100 J.. Previous Next LAb faiUI~S Notes Calculator

43
44
45
46
47
48
49
50
51
52
FA17 p 175.2
53
Common causes of pneumonia
54
NEONATES(< 4WK) CHILDREN (4WK- 18 YR) ADULTS (18-40YR) ADULTS (40-65 YR) ELDERLY
55
56 Group B streptococci Viruses (RSV) Mycoplasma S pneumoniae S pneumoniae
57 E coli Mycoplasma C pnewnoniae H influenzae 1nAuenza virus
58 C trachomatis S pneumoniae Anaerobes Anaerobes
59 (infants-3 yr) Viruses (eg, influenza) Viruses H influen:we
60 C pnewnoniae Mycoplasma Cram 8 rods
61 (school-aged
62 children)
63 S pneumoniae
64 Runts May Cough
65
Chunky Sputum
66
Special groups
67
• 68
Alcoholic Klebsiella, anaerobes usually due to aspiration (eg, Peptostreptococcus, Fusobacterium, Prevotella,
0 _,., ,. ..,.,.:,1,.,.\
.
Lock
8 s
Suspend
0
End Block
Item: 67 of ~ ,• Mark <::J [:::> ""I ~· ~'j
100 J.. Previous Next LAbfaiUI~S Notes Calculator

43
Common causes of pneumonia
44
NEONATES(< 4WK) CHILDREN (4 WK-18YR) ADULTS (18-40YR) ADULTS (40-65 YR) ELDERLY

45 Group B streptococci Viruses (RSV) Mycoplasma S pneumoniae S pneumoniae


46 E coli Mycoplasma C pnewnoniae H influenzae 1nAuenza virus
47 C trachomatis S pneumoniae Anaerobes Anaerobes
48 (infants-3 yr) Viruses (eg, influenza) Viruses H influen:we
49 C pnewnoniae Mycoplasma Cram 8 rods
50
(school-aged
51
children)
52
S pneumoniae
53
Runts May Cough
54
Chunky Sputum
55
Special groups
56
57 Alcoholic Klebsiella, anaerobes usually due to aspiration (eg, Peptostreptococcus, Fusobacterium, Prevotella,
58 Bacteroides)
59
IV drug users S pneumoniae, S aureus
60
61
Aspiration Anaerobes
62 Atypical M)'coplasma, Legionella, Chlamydia
63
Cystic fibrosis Pseudomonas, S aureus, S pneumoniae, Burkholderia cepacia
64
65
lmmunocompromised S aureus, enteric gram 8 rods, fungi, viruses, P jirovecii (with II IV)
66 Nosocomial (hospital S aureus, Pseudomonas, other enteric gram 8 rods
67 acquired)
• 68
. Postviral S bnewnoniae. S aureus. 1-I influenzae
8
Lock
s
Suspend
0
End Block
Item: 68 of - ,• Mark -<] 1:> ""'I ~· 1!';:'1
100 ~ Prev1o u s Next Labf a lu es Note s Calculator


43 It has been estimated that 1 in 10 healt hcare workers in t he United States will suffer a needlestick injury or splash exposure during their r~AJ
44 career. Needlestlcks can lead to the accidental t ransmission of diseases, such as hepatitis C, hepat it is B, and HIV. Rates of transmission of
45
these diseases from needlestick vary based on infectivity, or the given virus' ability to establish infection.
46
In an individual who is unvaccinated, which of the following is the correct order for the ability of each of t hese viruses to establish infection (from
47
greatest chance to least chance) assuming each needle is infected?
48
:
49
A. Hepatitis B, hepatitis C, HIV
50
51
B. Hepatitis B, HIV, hepatitis C
52 C. Hepatitis C, hepatitis B, HIV
53 D. Hepatitis C, HIV, hepatitis B
54
E. HIV, hepatitis B, hepatitis C
55
56
57
58
59
60
61
62
63
64
65
66
67
• 68

8
L.odt
s
Su~pl'nd
~
End Block
Item: 68 of ~ ,• Mark <::J [:::> ""I ~· ~'j
100 J.. Previous Next LAb faiUI~S Notes Calculator

43 The correct a nswer is A. 4 7°/o chose t h is.


44 The Centers for Disease Cont rol and Prevention report that an unvaccinat ed healt h care worker has up to a 30% chance of cont racting
45 hepatitis B but only up to 10% ch ance of cont ract ing hepat it is C from a needlest ick inj ury. The Occupat ional Safety and Hea lt h Administ ration
46
reports t hat t he chance of HI V infect ion if accidentally st uck wit h a needle used on an HI V patient is between 0.3% and 0.45%.
Occupational Safety and Health Administration Centers for Disease Control and Prevention Hepatitis C Needlestick injury Hepatitis 8 HIV Hepatitis Health care Health professional
47
48
B is not correct. 11 % chose this .
Although hepat it is B has the highest infectivit y rat e out of t he t hree, hepat it is C has a higher chance of infectivity t han HIV.
49
Hepatitis C Hepatitis B Hepatitis Infectivity
50
51
C is not correct. 2 3% chose this .
Hepatitis B act ually has t he highest infectiv ity of t he t hree pathogens in this quest ion. The order is wrong and therefore t his answer is
52
incorrect.
53 Hepatitis B Hepatitis Infectivity Pathogen

54
D is not correct. 11% c hose this.
55
Hepatitis C has a lower infect ivit y rat e than hepat itis B in health care workers. Also, in comparison with HI V, hepat it is B has a higher infect ivity
56 rat e.
Hepatitis C Hepatitis B Hepatitis HIV Infectivity Health care
57
58 E is not correct. 8°/o chose this .
59 HI V actu ally has the lowest rate of infectivity of the three list ed diseases. The order is wrong and therefore t his answer is incorrect.
HIV Infectivity
60
61
62 Bottom Line :
63 I nfectivity refers t o t he ability of a pat hogen t o est ablish infection. This concept is im portant when t hinking about accidental needlest icks t hat
64 can occur in a health care sett ing. The risk of being infect ed with hepatit is B is highest at 30%, next is hepat it is C at 10%, and last is HI V, at
65
0 .3%- 0.45% .
Hepatitis C Hepatitis B Hepatitis HIV Pathogen Infectivity Needlestick injury Infection
66
67
68
. .. ljl:fil·11•i f or vea r:l 2017 .,. I
8
Lock
s
Suspend
0
End Block
Item: 68 of - ,• Mark -<] 1:> ""'I ~- 1!';:'1
100 ~ Prev1o u s Next Labf a lu es Note s Calculator

43 FA17 p 170.1
44 Hepatitis serologic markers
45
46 Anti-HAV (lgM) Ig I antibody to II V; best test to detect acute hepatitis .
47 Anti-HAV (lgG) IgC antibody indicates prior HAV infection and/or prior vaccination; protects against reinfection.
48
HBsAg Antigen found on surface of I JBV; indicates hepatitis B infection.
49
50 Anti-HBs Antibody to HBsAg; indicates immunity to hepatitis B due to ,-accination or reco,·cry from
51 infection.
52 HBcAg Antigen associated with core of II B .
53
Anti-HBc ntibody to HBc g; I g~1 = acute/recent infection; IgC =prior exposure or chronic infection. lg\11
54
anti-JIBe may be the sole® marker of infection during window period.
55
56 HBeAg Secreted by infected hepatocyte into circulation. Not part of mature HBV ,·irion. Indicates active
57 viral replication and therefore high transmissibility and poorer prognosis.
58 Anti-HBe Antibody to HBeAg; indicates low transmissibil it).
59
60 Important Incubation Prodrome. Convalescence
diagnostic period acute disease Early Late
61 tests
HBsAg Anti· Ant1·HBs Surface antigen
62 HBsAg
tanb-HBc} HBc (ant1-HBcJ HBsAg \
DNA
63
Anti -HBc polymerase
64 DNA
Relative polymerase
65 concentration
of reactants H BV paructes DNA~ ,....,
66 Envelope
antigen (-)

I
67 HBsAg HBeAg - - - . (+)
68 \'"-
. . . ·- •
8
L.odt
s
Su~pl'nd
~
End Block
Item: 68 of - ,• Mark -<] 1:> ""'I ~- 1!';:'1
100 ~ Prev1o u s Next Labf a lu es Note s Calculator


43 Important Incubation Prodrome. Convalescence
diagnostic penod acute disease Early Late
44
tests
45 HBsAg Anb· Antt·HBs Surface antigen
HBsAg
(anb·HBcl HBc lantt· HBc l HBsAg \
46 DNA
Anti· HBc polymerase
47 DNA
Relative polymerase
48 concentration
HBV particles
of reactants
49
50 HBsAg
51
52 Anti· HBs
Core anttgen /
53 HBeAg Anti·HBe HBcAg . . /
54 Level of
detection
55
56 Months after 0 1 2 4 5 6 7 8
exposure Symptoms
57
58 SGPT!ALTJ
59
60 HBsAg Anti- HBs HBeAg Anti-HBe Anti-HBc
61
Acute HBV ./ ./ lg 1
62
63 Window lgM
64 Chronic HBV (high infectivity) ./ ./ lgC
65
Chronic HBV (low infectivity) ./ ./ lgC
66
67 Recovery ./ ./ lgC
68 Immunized ./ •
8
L.odt
s
Su~pl'nd
~
End Block
Item: 68 of ~ ,• Mark <::J [:::> ""I ~· ~'j
100 J.. Previous Next LAb faiUI~S Notes Calculator

43
FA17 p 168.1
44
45 Hepatitis Signs and symptoms of all hepatitis viruses: episodes of fever, jaundice, t ALT and AST. Naked viruses (HAV
46 viruses and HEV) lack an envelope and are not destroyed by the gut: the vowels hit your bowels.
47 HBV 0 A polymerase has 0 A- and RNA-dependent activities. Upon entry into nucleus, the polymerase
48 completes the partial dsD 1A. Host R A polymerase transcribes mRNA from \·iral D 'A to make vira l
49 proteins. The D A polymerase then reverse transcribes viral R1 A to D1 A, which is the genome of the
50 progeny v1rus.
51 HCV lacks 3'-5' exonuclease activity - no proofreading ability - variation in antigenic structures of HCV
52 envelope protei ns. Host antibody production lags behi nd production of new muta nt strai ns of HCV.
53
Virus HAV HBV HCV HDV HEV
54
55
FAMILY R A picorna\·irus D lA hepadnavirus Rl A Aavivi rus RNA dcltavirus R1 A hepevirus
56 TRANSMISSION Fecal-oral (shellfish , Parenteral (Blood), Primarily blood Parenteral, sexual, Fecal-oral,
57 travelers, clay care) sexual (Baby- (IVDU, post- perinatal especially
58 making), perinatal transfusion) waterborne
59 (Birthing)
60
INCUBATION Short {weeks) Long (months) Long Superinfection Short
61
(l lDV after
62
HBV) = short
63
64
Coinfcc tion (HDV
65
with HBV) = long
66 CLINICAL COURSE Asymptomatic Initially like serum May progress to Similar to HBV Fulminant hepatitis
67 {usually), Acute sickness {fever, C irrhosis or in E xpectant
68 arthralgias, rash); Carcinoma (pregnant) women
.
8
Lock
s
Suspend
0
End Block
Item: 68 of ~ ,• Mark <::J [:::> ""I ~· ~'j
100 J.. Previous Next LAb faiUI~S Notes Calculator

• •
43
FA17 p 181.2
44
Nosocomial infections E coli (UTI) and S aureus (wound infection) are the two most common causes.
45
RISK FACTOR PATHOGEN UNIQUE SIGNS/SYMPTOMS
46
47 Antibiotic use Clostridium difficile Watery diarrhea, leukocytosis
48 Aspiration (2° to Polymicrobial, gram 8 bacteria, often Right lower lobe in li It rate or right upper/
49 altered mental status, midd le lobe (patient recumbent); purulent
anaerobes
50 old age) malodorous sputum
51
52
Decubitus ulcers, S aureus (including MRSA), gram 8 anaerobes Erythema, tenderness, indu ration, drainage
53
surgical wounds, (Bacteroides, Prevotella, fr'usobacteriurn) from surgical wound sites
54 drains
55 Intravascular catheters S aureus (including MRSA), S epidermic/is {long Erythema, induration, tenderness, drainage
56 term), Enterobacter from access sites
57
Mechanical ventilation, Late onset: P aeruginosa, Klebsiella, ew infiltrate on CXR, t sputum production;
58
endotracheal Acinetobacter, S attreus sweet odor (Pseudomonas)
59
intubation
60
61 Renal dialysis unit, HBV, HCV
62 needlestick
63 Urinary catheterization E coli, Klebsiella, Proteus spp. Dysuria, leukocytosis, Aan k pain or
64
costovertebral angle tenderness
65
66 Water aerosols Legionella Signs of pneumonia, Cl symptoms (nausea,
67 vomiting), neurologic abnormalities
68

. •
8
Lock
s
Suspend
0
End Block
Item: 69 of - ,• Mark -<J [:::> "'I ~ · ~
100 ~ P~v1ous N @xt Labl lues N o tes Calcula to r
43
A 10-year-old girl who was born in Mexico is brought t o t he county health clinic by her parents. She has a 4-day hist ory of a runny nose ~~AI
44
and a violent, repetitive cough charact erized by a quick, noisy inspiratory effort and vomit ing. Cultures of throat and nasal swabs on
45 Bordet-Gengou agar are found to produce an exotoxin th at affects cAMP levels.
46
47 Which other organism also produces a toxin that increases cAMP levels?
48
:
49 A. Corynebacterium diphtheriae
50
B. Clostridium botulinum
51
52 C. Clostridium perfringens
53 0 . Clostridium tetani
54
E. Vibrio cholerae
55
56
57
58
59
60
61
62
63
64
65
66
67
68
. 69 •
8
Lode.
s
Suspe-nd
8
End Bloc:k
Item: 69 of ~ ,• Mark <::J [:::> ""I ~· ~'j
100 J.. Previous Next LAbfaiUI~S Notes Calculator

43
44 The correct answer is E. 70°/o chose this .
45 This patient 's sym ptoms are consist ent wit h whooping cough. This is a disease seen mainly in infants and young child ren, although it is not
46 seen frequent ly in the Un it ed Stat es because of t he nearly universa l use of t he pertussis vaccine. A loud, inspirat ory "whoop" fo llows v iolent
expirat ory coughing. The cause of whooping cough is Bordetel/a pertussis infection . This bact eriu m can be grown on Bordet -Gengou agar and
47 produces an exotoxin t hat ADP- ri bosylates and inact ivat es G; prot eins. By " inhibit ing the inhibit or," pe1t ussis toxin leads to increased cAMP
48 levels, which can inhibit t he chemokine rece pt or and cause lymphocytosis. Vibrio cholerae also produces an exotoxin t hat leads to increased
49 cAMP levels by permanent ly act ivat ing G5 ; it causes a profuse, wat ery dia rrh ea.
Vibrio cholerae Bordetella pertussis Pertussis toxin Exotoxin Chemokine Pertussis Lymphocytosis Diarrhea ADP-ribosylation Bordetella Cyclic adenosine monophosphate Bordet-Gengou agar
50
Pertussis vaccine Bacteria vaccine Agar Toxin Cough Receptor (biochemistry)
51
52 A is not correct. 14% chos e this.
53
Corynebacterium diphtheriae produces an exot oxin t hat inactivat es elongation factor- 2, which is involved in prot ein t ranslat ion, not cAM P
signaling . Dipht heri a is characteri zed by a sore t hroat , low-grade feve r, and t onsillar exudat es. The exotoxin produced by Pseudomonas
54 aeruginosa has a mechanism of act ion similar to that of Corynebacterium diphtheriae.
55 Corynebacterium diphtheriae Pseudomonas aeruginosa EEF2 Diphtheria Exotoxin Protein Pseudomonas Protein biosynthesis Corynebacterium Cyclic adenosine monophosphate Translation (biology) Fever

56 Sore throat

57 B is not correct. 5% chose this.


58 Clostridium botulinum produces a t oxin t hat inhibi ts t he release of acet ylcholine at t he neuromuscular j unct ion, causing a flaccid paralysis. It
59 does not induce cAMP production. C botulinum infect ion can be caused by ingestion of canned foods or in an infant by ingest ion of honey.
Clostridium botulinum Neuromuscular junction Flaccid paralysis Acetylcholine Cyclic adenosine monophosphate Botulinum toxin Toxin Paralysis Honey
60
61 C is not correct. 6 °/o chose this .
62 Clostridium perfringens- a gram-posit ive, rod-shaped anaerobe found in the soil- is responsible fo r gas gangrene. It produces an a t oxin (a
lecit hinase) t hat degrades t issue and cell membranes lead ing t o myonecrosis and hemolysis, but does not induce cAMP product ion .
63
C perfringens may also be ingest ed and may cause diarrhea.
64 Clostridium perfringens Gas gangrene Hemolysis Gangrene Gram-positive bacteria Lecithinase Anaerobic organism Diarrhea Clostridium Toxin Bacillus (shape) Cell membrane

65
D is not correct. 5°/o chose this.
66
Clostridium tetani, like other bact eria of t he Clostridium species, is a spore-form ing anaerobe found in soil. It produces an exotoxin that blocks
67 t he release of inhibit ory neu rotransm it ters GABA and glycine but does not induce cAMP production . This is t he organism responsible fo r
68 t et anus, which is characterized by severe muscle spasm (spastic paralysis), particularly of t he back and j aw.
Clostridium tetani Gamma-Aminobutyric acid Tetanus Exotoxin Glycine Anaerobic organism Clostridium Neurotransmitter Bacteria Spasm Cyclic adenosine monophosphate Paralysis Spasticity Organism
69 •
8
Lock
s
Suspend
0
End Block
Item: 69 of ~ ,• Mark <::J [:::> ""I ~· ~'j
100 J.. Previous Next LAbfaiUI~S Notes Calculator

43
44
Bottom Line:
45
Bordete/la pertussis infect ion causes whooping cough in part by producing an exotoxin that inhibits the inhibit ory G; protein, which results in
46 a net increase in cAM P levels. Vibrio cholerae similarly produces a toxin t hat increases cAM P levels, result ing in a prof use, watery diarrhea .
47 Vibrio cholerae Bordetella pertussis Pertussis Exotoxin Cyclic adenosine monophosphate Bordetella Diarrhea Protein Vibrio Cough Toxin Infection

48
49
50 i@l;fil·1i•J for year:[ 2017 • J
FIRST AID FACTS
51
52
FA17 p 128.1
53
Bugs with exotox ins
54
BACTERIA TOXIN MECHANISM MANIFESTATI ON
55
56
Inhibit protein synthesis
57 Corynebacterium Diphtheria toxin3 Pharyngitis with pseuclomembranes in th roat
58 diphtheriae Inactivate elongation factor and severe lymphadenopathy (bull neck)
59
Pseudomonas Exotoxin N (EF-2) Host cell death
60
aeruginosa
61
62 Shigella spp. Shiga toxin (ST)3 GI mucosal damage .... dysentery; ST also
63 Inactivate 60S ribosome by enhances cytokinc release, causing hemolytic-
64 removing adenine from uremic syndrome (HUS)
65
Enterohemorrhagic Shiga-like toxin rR A SLT enhances cytokine release, causing HUS
66
E coli (EHEC) (SLT)a (prototypically in EHEC serotype 0157:H7).
67
Unlike Shigella, EHEC does not invade host
68
cells
69

8
Lock
s
Suspend
0
End Block
Item: 69 of ~ ,• Mark <::J [:::> ""I ~· ~'j
100 J.. Previous Next faiUI~S
LAb Notes Calculator

43 •
44 Increase fluid secretion
45 Enterotoxigenic Heat-labile Overactivates adenylate Watery diarrhea: "labile in the Air (Adenylate
46 E coli (ETEC) toxin (LT)3 cyclase (t cAMP) ..... t Cl- cyclase), stable on the Ground (Guanylate
47
secretion in gut and H70 cyclase)"
48
efAux
49
I Ieat-stable Overactivates guanylate
50
toxin (ST) cyclase (t cGMP)
51
..... l resorption of NaCI
52
and H20 in gut
53
54 Bacillus anthracis Edema toxin" Mimics the adenylate Likely responsible for characteristic edematous
55 cyclase enzyme (t cA 1P) borders of black eschar in cutaneous anthrax
56
Vibrio cholerae C holera toxina Overactivates adenylate Voluminous "rice-water" diarrhea
57
cyclase (t cA 1P) by
58
permanently activating C5
59
..... t CI- secretion in gut
60
and H20 efAux
61
62 Inhibit phagocytic ability
63 Bordetella pertussis Pertussis toxina Overactivates adenylate Whooping cough -child coughs on expiration
64 cyclase (t cAMP) by and "whoops" on inspiration (toxin may
65
disabli ng C ;, impairing not actually be a cause of cough; can cause
66
phagocytosis to perm it "100-day cough" in adults)
67
survival of microbe
68
69
Inhibit release of neurotransmitter

8
Lock
s
Suspend
0
End Block
Item: 69 of - ,• Mark -<] 1:> ""'I ~· 1!';:'1
100 ~ Prev1o u s Next Labf a lu es Note s Calculator
43
Clostridium Botulinum toxin3 of proteins required for Flaccid paralysis, Aoppy baby; toxin prevents
44
45
botulinum neurotransmitter release release of stimulatory (ACh) signals at
46
via vesicular fusion neuromuscular junctions - Aaccid paralysis
47 a An AB toxin (aka, two-component toxin [or three for anthrax]} with B enabling binding and triggering uptake (endocytosis)
48 of the acti'e A component. The A components are usually ADP ribosylt ransferases; others have enzymatic activities as listed
49 in chart.
so
51 Lyse cell membranes
52 Clostridium Alpha toxin Phospholipase (lecithinase) Degradation of phospholipids - myonecrosis
53 perfringens that degrades tissue and ("gas gangrene") and hemolysis ("double zone"
54 cell membranes of hemolysis on blood agar)
55
Streptococcus Streptolysin 0 Protein that degrades cell l.yses RBCs; contributes to ~-h emolysis;
56
pyogenes membrane host antibodies against toxin (ASO) used to
57
58
diagnose rheumatic fe,·er (do not confuse
59
with immune complexes of poststreptococcal
60 glomerulonephritis)
61 Superantigens causing shock
62
Staphylococcus Toxic shock Binds to IIIC II and TC R Toxic shock syndrome: fever, rash, shock; other
63
aureus syndrome toxin outside of antigen binding toxins cause scalded skin syndrome (exfoliative
64
(TSS1~1 ) site to cause overwhelming to>.in) and food poisoning (heat-stable
65
66 release of IL-l, IL-2, enterotoxin)
67 Streptococcus Exotoxin A IF1 -y, and T lf'-a Toxic shock-like smdrome: fever, rash, shock;
68 pyogenes - shock scarlet fc,·cr
69 •
8
L.odt
s
Su~pl'nd
~
End Block
Item: 70 of - ,• Mark -<J [:::> "'I ~ · ~
100 ~ P~v1ous N @xt Labl lues N o tes Calcula to r
6

59
An HIV-positive 43-year-old woman comes to the clinic complaining of fever, headache, and vision problems that have lasted for the past ~~AI
60 few days. She reports t hat she has had to discont inue her daily visits to the park to feed the pigeons because of her worsening symptoms.
61 A spinal tap is ordered, and increased cerebrospinal f luid pressure is noted during the procedure.
62
63
Which of the fol lowing organisms is most likely to be seen on microscopic examination of t his patient's cerebrospinal flu id?
64 :

65
A. Broad-based budding yeast
66 B. Budding yeast with pseudohyphae
67
C. Encapsulated yeast with narrow-based budding
68
0 . Mold with hyphae that branch at 45-degree angles
69
. 70 E. Small intracellular yeast
• 71

' 72
• 73
• 74
. 75
• 76
· 77
' 78
' 79
. 80
. 81

' 82
' 83
' 84
.
8
Lode.
s
Suspe-nd
8
End Bloc:k
Item: 70 of ~ ,• Mark <:::1 t::> ""I ~· ~'j
100 J.. Previous Next LAbfaiUI~S Notes Calculator


59
The correct answer is C. 56°/o chose this.
60
Fever, headache, and vision changes in an HIV-infected pat ient who has had been exposed t o pigeon droppings and has an elevat ed opening
61 pressure on lumbar puncture should raise concerns about Cryptococcus neoformans meningoencephalit is. This is an AIDS -defining illness,
62 because it generally does not occur until t he CD4+ cell count is <50 . Under t he m icroscope, Clyptococcus appears as a heavily encapsulat ed
nondimorphic yeast wit h narrow-based budding .
63
Cryptococcus neoformans Lumbar puncture Headache Cryptococcus Yeast Fever Meningoencephalitis
64
A is not correct . 12% chos e this.
65
"Broad-based budding yeast " is the characterist ic descript ion of Blastomyces, a dimorphic fungus that forms a mold at lower t emperatures
66
and a yeast at body temperatures. Blastomyces is fo und in various wooded areas east of t he Mississippi River.
67 Mississippi River Dimorphic fungus Yeast Blastomyces dermatitidis Saccharomyces cerevisiae Fungus Mold Mississippi

68
B is not correct. 12% chose this .
69 Pseudohyphae are t ypical of Candida albicans, t he organism that causes thrush in immunocompromised pat ients. It can also cause
70 vulvovaginit is, disseminated cand idiasis (t o any organ), and chronic mucocutaneous candidiasis. Candida is ubiquit ous. Diagnosis is made
• 71 t houg h cultures and t issue biopsy fo r invasive syst emic disease .
Candida albicans Immunodeficiency Candidiasis Hypha Pseudohyphae Candida (fungus) Chronic mucocutaneous candidiasis Vaginitis Thrush (bird) Biopsy Systemic disease Organism
· 72
• 73 D is not correct. 9°/o chose this .
• 74
This patt ern is characteristic of Aspergillus fumigatus, a mold wit h septate hyphae t hat branch at a V-shaped 45-deg ree angle. A. fumigatus
infection can present as bronchopulmonary aspergillosis, lung cavity aspergillomas ("fungus balls"), or invasive aspergillosis.
. 75 Aspergillus fumigatus Aspergillosis Hypha Mold Aspergillus Septum Septate Lung Infection
• 76
E is not correct. 11°/o chos e this.
· 77
"Small int racell ular yeast " is t he ty pical descript ion of Histoplasma, fungi found in t he Mississippi and Ohio River valleys. Immunocom promised
• 78 ind ividuals who are exposed to bird or bat droppings in these areas are at increased risk for hist oplasmosis.
• 79 Histoplasmosis Fungus Ohio River Immunodeficiency Yeast Histoplasma Bird Mississippi Intracellular Bat Ohio

. 80
. 81
Bottom Line:
• 82
I n a pat ient with HI V infect ion, neurologic sym ptoms and an increased opening pressure on lumbar puncture suggest infect ion wit h
• 83 Cryptococcus neoformans, an encapsulat ed yeast wit h narrow-based budding . This is an AIDS -defining illness.
• 84 Cryptococcus neoformans Lumbar puncture HIV Yeast Cryptococcus Infection Lumbar

. •
8
Lock
s
Suspend
0
End Block
Item: 70 of - ,• Mark -<] 1:> ""'I ~· 1!';:'1
100 ~ Prev1o u s Next Labf a lu es Note s Calculator

59

FA17 p 149.1
60
61
Opportunistic fungal infections
62 Candida albicans cdba = white. Dimorphic; forms pseudoh) phac and budding yeasts at 20°C · , germ tubes at
63 37oc m .
64 Systemic or superficial fungal infection. Causes oral and esophageal thrush in
65 immunocompromised (neonates, steroids, diabetes, AI OS), vulvovaginitis (diabetes, use of
66 antibiotics), diaper rash, endocarditis (I drug users), disseminated candidiasis (especially in
67
neutropenic patients), chronic mucocutaneous candidiasis.
68
Treatment: oral fluconazole/topical a7olc for vaginal; nystatin, fluconazole, or caspofungin for oral/
69
esophageal; Auconazolc, caspofungin, or amphotericin B for systemic.
70
• 71 Aspergillus Septate hyphae that branch at 45° \ cute \ nglc . Produces conidia in radiating chains at end of
• 72 fumigatus conidiophore 1].
. 73 Causes invasive aspergillosis in immunocompromised, patients with chronic granulomatous disease.
• 74 Can cause aspergillomas in pre-existing lung cavities, especiall)' after TB infection .
. 75 Some species of Aspergillus produce AOatoxins (associated with hepatocellular carci noma).
• 76
Allergic bronchopulmonary aspergillosis (ABPA): h)'persensitivity response associated with
• 77
• 78
asthma and cystic fibrosis; may cause bronchiectasis and eosinophilia .
• 79 Cryptococcus 5-10 j.lm with narrow buckling. lleavily encapsulated yeast. r ot dimorphic.
. 80 neoformans Found in soil, pigeon droppings. Acquired through inhalation with hematogenous dissemination
. 81 to meninges. Culture on Sabouraud ag<Jr. ll ighlightcd with India ink (clear halo ) and
• 82 mucicarmine (red inner capsule [!1). Latex agglutination test detects polysaccharide capsular
• 83 antigen and is more specific.
• 84 Causes cryptococcosis, cryptococcalmcningitis, cryptococcal encephalitis ("soap bubble" lesions •
.
8
L.odt
s
Su~pl'nd
~
End Block
Item: 70 of ~ ,• Mark <:::1 t::> ""I ~· ~'j
100 J.. Previous Next LAb faiUI~S Notes Calculator

• •
59
FA17 p 173.1
60
61 Common d iseases of As CD4+ cell count ~. risks of reactivation of past infections (eg, TB, HSV, shingles), dissemination
62 HIV-posit ive adults of bacterial infections and funga l infections (eg, coccidioidomycosis), and non-Hodgkin
63 lymphomas t.
64
PATHOGEN PRESENTATION FINDINGS
65
66
CD4+ cell count < 500/mm 3
67 Candida albicans O ral thrush Scrapable wh ite plaque, pseudohyphae on
68 m1croscopy
69 EBV O ral hairy leukoplakia Unscrapable white plaque on lateral tongue
70
Bartonella henselae Bacillary angiomatosis Biopsy with neutrophilic in Aammalion
• 71
· 72 HHV-8 Kaposi sarcoma Biopsy with lymphocytic inflammation
• 73 HPV Squamous cell carcinoma, commonly of anus
• 74 (men who have sex with men) or cerYix
. 75 (women)
• 76
CD4+ cell count < 200/mm 3
· 77
• 78
Histoplasma Fever, weight loss, fatigue, cough, dyspnea, Oval yeast cells within macrophages
• 79 capsulatum nausea, vom iting, diarrhea
. 80 HIV Dementia
. 81
JC virus (reactivation) Progressive mu ltifoca l leukoencephalopathy onenhancing areas of demyelination on IRI
• 82
• 83
Pneumocystis jirovecii Pneumocrstis pneumonia "Ground-glass" opacities on CXR
• 84 CD4+ cell count< 100/ mm 3

. •
8
Lock
s
Suspend
0
End Block
Item: 70 of - ,• Mark -<] 1:> ""'I ~· 1!';:'1
100 ~ Prev1o u s Next Labf a lu es Note s Calculator

59 HIV Dementia
60 JC virus (reactivation) Progressive mu ltifocalleukoencephalopathy onenhancing areas of demyelination on r-. IRI
61
Pneumocystis jirovecii Pneumocystis pneumonia "Ground-glass" opacities on CXR
62

63
CD4+ cell count < 100/ mm3
64 Aspergillus fumigatus Hemoptysis, pleuritic pain Ca\ itation or infiltrates on chest imaging
65
Candida albicans Esophagitis White plaques on endoscopr; yeast and
66
pseudo h) phae on biopsy
67
68
CMV Retinitis, esophagitis, colitis, pneumonitis, Linear ulcers on endoscopy, cotton-wool spots
69
encephalitis on fundoscopy
70
Biopsy re,·eals cells with intranuclear {owl eye)
• 71 inclusion bodies
• 72 Cryptococcus Meningitis Encapsulated yeast on India ink stain or
. 73 neoformans capsular antigen Etl
• 74
Cryptosporidium spp. Chronic, ,,·atery diarrhea Acid-fast oocysts in stool
. 75
• 76
EBV B-celllymphoma {eg, non-Hodgkin lymphoma, C S lymphoma- ring enhancing, may be
• 77
C lS lymphoma) solitary (vs Toxoplasma)
• 78 Mycobacterium onspecifi e systemic symptoms (fever, night
• 79 avium- intracellulare, sweats, weight loss) or foca l lymphaden itis
. 80 Mycobacterium avium
. 81 complex
• 82
Toxoplasma gondii Brain abscesses Multiple ring-enhancing lesions on :\IRI
• 83
• 84

.
8
L.odt
s
Su~pl'nd
~
End Block
Item: 71 of - ,• Mark --<) [::> ""'I ~· 1!';:'1
100 ~ Prev1o u s Next Labf a lu es Notes Calculator


59 A 60-year-old woman with chronic rena l failu re is undergoing peri toneal dialysis. After 2-3 days of her hospi tal stay, she begins to r~AJ
60 complain of chills and severe abdominal pain . Her tempera tu re is 38.4°C (101.1 °F), heart rate is 86/min, blood pressure is 112/78 mm Hg,
61
and respiratory rate is 26/min. Physical examinat ion is notable for ascites, diffuse tenderness in t he abdomen, and erythema surrounding
the catheter site. Microscopic analysis of her effluent perit oneal fluid revea ls a cloudy fluid with neut rophilic leukocytosis.
62
63
A culture of the fluid obtained by paracentesis would most likely reveal which of the following organisms?
64
:
65
A. Gram-negative, lactose-fermenting, urease-positive bacilli
66
67
B. Gram-negative, non-lactose fermenting, urease-positive bacilli
68 C. Gram-negative, obligate anaerobic bacilli
69
D. Gram-positive, coagulase- negative cocci
70
• 71
E. Gram-positive, spore-forming, obligate anaerobic bacilli
• 72
. 73
• 74
. 75
• 76
• 77
• 78
• 79
. 80
. 81
• 82
• 83
• 84

.
8
L.odt
s
Su~pl'nd
~
End Block
Item: 71 of ~ ,• Mark <::J [:::> ""I ~· ~'j
100 J.. Previous Next LAbfaiUI~S Notes Calculator


59
60 The correct a nswer is 0. 4 2°/o chose t his.
61 The classic vignett e of fever in t he sett ing of peritonea l dialysis wit h erythema around the catheter sit e suggests an infection likely involv ing
bacteri a normally present on the skin. Staphylococcus epidermidis is part of t he normal hu man skin flora and can cause syst em ic infection
62
t hrough ent ry sit es into the body. These sites include t hose fo r Foley urine cat het ers, int ravenous lines, prost het ic devices, and peritonea l
63 dialysis catheters. S. epidermidis can m igrate along the tubing from the skin to the inside of t he body wit h t he help of biofilm creation . Fevers,
64 ch ills, diff use abdomina l t enderness, and rebound tenderness all suggest a diagnosis of peritonit is, wh ich can be caused by S. epidermidis. S.
epidermidis is also a common cause of endoca rdit is with prost hetic valves.
65
Biofilm Skin flora Peritonitis Staphylococcus epidermidis Endocarditis Erythema Peritoneal dialysis Catheter Intravenous therapy Staphylococcus Urine Bacteria Dialysis Infection Fever Systemic disease
66
Peritoneum Artificial heart valve
67
68
A is not correct . 2 4 % chos e this .
Klebsiella pneumoniae is implicated in aspiration pneumonia, wh ich manifests with cough product ive of t hick red sput um in a patient at high
69
risk for aspirat ion, as wit h alcoholism and altered menta l status.
70 Klebsiella pneumoniae Aspiration pneumonia Sputum Pneumonia Klebsiella Alcoholism Cough Altered level of consciousness

71
B is not correct. 16 % chose this .
· 72 This describes Proteus mirabilis, which is important t o consider in t he set t ing of uri nary t ract infect ions. It is also a potent ial cause of
• 73 spontaneous bact eri al perit onit is in patients wit h liver cirrhosis-associated ascit es. However, it is not a cause of cathet er-associat ed peritonit is,
• 74 as it is not a part of normal skin flora .
Peritonitis Ascites Skin flora Spontaneous bacterial peritonitis Urinary tract infection Liver Urinary system
. 75
• 76 C is not co rrect. 14% chos e this .
· 77 This describes Bacteroides fragilis, which is a normal member of t he gastroint estinal flora. It can cause disease when t rauma or surgery leads
t o perforation of t he bowel wall. Alt hough it is a common cause of perit oneal infections, it is not associated wit h cathet er-associat ed
• 78
perit onit is .
• 79 Bacteroides fragilis Peritonitis Gut flora Bacteroides Peritoneum Gastrointestinal perforation Human gastrointestinal tract Gastrointestinal tract Perforation

. 80
E is not correct. 4 °/o chos e this .
. 81 This describes Clostridium difficile, which can cause gast roent eri t is of va rying severity from mild wat ery diarrh ea to fu lminant
• 82 pseudomembranous colit is wit h bloody diarrhea. Alt hough C. difficile ent eri t is is associat ed with previous ant ibiot ic t reatment, it is not
• 83 associat ed with t he cont inual presence of an indwelling cat heter.
Clostridium difficile colitis Gastroenteritis Antibiotics Diarrhea Clostridium difficile (bacteria) Enteritis Colitis Clostridium Catheter
• 84
. •
8
Lock
s
Suspend
0
End Block
Item: 71 of - ,• Mark -<J
P~v1ous
[:::> "'I ~ ·· ~
100 ~ N @xt Labl lues N o tes Calcula to r
A A
59 Bottom Line:
60
Staphylococcus epidermidis is a part of normal skin f lora and can cause infections, including peritonit is, in a setting of prosthetic devices or
61 catheters. It is the most common cause of dialysis-associated peritonitis, with Staphylococcus aureus being t he second most commonly
62 implicated pathogen.
Shphylococcus ep1derrn1d1• Skm flora Pentomtis Staphylococcus aureus Staphylococcus Pathogen Cllthet~ Prosthesn~
63
64
65
l@l ;fii·11•J for year: 2017 •
66 Fl ltST AIO FACTS

67
68 FA17 p 130.1
69
Gram-positive lab algorithm
70
71 Gram<±> (purple/blue)
' 72
• 73
• 74 Branching
Bacilli Cocci
. 75 filaments
• 76
· 77
I
Aerobic Anaerobic Anaerobic/facultative Aerobic Anaerobic
' 78
' 79 Usteria
Nocardia Actinomyces
. 80 Bacillus Clostridium
Propionibacterium (weakly aod fast) (not acid fast)
Corynebacterium
. 81
Catalase
' 82 1
-~er---
' 83
' 84
Streptococcus Staphylococcus
. T T
8
Lode.
s
Suspe-nd
8
End Bloc:k
Item: 71 of - ,• Mark -<J
P~v1ous
[:::> "'I ~ ·· ~
100 ~ N @xt Labl lues N o tes Calcula to r

----~--------~~
A A
59
60
1
Streptococcus Staphylococcus
61
62 Hemolysis Coagulase
63
64 (Partial (Complete
65 a hemolysis,
green)
hemolysis,
clear)
'Y (No hemolygs.
grows 1n bile)
66
Saureus
Novobiocin
67 sensitivity
Optochin sensitivity Bacitracin sensitivity Growth '" 65% NaCI
68 and bile solubility
69
70
71 Group B Group A Ssaprophytkus Sep;Mrmidis
Sagal«tiae Spyogenes
' 72
• 73
B- '+
• 74 I
. 75 Vindans streptococci Group 0
(no capsule) Spneumoniae Nonenterococcus (enterococcus)
• 76 Smutans (encapsulated) S bovts Efaecium
· 77 Smitis Efaecalis
' 78 Important tests are 10 bold. Important pathogens are in bold italics.
' 79 Note: Enterococcus IS either a- or y-hemolytic.
. 80
. 81
FA17 p 133.3
' 82
' 83 Enterococci Cram <±> cocci. Enterococci (E faecalis and Enterococci, hardier than nonenterococcal
' 84 E {aecium) are normal colonic flora that arc group D, can grow in 6.5% 1 aCI and bile (lab
.
8
Lode.
s
Suspe-nd
8
End Bloc:k
Item: 71 of - ,• Mark -<J
P~v1ous
[:::> "'I ~ ·· ~
100 ~ N @xt Labl lues N o tes Calcula to r

59
Vindans streptococci Group D
60 (no capsule} Spneumoniae Nonenterococcus (enterococcus)
61 Smutans (encapsulated} Sbows Efaecium
Smitis Efaecalis
62
63 Important tests are tn bold. Important pathogens are in bold italics.
Note: Enterococcus ts either a· or y-hemolytic.
64
65
66 FA17 p 133.3
67
68
Enterococci Gram EEl cocci. Enterococci {E {aecalis and Enterococci, hardier than nonenterococcal
69 E {aecium) are normal colonic fl ora I hat arc group D, can grow in 6.5% l':aCI and bile {lab
70 penicillin G resistant and cause UTI, biliary test).
71 tract infections, and subacute endocarditis E11tero = intestine, faecalis = feces, slrepto =
' 72 (following Gl/GU procedures). Catalase 8, twisted (chains), coccus= berry.
• 73 PYR EEl, ,·ariable hemolysis.
• 74 VRE (vancomycin-resistant enterococci) are an
. 75 important cause of nosocomial infection .
• 76
· 77
FA17 p 131 .4
' 78
' 79 Staphylococcus Gram®, catalase EEl, coagu lase 8 , urease EEl cocci in clusters. Novobiocin sensitive. Does not
. 80 epidermidis ferment mannitol (vs S aureus).
. 81 onnal flora of skin; contaminates blood cultures.
' 82 Infects prosthetic devices (eg, hip implant, heart \'alve) and lV catheters by producing adherent
' 83 biofilms.
' 84
.
8
Lode.
s
Suspe-nd
8
End Bloc:k
Item: 72 of - ,• Mark --<) [::> ""'I ~· 1!';:'1
100 ~ Prev1o u s Next Labf a lu es Notes Calculator


59 A 12-year-old boy with no past medical history is brought to the emergency department by his mother after 3 days of fever, delirium, r~AJ
60 hydrophobia, and hypersalivation. About 1 month earlier his fami ly had been camping, and the mother remembers the boy encountering a
61 "sick" raccoon. He did not seek medical attention at that time.
62
The pathogen is most likely which of the following?
63
64 :
A. Encapsulated bacteria
65
66 B. Enveloped double-stranded DNA virus
67
C. Enveloped single-stranded RNA virus
68
0 . Naked single-stranded DNA virus
69
70 E. Naked single-stranded RNA virus
71
• 72
. 73
• 74
. 75
• 76
• 77
• 78
• 79
. 80
. 81
• 82
• 83
• 84

.
8
L.odt
s
Su~pl'nd
~
End Block
Item: 72 of ~ ,• Mark <:::1 t::> ""I ~· ~'j
100 J.. Previous Next LAbfaiUI~S Notes Calculator


59
60
61 The correct answer is C. 59°/o chos e this.
62 This boy is exhibiting symptoms of advanced ra bies infection . Hydrophobia is a very unique and specific symptom of ra bies. The ra bies v irus is
one of the rhabdov iruses, which are single-st randed, enveloped, and helical.
63
Rabies Rabies virus Virus Helix symptom Infection
64
65 A is not correct. 3°/o chos e this.
Encapsulated bacteria, such as Streptococcus pneumoniae, Haemophi/us influenzae, and Bacillus anthracis, would not produce t he above
66
symptomatology of hydrophobia and hypersalivat ion.
67 Bacillus anthracis Streptococcus pneumoniae Haemophilus influenzae Streptococcus Rabies Bacteria Bacillus Haemophilus

68
B is not correct. 11 % chose this .
69
Enveloped double-st randed DNA viruses like herpesviruses and poxviruses would not cause rabies-like symptoms. These viruses produce
70 vesicular and wart-l ike skin lesions.
71 DNA Virus Herpesviridae Poxviridae DNA virus Vesicle (biology and chemistry)

72 D is not correct. 8°/o chose this .


• 73 Non-enveloped single-st randed DNA viruses like parvovirus B19 cause aplast ic crises in sickle-cell pat ients, and a "slapped-cheeks" rash in
• 74 children .
Parvovirus 619 Parvovirus Virus DNA Sickle-cell disease Rash DNA virus
. 75
• 76 E is not correct. 19°/o chos e this .
· 77 Non-enveloped single-st randed RNA viruses like t he calciviruses cause diarrheal diseases.
Virus Diarrhea RNA RNA virus
• 78
• 79
. 80 Bottom Line:
. 81 Hydrophobia is a very unique and specif ic sympt om of rabies. The rabies virus is one of the rhabdoviruses, which are sing le-stranded,
• 82 enveloped, and helical.
Rabies Rabies virus symptom Virus Helix
• 83
• 84
. •
8
Lock
s
Suspend
0
End Block
Item: 72 of - ,• Mark --<) [::> ""'I ~· 1!';:'1
100 ~ Prev1o u s Next Labf a lu es Notes Calculator

59 Bottom Line:
60 Hydrophobia is a very unique and speci fic symptom of rabies. The rabies virus is one of the rhabdoviruses, which are sing le-stranded,
61
enveloped, and helical.
Rab1es Rab1es v1ru! 9.-mptom Virus Helix
62

63
64
141:l'i1!1i•l
ft-.ST AID fACTS
for y ear: 2017 •
65
66
67 FA17 p 167.1
68 Rabies virus Bullet-shaped vims B. i\egri bodies Infection more commonly from bat, raccoon, and
69 (cytoplasmic inclusions ) commonly skunk bites than from dog bites in the United
70 States; aerosol transmission (eg, bat caves) also
found in Purkinje cells of cerebellum and
71
in hippocampal neurons. Rabies has long possible.
72
incubation period (weeks lo months) before
. 73
symptom onset. Poslexposure prophylaxis
• 74
is wound cleaning plus immunization with
. 75
• 76
killed vaccine and rabies immunoglobulin.
• 77
Example of passive-active immunity.
• 78
Tra,·els to the C S by migrating in a rcl·rogradc
• 79 fashion (via dynein motors) up nerve axons
. 80 after binding to ACh receptors .
. 81 Progression of disease: fever, malaise
• 82 - agitation, photophobia, hydrophobia,
• 83 hypersa)i,·ation - paralysis, coma - death .
• 84
. •
8
L.odt
s
Su~pl'nd
~
End Block
Item: 73 of - ,• Mark -<J [:::> "'I ~ · ~
100 ~ P~v1ous N @xt Labl lues N o tes Calcula to r

59 A A 56-year-old woman who is undergoing chemot herapy for leukemia is admitted to t he hospit al for severe neut ropenia discovered on [.AA] A

60
routine laboratory tests. She is immediately started on intravenous antibiotics, but on the third day of hospitalization she becomes febrile,
develops a persistent cough, and has an episode of hemoptysis. CT of the chest, shown in image, reveals an apical radiopaque mass
61 surrounded by a cresentic-shaped layer of air
62
63
64
65
66
67
68
69
70
71
72
• 73
• 74
. 75
• 76
· 77
• 78
• 79 Image cowtesy of Radiopaedia/Dr. Frank Gaillard
. 80
. 81 Which of the fol lowing features accurately describes t he most likely infectious agent that is responsib le for her sympt oms?
• 82 :
• 83 A. 45-degree-angle branching septate hyphae with rare fruiting bodies
• 84 B. Irregular, broad nonseptate hyphae with wide-angle branching
.
8
Lode.
s
S uspe-nd
8
End Bloc:k
59
60
61
62

63
64
65
66
67
68
69
70
71
72
' 73 Image coUitesy of Radiopaedia/Dr. Frank Gaillard

' 74
. 75 Which of the fol lowing features accurately describes t he most likely infectious agent that is responsible for her sympt oms?
' 76 :
' 77
A. 45-degree-angle branching septat e hyphae with ra re fruit ing bodies
' 78 B. I rreg ular, broa d nonseptat e hyphae wit h wide-angle branching
' 79
C. Na rrow- based, unequal budding yeast wit h a wide capsular halo
. 80
. 81 D. Pseudohyphae and budding yeasts
' 82 E. Spherules with endospores
' 83
F. Tiny yeasts inside macrophages
' 84
. •
8
L.odt
s
Su~pl'nd
~
End Block
Item: 73 of - ,• Mark -<J [:::> "'I ~ · ~
100 ~ P~v1ous N @xt Labl lues N o tes Calcula to r
A A
59
60
The correct answer is A. 49°/o chose this.
61
The CT shows an "air-crescent sign," indicated by the red circle. This sign is caused by reabsorption of
62
necrotic t issue following an invasive fungal infection and is usually a sign of resolving neutropenia (a good
63 prognosis for the patient). It is commonly associated with resolving pulmonary aspergillosis, a form of
64 invasive aspergillosis commonly seen in neutropenic patients. The most common causative species by far is
Aspergillus fumigatus. This organism is described as having 45-degree-angle branching septate hyphae.
65
Defense against initial Aspergillus inoculation involves alveolar macrophages. Later layers of defense include
66 complement activation, neutrophil chemotactic factors, and neutrophil recruitment. Thus neutropenia
67 represents a significant risk for invasive aspergillosis, as are glucocorticoid therapy, bone marrow
transplant, solid organ transplant, AIDS, and chronic granulomatous disease. Symptoms of pulmonary
68
aspergillosis are fevers unresponsive to broad-spectrum antibiotics, pleuritic chest pain, cough, dyspnea,
69 and hemoptysis. CT findings of the chest may range from normal nodular lesions and patchy infiltrates, to
70 cavitary lesions. One such radiographic finding is the air-crescent sign, which represents the presence of air
between Aspergillus-infected/infarcted tissues and normal pulmonary parenchyma.
71
Aspergtllu fum•oatus Her '\Optysts Neutrophil Glucocorticoid Dyspnea Neutropenia Chronic granulomatous dtse .se Organ transplantation Aspergillosis
72
Pleunsy Parenchyma Aspergtllus Mycosts Macrophage Necrosis Hematopoietic stem cell transplantat:ton HIV/AIOS Granuloma Pulmonary alveolus
Image courtesy of
73 Radiopaedia/Dr. Frank Gaillard
Bone marrow Cough Chemotaxts Anbbtobcs Hypha Fungus Chest pain Septum Infectton Organtsm CT scan Inoculatton Prognosts Bone
• 74
. 75
B is not correct. 12°/o chose this .
I rreg ular, broad nonseptate hyphae wit h wide-angle branching describes Zygomycetes, a class of fung i t hat can cause a number of infections
• 76
in patients who are immunosuppressed or who have diabetes. Zygomycosis or mucormycosis causes a severe t ype of rhino-orbita l-cerebral
· 77 infection with symptoms includ ing facial pain, headache, and cranial nerve involvement. Less commonly it may also involve the lungs. Risks
• 78 for mucormycosis are sim ilar to t hose fo r invasive aspergillosis .
Zygom;costs Aspergtllosis Mucormycosts Fungus Immunosuppression Diabetes mellitus Hypha Headache Zygomycota Cramal nerves Lung
• 79
. 80 Cis not correct. 12% chose thi s .
. 81 Narrow-based, unequal budding yeasts with wide capsular halos describes C1yptococcus neoformans. This fungal agent is the most common
cause of meningitis in AIDS patients. All crypt ococcal infection begins in the lungs. Findings on X-ray of the chest include nodular lesions or
• 82
lobar infiltrates .
• 83 Cryptococcue: neofo1 nc.. ~ Memng1b., Chest radiograph HJV/AIDS Fungus Cryptococcus Nodule (med1c1ne) Yewt Lung X·ray

• 84
D is not correct. 10°/o chose this .
.
8
Lode.
s
S uspe-nd
8
End Bloc:k
Item: 73 of ~ ,• Mark <::J [:::> ""I ~· ~'j
100 J.. Previous Next LAbfaiUI~S Notes Calculator

59 D is not correct. 10% chose this .


60 Pseudohyphae and budding yeasts describes Candida species. Risk factors for invasive candidiasis are similar t o t hose fo r invasive
aspergillosis. Clinically, some signs and symptoms of invasive candidiasis include feve rs refract ory to broad-spect rum ant ibiot ics, characteristic
61
eye and skin lesions, and mult iorgan dysfunction. Pulmonary involvement in t he form of m icroabscesses may be seen at autopsy; however,
62 pneumonia is exceedingly ra re.
63 Aspergillosis Candidiasis Pneumonia Candida (fungus) Hypha Antibiotics Fungemia Yeast Autopsy Multiple organ dysfunction syndrome Species

64 E is not correct. 9°/o chose this.


65 Spherules wit h endospores describes Coccidioides immitis, a dimorphic fungus endemic t o t he southwestern part of the Unit ed Stat es. Most C.
66 immitis infection is subclinical. Some cases may present as community-acquired pneumonia wit h sympt oms like chest pain and cough. X-ray
of t he chest may show cavit at ion if there is hemoptysis.
67
Hemoptysis Coccidioides immitis Chest radiograph Pneumonia Endemism Community-acquired pneumonia Cavitation Coccidioides Endospore Chest pain Cough Fungus
68
69
F is not correct. 8°/o chose this.
Tiny yeasts inside macrophages describes Histoplasma capsulatum . Prima ry infect ion by t his agent is usually asym ptomatic, or m ild wit h flu-
70
like symptoms. In immunosuppressed ind ividuals such as AIDS patients, t his fu ngus can cause severe sympt oms known as hist oplasmosis.
71 Findings on x-ray of t he chest may be complet ely normal, or show f ibrosing mediast initis. Chronic histoplasmosis may mimic t uberculosis.
72 Histoplasma Histoplasmosis Mediastinitis Tuberculosis HIV/AIDS Macrophage Immunosuppression Fungus Influenza-like illness Infection X-ray Yeast Fibrosis

73
• 74 Bottom Line:
. 75
I nvasive aspergillosis occurs in immunosuppressed individuals such as those with AIDS, t ransplant patient s, chronic glucocort icoid users, and
• 76 neut ropen ic patient s. A specific form of invasive aspergillosis, pulmonary aspergillosis, is characterized by a unique radiographic find ing
· 77 called the air-crescent sign.
Glucocorticoid Aspergillosis Immunosuppression Neutropenia HIV/AIDS
• 78

• 79
• 80
l@);fil ~1hl for year:l 2o17 y
. 81 FIRST AID FACTS

• 82
• 83 FA17 p 149.1
• 84 Opportunistic fungal infections
.
8
Lock
s
Suspend
0
End Block
Item: 73 of - ,• Mark --<) [::> ""'I ~· 1!';:'1
100 ~ Prev1o u s Next Labf a lu es Notes Calculator

59
• FA17 p 149.1
60 Opportunistic fungal infections
61
Candida albicans alba = white. Dimorphic; forms pseudohyphac <md budding yeasts at 20°C · , germ tubes at
37°c m .
62
63
Systemic or superficial fun gal infection. Causes oral and esophageal thrush in
64
immunocompromised (neonates, steroids, diabetes, AI OS), vulvovaginitis (diabetes, use of
65
66
antibiotics), diaper rash, endocarditis (I drug users), disseminated candidiasis (especiallr in
67
neutropenic patients), chronic mucocutaneous candidiasis.
68
Treatment: oral fluconazole/topical a1olc for ''aginal ; nystatin, fluconazole, or caspofungin for oral/
69 esophageal; fluconazole, caspofungin, or amphotericin B for systemic.
70 Aspergillus Septate hyphae that branch at 45° \ cute \ ngle . Produces conidia in radiating chains at end of
71 fumigatus conidiophore 1).
72 Causes invasive aspergillosis in immunocompromised, patients" ith chronic granulomatous disease.
73 Can cause aspergillomas in pre-existing lung cavities, especially after TB infection .
• 74
Some species of Aspergillus produce \ natoxins (associated with hepatocellular carci noma).
. 75
• 76 Allergic bronchopulmonary aspergillosis (ABPA): hypersensitivity response associated with
• 77 asthma and cystic fibrosis; may cause bronchiectasis and eosinophilia .
• 78
Cryptococcus 5-10 j.lm with narrow buclcling. lleavily encapsulated yeast. lot dimorphic.
• 79
neoformans Found in soil, pigeon droppings. Acquired through inhalation wilh hematogenous dissem ination
. 80
to meninges. Culture on Sabouraud agar. llighlighted with India ink (clear halo 1]1) and
. 81
mucicarmine (red inner capsule ). Latex e~gglutination test detects polysaccharide capsular
• 82
antigen and is more specific.
• 83
• 84
Causes cryptococcosis, cryptococcal meningitis, cryptococcal encephalitis ("soap bubble" lesions
. in hraint nrima ril v in immunocomnromi~ecl. •
8
L.odt
s
Su~pl'nd
~
End Block
Item: 73 of - ,• Mark --<) [::> ""'I ~· 1!';:'1
100 ~ Prev1o u s Next Labf a lu es Notes Calculator
.
59
Mucor and Rh izopus Irregular, broad, nonseplale hyphae branching al "ide angles
60
spp. Mucormycosis. Causes disease mostly in keloaeidolic diabetic and/or neutropenic patients (cg,
61
62
leukemia). Fungi proliferate in blood vessel walls, penetrate cribriform plate, and enter brain.
63
Rhinocerebral, frontal lobe abscess; cavernous sinus thrombosis. Headache, facial pain, black
64
necrotic eschar on face; ma\' he~ve cmnial nerve involvement.
'
65 Treatment: surgical debridement, amphotericin B.
66
67
68
69
70
71
72
73
• 74
. 75 D
• 76
• 77
• 78

• 79
. 80
. 81
• 82
• 83 a
• 84

.
8
L.odt
s
Su~pl'nd
~
End Block
Item: 74 of - ,• Mark -<J [:::> "'I ~ · ~
100 ~ P~v1ous N @xt Labl lues N o tes Calcula to r
6
55
A 21-year-old U.S. student visit ing in Peru comes to a clinic wit h complaints of abdominal pain. She says she has been careful to drink only ~~AI
56 bottled water, but a few days earlier she decided to taste some local food being sold by a st reet vendor. Since t hen, she has been
57 experiencing fever and some bloody diarrhea. A stool sample is positive for a nonmotile gram-negative bacterium.
58
59 Which of the following is the most likely causative organism?
60 :
61 A. Clostridium difficile
62 B. Entamoeba histolytica
63
C. Giardia Iamblia
64
65
0 . Salmonella enteritidis
66 E. Shigella flexneri
67
68
69
70
71
72
73
• 74
. 75
• 76
. 77
• 78
• 79
. 80
.
8
Lode.
s
Suspe-nd
8
End Bloc:k
Item: 74 of ~ ,• Mark <::J [:::> ""I ~· ~'j
100 J.. Previous Next LAbfaiUI~S Notes Calculator

55 • The correct answer is E. 68°/o chose this .


56 This patient is most likely infected with Shigella flexneri, a nonmotile gram -negative facultat ive anaerobe t hat causes abdom inal pain, high
57
fever, and bloody diarrhea . Shigellosis is a common cause of bloody diarrhea in the developing worl d. The motilit y test is often used t o
different iat e bet ween Shigella and Salmonella; t he former is nonmot ile. Another differentiat ing fact or between Shigella and Salmonella is that
58 t he inoculum required for infection is very small fo r Shigella, but m uch larger fo r Salmonella.
59 Shigella flexneri Facultative anaerobic organism Shigellosis Gram-negative bacteria Shigella Diarrhea Anaerobic organism Salmonella Motility Abdominal pain Fever Flagellum Developing country Infection

60 A is not correct . 3°/o chos e this.


61 Clostridium difficile is a gram-posit ive rod, which may cause pseudomembranous colitis and/or diarrhea . C. difficile infect ions are usually
62 nosocomial and are caused by ant ibiot ic t reatments t hat red uce norma l gast roint estinal f lora and allow C. difficile t o f lourish.
Clostridium difficile colitis Gut flora Clostridium difficile (bacteria) Gram-positive bacteria Antibiotics Diarrhea Colitis Clostridium Hospital-acquired infection Human gastrointestinal tract Flora
63
Gastrointestinal tract
64
65 B is not correct. 6 % chose this.
66 Entamoeba histolytica is a prot ozoan pat hogen whose ova and parasitic fo rms can be found in stool cult ure . It is t ra nsmit ted t hrough the
fecal-oral route. Howeve r~ its present at ion is different f rom t he one described in t he vignet te. It common ly causes amebiasis, which is
67
charact erized by bloody diarrhea (dysent ery), liver abscesses, and right upper quadrant pain .
68 Entamoeba histolytica Fecal-oral route Amoebiasis Dysentery Diarrhea Microbiological culture Stool test Pathogen Protozoa Parasitism Liver Entamoeba Abscess Quadrant (abdomen) Egg cell

69
C is not correct. 3 °/o chos e this .
70
Giardia Iamblia is a protozoan pathogen that classically causes chronic nonbloody, fou l-smelling diarrhea.
71 Giardia Iambiia Giardia Diarrhea Pathogen Protozoa

72
D is not correct. 20% chose this .
73 Salmonella enteritidis is a motile gram-negat ive organism. This organism causes bloody diarrhea, usually after ingestion of cont aminat ed
74 poult ry or eggs. The motility test is often used to differentiat e between Shigella and Salmonella; the latt er is motile. For the purposes of t he
. 75 USMLE (and t riv ia to impress your classmat es), turt les are also a vector fo r S. enteritidis .
Salmonella enterica subsp. enterica Gram-negative bacteria Shigella Motility Diarrhea Salmonella Poultry Organism Egg (food) Egg Dysentery Vector (epidemiology)
• 76
· 77
• 78 Bottom Line:
• 79 Shigella flexneri is a nonmot ile, gram- negative bacterium that causes bloody diarrhea, feve r, and abdominal pain . Know all the organisms
. 80 t hat cause bloody diarrhea : Campylobacter, Salmonella, Shigella, enterohemorrhagic and ent eroinvasive Escherichia coli (EHEC and EI EC),
. Yersinia and Entamoeba .

8
Lock
s
Suspend
0
End Block
Item: 74 of - ,• Mark -<J [:::> .il ~ · ~
100 ~ P~v1ous N @xt LabValues N o tes Calcula to r
A A
55 FA17 p 141.1
56
Salmonella vs Shigella Both Salmonella and Shigella are gram 8 rods, non-lactose fermenters, oxidase 8, and can invade
57
58
the Gltract via ~ I cells of Pe)er patches.
59 Salmonella typhi Salmonella spp. Shigella
60 (exceptS tlf'hi)
61 RESERVOIRS Humans only Humans and animals Humans onh-,
62
SPREAD Can disseminate Can disseminate Cell to cell; no hematogenous spread
63
hematogenous!) hematogenous!)
64
65 H1SPRODUCTION Yes Yes ~0
66 flAGEllA Yes (salmon swim) Yes (\almon !I\\ im) No
67
VIRULENCE FACTORS Endotoxin; Vi capsule Endotoxin Endotoxin; Shiga toxin (enteroto\in)
68
69
INFECTIOUS DOSE UDso) High-large inoculum High Low-\·ery small inoculum required;
70 required because organism resistant to gastric acids
71 inactivated by gastric acids
72 EFFECT OF ANTIBIOTICS ONFECAL Prolongs duration Prolongs duration Shortens duration
73 EXCRETION
74 IMMUNE RESPONSE Primari ly monocytes p ,, s in disseminat·ed Primarilv PM I infiltration
. 75
disease
• 76
. 77
Gl MANIFESTATIONS Constipation, followed by Diarrhea (possibly bloody) Bloody diarrhea (bacillary dysentery)
• 78 diarrhea
• 79 VACCINE Oral \·accine contains li\·e o vaccme No vaccine
. 80 attenuated S typhi
.
8
Lode.
s
Suspe-nd
8
End Bloc:k
Item: 74 of ~ ,• Mark <::J [:::> ""I ~· ~'j
100 J.. Previous Next LAbfaiUI~S Notes Calculator

55 •
FA17 p 175.1
56
Bugs causing diarrhea
57
58 Bloody diarrhea
59 Campylobacter Comma- or S-shaped organisms; growth at 42°C
60
E histolytica Protozoan; amebic dysentery; liver abscess
61
62 Enterohemorrhagic 0157:1-17; can cause llUS; makes Shiga-like toxin
63 Ecoli
64 Enteroinvasive Ecoli Invades colonic mucosa
65
Salmonella (non- Lactose 8; Aagellar motility; has animal reservoir, especially poultry and eggs
66
typhoidal)
67
68 Shigella Lactose 8; very low 10;0; produces Shiga toxin (human reservoir only); bacillary dysentery
69 Yenterocolitica Day care outbreaks, pseudoappendicitis
70
71
Watery diarrhea
72 Cdifficile Pseudomembranous colitis; associated with antibiotics and PPis; occasionally bloody diarrhea
73 C perfringens Also causes gas gangrene
74
. 75
Enterotoxigenic E coli Tra,·elers' diarrhea; produces heat-labile (LT) and heat-stable (ST) toxins
• 76 Protozoa Giardia, Cryptosporidium
· 77
Vcholerae Comma-shaped organisms; rice-water diarrhea; often from infected seafood
• 78
• 79
Viruses Rota,·irus, norovirus, adenovirus
. 80
.
8
Lock
s
Suspend
0
End Block
Item: 74 of - ,• Mark --<) [::> ""'I ~· 1!';:'1
100 ~ Prev1o u s Next Labf a lu es Notes Calculator

55 • •
FA17 p 151 .1
56
Protozoa- gastrointestinal infections
57
58
ORGANISM DISEASE TRANSMISSION DIAGNOSIS TREATMENT
59 Giardia Iamblia Giardiasis-bloating, flatulence, Cysts in water ~ fultinucleated Metronidazole
60 foul-smelling, fatty diarrhea trophozoites or
61 (often seen in campers/hikers)- cysts : in stool,
62 think fat-rich Ghirardell i antigen detection
63 chocolates for fatty stools of
64 Giardia
65
Entamoeba Amebiasis- bloody diarrhea Cysts in water Serology and/or Metronidazole;
66
histolytica (dysentery), Ji,·er abscess trophozoites (with paromomycm or
67
("anchovy paste" exudate), engulfed RBCs (i iodoquinol for
68
69
RUQ pain; histology shows in the cytoplasm) asymptomatic cyst
70 flask-shaped ulcer or cysts with up to passers
71 4 nuclei in stool [!];
72 E ntamoeba Eats
73 Jt:ryt h rocytes,
74 antigen detection
. 75
• 76
Cryptosporidium Severe diarrhea in AIDS Oocrsts in water Oocysts on acid-fast Pre,·ention (by
. 77
lild disease (watery diarrhea) in sta in 11, antigen filtering city
• 78
immunocompetent hosts detection water supplies);
• 79
nitazoxanide in
. 80
. immunocompetent •
8
L.odt
s
Su~pl'nd
~
End Block
Item: 74 of - ,• Mark --<) [::> ""'I ~· 1!';:'1
100 ~ Prev1o u s Next Labf a lu es Notes Calculator

55 Entamoeba Cysts in water Serology and/or Metron idazole;


Amebiasis- bloody diarrhea
56
histolytica (dysentery), Ji,·er abscess trophozoites (with paromomycin or
57
("anchO\'}' paste" exudate), engulfed RBCs (!t iodoquinol for
58
RUQ pain; histology shows in the cytoplasm) asymptomatic C) st
59
Aask-shaped ulcer or cysts with up to passers
60
4 nuclei in stool • ;
61
Entamoeba Eats
62

63
l .r) I h rocytes,
64
antigen detection
65
66 Cryptosporidium Se,·ere diarrhea in AIDS Oocnts
, in water Oocrsts on acid-fast Pre,·ention (br
67 lvlild disease (watery diarrhea) in stain 0 , antigen filtering city
68 immunocompetent hosts detection water sup pi ies);
69 nitazoxanide in
70
immunocompetent
71
hosts
72
73
74
. 75
• 76
. 77
• 78

• 79
tl tl
. 80
. •
8
L.odt
s
Su~pl'nd
~
End Block
Item: 75 of - ,• Mark -<J [:::> "'I ~ · ~
100 ~ P~v1ous N @xt Labl lues N o tes Calcula to r
6
55
An 82-year-old woman with chronic obst ruct ive pulmonary disease is hospitalized for an acute exacerbat ion and is placed on ventilatory ~~AI
56 support. During the course of her hospitalizat ion, she becomes febrile with an associated elevation in her WBC count. X-ray of the chest
57 shows a new infiltrate in the right lower lobe. Sputum studies show that it is the gram-negative bacillus Acinetobacter baumannii. She is
58
treated with intravenous antibiotics. Several hours after starting intravenous antibiotics, t he patient has a seizu re that lasts 45 seconds. A
subsequent lumbar puncture is negative. The physician suspects a pharmacologic cause for th e seizure.
59
60
Administration of which antibiotic would most likely cause this effect?
61
:
62
A. Gentamicin
63
B. Imipenem
64
65 C. Levofloxacin
66 D . Linezolid
67
E. Rifampin
68
69
70
71
72
73
74
. 75
• 76
. 77
• 78
• 79
. 80
.
8
Lode.
s
Suspe-nd
8
End Bloc:k
Item: 7S of ~ ,• Mark <::J [:::> ""I ~· ~'j
100 J.. Previous Next LAb faiUI~S Notes Calculator

55 •
56 The correct a nswer is B. 48% chos e t his .
57 I mipenem is a broad -spectrum, 13- lactamase-resist ant antibiot ic of t he carbapenem class. I mipenem is st ruct urally sim ilar t o 13-lactam
ant ibiot ics but is 13- lactamase resistant and is administered with cilastatin t o decrease renal metabolism . Carbapenems can be used to t reat
58 bot h gram- posit ive and gram-negat ive infect ions, and it is first- line t herapy in the t reat ment of Acinetobacter and Enterobacter species
59 infections. Imipenem is not useful in t reating infect ions caused by met hicillin-resistant Staphylococcus aureus, Enterococcus faecium, or
60 Staphylococcus epidermidis. Adverse effects include gastroint estinal distress, t hrom bophlebit is, skin rashes, and seizu res.
Carbapenem Jmipenem Thrombophlebitis Enterococcus faecium Staphylococcus epidermidis Antibiotics Gram-negative bacteria Staphylococcus aureus Gram-positive bacteria Acinetobacter Enterobacter
61
Enterococcus Cilastatin Staphylococcus Rash Metabolism Broad-spectrum antibiotic Kidney Gastrointestinal tract
62
63 A is not correct . 15% chos e this.
64 Gentamicin is an aminog lycoside ant ibiotic used to t reat gram- negat ive bacilli that works by inhibit ing protein synt hesis. Adverse effects
include nephrot oxicity, otot oxicit y, and neuromuscular blockade. Gentamicin is effect ive in t reating Acinetobacter species infection, but it is
65 not as efficacious as im ipenem, wh ich is t he f irst- line choice. It is not associated wit h seizu res.
66 Aminoglycoside Jmipenem Gentamicin Gram-negative bacteria Antibiotics Ototoxicity Nephrotoxicity Acinetobacter Protein Bacillus (shape) Protein biosynthesis Epileptic seizure Protein synthesis inhibitor

67 Bacilli
68 C is not co rrect. 13% chos e this .
69 Levofloxacin is a fluo roquinolone ant ibiotic that acts by inhibiting DNA gyrase . I t is used t o t reat infections from gram- negative rods and some
70 gram-posit ive organ isms and t herefore is good for upper respiratory infections and urina ry t ract infections. Fluoroquinolones are generally well
71 t olerated but are associated wit h anaphylaxis, gastroint estinal discomfort, headache, and phot otoxicity. They are also associated wit h
damaged ca1t ilage in animal models and t hus are contraindicated in ch ildren. Other fluo roquinolones, such as ciprofloxacin, may lower the
72 seizure threshold in some pat ients, bu t are not typically used in t reating Acinetobacter infections.
73 Quinolone Levofloxacin Anaphylaxis DNA gyrase Gram-negative bacteria Ciprofloxacin Antibiotics Gram-positive bacteria Urinary tract infection Seizure threshold Acinetobacter Headache Urinary system

74 Phototoxicity DNA Contraindication Human gastrointestinal tract Gastrointestinal tract Upper respiratory tract infection Epileptic seizure

75 D is not correct. 16% c hose this .


• 76 Linezolid is used for community-acquired pneumon ias and acts by binding t o t he bact eri al 50S ri bosoma l RNA subunit and blocking prot ein
· 77 synt hesis. Adverse effects include gastroint estinal upset , headache, and myelosuppression. Alt hough there is evidence t hat linezolid can lower
• 78 t he seizure threshold, it is not used fo r managing gram-negat ive pat hogens and, consequent ly, is not the sing le best answer. Li nezolid is a
weak MAO! and should be used wit h caut ion in the sett ing of MAOis and SSRis .
• 79 Linezolid Bone marrow suppression Gram-negative bacteria Monoamine oxidase inhibitor 50S Ribosomal RNA Protein Protein biosynthesis Selective serotonin reuptake inhibitor Headache Seizure threshold
. 80 Protein synthesis Ribosome Pathogen Gastrointestinal tract RNA Epileptic seizure
.
8
Lock
s
Suspend
0
End Block
Item: 7S of ~ ,• Mark <::J [:::> ""I ~· ~'j
100 J.. Previous Next faiUI~S
LAb Notes Calculator

55 E is not correct. 8 °/o chose thi s.


56 Rifamp in is used t o t reat tuberculosis and acts by inhibi ting DNA-dependent RNA polymerase. I mportant adverse effects include a t ransient
57
rise in hepat ic aminot ransfe rase levels (which usually return t o normal without discontinuation of rifam pin) and hepat it is, which can occur
directly or due to rifam pin potentiat ing t he hepatic toxicity of ot her drugs. Rifampin also causes discoloration of body fluids. It does not cause
58 seizures.
59 Rifampicin Tuberculosis RNA polymerase Hepatitis Epileptic seizure RNA Liver Transaminase Toxicity

60
61
Bottom Line:
62
I mipenem, a broad-spect rum, 13-lactamase- resistant ant ibiot ic of the carbapenem class, may cause gastroint estinal distress,
63 t hrombophlebit is, skin rashes, and seizures.
64 Carbapenem Jmipenem Thrombophlebitis Antibiotics Rash Broad-spectrum antibiotic Gastrointestinal tract Epileptic seizure

65
66
67 l@ljl'il·1i•J for year:[ 2017 • J
FIRST AID FA CTS

68
69 FA17 p 186.1
70 Carbapenems lmipenem, meropenem, ertapenem, doripenem.
71
72
MECHANISM lmipenem is a broad-spectrum, ~- l actamase­ With imipenem, "the kill is lastin' with
73
resistant carbapenem. Always administered ci lastatin."
74
with cilastatin {inh ibitor of renal ewer carbapenems include ertapenem (limited
75 dehydropeptidase J) to l inactivation of drug Pseudom01ws coverage) and doripenem.
• 76 in renal tubules.
· 77 CLINI CAL USE Gram <±> cocci, gram 8 rods, and anaerobes.
• 78
Wide spectrum, but significant side effects
• 79
limit use to life-threatening infections or
• 80
. after other drugs have fa iled. Meropenem
8
Lock
s
Suspend
0
End Block
Item: 75 of - ,• Mark -<J [:::> "'I ~ . ~
100 ~ P~v1ous N@xt Labl lu es No tes Calcula t o r

55 •
FA17p183.1
56
57 Antimicrobial therapy
58
59 GYRASE
FOLIC ACID SYNTHESIS
60 AND REDUCTION
61 (DNA mtthy~tionl Metronidazole Auoroquinolones
Ciprofloxacin
62
Levofloxaon. etc
Sulfonamides
63 Ouinolone
Sulfamethoxazole BACTERIAl CEll
64 Sulfisoxazole Nalidixic acid
65
SulfadlclZJne

66
Tnmethopnm
67
68
69
PROTEIN SYNTHESIS
70
71 50S SUBUNIT
mRNA
72 Chloramphenicol
73 Cl1ndamye~n
CELL WALL SYNTHESIS
74 Linezolid
THF
75 r TIDOG AN S ~THESIS
Macrolides
Azithromycm
• 76 Celt Clarithromycin
Glycopeptides membrane
. 77 Vancomyc1n Erythromycin
• 78 Bacitrae~n Celt Wall Streptogramins
Ouinuprislln
• 79
DalfoprtSIIn
. 80
. r-::,_. .... . .... ... r ... 30SSUBUNIT

8
Lode.
s
S uspe-nd
8
End Bloc:k
Sulfonamides
Sulfamethoxazole BACTERIAL CELL Quinolone
56 Nalidixic acid
Sulfisoxazole PABA
57 Sulfadklzane
58
59 Tramethopr1m
60
61
62 PROTEIN SYNTHESIS
63
505 SUBUN""
64
Chloramphenicol
65
Cllndamycan
66 CULWALLSYNTHESIS
LJnezohd
67 Macrolides
r IDO" -.AN S "!THESIS
68 Az1thromyctn
Celt Clarithromycin
69 Glycopeptides membrane
Vancomycin Erythromycin
70 Bacltracan Celtwau Streptogramins
71 Ouinupristin
Dalfopnstin
72 r> TIDOG YCAN C OSS-LINKING
73 30SSUBUNIT
Penicillinase-sensitive penicillins Antipseudomonal Carbapenems
74 Penlcalhn G. V Ticarcillin lmipenem Aminoglycosides Glycyclines
Ampic1lhn Piperacillin Meropenem Gentamicin Tigecycline
75
Amoxicallln Cephalosporins (1-V) £rtapenem Neomycin
• 76 Tetracyclines
Penicillinase-resistant penicillins lst-Cefazolin. etc Doripenem Amikacin Tetracyclme
. 77 Oxac1lhn 2nd- Cefoxitin. etc Monobactams Tobramydn Doxycycline
• 78 Nafcllhn 3rd-Ceftriaxone. etc Aztreonam Streptomycin Minocycline
01cloxacall1n 4th-Cefepime
• 79 Sth- Ceftaroline J
. 80
.
8
Lode.
s
Suspe-nd
8
End Bloc:k
Item: 76 of - ,• Mark -<J [:::> "'I ~ · ~
100 ~ P~v1ous N @xt Labl lues N o tes Calcula to r
6
55
A 52-year-old man comes t o t he physician complai ning of a skin rash that Is extremely painfu l. A picture of t he rash is shown. A pathology ~~AI
56 specimen is obtained.
57
58
59
60
61
62
63
64
65
66
67
68
69
Which of the following results would most likely be seen on biopsy?
70
71 :
A . Auer bodies
72
73 B. Cabot ring bodies
74 C. Cali-Exner bodies
75
D. Cowdry A inclusion bodies
• 76
. 77 E. Mallory bodies
• 78
• 79
. 80
.
8
Lode.
s
Suspe-nd
8
End Bloc:k
Item: 76 of ~ ,• Mark <::J [:::> ""I ~· ~'j
100 J.. Previous Next LAb faiUI~S Notes Calculator

55
The correct a nswer is 0. 68°/o chose t his.
56 The image shows grouped vesicular lesions following t he distribution of a dermat ome unilaterally. This is t he classic appearance of herpes
57 zoster, caused by the varicella zoster virus (VZV). Cowdry A inclusion bodies are seen in pat hology preparat ions of herpes zoster skin rashes.
58
They are intranuclear eosinophilic inclusions surrounded by a clear ha lo.
varicella zoster virus Dermatome (anatomy) Shingles Somite Herpes simplex Chickenpox Virus Rash Vesicle (biology and chemistry) varicella vaccine Pathology
59
60 A is not correct . 7°/o chose this.
61 Auer bodies, or Auer rods, are rod-shaped bodies in myeloid cells. They appear pri marily in acut e promyelocytic leukemia (M3) and are made
of f used lysosomes. Care must be t aken in t reat ing t hese pat ients because release of Auer rods may lead t o dissem inat ed intravascular
62 coagulation.
63 Disseminated intravascular coagulation Acute promyelocytic leukemia Leukemia Auer rod Lysosome Myelocyte Coagulation Myeloid

64 B is not correct. 7% chose t his .


65 Cabot ri ng bodies are ring-shaped st ructures found in RBCs in severe cases of mega loblastic anemia.
66 Megaloblastic anemia Anemia

67 C is not correct. 11% chos e this .


68 Cali-Exner bodies are spaces bet ween granu losa cells in ovarian follicles and in granulosa cell t umors.
69 Caii-Exner bodies Granulosa cell OVarian follicle

70 E is not correct. 7°/o chos e this .


71 Mallory bodies, or alcoholic hyaline bodies, are accumulat ions of eosinophilic material in the cytoplasm of damaged hepatic cells. They are
72 common ly fou nd in hepatocytes of patients with alcoholic hepat it is whose livers would also show cirrhosis and fatty change.
Alcoholic hepatitis Mallory body Cytoplasm Cirrhosis Hepatocyte Steatosis Eosinophilic Hepatitis Liver Alcoholism
73
74
75 Bottom Line :
76 Grouped vesicles on an erythematous base in a dermat oma l distribut ion is a classic dermatologic descri ption of herpes zost er. Cells infected
· 77 with VZV, CMV, and HSV oft en show eosinophilic Cowdry t ype A inclusion bodies.
Shingles Herpes simplex virus Cytomegalovirus Herpes simplex Inclusion bodies Vesicle (biology and chemistry) Erythema Dermatology Varicella zoster virus
• 78

• 79
• 80
.
8
Lock
s
Suspend
0
End Block
Item: 76 of ~ ,• Mark <::J [:::> ""I ~· ~'j
100 J.. Previous Next LAbfaiUI~S Notes Calculator

55
FA17 p 162.1
56
57
HSV identification Viral culture for skin/genitalia.
58 CSF' PCR for herpes encephalitis.
59 Tzanck test-a smear of an opened skin vesicle to detect multinucleated giant cells commonlya
60 seen in HSV-1, IISV-2, and VZV infection. PCR of skin lesions is currently test of choice.
61 Intranuclear eosinophilic Cowdry A inclusions also seen with IISV-1, IISV-2, VZV.
62 Tzanck heavens I do not have herpes.
63
64
65
66
67 FA17 p 160.2
68
Herpesviruses Enveloped, OS, and linear viruses
69
70 VIRUS ROUTE OF TRANSMISSION CLI NICAL SIGNIFICANCE NOTES
71 Herpes Respiratory Gingivostomatitis, keratoconjunctivitis a, Most common cause of sporadic
72
simplex secretions, saliva herpes labial is 11], herpetic whitlow on finger, encephalitis, can present as altered
73
virus-1 temporal lobe encephalitis, esophagitis, mental status, seizures, and/or
74
erythema multiforme. aphasia.
75
76
Herpes Sexual contact, llerpes genital i s ~. neonatal herpes. Latent in sacral ganglia. ira!
· 77 simplex perinatal meningitis more common with
• 78 virus-2 HS -2 than with II SV-1.
• 79 Varicella- Respiratory Varicella-zoster (chickenpox m. shingles 1)}, Latent in dorsal root or trigeminal
• 80 Zoster virus secretions encephalitis, pneumonia. ganglia; C V1 branch
.
8
Lock
s
Suspend
0
End Block
Item: 76 of - ,• Mark -<J [:::> "'I ~ · ~
100 ~ P~v1ous N @xt Labl lues N o tes Calcula to r
A A
55
Epstein-Barr Respiratory Mononucleosis-fever, hepatosplenomegaly, Infects B cells through CD21.
56
virus (HHV-4) secretions, pharyngitis, and lymphadenopathy (especially Atypical lymphocytes on peripheral
57
sa li\·a· aka posterior cervical nodes ()). Avoid contact sports blood smear [!J-not infected B
58 '
"kissing disease," until resolution due to risk of splenic rupture. cells but reacth·e cytotoxic T cells.
59
(common in Associated with lymphomas (eg, endemic ~ fonospot test-heterophile
60
teens, young Burkitt l~mph oma}, nasopharyngeal antibodies detected by agglutination
61
adults) carcinoma (especially sian adults), of sheep or horse RBCs.
62
lymphoproliferati\'C disease in transplant Use of amoxicillin in mononucleosis
63
64
patients. can cause characteristic
65
maculopapular rash.
66 Cytomegalo- Congenital Mononucleosis (8 Monospot) in Infected cells have characteristic
67 virus (HHV-5) transfusion, immunocompetent patients; infection in "owl eye" inclusions Cl
68 sexual contact, immunocompromised, especially pneumonia Latent in mononuclear cells.
69 sali\·a, urine, in transplant patients; esophagitis; AIDS
70 transplant retinitis ("sightomega lovirus"): hemorrhage,
71 cotton-wool exudates, vision loss.
72
Congenital CM
73
Human Saliva Roseola infantum (exanthem subitum): high Roseola: fe\·er fi rst, Rosie (cheeks)
74
75
herpes- fevers for several days that can cause seizu res, later.
76
viruses 6 foll owed by diffuse macular rash D. HHV-7-less common cause of
. 77 and 7 roseola.
• 78 Human Sexual contact Kaposi sarcoma (neoplasm of endothelial cells). Can also affect C I tract and lungs.
• 79 herpesvirus Seen in HIV/AIDS and transplant patients.
. 80 8 Dark/violaceous plaques or nodules
.
8
Lode.
s
Suspe-nd
8
End Bloc:k
Item: 76 of ~ ,• Mark <::J [:::> ""I ~· ~'j
100 J.. Previous Next LAb faiUI~S Notes Calculator

55 •
56
57
58
59
60
61
62
63
64
65
66
67
68
69
70
71
72 FA17 p 160.1
73 DNA viruses
74 VIRAL FAMILY ENVELOPE ONA STRUCTURE MEDICAL IMPORTANCE
75 Herpesviruses Yes OS and linear See Herpesviruses entry
76
Poxvirus Yes OS and linear Smallpox eradicated world wide by use of the live-
· 77
• 78
(largest 0 1 A virus) attenuated vaccine
• 79
Cowpox ("mi lkmaid blisters")
. 80
Molluscum contagiosum -Aesh-colored papule with
. central umbilication
8
Lock
s
Suspend
0
End Block
Item: 76 of - ,• Mark --<) [::> ""'I ~· 1!';:'1
100 ~ Prev1o u s Next Labf a lu es Notes Calculator

55 • •
Hepadnavirus Yes Partially OS and circular li B
56
i\cule or chronic hepatitis
57
'ot a retrovirus but has reverse transcriptasc
58
59 Adenovirus 0 OS and linear Febrile pharyngitis r.J-sore throat
60 Acute hemorrhagic cystitis
61 Pneumonia
62 Conjunctivitis- .. pink eye"
63
64
65
66
67
68 OS and circular
Papillomavirus 0 II P - warts (serotypes 1, 2, 6, 11), Ch , cervical
69
cancer (most commonly 16, 18)
70
71
Polyomavirus 0 OS and circular JC virus-progressive multifocalleukocncephalopathy
72
(PM I.) in !lTV
73 BK virus-transplant patients, commonly targets kidney
74 JC: Junky Cerebrum; BK: Bad Kidney
75 Parvovirus 0 SS and linear B19 virus- aplastic crises in sickle cell disease,
76 (smallest D 1 virus) "slapped cheek" rash in children (erythema
. 77 infcctiosum, or fifth disease)
• 78
RBC destruction in fetus leads to hydrops fetal is
• 79
and death, in adults leads to pure RBC aplasia and
. 80
rheumatoid arthritis-like srmptoms •
.
8
L.odt
s
Su~pl'nd
~
End Block
Item: 77 of - ,• Mark -<J [:::> "'I ~· ~
100 ~ P~v1ous N @xt Labl lues N o tes Calcula to r
6
55
A 27-year-old man present s to the doctor's office with a history of chronic low-grade diarrhea of several years' durat ion. He reports that he ~~AI
56 sees no blood or pus in the diarrhea, but t hat recently he has developed copious gas and bloat ing when he ingests dairy products. Stool
57 analysis for ova and parasites revea ls the following image.
58
59
60
61
62
63
64
65
66
67
68
Image courtesy of CDC
69
70 What is the mechanism of pathogenesis of t he diarrhea?
71
:
72
A. Decreased absorptive capacity of t he intest ine due t o loss of brush border enzymes and blunt ing of v illi
73
B. Intracellular colonization by t he organism of int estinal epithelial cells
74
75 C. Invasion of the organism into the int est inal mucosa
76 D. Toxin production by t he organism resu lt ing in apoptosis of the intestinal mucosa
' 77
E. Toxin production by t he organism resulting in secret ion of ions into the intestinal lumen
' 78
' 79
. 80
.
8
Lode.
s
S uspe-nd
8
End Bloc:k
Item: 77 of ~ ,• Mark <:::1 t::> ""I ~· ~'j
100 J.. Previous Next LAbfaiUI~S Notes Calculator

55 •
56
The correct a nswer is A. 53°/o chose t h is.
57
The organism is Giardia Iamblia . Giardia can cause a long -term, smoldering infection of t he small intestine result ing in few or no symptoms.
58 The image shows the t rophozoite form of t he organism, wh ich proliferates in t he int estine. Sym ptoms occur due t o loss of brush border
59 enzymes and progressive blunting of villi. Lactase is one of t he brush border enzymes t hat is lost with Giardia infect ion and can cause
60 secondary lact ose intolerance.
Giardia Iambiia Lactose intolerance Lactase Giardia Small intestine Lactose Brush border Gastrointestinal tract Intestinal villus Organism Enzyme Infection Trophozoite
61
62 B is not correct. 15% chose this .
63 Whipple disease is a rare, systemic infectious disease caused by t he bacterium Tropheryma whipplei. Whipple disease is cha racterized by
fever, abdominal pain, diarrhea, weight loss, and m igratory polyarthralgia. The pri mary involvement is of t he small int estine and mesenteric
64 lymph nodes. Histologica lly t here is prom inent macrophage inf ilt ration and fat deposit ion. Charact eri st ic vacuoles within the macrophage that
65 stain with periodic acid-Schiff (PAS) stain are pat hognomonic of t he disease; t he int racellular and ext race llular PAS-posit ive material are
66 bacilli. Giardia is an extracellula r organism.
Macrophage Arthralgia Whipple's disease Pathognomonic Diarrhea Small intestine Infection Gastrointestinal tract Abdominal pain Bacteria Lymph node Weight loss Giardia Tropheryma whipplei
67
Periodic acid-Schiff Fever Histology Intracellular
68
69 C is not correct. 15% chos e this .
70 Several organisms have the ability t o invade t he lining of the int estine when t hey infect humans, such as Clyptosporidium and Entamoeba
histolytica . Giardia does not invade t he int est inal mucosa.
71 Entamoeba histolytica Cryptosporidium Giardia Gastrointestinal tract Entamoeba Mucous membrane
72
D is not correct. 7°/o chose t his.
73
Toxin production by Clostridium difficile results in apoptosis of t he intest inal mucosa . Giardia does not produce any toxins.
74 Apoptosis Clostridium difficile colitis Giardia Clostridium difficile (bacteria) Toxin Intestinal mucosa Mucous membrane Clostridium
75
E is not correct. 10°/o chose this.
76
This mechan ism is charact eristic of many organisms. Some strains of t he bacteria Escherichia coli possess heat-labile enterot oxin, which
77 increases cAMP by activating adenylate cyclase through t he guanosine t riphosphat e-dependent adenosine diphosphate ribosylation of specific
• 78 membrane prot eins. Heat - labile t oxin is cl osely relat ed in structure and function t o t he ent erotoxin expressed by Vibrio cholerae. Giardia,
• 79
however, does not produce any t oxins.
Heat-labile enterotoxin Vibrio cholerae Escherichia coli Adenosine diphosphate Adenylyl cyclase Cyclic adenosine monophosphate Bacteria Pyrophosphate Enterotoxin Toxin Guanosine Adenosine
. 80
.
8
Lock
s
Suspend
0
End Block
Item: 77 of ~ ,• Mark <:::1 t::> ""I ~· ~'j
100 J.. Previous Next LAbfaiUI~S Notes Calculator

55
56
Bottom Line:
57
Giardia Iamblia infect s t he small int estine, thereby causing lactose intolerance.
Giardia Iambiia Lactose intolerance Small intestine Lactose Giardia Gastrointestinal tract
58
59
60
141;fil·1i•J for year:[ 2017
FIRST AID FA C T S .
•j .
61
62
63 FA17 p 151 .1
64 Protozoa- gastrointestinal infections
65
ORGAN ISM DISEASE TRANSMISSION DIAGNOSIS TREATMENT
66
Giardia Iamblia Giardiasis-bloating, flatu lence, Cysts in water Multinucleated Metronidazole
67
foul-smelling, fatty diarrhea trophozoites fJ or
68
69
(often seen in campers/hikers)- rn
cysts in stoo1 J

70
think fat-rich Ghirardelli antigen detection
71 chocolates for fatty stools of
72 Giardia
73 Entamoeba Amebiasis-bloody diarrhea Cysts in water Serology and/or Metronidazole;
74 histolytica (dysentery), liver abscess trophozoites (with paromomycm or
75 ("anchovy paste" exudate), engulfed RBCs [i iodoguinol for
76
RUQ pain; histolog)' shows in 1·he cytoplasm) asymptomatic cyst
77
Aask-shaped ulcer or cysts with up to passers
• 78
4 nuclei in stool [!];
• 79
Entamoeba Eats
• 80
. Erythrocytes,
8
Lock
s
Suspend
0
End Block
Item: 77 of - ,• Mark -<J [:::> "'I ~ · ~
100 ~ P~v1ous N @xt Labl lues N o tes Calcula to r
A A
55
FA17p153.1
56
Protozoa- hematologic infections
57
ORGANISM DISEASE TRANSMISSION DIAGNOSIS TREATMENT
58
59
Plasmodium Malaria -fever, headache, anemia, Anopheles mosquito Blood smear: Chloroquine (for
60
P vivax/ovale splenomegaly trophozoite ring sensiti,·e species),
61 P falciparum J> vimx/omle-48-hr cycle (tertian; form'' ithin '' hich blocks
62 P malariae includes b·er on first day and RBC D. schizont Plasmodium heme
63 third day, thus fevers are actuall) containing polymerase; if
64 48 hr apart); dormant form mcrozoites; red resistant, usc
65 (hypnozoite) in liver granules (Schi.iffner mcAoquinc or
66 J> {alciparum-se\·ere; irregular stippling) rn atm·aquone/
67 fever patterns; pa rasitizcd RBCs throughout RBC proguanil
68 occlude capillaries in brain cytoplasm seen If life-threatening,
69 (cerebral ma laria), kidneys, lungs with P vivax/ovale use intravenous
70
J> malariae-72-hr cycle (quartan) quinidine or
71
artesunale (lest for
72
G6PD deficiency)
73
For P viwtxlovale,
74
add primaquine for
75
hypnozoitc (lest for
76
77
G6PD deficiency)
• 78 Babesia Babesiosis-fc,·er and hemolvl ic Ixodes I ick ( a me a) Blood smear: ring Ato\'aquone
'
• 79 anemia; predominantly in Borrelia burgdor{eri form , ".\ laltese + azithromycin
. 80 northeastern United States; of J., me disease; cross'' ; PCR
. r ' r r

8
Lode.
s
S uspe-nd
8
End Bloc:k
Item: 77 of - ,• Mark -<J [:::> "'I ~ · ~
100 ~ P~v1ous N @xt Labl lues N o tes Calcula to r

55
A
C6PD deficiency) A

56 F'or P vil'axlol'ale,
57 add primaquine for
58 hypnozoitc (test for
59 C6PD deficiency)
60
61
Babesia Babesiosis-fe,·er and hemolytic Ixodes I ick (same as Blood smear: ring Atovaquone
62
anemia; predominantly in Borrelia burgdorferi form , "~ lal tese + azithromycin
63 northeastern United States; of L) me disease; cross" ; PCR
64 asplenia t risk of se,ere disease ma) often coi nfect
65 humans)
66
67
68
69
FA17 p 124.2
70
71 Encapsulated bacteria Examples are Pseudomonas aeruginosa, Please Sl-li~ E mr SKiS.
72 Streptococcus pneumoniae D. llaemophilus Arc opsonized, and then cleared by spleen.
73 I Influen:we type B, Neisseria meningitic/is, Asplcnics have l opsonizing ability and thus
74 Escherichia coli, Salmonella, Klehsiella t risk for severe infections. GiveS pneumoniae,
75 ~ pnewnoniae, and group B Strep. Their H influen:we, meningitidis vaccines.
76
capsules serve as an antiphagocytic vi rulence
77
factor.
• 78
Capsular polysaccharide+ protein conjugate
• 79
sen·es as an antigen in \'accines.
. 80
.
8
Lode.
s
S uspe-nd
8
End Bloc:k
Item: 78 of - ,• Mark -<J [:::> "'I ~ · ~
100 ~ P~v1ous N @xt Labl lues N o tes Calcula to r

55
56
6

A 26-year-old female is brought to t he emergency department ( ED) by her husband for confu sion. He st at es she has been complaining of a ,..
headache and malaise fo r t he past 2 days. Physical examinat ion reveals a lethargic and confused fema le wit h a fever of 101.4° F. She A
AI
57 complains of pain when she f lexes her head forward to look at t he physician. She also complains of phot ophobia. A lumbar puncture is
58
performed in the ED. Cerebrospinal f luid (CSF) analysis reveals a polymorphonuclear neut rophil level of 400/ mm 3 , a protein level of 80 mg/dl,
and a glucose level of 25 mg/dl.
59
60
What is the mechanism of action fo r t he most appropriate t reatment for this patient?
61
:
62
A. Inhibit COX-1 and COX-2 to block prost aglandin synthesis
63
B. Inhibits bacterial cell wall synthesis
64
65 C. Inhibits bacterial DNA gyrase
66 D. Inhibits mycobacterial synt hesis of mycolic acid
67
E. Inhibits viral neuraminidase
68
69
70
71
72
73
74
75
76
77
. 78
• 79
. 80
.
8
Lode.
s
Suspe-nd
8
End Bloc:k
Item: 78 of ~ ,• Mark <::J [:::> ""I ~· ~'j
100 J.. Previous Next LAb faiUI~S Notes Calculator

55 •
The correct a nswer is B. 74 % chos e t his .
56 This patient 's CSF shows evidence of bacteri al mening it is. Lumbar puncture in a pat ient with bact erial mening it is usua lly shows increased
57 pressure, neut rophils, high prot ein levels, and low glucose levels. The most common cause of bact erial mening it is is Streptococcus
58
pneumoniae, though t he incidence of infect ion wit h other organisms varies by age group. I n infants, one should suspect group B streptococci
and Escherichia coli; in ch ild ren, t hink about S. pneumoniae, Neisseria meningitides, and Haemophilus influenzae; in teens and young adults
59 (especia lly college-aged), t he most likely cause is N. meningitidis. In t he elderly, S. pneumoniae is t he most common cause. Given the
60 patient's age and t he most likely organ ism (S. pneumoniae), ceftriaxone, a t hird-generation cephalosporin, is the ant ibiotic with the best CSF
61 penetrat ion and is a common choice for t reatment. I t is important to know t hat ceftriaxone is also an appropriate choice for empiri c t reat ment
if t he causat ive organ ism is not known. Cephalospori ns are 13-lactam ant ibiotics t hat inhibit bacterial cell wall synt hesis and are less sensit ive
62 t o penicillinases than t he penicillins.
63 Cephalosporin Lumbar puncture Escherichia coli Streptococcus pneumoniae Ceftriaxone Haemophilus influenzae Meningitis Antibiotics Bacterial meningitis Neisseria meningitidis Cell wall Streptococcus

64 Penicillin Streptococcus agalactiae Neutrophil Empiric therapy Protein Neisseria Glucose Cerebrospinal fluid

65 A is not correct . 2°/o chose this.


66 NSAIDs are used fo r cont rol of inf lammation and pain but are not indicated to t reat bacterial meningit is.
67 Meningitis Nonsteroidal anti-inflammatory drug Inflammation Bacterial meningitis

68 C is not correct. 15% chos e this .


69 Ciprofloxacin is t he most commonly prescribed f luoroquinolone in t he United States. The f luoroquinolones act by interfering wit h bact eri al DNA
70 gyrase duri ng cell division. Ciprofloxacin is primarily active against gram-negat ive bacteria. It is used t o t reat pseudomonal infect ions in cyst ic
f ibrosis and other hospi talized pat ients. It can also be used in cases of t raveler's diarrhea and ant hrax prophy laxis. Because many of t he
71
organ isms that cause bacterial meningit is are gram- posit ive, it would not be a f irst-line choice for t reat ment here.
72 Quinolone Cystic fibrosis Traveler's diarrhea Ciprofloxacin DNA gyrase Gram-negative bacteria Anthrax Meningitis Gram-positive bacteria Diarrhea Cell division Bacteria Pseudomonas DNA

73 Preventive healthcare Fibrosis United States

74
D is not correct. 5°/o chose t his.
75
Isoniazid is t he most potent of t he ant it uberculous drugs and is first -l ine t herapy for t uberculosis men ingit is. I n cont rast t o t his pat ient's
76 results, ana lysis of t he CSF in tuberculous meningit is t ypica lly reveals monocytic pleocytosis, elevat ed protein levels, and low glucose levels.
77 Isoniazid Tuberculous meningitis Pleocytosis Meningitis Tuberculosis Protein Glucose Monocyte Tuberculosis management

78 E is not correct. 4 °/o chos e this .


• 79 Osesltamivir is used t o t reat influenza A and B infections and works by blocking v iral neuraminidase, which decreases the release of progeny
. 80 viruses from infected cells. Viral meningit is is often caused by enterovirus, not influenza virus, and may present sim ilarly t o bact eri al
. men inqit is in t erms of a clinica l picture (stiff neck, headache, posit ive Kerniq and Brudzinski siqns), but the CSF f indinqs are different .

8
Lock
s
Suspend
0
End Block
Item: 78 of ~ ,• Mark <::J [:::> ""I ~· ~'j
100 J.. Previous Next LAb faiUI~S Notes Calculator

55 E is n ot correct. 4 °/o ch ose this.


56 Osesltamivir is used t o t reat influenza A and B infections and works by blocking viral neuram inidase, which decreases the release of progeny
57 viruses from infected cells. Viral mening it is is often caused by enterovirus, not influenza virus, and may present similarly t o bact eri al
meningitis in t erms of a clinical picture (stiff neck, headache, posit ive Kernig and Brudzinski signs), bu t the CSF f indings are diffe rent.
58
Pressure is usually normal or slightly increased, t here are lymphocytes in t he CSF, and prot ein and sugar levels are normal. Ent eroviruses are
59 t ypically the cause of viral meningitis. The prognosis of viral mening itis in adults is excellent , as most pat ients recover fully wit h m inimal
60 t reatment consisting of analgesics to help control pain and antipyretics if t here is a high fever. This treat ment is not sufficient for pat ients wit h
bact eri al mening it is, such as t he patient descri bed .
61
Enterovirus Viral neuraminidase Viral meningitis Neuraminidase Meningitis Influenza Headache Protein Influenza A virus Virus Lymphocyte Bacterial meningitis Ortholll(Xoviridae Fever Analgesic
62
63
64 Bottom Line:
65 I n bact erial meningit is, CSF contains WBCs, high protein, and low glucose, and t he mening it is is t reat ed with a third -generat ion
cephalosporin, usually ceftriaxone.
66 Cephalosporin Ceftriaxone Meningitis Bacterial meningitis Protein Glucose
67
68
69 l@ljl'il·1i•J f or yea r:[ 2017 • J
FIRST AID FA CTS
70
71
FA17 p 176.1
72
Common causes of meningitis
73
74 NEWBORN (0- 6MOl CHILDREN (6 M0- 6 YR) 6- 60YR 60YR+
75 Group B streptococci S pneumoniae S pneumoniae S pneumoniae
76 E coli N meningitidis N meningiticlis (#1 in teens) Gram 8 rods
77 Listeria H infiuenzae type B Enterovi ruses Listeria
78 Enteroviruses HSV
• 79
Give ceftriaxone and vancomycin empirically (add ampicillin if Listeria is suspected).
• 80
. Viral causes of meningitis: enteroviruses (especially coxsackievirus), llSV-2 (llSV-1 = encephalitis), liiV, West ile virus (also
8
Lock
s
Suspend
0
End Block
Item: 78 of ~ ,• Mark <::J [:::> ""I ~· ~'j
100 J.. Previous Next LAbfaiUI~S Notes Calculator
• ....__ • -·- I ' . • I
55 FIRS T AID FACTS

56
57 FA17 p 176.1
58 Common causes of meningitis
59 NEWBORN (0- 6MOl CHILDREN (6 M0- 6 YR) 6- 60YR 60YR+
60 Group B streptococci S pneumoniae S pneumoniae S pneumoniae
61
E coli N meningitidis N meningiticlis (#1 in teens) Gram 8 rods
62
Listeria H infiuenzae type B Enterovi ruses Listeria
63
Enteroviruses HSV
64
65 Give ceftriaxone and vancomycin empirically (add ampicillin if Listeria is suspected).
66 Viral causes of meningitis: enteroviruses (especially coxsackievirus), llSV-2 {llSV-1 = encephalitis), IIIV, West ile virus (also
67 causes encephalitis), VZV.
68 In HIV: Cryptococcus spp.
69 Note: Incidence of H infiuenzae meningitis has l greatly due to conjugate H infiuenzae vaccinations. Today, cases are usually
70 seen in unimmunized children.
71
72
FA17 p 176.2
73
74
CSF findings in meningitis
75 OPENING PRESSURE CELL TYPE PROTEIN GLUCOSE
76 Bacterial t t PM s t l
77
Fungai/TB t t lymphocytes t l
78
• 79 Viral Normal/t t lymphocytes Normal/t ormal
• 80
.
8
Lock
s
Suspend
0
End Block
Item: 79 of - ,• Mark -<J [:::> "'I ~· ~
100 ~ P~v1ous N @xt Labl lues N o tes Calculato r
74
Public health investigators are looking into a series of illnesses that have occurred in a small community. Many pat ients present ed with ~~AI
75
acute-onset hyperpyrexia and a part icularly severe pneumonia. A sample of a patient's sput um is shown.
76 ,
77
78 • •
. 79
. 80
. 81
• 82
• 83
• 84
. 85
...•
. 86
• 87

-
• 88
• 89
Image courtesy of CDC/ Dr. William Cherry
:
• 90
. 91
Which of the fol lowing organisms is most likely t o have caused t his outbreak?
• 92
:
• 93
A. Bordetella pertussis
• 94
• 95
B. Haemophilus influenzae type B
. 96 C. Legionella pneumophila
. 97
D. Mycobacterium tuberculosis
• 98
• 99
E. Streptococcus pneumoniae
. 100 •
8
Lode.
s
S uspe-nd
8
End Bloc:k
Item: 79 of ~ ,• Mark <::J [:::> ""I ~· ~'j
100 J.. Previous Next LAb faiUI~S Notes Calculator

74
75
The correct answer is C. 58°/o chos e this.
Legionel/a pneumophila is an aerobic, gram -negative rod t hat causes Legionnaires' disease, a condit ion in which pat ients develop acute,
76
severe pneumonia and a high fever. Legionnaires' disease is one of t he most common causes of community-acquired pneumonia but is
77 ident if ied as the cause in only 3% of cases. The organism is present only in wat er sources (eg, air condit ion ing systems, whirl pools, m ist
78 sprayers); indeed, t ransmission is not by person-to -person contact. Typically, more severe illness is seen in patient s who are >50 years of age
and t hose who smoke. Silver sta in can be used to visualize Legionella, which sta in black t o brown against a yellow backg round. Silver sta in is
79
also used t o v isulaize fungi (Coccidioides, Pneumocystisjirovecii), and Helicobacter pylori. Gram st ain shows neut rophils and very few
. 80 organ isms, as Legionella pneumophila is facult at ively int racellular.
. 81 Gram staining Legionella pneumophila Helicobacter pylori Gram-negative bacteria Legionnaires' disease Legionella Pneumocystis jirovecii Fungus Community-acquired pneumonia Pneumonia Neutrophil

• 82 Aerobic organism Fever Silver stain Intracellular

• 83 A is not correct . 5°/o chos e this.


• 84 Bordete/la pertussis is a gram -negative rod t hat causes whooping cough, charact eri zed by paroxysms of cough ing followed by a loud
. 85 inspirat ion, or "whoop." B. pertussis infect ion does not typically cause pneumonia .
Bordetella pertussis Gram-negative bacteria Pertussis Pneumonia Bordetella Cough
. 86
• 87 B is not correct. 13% chose this .
• 88 Haemophilus influenzae ty pe B is a gram- negative rod commonly associat ed with acute epiglott it is or meningit is. It is an exclusively human
pathogen t hat is t ransmitt ed by aerosolized droplets or direct cont act with secretions. The H. influenzae type B vaccine has re ndered these
• 89
infections fa r less common, making it an unlikely agent in this scenari o .
• 90 Epiglottitis Haemophilus influenzae Gram-negative bacteria Meningitis Pathogen Haemophilus Vaccine

. 91
D is not correct. 6°/o chose this .
• 92 Mycobacterium tuberculosis is an acid -fast mycobacteri um t hat causes t uberculosis. The first exposure t o M. tuberculosis, or pri mary
• 93 t uberculosis, is usually asym ptomatic. Secondary, or react ivat ion, tuberculosis occu rs after t he bact eri a have been dormant fo r some t ime and
• 94 re-emerge as a result of temporary weaken ing of t he immune system . This phase of disease is t ypically a chronic process charact erized by
low-grade feve r, night sweat s, weight loss, and productive cough. This chronic picture differentiat es tuberculosis from the acut e picture of
• 95 Leg ionnaires' disease.
. 96 Mycobacterium tuberculosis Mycobacterium Legionnaires' disease Tuberculosis Acid-fast Immune system Cough Asymptomatic Bacteria Night sweats Weight loss Fever

. 97
E is not correct. 18°/o chos e this .
• 98 Streptococcus pneumoniae manifest s acut ely and is a sign if icant cause of bacteria l pneumon ia in adults. Unlike Legione!la pneumophila,
• 99 however, sput um cultu re for S. pneumoniae infection would reveal sign if icant growth of gram- posit ive diplococci. S. pneumoniae is
• 100
t ransm itt ed by person-to -person contact.
~

8
Lock
s
Suspend
0
End Block
Item: 79 of - ,• Mark -<J [:::> "'I ~ · ~
100 ~ P~v1ous N @xt Labl lues N o tes Calculato r
74 •
75
Botto m Line:
76
L. pneumophi/a infection should be suspected in patients with acute hyperpyrexia and severe pneumonia. The organism can be visualized on
77 silver stain, but Gram stain will show neutrophils and poorly staining organisms.
78 Gn'm ~ta1n1no Pneumon•a Fe-..er Leg1onella pneumophila Neutrophil Silver stain Stainmg Infection Org.J~n•~m

79
. 80
. 81 l@l ;f.ii·11•J for year: 2017 •
FI RST AIO FACTS
• 82
• 83
FA17 p 139.2
• 84
. 85
Legionella Gram 8 rod. Gram stains poorly-use sih er Think of a French legionnaire (soldier) with
. 86 pneumophila slain. Grow on ch;ucoal yeast extract his ~ihcr helmet, sitting around a campfire
• 87 medium with iron and cysteine. Detected by (charcoal) with his iron dagger- he is no !.i~!.)'
• 88 presence of antigen in urine. Labs may show (cysteine).
• 89 hrponatremia. Aerosol transmission from
• 90 environmental water source hahilat (cg, air
. 91 conditioning systems, hot water tanks). No
• 92 person-to-person transmission .
• 93 Treatment: macrolide or quinolone .
• 94
Legionnaires' disease-severe pneumonia
• 95
(often unilateral and lobar rJ), fever, G l and
. 96
C 'S symptoms. Common in smokers and in
. 97
chronic lung disease .
• 98
Pontiac fever- mild Au-like S) ndrome.
• 99
. 100

8
Lode.
s
S uspe-nd
8
End Bloc:k
Item: 80 of - ,• Mark -<J [:::> "'I ~ · ~
100 ~ P~v1ous N @xt Labl lues N o t es Calcula to r
74
75 An 8-year-old boy com plains of a 10-day history of rig ht leg pain and decreasing ability to bear weight. He has a history of painful episodes ~~AI
in the past requ iring hospitalizat ion. His vital signs are notable for temperature 39°C (102.2°F), heart ra te 11 5/min, blood pressure 110/65
76
mm Hg, and oxygen saturation 98% on room air (normal range : 95%-99%). On physical examinat ion, t he area over the right t ibia is
77 swollen and erythematous. A blood smear confirms t he presence of numerous pathogenic bacteria and crescent- shaped blood cells.
78
79 This patient's condition predisposes him to infection with a particular bacterial species because of which of the fo llowing virulence factors?
. 80 :
. 81 A. Capsule
• 82 B. Flagella
• 83
C. Lack of cell wall
• 84
. 85 D. Mycolic acids
. 86 E. Pili
0 87
0 88
• 89
• 90
. 91
• 92
0 93
• 94
0
95
. 96
. 97
0 98
0 99
. 100 •
8
Lode.
s
Suspe-nd
8
End Bloc:k
Item: 80 of ~ ,• Mark <:::1 t::> ""I ~· ~'j
100 J.. Previous Next LAb faiUI~S Notes Calculator

74
The correct ans wer is A. 69°/o chose this.
75
This patient 's hist ory and current presentat ion raise suspicion fo r sickle cell disease . Aut osplenect omy typica lly occurs in sickle cell patients by
76
age 8 years as a result of repeat ed splen ic infarcts. Pat ients wit h impaired splenic function are suscept ible to infect ion by encapsulat ed
77 organ isms such as Salmonella typhi. The sympt oms suggest t hat the boy has ost eomyelit is of the t ibia, and t he most important det erm inant
78 of v irulence in sickle cell pat ients wit h osteomyelit is is the presence of a capsule. Staphylococcus aureus is the most common cause of
ost eomyelit is in all groups, but Salmonella ost eomyelit is has a particular associat ion wit h sickle cell disease, and it is t his associat ion that is
79
often test ed on the Step 1 USM LE.
80 Osteomyelitis Autosplenectomy Salmonella enterica subsp. enterica Staphylococcus aureus Sickle-cell disease Salmonella Staphylococcus Polysaccharide encapsulated bacteria Infarction Spleen Tibia

. 81 Bacterial capsule Virulence Infection

• 82
B is not correct. 14% chose this .
• 83
The flagellum of Salmonella is indeed a virulence factor that acts as an ant igen t hat can be va ried (antigenic variat ion or swit ching) . However,
• 84 it is not t he presence of t he f lagellum t hat makes Salmonella more pat hogenic in sickle cell patients .
Virulence factor Antigenic variation Salmonella Flagellum Antigen Virulence Pathogen
. 85
. 86 C is not correct. 5 °/o chose this .
• 87 Mycoplasma species are bact eri a t hat lack a cell wall and that most often cause pneumon ia (M. pneumoniae) or nongonococca l urethri t is (a lso
• 88 Ureaplasma urealyticum ) .
Non-gonococcal urethritis Ureaplasma urealyticum Urethritis Ureaplasma Pneumonia Mycoplasma Bacteria Cell wall Mycoplasma pneumoniae
• 89
• 90 D is not correct. 4°/o chose this .
. 91
Mycolic acids are found in t he out er cell membrane of mycobacteria . Mycobacteria can cause ost eomyelit is, but autosplenectomy does not
particularly put sickle cell patient s at a higher ri sk .
• 92 Osteomyelitis Autosplenectomy Mycobacterium Cell membrane Sickle-cell disease Mycolic acid
• 93
E is not correct. 8 °/o chos e this .
• 94
Pili are st ruct ures on some bacteria that aid in adhesion to host cells .
• 95 Pilus Bacteria Cell adhesion Adhesion

. 96
. 97
Bottom Line:
• 98
Salmonella ost eomyelit is has a pa1t icular associat ion with sickle cell disease. Sickle cell patients are vulnerable to encapsulat ed bact eri a
• 99
because of t he loss of splenic funct ion .
• 100 ~ Osteomyelitis Sickle-cell disease Polysaccharide encapsulated bacteria Salmonella Soleen Bacteria Sickle

8
Lock
s
Suspend
0
End Block
Item: 80 of ~ ,• Mark <:::1 t::> ""I ~· ~'j
100 J.. Previous Next LAb faiUI~S Notes Calculator

y p I p p I

75 because of t he loss of splenic function.


Osteomyelitis Sickle- cell disease Polysaccharide encapsulated bacteria Salmonella Spleen Bacteria Sickle
76
77
78
i@l;fil·1i•J for yea r:[ 201 7 • J
79 FIRST AID FACTS

80
. 81 FA17 p 176.4
• 82
Osteomyelitis RISK FACTOR ASSOCIATED INFECTION
• 83
Assume if no other information is available S aureus (most common overall)
• 84
. 85 Sexually active Neisseria gonorrhoeae (rare), septic arthritis more
. 86 common
• 87
Sickle cell disease Scdmonella and S aureus
• 88
• 89
Prosthetic joint replacement S aureus and S epidermidis
• 90 Vertebral involvement S aureus, Mycobacterium tuberculosis (Pott
. 91 disease)
• 92
Cat and dog bites Pasteurella multocida
• 93
• 94
IV drug abuse Pseudomonas, Candida, S aureus are most
• 95
common
. 96 Elevated C-reactive protein (CRP) and erythrocyte sedimentation rate common but nonspecific.
. 97 M Rl is best for detecting acute infection and detail ing anatomic involvement a.
Radiographs are
• 98 insensiti\·e early but can be useful in chronic osteomyelitis rn.
• 99
• 100 ~

8
Lock
s
Suspend
0
End Block
Item: 81 of - ,• Mark -<J
P~v1ous
[:::> "'I ~ ·· ~
100 ~ N @xt Labl lues N o tes Calcula to r
74
75 A 3-year-old boy presents t o th e pediat ri c emergency depart ment with his mother. The moth er explai ns t hat her son has been "burning up" ~~AI
and complaining that his head hurts. On physical examination he displays photophobia, and cervical stiffness is noted. Lumbar puncture
76
and cerebrospinal flu id analysis shows an increase in polymorphonuclear cells, an increased protein level, and a decreased glucose level.
77
78 After considering the most likely agent s in t his age group, you believe the Gram stain of the cerebrospinal fluid will most likely show which of the
79 following?
80 :
. 81 A. Gram-negative diplococci
• 82 B. Gram-negative rods
• 83
C. Gram-positive diplococci
• 84
. 85 D. Gram-positive rods
. 86 E. Negative Gram stain
• 87
• 88
• 89
• 90
. 91
• 92
• 93
• 94
• 95
. 96
. 97
• 98
• 99
. 100 •
8
Lode.
s
Suspe-nd
8
End Bloc:k
Item: 81 of - ,• Mark -<J
P~v1ous
[:::> "'I ~ ·· ~
100 ~ N @xt Labl lues N o tes Calcula to r
74 A A
The correct answer is C. 44°/o chose this.
75
This patient's clinical presentation and cerebrospinal fluid (CSF) analysis are consistent with a bacterial
76 meningit is. The most common cause of bacterial mening itis in children between 3 months and 10 years of '
77 age is Streptococcus pneumoniae, which shows up as gram-positive diplococci on Gram stain, as seen in

.. ,..... ...
78
the image. In adolescents and young adults, meningitis caused by Neisseria meningitidis is more common,
but Strepococcus pneumoniae again becomes the most common agent when it comes to adults.
79 Gr~ o;.t nrng N,. e 11 nenrngrbd ~ Diplococcus streptococcus pneumoniae Cerebrosprn~ flurd Menrng r Gl n·;>ottrb e boctena Baeten~ menrngitis

80 Streptococcus Ne.ssena .. .
81
•, "to
• 82 ' .
• 83 Image courtesy of CDC/ Or. Mike
• 84 Miller
. 85
A is not correct. 20% chose this .
. 86
Although Neisseria meningitidis, which shows on Gram stain as gram-negative diplococci, would have similar CSF findings, it is not the most
• 87 likely source of bacterial meningitis in this age group. It is more common in adolescents and young adults, specifically between the ages of 10
• 88 and 19 years of age .
Gram stainrno Nerssena men~ngrtrdrs Orplococcus Gram- negative bacteria Meningrtrs Bactenal menrngrtls Nerssena
• 89
• 90 B is not correct. 19°/o chose this .
. 91 Although Haemophilus influenzae, which is a gram-negative rod, would show similar CSF find ings, H. influenzae is a less common cause of
• 92
meningitis In children o f this age group. This is largely due to th e int roduction of the H. influenzae type B vaccine, which has greatly
decreased the incidence of t his t ype of mening it is .
• 93 Haemophilus influenzae Gram·negabve bacteria Meningitis Vaccine Haemophilus

• 94
D is not correct. 12°/o chose this .
• 95
Listeria species, which are gram-positive rods, would show similar CSF findings, but Listeria is not a common cause of meningit is in this age
. 96 group. It is much more commonly seen in newborns age 0-6 months and in the elderly.
. 97 Bacrlli Gram-posrttve bactena Memngrbs Llstena Species Cerebrospinal fluid

• 98 E is not correct . 5°/o chose this.


• 99 The lab results do not fit well with a viral meningitis. In viral meningitis, CSF typically shows an elevated lymphocyte cell count (although
. 100 polymorphonuclear cells may predominate in t he first 48 hours), moderately increased protein level, and normal glucose level.

8
Lode.
s
Suspe-nd
8
End Bloc:k
Item: 81 of ~ ,• Mark <::J [:::> ""I ~· ~'j
100 J.. Previous Next LAbfaiUI~S Notes Calculator

74
75
Bottom Line:
76
The most likely agent of bacteria l meningit is in ch ildren between 3 months and 10 years of age is Strepococcus pneumoniae.
77 Meningitis Bacterial meningitis
78
79
80 141;fil·1i•J for year:[ 2017
FIRST AID FA CTS .
•j .
81
• 82
• 83
FA17 p 176.1

• 84
Common causes of meningitis
. 85 NEWBORN (0- 6MOl CHILDREN (6 M0- 6 YR) 6- 60YR 60YR+
. 86 Group B streptococc i S pneumoniae S pneumoniae S pneumoniae
• 87 E coli N meningitidis N meningiticlis (#1 in teens) Gram 8 rods
• 88 Listeria H infiuenzae type B E nterovi ruses Listeria
• 89 Enteroviruses HSV
• 90
Give ceftriaxone and vancomycin empirically (add ampicillin if Listeria is suspected).
. 91
Viral causes of mening itis: enteroviruses (especially coxsackievirus), llSV-2 {llSV-1 = encephalitis), IIIV, West ile virus (also
• 92
• 93
causes encephalitis), VZV.
• 94
In HIV: Cryptococcus spp.
• 95 Note: Incidence of H infiuenzae meningitis has l g reatly due to conjugate H infiuenzae vaccinations. Today, cases are usually
. 96 seen in unimmun ized children .
. 97
• 98
FA17 p 132.2
• 99
. 100 Streptococcus Gram <±), lancet-shaped diplococci fJ. Pneumococcus is associated with "rusty"
8
Lock
s
Suspend
0
End Block
Item: 81 of ~ ,• Mark <::J [:::> ""I ~· ~'j
100 J.. Previous Next LAbfaiUI~S Notes Calculator

74
Common causes of meningitis
75
NEWBORN (0- 6MOl CHILDREN (6 M0- 6 YR) 6- 60YR 60YR+
76
77 Group B streptococci S pneumoniae S pneumoniae S pnewnoniae
78 E coli N meningitidis N meningiticlis (#1 in teens) Gram 8 rods
79 Listeria H infiuenzae type B Enterovi ruses Listeria
80 Enteroviruses HSV
81
Give ceftriaxone and vancomycin empirically (add ampicillin if Listeria is suspected).
• 82
Viral causes of meningitis: enteroviruses (especially coxsackievirus), llSV-2 {llSV-1 = encephalitis), l iiV, West ile virus (also
• 83
causes encephalitis), VZV.
• 84
. 85
In HIV: Cryptococcus spp.
. 86
ote: Incidence of H infiuenzae meningitis has l greatly due to conjugate H infiuenzae vaccinations. Today, cases are usually
• 87
seen in unimmunized children .
• 88
• 89
FA17 p 132.2
• 90
. 91
Streptococcus Gram E£), lancet-shaped diplococci fJ. Pneumococcus is associated with "rusty"
• 92 pneumoniae Encapsulated. IgA protease. Optoch in sputum, sepsis in patients with sickle cell
disease, and asplenic patients.
• 93
a ,, sensitive. Most common cause of:
o virulence without capsule.
• 94
, Meningitis
• 95 • Otitis media (in children)

. 96 Bacterial pneumonia
. 97
• 98
,..... •' . • Sinusitis

• 99
• 100

8
Lock
s
Suspend
0
End Block
Item: 82 of - ,• Mark -<J [:::> "'I ~ · ~
100 ~ P~v1ous N @xt Labl lues N o tes Calcula to r
74
A 28-year-old white man presents to his fam ily physician complaining of painful urination . Physical examination revea ls a purulent urethral ~~AI
75
discharge. Gram stain demonst rates infection with Neisseria gonorrhoeae. An HIV test is positive.
76
77
Which of the following genotypes would have helped prevent this patient from contracting HIV?
78
:
79
A. Expression of HLA-627
80
81
B. Heterozygous for the CCR5 mutation
• 82 C. Heterozygous for the CXCR4 mutation
• 83
0 . Homozygous for the CCR5 mutation
• 84
. 85
E. Homozygous for the CXCR4 mutation
. 86 F. Polymorphisms of the RANTES
• 87
• 88
• 89
• 90
. 91
• 92
• 93
• 94
• 95
. 96
. 97
• 98
• 99
. 100 •
8
Lode.
s
S uspe-nd
8
End Bloc:k
Item: 82 of - ,• Mark -<] 1:> ""'I ~· 1!';:'1
100 ~ Prev1o u s Next Labf a lu es Notes Calculator
74
75 The correct answer is D. 60°/o chose this.
76 The CCR5 gene encodes for the CCRS receptor, which is a chemokine receptor on macrophages that facilit at es viral entry into cells. Individuals
who are homozygous for the CCR5 mutat ion are immune to infection with certain common strains of HIV. The mutation is almost never seen
77
in those of Asian or African decent .
78 Zygos1t-, G4ffle CheMOk1ne Homor,gous CCRS Chemokine receptor Macrophage Mutation Virus Receptor (btochem•stry) Viral entry

79
A is not correct. 3°/o chose this.
80
Expression of HLA-B27, along with at least three other types of alleles, has been associated with a long-term delay in progression of HIV
81 disease, but does not confer immunity.
82 HLA·82 HI "' e

. 83 B is not correct. 11 °/o chose this.


• 84 Heterozygosity for the CCR5 gene does not confer immunity to HIV. However, in 20% of white patients who are heterozygous for the CCR5
. 85 mutation, progression of HIV infection may be delayed .
Gene HIV ee. 5 2¥9o••tv Muto'lbon Heterozygous Immumty (medocal) Infection
. 86
• 87 C is n ot correct. 9°/o chose this .
• 88 CXCR4 is the other major chemokine receptor used by HIV to enter host cells. Neither homozygosity nor heterozygosity of the CXCR4 gene
has been shown to confer immunity or delay progression of HIV infection .
• 89
ehemokone Gene exeR4 HIV ehemokone receptor Homozygous Receptor (boochemostry) Zygosoty lmmumty (medocal) lnfectoon
• 90
E is not correct. 16°/o chose this .
. 91
CXCR4 is the other major chemokine receptor used by HIV to enter host cells. Neither homozygosity nor heterozygosit y of t he CXCR4 gene
• 92
has been shown to confer immunity or delay progression of HIV infection .
• 93 Chemoktne Gene CXCR4 HIV Chemokine receptor Homozygous Receptor (biochemtstry) Zygostty Immuntty (medtcal) Infectton

• 94
F is not correct. 1 °/o chose this .
• 95 RANTES is the main natural ligand that binds to CCRS, and polymorphisms in this protein have been investigated for their role in conferring
. 96 immunity or nonprogressor status in HIV infection. However, several studies have shown that RANTES does not affect the course of HIV
. 97 infection .
eeLS eeRS HIV Logand (boochemostry) Ligand Proteon Polymorphism (boology) lmmunoty (medocal) lnfectoon
• 98
• 99
• 100 Botto m line : I.
8
L.odt
s
Su~pl'nd
~
End Block
Item: 82 of - ,• Mark -<J [:::> "'I ~ · ~
100 ~ P~v1ous N @xt Labl lues N o tes Calcula to r
74 •
75 Botto m Li n e:
76 People who are homozygous for mutations of the CCRS receptor on macrophages are immune to certain strains of HIV.
77 Homozygous CCRS Zygo51ty Macrophage Mutat1on

78

79
80 l@l ;fii ·11•J for year : 2017 •
FI RST A ID FACTS
81
82 FA17 p 171 .1
• 83
HIV
• 84
. 85 Envtlopt protttns Diploid genome (2 molecules of R 'A).
acquored through buddng from
. 86
host cell p4asma membrane The 3 structural genes (protein coded for):
• 87
p17: Matr1x protein env (gpl20 and gp41):
• 88
gpl20 Formed from cleavage of gpl60 to form
• 89 DOCking envelope glycoproteins.
gCycoproteon '-.
• 90 gpl20-attachment to host CD4+ T cell.
gp41
. 91 Transmembrane gp41 - fu sion and en try.
• 92 glycoprotein
gag (p24 and p17)-capsid and matrix
• 93 Reverse
RNA " " "
proleins, respectively.
• 94 transctlptase
pol- reverse tra nscriptase, asp a rtatc protease,
• 95
integrase.
. 96
. 97
Reverse lranscriptase synthesizes dsDNA from
• 98
genom ic R A; dsD 1A integrates into host
• 99 genome.
. 100 Virus binds CD4 as " ·ell as a corcccptor, either
8
Lode.
s
S uspe-nd
8
End Bloc:k
Item: 82 of - ,• Mark -<J [:::> "'I ~ · ~
100 ~ P~v1ous N @xt Labl lues N o tes Calcula to r
74 A
genome. A

75
Virus binds CD4 as well as a corcccptor, either
76
CCR5 on macrophages (early infection) or
77
C CR4 on T cells (late infection).
78 u l lomozygous CCR5 mutation =immunity.
79
Heterozygous CCR5 mutation = slower course.
80
81
82
FA17 p 171.2
• 83
• 84
HIV diagnosis Presumpti,·e diagnosis made with ELISA ELISA/Western blot tests look for antibodies to
. 85 (sensitive, high fa lse<±> rate and lo'' threshold, viral proteins; these tests often are fal selr e
. 86 rule out test);<±> results confirmed with in the first 1- 2 months of HI infection and
• 87 Western blot assay (specific, low false <±> rate falselr <±> initially in babies born to infected
• 88 and high threshold, rule in test). mothers (anti-gp120 crosses placenta). Use
• 89 Viral load tests determine the amount of PCR in neonates to detect viral load .
• 90 viral Rr A in the plasma. High viral load
. 91 associated with poor prognosis. Also use viral
• 92 load to monitor effect of drug therapy. Il lV
• 93 genotyping used to detcn nine appropriate
• 94
therapy.
• 95
AIDS diagnosis$; 200 CD4+ cells/mm3
. 96
(normal: 500-1500 cells/mm3). HI (±) with
. 97
AIDS-defining condition (eg, Pneumocystis
• 98
pneumonia) or CD4+ percentage < 14%.
• 99
. 100 •
8
Lode.
s
S uspe-nd
8
End Bloc:k
Item: 82 of - ,• Mark -<] 1:> ""'I ~- 1!';:'1
100 ~ Prev1o u s Next Labf a lu es Notes Calculator
74
75
FA17 p 172.1
76 Time course of untreated HIV infection
77
Window period +I- Acute HIV infection Slun and mucous Systemk immuno-
78 Viral dissemination membrane deficiency/AIDS·
Seeding of lymphoid organs Clinkallatency infections defining illnesses
79
80
81
1200 Anti·envtlope antibody (gpl20) 10
82
. 83
uoo
• 84
. 85 1000
COSt Tcetl
. 86
900
• 87
~

• 88 E
800
• 89 --
E
~
Qj
u 700
:J:
<
Xl
z
• 90 + >
QO n
. 91 0 0
u.._ 600 -o
• 92
• 93
0
+
v
0 500
--
~-
3
,.-
• 94 u
HIVRNA 10'
• 95 400
. 96
300
. 97
• 98 200
• 99
• 100
100 •
8
L.odt
s
Su~pl'nd
~
End Block
Item: 83 of - ,• Mark -<J [:::> "'I ~ · ~
100 ~ P~v1ous N @xt Labl lues N o tes Calcula to r
74
A A 45-year-old man who is HIV posit ive is being t reat ed with highly active antiretroviral therapy. Despite his aggressive medication reg imen, ~AI A

75 he has recently developed the lesions shown in th e image. His examination is also notable for nonpitting edema of t he feet and ankles.
76
77
78

79
80
81
82
• 83
• 84
. 85
. 86
• 87
• 88

• 89
• 90
. 91
• 92
• 93
• 94
• 95 Which of the following is most appropriate for t reat ing this patient's lesions?
. 96
:
. 97 A. a-Interferon
• 98
8. 13-Interferon
• 99
. 100 C. y - Interferon

8
Lode.
s
S uspe-nd
8
End Bloc:k
75
76
77
78
79
80
81
82
• 83
• 84
. 85
. 86
• 87
• 88
• 89

• 90
. 91
• 92 Which of the fol lowing is most appropriate for treat ing t his patient's lesions?
• 93
:
• 94 A . o-Interferon
• 95
B. ~-Interferon
. 96
. 97 C. y-Interieron
• 98 D. Tacrolimus
• 99
E. Tamoxifen
• 100 •
8
L.odt
s
Su~pl'nd
~
End Block
Item: 83 of ~ ,• Mark <::J [:::> ""I ~· ~'j
100 J.. Previous Next LAbfaiUI~S Notes Calculator

74
75 The correct ans wer is A. 45°/o chose this.
76 This patient 's lesions are consist ent wit h Kaposi's sarcoma, a cutaneous malignancy derived f rom lymphatic and vascular endot helial cells.
Kaposi's sarcoma is caused by human herpes virus-8 (HHV-8) and is most f requently seen in patients with AIDS . I t can present with t he skin
77
lesions seen in t he image, wit h lymphedema of t he extremit ies, genitalia, or face or with spread to t he oral m ucosa, gastrointest ina l t ract , or
78 respirato ry t ract. I nterferons are cytot oxic prot eins t hat int erfere wit h vira l protein expression and DNA synthesis. a-I nt erferon is used for it s
79 leukocytic effects in pat ients wit h hepat it is B, hepat it is C, Kaposi's sa rcoma, leukemias, and malignant melanoma. Ot her t reatments for
Kaposi's sarcoma include local radiation t reatment or chemot herapeutic agents such as doxorubicin or paclitaxel. Of not e, bacillary
80
angiomatosis (caused by Bartonella species) can present wit h cut aneous lesions that appear ident ical t o t hose of Kaposi's sarcoma. The only
81 way to definit ively different iat e between t he t wo diseases is to biopsy the lesion and ident ify Bartonella organisms with Warthin-St arry silver
82 staining. Bacillary angiomatosis can also present wit h systemic signs of infect ion (fever, chi lls, malaise), subcut aneous lesions, osseous
lesions, mucosal lesions, or cent ral nervous syst em lesions. The infection is t reated wit h erythromycin or doxycycline.
83
Erythromycin Doxorubicin Doxycycline Paclitaxel Human gastrointestinal tract Kaposi's sarcoma Hepatitis C Central nervous system Melanoma Lymphedema Hepatitis 8 Bacillary angiomatosis Biopsy
• 84
Kaposi's sarcoma-associated herpesvirus Gastrointestinal tract Endothelium HIV/AIDS Herpes simplex Chemotherapy Malignancy Sarcoma Oral mucosa Malaise White blood cell DNA replication Interferon
. 85
Bartonella Cytotoxicity Radiation therapy Lesion Lymphatic system Hepatitis Blood vessel Fever Mucous membrane Cancer Angiomatosis Virus Leukemia Sex organ Bone Protein DNA Nervous system
. 86
Respiratory tract Subcutaneous tissue Lymph Infection
• 87
• 88
B is not correct. 13% chose this .
Connective t issue fibroblasts produce 13-interferon; a synthet ic version is used t o t reat mult iple scl erosis. It has no role in t reating Kaposi's
• 89
sarcoma .
• 90 Multiple sclerosis Kaposi's sarcoma Connective tissue Fibroblast Sarcoma

. 91
C is not correct. 25% chos e this .
• 92
y- I nt erferon is produced by T lymphocytes and activat es tumoricida l macrophages. It is used for patients with ch ron ic granulomatous disease
• 93 and has no role in t reat ing Kaposi's sarcoma .
Chronic granulomatous disease Kaposi's sarcoma Lymphocyte Macrophage Chemotherapy T cell Sarcoma Granuloma
• 94
• 95 D is not correct. 11% chose this .
. 96 Tacrolimus is an immunosuppressive agent used in organ t ransplant recipients. It has no effect in t reat ing Kaposi 's sarcoma .
Tacrolimus Kaposi's sarcoma Organ transplantation Immunosuppressive drug Immunosuppression Sarcoma
. 97
• 98 E is not correct. 6 °/o chos e this.
• 99 Select ive est rogen receptor modulat ors such as tamoxifen block t he binding of est rogen to estrogen rece pt or- posit ive cells and are most
• 100
common ly used to t reat breast cancers. They have no effect in t reat ing Kaposi's sarcoma .
~

8
Lock
s
Suspend
0
End Block
Item: 83 of ~ ,• Mark <::J [:::> ""I ~· ~'j
100 J.. Previous Next LAb faiUI~S Notes Calculator

74
75 Bottom Line:
76
Kaposi 's sarcoma is a cutaneous malignancy most commonly seen in pat ients with AIDS. It can be treat ed with a-i nt erferon .
77 Kaposi's sarcoma Malignancy Cancer Sarcoma HIV/AIDS

78
79
80 l@l;fil·1i•l for year:[ 2017
FIRST AID FACTS .
•j .
81
82
FA17 p 117.2
83
• 84 Recombinant AGENT CliNICAl USES
. 85 cytokines and clinical Aldesleukin (IL-2) Renal cell carcinoma, metastatic melanoma
. 86 uses
Epoetin alfa (erythropoietin) Anemias (especially in renal failure)
• 87
• 88 Filgrastim (G-CSF') Recovery of bone marrow and WBC counts by
• 89 granulocyte stimulation
• 90 Sargramostim (GM-CSF) Reco,·ery of bone marro,,· and WBC counts by
. 91 granulocyte and monocyte stimulation
• 92
IF! -a Chronic hepatitis B and C, Kaposi sarcoma,
• 93
• 94
malignant melanoma, hairy cell leukemia,
• 95
condyloma acuminata, renal cell carci noma
. 96 lFr -~ Multiple sclerosis
. 97
IF! -y C hronic granulomatous disease
• 98
• 99
Romiplostim (thrombopoietin analog), Thrombocytopenia
. 100 eltrombopag (thrombopoietin receptor agonist)

8
Lock
s
Suspend
0
End Block
Item: 8 3 of ~ ,• Mark <::J [:::> ""I ~· ~'j
100 J.. Previous Next LAb faiUI~S Notes Calculator

74
75 FA17 p 173.1
76
Common d iseases of As CD4+ cell count ~. risks of reactivation of past infections (eg, TB, HSV, shingles), dissemination
77
HIV-posit ive adults of bacterial infections and funga l infections (eg, coccidioidomycosis), and non-Hodgkin
78
lymphomas t.
79
80 PATHOGEN PRESENTATION FINDINGS
81 CD4+ cell count < 500/mm 3
82
Candidaalbicans Oral thrush Scrapable white plaque, pseudohyphae on
83
m1croscopy
• 84
. 85 EBV Oral hairy leukoplakia Unscrapable white plaque on lateral tongue
. 86 Bartonella henselae Bacillary angiomatosis Biopsy with neutrophilic in Aammalion
• 87
HHV-8 Kaposi sarcoma Biopsy with lymphocytic inflammation
• 88
HPV Squamous cell carcinoma, commonly of anus
• 89
• 90
(men who have sex with men) or cerYix
. 91
(women)
• 92 CD4+ cell count < 200/mm 3
• 93
Histoplasma Fever, weight loss, fatigue, cough, dyspnea, Oval yeast cells within macrophages
• 94
capsulatum nausea, vom iting, diarrhea
• 95
. 96
HIV Dementia
. 97 JC virus (reactivation) Progressive mu ltifoca l leukoencephalopathy onenhancing areas of demyelination on IRl
• 98 Pneumocystis jirovecii Pneumocrstis pneumonia "Ground-glass" opacities on CXR
• 99
CD4+ cell count< 100/ mm 3
• 100 ~

8
Lock
s
Suspend
0
End Block
Item: 83 of - ,• Mark -<] 1:> ""'I ~· 1!';:'1
100 ~ Prev1o u s Next Labf a lu es Notes Calculator
74
HIV Dementia
75
76
JC virus (reactivation) Progressive mu ltifocalleukoencephalopathy onenhancing areas of demyelination on IRI
77 Pneumocystis jirovecii Pneumocystis pneumonia "Ground-glass" opacities on CXR
78
CD4+ cell count < 100/ mm3
79
80
Aspergillus fumigatus Hemoptysis, pleuritic pain Ca\ itation or infiltrates on chest imaging
81 Candida albicans Esophagitis White plaques on endoscopy; yeast and
82 pseudo h) phae on biopsy
83 CMV Retinitis, esophagitis, colitis, pneumonitis, Linear ulcers on endoscopy, cotton-wool spots
• 84 on fundoscopy
encephalitis
. 85
Biopsy re\·eals cells with intranuclear {owl eye)
. 86
inclusion bodies
• 87
• 88
Cryptococcus Meningitis Encapsulated yeast on India ink stain or
• 89
neoformans capsular antigen<±>
• 90 Cryptosporidium spp. Chronic, \\·atery diarrhea Acid-fast oocysts in stool
. 91 EBV B-celllymphoma {eg, non-Hodgkin lymphoma, C S lymphoma- ring enhancing, may be
• 92 solitary (vs Toxoplasma)
C S lymphoma)
• 93
Mycobacterium onspecific systemic symptoms (fever, night
• 94
• 95
avium- intracellulare, sweats, weight loss) or foca l lrmphadcn itis
Mycobacterium avium
. 96
. 97 complex
• 98 Toxoplasma gondii Brain abscesses Multiple ring-enhancing lesions on ~ IRI
• 99
• 100 •

8
L.odt
s
Su~pl'nd
~
End Block
Item: 84 of - ,• Mark -<J [:::> "'I ~ · ~
100 ~ P~v1ous N @xt Labl lues N o tes Calculato r
74
A 45-year-old man present s to his primary care physician wit h t he recent onset of product ive cough, dyspnea, and fever. A sample of the ~~AI
75
patient's sputum is smeared ont o a blood agar plate, which grows several colonies that t urn the surrounding medium green. An optochin
76
disc placed on one colony inhibits all growth surrou nding t he disc.
77
78 Which virulence factor contributes to this organism's ability to colonize the lung?
79
:
80 A. IgA protease
81
B. Ligand for cellular adhesion
82
83 C. Lipopolysaccharide
• 84 0 . Pili
. 85
E. Protein A
. 86
• 87
• 88
• 89
• 90
. 91
• 92
• 93
• 94
• 95
. 96
. 97
• 98
• 99
. 100 •
8
Lode.
s
S uspe-nd
8
End Bloc:k
Item: 84 of ~ ,• Mark <:::1 t::> ""I ~· ~'j
100 J.. Previous Next LAbfaiUI~S Notes Calculator

74
The correct ans wer is A. 58°/o chose this.
75
The organism ident ified is Streptococcus pneumoniae. S. pneumoniae can be dist inguished from other m icroorganisms by virtue of its ability
76 t o produce a-hemolysis (t urn ing blood agar green) and its sensit ivity t o opt ochin. Another import ant feature of S. pneumoniae is its ability t o
77 produce l gA protease enzymes that cleave secretory lgA antibodies. Secretory l gA normally provides the maj or defense fo r mucosal surfaces
78
against bact erial colonization. The compromise of this defense system st rongly cont ri but es t o t he virulence of S. pneumoniae. Ot her bacteria
t hat secrete lgA proteases include Neisseria meningitidis, N. gonorrhoeae, and Haemophi/us influenzae. Another key virulence fact or of S.
79 pneumoniae, and t he MOST important for coloni zing t he lu ngs, is t he presence of a polysacchari de capsule, which enables the organism to
80 resist phagocytosis.
Polysaccharide Neisseria meningitidis Streptococcus pneumoniae Phagocytosis Haemophilus influenzae Virulence factor Optochin Neisseria gonorrhoeae Streptococcus Virulence Neisseria Antibody
81
Microorganism Immunoglobulin A Blood agar Bacteria Protease Agar plate Organism Agar Enzyme Lung Mucous membrane
82
83 B is not correct. 9 % chose this.
84 Rhinovirus uses l CAM- 1 (intracellular adhesion molecule- 1), a prot ein expressed on respi rat ory epit helium, as a mode of ent ry to init iat e
. 85
infection . Rhinovirus is a common cause of upper respi ratory infections .
Rhinovirus ICAM-1 Respiratory epithelium Protein Upper respiratory tract infection Cell adhesion Epithelium
. 86
• 87 C is not correct. 13% chos e this .
• 88
Lipopolysacchari de (LPS), also known as endotoxins, are composed of a lipid and polysacharide, fo und on t he out er membrane of Gram -
negative bacteria. It induces strong immune response in t he host.
• 89 Lipopolysaccharide Gram-negative bacteria Immune system Lipid Bacterial outer membrane Mitochondrion Bacteria
• 90
D is not correct. 9°/o chos e this .
. 91
Pili can contribu te t o bact eri al cell adhesion and virulence but are found predominantly on gram -negative bacteria .
• 92 Gram-negative bacteria Pilus Cell adhesion Virulence Bacteria Adhesion
• 93
E is not correct. 11°/o chos e this .
• 94
Protein A binds Fc-lgG and inhibits complement fixat ion and phagocytosis. However, it is a virulence fact or associat ed with Staphylococcus
• 95 aureus, not Streptococcus pneumoniae .
. 96 Streptococcus pneumoniae Phagocytosis Virulence factor Staphylococcus aureus Protein A Protein Streptococcus Virulence Staphylococcus

. 97
• 98 Bottom Line:
• 99
Streptococcus pneumoniae diffe rs from other bacteria in t hat it produces a- hemolysis, result ing in green halos in blood agar. S. pneumoniae
• 100 ~ produces loA protease enzvmes t hat cleave secret ory l oA ant ibodies .
8
Lock
s
Suspend
0
End Block
Item: 84 of - ,• Mark -<J [:::> "'I ~ · ~
100 ~ P~v1ous N @xt Labl lues N o tes Calculato r
74 A A

75 l@l : fil ~!l•l for yea r: 20 17 "


FI R ST AID FACTS
76
77
FA17 p 132.2
78

79 Streptococcus Gram EEl, lancet-shaped diplococci · . Pneumococcus is associated with "rusty"


80 pneumoniae Encapsulated. lg protease. Optochin sput urn, sepsis in patients '' ith sickle cell
81 sensitive. ~ lost common cause of: disease, and asplenic patients.
82
rJ (' \ leningitis o 'irulence without capsule.
,
83 • Otitis media (in children)
84
Bacterial pneumonia
. 85
• Sinusitis
. 86
0 87
0 88 I

• 89
• 90
FA17 p 131 .1
. 91
• 92 a-hemolytic bacteria Gram EEl cocci. Partial reduction of hemoglobin causes greenish or brownish color without clearing
0 93 around growth on blood agar floInclude the following organisms:
• 94 Streptococcus pneumoniae (catalase 8 and optochin sensitive)
0
95 Viridans streptococci (catalase 8 and optochi n resistant)
. 96
. 97
0 98
0 99
0
100

8
Lode.
s
S uspe-nd
8
End Bloc:k
Item: 84 of - ,• Mark -<] 1:> ""'I ~· 1!';:'1
100 ~ Prev1o u s Next Labf a lu es Note s Calculator
74
FA1 7 p 101 .1
75
76 Immunoglobulin All isotypes can exist as monomers. \ Iature, na ive B cells prior to activation express lg I ancl lgD
77 isotypes on their surfaces. The) may differentiate in germinal centers of lymph nodes by isotype switching
78 (gene rearrangement; mediated by cytokines and CD40L) into plasma cells that secrete IgA, lgE,
79 or lgC.
80
81
lgG zo
Main antibody in {delayed) response to an antigen. lost abundant isotypc in serum. Fixes
82
complement, crosses the placenta {provides infants with passi'c immunity), opsonizcs bacteria,
83
neutralizes bacterial toxins and viruses.
84
. 85
lgA Pre,·ents attachment of bacteria and viruses to mucous membranes; does not fix complement.
. 86
.\ lonomer (in circulation) or dimer (with J chain when secreted). Crosses epithelial cells by
• 87
transcytosis. Produced in C l tract (eg, by Peyer patches) and protects against gut infections (cg,
• 88
Giardia). .\lost produced antibody overall, but has lower serum concentrations. Released into
• 89
secretions (tea rs, saliva, mucus) <mel breast milk. Picks up secretory component from epithelial cells,
• 90
. 91
which protects the F'c portion from luminal protcases.
• 92 lgM Produced in the 1° (immediate) response to an antigen. Fixes complement but docs not cross the
• 93 placenta. Antigen receptor on the surface of B cel ls. Monomer on B cell, pentamer with J chain
• 94 when secreted. Pentamer enables avid bi nding to antigen while humoral response evolves .
• 95
. 96
. 97
• 98 IgO Unclear function. Found on ~urf<ICC of manv

B cells and in serum.
• 99
• 100 •
8
L.odt
s
Su~pl'nd
~
End Block
Item: 85 of - ,• Mark -<J [:::> "'I ~ · ~
100 ~ P~v1ous N @xt Labl lues N o tes Calcula to r
74
A 32-year-old pregnant woman had a near fatal ill ness after ingesting river water on a trip abroad . Her symptoms included jaundice, ~~AI
75
anorexia, fever, severe abdominal pain, nausea, vomit ing, and malaise. Many other people on t he t rip exp erienced sim ilar, but less severe,
76
symptoms. Laboratory tests showed an alanine aminotransferase of 600 U/L and an aspartate aminot ransferase of 420 U/L.
77
78 What type of virus did this woman most likely contract ?
79
:
80 A. Enveloped RNA virus
81
B. Enveloped, circular DNA virus
82
83 C. Nonenveloped, single-stranded RNA virus
84 0 . Small defective circular RNA virus
. 85
. 86
0 87
0 88
• 89
• 90
. 91
• 92
0 93
• 94
0
95
. 96
. 97
0 98
0 99
0
100 •
8
Lode.
s
Suspe-nd
8
End Bloc:k
Item: as of ~ ,• Mark <::J [:::> ""I ~· ~'I
100 J.. Previous Next LAb faiUI~S Notes Calculator

74
75 The correct ans wer is C. 62°/o chos e this.
76 Hepatitis E is a RNA hepevirus (recent ly reclassif ied from being a Calicivirus) that resembles hepatit is A in its disease course, wit h nausea,
77 vomiting, fever, anorexia, and j aundice. The incubat ion peri od ranges from 15-60 days. The maj or dist inguishing fa ctor of hepatitis E is t hat it
78
causes a severe, often-fatal disease in pregnant women . This is known as fulm inant hepatic fai lure, and mort alit y reaches 20%. Hepatit is A is
an RNA picornavirus that has a course similar to that of hepatit is E. However, it does not have selectively high mortalit y in pregnant women.
79 Picornavirus Jaundice Hepatitis E Hepatitis A Caliciviridae Hepatitis Anorexia (symptom) Incubation period Nausea RNA Liver Vomiting Acute liver failure Fulminant Liver failure Fever Anorexia nervosa

80
A is not correct . 21% chos e this .
81
Hepatitis C is an RNA flavivirus t hat is t ransmitted primarily through t he blood and resem bles hepat it is B in course and severi ty. It is a chronic
82 infection .
83 Flavivirus Hepatitis C Hepatitis B Hepatitis RNA Infection

84 B is not corre ct. 12% chose this .


85 Hepatitis B is a DNA hepadnavirus that does not have enhanced t ropism for pregnant women, although it is important to know if a pregnant
. 86 woman has this infect ion, as it can be t ransmitted vert ically if proper medical t reat ment is not given perinatally.
Hepadnaviridae Hepatitis B Tropism Vertically transmitted infection Hepatitis Pregnancy DNA
• 87
• 88 D is not correct. 5°/o chos e this .
• 89 Hepatitis D is a defect ive RNA deltavirus t hat req uires hepat it is B to serve as an envelope and cannot be t ra nsmit ted wit hout the presence of
t he hepatitis B virus.
• 90
Hepatitis 8 Hepatitis Hepatitis 8 virus Virus Hepatitis 0 RNA
. 91
• 92
• 93
Bottom Line:
• 94 Hepatitis E is a RNA hepevirus t hat is spread by t he fecal-oral route, is most commonly water-borne, and is associat ed with high mortality in
pregnant women .
• 95 Fecal- oral route Hepatitis E Hepatitis RNA
. 96
. 97
• 98 l@);fil ~1hl for year:l 2o17 y
FIRST AID FACTS
• 99
• 100

8
Lock
s
Suspend
0
End Block
Item: 85 of - ,• Mark -<J [:::> "'I ~ · ~
100 ~ P~v1ous N@xt Labl lu es No tes Calcula t o r

74 A A
FA17 p 168.1
75
76 Hepatitis Signs and symptoms of all hepatitis viruses: episodes of fever, jaundice, t ALT and AST. aked viruses (I-L\V
77 viruses and I lEV) lack an envelope and arc not destroyed by the gut: the ' owch hit your bo" cls.
78 HBV DNA polymerase has D1 A- and RNA-dependent acti,·ities. Upon entry into nucleus, the polymerase
79 completes the partial dsDI'\A. Host RN poly mcrase transcribes mR 1A from ,·ira I DNA to make viral
80 proteins. The D1 A polymerase then re,·erse transcribes viral RNA to Of\A, which is the genome of the
81 progeny VIrus.
82 HCV lacks 3'-5' exonuclease activity - no proofreading ability - ,·ariation in antigenic structures of HCV
83
envelope proteins. Host antibody production lags behind production of new mutant strains of HC .
84
Virus HAV HBV HCV HDV HEV
85
. 86
FAMILY RNA picornavirus D 1A hcpadnavirus Ri':A Aavivi rus Ri A delta\'irus R1 A hcpcvi rus
0 87 TRANSMISSION Feca l-oral (shellfish, Parenteral (Blood), Primarily blood Parenteral, sexual, Fecal-oral,
0 88 travelers, day care) sexual (Baby- (IVDU, post- perinatal especially
• 89 making), perinatal Ira nsfusion) waterborne
• 90 (Birthing)
. 91
INCUBATION Short (weeks) Long (months) Long Superinfection Short
• 92
0 93
(llDV after
• 94
II BV) =short
0
95
Coinfection (HDV
. 96 with HB ) = long
. 97 CLINICAL COURSE \ symptomatic Initially like serum lay progress to Similar to HBV Fulminant hepatitis
0 98 (usually), Acute sickness (fever, Cirrhosis or in Expectant
99 Carcinoma
0
arthralgias, rash); (pregnant) women
0
100 • m'lP n.rn.nr~c-c- tn

8
Lode.
s
Suspe-nd
8
End Bloc:k
Item: 85 of - ,• Mark -<J [:::> "'I ~ · ~
100 ~ P~v1ous N @xt Labl lues N o tes Calcula to r
74 •
J JBV) = short
75
Coinfection (HDV
76
with HB ) = long
77
78 WNICAL COURSE \ symptomatic Initially like semm lay progress to Similar to HBV Fulminant hepatitis
79 (usually}, Acute sickness (fever, Cirrhosis or in Expectant
80 arthralgias, rash); Carcinoma (pregnant) \\ Omen
81 may progress to
82 carcmoma
83
PROGNOSIS Good dults ..... mostly lajority de\'elop Superinfection High mortality in
84
full resolution; stable, C hronic ..... worse prognos1s pregnant women
85
neonates ..... worse hepatitis C
. 86
0 87
prognoSIS
0 88 HCC RISK No Yes Yes Yes No
• 89 LIVER BIOPSY Lymphoid
Hepatocyte Granular Similar to HBV Patchy necrosis
• 90
swelling, eosinophilic aggregates with
. 91
monocyte "ground glass" focal areas of
• 92
in fi II ration, appea ranee; macrovcsicula r
0 93
Councilman cytotoxic T cells steatosis
• 94
bodies mediate damage
0
95
. 96 NOTES o carrier state Carrier state Carrier state ,·ery Defective ,·irus, Enteric, Epidemic,
. 97 ("Alone") common common Depends on HBV no carrier state
0 98 HBsAg
0 99
0
100 •
8
Lode.
s
Suspe-nd
8
End Bloc:k
Item: 86 of - ,• Mark -<J [:::> "'I ~ · ~
100 ~ P~v1ous N @xt Labl lues N o t es Calcula to r
74
A 23-year-old man present s to t he emergency department complaining of bloody diarrhea, abdominal pain, and fever. Laboratory tests of ~~AI
75
blood and stool show an oxi dase- negative, motile, gram-negat ive bacillus that grows as clear colonies on MacConkey agar at 37°C. When
76
plated on Hektoen enteric agar, the colonies appear light green with black centers.
77
78 Which of the following is the most likely causative organism?
79
:
80 A. Salmonella enterica enteritidis
81
B. Campy/obacter jejuni
82
83 C. Escherichia coli
84 0 . Shigella dysenteriae
85
E. Yersinia enterocolitica
. 86
0 87
0 88
• 89
• 90
. 91
• 92
0 93
• 94
0
95
. 96
. 97
0 98
0 99
0
100 •
8
Lode.
s
Suspe-nd
8
End Bloc:k
Item: 86 of - ,• Mark -<J [:::> "'I ~ · ~
100 ~ P~v1ous N @xt Labl lues N o t es Calcula to r
74 A A

75 The correct ans wer is A. 52°/o chose this.


76 Bloody diarrhea can have a number of bacteria l causes, including enterohemorrhag ic Escherichia coli
(0157:H7), Salmonella, Shigella, Campylobacter, and Yersinia enterocolitica. Both Salmonella and Shigella
77
species are gram- negative rods t hat do not ferment lactose and are oxidase-negative. In culture,
78 Salmonella are flagellated and mot ile; whereas Shigella propel themselves solely intracellularly via actin
79 polymerization and are nonmotile in cult ure. Nonlactose-fermenting bacteria appear clear or white on
MacConkey agar. Hektoen enteric agar is useful for differentiating between Salmonella and Shigella because
80
only Salmonella produces hydrogen sulfide. Hydrogen sulfide forms a black precipiate when it comes into
81 contact with thiosulfate in Hektoen agar, as shown in the image. However, Shigella does not produce
82 hydrogen sulfide, and therefore does not create a precipiate on Hektoen agar. Thus the colonies remain
green without a black center.
83
Yercm•a enterocollt H "rtoen entenc agar Hydrogen sulfide Escherichia col. Gram-negatJve bactena Sh•geja Mac:Cohkey agar Actm Carnpytobacter- Lactose
84 Otarrhea Salmonella Thoosulfate flagellum Bactena Motility Agar Yersonoa Polymenzatoon f'ermentatoon Hydrogen Gastroontesbnal tract
85
fermentation en food processmg
86
0 87 Image courtesy of Wikimedia
0 88 Commons
• 89
B is not correct . 7 °/o chose this .
• 90
Campylobacter species are curved, gram -negat ive rods which can cause bloody diarrhea. Campylobacter jejuni is also non-lactose fermenting
. 91 and would not grow on MacConkey agar. It only grows on special media (Skin·ow or Campy agar) cultured under microaerophilic conditions .
Campylobacter jejune Campylobacter Microaerophile Gram-negative bacteria MacConkey agar Diarrhea Fermentation Agar Fermentation 1n food processing Species Lactose intolerance
• 92
0 93 C is not correct. 16% chose this.
• 94 Certain strains of Escherichia coli (0157: H7) can cause bloody diarrhea . However, E. coli does fe rment lactose, which would result in the
0
95
growth of pink colonies on MacConkey agar. Of note, fever would not be characteristic of infect ion wit h E. coli 0157:H7.
Escherichia cole MacConkey agar Diarrhea Lactose Agar Fever Fermentation Fermentat1on 1n food proc~s1ng Infect1on
. 96
. 97 D is not correct. 18°/o chose this .
0 98
Shigella dysenteriae is a nonmotile, gram-negative bact eria. S. dysenteriae produces Shiga toxin, which inhibits prot ein synthesis in host
cells, which leads to bloody diarrhea. Unlike Salmonella species, Shigella is not sulfur producing and t herefore the does not form a black
0 99 percipitate on Hektoen agar, distinguishing Shigella from Salmonella.
0
100 Stugella d~f"fttet 1M ~ ga tcx 1n Gr-am-negative bacteria Shigella Diarrhea Salmonella Prote1n Proten "Y"'thes s e.cter11 Mobhty To :.an SUlfur Ragellum Agar

8
Lode.
s
Suspe-nd
8
End Bloc:k
Item: 86 of ~ ,• Mark <::J [:::> ""I ~· ~'j
100 J.. Previous Next LAbfaiUI~S Notes Calculator

74
E is not correct. 7 °/o chos e this .
75
I nfection wit h Yersinia enterocolitica occurs most commonly in young children and presents with feve r, abdomina l pain, and diarrhea (often
76
bloody) . Older child ren wit h the infection can present wit h right -sided abdominal pain and fever. Y. enterocolitica is an oxidase- negative, non-
77 lactose fe rment ing, gram- negative coccobacillus. However, alt hough it is motile at 25° C, it is nonmot ile at 37°C.
Yersinia enterocolitica Coccobacillus Gram-negative bacteria Diarrhea Motility Yersinia Abdominal pain I nfection Fever Flagellum Lactose intolerance
78
79
80 Bottom Line :
81 Common causes of bloody diarrhea include Salmonella, Shigella, E. coli, Campylobacte1~ and Yersinia . Of t hese, Salmonella is the on ly
82 prod ucer of hyd rogen sulf ide, which can be det ected on Hektoen agar cult ure.
Hydrogen sulfide Shigella Escherichia coli Campylobacter Diarrhea Salmonella Yersinia Agar Dysentery Hydrogen
83
84
85
86
l@ljl'il·1i•J for yea r:[2017 • J
FIRST AID FA CTS

• 87
• 88 FA17 p 141.1
• 89
Salmonella vs Shigella Both Salmonella and Shigella are gram 8 rods, non-lactose fermenters, oxidase 8, and can invade
• 90
. 91
the Gl tract via M cells of Peyer patches.
• 92 Salmonella typhi Salmonella spp. Shigella
• 93 (except S typhi)
• 94 RESERVOIRS Humans only Humans and animals Humans only
• 95
SPREAD Can dissem inate Can dissem inate Cell to cell; no hematogenous spread
. 96
hematogenously hematogenous!)'
. 97
• 98 H1SPRODUCTION Yes Yes 'o
• 99 FLAGELLA Yes (salmon swim) Yes (salmon swim) No
• 100 utnl u cure. CAr Tnnc 1: , .
1"':' 1 ' T:' 1 ,..., . I . \

8
Lock
s
Suspend
0
End Block
Item: 86 of - ,• Mark --<) [::> ""'I ~· 1!';:'1
100 ~ Prev1o u s Next Labf a lu es Note s Calculator
74
75
FA17 p 140.1
76
77 Escherichia coli Cram 8 rod. E coli virulence factors: fimbriae-cystitis and pyelonephritis (P-pili); K capsule-
78 pneumonia, neonatal meningitis; LPS endotoxin-septic shock.
79
STRAIN TOXIN AND MECHANISM PRESENTATION
80
81 EIEC Microbe invades intestinal mucosa and causes Invasive; dysenter). Clinical manifestations
82 necrosis and inAammation. similar to Shigella.
83 ETEC Produces heat-labile and heat-stable Tm ·clcrs' diarrhea (watery).
84 enteroToxins. No inAammation or invasion.
85
EPEC 1o toxin produced. Adheres to apical surface, Diarrhea, usually in children (Pediatrics).
86
• 87
Aattens \'illi, pre,·ents absorption .
• 88 EHEC Ol57:H7 is most common serotype in US. Often Dysentery (toxin alone causes necrosis and
• 89 transmilled via undercooked meat, raw leafy inAammat ion).
• 90 vegetables. Does not ferment sorbitol (vs other E coli).
. 91 Shiga-like tox in causes hemolytic-uremic II emorrhagic, Hamburgers, llemolyt ic-urcm ic
• 92 syndrome: triad of anemia, thrombocytopenia, syndrome.
• 93 and acute renal fa ilure clue to microthrombi
• 94
formi ng on damaged endothelium
• 95
..... mechanical hemolysis (with schistocytcs on
. 96
peripheral blood smear), platelet consumption,
. 97
and l renal blood Aow.
• 98
• 99
• 100 FA17 p 175.1 •
8
L.odt
s
Su~pl'nd
~
End Block
Item: 86 of ~ ,• Mark <::J [:::> ""I ~· ~'j
100 J.. Previous Next LAbfaiUI~S Notes Calculator

74
75 FA17 p 175.1
76 Bugs causing diarrhea
77 Bloody diarrhea
78
Campylobacter Comma- or S-shaped organisms; growth at 42°C
79
80 E histolytica Protozoan; amebic dysentery; liver abscess
81 Enterohemorrhagic 0157:H7; can cause IIUS; makes Shiga-like toxin
82
Ecoli
83
84
Enteroinvasive Ecoli Invades colonic mucosa
85 Salmonella (non- Lactose 8; Aagellar motility; has animal reservoir, especially poultry and eggs
86 typhoidal)
• 87
Shigella Lactose 8; very low 10 ;0; produces Shiga toxin (human reservoir only); bacillary dysentery
• 88
• 89 Yenterocolitica Day care outbreaks, pseudoappendicitis
• 90 Watery diarrhea
. 91
Cdifficile Pseudomembranous colitis; associated with antibiotics and PPis; occasionally bloody diarrhea
• 92
• 93 C perfringens Also causes gas gangrene
• 94 Enterotoxigenic E coli Tra,·elers' diarrhea; produces heat-labile (LT) and heat-stable (ST) toxi ns
• 95
Protozoa Giardia, Cryptosporidium
. 96
. 97 Vcholerae Comma-shaped organisms; rice-water diarrhea; often from infected seafood
• 98 Viruses Rota,·irus, norovirus, adenovirus
• 99
• 100

8
Lock
s
Suspend
0
End Block
Item: 87 of - ,• Mark --<) [::> ""'I ~· 1!';:'1
100 ~ Prev1o u s Next Labf a lu es Notes Calculator
74
75
A 2-day-old girl begins vomit ing, does not feed, and develops a fever of 39° C (102.2°F) . Lumbar puncture shows an extremely elevated
WBC count (with an especially high level of polymorphonuclear leukocytes), an extremely elevated protein level, and a reduced glucose
76 level .
77
78 I nfection with which of the following organisms is t he most likely cause of her symptoms?
79
:
80 A. Coxsackie virus
81
B. Group B Streptococcus
82
83 C. Haemophilus influenzae
84 0 . Herpes virus
85
E. Streptococcus pneumoniae
86
• 87
• 88
• 89

• 90
. 91
• 92
• 93
• 94
• 95
. 96
. 97
• 98
• 99
• 100 •

8
L.odt
s
Su~pl'nd
~
End Block
Item: 87 of ~ ,• Mark <::J [:::> ""I ~· ~'j
100 J.. Previous Next LAb faiUI~S Notes Calculator

74
75 The correct answer is B. 75% chos e this.
76 The baby's sympt oms and lumbar punct ure (LP) results suggest t hat she has acut e bact erial mening it is. The most common cause of neonat al
77 bact eri al mening it is is infect ion with Group B Streptococcus. Twenty -five percent of women's vaginal canals are colonized with t hese bact eria,
and t he baby can be exposed t o t hem duri ng delivery. Ant ibiotics should be started immed iat ely (even before LP) when mening it is is
78
suspect ed in neonat es.
79 Lumbar puncture Meningitis Bacterial meningitis Antibiotics Streptococcus Lumbar Bacteria Infant Infection Lumbar vertebrae

80
A is not correct . 3°/o chos e this.
81 Coxsackie virus causes asept ic (viral) rather than bact eri al mening it is. The LP f rom a pat ient with v iral meningit is shows a mild increase in
82 WBC count (predominant ly lymphocytes), a m ild increase in protein, and normal glucose.
Meningitis Viral meningitis Coxsackievirus Glucose Protein Virus
83
84 C is not correct. 10% chos e this .
85 The LP f rom a pat ient with Haemophilus influenzae infection would be sim ilar to t his pat ient's find ings. However, H. influenzae men ingit is is
86
not often seen in neonat es.
Haemophilus influenzae Meningitis Infant Haemophilus Infection
87
• 88 D is not correct. 2°/o chose this .
• 89
Herpes virus causes aseptic (viral) rat her t han bacteri al mening it is. The LP f rom a pat ient wit h v iral meningit is shows a mild increase in WBC
count (predominant ly lymphocyt es), a m ild increase in protein, and normal glucose .
• 90 Meningitis Viral meningitis Herpes simplex Herpes simplex virus Bacterial meningitis Protein Glucose Herpesviridae Virus Asepsis
. 91
E is not correct. 9 °/o chos e this .
• 92
Streptococcus pneumoniae is t he most common cause of bact eri al mening it is in adults. The LP from a patient wit h t his infect ion would be
• 93 similar to the patient described above. However, Streptococcus pneumoniae is not often seen in neonat es.
• 94 Streptococcus pneumoniae Meningitis Bacterial meningitis Streptococcus Infant

• 95
. 96
Bottom Line:
. 97
Group B streptococci are t he most common cause of meningit is in ch ildren <6 mont hs old. Pregnant women are screened for GBS, and GBS-
• 98 posit ive mot hers are t reat ed prophylactically with ant ibiot ics at t he t ime of delivery in order to prevent neonatal infect ion.
• 99 Neonatal infection Meningitis Streptococcus agalactiae Streptococcus Antibiotics Infant Preventive healthcare Infection

• 100 ~

8
Lock
s
Suspend
0
End Block
Item: 87 of ~ ,• Mark <::J [:::> ""I ~· ~'j
100 J.. Previous Next LAbfaiUI~S Notes Calculator

74
75 141;fil·1i•J
FIRST AID FACTS
for year:[2017
.
•j .
76
77 FA17 p 176.1
78
Common causes of meningitis
79
NEWBORN (0- 6MOl CHILDREN (6 M0- 6 YR) 6- 60YR 60YR+
80
81
Group B streptococci S pneumoniae S pneumoniae S pneumoniae
82
E coli N meningitidis N meningiticlis (#1 in teens) Gram 8 rods
83
Listeria H infiuenzae type B Enterovi ruses Listeria
84 Enteroviruses HSV
85 Give ceftriaxone and vancomycin empirically (add ampicillin if Listeria is suspected).
86 Viral causes of meningitis: enteroviruses (especially coxsackievirus), llSV-2 {llSV-1 = encephalitis), IIIV, West ile virus (also
87 causes encephalitis), VZV.
• 88
In HIV: Cryptococcus spp.
• 89
Note: Incidence of H infiuenzae meningitis has l greatly due to conjugate H infiuenzae vaccinations. Today, cases are usually
• 90
seen in unimmunized children .
. 91
• 92
• 93 FA17 p 176.2
• 94 CSF findings in meningitis
• 95 OPENING PRESSURE CELL TYPE PROTEIN GLUCOSE
. 96
Bacterial t t PM s t l
. 97
• 98 Fungai/TB t t lymphocytes t l
• 99 Viral Normal/t t lymphocytes Normal/t ormal
. 100 ~

8
Lock
s
Suspend
0
End Block
Item: 87 of - ,• Mark -<J [:::> "'I ~ · ~
~ P~v1ous
100
. . . N@xt
.. .. Labl lu es
;
No tes
;. . ..
Calcula t o r
;. . . . ..
75 seen in unimmunizcd children.
76
77
FA17 p 176.2
78
CSF findings in meningitis
79
OPENING PRESSURE CELL TYPE PROTEIN GLUCOSE
80
81 Bacterial f f P.\ INs f
82 Fungai/TB f f I) mphoc) tes f
83
84
Viral ormal/f f I) mphoc) tes orma1/f ormal
85
86 FA17 p 133.1
87
0 88
Streptococcus Cram ® cocci, bacitracin resistant, ~-hemolyti c, C roup B for Babies!
• 89
aga/actiae (group B colonizes ,·agina; causes pneumonia,
• 90
streptococci) meningitis, and sepsis, mainly in babies .
. 91 Produces CAMP factor, wh ich enlarges the
• 92 area of hemolysis formed by S (/I/ reus. ( otc:
0 93 CAMP stands for the authors of t·he test·, not
• 94 cycl ic A IP.) Hippurate test ®. PYR 8 .
0
95 Screen pregnant women at 35-37 weeks of
. 96 gestation wit·h rectal +vaginal swabs. Patients
. 97 with ® culture receive intrapartum penicillin
98
0
prophylaxis.
0 99
0
100 •
8
Lode.
s
Suspe-nd
8
End Bloc:k
Item: 88 of - ,• Mark -<J [:::> "'I ~ · ~
100 ~ P~v1ous N @xt Labl lues N o t es Calcula to r
74
A 24-year-old man present s to t he physician with profuse bloody diarrhea and abdominal cramps. On questi oning, it is learned t hat the ~~AI
75
pat ient went swimming in a lake during a cam ping t ri p 2 weeks earlier. A stool sample is sent fo r laborat ory evaluation.
76
77
Which of the following is most likely t o be seen on microscopic examination of this patient's stool?
78
:
79
A. Acid -fast staining cysts
80
81
B. Macrophages containing amastigotes
82 C. RBCs containing schizonts
83
D . Tear-shaped trophozoites with two nuclei
84
E. Trophozoites with ingested RBCs
85
86
87
' 88
• 89
• 90
. 91
• 92

' 93
• 94

' 95
. 96
. 97

' 98
' 99
' 100 •
8
Lode.
s
Suspe-nd
8
End Bloc:k
Item: 88 of ~ ,• Mark <::J [:::> ""I ~· ~'j
100 J.. Previous Next LAb faiUI~S Notes Calculator

74
75
76
The correct a nswer is E. 4 6°/o chose this .
77 This patient is infected wit h Entamoeba histolytica, which is t ransmit ted v ia cysts in water (fecal-oral t ransmission) . Infect ion manifests with
78 bloody diarrhea (dysent ery ), abdominal cramps wit h tenesmus, and pus in the stool. It can also cause right upper quad rant pain and liver
79
abscesses. On m icroscopy, one observes t rophozoites (t he amoeba) with ingest ed RBCs. Treat ment fo r E. histolytica infection includes
metron idazole followed by paromomycin, which t reats int ralumina l cysts specif ically.
80 Metronidazole Entamoeba histolytica Paromomycin Rectal tenesmus Dysentery Diarrhea Fecal-oral route Amoeba Quadrant (abdomen) Liver Lumen (anato,.,) Pus Microbial cyst Trophozoites Entamoeba
81 Cyst Abscess Apicomplexan life cycle Infection Feces Microscopy
82
A is not correct . 10% chose this.
83
Acid-fast sta ining cysts are found in several parasit ic infections, most not ably t he Cryptosporidium species and Toxoplasma gondii.
84 Cryptosporidium infection man ifests wit h severe, nonbloody diarrhea in HIV-posit ive patient s and wit h m ild watery diarrhea in HI V-negative
85 patients. It is t ransmit ted v ia cysts in water (fecal-oral t ransmission). Treat ment is supportive, although in healt hy patients cryptospori diosis is
86
usua lly self -resolv ing . T. gondii infect ion manifests with brain abscesses in HIV-posit ive pat ients and wit h bi1th defects if infection occurs
duri ng pregnancy (toxoplasmosis is one of the ToRCHeS organisms). T. gondii is t ransmitted v ia cysts in raw meat or cat feces. The definit ive
87 stage (sexual stage) occurs in cats. Sulfadiazine and py rimethamine are used to t reat t oxoplasmosis.
88 Toxoplasma gondii Toxoplasmosis Pyrimethamine Cryptosporidiosis Cryptosporidium Diarrhea Fecal-oral route Acid-fast Parasitism Feces Sulfadiazine Microbial cyst Cyst Cat HIV Congenital disorder Brain

• 89 Pregnancy Species Infection Abscess

• 90 B is not correct. 9 % chose t his .


. 91 Macrophages containing amastigotes are typical of infect ion by Leishmania donovani, which is t ransmitted by the sandfly. Pat ients present
• 92 with hepatosplenomega ly, malaise, anem ia, and weight loss. Sod ium stibogluconat e is used to t reat L. donovani infect ion .
Leishmania donovani Leishmania Hepatosplenomegaly Sodium stibogluconate Anemia Macrophage Sandfly Amastigote L. donovani Infection
• 93
• 94 C is not correct. 9°/o chose this .
• 95 RBCs cont aining sch izonts are charact eri st ic of infection by Plasmodium species, which cause malaria. The schizont form represent s a lat e
stage in t he development of the asexua l form of t he parasit e. Malari a is t ransmit ted by the fema le Anopheles mosquit o. I nfection presents
. 96
with anemia, fever, chills, and flu -like symptoms; t he t ime course and pattern of t hese sympt oms depend on the Plasmodium species wit h
. 97 which t he pat ient is infect ed. Treat ment is tailored t o t he geographic area of infect ion and the Plasmodium species involved; agents include
• 98 ch loroquine, hydroxychloroquine, and atovaquone-proguanil.
Schizont Chloroquine Hydroxychloroquine Malaria Plasmodium Mosquito Anopheles Anemia Parasitism Fever Infection Asexual reproduction Influenza-like illness
• 99
• 100 D is not correct. 26% c hose this .
8
Lock
s
Suspend
0
End Block
Item: 88 of ~ ,• Mark <::J [:::> ""I ~· ~'j
100 J.. Previous Next LAbfaiUI~S Notes Calculator

74
75 D is not correct. 26% chose this .
76
Tear-shaped t rophozoites with two nucl ei are charact erist ic of Giardia Iamblia infection. It presents wit h bloating, f latulence, fo ul-smelling but
nonbloody diarrhea, and light -colored fatty st ools. G. Iamblia is t ransm itted via cyst s in wat er (fecal-oral t ransm ission). Metronidazole is used
77 t o t reat giardiasis.
78 Giardia Iambiia Metronidazole Giardiasis Fecal-oral route Flatulence Diarrhea Giardia Trophozoites Bloating Cell nucleus Infection Microbial cyst Cyst

79
80
Bottom Line:
81
Bloody diarrhea aft er exposure t o unfilt ered f resh water is suggest ive of infect ion by Entamoeba histolytica, in which t rophozoit es wit h
82 ingested RBCs are seen under t he microscope.
83 Entamoeba histolytica Diarrhea Entamoeba Trophozoites Microscope Apicomplexan life cycle Dysentery Infection

84
85
86 l@ljl'il·1i•J for year:[2017 • J
FIRST AID FA CTS

87
88 FA17 p 151 .1
• 89
• 90
Protozoa- gastrointestinal infections
. 91 ORGAN ISM DISEASE TRANSMISSION DIAGNOSIS TREATMENT
• 92 Giardia Iamblia Giardiasis- bloating, flatulence, C ysts in water Multinucleated Metron idazole
• 93 foul-smelling, fatly diarrhea trophozoites t'J or
• 94 (often seen in campers/h ikers)- cysts I]) in stooI,
• 95 lhink fat-rich G hirardelli anligen delection
. 96 chocolates for fatty stools of
. 97
Giardia
• 98
Entamoeba Amebiasis- bloody diarrhea C ysts in water Serology and/or Metron idazole;
• 99
• 100
histolytica (dysentery), liver abscess trophozoites (with paromomycm or

8
Lock
s
Suspend
0
End Block
Item: 88 of - ,• Mark -<] 1:> ""'I ~· 1!';:'1
100 ~ Prev1o u s Next Labf a lu es Note s Calculator
74 • •
Giardia
75
76
Entamoeba Amebiasis-bloody diarrhea Cysts in water Serology and/or Metronidazole;
77
histolytica (d}'sentery), li,·er abscess trophozoitcs (with paromomycm or
78 ("ancho''Y paste" exudate), engulfed RBCs Bl iodoquinol for
79 RUQ pain; histology shows in the cytoplasm) asymptomatic C) st
80 Aask-shaped ulcer or cysts with up to passers
81 4 nuclei in stool • ;
82 Entamoeba Eats
83 l .r) throcytes,
84 antigen detection
85
86
Cryptosporidium Se,·ere diarrhea in AIDS Oocnts
, in water Oocysts on acid-fast Pre,·ention (by
87
Mild disease (watery diarrhea) in stain 0 , antigen filtering city
88
• 89
immunocompetent hosts detection water supplies);
• 90
nitazoxanide in
. 91 immunocompetent
• 92 hosts
• 93
fJ rn
• 94
• 95
. 96
. 97
• 98
• 99
• 100 D D L---~,.,.- D •
8
L.odt
s
Su~pl'nd
~
End Block
Item: 89 of - ,• Mark -<J [:::> "'I ~ · ~
100 ~ P~v1ous N @xt Labl lues N o tes Calcula to r
74
A 30-year-old woman comes to th e emergency department because of a fever of 38.4°C (101.1°F) and a diff use macular rash on her ~~AI
75
torso. Her blood pressure is 91/51 mm Hg. She reports she was seen a few days prior for a "nosebleed t hat would not stop" and given
76
nasal packing, which is st ill in place. She is admitted to the hospital for further treatment. Over the next few days, the rash on her torso
77 desquamates.
78
79 Which mechanism most likely produced this constellation of symptoms?
80 :
81 A. Binding of antibodies that recognize this toxin to a similar epitope on normal tissue
82 B. Cleavage of host cell rRNA to inhibit protein synthesis
83
C. Cross-linking of IgE on presensitized mast cells and basophils, triggering release of their contents
84
85 D. Disruption of cellular membranes by a lecithinase
86 E. Nonspecific cross-linking of MHC with T-cell receptors
87
88
• 89
• 90
. 91
• 92
• 93
• 94
• 95
. 96
. 97
• 98
• 99
. 100 •
8
Lode.
s
Suspe-nd
8
End Bloc:k
Item: 89 of - ,• Mark -<J [:::> "'I ~ · ~
100 ~ P~v1ous N @xt Labl lues N o tes Calcula to r
74 A A

75
The correct answer is E. 4 9°/o chose thi s.
76
Staphylococcus aureus is a gram-posit ive, catalase-positive coccus. It produces a superantigen called toxic
77
shock syndrome toxin (TSST-1 ). TSST- 1 cross-links class II molecules on antigen-presenting cells toT-cell
78 receptors on T cells independent of antigen. This results in T-cell proliferation, which can induce a cytokine
79 storm. The presence of these cyt okines gives rise t o t he symptoms: fever, hypotension, and a diffuse
macular rash that desquamates after a few days ( like that shown in the image). Retained foreign bodies,
80 such as nasal packing that has stayed in place for too long, can form a nidus for infection with s. aureus.
81 Another classic association with TSST- 1 is a tampon left in place for a prolonged period of time.
Super ..-ge"'\ C •o1 1e ..tor• C ltol.ne To ~c shock syndrome Stapllyiococcus au reus Gr~po ~b re ~llctt' •I ~· tt ._;otens•on Staph 1ococcus T cell
82
Antigen Tox.n T cell receptor Anbgen-presenbng cell Coccus Fever Infection Rash
83
84
85
86
87
88
89
• 90
. 91
• 92 Image copyright © 2007
• 93 Raghavendra eta/; licensee
• 94
BioMed Central Ltd.
• 95
A i s not correct. 20°/o chose thi s.
. 96 This is an example of a type II hypersensit ivity disorder such as rheumato id heart disease. However, it is not the mechanism for toxic shock
. 97 syndrome toxin-1. A type II reaction is cytotoxic (antibody-mediated) where IgM and/or IgG bind to a fixed ant igen leading to an immune
• 98
response against that cell by way of opsonizat ion, complement-mediated lysis, and antibody-dependent cell -mediated cytotoxicity.
Anbbody·dep~d~t ceU~med1ated cytotoxicity Toxic shock syndrome Opsonin Immunoglobulin ~ Type II hyperseo .,,t1v1ty Cytotox1c1ty Immunoglobulin G Immune system Hypersens1bvtty Antigen Lyses
• 99
ToJOcrty CardtOVB$cular d1sease
. 100

8
Lode.
s
Suspe-nd
8
End Bloc:k
Item: 89 of ~ ,• Mark <::J [:::> ""I ~· ~'j
100 J.. Previous Next LAb faiUI~S Notes Calculator

74
B is not correct. 6 % chose this.
75
This is t he mechanism fo r Shiga t oxin, which is not associat ed with toxic shock syndrome. The toxin is int ernalized aft er binding t o its
76
receptor, glycolipid Gb3, wit h t he B subunit. The A subunit ca rries t he fragme nt that acts as an enzyme t o cleave an adenine residue from 285
77 rRNA in the 60S ri bosoma l subunit, disrupting protein synt hesis and leading to apoptosis.
Glycolipid Toxic shock syndrome Apoptosis Enzyme Adenine Ribosome Protein Ribosomal RNA Protein synthesis Toxin 285 ribosomal RNA Eukaryotic large ribosomal subunit (60S) Protein biosynthesis
78
79 Receptor (biochemistry) Toxicity Amino acid

80 C is not correct. 13% chos e this.


81 This describes type I hypersensit ivity, which is not associated wit h any exotoxin re leased from Staphylococcus aureus. A t ype I reaction
82 involves free ant igen cross- linking of IgE on mast cells and basophils, which t riggers t he release of vasoactive a m ines such as hist amine in
anaphylact ic (eg, bee st ings) and at opic (eg, eczema) react ions.
83 Type I hypersensitivity Dermatitis Staphylococcus aureus Histamine Exotoxin Immunoglobulin E Hypersensitivity Mast cell Antigen Atopic dermatitis Atopy Anaphylaxis Staphylococcus Bee Vasoactive
84
D is not correct. 12% chose this .
85
This describes a-t oxin, produced by Clostridium perfringens, t he organ ism responsible for t he development of gas gangrene. This organism is
86 associat ed with cellulit is, myonecrosis, and diarrh ea but not wit h t oxic shock syndrome .
87 Toxic shock syndrome Clostridium perfringens Gas gangrene Gangrene Cellulitis Diarrhea Clostridium Organism

88
89
Bottom Line:
• 90
Staphylococcus aureus produces a superant igen TSST-1, which act ivat es a large population of T lymphocyt es by cross- linking MHC and T-cell
. 91 receptors, result ing in a t riad of feve r, shock, and desquamating rash .
• 92 Superantigen Staphylococcus aureus Staphylococcus Desquamation Lymphocyte T cell Toxic shock syndrome toxin T cell receptor Major histocompatibility complex Fever

• 93
• 94
• 95 141ifil·1i•J for year:[2017 • J
FIRST AID FACTS
. 96
. 97
FA17 p 131 .3
• 98
• 99
Staphylococcus aureus Cram EB, P-hemolytic, catalase ®, coagulase TSST-1 is a superantigen that binds to MHC
• 100
E!7 cocci in clusters rJ. Protein A (virulence II and T-cell receptor, resulting in polyclonal
8
Lock
s
Suspend
0
End Block
Item: 89 of - ,• Mark -<] 1:> ""'I ~· 1!';:'1
100 ~ Prev1o u s Next Labf a lu es Note s Calculator
74
75 141 : fi1~11ai for year: 201 7 •
fiRST AIO fACTS
76
77
FA17 p 131 .3
78
79 Staphylococcus au reus Gram (f), ~-hemolytic, catalase (f), coagulase TS T-1 is a superantigen that binds to MHC
80
81
82
83
- (f) cocci in clusters D. Protein (\ irulcnce
factor) binds Fc-lgG, inhibiting complement
acti,·ation and phagocytosis. Commonly
colonizes the narcs, axilla, and groin.
II and T-cell receptor, resulting in polyclonal
rr:cell act i\'ation.
taphylococcal toxic shock syndrome
(TS ) presents as fever, \'Omiting, rash,
84 Causes: desquamation, shock, end-organ failure. TSS
85
.,., ... . Inflammatory disease-skin infections, resulls in t AST, t LT, t bilirubin. ssociatcd
86
87 Jill"\ organ abscesses, pneumonia (often after
influenza virus infection}, endocarditis,
with prolonged use of ,·aginal tampons or nasal
packing.
88
septic arthritis, and osteomyelit is. Compare with Streptococcus pyogenes TSS (a
89
Toxin-mediated disease-toxic shock toxic shock- like syndrome associated with
• 90
. 91
syndrome (TSS1 ~l }, scalded skin syndrome painful skin infection).
• 92
(exfoliative toxin), rapid-onset food S aureus food poisoning due to ingestion of
• 93
poisoning (cntcrotoxins). preformed toxin _. short incubation period
• 94 MRSA (methicillin-resistantS aureus} (2-6 hr) followed by nonbloody diarrhea
• 95 infection- important cause of serious and emesis. Enterotoxin is heat stable - not
. 96 nosocomial and community-acquired destroyed by cooking.
. 97 infections; resistant· to methicillin and Bad staph (aureus) make coagulase and toxins .
• 98 nafcillin because of altered penicillin- Forms fibrin clot around self_. abscess .
• 99 binding protein .
• 100 •
8
L.odt
s
Su~pl'nd
~
End Block
Item: 89 of - ,• Mark -<] 1:> ""'I ~· 1!';:'1
100 ~ Prev1o u s Next Labf a lu es Note s Calculator
74 • •
FA17 p 128.1
75
76 Bugs with exotoxins
77 BACTERIA TOXIN MECHANISM MANIFESTATION
78 Inhibit protein synthesis
79
Corynebacterium Diphtheria toxina Phal) ngitis '' ith pseudomembranes in throat
80
diphtheriae Inactivate elongation factor and severe lymphadenopathy (bull neck)
81
82 Pseudomonas Exotoxin N (EF-2) Host cell death
83 aeruginosa
84 Shigella spp. Shiga toxin (ST)3 Gl mucosal damage -- d~·sentel)·; ST also
85
Inactivate 60S ribosome by enhances cytokine release, causing hemolytic-
86
removing adenine from uremic syndrome (HUS)
87
Enterohemorrhagic Shiga-like toxin rR 1A SLT en hances cytokine release, causing II U
88
89
E coli (EHEC) (SLT)3 (prototypically in EHEC serotype 0157:H7).
• 90 Unlike Shigella, EH EC does not invade host
. 91 cells
• 92 Increase flUid secretion
• 93
• 94
Enterotoxigenic Heat-labile Overactivates aclenylate Watery diarrhea: "labile in the Air (Adenylate
• 95
E coli (ETEC) toxin (LT)3 cyclase (t cAMP) -- t Cl- cyclase), stable on the Ground (Guanylate
. 96
secretion in gut and 1110 cyclase)"
. 97 efAux
• 98 lleat-sh1ble Overactivates guan)date
• 99 toxin (ST) cyclase (t cGMP)
• 100 -- l resorpl ion of 1 aCI •
8
L.odt
s
Su~pl'nd
~
End Block
Item: 90 of - ,• Mark -<J [:::> "'I ~ · ~
100 ~ P~v1ous N @xt Labl lues N o t es Calcula to r
74
A woman comes in with a complaint of vulvar pruritu s accompanied by discharge and dysuria. The physician diag noses vaginit is. Gram ~~AI
75
stain of her vaginal discharge is shown in t he image.
76
77
78
79
80
81
82
83
84
85
86
• .J.
87
88
Image courtesy
89
• 90
Which pH value is most likely on a litmus paper t est of her vagina?
. 91
• 92 :

• 93
A. l.O
• 94 B. 4.0
• 95 c. 6.0
. 96
D. 7.0
. 97
• 98 E. 8.0
• 99
. 100 •
8
Lode.
s
Suspe-nd
8
End Bloc:k
Item: 90 of - ,• Mark -<J [:::> "'I ~ · ~
100 ~ P~v1ous N @xt Labl lues N o t es Calcula to r
74 •
75
76 Th e correct an swer is B. 51 °/o chose this.
77 Vulvovaginal candidiasis is generally detect ed in a normal vaginal pH range ( <4.5}. The image is of a Gram stain showing hyphae and
78
blastospores.
Gr'm 'ltatntng C~od1d1 15 PH Hypha
79
80
A i s not correct. 6 °/o ch ose this.
The normal vaginal pH is 3.8 to 4.5. It is maintained by Lactobacilli species that secrete lactic acid and hydrogen peroxide. A pH of 1.0 is not a
81
typical finding in any of the vaginal infections.
82 H Hlro~en pero -"~~ L-.tac .cad Pt1 l.dctobac• .~..~s H rdrogen Peroxide Vaginrtis Spec•es

83
C is not correct. 23°/o chose this.
84
Bacterial vaginosis is generally associated with a vaginal pH range of 5.0-6.0, moderately elevated from normal values. Adding KOH to the
85 vaginal discharge would generate a strong fishy or amine odor, thereby confirming the diagnosis of bacterial vaginosis; this is called the whiff
86 test.
Bactenal vagtnO~I!: Amane PH Pota.;.stum hydroxide Vaginal discharge
87
88 D is not correct. 10°/o chose thi s .
89 Trichomonas vagina/is infection typically results in a vaginal pH between 5.0 and 7.0. Separat ely, amniot ic f luid has a pH that is usually
between 7.0 and 7.5.
90 Trichomonas vaginalts Amntobc flutd PH Tnchomonas Infection
. 91
E is n ot correct. 10°/o chose this .
• 92
A vaginal pH of 8.0 is sugges tive of contamination wit h either cerv ical mucus or sperm, both of which normally have a pH around 8.0. A more
• 93
moderately elevated pH suggests Trichomonas vag initis (5 .0-7.0) or bacterial vaginosis (5.0-6.0} .
• 94 Bacterial vaginosis Vag•n•t•s Cervtl< PH Mucus Cervical mucus Sperm Trichomonas

• 95
. 96
Botto m Line :
. 97
Vulvovaginal candidiasis generally is detected in a normal vaginal pH range ( <4.5} .
• 98
• 99
. 100

8
Lode.
s
Suspe-nd
8
End Bloc:k
Item: 90 of - ,• Mark -<] 1:> ""'I ~· 1!';:'1
100 ~ Prev1o u s Next Labf a lu es Note s Calculator
74
75
l@l :fi1·1hi for year: 2017 •
76 FIRST AIO ,ACTS

77
78 FA17 p 177.3
79 Common vaginal infections
80
Bacterial vaginosis Trichomonas vaginitis Candida vulvovaginitis
81
82
SIGNS AND SYMPTOMS 1 o inAammation InAammation ("stra\\ berry InAammation
83 T hin, white discharge fJ with cervix") Thick, white, "cottage cheese"
84 fishv odor Frothy, ) clio" -green, foul- discharge m
85 smelling discharge
86 lAB FINDINGS Clue cells ~ lot ile 1richomonads Pseudohyphae
87
pH > 4.5 pi I > 4.5 pH normal (4.0-4.5)
88
89
TREATMENT Metronidazole J\letronidazole -azoles
90
Treat sexual partner(s)
. 91
• 92
• 93
• 94
• 95
. 96
. 97
• 98
• 99
• 100 •
8
L.odt
s
Su~pl'nd
~
End Block
Item: 91 of - ,• Mark -<J
P~v1ous
[:::> "'I ~ ·· ~
100 ~ N @xt Labl lues N o tes Calcula to r
74
A 79-year-old malnourished man comes in complaining of pleuri tic chest pain, productive cough, and short ness of brea t h. On f urther ~~AI
75
questioning, the patient admits t o having fevers and chills. A sputum sample looks rusty in color. Blood tests show a left shift with
76
increased numbers of neutrophils. On spu tum cult ure, gram-positive diplococci are seen, and an x-ray fi lm of t he chest shows a lobar
77 effusion. Pneumonia is diagnosed.
78
79 Which of the following is characteristic of this patient's type of pneumonia?
80 :
81 A. Inflammation is localized to interstitial areas at alveolar walls
82 B. It can be defined as an "atypical" pneumonia
83
C. It is not associated with pneumonia causing consolidation
84
85 D. It is the most frequent cause of lobar pneumonia
86 E. It typically presents with patchy dist ribut ion involving 1 lobe
87
88
89
90
. 91
• 92
• 93
• 94
• 95
. 96
. 97
• 98
• 99
. 100 •
8
Lode.
s
Suspe-nd
8
End Bloc:k
Item: 91 of - ,• Mark -<] 1:> ""'I ~· 1!';:'1
100 ~ Prev1o u s Next Labf a lu es Note s Calculator
74
The correct a ns w e r is D. 7 6 °/o chose this.
75
The patient has Streptococcus pneumoniae ( pneumococcal) pneumonia, the most frequent cause of
76 community-acquired and lobar pneumonias.The image shows right lower lobe consolidation charact eristic of
77 lobar pneumonia.
Streptococcus pneumontae Pneumonia Lobar pneumonia Streptococcus
78
79
80
81
82
83
84 Image copyright © 2005
85
Parmar; licensee BioMed Central
Ltd.
86
87 A is not correct. 4 °/o chose this .
88 Interstitial (atypical) pneumonia, usually caused by viruses and some bacteria such as Mycoplasma pneumoniae, is characterized by diffuse,
89
patchy inflammation localized to interst it ial areas at alveolar walls.
Mycoplasma pneumoniae Pneumonia Mycoplasma Pulmonary alveolus Bacteria Vtrus Inflammation
90
91 B is not co rrect. 7 % chose this .
• 92
"Atypical" pneumonia, also known as interst it ial pneumonia, is not caused by Streptococcus pneumoniae. It is normally caused by viruses or
other bacteria such as Mycoplasma pneumoniae and Chlamydia pneumoniae .
• 93 Mycoplasma pneumoniae Streptococcus pneumoniae Pneumonia Chlamydophila pneumoniae Chlamydia tnfectton Chlamydia (genus) Mycoplasma Streptococcus Bacteria Virus Interstitial lung dtsease
• 94
C is not correct. 4 °/o chose this .
• 95
Streptococcus pneumoniae pneumonia involves consolidation of the lungs, resu lt ing in bronchial breat h sounds, dull resonance, and increased
. 96 tactile fremitus .
. 97 Streptococcus pneumontae Pneumonta Streptococcus Respiratory sounds Bronchtole Fremttus Lung

• 98 E is no t correct. 9 °/o chose t his .


• 99 Pneumococcal pneumonia usually presents as lobar pneumonia, not a patchy distribution, and it may involve an entire lobe of the lung .
• 100 PneuPlOn•a Lot, pneumon•a Pneumococcal pneumonia streptococcus pneumon•ae lung •
8
L.odt
s
Su~pl'nd
~
End Block
Item: 91 of ~ ,• Mark <::J [:::> ""I ~· ~'j
100 J.. Previous Next LAbfaiUI~S Notes Calculator

74
75 Bottom Line:
76 The most common organism causing loba r pneumonia is the pneumococcus. Atypical pneumonia, in cont rast , is more often caused by
77
Mycoplasma, Chlamydia, or viruses.
Atypical pneumonia Pneumonia Lobar pneumonia Streptococcus pneumoniae Mycoplasma Organism Chlamydia infection Chlamydia (genus) Virus
78
79
80
l@ljl'il·1i•J for year:[ 2017 • J
FIRS T AID FACTS
81
82
83 FA17 p 645.1
84 Pneumonia
85 TYPE TYPICAL ORGANISMS CHARACTERISTICS
86 Lobar S pneumoniae most frequently, also Legionella, Intra-alveolar exudate -+ consolidation rl; may
87 Klebsiella rn
iJwolve entire lobe or lung.
88
Bronchopneumonia S pneumoniae, S aureus, H influenzae, Acute inflammatory infiltrates [!1 from
89
Klebsiella bronchioles into adjacent alveoli; patchy
90
91
m.
distribution involving;;:: 1 lobe
• 92 Interstitial (atypical) Mycoplasma, Chlamydophila pneumoniae, Diffuse patchy inflammation localized to
• 93 pneumonia Chlamydia psittaci, Legionella, viruses (RSV, interstitial areas at alveolar walls; diffuse
• 94 C.MV, influenza, adenovirus) distribution involving ;;:: 1 lobe D. Generally
• 95 follows a more indolent course ("walking"
. 96 pneumonia).
. 97
Cryptogenic Formerly known as bronchiolitis obliterans
• 98
• 99
organizing organizing pneumonia (BOOP). Noninfectious
. 100
pneumonia pneumon ia characterized by inflammation of

8
Lock
s
Suspend
0
End Block
Item: 91 of ~ ,• Mark <::J [:::> ""I ~· ~'j
100 J.. Previous Next LAbfaiUI~S Notes Calculator

74
FA17 p 175.2
75
76
Common causes of pneumonia
77 NEONATES(< 4WK) CHILDREN (4 WK-18 YR) ADULTS (18-40YR) ADULTS (40-65 YR) ELDERLY
78 Group B streptococci Viruses (RSV) Mycoplasma S pneumoniae S pneumoniae
79 E coli M )1coplasma C pnewnoniae H influenzae 1nAuenza virus
80 C trachomatis S pneumoniae Anaerobes Anaerobes
81 (infants-3 yr) Viruses (eg, influenza) Viruses H influenzae
82 C pnewnoniae Mycoplasma C ram 8 rods
83
(school-aged
84
children)
85
S pneumoniae
86
Runts May Cough
87
Chunky Sputum
88
Special groups
89
90 Alcoholic Klebsiella, anaerobes usually due to aspiration (eg, Peptostreptococcus, Fusobacterium, Prevotella,
91 Bacteroides)
• 92
IV drug users S pneumoniae, S aureus
• 93
• 94
Aspiration Anaerobes
• 95 Atypical Mrcoplasma, Legionella, Chlamydia
. 96
Cystic fibrosis Pseudomonas, S aureus, S pneumoniae, Burkholderia cepacia
. 97
• 98 lmmunocompromised S aureus, enteric gram 8 rods, fungi, viruses, P jirovecii (with II IV)
• 99 Nosocomial (hospital S aureus, Pseudomonas, other enteric gram 8 rods
• 100 ;~rn11irPrl\

8
Lock
s
Suspend
0
End Block
Item: 91 of - ,• Mark -<J
P~v1ous
[:::> "'I ~ ·· ~
100 ~ N @xt Labl lues N o tes Calcula to r
74 A A

75
Alcoholic Klebsiella, anaerobes usually due to aspiration (eg, Peptostreptococcus, Fusobacterium, Prevotella,
76
Bacteroides)
77 IV drug users S pneumoniae, S aureus
78
Aspiration Anaerobes
79
80 Atypical Iycoplasma, Legionella, Chlamydia
81 Cystic fibrosis Pseudomonas, S aureus, S pneumoniae, Burkholderia cepacia
82
lmmunocompromised S aureus, enteric gram 8 rods, fungi , viruses, P iirol·ecii (with IIIV)
83
84 Nosocomial (hospital S aureus, Pseudomonas, other enteric gram 8 rods
85 acquired)
86 Postviral S pneumoniae, S au reus, I I influem:ae
87
88
89 FA17 p 132.2
90 Streptococcus Gram $, lancet-shaped diplococci tl Pneumococcus is associated with "rusty"
91 pneumonioe Encapsulated. lgA protease. Optochin sputum, sepsis in patients with sickle cell
• 92
sensitive. Most common cause of: disease, and asplenic patients.
• 93 fJ " Meningitis o virulence without capsule.
• 94 ,
• Otitis media (in children)
• 95
. 96
Bacterial pneumonia
. 97
• Sinusitis

• 98
• 99

. 100

8
Lode.
s
Suspe-nd
8
End Bloc:k
Item: 92 of - ,• Mark -<J [:::> "'I ~ · ~
100 ~ P~v1ous N @xt Labl lues N o tes Calcula to r
74
75 A 43-year-old man complains of a 3-day history of loose, bulky, greasy, foul-sme ll ing stools and excessi ve flatulence. He denies t he
presence of blood or mucus in th e st ool. The patien t also denies fever, nausea, vomit ing, recent t rave l, or sick contacts. However, he notes
76
that he was born and lived in Puerto Rico up until 10 years ago, when he moved to Miami. He has hypertension but no ot her medical or
77 surgical history. He works as a mortgage broker and lives with his male partner of 4 years. He is sexually act ive only with his partner. Physical
78 examination is unremarkable. The physician orders stool studies, which are positive for cysts and feca l ant igen.
79
80
Which of the following is the most likely diagnosis?
81 :
82
A. Amebiasis
83 B. Celiac sprue
84 C. Enteritis due to Salmonella
85
D. Giardiasis
86
87 E. Tropical sprue
88
89
90
91
• 92
• 93
• 94
• 95
. 96
. 97
• 98
• 99
. 100 •
8
Lode.
s
S uspe-nd
8
End Bloc:k
Item: 92 of ~ ,• Mark <::J [:::> ""I ~· ~'j
100 J.. Previous Next LAb faiUI~S Notes Calculator

74
The correct a ns wer is 0. 6 1 °/o chose t his.
75
This patient has gia rdiasis, caused by t he f lagellate Giardia Iamblia. Transmission is feca l-oral, and incidence r-"R"~---------- ....
76 is higher in men who have sex wit h men, t ravelers, immunocompromised patients, and pat ients wit h a ,, 0
77 hist ory of dri nking unf iltered water f rom streams. Onset of sympt oms is usually 1- 2 weeks after ingestion . ...
78
of t he organisms. I ngested cysts develop int o t ro phozoit es t hat infect t he duodenum and jejunum. Most ...
79
cases are asymptomatic. When sym ptoms occur~ t hey include acute and chronic diarrhea and
malabsorption, which typically last f rom 2- 6 weeks if unt reated. Stools are bulky, f rothy, greasy, and 0 ·e .-· 0 •
80 malodorous and free of pus and blood. Diagnosis is made by m icroscopic ident ificat ion of cysts (evident in

t he image) and t rophozoit es in st ool or by immunoassays to det ect feca l ant igen or cysts in the stool.
81
Tinidazole and met ro nidazole are t reat ment s of choice.
82 Giardia Iambiia Metronidazole Giardiasis Tinidazole Flagellate Jejunum Duodenum Immunodeficiency Giardia Malabsorption Asymptomatic Diarrhea Antigen
83 Cyst Microbial cyst Pus Trophozoites Feces Image cowtesy of CDC
84
A is not correct . 17 % chos e this .
85
Amebiasis is a parasit ic gast rointest inal infection t ransm it ted via the feca l-ora l rout e. I ntest ina l manifestat ions include abdom inal pain and
86 recurrent episodes of dia rrhea containing mucus and/or blood, rather t han the bulky, frot hy, ma lodorous st ools caused by giardiasis. Stool
87 studies would have shown amoebas or Entamoeba histolytica ant igen rather than t he feca l ant igen, which is diagnostic fo r gia rdiasis.
Entamoeba histolytica Giardiasis Fecal-oral route Amoebiasis Parasitism Diarrhea Mucus Abdominal pain Antigen Entamoeba Human gastrointestinal tract Amoeba Feces Infection Gastrointestinal tract
88
89 B is not correct. 5% chose t his .
90 Celiac sprue, otherwise known as gluten -sensit ive enteropathy, is an autoimmune disease of t he small int estinal m ucosa that occurs in
91 response to ingestion of glut en. It can have some of t he same manifestations as giardiasis, including chronic diarrhea, greasy-appeari ng stool,
f latulence, and ma labsorpt ion. However, st ool from pat ients wit h celiac sprue would not cont ain cyst s and the coproant igen. The diagnosis of
92 celiac sprue is made by biopsy revealing f lat tened villi. Ant it ransglut aminase ant ibodies can also be found in t he serum .
• 93 Giardiasis Altoimmune disease Coeliac disease Flatulence Diarrhea Malabsorption Biopsy Antibody Intestinal villus Gluten-related disorders Human gastrointestinal tract Non-celiac gluten sensitivity

• 94 Autoimmunity Gluten Blood plasma Enteropathy Mucous membrane Intestinal mucosa Serum (blood) Cyst Small intestine

• 95
C is not co rrect. 4 °/o chose this .
. 96 Nonty phoidal Salmonella species can cause bact erial gastroint estinal infect ion, which is cha racterized by nausea, vom it ing, abdomina l cramps,
. 97 and bloody diarrhea wit h mucus. Salmonella infections can be diagnosed via stool cult ure, rat her t han t hroug h detect ing coproantigen or cysts
• 98 in the stool. Whereas Salmonella ent eri t is is usually t ransmitted through eating contam inat ed foods such as eggs, dairy products, and poult ry,
t he rarer typhoida l Salmonella species are usually t ransm it ted v ia the feca l-oral rout e and cause t yphoid fever. Alt hough patients with t yphoid
• 99 fever can have nonbloody diarrhea, as seen in t his patient, these pat ients will also have a sust ained high fever, which t his pat ient denies, as
• 100 ~
well as ot her symptoms such as diaphoresis, bradyca rdia, and delirium . DiaQnoses are conf irmed via blood, st ool, and bone marrow cult ures. ~

8
Lock
s
Suspend
0
End Block
Item: 92 of ~ ,• Mark <::J [:::> ""I ~· ~'j
100 J.. Previous Next LAb faiUI~S Notes Calculator

74
E is not correct. 1 3°/o chose this .
75
Tropica l sprue is a possibility for any person who has lived a sign if ican t amount of t ime in Sout heast Asia or the Caribbean (the t ropics or
76 subtropics). Epidemiologic data report rat es of t ropical sprue of up to 8% in Pue1to Rico. This disease presents wit h acut e or ch ron ic diarrhea,
77 result ing in weight loss, fatigue, fever, and malabsorption. It is believed to be a result of an intest inal infection . The stool studies make the
78
diagnosis of giardiasis more likely than t ropical sprue.
Tropical sprue Giardiasis Malabsorption Diarrhea Coeliac disease Weight loss Caribbean Fatigue (medical) Southeast Asia Gastroenteritis Intestinal parasite Asia Feces Puerto Rico
79
80
81 Bottom Line :
82 Gia rdiasis is a gast roint estinal illness t ransm itt ed via t he feca l-ora l route, characterized by ch ronic and acute diarrhea wit h bulky, frot hy
83
stools as well as nausea and vom it ing . Met ronidazole is t he t reat ment of choice.
Metronidazole Giardiasis Fecal-oral route Diarrhea Gastrointestinal disease Nausea Gastroenteritis Vomiting Human gastrointestinal tract Gastrointestinal tract
84
85
86
141;fil·1i•J for yea r:[2017 •
FIRST AID FA CTS
87
88
89
FA17 p 175.1
90 Bugs causing diarrhea
91 Bloody diarrhea
92
Campylobacter Comma- or S-shaped organisms; growth at 42°C
• 93
• 94 E histolytica Protozoan; amebic dysentery; liver abscess
• 95 Enterohemorrhagic 0157:H7; can cause llUS; makes Shiga-like toxin
. 96 Ecoli
. 97
Enteroinvasive Ecoli Invades colonic mucosa
• 98
• 99 Salmonella (non- Lactose 8; Aagellar motility; has animal reservoir, especially poultry and eggs
• 100 typhoidal)
8
Lock
s
Suspend
0
End Block
Item: 92 of - ,• Mark -<] 1:> ""'I ~· 1!';:'1
100 ~ Prev1o u s Next Labf a lu es Note s Calculator
74 • •
FA17 p 151.1
75
76 Protozoa- gastrointestinal infections
77 ORGANISM DISEASE TRANSMISSION DIAGNOSIS TREATMENT
78
Giardia Iamblia Giardiasis-bloating, Aatulence, Cysts in water ~ fultinucleated Metronidazole
79
foul-smelling, fatty diarrhea trophozoites or
80
(often seen in campers/hikers)- cysts : in stool,
81
think fat-rich Ghirardell i antigen detection
82
chocolates for fatty stools of
83
84
Giardia
85 Entamoeba Amebiasis-bloody diarrhea Crsts in water Serology and/or Metronidazole;
86 histolytica (dysentery), Ji,·er abscess trophozoites (with paromomycm or
87 ("anchovy paste" exudate), engulfed RBCs 61 iodoquinol for
88 RUQ pain; histology shows in the cytoplasm) asymptomatic cyst
89 Aask-shaped ulcer or cysts with up to passers
90 4 nuclei in stool [!];
91
Entamoeba Eats
92
Jo:rythrocytes,
• 93
antigen detection
• 94
• 95
. 96 Cryptosporidium Severe diarrhea in AIDS Oocysts in water Oocysts on acid-fast Pre,·ention (by
. 97 lild disease (watery diarrhea) in stain 0 , antigen filtering city
• 98 immunocompetent hosts detection water supplies);
• 99 nitazoxanide in
• 100 immunocomoetent •
8
L.odt
s
Su~pl'nd
~
End Block
Item: 93 of - ,• Mark -<] 1:> ""'I ~· 1!';:'1
100 ~ Prev1o u s Next Labf a lu es Notes Calculator
74
75
A 65-year-old man presents t o a cardiologist with increasing shortness of breath. He has not seen a doctor in 10 years. He reports that his r~AJ
shortness of breath has been present for abou t two weeks and is worsening. He can walk only about 100 feet before becoming winded, and
76 he has taken to sleeping on four pillows at night. His heart rate is 80/min, blood pressure is 145/95 mm Hg, and respiratory rate is 16/min.
77 Physical examination reveals jugular venous distention, hepatomegaly, and bilateral lower-extremity edema . Laboratory studies indicate negative
78
troponin levels but an elevated B-type natriuret ic peptide. An s 3 is heard on cardiac examination.
79
Which infectious agent is most likely responsible for this disease?
80
81 :
A. Coronavirus
82
83 B. Coxsackievirus B virus
84
C. Parainfluenza virus
85
D. Streptococcus pyogenes
86
87 E. Streptococcus viridans
88 F. Treponema pallidum
89
90
91
92
• 93
• 94
• 95
. 96
. 97
• 98
• 99
• 100 •

8
L.odt
s
Su~pl'nd
~
End Block
Item: 93 of ~ ,• Mark <::J [:::> ""I ~· ~'j
100 J.. Previous Next faiUI~S
LAb Notes Calculator

74
75 The correct answer is B. 66% chos e this.
76 The pat ient has dilat ed cardiomyopathy caused by Coxsackievirus, resulting in congestive hea1t fa ilure (CHF) . Elevat ed B-type natri uret ic
77 peptide is highly sensit ive fo r CHF, and an 5 3 heart sound is highly specific fo r CHF. Ot her causes of dilated cardiomyopat hy include alcohol or
cocaine abuse, pregnancy, inherit ed genetic abnormalit ies, hemochromatosis, and doxorubicin. Notably, Trypanosoma cruzi, t he causat ive
78
agent of Chagas disease, is another infect ious cause of CHF, although it is chronic in onset.
79
A is not correct . 3°/o chos e this.
80 Coronaviruses are RNA viruses that cause t he common cold, severe acute respi ra tory syndrome (5AR5), and Middle East resp iratory
81 syndrome (M ER5). They are not associat ed with cardiomyopathy.
82 C is not correct. 4 °/o chose this .
83 Para influenza v irus, a paramyxovirus, is t he causative agent of croup, which manifest s with a seal- like barking cough. The F swface protein in
84
t hese viruses cause respira tory epit helial cells t o fuse and fo rm mult inucleat ed cells. Parainfluenza viruses are not associat ed with
cardiomyopat hy.
85
D is not correct. 11% chose this .
86
Streptococcus pyogenes is the causat ive agent of rheumat ic fever and can result in pancardit is, not t he diffuse syst olic fa ilu re evidenced in
87 t his pat ient.
88 E is not correct. 11°/o chos e this.
89 Streptococcus viridans can cause subacute bacterial endocardit is, a disease of cardiac valves, not cardiac m uscle.
90 F is not correct . 5°/o chos e this.
91 The causat ive agent of syphilis, Treponema pal/idum, can affect t he heart in t ertiary syphilis. At t his late stage, aortit is is possible as a result
92 of destruct ion of t he vasa vasorum t hat supplies the aortic media l layer. This can result in aneurysm of the aort a.
93
• 94 Bottom Line:
• 95 Dilat ed cardiomyopat hy causes systolic dysfunction of the heart, which results in t he signs and symptoms of CHF, such as an 5 3 hea1t sound,
. 96 and dyspnea, respect ively. Infectious agents leading to this disease include Coxsackievirus Band Trypanosoma cruzi (Chagas disease) .
. 97
• 98
• 99
l@l;fil·1i•l for year:[2017
FIRST AID FACTS .
•j .

• 100

8
Lock
s
Suspend
0
End Block
Item: 93 of - ,• Mark -<] 1:> ""'I ~· 1!';:'1
100 ~ Prev1o u s Next Labf a lu es Notes Calculator
74
75 FA17 p 297.1
76 Cardiomyopathies
77
Dilated Most common cardiomyopathy (90% of cases). Systolic dysfunction ensues.
78
cardiomyopathy Often idiopathic or familial. Other etiologies Eccentric hypertrophy fJ (sarcomcrcs added in
79
include chronic AJcohol abuse, wei Beriberi, series).
80
Coxsackie B \'iral nwocardil is, chronic
I
ABCCCD.
81
82
Cocaine use, C hagas disease, Doxorubicin Takotsubo cardiomyopathy: "broken heart
83
toxicity, hemochromatosis, sarcoidosis, syndrome"-ventricular apical ballooning
84 peripartum cardiomyopathy. likely due to increased sympathetic stimulation
85 Findi ngs: HF, S3, systolic regurgitant murmur, (stressful situations).
86 dilated heart on echocardiogram, balloon
87 appea rance of hea rt on CXR.
88 Treatment: 1 a+ restriction, ACE inhibitors,
89 ~-bloc kers, diuretics, digoxin, lC D, heart
90 transplant.
91
Hypertrophic 60-70% of cases are familial , autosomal Diastolic dysfunction ensues.
92
cardiomyopathy dom inant (most commonly due to mutations larked ventricular concentric hypertrophy
93
• 94
in genes encoding sarcomeric proteins, such as (sarcomcrcs added in parallel) rn,often septa l
• 95
myosin bind ing protein C and ~ -myos in heavy predominance. Myofibrillar disarray and
. 96
chain). Can be associated with Friedrcich fibrosis.
. 97 ataxia. Causes syncope during exercise and Hypertrophic obstructi,·e cardiomyopathy
• 98 may lead to sudden death in young athletes (subsel)-asymmetric septal h}pertrophy
• 99 due to ,-entricular arrh,,thmia. I
and systolic anterior motion of mitral valve
• 100 Findings: S4, systolic murmur. to. Ia) sec mitral - outAO\\ obstruction - dyspnea, possible •
8
L.odt
s
Su~pl'nd
~
End Block
Item: 93 of ~ ,• Mark <::J [:::> ""I ~· ~'j
100 J.. Previous Next LAbfaiUI~S Notes Calculator

74
FA17p154.1
75
Protozoa- others
76
77
ORGANISM DISEASE TRANSMISSION DIAGNOSIS TREATM ENT
78 Visceral infections
79 Trypanosoma Chagas disease- dilated Reduviid bug Trypomastigote in Benzn idazole
80
cruzi cardiomyopathy with ("kissing bug") blood smear fJ or nifurtimox;
81
apical atrophy, megacolon, feces, deposited C ruzing in my
82
megaesophagus; predominantly in a painless bite Benz, with a fur
83
in South America (much Iike a kiss) coat on
84
Unilateral periorbital swelling
85
86
(Romai'ia sign) characteristic of
87
acute stage
88 Leishmania Visceral leishmaniasis Sand fly Macrophages Amphotericin B,
89 donovani (kala-azar) - spiking fevers, contain ing sodium
90 hepatosplenomegaly, a mastigotes : stibogluconate
91
pancytopenia
92
Cutaneous leishmaniasis-skin
93
ulcers
• 94
• 95
. 96
. 97 Sexually transmitted infections
• 98
Trichomonas Vaginitis- foul-smelling, greenish Sexua l (cannot exist Trophozoites Metronidazole for
• 99
• 100
vagina/is discharge; itching and burning; outside human (motile) 9 on wet patient and partner

8
Lock
s
Suspend
0
End Block
Item: 94 of - ,• Mark -<J [:::> "'I ~ · ~
100 ~ P~v1ous N @xt Labl lues N o tes Calculato r
74
A 55-year-old male immigrant f rom Africa presents to t he emergency department after an episode of gross hemat uria. He st ates that he ~~AI
75
has seen small amounts of blood in his urin e from time t o t ime over the past several mont hs. His physical examination is remarkab le only
76
for mild hepatosplenomegaly. A urology consult is called, and the urolog ist performs a bedside cystoscopy. A large fungating mass is seen
77 adherent to the superior part of t he bladder. Results of a bi are shown in the image.
78
79
80
81
82
83
84
85
86
87
88
89
90
91 What is the most likely environmental exposu re associat ed with t his disease in this patient?
92 :
93 A. Cigarette smoking
• 94
B. Exposure to ani line dyes
• 95
. 96
C. Helminth infection
. 97 D. Infection with a papillomavirus
• 98
E. Long -term indwelling cathet er
• 99
. 100 •
8
Lode.
s
S uspe-nd
8
End Bloc:k
Item: 94 of - ,• Mark -<J [:::> "'I ~ · ~
100 ~ P~v1ous N@xt Labl lues No tes Calculato r

74 A A

75
The correct answer is c. 5 2 °/o chose this.
76
This patient is suffering from squamous cell carcinoma (SCC) of t he bladder. Although this condit ion is ra re in t he United States, it is the most
77 common type of bladder cancer in the world . The most common cause is chronic irritation by the eggs of the helminth Schistosoma
78 haematobium, which lodge in the bladder wall. Although smoking has also been linked to an increased risk of sec of th e bladder, in this
patient of African descent, helminth infection is the more likely risk factor.
79
80 Squamous cell carcinoma can be distinguished from transitional cell carcinoma pathologically by its appearance. This specimen shows keratin
pearl formation, a common finding in squamous cell cancers. In contrast to the United States, where transitional cell carcinoma of the bladder
81
is the most common type of bladder cancer (it accounts for 95% of cases), countries in which Schistosoma haematobium is endemic have
82 much higher rates of squamous cell carcinoma of the bladder.
83 SIN'Ider c nee Se-t ;ttv~o• a hae 4tobtum Hehnmths Transitional c~r carcinoma K.eratm Squamous·cefl carcmoma Unnary bladder Carcmoma Epithelium Schistosoma Cancer Tobacco smolong
84 Squ~f""M)U!: eptthel c Rtsl factor Umted states

85
A is not correct. 18% chose this .
86
Cigarette smoking is the most common risk factor associated with transitional cell bladder carcinoma,
87 demonstrated in this image of a t ransureth ral biopsy. I n this case, however, the patient's presentation and
88 pathology are more suggestive of squamous cell carcinoma (SCC). Smoking has also been linked to
increased risk of SCC, but helmint h infection remains the greater and more common risk fact or, particularly
89
outside the United States.
90 Helminths Squamous·cell carctnoma Btopsy Unnary bladder Bladder cancer Carctnoma Pathology Tobacco smoktng Squamous eptthehal cell Risk factor
91
92
93
94
• 95 Image courtesy of Wikimedia
Commons
. 96
. 97 B is not correct. 18°/o chose t his.
• 98 Aniline dyes are associated with t ransit ional cell carcinoma, the most common type of bladder cancer in the United States. They are not
• 99 associated with squamous cell carcinoma of t he bladder.
Trftnsrbon~ cen c c o na Anllme Squamous-cell carcinoma Bladder cancer Urinary bladder Carer noma Cancer Eprtl d•um Squamous eprthehal ceU
. 100

8
Lode.
s
S uspe-nd
8
End Bloc:k
Item: 94 of ~ ,• Mark <:::1 t::> ""I ~· ~'j
100 J.. Previous Next faiUI~S
LAb Notes Calculator

74
D is not correct. 6°/o chose this .
75
Papillomaviridae such as human papillomavirus are responsible fo r gen it al warts and cerv ical ca rcinoma. They are not associat ed with bladder
76 cancers.
77 Human papillomavirus Papillomaviridae Cervical cancer Genital wart Urinary bladder Carcinoma Cervix

78
E is not correct. 6 °/o chos e this.
79 Long-term cat het eri zat ion is associat ed with t he development of squamous cell carcinoma of the bladder. Based on t he pat ient 's history,
80 however, t here is no reason to assume this patient has had an indwelling cathet er at any t ime.
Bladder cancer Squamous-cell carcinoma Catheter Urinary bladder Carcinoma Squamous epithelial cell Urinary catheterization
81
82
83 Bottom Line:
84 This patient has bladder cancer caused by long-term irritat ion from the eggs of the t rematode Schistosoma haematobium.
85 Trematoda Schistosoma haematobium Bladder cancer Urinary bladder Schistosoma Cancer

86
87
88
l@);fil ~1hl for year:l2o17 y
FIRST AID FACTS
89
90 FA17 p 156.2
91 Trematodes (flukes)
92
ORGANISM DISEASE TRANSMISSION TREATMENT
93
94
Schistosoma Liver and spleen enlargement Snails are host; cercariae Praziquantel
• 95
(S mansoni, egg with penetrate skin of humans
. 96
lateral spine rl), fibrosis,
. 97 inflammation, portal
• 98 hypertension
• 99 Chronic infection with
• 100 1!1 0 ' 0 S haematobium (egg with
8
Lock
s
Suspend
0
End Block
Item: 94 of - ,• Mark -<J [:::> "'I ~ · ~
100 ~ P~v1ous N @xt Labl lues N o tes Calculato r
74 A A

75 l@l : fil ~11•1 fo r yea r: 2017 "


FI RST AID FACTS
76
77
FA17 p 156.2
78
Trematodes (flukes)
79
80
ORGANISM DISEASE TRANSMISSION TREATMENT
81 Schistosoma Li\·er and spleen enlargement Snails arc host; cercariae Praziquantel
82 (S mansoni, egg with penetrate skin of humans
83 lateral spine rJ), fibrosis,
84 inAammation, portal
85 h) pertension
86 Chronic infection with
87
[l] a 0 S haenwtobium (egg "it h
88 0.
0 terminal spine [i]) can lead
89
to squamous cell carcinoma
90
of the bladder (painless
91
hematuria) and pulmona ry
92
hypertension
93
94 Clonorchis sinensis Biliary tract inAammation Undcrcookcd fi~h Praziquant'CI
• 95 -+ pigmented gallstones

. 96 Associated with
. 97 cholangiocarcinoma
• 98
• 99
. 100

8
Lode.
s
S uspe-nd
8
End Bloc:k
Item: 94 of - ,• Mark --<) [::> ""'I ~· 1!';:'1
100 ~ Prev1o u s Next Labf a lu es Notes Calculator
74
FA17 p 569.2
75
76 Transitional cell Most common tumor of urinary tract system
77 carcinoma (can occur in rena l calyces, renal pelvis,
78 ureters, and bladder) : . Can be suggested
79 by painless hematuria (no casts).
80 Associated with problems in )OU r Pee S \ C:
81
Phenacetin, Smoking. \ niline dres, and
82
Cyclophosphamide.
83
84
85
86
87
88
89
FA17 p 154.1
90 Protozoa- others
91 ORGANISM DISEASE TRANSMISSION DIAGNOSIS TREATMENT
92 Visceral infections
93
Trypanosoma Chagas disease-dilated Reduviid bug Trypomastigote in Bcnznidazole
94
cruzi cardiomyopathy with ("kissing bug") blood smear rJ or nifurtimox;
• 95
apical atrophy, megacolon, feces, deposited C ruzing in my
. 96
. 97
megaesophagus; predominantly in a painless bite Benz, with a fur
• 98
in South merica (much like a kis'>) coat on
• 99 Unilateral periorbital swelling
• 100 (Romana sign) characteristic of •
8
L.odt
s
Su~pl'nd
~
End Block
Item: 95 of - ,• Mark -<J [:::> "'I ~ · ~
100 ~ P~v1ous N @xt Labl lues N o tes Calcula to r
74
75
A homeless man arrives at the clinic with diarrhea, multiple mouth sores, and a low-grade fever. He complains of int ense pain on
swallowing. Endoscopy reveals mult iple diffu se white plaques on the esophagus. Gram-stained sample from scrapings of the plaques
76 appear in the image. A resident remarks that a chronic syndrome has faci litated the formation of the current , more acute, infection.
77
78
79
80
81
82
83
84
85
86
..
87 • J.
88
89
Image courtesy of CDC/ Or. Stuart Brown
90
91 According to the Centers for Disease Cont rol and Prevent ion (CDC), which of the fo llowing criteria must be met t o diagnose AIDS?
92
:
93 A. A CD4+ cell count <200/mm 3 only
94
B. Serologic evidence of HIV infect ion only
• 95
. 96 C. Serologic evidence of HIV infect ion plus a CD4+ cell count <250/mm3
. 97 D. Serologic evidence of HIV infection plus either a CD4+ <200/mm 3 , or one or more defining opportunist ic infections
• 98
E. The presence of two or more defining opportunistic infections
• 99
. 100 •
8
Lode.
s
Suspe-nd
8
End Bloc:k
Item: 9S of ~ ,• Mark <::J [:::> ""I ~· ~'j
100 J.. Previous Next LAbfaiUI~S Notes Calculator

74
75 The correct a nswer is 0. 75°/o chose t his.
76 The pat ient in this vignett e presents wit h Candida (as shown in t he image) esophagit is, which is an AIDS -defining illness. Alt hough Candida is
77
a fu ngus, it is unique in t hat it shows up on Gram stain . AIDS results f rom T-lymphocyt e dest ruction fo llowing infection wit h HIV. I n 1993 the
CDC ref ined its definit ion of AIDS to include all HI V-infected persons wit h either a CD4+ cell count <200/mm 3 or a CD4 percentage < 14% . Of
78 not e, CD4+ T- lymphocyte count t akes precedence over the CD4 + T-lymphocyte percent age, and the percent age is considered only if the
79 CD4+ count is m issing . Alt ern at ively, t he CDC guidelines also def ined AIDS as t he presence of an "AIDS-defining" opportunistic infect ion (eg,
80
Candida esophagit is) in the cont ext of a known HI V infect ion . Therefore, if serology is positive for HIV infect ion, t his patient meets t he CDC
definit ion of AIDS because of the esophageal candidiasis.
81 Opportunistic infection Gram staining HIV T cell HIV/AIDS Candida (fungus) Serology Candidiasis CD4 Esophagitis Esophageal candidiasis AIDS-defining clinical condition
82 Centers for Disease Control and Prevention Fungus
83
A is not correct . 14% chose this .
84
A CD4+ cell count of <200/mm 3 o nly is not a complete def init ion of AIDS, because there is no evidence of HI V infect ion. If a CD4+ cell count
85 < 200/mm 3 was obtained in t his pat ient who presents wit h cand ida esophagit is, an AIDS-def ining illness, t hen AIDS could be diagnosed.
86 However, answer D is a bet ter option (serologic evidence of HIV infect ion plus eit her a CD4+ cell percentage <200/mm3 , or one or more
87
defining opport unist ic infections) since this quest ion asks solely t he general diagnostic criteria for def ining AIDS.
HIV Esophagitis HIV/AIDS AIDS-defining clinical condition Opportunistic infection Serology CD4 Medical diagnosis
88
89
B is not correct. 3% chose t his .
Serologic evidence of HI V infect ion only is not a complet e def init ion of AIDS, because the CD4+ count may not (yet) be sufficient ly depleted
90
t o cause severe immunodeficiency.
91 HIV HIV/AIDS Immunodeficiency Serology Infection
92
C is not co rrect. 5°/o chose this .
93
Serologic evidence of HI V infect ion and a CD4+ cell count of <250/mm 3 do not constitut e a complete def init ion of AIDS. The definit ion
94 requires a CD4 + count of <200/mm 3 . This cut -off is not random- at this level of CD4 + deplet ion, the pat ient becomes more susceptible t o
95 t he AIDS-defining opport unistic infect ions.
HIV HIV/AIDS Serology CD4 Infection
. 96
. 97 E is not correct. 3°/o chose this .
• 98 Despit e t he name, the presence of one or more "AIDS-defining" illnesses does not t echnically ensure a diagnosis of AIDS unt il there is
evidence of HI V infection .
• 99
HIV HIV/AIDS Infection
• 100

8
Lock
s
Suspend
0
End Block
Item: 95 of - ,• Mark -<] 1:> ""'I ~· 1!';:'1
100 ~ Prev1o u s Next Labf a lu es Note s Calculator
74
75
. .-------------------------------
Bottom Line:
-------------------------------- -------------------------------- -------------
76 According to the CDC, AIDS is defined by the CDC by a CD4+ count <200/mm 3 , CD4 percentage <14%, and/or presence of one or more
77 defining opportunistic infections, all in t he setting of positive HIV serology.
C04 Seroloo~ H V HIV/AIOS Centers for Disease Control and Prevention Opportunistic anfect1on
78
79
80
l@l;fjl;1i•J for year: 2017 •
81 FI~ST AIO FACTS

82
83 FA17 p 171 .2
84
HIV diagnosis Presumpti,·e diagnosis made with ELISA ELISA/Western blot tests look for antibodies to
viral proteins; these tests often are fahcly e
85
(sensiti,·e, high false (f) rate and lo" thrc~hold ,
86
rule out test); (f) results confirmed with in the fi rst 1- 2 months of HIV infection and
87
88
Western blot assay (speci fie, low fa lsc <t> rate falsely (f) initially in babies born 1·o infected
89
and high threshold, rule in test). mothers (anti-gpl20 crosses placenta). Use
90
Viral load tests determine the amount of PCR in neonates to detect viral load.
91 viral Rr A in the plasma. High viral load
92 associated with poor prognosis. Also use vi ral
93 load to monitor effect of drug therapy. Ill V
94 genotyping used to determine appropriate
95 therapy.
. 96 AIDS diagnosis ~ 200 CD4+ cclls/mm 3
. 97 (normal: 500-1500 cells/mm 3). HIV<t> with
• 98 AIDS-defining condition (eg, Pneumocystis
• 99 pneumonia) or CD4+ percentage < 14%.
• 100 •
8
L.odt
s
Su~pl'nd
~
End Block
Item: 95 of - ,• Mark -<] 1:> ""'I ~· 1!';:'1
100 ~ Prev1o u s Next Labf a lu es Note s Calculator
74
FA17 p 172.1
75
76 Time course of untreated HIV infection
77 Window period +I- Acute HIV infection Skm and mucous Systemic immuno-
78 Viral dissemination membrane deficiency/AIDS·
Seeding of lymphoid organs Clinical latency infections defining ~lnesses
79
80
81
1200 Anti·envdope antibody (gpl20) t lO'
82
83 uoo
84
85
1000
86
900
87 ~

E
88 E
.._ 800 :I:
89
.!:!!
Q)
<
90
u
+
00
700
n
~
91 0
u.... 600 ..,
0

0 ~·
92 .._
+ 3
93 v 500 ,.-
0
u
94
HIVRNA
400 10'
95
. 96 300
. 97
• 98 200
• 99
100
• 100 o.:-·-· •
8
L.odt
s
Su~pl'nd
~
End Block
Item: 96 of - ,• Mark -<J [:::> "'I ~ · ~
100 ~ P~v1ous N @xt Labl lues N o tes Calcula to r
74
75 A 57-year-old man with a past medical hist ory of mitral valve prolapse presents to his primary care physician with complaints of 2 months
of low-grade fever, malaise, and a 4.5-kg (10- lb} weight loss. On history, the patient is not a smoker, has undergone recommended cancer
76
screening, and denies cough or night sweats. He has no history of incarceration or intravenous drug use. He denies feelin g palpitations and
77 has not experienced paroxysmal nocturnal dyspnea. His heart ra te is 76/min and regular, blood pressu re is 134/78 mm Hg, respiratory rate is
78 16/min, and temperature is 38.2°C (100.8°F}. On cardiac auscultation the physician notes a pronounced midsystolic click. Blood cultures revea l
bacteremia.
79
80
What are the characteristics of the offending microbe?
81
:
82
A . Gram positive, optochin-resistant, a-hemolytic, cocci growing in chains
83
84
B. Gram-positive, catalase-positive, coagulase-negative, cocci in clusters
85 C. Gram-positive, catalase-positive, coagulase-positive, cocci in clusters
86
D . Gram-positive, optochin-sensitive, cat alase-negative, coagulase-negative, a-hemolytic, cocci growing in pairs
87
88
E. Not visible microscopically but pathogenic t o mice

89 F. Weakly gram-positive, acid-fast rods, growing in straight or branching chains, and subject t o Ziehi-Nielsen stain
90
91
92
93
94
95
. 96
. 97
• 98
• 99
. 100 •
8
Lode.
s
Suspe-nd
8
End Bloc:k
Item: 96 of - ,• Mark -<J [:::> "'I ~ · ~
100 ~ P~v1ous N @xt Labl lu es N o tes Calcula t o r
74 A A

The correct answer is A . 51 °/o chose this .


75
76
77
78
These are characteristics of viri dans st reptococci, shown in the image. Fifty to sixty percent of
cases of endocardit is of native, not prosthetic, valves are caused by these bacteria. Viridans
streptococci usually attack valves t hat have previous damage. This is evident in this patient, who
has a history of mitral valve prolapse. The presence of a changed cardiac murmur in the setting
of low-grade fever, malaise, and weight loss is highly suggestive of subacute bacterial
•••0 . •• ~


79
80
endocarditis, which is most often caused by this class of bacteria.
M•tnl e pro v- t; Et•doc.rd•b5 Vtndans streptococci Infective endocarditis Mitral vatve

He~Wt m rmur Mal:atse
• ~

• •
81
82
83
84
Subacute bactenal endocardrbs streptococcus Bactena Fever Weght loss

Jo.,
85
Image courtesy of CDC/Or. Mike Miller
86
87 B is not correct. 15% c hose this.
88 Staphylococcus epidermidis accounts for a significant number of cases of bacterial endocardit is. Although seen to a lesser extent than S.
89 aureus, this bug is seen in intravenous dru g users. S. epidermidis also more often affects the tricuspid valve.
Staphylococcus ep1derm1d1s Tncusp1d valve Endocarditis Staphylococcus Infective endocard1t1s Intravenous thentpy Staphylococcus aureus Drug injection
90
91 C is not correct. 17% chose this .
92 Staphylococcus aureus is respo nsible for 10% - 20% of cases of bacterial endocarditis. These cases are usually much more acute than the 2-
mont h insidious cou rse that th is patient descri bes, and are mos t commonly associat ed with intravenous drug use.
93 Staphylococcus aureus Endocard1bs Staphylococcus Infective endocarditis IntrCN"enous therapy Oruo injection
94
D is not correct. 13°/o ch ose this .
95
Alt hough Streptococcus pneumoniae is the most common cause of pneumonia in the elderly, this patient's symptoms are not consistent with
96 this diagnosis. He is not tachypneic and does not report a cough productive of rusty brown sputum.
. 97 Streptococcus pneumonute Tachypnea Sputum Pneumonia Streptococcus Cough

• 98
E is not correct. 2 °/o chose t h is.
• 99
Coxsackievirus B is a frequent cause of viral myocarditis. Patients with this disease present with symptoms that range from palpitations to
. 100 severe arrhythmias to congestive heart fa ilure. Because this patient denies these symptoms, this diagnosis is unlikely.

8
Lode.
s
Suspe-nd
8
End Bloc:k
Item: 96 of ~ ,• Mark <::J [:::> ""I ~· ~'j
100 J.. Previous Next LAbfaiUI~S Notes Calculator

74
F is not correct. 2°/o chose this.
75
These are characteristics of Mycobacterium tuberculosis . Tuberculosis is a possibilit y in any patient who presents with low-g rade feve r, weight
76 loss, and malaise. Howeve r~ the absence of cough and lack of history of exposure make t his diagnosis unlikely.
77 Mycobacterium tuberculosis Tuberculosis Malaise Mycobacterium Cough Weight loss Fever

78
79
Bottom Line:
80
Subacute bacterial endocarditis occurs in individuals who have pre-exist ing valvular disease. Symptoms include low-grade fever, malaise, and
81 weight loss over a peri od of weeks to mont hs. The causat ive bacteria are most likely v iridans st rept ococci, which are gram-posit ive,
82 optochin-resistant , a-hemolyt ic cocci.
Subacute bacterial endocarditis Endocarditis Viridans streptococci Gram-positive bacteria Coccus Infective endocarditis Streptococcus Malaise Bacteria Fever Valvular heart disease Acute (medicine)
83
84
85
86
i@l;fil·1i•J for year:[2017 • J
FIRST AID FACTS

87
88 FA17 p 299.2
89
Bacterial endocarditis Fever (most common symptom), new murmur, Mitral valve is most frequen tly involved.
90
Roth spots (round white spots on retina Tricuspid valve endocarditis is associated with
91
surrounded by hemorrhage fl), Osler nodes IV drug abuse (don't "tri" drugs). Associated
92
(tender raised lesions on Finger or toe pads (lJ with S aureus, Pseudomonas, and Candida.
93
94
due to immune complex deposition}, Janeway Culture 8; most likely Coxiella burnetii,
95
lesions (sma II, painless, erythematous lesions Bartonella spp., II ACEK (I laemofJhilus,
96 on palm or sole) m,
glomerulonephritis, Aggregatibacter (formerly Actinobacillus),
. 97 septic arterial or pulmonary emboli, splinter Cardiobaclerium, Eikenella, Kingella)
• 98 hemorrhages [!] on nail bed. Multiple blood • Bacteria FROM JANE • :
• 99 cultures necessary for die~gnosis . Fever
• 100 ~
• Acute-S aureus (high virulence). Roth spots
8
Lock
s
Suspend
0
End Block
Item: 96 of - ,• Mark --<) [::> ""'I ~· 1!';:'1
100 ~ Prev1o u s Next Labf a lu es Note s Calculator
74
lesions (sma ll, pai nless, erythematous lesions 13artonella spp., IIACEK (1 /aelllophi/us,
75
76
on palm or sole) m.
glomerulonephrit is, Aggregatibacter (formerly Actinobacillus},
77
septic arteria l or pulmonmy emboli, splinter Cardiobaclerium, Eikenella, Kingella)
78
hemorrhages [!J on nail bed. Multiple blood \f Bacteria FRO~ l JA.'IE '1:
79
cultures necessary for diagnosis. F C\Cr

80 Acute-S aureus (high virulence). Roth spots


81 Large ,·egetations on pre,·iouslr normal Osler nodes
82 ,·alves (J. Rapid onset. \ lum1ur
83 Subacute-\'iridans streptococci (lo" Jane"'a) lesions
84 ,·irulence). Smaller ,·egetations on \ nemia
85 congenitally abnormal or diseased vaJ,es. 1\ail-bed hemorrhage
86 Sequela of dental procedures. Gradual Emboli
87 onset.
88 S bovis (gallolyticus) is present in colon cancer,
89 S epidermidis on prosthetic ,·a lvcs.
90
Endoca rditis may also be nonbactcrial
91
(marantic/thrombotic) 2° to malignancy,
92
hypercoagulablc state, or lupus.
93
94
95
96
. 97
• 98
• 99
• 100 •
8
L.odt
s
Su~pl'nd
~
End Block
Item: 97 of - ,• Mark -<J [:::> "'I ~· ~
~ P~v1ous
100
74 A
.... ... ... . . ..
N @xt
..
Labl
..
lues
....
N o tes
...
Calcula to r
... ... ... .. . .. . ... . .. .. ... -- A
and a nonproductive cough. Initially, her cough was minimal but it had worsened over the last 2 days. Physical exam is notable for a fever
75
of 103.10F and dullness to percussion over her left lower lung. Gram stain and routine culture of induced sputum is unrevealing. A silver stain of
76 an induced sputum specimen is shown in the image.
77
78
79
- •
80
81
...
- •
,

82
83
' • •

84
l
85 •

.,•
86
87
88 .- ' • • '
89
90 Image courtesy of CDC/ Dr. William Cheny
91
92 Which of the fol lowing is the most likely diagnosis?
93
94 A. Aspergillosis
95 B. Brucellosis
96
C. Legionnaires' disease
. 97
• 98 D. Mycoplasma pneumonia
• 99 E. Pneumocystis jirovecii pneumonia
. 100

8
Lode.
s
Suspe-nd
8
End Bloc:k
Item: 97 of ~ ,• Mark <::J [:::> ""I ~· ~'j
100 J.. Previous Next LAb faiUI~S Notes Calculator

74
The correct a nswer is C. 66°/o chose this .
75
Legionnaires' disease oft en occurs in the elderly and manifests wit h feve r, chills, and nonproductive cough. Diarrhea is present in up t o 50% of
76 patients. I nitially, the the respirat ory symptoms are m inimal. The causat ive bact erium, Legionel/a pneumophila, Gram st ain poorl y, so silver
77 stain is used to visualize the rods. Transmission occurs by aerosolized wat er via bu ilding systems such as air condit ioners.
Gram staining Legionella pneumophila Legionnaires' disease Legionella Diarrhea Silver stain Bacteria Cough Fever Chills
78
79 A is not correct . 6°/o chose this.
80 Aspergillosis is caused by the fungus Aspergillus. Sympt oms include feve r and a productive cough. I nvasive infect ion oft en occurs in an
immunocompromised host. Aspergilloma (fungus ball) often occurs in a t uberculosis cavity. Silver stain of Aspergillus would show sept at e
81
hyphae t hat branch int o a V shape.
82 Aspergillosis Tuberculosis Aspergilloma Aspergillus Immunodeficiency Hypha Silver stain Fungus Cough Fever Infection

83
B is not correct. 4 % chose t his .
84
Brucellosis is caused by t he bacteria of the genus Brucella. Humans become infect ed by coming in contact with animals or animal products
85 t hat are contaminat ed with t hese bact eri a. Brucellosis can cause a range of symptoms that may include fever, sweats, headaches, back pains,
86 and physical weakness.
Brucellosis Brucella Bacteria Fever
87
88 D is not correct. 11% c hose this.
89 Mycoplasma pneumonia is a common, m ild pneumonia t hat usually affect s people <40 years old. Its symptoms are very similar to those of
Legionnaires' disease, with the except ion of diarrhea. Add it ionally, silver st ain would not be used to diagnose Mycoplasma pneumoniae
90 infection . I nstead, serology, cold agglutinins, or polymerase chain react ion test ing for mycoplasma would be in order.
91 Mycoplasma pneumoniae Mycoplasma Pneumonia Polymerase chain reaction Serology Legionnaires' disease Diarrhea Cold agglutinin disease Agglutination (biology)

92
E is not correct. 1 3°/o chose this.
93 Pneumocystis jirovecii (formerly carinii) pneumonia manifests wit h feve r, shortness of breat h, and nonproduct ive cough . It targets mainly
94 immunocompromised pat ients (eg, those with AIDS) and it is identified by the immunofluorescent staining of sputum or lavage fluid. Silver
95 stain is also used, but would show cysts, not t he rod-shaped bacteri a seen in t his image.
Pneumocystis jirovecii Pneumonia Immunodeficiency Sputum Immunofluorescence HIV/AIDS Pneumocystis pneumonia Cough Dyspnea Bacteria Bacillus (shape) Silver stain Fever Staining Cyst
96
97
• 98 Bottom Line :
• 99 Legionnaires' disease presents mainly in older pat ients, with fever, chills, nonproductive cough, diarrhea, malaise, chest pain, shortness of
• 100 ~
breat h, and at axia; and its cause, Legione/la pneumophila rods, can be seen on silver stain .

8
Lock
s
Suspend
0
End Block
Item: 97 of - ,• Mark -<J [:::> "'I ~ · ~
100 ~ P~v1ous N @xt Labl lues N o tes Calcula to r
74 A A

75
76 Ulifil·!ltl fo r yea r : 2017
fiRS T AID f A CTS

77
78 FA17 p 139.2
79
80
Legionella Gram 8 rod. Gram stains poorly-use sih er Think of a French legionnaire (soldier) with
81
pneumophifa stain. Grow on charcoal yeast extract his ~ih cr helmet. sitting around a campfire
82 medium with iron and cysteine. Detected by (charcoal) with his iron dagger-he is no ~i~~r
83 presence of antigen in urine. Labs may show (cysteine).
84 hyponatremia. Aerosol transmission from
85 em·ironmental water source habitat (eg, air
86 conditioning systems, hot water tanks). 'o
87 person-to-person transmission.
88 Treatment: macrolide or quinolone.
89 Legionnaires' disease-severe pneumonia
90 (often unilateral and lobar rJ), fever, G l and
91
CNS symptoms. Common in smokers and in
92
chronic lung disease.
93
Pontiac fever-mild Au-like syndrome.
94
95
96 FA17 p 122.1
97 Stains
• 98 Gram stain First-line lab test in bacterial identification. Bacteria with thick peptidoglycan layer retain crystal
• 99
violet dye (gram EB); bacteria with thin peptidoglycan layer turn red or pink (gram 8 ) \\ith
. 100

8
Lode.
s
Suspe-nd
8
End Bloc:k
Item: 97 of - ,• Mark --<) [::> ""'I ~· 1!';:'1
100 ~ Prev1o u s Next Labf a lu es Notes Calculator
74
75 FA17 p 122.1
76 Stains
77 Gram stain First-line lab test in bacterial identification. Bacteria with thick peptidoglycan layer retain crystal
78 violet dye (gram®); bacteria with thin peptidoglycan layer turn red or pink (gram 8 ) with
79
counterstain.
80
These bugs do not Gram stain well ('I hese Little \ licrobes t\lay Unfortunately Lack Real C olor
81
But Are Everp\ here).
82
83 T reponema. Leptospira Too thin to be visualized.
84 ~lycobacteria Cell wall has high lipid content.
85
'1ycoplasma, Ureaplasma t o cell wall.
86
87 Legionella, Rickettsia, Chlamydia, Bartonella, Primarily intracellular; also, Chlamydia lack
88 Anaplasma, Ehrlichia classic peptidoglycan because of ! muramic
89 acid.
90 Giemsa stain Chlamydia, Borrelia, Rickettsia, C ertain Bugs Really Try my Patience.
91 Trypanosomes t:J, Plasmodium
92
Periodic acid- Schiff Stains glycogen, mucopolysaccha rides; used PaSs the sugar.
93
94
stain lo diagnose Whipple disease (Tropherynw
95
whipplei I}])
96 Ziehi-Neelsen stain Acid-fast bacteria (cg, Mrcobacteria Current standard of care is auram ine-
97 (carbolfuchsin) Nocardia; stains mycolic acid in cell wall); rhodamine stain for screening (inexpensi\·e,
• 98 protozoa (eg, Cryptosporidium oocysts) more sensitive but less specific).
• 99
India ink stain Cryptococcus neo{ormans • ; mucicarmine
• 100 .- - .. . •
8
L.odt
s
Su~pl'nd
~
End Block
Item: 97 of - ,• Mark --<) [::> ""'I ~· 1!';:'1
100 ~ Prev1o u s Next Labf a lu es Notes Calculator
74
FA17 p 123.1
75
76
Special culture requirements
77 BUG MEDIA USED FOR ISOlATION MEDIA CONTENTS/OTHER
78 Hinfluenzae Chocolate agar Factors V (NAD+) and X (hematin)
79
N gonorrhaeae, T ha~er-Martin agar electivel) favors growth of Neisseria by
80
N meningitidis inhibiti ng growth of gram $ organisms
81
'' ith Vancomycin, gram 8 organisms except
82 1eisseria
with Trimethoprim and Colistin,
83
and fungi with '\ystatin
84
85
Very Typically Cultures :'\eisseria
86 Bpertussis Bordet-Gengou agar (Bordct for Bordetella) Polato extract
87 Regan-Lo\\·e medium Charcoal, blood, and antibiotic
88
Cdiphtheriae Tellurile agar, Lofner medium
89
90
M tuberculosis Lowcnstein-Jcnscn agar
91 M pneumoniae Eaton agar Requires cholesterol
92
Lactose-fermenting lacConkey agar l'crmenlalion produces acid, causing colonies to
93
enterics turn pink
94
95 Ecoli Eosin-methylene blue (EM B) agar Colonies with green metallic sheen
96 Legionella Charcoal yeast extract agar buffered with
97 crsteine and iron
• 98
Fungi Sabouraud agar "S
• a b'sa f un guyI"
.
• 99
• 100 •
8
L.odt
s
Su~pl'nd
~
End Block
Item: 98 of - ,• Mark -<J [:::> "'I ~ · ~
100 ~ P~v1ous N @xt Labl lues N o tes Calcula to r
74
Parents bring their 10-day-oid infa nt t o t he emergency depart ment. He is febri le and appears let hargic and irrita ble. The infant is ~~AI
75
immediately worked up for sepsis, and the appropriate resuscitation is started. Laborat ory culture of cerebrospinal fl uid reveals growth of
76
gram-posit ive bacilli, with ~ hemolysis on sheep blood agar. The microbiologist notes that the same organism may cause meningit is in an
77 immunocompromised adult.
78
79 What is the likely mode of pathogen t ransm ission to the immunocompromised adult?
80 :
81 A. An ascending urinary tract infection
82 B. Direct inoculation into an open wound
83
C. Ingestion of unpasteurized milk
84
85 D. Inhalation of aerosolized bacteria
86
87
88
89
90
91
92
93
94
95
96
97
. 98
• 99
. 100 •
8
Lode.
s
Suspe-nd
8
End Bloc:k
Item: 98 of ~ ,• Mark <::J [:::> ""I ~· ~'j
100 J.. Previous Next LAb faiUI~S Notes Calculator

74
75
The correct answer is C. 58°/o chos e this.
76 The organism descri bed, Listeria monocytogenes, causes meningit is and sepsis in neonates and the immunocompromised. I ngestion of poorly
77 past eurized milk and soft cheese are most commonly im plicat ed in t he pat hogenesis of listeriosis in t he immunocompromised population and
78 pregnant women.
Other common bacteria causing neonat al mening it is include Escherichia coli and Group B streptococci (GBS) . GBS are t he most common
79 cause of neonat al mening it is.
80 Listeria monocytogenes Escherichia coli Sepsis Meningitis Listeriosis Listeria Streptococcus Pasteurization Organism Bacteria Group 8 streptococcal infection Immunodeficiency Infant Milk Cheese

81 Pathogenesis

82
A is not correct . 11% chos e this.
83 While Escherichia coli, a common gram-negat ive bact eri al cause of neonatal meningit is, can produce urina ry t ract infections in bot h well and
84 immunocomprom ised ad ults, t he organ ism described in t he cl inical case is the gram -posit ive bacilli Listeria monocytogenes.
Listeria monocytogenes Escherichia coli Gram-negative bacteria Meningitis Gram-positive bacteria Immunodeficiency Listeria Urinary tract infection Bacillus (shape) Bacilli Infant Organism Urinary system
85
Neonatal meningitis
86
87 B is not correct. 11 % chose this .
88 Direct inoculat ion is a common route of t ransmission of bacterial infect ion. I n cont rast , list eri osis in t he immunocomprom ised is most often
89 f rom ingestion.
Listeriosis Immunodeficiency Pathogenic bacteria Inoculation
90
91 D is not correct. 20% chose this .
92
Listeria monocytogenes is not an airborne infection. Even t he ent eroviruses t hat most often cause asept ic men ingit is are not airborne
pathogens.
93 Listeria monocytogenes Aseptic meningitis Meningitis Listeria Pathogen Enterovirus Infection
94
95
Bottom Line:
96
97
Listeria monocytogenes, wh ich causes men ingit is in neonat es and t he immunocom promised, is commonly found in unpasteuri zed m ilk, soft
cheeses, coleslaw, and packaged cold cuts.
98 Listeria monocytogenes Meningitis Immunodeficiency Listeria Pasteurization Infant Lunch meat Milk Rem milk
• 99
• 100 ~

8
Lock
s
Suspend
0
End Block
Item: 98 of ~ ,• Mark <::J [:::> ""I ~· ~'j
100 J.. Previous Next LAb faiUI~S Notes Calculator

74
75 i@l;fil·1i•J for year:[2017 • J
FIRST AID FACTS
76
77
FA17 p 135.2
78
79 Listeria Gram <33, facu ltative intracellular rod; acquired by ingestion of unpasteurized dai ry products and
80 monocytogenes cold deli meals, via lransplacenlallransmission, or by vaginal transmission during birth . Grows
81
A ... well at refrigeration temperatures (4°-l0°C; "cold enrichment").
82 Forms "rocket tails" (red in fJ) via acti n polymerization that allow intracellular movement and cell-
83 to-cell spread across cell membranes, thereby avoiding antibody. Characteristic tumbling motility
;

84 in broth.
I
85
4 I Can cause amnionitis, septicemia, and spontaneous abortion in pregnant women; granulomatosis
86
infantiseptica; neonatal meningitis; meningitis in immunocompromised patients; mild, self-
87 .,• limited gastroenteritis in healthy individuals.
88 •
Treatment: ampicillin.
89
90
91 FA17 p 176.1
92 Common causes of meningitis
93 NEWBORN (0- 6MOl CHILDREN (6 M0- 6 YR) 6- 60YR 60YR+
94
Group B streptococci S pneumoniae S pneumoniae S pneumoniae
95
E coli N meningitic/is N meningitic/is (f/ 1 in teens) Gram 8 rods
96
Listeria H influenzae type B Enterovi ruses Listeria
97
En terovi ruses HSV
98
• 99 Give ceftri axone and vancomycin empirically (add ampicill in if Listeria is suspected).
. 100 Viral causes of meningitis: enteroviruses (especially coxsackievirus), HSV-2 (HSV-1 = encephalitis), HlV, West ile virus (also
8
Lock
s
Suspend
0
End Block
Item: 98 of ~ ,• Mark <::J [:::> ""I ~· ~'j
100 J.. Previous Next LAbfaiUI~S Notes Calculator

75
76
FA17 p 176.1
77
Common causes of meningitis
78
NEWBORN (0- 6MOl CHILDREN (6 M0- 6 YR) 6- 60YR 60YR+
79
80
Group B streptococci S pneumoniae S pneumoniae S pneumoniae
81 E coli N meningitidis N meningiticlis (f/ 1 in teens) Gram 8 rods
82 Listeria H influenzae type B Enterovi ruses Listeria
83 Enteroviruses HSV
84 Give ceftri axone and vancomycin empirically (add ampicill in if Listeria is suspected).
85 Viral causes of meningitis: enteroviruses (especially coxsackievirus), HSV-2 (HSV-1 = encephalitis), HIV, West ile virus (also
86 causes encephalitis), VZV.
87
In HIV: Cryptococcus spp.
88
ote: Incidence of H influenzae meningitis has l greatly due to conjug<ltc H influenzae vaccinations. Today, cases are usually
89
seen in unimmunized children.
90
91
92 FA17 p 176.2
93 CSF findings in meningitis
94 OPENING PRESSURE CEl l TYPE PROTEIN GLUCOSE
95
Bacterial t t P II s t l
96
97 Fungai/TB t t lymphocytes t l
98 Viral orma 1/t t lymphocytes Norma lit Normal
• 99
• 100 ~

8
Lock
s
Suspend
0
End Block
Item: 99 of - ,• Mark -<J [:::> "'I ~ · ~
100 ~ P~v1ous N @xt Labl lues N o tes Calcula to r
74
A 16-year-old girl in a rural community has swollen, painful lymph nodes in her right axilla. Medical hist ory is significant for unt reated HIV ~~AI
75
infection. Physical examinat ion revea ls m ult iple healed scratches on her right arm with a papule associated wit h one of t he scrat ch marks.
76 She states that the scrat ches occurred about 5 weeks ago.
77
78 What organism is the most likely cause of this patient 's infection?
79
:
80 A. Gram-negative bacterium; oxidase, catalase, and urease positive
81
B. Gram-negative bacterium; visualized with Warthin-Starry stain
82
83 C. Gram-negative coccobacillus, oxidase and catalase positive
84 0 . Gram-positive rod, spore producing
85
E. Yellow-white round worm
86
87
88
89
90
91
92
93
94
95
96
97
98
. 99
. 100 •
8
Lode.
s
Suspe-nd
8
End Bloc:k
Item: 99 of - ,• Mark -<J [:::> "'I ~ · ~
100 ~ P~v1ous N @xt Labl lues N o tes Calcula to r
74 A A

75 The correct answer is B. 68% chose this.


76 Ba1tonella henselae is a very small, gram- negative bact erium t hat is closely related to Rickettsia, although
it is able to grow on lifeless media. It is the cause of cat scrat ch fever (a local, chronic lymphadenitis most
77
commonly seen in children) and bacillary angiomatosis (seen particularly in patients with AIDS). I n this
78 latter patient population, the organism causes proliferation of blood and lymphatic vessels, causing a
79 characteristic "mulberry" lesion in the skin and subcutaneous tissues of the afflicted individual. As shown in
the image, Bartonella henselae bacteria appear as darkly staining bacilli on a Warthin-Starry stain, which is
80 a silver nitrate- based method.
81 QtO~ artJ ·SIMrv >tam Bartonella henselae G<am-negabve bactena Bartonol.a Rtckettsta Lymphadenopathy Cat-scratch dtsease HIV/AIDS
82 Bactena Bacdh Ano•omatos•s f1:'ver Cat leston Organism

83 Image courtesy of CDC


84
85
A is not correct. 7°/o ch ose t h is.
86
Brucella abortus is a gram-negative rod known for its ability to cause abortions in cattle. Though normally found in cattle, it is also a zoonotic
agent. When it gains access to humans, it causes an undulating fever with malaise, lymphadenopathy, and hepat osplenomegaly. The normal
87 route of exposure is via ingestion of the organism, which may be found in products of infected animals (unpasteurized milk, undercooked
88 meat).
Brucella Lymphadenopathy Gram-negattve bactena Zoonosis Malaise Hepatosplenomegaly Fever M1lk
89
90 C is not correct. 15°/o chose this .
91 Pasteurella multocida is a gram-negat ive rod that is norma l f lora of the oral cavity of dogs and cat s. It oft en causes cellulit is following
92
introduction under a person's skin via an animal bit e. Most cases occur in adults who are injured while playing wit h a pet.
Pasteurella multocida Gram-negative bacteria Cellulitis Pasteurella Human microbiota Mouth
93
94
D is not correct. 7°/o chose t his.
Clostridium tetani is a gram-posit ive spore-fo rming anaerobic rod. It causes tetanus (a spast ic pa ra lysis caused by tetanospasmin, which
95
blocks the release of the inhibitory neurotransmitters glycine and y-aminobutyric acid). There may be no lesion at the site of inoculation, and
96 exu dation would be extremely ra re. Clostridium perfringens is also a gram- posit ive spore-form ing anaerobic rod that produces a-toxin
97 (lecithinase) which induces platelet aggregation, causing local ischemic myonecrosis and eventual gas gangrene.
Clostridtum tet11n1 Clostndtum perfnngens Tetanospasmin Gas gangrene Tetanus Gangrene Glyctne An.>terobtc organtsm Gram-postbve bacteria Platelet Clostridium Lecithinase Ischemia
98
Hypox1a (environmental) Les1on ParaJys1s Neurotransmitter Inoculation
99
. 100 E is not correct. 3°/o chose t his .

8
Lode.
s
Suspe-nd
8
End Bloc:k
Item: 99 of ~ ,• Mark <::J [:::> ""I ~· ~'j
100 J.. Previous Next LAb faiUI~S Notes Calculator
II • - I - • ;;;: I - • -o I • - I I. I
74
75 E is n ot correct. 3°/o ch ose thi s.
76 Toxocara canis, a common int estinal parasite of dogs, is a metazoan parasit e that causes visceral larva m igrans. Young children are most
77 likely to be affected because t hey are most likely t o ingest soil cont aminated wit h eggs of the parasite.
Larva Intestinal parasite Toxocara canis Animal Visceral larva migrans Parasitism Toxocaridae Egg Cutaneous larva migrans
78
79
80 Bottom Line:
81 Bartonella hense/ae causes the loca l lymphaden it is called cat scrat ch feve r~ commonly in children. The organ ism also causes bacillary
82
angiomatosis.
Bacillary angiomatosis Bartonella henselae Bartonella Lymphadenopathy Cat-scratch disease Angiomatosis Cat Fever Organism
83
84
85
l@l;fil·1i•l f or yea r:[ 2017
FIRST AID FACTS .
•j .
86
87
88 FA17 p 144.1
89 Zoonotic bacteria Zoonosis: infectious disease transmitted between animals and humans.
90
SPECIES DISEASE TRANSMISSION AND SOURCE
91
92 Anaplasma spp. Anaplasmosis Ixodes ticks (live on deer and mice)
93 Bartonella spp. Cat scratch disease, bacillary angiomatosis Cat scratch
94
Borrelia burgdorferi Lyme disease Ixodes ticks (live on deer and mice)
95
96 Borrelia recurrentis Relapsing fever Louse (recurrent due to variable surface
97 antigens)
98
Brucella spp. Brucellosis/undulant fever Unpasteurized dairy
99
. 100 Campy/obacter Bloody diarrhea Feces from infected pets/animals; contaminated
8
Lock
s
Suspend
0
End Block
Item: 99 of - ,• Mark -<] 1:> ""'I ~· 1!';:'1
100 ~ Prev1o u s Next Labf a lu es Note s Calculator
74 • •
75
Borrelia burgdorferi Lyme disease Ixodes I icks (live on deer and mice)
76 Borrelia recurrentis Relapsing fever Louse (recurrent due to variable surface
77 antigens)
78
Brucella spp. Brucellosis/undulant fe,·er Unpasteurized dairy
79
80 Campylobacter Bloody diarrhea Feces from infected pets/animals; contaminated
81 mcats/foods/ha nels
82 Chlamydophila psittaci Psittacosis Parrots, other birds
83
Coxiella burnetii Q fever erosols of cattle/sheep amniotic fluid
84
85 Ehrlichia chaffeensis Ehrlichiosis Amblyomma (Lone Star tick)
86 Francisella tularensis Tularemia Ticks, rabbits, deer flies
87
Leptospira spp. Leptospirosis Animal urine in water; recreational water usc
88
89 Mycobacterium leprae I .eprosy ll umans with lepromatous leprosy; armadillo
90 (rare)
91 Pasteurella multocida Cellul il'is, ostcomyelit is Animal bite, cats, dogs
92
93
Rickettsia prowazekii Epidemic typhus Human to human via human body louse
94 Rickettsia rickettsii Rocky Mountain spotted fever Dermacentor (dog tick)
95 Rickettsia typhi Endemic typhus Fleas
96
Salmonella spp. Diarrhea (which may be bloody), vomiting, Reptiles and poultry
97
98
(exceptS typhi) fe, er, abdominal cramps
99 Yersinia pestis Plague Fleas (rats and prairie dogs are resen·oirs)
• 100 •
8
L.odt
s
Su~pl'nd
~
End Block
Item: 100 of - ,• Mark -<] 1:> ""'I ~· 1!';:'1
100 ~ Prev1o u s Next Labf a lu es Note s Calculator
74
75
A 27-year-old man with no significant med ical history complains of pain in his right knee fo r the past 2 days. He also complains of dry, r~AJ
itchy eyes. Physical examination reveals tenderness, erythema, and decreased range of motion in the right knee, and some clear discharge
76 from the urethra. His temperature is 36.7°C (98.1°F), heart rate is 78/min, blood pressure is 111/78 mm Hg, and respirations are 14/min.
77 A complete blood cell count is within normal limits, and urinalysis shows 0-1 RBCs/mm 3 , 8-9 WBCs/mm 3 , and no protein or glucose. Multiple
78 cultures of synovial aspirate show no bacterial growth.
79
Which of the following is the most likely diagnosis?
80
81 :
A . Psoriatic arthritis
82
83 B. Reactive arthritis
84
C. Rheumatoid arthritis
85
D. Sjogren syndrome
86
87 E. Systemic lupus erythematosus
88
89
90
91
92
93
94
95
96
97
98
99
• 100 •

8
L.odt
s
Su~pl'nd
~
End Block
Item: 100 of - ,• Mark -<J [:::> "'I ~ · ~
100 ~ P~v1ous N @xt Labl lues N o t es Calcula to r
74 A A

75
76
The correct answ er is B. 85°/o chose this.
77
Reactive arthritis (formerly called Reiter syndrome) is characterized by urethritis, conjunctivitis / uveitis
(shown in this image), and arthritis. A useful mnemonic for this triad is "can't pee, can't see, can't climb a
78 tree." Reactive arthritis typically manifests 1- 3 weeks after infection with Chlamydia trachoma tis, Neisseria
79 gonorrhoeae, Salmonella, Shigella, Yersinia, Campylobacter, or Ureaplasma urealyticum. Affected patients
80
are usually male, with the human leukocyte antigen-B27 phenotype. Culture of synovial fluid drawn from an
affected joint is usually negative, suggesting an autoimmune-mediated reaction.
81 C~l ...t a ... v._orn It•• eocu 'e arthnb5 Nessen a gonorrhoeae Ureaplasma ureal'fticum Urethr .,.. S., gel a C o~Gtter SynoY1iM fluod Whole blood cell

82 Chlamydta onfectlon Salmonella Arthnbs Yersonoa Phenotype Ureaplasma Chlamydoa (genus) Mnemontc
83
84
85 Image copyright © Indian
Journal of Ophthalmology
86
87 A is not correct . 3°/o chose this.
88 Psoriatic arthritis is characterized by nail pitting, dist al interphalangeal joint involvement, and the dermatologic feat ures of psoriasis (plaques
89 with scales on extensor surfaces such as t he elbows and knees).
Psoriatic arthntis Interphalangeal art1culat1ons of hand Psoriasis Arthritis Distal Interphalangeal JOint Anatom1cal terms of location
90
91 C is not correct. 1 °/o chose thi s.
92 Rh eumatoid arthritis is an aut oimmune disease charact erized by symmetric Inflammatory arthri t is t hat is worse in t he morning and affects the
93
knees, feet, and metacarpophalangeal and proximal int erph alangeal joints. It affects women more often t han men (3 : 1). Screening laboratory
t ests are rheumatoid factor ( RF), which is an I gM ant ibody directed against IgG, and ant i-cit ric cit rullinated pept ide (CCP) antibodies, though
94 t hey are not positive in all cases.
95 Screening (med1c1ne) Rheumatoid arthnbs Autoimmune disease Rheumatoid factor Immunoglobulin M Arthnt1s Ant1body Immunoglobulin G Autoimmunity Interphalangeal articulations of hand

96 Metacarpophalang~l JOint Peptide Anatomical terms of location Inflammation

97 0 is not correct . 9 °/o chose this .


98 Sjogren syndrome is an autoimmune disease that manifests with the classic triad of keratoconjunctivitis sicca (dry eyes), xerostomia (dry
99 mouth), and arthritis. It is associated with human leukocyte antigen-DR3 and an increased risk of lymphoma.
~ePtocon)unc:tJvlb a•cca X:erostolllla Auto1rrvnune disease White blood cetl Auto1mmun1ty :erophthAim~a Ltmt o na ~atoconJunctivltis Arthritis
100 •
8
Lode.
s
Suspe-nd
8
End Bloc:k
Item: 100 of ~ ,• Mark <::J [:::> ""I ~· ~'j
100 J.. Previous Next LAb faiUI~S Notes Calculator

74
75 E is not correct. 2 °/o chos e this.
76 Systemic lupus e rythematosus is cha racte ri zed by a numbe r of potential symptoms including fever, fatigue, weight loss, non bacte rial
ve rrucous endocarditis, hilar adeno pathy, and Raynaud phenomenon. Pote ntia l lab findi ngs include anti-nuclear ant ibodies, anti-double-
77
stranded DNA antibodies, anti-smoot h muscle antibodies, or a nti-histone a ntibodies.
78 Anti-nuclear antibody Raynaud's phenomenon Systemic lupus erythematosus Endocarditis Lymphadenopathy Antibody Anti-histone antibodies Lupus erythematosus Fatigue (medical) Weight loss DNA Muscle

79 Fever

80
81
Bottom Line:
82
Reactive a rthritis is a postinfectious process characterized by conjunctivitis, urethri tis, a nd a rthritis. Joint aspirates of synovial fluid a re
83
usually uninfected, diffe rentiat ing t his illness from septic a tt hritis.
84 Arthrocentesis Reactive arthritis Septic arthritis Urethritis Conjunctivitis Synovial fluid Arthritis Synovial membrane Synovial joint

85
86
87 141;fil·1i•J for year:[2017
FIRST AID FA CTS .
•j .
88
89
FA17 p442.1
90
91 Seronegative Arthritis without rheumatoid fac tor (no anti-IgG antibody). Strong association with HLA-B27
92 spondyloarthritis (MHC class I sero type). Subtypes (PAIR) share va riable occurrence of inflammato ry back
93 pain (associated wit h morn ing stiffness, improves with exercise), peripheral arthritis, enthesitis
94 (inflame d insertio n sites of te ndons, eg, Ac hi lles), dactylitis ("sausage fingers"), uveitis.
95 Psoriatic arthritis Associated with ski n psoriasis and nail lesions. Seen in fewer than 1/3 of patients with psoriasis.
96
Asymme tric and patchy involve me nt fJ.
97
Dactylitis and "pencil-in-cup" deformity of
98
D l P on x-ray 11).
99
100
Ankylosing Symmetric invoke ment of spine and sacroiliac Bamboo spine (vertebral fusion) l!l Can cause

8
Lock
s
Suspend
0
End Block
Item: 100 of - ,• Mark -<] 1:> ""'I ~· 1!';:'1
100 ~ Prev1o u s Next Labf a lu es Note s Calculator
74
75 Ankylosing Symmetric invoh-ement of spi ne and sacroiliac Bamboo spine (vertebral fusion) l!l Can cause
76 spondylitis joints - ankylosis (joint f11sion), u\·eitis, aortic restrictive lung disease due to limited chest
77 regurgitation. wall expansion (costo,·ertebral and costosternal
78 ani..) los is).
79 \1ore common in males.
80
Inflammatory bowel Crohn disease and ulcerative colitis arc often
81
disease associated ,,·ith spondyloarthritis.
82
83 Reactive arthritis Formerly known as Reiter syndrome. "Can' t see, can't pee, can' t bend m~ knee."
84 C lassic triad: Shigella, ) ersinia, Chlamydia, Camprlobacler,
85 Con junctivitis Salmonella (Sh\ C hiCS).
86 Urethritis
87 ,\rthritis
88
89
90
91
92
93
94
95
96
97
98
99
100 •
8
L.odt
s
Su~pl'nd
~
End Block
Item: 100 of - ,• Mark -<] 1:> ""'I ~· 1!';:'1
100 ~ Prev1o u s Next Labf a lu es Note s Calculator
74
Arthritis
75
76
77
78
79
80
81
82
83
84
85
86
87
88
FA17 p 506.2
89
90 Uveitis lnAammation of uvea; specific name based on location within affected eye. Anterior uveitis: iritis;
91 posterior uveitis: choroid it is and/or retinitis. May have hypopron (accumulation of pus in anterior
92 chamber · ) or conjunctival redness. Associated wit h systemic inAammatory disorders (eg,
93 sarcoidosis, rheumatoid arthritis, juvenile id iopath ic arth ritis, HLA-B27-associated condit ions).
94
95
96
97
98
99
100 •

8
L.odt
s
Su~pl'nd
~
End Block
Item: 1 of 60
QIO: 1816
- ,•
~
Mark -<J
P~v•ous
[:::>
N@xt Labl
"'I
lues
~ ·
No tes
~
Calculato r

A
. 1 A

A fam ily who recently emigrated to the Unit ed Stat es f rom Romania brings their 7-year-old child t o th e pediat ri cian with complaints of red, ~~AI
•2
itchy eyes and some swelling around them . The child has had coughing with a runny nose and high feve r fo r 3 days. Small lesions with
·3 white centers are seen in his oral cavity. A day after the oral lesions were observed, a maculopapular rash began at t he head and became
• 4 increasingly more prominent on the child's t runk and legs over time.
·5
·6
Which of the following is the most likely cause of this child's symptoms?
·7 :
A. Diphtheria
·8
•9 B. Pertussis
• 10
C. Roseola
· 11
• 12
D. Rubella
• 13 E. Rubeola
• 14

• 15
. 16
. 17
• 18
• 19

• 20
. 21
. 22
. 23
• 24
• 25
• 26
. 27

8
Lode.
s
S uspe-nd
8
End Bloc:k
Item: 1 of 60 ~ ,• Mark <::J [:::> ""I ~· ~'j
QID: 18 16 J.. Previous Next LAbfaiUI~S Notes Calculator

1
•2
The correct answer is E. 6 1 °/o chose this.
•3 Rubeola, also called m easles, is a relatively rare illness in t he United States because of t he ubiquity of t he measles/mumps/rubella (M MR)
•4 vaccine. I t manifests wit h t he prodrome described in this patient . The classic prodrome of measles is commonly referred to as the 3 C's:
. 5 Conjunct ivit is, Coryza, and Cough, all of which are present in this patient. The rash that spreads from head t o toe over a 3-day period
develops 1 or 2 days after t he appearance of Koplik spots- red oral lesions with white centers.
.6 Measles Prodrome vaccine Rash MMR vaccine United States
• 7
A is not correct . 3°/o chose t his.
•8
The immigrat ion status of the pat ient places int o doubt t he vaccine status of the patient. Diphtheria is an upper respirat ory illness that can
•9 quickly progress to a syst emic disease. Alt hough t his disease is virtua lly unknown in t he United St at es because of the prevalence of the
• 10 diphtheria/tetanus/pertussis (DTaP) vaccine, it remains an important disease worl dwide. It is caused by virulent strains of Corynebacterium
· 11
diphtheriae - a club-shaped, gram -posit ive rod expressing dipht heria t oxin t hat inhibi ts eukaryotic prot ein synthesis. The disease begins wit h a
sore throat, low-grade fever, and malaise. Over t ime, an adherent grayish membrane (known as a pseudomembrane fo rms on the lining
• 12 t issues of t he tonsils, pharynx, and/or nasal cavity. Cont inued growt h of the pseudomembrane into the larynx and t rachea can lead to
• 13 obst ruct ion of t he airway wit h subsequent suffocat ion and deat h. Individuals wit h severe disease may also develop neck swelling and enlarged
• 14
neck lymph nodes, leading to a "bull- neck" appearance; prolonged infection can cause systemic organ involvement such as myocardit is
(inf lammat ion of t he heart) . I n this quest ion the presence of a pseudomembrane is not not ed, decreasing t he possibilit y of diphtheria as the
• 15 cause of this patient's symptoms.
. 16 Corynebacterium diphtheriae Diphtheria Myocarditis Larynx Trachea Pharynx Diphtheria toxin Gram-positive bacteria Protein Sore throat Vaccine Lymph node Respiratory disease Systemic disease Tonsil

. 17 Toxin Inflammation Eukaryote Respiratory tract Fever Corynebacterium Asphyxia Malaise United States

• 18
B is not correct. 2% chose t his .
• 19 The immigrat ion status of the pat ient places int o doubt t he vaccine status of the patient. Pert ussis, or whooping cough, is also rare due to
• 20 vaccinat ion protocols in t he developed world . This disease is caused by Bordete/la pertussis, a gram -negat ive coccobacillus. This organism
. 21
produces several t oxins such as pertu ssis t oxin (ADP-ri bosylates G-proteins), adenyl cyclase, and t racheal cytotoxin. Untreated, t he disease
occurs in t hree st ages: ( 1) a catarrhal st age in which upper respirat ory sympt oms indist inguishable f rom t hose of m inor respirat ory t ract
. 22 infections stages; (2) a paroxysmal st age t hat involves a cough that cases paroxysms t erm inating wit h an inspiratory whoop; and (3) a
• 23 convalescent stage in which t he coughing persists over t he course of several months. There has been a recent uptick in reported cases, due to
• 24
bot h a re lat ively decreased DTaP vaccinat ion rate and t o new strains of t he bacteria t hat are not protect ed by t he vaccine. I n this quest ion t he
patient demonstrat es upper respiratory sympt oms wit hout t he paroxysmal cough, decreasing t he possibility of pert ussis as a diagnosis .
• 25 Cytotoxicity Bordetella pertussis Pertussis toxin Tracheal cytotoxin OPT vaccine Coccobacillus Gram-negative bacteria Bordetella Pertussis Adenylyl cyclase ADP-ribosylation vaccination Cough G protein
• 26 Respiratory tract vaccine Bacteria Toxin Paroxysmal attack
. 27

8
Lock
s
Suspend
0
End Block
Item: 1 of 60 ~ ,• Mark <::J [:::> ""I ~· ~'j
QID: 18 16 J.. Previous Next LAbfaiUI~S Notes Calculator

1 • B is not correct. 2% chose this.


•2 The immigrat ion status of the pat ient places int o doubt t he vaccine status of the patient. Pert ussis, or whooping cough, is also rare due to
vaccinat ion protocols in t he developed world . This disease is caused by Bordete/la pertussis, a gram -negat ive coccobacillus. This organism
•3
produces several t oxins such as pertu ssis t oxin (ADP-ri bosylates G-proteins), adenyl cyclase, and t racheal cytotoxin. Untreated, t he disease
•4 occurs in t hree st ages: ( 1) a catarrhal st age in which upper respirat ory sympt oms indist inguishable f rom t hose of m inor respirat ory t ract
. 5 infections stages; (2) a paroxysmal st age t hat involves a cough that cases paroxysms t erm inating wit h an inspiratory whoop; and (3) a
.6
convalescent stage in which t he coughing persists over t he course of several months. There has been a recent uptick in reported cases, due to
bot h a re lat ively decreased DTaP vaccinat ion rate and t o new strains of t he bacteria t hat are not protect ed by t he vaccine. I n this quest ion the
• 7 patient demonstrat es upper respiratory sympt oms wit hout the paroxysma l cough, decreasing t he possibility of pert ussis as a diagnosis.
•8 Cytotoxicity Bordetella pertussis Pertussis toxin Tracheal cytotoxin OPT vaccine Coccobacillus Gram-negative bacteria Bordetella Pertussis Adenylyl cyclase ADP-ribosylation vaccination Cough G protein

•9 Respiratory tract vaccine Bacteria Toxin Paroxysmal attack

• 10 C is not correct. 11% chose this.


· 11 Roseola is a generally m ild infection t hat usually affects children by age 2. There is no vaccine in current use against roseola. I t is t he result of
• 12 a herpes virus that begins wit h a fever and evolves into an exanthem t hat looks like measles. The age of the patient is inconsistent with the
case description .
• 13
Measles Exanthem Herpesviridae Herpes simplex Roseola Vaccine Herpes simplex virus Virus Fever Infection
• 14

• 15
D is not correct. 23% chose thi s.
The pat ient's immigration status places into dou bt her vaccine statu s (MMR vaccine) . Ru bella is a vira l infection t hat is also known as German
. 16
measles. I n this infect ion, a rash generally precedes t he upper respiratory infections. It causes a less severe vira l exant hem t han rubeola
. 17 (measles). Many infections are subclinical, but rubella can cause severe birth defects when infect ion occurs during t he prenatal period . I n this
• 18 case description, the upper respiratory t ract infect ions precede t he rash . This is inconsistent with t he case descript ion .
MMR vaccine Measles Rubella Vaccine Exanthem Viral disease Respiratory tract Virus Upper respiratory tract infection Rash Congenital disorder Asymptomatic Infection
• 19
• 20
. 21 Bottom Line:
. 22 Remember t he 3 C's of measles (rubeola) : Cough, Coryza ( runny/stuffy nose), and Conjunct ivit is - and Koplik spots. The rash of rubeola
• 23 t ypically progresses in a cranial-to -caudal fashion beginning 1 t o 2 days after t he appearance of Koplik spots .
Measles
• 24
• 25
• 26
l@ljl'il·1i•J for year:[ 201 7
F'IQ: ~ T A tn F' AC" TC:
• J
. 27 •

8
Lock
s
Suspend
0
End Block
Item: 1 of 60
QIO: 1816
- ,•
~
Mark -<J
P~v•ous
[:::>
N@xt Labl
"'I
lues
~ ·
No tes
~
Calculato r

A A
1
•2
·3
l@l;fil·!ltl
FIRST AID FACTS
for y ea r: 2017 •

• 4

·5 FA17 p 166.2
·6
Measles (rubeola) A para myxovirus that causes measles. Usual 3 C's of measles:
·7
virus presentation itwolves prodromal fe,er "ith Cough
•9
cough, coryza, and conjuncti\'itis, then Coryza
• 10
C\entually Koplik spots (bright red spots" ith Conjuncti\'it is
· 11
blue-white center on buccal mucosa ), Vitamin A supplementation can reduce
• 12 followed 1-2 dars later by a maculopapular morbiditr. and mortalitr. from measles,
• 13 rn
rash that starts at the head/neck and spreads particularly in malnourished children.
• 14 downward. Lymphadenitis " ith Warthin-
• 15 Finkeldey giant cells (fused lymphocytes) in
. 16 a background of p;tracortical hyperplasia .
. 17 SSPE (subacute sclerosing panencephalitis,
• 18 occurring years later), encephalitis (1:2000),
• 19 and giant cell pneumonia (rarely, in
• 20 immunosuppressed) are possible sequelae.
. 21
. 22
. 23
• 24
• 25
• 26 FA17 p 182.1
. 27 - ··-- - U - __._a _ - ••-··- __ , _ _ .._ _ _.I - L!I-1 - - -

8
Lode.
s
S uspe-nd
8
End Bloc:k
•2
•3
•4 FA17 p 182.1
. 5 Bugs affecting unvaccinated children
. 6 CLINI CAL PRESHHATION FINDINGS/LABS PATHOGEN
• 7 Dermatologic
•8
Rash Begin ning at head and moving down with Rubella virus
•9
postauricular lymphadenopathy
• 10
· 11 Begin ning at head and moving down; rash Measles virus
• 12 preceded by cough, coryza, conjunctivitis, and
• 13 blue-white (Koplik) spots on buccal mucosa
• 14
Neurologic
• 15
. 16
Meningitis Microbe colonizes nasopharynx II influenzae type B
. 17 Can also lead to myalgia and paralysis Poliovirus
• 18
Respiratory
• 19
• 20
Epiglottitis Fever with dysphagia, drooling, and difficulty H influenzae type B (also capable of causing
. 21
breathing due to edematous "cherry red" epiglottitis in fully immunized children)
. 22 epiglottis; "thumbprint sign" on x-ray
• 23 Pharyngitis Grayish oropharyngeal exudate Corynebacterium diJJhtheriae (elaborates toxin
• 24 ("pseudomembranes" may obstruct airway); that causes necrosis in pharynx, cardiac, and
• 25 painful throa t C 'S tissue)
• 26
. 27 •
8
Lock
s
Suspend
0
End Block
Item: 2 of 60 - ,• Mark -<J [:::> "'I ~ · ~
QIO: 1099 ~ P~v•ous N@xt Labl lues No tes Calcula t o r

A A
1
A homeless 37-year-old woman wit h HI V infection comes t o th e clinic with a 4 -week history of worsening hem iparesis, visual fiel d deficits, ~~AI
•2
and cognitive impairment. The patient 's CD4+ count is 22/mm 3 . A lumbar puncture shows a normal opening pressure, and cerebrospinal
·3 fluid analysis is largely normal with negat ive PCR analysis. An MRI is performed and a T2-weight ed image is shown below.
• 4

·5
·6
·7

•9
• 10

· 11
• 12
• 13
• 14

• 15
. 16
. 17
• 18
• 19

• 20
. 21
Which of the fol lowing ent it ies is most likely responsib le fo r this patient's clinical picture?
. 22
:
. 23
A. Cortical tuberculoma
• 24
• 25 B. Cytomegalovirus encephalitis
• 26 C. JC virus
. 27

8
Lode.
s
Suspe-nd
8
End Bloc:k
•2
·3
•4

·5
·6
·7
·8
·9
· 10
· 11
• 12
• 13
• 14
. 15
. 16
. 17
• 18
Which of the fol lowing entities is most likely responsib le fo r t his patient's clinical picture?
• 19
• 20 :
A . Cortical tuberculoma
. 21
. 22 B. Cytomegalovirus encephalitis
• 23 C. JC virus
• 24
D. Primary central nervous system lymphoma
• 25
• 26 E. Toxoplasmosis
. 27 •
8
L.odt
s
Su~pl'nd
~
End Block
Item: 2 of 60
QIO: 1099
- ,•
~
Mark -<J
P~v•ous
[:::>
N@xt Labl
"'I
lues
~ ·
No tes
~
Calcula t o r

1 A
The correct answ er is c. 60°/o chose this. A

2 The clinical picture and imaging are consistent with progressive mult ifoca l leukoencephalopathy (PML)
·3 secondary to reactivation of latent JC virus infection, which can occur with CD4+ counts <200/mm 3 . PML
typically manifests with rapidly progressive foca l neurologic deficits without signs of increased intracranial
• 4
pressure. Ataxia, aphasia, and cranial nerve deficits also may occur. CSF analysis is not usually sufficient fo r
·5 diagnosis but generally shows mildly elevated protein, and WBCs and can sometimes reveal the presence of
·6 myelin basic protein, which is associated with the demyelination caused by JC virus. On imaging, PML
typically manifests as multiple nonenhancing T2-hyperintense lesions, as shown by arrows in the image.
·7
When PML is suspected, stereotactic biopsy is required for definitive diagnosis, but a positive CSF
polymerase chain reaction for JC virus is diagnostic in the appropriate clinical setting. Histology of the
•9 lesions shows nuclear inclusions in oligodendrocytes. Although there is no definitive treatment, clearance of
JC virus DNA can be observed with response to highly active antiretroviral therapy.
• 10
Progrec. •ve t'l"t 1f leul oencephalopath JC V1rus Oemyehnating disease Myehn Aphas•a H•'\tolog Pol,-. "'ct,a. rec.cbon Management of HIV/AIDS
· 11
Myet.n bas•c protem Atax•a Intracrc~mal pressure Oligodendrocyte &opsy Leukoencephalopathy Proteen Neurolooy ONA Virus Infection Cerebrospinal fluid
• 12
CD4
• 13
• 14

• 15
. 16 Image copyright© 2013 Lind~
and von Heijne
. 17
• 18
A is not correct . 4 °/o chose this .
• 19
Uncommon in t he developed world, but present ing wit h increased risk In homeless and HIV pat ients, cort ical t uberculomas are caseating foci
• 20 within t he cort ical parenchyma occurring f rom previous hematogenous mycobacterial bacillemia. The clinical presentat ion may be similar to
. 21 t hat of the current patient; however, present at ion would include enhancing nodu lar lesions on imaging and elevated prot ein and low glucose
on CSF examination .
. 22 Parenchyma HIV Prote1n Glucose Developed country Tuberculosis Mycobactenum Cerebrospinal flu1d Bacteremia
. 23
B is not correct. 1 3°/o chose t his .
• 24
Cytomegalovirus (CMV) encephalitis can mimic t he appearance of PML, but would be associated wit h enhancing periventricular (ie, next to the
• 25 ventrices) white matter lesions in cortical and subependymal regions. CMV encephalitis also is associated typically with more systemic signs
• 26 and symptoms. Polymerase chain reaction analysis of CSF would be positive for CMV, and histologic exam shows giant cells with eosinophilic
. 27
inclusions in both the cytoplasm and the nucleus .

8
Lode.
s
Suspe-nd
8
End Bloc:k
Item: 2 of 60 ~ ,• Mark <::J [:::> ""I ~· ~'j
QID: 1099 J.. Previous Next LAb faiUI~S Notes Calculator

1
D is not correct. 10% chose thi s.
2
Cent ra l nervous syst em lymphoma typically affects t hose with CD4+ cell counts <50/mm 3 MRI will demonstrate one or more enhancing
•3
lesions (SO% are mult iple and 50% are sing le) t hat typically are surrounded by edema and can produce a mass effect. Central nervous
•4 system lymphoma can manifest wit h polymerase chain react ion findings posit ive fo r Epst ein-Barr virus on CSF.
Epstein-Barr virus Polymerase chain reaction Central nervous system Lymphoma Edema Magnetic resonance imaging Nervous system Virus Mass effect (medicine) CD4
. 5
.6 E is not correct. 13°/o chose this.
• 7 Space-occupying lesions due t o t oxoplasmosis infect ion represent t he most common cause of cerebral mass lesions in HIV- infected pat ients
•8 and typically manifest wit h m ult iple enhancing lesions on MRI. The lesions typically are located at t he corticomedullary j unction and are
surrounded by edema t hat frequent ly produces a mass effect and distinguishes its appearance from progressive mult ifoca l
•9 leukoencephalopathy. Posit ive Toxoplasma serologies can assist in diagnosis, and clinical improvements will resu lt from t reat ment with
. 10 suIfad iazi ne/pyri met ha mine or t ri met hopri m/su lfamet hoxa zole .
Trimethoprim/sulfamethoxazole Progressive multifocal leukoencephalopathy Toxoplasmosis Toxoplasma gondii Edema Serology Infection HIV
· 11
• 12
• 13 Bottom Line:
• 14 PML occurs secondary to react ivat ion of lat ent JC virus infection. It manifests wit h rapidly prog ressive foca l neurologic deficits. Imaging
• 15 shows mult iple nonenhancing T2-weighted hy perint ensities. St ereot act ic biopsy is req uired for def init ive diagnosis, but a positive CSF
. 16
polymerase chain react ion fo r JC virus is diagnostic in t he appropriate clinical set t ing .
JC virus Polymerase chain reaction Biopsy Neurology Infection Virus
. 17
• 18
• 19
i@l;fil·1i•J for year:[ 2017 • J
. 20 FIRST AID FACTS

. 21
. 22 FA17 p 494.1
• 23 Other demyelinating and dysmyelinating diseases
• 24 Acute disseminated Multifocal in Aammation and demyelination after infection or vaccination. Presents with rapidly
• 25 (post infect ious) progressive multifocal neurologic symptoms, altered mental status.
• 26
encephalomyelitis
. 27
8
Lock
s
Suspend
0
End Block
Item: 2 of 60 - ,• Mark --<) [::> ""'I ~· 1!';:'1
QIO: 1099 ~ Prev1o u s Next Labf a lues Notes Calculator

1
Charcot-Marie-Tooth Also known as hereditary motor and sensory neuropathy (llMSr ). Group of progressive hereditary
2
disease nen·e disorders related to the defective production of proteins invok ed in the structure and
·3
•4
function of peripheral nen·es or the myelin sheath. Typically autosomal dominant inheritance
·5
pattern and associated with foot deformities (eg, pes can1s, hammer toe), lower extremity
·6
weakness (eg, foot drop) and sensory deficits.
·7 Krabbe disease utosomal recessive lysosomal storage disease due to deficiency of galactocerebrosidase. Buildup
·8 of galactocerebroside and psychosine destrO)'S myelin sheath. F'indings: peripheral neuropathy,
·9 de,elopmental delar, optic atroph}. globoid cells.
· 10
Metachromatic utosomal recessi,·e lysosomal storage disease, most commonly due to arylsulfatase defi ciency.
· 11
• 12
leukodystrophy Buildup of sulfatides .... impaired production and destruction of myelin sheath. Findings: central
• 13
and peripheral demyelination with ataxia, dementia .
• 14 Progressive multifocal Demyelination of C S fJ due to destruction of oligodendrocytes. Seen in 2-4% of AIDS patients
. 15 leukoencephalopathy (reactivation of latent JC virus infection). Rapidly progressive, usually fatal. t risk associated" ith
. 16 natal izumab, rituximab.
. 17
• 18
• 19
• 20
. 21
. 22
• 23
• 24
• 25
• 26 FA17 p 176.2
. 27 •
8
L.odt
s
Su~pl'nd
~
End Block
Item: 2 of 60 ~ ,• Mark <::J [:::> ""I ~· ~'j
QID: 1099 J.. Previous Next LAbfaiUI~S Notes Calculator

1 • •
FA17 p 176.2
2
CSF findings in meningitis
•3
OPENING PRESSURE CELL TYPE PROTEIN GLUCOSE
•4
. 5 Bacterial t t PM s t l
. 6 Fungai/TB t t lymphocytes t l
• 7
Viral Normal/t t lymphocytes Normal/t ormal
•8
•9
• 10 FA17 p 160.1
· 11 DNA viruses
• 12 VIRAL FAMILY ENVELOPE DNA STRUCTURE MEDICAL IMPORTANCE
• 13
Herpesviruses Yes OS and linear See Herpesviruses entry
• 14
• 15 Poxvirus Yes OS and linear Smallpox eradicated world wide by use of the live-
. 16 (largest 0 A virus) attenuated vaccine
. 17 Cowpox ("milkmaid blisters")
• 18 Molluscum contagiosum -Aesh-colored papule with
• 19 centra I umbilication
• 20 Hepadnavirus Yes Partially OS and circular HBV:
. 21
Acute or chron ic hepatitis
. 22
Not a retrovirus but has reverse transcriptase
• 23
• 24
Adenovirus 0 OS and linear Febrile pharyngitis r.J-sore throat
• 25
Acute hemorrhagic cystitis
• 26
Pneumonia
. 27 • Conjunctivitis- "pink eve" •

8
Lock
s
Suspend
0
End Block
Item: 2 of 60 - ,• Mark --<) [::> ""'I ~· 1!';:'1
QIO: 1099 ~ Prev1o u s Next Labf a lues Notes Calculator

1 • i\cule or chronic hepatitis •


2 1
ot a retrovirus but has re,·erse transcriptase
·3
Adenovirus 0 OS and linear Febrile pharyngitis r.J-sore throat
•4
Acute hemorrhagic cystitis
· 5
Pneumonia
·6
Conjuncti\'itis-.. pink e~·e"
· 7
·8
·9
· 10
· 11
• 12
• 13 Papillomavirus I 0 OS and circular II PV-warts (serotypes 1, 2, 6, 11), C l1 , cen ical
• 14 cancer (most commonly 16, 18)
. 15
Polyomavirus I 0 OS and circular JC virus-progressive multifocalleukocncephalopathy
. 16
(P ll.) inlll
. 17
BK virus-transplant patients, commonly targets kidney
• 18
JC: Jtmky Cerebrum; BK: Bad Kidney
• 19
• 20 Parvovirus No SS and linear BI9 virus-aplastic crises in sickle cell disease,
. 21 (smallest 0 virus) "slapped cheek" rash in children (erythema
. 22 infcctiosum, or fifth disease)
• 23 RBC destruction in fetus leads to hydrops fetal is
• 24 and death, in adu lts leads to pure RBC aplasia and
• 25 rheumatoid arthritis-like symptoms
• 26
. 27

8
L.odt
s
Su~pl'nd
~
End Block
Item: 3 of 60
QIO: 3 197
- ,•
~
Mark -<J
P~v•ous
[:::>
N@xt Labl
"'I
lues
~ ·
No tes
~
Calcula t o r

1 A A 36-year-old woman with a hist ory of leu kemia receives a bone m arrow transplant. Two and a half months later, she experiences fever,
2 cough, and dyspnea . Bronchoalveolar lavage revea ls large cells with prominent intra nuclear inclusions, as shown in t he image.
·3
• 4

·5
·6
·7

•9
• 10

· 11
• 12
• 13
• 14

• 15
. 16
. 17
• 18 What is t he m ost likely cause of this patient 's infection?
• 19 :

• 20
A. Adenovirus
. 21 B. Candida albicans
. 22
C. Cytomegaloviru s
. 23
• 24
D. Herpes simplex virus
• 25 E. Mycobacterium tuberculosis
• 26
F. Pneumocystis jirovecii
. 27

8
Lode.
s
Suspe-nd
8
End Bloc:k
2
3
•4

·5
·6
·7
·8
·9
· 10
· 11
' 12
' 13
. 14

' 15
. 16
What is the most likely cause of this patient 's infection'
. 17
A. Adenovirus
' 18
' 19 B. Candida albicans
' 20 C. Cytomegalovirus
. 21
D. Herpes simplex virus
. 22
' 23 E. Mycobacterium tuberculosis
' 24 F. Pneumocystis jirovecii
' 25
G. Toxoplasma gondii
' 26
. 27 •
8
Lock
s
Su~pcnd
:I
End Block
Item: 3 of 60
QIO: 3 197
- ,•
~
Mark -<J
P~v•ous
[:::>
N@xt Labl
"'I
lues
~ ·
No tes
~
Calcula t o r

A A
1
2
The correct answer is c. 61°/o chose this.
3
Cytomegalovirus (CMV) infection is a common complication in immunocompromised patients after bone
marrow transplantation. Histopat hology shows large cells with intranuclear basophilic inclusions ("owl's
• 4 eyes"-see black arrow in image); these are typical of CMV infection.
·5 Cyt:omeg lov1ru H1atopathology Immunodeficiency Hematopoietic stem cell transplantation aa.-;oph1hc Bone rna, row Bone Organ transplantation

·6
·7

•9
I
• 10
· 11
• 12 A is not correct. 3°/o chose t his .
• 13 Histopathology of adenovirus would show int ranuclear basophilic inclusions, indicated by the arrow in this
image. Adenovirus causes respiratory illness, ranging f rom the common cold to pneumonia. Depending on
• 14
degree of immunosuppression and the strain of adenovirus, patients with adenovirus could present wit h
• 15 symptoms of fever, rh inorrhea, cough, and sore t hroat of 3-5 days' duration, or they could present with
. 16 more dramatic symptoms .
Rh1norrhea Common cold Immunosuppression Adenoviridae Pneumonia Histopathology Cough Sore throat Resp1ratory d1sease Fever
. 17
• 18
• 19
• 20
. 21 I mage courtesy of Yale Rosen,
. 22 MD
. 23
B is not correct. 4 °/o chose this.
• 24
Histopathology of Candida albicans would show budding cells with pseudohyphae, like in this image.
• 25
Candida pneumonia rarely develops alone. It can be manifested as multiple lung abscesses or it can come •
• 26 with disseminated disease. Patients may present with shortness of breath, cough, fever, and respiratory
. 27 distress .

8
Lode.
s
Suspe-nd
8
End Bloc:k
Item: 3 of 60
QIO: 3197
- ,•
~
Mark -<J
P~v•ous
[:::>
N@xt Labl
"'I
lues
~ ·
Notes
~
Calculator

A A
1
2
B is not corre ct. 4 % chose this .
3
Histopathology of Candida albicans would show budding cells with pseudohyphae, like in this image.
Candida pneumonia ra rely develops alone. It can be manifested as mult iple lung abscesses or it can com e
• 4 with disseminated disease. Patients may present with shortness of breath, cough, fever, and respi ra tory
·5 distress.
C•ndtda btc~o t'neumonta Hypha Cand•da (fungus) Htstopathology Dyspnea Cough Abscess Fever L1Jng
·6
·7

•9
• 10
· 11
• 12
Image courtesy of CDC/Or.
Stewart Brown
I
• 13 0 is not correct. 6°/o chose this .
• 14 Histopathology of herpes simplex virus ( HSV) would show intranuclear acidophilic Cowdry type A inclusions,
indicated by the arrow in this image. HSV causes wat ery blisters of skin and mucous membranes, but rarely
• 15
infects the resp iratory epithelium .
. 16 Herpes stmple)C vtrus Resptratory eptthehum Htstopathology Epithelium Virus Herpes simplex Mucous membrane

. 17
• 18
• 19
• 20
. 21
. 22
. 23 E is not correct. 7°/o chose this.
• 24 Histopathology of Mycobacterium tuberculosis would show a granulomatous reaction, with th e possible
presence of giant cells, as indicated by the arrows in the image. Symptoms of infection wit h M. tuberculosis
• 25 include cough (often productive), weight loss, night sweats, and fever.
• 26 Mycob~e tum tu uculot. ~ Tuberculosts Granuloma Mycobacterium Weight loss Htstopathology Ntght <C em Cough Fever

. 27

8
Lode.
s
Suspe-nd
8
End Bloc:k
Item: 3 of 60
QIO: 3197
- ,•
~
Mark -<J
P~v•ous
[:::>
N@xt Labl
"'I
lues
~ ·
Notes
~
Calculator

A A
1
E is not corr ect . 7 °/o c hose thi s.
2
Histopathology of Mycobacterium tuberculosis would show a granulomatous reaction, with the possible
3
presence of giant cells, as indicated by the arrows in the image. Symptoms of infection with M. tuberculosis
• 4 include cough (often productive), weight loss, night sweats, and fever.
Mycobactenum tuberculos 5 Tuberculos1s Granuloma Mycobacterium Weight loss Histopathology N1ght e.weats Cough Fever
·5
·6
·7

•9
• 10
· 11
• 12
• 13
• 14
F is not correct. 14°/o chose t his .
Histopathology of Pneumocystis jirovecii would show acellular, foamy material, as indicated by the arrows in
this image. The P. jirovecii fungus rarely causes illness in healthy people, but in the immunocompromi5ed,
this type of fungal pneumonia would manifest with slowly progressive dry cough, fever, and shortness of
breath .
ril'~~~,.....-,..,.-~-::o"""::r-.'""1
I
• 15
Pneumoeystls JlrOvecu Immunodeficiency Pneumonia Cough Histopathology Dyspnea Fungal pneumon1a Pneumocyst1s pneumoma Fungus Fever
. 16
. 17
• 18
• 19
• 20
. 21
G is not correct . 5% ch ose thi s .
. 22 Histopathology of Toxoplasma gondii would show cysts with bradyzoites, as indicated by the arrow in t his
. 23 image. Most people infected with Toxoplasma gondii are asymptomatic or have a brief flu-like phase
• 24 consisting of swollen lymph glands or muscle aches. Severe toxoplasmosis can happen in an acut e setting
or as a reactivation of a latent infect ion and can cause severe damage to the brain, eyes, or other organs .
• 25 To opll'<ma gond11 Toxopl&.>mo> • Asymptomatic Lymph node Myalg1a Histopathology Brl>dyzOI!.. Lymph/tde-,op othy Brain Infection Cyst Human bra1n
• 26 M1crob1al cyst
. 27

8
Lode.
s
Suspe-nd
8
End Bloc:k
Item: 3 of 60 ~ ,• Mark <::J [:::> ""I ~· ~'j
QID: 319 7 J.. Previous Next LAb faiUI~S Notes Calculator

1
G is not correct. 5 % ch ose this .
2
Hist opat hology of Toxoplasma gondii would show cysts with bradyzoit es, as indicat ed by t he arrow in t his
3 image. Most people infect ed with Toxoplasma gondii are asympt omat ic or have a brief flu -like phase
•4 consisting of swollen lymph glands or muscle aches. Severe toxoplasmosis can happen in an acut e set t ing
. 5
or as a reactivat ion of a latent infection and can cause severe damage t o t he brain, eyes, or other organs .
Toxoplasma gondii Toxoplasmosis Asymptomatic Lymph node Myalgia Histopathology Bradyzoites Lymphadenopathy Brain Infection Cyst Human brain
.6
Microbial cyst
• 7
•8
•9
. 10
· 11 Image cowtesy of Yale Rosen,
. 12
MD

I
• 13
• 14
Bottom Line:
• 15
Cytomegalovirus (CMV) is indicat ed by large cells with int ranuclear inclusions ( "owl's eyes") . CMV infect ions in immunocompet ent patients
. 16 often resemb le mononucleosis, bu t in t he set t ing of immunocompromise, more severe infect ions are common. I n pa1t icular, t ransplant
. 17 patients are suscept ible to CMV pneumonia .
Immunodeficiency Cytomegalovirus Infectious mononucleosis Pneumonia Immunocompetence Human cytomegalovirus Inclusion bodies
• 18
• 19
. 20
l@ljl'il·1i•J f or yea r:[ 2017 • J
. 21 FIRST AID FACTS

. 22
• 23 FA17 p 160.2
• 24
Herpesviruses Enveloped, OS, and linear viruses
• 25
• 26 VIRUS ROUTE OF TRANSMISSION CLI NICAL SIGNIFICANCE NOTES
. 27
8
Lock
s
Suspend
0
End Block
Item: 3 of 60 ~ ,• Mark <::J [:::> ""I ~· ~'j
QID: 319 7 J.. Previous Next LAb faiUI~S Notes Calculator

1 • •
2 FA17 p 160.2
3 Herpesviruses Enveloped, OS, and linear viruses
•4
. 5
VIRUS ROUTE OF TRANSMISSION CLI NICAL SIGNIFICANCE NOTES
. 6 Herpes Respiratory Gingivostomatitis, keratoconjunctivitis fJ, Most common cause of sporadic
• 7 simplex secretions, saliva herpes labialis [l), herpetic whitlow on finger, encephalitis, can present as altered
•8 virus-1 temporal lobe encephalitis, esophagitis, mental status, seizures, and/or
•9 erythema mu lti forme. aphasia .
• 10
Herpes Sexua I contact, t-Ierpes genital i s ~, neonatal herpes. Latent in sacral ganglia. Vi ral
· 11
simplex perinatal meningitis more common with
• 12
virus-2 HSV-2 than with HSV-1.
• 13
• 14
Varicella- Respiratory Varicella-zoster (chickenpox 1!], shingles 0}, Latent in dorsal root or trigeminal
• 15 Zoster virus secretions encephalitis, pneumonia. ganglia; Cl V1 branch
. 16
. 17
• 18
• 19
(HHV-3)

Epstein-Barr
virus (HHV-4)
Respiratory
secretions,
Most common complication of shingles is post-
herpetic neuralgia .
Mononucleosis- fever, hepatosplenomegaly,
pharyngitis, and lymphadenopathy (especially
involvement can cause herpes
zoster ophthalmicus.
Infects B cells through CD21.
Atypical lymphocytes on peripheral
I
• 20
saliva; aka posterior cervical nodes 0 ). Avoid contact sports blood smear [!!- not infected B
. 21
"kissing disease," until resolution due to risk of splenic rupture. cells but reactive cytotoxic T cells.
. 22
(common in Associated with lymphomas (eg, endemic (±) Monospot test-heterophile
• 23
teens, young Burkitt lymphoma), nasopharyngeal antibodies detected by agglutination
• 24
adults) carcinoma (especially Asian adults), of sheep or horse RBCs.
• 25
lymphoprol iferative disease in transplant Use of amoxicillin in mononucleosis
• 26

patients. can cause characteristic •
. 27

8
Lock
s
Suspend
0
End Block
Item: 3 of 60 - ,• Mark --<) [::> ""'I ~· 1!';:'1
QIO: 3 197 ~ Prev1o u s Next Labf a lues Note s Calculator

1
Congenital CM
2
3
Human Saliva Roseoh1 infantum (exanthem subitum): high Roseola: fe,·cr first, Rosie (checks)
•4
herpes- fevers for several days that can cause seizures, later.
·5
viruses 6 followed by diffuse macular rash HHV-7-lcss common cause of
·6 and 7 roseola.
·7 Human Sexual contact Kaposi sarcoma (neoplasm of endothelial cells). Can also affect CI tract and lungs.
·8 herpesvirus Seen in HIV/AIDS and transplant patients.
·9 8 Dark/violaceous plaques or nodules
· 10 representing vascular pro Iiferations.
· 11
• 12
• 13
• 14
. 15
. 16
. 17
• 18
• 19
• 20
. 21
. 22
• 23
I
• 24
• 25
• 26
. 27 •
8
L.odt
s
Su~pl'nd
~
End Block
Item: 3 of 60 ~ ,• Mark <::J [:::> ""I ~· ~'j
QID: 319 7 J.. Previous Next faiUI~S
LAb Notes Calculator

1 • •
FA17 p 160.1
2
3
DNA viruses
•4 VIRAL FAMILY ENVELOPE ONA STRUCTURE MEDICAL IMPORTANCE
. 5 Herpesviruses Yes OS and linear See Herpesviruses entry
. 6
Poxvirus Yes OS and linear Smallpox eradicated world wide by use of the live-
• 7
(largest 0 1 A virus) attenuated vaccine
•8
Cowpox ("milkmaid blisters")
•9
Molluscum contagiosum -Aesh-colored papule with
• 10
central umbilication
· 11
• 12 Hepadnavirus Yes Partially OS and circular HBV:
• 13 Acule or chron ic hepatitis
• 14 lot a retrovirus but has reverse transcriptase
• 15 Adenovirus 0 OS and linear Febrile pharyngitis rJ-sore throat
. 16 Acute hemorrhagic cystitis
. 17
Pneumonia
• 18
Conjunctivitis- "pink eye"
• 19
• 20
. 21
. 22
• 23
• 24
• 25
• 26
Papillomavirus No OS and circular HPV-warts (serotypes 1, 2, 6, 11), Cl , cervical
cancer (most commonly 16, 18)
I•
. 27 • Dnlunl"r''l::t.uif'• u:. l\f,., 1\C:: ., ,. . . ,-~ ~; ..,.. ,,)., .. lr " ;r l 1 (' If.-; fn,..., l ),o,,
nri"\I"Tr,.,.C"C' ;\Ib h., lnn."l l-h "
n"\t 1 1.,1"\£\1"\l"obn

8
Lock
s
Suspend
0
End Block
Item: 4 of 60
QIO: 2713
- ,•
~
Mark -<J
P~v•ous
[:::>
N@xt Labl
"'I
lues
~ ·
No tes
~
Calcula t o r

A A
1
A 40-year-old woman develops bloody mucoid diarrhea wit h abdominal cramping after return ing f rom a mission trip in a small Mexi can ~~AI
2
village. Four months later, she still has intermittent abdominal pain in add ition to worsening cramping and increased frequency of the
3 diarrhea. Physical examination shows right upper quadrant tend erness. Ultrasound shows a large, well-circumscribed, hypoechoic mass in
• 4 the liver. Her serum laborat ory studies have the fol lowing results :
·5 Aspartate aminotransferase: 18 U/L
·6 Alanine aminotransferase: 20 U/L
·7 Alkaline phosphatase: 300 U/L
Bilirubin, total serum: 1.1 mg/dL
Bilirubin, direct: 0.3 mg/dL
•9
Stool microscopy reveals the following:
• 10
· 11
• 12
• 13
• 14

• 15
. 16
. 17
• 18
• 19
• 20
. 21
. 22
. 23 Which of the fol lowing is the best treatment for this patient?
• 24
:
• 25 A. Metronidazole
• 26
B. Praziquantel
. 27

8
Lode.
s
Suspe-nd
8
End Bloc:k
Item: 4 of 60
QIO: 2713
- ,•
~
Mark -<J
P~v•ous
[:::>
N@xt Labl
"'I
lues
~ ·
No tes
~
Calcula t o r

1
I. p .. .. • I • ; I
Alanine aminotransferase: 20 U/L
2 Alkaline phosphatase: 300 U/L
3 Bilirubin, total serum : 1.1 mg/dL
• 4 Bilirubin, direct: 0.3 mg/dL
·5 Stool microscopy reveals the following:
·6
·7

•9
• 10
· 11
• 12
• 13
• 14

• 15
. 16
. 17
• 18
• 19
Which of the following is the best t reatment for t his patient?
• 20 :

. 21
A. Metronidazole
. 22 B. Praziquantel
. 23
C. Surgical excision of the hepat ic lesion
• 24
D. Ultrasound-guided aspiration of t he hepatic lesion
• 25
• 26 E. Vancomycin
. 27

8
Lode.
s
Suspe-nd
8
End Bloc:k
Item: 4 of 60 ~ ,• Mark <::J [:::> ""I ~· ~'j
QID: 2713 J.. Previous Next LAbfaiUI~S Notes Calculator

1
The correct a nswer is A. 58°/o chose t h is.
2
This patient has amebiasis due to infect ion wit h Entamoeba histolytica. She has classic features of amebiasis, includ ing bloody mucoid
3 diarrhea, recent t rave l t o a developing country, and hepat ic abscess. Amebic hepatic abscesses may show liver f unction tests consist ent wit h
4 an infilt rative pat tern of liver injury (ie, increased alkaline phosphatase wit h near-n ormal or slight ly elevated aspartate aminot ransfe rase,
. 5 alanine aminot ra nsferase, and bilirubi n) . The treat ment of hepat ic abscess due to E. histolytica infection consists of a course of met ronidazole,
which usually leads t o complet e resolution of pat hology. Met ronidazole is an antibiotic used for protozoan infect ions such as Giardia,
.6 Entamoeba, and Trichomonas. Addit ionally, subsequent t reat ment wit h paromomycin t o elim inate int ra lum inal cysts is often
• 7 necessary. Metronidazole is well t olerated but can cause adverse effects, includ ing abdominal discomfort , nausea, and a disulfiram -like effect
•8 when combined wit h alcohol.
Metronidazole Entamoeba histolytica Paromomycin Bilirubin Alanine transaminase Aspartate transaminase Liver function tests Alkaline phosphatase Amoebiasis Antibiotics Diarrhea Abscess Protozoa Giardia
•9
Liver Nausea Aspartic acid Developing country Cyst Infection Alkalinity Mucus Lumen (anatomy) Abdominal pain Pathology Alanine
• 10
· 11 B is not correct. 21% chose this .
• 12
Praziquant el is used t o t rea t schistosomiasis, or infect ion wit h flukes. Although chronic infect ion wit h schistosomes can cause liver disease due
t o granu lomatous reactions t he immune syst em mounts against eggs deposit ed in the liver, this reaction does not charact eri st ically cause
• 13 large abscesses.
• 14 Praziquantel Schistosomiasis Trematoda Schistosoma Immune system Liver Schistosomatidae Granuloma Liver disease Infection

• 15 C is not correct. 10% chose this .


. 16 Surgical excision of hepatic lesions is t he standard treat ment for Echinococcus infection causing a hydatid cyst in t he liver. Hydatidosis is oft en
. 17 caused by eat ing cont aminated meat, but t he cl inical pict ure can sometimes be confused with t hat of an amebic liver abscess. Echinococcus
• 18
infection would not manifest with bloody mucoid diarrhea as seen in t his vignette. Rath e r~ it is more likely to manifest with obst ruct ive biliary
symptoms, as a hydatid cyst can compress t he biliary t ree. Hydat idosis lesions are walled off and potent ially anaphylactic if t hey burst; t hus,
• 19 careful surgical removal is first- line t reat ment.
• 20 Echinococcosis Liver abscess Echinococcus Abscess Biliary tract Diarrhea Cyst Liver Anaphylaxis Bile duct Infection Surgery

. 21
D is not correct. 7°/o chose this.
. 22 The cause of t he infect ion in t his scenari o, given t he patient 's diarrheal symptoms, is most likely a recent history of fo reign t ravel and crampy
• 23 abdominal pain. Therefore, t he pat ient is more likely to have a liver abscess t han a hepatic cyst , and aspiration would not be warranted .
• 24
Percutaneous aspiration using ult rasound or CT guidance was prev iously used in t he t reatment of hepatic cysts, but is no longer recommended
because cyst s tended to recur afterward .
• 25 Medical ultrasound Liver abscess Abscess Cyst Ultrasound Abdominal pain Percutaneous Etiology Liver
• 26
E is not correct. 4 °/o chos e this .
. 27

8
Lock
s
Suspend
0
End Block
Item: 4 of 60 ~ ,• Mark <::J [:::> ""I ~· ~'j
QID: 2713 J.. Previous Next faiUI~S
LAb Notes Calculator

1 E is not correct. 4 °/o chos e this .


2 Vancomycin, when administered intravenously, is useful for t reatment of gram-posit ive organ isms such as methicillin- resistant Staphylococcus
3 aureus. Adm inist ered ora lly, it is used to t reat diarrhea secondary t o Clostridium difficile infection . Predisposing factors for C. difficile
ent erocolit is include prior antibiot ic use and recent hospita lizat ion, which this patient does not have . Also, C. difficile does not cause hepatic
4
disease .
. 5 vancomycin Staphylococcus aureus Antibiotics Diarrhea Clostridium difficile colitis Intravenous therapy Clostridium difficile (bacteria) Staphylococcus Enterocolitis Clostridium Liver Liver disease

.6
• 7
Bottom Line:
•8
Classic signs of amebiasis include bloody m ucoid diarrhea, recent t ravel t o a developing country, and hepat ic abscess. Treatment is wit h
•9
metron idazole.
• 10 Metronidazole Amoebiasis Diarrhea Abscess Liver Developing country Mucus

· 11
• 12
• 13 l@ljl'il·1i•J for year:[2017 • J
FIRST AID FA CTS
• 14

• 15
FA17 p 191 .3
. 16
Metronidazole
. 17
• 18 MECHANISM Forms toxic free radical metabolites in the
• 19 bacterial cell that damage Dr A. Bactericidal,
• 20 anti protozoal.
. 21 CLINI CAL USE Treats Giardia, Entamoeba, Trichomonas, GET GAP on the Metro with metronidazole!
. 22
Cardn.erella vaginalis, Anaerobes (Bacteroides, Treats anaerobic infection below the diaphragm
• 23
C dif{tcile). Can be used in place of amoxicill in vs clinclamycin (anaerobic infections above
• 24
in H jJylori "triple therapy" in case of penicillin diaphragm).
• 25
allergy.
• 26
. 27 ADVERSE EFFECTS Disulfiram-like reaction (severe Rushing,
8
Lock
s
Suspend
0
End Block
Item: 4 of 60
QIO: 2713
- ,•
~
Mark -<J
P~v•ous
[:::>
N@xt Labl
"'I
lues
~ ·
No tes
~
Calcula t o r

A
1 FA17 p 151.1
A

2
3
Protozoa- gastrointestinal infections
4 ORGANISM DISEASE TRANSMISSION DIAGNOSIS TREATMENT
· 5 Giardia Iamblia Giardiasis- bloating, Aatulence, Cysts in water :\ lultinucleated ~ letronidazole
· 6 foul-smelling, fatty diarrhea trophozoites or
· 7 (often seen in campers/hikers)- C)'StS IE) in stool,
·8 thi nk fat-rich Ghirardelli a ntigen detection
•9 chocolates for fatty stools of
• 10
Giardia
· 11
Entamoeba Amebiasis-bloody diarrhea Cysts in water Serology and/or ~ letron idazole;
• 12
• 13
histolytica (dysentery), li\'er abscess trophozoites (with paromomycm or
• 14 ("anchO\')' paste" exudate), e ngulfed RBCs [i iodoquinol for
• 15 RUQ pain; histology shO\\ S in the cytoplasm) asymptomatic cyst
. 16 Aask-shaped ulcer or cysts with up to passers
. 17 4 nuclei in stool (!];
• 18 E ntamoeba Eats
• 19 Erythrocytes,
• 20 antigen detection
. 21
. 22
Cryptosporidium Se\·ere diarrhea in AIDS O ocysts in water O ocysts on acid-fast Pre\·ention (by
. 23
1ild disease (watery diarrhea) in stain 0 , antigen filtering city
• 24
immunocompetent hosts detection water supplies);
• 25
nitazoxan ide in
• 26
. 27 • immunocompetent
8
Lode.
s
Suspe-nd
8
End Bloc:k
Item: S of 60
QIO: 38M
- ,•
~
Mark -<J
P~v•ous
[:::>
N@xt Labl
"'I
lues
~ ·
No tes
~
Calcula t o r

A A
1
A 7-day- old boy is brought t o a clinic in su b-Saharan Africa because of decreased oral intake. Physical examinat ion reveals erythema ~~AI
2
around the umbilical stump, which was wrapped in unclean linen. The child begins convulsing abruptly and his spine becomes locked in an
3 arched position.
4
·5 What is the main virulence factor of t he offending pathogen?
·6 :
·7 A. Bacterial membrane components inducing cytokine release
·8 B. Degradation of host cell membranes
•9
C. Inhibition of glycine release
• 10
· 11 0 . Inhibition of mRNA translation in host cells
' 12 E. Nonspecific activation of host T-lymphoctyes
' 13
' 14
' 15
. 16
. 17
• 18

' 19
' 20
. 21
. 22
' 23
' 24
' 25
' 26
. 27

8
Lode.
s
Suspe-nd
8
End Bloc:k
Item: S of 60 ~ ,• Mark <::J [:::> ""I ~· ~'j
QID: 3894 J.. Previous Next LAb faiUI~S Notes Calculator

1
2
The correct a nswer is C. 68°/o chos e this .
3 The clinical vignette is the classic prese ntation of neonat al t et anus (cessat ion of feeding, peri umbilical infection, convulsions, arched back) .
4 Although exceeding ly rare in developed count ri es, neonat al t et anus is st ill a prominent disease in the third worl d, where mot hers are not
5
vaccinat ed (cannot confe r immunity to t heir infa nts) and t he um bilical st ump is not kept hyg ienic (t his is t he site of the actual infection). The
causative agent of t et anus is the gram- posit ive, spore-fo rming rod Clostridium tetani. The bacteria re lease the exotoxin tetanospasmin, which
.6 ent ers t he circulation and is endocytosed by neurons at the neurom uscular junct ion. Following ret ro grade t ra nsport up the axon and
• 7 intercellular t ransfer, tetanospasmin ult imately impairs t he ability of inhibitory Renshaw neurons t o release t he neurot ra nsmitt er glycine. The
•8 symptoms of t et anus are due to excessive st imulat ion of muscles due to the removal of glycine's inhibitory effect.
Clostridium tetani Axon Tetanospasmin Neuromuscular junction Tetanus Glycine Neurotransmitter Exotoxin Gram-positive bacteria Endocytosis Bacteria Infant Neonatal tetanus Clostridium Neuron
•9
Infection Immunity (medical) Convulsion Vaccine Developed country
• 10
· 11 A is not correct . 13% chos e this.
• 12
This choice descri bes endot oxin (lipopolysacchari de), which exists in the outer membra ne of gram -negative bacteria. Endotoxin is
cont inuously shed f rom gram-negat ive bact eri a during their life span . Endot oxin induces Toll- like rece pt or signaling in host immune cells,
• 13 which ult imat ely leads t o t he generat ion of proinf lammat ory cytokines and, potentially, endotoxic shock.
• 14 Lipopolysaccharide Gram-negative bacteria Toll-like receptor Bacteria Cytokine Bacterial outer membrane Immune system White blood cell Septic shock Proinflammatory cytokine Mitochondrion

• 15 Toxic shock syndrome Biological membrane Membrane Cell membrane Receptor (biochemistry)

. 16 B is not correct. 7% chose t his .


. 17 A num ber of bacterial species produce pore-fo rming exot oxins that can induce apoptosis in host cells. These include the a t oxin of
• 18 Staphylococcus aureus, st re ptolysin 0 of Streptococcus pyogenes, and 13- hemolysin of Streptococcus agalactiae. Of t hese pat hogens, t he only
• 19
one that might be associated wit h this cl inical scenario is S . agalactiae, a leading cause of neonatal meningit is. Meningit is in newborn s can
manifest with cessation of feeding, irri tabilit y, and convulsions. However, the arched spine and omphalit is in this case make tetanus a more
• 20 likely diagnosis. Cerebrospinal f luid cytology in S . agalactiae meningit is would show decreased glucose, increased protein, and increased
. 21 leukocytes (mainly neut rophils) .
Streptococcus pyogenes Cerebrospinal fluid Streptococcus agalactiae Staphylococcus aureus Apoptosis Tetanus Meningitis White blood cell Neutrophil Pathogen Streptococcus Staphylococcus Protein
. 22
Glucose Exotoxin Toxin Infant Irritability Cell biology Cytopathology
• 23
• 24 D is not correct. 6°/o chose this .
• 25 This choice descri bes t he exot oxin f rom Corynebacterium diptheriae, t he cause of dipht heria . Only st ra ins of C. diptheriae t hat have been
• 26
lysogenized by a particular t em perat e bact eri ophage possess t he gene for t his exot oxin and are pathogenic.
Bacteriophage Diphtheria Exotoxin Gene Lysis Lysogenic cycle Corynebacterium Pathogen
. 27

8
Lock
s
Suspend
0
End Block
Item: S of 60 ~ ,• Mark <::J [:::> ""I ~· ~'j
QID: 3894 J.. Previous Next LAbfaiUI~S Notes Calculator

1
D is not correct. 6°/o chose this .
2
This choice descri bes t he exot oxin f rom Corynebacterium diptheriae, t he cause of dipht heria . Only st rains of C. diptheriae t hat have been
3 lysogenized by a particula r t em perat e bact eri ophage possess t he gene for t his exot oxin and are pathogenic.
Bacteriophage Diphtheria Exotoxin Gene Lysis Lysogenic cycle Corynebacterium Pathogen
4
5 E is not correct. 6 °/o chose this.
.6 This choice descri bes t he activ ity of superant igens, which lead to non-specific, massive T-l ymphocyt e act ivat ion and t oxic shock syndrome.
• 7 Two not eworthy superant igens are TSST- 1 of Staphylococcus aureus and pyrogen ic toxin of Streptococcus pyogenes.
Streptococcus pyogenes Toxic shock syndrome Staphylococcus aureus Superantigen Staphylococcus Fever Streptococcus Toxic shock syndrome toxin T cell Toxin
•8
•9

. 10 Bottom Line:
· 11 Tetanospasmin, t he exotoxin re leased by Clostridium tetani, is endocytosed by neurons and impairs glycine release. Because glycine is an
. 12 inhibitory n e u rotransm i tte r~ prolonged muscl e cont raction occur due t o excessive st imulation .
Clostridium tetani Tetanospasmin Exotoxin Neurotransmitter Glycine Endocytosis Clostridium Muscle contraction Neuron Muscle
• 13
• 14

• 15
i@l;fil·1i•J for year:[ 2017 • J
. 16 FIRST AID FACTS

. 17
• 18 FA17 p 128.1
• 19 Bugs with exotoxins
. 20 BACTERIA TOXIN MECHANISM MANIFESTATI ON
. 21 Inhibit protein synthesis
. 22
Corynebacterium Diphtheria toxin3 Pharyngitis with pseuclomembranes in th roat
• 23
• 24
diphtheriae Inactivate elongation factor and severe lymphadenopathy (bull neck)
• 25 Pseudomonas Exotoxin N (EF-2) Host cell death
• 26 aeruginosa
. 27
8
Lock
s
Suspend
0
End Block
Item: S of 60 ~ ,• Mark <::J [:::> ""I ~· ~'j
QID: 3894 J.. Previous Next faiUI~S
LAb Notes Calculator

1 • FA17 p 128.1 •
2
Bugs with exotoxins
3
BACTERIA TOXIN MECHANISM MANIFESTATI ON
4
Inhibit protein synthesis
5
. 6 Corynebacterium Diphtheria toxin3 Pharyngitis with pseudomembranes in th roat
• 7 diphtheriae Inactivate elongation factor and severe lymphadenopathy (bull neck)
•8 (EF-2)
Pseudomonas Exotoxin N Host cell death
•9
aeruginosa
• 10
· 11 Shigella spp. Shiga toxin (ST)3 GI mucosal damage .... dysentery; ST also
• 12 Inactivate 60S ribosome by enhances cytokinc release, causi ng hemolytic-
• 13 removing adenine from uremic syndrome (HUS)
• 14 Enterohemorrhagic Shiga-like toxin rR A SLT en hances cytokine release, causing HUS
• 15 E coli (EHEC) (SLT)a (prototypically in EHEC serotype 0157:H7).
. 16
Unlike Shigella, EHEC does not invade host
. 17
cells
• 18
• 19 Increase flu id secretion
• 20 Enterotoxigenic Heat-labile Overactivates adenylate Watery diarrhea: "labile in the Air (Aclenylate
. 21 E coli (ETEC) toxin (LT)3 cyclase (t cAMP) .... t CI- cyclase), stable on the Ground (Guanylate
. 22 secretion in gut and H7 0 cyclase)"
• 23 efAux
• 24
I Ieat-stable Overactivates guanylate
• 25
toxin (ST ) cyclase (t cGMP)
• 26
- l resorption of NaCI •
. 27 •
8
Lock
s
Suspend
0
End Block
Item: 5 of 60 - ,• Mark --<) [::> ""'I ~· 1!';:'1
QIO: 3894 ~ Prev1o u s Next Labf a lu es Note s Calculator

1 Clostridium Botulinum toxin 3 of proteins required for Flaccid paralysis, Aoppy baby; toxin prevents
2
botulinum neurotransmitter release release of stimulatory (ACh) signals at
3 via vesicular fusion neuromuscular junctions - Aaccid paralysis
4
a An AB toxin (aka, two-component toxin [or three for anthrax]} with B enabling binding and triggering uptake (endocytosis)
5
·6
oft he act i' e A component. T he A components are usually ADP ribosylt ransferases; others ha\'e enzymatic activities as listed
·7 in chart.
·8
Lyse cell membranes
·9
· 10
Clostridium lpha toxin Phospholipase (lecithinase) Degradation of phospholipids - myonecrosis
· 11 perfringens that degrades tissue and ("gas gangrene") and hemolysis ("double zone"
• 12 cell membranes of hemolysis on blood agar)
• 13 Streptococcus Streptolysin 0 Protein that degrades cell Lyses RBCs; contributes to ~-h em olysis;
• 14 host antibodies against toxin (ASO) used to
pyogenes membrane
. 15
diagnose rheumatic fe,·er (do not confuse
. 16
with immune complexes of poststreptococcal
. 17
glomerulonephritis)
• 18
• 19 Superantigens causing shock
• 20 Staphylococcus Toxic shock Binds to .MI IC II and TC R Toxic shock syndrome: fever, rash, shock; other
. 21 aureus syndrome toxin outside of antigen binding toxins cause scalded skin syndrome (exfoliative
. 22 (TSS1 ~ 1 ) site to cause overwhelming to>. in) and food poisoning (heat-stable
• 23 enterotoxin)
release of IL-1, IL-2,
• 24
Streptococcus Exotoxin A IFN-y, and T N F-a Toxic shock-like svndrome: fe\'er, rash, shock;
• 25
pyogenes - shock scarlet fc,·cr
• 26
. 27 •
8
L.odt
s
Su~pl'nd
~
End Block
Item: 5 of 60 - ,• Mark --<) [::> ""'I ~· 1!';:'1
QIO: 3894 ~ Prev1o u s Next Labf a lu es Note s Calculator

1 • •
FA17 p 134.2
2
Clostridia (with Gram Ei1, spore-forming, obligate anaerobic rods.
3
4
exotoxins)
5 Ctetani Produces tetanospasmin, an exotoxin causing Tetanus is tetanic paralysis.
·6 tetanus. Tetanus toxin (and botulinum toxin)
· 7 are proteases that clea,·e SNARE proteins for
·8 neurotransmitters. Blocks release of inhibitory
·9 neurotransmitters, GABA and glycine, from
· 10 Renshaw cells in spinal cord.
· 11
Causes spastic paralysis, trismus (lockjaw), risus
• 12
sardonicus (raised eyebrows and open grin),
• 13
opisthotonos (spasms of spinal extensors).
• 14
. 15
Pre\ enl with tetanus vaccine. Treat'' ith
. 16
antitoxin +1- vaccine booster, diazepam (for
. 17
muscle spasms), and wound debridement.
• 18 Cbotulinum Produces a heat-labile toxin that inhibits Symptoms of botulism (the 4 D's): Diplopia,
• 19 ACh release at the neuromuscular june!ion, Dysarthria, Dysphagia, Dyspnea.
• 20 causi ng botulism. In adults, disease is caused Botulinum is from bad bottles of food, ju ice, and
. 21 by ingestion of preformed toxin. In babies, honey (causes a descending flaccid paralysis) .
. 22 ingestion of spores (eg, in honey) leads to l .oca I botox injections used to treat foca I
• 23 dyston ia, achalasia, and muscle spasms. Also
disease (flopp) baby syndrome). Treat with
• 24
antitoxin . used for cosmetic reduction offa cial wrinkles.
• 25
• 26
. 27
Cperfringens
- .. Produces a toxin (lecithinase, a phospholipase)
that can cause myonecrosis (gas gangrene )
Perfringens perforates a gangrenous leg.

8
L.odt
s
Su~pl'nd
~
End Block
Item: S of 60
QIO: 38M
- ,•
~
Mark -<J
P~v•ous
[:::>
N@xt Labl
"'I
lues
~ ·
No tes
~
Calcula t o r

1 •) •y p g ' •y p
2 causing botulism. In adults, disease is caused Botulinum is from bad bottles of food, juice, and
3 by ingestion of preformed toxin. In babies, honey (causes a descending flaccid paralysis).
4 ingestion of spores (eg, in honey) leads to Local botox injections used to treat focaI
5 disease (Aopp) baby s~ ndrome). Treat'' ith dystonia, achalasia, and muscle spasms. Also
· 6 antitoxin. used for cosmetic reduction of facial wrinkles.
· 7
C perfringens Produces a toxin (lecithinase, a phospholipase) Perfringens perforates a gangrenous leg.
·8
that can cause myonecrosis (gas gangrene )
•9
and hemolysis.
• 10
Spores can sun·i,·e in undercooked food;
· 11
when ingested, bacteria release heat-labile
' 12
' 13
enterotoxin ..... food poisoning.
' 14
' 15
. 16
. 17 c difflcile Produces 2 toxins. Toxin A, an enterotoxin, Dif{icile causes diarrhea. Treatment:
• 18 binds to brush border of gut and alters Auid metronidazole or oral vancomycin. For
' 19 secretion. Toxin B, a cytotoxin, disrupts recurrent cases, consider repeating prior
' 20 cytoskeleton via actin depolymerization. Both regimen, fidaxomicin, or fecal microbiota
. 21 toxins lead to diarrhea ..... pseudomembranous transplant.
. 22 zo
colitis B Often to antibiotic usc, cspccia lly
' 23 clindamycin or ampicillin ; associated with PPI
' 24 use. Diagnosed by detecting one or both toxins
' 25
in stool by antigen detection or PC R.
' 26
. 27 •
8
Lode.
s
Suspe-nd
8
End Bloc:k
Item: 6 of 60
QIO: 2056
- ,•
~
Mark -<J
P~v•ous
[:::>
N@xt Labl
"'I
lues
~ ·
No tes
~
Calcula t o r

A A
1
A 47-year-old man with a history of int rave nous drug use presents with a complaint of 2 days of acut e abdominal swelling and discomfort. ~~AI
2
His wife reports that his skin and eyes have taken on a yellow hue recently. Physical examination reveals a jaundiced patient with a flu id
3 wave on abdominal examination. His serum a-fetoprotein level is 550 ~/L (normal: 10-20 ~/ L).
4
5 The RNA virus responsible for this presentat ion belongs to which of the following families?
·6 :
·7 A. Deltavirus
·8 B. Flavivirus
•9
C. Hepadnavirus
• 10
· 11 D. Picornavirus
' 12 E. Retrovirus
' 13
' 14
' 15
. 16
. 17
• 18

' 19
' 20
. 21
. 22
' 23
' 24
' 25
' 26
. 27

8
Lode.
s
Suspe-nd
8
End Bloc:k
Item: 6 of 60 ~ ,• Mark <::J [:::> ""I ~· ~'j
QID: 2056 J.. Previous Next LAbfaiUI~S Notes Calculator

1
2
The correct a nswer is B. 53% chos e t his .
3
HCV is t ransmitted primarily via t he parenteral rout e and is more common in intravenous ( I V) drug users. I n a pat ient with hepatic cirrh osis,
4 t he acut e development of j aundice and ascit es in the sett ing of a markedly elevated serum o-fetoprotein level is highly suggestive of
5 hepatocellular carcinoma (HCC) . Bot h hepat it is B virus (HBV) and hepat it is C virus (HCV) are common causes of acute hepat it is, cirrhosis, and
HCC. However, the pat ient's history of IV drug abuse indicates that HCV, as opposed t o HBV, is the cause. A key difference between t hese t wo
6
viruses is t hat HBV is a DNA virus and HCV is an RNA v irus.
• 7 DNA virus Ascites Hepatocellular carcinoma Hepatitis C virus Jaundice Hepatitis C Hepatitis 8 Hepatitis 8 virus Cirrhosis Hepatitis Intravenous therapy Liver Blood plasma RNA virus Virus Parenteral RNA
•8 DNA Serum (blood) Substance abuse Carcinoma
•9
A is not correct . 5°/o chose this .
• 10
Hepatitis D virus (HDV) (delta agent ) is a defect ive virus (subviral satellit e) t hat requ ires the hepat it is B surface antigen as its envelope.
· 11 Sim ultaneous infection with HDV and HBV is known as co infect ion and resu lt s in f ulm inant liver fa ilure in 1% of pat ients. Complete clinical
• 12 recovery and cl earance of HBV and HDV coinfection is the most common out come . Chronic infection with HBV and HDV occurs in <5% of
patients .
• 13
Satellite (biology) Acute liver failure Hepatitis 8 Hepatitis Hepatitis 0 Virus Liver Antigen Hepatitis 8 virus Coinfection
• 14

• 15
C is not co rrect. 28% chos e this .
HBV is a hepadnavirus t ransmitt ed primarily by parenteral, sexual, and vertical routes. HBV is a DNA virus; it carries out reverse t ranscri ption
. 16
via a DNA-dependent DNA polymerase. Some patients never clear t he virus, leading to chro nic act ive hepatit is, cirrhosis, and HCC.
. 17 DNA virus HepadnCPJiridae Cirrhosis Reverse transcription DNA polymerase Hepatitis Transcription (genetics) DNA Hepatitis 8 virus Virus

• 18
D is not correct. 10% c hose this .
• 19
Hepatitis A virus ( HAV) is a picornavirus that is t ransmit ted pri marily by the feca l-oral rout e. I n the vast majority of patients, HAV infection is
• 20 self-limit ed and complet e recovery occurs. A chronic carrier state is not seen with HAV infection. Fu lminant hepatit is due to HAV is ra re and
. 21 has a case fata lity rate of 0.4% .
Picornavirus Fecal-oral route Hepatitis A Hepatitis A virus Hepatitis Virus Case fatality rate Acute liver failure Fulminant
. 22
• 23 E is not correct. 4 °/o chos e this .
• 24 HIV has not been shown to cause cirrhosis or HCC. Alt hough t his patient may have coinfection with HI V, his present ation is not typical of this
disease .
• 25 Cirrhosis HIV Coinfection
• 26
. 27

8
Lock
s
Suspend
0
End Block
Item: 6 of 60 ~ ,• Mark <::J [:::> ""I ~· ~'j
QID: 2056 J.. Previous Next LAbfaiUI~S Notes Calculator

1
Bottom Line:
2
I nt ravenous drug abusers are at high risk of contracting HCV (an RNA virus in the Flavivirus fa mily), which may lead to hepat itis, cirrhosis,
3
and hepatocellu lar ca rcinoma.
4 Hepatocellular carcinoma FICPJivirus Cirrhosis Hepatitis Intravenous therapy RNA virus Hepatitis C virus Drug injection Virus RNA

5
6
• 7 l@l;fil·1i•l for year:[ 2017
FIRST AID FACTS .
•j .
•8
•9
FA17 p 168.1
• 10
· 11 Hepatitis Signs and symptoms of all hepatitis viruses: episodes of fever, jaundice, t ALT and AST. Naked viruses (HAY
• 12 viruses and I lEV) lack an envelope and are not destroyed by the gut: the vowels hit your bowels.
• 13 HBV D lA polrmerase has D A- and R IA-dependent activities. Upon entry into nucleus, the polymerase
• 14 completes the partial dsDt A. Host Rt A polymerase transcribes mRNA from \·iral D lA to ma ke vira l
• 15 proteins. The D A polymerase then re\·erse transcribes viral R A to DNA, which is the genome of the
. 16 progeny v1rus.
. 17 HCV lacks 3'-5' exonuclease activity -+ no proofreading ability -+ variation in antigenic structures of HCV
• 18 envelope proteins. Host antibody production lags behind production of new mutant strains of HCV.
• 19
Virus H~ H~ HCV H~ HEV
• 20
. 21
FAMILY RNA picornavirus DNA hepadnavirus RNA Aavivirus Rl A deltavirus RNA hepcvirus
. 22 TRANSMISSION Feca l-oral (shell fish , Parenteral (Blood), Primarily blood Parenteral, sexual, Fecal-oral,
• 23 travelers, day care) sexual (Baby- (IVDU, post- perinatal especially
• 24 making), perinatal transfusion) waterborne
• 25 (Birthing)
• 26
INCUBATION Short (weeks) Long (months) Long Superinfection Short
. 27

8
Lock
s
Suspend
0
End Block
Item: 6 of 60
QIO: 2056
- ,•
~
Mark -<J
P~v•ous
[:::>
N@xt Labl
"'I
lues
~ ·
No tes
~
Calcula t o r

A
1 A
INCUBATION Short (weeks) Long (months) Long Superinfection Short
2
(llDV after
3
II BV) =short
4
Coinfection (HOV
5
with HB ) = long
6
· 7 CliNICAL COURSE \ symptomatic Initially like serum J\ lay progress to Similar to HBV Fulminant hepatitis
·8 (usually), Acute sickness (fever, C irrhosis or in Expectant
•9 arthralgias, rash); Carcinoma (pregnant) "omen
• 10 may progress to
· 11 carcmoma
• 12
PROGNOSIS Good Adults ..... mostly t\ lajority de,·elop Superinfection High mortality in
• 13
• 14
full resolution; stable, C hronic ..... worse prognOSIS pregnant women
• 15
neonates ..... worse hepatitis C
. 16
prognosis
. 17 HCC RISK I 0 Yes Yes Yes 0
• 18 LIVER BIOPSY Hepatocyte Granular Lymphoid Similar to 1-IBV Patchy necrosis
• 19
swelling, eosinophilic aggregates with
• 20
monocyte "ground glass" foca l areas of
. 21
in fi It ration, appearance; macrO\eSicuJa r
. 22
. 23
Councilman cytotoxic T cells steatosis
• 24
bodies mediate damage
• 25 NOTES No carrier state Carrier state Carrier state ,·ery Defecti,·c ,·irus, Enteric, Epidemic,
• 26 ("Alone") common common Depends on H BV no carrier state
. 27 • U U ~ &-

8
Lode.
s
Suspe-nd
8
End Bloc:k
Item: 6 of 60 ~ ,• Mark <::J [:::> ""I ~· ~'j
QID: 2056 J.. Previous Next LAb faiUI~S Notes Calculator

1 • •
FA17 p 163.1
2
RNA viruses
3
VIRAL FAMILY ENVELOPE RNA STRUCTURE CAPSID SYMMETRY MEDICAL IMPORTANCE
4
5 Reoviruses 0 OS linear Icosahedral Coltivirus"- Colorado tick fever
6 10-12 segments (double) Rotavirus-cause offatal diarrhea in children
• 7 Picornaviruses No SS ®linear Icosahedral Poliovirus-polio-Salk/Sabin vaccines-IPV/OPV
•8 Echovirus-aseptic meningitis
•9 Rhi novirus-"common cold"
• 10 Coxsackievirus-aseptic meningitis; herpangina
· 11 (mouth blisters, fever); hand, foot, and mouth
• 12 disease; myoca rditis; pericarditis
• 13 IIAV-acute viral hepatitis
• 14
PERCI I
• 15
Hepevirus No SS ®linear Icosahedral HEV
. 16
. 17 Caliciviruses No SS ® linear Icosahedral 'orovirus-vira l gastroenteritis
• 18
Flaviviruses Yes SS ® linear Icosahedral HCV
• 19
Yellow fever1
• 20
Dengue3
. 21
St. Louis encephalitis3
. 22
West Tile virus3 (meningoencephalitis)
• 23
Zika virus
• 24
• 25 Togaviruses Yes SS ®l inear Icosahedral Rubella
• 26 Western and Eastern equine encephalitis"
• Chikum!llnva ,·irus •
. 27

8
Lock
s
Suspend
0
End Block
Item: 6 of 60
QIO: 2056
- ,•
~
Mark -<J
P~v•ous
[:::>
N@xt Labl
"'I
lues
~ ·
No tes
~
Calcula t o r

A
1 A

Retroviruses Yes SS ® linear Icosahedral ! lave re,·erse transcriptase


2
2 copies (HTLV), HTLV-T-cellleukemia
3
complex II IV-AIDS
4
and conical
5
(II IV)
6
· 7 Coronaviruses Yes SS ® linear Helical "Common cold," SARS, \ IERS
·8
•9
Orthomyxoviruses Yes ss e linear Helical lnAuenza virus
8 segments
• 10
· 11
Paramyxoviruses Yes ss e linear Helical PaRaMno,·irus:

• 12 1 onsegmented Parain Aucnza-croup
• 13 RSV-bronchiolitis in babies; R\:-ribavirin
• 14 \tcaslcs, \lumps
• 15 Rhabdoviruses Yes ss e linear Helical Rabies
. 16
. 17
Filoviruses Yes ss e linear Helical Ebolall\ farburg hemorrhagic fever- often fatal!
• 18 Arenaviruses Yes ss ®a nd e f felica l I.CMV- lymphocytic choriomeningitis virus
• 19 circular Lassa fever encephalitis-spread by rodents
• 20 2 segments
. 21
. 22
Bunyaviruses Yes ss e circular Helical Ca Iifornia encephal itis 3
3 segments S<llldny/Rift Valley fevers 3
. 23
Crimean-Congo hemorrhagic fe,·e~
• 24
llantavirus- hemorrhagic fe\'er, pneumonia
• 25
• 26 Delta virus Yes ss e circular Uncertain HDV is a "defective'' ,-irus that requires the
. 27 • presence of II BV to rep Iicate
8
Lode.
s
Suspe-nd
8
End Bloc:k
Item: 6 of 60
QIO: 2056
- ,•
~
Mark -<J
P~v•ous
[:::>
N@xt Labl
"'I
lues
~ ·
No tes
~
Calcula t o r
• • • - ~ - w .. •

A A
1
3 segments $;111d Oy/Ri ft Valley fevers 3
2
Crimean-Congo hemorrhagic fe,·e~
3
llantavirus-hemorrhagic fever, pneumonia
4
5 Delta virus Yes sse circular Uncertain HDV is a "defective'' ,-irus that requires the
6 presence of HBV to replicate
· 7
SS, single-stranded: OS, double-stranded;®, positi,·e sense; e. negative sense; 3 = arJX>,·ims, arthropod borne (mosquitoes, ticks).
·8
•9
• 10 FA17 p 375.1
· 11
Hepatocellular Most conunon 1o malignant tumor of liver
• 12
carcinoma/hepatoma in adults . Associated with II BV (+/-
• 13
cirrhosis) and all other causes of cirrhosis
• 14
(including HC , alcoholic and nonalcoholic
• 15
fatty liver disease, autoimmune disease,
. 16
. 17
hemochromatosis, a 1-antit rypsi n deficiency)
• 18
and specific carcinogens (eg, aflatoxin
• 19 from Aspergillus). May lead to Budd-Chiari
• 20 syndrome.
. 21 Findings: jaundice, tender hcpatomcga ly,
. 22 ascites, polycythemia, anorexia. Spreads
. 23 hematogenously.
• 24 Diagnosis: t a-fetoprotein; ultrasound or
• 25 contrast CT/MRI , biopsy.
• 26
. 27 •
8
Lode.
s
Suspe-nd
8
End Bloc:k
Item: 7 of 60
QIO: 2 108
- ,•
~
Mark -<J
P~v•ous
[:::>
N@xt Labl
"'I
lues
~ ·
No tes
~
Calcula t o r

1
A 54-year-old man with poorly controlled diabetes mellitus is brought to the emergency department ( ED) by emergency medical services.
2
He was unconscious and hypotensive when fou nd, but in t he ED he is alert and complains of severe abdominal pain and shortness of
3 breath . The patient is also febri le and has sinus tenderness with erythema in the paranasal sinus, and a bloody discharge around his nose.
4 He states that he has had double vision for t he past 3 days. Physical examination shows t hat he cannot move his left eye in any direction. The
physician is concerned that the pat ient , in additi on to diabetic ketoacidosis, has ophthalmic sympt oms consistent wit h an infectious process.
5
6
Which of the following is the most likely cau sal organism?
·7
:
·8
A. Aspergillus species
•9
• 10 B. Blastomyces dermatitidis
· 11 C. Cryptococcus neoformans
• 12
D. Histoplasma capsulatum
• 13
• 14
E. Mucor species
• 15
. 16
. 17
• 18
• 19
• 20
. 21
. 22
. 23
• 24
• 25
• 26
. 27

8
Lode.
s
Suspe-nd
8
End Bloc:k
Item: 7 of 60 ~ ,• Mark <::J [:::> ""I ~· ~'j
QID: 2108 J.. Previous Next LAb faiUI~S Notes Calculator

1
2 The correct a nswer is E. 68°/o chose this .
3 This diabetic patient wit h sinusit is as well as a palsy involving several cranial nerves has sympt oms
suggest ive of invasive zygomycosis f rom Mucor species. Mucor species are molds with irregular nonseptate Image cowtesy of Yale Rosen,
4
hyphae, branching at wide angles ( > 90 degrees), as shown in t he image. I nvasive infect ion with t hese MD
5 ubiquitous fungi is most ly seen in pat ients wit h diabet ic ket oacidosis and t hose who are immunosu ppressed
6 (eg, because of hemat ologic ma lignancies) . Surgical debridement and high doses of amphot ericin B are used to t reat m ucormycosis, but
mortality is nevertheless very high.
7
Diabetic ketoacidosis Amphotericin B Zygo,.,cosis Mucormycosis Fungus Debridement Sinusitis Cranial nerves Immunosuppression Ketoacidosis Hypha Mucor Diabetes mellitus Malignancy Cancer
•8
•9
A is not correct . 9°/o chose t his .
Aspergillus fumigatus can cause bronchopulmonary aspergillosis, lung cavity aspergillomas ("fungus balls" ), and invasive aspergillosis. A.
• 10
fumigatus is a mold wit h septate hyphae that branch at a V-shaped 45-degree angle. A. fumigatus is not dimorphic.
· 11 Aspergillus fumigatus Aspergillosis Aspergillus Hypha Septum Mold Lung Septate Polymorphism (biology)

• 12
B is not correct. 6% chose t his .
• 13
Blastomycosis can manifest wit h f lu-like symptoms, feve rs, chills, productive cough, myalgia, arthra lgia, and pleurit ic chest pain. Some
• 14 patients will fail t o recover from an acute infection and prog ress t o develop chronic pulmonary infect ion or widespread disseminat ed infection .
• 15 Fluconazole or ketoconazole is used to t reat m ild pulmonary blast omycosis; amphotericin B is used t o t rea t severe pulmonary disease or for
systemic infect ions .
. 16 Arthralgia Amphotericin 8 Ketoconazole Blastomycosis Fluconazole Myalgia Pleurisy Cough Influenza-like illness Chills Chest pain Infection Acute (medicine)
. 17
C is not correct. 10% chose this .
• 18
Cryptococcus neoformans infect ion does not often produce symptoms in an immunocom petent host. However, in an immunocom promised
• 19 individual, it can manifest with meningoencephalit is. Cryptococcus is a heavily encapsulated yeast t hat is not dimorp hic. The f ungus is fo und
• 20 in soil and pigeon droppings .
Cryptococcus neoformans Immunodeficiency Cryptococcus Yeast Fungus Immunocompetence Infection Meningoencephalitis
. 21
. 22 D is not correct. 7°/o chose this .
• 23 Hist oplasmosis most oft en does not lead to any sympt omat ic presentat ion. When sym pt oms are present , t hey usually constit ut e a f lu- like
• 24
illness with feve r~ cough, headaches, and myalgias. Histoplasmosis can result in a lung disease resem bling tuberculosis and widespread
disseminat ed infection affect ing t he liver, spleen, adrenal glands, mucosal surfaces and meninges. Histoplasmosis occurs most commonly in
• 25 t he Mississippi and Oh io river valleys.
• 26 Histoplasmosis Meninges Tuberculosis Spleen Liver Adrenal gland Cough Fever Lung Mucous membrane

. 27

8
Lock
s
Suspend
0
End Block
Item: 7 of 60 ~ ,• Mark <::J [:::> ""I ~· ~'j
QID: 2108 J.. Previous Next LAbfaiUI~S Notes Calculator

1
Botto m Line:
2
3
Cranial nerve palsies and sinusit is in a pat ient with diabet es are highly suggestive of infect ion wit h Mucor species.
Sinusitis Cranial nerves Diabetes mellitus Skull
4
5
6
141;fil·1i•J for yea r:[2017
FIRST AID FA CTS .
•j .
7
•8
•9 FA17 p 149.1
. 10 Opportunistic fungal infections
· 11 Candida albicans alba = white. Dimorphic; Forms pseudohyphae and budding yeasts at zooc fJ, germ tubes at
. 12 37°C rn.
• 13
Systemic or superficial fungal in Feclion. Causes ora l ~ and esophageal lh rush in
• 14
immunocompromised (neonates, steroids, diabetes, AIDS), vulvovaginitis (diabetes, use of
• 15
antibiotics), diaper rash, endocarditis (IV drug users), disseminated candidiasis (especially in
. 16
neutropenic patients), chronic mucocutaneous candidiasis .
. 17
Treatment: oral Auconazolc/topical azoic for vagina l; nystati n, fluconazole, or caspofungin for oral!
• 18
esophageal; fluconazole, caspofungin, or amphotericin B for systemic.
• 19
. 20 Aspergillus Septate hyphae that branch at 45° Acute Angle [!]. Produces con idia in radiating cha ins at end of
. 21 fumigatus conid iophore D.
. 22 Causes invasive aspergillosis in immunocompromised, patients with chronic granulomatous disease.
• 23 Can cause aspergillomas in pre-existing lung cavities, especially after T B infection .
• 24 Some species of Aspergillus produce AAaloxins (associated with hepatocellular carci noma).
• 25
• 26
Allergic bronchopulmonary aspergillosis (ABPA): hypersensitivity response associated with
. 27 asthma and cystic fibrosis; may cause bronchiectasis and eosinophilia.
8
Lock
s
Suspend
0
End Block
Item: 7 of 60 - ,• Mark --<) [::> ""'I ~· 1!';:'1
QIO: 2 10 8 ~ Prev1o u s Next Labf a lues Note s Calculator

1 • Allergic bronchopulmonary aspergillosis (ABPA): hypersensitivity response associated with


2 asthma and cystic ~brosis; may cause bronchiectasis and eosinophilia.
3
Cryptococcus 5-10 11m with narrow buckling. l lea\'ily encapsulated yeast. 1 ot dimorphic.
4
neoformans Found in soil, pigeon droppings. Acquired through inhalation with hematogenous dissemination
5
6
to meninges. Culture on Sabouraud agar. ll ighlighted \\ ith India ink (clear halo ) and
7
mucicarmine (red inner capsule ). Latex agglutination test detects polysaccharide capsular
·8
antigen and is more specific.
·9
Causes cryptococcosis, cryptococca lmeningitis, cryptococcal encephalitis ('"soap bubble" lesions
· 10 in brain), primarily in immunocompromi eel.
· 11 Treahnent: amphotericin B + Aucytosine followed by Auconazole for cryptococcal meningitis.
• 12 Mucor and Rhizopus Irregular, broad, nonseptate h~ phae branching at \\ide angles C).
• 13
spp. Mucormycosis. Causes disease mostly in ketoacidotic diabetic and/or neutropenic patients (eg,
• 14
leukemia). Fungi proliferate in blood vessel walls, penetrate cribriform plate, and enter brain .
. 15
Rhinocercbral, frontal lobe abscess; cavernous sinus thrombosis. Headache, facial pain, black
. 16
necrotic eschar on face; may have cranial nerve involvement.
. 17
Treatment: surgical debridement, amphotericin B.
• 18
• 19
• 20
. 21
. 22
• 23
• 24
• 25
'
• 26
. 27 •
8
L.odt
s
Su~pl'nd
~
End Block
2
3
4

5
6
7
·8
·9 D
· 10
· 11 FA17 p 502.2
• 12 Facial nerve lesions
• 13
• 14
Upper motor neuron Destruction of motor cortex or connection
. 15
lesion between motor cortex and facia l nucleus in r.u-
""*" •
c
ol
. 16
pons - contralateral paralysis of lower muscles contJ
. 17 of facia l expression. Forehead is spared due to
Con.coll<llbaf
• 18 its bilateral U li innervation . IrK\

Lower motor neuron Destruction of facia I nucleus or CN VII


lUI'~___--;.::
-
• 19 IJpp«
dMsron
• 20 lesion anywhere along its course - ipsi latera l lnron• lower
. 21 c-JI dMsron
paralysis of upper and lower muscles of 11010 ll''l
. 22
facial expression · , hyperaeusis, loss of taste llliOtt
• 23
sensation to anterior tongue.
• 24
When idiopathic (most commou), facia l ncn·c
• 25
palsr is called Bell palsy. Ma) also be caused
• 26
. 27
by Lrme disease, herpes simplex, herpes •
8
L.odt
s
Su~pl'nd
~
End Block
Item: 7 of 60
QIO: 2 108
- ,•
~
Mark -<J
P~v•ous
[:::>
N@xt Labl
"'I
lues
~ ·
No tes
~
Calcula t o r

A A
1 FA17 p 182.2
2
Bug hints (if all else CHARACTERISTIC ORGANISM
3
fails) Asplen ic patient (clue to smgica I splenectomy Encapsulated microbes, especially S ll i~
4
5 or autosplenectomy, eg, chronic sickle cell (S pneumoniae >> H inf/.uen;:ae type B >
6 disease) 1\ meningitidis)
7 Branching rods in oral infection, sulfur granules Actinomyces israelii
·8
•9
Chronic granulomatous disease Catalase (±) microbes, especiall) S aureus
• 10 "Currant jelly" sputum Klebsiella
· 11
Dog or cat bite Pasteurella multocida
• 12
• 13
Facial ner\'e palsy (typically bilateral) Borrelia burgdorferi (Lyme disease)
• 14 Fungal infection in diabetic or Mucor or Rhizopus spp.
• 15 immunocompromiscd patient
. 16
Health care provider HB (from needlestick)
. 17
• 18
Neutropenic patients Candida albiccms (systemic), Aspergillus
• 19 Organ transplant recipient CMV
• 20
PAS (±) TrofJheryma whipplei (Whipple disease)
. 21
. 22 Pediatric infection Haemophilus in{luen:w e (including epiglottitis)
. 23 Pneumonia in cystic fibrosis, burn infection Pseudomonas aeruginosa
• 24
Pus, emprema , abscess S aureus
• 25
• 26 Rash on hands and feet Coxsackie A ,·irus. Treponema pallidum,
. 27 Hickellsia rickettsii
8
Lode.
s
Suspe-nd
8
End Bloc:k
Item: 8 of 60
QIO: 2861
- ,•
~
Mark -<J
P~v•ous
[:::>
N@xt Labl
"'I
lues
~ ·
No tes
~
Calcula t o r

A A
1
A 22-year-old woman presents to the physician because of an erythematous rash on her hands and the soles of her feet. She is slightly ~~AI
2
febri le with swollen, nont ender submandibular, axillary, and inguinal lymph nodes. On questioning she reports having unprotected sex with
3 five partners within the past year. Several months ago she noticed a sing le, painless, and fi rm lesion on her labia that resolved without
4 treatment.
5
6
If treatment is not begun, which of the following is t his patient at increased risk of developing?
7 :
A. Aortic insufficiency
·8
•9 B. Migratory arthritis
• 10
C. Opportunistic infections
· 11
• 12
D. Transient monocular blindness
• 13 E. Upper right quadrant abdominal tenderness
• 14

• 15
. 16
. 17
• 18
• 19
• 20
. 21
. 22
. 23
• 24
• 25
• 26
. 27

8
Lode.
s
Suspe-nd
8
End Bloc:k
Item: 8 of 60
QIO: 2861
- ,•
~
Mark -<J
P~v•ous
[:::>
N@xt Labl
"'I
lu es
~ ·
No tes
~
Calcula t o r

A A
1
2
The correct answer is A . 65°/o chose this.
3
This patient's symptoms are explained best by secondary, or disseminated, stage syphilis.
4 Syphilis is a sexually transmitted disease caused by the spirochete Treponema pallidum. It is
5 characterized by fever, sore throat, malaise, a generalized maculopapular rash involving the
6
palms and soles (like that shown in the images), condylomata lata (wart-like papules on the
genitals), and generalized nontender lymphadenopathy. A preceding primary stage is
7 characterized by a painless, indurated ulcer or chancre at the site of inoculation. If left
8 untreated, syphilis can progress to a late stage that is characterized by end-organ damage to the
•9 central nervous system, aorta, or musculoskeletal system. Long-term Treponema infection can
destroy the vasa vasorum of the ascending aorta, leading to destruction and dilation of the
• 10
vessel. The distorted aorta consequently affects the patency of the aortic valve. Treatment with
· 11 penicillin is essential. Images courtesy of CDC/ Robert Sumpter
• 12 CondytornP"ta I-ta S... ual trans,... tted mfecbon Sptrochaete Treponema pal.•dum Pen•c•llm Syph1hs Central nervous system Maculopapular rash lymphadenopathy Chancre Malatse Cutaneou! cond•tlon

• 13 Ulcer (dermatology) Ner.ous system Sore throat Papule Aorta Ulcer fever Inoculation Genttal wart Human musculoskeletal system Peptic ulcer Rash Aortic valve

• 14
B is n ot correct . 14°/o chose this .
• 15
Gonorrhea is a sexually transmitted disease caused by the gram-negative diplococcus Neisseria gonorrhoeae, which lacks both a capsule and
. 16 the ability to fe rment maltose (in contrast toN. meningitides). N. gonorrhoeae infection predisposes t o pelvic inflammatory disease, urethritis,
. 17 pharyng itis, proctitis, oph thalmia neonat orum, and septic arthritis. While arthritis can be associated wit h secondary syphilis, mig ratory
arthrit is is more classically associat ed with gonorrhea .
• 18
Neonatal conjunctivitiS Pelvtc tnflammatory disease Sexually transmitted infection Gonorrhea Netsseria gonorrhoeae Syphths Maltose Septic arthritis Urethritis Gram-negative bacteria Pharyngitis Proct1t1s
• 19
Diplococcus Neisseria men1nQ1bd1s Ne1sseria Arthritis I nflammation
• 20
C is not correct . 6°/o chose this .
. 21
Opportunistic infection, especially by Pneumocystisjirovecii, Mycobacterium avium-intracellulare, John Cunningham (or JC) virus,
. 22 toxoplasmosis, or Cryptococcus species, is indicative of immunosuppression and/or AIDS. AIDS is the progression of HIV infection, caused by
. 23 a single-stranded RNA retrovirus enveloped wit h reverse transcriptase. AIDS can lead to a variety of symptoms, but this patient's presentation
• 24 is best explained by syphilis infection .
OpportuniStiC 1nfect1on Toxoplasmosis Retrovirus Reverse transcriptase Pneumocyst1s j1rovecii Syph1hs Immuno5uppress1on HIV/AIDS HIV Mycobacterium Virus Mycobactenum av1um-1ntracellulare 1nfect1on
• 25
Cryptococcu Pneu nocy5t15 pneumoma RNA Infection
• 26
. 27 D is not correct. 11 °/o chose this .

8
Lode.
s
Suspe-nd
8
End Bloc:k
Item: 8 of 60 ~ ,• Mark <::J [:::> ""I ~· ~'j
QID: 2861 J.. Previous Next faiUI~S
LAb Notes Calculator

1 D is n ot cor rect. 11% c h ose this.


2 Amauros is fugax, or t ransient monocular blindness, is a condition in which tempora ry blindness occurs in one eye due to lack of blood flow t o
3
t he retina. Atherosclerotic disease of t he intracranial and carot id vasculature is the most common cause of t his condit ion, and risk facto rs
include diabet es, hy pertension, hyperlipidemia, and coronary a1tery disease. The Argyll Robertson pupils result from unt reat ed tertiary
4 syphilis. They are bilaterally small (miot ic) and fail to constrict fwther in response t o light , bu t do demonst ra te normal const rict ion to
5 accommodation.
Amaurosis fugax syphilis Coronary artery disease Retina Hyperlipidemia Hypertension Diabetes mellitus Miosis Amaurosis Atherosclerosis Visual impairment Circulatory system Blood flow
6
7 E is n ot correct. 4 °/o ch ose thi s.
8 Fit z-Hugh-Curtis syndrome is not a complication of long-term syphilis. It is due to adhesions caused by a chlamydia! or gonococcal infection
•9 t hat has spread t o t he f ibrous capsule of t he liver. This perihepat it is manifests wit h ri ght upper quadrant or bilateral upper abdominal
t enderness .
. 10 Fitz-Hugh-Curtis syndrome Syphilis Perihepatitis Quadrant (abdomen) Liver Chlamydia infection Neisseria gonorrhoeae Adhesion (medicine) Gonorrhea Infection
· 11
• 12
Bottom Line:
• 13
• 14
Syphilis infections can lead t o aortic insufficiency due t o dest ruct ion of t he vasa vasorum .
Aortic insufficiency vasa vasorum Syphilis
• 15
. 16
. 17
l@ljl'il·1i•J for yea r:[ 2017 • J
FIRST AID FA CTS
• 18
• 19
. 20 FA17 p 143.1
. 21 Syphilis Caused by spirochete Treponema pallidwn.
. 22 Primary syphilis Local ized disease presenti ng with painless chancre f.J. If available, use dark-field microscopy to
• 23 visualize treponemes in fl uid from chancre Ill VORL ED in ~ 80%.
• 24
Secondary syphilis Disseminated disease with constitutional symptoms, maculopapu lar rash 1!1 (including palms [!]
• 25
• 26
and soles), condylomata lata 0 (smooth, moist, painless, wart-like wh ite lesions on genitals),
. 27
lymphadenopathy, patchy hair loss; also con fi rmable with dark-fi eld microscopy.
8
Lock
s
Suspend
0
End Block
Item: 8 of 60
QIO: 2861
- ,•
~
Mark -<J
P~v•ous
[:::>
N@xt Labl
"'I
lues
~ ·
No tes
~
Calcula t o r

A A
1 FA17 p 143.1
2 Syphilis Caused by spirochete Treponema pallidum .
3

4
Primary syphilis Localized disease presenting with pain less chancre · . If available, use dark-fi eld microscopy to
5
visualize treponemes in Auid from chancre : . VORL Ef> in ~ 80%.
6 Secondary syphilis Disseminated disease '' ith constitut ional S) mptoms, maculopapular rash 1!1 (including palms
7 and soles), condylomata lata (smooth, moist, painless, wart-like white lesions on genitals),
8 I) mphadenopathy, patchy hair loss; also confirmable '' ith dark-fi eld microscopy.
•9 Serologic testing: VDRURPR (nonspecific), confirm diagnosis with specific test (eg, FTA-ABS).
• 10
Secondaf) syphilis = Systemic. Latent syphilis (Ef.> serology without S) mptoms) ma) folio''·
· 11
• 12
Tertiary syphilis Gummas D (chronic granulomas), aortitis (\·asa ,·asorum destruction), neurosyphilis (tabes dorsalis,
• 13
"general paresis"), Argyll Robertson pupil (constricts with accommodation but is not reactive to
• 14 light; also called "prostitute's pupil" since it accommodates but does not react).
• 15 Signs: broad-based ataxia, Ef> Romberg, Charcot joint, stroke without hypertension.
. 16 For neurosrphil is: test spinal Auid with VORL, F'TA-ABS, and PCR.
. 17 Congenital syphilis Presents with facial abnormalities such as rhagades (linear scars at angle of mouth, black arrO\\
• 18
in ~). snufAes (nasal discharge, red arro'' in ~). saddle nose, notched (Hutchinson) teeth CJ.
• 19
mulberry molars, and short maxi lla; saber shins; C VIII deafness.
• 20
To prevent, treat mother early in prcgm111cy, as placental transmission typically occurs after fi rst
. 21
trimester.
. 22
. 23
• 24
• 25
• 26
. 27

8
Lode.
s
Suspe-nd
8
End Bloc:k
Item: 8 of 60 - ,• Mark --<) [::> ""'I ~· 1!';:'1
QIO: 2861 ~ Prev1o u s Next Labf a lues Notes Calculator

1
- -
For neurosyphilis: test spinal fluid \\·ith VORL, FTA-ABS, and PCR.
2
3
Congenital syphilis Presents with facial abnormalities such as rh;1gades (linear scars at angle of mouth, black arrO\\
4
in [!!), snuffl es (nasal discharge, red arrO\\ in [!!), saddle nose, notched (Hutchinson) Leeth CJ,
5 mulberry molars, and short maxilla; saber shins; C 1 Ill deafness.
6 To prevent. treat mother early in pregnancy, as placental transmission typically occurs after first
7 trimester.
8
·9
· 10
· 11
• 12
• 13
• 14
. 15
. 16
. 17
• 18
• 19
• 20
. 21
. 22
• 23
• 24
• 25
• 26
. 27

8
L.odt
s
Su~pl'nd
~
End Block
Item: 9 of 60
QIO: 1051
- ,•
~
Mark -<J
P~v•ous
[:::>
N@xt Labl
"'I
lues
~ ·
Notes
~
Calculator

1
A 29-year-old woman presents to the emergency departm ent with a 9- hour history of worsening letharg y and a t hrobbing headache. On
2
arrival the patient's temperature is 102.4°F. Her blood pressure is 102/68 mm Hg, and she has a heart rate of 97/min. Physical exam
3 reveals several small red spots on her trunk and arms that do not blanch with pressure. The patient has tense neck stiffness. A lumbar
4 puncture and blood cultures are initially positive for bacteria.
5
Opening wac (cells/mm,) Protein Glucose
pressure (mg/dL)
6 lmm H20l
A 200 50 (70% lymphocytes) 50 !
7
B 150 3 35 -
c ~tes)

1 12~
8 250 100(80%~m 54 !

I~I
·9 250 (90% PMmr 55
200 80
• 10
· 11
Using the laboratory findings in the table above, what analysis of the cerebrospinal fluid would you expect?
• 12
• 13
A
• 14
0
15
B
. 16 c
. 17
D
• 18
0 19
E
0
20
. 21
. 22
0
23
0 24
0
25
0
26
. 27

8
Lode.
s
Suspe-nd
8
End Bloc:k
Item: 9 of 60 ~ ,• Mark <::J [:::> ""I ~· ~'j
QID: 1051 J.. Previous Next LAb faiUI~S Notes Calculator

1
2
3
The correct answer is 0. 82°/o chose this.
This patient present s with typical symptoms of bact erial meningit is: fever, headache, and nuchal ri gidity. On physical examinat ion, she has the
4
petechial rash that is associated with infection by Neisseria meningitidis as well. Cerebrospinal f luid analysis will show an elevated opening
5 pressure, elevated prot ein, decreased glucose concentrat ion, and signif icant pleocytosis of WBCs wit h a predominance of neut rophils.
6 Neisseria meningitidis Cerebrospinal fluid Meningitis Pleocytosis Petechia Meningism Neutrophil Neck stiffness Headache Purpura Protein Rash Bacterial meningitis Glucose Physical examination Fever

7 A is not correct. 5°/o chose this.


8 This cerebrospinal f luid profile, wit h normal opening press ure, slight ly elevated WBC count, normal protein levels, and low-norma l glucose
9 would be most consistent with viral meningit is. The different ial would show a predom inance of lymphocytes.
Cerebrospinal fluid Meningitis Viral meningitis Protein Glucose Lymphocyte Virus
• 10
· 11 B is not correct. 3% chose this.
• 12 This is a normal cere brospinal fluid prof ile. I n a healt hy pat ient, CSF is expect ed to be acellular with low protein content.
Cerebrospinal fluid Protein
• 13
• 14 C is not correct. 9 °/o chose this .
• 15 This cerebrospinal f luid profile is most consist ent wit h fungual/tuberculous meningit is: elevat ed opening pressure, leukocytic pleocytosis, a
reduced glucose level, and an elevated protein level. The rash seen in this patient indicates an infection wit h Neisseria, not mycobacteria .
. 16 Cerebrospinal fluid Pleocytosis Meningitis Mycobacterium Protein Glucose White blood cell Rash Neisseria Infection
. 17
E is not correct. 1 °/o chos e this .
• 18
The CSF findings here show an acellula r f luid with elevated prot ein. These charact eri st ics are indicative of albuminocytologic dissociation and
• 19
are charact erist ic of Guillain -Barre syndrome (GBS) .
• 20 Protein Dissociation (chemistry)

. 21
. 22
Bottom Line:
• 23
I n bact erial men ingit is, cerebrospinal flu id analysis will show an elevated opening pressure, elevated prot ein, decreased glucose
• 24 concentration, and significant pleocyt osis of WBCs with a predom inance of neut rophils.
• 25 Cerebrospinal fluid Pleocytosis Meningitis Protein Neutrophil Bacterial meningitis Glucose

• 26
. 27

8
Lock
s
Suspend
0
End Block
Item: 9 of 60 ~ ,• Mark <::J [:::> ""I ~· ~'j
QID: 1051 J.. Previous Next LAbfaiUI~S Notes Calculator

1
2
3

4 FA17 p 176.2
5 CSF findings in meningitis
6 OPENING PRESSURE CELLTYPE PROTEIN GLUCOSE
7
Bact erial t t P II s t l
8
9 Fungai/TB t t lymphocytes t l
• 10 Viral orma 1/t t lymphocytes Norma lit Normal
· 11
• 12
• 13 FA17 p 176.1
• 14 Common causes of meningitis
• 15 NEWBORN (0- 6MOl CHILDREN (6 M0- 6 YR) 6- 60YR 60YR+
. 16 Group B streptococci S pneumoniae S pneumoniae S pneumoniae
. 17 E coli N meningitidis N meningiticlis (111 in teens) Gram 8 rods
• 18
Listeria H influenzae type B Enterovi ruses Listeria
• 19
Enterovi ruses HSV
• 20
. 21
Give ceftri axone and vancomycin empirically (add ampicill in if Listeria is suspected).
. 22
Viral causes of meningitis: enteroviruses (especially coxsackievirus), HSV-2 (HSV-1 = encephalitis), HIV, West ile virus (also
• 23 causes encephalitis), VZV.
• 24 In HIV: Cryptococcus spp.
• 25 ote: Incidence of H influenzae meningitis has l greatly due to conjugate H inflt~enzae vaccinations. Today, cases are usually
• 26 seen in unimmunized children .
. 27

8
Lock
s
Suspend
0
End Block
Item: 9 of 60
QIO: 1051
- ,•
~
Mark -<J
P~v•ous
[:::>
N@xt Labl
"'I
lues
~ ·
No tes
~
Calcula t o r

A A
1
FA17 p 138.2
2
3 Neisseria Gram 8 diplococci. Metabolize glucose \ leninGococci ferment .\ laltose and Glucose.
4 and produce lgA proteases. Contain Gonococci ferment Glucose.
5 Iipool igosaccharides (LOS) with strong
6 endotoxin acti\'ity. gonorrhoeae is often
7 intracellular (within neutrophils)
8
9
Gonococci Meningococci
• 10 o polysaccharide capsule Polysaccharide capsule
· 11
o maltose metabolized Maltose fermentation
• 12
• 13 o \'accine due to antigenic \'ariation of pilus Vaccine (type B \'accine not widely a\'ailablc)
• 14 proteins
0
15 Sexually or peri natally transmitted Transmitted via respiratory and oral secretions
. 16
Causes gonorrhea, septic arthritis, neonatal Causes meningococcemia with petechial
. 17
• 18
conjuncti,·itis (2-5 days after birth), pelvic hemorrhages and gangrene of toes rn.
0 19
inAammatory disease (PI D), and Fitz-llugh- meni ngitis, Waterhouse-Friderichsen
0
20
Curtis syndrome syndrome (adrenal insufficiency, fever, DIC,
. 21 shock)
. 22 Condoms l sexual transmission, erythromycin Rifampin, ciproAoxacin, or ceftriaxone
0
23 eye ointment prevents neonatal blindness prophylaxis in close contacts
0 24
Treatment: ceftriaxone + (azithromycin Treatment: ceftriaxone or penicillin G
0
25
or doxycycline) for possible chlamydia!
0
26
. 27
coinfection
8
Lode.
s
Suspe-nd
8
End Bloc:k
Item: 10 of 60 - ,• Mark -<J [:::> "'I ~ · ~
QIO: 1860 ~ P~v•ous N@xt Labl lues No tes Calcula t o r

1
An HIV-positive man is admitted to the hospital wit h left-sided weakness and paresthesias. History reveals t hat the patient suffers from
2
seizures and a recent decline in neurologic functioning . An MRI shows numerous lesions throughout his white matter. Despite his
3 physician's best efforts over the next 8 months, the patient continu es to exhibit neurologic decline and dies.
4
5 Which of the following is the most likely cause of this patient's death?
6 :
7 A. BK virus infection
8 B. Cytomegalovirus infection
9
C. Epstein-Barr virus infection
• 10
· 11 D . JC virus infection
• 12 E. Lyme infection
• 13
• 14
0
15
. 16
. 17
• 18
0 19
0
20
. 21
. 22
0
23
0 24
0
25
0
26
. 27

8
Lode.
s
Suspe-nd
8
End Bloc:k
Item: 10 of 60 - ,• Mark -<J [:::> "'I ~ · ~
QIO: 1860 ~ P~v•ous N@xt Labl lues No tes Calcula t o r

A A
1
2
Th e correct an sw er is D. 77°/o chose this.
3
This is a case of progressive multifocal leukoencephalopathy (PML) associated with react ivati on of the
polyomavirus JC (JC virus) in an immunosuppressed patient. PML occurs predominantly in individuals who
4 are immunocompromised secondary to immunosuppressants after an organ transplantation, but also
5 following chemotherapy or HIV infection. JC virus in immunocompetent individuals usually remains a
subclinical infection.
6
7 Asymptomatic primary infection with JC virus can happen in childhood, and the antibodies can be found in a
majority of individuals. It remains latent but can be reactivated in the setting of immunosuppression, and it
8
can spread to the brain and cause an infection of oligodendrocytes.
9
PML lesions are usually located in the white matter (as indicated by the arrows in the image) and can
10 manifest as a progressive neurologic illness including mental status changes, motor deficits, ataxia, and
· 11 visual symptoms. Often, it can lead t o death in 6 months to 1 year from the time of reactivation.
• 12 The gold standard for diagnosis is brain biopsy; however, if that is not possible, a thorough evaluation of
• 13 risk factors, history, neuroimaging, and a cerebrospinal fluid JC virus polymerase chain reaction should be
checked .
• 14
Progr6$1Ve mulbfocal leukoencephalopathy JC virus Cerebrospinal fluid Polymerase chain react1on Immunodeficiency Immunosuppression Chemotherapy
0
15
Wh1te matter HIV Subchn1cal mfect1on .tta>Cia Oligodendrocyte Organ transplantation B1opsy Antibody Polyomav1r1dae Neuro1magmg Leukoencephalopathy
. 16
. 17
Human brain Immunocompetence Asymptomatic Immunosuppressive drug Virus Bra1n b1opsy Bra1n Infection Neurology Polymerase Image copyright © 2013 Lind~
and von Heijne
• 18
0 19 A is n ot correct . 4 °/o chose this.
0
20 BK virus usually produces subclinical kidney infect ions and is not associated with neurologic illness. BK virus nephropat hy is often encountered
. 21
in renal allograft patient s and may clinically manifest as ureteral stenosis or tubulointerstit ial nephritis (TI N) .
Nephritis Interstitial nephritiS BK v1rus Kidney disease Stenosis Asymptomatic Kidney V1rus Pyelonephr1t1s
. 22
0
23
B is not correct. 11 °/o chose this.
0 24
Cytomegalovirus (CMV) is an important cause of morbidity and mortality in transplant recipients. Without prophylaxis, CMV reactivation occurs
in approximately 50%-60% of patients. CMV can manifest with a variety of symptoms, most commonly fevers, leukopenia,
0
25 thrombocytopenia, malaise, ret initis, and arthralgias. CMV manifests less frequently with gast roenteritis, hepatitis, pneumonia, and
0
26 encephalitis. CMV is definitely in the differential for neurologic symptoms in an immunocompromised host , but t he whit e matter lesions
. 27
described are more characteristic of progressive mult ifocal leukoencephalopathy. CMV would be more likely to have periventricular and

8
Lode.
s
Suspe-nd
8
End Bloc:k
Item: 10 of 60 ~ ,• Mark <::J [:::> ""I ~· ~'j
QID: 1860 J.. Previous Next LAb faiUI~S Notes Calculator

1
B is not correct. 11 % chose this .
2
Cytomegalovirus (CMV) is an importan t cause of morbidity and mortality in t ransplant recipient s. Wit hout prophylaxis, CMV react ivat ion occurs
3 in approximat ely 50%- 60% of pat ients. CMV can manifest with a variety of sym ptoms, most commonly feve rs, leukopenia,
4 t hrombocyt openia, malaise, ret init is, and arthra lgias. CMV manifest s less frequent ly with gast roent eri t is, hepat it is, pneumonia, and
encephalit is. CMV is definitely in the different ial for neurologic symptoms in an immunocompromised host , but the white matt er lesions
5
descri bed are more characterist ic of progressive multifocal leukoencephalopathy. CMV would be more likely t o have peri vent ri cular and
6 meningeal enhancement.
7 Progressive multifocal leukoencephalopathy Cytomegalovirus Leukopenia Thrombocytopenia Gastroenteritis Pneumonia White matter Encephalitis Malaise Arthralgia Morbidity Hepatitis Immunodeficiency

8 Meninges Preventive healthcare Retinitis Neurology

9 C is not co rrect. 5 °/o chose this .


10 Epstein-Barr virus ( EBV) is a herpesvirus that causes infect ious mononucleosis; it is also associat ed with Burkitt lymphoma. Alt hough EBV is
· 11 associat ed with postt ransplantation lym phoprolife rative disease, t he physician would not expect neurologic f indings. I n addition, primary CNS
lymphoma is quite rare with an incidence of 3 .1% of all primary brain tumors .
• 12
Infectious mononucleosis Burkitt's lymphoma Epstein-Barr virus Primary central nervous system lymphoma Lymphoma Virus Herpesviridae Lymphoproliferative disorders Brain tumor Central nervous system
• 13
Kaposi's sarcoma-associated herpesvirus Neoplasm Neurology
• 14

• 15
E is not correct. 3°/o chose this .
The pat ient does not have any known exposures t hat would put him at ri sk for a Lyme infect ion. Alt hough Lyme can have neurologic
. 16
manifestat ions such as meningit is, cranial neuropathy, and mot or or sensory radiculoneuropathy, it likely would not cause t he white matter
. 17 changes and rapid cl inical decline seen in t his pat ient .
• 18 Meningitis White matter Neurology Peripheral neuropathy Neurological disorder Infection

• 19
• 20 Bottom Line :
. 21
PML is a demyelinat ing disease of the cent ral nervous system t hat manifest s secondary to JC virus reactivation in immunocompromised
. 22 patients (eg, st at us postt ransplant, AIDS) .
JC virus oe,.,elinating disease Central nervous system Immunodeficiency HIV/AIDS Nervous system Progressive multifocalleukoencephalopathy Virus Myelin
• 23
• 24

• 25
• 26
141;fil·1i•J for yea r:[2017 •
FIRST AID FA CTS

. 27

8
Lock
s
Suspend
0
End Block
Item: 10 of 60 - ,• Mark --<) [::> ""'I ~· 1!';:'1
QIO: 1860 ~ Prev1o u s Next Labf a lu es Notes Calculator

1 •
FA17 p 494.1
2
Other d emyelinating and dysmyelinating diseases
3
4 Acute disseminated Multifocal inflammation and demyelination after infection or \'accination. Presents with rapidly
5 (postinfectious) progressi\·e multifocal neurologic symptoms, altered mental status.
6 encephalomyelitis
7 Charcot-Marie-Tooth Also kno\m as hereditary motor and sensory neuropathy ( ll~ lSN). Group of progressi\'e hereditary
8 disease nerve disorders related to the defecti\'e production of proteins in\'oh-ed in the structure and
9
function of peripheral nen·es or the myelin sheath. Typically autosomal dominant inheritance
10
pattern and associated with foot deformit ies (eg, pes can1s, hammer toe), lower extremity
· 11
weakness (eg, foot drop) and sensory defi cits.
• 12
• 13 Krabbe disease Autosomal recessive lysosomal storage disease due to defi ciency of galactocerebrosidase. Buildup
• 14 of galactocerebroside and psychosine destroys myel in sheath. Findings: peripheral neuropathy,
. 15 developmental delay, optic atroph)', globoid cells.
. 16 Metachromatic Autosomal recessi\·e lysosomal storage disease, most commonly due to arylsulfatase A deficiency.
. 17
leukodystrophy Buildup of sulfatides -- impaired production and destruction of myelin sheath. Findings: central
• 18
and peripheral demyelination with ataxia, dementia .
• 19
• 20
Progressive multifocal Demyelination of C S fJ clue to destruction of oligodend rocytes. Seen in 2-4% of AIDS patients
. 21 leukoencephalopathy (reactivation of latent JC virus in feet ion). Rapidly progressive, usually fatal. f risk associated with
. 22 natalizumab, rituximab.
• 23
• 24
• 25
• 26
. 27 •
8
L.odt
s
Su~pl'nd
~
End Block
Item: 10 of 60 ~ ,• Mark <::J [:::> ""I ~· ~'j
QID: 1860 J.. Previous Next LAb faiUI~S Notes Calculator

1 • •
FA17 p 160.1
2
DNA viruses
3
VIRAL FAMILY ENVELOPE ONA STRUCTURE MEDICAL IMPORTANCE
4
5
Herpesviruses Yes OS and linear See Herpesviruses entry
6 Poxvirus Yes OS and linear Smallpox eradicated world wide by use of the live-
7 (largest 0 A virus) attenuated vaccine
8 Cowpox ("milkmaid blisters")
9 Molluscum contagiosum -Aesh-colored papule with
10 centra I umbilication
· 11
• 12
Hepadnavirus Yes Partially OS and circular HBV:
• 13
Acute or chron ic hepatitis
• 14
Not a retrovirus but has reverse transcriptase
• 15 Adenovirus 0 OS and linear Febrile pharyngitis r.J-sore throat
. 16 Acute hemorrhagic cystitis
. 17 Pneumonia
• 18 Conjunctivitis-"pink eye"
• 19
• 20
. 21
. 22
• 23
• 24
Papillomavirus 0 OS and circular HPV-warts (serotypes 1, 2, 6, 11), Cl , cervical
• 25
cancer (most commonly 16, 18)
• 26
• Polvomavirus No OS and circular IC virus-oro!!rcssivc multifoca lleukocnceohalooathv •
. 27

8
Lock
s
Suspend
0
End Block
Item: 11 of 60
QIO: 20 15
- ,•
~
Mark -<J
P~v•ous
[:::>
N@xt Labl
"'I
lues
~ ·
No tes
~
Calcula t o r

A A
1
A 5-year-old girl visiting from Nicaragua is brought to the emergency department by her aunt because of a sore t hroat and general malaise ~~AI
2
for the past 3 days. Physical examination reveals t emperat ure of 38° C ( 100.4° F) and a grayish-white membrane on t he pharynx that
3 bleeds on attempted dislodgement.
4
5 Which of the following is the most appropri at e cult ure media for diagnosing this patient's infection?
6 :
7 A. Bordet-Gengou agar
8 B. Chocolate agar with factors V and X
9
C. Loeffler medium
10
· 11 0 . Sabouraud agar
' 12 E. Thayer-Martin agar
' 13
' 14
' 15
. 16
. 17
• 18

' 19
' 20
. 21
. 22
' 23
' 24
' 25
' 26
. 27

8
Lode.
s
Suspe-nd
8
End Bloc:k
Item: 11 of 60
QIO: 20 15
- ,•
~
Mark -<J
P~v•ous
[:::>
N@xt Labl
"'I
lues
~ ·
No tes
~
Calcula t o r

A A
1
The correct answer is c. 58°/o chose this .
2
This patient likely has diphtheri a, an infection caused by t he gram-positive rod Co1ynebacterium
3 diphtheriae. Diphtheria classically presents with a grayish-white pseudomembrane on the pharynx or t onsils
4 ( like that shown in the image). This pseudomembrane should not be disrupted so as to avoid increased
5
absorption of the lethal exotoxin. Fever is usually mild or absent. It is seen very rarely in vaccinated
populations but is endemic to certain parts of t he world. Typically Loeffler medium and tellurite agar are
6 used in conjunction for the diagnosis of C. diptheriae infection. Growth of Cornyebacterium spp on Loeffler
7 medium appear as minute cream-colored colonies. This helps preferentially grow C. diptheriae. Tellurite
8
agar is used for further identification . Tinsdale agar can be used to differentiate C. diphtheriae and related
strains from other Corynebacterium spp by the formation of brown to black colonies. Black colonies are
9 formed as a result of potassium tellurite being reduced to metallic tellurite within the colonies. Colonies will
10 become gray to black within 24 hours.
CoNnebKt:MI 1m 1phth~1ae Olphthena Gram-pos•tive bacteria Exotoxin Pharynx Corynebacterium AQar TeiJunte
11

' 12
' 13
' 14
' 15
. 16
. 17
• 18 I mage courtesy of Wikimedia
' 19 Commons
' 20
A is not co rrect . 12°/o chose thi s .
. 21
Bordet-Gengou agar is used to culture Bordete/la pertussis. Pertussis presents with paroxysmal coughing spells and whooping sounds on
. 22 inspiration .
Bordetella pertuSSIS Bordet·Gengou agar PertusSis Bordetella Agar Cough
' 23
' 24 B is not co rrect . 12°/o chose t his.
' 25 Chocolate agar is used to grow Haemophi/us influenzae. Encapsulated strains of H. influenzae cause invasive diseases such as septicemia,
' 26 meningitis, cellulitis, septic arthritis, epiglottitis, and pneumonia . Nonencapsulated strains are likely to cause otit is media, conjunctivitis,
bronchitis, and sinusitis .
. 27

8
Lode.
s
Suspe-nd
8
End Bloc:k
Item: 11 of 60 ~ ,• Mark <::J [:::> ""I ~· ~'j
QID: 2015 J.. Previous Next LAbfaiUI~S Notes Calculator

1
D is not correct. 9°/o chose this.
2 Sabouraud agar is used t o grow fung i.
3 Sabouraud agar Fungus Agar

4
E is not correct. 9 °/o chose this.
5 Thayer-Mart in agar is a chocolat e agar plat e containing VCN antibiot ics (va ncomycin, colistin, and nyst at in) t hat suppress the growth of
6 endogenous f lora while support ing Neisseria gonorrhoeae growt h. This pat ient does not have sym ptoms of gonorrh ea.
Nystatin Colistin Gonorrhea Neisseria gonorrhoeae Chocolate agar Agar plate Thayer-Martin agar Agar Neisseria Antibiotics
7
8
9 Bottom Line:
10 A patient wit h pharyngit is and a pseudomembrane on t he pharynx is likely infected wit h Co1ynebacterium diphtheriae, which is best cult ured
11 on a Loeff ler med ium.
Corynebacterium diphtheriae Pharyngitis Pharynx Corynebacterium
. 12
• 13
• 14

• 15
141;fil·1i•J for year:[ 2017 •
FIRST AID FA CTS

. 16
. 17 FA17 p 123.1
• 18 Special culture requirements
• 19 BUG MEDIA USED FOR ISOLATION MEDIA CONTENTS/OTHER
. 20
H influenzae Chocolate agar Factors V (NAD+) and X (hematin)
. 21
. 22
N gonorrhoeae, Thayer-Marti n agar Selectively favors growth of J eisseria by
• 23 N meningitidis inhibiting growth of gram EEl organisms
• 24 with Vancomycin, gram 8 organ isms except
• 25 Neisseria with Trimethoprim and Colistin,
• 26 and fu ngi with 1\ystatin
. 27 V P r \.1 T \/nl f""l II" r .. 1+-nn')(' V oi<'<-nr-il"1
8
Lock
s
Suspend
0
End Block
Item: 11 of 60 - ,• Mark --<) [::> ""'I ~· 1!';:'1
QIO: 2015 ~ Prev1o u s Next Labf a lues Notes Calculator

1 • •
FA17 p 123.1
2
Special culture requirements
3
BUG MEDIA USED FOR ISOLATION MEDIA CONTENTS/OTHER
4
5 Hinfluenzae Chocolate agar Factors V (NAD+) and X (hematin)
6 N gonorrhoeae, Tha~e r-Ma rtin agar elect ivel) favors growth of I eisseria by
7 N meningitidis inhibiting growth of gram $ organisms
8
'' ith Vancomycin, gram 8 organisms except
9 'eisseria with Trimethoprim and Colistin,
10
and fungi with '\ystatin
11
\'cry Typically Cultures :'\eisseria
• 12
• 13 Bpertussis Bordet-Gengou agar (Bordct for Bordetelfa) Potato extract
• 14 Regan-Lowe medium Charcoal, blood, and antibiotic
. 15 Cdiphtheriae Tellurite agar, Lofner medium
. 16
M tuberculosis Lowcnstein-Jcnscn agar
. 17
• 18 M pneumoniae Eaton agar Requires cholesterol
• 19 Lactose-fermenting lacConkey agar l'crmenlalion produces acid, causing colonies to
• 20 enterics turn pink
. 21
. 22
Ecoli Eosin-methylene blue (EM B) agar Colonies with green metallic sheen
• 23 Legionella Charcoal yeast extract agar buffered with
• 24 cysteine and iron
• 25
Fungi Sabouraud agar • ab'sa fun guy.I"
"S
• 26
. 27 •
8
L.odt
s
Su~pl'nd
~
End Block
Item: 11 of 60
QIO: 20 15
- ,•
~
Mark -<J
P~v•ous
[:::>
N@xt Labl
"'I
lues
~ ·
No tes
~
Calcula t o r

A A
1 FA17p 135.1
2
3
Corynebacterium Gram <±) rod; transmitted via respiratory Coryne =club shaped.
4
diphtheriae droplets. Causes diphtheria via exoto'l:in Black colonies on cystine-tellurite agar.
5 encoded by ~-proph age. Potent e;..otoxin \ BC DEFG:
6 inhibits protein synthesis via DP-ribosylation \ DP-ribosylation
7 of EF-2. ~ -proph age
8 Symptoms include pseudomembranous Cor)'nebacterium
9 pharyngitis (grayish-\\ hite membrane ) Diphtheriae
10 ,,·ith lymphadenopathy, myocarditis, and Elongation Factor 2
11 arrhythmias. Granules
' 12 Lab diagnosis based on gram <±) rods \\'ith
' 13 metachromatic (blue and red) granule~ and
' 14 <±) Elek test for toxin.
' 15
Toxoid vaccine prevents diphtheria .
. 16
. 17
• 18 FA17 p 128.1
' 19 Bugs with exotoxins
' 20 BACTERIA TOXIN MECHANISM MANIFESTATION
. 21 Inhibit protein synthesis
. 22
Corynebacterium Diphtheria toxin 3 Phar) ngitis with pseudomembranes in throat
' 23
diphtheriae Inactivate elongation factor and severe lymphadenopathy (bull neck)
' 24
' 25 Pseudomonas Exotoxin N (EF-2) llost cell death
' 26 aeruginosa
. 27 ~· . ~ -,....., ,-
~· • •
8
Lode.
s
Suspe-nd
8
End Bloc:k
Item: 11 of 60 ~ ,• Mark <::J [:::> ""I ~· ~'j
QID: 2015 J.. Previous Next faiUI~S
LAb Notes Calculator

1 • FA17 p 128.1 •
2 Bugs with exotoxins
3
BACTERIA TOXIN MECHANISM MANIFESTATI ON
4
Inhibit protein synthesis
5
6 Corynebacterium Diphtheria toxin3 Pharyngitis with pseudomembranes in th roat
7 diphtheriae Inactivate elongation factor and severe lymphadenopathy (bull neck)
8
Pseudomonas Exotoxin N (EF-2) Host cell death
9
aeruginosa
10
11
Shigella spp. Shiga toxin (ST)3 GI mucosal damage .... dysentery; ST also
• 12 Inactivate 60S ribosome by enhances cytokinc release, causi ng hemolytic-
• 13 removing adenine from uremic syndrome (HUS)
• 14 Enterohemorrhagic Shiga-like toxin rR A SLT en hances cytokine release, causing HUS
• 15 E coli (EHEC) (SLT)a (prototypically in EHEC serotype 0157:H7).
. 16 Unlike Shigella, EHEC does not invade host
. 17
cells
• 18
• 19
Increase flu id secretion
• 20 Enterotoxigenic Heat-labile Overactivates adenylate Watery diarrhea: "labile in the Air (Aclenylate
. 21 E coli (ETEC) toxin (LT)3 cyclase (t cAMP) .... t CI- cyclase), stable on the Ground (Guanylate
. 22 secretion in gut and H7 0 cyclase)"
• 23 efAux
• 24 I Ieat-stable Overactivates guanylate
• 25 toxin (ST ) cyclase (t cGMP)
• 26
- l resorption of NaCI
. 27 • . ...... -. •

8
Lock
s
Suspend
0
End Block
Item: 11 of 60 - ,• Mark --<) [::> ""'I ~· 1!';:'1
QIO: 2015 ~ Prev1o u s Next Labf a lues Notes Calculator

1
Clostridium Botulinum toxin 3 of proteins required for Flaccid paralysis, Aoppy baby; toxin prevents
2
3
botulinum neurotransmitter release release of stimulatory (ACh) signals at
4
via vesicular fusion neuromuscular junctions - Aaccid paralysis
5 3 An AB toxi n (aka, two-component toxin [or three for anthrax]} with B enabling binding and triggering uptake (endocytosis)
6 oft he act i' e A component. T he A components are usually ADP ribosylt ransferases; others ha\'e enzymatic activities as listed
7 in chart.
8
9
Lyse cell membranes
10 Clostridium Alpha toxin Phospholipase (lecithinase) Degradation of phospholipids - myonecrosis
11 perfringens that degrades tissue and ("gas gangrene") and hemolysis ("double zone"
• 12 cell membranes of hemolysis on blood agar)
• 13
• 14
Streptococcus Streptolysin 0 Protein that degrades cell l.yses RBCs; contributes to ~-h em olysis;
. 15
pyogenes membrane host antibodies against toxin (ASO) used to
. 16
diagnose rheumatic fe,·er (do not confuse
. 17 with immune complexes of poststreptococcal
• 18 glomerulonephritis)
• 19 Superantigens causing shock
• 20
Staphylococcus Toxic shock Binds to IIIC II and TC R Toxic shock syndrome: fever, rash, shock; other
. 21
aureus syndrome toxin outside of antigen binding toxins cause scalded skin syndrome (exfoliative
. 22
(TSST-1) site to cause overwhelming IOAin) and food poisoning (heat-stable
• 23
• 24 release of I L-1, I L-2, cnterotoxin)
• 25 Streptococcus Exotoxin A IF.N-y, and T N F-a Toxic shock-like smdrome: fe\'er, rash, shock;
• 26 pyogenes - shock scarlet b ·cr
. 27 •
8
L.odt
s
Su~pl'nd
~
End Block
Item: 12 of 60 - ,• Mark -<J [:::> "'I ~ · ~
QIO: 2046 ~ P~v•ous N@xt Labl lues Notes Calculator

A A
1
A 73-year-old woman is admitted t o the hospital because of an acute exacerbation of congestive heart fai lure. A cent ra l venous catheter is ~~AI
2
inserted on the second hospital day. On hospital day 6, the nurse notes that the patient can no longer reca ll her name and is confused
3 about where she is. The patient's t emperat ure is 38.9°C (102°F), pulse is 95 beats/min, respirations are 24 breaths/min, and blood
4 pressure is 130/90 mm Hg. Blood culture reveals a gram-negative rod that produces a red pigment.
5
6
Which of the following organisms is the most likely cause of this patient's infection?
7 :
A. Escherichia coli
8
9 B. Klebsiella pneumoniae
10 C. Proteus mirabilis
11
• 12
D. Serratia marcescens
• 13 E. Staphylococcus saprophyticus
• 14

• 15
. 16
. 17
• 18
• 19
• 20
. 21
. 22
. 23
• 24
• 25
• 26
. 27

8
Lode.
s
Suspe-nd
8
End Bloc:k
Item: 12 of 60 - ,• Mark -<J [:::> "'I ~ · ~
QIO: 2046 ~ P~v•ous N@xt Labl lues Notes Calculator

A A
1
The correct answer is 0. 67°/o chose this.
2
This patient likely has a nosocomial infection related to her cent ra l venous catheter (CVC), as indicated by
3 the change in mental status and her feve r wit hout another obvious cause. Serratia marcescens is t he most
4 likely culprit here, in light of the blood cult ure find ings: It is a gram-negative rod that produces red
5
pigment, like that shown in the image (think of red m a raschino cherries). Unlike the other bacteria here,
Serratia is a frequent cause of eve infections rat her than urinary catheter infections.
6 Hosp•tal·~ttQutred tectlon Ce,tJal venous catheter Serratia marcescens Blood culture c:;,-a.m..neg1f".Jve b~e 11 U 1n 'f cGtt etenzMJon Bacteria Catheter

7 AJtered level of consc•ousness Fever- Ptgment Infection Serratia


8
9
10
11

12
Image courtesy of CDC/Or.
• 13
Negut
• 14

• 15 A is not correct. 7 °/o chose this .


. 16 Escherichia coli is the most common cause of urinary t ract infection. It is a gram-negative rod, but it produces no pig ments .
Unnary tract tnfectlon Eschenchta coh Gram-negative bacteria Urinary system
. 17
• 18 B is not correct. 17°/o chose this .
• 19 Klebsiella pneumoniae is a gram-negat ive rod t hat is responsible for approximately 8% of nosocomial infections. It is a signif icant cause of
urinary tract infection and pneumonia in hospit alized and ambulatory patients, especially in alcoholic and diabet ic ones . Klebsiella pneumoniae
• 20
does not produce pigment s when cultured. Clinically, Klebsiella pneumonia is associated wit h a mucoid, bloody sputum, somet imes referred to
. 21 as "currant jelly sputum."
. 22 Unnary tract tnfectton Klebstella pneumomae Gram-negative bacteria Sputum Klebsiella Hospttal-acquired tnfectton Pneumoma Unnary system Hemoptysis Diabetes mellitus Infectton Alcoholtsm

. 23 C is not correct . 7 °/o chose this .


• 24 Proteus mirabilis is a gram-negat ive bacillus t hat is a frequent cause of nosocomial urinary tract infect ions. It produces the enzyme urease,
• 25 which serves to create a more alkaline environment for itself. The hallmark of a urease-producing organism such as Proteus is the production
of kidney stones along with infection. It does not produce pigment.
• 26 UreMe Proteu m bd•a Gr-a.n-negab e bacteria Enzyme Proteus (bacterium) Bac- Ius Unnllry tract 1nfect•on t:JaC llus (shape) Hosp1tal-acquired mfection Kidney l(jdney stone P1gment Unnary SyStem
. 27

8
Lodt
s
Suspe-nd
8
End Bloc:k
Item: 12 of 60 ~ ,• Mark <::J [:::> ""I ~· ~'j
QID: 2046 J.. Previous Next LAb faiUI~S Notes Calculator

1
E is not correct. 2°/o chose thi s.
2
Staphylococcus saprophyticus is t he second most common cause of urinary t ract infection in young women . It is a gram- posit ive coccus and
3
does not produce any pigments.
4 Urinary tract infection Staphylococcus saprophyticus Gram-positive bacteria Urinary system Staphylococcus Coccus

5
6
Bottom Line:
7
Serratia marcescens is a common cause of cat heter-relat ed infect ion.
8 Serratia marcescens Serratia I nfection
9
10
11 i@l;fil·1i•J for year:[ 2017 • J
FIRST AID FACTS
12
• 13
FA17 p 177.2
• 14

• 15
UTI bugs
. 16 SPECIES FEATURES COMMENTS
. 17 Escherichia coli Leading cause of UTI. Colonies show green Diagnostic markers:
• 18 metallic sheen on EMB agar. Ef> Leukocyte esterase= evidence ofWBC
• 19
Staphylococcus 2nd leading cause of UTI in sexua lly active activity.
. 20 Ef> litrite test = reduction of urinary nitrates
saprophyticus women.
. 21 by bacterial species (eg, E coli).
. 22 Klebsiella pneumoniae 3rd leading cause of UTI. Large mucoid capsule
Ef> Urease test = urease-producing bugs (eg,
• 23 and viscous colon ics .
S sa(Jrophyticus, Proteus, Klebsiella).
• 24 Serratia marcescens Some strains produce a red pigment; often
• 25 nosocomial and drug resistant.
• 26
Enterococcus Often nosocomial and drug resistant.
. 27
8
Lock
s
Suspend
0
End Block
Item: 12 of 60 ~ ,• Mark <::J [:::> ""I ~· ~'j
QID: 2046 J.. Previous Next LAb faiUI~S Notes Calculator
.., FIRST AID FACTS
1
2
FA17 p 177.2
3

4
UTI bugs
5 SPECIES FEATURES COMMENTS
6 Escherichia coli Leading cause of UTI. Colonies show green Diagnostic markers:
7 metallic sheen on EMB agar. (f) Leukocyte esterase= evidence ofWBC
8
Staphylococcus 2nd leading cause of UTI in sexua lly active activity.
9
saprophyticus women. Ef> litrite test = reduction of urinary nitrates
10 by bacterial species (eg, E coli).
11 Klebsiella pneumoniae 3rd leading cause of UTI. Large mucoid capsule
Ef> Urease test = urease-producing bugs (eg,
12 and viscous colonies.
S sa(Jrophyticus, Proteus, Klebsiella).
• 13
Serratia marcescens Some strains produce a red pigment; often
• 14
nosocomial and drug resistant.
• 15
. 16
Enterococcus Often nosocomial and drug resistant.
. 17 Proteus mirabilis Motility causes "swarming" on agar; produces
• 18 urease; associated with struvite stones .
• 19
Pseudomonas Blue-green pigment and fruity odor; usually
• 20
aeruginosa nosocomial and drug resistant.
. 21
. 22
• 23 FA17 p 137.2
• 24
Gram-negative lab algorithm
• 25
• 26 Gram 8 (pink)
. 27

8
Lock
s
Suspend
0
End Block
Item: 12 of 60 - ,• Mark --<] [::> ""'I ~- 1!';:'1
QIO: 2046 ~ Prev1o u s Next Labf a lu es Notes Calculator

1 FA1 7 p 137.2
2
3
Gram-negative lab algorithm

Gram 8 (ptnk)
4

5
6
7
Diplococci Coccobacilli Comma-shaped rods
8
9
10
Aerobic HMmOphilus infl~m~ZM Oxidase ~±~
11 (requires factQ(S Vand XJ
12 Maltose utilization Pasteurella
• 13
Brucella
Bordetella pertussis Grows in 42"C Grows in alkaline media Produces urease
• 14 Francisella tularens1s
. 15 N gonorrhoeae Campylobldrrjtjuni Vibrio cholerae Helicoblder pylori
N meningitidis
. 16 Moraxella
. 17
• 18
• 19
• 20 Bacilli
. 21
. 22 lactose fermentation
• 23
• 24 r~--~e~----L---~~
• 25 Oxidase Fast Slow
• 26
. 27 •
8
L.odt
s
Su~pl'nd
~
End Block
Item: 12 of 60 - ,• Mark --<] [::> ""'I ~· 1!';:'1
QIO: 2046 ~ Prev1o u s Next Labf a lu es Notes Calculator

1 • I
lactose fermentation
2
3
4
r'----e~----~--~~
Oxidase Fast Slow
5
6
Kttb*lu
7
Ecoli
8 H2Sproduction Enterobacter
9 on TSI agar
10
11

12
Shigttu Satmontlu
• 13 Yersim. Proteus
• 14
Important tests are in bold. Important pathogens are in bold italics.
. 15
. 16
. 17
FA17 p 177.1
• 18
• 19
Urinary tract Cystitis presents with dysuria, frequency, urgency, suprapubic pain, and WBCs (but not WBC
• 20 infections casts) in urine. Primarily caused by ascension of microbes from ureth ra to bladder. Males-
. 21 infants with congenital defects, vesicoureteral reAux. Elderly- enlarged prostate. Ascension to
. 22 kidney results in pyelonephritis, which presenls '' il h fever, chil ls, flank pain, costovertebra l angle
• 23 tenderness, hematuria, and WBC casts.
• 24 Ten times more common in \\ Omen (shorter urethras colonized by feca l fl ora). Other predisposing
• 25 factors: obstruction, kidney surger}. catheterization, C U malformation, diabetes, pregnancy.
• 26
. 27

8
L.odt
s
Su~pl'nd
~
End Block
Item: 13 of 60 - ,• Mark -<J [:::> "'I ~ · ~
QIO: 3872 ~ P~v•ous N@xt Labl lues No tes Calcula t o r

A A
1
A 24-year-old woman present s with severe ri ght lower quadrant pain. She has a history of multiple sexual part ners bu t is now married and ~~AI
2
sexually active with only her husband for t he past 3 years. A urine S-human chorionic gonadotropin t est is posit ive, but no gestational sac
3 is visualized on transvaginal ultrasound imag ing of the uterus. During the examination, her pain increases and she is becoming tachycardic.
4
5 Which of the following best characteri zes t he organism that most likely predisposed this woman to her current clinical picture?
6 :
7 A. It is a gram-negative bacillus
8 B. It is a gram-variable bacillus
9
C. It is a spirochete
10
11 0 . It is a teardrop-shaped trophozoite
12 E. It is an obligate intracellular organism
. 13
• 14

• 15
. 16
. 17
• 18
• 19
• 20
. 21
. 22
. 23
• 24
• 25
• 26
. 27

8
Lode.
s
S uspe-nd
8
End Bloc:k
Item: 13 of 60 ~ ,• Mark <::J [:::> ""I ~· ~'j
QID: 3872 J.. Previous Next faiUI~S
LAb Notes Calculator

1
2
The correct answer is E. 6 4 °/o chose this.
3 The woman is present ing wit h classic signs of ect opic pregnancy. Once t he 13-human chorionic gonadot ropin is above 1500 miU/m L, and
4 int rauterine pregnancy should be visible on t ransvag inal ult rasound imaging. If it is not , ectopic pregnancy m ust be suspected. If embedded in
5
t he fallopian t ube, t he growing fet us will event ually rupture the organ, leading t o life-t hreatening int ra -abdominal bleeding or it will die and
spontaneously abort . One risk factor for the development of ectopic preg nancy is previous pelvic inflammatory disease (PID) due t o its
6 associat ed damage to fal lopian tubes. Scarri ng of t he lining of the tubes renders them unable to propel t he fertilized ovum toward the uterus.
7 Organisms associated wit h PID are Chlamydia trachomatis, an obligat e intracellular bacterium, and Neisseria gonorrhoeae, a gram -negative
8 diplococcus.
9 A is not correct . 14% chose this.
10
Escherichia coli, a gram-negat ive bacillus, is a common cause of uri nary t ract infect ions in women, but is not associated wit h pelvic
inflammatory disease or ectopic pregnancy.
11
B is not correct. 7% chose t his.
12
Gardnerel/a vagina/is, a bacillus t hat has variable Gram st aining, is a cause of bact eri al vaginosis, which is not associated wit h an increased
13 risk of pelvic inf lammat ory disease or ect opic pregnancy.
• 14 C is not correct. 9 °/o chose this.
• 15 Treponema pal/idum, a spirochete, causes t he sexually t ransmitted disease syphilis, which is not associat ed wit h pelvic inf lammat ory disease
. 16 or ectopic pregnancy.
. 17 D is not correct. 6°/o chose this .
• 18 Trichomonas vagina/is is a t eardrop-shaped t rophozoit e t hat is spread t hrough sexual cont act. I n fema les, it coloni zes t he vagina and
produces a greenish, watery, and foul-smelling vaginal discharge and pruritus .
• 19
• 20
. 21
Bottom Line:
. 22 An important risk fact or for ect opic pregnancy is a history of PID because of its associated damage to t he fa llopian t ubes. Organisms
associat ed with PID are C. trachomatis, an obligate int racellu lar bacterium, and Neisseria gonorrhoeae, a gram-negat ive diplococcus .
• 23
• 24

• 25
l@l;fil·1i•l for yea r:[ 2017
FIRST AID FACTS .
•j .
• 26
. 27

8
Lock
s
Suspend
0
End Block
Item: 13 of 60 - ,• Mark -<J [:::> "'I ~ · ~
QIO: 3872 ~ P~v•ous N@xt Labl lues Notes Calculator
• • I I I J I

2
3
4
l@l ; fii ~11•J fo r year: 2017 •
FI RST AID FA CTS

5
6 FA17 p 181 .1
7
Pelvic inflammatory Top bugs- Chlamydia trachomatis (subacute, Salpingitis is a risk factor for ectopic pregnancy,
8
disease often undiagnosed), Neisseria gonorrhoeae infertility, chronic pekic pain, and adhesions.
9
(acute). C trachomatis-most common Can lead to Fitz-Hugh- Curtis syndrome-
10
11
bacterial STI in the United Stales. Cer\'ical infection of the li,·er capsule and '\·iolin string"
12
motion tenderness (chandelier sign), purulent adhesions of peritoneum to li\'er rn.
13 cer\'ical discharge · . PI D may include
' 14 salpingitis, endometritis, hydrosa lpinx, and
' 15 lubo-ovarian abscess.
. 16
. 17
• 18

' 19
' 20
. 21
. 22
' 23
' 24
' 25
' 26
. 27 •
8
Lode.
s
Suspe-nd
8
End Bloc:k
Item: 14of60 - ,• Mark -<J
P~v•ous
[:::> "'I ~ ·· ~
QIO: 2 104 ~ N@xt Labl lues No tes Calcula t o r

12
A 43-year-old man who recent ly returned from a 1-week hiking trip around the Great Lakes presents to t he clinic with flu-like symptoms, ~~AI
13 productive cough, and alt ered mental status. Review of symptoms is remarkable for new-onset diffuse bone and joint pain. Physical
. 14 examination is remarkab le for pleuritic ch est pain and multiple well-circumscribed cutaneous lesions. Cultures are taken from the patient's
• 15
. 16
blood, a slide from which is shown in t he image.
.... ·-
~------~wr-----------------,.-~----


0


. 17
• 18
• 19
• 20
. 21
. 22
'~• ,. t
0 23
0 24

0
25 •
0
26 •
. 27
• 28
• 29 Which of the fol lowing agents is t he most appropriate t reatment for this patient ?
• 30 :
0
31 A. Amphotericin B
. 32 B. Fluconazole
. 33
C. Potassium iodide
0 34
0 35 D. Selenium sulfide
0
36 E. Sodium stibogluconate
. 37
.
8
Lode.
s
Suspe-nd
8
End Bloc:k
Item: 14of60 - ,• Mark -<J
P~v•ous
[:::> "'I ~ ·· ~
QIO: 2 104 ~ N@xt Labl lues No tes Calcula t o r

A A
12
13 The correct answ er is A . 6 4 °/o chose this .
14 This patient has likely been infected by Blastomyces dermatitidis, the fungus that causes blastomycosis. It
is endemic in many parts of North America, including states east of the Mississippi River and in t he areas
• 15
surrounding the Great Lakes. The broad-based budding yeast shown in the stem image, indicated with the

I
. 16 red arrow, is classic for Blastomyces. Blastomycosis usually presents with flu - like symptoms: fevers, chills,
. 17 productive cough, myalgias, arthralgias, and pleuritic chest pain. Patients commonly progress t o develop
chronic pulmonary infection or widespread disseminated infection, the most common manifestation of which
• 18
is rash, as seen in this patient and like that shown in this image. The presence of organisms in this patient's
• 19 blood, along with the rash and altered mental status, indicates systemic infection, which is treated most
• 20 effectively with amphotericin B. Less severe, limited disease, such as mild pulmonary infection, is treated
with fluconazole or ketoconazole .
. 21
Ketoconi'Zole 81 •o -o··· Blosto oces dennabbdos Fluconazole Endertllsm Pleunsy Arthralgoa Yeast Cough Fungus North Amenca Rash Amphotencon B Image courtesy of CDC
. 22
Influenza~hke 1 lne Infect1on
• 23
B is not correct . 2 2% c hose this .
• 24
Fluconazole or ketoconazole are effective t reatments for local mild pulmonary Blastomyces infections bu t are ineffecti ve if t he infection has
• 25
already become systemic .
• 26 Ketoconazole Fluconazole Blastom,-ces dermabbd1s

. 27
C is not correct. 5°/o chose t his .
• 28
Potassium iodide and it raconazole are used to t reat Sporothrix schenckii infections (sporotrichosis). When
• 29 5. schenckii Is introduced into t he skin, usually by a t horn prick, it causes the fo rmation of a local pustule or
• 30 ulcer with nodules along t he draining lympha tics (ascending lymph angit is) . Under m icroscopy, 5. schenckii
appears as a dimorphic fungus wit h cigar-shaped, budding yea st , as shown in t his image .
• 31 Sporothrix schencku ltraconazole Sporotnchosis Lymphangitis Dimorphic fungus Lymphatic system Yeast Fungus Ulcer (dermatology)
. 32
. 33
• 34
• 35 •
I mage courtesy of CDC/Or.
• 36
Lucille K. Georg
. 37
. • "· - •• - 4 - - · · - - _ .,_ A ftl - L ...._L •

8
Lode.
s
Suspe-nd
8
End Bloc:k
Item: 14of60 - ,• Mark -<J
P~v•ous
[:::> "'I ~ ·· ~
QIO: 2 104 ~ N@xt Labl lues No tes Calcula t o r

A A
12 D is not correct. 4 °/o chose this .
13 Selenium sulfide (Selsun) is used to treat Ma/assezia fudur, the cause of tinea versicolor. Symptoms of this
14 infection include hypopigmented skin lesions t hat occur in hot and humid conditions. M. furfur has a
• 15
"spaghetti and meatballs" appearance under microscopy, as shown in this image .
T1nea vere.1color f¥1 I, liezla Hypop1gmentation T1nea Selenium Selsun Blue Sulfide Infection
. 16
. 17
• 18
• 19
• 20
. 21
. 22
• 23 E i s no t correct. 5% chose this .
Image courtesy of CDC/ Or.
Lucille K. Georg I
• 24 Sodium stibogluconate is used to t reat Leishmania donovani infection. L. donovani infection presents with
• 25 hepatomegaly and splenomegaly, malaise, anemia, and weight loss. It is a protozoal parasite transmitted
via the sandfly. Microscopically, macrophages cont aining amastigotes are observed, as shown in this image .
• 26 Le1shman1a donovan1 Hepatomegaly Splenomegaly Le1shmania Anemia Macrophage L. donovani Protozoa Sandfly Paras1t1sm Amastlgote Mala1se
. 27
• 28
• 29
• 30
• 31
Image courtesy of CDC/Or.
. 32
Francis W. Chandler
. 33
• 34
• 35 Bottom Line :
• 36 Blastomycosis, endemic to many parts of eastern North America, presents with flu-like symptoms. Mild pulmonary infections are treated with
. 37 fluconazole or ketoconazole; systemic infection requires amphotericin B. The presence of cutaneous f indings is indicative of systemic
. infection .

8
Lode.
s
Suspe-nd
8
End Bloc:k
Item: 14of60 - ,• Mark -<J
P~v•ous
[:::> "'I ~ ·· ~
QIO: 2 104 ~ N@xt Labl lues No tes Calcula t o r

A A
12
13 FA17 p 147.1
14
Systemic mycoses All of the following can cause pneumonia and can disseminate.
• 15
All are caused by dimorphic fun gi: cold (20°C) = mold ; heat (37°C) = )Cast. Only exception is
. 16
Coccidioides, which is a spherule (not yeast) in tissue.
. 17
Systemic mycoses can form granulomas (til e TB); cannot be transmitted person-to-person (unlike
• 18
• 19
T B).
• 20
Treatment: Auconazole or itraconazolc for local infection; amphotericin B for systemic infection .
. 21 DISEASE ENDEMIC lOCATION PATHOLOGIC FEATURES UNIQUE SIGNS/SYMPTOMS NOTES
. 22
Histoplasmosis Vlississippi and Ohio Macrophage fill ed Palatal/tongue ulcers, ll isto hides (\\ ithin

I
• 23
Rh·er Valleys with 1-Iistoplasma splenomegaly macrophages)
• 24
(smaller I han Bird (eg, starlings) or
• 25
RBC)FJ bat droppings
• 26
Diagnosis via urine/
. 27
• 28
serum antigen
• 29
• 30 Blastomycosis Eastern and Broad-based buclcling lnnammatory Blasto buds broadly
• 31 Central US of Blastomyces (sa me lung disease, can
. 32
size as RI3C) [J disseminate lo skin/
. 33
bone
• 34
Verrucous skin lesions
can simulate sec
• 35
• 36
Forms granulomatous
. 37
. nodules
8
Lode.
s
Suspe-nd
8
End Bloc:k
Item: 14 of 60 ~ ,• Mark <::J [:::> ""I ~· ~'j
QID: 2104 J.. Previous Next faiUI~S
LAb Notes Calculator

12
Coccidioidomycosis Southwestern US, Spheru le (much larger Disseminates to skin/
13
California than RBC) fill ed bone
14
with endospores of Erythema nodosum
• 15
. 16
Coccidioides [!1 (desert bumps) or
. 17
mull iforme
• 18 Arthralgias (desert
• 19 rheumatism)
• 20 Can cause meningitis
. 21 Para- Latin America Budding yeast of Similar to Paracoccidio parasa ils
. 22 coccidioidomycosis Paracoccidioides with with the captain's
Coccidioidomycosis,
• 23
"captain's wheel" males> fema les wheel all the way to
• 24
formation (much Latin America
• 25
larger than RBC) (!]
• 26
. 27
• 28
• 29
• 30
I
• 31
FA17p154. 1
. 32
Protozoa- others
. 33
ORGANISM DISEASE TRANSMISSION DIAGNOSIS TREATM ENT
• 34
• 35
Visceral infections
• 36 Trypanosoma Chagas disease-dilated Recluvi icl bug Trypomastigote in Benznidazole
cruzi ("kissing bug") blood smear fJ or nifurtimox;
. 37
. ----- - cardiomyopathy with

8 s 0

Lock Suspend End Block


Item: 14 of 60 ~ ,• Mark <::J [:::> ""I ~· ~'j
QID: 2104 J.. Previous Next LAbfaiUI~S Notes Calculator

• •
12 FA17p154.1
13 Protozoa- others
14 DIAGNOSIS
ORGANISM DISEASE TRANSMISSION TREATM ENT
• 15
Visceral infections
. 16
. 17 Trypanosoma Chagas disease- dilated Reduviid bug Trypomastigote in Benzn idazole
• 18 cruzi cardiomyopathy with ("kissing bug") blood smear fJ or nifurtimox;
• 19 apical atrophy, megacolon, feces, deposited C ruzing in my
• 20 megaesophagus; predominantly in a painless bite Benz, with a fur
. 21 in South America (much Iike a kiss) coat on
. 22 Unilateral periorbital swelling
• 23 (Romai'ia sign) characteristic of
• 24 acute stage
• 25
• 26
Leishmania Visceral leishmaniasis Sand fly Macrophages Amphotericin B,
. 27
donovani (kala-azar) - spiking fevers, contain ing sodium
• 28 hepatosplenomegaly, a mastigotes : stibogluconate
• 29
• 30
• 31
. 32
pancytopenia
Cutaneous leishmaniasis-skin
ulcers I
. 33
• 34
• 35 Sexually transmitted infections
• 36 Trichomonas Vaginitis- foul-smelling, greenish Sexua l (cannot exist Trophozoites Metronidazole for
. 37 vagina/is discharge; itching and burning; outside human (motile) 9 on wet patient and partner •
. •
8
Lock
s
Suspend
0
End Block
Item: 14 of 60 ~ ,• Mark <::J [:::> ""I ~· ~'j
QID: 2104 J.. Previous Next faiUI~S
LAb Notes Calculator

12 FA17 p 148.1
13 Cutaneous mycoses
14
Tinea Tinea is the cli nica l name given to dermatophyte (cutaneous fungal} infections. Dermatophytes
• 15
(dermatophytes) include Microsporum, Trichophyton, and Epidermophyton. Branching septate hyphae visible on
. 16
. 17
KOH preparation with blue fungal sta in rJ. Associated with pruritus.
• 18 Tinea capitis Occurs on head, scalp. Associated with lymphadenopathy, alopecia, scaling 1]).
• 19
Tinea corporis Occurs on torso. Characterized by erythematous scal ing rings ("ringworm") and central
• 20
clearing 9 . Can be acquired from contact with an infected cat or dog.
. 21
. 22
Tinea cruris Occurs in inguinal area 1!]. Often does not show the central clearing seen in tinea corporis.
• 23 Tinea pedis T hree varieties:
• 24 • Interdigital I]; most common
• 25 • Moccasin distribution I]
• 26 • Vesicular type
. 27
Tinea unguium Onychom)'COsis; occurs on nails.
• 28
• 29 Tinea (pityriasis) Caused by Malassezia spp. (Pityrosporum spp.), a yeast-like fu ngus (not a dermatophyte despite
• 30 versicolor being called tinea). Degradation of lipids produces acids that damage melanocytcs and cause
• 31 hypopigmented [!!, hrperpigmented, and/or pink patches. Less pruritic than dermatophytes.
. 32 Can occur any time of year, but more common in summer (hot, humid weather). "Spaghetti and
meatballs" appearance on microscopy CJ.

I
. 33
• 34 Treatment: selenium sulfide, topical and/or oral antifunga l medications.
• 35
• 36
. 37
. •
8
Lock
s
Suspend
0
End Block
Item: 14 of 60 - ,• Mark --<] [::> ""'I ~· 1!';:'1
QIO: 2104 ~ Prev1o u s Next Labf a lues Notes Calculator

12 Tinea unguium Onychomycosis; occurs on nails.


13
Tinea (pityriasis) Caused by Malassezia spp. (Pityrosporum spp.), a yeast-like fungus (not a dennatophyte despite
14
versicolor being called tinea). Degradation of lipids produces acids that damage mclanocytcs and cause
• 15
. 16
hypopigmented [!1, hyperpigmented, and/or pink patches. Less pruritic than dermatophytes.
. 17
Can occur any time of year, but more common in summer (hot, humid weather). "Spaghetti and
• 18 meatballs" appearance on microscopr
. 19 Treatment: selenium sulfide, topical and/or oral antifungal medications.
• 20
. 21
. 22

• 23
• 24
• 25
• 26
. 27
• 28
• 29
• 30
• 31
• 32
. 33
• 34
• 35
• 36
. 37
. •
8
L.odt
s
Su~pl'nd
~
End Block
Item : l S o f 60 - ,• Ma rk -<J [:::> "'I ~ · ~
QIO: 3510 ~ P~v•ous N@xt Labl lues Notes Calcu lator

12
A 32-year-old woman presents to her physician complaining of 1 week of vaginal itchiness. She also not ices increased vaginal discharge. ~~AI
13 She is afebrile, and physical examination reveals no abdominal or adnexal tenderness. Speculum examination demonstrates a frothy
14 discharge with a greenish hue. The cervix bleeds during the collection of fluid with a cotton swab.
. 15
. 16
Which of the following is most likely to be observed on light microscopy?
. 17 :

• 18
A. Flagellated organisms on wet mount
• 19 B. Fuzzy epithelial cells on wet preparation
• 20
C. Gram-negative diplococci on Gram stain
. 21
0 . Gram-positive cocci on Gram stain
. 22
• 23 E. Spores and hyphae-like growt h on Giemsa stain
• 24
• 25
• 26
. 27
• 28
• 29
• 30
• 31
. 32
. 33
• 34
• 35
• 36
. 37
.
8
Lode.
s
Sus pe-nd
8
End Bloc:k
Item: lS of 60 ~ ,• Mark <::J [:::> ""I ~· ~'j
QID: 3510 J.. Previous Next LAb faiUI~S Notes Calculator


12
13
The correct a nswer is A. 72°/o chose t h is.
14 The presentation in this case is typical for infection with Trichomonas. Greenish vaginal discharge, fri able cerv ix, and motile cells on wet prep
15 are common featu res. There are 3 million t richomonal infections annua lly in t he United States. Men also become infected, but they are
. 16
asymptomat ic. Bot h part ners must be t reat ed with met ronidazole. Trichomonas vagina/is can t ypically be seen as flagellat ed t rophozoites on
wet mount and microscopy. PCR may also be used to ident ify t he organ ism . Because t he frequency of m ixed sexua lly t ransmitt ed infect ions
. 17 (STis) is relat ively high, t he f inding of mot ile prot azoans means fu rt her work-up should be perfo rmed in t his pat ient.
• 18 Metronidazole Trichomonas vaginalis Cervix Vaginal discharge Asymptomatic Sexually transmitted infection Polymerase chain reaction Infection Flagellum Motility Microscopy Trichomonas Microscope slide

• 19 Organism

• 20 B is not correct. 10% chose this .


. 21 Gardnerella is a pleomorph ic rod t hat is a norma l part of t he vag inal flora, but it can become overgrown, and t his leads t o bact erial vaginosis,
. 22 which is charact eri zed by pruritu s and a f ishy-smelling vag inal discharge . Clue cells are often fo und on wet prep. Clue cells are epithelia l cells
• 23
t hat appea r fu zzy and are covered with adherent gram- posit ive rods wit h some gram -negative rods as well. Metronidazole is t he t reatment of
choice .
• 24 Bacilli Metronidazole Bacterial vaginosis Gram-negative bacteria Itch Gram-positive bacteria List of microbiota species of the lower reproductive tract of women Gardnerella vaginalis vaginal flora Epithelium
• 25 vaginal discharge Clue cell
• 26
C is not correct. 8 °/o chose this .
. 27
Gram -negative diplococci seen in the vagina are most likely Neisseria gonorrhoeae. The discharge in gonococca l infect ions is t ypically purulent
• 28 or m ucopurulent, not frot hy, and can range f rom yellow to green in color. Alt hough t his pat hogen can cause vaginal discharge and pruritu s,
• 29 t his pat ient's clinica l presentat ion is more consistent wit h t richomoniasis .
Gram-negative bacteria Trichomoniasis Diplococcus Neisseria gonorrhoeae Itch Pus vaginal discharge Pathogen vagina Gonorrhea Neisseria
• 30
• 31 D is not correct. 3°/o chose t his .
. 32 Staphylococcus saprophyticus is a common cause of urinary t ract infections in patients of t his age group, but gram -positive cocci are not
generally associat ed with vaginit is .
. 33
Staphylococcus saprophyticus Vaginitis Gram-positive bacteria Coccus Urinary system Staphylococcus Urinary tract infection
• 34
• 35
E is not correct. 7°/o chos e this .
Spores and pseudohyphae on Giemsa st ain of vaginal discharge are indicative of Candida albicans vaginit is. Candidial vaginit is causes a t hick,
• 36
"cott age-cheese" discharge. This infect ion can be t reat ed with cl otrimazole cream or f luconazole .
. 37 Candida albicans Clotrimazole Fluconazole Vaginitis Candida (fungus) vaginal discharge Hypha Spore Pseudohyphae Infection
. •
8
Lock
s
Suspend
0
End Block
Item: lS of 60 ~ ,• Mark <::J [:::> ""I ~· ~'j
QID: 3510 J.. Previous Next LAb faiUI~S Notes Calculator

• •
12
Bottom Line:
13
Trichomonas infection manifests with greenish vaginal discharge, a fria ble cervix, and motile cells on wet prep.
14 Cervix vaginal discharge Trichomonas Motility Infection

15
. 16
. 17 ljl:fil·11•i for year:[ 201 7
FIRST AID FA CTS
•I
• 18
• 19
FA17p154.1
• 20
Protozoa- others
. 21
ORGANISM DISEASE TRANSMISSION DIAGNOSIS TREATM ENT
. 22
• 23 Visceral infections
• 24 Trypanosoma Chagas disease- dilated Reduvi id bug Trypomastigote in Benzn idazole
• 25 cruzi cardiomyopathy with ("kissing bug") blood smear fJ or nifurtimox;
• 26
apical atrophy, megacolon, feces, deposited C ruzing in my
. 27
megaesophagus; predominantly in a painless bite Benz, with a fur
• 28
in South America (much Iike a kiss) coat on
• 29
Unilateral periorbital swelling
• 30
(Romai'ia sign) characteristic of
• 31
acute stage
. 32
. 33 Leishmania Visceral leishmaniasis Sand fly Macrophages Amphotericin B,
• 34 donovani (kala-azar) - spiking fevers, contain ing sodium
• 35 hepatosplenomegaly, a mastigotes stibogluconate
• 36 pancytopenia
. 37
Cutaneous leishmaniasis-skin •
. •
8
Lock
s
Suspend
0
End Block
Item: lS of 60 - ,• Mark -<J [:::> "'I ~ · ~
QIO: 3510 ~ P~v•ous N@xt Labl lues No tes Calcula t o r

A A
12
13 FA17 p 177.3
14 Common vaginal infections
15 Bacterial vaginosis Candida v ulvovaginitis
Trichomonas vaginitis
. 16
. 17
SIGNS AND SYMPTOMS o inflammation I nAammation ("strawberry Inflammation
• 18
Thin, white discharge with cervi, ") Thick,'' hite, "cottage cheese"
• 19 fi shy odor Frothy, yellow-green, foul- discharge ~
• 20 smelling discharge
. 21 lAB FINDINGS Clue cells 1otile trichomonads Pseudohrphae
. 22 pH > 4.5 pi I > 4.5 pH normal (4.0-4.5)
0 23
TREATMENT VIetronidazole t\ lctronidazole -azoles
0 24
Treat sexua I partner(s)
0
25
0
26
. 27
• 28
• 29
• 30
0
31
. 32
. 33
0 34
0 35 \ f
0
36
. 37
.
8
Lode.
s
Suspe-nd
8
End Bloc:k
Item: lS of 60 ~ ,• Mark <::J [:::> ""I ~· ~'j
QID: 3510 J.. Previous Next LAbfaiUI~S Notes Calculator

• •
12 FA17 p 180.1
13 Sexually transmitted infections
14 DISEASE CLINICALFEATURES ORGANISM
15
AIDS Opportunistic infections, Kaposi sarcoma, HIV
. 16
lymphoma
. 17
• 18 Chancroid Pain fu l genita l ulcer with exudate, inguinal Haemophilus ducreyi (it's so pain ful , you "do
• 19 adenopathy cry")
• 20 Chlamydia Urethritis, cervicitis, epididymitis, Chlamydia trachomatis (0 - K)
. 21
conjunctivitis, reactive arth ritis, PI D
. 22
• 23
Condylomata Genital warts, koilocytes HPV-6 and -11
• 24
acuminata
• 25 Genital herpes Painful pen ile, vulvar, or cervical vesicles and HSV-2, less commonly HSV-1
• 26 ulcers; can cause systemic symptoms such as
. 27 b·er, headache, myalgia
• 28
Gonorrhea Urethritis, cervicitis, PI D, prostatitis, eisseria gonorrhoeae
• 29
epididymitis, arthritis, creamy purulent
• 30
• 31
cl iscl,;1rge
. 32 Granuloma inguinale Painless, beefy red ulcer that bleeds readily on Klebsiella (Calymmatobacterium) granulomatis;
. 33 (Donovanosis) contact fJ cytoplasmic Donovan bodies (bipolar staining)
• 34 Not common in US seen on m1croscopy
• 35
• 36
. 37

. •
8
Lock
s
Suspend
0
End Block
Item: 16 of 60 - ,• Mark -<J [:::> "'I ~ · ~
QIO: 2 199 ~ P~v•ous N@xt Labl lues No tes Calcula t o r

12
A construction worker accident ally drives a nail int o his t high with a nail gun. The nail is removed, but t he area remains painful and ~~AI
13 edematous, and a diagnosis of cellulit is is made. After a few days the skin over the affected area becomes spongy and produces a crackling
14 sound on palpation. A blood-tinged exudate is also observed.
15
. 16
To which of the following organisms was this man most likely exposed?
. 17 :

• 18
A. Anaerobic filamentous rod that forms yellow "sulfur granules"
• 19 B. Dimorphic fungus that at body temperature appears as cigar-shaped, unequal budding yeast
• 20
C. Facultative anaerobic rod with a protein capsule
. 21
D. Obligate anaerobic rod that produces a neurotoxin .
. 22
0 23 E. Obligate anaerobic rod that produces a phospholipase toxin
0 24
0
25
0
26
. 27
• 28
• 29
• 30
0
31
. 32
. 33
0 34
0 35
0
36
. 37
.
8
Lode.
s
Suspe-nd
8
End Bloc:k
Item: 16 of 60 ~ ,• Mark <::J [:::> ""I ~· ~'j
QID: 2199 J.. Previous Next LAb faiUI~S Notes Calculator


12
13
The correct answer is E. 700% chose this.
Clostridium pedringens can cause t issue necrosis due t o t he production of a-toxin (lecit hinase, a phospholipase) . Alt hough t he
14
pathophysiology is not well undesrt ood, t he exotoxin causes bot h hemolysis and myonecrosis, which produces gas in the dying t isse. This
15 subcutaneous gas causes t he classic crackling sound heard on palpation (crepit us) . C. perfringens may also infect j ust the skin, lead ing t o
16 cellulit is, and its enterot oxin is a lead ing cause of food poisoning .
Clostridium perfringens Exotoxin Gas gangrene Crepitus Cellulitis Hemolysis Necrosis Lecithinase Clostridium Enterotoxin Phospholipase Foodborne illness Subcutaneous tissue Pathophysiology
. 17
• 18 A is not correct. 4°/o chose this .
• 19 This describes Actinomyces israelii, which typically manifests as a chronic, slowly progressing abscess t hat eventu ally drains t hrough sinus
t racts. It is not charact eri zed by cellulit is or crepitus.
• 20
Actinomyces israelii Actinomyces Cellulitis Abscess Crepitus
. 21
B is not correct. 2% chose this .
. 22
This describes Sporothrix schenckii, which is a f ungus t hat lives on vegetat ion. Infection occurs aft er it is introduced into the skin, often on t he
• 23
t ip of a t horn ( "rose gardener's disease") and results in a local ulcer wit h ascending lymphangit is. Not on ly do t he symptoms of t his pat ient
• 24 not match t his infect ion, but t he met hod of int roduct ion also does not suggest Sporothrix infect ion .
Sporothrix schenckii Lymphangitis Ulcer (dermatology) Fungus Infection
• 25
• 26 C is not correct. 9 °/o chose this .
. 27 This describes Bacillus anthracis, the on ly med ically relevant bacteri um with a prot ein capsule. It can cause cutaneous ant hrax, wh ich is
• 28 charact erized by a painless ulcer wit h a black scab, called an escha r. It is not associated wit h cellulit is or crepit us .
Eschar Bacillus anthracis Cellulitis Anthrax Protein Bacillus Bacteria Ulcer Peptic ulcer Wound healing
• 29
• 30 D is not correct. 15% chose this .
• 31
This describes Clostridium tetani, which causes tetanus, characterized by severe m uscle spasms. I t is not associat ed wit h cellulit is .
Clostridium tetani Tetanus Cellulitis Clostridium Spasm Muscle
. 32
. 33
• 34 Bottom Line:
• 35 Clostrdium perfringens is a gram -positive obligat e anaerobic rod t hat produces lecithinase (a-toxin), which can cause cellulit is, myonecrosis
• 36
(leading to crepitu s), and hemolysis .
Cellulitis Gas gangrene Gram-positive bacteria Hemolysis Lecithinase Anaerobic organism Obligate anaerobe Crepitus
. 37
. •
8
Lock Suspend
s 0
End Block
Item: 16 of 60 - ,• Mark --<] [::> ""'I ~· 1!';:'1
QIO: 2199 ~ Prev1o u s Next Labf a lues Notes Calculator

• •
12 FA17 p 134.2
13
Clostridia (with Gram EB, spore-forming, obligate anaerobic rods.
14
exotoxins)
15
16 Ctetani Produces tetanospasmin, an exotoxin causing Tetanus is tet anic paralysis.
. 17 tetanus. Tetanus toxin (and botulinum toxin)
• 18 are proteases that clea,·e SNARE proteins for
. 19 neurotransmitters. Blocks release of inhibitory
• 20 neurotransmitters, GABA and glrci ne, from
. 21 Renshaw cells in spinal cord .
. 22 Causes sp astic paralysis, trismus (lockjaw), risus
• 23 sardonicus (raised eyebrows and open gri n),
• 24
opisthotonos (spasms of spinal extensors).
• 25
Pre, en! with tetanus vaccine. Treat'' ith
• 26
antitoxin +1- vaccine booster, diazepam (for
. 27
muscle spasms), and wou nd debridement.
• 28
• 29 Cbotulinum Produces a heat-labile toxin that inhibits Symptoms of botulism (the 4 D 's): D iplopia,
• 30 ACh release at the neuromuscular june! ion, O )rsarthria, D ysphagia, D yspnea .
• 31 causi ng botulism. In adults, disease is caused Botulinum is from bad bottles of food , ju ice, and
. 32 by ingestion of preformed toxin. In babies, honey (causes a descending flaccid paralysis).
. 33 ingestion of spores (eg, in honey) leads to Local botox injections used to treat foca l
• 34 disease (flopp~ baby syndrome). Treat with dystonia, achalasia, and muscle spasms. Also
• 35 antitoxin . used for cosmetic reduction offacial wrinkles.
• 36
. 37
.
C perfringens
.. Produces a toxin (lecithinase, a phospholipase)
that can cause m\·onecrosis (gas gangrene )
Perfringens perforates a gangrenous leg.

8
L.odt
s
Su~pl'nd
~
End Block
Item: 16 of 60 - ,• Mark --<] [::> ""'I ~· 1!';:'1
QIO: 2199 ~ Prev1ous Next Labfa lues Notes Calculator

12
e o • y~
•ysp g , • ysp
causing botulism. In adults, disease is caused Botulinum is from bad bottles of food, ju ice, and
13
14
by ingestion of preformed toxin. In babies, honey (causes a descending Aaccid paralysis).
15
ingestion of spores (eg, in honey) leads to Local botox injections used to treat focal
16 disease (Aopp~ baby syndrome). Treat'' ith dystonia, achalasia, and muscle spasms. Also
. 17 antitoxin . used for cosmetic reduction of fa cial wrinkles.
• 18 C perfringens Produces a toxin (lecithinase. a phospholipase) Perfringens perforates a gangrenous leg.

....
. 19 that can cause myonecrosis (gas gangrene )
• 20

:r , and hemolysis.
. 21 • Spores can sun·i,·e in undercookcd food;
. 22
,
when ingested, bacteria release heat-labile
• 23 ' ~ enterotoxin ..... food poisoning.
• • '
• 24
• 25
• 26 .,..
. 27
• 28
C difficile Produces 2 toxins. Toxin A, an enterotoxin, /)i{ficile causes diarrhea. Treatment:
• 29
binds to brush border of gul and allers Auid metronidazole or oral vancomycin. For
• 30 secretion. Toxin B, a cytotoxin, disrupts recurrent cases, consider repeating prior
• 31 cytoskeleton via actin depolymerizalion. Both regimen, fidaxomicin, or fecal microbiota
• 32 toxins lead to diarrhea ..... pseudomembranous transplant.
. 33 zo
colitis (lJ. Often to antibiotic usc, especially
• 34 clindamrcin or ampicillin ; <~ ssoci <~t cd with PPI
• 35 use. Diagnosed by detecting one or both toxins
• 36 in stool by antigen detection or PCR.
. 37

.
8
L.odt
s
Su~pl'nd
~
End Block
Item: 17 of 60 - ,• Mark -<J [:::> "'I ~ · ~
QIO: 3889 ~ P~v•ous N@xt Labl lues No tes Calcula t o r

12
A 27-year-old man present s to his family practice doctor complaining of a painfu l and swollen right knee. He asks if t hese problems might ~~AI
13 be related to a new skin rash on the soles of his feet. He states that he has been healthy except for a bout of gastroenteritis that he and
14 his wife both had 3 weeks ago. His family history is notable for his father having psoriati c a1thri tis. Synovial fluid analysis from his right
knee shows:
15
Volume: 5 mL (normal: <3.5 mL)
16
Viscosity: low (normal: high)
. 17 WBC count: 10,000/mm 3 with 65% polymorphonuclear leukocytes
• 18 Glucose: 89 mg/dl (normal: nearly equal to that in blood)
• 19
Gram stain: negative
Culture: negative
• 20
. 21
. 22 Which of the following findings is the patient most likely to have in addition to the signs and symptoms above?
0 23 :

0 24
A . Anti-IgG antibodies
0
25 B. Conjunctivitis
0
26 C. Heberden nodes on fingers
. 27
D. Malar rash
• 28
• 29 E. Ulnar devi ation of fingers
• 30
0
31
. 32
. 33
0 34
0 35
0
36
. 37
.
8
Lode.
s
Suspe-nd
8
End Bloc:k
Item: 17 of 60 - ,• Mark -<J [:::> "'I ~ · ~
QIO: 3889 ~ P~v•ous N@xt Labl lues No tes Calcula t o r

A A
12 Th e correct an sw er is B. 55% chose this.
13 [.,;. '
14 The patient most likely has reactive arthrit is. This is one of the seronegative spondyloarthropathies that is
15 characterized by asymmetric arthrit is accompanied by ot her extra-articular manifestations, which follows
either a gastrointestinal (GI) or genit ourinary (GU) infection by 2-4 weeks. The extra-articular
16
manifestations include conjunctivitis (which often precedes the arthritis), keratoderma blennorrhagica (a
17 papulosquamous skin rash on the palms and soles, like that shown in the image), urethritis (especially
• 18 when the preceding illness is GU), circinate balanitis (a lesion on the penile shaft), and occasionally aortitis
• 19 (which becomes aortic valve regurgitation). The symptoms can be remembe red with the mnemonic " can't
see (conjunctivitis), can't p ee (urethritis), can' t bend my k nee (arthritis), and n o a nti- IgG." Reactive
• 20 arthritis is associated with human leukocyte antigen B27, as is psoriatic arthritis (an important finding from
. 21 his family history). As one of the seronegative spondyloarthropathies, there will be no anti-IgG antibodies .
. 22 Normal characteristics of synovial fluid include a WBC count of <200 ceiiS/ I.JL, <25% polymorphonuclear Image courtesy of CDC/ Dr. M.
neutrophils, high viscosity, and a glucose level similar to the patient's serum glucose level. These values are F. Rein
0 23 contrary in this patient.
0 24 Spondyloarthropath~ Human leukocyte antigen Psoriatic arthritis Reactive arthritis Urethntis Synov1al flUid ConjunctiVItis Wh1te blood cell Keratoderma blennorrhagicum Arthritis Mnemontc Antigen

0
25 Neutrophil Gen1tounnar,. system Glucose Rash Serostatus Blood plasma Infection Liver funct1on tests Seroneoat1ve arthnt•s Synov•al membrane Synovial joint

0
26 A is n ot correct . 19 °/o chose this .
. 27 Reactive arthritis is a seronega t ive spondyloart hropat hy. Thu s, it will not have anti-IgG antibodies. An example of an anti-I gG antibody are
• 28 rheumatoid factors found in the majori ty of pat ien ts wit h rheumatoid arthrit is. These are au toant ibodies against t he Fe port ion of IgG .
Reactive arthritis Rheumato•d arthntls Spondyloarthropathy Arthritis Serostatus Autoantibody Ant1body Seronegative spondyloarthropathy Seronegative arthritis
• 29
• 30 C is n ot correct . 10% chose this .
0
31 Heberden nodes on t he distal interphalangeal joints of fingers are a manifestation of ost eoarthrit is, which is characterized by t he destruction
of articular cartilage. These are not consist ent wit h t his pat ient 's presentation .
. 32
Osteoarthr1t1s Interphalangeal art1culabons of hand Articular cartilage Anatomical terms of location Cartilage 01stal Interphalangeal art1culat1ons of foot
. 33
0 34
D is n ot co rrect . 9 °/o chose this .
35
Malar rash, or butterfly rash, is a common manifest at ion of systemic lupus erythematosus (SLE). Although art hrit is can be a feature of SLE,
his history and presentation is more consistent with reactive arthritis.
0

0
36 Reacbve ,.,..a., tb ~te-n1c lupus erythematosus Arthritis Malar rash Lupus erythematosus R~h

. 37
E is n ot correct. 7 °/o chose thi s .
.
8
Lode.
s
Suspe-nd
8
End Bloc:k
Item: 17 of 60 ~ ,• Mark <::J [:::> ""I ~· ~'j
QID: 3889 J.. Previous Next LAb faiUI~S Notes Calculator

12 E is not correct. 7 °/o chose this.


13 Ulnar deviat ion of f ingers is a late man ifest at ion of rheumatoid arthrit is. It is not associat ed with reactive arthri t is.
Reactive arthritis Rheumatoid arthritis Arthritis Ulnar deviation
14
15
16 Bottom Line:
17 Reactive arthritis, a seronegative spondyloarthropathy, is characterized by arthri t is accompanied by ext ra -articular manifestat ions
• 18 (conjunct ivit is and urethri t is) following GI or GU infect ions. Charact eristics of synovial f luid include a WBC count > 2000 cells/I-ll, PMNs >
SO%, low viscosity and a lower or similar t o t he glucose level of t he pat ient's serium glucose .
• 19 Reactive arthritis Spondyloarthropathy Urethritis Conjunctivitis synovial fluid Arthritis Serostatus Neutrophil Synovial membrane Glucose Granulocyte Synovial joint Viscosity
. 20
. 21
. 22 i@l;fil·1i•J for year:[ 2017 • J
FIRST AID FACTS
• 23
• 24
FA17 p442.1
• 25
• 26 Seronegative Arthritis without rheumatoid fac tor (no anti-IgG antibody). Strong association with HLA-B27
. 27 spondyloarthritis (M HC class I serotype). Subtypes (PAIR) share variable occurrence of inflammatory back
• 28 pain (associated with morning stiffness, improves with exercise), peripheral arthritis, enthesitis
• 29 (inflamed insertion sites of tendons, eg, Achi lles), dactylitis ("sausage fingers"), uveitis.
• 30
Psoriatic arthritis Associated with ski n psoriasis and nail lesions. Seen in fewer than 1/3 of patients with psoriasis.
• 31
Asymmetric and patchy involvement fJ.
. 32
. 33
Dactylitis and "pencil-in-cup" deformity of
• 34
Dl P on x-ray 11).
• 35 Ankylosing Symmetric invoh-ement of spine and sacroiliac Bamboo spine (vertebral fusion) l!l Can cause
• 36 spondylitis joints ..... an kylosis (joint fusion), uveitis, aortic restrictive lung disease due to limited chest
. 37 regurgitation. wall expansion (costovertebral and costosternal
.
8
Lock
s
Suspend
0
End Block
Item: 17 of 60 - ,• Mark --<] [::> ""'I ~· 1!';:'1
QIO: 388:9 ~ Prev1o u s Next Labf a lu es Notes Calculator

12 Ankylosing Symmetric invoh-ement of spi ne and sacroiliac Bamboo spine (vertebral fusion) l!l C an cause
13 spondylitis joints - ankylosis (joint fttsion), li\'Cilis, aortic restrictive lung disease due to limited chest
14 regurgitation. wall expansion (costovertebral and costosterna l
15 anl-ylosis).
16 \!lore common in males.
17
Inflammatory bowel C rohn disease and ulcerative colitis are often
• 18
. 19
disease associated ,,·ith spondyloarthritis.
• 20 Reactive arthritis Formerly known as Re iter syndrome. "Can' t see, can't pee, can' t bend m~ knee."
. 21 C lassic triad: Shigella, ) ersinia, Chlamydia, Campylobacter,
. 22 Conjunctivitis Salmonella (Sh\ C hiCS).
• 23 Urethritis
• 24 ,\rthritis
• 25
• 26
. 27
• 28
• 29
• 30
• 31
• 32
. 33
• 34
• 35
• 36
. 37

.
8
L.odt
s
Su~pl'nd
~
End Block
Item: 18 of 60 - ,• Mark -<J [:::> "'I ~ · ~
QIO: 20 1 2 ~ P~v•ous N@xt Labl lues No tes Calcula t o r

12
A 25-year-old woman comes to th e gynecologist because her new sexu al partner has dysuria and a puru lent discharge from his urethra. ~~AI
13 She admits to engaging in unprotected sexual activity over the past few months but denies any dysuria, discharge, or abdominal pain.
14
15 Which of the fol lowing would best ident ify the cause of her partner's urinary symptoms?
16 :
17 A. Bordet-Gengou agar
. 18 B. Charcoal yeast extract agar
• 19
C. Lowenstein-Jensen agar
• 20
. 21 0 . MacConkey agar
. 22 E. Thayer-Martin agar
• 23
• 24
• 25
• 26
. 27
• 28
• 29
• 30
• 31
. 32
. 33
• 34
• 35
• 36
. 37
.
8
Lode.
s
Suspe-nd
8
End Bloc:k
Item: 18 of 60 ~ ,• Mark <::J [:::> ""I ~· ~'j
QID: 2012 J.. Previous Next faiUI~S
LAb Notes Calculator


12
The correct a nswer is E. 7 1 °/o chose this.
13
This woman's partner is likely infect ed with Neisseria gonorrhoeae . Nucl eic acid amplif ication is now t he first - line t est fo r gonorrhea, but
14 cult ure on Thayer- Mart in agar was long considered the gold st andard and is still used t o eva luat e antibiot ic-resist ant infect ions. Thayer-Martin
15 agar is a chocolat e agar plat e suffused with vancomycin, t rimethoprim, colist in, and nystatin (VTCN) ant ibiot ics that suppress t he growt h of
endogenous f lora while support ing t he growt h of N. gonorrhoeae .
16
17 The maj ority of people (up t o 60% of men and 70% of women) infect ed with N. gonorrhoeae are asymptomatic. However, females wit h
asymptomat ic infections are at the same risk for pelvic inflammatory disease and ot her complicat ions as symptomatic women. As a result , t he
18
woman in t his question, who likely has an asymptomatic infect ion, should be t reated. Ch lamydia often co -infects wit h gonorrhea, so ant ibiot ic
• 19 t herapy should include coverage for bot h organisms .
• 20 Pelvic inflammatory disease Nystatin Gonorrhea Colistin Neisseria gonorrhoeae Vancomycin Chocolate agar Antibiotics Trimethoprim Chlamydia infection Nucleic acid Agar plate Neisseria
. 21 Antimicrobial resistance Polymerase chain reaction Gold standard (test) Thayer-Martin agar Agar Endogeny (biology) Infection
. 22
A is not correct . 5°/o chose t his .
• 23
Bordet-Gengou (potato) agar is used to culture Bordetel/a pertussis, which causes whooping cough, a respirat ory illness .
• 24 Bordetella pertussis Pertussis Bordetella Cough Agar

• 25
B is not correct. 6% chose t his .
• 26
Charcoal yeast extract agar buffered with increased iron and cysteine is used t o cultu re Legionella pneumophila, which causes leg ionella
. 27 pneumonia, another respirat ory illness .
• 28 Legionella pneumophila Legionella Cysteine Pneumonia Yeast extract Yeast Buffered charcoal yeast extract agar Agar Charcoal Iron

• 29 C is not correct. 7 °/o chose this .


• 30 Lowenstein-Jensen agar is used to cult ure Mycobacterium tuberculosis, t he pri mary agent in tuberculosis. Th is agar selects against gram-
• 31 posit ive bacteria in resp iratory f lora .
Mycobacterium tuberculosis Gram-positive bacteria Tuberculosis Mycobacterium Bacteria Agar
. 32
. 33 D is not correct. 11% c hose this .
• 34 MacConkey agar is used t o cult ure lactose-ferment ing ent eri c bacteria such as Escherichia coli and Klebsiella species. These two bact eria
cause urinary t ract infections, which would result in dysuria but not a purulent discharge .
• 35 Dysuria Escherichia coli MacConkey agar Bacteria Urinary system Klebsiella Urinary tract infection Pus Agar Gut flora
• 36
. 37
. • Bottom Line :
8
Lock
s
Suspend
0
End Block
Item: 18 of 60 ~ ,• Mark <::J [:::> ""I ~· ~'j
QID: 2012 J.. Previous Next LAb faiUI~S Notes Calculator

12
13
Bottom Line:
14 The presence of dysuria and purulent uret hral discharge is suggest ive of Neisseria gonorrhoeae infection . Thayer-Martin agar is the most
appropriate medium for cult ure and diagnosis.
15 Dysuria Neisseria gonorrhoeae Neisseria Thcrter-Martin agar Pus Agar Urethra I nfection
16
17
18 i@l;fil·1i•J f or yea r:[ 2017 • J
FIRST AID FACTS
• 19
• 20
FA17 p 123.1
. 21
. 22
Special culture requirements
• 23 BUG MEDIA USED FOR ISOLATION MEDIACONTENTS/OTHER
• 24 H influenzae Chocolate agar Factors V (1 AD+) and X (hematin)
• 25
N gonorrhoeae, Thayer-Martin agar Selectively favors growth of eisseria by
• 26
N meningitidis inhibiting growth of gram Etl organisms
. 27
with Vancomycin, gram 8 organisms except
• 28
Neisseria with Trimethoprim and Colistin,
• 29
and fungi with ;\'ystatin
• 30
• 31
Very Typically C ultures Neisseria
. 32 Bpertussis Bordet-Cengou agar (Bordet for Bordetella) Potato extract
. 33 Regan-Lowe medium C harcoal, blood, and antibiotic
• 34
Cdiphtheriae Tellurite agar, LofAer medium
• 35
• 36
M tuberculosis Lowenstein-Jensen agar
. 37 M pneumoniae Eaton agar Requires cholesterol
.
8
Lock
s
Suspend
0
End Block
Item: 18 of 60 ~ ,• Mark <::J [:::> ""I ~· ~'j
QID: 2012 J.. Previous Next LAbfaiUI~S Notes Calculator

• •
12 FA17 p 123.1
13 Special culture requirements
14
BUG MEDIA USED FOR ISOLATION MEDIACONTENTS/OTHER
15
H influenzae Chocolate agar Factors V ( AD+) and X (hematin)
16
17 N gonorrhoeae, Thayer-Martin agar Selectively favors growth of eisseria by
18 N meningitidis inhibiting growth of gram Etl organisms
• 19 with Vancomycin, gram 8 organisms except
• 20 Neisseria with Trimethoprim and Colistin,
. 21 and fungi with :\'ystatin
. 22 Very Typically Cultures :-.!eisseria
• 23
Bpertussis Bordet-Cengou agar (Bordet for Bordetella) Potato extract
• 24
Regan-Lowe medium Charcoal, blood, and antibiotic
• 25
• 26 Cdiphtheriae Tellurite agar, LofAer med ium
. 27 M tuberculosis Lowenstein-Jensen agar
• 28
M pneumoniae Eaton agar Requires cholesterol
• 29
• 30 Lactose-fermenting MacConkey agar Fermentation produces acid, causing colonies to
• 31 enterics turn pink
. 32
E coli Eosin-methylene blue (EMB) agar Colonies with green metallic sheen
. 33
• 34
Legionella Charcoal yeast extract agar buffered with
• 35
cysteine and iron
• 36 Fungi Sabouraud agar "Sab's a f un guy1"
.
. 37

. •
8
Lock
s
Suspend
0
End Block
Item: 18 of 60 - ,• Mark --<] [::> ""'I ~· 1!';:'1
QIO: 201 2 ~ Prev1o u s Next Labf a lues Note s Calculator

12 FA17 p 180.1
13 Sexually transmitted infections
14
DISEASE CLINICAL FEATURES ORGANISM
15
AIDS Opportunistic infections, Kaposi sarcoma, Ill
16
lymphoma
17
18 Chancroid Painful genital ulcer with exudate, inguinal Haemophilus ducreyi (it's so painful, you "do
. 19 adenopathy Cf) " )
• 20
Chlamydia Urethritis. cen·icitis, epididymitis, Chlamydia trachomatis (0 - K)
. 21
conjuncti\"itis, reacti\ e arthritis, PID
. 22

• 23
Condylomata Genital warts, koilocytes HP -6 and -11
• 24 acuminata
• 25 Genital herpes Painful penile, nilvar, or cer\"ical vesicles and HS -2, less commonly HSV-1
• 26 ulcers; can cause systemic symptoms such as
. 27 b ·er, headache, myalgia
• 28
Gonorrhea Urethritis, cervicitis, PID, prost ~1 1 it is, eisseria gonorrhoeae
• 29
• 30
epididymitis, arthritis, creamy purulent
• 31
discharge
• 32 Granuloma inguinale Painless, beefy red ulcer that bleeds readily on Klebsiella (Calymmatobacterium) granulomatis;
. 33 (Donovanosis) contact rJ cytoplasmic Donovan bodies (bipolar staining)
• 34 1 ot common in US seen on m1croscopy
• 35
• 36
. 37
. •
8
L.odt
s
Su~pl'nd
~
End Block
Item: 18 of 60 - ,• Mark --<] [::> ""'I ~· 1!';:'1
QIO: 201 2 ~ Prev1o u s Next Labf a lues Note s Calculator

• •
12 Gonorrhea Urethritis, cervicitis, PID, prostatit is, Neisseria gonorrhoeae
13 epididymitis, arthritis, creamy purulent
14
discharge
15
16
Granuloma inguinale Painless, beefy red ulcer that bleeds readil) on Klebsiella (Calymmatobacterium) granulomatis;
17
(Oonovanosis) contact rJ C) toplasmic Donovan bodies (bipolar staining)

18 1 ot common in US seen on m1croscopy


. 19
• 20
. 21
. 22
• 23
• 24
• 25 ll
• 26 Hepatitis B Jaundice HBV
. 27
Lymphogranuloma Infection of lymphatics; painless genital ulcers, C trachomatis (Ll- L3)
• 28
• 29
venereum painful lymphadenopathy (ie, buboes)
• 30 Primary syphilis Painless chancre Treponema pallidum
• 31 Secondary syphilis Fever, lymphadenopath)', skin rashes,
. 32
condylomata lata
. 33
• 34
Tertiary syphilis Gummas, tabes dorsalis, general paresis, aortitis,
• 35 Argyll Robertson pupil
• 36 Trichomoniasis Vaginitis, strawberry cer\"ix, motile in wet prep Trichomonas vagina/is
. 37
. •
8
L.odt
s
Su~pl'nd
~
End Block
Item: 19 of 60 - ,• Mark -<J [:::> "'I ~ · ~
QIO: 3969 ~ P~v•ous N@xt Labl lues Notes Calculator

12
An 18-month-old boy who recently im migrat ed t o t he Unit ed St ates with his family from Laos, comes to t he emergency department with ~~AI
13 fever and lethargy. On examinat ion he has a very stiff neck. The pediatrician pe1forms a lumbar punct ure. A Gram stain of t he
cerebrospinal flu id Is performed, revealing gram- negative coccobacilli.

-
14
15
I c-. I ........
Opef'llng I
wac oovnt GkiCOM

16 A
~ ~ f
17
8 ~ f ~ ~
18
c ~ f
-- ~ I
• 19
1
0 i ~ f 1
• 20
. 21
E
t f ~ t l
. 22
0 23 Which of the sets of laboratory values shown in the table will most likely be present?
0 24 :
A
0
25
0
26 B
. 27
c
• 28
• 29
D
• 30 E
0
31
. 32
. 33
0 34
0 35
0
36
. 37
.
8
Lode.
s
Suspe-nd
8
End Bloc:k
Item: 19 of 60 ~ ,• Mark <::J [:::> ""I ~· ~'j
QID: 3969 J.. Previous Next LAb faiUI~S Notes Calculator


12
13
The correct a nswer is E. 80°/o chose this .
14 This infant's signs and symptoms are consistent with bact erial meningit is, likely caused by Haemophilus influenzae. Most cases occur in
15 nonimmunized infant s. H. influenzae is a gram-negat ive coccobacillus. The t reat ment of choice is a t hird-generat ion cephalosporin, such as
16
ceftriaxone. I n addit ion to increased opening pressure on lumbar puncture, laboratory results on t he t apped cerebrospinal f luid will most likely
reveal elevat ed WBCs (with a neutrophil predominance), decreased glucose, and increased prot eins.
17 Cephalosporin Neutrophil Lumbar puncture Ceftriaxone Cerebrospinal fluid Haemophilus influenzae Meningitis Gram-negative bacteria Coccobacillus Bacterial meningitis Glucose Haemophilus Protein
18
A is not correct . 3°/o chose t his.
19
This set of cerebrospinal fluid laboratory values is not consist ent wit h bacterial meningit is. The opening pressure and WBC count are
• 20 decreased, which is opposite to that expect ed in bacterial meningit is.
. 21 Cerebrospinal fluid Meningitis Bacterial meningitis

. 22 B is not correct. 5% chose t his .


• 23 This set of cerebrospinal fluid laboratory values is not consist ent wit h bacterial meningit is. The opening pressure and prot eins would be
• 24 elevated, not decreased, as shown here .
Cerebrospinal fluid Meningitis Bacterial meningitis Protein
• 25
• 26 C is not correct. 8 °/o chose this .
. 27 This set of cerebrospinal fluid laboratory values is not consist ent wit h bacterial meningit is. The opening pressure in bact eri al meningit is is
elevated .
• 28
Cerebrospinal fluid Meningitis Bacterial meningitis
• 29
• 30
D is not correct. 4 °/o chose t his .
This set of cerebrospinal fluid laboratory values is not consist ent wit h bacterial meningit is. The WBC count is elevat ed in bacterial meningit is .
• 31
Cerebrospinal fluid Meningitis Bacterial meningitis
. 32
. 33
• 34
Bottom Line :
• 35 I n addit ion to elevat ed opening press ure upon lumbar pu nct ure in patients wit h bacterial meningit is, laborat ory test ing of t he obtained f luid
will reveal elevated WBCs and protein levels, but decreased glucose .
• 36 Lumbar puncture Meningitis Protein Lumbar Glucose Bacterial meningitis Lumbar vertebrae
. 37
. •
8
Lock
s
Suspend
0
End Block
Item: 19 of 60 - ,• Mark -<J [:::> "'I ~ · ~
QIO: 3969 ~ P~v1ous N@xt Labl lues Notes Calculator

12

13
l@l : fil ~11•1 for yea r: 2017 •
14 FI RST AIO FACTS

15
16 FA17 p 138.3

Small gram e (coccobacillary) rod. erosol


17
Haemophilus Vaccine contains type b capsular polysaccharide
18
influenzae transmission. l'\ontypeable (unencapsulated) (pol) ribos) Iribitol phosphate) conjugated
19
strains are the most common cause of mucosal to diphtheria toxoid or other protein. Ci,en
• 20
. 21
infections (otitis media, conjuncti' itis, bet"een 2 and 18 months of age.
. 22 bronchitis) as well as in"asi' e infections since Does not cause the Au (influenza virus does).
0 23 the \'accine for capsular type b was introduced.
0 24 Produces IgA protease. Culture on chocolate
0
25 agar, which contains factors V ( lAD+) and
0
26 (hematin) for growth; can also be grown with
. 27 S aureus, which provides factor V through the
• 28 hemolysis of RBCs. Ha EMOJ>hilus ce~ u scs
• 29 Epiglottitis (endoscopic appearance in r.J,
• 30 can be "cherry red" in chi ldren; "t·humb sign"
0
31 on x-ray m), Men ingitis, O titis media, and
• 32
Pneumonia .
. 33
Treatment: amoxicilli n +1- clavulanate for
34
mucosal infections; ceflriaxone for meningitis;
0

0 35
rifampin prophylaxis for close contacts.
0
36
. 37
.
8
Lode.
s
Suspe-nd
8
End Bloc:k
Item: 19 of 60 ~ ,• Mark <::J [:::> ""I ~· ~'j
QID: 3969 J.. Previous Next LAbfaiUI~S Notes Calculator


12
13
14 FA17 p 176.1
15 Common causes of meningitis
16 NEWBORN (0- 6MOl CHILDREN (6 M0- 6 YR) 6- 60YR 60YR+
17 Group B streptococci S pneumoniae S pneumoniae S pneumoniae
18 E coli N meningitidis N meningiticlis (#1 in teens) Gram 8 rods
19
Listeria H infiuenzae type B Enterovi ruses Listeria
• 20
Enteroviruses HSV
. 21
. 22
Give ceftriaxone and vancomycin empirically (add ampicillin if Listeria is suspected).
• 23 Viral causes of meningitis: enteroviruses (especially coxsackievirus), llSV-2 {llSV-1 = encephalitis), IIIV, West ile virus (also
• 24 causes encephalitis), VZV.
• 25 In HIV: Cryptococcus spp.
• 26 Note: Incidence of H infiuenzae meningitis has l greatly due to conjugate H infiuenzae vaccinations. Today, cases are usually
. 27 seen in unimmunized children .
• 28
• 29
FA17 p 176.2
• 30
• 31
CSF findings in meningitis
. 32
OPENING PRESSURE CELL TYPE PROTEIN GLUCOSE
. 33 Bacterial t t PM s t l
• 34
Fungai/TB t t lymphocytes t l
• 35
• 36
Viral Normal/t t lymphocytes Normal/t ormal
. 37
. •
8
Lock
s
Suspend
0
End Block
Item: 20 of 60 - ,• Mark -<J [:::> "'I ~ · ~
QIO: 4 958 ~ P~v•ous N@xt Labl lues No tes Calcula t o r

12 A A 34-year-old man presents to t he emergency department with a 3-day history of fat igue, malaise, headache, fever, and a rash on the ~A] A

13 buttocks. Past medical history is nonsignifi cant. Physical examination reveals an erythematous lesion on t he abdomen wit h central clearing,
as shown in the image.
14

15
16
17
18
19
• 20
. 21
. 22
0 23
0 24
0
25
0
26
. 27
• 28
• 29
• 30
0
31
• 32
. 33
What is a side effect of the treat ment for his cond it on?
0 34 :

0 35
A. Achilles tendon rupt ure
0
36 B. Nausea, vomitting, and flushing wit h alcohol consumption
. 37
C. Orange tears
.
8
Lode.
s
Suspe-nd
8
End Bloc:k
12
13
14

15
16
17
18
19
• 20
. 21
. 22

• 23
• 24
• 25
• 26
. 27
• 28
• 29
What is a side effect of the t reat ment fo r his cond iton?
• 30
:
• 31
A. Achilles tendon rupture
• 32
. 33 B. Nausea, vomitting, and flushi ng wit h alcohol consumption
• 34 C. Orange tears
• 35
D. Red Man Syndrome
• 36
. 37
E. Sun sensitivity
. •
8
L.odt
s
Su~pl'nd
~
End Block
Item: 20 of 60 ~ ,• Mark <::J [:::> ""I ~· ~'j
QID: 4958 J.. Previous Next LAbfaiUI~S Notes Calculator

• The correct a nswer is E. 52°/o chose this .


12
13
This patient has manifestations of earl y Lyme disease, which manifests wit h erythema chronicum migrans (ECM, which is sometimes referred
t o j ust as "erythema migrans") and nonspecif ic complaints of malaise, headache, fat igue, muscle pain, and j oint pain. The ECM rash typically
14 manifests wit hin 1 mont h of the tick bi te, bu t most patients do not actually recall a t ick bite. The rash typically has a "bull's eye" appearance
15 with an erythematous border surrounding an area of cent ral clearing . Lyme disease is caused by t he spirochet e Borrelia burgdorferi and
16
t ra nsmitt ed by t he Ixodes deer t ick. The most appropriat e t reat ment fo r Lyme disease is oral doxycycline. Doxycline, like t he ot her
t et racycline ant ibiot ics, causes sun sensitivity as well as discoloration of teet h in young children.
17 Ixodes scapularis Spirochaete Lyme disease Doxycycline Borrelia burgdorferi Tetracycline Erythema chronicum migrans Tetracycline antibiotics Myalgia Tick Arthralgia Malaise Erythema Headache Borrelia
18 Ixodes Antibiotics Rash Fatigue (medical)
19
A is not correct . 14% chos e this.
20
Ciprofloxacin is a quinolone used t o t rea t infect ions of the uri nary t ract, lungs and more. Achilles tendon rupt ure or ten donitis is a side effect
. 21 associat ed with t he fluo roquinolones, especially ciprof loxacin .
. 22 Quinolone Ciprofloxacin Tendinitis Achilles tendon Achilles tendon rupture Tendon Adverse effect Urinary system Lung

• 23 B is not correct. 14% chose this .


• 24 Met ronidazole is used to treat infections with protozoans and anaerobic bacteria, typically below t he diaphragm. Consum ing alcohol when
• 25 t aking metronidazole causes a disulfram like reaction wit h effects t hat can include nausea, vomitt ing, flushi ng of the skin, tachycardia and
• 26
shortness of breath .
Metronidazole Tachycardia Vomiting Nausea Dyspnea Anaerobic organism Protozoa Bacteria Alcoholic beverage Thoracic diaphragm Alcohol Flushing (physiology)
. 27
• 28 C is not co rrect. 8 °/o chose this .
Rifamp in is part of t he reg imen used to t reat t uberculosis. It causes orange discolorat ion of body f luids as a side effect.
• 29
Rifampicin Tuberculosis Adverse effect Side effect
• 30
D is not correct. 12% c hose this .
• 31
Vancomycin is t ypically reserved for t he t reatment of infections wit h ant ibiotic- resist ant bacteria such as methicillin- resistant Staphlococcus
. 32
aureus. I t can cause "red man syndrome," which consists of prurit is, an erythematous rash that involves the face , neck, and upper torso .
. 33 vancomycin Itch Erythema Antimicrobial resistance Rash Bacteria Torso

• 34
• 35
Bottom Line :
• 36
Early Lyme disease manifests with eryt hema chronicum migrans and nonspecific symptoms of malaise, headache, fatigue, muscle pain, and
. 37
j oint pain. Oral doxycycline is the t reat ment of choice .
. •
8
Lock
s
Suspend
0
End Block
Item: 20 of 60 ~ ,• Mark <::J [:::> ""I ~· ~'j
QID: 4958 J.. Previous Next faiUI~S
LAb Notes Calculator

12
13
14
141;fil·1i•J for year:[ 201 7
FIRST AID FA C T S .
•j .

15
16 FA17 p 142.4
17 Lyme disease Caused by Borrelia burgdorferi, which is A Key Lyme pie to the FACE:
18
transmitted by the Ixodes deer tick rJ (also Facial nerve palsy (typically bilatera I)
19
vector for Anaplasma spp. and protozoa Arthritis
20
Babesia). atural reservoir is the mouse (and Cardiac block
. 21
important to tick life cycle). Erythema migrans
. 22
• 23
Common in northeastern United States. Treatment: doxycycline (1st line); amoxicillin
• 24
Stage 1-early localized: erythema migrans and cefuroxime in pregnant women and
• 25
(typical "bu lls-eye" configuration is rn children.
• 26 pathognomonic but not a!ways present),
. 27 Au-l ike symptoms.
• 28 Stage 2-early disseminated: secondary lesions,
• 29 carditis, AV block, facial nerve (Bell) palsy,
• 30 migratory myalgias/transient arthritis.
• 31 Stage 3- late cl issemina ted : encephalopathies,
. 32 chronic arthritis.
. 33
• 34
• 35
• 36 FA17 p 188.1
. 37 Tetracyclines Tetracycline, doxycycline, minocycline.
.
8
Lock
s
Suspend
0
End Block
12
13
pathognomonic but not always present),
14
Au-like symptoms.
15
Stage 2-early disseminated: secondary lesions,
16
carditis, V block, facial nen e (Bell) pa ls),
17 migratory myalgias/transient arthritis.
18 Stage 3-late disseminated: encephalopathies,
19 chronic arthritis.
20
. 21
. 22

• 23 FA17 p 188.1
• 24 Tetracyclines Tetracycline, do:-.:ycycline, minocycline.
• 25 MECHANISM Bacteriostatic; bind to 30S and pre, ent attachment of aminoac) l-tR1 ; limited CNS penetration.
• 26 Doxycycline is fecally eliminated and can be used in patients with renal failure. Do not take
. 27
tetracyclines with milk (Ca 2+), antacids (Ca2+ or Mg2+), or iron-containing preparations because
• 28
di,·alent cations inhibit drugs' absorption in the gut .
• 29
• 30
CLINICAL USE Borrelia burgdor{eri, M pneumoniae. Drugs' abi lity to accumulate intracellularly makes them very
• 31
effective against Rickettsia and Chlamydia. Also used to treat acne. Doxycycl ine effective against
• 32 MRSA.
. 33 ADVERSE EFFECTS Cl distress, discoloration of teeth and inhibition of bone growth in children, photosensitivity.
• 34 Contraindicated in pregnancy.
• 35
MECHANISM OF RESISTANCE l uptake or t efflux out of bacterial cells by plasmid-encoded transport pumps.
• 36
. 37

.
8
L.odt
s
Su~pl'nd
~
End Block
Item: 21 of 60 - ,• Mark -<J [:::> "'I ~ · ~
QIO: 2208 ~ P~v•ous N@xt Labl lues No tes Calcula t o r

12
A 55-year-old man comes t o his physician wit h a tend er, swollen, and erythematous left knee. He has limit ed range of mot ion in his leg . On ~~AI
13 aspiration of the synovial f luid from his knee, t he f luid is found to be yellow and cloudy and has 150,000 neutrophils/mm 3 . Gram stain of
14 the aspirate shows gram-positive cocci in clusters.
15
16
The organism most likely responsible for t his pat ient' s symptoms has which of the fol lowing properties?
17 :

18
A. a-Hemolysis
19 B. Catalase-positive and coagulase -negative
20 C. Catalase-positive and coagulase -positive
. 21
0 . Glycoprotein capsule
. 22
0 23 E. Soluble in bile
0 24
0
25
0
26
. 27
• 28
• 29
• 30
0
31
. 32
. 33
0 34
0 35
0
36
. 37
.
8
Lode.
s
S uspe-nd
8
End Bloc:k
Item: 21 of 60 ~ ,• Mark <::J [:::> ""I ~· ~'j
QID: 2208 J.. Previous Next LAbfaiUI~S Notes Calculator


12
The correct answer is C. 81 °/o chose this.
13
This man is suffering from sept ic arthri t is, commonly charact eri zed by a swollen, tender, and eryt hematous j oint . The organism most
14 common ly responsible fo r this infect ion is Staphylococcus aureus. The infect ion result s from the invasion of t he bact eria into the synovial f luid .
15 The diagnosis of sept ic arthrit is requires aspirat ion of the synovial fluid, which appears yellow and t urbid with a predom inance of neut rophils.
When Staphylococcus is t he causative agent , Gram stain and cultu re of t he synovial f luid show gram-posit ive cocci in clust ers. S. aureus is
16
cat alase- posit ive and coagulase- posit ive.
17 Gram staining Septic arthritis Synovial fluid Staphylococcus aureus Gram-positive bacteria Staphylococcus Neutrophil Arthritis Coccus synovial membrane Erythema Bacteria Organism Infection
18
A is not correct. 3°/o chose this.
19
S. aureus displays a 13-hemolytic pattern, not an a- hemolyt ic pat tern .
20 Staphylococcus aureus

21
B is not correct. 10% chose this .
. 22
While S. aureus is cata lase-posit ive, it is not coagulase- negative. This answer instead descri bes t he properties of Staphylococcus epidermidis,
• 23 which is not a cause of septic arthrit is .
• 24 Staphylococcus epidermidis Septic arthritis Staphylococcus Staphylococcus aureus Arthritis

• 25 D is not correct. 4°/o chose this .


• 26 S. aureus does not possess a glycoprotein capsule. Streptococcus pneumoniae is a gram- posit ive cocci t hat does possess a glycoprot ein
. 27 capsule but grows in chains .
Streptococcus pneumoniae Glycoprotein Gram-positive bacteria Coccus Streptococcus Staphylococcus aureus
• 28
• 29 E is not correct. 2°/o chose this .
• 30 Bile solubility is not a cha racteristic prope1ty of S. aureus; instead, it is a property of Streptococcus pneumoniae, which is not a likely cause of
sept ic arthrit is.
• 31 Streptococcus pneumoniae Septic arthritis Streptococcus Staphylococcus aureus Bile Arthritis Solubility
. 32
. 33
• 34
Bottom Line:
• 35 S. aureus is a gram -posit ive, catalase -posit ive, coagulase-posit ive cocci in clust ers t hat is a common cause of septic art hrit is .
Septic arthritis Gram-positive bacteria Coccus Staphylococcus aureus Arthritis
• 36
. 37
. •
8
Lock
s
Suspend
0
End Block
Item: 21 of 60 - ,• Mark --<] [::> ""'I ~· 1!';:'1
QIO: 2 208 ~ Prev1o u s Next Labf a lu es Notes Calculator

12
13 l@l :fi1·1hi for year: 2017 •
fiRST AID ,ACTS
14

15 FA17 p 131 .3
16
17
Staphylococcus au reus Gram EB, ~-hemolytic, catalase EB, coagulase TS T-1 is a superantigen that binds to 1HC
18
(f) cocci in clusters Ll Protein (virulence II and T-cell receptor, resulting in polyclonal
19 factor) binds Fc-lgG, inhibiting complement '(: cell activation.
20 acti,·ation and phagocytosis. Commonly Staphylococcal toxic shock syndrome
21 colonizes the narcs, axilla, and groin. (T ) presents as fever, vomiting, rash,
. 22 Causes: desquamation, shock, end-organ failure. TSS
• 23 Inflammatory disease-skin infections, results in t AST, t ALT, t bilirubin. ssociated
• 24 organ abscesses, pneumonia (often after with prolonged use of ,·aginal tampons or nasal
• 25 influenza virus infection}, endocarditi ~. packing.
• 26 septic arthritis, and osteomyelit is. Compare with Streptococcus pyogenes TSS (a
. 27
Toxin-mediated disease-toxic shock toxic shock- like syndrome associated with
• 28
syndrome (TSS1 ~l }, scalded skin syndrome painful skin infection).
• 29
(exfoliative toxin), rapid-onset food S aureus food poisoning due to ingestion of
• 30
poisoning (cntcrotoxins). preformed toxin _. short incubation period
• 31
MRSA (methicillin-resistantS aureus) (2-6 hr) followed by nonbloody diarrhea
• 32
infection- important cause or serious and emesis. Enterotoxin is heat stable -+ not
. 33
• 34
nosocomial and community-acquired destroyed by cooking.
• 35
infections; resistant to methicillin and B<1d staph (aureus) make coagulase and to-:ins .
• 36
nafcillin because of altered penicillin- Forms fibrin clot around self _. abscess .
. 37 binding protein .
. •
8
L.odt
s
Su~pl'nd
~
End Block
Item: 21 of 60 - ,• Mark -<J [:::> "'I ~ · ~
QIO: 2208 ~ P~v•ous N@xt Labl lues No tes Calcula t o r

A A
12 FA17 p 130.1
13
Gram-positive Jab algorithm
14

15
Gram<±> (purple/blue)
16
17
18
Branching
Bacllb COCCI
19 filaments
20
21
Aerobic Anaerobic Anaerobic/facultabve Aerobic Anaerobic
. 22
0 23
Listem
Nocardia Actinomyces
0 24 Baciffus Clostridium
Corynebacterium Propion1bactenum (weakly add fast) (not acid fast)
0
25
0
26 Catalase
. 27 r ---18>-- -
• 28 Streptococcus Staphylococcus
• 29 T.
Hemolysas Coagulase
• 30
j_
0
31
(Partial (Complete
• 32
. 33
a hemolysis,
green)
hemolysis,
clear)
y (No hemolysis,
grows in bile)
Saureus
0 34 Novobiocin
35 Optochin sensitivity sensitivity
0
Bacitracin sensitivity Growth in 6.5% NaCI
0
36 and bile solubility
. 37
.
8
Lode.
s
S uspe-nd
8
End Bloc:k
Item: 21 of 60 - ,• Mark -<J [:::> "'I ~ · ~
QIO: 2208 ~ P~v•ous N@xt Labl lues No tes Calcula t o r

12
• •
Usteria
Nocardia Actinomyces
13 Bacillus Clostridium
Propion1bactenum (weakly acid fast) (not acid fast)
14
Corynebacterium

15 Catalase
.!__
16
-~e>---
17 Streptococcus Staphylococcus
18
Hemolysis Coagulase
19
20
(Partial (Complete
. 22
21
a. hemolysis, hemolysis.
clear)
(No hemolysis.
grows in bile)
green) Saureus
0 23 Novobiocin
0 24 Optochin sensitivity Bacitracin sensitivity Growth in 6.5% NaCI sensitivity
0
25 and bile solubility
0
26
. 27
Group B Group A SSilprophyticus Sepidermidis
• 28
Sagalilctiae Spyogenes
• 29
• 30
0
31
Viridans streptococci Group D
. 32 (no capsule) Spneumoniae Nonenterococcus (enterococcus)
. 33 Smutans (encapsulated) SOOVIS Efaecium
Smitis Efaecalis
0 34
0 35 Important tests are 1n bold. Important pathogens are in bold ilillics.
Note: Enterococcus 1S either a- or y-hemolytic.
0
36
. 37
.
8
Lode.
s
S uspe-nd
8
End Bloc:k
Item: 22 of 60 - ,• Mark -<J [:::> "'I ~ · ~
QIO: 402 1 ~ P~v•ous N@xt Labl lues No tes Calcula t o r

12 A A 50-year-old homeless man presents t o t he emergency departm ent wit h several days of fever, chills, and general malaise coupled wit h th e ~AI A

13 inability to bear weight on his ri ght foot . On quest ioning, the pat ient admits to intravenous drug abuse, and physical examinat ion reveals
warmth and erythema over the dorsolat eral aspect of his foot. A raph of his foot is shown.
14

15
16
17
18
19

20
21
. 22
0 23
0 24
0
25
0
26
. 27
• 28
• 29
• 30 What is t he most likely pathogen responsible for t he pat ient's illness?
0
31 :

• 32
A. Mycobacterium tuberculosis
. 33 B. Neisseria gonorrhoeae
34
0
C. Salmonella typhi
0 35
D. Staphylococcus aureus
0
36
. 37 E. Staphylococcus epidermidis
.
8
Lode.
s
Suspe-nd
8
End Bloc:k
Item: 22 of 60 - ,• Mark --<] [::> ""'I ~· 1!';:'1
QIO: 402 1 ~ Prev1o u s Next Labf a lues Notes Calculator

12

13
The correct a nsw er is D. 80°/o chose this.
14
Staphylococcus aureus is the most common cause of osteomyelitis in the general population. In the intravenous drug user population,
15 Pseudomonas aeruginosa should also be considered as a highly likely candidate, but it is still less common t han S. aureus.
16 Osteol'""' el1t1~ Pseudomon~ aerug1nosa Drug injection Staphylococcus au reus Staph lococcu~ Intra....enou~ therap Pseudomonas

17 A is not correct. 2°/o c hose this.


18 Mycobacterium tuberculosis should be considered as the most likely pathogen in vertebral osteomyelitis (also known as Pott disease).
19 Osteo reub~ ,.. ob,l!"MH um tuberculos•s Pott disease Tuberculosis Mycobacterlum Pathogen lertebra. oste 1 ,..,t•s

20 B is not correct. 4 °/o chose this.


21 Neisseria gonorrhoeae is a rare cause of osteomyelitis in sexually active individuals, but more often results in septic arthritis.
22 Osteo nye •t Nt. 'e a oonorrhoeae Sepbc arthritis Neisseria Arthritis

• 23 C is not correct. 5°/o chose this.


• 24 Salmonella species are most often seen as the cause of osteomyelitis in patients with sickle cell disease .
• 25 Osteomyeht1tt ;i.•ckle~cell d•seMe St.lmonella Sickle Species

• 26 E is not correct. 9°/o chose thi s .


. 27 Staphylococcus epidermidis is not as common a cause of osteomyelitis as isS. aureus .
• 28 OsteomyelitiS Staphylococcus ep•derm1d1s Staphylococcus Staphylococcus aureus

• 29
• 30 Bottom Line:
• 31
Staphylococcus aureus is t he most common cause of osteomyelit is.
. 32 OsteomyelitiS Staphylococcus aureus Staphylococcus

. 33
• 34
• 35 141:fi1·1i•i
,-lltST AID ,ACTS
for yea r: 2017 •

• 36
. 37
FA17 p 176.4 •
.
8
L.odt
s
Su~pl'nd
~
End Block
Item: 22 of 60 ~ ,• Mark <::J [:::> ""I ~· ~'j
J.. faiUI~S
QID: 4021


.. . Previous Next LAb Notes Calculator

12
Staphylococcus au reus is t he most common cause of osteomyelit is.
13 Osteomyelitis Staphylococcus aureus Staphylococcus

14
15
16 141;fil·1i•J toryear:[ 2a1 7
FIRST AID FA CTS -
•J .
17
18
FA17 p 176.4
19
20 Osteomyelitis RISK FACTOR ASSOCIATED INFECTION
21 Assume if no other information is available S aureus (most common overall)
22
Sexually active Neisseria gonorrhoeae (rare), septic arthritis more
• 23
common
• 24
• 25 Sickle cell disease Salmonella and S aureus
• 26 Prosthetic joint replacement S aureus and S epidennidis
. 27
Vertebra I involvement S aureus, Mycobacterium tuberculosis (Pott
• 28
disease)
• 29
• 30 Cat and clog bites Pasteurella multocida
• 31 IV drug abuse Pseudomo11as, Ca11dida, S aureus are most
. 32
common
. 33
• 34
Elevated C-reactive protein (C RP) and eryth rocyte sedimentation rate common but nonspecific.
• 35
M Rl is best for detecting acute infection and detailing anatomic involvement f.i.l Radiographs are
• 36
insensitive early but can be usefu l in chronic osteomyelitis rn.
. 37
. •
8
Lock
s
Suspend
0
End Block
Item: 23 of 60 - ,• Mark --<] [::> ""'I ~· 1!';:'1
QIO: 2053 ~ Prev1o u s Next Labf a lu es Notes Calculator

12

A 10-year-old boy is re ferred to a neurologist after declining school performance, mood swings, generalized seizures, and visual
13 disturbances that have worsened over the past fe w month s. Cerebrospinal fluid analysis shows a WBC count of 2 cells/u l , glucose level of
14 SO mg/dL and protein of 30 mg/d L, and culture shows no bact erial growth. The patient is afebrile and reports no headache. On rev iew of
the medical record, a pathology report of respirat ory secretions obtained from the patient at the age of 3 was posit ive for multinucleated giant
15 cells with intracytoplasmic and intranuclear inclusions. At that ti he exhibited a rash resembling the one shown in the image.
16
17
18
19
20
21
22
• 23
• 24
• 25
• 26
. 27
• 28 Image cowtesy . Heinz F. t:ichenwald
• 29
• 30 If th e patient were to undergo lumba r puncture now, ant ibodies against what pathogen are likely t o be fou nd in the sample?
• 31 42883S :
. 32 A . Herpes simplex virus t ype 2
. 33 B. Measles virus
• 34
C. Mumps virus
• 35
• 36 D. Neisseria meningitidis
. 37 E. Rubella virus
. •
8
L.odt
s
Su~pl'nd
~
End Block
Item: 23 of 60 - ,• Mark -<J [:::> "'I ~ · ~
QIO: 2053 ~ P~v•ous N@xt Labl lues Notes Calculator

12 A 50 mg/dL and protein of 30 mg/dL, and cultu re shows no bacterial growth. The patient is afebri le and reports no headache. On rev iew of A

the medical record, a pathology report of respiratory secretions obtained from the pat ient at the age of 3 was posit ive for mult inucleated giant
13
cells with intracytoplasmic and intranuclear inclusions. At t hat t ime, he exhibited a rash resembling t he one shown in t he image.
14

15
16
17
18
19

20
21
22
' 23
' 24
' 25
' 26 I mage courtesy CDC/ Or. Heinz F. Eichenwald
. 27
• 28 If th e pat ient were t o undergo lumbar puncture now, ant ibodies against wh at pathogen are likely t o be fou nd in the sample?
• 29 :
• 30 A. Herpes simplex virus t ype 2
' 31 B. Measles virus
. 32
C. Mumps virus
. 33

' 34 D. Neisseria meningitidis


' 35 E. Rubella virus
' 36
F. Treponema pallidum
. 37
.
8
Lode.
s
Suspe-nd
8
End Bloc:k
Item: 23 of 60 - ,• Mark -<J [:::> "'I ~ · ~
QIO: 2053 ~ P~v•ous N@xt Labl lues Notes Calculator

A A
12
13 The correct answ er is B. 53% chose this.
14 This patient is most likely suffering from subacute sclerosing panencephalitls. This is a rare
progressive demyelinating disease associated with chronic central nervous system infection with
15 measles virus. There is often a history of primary measles infection at an early age
16 (approximately 2 years) followed by a latent interval of 6-8 years. Initial manifestations include
17 poor school performance and mood and personality changes. Fever and headache do not occur.
As the disease progresses, patients develop progressive intellectual deterioration, focal and/ or
18
generalized seizures, myoclonus, ataxia, and visual disturbances. The cerebrospinal fluid (CSF) is
19 acellular with normal or mildly elevated protein and markedly elevated gamma-globulin (>20%
20 of total CSF protein). CSF antimeasles antibodies are also elevated. CT and MRI show evidence
of multifocal white matter lesions (indicated by the arrows in this image), cortical atrophy, and
21
ventricular enlargement. Classic measles presentation includes a red maculopapular rash that
22 spreads downward from the head, cough, coryza, and high fever. Koplik spots are seen in the
23
buccal mucosa, and Warthin-Finkeldey cells in the respira tory secretions are pathognomonic for Images copyright© Indian 1 Radio/ !mag
measles .
• 24
Subacute ~clerostng panencephallbs Cerebrospinal fluid Measles Oem,telinating dtsease Central nervous svster .. Myoclonus Pathognomonic Maculopapular rash White matter Pt:axia Rhtnttts Anttbody
• 25
Headache Measles vtrus V~rus Oral mucosa Magnetic resonance imaging Epileptic seizure Atrophy Cough Protem Rash Nervous system Mucous membrane Fever Infection Acute (medtctne) Myeltn
• 26
Gamma globul1n
. 27
A is not correct. 18°/o chose this .
• 28
Herpes simplex virus 2 (HSV-2) can cause a recurrent meningitis. As with the other examples, one would expect to see signs of meningeal
• 29
irritation as well as an increase in CSF lymphocyt es. Infection with HSV-2 is often associated with genit al lesions .
• 30 Herpes simplex virus Mentngtbs Vtrus Herpes simplex Lymphocyte Meninges Sex organ

• 31
C is not correct . 7°/o chose this .
. 32
The patient's symptoms are not consistent with mumps infection. Mumps virus can cause an acute viral meningitis, but one would expect to
. 33 see classic signs of meningit is (eg, headache and nuchal rigidity), as well as an increase in lymphocytes in the CSF.
Mumps V1ral men1ng1tht Men1ng1t1s Neck stiffness Headache Virus Mumps virus Memng1sm Infect1on Lymphocyte
• 34
• 35 0 is not correct . 6 °/o c hose this .
• 36 Neisseria meningitidis can cause bacterial meningitis. These organisms would likely be discovered on cult ure of t he CSF. Patients with bacterial
. 37 meningitis typically have a high fever with meningeal signs, as well as CSF abnormalities, including decreased glucose, increased protein, and
. the presence of mononuclear and/or polymorphonuclear cells .

8
Lode.
s
Suspe-nd
8
End Bloc:k
Item: 23 of 60 ~ ,• Mark <::J [:::> ""I ~· ~'j
QID: 2053 J.. Previous Next LAb faiUI~S Notes Calculator

12 E is not correct. 1 2°/o chose this.


13 Rubella virus causes German measles, which is generally characterized by fever and upper respiratory symptoms t hat resolve with subsequent
14 rash. The maculopapular rash usually starts with th e face and descends to the extremit ies, last ing only several days.
Measles Rubella Maculopapular rash Rubella virus Virus Fever Rash
15
16 F is not correct . 4 °/o chose this.
17 I nfection wit h Treponema pal/idum can eventu ally lead t o neurosyphilis, which can include some of the symptoms described in this case. The
patient has neit her elevated CSF protein nor the presence of mononuclear cells. There is also litt le evidence in the history t o suggest prior or
18
current infect ion wit h T. pal/idum .
19 Neurosyphilis Treponema pallidum Syphilis Treponema Protein Infection Cerebrospinal fluid

20
21
Bottom Line :
22
Subacute sclerosing panencephalit is is a rare complication of prior measles infection t hat causes progressive neurologic disease leading t o
23
death. I t usually causes a bland CSF with normal glucose and norma l or mildly elevat ed prot ein .
• 24 Subacute sclerosing panencephalitis Measles Protein Neurological disorder Glucose Acute (medicine) Neurology Infection

• 25
• 26
. 27 i@l;fil·1i•J for yea r:[2017 • J
FIRST AID FACTS
• 28
• 29
FA17 p 166.2
• 30
• 31 Measles (rubeola) A paramyxovirus that causes measles. Usual 3 C 's of measles:
. 32 virus presentation involves prodroma l fever with Cough
. 33 cough, coryza, and conjunctivitis, then Coryza
• 34 eventually Koplik spots (bright reel spols with Con junctivit is
• 35 blue-white center on buccal mucosa D ), Vitamin A supplementation can reduce
• 36 followed 1- 2 days later by a maculopapular morbidity and mortality from measles,
. 37
.
rn
rash that starts at the head/neck and spreads particularly in malnourished children .

8
Lock
s
Suspend
0
End Block
12 Subacute scleros1ng panencephahbs Measles Protein Neurological disorder Glucose Acute (medicine) Neurology Infect1on

13
14

15 l@l ; fii ~11•J for year: 2017 •


FI RST AIO FACTS
16
17 FA17 p 166.2
18
19
Measles (rubeola) A para myxovirus that causes measles. Usual 3 C's of measles:
20
virus presentation im·olves prodromal fe\er \\ ith Cough
21
cough, coryza, and conjunctivitis, then Coryza
22 e\entually Koplik spots (bright red spots\\ ith Conjunctivitis
23 blue-white center on buccal mucosa · ), Vitamin A supplementation can reduce
• 24 followed 1-2 days later by a maculopapular morbidity and mortality from measles,
• 25 rn
rash that starts at the head/neck and spreads particularlr in malnourished ch ildren .
• 26 downward. Lymphadenitis with Warthin-
. 27 Finkeldey giant cells (fused lymphocytes) in
• 28 a background of p;~racort i cal hyperplasia .
• 29 SSPE (subacute sclerosing panencephalitis,
• 30
occurring years later), encephalitis (1:2000),
• 31
and giant cell pneumonia (rarely, in
• 32
immunosuppressed) arc possible sequelae.
. 33

• 34
• 35
• 36
. 37
.
8
Lode.
s
Suspe-nd
8
End Bloc:k
Item: 24 of 60 - ,• Mark -<J [:::> "'I ~ · ~
QIO: 24 35 ~ P~v•ous N@xt Labl lues No tes Calcula t o r

12
A 57-year-old man with a history of alcoholism is found unconscious on the floor of his bedroom and taken t o t he hospital. In the ~~AI
13 subsequent days, he develops shortness of breath and a cough productive of foul-smelling sputum. On physical examination, wheezing is
14 prominent over the posterior right lower lobe. Based on the patient's resp iratory culture results, he is started on int ravenous clindamycin.
15
16
Which of the following is the major category of organisms that is most likely responsible for this patient's condition?
17 :

18
A. Anaerobes
19 B. Enterics
20 C. Gram-positive aerobes
21
0 . Obligate aerobes
22
23 E. Obligate intracellular bacteria
. 24
• 25
• 26
. 27
• 28
• 29
• 30
• 31
. 32
. 33
• 34
• 35
• 36
. 37
.
8
Lode.
s
Suspe-nd
8
End Bloc:k
Item: 24 of 60 ~ ,• Mark <::J [:::> ""I ~· ~'j
QID: 2435 J.. Previous Next LAbfaiUI~S Notes Calculator

12 The correct a nswer is A. 57°/o chose t h is.


13 This is a classic case of a lung abscess caused by probable aspirat ion of anaerobic organisms f rom t he oral cavit y. Those who have lost
14 consciousness, are debilitat ed, or are alcoholics are particularl y suscept ible to anaerobic lung infection due to ineffective gag ref lex and
increased ri sk of inhaling bact eri a f rom t he oral cavity. These abscesses are oft en m ixed infections t hat also include m icroaerophiles. The fo ul-
15
smelling sputum and history of unconsciousness point to a diagnosis of aspiration pneumonia. I ntravenous clindamycin or penicillin are usually
16 f irst- line choices to treat aspirat ion pneumonia.
17 Clindamycin Penicillin Lung abscess Aspiration pneumonia Sputum Anaerobic organism Pneumonia Pharyngeal reflex Abscess Intravenous therapy Bacteria Lung Mouth Pulmonary aspiration

18 Respiratory tract infection Infection Alcoholism

19 B is not correct. 16% chose this .


20 Enteric bacteri a are a diverse group of bact eri a t hat populate t he int estinal t ract. One common exam ple of an enteric bacteria associat ed with
21 pneumonia is Klebsiella, which is a mem ber of the Ent erobact eriaceae family. All of t he members of t his family are facultat ive anaerobes.
Klebsiella ty pically causes a lobar pneumonia (often seen in alcoholics) with a cough product ive of sput um t hat is classically described as
22
"currant jelly." The fact t hat t his patient is being t reat ed wit h cl indamycin is an addit ional clue to the correct answe r~ as clindamycin cannot
23 permeate the outer envelope of t he microbe and t hus is not effect ive against the Ent erobact eriaceae.
24 Enterobacteriaceae Clinda,.,cin Sputum Facultative anaerobic organism Microorganism Bacteria Pneumonia Klebsiella Lobar pneumonia Gut flora Anaerobic organism Cough Gastrointestinal tract

• 25 C is not correct. 11% chose this .


• 26 Gram -posit ive aerobes such as Streptococcus pneumoniae ra rely cause abscesses alone. These infect ions t end to be more patchy or lobar
. 27 with consolidation rather t han cavita ry.
Streptococcus pneumoniae Gram-positive bacteria Streptococcus Anaerobic organism Aerobic organism Abscess Cellular respiration
• 28
• 29 D is not correct. 10% c hose this .
• 30 Obligate aerobes such as Pseudomonas can cause lung abscesses, especially nosocomially, but the history points t o aspirat ion pneumonia .
Aspiration pneumonia Pneumonia Anaerobic organism Aerobic organism Pseudomonas Abscess Hospital-acquired infection Cellular respiration Lung Pulmonary aspiration
• 31
. 32 E is not correct. 6°/o chose this .
. 33 Obligate int racellu lar bact eria such as Chlamydia or Mycoplasma can cause atypical, or "walking, " pneumonia, which does not manifest wit h
such striking lung f indings on physical examinat ion. Atypical pneumonia charact eri st ically manifests with mild-to -moderat e changes in lung
• 34
sounds in all lung f ields as well as a nonprod uct ive cough .
• 35 Atypical pneumonia Pneumonia Bacteria Mycoplasma Intracellular Cough Obligate intracellular parasite Chlamydia (genus) Chlamydia infection Lung Physical examination

• 36
. 37
Bottom Line :
.
8
Lock
s
Suspend
0
End Block
Item: 24 of 60 ~ ,• Mark <::J [:::> ""I ~· ~'j
QID: 2435 J.. Previous Next LAbfaiUI~S Notes Calculator

12
Bottom Line:
13
Anaerobic organisms commonly cause lung abscess in pat ients who have lost conciousness, alcoholics, and debilitated persons. One key sign
14 is fou l-smelling sputu m .
15 Sputum Lung abscess Abscess Anaerobic organism Lung Alcoholism

16
17
18 l@l;fil·1i•l f or yea r:[ 2017
FIRST AID FACTS .
•j .
19
20 FA17 p 645.1
21
Pneumonia
22
TYPE TYPICAL ORGANISMS CHARACTERISTICS
23
24
Lobar S pneumoniae most frequently, also Legionella, Intra-alveolar exudate -+ consolidation rl; may
• 25
Klebsiella rn
iJwolve entire lobe or lung.
• 26 Bronchopneumonia S pneumoniae, S aureus, H influenzae, Acute infl ammatory infiltra tes [!1 from
. 27 Klebsiella bronchioles into adjacent alveoli; patchy
• 28 m.
distribution involving;;:: 1 lobe
• 29
Interstitial (atypical) Mycoplasma, Chlamydophila pneumoniae, Diffuse patchy inflammation localized to
• 30
pneumonia Chlamydia psittaci, Legionella, viruses (RSV, interstitial areas at alveolar walls; diffuse
• 31
. 32
C.MV, influenza, adenovirus) distribution involving;;:: 1 lobe D. Generally
. 33
follows a more indolent course ("walking"
• 34
pneumonia).
• 35 Cryptogenic Formerly known as bronchiolitis oblitera ns
• 36 organizing organizing pneumonia (BOOP). Noninfectious
. 37 pneumonia pneumonia characterized by inflammation of
.
8
Lock
s
Suspend
0
End Block
Item: 24 of 60 - ,• Mark --<] [::> ""'I ~· 1!';:'1
QIO: 24 35 ~ Prev1o us Next Labf a lues Notes Calculator

12

13
14

15
16
17
18
19
20
21
FA17 p 646.1
22
23 Lung abscess Localized collection of pus within Air-fluid b ·els [E) often seen on CXR. Fluid
24 parenchyma . Caused by aspiration of le' els common in ca\'ities; presence suggests
• 25 oropharyngeal contents (especia lly in patients cavitation. Due to anaerobes (eg, Bacteroides,
• 26 predisposed to loss of consciousness [cg, Pusobacterium, Peptostreptococcus) or S aureus .
. 27
alcoholics, epileptics]) or bronchial obstruction Lung abscess 2° to aspiration is most often found
• 28
(eg, cancer) . in right lung. Location depends on patient's
• 29
Treatment: clindamycin. position during aspiration .
• 30
• 31
. 32
. 33
• 34
• 35
• 36
. 37
. •
8
L.odt
s
Su~pl'nd
~
End Block
Item: 25 of 60 - ,• Mark -<J [:::> "'I ~ · ~
QIO: 2029 ~ P~v•ous N@xt Labl lues Notes Calculator

12
A 26-year-old woman presents with purulent vag inal discharge, lower abdominal pain, and fever. She reports having engaged in ~~AI
13 unprotected sexual intercourse on multiple occasions. Physical examination reveals cervical motion tenderness, right adnexal tenderness,
14 and bilateral abdominal tenderness.
15
16
In light of this presentation, treatment with which of the following antimicrobial drug regimens should be initiated?
17 :

18
A. Acyclovir
19 B. Ceftriaxone-azithromycin
20 C. Metronidazole
21
0 . Penicillin
22
23 E. Trimethoprim-sulfamethoxazole
24
' 25
' 26
' 27
• 28
• 29
• 30

' 31
. 32
. 33

' 34
' 35
' 36
. 37
.
8
Lode.
s
Suspe-nd
8
End Bloc:k
Item: 2S of 60 ~ ,• Mark <::J [:::> ""I ~· ~'j
QID: 2029 J.. Previous Next LAbfaiUI~S Notes Calculator

12
13 The correct answer is B. 70% chos e this.
14 This patient present s with signs and symptoms consistent with pelvic inf lammat ory disease (PID), which is caused most commonly by
Chlamydia trachomatis and/or Neisseria gonorrhoeae. Azi thromycin is a macrolide antibiot ic and the first -line drug for t reating chlamydia.
15 Ceftriaxone is a t hird-generat ion cephaslospori n and t he drug of choice for t rea t ing gonococcal infect ions. Although Chlamydia is the most
16 common cause of PID, t he t wo ant ibiotics are typically given t oget her because of t he high rat e of coinfection wit h Chlamydia and N.
17 gonorrhoeae.
Pelvic inflammatory disease Macrolide Chlamydia trachomatis Azithromycin Ceftriaxone Neisseria gonorrhoeae Antibiotics Chla,.,dia infection Chlamydia (genus) Gonorrhea Neisseria Inflammation
18
19 A is not correct. 3°/o chose this.
20
Acyclovir is an ant iviral drug used to t reat herpes simplex infections. Herpes simplex virus-2 (HSV-2) may cause genit al herpes but would not
account for t he purulent vaginal discharge or signs and symptoms of pelvic inf lammat ory disease seen in this patient .
21 Pelvic inflammatory disease Antiviral drug Aciclovir Herpes simplex virus Genital herpes Herpes simplex Vaginal discharge Pus Inflammation
22
C is not correct. 12% chose this.
23
Met ronidazole is the agent of choice fo r t reat ing vaginit is caused by both Trichomonas vagina/is and Gardnere/la vagina/is. These organisms
24 cause vulvovaginit is, associated wit h genital pruri tus and foul-smelling vaginal discharge, but would not cause the signs and sympt oms of
25 pelvic inf lammat ory disease seen in this patient .
Pelvic inflammatory disease Metronidazole Trichomonas vaginalis Gardnerella vaginalis Vaginitis Vaginal discharge Itch Trichomonas Pelvis
• 26
. 27 D is not correct. 7°/o chose this .
• 28 Penicillin is the t reat ment choice for syphilis infections caused by t he spirochete Treponema pa/lidum . Syphilis is not a cause of pelvic
• 29
inflammatory disease (PID) and would t herefore not present with t he physical exam f indings seen in this patient.
Pelvic inflammatory disease Spirochaete Treponema pallidum Penicillin Syphilis Treponema Inflammation
• 30
• 31 E is not correct. 8°/o chose this .
. 32
Trimet hoprim -sulfamet hoxazole, also known as Bactri m, is t he drug of choice fo r urina ry t ract infections (UTis), t ypically caused by gram-
negative organisms such as Escherichia coli. Sexually t ransmitted diseases and UTis may bot h present wit h dysuria, but a UTI would not
. 33 cause purulent vaginal discharge or t he signs and symptoms of pelvic inf lammat ory disease seen in this patient .
• 34 Pelvic inflammatory disease Dysuria Escherichia coli Trimethoprim/sulfamethoxazole Gram-negative bacteria vaginal discharge Urinary tract infection Urinary system Pus Sexually transmitted infection

• 35 Inflammation

• 36
. 37
Bottom Line:
.
8
Lock
s
Suspend
0
End Block
Item: 25 of 60 - ,• Mark --<] [::> ""'I ~· 1!';:'1
QIO: 2029 ~ Prev1o u s Next Labf a lu es Notes Calculator


12 Bottom Line:
13 PID is caused most commonly by C. trachomatis and/or N. gonorrhoeae. Treatment with both ceftriaxone and azithromycin is typically
14 initiated in patients with PID because of the high rate of coinfection with the two organisms.
Aztthrom'y\,;ln Ceftr~a one Ne•~-seria gonorrhoeae Pelvic inflammatory disease Chlamydta trachomat1s
15
16
17
18
141;t'il;1i•J for year: 2017 •
FlltST AIO FACTS

19
20 FA17 p 146.1
21
Chlamydiae Chlamydiae cannot make their own TP. They Chlam)'S =cloak (intracellular).
22
are obligate intracellular organisms thai cause C f' sillaci-has an avian reser\'oir (parrots),
23
24
mucosal infections. 2 forms: causes at~ pica I pneumonia.
25
Elementary body (sma ll, dense) Lab diagnosis: PC R, nucleic acid amplification
• 26
is "Enfcctious" and Enters cell via lest. Cytoplasmic inclusions (reticulate bodies)
. 27 Endocrtosis; transforms into reticulate body. seen on Cicmsa or Auorescent antibody-
• 28 Reticulate body Replicates in cell by fission; stained smcm.
• 29 Reorganizes into elementary bodies. T he chlamydia! cell wall lacks classic
• 30 Chlamydia trachomatis causes reactive arthritis peptidoglycan (due to reduced muramic acid),
• 31 (Reiter syndrome), follicular conjunctivitis I'J, rendering ~-lactam antibiotics ineffective.
. 32 nongonococcal urethritis, and PI D.
. 33 Chlamydophila pneumoniae and Chlamydophila
• 34 psittaci cause atypical pneumonia; Ira nsmittcd
• 35 by aerosol.
• 36 Treatment: azithromycin (f<l\'Ored because one-
. 37
time treatment) or doxycycline (+ ceftria\one •
.
8
L.odt
s
Su~pl'nd
~
End Block
Item: 2S of 60 ~ ,• Mark <::J [:::> ""I ~· ~'j
QID: 2029 J.. Previous Next LAbfaiUI~S Notes Calculator

• •
12 FA1 7 p 146.2
13 Chlamydia trachomatis serotypes
14
Types A, B, and C Chronic infection, cause bli ndness due to ABC = Afric<l, Blindness, C hronic infection.
15
follicular conjunctivitis in Africa.
16
17 Types D- K Urethritis/PID, ectopic pregnancy, neonatal 0 - K = everythi ng else.
18 pneu monia (staccato cough) with eosinophilia, eonatal disease ca n be acquired during
19 neonatal conjunctivitis (l-2 weeks after birth). passage through infected birth canal.
20
Types L1, L2, and L3 Lymphogranuloma venereum-small, painless
21
ulcers on genitals ..... S\\·ollen, painful inguinal
22
lymph nodes that ulcerate (buboes). Treat with
23
doxycycl ine.
24
25
• 26 FA17 p 180.1
. 27 Sexually transmitted infections
• 28 DISEASE CLINICALFEATURES ORGANISM
• 29
AIDS Opportunistic infections, Kaposi sarcoma, HIV
• 30
lymphoma
• 31
. 32
Chancroid Painfu l genital ulcer with exudate, inguinal Haemophilus ducreyi (it's so painful, you "do
. 33
adenopathy cry")
• 34 Chlamydia Urethritis, cervicitis, epididymitis, Chlam)'dia trachomatis (0-K)
• 35 conjunctivitis, reactive arthritis, PI 0
• 36
Condylomata Genital warts, koilocytes HPV-6 and -II
. 37
. • acuminata •

8
Lock
s
Suspend
0
End Block
Item : 26 o f 60 - ,• Ma rk -<J [:::> "'I ~ · ~
QIO: 3285 ~ P~v•ous N@xt Labl lues Notes Calcu lator

12
An elderly New York businessman presents t o t he clinic because of several weeks of breathing difficulty, product ive cough, and joint pain.
13 When asked about recent travel, he stat es t hat he works as a consultant for numerous firms across the nation, which requires him to live
14 in different reg ions of the country for 1- 2 months at a time. The physician orders an X-ray of the chest, which reveals infiltrates in the
lungs with evidence of granulomas. A PPD t est is read as negative a few days later. A lung biopsy of the lesion reveals a fungus with spherules
15
containing endospores. Spherules with endospores are also seen from culture at 37°C on special media, while branched hyphae with
16 arthroconidia are seen in cultures at 25°C.
17
18 What is the most likely cause of his symptoms?
19 :
20 A. Aspergillus fumigatus
21 B. Blastomyces dermatitidis
22
C. Candida albicans
23
24 D. Coccidioides immitis
25 E. Histoplasma capsulatum
• 26
. 27
• 28
• 29
• 30
• 31
. 32
. 33
• 34
• 35
• 36
. 37
.
8
Lode.
s
Sus pe-nd
8
End Bloc:k
Item: 26 of 60 ~ ,• Mark <::J [:::> ""I ~· ~'j
QID: 3285 J.. Previous Next LAb faiUI~S Notes Calculator

12
13 The correct answer is 0. 55°/o chose this.
14 Coccidioides is a fungus charact erized by endospores containing spherules when cult ured at 37° C and branched hyphae when t he organism is
cult ured at 25° C. Coccidioides infection is most common ly seen in immunocompromised patients or t he elderly and is most prevalent in t he
15
Sout hwest United States. Alt hough normal hosts are susceptible to infection, t hey usually do not present wit h sympt oms. This patient's
16 hist ory of t ravel likely includes t he Sout hwest region and makes C. immitis the most likely candidate. Also consider Coccidioides in pat ients
17 who have t raveled t o Californ ia and have possibly been exposed t o dust or sand out West.
Immunodeficiency Coccidioides Southwestern United States Hypha Endospore Coccidioides immitis Fungus United States Infection
18
19 A is not correct . 17% chos e this.
20 Aspergillus infect ion is also seen almost exclusively in t he immunocom promised patient; exceptions include allergic bronchopulmonary
aspergillosis, an allergic disease that occurs in ot herwise healthy individuals. Aspergillus most often causes feve r, hemoptysis, and pneumonia.
21
A t issue biopsy will show branching hyphae with septae, but a sputum cult ure will show radiat ing chains of spores.
22 Hemoptysis Allergic bronchopulmonary aspergillosis Sputum culture Aspergillosis Sputum Immunodeficiency Pneumonia Aspergillus Biopsy Spore Hypha Septum Fever Infection Allergy

23
B is not correct. 8% chose this.
24
Blastomyces grows as thick-wa lled budding yeast s at 37°C and hyphae with sma ll con idia at 25° C. It is mostly seen along the Mississippi
25 River and in Cent ral Ameri ca . It also causes a pneumonia t hat can progress to disseminated granulomat ous disease .
Conidium Mississippi River Pneumonia Central America Blastomyces dermatitidis Granuloma Hypha Yeast Mississippi
26
. 27 C is not correct. 7 °/o chose this .
• 28 Candida is t he cause of t hrush and is seen most often in t he immunocompromised pat ient , such as t hose with HIV infection. I n a t issue biopsy
• 29 it will be seen as pseudohyphae and budding yeast. Candida ! infect ion, however, is more likely t o affect t he esophagus than t he airways and
lungs .
• 30
Immunodeficiency Esophagus HIV Hypha Yeast Biopsy Saccharomyces cerevisiae Candida (fungus) Pseudohyphae Lung Candida albicans Thrush (bird) Infection
• 31
E is not correct. 13°/o chose this .
. 32
Histoplasma grows as branched hyphae at 25°C and as full yeast cells at 37° C t hat are 2- 5 11m in diameter and live wit hin macrophages .
. 33
Histoplasma infection causes both a pulmonary disease sim ilar to a pneumonia and a severe granulomat ous disease t hroughout t he body,
• 34 especially in t he adrenals, liver, and spleen. I t is preva lent in the Mississippi and Ohio River valleys .
Pneumonia Ohio River Spleen Yeast Macrophage Histoplasma Hypha Granuloma Liver
• 35
• 36
. 37 Bottom Line:
.
8
Lock
s
Suspend
0
End Block
Item: 26 of 60 ~ ,• Mark <::J [:::> ""I ~· ~'j
QID: 3285 J.. Previous Next LAbfaiUI~S Notes Calculator

12
Bottom Line:
13
Coccidioides immitis is a dimorph ic f ungus, which grows in desert sand as filaments t hat develop arthrocon idia t hat are re leased and blow in
14
dust and may be inhaled . I n t he human body t hey develop int o spherules with interi or endospores. C. immitis infection can cause
15 granulomas in t he lu ngs and does not cause a posit ive PPD test.
Coccidioides immitis Mantoux test Granuloma Arthroconidium Endospore Coccidioides Fungus Dimorphic fungus Lung
16
17
18
19 l@ljl'il·1i•J f or yea r:[ 2017 • J
FIRS T AID FACTS

20
21 FA17 p 147.1
22
Systemic mycoses All of the following can cause pneumonia and can disseminate.
23
All are caused by dimorphic fungi: cold (20°C) = mold; heat (37°C) = yeast. Only exception is
24
25
Coccidioides, which is a spherule (not yeast) in tissue.
26
Systemic mycoses can form granulomas (like TB); cannot be transmitted person-to-person (unlike
. 27 TB).
• 28 Treatment: Auconazole or itraconazole for local infection; amphotericin B for systemic infection .
• 29
DISEASE ENDEMIC LOCATION PATHOLOGICFEATURES UNIQUE SIGNS/SYMPTOMS NOTES
• 30
Hist oplasmosis Mississippi and Ohio Macrophage fill ed Palatal/tongue ulcers, fl isto h ides (with in
• 31
River Valleys with f-listoplasma splenomegaly macrophages)
. 32
. 33
(smaller than Bird (eg, starlings) or
• 34
RBC) rl bat droppings
• 35 Diagnosis via urine/
• 36 serum antigen
. 37
.
8
Lock
s
Suspend
0
End Block
Item: 26 of 60 - ,• Mark --<] [::> ""'I ~· 1!';:'1
QIO: 328 5 ~ Prev1o u s Next Labf a lu es Notes Calculator

12 Blastomycosis Eastern and Broad-based budd ing lnAammatory Blasto buds broadly
13 Central US of BlcJstomyces (same lung disease, can
14 size as RBC) disseminate to skin/
15 bone
16 Verrucous skin lesions
17
can simulate sec
18
Forms granulomatous
19
nodules
20
21 Coccidioidomycosis Southwestern US, Spherule (much larger Disseminates to skin/
22 Californ ia than RBC) fill ed bone
23 with endospores of Er) thema nodosum
24 Coccidioides ~ (desert bumps) or
25 multi forme
26 rt hra lgias (desert
. 27 rheumatism)
• 28 Can cause meningitis
• 29
• 30
Para- Latin America Budding yeast of Similar to Paracoccidio parasails
coccidioidomycosis Pare~coccidioicles
with Coccid ioidomycosis, with the captain\
• 31
. 32
"captain\ '' heel" males> females wheel all the way to
. 33
formation (much Latin America
• 34 larger than RBC) [!]
• 35
• 36
. 37
. •
8
L.odt
s
Su~pl'nd
~
End Block
Item: 26 of 60 - ,• Mark --<] [::> ""'I ~· 1!';:'1
QIO: 328 5 ~ Prev1o u s Next Labf a lu es Notes Calculator

12

FA17 p 149.1
13
14 Opportunistic fungal infections
15 Candida albicans cdba = white. Dimorphic; forms pseudoh) phac and budding yeasts at 20°C · , germ tubes at
16 37oc m .
17 Systemic or superficial fun gal infection. Causes oral and esophageal thrush in
18 immunocompromised (neonates, steroids, diabetes, AIDS), vulvovaginitis (diabetes, usc of
19 antibiotics), diaper rash, endocarditis (I drug users), disseminated candidiasis (cspeciallr in
20 neutropenic patients), chronic mucocutaneous candidiasis.
21
Treatment: oral Auconazole/topical a7olc for \'aginal; nystatin, Auconazole, or caspofungin for oral/
22
esophageal; Auconazole, caspofungin, or amphotericin B for systemic.
23
24 Aspergillus Septate hyphae that branch at 45° \ cute \ nglc . Produces conidia in radiating chains at end of
25 fumigatus conidiophore O.
26 Causes invasive aspergillosis in immunocompromiscd, patients with chronic granulomatous disease.
. 27 Can cause aspergillomas in pre-existing lung cavities, especially after T B infection .
• 28 Some species of Aspergillus produce AOatoxins (associated with hepatocellular carcinoma).
• 29
Allergic bronchopulmonary aspergillosis (ABPA): hypersensitivity response associated with
• 30
• 31
asthma and cystic fibrosis; may c<tuse bronchiectasis and eosinophilia.
. 32 Cryptococcus 5-lO J.lm with narrow buckling. llea"ily encapsulated yeast. lot dimorphic.
. 33 neoformans Found in soil, pigeon droppings. Acquired through inhalation with hematogenous dissemination
• 34 to meninges. Culture on Sabouraud agar. llighlighted with India ink (clear halo 0 ) and
• 35 mucicarmine (red inner capsule tal. Latex agglutination test detects polysaccharide capsular
• 36 antigen and is more specific.
. 37 Causes cry ptococcosis, cryptococcalmeningitis, cryptococcal encephalitis ("soap bubble" lesions
. •
8
L.odt
s
Su~pl'nd
~
End Block
Item: 26 of 60 - ,• Mark --<] [::> ""'I ~· 1!';:'1
QIO: 328 5 ~ Prev1o u s Next Labf a lu es Notes Calculator
.
12
Mucor and Rh izopus Irregular, broad, nonseplale hyphae branching al "ide angles
13
spp. Mucormycosis. Causes disease mostly in keloacidolic diabetic and/or neutropenic patients (eg,
14

15
leukemia). Fungi proliferate in blood vessel walls, penetrate cribriform plate, and enter bra in.
16
Rhinocerebral, frontal lobe abscess; cavernous sinus thrombosis. Headache, facial pain, black
17
necrotic eschar on fa ce; ma\' he~ve cmnial nerve involvement.
'
18 Treatment: surgical debridement, amphotericin B.
19
20
21
22
23
24
25
26
. 27
• 28 D
• 29
• 30
• 31

. 32
. 33
• 34
• 35
• 36 a
. 37

.
8
L.odt
s
Su~pl'nd
~
End Block
Item: 27 of 60 - ,• Mark -<J [:::> "'I ~ · ~
QIO: 3900 ~ P~v•ous N@xt Labl lues No tes Calcula t o r

12
A 32-year-old homeless man from Florida presents t o th e emergency department complaining of a 3-month history of severe, progressive ~~AI
13 upper back pain. He is HIV-positive but has ref used antiret rov iral treatment in the past. Examination of his thoracic spine revea ls mild
14 kyphosis. CT of the spine reveals lytic destruction and partial collapse of the anterior portions of T7 and T8, wit h surrounding abscess
formation. CT-guided biopsy of the bone shows areas of caseating necrosis on pathology. A purified protein derivative test is negative and his
15
CD4 count is 90 ceiiS/IJL
16
17 Which of the following is the most appropriate treatment for this man's condition?
18
:
19 A. Combination chemotherapy with vincristine, doxorubicin, and dexamethasone
20
B. Combination therapy with isoniazid, rifampin, and pyrazinamide
21
22 C. Intravenous amphotericin
23 D . Intravenous gentamicin
24
E. Oral penicillin
25
26
. 27
• 28
• 29
• 30
• 31
. 32
. 33
0 34
0 35
0
36
. 37
.
8
Lode.
s
Suspe-nd
8
End Bloc:k
Item: 27 of 60 - ,• Mark -<J [:::> "'I ~ · ~
QIO: 3900 ~ P~v•ous N@xt Labl lues No tes Calcula t o r

A A
12
13
14 The correct answer is B. 70% chose this.
15 The presence of caseation on CT suggests that the patient is suffering from extrapulmonary tuberculosis of
the spine, also known as Pott disease. The classic clinical presentation includes spinal pain, kyphosis (due
16
to compromised vertebral integrity, shown in t he image), and neurologic signs (ranging from radicular pain
17 to cord compression and paralysis). The most common sites for Pott disease are the lower thoracic and
18 upper lumbar spines. Like pulmonary tuberculosis, it is treated with a combination of antimycobacterial
therapy, often composed of isoniazid, rifampin, and pyrazinamide. It is important to note that individuals
19
with Pott disease may or may not show symptoms of pulmonary tuberculosis at the time they present with
20 back pain. Also, a negative purified protein derivative test is insufficient to rule out tuberculosis (pulmonary
21 or extrapulmonary) in a patient with advanced HIV infection. This is because the suppression of CD4+ cells
by HIV infection can result in anergy toward the tuberculin antigen.
22
l<.l'J)hO!IS Isonraz• Pott drsewe Tuberculosrs Rifampicin Pyrazinamide HIV Caseous necro~L ,.uberculln Mantoux test lung Pulmonary tuberculosrs Protein
23 Infectoon Anbgen T cell
24
25
26
27
• 28
• 29
• 30 Image copyright ©
Andrianarimanitra HU, eta/.
• 31
. 32 A is n ot correct. 8°/o chose this .
. 33 The differential diagnosis of a patient presenting with recent-onset back pain includes multiple myeloma of t he spine. This can be treated with
0 34 combination chemotherapy comprising vincristine, doxorubicin, and dexamethasone (doxorubicin = Adriamycin). The biopsy, however, does
not revea l neoplastic plasma cells and is not consistent with a diagnosis of mult iple myeloma.
0 35
Vrncnstrne Doxorubrcrn Multiple onyeloma Dexamethasone Chemotherapy Differential diagno~is Biopsy PIMma cell Neoplwm Combrnation chemotherapy Blood plasma Back parn
0
36
. 37
. C is n ot correct. 11 °/o chose this.
8
Lode.
s
Suspe-nd
8
End Bloc:k
Item: 27 of 60 ~ ,• Mark <::J [:::> ""I ~· ~'j
QID: 3900 J.. Previous Next LAb faiUI~S Notes Calculator

12 C is not co rrect. 11% chos e this .


13 Amphot eri cin, which works by binding ergosterol and fo rming membrane pores in fungal walls, is the t reat ment of choice for a number of
14 dissem inat ed mycoses (eg, blastomycosis, crypt ococcosis, and hist oplasmosis) . Although t hese cond it ions can occasionally manifest wit h
lesions of j oints and bones, a solit ary compla int of back pain would not be a typical presentation. Addit ionally, th e patient is f rom Florid a,
15 which is outside of most of t he fungal endemic areas.
16 Cryptococcosis Ergosterol Blastomycosis Histoplasmosis Mycosis Endemism Fungus Back pain Amphotericin 8

17 D is not correct. 6°/o chose this .


18
Anot her possible explanat ion fo r t his pat ient 's presentat ion is vert ebral ost eomyelit is with fo rmat ion of a paraspinal abscess. The most
19 common organism fo und in paraspinal abscesses is Staphylococcus aureus, wh ich would be t reat ed with int ravenous vancomycin (especially if
20 it is methicillin-resist ant) . I nt ravenous gent amicin would be an inappropriat e choice because gentamicin mainly has gram -negat ive coverage.
Regardless, t he presence of caseating necrosis in t he pathology specimen is an indicator of t uberculous spondylit is ( Pot t disease) inst ead.
21 Gentamicin Osteo,.,elitis Pott disease Vanco,.,cin Staphylococcus aureus Gram-negative bacteria Abscess Staphylococcus Intravenous therapy Necrosis Spondylitis Pathology Tuberculosis
22 Vertebral osteomyelitis
23
E is not correct. 5°/o chos e this .
24
Oral penicillin works by binding penicillin-binding proteins and blocking th e cross-linking of cell walls. The drug is bactericidal fo r many gram -
25 posit ive cocci, gram -positive rods, gram- negat ive cocci, and spirochet es. Penicillin, however, is not a t reatment for tu berculosis.
26 Bacilli Coccus Penicillin Gram-negative bacteria Penicillin binding proteins Tuberculosis Gram-positive bacteria Bactericide Spiral bacteria Spirochaete Protein

27
• 28
Bottom Line :
• 29
Pott disease represent s t uberculous infection of the spine and cl assically manifests wit h back pain, kyphosis, and neurologic signs. Like
• 30
pulmonary tuberculosis, it is t reat ed with combination ant imycobacteria l t herapy : isoniazid, rifampin, and pyrazinamide .
• 31 Isoniazid Pott disease Pyrazinamide Kyphosis Rifampicin Tuberculosis Pulmonary tuberculosis Infection

. 32
. 33
• 34 l@ljl'il·1i•J for yea r:[2017 • J
FIRST AID FACTS
• 35
• 36
FA17 p 136.1
. 37
. Primarv and secondarv tuberculosis
8
Lock
s
Suspend
0
End Block
Item: 27 of 60 - ,• Mark --<) [::> ""'I ~· 1!';:'1
QIO: 3900 ~ Prev1o u s Next Labf a lu es Notes Calculator

12
FA17 p 136.1
13
14 Primary and secondary tuberculosis
15
..._,, . . MycobacteriUm PPD EEl if current infection or past exposure.
16 .r •'' t tuberculOSIS
e
PPD if no infection and in sarcoidosis or
17
II IV infection (especial!) "ith IO\\ C D4+ cell
18 Ghon
complex count).
19
lnterferon-y release assay {IGRA) has fewer fa lse
20
Primaty tubtrcutosis positives from BCG \<lCcination.
21
22
>90 <l!r. Caseating granulomas rJ with central necrosis
23
Healwlg by fibrosis Progressr;e pnrnary tuberculosis (upper left) and Langhans giant cells (arrow)
WOS. malnutntlon)
are characteristic of zotuberculosis.
Ca1ofication I
24 (tuberculin @)
1 l
25
26
ReactiVation

2' tubtrtulosis
j Progressr;e
lungdi~a~

27 Fiococaseous
• 28 UVItary leston
• 29 (usually upper
lobes)
..
_. Miliary {.,. r Menlnges
• 30
,~ ~tosis
~Vertebrae
• 31
. 32 1
localized destructive disease lymph nodes
..
f .I r"' (Pott diSeaSe)

. 33
Cavtty
• 34 Caseation
Caseat100
• 35
• 36 Adrenal
gt.Jnd
. 37
..,__.loons and •
. 8
L.odt
s
Su~pl'nd
~
End Block
Item: 27 of 60 ~ ,• Mark <::J [:::> ""I ~· ~'j
QID: 3900 J.. Previous Next LAbfaiUI~S Notes Calculator

12 FA17 p 192.1
13 Antimycobacterial drugs
14
BACTERIUM PROPHYLAXIS TREATMENT
15
M tuberculosis Isoniazid Rifampin, Isoniazid, Pyrazinamide,
16
Ethambutol (RIPE for treatment)
17
18 M avium-intracel/ulare Azithromycin, rifabutin More drug resistant than M tuberculosis.
19 J\zithromrcin or clarithromycin +ethambutol.
20 Can add rifabutin or ciproAoxacin.
21
M /eprae N/A Long-term treatment with dapsone and rifamp in
22
for tuberculoid form. Add clofazimine for
23
lepromatous form.
24
25 MYCOBACTERIAL CELL Plasma
Cell wall membrane Interior of cell
26
27
• 28
• 29 mRNA mRNASYNTHESIS
(ONA-deJlendent
• 30 MYCOLIC ACID RN otyme asel

l
• 31 SYNTHESIS
. 32
Rifabutin
. 33 RNA
polymerase Rifampin
• 34 '------------"
• 35
ARABINOGAlACTAN
• 36 SYNTHESIS INTRACELLUlAR
. 37 (arabinosyt transferase) (unclear mechanism)
. •
8
Lock
s
Suspend
0
End Block
Item: 27 of 60 - ,• Mark -<J [:::> "'I ~ · ~
QIO: 3900 ~ P~v•ous N@xt Labl lu es No tes Calcula t o r

A A
12
MYCOBACTERIAL CELL Plasma
13 Cell wall membrane Interior of cell
14
15
16
mRNA
17
18
19
20
21
22
23
( lson~azld

ARAIINOGALACTAN
RNA
polymerase
RilabutJn
Rilampin
J
SYNTHESIS
24
(~rabinosyl transferase)
25
26
27
1-----1~zinamide
• 28
• 29
• 30
FA17 p 176o4
• 31
. 32
Osteomyelitis RISKFACTOR ASSOCIATED INFECTION

. 33 Assume if no other information is avai lable S aureus (most common overall)


34
0
Sexually active I eisseric1 gonorrhoeae (rare), septic arthritis more
35
0
common
0
36
. 37
Sickle cell disease Salmonella and S aureus
. n .t •· · · , ... .
8
Lode.
s
Suspe-nd
8
End Bloc:k
1--...,~azinamide )
14

15
16
17
18
FA17 p 176.4
19

20 Osteomyelitis RISK FACTOR ASSOCIATED INFECTION


21 Assume if no other in formation is a,·ailable S aureus (most common 0\·erall)
22
Sexually active I eisseria gonorrhoeae (rare), septic arthritis more
23
common
24
25 Sickle cell disease Salmonella and S aureus
26 Prosthetic joint replacement S aureus and S epidermidis
27
Vertebral involvement S aureus, J'vfycobacterium tuberculosis (Pott
• 28
disease)
• 29
• 30 Cat and clog bites Pasteurella multocida
• 31 IV drug abuse Pseudomonas, Candida, S aureus are most
. 32 common
. 33
• 34
Elevated C-reactive protein (CRP) and erythrocyte sedimentation rate common but nonspecific.
• 35
M Rl is best for detecting acute infection and detailing anatomic im·olvement fJ. Radiographs are
• 36
insensiti\·e early but can be useful in chronic osteomyelitisrn.
. 37
.
8
Lode.
s
Suspe-nd
8
End Bloc:k
Item: 28 of 60 - ,• Mark -<J [:::> "'I ~ · ~
QIO: 4480 ~ P~v•ous N@xt Labl lues No tes Calcula t o r

12 A A concerned mother brings her 9-year-old son to your clinic because he has been com plaining of a headache. On further questioning, you
13
the discover the boy just returned home from a summer camp in North Carolina 5 days ago. Physical examination reveals a fever, but no
other abnormalities are noted. A few days later~ he returns to clinic and his physical examination is notable for a rash on his hands and
14 feet, as shown in the image below. The rash t hen spreads t o involved the trunk.
15
- - --
16
17
18
19

20
21
22
23
24
25
26
27
• 28
• 29
• 30
• 31
• 32 Image cowtesyofCDC
. 33

0 34 What is the most likely diagnosis?


0 35
:
0
36 A. Chickenpox
. 37
. B. German measles

8
Lode.
s
Suspe-nd
8
End Bloc:k
Item: 28 of 60 - ,• Mark --<] [::> ""'I ~· 1!';:'1
QIO: 4480 ~ Prev1o u s Next Labf a lu es Notes Calculator

12

13
14
15
16
17
18
19
20
21
22
23
24
25
26
27
Image COUitesy of CDC
• 28
• 29
What is t he most likely diagnosis?
• 30
:
• 31
A . Chickenpox
. 32
. 33 B. German measles
• 34 C. Lyme disease
• 35
D. Measles
• 36
. 37 E. Rocky Mountain spotted fever
. •
8
L.odt
s
Su~pl'nd
~
End Block
Item: 28 of 60 ~ ,• Mark <::J [:::> ""I ~· ~'j
QID: 4480 J.. Previous Next LAbfaiUI~S Notes Calculator

12
13 The correct answer is E. 85°/o chose this.
14
Only a small number of infections, including Rocky Mount ain spot ted fever (RMSF), meningococcemia, and secondary syphilis, can cause a
pattern of rashes involving t he palms and soles. RMSF is caused by Rickettsia rickettsii and is t ra nsmitt ed by Dermacentor t icks. Three to 12
15 days after a t ick bit e, pat ients develop malaise, f rontal headache, and fever. Several days later a maculopapular rash involving t he hands and
16 feet and t hen t he t runk develops. Ot her manifestations of RMSF inclu de hepatosplenomegaly, t hrom bocytopenia, and (pot entially fata l)
17 disseminat ed intravascular coagulation.
Rocky Mountain spotted fever Disseminated intravascular coagulation Rickettsia rickettsii Thrombocytopenia Syphilis Meningococcemia Meningococcal disease Maculopapular rash Rickettsia Secondary syphilis
18
Headache Coagulation Malaise Tick Hepatosplenomegaly Rash Spotted fever Dermacentor Blood vessel Fever
19
20 A is not correct . 3°/o chose this.
21
Chickenpox, or vari cella, is characterized by maculopapules t hat evolve into vesicles over hours t o days and event ually form crusts. Typical
lesions f irst appear on t he t runk and face and rapidly spread t o involve ot her areas.
22 Chickenpox Cutaneous condition Maculopapular rash Varicella vaccine Vesicle (biology and chemistry)
23
B is not correct. 4% chose this.
24
The maculopapular rash of German measles, or rubella, usually begins on the face and then spreads down t he body. German measles is also
25 associat ed with syst emic symptoms including malaise, low-grade feve r, and arthra lgias. Pat ients may have lymphadenopat hy, particularly in
26 t he post auricular area .
Measles Rubella Maculopapular rash Lymphadenopathy Malaise Arthralgia Rash Fever
27
28 C is not correct. 3 °/o chose this .
• 29 The rash of Lyme disease, erythema migrans, is characterized by a ring of expanding eryt hema encircling the bit e site. Fever, headache,
• 30
myalgias, and arthralgias may also be present .
Lyme disease Erythema chronicum migrans Erythema Arthralgia Headache Rash Fever
• 31
. 32
D is not correct. 5°/o chose this .
Although t he maculopapular rash of measles, or rubeola, can include t he palms and soles, it t ypically begins along the hairl ine in front al and
. 33
t emporal regions, and t hen spreads down the t run k to t he lim bs. Systemic sym ptoms include prodromal feve r and malaise several days pri or
• 34 t o rash eruption. Koplik's spot s on the buccal m ucosa also typically precede the rash and are pat hognomonic for measles.
• 35 Measles Maculopapular rash Pathognomonic Prodrome Oral mucosa Malaise Mucous membrane Rash Fever Cheek

• 36
. 37
Bottom Line:
.
8
Lock
s
Suspend
0
End Block
Item: 28 of 60 ~ ,• Mark <::J [:::> ""I ~· ~'j
QID: 4480 J.. Previous Next LAbfaiUI~S Notes Calculator

12
Bottom Line:
13
RMSF begins wit h feve r, and th en causes a rash on the hands and feet that spreads t o t he t runk. It may also manifest with
14
hepatosplenomegaly, dissem inated intravascular coagulation, and thrombocytopenia.
15 Disseminated intravascular coagulation Hepatosplenomegaly Thrombocytopenia Rash Coagulation Fever
16
17
18 141;fil·1i•J toryea r:[2a17
FIRS T AID FACTS -
•J .
19
20
FA17 p 145.1
21
22 Rickettsial diseases Treatment: doxycycline (caution during pregnancy; alternative is chloramphenicol).
23 and vector-borne
24 illnesses
25 RASH COMMON
26
Rocky Mountain Rickettsia rickettsii, vector is tick. Despite its Classic triad- headache, fever, rash (vasculitis).
27
spotted fever name, disease occurs primarily in the South Palms and soles rash is seen in Coxsackievirus
28
Atlantic states, especially orth Carolina. A infection (hand, foot, and mouth disease),
• 29
Rash typically starts at wrists · and ankles and Rocky Mountain spotted fever, and 2° Syphilis
• 30
• 31
then spreads to trunk, palms, and soles. (you drive CARS using your palms and soles).
. 32 Typhus Endemic (Aeas)-R typhi. Rickettsii on the wRists, Typhus on the Trunk.
. 33 Epidemic (human body lousc)- R prowazekii .
• 34 Rash starts centrally and spreads out, sparing
• 35 palms and soles.
• 36 RASH RARE
. 37
. Ehrlichiosis Ehrlichia, vector is tick. \itonocytes with MEGA berry-
8
Lock
s
Suspend
0
End Block
Item: 28 of 60 ~ ,• Mark <::J [:::> ""I ~· ~'j
QID: 4480 J.. Previous Next LAbfaiUI~S Notes Calculator


12 Typhus Endemic (Aeas)-R typhi. Rickettsii on the wRists, Typhus on the Trunk.
13 Epidemic (human body lousc)- R prowazekii.
14 Rash starts centrally and spreads out, sparing
15 palms and soles.
16
RASHRARE
17
18
Ehrlichiosis Ehrlichia, vector is tick. \ito nocytes with MEGA berry-
19
rn
morulae (mulberry-like inclusions) in Monocytes = Ehrlichiosis
20 cytoplasm. Granulocytes = Anaplasmosis
21 Anaplasmosis Anaplasma, vector is tick. GranulOC)ies with
22 morulae 9 in cytoplasm.
23
Qfever Coxiella bumetii, no arthropod vector. Spores Q fever is Queer because it has no rash or vector
24
in haled as aerosols from cattle/sheep amniotic and its causative organism can survive outside
25
26
Auid. Presents as pneumonia. Common cause in its endospore form. ot in the Rickettsia
27
e
of culture endocarditis. genus, but closely related.
28
___;::::,;-;:;;;:-;;;:::;::;-:;:::::;
• 29
• 30
• 31
. 32
. 33
• 34
• 35
• 36
. 37
.
8
Lock
s
Suspend
0
End Block
Item: 28 of 60 - ,• Mark --<] [::> ""'I ~· 1!';:'1
QIO: 4480 ~ Prev1o u s Next Labf a lu es Notes Calculator

12
• •
FA17 p 14401
13
Zoonotic bacteria Zoonosis: infectious disease transmitted between animals and humans.
14
15 SPECIES DISEASE TRANSMISSION AND SOURCE
16 Anaplasma spp. Anaplasmosis Ixodes ticks (live on deer and mice)
17
Bartonella spp. Cat scratch disease, bacillary angiomatosis Cat scratch
18
19 Borrelia burgdorferi Lyme disease Ixodes ticks (live on deer and mice)
20 Borrelia recurrentis Relapsing fever Louse (recurrent due to \<Iriable surface
21 antigens)
22
23
Brucella spp. Brucellosis/undulant fe,oer Unpasteurized dairy
24 Campylobacter Bloody diarrhea Feces from infected pets/animals; contaminated
25 meats/foods/hands
26
Chlamydophila psittaci Psittacosis Parrots, other birds
27
28
Coxiella burnetii Q fever Aerosols of cattle/sheep am niotic Auid
• 29 Ehrlichia chaffeensis E:hrlichiosis Amblyomma (Lone Star tick)
• 30
Francisella tularensis Tul arc mia Ticks, rabbits, deer Aies
• 31
. 32 Leptospira spp . Leptospirosis Animal urine in water; recreationa l water use
033 Mycobacterium leprae Leprosy ll umans ''ith lepromatous leprosy; armadillo
• 34 (rare)
• 35
Pasteurella multocida Cellulitis, osteomvclitis

nimal bite, cats, dogs
• 36
037 Rickettsia prowazekii Epidemic typhus ! Iuman to human via human body louse
. . . - •
8
L.odt
s
Su~pl'nd
~
End Block
Item: 29 of 60 - ,• Mark -<J [:::> "'I ~ · ~
QIO: 2180 ~ P~v•ous N@xt Labl lues Notes Calculator

12
A 24-year-old woman comes t o the physician complaining of recurrent upper abdominal pain that worsens with food intake. She denies any ~~AI
13 history of vomit ing, nausea, or diarrhea. After fa iling 3 months of treatment with omeprazole for a presumed gastric ulcer, the patient
14 undergoes upper endoscopy, which shows patches of gastrit is in the antrum.
15
16
This patient is most at risk for which of t he following?
17 :

18
A. Autoimmune intolerance of gliadin
19 B. Low-grade clonal proliferation of extranodal B lymphocytes
20 C. Monoclonal plasma cell proliferation in the bone marrow
21
0 . Painful mucosal lacerations at the gastroesophageal junction
22
23 E. Replacement of squamous epithelium wit h columnar epithelium in the distal esophagus
24
25
26
27
28
• 29
• 30
• 31
. 32
. 33
0 34
0 35
0
36
. 37
.
8
Lode.
s
Suspe-nd
8
End Bloc:k
Item: 29 of 60 ~ ,• Mark <::J [:::> ""I ~· ~'j
QID: 2180 J.. Previous Next LAbfaiUI~S Notes Calculator


12
13
The correct answer is B. 53% chos e this.
This patient present s with symptoms consistent with dyspepsia secondary to gast ri t is. In t he absence of predisposing facto rs such as chro nic
14
NSAID use, Helicobacter pylori infect ion is t he most common culprit. The pathology described is MALT (mucosa -associat ed lymphoid t issue)
15 lymphoma, which is a type of ext ranoda l non-Hodgkin lymphoma. Patient s chronically infected wit h H. pylori are at an increased risk for
16 developing this malignancy, and pat ients wit h H. pylori- induced chronic gast rit is are particularl y at risk. Eradication of H. pylori is t he first step
in t reat ing localized gast ric MALT lymphoma and often results in com plet e remission. The f irst-line t reat ment fo r H. pylori infection is "t riple
17
t herapy, " which consists of a proton pum p inhibitor ( PPI), amoxicillin, and cla rit hromycin. Metron idazole can be used in place of amoxicillin in
18 penicillin-allergic patients. Quadruple therapy, which consists of a PPI , bismut h, metronidazole, and t et racycline, is less commonly prescribed,
19 but can be used in pat ients wit h clari thromycin-resist ant H. pylori.
Proton-pump inhibitor Non-Hodgkin lymphoma Nonsteroidal anti-inflammatory drug Metronidazole Clarithromycin Helicobacter pylori Indigestion Amoxicillin Tetracycline MALT lymphoma Gastritis Bismuth
20
Malignancy Lymphatic system Proton Cancer Mucosa-associated lymphoid tissue Lymphoma Pathology Therapy Helicobacter Infection
21
22 A is not correct . 6°/o chos e this.
23 This describes celiac sprue, wh ich is not associat ed with Helicobacter pylori infection or chronic gast ri t is. On endoscopic examination, earl y
hist opat holog ic changes include increased int raep it helial lym phocyt es and prolife rat ion of crypts of LieberkUhn. I n advanced stages, it will lead
24
t o villous atrophy and hypoplasia of the small intest ine.
25 Helicobacter pylori Coeliac disease Gastritis Small intestine Gastrointestinal tract Endoscopy Histopathology Intestinal villus Lymphocyte Atrophy Esophagogastroduodenoscopy Chronic gastritis Infection

26 Helicobacter Hypoplasia

27
C is not correct. 8 °/o chos e this .
28
This describes m ult iple myeloma . Patients with mult iple myeloma usually present wit h bone pain, anemia, hypercalcemia and rena l fa ilure. It
29 is not associated wit h Helicobacter pylori infection or chronic gast rit is.
• 30 Multiple myeloma Helicobacter pylori Hypercalcaemia Gastritis Anemia Bone Bone pain Infection Kidney

• 31 D is not correct. 12% chose this .


. 32 This describes Mallory-Weiss syndrome, which is often caused by severe vomit ing. This pat ient is not experi encing vomit ing, and Mallory-
. 33 Weiss syndrome is not associated wit h Helicobacter pylori infection .
Helicobacter pylori Vomiting Helicobacter
• 34
• 35 E is not correct. 21 °/o chos e this .
• 36 This describes Barret t esophagus, which can result from gast roesophageal reflux disease (GERD). GERD is caused by esophageal inflammation
(esophagitis) due to reflux of stomach cont ents and will present with symptoms of reflux such as food coming up after meals and a
. 37
retrost ernal pain or burning sensat ion. This pat ient 's history is more consistent with dyspepsia secondary to gastri t is.
. •
8
Lock
s
Suspend
0
End Block
Item: 29 of 60 ~ ,• Mark <::J [:::> ""I ~· ~'j
QID: 2180 J.. Previous Next LAbfaiUI~S Notes Calculator

12
13 Bottom Line:
14 Chronic gast rit is, especially in t he antrum, is most often caused by Helicobacter pylori infection. I nfect ion wit h H. pylori is associated wit h an
15 increased ri sk for MALT lymphoma, a low-grade type of non- Hodg kin lymphoma .
Non-Hodgkin lymphoma Helicobacter pylori MALT lymphoma Gastritis Chronic gastritis Lymphoma Pyloric antrum
16
17
18
l@ljl'il·1i•J for year:[ 2017 • J
19 FIRS T AID FACTS

20
21 FA17 p 142.1
22 Helicobacter pylori Curved, flagellated (motile), gram 8 rod fJ that is triple EB: catalase EB, oxidase EB, and urease EB
23
(can use urea breath test or feca l antigen test for diagnosis). Urease produces ammonia, creating
24
an alkaline environment, which helps H pylori survive in acidic mucosa. Colonizes mainly
25
antrum of stomach; causes gastritis and peptic ulcers (especially duodenal). Risk factor for peptic
26
ulcer disease, gastric adenocarcinoma, and MALT lymphoma.
27
Most common initial treatment is triple therapy: Amoxicill in (metronidazole if penicillin allergy)
28
29
+ Clarithromycin + Proton pump inhibitor; Antibiotics C ure Pylori.
• 30
• 31
. 32
. 33
FA17 p 362.1
• 34 Gastritis
• 35 Acute gastritis Erosions can be caused by: Especially common among alcoholics and
• 36 • SA IDs- l PCE 2 - l gastric mucosa patients taking daily NSA IDs (eg, patients with
. 37 protection rheumatoid arthritis).
.
8
Lock
s
Suspend
0
End Block
12
common
13
+ Clarithromycin + Proton pump inhibitor; Antibiotics Cure Pylori.
14

15
16
17 FA17 p 362.1
18 Gastritis
19 Acute gastritis Erosions can be caused bv: Especially common among alcoholics and
'
20 NSAIDs- l PC E2 - l gastric mucosa patients taking daily i\ SAIDs (eg, patients" ith
21 protection rheumatoid arthritis).
22
Burns (Curli ng ulcer)- h) povolemia
23
- mucosal ischemia Burned by the Curling iron.
24
Brain injury (Cushing ulcer)- t vagal
25
stimulation - t ACh - t [(+ production Alwavs Cushion the brain.
26 '
27
28 Chronic gastritis Mucosal inflammation , often leading to atrophy
29 (hypochlorhydria - hypergastrincmia) and
• 30 intestinal C-cell metaplasia (t risk of gastric
• 31 cancers).
• 32
H pylori Most common. t risk of peptic ulcer disease, Affects antrum first and spreads to body of
. 33
I LT lymphoma . stomach.
• 34
• 35 Autoimmune Autoantibodies to parietal cells and intrinsic Affects body/fundus of stomach.
• 36 factor. t risk of pernicious anemia .
. 37
.
8
Lode.
s
Suspe-nd
8
End Bloc:k
Item: 3 0 of 60 - ,• Mark -<J [:::> "'I ~ · ~
QIO: 2 179 ~ P~v•ous N@xt Labl lues No tes Calcula t o r

12 A 12-year-old girl is brought to t he pediatrician because of a fever of 39.7°C (103.5°F), t rouble swallowing, and drooling. Her mother
13 states that she rarely brings her to the doctor and t hat she doesn't believe in vaccines. Wit hin a few minutes of arrivi ng at t he office, she
14
develops inspiratory stridor and respi ra tory distress. An X-ray film of her neck is shown in the image.
15
16
17
18
19
20
21
22
23
24
25
26
27
28
29
• 30
• 31
. 32
. 33
• 34 Which of the following organisms is most likel y responsible fo r this patient's cond ition?
• 35 :
• 36 A. Gram-negative coccobacilli
. 37
B. Gram-positive coccus growing in clusters
.
8
Lode.
s
Suspe-nd
8
End Bloc:k
12
13
14

15
16
17
18
19

20
21
22
23
24
25
26
27
28
29
Which of the fol lowing organisms is most likely responsible for t his patient's condit ion?
• 30
:
• 31
A. Gram- negative coccobacilli
• 32
. 33 B. Gram- posit ive coccus growing in clust ers
• 34 C. Gram- posit ive sphere growing in chains
• 35
D. Negative-sense sing le-stra nded virus
• 36
. 37 E. Non-Gram-staining bacteria lacking a cell wall
.
8
Lode.
s
Suspe-nd
8
End Bloc:k
Item: 30 of 60 ~ ,• Mark <::J [:::> ""I ~· ~'j
QID: 2179 J.. Previous Next LAb faiUI~S Notes Calculator

12
13
14 The correct a nswer is A. 57°/o chose t h is .
High feve r, dysphagia, drooling, inspiratory strido r~ and respiratory dist ress are all consist ent wit h t he diagnosis of epiglottit is. The X- ray f ilm
15
shows t hicken ing of t he epiglottis (thumbpri nt sign) and aryepiglott ic folds. The most common causal agent associated wit h epiglot t it is is
16 Haemophilus influenzae, which is a gram- negative coccobacilli.
17 Stridor Epiglottitis Dysphagia Haemophilus influenzae Epiglottis Thumbprint sign Gram-negative bacteria Coccobacillus Dyspnea Radiography Fever X-ray

18 B is not correct. 9 % chose this .


19 Staphylococcus aureus is commonly found in t he skin and nasal f lora of healthy pat ients. It can cause nosocom ial or wound infect ions.
20 Presentations of infection are diverse but include skin involvement , pneumonia, and meningit is.
Staphylococcus aureus Meningitis Pneumonia Staphylococcus Hospital-acquired infection Flora
21
22 C is not co rrect. 10% chose this .
23 Streptococcus pyogenes (group A streptococcus) can cause pharyngit is. Pat ients wit h t his illness present wit h a sore t hroat , headache, and
abdominal pain. The patient's symptoms are not consist ent wit h t his diagnosis, nor is t here a charact eri st ic X- ray f inding for ma king this
24
diagnosis.
25 Streptococcus pyogenes Streptococcus Pharyngitis Sore throat Abdominal pain Headache X-ray

26
D is not correct. 19% c hose this.
27
Bot h respirat ory syncytial virus and parainfluenza virus are negative-sense single-st randed viruses in t he Paramyxoviridae fam ily. Respiratory
28 syncyt ial virus is a causal agent of bronchiolitis, which is characterized by gradually developing respiratory distress and paroxysmal wheezing.
29 Parainfluenza v irus is associat ed with croup. That disease can manifest with inspirat ory st ridor, seal-like barking cough, retract ions, and
coryza. The X- ray film (frontal neck) in t hat case would show a charact eristic st eeple sign, which is a narrowing of t he upper t rachea.
30
Stridor Paramyxoviridae Human respiratory syncytial virus Steeple sign Croup Rhinitis Trachea Human parainfluenza viruses Dyspnea Bronchiolitis Sense (molecular biology) Virus Cough Syncytium Wheeze
• 31
X-ray Radiography Negative-sense
. 32
E is not correct. 5°/o chose this .
. 33
Mycoplasma pneumoniae causes aty pical pneumonia t hat presents with cough, shortness of breat h, and bilatera l diff use infilt rat es on an X-ray
• 34
f ilm . This patient's symptoms and X-ray f indings are not consist ent with this disease .
• 35 Mycoplasma pneumoniae Atypical pneumonia Pneumonia Mycoplasma Cough Dyspnea X-ray

• 36
. 37
Bottom Line :
.
8
Lock
s
Suspend
0
End Block
Item: 30 of 60 ~ ,• Mark <::J [:::> ""I ~· ~'j
QID: 2179 J.. Previous Next faiUI~S
LAb Notes Calculator

12 Bottom Line:
13 A child wit h high feve r~ drooling, diff icult y breat hing, and inspirat ory distress should be evaluat ed for epiglottit is. This can be seen on X-ray
14 f ilms and is most oft en caused by Haemophilus influenzae infection, a gram -negative coccobacillus.
Epiglottitis Haemophilus influenzae Coccobacillus Gram-negative bacteria X-ray Haemophilus Dyspnea Infection Fever
15
16
17
18
141;fil·1i•J for year:[ 2017
FIRST AID FA CTS .
•j .

19
20 FA17 p 138.3
21
Haemophilus Small gram 8 (coccobacillary) rod. Aerosol Vaccine conta ins type b capsular polysaccharide
22
influenzae transmission. lontypeable (unencapsulated) (polyribosylribitol phosphate) conjugated
23
strains are the most common cause of mucosal to diphtheria toxoid or other protein. Given
24
25
infections (otitis media, conjunctivitis, between 2 and 18 months of age.
26
bronchitis) as well as invasive infections since Does not cause the Au (influenza virus does).
27 the vaccine for capsular type b was introduced.
28 Produces lgA protease. Culture on chocolate
29 agar, which contains factors V (NAD+) and X
30 (hematin) for growth; can also be grown with
• 31 S aureus, which provides factor V through the
. 32 hemolysis of RBCs. HaEMOPhilus causes
. 33 Epiglottitis (endoscopic appearance in Ill,
• 34 can be "cherry red" in children; "thumb sign"
• 35
on x-ray IE)), Meningitis, O titis media, and
• 36
Pneumonia .
. 37
Treatment: amoxicillin +1- clavulanate for
.
8
Lock
s
Suspend
0
End Block
Item: 3 0 of 60 - ,• Mark -<J [:::> "'I ~ · ~
QIO: 2 179 ~ P~v•ous N@xt Labl lues No tes Calcula t o r

12
"' . . . . .
rifampin prophylaxis for close contacts.
13
14

15 FA17 p 165.3
16
17
Paramyxoviruses Paramyxo\'iruses cause disease in children. They include those that cause parainfluenza (croup:
18
seal-like barking cough), mumps, measles, RS , and human metapneumo,·irus, which causes
19
respiratory tract infection (bronchiolitis, pneumonia) in infants. All contain surface F (fusion)
20 protein, which causes respiratory epithelial cells to fuse and form multinucleated cells.
21 Pali\'izumab (monoclonal antibod) against F protein) prevents pneumonia caused by RSV
22 infection in premature infants.
23 Pali"izumab for Paramyxovirus (RSV) Proph)'la\is in Prem ies.
24
25
FA17 p 146.3
26
27 Mycoplasma Classic cause of atypical "walking" pneumonia o cell wall. Not seen on Gram stain.
28 pneumoniae (insid ious onset, headache, nonproductive Pleomorphic t:J.
29 cough, patchy or diffuse interstitial infi ltrate). Bacterial membrane contains sterols for stability.
30 X-ray looks worse than patient. ll igh titer of ~ycoplasma l pneumonia is more common in
• 31 cold agglutinins (IgM), which can agglutinate patients < 30 years old.
• 32 or lyse RBCs. Grown on Eaton agar. Frequent· outbreaks in mil itary recruits and
. 33 Treatment: macrolides, doxycycline, or pnsons.
0 34 Auoroquinolone (penicillin ineffecti\'c since Mycoplasma gets cold \\'ithout a coat (cell wall).
0 35 Mycoplasma ha,·e no cel l wall).
0
36
. 37
.
8
Lode.
s
Suspe-nd
8
End Bloc:k
Item: 31 of 60 - ,• Mark -<J [:::> "'I ~ · ~
QIO: 2 19 1 ~ P~v•ous N@xt Labl lues No tes Calcula t o r

12
A 29-year-old man who has just ret urned from camping in Arkansas complains of fever and malaise after sustaining a t ick bite. He has a ~~AI
13 well-demarcated skin lesion with a black base on his finger. The patient recal ls that shortly after being bitten, a small itchy bump emerged
14 at the site of the lesion. He says that he has not had diarrhea. The responsible organism is observed to be a coccobacillus that is
predominantly intracellular.
15
16
17
18
19
20
21
22
23
24
25
26
27
28
29
30
• 31
. 32
. 33 Image cowtesy of CDC/ Emory University, Dr.
0 34
0 35 Which of the following organisms is most likely responsible for this patient's disease?
0
36 :
. 37 A. Borrelia burgdorieri
.
8
Lode.
s
Suspe-nd
8
End Bloc:k
19
20
21
22
23
24

25
26
27
28
Which of the fol lowing organisms is most likely responsible for t his patient's disease?
29
:
30
A. Borrelia burgdorferi
• 31
. 32 B. Brucella melitensis
. 33 C. Francisella tularensis
• 34
D. Nocardia asteroides
• 35
• 36 E. Yersinia pestis
. 37

.
8
L.odt
s
Su~pl'nd
~
End Block
Item: 31 of 60 ~ ,• Mark <::J [:::> ""I ~· ~'j
QID: 2191 J.. Previous Next LAb faiUI~S Notes Calculator

12
13
14 The correct a nswer is C. 4 6 °/o chose this .
15 One of the diseases caused by the facult at ive int racellular gram- negative rod -shaped Francisella tularensis is ulceroglandular t ularemia. This
disease is characterized by a well-demarcat ed skin lesion with a black base. The bacterium, which is carried by rabbits, can be t ransmitted to
16
human beings in a t ick or deerfly bi te. The most common t ransmission of tu laremia for ulceroglandular disease (80% of cases) is via
17 contaminated ra bbi t blood. Im p01tant t o not e, t he form of F tularensis may vary; it can be descri bed as a either coccobacillus or "rod-shaped,"
18 bot h of which may show up on the USMLE in the descri ption of t his bacterium.
Tularemia Francisella tularensis Cutaneous condition Gram-negative bacteria Coccobacillus Intracellular Tick Bacteria Bacillus (shape) Intracellular parasite Lesion Rabbit
19
20 A is not correct . 21% chose this .
21 Borrelia burgdorferi is a spirochet e t hat causes Lyme disease . The organism is t ransmitt ed t hrough the bit e of t he Ixodes t ick. Infect ion does
not result in an ulcer wit h a black base. This organism is an extracellular pat hogen.
22
Spirochaete Lyme disease Borrelia burgdorferi Tick Borrelia Pathogen Ixodes Organism Extracellular Peptic ulcer
23
24
B is not correct. 13% chose this .
Brucella species ent er t he body in cont aminat ed m ilk products or through direct cont act with contaminated livestock. Brucella melitensis is an
25
intracellular rod-shaped bacterium, but it is not t ransm it ted by the bit e of a t ick or f lea. Infect ion is not associated wit h an ulcer wit h a black
26 base.
27 Brucella Bacteria Tick Brucella melitensis Milk Intracellular Livestock

28 D is not correct. 6°/o chose this.


29 Nocardia asteroides is an acid-fast aerobe found in soil. This organism causes pulmonary infect ions, pri marily in immunocompromised
30 ind ividuals, but is not associat ed with skin lesions such as t he one described in the stem of the question. It is a branching f ilamentous
bact eriu m .
31
Nocardia asteroides Aerobic organism Nocardia Immunodeficiency Cellular respiration Acid-fast Bacteria Organism Hypha
. 32
. 33
E is not correct. 14°/o chose this .
Yersinia pestis, a gram-negat ive rod, causes t he plague. Human infection occurs when a person is bit ten by a flea t hat has previously bit ten
• 34
an infect ed rodent.
• 35 Rodent Yersinia pestis Gram-negative bacteria Plague (disease) Bubonic plague Yersinia Flea

• 36
. 37
Bottom Line :
.
8
Lock
s
Suspend
0
End Block
Item: 31 of 60 ~ ,• Mark <::J [:::> ""I ~· ~'j
QID: 2191 J.. Previous Next faiUI~S
LAb Notes Calculator

12
13 Bottom Line:
14 Francisel/a tularensis is carried by ra bbits and can be transferred to humans in t ick or deerfly bit e or through direct cont act with an infected
animal. I nfection may lead to ulceroglandu lar tularemia, characterized by a well -dema rcat ed skin lesion with a black base, along with
15
systemic symptoms.
16 Cutaneous condition Francisella tularensis Tularemia Tick Francisella Lesion

17
18
19 l@l;fil·1i•l f or yea r:[ 2017
FIRST AID FACTS .
•j .
20
21
FA17 p 144.1
22
23 Zoonotic bacteria Zoonosis: infectious disease transmitted between animals and humans.
24
SPECI ES DISEASE TRANSMISSION AND SOURCE
25
Anaplasma spp. Anaplasmosis Ixodes ticks (live on deer and mice)
26
27 Bartonella spp. Cat scratch disease, bacilla ry angiomatosis Cat scra tch
28 Borrelia burgdorferi Lyme cl iseasc Ixodes ticks (live on deer and mice)
29
Borrelia recurrentis Relapsing fever Louse (recurrent due to variable surface
30
31
antigens)
. 32 Brucella spp. Brucellosis/undulant fever Unpasteurized dairy
. 33
Campylobacter Bloody diarrhea Feces from infected pets/animals; contaminated
• 34
meats/foods/hands
• 35
• 36
Chlamydophila psittaci Psittacosis Parrots, other birds
. 37 Coxiella burnetii Q fever Aerosols of cattle/sheep amniotic Auid
.
8
Lock
s
Suspend
0
End Block
Item: 31 of 60 - ,• Mark --<] [::> ""'I ~· 1!';:'1
QIO: 2 19 1 ~ Prev1o u s Next Labf a lues Note s Calculator

12
FA17 p 211.2
13
14 Granulomatous Bacterial: Granulomas are composed of epithelioid
15 diseases J\lycobacteria (tuberculosis, leprosy) cells (macrophages with abundant pink
16 Bartonella henselae (cat scratch disease) cytoplasm) with surrounding multinucleated
17 Listeria monocrtogenes (granulomatosis giant cel ls and lymphocytes. Th 1 cells secrete
18 infantiseptica) IFN-y, acth·ating macrophages. T N F-a
19 Treponema pallidum (3° syphilis) from macrophages induces and m a i n t~1ins
20 Fungal: endemic mycoses (eg, histoplasmosis) granuloma formation. Anti-TNF drugs can, as
21 Parasitic: schistosomiasis a side effect, cause sequestering granulomas to
22 C hronic granulomatous disease break down, leading to disseminated disease.
23 AutoinAammatory: Always lest for latent TB before starting anti-
24
Sarcoidosis T ' F therapy.
25
C rohn disease Associated" ith hypercalcemia due to calcitriol
26
Primary biliary cirrhosis (I ,25-[011],- vitamin 0 .)
>
production.
27
Subacute (de Querva in/granulomatous) Caseating necrosis is more common with
28
thyroiditis an infectious etiology (eg, TB). Diagnosis
29
Granulomatosis with pol)rangiitis (Wegener) of sarcoidosis requires noncascating
30
31
Eosinophilic granulomatosis with m
granulomas on biopsy.
. 32
polyangiitis (Churg-Strauss)
. 33 Giant cell (temporal) arteritis
0 34 Takayasu arteritis
0 35 F'oreign material: berylliosis, talcosis,
0
36 hypersensitivity pneumonitis
. 37

.
8
L.odt
s
Su~pl'nd
~
End Block
Item: 32 of 60 - ,• Mark -<J [:::> "'I ~ · ~
QIO: 4486 ~ P~v•ous N@xt Labl lues No tes Calcula t o r

12 ~~AI
A 35-year-old white man presents t o t he emergency depart ment 15 days after retu rn ing from a boating t ri p t o t he boundary wa ters of
13 Minnesota. He reports a 4-day hist ory of headache; fat igue; low back pain; myalgia, especially of the calves; vomit ing; and a fever as high
14 as 40.6°C ( 105.1 ° F). On physical examinat ion, the man has erythema of the left flank, with scattered t iny red macules on the legs.
Laboratory studies show proteinuria and a creat inine level of 4.0 mg/dL. Results of cerebrospinal fluid examinat ion are normal. A monospot test
15 is negative.
16
17 What is the most likely diagnosis?
18
:
19 A. Hepatitis A
20
B. Infectious mononucleosis
21
22 C. Leptospirosis
23 D. Lyme disease
24
E. Naegleria meningitis
25
26
27
28
29
30
31
. 32
. 33
0 34
0 35
0
36
. 37

s
0

8 S uspe-nd
8
End Bloc:k
Lode.
Item: 32 of 60 ~ ,• Mark <::J [:::> ""I ~· ~'j
QID: 4486 J.. Previous Next LAbfaiUI~S Notes Calculator


12 The correct a nswer is C. 66°/o chose this .
13 Leptospirosis is common after recreat ional freshwater exposures. Salient clinical featu res include high feve r~ ri gors, myalgias,
14
t hrombocyt openia, and headache. Pat ients commonly have characteristic red eyes (up t o 55% of t hose infected) from conj unctival suffusion
with subconj unctival hemorrhage. While not diagnostic, it is uncommon in ot her infect ious diseases t hat present similarly and can help
15 different iat e leptospirosis. Lept ospirosis may last fo r a few days to 3 weeks; wit hout t reat ment , it may take pat ients mont hs to recover. The
16 combination of leptospirosis wit h rena l failure and hepat it is is known as Wei I disease . other complications include pulmonary hemorrhage,
17
acute respiratory dist ress syndrome (ARDS), myocardit is, rhabdomyolysis, and uveit is.
Conjunctival suffusion Leptospirosis Acute respiratory distress syndrome Subconjunctival hemorrhage Myocarditis Uveitis Thrombocytopenia Pulmonary hemorrhage Hepatitis Headache Bleeding
18
Conjunctiva Infection Dyspnea Fever
19
20
A is not correct . 4 °/o chose this.
Given t he hist ory of freshwat er exposure, hepatitis A should be considered . However, hepatit is A does not usually manifest with renal fa ilure :
21
in a patient wit h suspected hepat it is A, look for diarrhea, nausea, vomiting, dark urine, itching, or jaundice.
22 Jaundice Hepatitis A Diarrhea Hepatitis Itch Nausea Vomiting Urine Fresh water

23
B is not correct. 3% chose t his .
24
Because t he monospot t est was negat ive, infect ious mononucleosis can be ruled out. Furt hermore, infectious mononucleosis t ends to be
25 associat ed with malaise, fat igue, sore t hroat , and splenomegaly, symptoms that do not fit t his pat ient's presentat ion.
26 Infectious mononucleosis Splenomegaly Heterophile antibody test Malaise Sore throat Fatigue (medical)

27 D is not correct. 1 2% c hose this.


28 Lyme disease usually begins wit h a t ick bite and present s with a classic circular rash. I t is not associated wit h nausea or vomit ing, though it
29 can cause f lu-like symptoms.
Lyme disease Tick Nausea Influenza-like illness Vomiting Rash Tick-borne disease
30
31 E is not correct. 1 5°/o chose this.
32 Because t he cerebrospinal fluid examinat ion was normal, Naegleria mening it is can be ru led out. This disease is rapidly fat al, and t ends to be
associat ed with severe headaches and fever t hat progress t o stiff neck, seizures, alt ered ment al status, and coma .
. 33
Cerebrospinal fluid Meningitis Epileptic seizure Coma Altered level of consciousness Fever Mental status examination Meningism Neck stiffness
• 34
• 35
• 36
Bottom Line :
. 37 Leptospirosis commonly occurs after f reshwat er ingest ion and causes high fever, re nal impairment , and mild hepat it is .
. Leptospirosis Hepatitis Fever Kidney

8
Lock
s
Suspend
0
End Block
Item: 32 of 60 ~ ,• Mark <::J [:::> ""I ~· ~'j
QID: 4486 J.. Previous Next LAb faiUI~S Notes Calculator
• I I I

12
13
D is not correct. 12% chose this .
14
Lyme disease usually begins wit h a t ick bite and present s with a classic circular rash. I t is not associated wit h nausea or vomit ing, though it
can cause f lu-like symptoms.
15 Lyme disease Tick Nausea Influenza-like illness Vomiting Rash Tick-borne disease

16
E is not correct. 15°/o chose this.
17
Because t he cerebrospinal fluid examinat ion was normal, Naegleria mening it is can be ru led out. This disease is rapidly fat al, and t ends to be
18 associat ed with severe headaches and fever t hat progress t o stiff neck, seizures, alt ered ment al status, and coma.
19 Cerebrospinal fluid Meningitis Epileptic seizure Coma Altered level of consciousness Fever Mental status examination Meningism Neck stiffness

20
21 Bottom Line:
22
Leptospirosis commonly occurs after f reshwat er ingest ion and causes high fever, re nal impairme nt , and m ild hepat it is.
23 Leptospirosis Hepatitis Fever Kidney

24
25
26 l@);fil ~1hl for year:l 2o17 y
FIRST AID FACTS
27
28
FA17 p 142.3
29
30 Leptospira interrogans Spirochete with hook-shaped ends found in water contaminated with animal urine.
31 Leptospirosis -Au-li ke symptoms, myalgias (cla ssic<~ lly of c<llves), jaund ice, photophobia with
32 conjunctival suffusion (erythema \\·ithout exudate). Pre,·alent among surfers and in tropics (eg,
. 33 Hawaii).
• 34
Weil disease (icterohemorrhagic leptospirosis)- severe form with jaundice and azotemia from liver
• 35
• 36
and kidney dysfunction, fever, hemorrhage, and anemia .
. 37
. •
8
Lock
s
Suspend
0
End Block
Item: 33 of 60 - ,• Mark -<J [:::> "'I ~ · ~
QIO: 3544 ~ P~v•ous N@xt Labl lues No tes Calcula t o r

12
A 33-year-old HIV- posit ive man is taken t o t he emergency depart ment by his room mate. Initially t he pat ient complained of generalized ~~AI
13 weakness and vision difficulties, which started severa l weeks ago. Later, the patient lost his peripheral vision and became unable to talk or
14 walk. Lumbar puncture reveals unremarkable cerebrospinal flu id. MRI of the brain reveals multiple, non-cont rast-enhancing lesions in the
white matter, primarily in the parieta l and occipital lobes. His CD4 count is 35/mml.
15
16
Which of the following is the most likely diagnosis?
17
:
18
A. Acute disseminated encephalomyelitis
19
20
B. Cryptococcal meningitis
21 C. Guillaine-Barre syndrome
22
D. Herpes simplex meningitis
23
24
E. Progressive multifocal leukoencephalopathy
25
26
27
28
29
30
31
32
. 33
0 34
0 35
0
36
. 37
.
8
Lode.
s
Suspe-nd
8
End Bloc:k
Item: 33 of 60 ~ ,• Mark <::J [:::> ""I ~· ~'j
QID: 3544 J.. Previous Next LAb faiUI~S Notes Calculator

12
13 The correct a nswer is E. 67°/o chose this .
14 This patient most likely now has AIDS and progressive mult ifocal leukoencephalopathy (PML), an opportunist ic infection t hat affects t he
cent ra l nervous syst em (CNS) . PML is t he result of t he reactivation of lat ent JC papovavirus, usually after the pat ient's CD4 count fa lls to less
15
t han 200/mm 3 . Approximately 75% of all humans have been exposed to t he JC virus. PML is a fa tal CNS disease t hat causes demyelination of
16 t he whit e mat ter. Disease progression is su bacute, and it is init ially marked by visual field deficits, mental st atus changes, and weakness. The
17 disease progresses t o blindness, dement ia, coma, and death, t ypically within 6 mont hs. Cerebrospinal f luid analysis is usually unremarka ble,
alt hough t he fact t hat PML does not enhance on MRI with contrast is a key feat ure.
18
Opportunistic infection Progressive multifocalleukoencephalopathy JC virus Cerebrospinal fluid Central nervous system Demyelinating disease CD4 White matter Papovavirus HIV/AIDS Dementia
19
Leukoencephalopathy Magnetic resonance imaging Nervous system Virus Coma Infection
20
21
A is not correct . 8°/o chose t his.
Acute disseminated ( post infect ious) encephalomyelit is (ADEM) is a multifocal periventricular inf lammation and demyelinat ion after infect ion
22
(eg, chickenpox, measles) or certain vaccinations (eg, ra bies, smallpox) . ADEM produces m ult iple inf lammatory lesions in t he brain and spinal
23 cord, particularl y in t he whit e matter. Hence, it presents sim ilarly to mult iple sclerosis because of its demyelinating disease process. Unlike
24 PML, it does not have an increased incidence in patients who have an HI V infect ion or AIDS.
Multiple sclerosis Chickenpox Demyelinating disease Measles Rabies Smallpox White matter HIV Encephalomyelitis HIV/AIDS Spinal cord Acute disseminated encephalomyelitis Inflammation
25
Progressive multifocal leukoencephalopathy Vaccine Brain
26
27 B is not correct. 15% chose this .
28 Cryptococcal meningit is is classically associated wit h the acut e onset of a severe headache, fever, nuchal rig id ity, a change in mental st atus,
focal neurologic signs, and high int racranial pressures wit h papilledema . The clinical presentat ion descri bed is more consist ent wit h
29
prog ressive m ult ifoca lleukoencephalopat hy t han crypt ococcal meningitis.
30 Progressive multifocal leukoencephalopathy Papilledema Meningitis Cryptococcosis Meningism Headache Neck stiffness Altered level of consciousness Fever Cryptococcus neoformans Focal neurologic signs

31
C is not correct. 3 °/o chose this .
32
Guillain-Barre syndrome (GBS) generally causes symmet ric weakness that begins in t he distal ext rem it ies and t hat ascends to affect t he
33 proximal extremit ies and t he t runk. It is associat ed with recent infect ions of t he upper respirat ory and gastrointest inal t racts. The classic
• 34 cerebrospinal fluid f inding is albuminocytologic dissociat ion, which involves increased protein wit hout increased WBCs. GBS is caused by
immune reaction against epitopes in Schwann cell surface membrane as a result of molecular mimicry. The most commonly described
• 35
precipit ant of GBS is Campylobacter jejuni. GBS is also seen in patients with HI V infect ion .
• 36 Schwann cell Campylobacter jejuni Cerebrospinal fluid Molecular mimicry HIV Protein Campylobacter Cell membrane Epitope Gastrointestinal tract Immune system Infection

. 37
D is not correct. 7°/o chose this .
.
8
Lock
s
Suspend
0
End Block
Item: 33 of 60 ~ ,• Mark <::J [:::> ""I ~· ~'j
QID: 3544 J.. Previous Next LAbfaiUI~S Notes Calculator

12 D is not correct. 7°/o chose this.


13 Herpes sim plex virus (HSV) may cause meningit is and encephalit is, and it is usually characterized by a necrotic temporal lobe lesion. Classic
symptoms include fever, headache, alt ered ment al stat us, olfactory hallucinat ions, seizures, and vomit ing . Cerebrospinal f luid analysis
14
t ypically revea ls an increased number of lymphocytes and an elevated prot ein level. The clinical course is usually acute or subacut e.
15 Cerebrospinal fluid Meningitis Herpes simplex virus Temporal lobe Encephalitis Herpes simplex Headache Lesion Virus Protein Vomiting Altered level of consciousness Epileptic seizure Lymphocyte Olfaction

16 Necrosis Hallucination Fever

17
18
Bottom Line :
19
I n severely immunocompromised pat ients wit h su bacute altered mental st atu s, neurologic def icit , v isua l and mot or changes, and ataxia,
20
consider PM L as one of t he diagnoses.
21 Immunodeficiency Ataxia Altered level of consciousness Neurology

22
23
24 i@l;fil·1i•J for yea r:[2017 • J
FIRST AID FACTS
25
26
FA1 7 p 160.1
27
DNA viruses
28
VIRAL FAMILY ENVELOPE DNA STRUCTURE MEDICAL IMPORTANCE
29
30
Herpesviruses Yes OS and linear See Herpesviruses entry
31 Poxvirus Yes OS and linear Smallpox eradicated world wide by use of the live-
32 (largest 0 A virus) attenuated vaccine
33 Cowpox ("mi lkmaid blisters")
• 34 Molluscum contagiosum - Aesh-colored papule with
• 35
central umbilication
• 36
. 37
Hepadnavirus Yes Partially OS and circular HBV:
. • Ac ul·e or chronic heoatitis
8
Lock
s
Suspend
0
End Block
Item: 33 of 60 - ,• Mark --<] [::> ""'I ~· 1!';:'1
QIO: 3 544 ~ Prev1o u s Next Labf a lu es Notes Calculator

12
• •

13 FA17 p 160.1
14 DNA viruses
15 VIRAL FAMILY ENVELOPE DNA STRUClURE MEDICAL IMPORTANCE
16 Herpes viruses Yes OS and linear See Hcrpesviruses entry
17
Poxvirus Yes OS and linear Smallpo'l. eradicated world wide by use or the li\C-
18
(largest 01'\A virus) attenuated vaccine
19
Co" pox ("milkmaid blisters")
20
Molluscum contagiosum- flesh -colored papule with
21
22
central umbilication
23 Hepadnavirus Yes Partialk OS and circular II BV:
'
24 Acute or chronic hepatitis
25 Not a retrovirus but has re,·crse transcriptase
26
Adenovirus No OS and linear Febrile pharyngitis rJ-sore throat
27
Acute hemorrhagic cystitis
28
Pneumonia
29
Conjunctivitis-"pink eye"
30
31
32
33
0 34
0 35
0
36 Papillomavirus 0 OS and circuhtr II P - warts (scrotypcs I 2, 6, II}, Cli cervical
0 0

0
37 cancer (most commonly 16, 18)
. •
8
L.odt
s
Su~pl'nd
~
End Block
Item: 33 of 60 - ,• Mark --<] [::> ""'I ~· 1!';:'1
QIO: 3 544 ~ Prev1o u s Next Labf a lu es Notes Calculator

12

FA17 p 494.1
13
Other d emyelinating and dysmyelinating diseases
14
15 Acute disseminated Multifocal in flammation and demyelination after infection or \'accination. Presents with rapidly
16 (postinfectious) progressi,·e multifocal neurologic symptoms, altered mental status.
17 encephalomyelitis
18
Charcot-Marie-Tooth Also kno'm as hereditary motor and sensory neuropathy ( H~ lSN). Group of progressive hereditary
19
disease nen·e disorders related to the defecti\'e production of proteins invoked in the structure and
20
function of peripheral nen·es or the myelin sheath. Typically autosomal dominant inheritance
21
pattern and associated with foot deformities (eg, pes can1s, hammer toe}, lower extremity
22
weakness (eg, foot drop) and sensory deficits.
23
24 Krabbe disease Autosomal recessive lysosomal storage disease due to deficiency of galactocerebrosidase. Buildup
25 of galactocerebroside and psychosine destroys myelin sheath. Findings: peripheral neuropathy,
26 developmental delay, optic atrophy, globoicl cells.
27
Metachromatic Autosomal recessi,·e lysosomal storage disease, most commonly due to arylsulfatase A deficiency.
28
leukodystrophy Buildup of sulfatides -- impaired production and destruction of myelin sheath. Findings: central
29
and peripheral demyelination with ataxia, dementia.
30
31 Progressive multifocal Dem~'el i nationof C S fJ clue to destruction of oligodencl rocytes. Seen in 2-4% of AIDS patients
32 leukoencephalopathy (reactivation of latent JC virus infection). Rapidly progressive, usually fatal. f risk associated with
33 natalizumab, rituximab.
0 34
0 35
0
36
. 37
. •
8
L.odt
s
Su~pl'nd
~
End Block
Item: 33 of 60 ~ ,• Mark <::J [:::> ""I ~· ~'j
QID: 3544 J.. Previous Next LAb faiUI~S Notes Calculator

12 FA17 p 173.1
13
Common d iseases of As CD4+ cell count ~. risks of reactivation of past infections (eg, TB, HSV, shingles), dissemination
14
HIV-posit ive adults of bacterial infections and funga l infections (eg, coccidioidomycosis), and non-Hodgkin
15
lymphomas t.
16
17 PATHOGEN PRESENTATION FINDINGS
18 CD4+ cell count < 500/mm 3
19
Candidaalbicans Oral thrush Scrapable white plaque, pseudohyphae on
20
m1croscopy
21
22
EBV Oral hairy leukoplakia Unscrapable white plaque on lateral tongue
23 Bartonella henselae Bacillary angiomatosis Biopsy with neutrophilic inAammalion
24 HHV-8 Kaposi sarcoma Biopsy with lymphocytic inflammation
25
HPV Squamous cell carcinoma, commonly of anus
26
(men who have sex with men) or cerYix
27
28
(women)
29 CD4+ cell count < 200/mm 3
30 Histoplasma Fever, weight loss, fatigue, cough, dyspnea, Oval yeast cells within macrophages
31 capsulatum nausea, vom iting, diarrhea
32
HIV Dementia
33
• 34 JC virus (reactivation) Progressive mu ltifocal leukoencephalopathy onenhancing areas of demyelination on IRl
• 35 Pneumocystis jirovecii Pneumocrstis pneumonia "Ground-glass" opacities on CXR
• 36
CD4+ cell count< 100/ mm 3
. 37
.
8
Lock
s
Suspend
0
End Block
Item: 33 of 60 - ,• Mark --<] [::> ""'I ~· 1!';:'1
QIO: 3 544 ~ Prev1o u s Next Labf a lu es Notes Calculator

12 HIV Dementia
13 JC virus (reactivation) Progressive mu ltifocalleukoencephalopathy onenhancing areas of demyelination on IRI
14
Pneumocystis jirovecii Pneumocystis pneumonia "Ground-glass" opacities on CXR
15
16 CD4+ cell count < 100/ mm3
17 Aspergillus fumigatus Hemoptysis, pleuritic pain Ca\ itation or infiltrates on chest imaging
18
Candida albicans Esophagitis White plaques on endoscopy; yeast and
19
pseudoh) phae on biopsy
20
21
CMV Retinitis, esophagitis, colitis, pneumonitis, Linear ulcers on endoscopy, cotton-wool spots
22
encephalitis on fundoscopy
23 Biopsy re\·eals cells with intranuclear {owl eye)
24 inclusion bodies
25 Cryptococcus Meningitis Encapsulated yeast on India ink stain or
26 neoformans capsular antigen <±>
27
Cryptosporidium sppo Chronic, \\·atery diarrhea Acid-fast oocysts in stool
28
29
EBV B-celllymphoma {eg, non-Hodgkin lymphoma, C S lymphoma- ring enhancing, may be
30 C S lymphoma) solitary (vs Toxoplasma)
31 Mycobacterium onspecific systemic symptoms (fever, night
32 avium- intracellulare, sweats, weight loss) or foca l lr mphadcnitis
33 Mycobacterium avium
0 34 complex
0 35
Toxoplasma gondii Brain abscesses Multiple ring-enhancing lesions on ~IRI
0
36
. 37

.
8
L.odt
s
Su~pl'nd
~
End Block
Item: 34 of 60 - ,• Mark -<J [:::> "'I ~ · ~
QIO: 2035 ~ P~v•ous N@xt Labl lues No tes Calcula t o r

12 ~~AI
A 70 year-old man with type 2 diabet es mellit us and previous infection with Mycobacterium tuberculosis present ed t o his fam ily physician
13 complaining of several weeks of cough, night sweats, and a 6.8-kg (15- lb) weight loss. An X-ray of t he chest is shown in the image.
14 Physical examination revea ls an enlarged spleen and liver.
15
16
17
18
19
20
21
22
23
24
25
26
27
28
29
30
31
32
33
' 34
' 35 Tissue biopsy of this patient's liver is most likely to show which of the following?
' 36 :
. 37 A. Bridging fibrous septae
.
8
Lode.
s
S uspe-nd
8
End Bloc:k
12
13
14

15
16
17
18
19
20
21
22
23
24
25
26
27
28
29
Tissue biopsy of this patient 's liver is most likely to show which of the fo llowing?
30
:
31
A . Bridging fibrous septae
32
33 B. Centrolobular hemorrhagic necrosis
• 34 C. Clear macrovesicular globules
• 35
D. Diffuse nodularity
• 36
. 37 E. Multiple fine granulomas
. •
8
L.odt
s
Su~pl'nd
~
End Block
Item: 34 of 60 - ,• Mark --<) [::> ""'I ~· 1!';:'1
QIO: 2035 ~ Prev1o u s Next Labf a lu es Note s Calculator

12

The correct answ er is E. 58°/o chose this.
13 This patient has miliary tuberculosis (TB), likely following reactivation of a latent focus of TB
14 f rom his prior infection. His chest x-ray clearly shows many small opacities distributed
t hroughout the lung, indicated by arrows in the image. Systemic miliary TB is due to
15
hematogenous spread of Mycobacterium tuberculosis via seeding of the pulmonary venous
16 return to the heart. Almost every organ can be infected; most commonly affected are the lungs
17 (through reseeding), liver, bone marrow, spleen, adrenal glands, and meninges. The lesions in
these organs on biopsy resemble those in t he lungs: 1- to 2-mm yellowish granulomas. Miliary
18
TB is the most deadly progression of the disease.
19 M •arv tub c..Jjos•~ ., ob1d:er •m tuberculos•s Tuberculosis Meninges Chest rac:hograph &opsy Granulo Lim .-\drenal gland Spleen
20 Mycobac:tenum Bone marrow Lung Bacteren"la X-ray Bone Infection
21
22
23
24
25
26
27
28
29
30
31
A is not correct . 9 °/o chose this.
32 Cirrhosis in end-stage liver disease is manifested by bridging fib rous septae that appear as broad scars linking t he portal tracts with one
33 another. This pattern is the resu lt of repeated attempts at regeneration followed by injury. It is therefore a result of chronic injury and repair.
C1rrhos1s L1-..er L1ver d1-sease Hepat1c portal system
34
• 35 B is not correct . 11 °/o c hose thi s .
• 36 Centrilobular necrosis of the liver parenchyma results from t he relatively lower oxygenation of the hepat ocytes surrounding the central veins
. 37 (zone III in the acinar model of hepatic anatomy) versus the hepatic arteries (zone 1). The decreased oxygenation reduces the ability of the
hepatocytes to detoxify reactive species and also makes the central hepatocytes more sensitive to changes in perfusion. Therefore •
.
8
L.odt
s
Su~pl'nd
~
End Block
Item: 34 of 60 ~ ,• Mark <::J [:::> ""I ~· ~'j
QID: 2035 J.. Previous Next LAb faiUI~S Notes Calculator

12 B is not correct. 11 % chose thi s.


13 Cent ri lobula r necrosis of the liver parenchyma results f rom t he relatively lower oxygenation of t he hepat ocytes surrounding the cent ra l veins
(zone III in the acinar model of hepat ic anatomy) versus t he hepatic arteri es (zone I). The decreased oxygenation reduces t he ability of t he
14
hepatocyt es t o detoxify react ive species and also makes t he cent ral hepat ocytes more sensit ive t o changes in perfusion. Therefore
15 cent rilobular necrosis may be caused by poor perfusion (eg, left - and right-side cardiac fa ilure, shock) or toxins (eg, acetaminophen, ca rbon
16 t et rachloride, alcohol).
Carbon tetrachloride Parenchyma Paracetamol Necrosis Liver Hepatocyte Common hepatic artery
17
18 C is not co r r ect. 7 °/o chose this .
19 Clear macrovesicular globules are characteristic of hepat ic steatosis due to chronic alcohol consumpt ion, wherein met abolism has sh ifted to
increased fatt y acid synt hesis to resto re NAD+ levels (necessary for oxidation of et hanol) . The liver is la rge, yellow, and greasy. This is
20
reversible with t he cessat ion of alcohol ingest ion.
21 Fatty acid Ethanol Fatty liver Fatty acid synthesis Steatosis Metabolism Liver Alcohol Alcoholic beverage Redox Nicotinamide adenine dinucleotide Alcoholism

22
D is n ot cor rect. 15% c h ose this.
23
Diffuse nodularity is cha racteristic of irreversible alcoholic cirrhos is. The "bridg ing fibrous septae" described in choice A have expanded to
24 more scar t issue relat ive to funct iona l hepat ic parenchyma.
Parenchyma Cirrhosis Liver Scar Alcoholism
25
26
27 Bottom Line:
28
Pat ients wit h previous Mycobacterium tuberculosis infection are at risk of developing miliary TB, cha racterized by 1- to 2- mm granulomas in
29 t he lung, liver, bone ma rrow, spleen, adrenal glands, and meninges.
Mycobacterium tuberculosis Meninges Tuberculosis Miliary tuberculosis Mycobacterium Spleen Granuloma Adrenal gland Bone marrow Liver Lung Bone
30
31
32
33
i@l;fil·1i•J f or yea r:[ 2017 • J
FIRST AID FACTS
34
• 35 FA17 p 136.1
• 36
Primary and secondary tuberculosis
. 37
. n n T"'\ r.-.. · r • . c

8
Lock
s
Suspend
0
End Block
Item: 34 of 60 - ,• Mark --<] [::> ""'I ~· 1!';:'1
QIO: 2035 ~ Prev1o u s Next Labf a lu es Note s Calculator

12
FA17 p 136.1
13
14 Primary and secondary tuberculosis
15 PPD Ef) if current infection or past exposure.
16
r Hll.v nodes PPD 8 if no infection and in sarcoidosis or

1 ;ocus ~
17
II IV infection (especially with low C D4+ cell
18
19
c:X cmn
(t&Jally ITlldl
count).
lnterferon-y release assay (IGRA) has fewer fa lse
20 lower lobes!
Prinlily tuberculosis positives from BCG \<lCcination.
21
>904 l <l~ Caseating granulomas rJ with central necrosis
22 •
Healing by fi.brosls ProgressiVe pmnary wberculosls (upper left) and Langhans giant cells (arrow)
23
24
Calofication
(tuberculin <±>l
WDS. matmMuon)
I zo
arc characteristic of tuberculosis.
l
25
26
27
1 ReactiVatiOn
z· tuberculosis
FdlC·ocaseous
I ProgresSIVe
tung disease

J
28
29
cavitary tes1011
(usuattyupper
lobes)
_..• Miliary
30
, ~ ~tosis
31
32 !
localized destructive disease lymph nodes .
33
Cavity
,,' •
34 caseation
caseation
• 35 Scar
• 36 Adrenal
gland
. 37
~ Joints and •
. 8
L.odt
s
Su~pl'nd
~
End Block
Item: 35 of 60 - ,• Mark -<J [:::> "'I ~ · ~
QIO: 4019 ~ P~v•ous N@xt Labl lues No tes Calcula t o r

12
A 46-year-old man present s with a t emperature of 38.6°C ( 101.5° F). He was fine 2 weeks ago, but started feeling poorly a few days ago ~~AI
13 after a dental examination. Physical examination reveals ten der raised lesions on the beds of his f ingers and toes, and painless,
14 erythematous lesions on his palms and soles, like those in t he images. On further questioning, the physician discovers t hat the man has a
history of rheumatic fever as a child. Blood cult ures are drawn.
15
16
17
18
19
20
21
22
23
24
25
The most likely causative organism will have which characteristics?
26
27 :
A. Gram-posit ive cocci, catalase -negat ive, 13-hemolytic, bacitracin -resistant
28
29 B. Gram-posit ive cocci, catalase -negat ive, 13-hemolytic, bacitracin -sensitive
30 C. Gram-posit ive cocci, catalase -negat ive, a- hemolytic, op tochin- res istant
31
D. Gram-posit ive cocci, catalase -positi ve
32
33 E. Gram-posit ive, weakly acid-fast rod
34
. 35
• 36
. 37
.
8
Lode.
s
Suspe-nd
8
End Bloc:k
Item: 3S of 60 ~ ,• Mark <::J [:::> ""I ~· ~'j
QID: 4019 J.. Previous Next LAb faiUI~S Notes Calculator


12 The correct a nswer is C. 52°/o chose this .
13 The pat ient is likely sufferi ng from subacute bacterial endoca rdit is, likely precipitated by his recent denta l examinat ion. He has classic
14 symptoms of Osier's nodes (tender raised lesions on finger or t oe pads-- left- hand image) and Janeway's lesions (non-tender eryt hemat ous
lesions on t he palms or soles, right- hand image). Viri dans st reptococci are a common cause of this form of endocardit is, wh ich generally
15
occurs in t he setting of dental procedures on patient s with diseased heart valves. Viridans streptococci are gram-posit ive, catalase- negative,
16 a-hemolyt ic cocci.
17 Viridans streptococci Gram-positive bacteria Endocarditis Infective endocarditis Coccus Streptococcus valvular heart disease Subacute bacterial endocarditis Heart valve Catalase Acute (medicine)

18 A is not correct . 12% chose this .


19 Streptococcus aga/actiae, or Group B streptococci, are gram-posit ive, 13-hemolytic cocci. They are bacit racin resistant . They cause pneumonia,
20 men ingit is, and sepsis, mainly in babies, and thus are unlikely to be responsible for t his pat ient's con dit ion.
Streptococcus agalactiae Sepsis Gram-positive bacteria Meningitis Bacitracin Coccus Pneumonia Streptococcus Group 8 streptococcal infection
21
22 B is not correct. 26% chose this .
23 Streptococcus pyogenes is a gram-posit ive, 13-hemolytic group A streptococcus. These organ isms are bacit racin sensit ive. This organism is
most commonly associated wit h illnesses such as pharyngit is, cellulit is, impet igo, scarlet fever, toxic shock syndrome, rheumatic feve r, and
24
acute glomerulonephrit is- but not with endoca rdit is.
25 Scarlet fever Streptococcus pyogenes Rheumatic fever Toxic shock syndrome Bacitracin Impetigo Streptococcus Gram-positive bacteria Pharyngitis Cellulitis Endocarditis Organism

26
D is not correct. 7°/o chose this.
27
Staphylococcus aureus is a common cause of acut e endoca rdit is. These organisms often are the culprit in the sett ing of previously healt hy
28 intravenous drug users. They are gram- posit ive, cat alase- posit ive cocci.
Staphylococcus aureus Endocarditis Gram-positive bacteria Coccus Intravenous therapy Staphylococcus Drug injection
29
30 E is not correct. 3°/o chose this .
31 Nocardia asteroides is a gram- posit ive, weakly acid-fast rod. It is known to cause pulmonary infection in immunocompromised individuals. It
32 is not known to cause bacterial endocardit is.
Nocardia asteroides Nocardia Gram-positive bacteria Endocarditis Immunodeficiency Acid-fast Infective endocarditis
33
34
35 Bottom Line :
• 36 Viridans streptococci are a group of gram- posit ive, catalase- negative cocci that are also alpha hemolytic. They can cause subacut e
. 37 endocardit is, generally in t he setting of previously diseased hea1t valves .
. • Viridans streptococci Endocarditis Gram-positive bacteria Coccus Streptococcus Heart valve Subacute bacterial endocarditis valvular heart disease Hemolysis Catalase

8
Lock
s
Suspend
0
End Block
Item: 35 of 60 - ,• Mark -<J [:::> "'I ~ · ~
QIO: 40l9 ~ P~v1ous N@xt Labl lues No tes Calcula t o r


12 FA17 p 132.3
13
Viridans group Gram<±>, a -hemolytic cocci. T hey arc normal Sanguinis= blood. Think, "there is lots of
14
streptococci Aora of the oropharynx that cause dental blood in the heart" {endocarditis). S sanguinis
15
caries (Streptococcus mu/ans and S mitis) makes dextrans, which bind to fibrin-platelet
16
17
and subacute bacterial endocarditis at aggregates on damaged heart \'alves.
18 damaged heart ,·alves (S sanguinis). Resistant Viridans group strep )i,·e in the mouth because
19 to optochin, diffe rentiating them from they are not afraid of-the-ch in (op-to-chin
20 S pneumoniae, which is a-hemolytic but is resistant).
21 optochin sensitive.
22
23
FA17 p 299.2
24
25 Bacterial endocarditis Fe,·er (most common symptom), new murmur, Mitral valve is most frequently involved.
26 Roth spots (round white spots on retina Tricuspid valve endocarditis is associated with
27 surrounded by hemorrhage []), Osler nodes IV drug abuse (don' t "tri" drugs). Associated
28 (tender raised lesions on Finger or toe pads rn with S aureus, Pseudomonas, and Candida.
29 due to immune complex deposition), Janeway Culture 8; most likely Coxiella bumetii,
30 lesions (small, painless, erythematous lesions 13arlollella spp., IIACEK (1 /aemophilus,
31 on palm or sole) [i, glomerulonephritis, Aggregatibacter (formerly Actinobacillus),
32 septic arterial or pulmonary emboli, splinter Cardiobacterium, Eikenella, Kingella)
33
m
hemorrhages on nail bed. 1ultiple blood • Bacteria FRO~I JANE • :
34
cultures necessa ry for diagnosis. Fe\'er
35
Acute-$ aureus (high virulence). Roth spots
• 36
Large ,·egetations on pre\'iously normal Osler nodes
. 37
. \'alves D. Rapid onset. J\ lurmur
8
Lode.
s
Suspe-nd
8
End Bloc:k
-<J [:::> "'I ~ · ~

. .
Item: 35 of 60 - ,• Mark
QIO: 4019

12
~ P~v•ous N@xt

.Labl lues

'
No tes Calcula t o r

on palm or sole) [i, glomerulonephritis,


' , '
Aggregatibacter (formerly Actinobacillus),
13
septic arterial or pulmonary emboli, splinter Cardiobacterium, Eikenella, Kingella)
14

15
m
hemorrhages on nail bed. i\ lultiple blood '1 Bacteria FR0:\1 JANE '1:
16
cultures necessarr for diagnosis. Fever
17
Acute-$ aureus (high virulence). Roth spots
18 Large ,·egetations on previously normal Osler nodes
19 ,·alves . Rapid onset. \lurmur
20 Subacute-,·iridans streptococci (low Janewar lesions
21 ,·irulence). Smaller' egetat ions on \ nemia
22 congenitallr abnormal or diseased \'ahes. l\ail-bed hemorrhage
23 Sequela of dental procedures. Gradual Emboli
24 onset.
25 S bovis (gallolyticus) is present in colon cancer,
26 S epidermidis on prosthetic valves.
27
Endocarditis may also be non bacterial
28
(marantic/thrombotic) 2° to ma lignancy,
29
hypercoagulablc state, or lupus.
30
31
32
33
34
35
• 36
. 37
.
t
8
Lode.
s
Suspe-nd
8
End Bloc:k
Item: 36 of 60 - ,• Mark -<J [:::> "'I ~ · ~
QIO: 1825 ~ P~v•ous N@xt Labl lues No tes Calcula t o r

12
A 27-year-old accountant present s to his physician 5 weeks after returning from a trip to his gra ndfat her's home in rura l India, where he ~~AI
13 lived with locals. He is jaundiced and complai ns of nausea, vomiting, fever, and abdominal pain. He denies any new sexual contacts and
14 has never used recreational drugs. He has no significant medical history. Hepatomegaly is noted on examinat ion. His laboratory results are
as fol lows:
15
16 Aspartate aminotransferase: 425 U/L
17 Alanine aminotransferase: 574 U/L
Total bilirubin: 4.6 mg/dL
18
19
What is the genomic structure of the virus mostly likely causing this patient's symptoms?
20
:
21
A . Double-stranded linear DNA
22
23 B. Double-stranded linear RNA
24 C. Double-stranded partially circular DNA
25
D. Single-stranded circular RNA
26
27
E. Single-stranded linear DNA
28 F. Single-stranded linear RNA
29
30
31
32
33
34
35
. 36
. 37
.
8
Lode.
s
Suspe-nd
8
End Bloc:k
Item: 36 of 60 ~ ,• Mark <::J [:::> ""I ~· ~'j
QID: 18 25 J.. Previous Next LAb faiUI~S Notes Calculator


12
13 The correct an swer is F. 5 1 °/o ch ose this.
14 Hepatitis E v irus (HEV) is t he most likely cause of t his pat ient's disease, though hepat it is A virus (HAV) is also a possibilit y. Both are single-
stranded linear RNA viruses t ransmitted v ia t he feca l-oral rout e t hat are endemic in developing count ries. Given the history of t ravel t o rural
15
I ndia and t he geographic distribut ion of hepat it is viruses (hepat it is E is the most common hepatit is infect ion in t he developing world), HEV is
16 most likely. The incubation period following exposure t o HEV is 3 - 8 weeks, wit h a mean of 40 days. I nfection with HAV and HEV is often
17 charact erized by jaundice and syst emic sym pt oms in adults, although asym ptomatic infect ions do occur. Laboratory f indings include elevated
aminot ransferases (ALT > AST) and increases in bilirubin . Alt hough direct bilirubin level is higher than indirect bilirubin level in t his pat ient,
18
bilirubin f ractionat ion is not useful for diagnosing HEV and HAV infection. Recovery ensues within a few mont hs, with no risk of chro nic disease
19 except for HEV in immunocompromised hosts. There is no indicated medical t reatment beyond supportive care. HAV is part of t he
20 Pico rnavi rus family, and HEV is part of the He pevirus fami ly. Hepat it is C virus (HCV) is also a single-st randed linear RNA virus of the
Flaviv irus fami ly but is likely not t he cause of t his pat ient's disease due to lack of risk fact ors fo r blood-born e exposure to HCV. The ot her
21
answer choices do not accurat ely descri be hepatit is A or E.
22 Bilirubin Jaundice Hepatitis E Hepatitis C virus Hepatitis C Fecal-oral route FICPJivirus Hepatitis A virus Hepatitis Hepatitis A Incubation period Hepatitis E virus RNA virus Picornavirus Orthohepevirus
23 Viral hepatitis Asymptomatic Virus Immunodeficiency Aspartate transaminase India RNA Transaminase Chronic condition Developing country Infection
24
A is n ot correct . 11% ch ose this.
25
Double-stranded linear DNA viruses include Herpesviruses, Adenovirus, and Poxvirus. Herpesviruses include (1) herpes simplex viruses (HSV l
26 and HSV2), which cause orolabial and genit al herpes, (2) varicella zost er virus (VZV), which causes chickenpox and shingles, (3) Epst ein -Barr
27 virus, which causes infectious mononucleosis, and (4) cytomegalovirus. Adenovirus ty pically causes a respirato ry illnesses, especially in
children. Poxviridae includes orthopoxvirus (small pox) and molluscipoxvirus (molluscum contagiosum).
28
Infectious mononucleosis varicella zoster virus Epstein-Barr virus Poxviridae Chickenpox Cytomegalovirus Molluscum contagiosum Orthopoxvirus Smallpox Adenoviridae Genital herpes Herpesviridae
29
Herpes simplex virus Herpes simplex Shingles Virus Molluscipoxvirus DNA DNA virus
30
31
B is not correct. 8% ch ose t his .
Double-stranded linear RNA viruses include Reoviruses, which encompass rotavirus and the causat ive agents of Colorado t ick fever. Rotavirus
32
is the most common cause of vomit ing and diarrhea in young children .
33 Colorado tick fever Rotavirus Diarrhea RNA virus Reoviridae Virus Tick Vomiting RNA Fever

34
C is not corr ect. 10% ch os e thi s .
35
Double-stranded partially circular DNA viruses include Hepadnavirus, t he causat ive agent of hepat it is B. Chronic infect ion wit h hepat it is B
36 virus can result in liver cirrhosis and hepat ocellular carcinoma.
. 37 Hepatocellular carcinoma Cirrhosis Hepatitis Liver Virus Carcinoma DNA DNA virus

. •
8
Lock
s
Suspend
0
End Block
Item: 36 of 60 ~ ,• Mark <::J [:::> ""I ~· ~'j
QID: 18 25 J.. Previous Next LAbfaiUI~S Notes Calculator

12 D is n ot cor rect. 11% c h ose this.


13 Single-stranded circular RNA viruses include Arenaviruses, Bunyaviruses, and Deltavirus. Although arenaviruses are ra re in humans, t hey can
14 occasionally result in aseptic meningit is. Bunyaviruses are t ransmitted by art hroprods and can cause hemorrhagic feve rs. Deltavirus includes
15 hepatit is D virus, which can co- infect wit h hepat it is B virus.
Hepatitis 0 Aseptic meningitis Hepatitis 8 Meningitis Hepatitis RNA virus Circular RNA Hepatitis 8 virus Virus Bunyaviridae RNA Coinfection
16
17 E is n ot correct. 9°/o ch ose this.
18
Single-stranded linear DNA viruses include Parvovirus. Parvovirus t ypically causes erythema infectiosum (fift h disease) in young children.
Fifth disease Erythema Virus Parvovirus DNA virus DNA
19
20
21 Bottom Line:
22 The hepat it is A and hepat it is E viruses contain a single-stranded linear RNA genome and are t ransm itt ed by t he fecal-oral route. Infect ion is
often characterized by j aundice and systemic symptoms in adults. Recovery occurs within a few mont hs in immunocompet ent hosts. Neither
23
virus persist s in a carrier state except for hepatitis E as a chronic carrier stat e in immunocompromised hosts.
24 Jaundice Hepatitis E Fecal-oral route Hepatitis A Hepatitis Immunodeficiency Virus Genome RNA Immunocompetence Infection

25
26
27 i@l;fil·1i•J f or yea r:[ 2017 • J
FIRST AID FACTS
28
29
FA17 p 168 .1
30
31 Hepatitis Signs and symptoms of all hepatitis viruses: episodes of fever, jaundice, t ALT and AST. Naked viruses (HAY
32 viruses and I lEV) lack an envelope and are not destroyed by the gut: the \"Owcls hit your bowels.
33 HBV D 'A polrmerase has D A- and RNA-dependent activities. Upon entry into nucleus, the polymerase
34 completes the partial dsD 'A. Host R1 A polymerase transcribes mRNA from \·iral D 'A to ma ke vira l
35 proteins. The DNA polymerase then re\·erse transcribes viral R A to D1 A, which is the genome of the
36 progeny v1rus.
. 37
HCV lacks 3'-5' exonuclease activity -+ no proofreading ability -+ variation in antigenic structures of HCV
.
8
Lock
s
Suspend
0
End Block
Item: 36 of 60 ~ ,• Mark <::J [:::> ""I ~· ~'j
QID: 18 25 J.. Previous Next LAb faiUI~S Notes Calculator

• •
12 Virus HAV HBV HCV HDV HEV
13 FAMILY RNA picornavirus DNA hepadnavirus RNA Aavivirus Rl A deltavirus RNA hepcvirus
14
TRANSMISSION Feca l-oral (shellfish, Parenteral (Blood), Primarily blood Parentera l, sexual, Fecal-oral,
15
16 travelers, day care) sexual (Baby- (IYDU, post- perinatal especially
17 making), perinatal transfusion) waterborne
18 (Birthing)
19 INCUBATION Short (weeks) Long (months) Long Superinfection Short
20 (HOY after
21
!!BY) = short
22
Coinfection (HOY
23
with HBV) = long
24
CLINICAL COURSE Asymptomatic Initially li ke serum l\llay progress to Similar to HBY Fulminant hepatitis
25
26 (usually), Acute sickness (fever, Cirrhosis or in Expectant
27 arth ralgias, rash); Carcinoma {pregnant) women
28 may progress to
29 carcmoma
30 PROGNOSIS Good Adults ..... mostly Nlajority develop Superinfection High mortality in
31
full resolution; stable, C hronic ..... worse prognosts pregnant women
32
neonates ..... worse hepatitis C
33
prognosis
34
35
HCC RISK lo Yes Yes Yes 'o
36 LIVER BIOPSY Hepatocyte Granular Lymphoid Similar to HBY Patchy necrosis
. 37 swelling, eosinophilic aggregates with •
. •
8
Lock
s
Suspend
0
End Block
Item: 36 of 60 - ,• Mark --<] [::> ""'I ~· 1!';:'1
QIO: 1825 ~ Prev1o u s Next Labf a lues Notes Calculator

12
• FA17 p 163.1

13 RNA viruses
14 VIRAL FAMILY ENVELOPE RNA STRUCTURE CAPSID SYMMETRY MEDICAL IMPORTANCE
15
Reoviruses No OS linear Icosahedral Coltivirus3 - Colorado tick fever
16
10-12 segments (double) Rotavirus-cause of fatal diarrhea in children
17
18 Picornavi ruses [ O SS Et> linear Icosahedral Poliovirus-polio-Salk/Sabin vaccines- IPV/OP
19 l .cho\·irus-aseptic meningitis
20 Rhino\ irus-"common cold"
21 c o, sackievirus-aseptic meningitis; herpangina
22 (mouth blisters, fever); hand, foot, and mouth
23 disease; myocarditis; pericarditis
24 II V-acute \'ira) hepatitis
25 PERCII
26 Hepevirus I 0 SS Et> linear icosahedral HEV
27
28
Caliciviruses No SS ® linear Icosahedral 'orovirus-viral gastroenteritis
29 Flaviviruses Yes SS Et> linear Icosahed ral HCV
30 Yellow fever:1
31 Denguea
32 St. Louis encephalitisa
33 West 1ile virus3 (meningoencephalitis)
34 Zi~a virus
35
Togaviruses Yes SS ® linear lcosa hedral Rubella
36
\ estern and Eastern equine eneepha litis3
. 37
. Chikungunya \·irus •
8
L.odt
s
Su~pl'nd
~
End Block
Item: 36 of 60 ~ ,• Mark <::J [:::> ""I ~· ~'j
QID: 18 25 J.. Previous Next LAb faiUI~S Notes Calculator

12 FA17 p 160.1
13 DNA viruses
14 VIRAL FAMILY ENVELOPE DNA STRUCTURE MEDICAL IMPORTANCE
15
Herpesviruses Yes OS and linear See Herpesviruses entry
16
17 Poxvirus Yes OS and linear Smallpox eradicated world wide by use of the live-
18 (largest 0 A virus) attenuated vaccine
19 Cowpox ("mi lkmaid blisters")
20 Molluscum contagiosum - Aesh-colored papule with
21 centra I umbiI ication
22 Hepadnavirus Yes Partially OS and circular HBV:
23
• cute or chronic hepatitis
24
• I ot a retrovirus but has re,·erse transcriptase
25
26
Adenovirus 0 OS and linear F'ebrile pharyngitis f.J - sore throat
27
Acute hemorrhagic cystitis
28
Pneumonia
29 Conju ncti,·itis-"pink eye"
30
31
32
33
34
35 Papillomavirus No OS and circular 1--1PV- warts (serotypes l, 2, 6, ll), Cl1 , cervica l
36 cancer (most com monly 16, 18)
. 37
. Polyomavirus 0 OS and circular JC virus-progressive multifocal leukoencephalopathy
8
Lock
s
Suspend
0
End Block
Item: 37 of 60 - ,• Mark -<J [:::> "'I ~ · ~
QIO: 2032 ~ P~v•ous N@xt Labl lues No tes Calcula t o r

12 A A 32-year-old woman presents to her fa mily doctor complaining of fatigue, myalgia, and anorexia fo r nearl y 1 week. Physical exam ination i.AAj A

revea ls cervical lymphadenopat hy and t he rash seen in t he im,.,m,


13
14

15
16
17
18
19

20
21
22
23
24
25
26
27
28
29
30
31
32
33 If this illness is left untreated, which of t he following symptoms or conditions is most likely t o occur next ?
34 :
35 A. Aortic aneurysm
36 B. Argyll Robertson pupil
. 37
. C. Autoimmune polyarthritis
8
Lode.
s
Suspe-nd
8
End Bloc:k
12
13
14

15
16
17
18
19
20
21
22
23
24
25
26
27
28
29
If thi s illness Is left untreated, which of t he following symptoms or conditions Is most likely to occur next?
30
:
31
A . Aortic aneurysm
32
33 B. Argyll Robertson pupil
34 C. Autoimmune polyarthritis
35
D. Facial nerve palsy
36
. 37 E. Opportunistic infection with Pneumocystis jiroveci
. •
8
L.odt
s
Su~pl'nd
~
End Block
Item: 37 of 60 ~ ,• Mark <::J [:::> ""I ~· ~'j
QID: 2032 J.. Previous Next faiUI~S
LAb Notes Calculator


12 The correct a nswer is 0. 79°/o chose t his.
13 This patient present s in t he early localized stage (st age 1) of Lyme disease, caused by infect ion wit h the spiroc hete Borrelia burgdoderi. This
14 organism is carried by several species of the Ixodes t ick and is common in t he northeastern Unit ed Stat es. This first stage is charact erized by
a flu-like illness and t he erythema migrans rash, which cl assically spreads over time and develops a cent ra l clearing (see image) . The second
15
stage (early disseminated disease) targets four organ syst ems: skin, cent ra l nervous syst em, hea1t, and joints. Bilateral cranial nerve VII
16 palsy is a common early effect of B. burgdoderi infectio n. This bilateral cranial nerve VII palsy is not t he same as Bell's palsy, which is a term
17 for idiopathic cranial nerve VII palsy.
Bell's palsy Spirochaete Lyme disease Borrelia burgdorferi Erythema chronicum migrans Central nervous system Erythema Cranial nerves Tick Ixodes Idiopathy Nervous system Borrelia Rash Facial nerve
18
Infection Organism United States Influenza-like illness
19
20 A is not correct . 4 °/o chose this.
21 A01tic aneurysms fo rm in approximately 10% of pat ients wit h t ert iary syphilis as the spirochete blocks t he vasa vesorum and t hus causes
t issue breakdown of the advent itia. The cardiovascular involvement t ypical of disseminated Lyme disease, on t he other hand, is
22
atri oventricularnodal block, myocarditis, or left ventri cular function.
23 Spirochaete Lyme disease Myocarditis Syphilis Aneurysm Heart Circulatory system Ventricle (heart)

24
B is not correct. 3% chose this .
25
Argyll Robertson pupil is charact eristic of te1t iary syphilis. As a resu lt of a lesion of t he midbrain t ectum, the pupil constricts during
26 accommodation but not in response to light. Efferent f ibers f rom t he Edinger-Westphal nucleus are believed to be damaged by Treponema
27 pal/idum, causing a disruption in the efferent signal f rom t he nucleus, which med iat es the response t o light. Borrelia burgorferi does not cause
such lesions.
28
Edinger-Westphal nucleus Argyll Robertson pupil Syphilis Midbrain Treponema pallidum Lesion Efferent nerve fiber Midbrain tectum
29
C is not co rrect. 13% chose this .
30
Aut oimmune polyarthri tis does occur in untreat ed Lyme disease . However, t his phenomenon occurs mont hs t o years after t he init ial
31 presentat ion of t he disease (late stage). This is distinct , h oweve 1~ from t he mya lgias and arthralgias t hat are often the chief complaint in Lyme
32 disease.
Lyme disease Polyarthritis Arthralgia Autoimmune disease Autoimmunity
33
34 E is not correct. 1 °/o chose this .
35 Opportunistic infection wit h Pneumocystis jiroveci is found in immunocompromised patients, such as those in t he lat er st ages of HIV infection
36
or wit h a hereditary immune disease.
Opportunistic infection Immunodeficiency HIV Pneumocystis jirovecii Pneumocystis pneumonia Pneumocystidomycetes Infection
37
. •
8
Lock
s
Suspend
0
End Block
Item: 37 of 60 - ,• Mark -<J [:::> "'I ~ · ~
QIO: 2032 ~ P~v•ous N@xt Labllues Notes Calculator

A A
12
Bottom Line:
13
Stage 1 Lyme disease, characterized by flu -like symptoms and erythema m igrans, can progress to stages 2 and 3 disease if untreated; these
14
later stages can involve the skin, central nervous system, heart , and joints.
15 Lyme d1!C~e Erythema chromcum m~grans Central nervous system Erythema Influenza-like 1llness Nervous ~ystem

16
17
18 14lif.il·1i•J
FllltST AID FAc·s
for y ear : 2017 •
19
20
FA17 p 142.4
21
22
Lyme disease Caused by Borrelia burgdor{eri, which is Key L) me pie to the E \CE:
23 transmitted by the Ixodes deer tick (also Facial nen·e palsr (typically bilateral)
24 vector for Anaplasma spp. and protozoa Arthritis
25 Babesia). 1 atural rcscn·oir is the mouse (and Cardiac block
26 important to tick life cycle). Erythema migrans
27 Common in northeastern United States. Treatment: doxycrcline (lst line); amoxiei ll in
28 Stage 1-early localized: erythema migrans and cefuroxime in pregnant women and
29 (typical "bulls-eye" configmation is children.
30 pathognomonic but· not always present),
31 Au-like symptoms.
32
Stage 2-early disseminated: secondary lesions,
33
carditis, AV block, facial nerve (Bell) palsy,
34
migratory myalgias/transient arthritis.
35
Stage 3-late disseminated: encephalopathies,
36
chronic arthritis.
37
.
8
Lode.
s
Suspe-nd
8
End Bloc:k
Item: 37 of 60 - ,• Mark --<] [::> ""'I ~· 1!';:'1
QIO: 2032 ~ Prev1o u s Next Labf a lu es Notes Calculator

12 FA17 p 188.1
13 Tetracyclines Tetracycline, doxycycline, minocycline.
14
MECHANISM Bacteriostatic; bind to 30S and pre\'ent attachment of aminoac}l-t RI ; limited CNS penetration.
15
16
Doxycycline is fecally eliminated and can be used in patients with renal failure. Do not take
17
tetracyclines with milk (Ca 2+), antacids (Ca 2+ or \1g 2•), or iron-containing preparations because
18
di,·alent cations inhibit drugs' absorption in the gut.
19 CLINICAL USE Borrelia burgdor{eri, M pneumoniae. Drugs' abilit) to accumulate intracellularlr makes them very
20 effective against Rickettsia and Chlamydia. Also used to treat acne. Doxycycline effecti,·e against
21 XlRSA.
22
ADVERSE EFFECTS Gl distress, discoloration of teeth and inhibition of bone growth in children, photosensitivity.
23
24
Contraindicated in pregnane).
25 MECHANISM OF RESISTANCE ! uptake or t efAux out of bacterial cells b) plasmid-encoded transport pumps.
26
27
FA17 p 502.2
28
29
Facial nerve lesions
30 Upper motor neuron Destruction of motor cortex or connection
31 lesion between motor cortex and facial nucleus i11
32 pons - contralateral paralysis of lower muscles
33
of facial expression. Forehead is spared due to
34
itS bilateral U~ (. innerV<Ition. IrK!
35
36
Lower motor neuron Destruction of facial nucleus or C VI I
37
lesion an) where along its course - ipsi latera I
. •
8
L.odt
s
Su~pl'nd
~
End Block
Item: 37 of 60 - ,• Mark --<] [::> ""'I ~· 1!';:'1
QIO: 2032 ~ Prev1o u s Next Labf a lu es Notes Calculator

12 Contraindicated in pregnancy.
13
MECHANISMOF RESISTANCE l uptake or t efAux out of bacterial cells by plasmid-encoded transport pumps.
14

15
16 FA17 p 502.2
17 Facial nerve lesions
18
Upper motor neuron Destruction of motor cortex or connect ion
19
lesion between motor cortex and facial nucleus in
20
pons - contralateral paralysis of lo\\er muscles
21
22
of facial expression. Forehead is spared clue to
CcnalloAINr
23
its bilateral U ~h innervation. ~11(1

24 lower motor neuron Destruction of facial nucleus or Cl'\ II


(\JII'f _ _ - - ; : ; : :
25 lesion anywhere along its course - ipsilateral "'"' ~ · nudtus
Centnl
26 paralysis of upper and lower muscles of IK>IO
27 facia I expression , , hr peracusis, loss of taste
28 sensation to anterior tongue.
29
When idiopathic (most common), faci<ll nerve
30
palsy is called Bell palsy, May also be caused
31
by Lyme disease, herpes simplex, herpes
32
zoster (Ramsay Hunt syndrome), sarcoidosis,
33
tumors (eg, parotid gland), diabetes mell itus.
34
35
Treatment is corticosteroids, acyclo\'ir. \lost
36
patients gradually recover function.
37

8
L.odt
s
Su~pl'nd
~
End Block
Item : 38 o f 60 - ,• Ma rk -<J [:::> "'I ~ · ~
QIO: 2101 ~ P~v•ous N@xt Labl lues Notes Calcu lator

12 6

A 38-year-old patient, currently undergoing chemot herapy treatment for a malignancy, comes t o t he physician complaining of vag inal
13
burning and itching. On physical examination, inflammation of the walls of the vagina and vulva are observed, as well as a vaginal
14 discharge. A sample is taken, and a photomicrograph of the vaginal discharge fol lowing Gram staining (lOOOx magnification) is shown in
15 the image.
16
17
18
19
20
21
22
23
24
25
26
27
28
29
30 What is another common clinical sym ptom associated wit h a different infection by this organism?
31 :
32 A. Chronic lung disease resembling t uberculosis
33 B. Esophagitis
34
C. Lesions in lung cavities
35
36 D. Meningoencephalitis
37 E. Migrating synovitis
. 38

8
Lode.
s
Suspe-nd
8
End Bloc:k
Item: 38 of 60 ~ ,• Mark <::J [:::> ""I ~· ~'j
QID: 2101 J.. Previous Next faiUI~S
LAb Notes Calculator

12
13 The correct answer is B. 62% chos e this.
14 This patient has sympt oms of Candida albicans vulvovaginit is. Infection with t his organism also can cause thrush, usually in
15
immunocompromised pat ients. Candida ! esophagit is is common in patients with HI V infect ion . C. albicans also can cause disseminat ed disease
of any organ.
16 Candida albicans Immunodeficiency HIV Candida (fungus) Esophagitis vaginitis Organism Infection
17
A is not correct . 10% chos e this .
18
Chronic lung disease resembling t uberculosis is suggest ive of infect ion with Histoplasma organisms, among others. Histoplasmosis typically
19 does not present symptomatically, alt hough some pat ients experience a flu -like illness with feve r~ cough, headaches, and myalgias. In addit ion
20 t o causing lung disease resembling tuberculosis, histoplasmosis can disseminate, affect ing t he liver, spleen, adrenal glands, mucosal surfaces,
and meninges. Histoplasmosis occurs most commonly in t he Mississippi and Ohio River valleys.
21
Histoplasmosis Meninges Tuberculosis Spleen Histoplasma Liver Adrenal gland Chronic obstructive pulmonary disease Fever Cough Infection Ohio River Lung Mucous membrane
22
23
C is not correct. 12% chos e this.
Lesions in lung cavit ies ( "f ungus balls") are typical of Aspergillus fumigatus infection . I n the lungs A. fumigatus also can cause a patchy
24
pneumonia or pu lmonary nodules. A. fumigatus is a nondimorp hic mold wit h septat e hyphae that branch at a V-shaped 45-degree angle.
25 Aspergillus fumigatus Pneumonia Hypha Aspergillus Septum Mold Infection Lung Nodule (medicine)

26
D is not correct. 9°/o chos e this.
27
I n immunocompromised hosts, several fungi cause meningoencephalit is, most not ably Cryptococcus neoformans. Clyptococcus is a
28 nondimorphic, heavily encapsulat ed yeast found in soil and pigeon droppings. I n an immunocompetent host , infection often is asymptomatic.
29 Cryptococcus neoformans Fungus Immunodeficiency Meningoencephalitis Asymptomatic Yeast Immunocompetence Cryptococcus Infection

30 E is not correct. 7 °/o chos e this.


31 Migrating synovit is is a feat ure of disseminated gonococcal infection wit h Neisseria gonorrhoeae. N. gonorrhoeae is a gram -negative
32 diplococcus that causes uret hrit is in both sexes and pelvic inflammato ry disease in women . It is treat ed with ceftri axone .
Pelvic inflammatory disease Gonorrhea Ceftriaxone Neisseria gonorrhoeae Gram-negative bacteria Urethritis Diplococcus Neisseria Infection Inflammation
33
34
35 Bottom Line:
36 Vaginal it ching wit h a whit ish, curd -like discharge is suggest ive of Candida infectio n, which also can cause thrush and disseminated infect ion
37 in immunocomprom ised pat ients.
Immunodeficiency Candida (fungus) Itch Infection
38

8
Lock
s
Suspend
0
End Block
Item: 38 of 60 - ,• Mark --<] [::> ""'I ~· 1!';:'1
QIO: 2 10 1 ~ Prev1o u s Next Labf a lues Note s Calculator
12
FA17 p 149.1
13
Opportunistic fungal infections
14

15 Candida albicans alba = white. Dimorphic; forms pseudohyphac and budding yeasts at 20°C · , germ tubes at
16 37°c m .
17 Systemic or superficial fungal infection. Causes om I and esophageal thrush in
18 immunocompromised (neonates, steroids, diabetes, AI OS), vulvovaginitis (diabetes, use of
19 antibiotics), diaper rash, endocarditis (I drug users), disseminated candidiasis (espcciallr in
20 neutropenic patients), chronic mucocutaneous candidiasis.
21 Treatment: oral fluconazole/topical a;rolc for ''aginal ; nystatin, fluconazole, or caspofungin for oral/
22 esophageal; fluconazole, caspofu ngin, or amphotericin B for systemic.
23
Aspergillus Septate hyphae that branch at 45° \ cute \ ngle . Produces conidia in radiating chains at end of
24
fumigatus conidiophore 0 .
25
26
Causes in\'asive aspergillosis in immunocompromiscd, patients " ith chronic granulomatous disease.
27
Can cause aspergillomas in pre-existing lung cavities, especially after TB infection.
28
Some species of Aspergillus produce \ natoxins (associated with hepatocellular carcinoma).
29
Allergic bronchopulmonary aspergillosis (ABPA): hypersensitivity response associated with
30
asthma and cystic fibrosis; may c<~u se bronchiectasis and eosinophilia.
31
32
Cryptococcus 5- 10 jlm with narrow buckling. lleavily encapsulated yeast. lot dimorphic.
33 neoformans Found in soil, pigeon droppings. Acquired through inhalation with hematogenous dissem ination
34 to meninges. Culture on Sabouraud aga r. ll ighlightcd with India ink (clear halo 1]1) and
35 mucicarmine (red inner capsule ). Latex e~gglutinati on test detects polysaccharide capsular
36 antigen and is more specific.
37 Causes cry ptococcosis, cryptococcal meningitis, cryptococcal encephalitis ("soap bubble" lesions
38 in brain). orimarilv in immunocomoromised. •
8
L.odt
s
Su~pl'nd
~
End Block
Item: 38 of 60 - ,• Mark --<] [::> ""'I ~· 1!';:'1
QIO: 2 10 1 ~ Prev1o u s Next Labf a lues Note s Calculator
12
Mucor and Rhizopus Irregular, broad, nonseplale hyphae branching al wide angles 1'!].
13
14
spp. Mucormycosis. Causes disease mostly in keloaeidolic diabetic and/or neutropenic patients (eg,
15
leukemia). Fungi proliferate in blood vessel walls, penetrate cribri form plate, and enter bra in.
16 Rhinoccrebral, frontal lobe abscess; cavernous sinus thrombosis. Headache, facial pain, black
17 necrotic eschar on face; mav• have cranial nerve invohement.
18 Treatment: surgical debridement, amphotericin B.
19
20
21
22
23
24
25
26
27
28
29
30
31
32
33
34
35
36
37
38 •
8
L.odt
s
Su~pl'nd
~
End Block
Item : 38 o f 60 - ,• Ma rk -<J [:::> "'I ~ · ~
QIO: 2101 ~ P~v•ous N@xt Labl lues Notes Calcu lator

12 A A

13 FA17 p 181 .2
14 Nosocomial infections £coli (UTI) and S aureus (wound inrection) are the two most common causes.
15 RISK FACTOR PATHOGEN UNIQUE SIGNS/SYMPTOMS
16
Antibiotic use Clostridium dif{icile Watery diarrhea, leukocytosis
17
18
Aspiration (r to Polymicrobial, gram 8 bacteria, ortcn Right lower lobe infiltrate or right upper/
19 altered mental status, anaerobes middle lobe (patient recumbent); purulent
20 old age) malodorous spuhtm
21 Decubitus ulcers, S au reus (including 1 RSA), gram 8 anaerobes Eq thema, tenderness, induration, drainage
22 surgical wounds, (Bacteroides, Prevotella, Fusobacterium) rrom surgical wound sites
23
drains
24
25
Intravascular catheters S aureus (including MRS ), S epidennidis (long Erythema, induration, tenderness, drainage
26
term), £nterobacter rrom access sites
27 Mechanical ventilation, I .ate onset: P aeruginosa, Klebsiella, infiltrate on CXR, t sputum production;
C\\'

28 endotracheal Acinetobacter, S aureus sweet odor (Pseudomonas)


29 intubation
30
Renal dialysis unit, HBV, HCV
31
needlestick
32
33 Urinary catheterization £ coli, Klebsiella, Proteus spp. Dysuria, leukocytosis, Rank pain or
34 costovertebral angle tenderness
35 Water aerosols Legionella Signs of pneumonia, Gl symptoms (nausea,
36
\'Omiting). neurologic abnormalities
37
38

8
Lode.
s
Suspe-nd
8
End Bloc:k
Item: 38 of 60 - ,• Mark -<J [:::> "'I ~ · ~
QIO: 2101 ~ P~v•ous N@xt Labl lues Notes Calculator

12 A A

13 FA17 p 138.2
14
Neisseria Gram 8 diplococci. Metabolize glucose \ leninGococci ferment :\ laltose and Glucose.
15
and produce IgA proteascs. Contain Gonococci ferment Glucose.
16
lipooligosaccharides (LOS) with strong
17
endotoxin acti\'ity. N gonorrhoeae is often
18
intracellular (within neutrophils)
19
20 Gonococci Meningococci
21
o polysaccharide capsule Pol}saccharide capsule
22
23 o maltose metabolized \llaltose fermentation
24 l o ,·accine due to antigenic variation of pilus Vaccine (type B ,·accine not widely available)
25
proteins
26
27
Sexually or peri nata lly transmitted Transmitted via respiratory and oral secretions
28 Causes gonorrhea, septic arthritis, neonatal Causes meningococcemia with petechial
29 conju nctivitis (2-5 clays <I ncr birth), pelvic hemorrhages and gangrene of toes D.
30 inflammatory disease (PID), and Fitz-1Iugh- meningitis, Waterhouse-Friderichsen
31 Curtis syndrome syndrome (adrenal insufficiency, b ·cr, DIC,
32 shock)
33
Condoms ~ sexual transm ission, erythromycin Rifampin, ciproAoxacin, or ceftriaxone
34
eye ointment prevents neonatal blindness prophylaxis in close contacts
35
36 Treatment: ceftriaxone + (azithromycin Treatment: ceftria.xone or penicillin G
37 or doxycycline) for possible ch lam)dial
38 coinfection
8
Lode.
s
Suspe-nd
8
End Bloc:k
Ite m : 39 o f 60 - ,• Ma rk -<J [:::> "'I ~ · ~
QIO: 2183 ~ P~v•ous N@xt Labl lues Notes Calcu lator

13
A 31-year-old man comes t o t he physician because he is experi encing pain during urination and a purulent urethral discharge. Gram ~~AI
14
staining of the discharge revea ls numerous gra m -negat ive diplococci within leukocytes.
15
16
Which of the following describes a component of the most likely causative organism?
17
:
18
A. An ability to metabolize urea int o ammonia and carbon dioxide
19
20
B. An infectious extracellular form and an intracellular form capable of replication
21 C. Helical shape with two membranes and two flagella
22
D. Polysaccharide capsule with the ability to ferment maltose
23
E. Variable antigenic pili
24
25
26
27
28
29
30
31
32
33
34
35
36
37
38
. 39 •
8
Lode.
s
Suspe-nd
8
End Bloc:k
Item: 39 of 60 ~ ,• Mark <::J [:::> ""I ~· ~'j
QID: 218 3 J.. Previous Next LAbfaiUI~S Notes Calculator

34
35 The correct ans wer is E. 55°/o chose this .
36 Neisseria gonorrhoeae may present in male pat ients as uret hrit is and a purulent discharge. Cult ure showing gram -negative diplococci wit hin
neut rophils is sufficient fo r a diagnosis of gonorrhea in ma le pat ients. N. gonorrhoeae has specialized pili t hat mediat e its att achment to
37
mucosal surfaces but are continua lly va ri ed to evade host defenses and vaccine development.
38 Diplococcus Gonorrhea Neisseria gonorrhoeae Urethritis Gram-negative bacteria Pilus Neisseria Pus Neutrophil Vaccine Mucous membrane
39
A is not correct. 8°/o chos e this .
• 40
Ureaplasma urealyticum has no Gram st ain react ion and is a cause of nongonococcal uret hrit is. This organism has t he abilit y to conve1t urea
• 41 into ammonia and ca rbon dioxide. Other urease-posit ive organ isms include Helicobacter pylori, Nocardia, Cryptococcus, Proteus ( includ ing
• 42 Proteus mirabilis), and Klebsiella .
Gram staining Helicobacter pylori Uredp!asma ured!yticum Proteus mirabi/is Ammonia Carbon dioxide Nocardia Ure.:tp/asma Urethritis Ured Klebsiella Proteus (bacterium) Cryptococcus
. 43
• 44 B is not correct. 12% chose this .
• 45 These descri be t he element ary and ret iculate bodies, respectively, of Chlamydia trachomatis. C. trachomatis can also cause urethri t is but does
not show gram -negative diplococci wit hin polymorphonucl ear neut roph ils. I t is an obligate intracellular bacteri um t hat cannot be det ected wit h
• 46
t he Gram sta in .
• 47 Gram staining Chlamydia trachomatis Gram-negative bacteria Urethritis Diplococcus Chlamydia infection Bacteria Chla,.,dia (genus) Neutrophil Granulocyte Intracellular

. 48
C is not correct. 5 °/o chos e this .
. 49
This is a charact eristic of Treponema pa/lidum, wh ich causes syphilis. Primary syphilis typically presents wit h a painless chancre, not uret hral
• 50 discharge .
• 51 Treponema pallidum Primary syphilis Syphilis Chancre Treponema Urethra

• 52 D is not correct. 20% chose this .


. 53 Neisseria meningitidis has a polysacchari de capsule and can fe rment maltose but is not associated wit h uret hrit is. Other diffe rences between
. 54 N. meningitidis and N. gonorrhoeae include mode of t ransmission and t he exist ence of an N. meningitidis vaccine .
Polysaccharide Neisseria meningitidis Maltose Neisseria gonorrhoeae Urethritis Neisseria vaccine Fermentation
• 55
• 56
• 57 Bottom Line:
• 58 Urethri t is and a purulent uret hral discha rge point towa rd an infection wit h N. gonorrhoeae, a gram -negative diplococcus with va riable
. 59 ant igen ic pili that mediate its v irulence in t he human host.
Urethritis Gram-negative bacteria Neisseria gonorrhoeae Diplococcus Pilus Pus Virulence Urethra Infection Antigen
• 60 ~

8
Lock
s
Suspend
0
End Block
Item: 3 9 of 60 - ,• Mark -<J [:::> "'I ~ · ~
QIO: 2 183 ~ P~v•ous N@xt Labl lues No tes Calcula t o r

34 A A
FA17 p 138.2
35
36 Neisseria Gram 8 diplococci. Metabolize glucose \ leninGococci ferment :\ laltose and Glucose.
37 and produce lgA proteases. Contain Gonococci ferment Glucose.
38 Iipool igosaccharides (LOS) with strong
39 endotoxin acti\'ity. gonorrhoeae is often
• 40
intracellular (within neutrophils)
• 41
• 42
. 43
- Gonococci Meningococci
o polysaccharide capsule Pol} saccharide capsule
• 44 D
• 45
1 o maltose metabolized .Ylaltose fermentation
• 46 1 o \'accine due to antigenic \'ariation of pilus Vaccine (type B \'accine not widely a\'ailable)
• 47 proteins
. 48
Sexually or peri natally transmitted Transmitted via respiratory and oral secretions
. 49
• 50
Causes gonorrhea, septic arthritis, neonatal Causes meningococcemia with petechial
• 51 conjuncti,·itis (2-5 days after birth), pelvic hemorrhages and gangrene of toes rn.
• 52 inflammatory disease (PI D), and Fitz-llugh- meni ngitis, Waterhouse-Friderichscn
. 53 Curtis syndrome syndrome (adrenal insufficiency, fever, DIC,
. 54 shock)
• 55
Condoms l sexual transm ission, erythromycin Rifampin, ciproAoxacin, or ceftriaxone
• 56
eye ointment prevents neonatal blindness prophylaxis in close contacts
. 57
• 58
Treatment: ceftriaxone + (azithromycin Treatment: ceftria.xone or penicillin C
. 59 or doxycycline) for possible ch lamydial
. 60 coinfection
8
Lode.
s
Suspe-nd
8
End Bloc:k
Item: 40 of 60 - ,• Mark -<J [:::> "'I ~ · ~
QIO: 4886 ~ P~v•ous N@xt Labl lues No tes Calcula t o r

34 6

35
An 8-year-old boy presented to the emergency wit h nausea, abdominal cramping, bloat ing, and watery diarrhea . He is otherwise healthy.
On physical exam, he is dehydrat ed and his abdomen was tender in all four quadrants, but t here was no guarding or rebound tenderness.
36 He went camping 2 days ago. A duodenal aspirate revea ls the organism shown in the image.
37
38
39
• 40
• 41
• 42
. 43
• 44

• 45
• 46 •
• 47
. 48
. 49
• 50
Image courtesy of CDC/ Dr. Mae Melvin
• 51
• 52
What is the most likely explanation for his sympt oms?
. 53
:
. 54
A. Activation of ent era l adenylate cyclase
• 55
B. Depletion of an enzyme
• 56
. 57 C. Invasion of the lining of t he int est ine
• 58 D. Small bowel inflammation and villous atrophy
. 59
E. Viral infection of the cells of t he small intestinal villi
. 60 •
8
Lode.
s
Suspe-nd
8
End Bloc:k
Item: 40 of 60 ~ ,• Mark <::J [:::> ""I ~· ~'j
QID: 4886 J.. Previous Next faiUI~S
LAb Notes Calculator

34
35

36 The correct a nswer is 0. 44°/o chose this.


37 Colon izat ion of t he gut by Giardia t rophozoit es results in inflammation and v illous at rophy, reducing the gut 's absorpt ive capabilit y. The image
38 shows t hese pear-shaped t rophozoit es from a duodena l aspirate. Duodena l aspiration can be used t o diagnose infection with Giardia.
Trophozoit es have four pairs of flagella and t wo nuclei. They attach t o epit helial cells in duodenal and jejunal crypts t hrough a sucking disk on
39
t heir ventral surf ace. They absorb nut rients from the host and cause inflammation and malabsorption, but t hey do not invade t he intest inal
40 cells.
• 41 Flagellum Malabsorption Giardia Intestinal villus Epithelium Inflammation Duodenum Ptrophy Cell nucleus Jejunum Gastrointestinal tract Anatomical terms of location

• 42 A is not correct . 17 % chos e this .


. 43 Some strains of t he bact eria Escherichia coli possess heat- labile ent erot oxin, which increases cyclic adenosine monophosphate by activating
• 44 adenylat e cyclase t hrough t he GTP-dependent ADP ri bosylation of specific membrane proteins. This gives ri se t o secret ory dia rrhea . The heat -
labile toxin is closely re lat ed in st ructure and funct ion to the enterot oxin expressed by Vibrio cholerae .
• 45
Heat-labile enterotoxin Vibrio cholerae Escherichia coli Cyclic adenosine monophosphate Adenylyl cyclase Enterotoxin Bacteria Diarrhea Adenosine diphosphate Toxin Adenosine Adenosine monophosphate
• 46
• 47
B is not correct. 9 % chose t his .
Deplet ion of t he enzyme lactase (called lact ose intolerance) allows undigested lact ose t o pass int o t he colon, wh ich causes an osmotic
• 48
diarrhea. Colonic bacteria can furthe r ferment t he suga r ( lact ose) to hydrogen gas and organic acids, producing bowel dist ent ion, bloat ing,
. 49 cramping, and abdominal pain . Lactase deficiency can be a postinfect ion complicat ion of giardiasis, but is not seen in t he acute setting .
• 50 Giardiasis Lactose intolerance Lactase Lactose Enzyme Diarrhea Gut flora Bloating Colon (anatomy) Abdominal pain Bacteria Sugar Hydrogen Fermentation Large intestine Osmosis

• 51 Fermentation in food processing

• 52 C is not correct. 27% chos e this .


. 53 Several organisms have the ability t o invade t he lining of the int estine when t hey infect humans. Cryptospori diosis, caused by infection wit h
. 54 t he protozoan Cryptosporidium, is one example. It causes nonbloody, mucousy diarrh ea; flatu lence and bloat ing; and a low-grade fever.
Amebiasis, an infection caused by t he protozoa l organ ism Entamoeba histolytica, is another invasive organism. Patients with amebic colit is
• 55
t ypically present with a hist ory of several weeks of abdominal pain, diarrh ea, and bloody st ools. A cha racterist ic f ind ing of E. histolytica
• 56 t ro phozoit es is that t hey often have ingested RBCs in their cytoplasm, whereas the cyst ic forms have a dist inctive " ri ng and dot " nucleus and
• 57 ch romatoid bodies .
Entamoeba histolytica Cryptosporidiosis Cryptosporidium Amoebiasis Cytoplasm Flatulence Protozoa Gastrointestinal tract Diarrhea Colitis Abdominal pain Entamoeba Fever Organism Trophozoites
• 58
Cell nucleus Bloating Infection Apicomplexan life cycle
. 59
• 60 E is not correct. 3 °/o chos e this .
8
Lock
s
Suspend
0
End Block
Item: 40 of 60 ~ ,• Mark <::J [:::> ""I ~· ~'j
QID: 4886 J.. Previous Next LAbfaiUI~S Notes Calculator

34
E is not correct. 3°/o chos e this .
35
Vira l ent erit is is caused by v ira l infect ion of t he epit helial cells of the small intest inal villi, especially those near t he t ips of t he villi. Because t he
36 villi are important fo r the digest ion of carbohydrat es and absorption of flu id and elect rolytes, ent eric viruses can lead to malabsorption by
37 decreased hydrolysis of carbohydrat es and excess fluid loss f rom t he intest ine. On microscopy, there is often only an inflammatory infilt rate
38 and intest inal villus blunt ing . Wit h extensive damage, one may see fusion of adjacent villi resulting in significant ly decreased absorptive
surface area.
39 Intestinal villus Malabsorption Epithelium Enteritis Hydrolysis Gastrointestinal tract Digestion Virus Carbohydrate Gastroenteritis Infection Electrolyte Microscopy Small intestine Human gastrointestinal tract
40
• 41

• 42
Bottom Line :
. 43 Giardia t rophozoites can cause inflammation and villous atrophy in t he gut , which reduce its absorptive capabilit y because they absorb
nutrients and cause malabsorption. Giardia infect ion is diagnosed by duodenal aspira t ion .
• 44 Giardia Malabsorption Intestinal villus Duodenum Inflammation Atrophy
• 45
• 46
• 47 l@ljl'il·1i•J for yea r:[2017 • J
FIRST AID FA CTS
• 48
. 49
FA17 p 151.1
• 50
• 51 Protozoa- gastrointestinal infections
• 52 ORGAN ISM DISEASE TRANSMISSION DIAGNOSIS TREATMENT
. 53
Giardia Iamblia Giardiasis- bloating, flatulence, Cysts in water Multinucleated Metron idazole
. 54
foul-smelling, fatly diarrhea trophozoites fJ or
• 55
(often seen in campers/h ikers)- cysts I]) in stoo I,
• 56
lhink fat-rich G hirardelli anligen delection
• 57
chocolates for fatty stools of
• 58
. 59
Giardia
• 60 Entamoeba Amebiasis- bloody diarrhea C ysts in water Serology and/or Metron idazole;
8
Lock
s
Suspend
0
End Block
Item: 40 of 60 - ,• Mark -<J [:::> "'I ~ · ~
QIO: 4886 ~ P~v•ous N@xt Labl lues No tes Calcula t o r

34 A A
FA17 p 151 .1
35
36 Protozoa- gastrointestinal infections
37 ORGANISM DISEASE TRANSMISSION DIAGNOSIS TREATMENT
38 Giardia Iamblia Giardiasis- bloating, Aatulence, Cysts in water ~ lultinucleated Metronidazole
39 foul-smelling, fatty diarrhea trophozoites or
40
(often seen in campers/hikers)- C)'StS 11] in stool,
• 41
think fat-rich Ghirardelli a ntigen detection
• 42
chocolates for fatty stools of
. 43
G iardia
• 44

• 45 Entamoeba Amebiasis- bloody diarrhea Cysts in water Serology and/or ~ letron idazole;
• 46 histolytica (dysentery), Ii\'er abscess trophozoites (with paromomycm or
• 47 ("ancho''Y paste" exudate), engulfed RBCs [i iodoquinol for
. 48 RUQ pain; histology sho" s in the cytoplasm) asymptomat ic cyst
. 49 Aask-shaped ulcer or cysts with up to passers
• 50 4 nuclei in stool f!l;
• 51 E ntamoeba Eats
• 52 Ery throcytes,
. 53 antigen detection
. 54
• 55
• 56
Cryptosporidium Se,·ere diarrhea in AIDS Oocysts in water Oocysts on acid-fast Pre,·ention (by
. 57
.M ild disease (watery diarrhea) in stain 0 , antigen filtering city
• 58 immunocompetent hosts detection water supplies);
. 59 nitazoxanide in
. 60 •
immunocompetent
8
Lode.
s
Suspe-nd
8
End Bloc:k
Item: 40 of 60 ~ ,• Mark <::J [:::> ""I ~· ~'j
QID: 4886 J.. Previous Next LAbfaiUI~S Notes Calculator

34
FA17 p 153.1
35
36
Protozoa- hematologic infections
37
ORGANISM DISEASE TRANSMISSION DIAGNOSIS TREATMENT
38 Plasmodium Malaria - fever, headache, anemia, Anopheles mosquito Blood smear: Chloroquine (for
39 P vivax/ovale splenomegaly trophozoite ring sensitive species),
40 P falciparum P vivax/ovale- 48-hr cycle (tertian; form within which blocks
• 41 P malariae includes fever on first day and RBC fJ, schizont· Plasmodium heme
• 42 third day, thus fevers are actually containing polymerase; if
. 43
48 hr apart); dormant form merozoites; red resistant, use
• 44
(hypnozoite) in liver granules (Schi.iffner mefloquine or
• 45
• 46
P falciparum- sevcrc; irregular stippling) rn atovaquonc/
fever patterns; parasitized RBCs throughout RBC proguanil
• 47
occlude capillaries in brain cytoplasm seen If life-threatening,
. 48
. 49
(cerebral malaria), kidneys, lungs with P vivaxlovale use intravenous
• 50
P nwlariae- 72-hr cycle (quartan) quinidine or
• 51
artesunate (test for
• 52
G6PD defi ciency)
. 53 For P vivaxlovale,
. 54 add prim<lquine for
• 55 hypnozoite (test for
• 56 G6PD defi ciency)
• 57 Babesia Babesiosis-fever and hemolytic Ixodes tick {same as Blood smear: ring Atovaquone
• 58
anemia; predominantly in Borrelia burgdorferi form m," laltese + azithromycin
. 59
northeastern United States; of Lyme disease; cross" [!); PCR
• 60 ~ ___ ._+_ :_ . __! _ ) _ {" __ - - - -•=----- ----- _r..___ __ : __ c __ ._

8
Lock
s
Suspend
0
End Block
Item: 40 of 60 ~ ,• Mark <::J [:::> ""I ~· ~'j
QID: 4886 J.. Previous Next LAbfaiUI~S Notes Calculator

34
35 FA17 p 175.1
36 Bugs causing diarrhea
37 Bloody diarrhea
38
Campylobacter Comma- or S-shaped organisms; growth at 42°C
39
40 E histolytica Protozoan; amebic dysentery; liver abscess
• 41 Enterohemorrhagic 0157:1-17; can cause IIUS; makes Shiga-like toxin
• 42
Ecoli
. 43

• 44
Enteroinvasive Ecoli Invades colonic mucosa
• 45 Salmonella (non- Lactose 8; Aagellar motility; has animal reservoir, especially poultry and eggs
• 46 typhoidal)
• 47
Shigella Lactose 8; very low 10 ;0; produces Shiga toxin (human reservoir only); bacillary dysentery
• 48
. 49 Yenterocolitica Day care outbreaks, pseudoappendicitis
• 50 Watery diarrhea
• 51
Cdifficile Pseudomembranous colitis; associated with antibiotics and PPis; occasionally bloody diarrhea
• 52
. 53 C perfringens Also causes gas gangrene
. 54 Enterotoxigenic E coli Tra,·elers' diarrhea; produces heat-labile (LT) and heat-stable (ST) toxins
• 55
Protozoa Giardia, Cryptosporidium
• 56
• 57 Vcholerae Comma-shaped organisms; rice-water diarrhea; often from infected seafood
• 58 Viruses Rota,·irus, norovirus, adenovirus
. 59
• 60

8
Lock
s
Suspend
0
End Block
Item: 41 of 60 - ,• Mark -<J [:::> "'I ~ · ~
QIO: 4487 ~ P~v•ous N@xt Labl lues No tes Calcula t o r

34
35 The im age depict s th e cell wall of a gram- negative bact eri um.
D
36
37
38 A

39
40
• 41
• 42 E ~~ }
. 43

c{ ~
• 44

' 45
' 46
' 47
. 48 What st ructure is responsible for t he pathogenicity of the organism?
. 49 :
• 50 A
• 51
B
• 52
. 53
c
. 54 D
' 55 E
' 56
' 57
' 58
. 59

' 60 •
8
Lode.
s
Suspe-nd
8
End Bloc:k
Item: 41 of 60 - ,• Mark -<J [:::> "'I ~ · ~
QIO: 4487 ~ P~v•ous N@xt Labl lues No tes Calcula t o r

34 A A

35 The correct answer is A. 72°/o chose this.


36
The pathogenicity of a gram-negat ive bacterium is largely due to structures in the out er
37
membrane. This membrane consists of a lipopolysaccharide (diagrammed in the image) and
various proteins such as endotoxins. Endotoxins, specifica lly lipid A, can activate the complement
\
38 cascade, the coagulation cascade, and macrophages. The outer membrane proteins have
39 enormous genetic and antigenic variability.
l.Jpopol, oc:c~ de Ghon- t9ab e b..ctena LJp1d A Coagulation Pathogen Macrophage l.lp1d Complement svstem 81octena M1tochondnon Outer
40 membrane
Bactenal outer membrane &olog•cal membrane Protein Membrane protein Antigen CeU membrane
41
• 42 Peptidoglycan

. 43
• 44
Inner
membrane
' 45
' 46 Cytosol
Mtmbt~ne prottlns
' 47
. 48 B is not correct. 8 °/o chose this.
. 49 The periplasmic space is immediat ely beneat h the outer membrane and is adjacent t o t he peptidoglycan, which forms t he major cell wall
• 50 constituent .
Peptidoglycan Penplasm Cell wall Bactenal outer membrane Mitochondrion Biological membrane Membrane
• 51

• 52 C is not correct. 6°/o chose this .


. 53 The inner cytoplasmic membrane contains penicillin-binding prot eins.
Cell membrane Pen~e1ll1n b1nd1ng prote1ns Protein Cytoplasm
. 54

' 55 D is not correct. 7 °/o chose this.


' 56 Porins are beta barrel proteins t hat cross the cell membrane and allow passive diffusion of molecules.
Beta barrel Cell membrane Ponn (protem) D1ffus•on Protein Biological membrane Membrane ~~~ive transport
' 57
' 58 E is not correct. 7 °/o chose this.
. 59 Choice E represents the peptidoglycan layer. Inhibition of peptidoglycan synthesis is the mechanism of action for the penicillins and
cephalosporins.
' 60 n-._.1 ..... 1 ........ ,.._ ' .. ... .... ......._ ... , - u-.-L.-.. ;.. _,. ..& ~ ......

8
Lode.
s
Suspe-nd
8
End Bloc:k
Item: 41 of 60 - ,• Mark --<] [::> ""'I ~· 1!';:'1
QIO: 4487 ~ Prev1o u s Next Labf a lu es Notes Calculator
34
35 Bottom Line:
36 The cell wall includes the outer membrane, peptidoglycan layer, and inner membrane components. Endotoxins and lipopolysaccharide present
37 in the cell wall of gram-negative bacteria are largely responsible for their pathogenicity.
Llpopolysacchande Peptidoglycan Cell wall Gram-negative bacteria Bacterial outer membrane Pathogen M•tochondr1on Bactena Inner membrane Inner mitochondrial membrane Blologtcal mefllbrane
38
Cell membrane Membrane
39
40
41
141;f;l·1hl for year: 2017 •
fiRST A ID fACTS
• 42
. 43
• 44 FA17 p 120.2
• 45 Cell walls
• 46
Unique to Common to both Unique to
• 47 gram (±) gram e
. 48
(---~'"'----l
. 49
• 50
• 51
• 52
. 53
. 54
• 55
• 56
. 57
• 58
Gram (±) Gram e
. 59
• 60 •
8
L.odt
s
Su~pl'nd
~
End Block
Item: 41 of 60 ~ ,• Mark <::J [:::> ""I ~· ~'j
QID: 4487 J.. Previous Next LAbfaiUI~S Notes Calculator

34 • •
FA17 p 120.1
35
Bacterial structures
36
37
STRUCTURE CHEMICAL COMPOSITION FUNCTION
38 Appendages
39 Flagellum Proteins. Motility.
40
Pilus/fimbria G lycoprotein. Mediate adherence of bacteria to cell surface;
41
• 42
sex pi h1s forms during conjugation .
. 43 Specialized structures
• 44
Sp ore Keratin-like coat; dipicolinic acid; Cram (f) only.
• 45
peptidoglrcan, D NA. Survival: resist dehydration, heat, chem icals.
• 46
• 47 Cell envelope
• 48 Cap sule Organized, d iscrete polysaccharide layer (except Protects aga inst phagocytosis.
. 49
poly-o-glutamate on B anthracis).
• 50
Glyco calyx Loose network of polysaccharides. Mediates adherence to surfaces, especially
• 51
foreign surfaces (eg, indwelling catheters).
• 52
. 53 Oute r me mbra n e Outer leaflet: contains endotoxin (LPS/ LOS). C ram 8 only.
. 54 Embedded proteins: porins and other outer Endotoxin: lipid A induces T J F and IL-l;
• 55 membrane proteins (O~lPs) antigen ic 0 polysaccharide component.
• 56 Inner leaflet: phospholipids. Most OMPs are antigenic .
• 57 Porins: transport across outer membrane.
• 58
Peri plasm Space bel ween cytoplasm ic membrane Accu mu lates components exiling gram
. 59
• 60 •
and outer membrane in gram bacteria. e e cells, including hydrolytic enzymes •

8
Lock
s
Suspend
0
End Block
Item: 42 of 60 - ,• Mark -<J [:::> "'I ~ · ~
QIO: 3281 ~ P~v•ous N@xt Labl lues No tes Calcula t o r

6
27 ~~AI
A 28-year-old man who lives in rura l upstate New York visits his physician because he has noticed a slight droo ping of t he left side of his
28 face in the past 3 days. He is an avid hiker, and recent ly v isit ed the Adirondacks on a 4-day excursion. The pat ient is very concerned that
29 he may be experiencing a stroke. He is unable to smile, close his eye, or wrinkle his forehead on the left side, but has no other neurologic
deficits.
30
31
What signs and symptoms are most likely to be associated with this patient's condition?
32
:
33
A. Atrial fibrillation
34
35 B. Atrioventricular conduction delay
36 C. Carotid stenosis
37
D. Eeduced ankle:brachial index
38
39
E. Myocardial ischemia
40
41
• 42
. 43
• 44
• 45
• 46
• 47
. 48
. 49
• 50
• 51
• 52

8
Lode.
s
Suspe-nd
8
End Bloc:k
Item: 42 of 60 ~ ,• Mark <::J [:::> ""I ~· ~'j
QID: 3281 J.. Previous Next LAb faiUI~S Notes Calculator

27 •

28
The correct answer is B. 68% chose this.
29
This patient has a facial nerve palsy secondary t o Lyme disease, which is caused by Borrelia burgdorferi. The vector is t he Ixodes t ick, wh ich
30
is fou nd commonly in t he northeastern Unit ed Stat es. Often, a t ick bite is not recalled, and not all pat ients develop t he characterist ic "bull's-
31 eye" rash. Facial nerve palsy can be diffe rentiated f rom stroke by asking t he patient to f urrow his forehead: the occipit of ront alis muscle is
32 cont rolled by bot h mot or cortices, so inability t o f urrow is either t he result of t wo strokes in the same precise locat ion in bot h hemispheres
(unlikely) or a result of dysfunction of t he lower motor neuro n of cranial nerve VII ( likely) . I n addit ion to facial nerve palsy, Lyme disease can
33
man ifest with t ransient arthralgias, atri oventri cular noda l block, secondary annu lar rashes, and cardi t is. Cardiac involvement is seen in 5% of
34 patients, whereas neurolog ic involvement is seen in 15%. Atriovent ricula r block can be symptomatic and high-grade ( ie, third -deg ree as seen
35 in the ECG). Doxycycline is t he antibiot ic t reatment of choice fo r most patient s; h oweve r~ fo r those with cardiovascular or neurologic
involvement , int ravenous ceftriaxone is t he t reat ment of choice.
36
37
Of not e- Bell palsy is not synonymous wit h facia l nerve palsy. Bell palsy is a facial nerve palsy of idiopathic origin . Therefore, t his patient
does NOT have Bell palsy; t he facial nerve palsy has an obvious pri mary cause (Lyme disease). This important considerat ion is often
38 m isunderst ood by medical stu dents.
39 Bell's palsy Lyme disease Doxycycline Borrelia burgdorferi Ceftriaxone Neuron Lower motor neuron Antibiotics Facial nerve Cranial nerves Facial nerve paralysis Arthralgia Intravenous therapy Motor neuron
40 Borrelia Idiopathy Rash Stroke Heart Ixodes Tick Muscle Vector (epidemiology) Motor cortex United States
41
42 t Itt T : fI ~t· ·t :7 Itt I +tt
. 43 ·- j ~ ·~ j ~ ~ ''i::Y
• 44
-' 1- -' - 1-
.c
./\ ~
• 45 ..
tT,e- j : i- ~ i- ~ 1-
• 46 I ,.
tTTT" :::;:.. . ~H ii
f+t+
• 47
. 48 A is not correct . 14% chose this .
. 49 Pat ients wit h at rial fibri llation who are not ant icoagulated are at ri sk of developing an atri al t hrombus t hat may occlude an artery when
dislodged . I f an artery in t he brain is occluded, st roke results. The embolus can get st uck in other vessels as well, such as a mesent eric artery .
• 50 This patient did not suffer a st roke, however.
• 51 Atrial fibrillation Thrombus Embolism Embolus Stroke Anticoagulant Atrium (heart) Human brain vascular occlusion

• 52 C is not correct. 7 °/o chose this.

8
Lock
s
Suspend
0
End Block
Item: 42 of 60 ~ ,• Mark <::J [:::> ""I ~· ~'j
QID: 3281 J.. Previous Next LAbfaiUI~S Notes Calculator

27
A is not co rre ct . 14% chos e this .
28 Pat ients wit h at rial fibrillation who are not anticoagulated are at ri sk of developing an atri al t hrom bus that may occlude an artery when
29 dislodged . I f an artery in the brain is occluded, st roke results. The embolus can get st uck in other vessels as well, such as a mesent eric artery.
30
This patient did not suffer a st ro ke, however.
Atrial fibrillation Thrombus Embolism Embolus Stroke Anticoagulant Atrium (heart) Human brain vascular occlusion
31
32 C is not co rrect. 7 °/o chose this .
Carotid stenosis is anot her risk fact or for t he development of stroke. It can be managed medically or, if severe or sympt omat ic, with surge ry
33
(carot id endarterectomy) . I t can often be detected by heari ng a bruit of t he carot id artery. However, t his is not t he disease process underlying
34 t his pat ient's presentation .
35 Carotid endarterectomy Carotid artery stenosis Stenosis Endarterectomy Carotid artery Risk factor Stroke Common carotid artery

36 D is not corre ct. 6°/o chos e this.


37 A red uced ankle:brachial index may be seen in patients with st enosis of peripheral arteries. I t oft en man ifests wit h claudicat ion that may
38 occur wit h exercise or even at rest in more severe cases. Patients who suffe r an ischemic stroke due to a vasculopat hic process are at ri sk of
developing ischemia in other parts of t he body. However, this patient did not have a stroke.
39
Ischemia Ischemic stroke Stroke Stenosis Claudication Artery Peripheral vascular system
40
41
E is not correct. 5 °/o chose this .
Pat ients t hat have luminal narrowing of a vessel in the bra in due to art hrosclerosis are at risk fo r stroke. If the disease process is also present
42
in the coronary vessels, myocardial ischem ia can resu lt. However, this patient did not suffer a st roke .
. 43 Ischemia Coronary artery disease Lumen (anatomy) Coronary circulation Stroke Brain
• 44
• 45
Bottom Line :
• 46
Lyme disease is caused by t he spirochet e Borrelia burgdoderi. This disease can move from a primary phase to a secondary phase if
• 47
untreat ed . One possible secondary phase complication is facial nerve palsy. Facial nerve palsy can be different iated from stroke by a
• 48 neurologic examinat ion .
. 49 Spirochaete Lyme disease Borrelia burgdorferi Facial nerve Facial nerve paralysis Borrelia Neurological examination Neurology Stroke

• 50
• 51
• 52
i@l;fil·1i•J fo r yea r:[2017 • J
FIRST AID FACTS

8
Lock
s
Suspend
0
End Block
Item: 42 of 60 - ,• Mark -<J [:::> "'I ~ · ~
QIO: 3281 ~ P~v•ous N@xt Labl lues No tes Calcula t o r

27
28
29 l@l ; fii ~11•J for year: 2017 •
FI RST AID FACTS
30

31
FA17 p 142.4
32
33 Lyme disease Caused by Borrelia burgdorferi, which is Key L) me pie to the E \ CE:
34 transmitted br the Ixodes deer lick (also Facial ner\'e palsr (typically bilateral)
35 vector for Anaplasma spp. and protozoa Arthritis
36 Babesia). Natural reservoir is the mouse (and Cardiac block
37 important to tick life cycle). ~ f) thcma migrans
38 Common in northeastern United Stales. Treatment: doxrcrcline (1st line); amoxicillin
39 Stage 1-early localized: erythema migrans and cefuroxime in pregnant women and
40
(typical "bu lls-eye" configmation is children.
41
pathognomonic but not always present),
42
Au-like symptoms.
. 43
Stage 2-early disseminated: secondary lesions,
• 44
carditis, AV block, facial nerve (Bell) palS)',
• 45
• 46
migratory myalgias/lransicnl arthritis .
• 47
Stage 3-late dissem inated: encephalopathies,
. 48
chronic arthritis.
. 49
• 50
• 51
FA17 p 502.2
• 52
Facial nerve lesions
8
Lode.
s
Suspe-nd
8
End Bloc:k
30

31 FA17 p 502.2
32 Facial nerve lesions
33
Upper moto r neuron Destruction of motor cortex or connection
34
35
lesion between motor cortex and facial nucleus in
36
pons - contralatcra I paralrsis of lo" cr muscles
37
of facial expression. Forehead is spared due to
38 its bilateral U~l:"\1 innervation.
39 Lower motor neuron Destruction of facial nucleus or C II
40 lesion anywhere along its course - ipsi latera I
41 paralysis of upper and lower muscles of
42
facial expression fJ, hypcracusis, loss of taste
. 43
sensation to anterior tongue.
• 44
When idiopathic (most common), facia l nerve
• 45
palsy is called Bell palsy. May <llso be cau~ed CHVII
IU4Nieoon •
• 46
by Lyme disease, herpes simplex, herpes fkNI .....
polsyl
• 47
. 48
zoster (Ramsay Hunt syndrome), sarcoidosis,
. 49
tumors (eg, parotid gland), diabetes me II it us .
• 50
Treatment is corticosteroids, acyclo' ir. Most
• 51 patients gradually recover function .
• 52

8
Lode.
s
Suspe-nd
8
End Bloc:k
Item: 43 of 60 - ,• Mark -<J [:::> "'I ~ · ~
QIO: 5264 ~ P~v•ous N@xt Labl lues Notes Calculator

6
27 ~~AI
A 64-year-old man who likes t o hunt and fish in t he north woods of Wisconsin presents t o his intern ist because his wife t old him he started
28 "smiling funny" over t he weekend . He does recall feel ing feveri sh and weak, with a strange red rash surrounding a clear center, after his
29 last hunting trip, which was about 3 weeks ago. A porti on of his examination is shown in th e video clip.
30 OPEN MEDIA

31
32
Which of the following organisms did the patient most likely encounter on his last hunting trip?
33
:
34
A. Borrelia burgdotferi
35
36 B. Orientia tsutsugamushi
37 C. Rickettsia rickettsii
38
D. Treponema pallidum
39
40
E. Varicella zoster
41
42
. 43
• 44
• 45
• 46
• 47
. 48
. 49
• 50
• 51
• 52

8
Lode.
s
Sus pe-nd
8
End Bloc:k
Item: 43 of 60 - ,• Mark -<J [:::> "'I ~ · ~
QIO: 5264 ~ P~v•ous N@xt Labl lues Notes Calculator

6
27 ~~AI
A 64-year-old man who likes t o hunt and fish in t he north woods of Wisconsin presents t o his intern ist because his wife t old him he started
28 "smiling funny" over t he weekend . He does recall feel ing feveri sh and weak, with a strange red rash surrounding a clear center, after his
29 last hunting trip, which was about 3 weeks ago. A portion of his examination is shown in th e video clip.
30 OPEN MEDIA

31
32
33
34
35
36
37
38
39
40
41
42
. 43
• 44
• 45
• 46
• 47
. 48
. 49
• 50
• 51
Which of the following organisms did the patient most likely encounter on his last hunting trip?
• 52 :
A Rorrpfiff humrlorfPri

8
Lode.
s
Sus pe-nd
8
End Bloc:k
Item: 43 of 60 - ,• Mark -<J [:::> "'I ~ · ~
QIO: 5264 ~ P~v•ous N@xt Labl lues Notes Calculator

6
27 ~~AI
A 64-year-old man who likes t o hunt and fish in t he north woods of Wisconsin presents t o his intern ist because his wife t old him he started
28 "smiling funny" over t he weekend . He does recall feel ing feveri sh and weak, with a strange red rash surrounding a clear center, after his
29 last hunting trip, which was about 3 weeks ago. A portion of his examination is shown in th e video clip.
30 OPEN MEDIA

31
32
33
34
35
36
37
38
39
40
41
42
. 43
• 44
• 45
• 46
• 47
. 48
. 49
• 50
• 51
Which of the following organisms did the patient most likely encounter on his last hunting trip?
• 52 :
A Rorrpfiff humrlorfPri

8
Lode.
s
Sus pe-nd
8
End Bloc:k
Item: 43 of 60 - ,• Mark -<J [:::> "'I ~ · ~
QIO: 5264 ~ P~v•ous N@xt Labl lues Notes Calculator

6
27 ~~AI
A 64-year-old man who likes t o hunt and fish in t he north woods of Wisconsin presents t o his intern ist because his wife t old him he started
28 "smiling funny" over t he weekend . He does recall feel ing feveri sh and weak, with a strange red rash surrounding a clear center, after his
29 last hunting trip, which was about 3 weeks ago. A portion of his examination is shown in th e video clip.
30 OPEN MEDIA

31
32
33
34
35
36
37
38
39
40
41
42
. 43
• 44
• 45
• 46
• 47
. 48
. 49
• 50
• 51
Which of the following organisms did the patient most likely encounter on his last hunting trip?
• 52 :
A Rorrpfiff humrlorfPri

8
Lode.
s
Sus pe-nd
8
End Bloc:k
Item: 43 of 60 - ,• Mark -<J [:::> "'I ~ · ~
QIO: 5264 ~ P~v•ous N@xt Labl lues Notes Calculator

6
27 ~~AI
A 64-year-old man who likes t o hunt and fish in t he north woods of Wisconsin presents t o his intern ist because his wife t old him he started
28 "smiling funny" over t he weekend . He does recall feel ing feveri sh and weak, with a strange red rash surrounding a clear center, after his
29 last hunting trip, which was about 3 weeks ago. A portion of his examination is shown in th e video clip.
30 OPEN MEDIA

31
32
33
34
35
36
37
38
39
40
41
42
. 43
• 44
• 45
• 46
• 47
. 48
. 49
• 50
• 51
Which of the following organisms did the patient most likely encounter on his last hunting trip?
• 52 :
A Rorrpfiff humrlorfPri

8
Lode.
s
Sus pe-nd
8
End Bloc:k
Item: 43 of 60 ~ ,• Mark <::J [:::> ""I ~· ~'j
QID: 5264 J.. Previous Next LAbfaiUI~S Notes Calculator


27 The correct answer is A. 88°/o chos e this.
28 Physical exam reveals a facial nerve palsy. Facial nerve palsy may ari se as a complication of Lyme disease which is caused by Borrelia
29 burgdorferi. B. burgdorferi is t ransm itt ed by Ixodes t ick bit es. Earl y symptoms include a flu -l ike illness with a charact eri st ic circular rash
known as erythema m igrans. Left unt reated, later symptoms include mening it is and facial nerve palsy.
30 Lyme disease Borrelia burgdorferi Erythema chronicum migrans Facial nerve Meningitis Erythema Facial nerve paralysis Rash Tick Borrelia Ixodes Physical examination
31
B is not correct. 2% chose this.
32
Orientia tsutsugamushi is the causat ive organism of scrub typhus, wh ich produces an acut e illness that includes feve r, cough, headache, and
33 gast roint estinal sympt oms. A more severe version can cause disseminat ed intravascula r coagu lation. Complicat ions include encephalit is, but
34 not a facia l nerve palsy.
Orientia tsutsugamushi Disseminated intravascular coagulation Scrub typhus Typhus Encephalitis Orientia Headache Cough Fever Coagulation
35
36 C is not correct. 6 °/o chos e this .
37 Rickettsia rickettsii causes Rocky Mounta in spotted feve r ( RMSF), anot her t ick-borne illness that causes feve r, headache, and rash . Later
38
neural sequelae of RMSF include part ial paralysis of the lower ext remit ies, movement disorders, and language disorders. facia l nerve palsy,
however, is not as common ly associat ed with RMSF, which also causes a petechia l rash, not erythema m igrans. RMSF is also less common in
39 t he Midwest.
40 Rocky Mountain spotted fever Rickettsia rickettsii Erythema chronicum migrans Petechia Erythema Rickettsia Purpura Sequela Headache Tick-borne disease Facial nerve paralysis Paralysis Rash Spotted fever

41 Fever

42 D is not correct. 2°/o chos e this.


43 Treponema pa/lidum causes syphilis, a sexua lly t ransmitt ed infect ion that manifests wit h characteristic symptoms in a seri es of st ages.
• 44 Neurologic complications of syphilis can occur du ri ng t he second stage, 1- 6 months after the init ial infection and include mening it is and optic
• 45 neurit is. In t he t ertiary st age, usually years after t he pri mary infection, pat ients present wit h a va riety of neurolog ic sympt oms such as Argyll
Robert son pupils, hy perref lexivit y, and tabes dorsalis, degenerat ion of t he dorsal columns of t he spinal cord . Syphilis is not , however,
• 46 common ly associated wit h facial nerve palsy, nor does t he chancre of primary syph ilis appear as eryt hema m igrans.
• 47 Sexually transmitted infection Tabes dorsalis Optic neuritis Treponema pallidum syphilis Erythema chronicum migrans Meningitis Facial nerve Erythema Chancre Spinal cord Argyll Robertson pupil

. 48 Posterior column Infection Neurology Facial nerve paralysis

. 49
E is not correct. 2 °/o chos e this .
• 50 I nfection wit h the vari cella-zost er virus init ia lly produces chicken pox . Following t he init ial infect ion, the virus will remain dormant in the
• 51 t rigeminal gang lia and t he dorsal root gang lia and can be react ivat ed later in life to produce an illness, often wit h a rash in a dermatomal
• 52
dist ri but ion, known as herpes zoster, or sh ing les. Complications of zoster include postherpetic neu ralgia, wit h pain and altered sensation in the
same dermat omal dist ribut ion as the zost er rash. A vari cella-zost er is not widely implicated in facial nerve palsy, bu t is t hought t o be a rare

8
Lock Suspend
s 0
End Block
Item: 43 of 60 ~ ,• Mark <::J [:::> ""I ~· ~'j
QID: 5264 J.. Previous Next LAbfaiUI~S Notes Calculator

27 E is not correct. 2°/o chose this .


28 I nfection wit h the vari cella-zost er virus init ia lly produces chicken pox . Following t he init ial infect ion, the virus will remain dormant in the
29 t rigeminal gang lia and t he dorsal root gang lia and can be react ivat ed later in life to produce an illness, often wit h a rash in a derm atomal
dist ri but ion, known as herpes zoster, or sh ingles. Complications of zoster include postherpetic neuralgia, wit h pain and altered sensat ion in the
30
same derm at omal dist ribut ion as the zost er rash. A vari cella-zost er is not widely implicated in facial nerve palsy, bu t is t houg ht t o be a rare
31 cause.
32 Postherpetic neuralgia Shingles Dorsal root ganglion Chickenpox Varicella zoster virus Trigeminal ganglion Facial nerve Facial nerve paralysis Herpes simplex Virus Ganglion Dermatome (anatomy) Neuralgia

33 Rash Infection Trigeminal nerve

34
35
Bottom Line :
36
Neurologic sequelae of infect ious diseases are not uncommon. One complicat ion of Lyme disease is facial nerve palsy.
37 Lyme disease Facial nerve Facial nerve paralysis Sequela Infection Neurology

38
39
40 141;fil·1i•J
FIRS T AID FACTS
for yea r:[2017 •
41
42
FA17 p 502.2
43
Facial nerve lesions
• 44
• 45 Upper motor neuron Destruction of motor cortex or connection
• 46 lesion between motor cortex and facial nucleus in
• 47 pons - contralateral paralysis of lower muscles
• 48 of facial expression. Forehead is spared due to
COIIIColl<llba<
. 49 its bilateral U IN innervation. tract
• 50
Lower motor neuron Destruction of facia l nucleus or C V II
• 51 (UMN _ _ - - ; . : : :
lesion anywhere along its course - ipsilateral lt!IOft • nucleus
• 52 Ctnlrll
paralysis of upper and lower muscles of fildoO ·~

8
Lock
s
Suspend
0
End Block
Item: 43 of 60 - ,• Mark --<] [::> ""'I ~· 1!';:'1
QIO: 5264 ~ Prev1o u s Next Labf a lu es Notes Calculator

27
28
29 141:fi1 ·1hi for year: 2017 •
,JitST AID ,ACTS
30
31 FA17 p 502.2
32 Facial nerve lesions
33
34
Upper motor neuron Destruction of motor cortex or connection
35
lesion between motor cortex and facial nucleus in
36
pons - contralateral pa ra lysi ~ of IO\\ er muscles
37 of facial expression. Forehead is spared due to
~
38 its bilateral U~h innervation. ~KI

39 lower motor neuron Destruction of facial nucleus or C N II (IJII'I ____-;:::


40 lesion anywhere along its course - ipsilateral lf'.oon. - Lower
41 Centnl
paralysis of upper and lower muscles of IOOID
42
facia I expression , , h)'peracusis, loss of taste
43
sensation to anterior tongue.
• 44
When idiopathic (most common), facia l nerve
• 45
palsy is called Bell palsy. May also be caused CHVII
• 46 ~N iosoon

• 47
by Lyme disease, herpes simplex, herpes fKIII-
potsyl
. 48
zoster (Ramsay Hunt syndrome), sarcoidosis,
. 49
tumors (eg, parotid gland), diabetes mellitus.
• 50 Treatment is corticosteroids, acyclovir. \lost
• 51 patients gradually reco\'er function.
• 52

8
L.odt
s
Su~pl'nd
~
End Block
27
zoster (Ramsay Hunt syndrome), sarcoidosis,
28
29
tumors (eg, parotid gland), diabetes meII it us.
30
Treatment is corticosteroids, acyciO\ ir. lost
31
patients graduall)' reco\'er function.
32
33
FA17 p 142.4
34
35
Lyme disease Caused by Borrelia burgdor(eri, which is A Ke) L} me pie to the E \ CE:
36 transmitted br the Ixodes deer tick (also Facia I nerYe palsy (tr pically bilateral)
37 \'ector for Anaplasma spp. and proto.wa \ rthritis
38 Babesia). Natural reser\'oir is the mouse (and Cardiac block
39 important to tick life cycle). Er) thema migrans
40 Common in northeastern United States. Treatment: doxycycline (lst line); amoxicillin
41 Stage 1-ea rly localized: erythema migrans and ccfuroxime in pregnant women and
42 (typical "bu lls-ere" configuration is children.
43 pathognomonic but not always present),
• 44 Au-1ike symptoms.
• 45
Stage 2-early disseminated: secondary lesions,
• 46
carditis, AV block, facial nerve (13ell) palsy,
• 47
migratory myalgias/transient artluit is.
. 48
Stage 3- late d issem ina ted: encephalopathies,
. 49
chronic arthritis.
• 50
• 51
• 52

8
Lode.
s
Suspe-nd
8
End Bloc:k
Item: 44 of 60
QIO: 1823
- ,•
~
Mark -<J
P~v•ous
[:::>
N@xt Labl
"'I
lues
~ ·
No tes
~
Calcula t o r

6
27 ~~AI
A 28-year-old man who is HIV posit ive comes to t he physician because of worsening sight. He states that his loss of vision developed qu ite
28 rapidly. After a funduscopic examinat ion, the physician suspects that cytomegalovirus (CMV) ret initis is the cause of t his pat ient's vision
29 loss.
30
31 For individuals with CMV retin itis, which of t he following is a late find ing on funduscopic examination?
32 :
33
A. Macular cherry-red spot
34 B. Microaneurysms
35
C. Neovascularization
36
0 . Papilledema
37
38 E. Retinal detachment
39
40
41
42
43
• 44
• 45
• 46
• 47
. 48
. 49
• 50
• 51
• 52

8
Lode.
s
Suspe-nd
8
End Bloc:k
Item: 44 of 60 ~ ,• Mark <::J [:::> ""I ~· ~'j
QID: 18 23 J.. Previous Next LAb faiUI~S Notes Calculator

27 •

28
The correct a nswer is E. 4 0°/o chose this .
CMV ret init is is a common cause of blind ness in pat ient s with AIDS. I t presents with rapidly dim inish ing sight (patients will report loss of
29
cent ral v ision with floaters and blind spots) and, on funduscopic exam, cotton -wool exudates, necrotizing ret init is, peri vascular hemorrhages,
30 and, ult imately, ret inal detachment. CMV retinit is is t reat ed wit h t he ant ivirals gancicl ovir and foscarnet.
31 Retinal detachment Foscarnet Ganciclovir Cytomegalovirus retinitis HIV/AJDS Ophthalmoscopy Visual impairment Cytomegalovirus Retinitis Antiviral drug Human cytomegalovirus Necrosis

32 A is not correct . 15% chose this .


33 Cherry- red spot on t he macula is found in Tay -Sachs disease and Niemann-Pick disease (bot h lysosom al st orage diseases) . The absence of
34 ganglion cells within the fovea allows t he choroid's normal color t o be seen in this area, result ing in v isibility of t he cherry -red spot. Tay-Sachs
is also associated wit h progressive neurodegeneration, and Nieman n-Pick disease is also associat ed with hepatosplenomega ly and neurological
35
deficits.
36 Cherry-red spot Tay-Sachs disease Hepatosplenomegaly Macula of retina Fovea centralis Choroid Retinal ganglion cell Lysosomal storage disease Lysosome

37
B is not correct. 15% chose this .
38 Microaneurysms are t iny, ro und, red spots seen around t he macular area. They are a nonspecif ic finding that can be caused by diabet ic
39 retinopat hy.
40 Diabetic retinopathy Retinopathy Diabetes mellitus

41 C is not co rrect. 16% chose this .


42 Neovascula rizat ion, t he process in which blood vessels prolife rate in an abnormal man n e r~ is not commonly seen in CMV ret init is. It is
43 common, however, in prolife rat ive (late-st age) diabet ic retinopathy, in which retinal hypoxemia results in neovascularization.
Diabetic retinopathy Retinopathy Cytomegalovirus retinitis Diabetes mellitus Blood vessel Cytomegalovirus Neovascularization
44
• 45 D is not correct. 14% c hose this .
• 46 Papilledema (edema of t he optic disc, often caused by increased int racranial pressure) is not a common f inding of CMV retinit is .
Papilledema Intracranial pressure Optic disc Cytomegalovirus retinitis Retinitis Cytomegalovirus
• 47
. 48
. 49 Bottom Line :
• 50 CMV ret init is is a common cause of ra pidly progressing blindness in immunocompromised pat ients, and presents wit h cotton wool exudates,
• 51 perivascular hemorrhage, and necrot izing retinit is on funduscopic examination. Treat wit h ganciclovir and foscarnet.
Foscarnet Ganciclovir Ophthalmoscopy Cytomegalovirus retinitis Immunodeficiency Visual impairment Cytomegalovirus Human cytomegalovirus Retinitis Necrosis
• 52

8
Lock
s
Suspend
0
End Block
Item: 44 of 60 ~ ,• Mark <::J [:::> ""I ~· ~'j
QID: 18 23 J.. Previous Next LAb faiUI~S Notes Calculator

27 • •
FA17 p 160.2
28
29 Herpesviruses Enveloped, OS, and linear viruses
30 VIRUS ROUTE OF TRANSMISSION CLI NICAL SIGNIFICANCE NOTES
31
Herpes Respiratory Gingivostomatitis, keratoconjunctivitis fJ, Most common cause of sporadic
32
simplex secretions, saliva herpes labialis m , herpetic whitlow on finger, encephalitis, can present as altered
33
virus-1 temporal lobe encephalitis, esophagitis, mental status, seizures, and/or
34
erythema mu lti fo rme. aphasia.
35
36 Herpes Sexua I contact, Herpes genital i s ~. neonatal herpes. Latent in sacral ganglia. Vi ral
37 simplex perinatal meningitis more common with
38 virus-2 HSV-2 than with HSV-1.
39 Varicella- Respiratory Varicella-zoster (chickenpox [!], shingles 0}, Latent in dorsal root or trigeminal
40
Zoster virus secretions encephalitis, pneumonia. ganglia; C V1 bra nch
41
(HHV-3) Most common complication of shingles is post- involvement can cause herpes
42
herpetic neuralgia. zoster ophthalmicus.
43
44 Epstein-Barr Respiratory Mononucleosis- fever, hepatosplenomegaly, Infects B cells through CD21.
• 45 virus (HHV-4) secretions, pharyngitis, and lymphadenopathy (especially Atypical lymphocytes on peripheral
• 46 saliva; aka posterior cervical nodes 0 ). Avoid contact sports blood smear [!!-not infected B
• 47 "kissing disease," until resolution due to risk of splenic rupture. cells but reactive cytotoxic T cells.
. 48 (common in Associated with lymphomas (eg, endem ic EE> Monospot test-heterophile
. 49 teens, young Burkitt lymphoma), nasopharyngeal antibodies detected by agglutination
• 50 adults) carcinoma (especially Asian adults), of sheep or horse RBCs.
• 51 lymphoproliferative disease in transplant Use of amoxicillin in mononucleosis
• 52 patients. can cause characteristic •

8
Lock
s
Suspend
0
End Block
28
29
FA17 p 507.3
30
31
Retinal detachment Separation of neurosensory layer of retina (photoreceptor layer with rods and cones) from
32
outermost pigmented epithelium (normally shields excess light, supports retina) - degeneration
33 of photoreceptors - \ision loss. lay be zoto retinal breaks, diabetic traction, inflammatory
34 effusions. Visualized on fundoscop) as crinkling of retinal tissue and changes in vessel
35 direction.
36 Breaks more common in patients" ith high myopia and/or history of head trauma. Often preceded
37 by posterior vitreous detachment ("Aashes" and ''Aoalers") and eventual monocular loss of vision
38 like a "curtain drawn down." Surgical emergenc).
39
40
41
FA17 p 198.1
42
Ganciclovir
43
44 MECHANISM 5'-monophosphate formed by a CM viral kinase. Guanosine analog. Triphosphate formed by
• 45 cellular kinases. Preferentially inhibits viral DNA polymerase.
• 46
CLINICAL USE CMV, especially in immunocompromiscd patients. Valganciclovir, a prodrug of ganciclo,·ir, has
• 47
better oral bioavailability.
. 48
. 49
ADVERSE EFFECTS Bone marrow suppression (leukopenia, neutropenia, thrombocytopenia), renal toxicity. lore toxic
• 50
to host enzymes than acyclo,·ir.
• 51 MECHANISM OF RESISTANCE Mutated viral kinase.
• 52

8
L.odt
s
Su~pl'nd
~
End Block
Item : 4 5 o f 60 - ,• Ma rk -<J [:::> "'I ~ · ~
QIO: 3977 ~ P~v•ous N@xt Labl lues Notes Calcu lator

6
27 ~~AI
A 32-year-old recent male immigrant present s to his primary care physician complaining of diarrhea, abdominal pain, and unintentional
28 weight loss of 2.3 kg (5 lb) over t he past several weeks. His symptoms were initially mild but have persisted intermitt ently. Presently, his
29 stools are more frequent, have a liquid consistency, and are streaked with blood and mucus. On physical examinat ion, he has diffuse
abdominal tenderness to palpation, increased in th e right upper quadrant. Stool is heme posit ive. Basic laborat ory test s show:
30
31 WBC count: 13,000/mm 3
32 Neutrophils: 65%
Lymphocytes: 25%
33
Eosinophils: 3%
34 Hemoglobin: 12 g/dl
35 Hematocrit: 35%
Platelet count: 250,000/mm 3
36
37 The physician is able to arrive at a final diagnosis when the patient's stool sample tests positive for a specific antigen.
38
39 What is the most likely diagnosis?
40 :
41 A . Amebic colitis
42
B. DiveJt iculitis
43
C. Enterocolitis due t o Campylobacter
44
• 45 D. Enterocolitis due t o Yersinia
• 46
E. Salmonellosis
• 47
F. Shigellosis
. 48
. 49 G. Ulcerative colitis
• 50
• 51
• 52

8
Lode.
s
Suspe-nd
8
End Bloc:k
Item : 4 5 o f 60 - ,• Ma rk -<J [:::> "'I ~ · ~
QIO: 3977 ~ P~v•ous N@xt Labl lues Notes Calcu lator

A A
27
28 The correct ans wer is A. 41 °/o chose this.
29 This patient has dysent eri c colitis caused by infection wit h the parasite Entamoeba histolytica. Amebiasis
30
(indicated in the image by the typical f lask-shaped ulcer in the intestinal mucosa) is transmitt ed via t he
fecal-oral route and is prevalent in tropical countries with poor sanitation. It can affect the intest ines and/or
31 liver by invading the po1tal vein and producing hepatic abscesses, which cause right upper quadrant pain.
32 Intestinal manifestations include recurrent episodes of diarrhea containing mucus and/or blood, and
33
abdominal pain. In severe colitis, patients can have fever, ileus, peritonitis, and hemorrhage. Laboratory
diagnosis is achieved by detecting amoebas or E. histolytica antigen in a stool sample or liver abscess
34 aspirate. Though eosinophilia is seen in helminth infections, it is not associated with E. histolytica infection.
35 Ent?I'I"''eba tu"-tOI'"' H 1tf'-s Pentombs Fecal-oral route Dysentery Amoeb1asis Ileus LJver "tb'\Ce\ 0. rrt- Eo • lOP"'' •a Abscess Cohtis Helminthiasis

36 Abdorronal p11n Mucus Mucous membrane Antigen Pa<asmsm Stool test Peptic ulcer SanotatJon I.Jv.,. Gastroontestmal tract Bleedong Quadrant (abdomen)
Entamoeba fever Ulc.,. lnfecbon
Image courtesy of CDC/ Dr. Mae
37
Melvin
38
39 B is not correct. 4°/o chose this.
40 Diverticulitis presents as left lower quadrant pain, fever, and leukocytosis. Bleeding per rectum is a separat e complicat ion of diverticulosis, and
is not a feature of diverticulitis.
41
Diverticulitis O•vert1culos•s Leukocytosis Quadrant (abdomen) Rectum Fever Enema
42
43
C is not correct. 9°/o chose this.
44
Campylobacter jejuni is t he most common bact erial cause of bloody diarrhea in the United States. Epidemics have been associat ed with the
consumption of unpasteurized milk. Infecti on causes loose, watery, or bloody stools; fever; and abdominal pain. Mucus in t he st ool and
45 eosinophilia are not common feat ures of Campylobacter ent erocolitis.
• 46 Campylobacter jeJUnl Campylobacter Diarrhea Mucus Abdominal pain Pasteurization Dysentery Enterocolitis Fever M1lk Feces

• 47 D is not correct. 10°/o chose this.


. 48 Yersinia enterocolitis is in th e differe ntial diagnosis for bloody diarrh ea. It most often affects children ages 1- 4 years and has been associated
. 49 with outbreaks in daycare settings. Although it is possible for other family members t o be affected via fecal-ora l t ransmission, t he pattern of
• 50
symptoms, presence of mucus in the st ool, and recent travel make amebic colit is more likely. Mesent eric adenit is due to yersinia infection may
be difficu lt to distinguish from appendicitis as right lower abdominal pain is common in both diseases .
• 51 Aden.be: Append•c•b., Ola. rhea Abdom~nal pa1n Mucus Fecal-oral route Colitis Differenbal d1agno..-" Ye ~1n1a Mese"''tery Dysentery Feces

• 52
E is not correct. 10°/o chose this .

8
Lode.
s
Sus pe-nd
8
End Bloc:k
Item: 4S of 60 ~ ,• Mark <::J [:::> ""I ~· ~'j
QID: 3977 J.. Previous Next LAbfaiUI~S Notes Calculator

27
D is not correct. 10% chose this .
28 Yersinia enterocolit is is in t he different ial diagnosis fo r bloody diarrhea. It most often affects children ages 1 - 4 yea rs and has been associat ed
29 with out breaks in daycare sett ings. Although it is possible fo r other fami ly members to be affect ed via feca l-ora l t ransmission, the pattern of
30
symptoms, presence of mucus in t he st ool, and recent t rave l make amebic colit is more likely. Mesenteric aden it is due to yersinia infect ion may
be difficult to distinguish f rom append icit is as right lower abdominal pain is common in bot h diseases.
31 Adenitis Appendicitis Diarrhea Abdominal pain Mucus Fecal-oral route Colitis Differential diagnosis Yersinia Mesentery Dysentery Feces

32
E is not correct. 10°/o chos e this.
33
Sa lmonellosis is another cause of bloody diarrhea and abdominal pain . However, t he onset of illness occurs wit hin 48 hours of exposure to the
34 pathogen, as opposed t o 2- 4 weeks' incubation t ime for amebic colit is. I n addit ion, the course is 3- 5 days, not severa l weeks.
35 Salmonellosis Diarrhea Pathogen Colitis Abdominal pain Dysentery Incubation period

36 F is not correct. 18% chos e this .


37 Shigellosis can be clinically sim ilar to amebic colit is, as bot h involve diarrhea containing blood and m ucus. Howeve r~ onset of symptoms is
38 abrupt, not gradua l, and tenesmus is a prominent featu re of shigellosis, not amebic colit is.
Shigellosis Rectal tenesmus Diarrhea Colitis Mucus
39
40 G is not correct. 8 % chos e this .
41 Ulcerative colit is could present in the manner that t he pat ient descri bed . However, t he diagnosis of ulcerat ive colit is is made by colonoscopic
evidence in t he cont ext of negative stool stud ies. The fact t hat t he pat ient's stool was positive fo r a specif ic ant igen support s t he diagnosis of
42
amebic colit is over ulcerat ive colit is.
43 Ulcerative colitis Colonoscopy Colitis Antigen

44
45
Bottom Line:
• 46
Entamoeba histolytica causes amebic colitis, which man ifests wit h dia rrhea, abdominal pain, and weight loss, and is confirmed by det ecting
• 47
t he organ ism or its ant igen . I t is common in t ropical countri es wit h poor sanitation.
• 48 Entamoeba histolytica Diarrhea Abdominal pain Colitis Antigen Sanitation Entamoeba Weight loss Organism

. 49
• 50
• 51 141;fil·1i•J for year:[2017 •
FIRST AID FA CTS
• 52

8
Lock
s
Suspend
0
End Block
Item: 4S of 60 ~ ,• Mark <::J [:::> ""I ~· ~'j
QID: 3977 J.. Previous Next LAbfaiUI~S Notes Calculator

27 • FA17 p 151 .1 •

28
Protozoa- gastrointestinal infections
29
ORGAN ISM DISEASE TRANSMISSION DIAGNOSIS TREATMENT
30
31
Giardia Iamblia Giardiasis- bloating, flatulence, Cysts in water Multinucleated Metronidazole
32 fou l-smelling, fa tly diarrhea trophozoites t'J or
33 (often seen in campers/hikers)- cysts I]) in stooI,
34 think fat-rich Ghirardelli antigen detection
35 chocolates for fatty stools of
36 Giardia
37 Entamoeba Cysts in water Metronidazole;
Amebiasis- bloody diarrhea Serology and/or
38
histolytica (dysentery), liver abscess trophozoites (with paromomycm or
39
("anchovy paste" exudate), engulfed RBCs [i iodoguinol for
40
RUQ pain; histolog)' shows in t·he cytoplasm) asymptomatic cyst
41
Aask-shaped ulcer or cysts with up to passers
42
4 nuclei in stool I!];
43
44
Entamoeba Eats
45 Erythrocytes,
• 46 antigen detection
• 47
. 48 Cryptosporidium Se,·ere diarrhea in AIDS Oocrsts in water Oocysts on acid-fast Pre\·ention (by
. 49 Mild disease (watery diarrhea) in stain 1), antigen fi ltering city
• 50 water supplies);
immunocompetent hosts detection
• 51
nitazoxan icle in
• 52
immunocompetent
8
Lock
s
Suspend
0
End Block
Item: 45 of 60 - ,• Mark -<J [:::> .il ~ · ~
QIO: 3977 ~ P~v•ous N@xt LabValu es No tes Calcula t o r

27
A • A

28
29
30
31
32
33
34
a a L____ ~
a
35
36
37 FA17 p 175.1
38 Bugs causing diarrhea
39 Bloody diarrhea
40
Campylobacter Comma- or S-shaped organisms; growth at 42°C
41
42 Ehistolytica Protozoan; amebic dysentery; liver abscess
43 Enterohemorrhagic 0157:117; can cause liUS; makes Shiga-like toxin
44 Ecoli
45
• 46
Enteroinvasive Ecoli lnvades colonic mucosa
• 47 Salmonella (non- I .actose 8; Aagellar motility; has animal reservoir, especially poultry and eggs
. 48 typhoidal)
. 49
Shigella Lactose 8; ,·ery low 1050; produces Shiga toxin (human resen·oir onlr); bacillary dysentery
• 50
• 51 Yenterocolitica Day care outbreaks, pseudoappendicitis
• 52 Watery diarrhea
---------
8
Lode.
s
Suspe-nd
8
End Bloc:k
Item: 45 of 60 - ,• Mark -<J [:::> "'I ~ · ~
QIO: 3977 ~ P~v•ous N@xt Labl lu es No tes Calcula t o r

27
A
Watery diarrhea A

----
28 c difficile Pseudomembranous colitis; associated with antibiotics and PPls; occasionallr bloody diarrhea
29
Cperfringens Also causes gas gangrene
30
31 Enterotoxigenic Ecoli Tmelers' diarrhea; produces heal-labile (LT) and heal-stable (ST) toxins
32 Protozoa Giardia, Cryptosporidium
33
Vcholerae Comma-shaped organisms; rice-water diarrhea; often from infected seafood
34
35 Viruses Rota,·irus. noro,·irus, adenO\·irus
36
37
FA17 p 365.1
38
Inflammatory bowel disease
39
40 Crohn disease Ulcerative colitis
41 LOCATION Anr portion of the GI tract, usually the termina l Colitis = colon inAammation. Continuous
42
ileum and colon. Skip lesions, rectal sparing. colon ic lesions, always with recta l involvemen t.
43
44
GROSS MORPHOLOGY Transmural inAammation - fistu las. Mucosal and submucosal inAammation only.
45
Cobblestone mucosa, creeping fat , bowel wa ll Friable mucosa with superficial and/or
• 46 th ickeni ng ("string sign" on bariu m swallow deep ulcerations (compare normal with rn
• 47 x-ray r.i.l), linear ulcers, fissures . diseased 1!11). Loss of haustra- "lead pipe"
. .
. 48 appearance on 1magmg.
. 49 MICROSCOPICMORPHOLOGY loncaseating granulomas and lymphoid Crypt abscesses and ulcers, bleeding, no
• 50
aggregates. Th I mediated . granulomas. Th2 mediated.
• 51
COMPLICATIONS Malabsorption/malnutrition, colorectal cancer (t risk with pancolitis).
• 52
~· a- I I ......... , ........... ,..,.,· .... 1 C • I I ' I L-. I • .... _,. __ ,_... r.... ••
8
Lode.
s
Suspe-nd
8
End Bloc:k
Item: 45 of 60 - ,• Mark -<J [:::> "'I ~ · ~
QIO: 3977 ~ P~v•ous N@xt Labl lues No tes Calcula t o r

27
. . g g
28
gallstones. Mar be Ef> for anti-Sacclwromrces p-A1 CA.
29
cervisiae antibodies (ASCA).
30 TREATMENT Corticosteroids, azalh ioprine, ani ibiol ics (eg, 5-aminosalicylic preparations (eg, mesalamine),
31 ciproAoxacin, metronidazole), inAiximab, 6-mercaptopurine, inAiximab, colectomy.
32 adalimumab.
33
For Crohn. think of a fat granny and an old Ulcerati,·e colitis causes ULCCCERS:
34
crone l>kipping down a cobble\tone road away Ulcers
35
from the \Heck (rectal sparing). Large intestine
36
37
Continuous, Colorectal carcinoma, C rypl
38
abscesses
39 l•.xtends proximally
40 Red diarrhea
41 Sclerosing cholangitis
42
43
44
45
• 46
• 47
. 48
. 49
• 50
• 51
• 52

8
Lode.
s
Suspe-nd
8
End Bloc:k
Item: 46 of 60 - ,• Mark -<J [:::> "'I ~ · ~
QIO: 2 184 ~ P~v•ous N@xt Labl lues No tes Calcula t o r

6
27 ~~AI
A 50-year-old man begins to have profuse, nonbloody, watery diarrhea 2 days after arriving in Bangladesh to serve as an aid worker. A
28 stool sample looks like wat er after rice has been cooked in it , and a smear shows no WBCs. His mucous membranes are dry, and his skin
29 shows signs of tenting. Subsequent ly, elect rolyt e abnormalit ies develop that lead to cardiac and renal fai lure.
30
31 What is the mechanism of action of the exotoxin produced by the most likely causative organism?
32 :
33
A. Cleaves host cell rRNA
34 B. Directly acts as an adenylate cyclase
35
C. Inactivates elongation factor 2
36
D. Permanently activates G5
37
38 E. Permanently inactivates G;
39
40
41
42
43
44
45
• 46
• 47
. 48
. 49
• 50
• 51
• 52

8
Lode.
s
Suspe-nd
8
End Bloc:k
Item: 46 of 60 ~ ,• Mark <::J [:::> ""I ~· ~'j
QID: 2184 J.. Previous Next LAb faiUI~S Notes Calculator

27 •

28 The correct answer is 0. 64°/o chose this.


29
This patient most likely has cholera, caused by t he bacterium Vibrio cho/erae, which is ma rked by watery st ools (often called " rice-wat er"
stool) . This illness is not accompanied by abdominal pain. Cholera t oxin ent ers t he cell and constitu t ively activates the Gs protein t hrough ADP
30 ri bosylation . This results in a high cAMP level, wh ich activates the CFTR (cyst ic f ibrosis t ransmemb rane conductance regulat or) channel,
31 lead ing t o a large efflux of chlori ne and other ions int o t he gast roint estinal lumen . The result is ext remely wat ery dia rrhea accompanied by
32
electrolyte imbalances.
Cystic fibrosis Cystic fibrosis transmembrane conductance regulator Vibrio cholerae Cholera toxin Cholera Electrolyte Cyclic adenosine monophosphate Chlorine Diarrhea Protein Bacteria Lumen (anatomy)
33
Adenosine diphosphate Toxin Fibrosis Abdominal pain Vibrio Human gastrointestinal tract Gastrointestinal tract Feces Transmembrane protein
34
35 A is not correct. 3°/o chose this.
This is a charact eristic of Shiga t oxin, which typically causes bloody diarrhea (eg, dysent ery) .
36
Shiga toxin Dysentery Diarrhea Toxin Enterohemorrhagic
37
38 B is not correct. 19% chose this .
Bacillus anthracis produce a toxin that acts as an adenylate cyclase (edema fact or) but is not associated wit h severe watery diarrhea .
39
Bacillus anthracis Adenylyl cyclase Diarrhea Bacillus Toxin
40
41
C is not correct. 6 °/o chose this .
Dipht heria t oxin and exotoxin A from Pseudomonas inact ivate elongation factor 2, but neither is a likely cause of wat ery diarrhea.
42 Diphtheria Exotoxin Diphtheria toxin Pseudomonas exotoxin Diarrhea Elongation factor EEF2 Toxin Pseudomonas
43
E is not correct. 8 °/o chos e this.
44
Bordetel/a pertussis produces an exotoxin that increases the cAMP level by inactivat ing t he inhibit ory G;. However, pertussis causes whooping
45 cough, not severe watery diarrhea .
46 Bordetella pertussis Pertussis Exotoxin Bordetella Cyclic adenosine monophosphate Diarrhea Cough

• 47
. 48
Bottom Line:
. 49
Vibrio cholerae produces an exotoxin t hat permanent ly activates Gs protein, result ing in an excess of cAMP, which leads t o a severe
• 50 nonbloody diarrhea .
• 51 Vibrio cholerae Exotoxin Cyclic adenosine monophosphate Diarrhea Vibrio Protein

• 52

8
Lock
s
Suspend
0
End Block
Item: 46 of 60 ~ ,• Mark <::J [:::> ""I ~· ~'j
QID: 2184 J.. Previous Next faiUI~S
LAb Notes Calculator

27 FA17 p 128.1
28 Bugs with exotox ins
29
BACTERIA TOXIN MECHANISM MANIFESTATI ON
30
Inhibit protein synthesis
31
32 Corynebacterium Diphtheria toxin3 Pharyngitis with pseuclomembranes in th roat
33 diphtheriae Inactivate elongation factor and severe lymphadenopathy (bull neck)
34 Exotoxin N (EF-2)
Pseudomonas Host cell death
35
aeruginosa
36
37
Shigella spp. Shiga toxin (ST)3 GI mucosal damage ..... dysentery; ST also
38 Inactivate 60S ribosome by enhances cytokinc release, causi ng hemolytic-
39 removing adenine from uremic syndrome (HUS)
40 Enterohemorrhagic Shiga-like toxin rR A SLT en hances cytokine release, causing HUS
41 E coli (EHEC) (SLT)a (prototypically in EHEC serotype 0157:H7).
42 Unlike Shigella, EHEC does not invade host
43
cells
44
45
Increase flu id secretion
46 Enterotoxigenic Heat-labile Overactivates adenylate Watery diarrhea: "labile in the Air (Aclenylate
• 47 E coli (ETEC) toxin (LT)3 cyclase (t cAMP) ..... t CI- cyclase), stable on the Ground (Guanylate
• 48 secretion in gut and H7 0 cyclase)"
. 49 efAux
• 50
I Ieat-stable Overactivates guanylate
• 51
toxin (ST ) cyclase (t cGMP)
• 52
..... l resorption of NaCI
8
Lock
s
Suspend
0
End Block
Item: 46 of 60 ~ ,• Mark <::J [:::> ""I ~· ~'j
QID: 2184 J.. Previous Next LAbfaiUI~S Notes Calculator

27 •
FA17 p 175.1
28
Bugs causing diarrhea
29
30 Bloody diarrhea
31 Campylobacter Comma- or S-shaped organisms; growth at 42°C
32
E histolytica Protozoan; amebic dysentery; liver abscess
33
34 Enterohemorrhagic 0157:H7; can cause llUS; makes Shiga-like toxin
35 Ecoli
36 Enteroinvasive Ecoli Invades colonic mucosa
37
Salmonella (non- Lactose 8; Aagellar motility; has animal reservoir, especially poultry and eggs
38
typhoidal)
39
40 Shigella Lactose 8; very low 10 ;0; produces Shiga toxin (human reservoir only); bacillary dysentery
41
Yenterocolitica Day care outbreaks, pseudoappendicitis
42
43
Watery diarrhea
44 Cdifficile Pseudomembranous colitis; associated with antibiotics and PPis; occasionally bloody diarrhea
45
C perfringens Also causes gas gangrene
46
• 47 Enterotoxigenic E coli Tra,·elers' diarrhea; produces heat-labile (LT) and heat-stable (ST) toxins
. 48 Protozoa Giardia, Cryptosporidium
. 49
Vcholerae Comma-shaped organisms; rice-water diarrhea; often from infected seafood
• 50
• 51 Viruses Rota,·irus, norovirus, adenovirus
• 52

8
Lock
s
Suspend
0
End Block
Item: 47 of 60 - ,• Mark -<J [:::> "'I ~ · ~
QIO: 2039 ~ P~v•ous N@xt Labl lues No tes Calcula t o r

27 A A 56-year-old man presents to his physician's office complaining of nausea, bloating, and int ermittent dull and gnawing epigastri c pain
28 after meals over the past 3 months. He does not have any fatigue or weight loss. Fecal occult blood t est was posit ive. A biopsy of the
gastric mucosa is obtained and shown in t he •m,•no
29
30
31
32
33
34
35
36
37
38
39
40
41
42
43
44
Which of the fol lowing statements is true?
45
46 :

• 47
. 48 A. Bismuth salts such as Pept o- Bismol alone are often sufficient for erad ication of t his inf ect ion
. 49
B. Decreasing the pH in t he st omach by blockin g stomach acid production helps eradicate t his infect ion
• 50
C. This infection is a risk factor for diffuse gastric adenocarcinoma characterized by thickening of t he stomach wall (linitis plastica) and
• 51
signet ring cells
• 52
D. This infection is a risk factor for intestinal gastric adenocarcinoma and B-cell non-Hodgkin gast ric lymphoma

8
Lode.
s
Suspe-nd
8
End Bloc:k
Item: 47 of 60 - ,• Mark --<] [::> ""'I ~· 1!';:'1
QIO: 2039 ~ Prev1ous Next Labfa lues Notes Calculator

27
g I I I

28
29
30
31
32
33
34
35
36
37
38
39
40 •
Image coUttesy of £:d Uthman, /110
41
42
43 Which of the fol lowing statement s is t rue?
44 :
45
46 A . Bismuth salts such as Pepto- Bismol alone are often su ff icient for eradication of t his infect ion
• 47
B. Decreasing the pH in t he stomach by blocking st omach acid production helps eradicat e t his infect ion
. 48
. 49 C. This infection is a risk factor fo r diffuse gastric adenocarcinoma characterized by t hickening of the st omach wall (linitis plastica) and
signet ring cells
' 50
D. This infection is a risk factor for intestinal gast ric adenocarcinoma and B-cell non-Hodgkin gastric lymphoma
' 51
' 52 E. This infection is more likely to be found in gastric ulcers than duodenal ulcers

8
L.odt
s
Su~pl'nd
~
End Block
Item: 47 of 60 ~ ,• Mark <::J [:::> ""I ~· ~'j
QID: 2039 J.. Previous Next LAbfaiUI~S Notes Calculator

27 •

28 The correct a nswer is 0. 60°/o chose t his.


29 The image shows bact eri a colonizing t he gast ric mucosa (note t he dozens of curved bacteria fil ling t he lumen), which indicates Helicobacter
pylori infection. H. pylori is an organism t hat makes urease, wh ich creat es a more suitable alkaline env ironment for its surviva l in t he acidic
30
environment of the st omach. A 3- t o 5-fo ld increased risk of intest ina l gast ric carcinoma and a 6- to 50-fo ld increased risk of B-cell non-
31 Hodgkin gast ric lym phoma (also known as MALT lymphoma) have been associat ed with H. pylori infect ion. I nterestingly, a growing body of
32 evidence has shown t hat colonizat ion wit h H. pylori prot ects aga inst adenocarcinoma of t he esophagus.
Helicobacter pylori Urease MALT lymphoma Gastric lymphoma Adenocarcinoma Stomach cancer Bacteria Esophagus Gastric mucosa B cell Lumen (anatomy) Mucous membrane Lymphoma Helicobacter
33
Organism Carcinoma Alkalinity Infection
34
35 A is not correct . 2°/o chose t his.
36 Treatment of peptic ulcer disease requires t riple t herapy. This includes combinat ions of bismut h subsalicylate and omeprazole to protect t he
gast ric mucosa and met ron idazole and tetracycline or amoxicillin to erad icate t he Helicobacter pylori infection .
37
Peptic ulcer Metronidazole Omeprazole Amoxicillin Tetracycline Helicobacter pylori Bismuth subsalicylate Bismuth Mucous membrane Ulcer Helicobacter Infection Ulcer (dermatology)
38
39
B is not correct. 9 % chose t his .
Triple therapy is t he commonly ut ilized reg imen for eradication of Helicobacter pylori and t reatment of it s associated gast ri t is. Triple t herapy
40
usua lly consists of a proton pump inhibi tor and t wo ant ibiot ics, such as met ronidazole, azit hromycin, cla rit hromycin, or amoxicillin . Stu dies
41 have shown t hat by increasing t he pH t o lower acidit y (not decreasing as stated in t his answer choice), PPis help to both heal the gastric
42 lining and eliminat e t he H. pylori infection.
Proton-pump inhibitor Metronidazole Azithromycin Clarithromycin Helicobacter pylori Amoxicillin Gastritis PH Acid Antibiotics Proton Enzyme inhibitor Proton pump Helicobacter Infection
43
44 C is not co rrect. 1 7% chose this .
45 Helicobacter pylori infect ion is associated wit h int est inal gastric adenocarcinoma, not t he diff use t ype. Int estinal gast ri c adenocarcinoma
resembles an ulcer with raised ma rgins and usually arises fo llowing H. pylori infection. Diffuse gastri c adenocarcinoma is usually the result of
46
biallelic inactivation of E-cadheri n and does not seem t o be associat ed with H. pylori infect ion .
47 Allele Helicobacter pylori CDHl (gene) Stomach cancer Peptic ulcer Adenocarcinoma Ulcer Infection

. 48
E is not correct. 1 2°/o chose this .
. 49
Helicobacter pylori is found in 90% of duodenal ulcers and 70% of gast ric ulcers.
• 50 Helicobacter pylori Peptic ulcer Duodenum Ulcer Helicobacter Equine gastric ulcer syndrome Ulcer (dermatology)

• 51
• 52
Bottom Line :
8
Lock
s
Suspend
0
End Block
Item: 47 of 60 ~ ,• Mark <::J [:::> ""I ~· ~'j
QID: 2039 J.. Previous Next LAbfaiUI~S Notes Calculator

27
Bottom Line:
28
Helicobacter pylori infection is associated wit h a 3- Sx increased risk of gast ric carcinoma and a 6- SOx increased risk of MALT lymphoma . It
29 is also high ly associated wit h duodenal and gast ric ulcers.
30 Helicobacter pylori MALT lymphoma Stomach cancer Peptic ulcer Duodenum Lymphoma Ulcer Helicobacter Carcinoma Infection

31
32
33 l@l;fil·1i•l for year:[ 2017
FIRST AID FACTS .
•j .
34
35
FA17 p 142.1
36
37
Helicobacter pylori Curved, flagellatee! (motile), gram 8 roc! fJ that is triple EB: catalase EB, oxidase EB, and urease EB
38 (can use urea breath test or feca l antigen test for diagnosis). Urease produces ammonia, creating
39 an alkaline environment, which helps I-l pylori survive in acidic mucosa. Colonizes mainly
40 antrum of stomach; causes gastritis and peptic ulcers (especially duodenal). Risk factor for peptic
41 ulcer disease, gastric adenocarcinoma, and MALT lymphoma.
42 Most common initial treatment is triple therapy: Amoxicillin (metronidazole if penicillin allergy)
43 + Clarithromycin + Proton pump inhibitor; Antibiotics C ure Pylori.
44
45
46
47 FA17 p 219.1
• 48
. 49
Oncogenic microbes Microbe Associated cancer
• 50 EBV Burkitt lymphoma, Hodgkin lymphoma,
• 51 nasopharyngeal carcinoma, 1° C lS
• 52 lymphoma (in immunocompromised pat ienls)

8
Lock
s
Suspend
0
End Block
Item: 47 of 60 - ,• Mark -<J [:::> "'I ~ · ~
QIO: 2039 ~ P~v•ous N@xt Labl lu es No tes Calcula t o r

A A
27 FA17p 219.1
28
Oncogenic microbes Microbe Associated cancer
29
30 EB Burkitt lymphoma, Hodgkin I) mphoma,
31 nasopharyngeal carcinoma, 1° CNS
32 lymphoma (in immunocompromised patients)
33 HBV. HC llepatocellular carcinoma
34
IIII V-8 Kaposi sarcoma
35
36 HPV Cen ical and penile/anal carcinoma (types 16,
37 18), head and neck cancer
38 11 pylori Cast ric adenocarcinoma and IAL'T I) mphoma
39
40
HTLV-1 dult T-cell leukemia/lrmphoma
41 Liver Auke (Clonorchis sinensis) Cholangiocarcinoma
42 Schistosoma haematobium Bladder cancer (squamous cell)
43
44
45 FA17 p 362.3
46
Gastric cancer Most commonly gastric adenocarcinoma; Virchow node-involvement of left
47
lymphoma, C l stromal tumor, carcinoid (rare). supraclavicu lar node by metastasis from
. 48
Early aggressive local spread with node/liver stomach.
. 49
metastases. Often presents late, with weight Krukenberg tumor-bilateralmetastases to
• 50
. and in some cases acanthosis
loss, earh-' satietY, O\·arics. Abundant mucin-secreting, signet ring
• 51
• 52
nigricans or Leser-Trclat sign. cells.
• Intestinal- associated with H bvlori dietarv Sister Marv Joseoh nodule- subcutaneous
8
Lode.
s
Suspe-nd
8
End Bloc:k
Item: 47 of 60 - ,• Mark --<] [::> ""'I ~· 1!';:'1
~
QIO: 2039 Prev1o u s Next Labf a lu es Notes Calculator
. .
27
28
II p)'lori Castric adenocarcinoma and i\ IALT lympho111a
29 HTLV-1 Adult T-cell leukemiallymphoma
30
Liver Auke (Clonorchis sinensis) Cholangiocarcinoma
31
32
Schistosoma haematobium Bladder cancer (squamous cell)
33
34
FA17 p 362.3
35
36
Gastric cancer Most commonly gastric adenocarcinoma; Virchow node-im·oh-ement of left
37 I} mphoma, C l stromal tumor, carcinoid (rare). supracla' icular node by metastasis from
38 Early aggressi,·e local spread with node/li\'er stomach.
39 metastases. Often presents late, with weight Krukenberg tumor-bilateral metastases to
40 loss, early satiety, and in some cases acanthosis ovaries. Abundant mucin-secreting, signet ring
41 nigricans or Leser-Trclat sign. cells.
42 Intestinal - associated with H P>•lori, dietary Sister Mary Joseph nodule- subcutaneous
43 nitrosamines (smoked foods), tobacco perium bi Iica I metastasis.
44 smoking, achlorhydria, chronic gastritis.
45 Common ly on lesser curvature; looks like
46
ulcer with raised margins.
47
• Diffuse -not associated with II pylori; signet
. 48
ring cells (mucin-filled cells with peripheral
. 49
nuclei) fJ; stomach \\'all grossly thickened
' 50
and leathery (linitis plastica).
' 51
' 52

8
L.odt
s
Su~pl'nd
~
End Block
Item: 48 of 60 - ,• Mark -<J [:::> "'I ~ · ~
QIO: 3892 ~ P~v•ous N@xt Labl lues No tes Calcula t o r

6
27 ~~AI
A microbiology laborat ory is investi gating the possibility of using enzymes as a novel class of antibact eria l. These enzymes cannot traverse
28 the double- layer lipid membrane. I nstead, they bind directly to the surface peptidog lycans and hydrolyze bonds within t hem.
29
30 Which of the following organisms would be a logical target for this type of drug?
31 :
32 A. Brucella abortis
33 B. Chlamydia trachoma
34
C. Escherichia coli
35
36 0 . Staphylococcus aureus
37 E. Ureaplasma urealyticum
38
39
40
41
42
43
44
45
46
47
. 48
. 49
• 50
• 51
• 52

8
Lode.
s
Suspe-nd
8
End Bloc:k
Item: 48 of 60 ~ ,• Mark <::J [:::> ""I ~· ~'j
QID: 3892 J.. Previous Next LAb faiUI~S Notes Calculator

27 •
The correct answer is 0. 65°/o chose this.
28
Kn owledge of t hese enzymes is not required t o answer t he question . It is sim pl y test ing for a basic underst anding of bact erial cell structure.
29 The quest ion stat es t hat the enzymes are able to bind surface peptidoglycans directly from the extracellula r space. Pept idoglycans make up
30 bact eri al cell walls. I n other words, the question is merely asking : "In what type of bact eria is the cell wall the outermost st ructu re?" The
answer is the gram- posit ive bacteria, wh ich have a thick pept idoglycan cell wall . Of t he answer choices, t he only gram-posit ive bact erium is
31
Staphylococcus au reus.
32 Peptidoglycan Cell wall Gram-positive bacteria Staphylococcus aureus Bacteria Staphylococcus Enzyme Extracellular

33
A is not correct. 4°/o chose this.
34
Brucella abo1tis is a gram- negat ive bact erium .
35 Gram-negative bacteria Brucella Bacteria

36
B is not correct. 8% chose this.
37
Chlamydia trachoma is an int racellular pat hogen, and would be inaccessible to enzymes in the ext racellular space.
38 Trachoma Chlamydia infection Intracellular Extracellular Pathogen Intracellular parasite

39
C is not correct. 16% chos e this.
40 Escherichia coli is a gram-negat ive organism. Thus it has an outer membrane surrounding a t hin peptidog lycan layer. The cell wall would not
41 be direct ly accessible to an enzyme in the extracellula r space.
Escherichia coli Peptidoglycan Gram-negative bacteria Enzyme Cell wall Bacterial outer membrane Extracellular Mitochondrion Organism Biological membrane
42
43 E is not correct. 6°/o chose this.
44 Ureaplasma urealyticum lacks a cell wall ent irely.
Ureaplasma urealyticum Cell wall Ureaplasma
45
46
47 Bottom Line:
48 For a hypot hetical antibiotic t hat hydrolyzes surface peptidoglycans, gram -posit ive bacteria would be most susceptible since their outermost
. 49 structure is a t hick peptidog lycan cell wall. Intracellula r bacteria, bacteria wit hout cell walls, or gram -negative bacteria, whose thin
• 50 peptidog lycan cell wall is enclosed by an outer lipopolysacchari de membrane, would be less susceptible to this mechan ism of act ion .
Lipopolysaccharide Peptidoglycan Cell wall Gram-positive bacteria Antibiotics Gram-negative bacteria Bacteria Hydrolysis Mechanism of action Intracellular Biological membrane
• 51
• 52

8
Lock
s
Suspend
0
End Block
Item: 48 of 60 - ,• Mark --<] [::> ""'I ~· 1!';:'1
QIO: 389 2 ~ Prev1o u s Next Labf a lu es Note s Calculator

27 FA17 p 120.2
28
Cell walls
29
30 Unique to Common to both Unique to
gram (!;) gram e
31
- - - - Flagellum - - - - . : -..'1 (
.. )
32
33
u-~------ Pilus - - - - 4
--__- Clpsule--~~~~~~f~~~~&~
34
35
36 Endotoxin/LPS Outer
37 jS5~~- Porin membrane
38
39
40 CytoplasmiC
41 ~~)OI;)O;)OOOOQ~~~~~--
. membrane -~~~0000006!l~~~~~
42 Gram $ Gram e
43
44
FA1 7 p 131 .3
45
46 Staphylococcus aureus Cram <±>, ~-hemolyti c, catalase <±>, coagulase TSST-1 is a superantigen that binds to MHC
47 <±>cocci in clusters fl. Protein A (virulence II and T-ccll receptor, resulting in polyclonal
48 factor} binds Fc-lgC, inhibiting complement T-cell activation.
0
49 activation and phagoc) tosis. Commonly Staphylococca l toxic shock syndrome
• 50 colonizes the nares, axilla, and groin . (TSS) presents as fever, vomiting, rash,
• 51 Causes: desquamation, shock, end-organ failure. TSS
• 52 Inflammatory disease-skin infections, results in t AST, t ALT, t bilirubin. Associated •
8
L.odt
s
Su~pl'nd
~
End Block
Item: 48 of 60 - ,• Mark -<J [:::> "'I ~ · ~
QIO: 3892 ~ P~v•ous N@xt Labl lues No tes Calcula t o r

~~--~~-- -------------~----
27 Gram$ Gram e
28
29
FA17 p 131.3
30
31 Staphylococcus au reus Gram EB, ~-hemolytic, catalase <:!:>, coagulase TSST-1 is a superantigen that binds to MIIC
32 <:!:> cocci in clusters rJ. Protein A (' irulence II and T-cell receptor, resulting in polyclonal
33
• factor) binds Fc-lgG, inhibiting complement T-cell acti,·ation.
34 acti,·ation and phagocytosis. Commonly Staphrlococcal toxic shock syndrome
35 colonizes the nares, axilla, and groin. (TSS) presents as fe,•er, vomiting, rash,
36 Causes: desquamation, shock, end-organ failure . T S
37
• Inflammatory disease-skin infections, results in t AST, t ALT, t bilimbin. ssociated
38
39
J•..'\ organ abscesses, pneumonia (often after
influenza virus infection), endocarditis,
"ith prolonged use of \'aginaltampons or nasal
packing.
40
septic arthritis, and osteomyelit is. Compare'' ith Streptococcus pyogenes TSS (a
41
1oxin-mediated disease-toxic shock toxic shock-like syndrome associated with
42
syndrome (TSST-1), scalded skin syndrome p<tinful skin infection).
43
(exfoliative toxin), rapid-onset food S aureus food poisoning due to ingestion of
44
45
poisoning (enterotoxins). preformed toxin - short incubation period
46
1RSA (methicillin-resistantS aureus) (2-6 hr) followed by nonbloody diarrhea
47
infection-important cause of seri011S and emesis. Enterotoxin is heat stable - not
48 nosocomial and community-acquired destroyed by cooking.
. 49 infections; resistant to methicillin and Bad staph (aureus) make coagulase and toxins.
• 50 nafcillin because of altered penicillin- Forms fibrin clot around self - abscess.
• 51 binding protein .
• 52

8
Lode.
s
Suspe-nd
8
End Bloc:k
Item: 49 of 60 - ,• Mark -<J [:::> "'I ~ · ~
QIO: 203 1 ~ P~v•ous N@xt Labl lues No tes Calcula t o r

6
27 ~~AI
A 22-year-old man present s to t he emergency department com plaining of a spreading rash aft er several days of high fevers, severe
28 headaches, and myalgia. He had recentl y return ed from a Peace Corps trip to Rwanda. He first not iced t he rash on his chest, but it has now
29 spread to his legs and arms. His t emperature is 38°C ( 100.4° F), pulse is 88/min, resp iratory rate is 20/min, and blood pressure is 125/87
mm Hg. Physical examination revea ls erythematous maculopapular eruptions on his chest, fo rearms, and thighs. A Weii-Felix reaction test is
30
positive for OX-19, but negative for OX- 2 and OX- K.
31
32 Which of the following is the most appropriat e treatment?
33
:
34 A. Ceftriaxone
35
B. Doxycycline
36
37 C. Penicillin
38 D . Supportive care
39
E. Vancomycin
40
41
42
43
44
45
46
47
48
0
49
0 50
0 51
0
52

8
Lode.
s
Suspe-nd
8
End Bloc:k
Item: 49 of 60 ~ ,• Mark <::J [:::> ""I ~· ~'j
QID: 2031 J.. Previous Next LAb faiUI~S Notes Calculator

27 •

28
The correct a nswer is B. 64 % chose t his .
29
This pat ient present s with t he cla ssic t ri ad of ri ckettsial illness (headache, fever, and rash) caused in t his case by the gram -negative organism
30 Rickettsia prowazekii (epidemic t yphus) . The rash of ty phus differs from t hat of Rocky Mountain spott ed fever because it starts on t he t runk
31 and moves outward, rather than st arting on t he ext remit ies and moving inward . Nevertheless, the treat ment of choice for all rickettsial
illnesses is doxycycline. Though immunoflourescence an t ibody test ing is preferred for diagnosis, Weii-Felix reactions are also posit ive in
32
ri ckettsial diseases, and so can be used t o help differentiat e them from ot her t ick-born e diseases. The Weii- Felix react ion is a nonspecif ic
33 agg lut ination t est that detects ant irickettsial an t ibodies in a pat ient's serum.
34 Rocky Mountain spotted fever Rickettsia prowazekii Doxycycline Rickettsia Typhus Gram-negative bacteria Headache Antibody Organism Tick-borne disease Spotted fever Rash Fever Agglutination (biology)

35 A is not correct . 10% chose this .


36 Ceftriaxone is a t hird-generat ion cephalosporin used most commonly to t reat Neisseria gonorrhoeae infection.
Cephalosporin Ceftriaxone Neisseria gonorrhoeae Neisseria Infection
37
38 C is not correct. 8°/o chose this .
39 Penicillin, an inhibit or of cell wall synthesis, is t he t reat ment of choice for syphilis, caused by the spirochet e Treponema pa/lidum.
Spirochaete Treponema pallidum Penicillin Syphilis Cell wall Treponema Enzyme inhibitor
40
41 D is not correct. 14% c hose this.
42 Supportive care would be appropriat e for viral infections. However, t he Weii- Felix react ion confirms t he bacteria l origin, and ant ibiot ic
43 t reatment should be init iated early.
Antibiotics
44
45 E is not correct. 4 °/o chose this .
46 Vancomycin works by inhibit ing cell wall synthesis, but is effective only against gra m -posit ive organisms.
vancomycin Cell wall Gram-positive bacteria
47
48
49 Bottom Line :
• 50 The rash of Rickettsia prowazekii, or epidemic t yphus, charact eri stica lly begins cent rally and spreads out ; it is best treat ed with tet racyclines .
Rickettsia prowazekii Epidemic typhus Typhus Rickettsia Tetracycline antibiotics Rash Epidemic
• 51
• 52

8
Lock
s
Suspend
0
End Block
Item: 49 of 60 ~ ,• Mark <::J [:::> ""I ~· ~'j
QID: 2031 J.. Previous Next faiUI~S
LAb Notes Calculator

27
28
29
l@ljl'il·1i•J for year:[2017 • J
FIRS T AID FACTS

30
31 FA17 p 188.1
32 Tetracyclines Tetracycline, doxycycline, minocycline.
33
MECHANISM Bacteriostatic; bind to 30S and prevent attachment of ami noacyl-tRNA; limited C l S penetration.
34
Doxycycline is fecally eliminated and can be used in patients with renal failure. Do not take
35
tetracyclines with milk (Ca 2+), antacids (Ca 2+ or Mg2+), or iron-containing preparations because
36
divalent cations inh ibit drugs' absorption in the gut.
37
38 CLINICAL USE Borrelia burgdorferi, M pnewnoniae. Drugs' ability to accumulate intracellula rl y makes them very
39 effective against Rickettsia and Chlamydia. Also used to treat acne. Doxycycline effective against
40 MRSA.
41
ADVERSE EFFECTS Gl distress, discoloration of teeth and inhibition of bone growth in children, photosensitivity.
42
Contraindicated in pregnancy.
43
44 MECHANISM OF RESISTANCE l uptake or t efAux out of bacterial cells by plasmid-encoded transport pumps.
45
46
FA17 p 145.1
47
48 Rickettsial diseases Treatment: doxycycline (caution during pregnancy; alternative is chloramphenicol).
49 and vector-borne
• 50 illnesses
• 51 RASH COMMON
• 52
Rocky Mountain Rickettsia rickettsii, vector is tick. Despite its C lassic triad- headache, fever, rash (vascu litis).
8
Lock
s
Suspend
0
End Block
Item: 49 of 60 ~ ,• Mark <::J [:::> ""I ~· ~'j
QID: 2031 J.. Previous Next LAbfaiUI~S Notes Calculator

27 FA17 p 145. 1
28
Rickettsial diseases Treatment: doxycycline (caution during pregnancy; alternative is chloramphenicol).
29
and vector-borne
30
illnesses
31
RASH COMMON
32
33 Rocky Mountain Rickettsia rickettsii, vector is tick. Despite its Classic triad- headache, fever, rash (vasculitis).
34 spotted fever name, disease occurs primarily in the South Palms and soles rash is seen in Coxsackievirus
35 Atlantic states, especially orth Carolina. A infection (hand, foot, and mouth disease),
36 Rash typically starts at wrists · and ankles and Rocky Mountain spotted fever, and 2° Syphilis
37 then spreads to trunk, palms, and soles. (you drive CARS using your palms and soles).
38
Typhus Endemic (Aeas)-R typhi. Rickettsii on the wRists, Typhus on the Trunk.
39
40
Epidemic (human body lousc)- R prowazekii.
41
Rash starts centrally and spreads out, sparing
42
palms and soles.
43 RASH RARE
44 Ehrlichiosis Ehrlichia, vector is tick. \ito nocytes with MEGA berry-
45 rn
morulae (mulberry-like inclusions) in Monocytes = Ehrlichiosis
46
cytoplasm. Granulocytes = Anaplasmosis
47
Anaplasmosis Anaplasma, vector is tick. GranulOC)i es with
48
49
morulae 9 in cytoplasm.
• 50 Qfever Coxiella bumetii, no arthropod vector. Spores Q fever is Queer because it has no rash or vector
• 51 inhaled as aerosols from cattle/sheep amniotic and its causative organism can survive outside
• 52 Auid. Presents as pneumonia. Common cause in its endospore form. ot in the Rickettsia

8
Lock
s
Suspend
0
End Block
Item: 49 of 60 - ,• Mark -<J [:::> "'I ~ · ~
QIO: 203 1 ~ P~v•ous N@xt Labl lues No tes Calcula t o r

Epidemic (human body lousc)-H prowa;:ekii.


A A
27
28 Rash starts centrally and spreads out, sparing
29 palms and soles.
30 RASH RARE
31
Ehrlichiosis Ehrlichia, ,·ector is tick. \lonOC) tes "ith \ 1EC\ bem-
32
33
morulae rn
(mulbern -like inclusions) in \1onocytes = E hrlichiosis
34
C) toplasm. C ranulOC) tes = \ naplasmosis
35 Anaplasmosis Anaplasma. ,·ector is tick. G ranuloc)1Cs with
36 morulae [!1 in cytoplasm.
37
Q fever Coxiella burnetii, no arthropod vector. Spores Q fe,er is Q ueer because it has no rash or \'ector
38
inhaled as aerosols from cattle/sheep amniotic and its causative organism can sur\'i,·e outside
39
Auid. Presents as pneumonia. Common cause in its endospore form. Not in the Rickettsia
40
41
e
of culture endocarditis. genus, but closely related.
42
43
44
45
46
47
48
49
• 50
• 51
• 52

8
Lode.
s
Suspe-nd
8
End Bloc:k
Item: SO of 60 - ,• Mark -<J [:::> "'I ~ · ~
QIO: 2891 ~ P~v•ous N@xt Labl lues No tes Calcula t o r

6
27 ~~AI
A 53-year-old Hispanic man presents to the gastroenterolgist complaining of interm ittent post prandial epigast ric pain of 1 year' s du ration.
28 An upper endoscopy is performed, and t he biopsy specimen is sent to the pathology lab. Alt hough a simple Steiner silver stain could easily
29 reveal the responsible microbe on t he gastric luminal surface, due to a shortage of inventory, no stain is available. I nstead of disappointing
the gastroenterologist, the pathologist decides to make the diagnosis based on histologic change alone.
30
31
Which of the following changes in the gastric mucosa is likely to be present in this patient?
32
:
33
A. Diffuse mucosal disruption with hemorrhage
34
35 B. Diffuse signet ring cells surrounding gastric mucosal glands
36 C. Hyperplasia of surface mucosal cells with enlarged rugae
37
D. Partial squamous metaplasia of gastric mucosal epithelium
38
39
E. Small, sharply demarcated erosions filled with nonspecific inflammatory infiltrate
40
41
42
43
44
45
46
47
48
49
. 50
• 51
• 52

8
Lode.
s
Suspe-nd
8
End Bloc:k
Item: 50 of 60 - ,• Mark --<) [::> ""'I ~· 1!';:'1
QIO: 2891 ~ Prev1o u s Next Labf a lues Note s Calculator

27 •
The correct answer is E. 51°/o chose this.
28 Chronic upper gastrointestinal discomfort is suggestive of chronic peptic ulcer disease, and th is is
29 often associated with Helicobacter pylori infection (organisms appear in the lumen in the image).
30 H. pylori contributes to ulcer formation by increasing gastric acid secretion, facilitating gastric
metaplasia, generating an immunologic response within the gastric tissue, and downregulating
31 normal mucosal protective barriers. A histologic manifestation of chronic H. pylori-associated
32 gastric ulcers is "punched-out" solitary lesions with underlying tissue replaced with chronic
33 inflammatory cells. These ulcers are typically benign and exhibit the following features: regular,
sharply demarcated borders, and a smooth base filled with exudate. Malignant ulcers, on the
34 other hand, are typically large, irregularly shaped, and have heaped- up margins. A noninvasive
35 diagnosis of H. pylori can be made by the urea breath test. H. pylori utilizes urease to hydrolyze
36 urea into ammonia and C0 2 . In the urea breath test, a urea labeled with carbon isotope (C-14 or
C-13) is given orally. The H. pylori, if present, will subsequently liberate the tagged C0 2 that can
37
be detected in the breath samples.
38 Pepbc ulce Urea bre,H+, ter.t HehcobN:ter pytori Urease Ammonia Gastric acid Urea Isotope HLstology HydtolyStS Immune system

39 Lumen (anatomy) Gastrotntesbnal trKt Helicobacter Malignancy Cancer Infection Mucous membrane Secret1on Carbon
Image courtesy of Ed Uthman, MD
40 Human gastro~ntesbn"l tract Oownregulat1on and upregulation

41
A i s no t correct . 1 1°/o chose thi s.
42 Diffuse mucosal disruption with hemorrhage occurs with acute gastritis and not with Helicobacter pylori-associated peptic ulcer disease.
43 Symptoms of this disorder include melena, anorexia, epigastric pain, and hematemesis.
Peptic ulcer Hematemesis Melena Helicobacter pylori Gastritis Abdominal pain Bleedtng Eptgastnum Anorexia (symptom) Anorexia nervosa Ulcer Gastroenteritis Mucous membrane
44
45 B is not correct. 11 °/o chose thi s.
46 Diffuse signet ring cells are fo und in infilt rating gastri c carcinoma, which is not associated with Helicobacter pylori infect ion and/or chronic
47 gastritis. Infiltrating gast ric carcinoma present s with weight loss, melena or hematemesis, and occasionally distant metastatic foci.
Hematemests Melena Hel1cobacter pylori Stomach cancer Gastritis Signet ring cell We1ght loss MetastasiS Chron1c gastr1t1s Carcinoma Hehcobacter Infection
48
49 C is not correct . 14°/o chose this .
Extensive gastric mucosal hypertrophy is called Menetrier disease, a disorder in which parietal cells atrophy and mucosal cells overproliferate.
50
This precancerous disorder often leads to a protein-losing enteropathy. Symptoms include postprandial pain, cachexia, and peripheral edema
• 51 due to albumin loss.
• 52 Cachex1a Proteen losmg enteropMhy Peripheral edema Albumin Edema Mucous membrane Hypertrophy Human serum album•n Atrophy

8
L.odt
s
Su~pl'nd
~
End Block
Item: SO of 60 ~ ,• Mark <::J [:::> ""I ~· ~'j
QID: 2891 J.. Previous Next LAb faiUI~S Notes Calculator

27 Cis not correct. 14% chos e this .


28 Extensive gast ri c mucosal hypertrophy is called Menetri er disease, a disorder in wh ich parietal cells atrophy and mucosal cells overprolife rate.
This precancerous disorder often leads to a protein-losing enteropathy. Symptoms include postprandia l pain, cachexia, and peri pheral edema
29
due to albumin loss.
30 Cachexia Protein losing enteropathy Peripheral edema Albumin Edema Mucous membrane Hypertrophy Human serum albumin Atrophy
31
D is not correct. 13% chose this .
32
Squamous metaplasia of t he gastric m ucosal epit helium is not a hist olog ic f ind ing in chronic peptic ulcer disease. This condition is extremely
33 rare and carries a t heoretical risk of adenosquamous cell carcinoma. No associat ion with Helicobacter pylori infection is known. Squamous cell
34 metaplasia is usually asymptomatic.
Peptic ulcer Helicobacter pylori Metaplasia Epithelium Histology Asymptomatic Carcinoma Squamous metaplasia Ulcer Ulcer (dermatology) Helicobacter Squamous epithelial cell Mucous membrane
35
36
37 Bottom Line:
38 Chronic Helicobacter pylori-associated gastric ulcers man ifest punched-out lesions with scar t issue and chronic inflammatory cells.
39 Helicobacter pylori Peptic ulcer Helicobacter Ulcer Scar Ulcer (dermatology)

40
41
42
141;fil·1i•J
FIRS T AID FACTS
for year:[2017 •
43
44 FA17 p 142.1
45
46
Helicobacter pylori Curved, flagellated (motile), gram 8 rod fJ that is triple EB: catalase EB, oxidase EB, and urease EB
47
(can use urea breath test or feca l antigen test for diagnosis). Urease produces ammonia, creating
48
an alkaline environment, which helps H pylori survive in acidic mucosa. Colonizes mainly
49 antrum of stomach; causes gastritis and peptic ulcers (especially duodenal). Risk factor for peptic
50 ulcer disease, gastric adenocarcinoma, and MALT lymphoma.
• 51 Most common initial treatment is triple therapy: Amoxicillin (metronidazole if penicillin allergy)
• 52 + Clarithromycin + Proton pump inhibitor; Antibiotics C ure Pylori.

8
Lock
s
Suspend
0
End Block
Item: 50 of 60 - ,• Mark --<) [::> ""'I ~· 1!';:'1
QIO: 2891 ~ Prev1o u s Next Labf a lu es Note s Calculator

27 • •
FA17p363. 1
28 Peptic ulcer disease
29
30
Gastric ulcer Duodenal ulcer
31 PAIN Can be Greater \\;th meals-weight loss Decreases with meals- weight gain
32 HPYLORI INFECTION - 70% - 90%
33
34
MECHANISM l mucosal protection against gastric acid l mucosal protection or t gastric acid secretion
35 OTHER CAUSES lSAIDs Zollinger-EIIison syndrome
36 RISK OF CARCINOMA t Generally benign
37
38
OTHER Biopsy margins to rule out malignanc) l lypertrophy of Bnmner glands
39
40
FA17 p 362.1
41
Gastritis
42
43
Acute gastritis F.rosions can be caused by: F.specially common among alcoholics and
44
• NSA IDs- l PGE 2 - l gastric mucosa patients taking daily NSA IDs (eg, patients wit h
45 protection rheu matoid arthritis).
46 • Burns (Curling ulcer)- hypovolemia
47 - mucosal ischemia Burned by the Curling iron.
48 • Brain injury (Cushing ulcer)- t vaga l
49 stimulation - t ACh - t H+ production lwavs Cushion the brain.
'
50
• 51
Chronic gastritis Mucosal inAammation, often leading to atrophy
• 52
(hypochlorhydria - hypergastrincmia) and •
8
L.odt
s
Su~pl'nd
~
End Block
Item: Sl of 60 - ,• Mark -<J [:::> "'I ~ · ~
QIO: 17 17 ~ P~v•ous N@xt Labl lues No tes Calcula t o r

6
27 ~~AI
After spending 5 days in the hospital being t reated fo r exacerbation of congestive heart failure, a 76-year-old man develops a fever. Urine
28 cultures revea l Enterococcus. An ant ibiot ic is administered, and the patient's fever subsides. However, 3 days lat er, the pat ient develops
29 watery stools and complains of abdominal pain.
30
31 Which of the following antibiotics was most likely init ially administered to this patient to treat the Enterococcus infection?
32 :
33
A. Ampicillin
34 B. Ciprofloxacin
35
C. Clindamycin
36
0 . Metronidazole
37
38 E. Trimethoprim-sulfamethoxazole
39
40
41
42
43
44
45
46
47
48
49
50
. 51
• 52

8
Lode.
s
Suspe-nd
8
End Bloc:k
Item: Sl of 60 - ,• Mark -<J [:::> "'I ~ · ~
QIO: 17 17 ~ P~v•ous N@xt Labl lues No tes Calcula t o r

A A
27
28
The correct answ er is A . 3 6 °/o chose this.
29
This patient had a urinary t ract infection from Enterococcus, for which ampicillin is considered to be first-
line therapy. A few days later he developed waterty diarrhea. In the context of recent antibiotic use, watery
30 diarrhea should raise suspicion for Clostridium difficile infection, particularly when there is an associated
31 leukocytosis. C. difficile releases a toxin that causes watery diarrhea and well as an exotoxin that is
32
cytotoxic to the colonic mucosa, which leads to pseudomembrane formation in later stages of disease (like
that shown in the image). Though ampicillin and clindamycin are common culprits leading to C. difficile
33 overgrowth and pseudomembranous colitis, many classes of antibiotics predispose patients to C. difficile
34 infection. The antibiotic treatment for C. difficile infection is metronidazole or, in more severe cases, oral
35
vancomycin. Oral vancomycin is not absorbed in the GI tract and therefore allows for maximal concentration
at the site of infection.
36 Urmaf'l tnct rnfer- ror Closttldrur drfficrle cohtis Metronidazole Clindamycin Ampicil •n VantOITt(Cin Ant•b•otres Leukocytosrs Exotoxm

37 Clostrtdrum drffrcde (bacterta) O.arrhea Colitis Enterococcus Clostridium Human gastro•ntestmal tract Gastro.ntestmal tract Cytotoxrctty Toxin

38 Mucous membrane Unnary system large intestine


39
Image copyright ©2012 Abe et
40 a/.; licensee BioMed Central Ltd.
41
42 B is no t correct. 11 °/o chose thi s.
43
Ciprofloxacin is used to t reat uri nary t ract infections due t o gram-negative rods, but it is not used for Enterococcus infections. It would not be
used in this case.
44 Ciprofloxacin Gram·negatrve bactena Unnary system Enterococcus Urinary tract infectron
45
C is not correct . 39 % chos e this.
46
Do not automatically choose clindamycin as t he cause when presented with a Clostridium difficile infect ion . Alt hough t reatment with
47 clindamycin may increase t he risk of developing C. diffici/e infection, other antibiotics (eg, cephalospori ns and ampicillin) can also lead to C.
48 difficile overgrowth . Clindamycin is not effective against Enterococcus, whereas ampicillin is considered fi rst- line t herapy. Therefore,
49
clindamycin likely was not given to t his pat ient.
Clindamycrn Amprcrlltn Clostridrum drfficrle cohbs Clostridium difficile (bacteria) Cephalosponn Enterococcus Clo5tnd1um Anbb1obcs Infect1on
50
51
0 is not correct. 8°/o c hose this .
• 52
Metronidazole is the first-line treatment for Clostridium difficile infections. It is also used extensively in the treatment of infections caused by
Giardia Iamblia, Entamoeba histolytica, and Trichomonas vagina/is.

8
Lode.
s
Suspe-nd
8
End Bloc:k
Item: Sl of 60 ~ ,• Mark <::J [:::> ""I ~· ~'j
QID: 1717 J.. Previous Next LAbfaiUI~S Notes Calculator

27 C is not co rrect. 39 % chose this .


28 Do not aut omatically choose clindamycin as t he cause when present ed with a Clostridium difficile infect ion. Alt hough t reat ment wit h
29 clindamycin may increase t he risk of developing C. diffici/e infect ion, other ant ibiotics (eg, cephalospori ns and ampicillin) can also lead t o C.
diffici/e overgrowth . Clindamycin is not effect ive against Enterococcus, whereas ampicillin is considered first -line t herapy. Therefore,
30
clindamycin likely was not given to this patient.
31 Clindamycin Ampicillin Clostridium difficile colitis Clostridium difficile (bacteria) Cephalosporin Enterococcus Clostridium Antibiotics Infection

32
D is not correct. 8°/o chose t his.
33 Met ronidazole is the first -line t reatment fo r Clostridium difficile infect ions. It is also used ext ensively in t he t reatment of infections caused by
34 Giardia Iamblia, Entamoeba histolytica, and Trichomonas vagina/is.
Giardia Iambiia Metronidazole Entamoeba histolytica Trichomonas vaginalis Clostridium difficile colitis Giardia Clostridium Clostridium difficile (bacteria) Trichomonas Entamoeba
35
36 E is not correct. 6°/o chose this .
37 Trimet hoprim -su lfamet hoxazole is used t o t reat recurrent uri nary t ract infections and respirat ory tract infect ions, as well as for prophylaxis
38
against and treat ment of Pneumocystisjirovecii pneumonia. Although TMP-SMX can effectively t reat UTI caused by E. coli, it is not
recommended for Enterococcus infect ions. I n vivo, ent erococci have shown the ability t o ut ilize exogenous folic acid, which allows it t o bypass
39 block in fo lat e synt hesis by TMP-SMX.
40 Folic acid Trimethoprim/sulfamethoxazole Pneumocystis jirovecii Escherichia coli Pneumonia Urinary tract infection Enterococcus Respiratory tract infection Urinary system In vivo Respiratory tract

41 Folate synthesis Preventive healthcare Pneumocystis pneumonia

42
43
Bottom Line :
44
Remember t hat the ant ibiotics t hat most commonly cause Clostridium diffici/e overgrowth are clindamycin, ampicillin, and cephalosporins.
45 Furthermore, ampicillin is f irst-line therapy for Enterococcus infections. For ampicillin- resistant Enterococcus infect ions, vancomycin would be
46 ut ilized.
Clindamycin Ampicillin vancomycin Clostridium difficile colitis Enterococcus Clostridium difficile (bacteria) Cephalosporin Clostridium Antibiotics
47
48
49
l@);fil ~1hl for yea r:l 2o17 y
50 FIRST AID FACTS

51
• 52 FA17 p 134.2

8
Lock
s
Suspend
0
End Block
Item: 51 of 60 - ,• Ma rk --<] [::> ""'I ~· 1!';:'1
QIO: 17 17 ~ Prev1o u s Next Labf a lues Note s Calculator

27 • •

28 FA17 p 134.2
29 Clostridia (with Gram Ei1, spore-forming, obligate anaerobic rods.
30 exotoxins)
31
Ctetani Produces tetanospasmin. an exotoxin causing Tetanus is tetanic paralysis.
32
33
tetanus. Tetanus toxin (and botulinum toxin)
34
are proteases that clea,·e SJ ARE proteins for
35
neurotransmitters. Blocks release of inhibitory
36 neurotransmitters, GABA and glrcine, from
37 Renshaw cells in spinal cord.
38 Causes spastic paralysis, trismus (lockjaw), risus
39 sardonicus (raised eyebrows and open gri n),
40 opisthotonos (spasms of spinal extensors).
41 Pre, en! with tetanus vaccine. Treat'' ilh
42 antitoxin +/-vaccine booster, diazepam (for
43 muscle spasms), and wou nd debridement.
44
( botulinum Produces a heat-labile toxin that inhibits Symptoms of botulism (the 4 D's): Diplopia,
45
46
ACh release at the neuromuscular june!ion, Dysarthria, Dysphagia, Dyspnea.
causi ng botulism. In adults, disease is caused Botulinum is from bad bottles of food , ju ice, and
47
48
by ingestion of preformed toxin. In babies, honey (causes a descending flaccid paralysis).
49 ingestion of spores (eg, in honey) leads to Local botox injections used to treat focal
50 disease (Aopp~ baby synd rome). Treat with dystonia, achalasia, and muscle spasms. Also
51 antitoxin. used for cosmetic reduction offa cial wrinkles.
• 52 Cp erfringens Produces CL toxin (lecithinase, a phospholipase) Perfringens perforates a gangrenous leg. •
8
L.odt
s
Su~pl'nd
~
End Block
Item: Sl of 60 - ,• Mark -<J [:::> "'I ~ · ~
QIO: 17 17 ~ P~v•ous N@xt Labl lues No tes Calcula t o r

A A
27 FA17 p 184.1
28
29
Penicillinase-sensitive Amoxicillin, ampicillin; aminopenicill ins.
30
penicillins
31 MECHANISM Same as penicillin. Wider spectrum; \ \finoPenicillins are A\lPed-up penicillin.
32 penicillinase sensiti,·e. lso combine with AmO,icillin has greater O ral bioavailabilit)
33 clavulanic acid to protect against destruction than ampicillin.
34 by ~-lactamase.
35
CliNICAL USE Extended-spectrum penicillin-I I ir1/1uen;:ae, Co,cragc: ampicillin/amoxicillin III IELPSS
36
I I pylori, E coli, Listeria monocytogenes, kill enterococci.
37
38
Proteus mirabilis, Salmonella, Shigella,
39
enterococci.
40 ADVERSE EFFECTS llrpersensitivity react ions; rash;
41 pseudomembranous colitis.
42
MECHANISM OF RESISTANCE Penicillinase in bacteria (a type or ~-lac lamasc)
43
cleaves ~-lactam ring.
44
45
46 FA17 p 133.3
47
Enterococci Gram$ cocci. Enterococci (E {aecalis and Enterococci, hardier than nonenterococcal
48
E faecium) are normal colonic Aora that are group D, can grow in 6.5% l aCI and bile (lab
49
penicillin G resistant and cause UTI, biliary test).
50
51
tract infections, and subacute endocarditis Entero = intestine, faecalis = feces, strepto =
• 52
(following GI/GU procedures). Catalase 8 , twisted (chains), coccus= berrr-
PYR t+l ~"'ri:~hiP hl"mnlvcio

8
Lode.
s
Suspe-nd
8
End Bloc:k
Item: 52 of 60 - ,• Mark -<J [:::> "'I ~ · ~
QIO: 1848 ~ P~v•ous N@xt Labl lues No tes Calcula t o r

6
27 ~~AI
A 30-year-old missionary who recent ly ret urned f rom a trip t o South America presents to t he clinic with sympt oms of high fever, headache,
28 and back pain. Physical examination revea ls yellow pigment ation at the conjunctiva of t he eyes, and the patient vomits during the visit.
29 The vomitus contains dark-colored blood . Results of a reverse t ranscription polymerase chain react ion test are positive.
30
31 Which of the following viruses is a member of the same virus family as the one most likely causing t his patient's disease?
32 :

33
A. Alpha virus
34 B. Arenavirus
35
C. Coronavirus
36
0 . Hepatitis C virus
37
38 E. Influenza virus
39
40
41
42
43
44
45
46
47
48
49
50
51
. 52

8
Lode.
s
Suspe-nd
8
End Bloc:k
Item: S2 of 60 ~ ,• Mark <::J [:::> ""I ~· ~'j
QID: 1848 J.. Previous Next LAb faiUI~S Notes Calculator

27 •
The correct answer is 0. 57°/o chose this.
28 Yellow fever is caused by a mosquito -born e virus belonging to the fami ly Flaviviri dae; t his virus is generally t ra nsmit ted by Aedes mosquitoes.
29 Sympt oms generally include high feve r, black vomitus (dark-colored blood in the vomit us as a result of gast roint estinal bleeding), j aundice,
30 headache, and back pain. Diagnosis can be confirmed by t esting wit h reverse t ran scriptase polymerase chain reaction wit hin 6 - 10 days. Li ver
biopsy can show signs of Councilman bodies (acidophilic inclusions in liver); however, due to t he bleeding t endency of yellow fever pat ients,
31 liver biopsy is generally not recommended and only advisable post m01tem t o confirm t he cause of deat h. Prevent ive treat ment includes t he
32 use of live-at tenuated vaccines before one t ravels t o an endemic area (eg, Sout h America, Africa) . The hepat it is C virus is another member of
33 t he fa mily Flaviviridae, a linear SS( + )RNA vira l fa m ily wit h icosahedral capsids. Other members of the fami ly Flaviviridae of viruses include
dengue, St. Lou is encephalit is, Japanese encephalitis, West Nile virus, and Zika virus.
34
Reverse t ra nscription PCR (RT-PCR) is used to synthesize DNA from mRNA, making it ideal fo r the detection of RNA viruses. This takes t he
35
mRNA sample and transforms it int o a DNA sample via reverse transcri ptase activity. The sam ple can t hen be readily amplified during t he PCR
36 process.
37 Reverse transcription polymerase chain reaction Flaviviridae West Nile virus Yellow fever Hepatitis C Saint Louis encephalitis Jaundice Reverse transcriptase Liver biopsy Hepatitis C virus Japanese encephalitis
38 Encephalitis Endemism Polymerase chain reaction Arbovirus Dengue fever Hepatitis RNA virus Zika virus Aedes Messenger RNA Virus Liver Biopsy Vomiting Headache RNA South America
39 Gastrointestinal bleeding Transcription (genetics) DNA Mosquito Councilman body Mosquito-borne disease Fever Capsid Gastrointestinal tract Human gastrointestinal tract Autopsy Family (biology)
40
A is not correct. 13% chos e this .
41
Alphavirus is a mem ber of the fami ly Togaviridae. It is a linear SS( +)RNA virus wit h an icosahedral capsid. The fami ly Togaviridae includes
42 rubella virus (German measles) and Alphavirus (eg, East and West equine encephalit is).
43 Togaviridae Alphavirus Measles Capsid Rubella Encephalitis Rubella virus Regular icosahedron Virus Western equine encephalitis virus Icosahedron

44 B is not correct. 14% chose this .


45 Arenavirus is a mem ber of the fami ly Arenaviri dae. I t is a circu lar SS(-)RNA virus (in t wo segments) and possesses a helical nucleocapsid.
46 Other members in t he family include lymphocyt ic choriomeningitis virus (which causes lymphocyt ic choriomeningit is) and Lassa fever
47
encephalit is.
Arenavirus Lassa fever Lymphocytic choriomeningitis Encephalitis Capsid Virus Fever Lassa virus
48
49 C is not correct. 13% chos e this.
Coronavirus is a member of t he fam ily Coronaviridae. It is a linear SS( + )RNA virus and possesses a helical nucleocapsid. This family is t he #2
50
cause of "common colds" (after rh inovirus) and is also associated wit h the 2002- 2003 epidemic of severe acut e respi ratory syndrome (SARS) .
51 Rhinovirus Severe acute respiratory syndrome Coronaviridae Capsid Virus Coronavirus Epidemic

52
E is not correct. 3°/o chos e this.
8
Lock
s
Suspend
0
End Block
Item: S2 of 60 ~ ,• Mark <::J [:::> ""I ~· ~'j
QID: 1848 J.. Previous Next LAb faiUI~S Notes Calculator

27 E is not correct. 3°/o chose this .


28 The inf luenza virus is a member of the fami ly Ort homyxoviri dae. It is a linea r SS(-)RNA virus (wit h eight segments) and possesses a helical
29 nucleocapsid.
Orthomyxoviridae Capsid Influenza Virus Helix
30
31
32 Bottom Line :
33 Yellow feve r is a mosquit o-borne viral illness caused by a Flavivirus, a fami ly of single-st randed ( + ) linear RNA v iruses responsible fo r
34 hepatit is C and t wo of the hemorrhag ic feve rs (yellow fever, dengue) . Yellow feve r manifests with fever, nausea, and pain, and can cause
j aund ice.
35
Yellow fever Hepatitis C Jaundice Flavivirus Hepatitis Dengue fever Virus Nausea RNA virus RNA Fever
36
37
38 141;fil·1i•J for yea r:[2017
FIRST AID FA CTS .
•j .
39
40
41
FA17 p 163. 1

42
RNA viruses
43 VIRAL FAMILY ENVELOPE RNA STRUCTURE CAPSID SYMMETRY MEDICAL IMPORTANCE
44 Reoviruses No OS linear Icosahedral Coltivirus" - C olorado tick fever
45 10-1 2 segments (double) Rotavirus- cause offa tal diarrhea in children
46
Picornaviruses No SS ® linear Icosahedral Poliovirus-polio-Salk/Sabin vaccines- IPV/OPV
47
Echovirus- aseptic meningitis
48
Rhi novirus-"common cold"
49
C oxsackievints- aseptic mening itis; herpangina
50
(mouth blisters, fever); hand, foot, and mouth
51
disease; myoca rditis; pericarditis
52
IIAV-acute viral hepatitis
8
Lock
s
Suspend
0
End Block
Item: 52 of 60 - ,• Mark -<J [:::> "'I ~ · ~
QIO: 1848 ~ P~v•ous N@xt Labl lu es No tes Calcula t o r

A A
27 Rhabdoviruses Yes SS 8 lincar Helica l Rabies
28
Filoviruses Yes SS 8linear Helica l Ebola/:.. larburg hemorrhagic fever-often fatal!
29
30 Arenaviruses Yes SS (f) and 8 ll elical LCM - lymphocytic choriomeningitis\ irus
31 circular Lassa fe\'er encephalitis-spread by rodents
32 2 segments
33
Bunyaviruses Yes SS 8 circular llelical California encephalitis3
34
3 segments SandOy/Rift Va lley fe\'ers3
35
Crimean-Congo hemorrhagic b-e~
36
ll anla\ irus-hemorrhagic fever, pneumonia
37
38 Delta virus Yes SS 8 circular Uncertain HDV is a "defecti,·e" ,-irus that requires the
39 presence of IIBV to replicate
40 SS, single-stranded; OS, double-stranded; Cf>, positi\'e sense; 8, negative sense; a= arbO\irus, arthropod borne {mosquitoes, ticks).
41
42
43 FA17 p 164.5
44 Yellow fever virus A Aavivirus (also an arbovirus) transmitted by Flavi = yellow, jaund ice.
45
Aedes mosquitoes. Virus h<ls a monkey or
46
human reservoir.
47
Symptoms: high fever, black \'Omit us, and
48
jaundice. May see Council man bodies
49
(eosinophilic apoptotic globules) on liver
50
biopsy.
51
52

8
Lode.
s
Suspe-nd
8
End Bloc:k
Item: S3 of 60 - ,• Mark -<J [:::> "'I ~ · ~
QIO: 1844 ~ P~v•ous N@xt Labl lues No tes Calcula t o r

34 6

A 45-year-old male presents complaining of skin lesions on his body. He says he first noticed t hem a few days ago around his buttocks and
35
genitalia, but they have now spread t o his arms. He says th e lesions are not painful, but they occasionally are prurit ic. The pat ient states
36 he's had atopic dermatitis since he was a child. He admits t o being with multiple sexual partners in the past few months but denies any
37 intravenous drug use. Of note, he denies any fever~ fatigue, recent f lu- like symptoms, or recent urethra l discharge. His exam is otherwise
unremarkable except for the umbilicated papules on his gentialia, perineum, buttocks, lower abdomen and extremities (like those shown in the
38
image). Excoriations are also present on the distal extremities.
39
40
41
42
43
44

45
46
47
48
49
50
51 I mages cowtesy CDC/ Dave Bray, MD, Walter Reed Army Medical Center

52
. 53 Which of the fol lowing types of virus most likely caused this patient's clinical findings?
. 54 :
• 55 A. Cytomegaloviru s
' 56 B. Human herpesvirus 3
' 57
C. Human herpesvirus 8
' 58
. 59
D. Human papillomavirus type 1

' 60 E. Poxvirus

8
Lode.
s
Suspe-nd
8
End Bloc:k
Item: S3 of 60 ~ ,• Mark <::J [:::> ""I ~· ~'j
QID: 1844 J.. Previous Next LAbfaiUI~S Notes Calculator

34
35 The correct answer is E. 60°/o chose this .
36 This is a case of molluscum contagiosum caused by t he poxvirus. Physical examinat ion is significant fo r pearl y white or skin-colored nodules
37
with cent ra l umbilication . In children it is often t ransmitted through skin -to-skin contact or indirect contact with fomites (gymnasium
equipment , for example) . Lesions often occur on the chest , arms, t runk, legs, face, and int ertrig inous areas. I n immunocompetent adults, it
38 can be a sexually t ransmitted disease (STD), and lesions t end t o be locat ed around gentialia and surrounding areas. However, in t his case, t he
39 lesions may have spread to t he ext rem it ies because of t rauma induced by excoriation as a means to provide t emporary relief from his
40 eczema . Widespread, pers istent , and atypical present at ions may be seen in immunocompromised pat ients. It often resolves spontaneously
but may require topical therapy or surgery in immunocompromised patients.
41 Cutaneous condition Sexually transmitted infection Poxviridae Dermatitis Molluscum contagiosum Immunodeficiency Fomite Physical examination Immunocompetence
42
A is not correct. 6°/o chos e this.
43
Cytomegalovirus, a member of the herpesvirus family, causes a diverse spectrum of huma n illnesses, wit h substant ial morbidity in
44 immunocompromised pat ients. It is t ransmit ted via mult iple routes such as sexual, cont act, blood or t issue, and perinatal exposure. It does
45 not ty pically cause infection localized t o t he skin, such as t his pat ient's symptoms.
Cytomegalovirus Immunodeficiency Herpesviridae Morbidity Perinatal
46
47 B is not correct. 9% chose this.
48 Human herpesvirus 3, also known as varicella -zoster virus, is t he causative agent in chickenpox. It is a highly cont agious DNA herpesvirus
49
t ra nsmitt ed by respiratory droplet s or by direct contact. The virus, charact erized by recu rrent crops of lesions classically descri bed as "dew-
drops on a rose petal, " is commonly seen on the face and trunk. Varicella-zost er infect ions are often accompanied by prodromal sympt oms of
50 fever and malaise.
51 varicella zoster virus Chickenpox Herpesviridae Prodrome Shingles Virus Malaise Fever DNA

52 C is not correct. 9 °/o chose this .


53 Human herpesvirus 8 is thought to be t he causative fa ct or of Kaposi sarcoma . These lesions are purple/ red/blue and are most common in
. 54 patients with AIDS.
Kaposi's sarcoma-associated herpesvirus Kaposi's sarcoma Sarcoma Herpesviridae HIV/ AIDS
• 55
• 56 D is not correct. 16% chose this .
• 57 Human papillomavirus type 1 is a DNA virus that has predilection fo r t he cut aneous epit helium . It commonly causes plantar warts that appear
as verrucous papules on t he sole or t oes of t he foot. I t is not t o be confused with other types of HPV, which are associat ed with squamous cell
• 58
carcinoma of the cerv ix and ot her mucosal sites .
. 59 DNA virus Human papillomCPJirus Squamous-cell carcinoma Cervix Virus Epithelium Carcinoma Papule DNA Papillomaviridae Squamous epithelial cell Mucous membrane
• 60 ~

8
Lock
s
Suspend
0
End Block
Item: S3 of 60 ~ ,• Mark <::J [:::> ""I ~· ~'j
QID: 1844 J.. Previous Next LAb faiUI~S Notes Calculator

34
35 Bottom Line:
36 Molluscum contagiosum manifests as white or skin-colored papules or nodules with cent ra l umbilication. It often resolves spontaneously.
Cutaneous condition Molluscum contagiosum Papule
37
38
39
40
141;fil·1i•J
FIRS T AID FACTS
for year:[ 2017
.
•j .

41
42 FA17 p 160.1
43 DNA viruses
44 VIRAL FAMILY ENVELOPE ONA STRUCTURE MEDICAL IMPORTANCE
45
Herpesviruses Yes OS and linear See Herpesviruses entry
46
47 Poxvirus Yes OS and linear Smallpox eradicated world wide by use of the live-
48 (largest 0 1 A virus) attenuated vaccine
49 Cowpox ("mi lkmaid blisters")
50 Molluscum contagiosum -Aesh-colored papule with
51 central umbilication
52 Hepadnavirus Yes Partially OS and circular HBV:
53 Acule or chronic hepatitis
. 54
lot a retrovirus but has reverse transcriptase
• 55
• 56
Adenovirus 0 OS and linear Febrile pharyngitis r.J-sore throat
• 57
Acute hemorrhagic cystitis
• 58
Pneumonia
. 59 Conjunctivitis-"pink eye"
• 60

8
Lock
s
Suspend
0
End Block
Item: 53 of 60 - ,• Mark --<] [::> ""'I ~· 1!';:'1
QIO: 1844 ~ Prev1o u s Next Labf a lues Note s Calculator
34
FA17 p 180.1
35
36
Sexually transmitted infections
37 DISEASE CLINICAL FEATURES ORGANISM
38 AIDS Opportunistic infections, Kaposi sarcoma, Ill
39 lymphoma
40
Chancroid Painful genital ulcer with exudate, inguinal Haemophilus ducreyi (it's so painful, you "do
41
adenopathy en " )
42
43 Chlamydia Urethritis, cen·icitis, epididrmitis, Chlamydia trachomatis (0 - K)
44 conjuncti\'itis, reacti\e arthritis, PI D
45 Condylomata Genital warts, koilocytes TIP -6and-ll
46
acuminata
47
48
Genital herpes Painful penile, vulvar, or cer\'ical \'esicles and li S -2, less commonly IISV-1
49
ulcers; can cause system ic symptoms such as
50
fever, headache, myalgia
51 Gonorrhea Urethritis, cervicitis, PID, prostatitis, eisseria gonorrhoeae
52 epididymitis, arthritis, creamy p11rulclll
53 discharge
. 54
Granuloma inguinale Painless, beefy red ulcer that bleeds ree~dily 011 Klebsiella (Calymmatobacterium) granulomatis;
• 55
• 56
(Donovanosis) contact fJ cytoplasmic Donovan bodies (bipolar staining)
. 57
Not common in US seen on microscopy
• 58
. 59
• 60 •
8
L.odt
s
Su~pl'nd
~
End Block
Item: 53 of 60 - ,• Mark --<] [::> ""'I ~· 1!';:'1
QIO: 1844 ~ Prev1o u s Next Labf a lues Note s Calculator
34
Gonorrhea Urethritis, cervicitis, PID, prostatitis, eisseria gonorrhoeae
35
epididymitis, arthritis, creamy pmulent
36
discharge
37
38 Granuloma inguinale Painless, beefy red ulcer that bleeds readil) 011 Klebsiella (Calymmatobacterium) granulomalis;
39 {Donovanosis) contactrJ C) toplasmic Oono,·an bodies (bipolar staining)
40 1 ot common in US seen on microscopy
41
42
43
44

45
46
47
48
Hepatitis B Jaundice IIBV
49
50 Lymphogranuloma Infection of lymphatics; painless genital ulcers, C /rachomatis (Ll-L3)
51 venereum painful lymphadenopathy (ic, buboes)
52 Primary syphilis Painless chancre Treponema pallidum
53
Secondary syphilis l<'ever, lymphadenopathy, skin rashes,
. 54
• 55
condylomata lata
• 56 Tertiary syphilis Gummas, tabes dorsalis, general paresis, aortitis,
. 57 Argyll Robertson pupil
• 58
Trichomoniasis Vaginitis, strawberry cer\'ix, motile in wet prep Trichomonas vagina/is
. 59
• 60 •

8
L.odt
s
Su~pl'nd
~
End Block
Item: S4 of 60
QIO: 2927
- ,•
~
Mark -<J
P~v•ous
[:::>
N@xt Labl
"'I
lues
~ ·
Notes
~
Calculator

34
An obese 43-year-old woman has epigast ric discomfort, feve r~ and scleral icterus of 2 days duration. Physical examinat ion revea ls slight ~~AI
35
jaundice and right upper quadrant tenderness without peritoneal signs. There is no hepatosplenomegaly. The pat ient 's history is significant
36 for m ild, intermittent epigastric pain after meals over the past several months. Laboratory tests show a WBC count of 14,200/mm3, with
37 74% segmented neutrophils, and a serum lipase level of 78 U/L (normal range 0-160 U/L).
38
39 Which of the following is the most likely diagnosis of this patient's acute symptoms?
40 :
41 A. Acute pancreatitis
42 B. Acute viral hepatitis
43
C. Bacterial cholangitis
44

45 D. Cholelithiasis
46 E. Primary biliary cirrhosis
47
48
49
50
51
52
53
. 54
• 55
' 56
' 57
' 58
. 59
' 60 •
8
Lode.
s
Suspe-nd
8
End Bloc:k
Item: S4 of 60 ~ ,• Mark <::J [:::> ""I ~· ~'j
QID: 2927 J.. Previous Next LAbfaiUI~S Notes Calculator

34
35
36 The correct a nswer is C. 5 0 °/o chose this .
37 The Charcot classic t riad of right upper quadrant pain, jaund ice, and feve r is ty pically found in pat ients with cholangit is. Patients may describe
38 symptoms consistent with long-stand ing gallstones, such as abdominal pain after eat ing fatty foods. These gallstones could have blocked the
39
bile duct , lead ing to infection. Bacterial cholang it is is defined as bact eri al infect ion of the bile ducts. Cholangit is can result f rom any cond it ion
obst ruct ing t he bile flow, most commonly choledocholit hiasis. The bacteria are usually ent eric gra m -negat ive rods, such as Escherichia coli,
40 Klebsiella species, Bacteroides species, or Enterobacter species, t hat enter t he bile duct via the am pulla of Vat er. Leukocytosis wit h
41 neut rophi lia and elevated band cell count is commonly present as well. I n t he set ting of cholecystit is, t he pat ient will present with persist ent
42
colicky right upper quadrant pain and nausea but less often displays marked fever and leukocytosis, unless there is signif icant bact eri al
prolife ration within the gallbladder.
43 Ampulla of vater Choledocholithiasis Cholecystitis Escherichia coli Jaundice Leukocytosis Gram-negative bacteria Bacteroides Klebsiella Gallbladder Quadrant (abdomen) Enterobacter Nausea Abdominal pain
44 Bile duct Bile Ascending cholangitis Gallstone Bacteria Biliary colic Fever Pathogenic bacteria Human gastrointestinal tract Gastrointestinal tract Infection Neutrophilia
45
A is not correct . 6°/o chose this.
46
I n cont rast to the norma l lipase level in t his pat ient, acut e pancreatitis usually presents with epigast ric pain radiating to the back and
47 increased serum amylase and lipase levels. Moreover, jaundice is not a classic finding in acute pancreat it is. Gallst ones and alcohol use are t he
48 t wo most common causes of acut e pancreatit is.
Jaundice Acute pancreatitis Amylase Pancreatitis Lipase Abdominal pain Epigastrium Gallstone Blood plasma Alcohol Serum (blood)
49
50 B is not correct. 7% chose t his .
51 Acute viral hepatitis also presents with j aund ice and fever; however, find ings of neut rophi lia and a history suggest ive of cholelit hiasis reduce
t he clinical suspicion fo r vira l hepatit is. A det ailed history can often elucidate t he cause of hepat it is; fo reign t ravel, int rave nous drug use,
52
medicat ions, and unprot ected sex can ra ise t he clinical suspicion for hepatit is.
53 Gallstone Jaundice Viral hepatitis Hepatitis Intravenous therapy Virus Safe sex Drug injection Fever
54
D is not correct. 30% c hose this .
• 55
Cholelit hiasis presents wit h int erm ittent postprandial pain in the epigast ri c reg ion. The consum ption of fatty food exacerbat es t he pain, as t he
• 56 release of cholecystokinin causes cont raction of t he gallbladder. This structure can be temporarily blocked by t he ga llstones, result ing in
• 57 colicky pain. An easy way to remembe r the fact ors that increase t he ri sk of cholelithiasis is t he "4 Fs :" Fat, Female, Forty years or age or
older, and Fertile. Unlike cholangit is, cholelit hiasis does not present wit h leukocytosis, fever, or jaundice .
• 58
Gallstone Cholecystokinin Leukocytosis Gallbladder Jaundice Renal colic Ascending cholangitis Epigastrium Fat Fever Biliary colic
. 59
• 60
E is not correct. 7°/o chose this .

8
Lock
s
Suspend
0
End Block
Item: S4 of 60 ~ ,• Mark <::J [:::> ""I ~· ~'j
QID: 2927 J.. Previous Next LAbfaiUI~S Notes Calculator

34
E is not correct. 7 °/o chos e this .
35
Primary biliary cirrhosis is an aut oimmune reaction t hat leads to lymphocytic inf iltrate and granuloma fo rmat ion in t he biliary t ract. I t is
36 associat ed with elevated serum levels of ant imitochondrial ant ibodies and manifests wit h prurit us, jaundice, dark uri ne, light stools, and
37 hepatosplenomegaly. I mportant laborat ory findings are elevated levels of conjugated bilirubin, cholest erol, and alkaline phosphat ase. Although
38
t he patient in t his scenario does present wit h j aundice, t here is no mention of the color of her urine or st ool. More import antly, she does not
com plain of any itching and has no hepat osplenomegaly on physical exam, t wo hallmarks of t his condit ion. Prima ry biliary cirrhosis is more
39 common in women t han in men and is associated wit h autoimmune diseases, such as celiac disease, Hashimot o t hyroidit is, and rh eumatoid
40 arthri t is.
Hashimoto's thyroiditis Primary biliary cirrhosis Coeliac disease Bilirubin Rheumatoid arthritis Alkaline phosphatase Cirrhosis Hepatosplenomegaly Jaundice Granuloma Biliary tract Itch Altoimmunity
41
Antibody Altoimmune disease Cholesterol Urine Serum (blood) Arthritis Thyroiditis Bile Blood plasma Physical examination Bile duct
42
43
44 Bottom Line:
45 Pat ients present ing wit h fever, jaundice, and upper right quadrant pain (Charcot t riad) should raise t he suspicion of cholangit is.
46 Jaundice Ascending cholangitis Fever

47
48
49 141;fil·1i•J for year:[2017
FIRST AID FA CTS .
•j .
50
51 FA17 p 379.1
52
53
Gallstones t cholesterol and/or bilirubin, l bile salts, and Risk factors (4 F's):
54 (cholelithiasis) gallbladder stasis aII cause stones. l. Female
• 55 2 types of stones: 2. Fat
• 56 Cholesterol stones (radiolucent with 10-20% 3. Fertile (pregnant)
• 57 opaque due to calcifications)- 80% of stones. 4. Forty
• 58 Associated with obesity, Crohn disease, Diagnose with ultrasound rn.Treal· with elective
. 59 advanced age, estrogen therapy, multiparity, cholecystectomy if symptomatic.
• 60 ~ ranid weight loss. ative American orif!in . Can cause fistula between gaJlblaclcler and
8
Lock
s
Suspend
0
End Block
Item: S4 of 60 ~ ,• Mark <::J [:::> ""I ~· ~'j
QID: 2927 J.. Previous Next LAb faiUI~S Notes Calculator

34
35 FA17 p 379.1

36 Gallstones t cholesterol and/or bilirubin, ! bile salts, and Risk factors (4 F's):
37 (cholelithiasis) gallbladder stasis aII cause stones. l. Female
38
2 types of stones: 2. Fat
39
Cholesterol stones (radiolucent with 10-20% 3. Fertile (pregnant)
40
opaque due to calcifications)- 80% of stones. 4. Forty
41
Associated with obesity, Crohn disease, Diagnose with ultrasound Ul. Treat with elective
42
advanced age, estrogen therapy, multiparity, cholecystectomy if symptomatic.
43
rapid " ·eight loss, ative American origin. Can cause fistula between gallbladder and
44
45
Pigment stones r.l1 (black= radiopaque, Ca 2+ Gl tract ..... air in biliary tree (pneumobilia)
46
bil irubinate, hemolysis; brown = radiolucent, ..... passage of gallstones into intestinal tract
infection). Associated with Crohn disease, -+ obstruction of ileocecal valve (gallstone
47
48 chronic hemolysis, alcoholic cirrhosis, ileus).
49 advanced age, biliary in fections, total
50 parenteral nutrition (TP ).
51 Most common complication is cholecystitis;
52 can also cause acute pancreatitis, ascending
53 chola ngitis.
54
RELATED PATHOLOGIES CHARACTERISTICS
• 55
• 56 Biliary colic Associated with nausea/vomiting and dull RUQ pain . eurohormonal activation (eg, by CCK after
• 57 a fatty meal) triggers contraction of gal lbladder, forcing stone into cystic duct. Labs are normal,
• 58 ultrasound shows cholelithiasis.
. 59 Presence of gallstone(s) in common bile duel, often leading to elevated ALP, GGT, direct bilirubin,
Choledocholithiasis
• 60 ., ~A J~. &C::T/111 T

8
Lock
s
Suspend
0
End Block
Item : S4 o f 60 - ,• Ma rk --<) [::> ""'I ~· 1!';:'1
QIO: 2927 ~ Prev1ous Next Labfa lues Notes Calcu lator
34
RELATED PATHOLOGIES CHARACTERISTICS
35
36
Biliary colic Associated with nausea/vomiting and dull RUQ pain. I eurohormonal activation (eg, by CCK after
37 a fatty meal) triggers contraction of gallbladder, forcing stone into cystic duct. Labs are normal,
38 ultrasound shows cholelith iasis.
39 Choledocholithiasis Presence of ga llstone(s) in common bile duct, often leading to elevated ALP, GGT, direct bilirubin,
40 and/or AST/ALT.
41
Cholecystitis Acute or chronic inflammation of gallbladder usually from cholel ithiasis (stone at neck of
42
43
gallbladder [red arrow in 8] with gallbladder wall thickening [yellow arrows]).
44
Calculous cholecystitis: mo~t common I ) pe; clue to gallstone impaction in the cystic duct re~ultin g
45 in inflammation; can produce zoinfection.
46 Acalculous cholecystitis: due to gallbladder stasis, hypoperfusion, or infection (C~ IV); seen in
47 critically ill patients.
48 Vlurphy sign: inspiratory arrest on RUQ palpation due to pain. t ALP if bile duct becomes im olved
49 (eg, ascending cholangitis).
50 Diagnose with ultrasound or cholesci nt igraphy (IIIDA scan). Failure to visualize gallbladder on
51 HIDA scan suggests obstruction.
52
53
Porcelain gallbladder m.
Ca lcified gallbladder due to chronic cholecyst itis; IISllally found incidentally on imaging
Treatment: prophylactic cholecystectomy clue to high rates of gallbladder cancer (mostly
54
adenocarcinoma).
• 55
• 56
. 57
• 58
. 59
• 60 •
8
L.odt
s
S u~ pl'nd
~
End Block
Item : S4 o f 60 - ,• Ma rk --<) [::> ""'I ~· 1!';:'1
QI O: 2927 ~ Prev1ous Next Labfa lues Notes Calcu lator
34 • •
FA17 p 378.3
35
36 Biliary tract disease May present with prurihts, jaundice, dark urine, light-colored stool, hepatosplenomegalr. Typically
37 with cholestatic pattern of LFTs (t conjugated bilirubin, t cholesterol, t ALP).
38 PATHOLOGY EPIDEMIOLOGY ADDITIONAL FEATURES
39 Primary sclerosing Unknown cause of concentric Classicall} in middle-aged men ssociated '' ith ulcerative
40 " onion skin" bile duct ,, ith lBO.
cholangitis colitis. p-A 'CA $ . t lg \1.
41
fibrosis - alternating Can lead to 2° biliary
42
strictures and dilation with cholangitis. t risk of
43
"beading" of intra- and cholangiocarcinoma and
44
extrahepatic bile ducts on gallbladder cancer.
45
46
ERCP, magnetic resonance
47
cholangiopancreatography
48
(1\IRCP).
49 Primary biliary Autoimmune reaction Clas~icall) in middle-aged Anti-mitochondrial antibody®,
50 cholangitis - lymphocytic infiltrate women. t IgM. Associated with other
51 +granulomas - destruction autoimmune conditions
52 of intralobular bile ducts. (eg, Sjogren syndrome,
53 Hashimoto thyroiditis,
54
CREST, rheumatoid arthritis,
• 55
celiac disease).
• 56
. 57 Secondary biliary Extrahepatic biliary obstruction Patients with known :\ lay be complicated by
• 58 cholangitis - t pressure in intrahepatic obstructive lesions (gallstones, ascending cholangitis.
. 59 ducts - injury/ fibrosis and biliary strictures, pancreatic
• 60 bile stasis. carcinoma). •
8
L.odt
s
S u~ pl'nd
~
End Block
Item: SS of 60 - ,• Mark -<J [:::> "'I ~ · ~
QIO: 24 23 ~ P~v•ous N@xt Labl lues No tes Calcula t o r

34
A 53-year-old man who was diagnosed wit h HI V infection 8 years ago presents to his physician with several purple-colored, painless ~~AI
35
plaques on his upper back. The image shows t he result of a biopsy of one of these lesions.
36
37
38
39
40
41
42
43
44

45
46
47
48
49 Which of the following is the etiologic agent responsible fo r these lesions?
50 :
51 A. Epstein-Barr virus
52 B. Hepatitis C virus
53
C. Human herpesvirus 8
54
• 55 D. Human papillomavirus
• 56 E. Human T-cell lymphoma virus type 1
. 57
• 58
. 59
. 60 •
8
Lode.
s
Suspe-nd
8
End Bloc:k
Item: SS of 60 ~ ,• Mark <::J [:::> ""I ~· ~'j
QID: 2423 J.. Previous Next LAbfaiUI~S Notes Calculator

34
35
36
The correct answer is C. 77°/o chos e this.
37 The image shows t he spindle-shaped tumor cells diagnost ic of Kaposi's sarcoma . The lesions classically appear as described in t he quest ion.
38 Human herpesvirus 8, also known as Kaposi's sarcoma-associated herpesvirus, is t he oncogenic virus t hat infects endothelial and spindle cells,
39
giving rise to lesions like the one shown here.
Kaposi's sarcoma-associated herpesvirus Kaposi's sarcoma Neoplasm Carcinogenesis Oncogene Sarcoma Herpesviridae Endothelium Spindle apparatus Virus
40
41 A is not correct. 7°/o chos e this.
Epstein-Ba rr virus is associat ed with Burkitt 's lymphoma and nasopharyngeal carcinoma .
42
Burkitt's lymphoma Epstein-Barr virus Nasopharynx cancer Carcinoma Lymphoma Nasopharynx Virus Pharynx
43
44
B is not correct. 3 % chose this.
45
Hepatitis C virus is associat ed wit h hepatocellular carcinoma .
Hepatitis C virus Hepatocellular carcinoma Hepatitis C Hepatitis Virus Carcinoma
46
47 D is not correct. 5°/o chos e this.
Human papillomavirus (types 16 and 18) is associated wit h cervica l and penile/anal carcinoma.
48
Human papillomavirus Carcinoma Papillomaviridae Cervix Cervical cancer
49
E is not correct. 8 °/o chos e this .
50
Human T-cell lymphoma virus t ype 1 is associat ed with adu lt T-cell leukemia.
51
Adult T-cell leukemia/lymphoma Leukemia T cell T-cell lymphoma T-cell leukemia Lymphoma Virus
52
53
54
Bottom Line:
55 Kaposi's sarcoma (wit h its spind le-shaped tu mor cells) is due to human herpesvirus 8 and is found common ly in AIDS patients.
Kaposi's sarcoma Kaposi's sarcoma-associated herpesvirus Sarcoma Herpesviridae HIV/AIDS Neoplasm
• 56
• 57
• 58
l@ljl'il·1i•J for year:[2017 • J
. 59 FIRST AID FACTS

• 60

8
Lock
s
Suspend
0
End Block
Item: SS of 60 ~ ,• Mark <::J [:::> ""I ~· ~'j
QID: 2423 J.. Previous Next LAb faiUI~S Notes Calculator

34 •
35 FA17 p 160.2
36
Herpesviruses Enveloped, OS, and linear viruses
37
38 VIRUS ROUTE OF TRANSMISSION CLI NICAL SIGNIFICANCE NOTES
39 Herpes Respiratory Gingivostomatitis, keratoconjunctivitis fJ, Most common cause of sporadic
40 simplex secretions, saliva herpes labialis [l), herpetic whitlow on finger, encephalitis, can present as altered
41 virus-1 temporal lobe encephalitis, esophagitis, mental status, seizures, and/or
42 erythema mu lti forme. aphasia.
43
44
Herpes Sexua I contact, t-Ierpes genital i s ~, neonatal herpes. Latent in sacral ganglia. Vi ral
45
simplex perinatal meningitis more common with
46
virus-2 HSV-2 than with HSV-1.
47 Varicella- Respiratory Varicella-zoster (chickenpox 1!], shingles 0}, Latent in dorsal root or trigeminal
48 Zoster virus secretions encephalitis, pneumonia. ganglia; Cl V1 bra nch
49 (HHV-3) Most common complication of shingles is post- involvement can cause herpes
50 herpetic neuralgia. zoster ophthalmicus.
51
Epstein-Barr Respiratory Mononucleosis- fever, hepatosplenomegaly, Infects B cells through CD21.
52
virus (HHV-4) secretions, pharyngitis, and lymphadenopathy (especially Atypical lymphocytes on peripheral
53
54
saliva; aka posterior cervical nodes 0 ). Avoid contact sports blood smear [!!- not infected B
55
"kissing disease," until resolution due to risk of splenic rupture. cells but reactive cytotoxic T cells.
• 56 (common in Associated with lymphomas (eg, endemic (±) Monospot test-heterophile
• 57 teens, young Burkitt lymphoma), nasopharyngeal antibodies detected by agglutination
• 58 adults) carcinoma (especially Asian adults), of sheep or horse RBCs.
. 59 lymphoprol iferative disease in transplant Use of amoxicillin in mononucleosis
• 60 ~
patients. can cause characteristic ~

8
Lock
s
Suspend
0
End Block
42
43
44
FA17 p 219.1

45 Oncogenic microbes Microbe Associated cancer


46
EB Burkitt lymphoma, Hodgkin I) mphoma,
47
nasopharyngeal carcinoma, 1° C lS
48
49
lymphoma (in immunocompromised pal ienls)
50 HBV, l1CV llcpatocellular carcinoma
51 HH V-8 Kaposi sarcoma
52
HPV Cervical and penile/anal carcinoma (lypes 16,
53
54
18), head and neck cancer
55 II P)'lori C astric adenocarcinoma and IALT lr mphonm
• 56
HTLV-1 Adult T-cellleukemiallrmphoma
. 57
• 58
Li\'er Auke (Clonorchis sinensis) Cholangiocarcinoma
. 59 Schistosoma haematobium Bladder cancer (squa mous cell)
• 60 •
8
L.odt
s
Su~pl'nd
~
End Block
Item: 56 of 60 - ,• Mark -<J [:::> "'I ~ · ~
QIO: 3911 ~ P~v•ous N@xt Labl lues No tes Calcula t o r

34
A full-term baby is born t o a 26-year-old woman who did not seek medical care during her pregnancy. The baby does well initially, but ~~AI
35
develops red eyes with purulent discharge on day 13. The responsible organism is shown in the image.
36
37
38
39
40
41
42
43
44

45
46
47
48 Image courtesy of CDC/ Or. Wiesner, Dr. Kaufman
49
50 The infectious agent most likely responsible fo r this baby's condit ion is also involved in the pathogenesis of which neonatal disease?
51 :
52 A. Chorioretinit is
53 B. Meningit is
54
C. Pneumonia
55
' 56 D. Sepsis
' 57 E. Urethritis
' 58
. 59
' 60 •
8
Lode.
s
Suspe-nd
8
End Bloc:k
Item: S6 of 60 ~ ,• Mark <::J [:::> ""I ~· ~'j
QID: 3911 J.. Previous Next LAbfaiUI~S Notes Calculator

34
35 The correct a ns wer is C. 4 6 °/o chos e this .
This baby has acute bacterial conj unctivit is. The most likely cause of which (and the organism in t his case) is Chlamydia trachomatis
36
(seroty pes D t hrough K) - the most common sexua lly t ransmitted disease in t he U.S . Neonates are exposed to the organism duri ng vaginal
37 delivery. Symptoms of chlamydia I (inclusion) conjunct iv it is beg in 5- 14 days after birth, and include inflammat ion, swelling around t he eyelids,
38 and t he presence of a yellow purulent discharge. Neisseria gonorrhoeae is another common cause of neonat al conjunct iv it is; however,
39
symptoms t ypically present between 2 and 5 days after bi1t h. All neonates born in the U.S. are given prophy lactic erythromycin eye drops,
alt hough t his pract ice does not effectively prevent ch lamyd ia! conj unctiv it is. I n neonates with inclusion conjunct ivit is, agg ressive therapy wit h
40 t opical and oral erythromycin is undert aken t o prevent t he development of neonatal pneumonia as a result of descending infection f rom t he
41 eyes to t he nasopharynx (presumably via the nasolacrimal duct ) and downward to t he lungs. Such infants develop cough, respiratory distress,
42
and pneumon ia between 4 and 11 weeks of life. A CBC may show eosinophilia (this is a commonly t ested point !) . The image displays
eukaryot ic cells infected wit h C. trachomatis. The diffuse inclusion body in t he cent er of t he cell pushes the nucleus to the cell's peri phery.
43 Sexually transmitted infection Erythromycin Chlamydia trachomatis Neisseria gonorrhoeae Neonatal conjunctivitis Pneumonia Conjunctivitis Pus Eukaryote Chlamydia infection Chlamydia (genus) Neisseria
44 Childbirth Serotype Nasopharynx Eosinophilia Inflammation Nasolacrimal duct Cough Preventive healthcare Dyspnea Eye drop Inclusion bodies Infant Organism Lung
45
A is not correct . 17 % chos e this .
46
Chorioret init is is inflammat ion of the choroid and retina. Newborns with chorioret init is are cl assica lly associat ed with congenita l toxoplasmosis,
47 caused by the parasite Toxoplasma gondii.
48 Toxoplasma gondii Toxoplasmosis Chorioretinitis Retina Choroid Inflammation Parasitism Congenital disorder

49 B is not correct. 19 % chose this .


50 Neit her Chlamydia trachomatis nor Neisseria gonorrhoeae cause meningit is. The most feared complication of C. trachomatis infection in
51 neonates is pneumonia .
Chlamydia trachomatis Neisseria gonorrhoeae Meningitis Pneumonia Chlamydia infection Chlamydia (genus) Neisseria Infant
52
53 D is not correct. 6°/o chose this .
54 Neonata l sepsis is usually caused by vertical t ransmission from mot her t o child t hrough contaminated amniotic fluid or by bact eria from the
mother's lower genital t ract during delivery. Neonatal sepsis is most commonly caused by Group B Streptococcus and Escherichia coli. other
55
etiolog ic agents include Listeria, Staphylococcus aureus, Enterococcus, and HSV.
56 Escherichia coli Staphylococcus aureus Amniotic fluid Sepsis Neonatal sepsis Vertically transmitted infection Streptococcus Staphylococcus Listeria Bacteria Enterococcus Infant

• 57
E is not correct. 1 2°/o chose this .
• 58
Urethri t is is an infect ion of t he urethra. Common causat ive agents include Neisseria gonorrhoeae and Chlamydia trachomatis. Often pat ients
. 59 are asymptomat ic. This is especially t rue of gonococcal infection in men. When sympt omat ic, t he most freq uent complaints are dysuri a and
• 60 ~
discharge. It is not a sequela of neonatal chlamyd ia! infection . ~

8
Lock
s
Suspend
0
End Block
Item: S6 of 60 - ,• Mark -<J [:::> "'I ~ · ~
QIO: 3911 ~ P~v•ous N@xt Labl lu es No tes Calcula t o r

34 A A

35
Bo ttom Li ne:
36 Neonatal chlamydia! infect ion (serotypes D throug h K) produces inclusion conjunctivit is 5-14 days after birth. Another complication of
neonatal chlamydia! infection is pneumonia, which occurs between 4 and 11 weeks after birth.
37 Con)unct1v1bs Pneumon1a Serotype Chlamydia Infection Infant Infection
38
39
40 l@l;fii·11•J for yea r : 2017 •
FI RST A ID FACTS
41
42
FA17 p 146.2
43
44
Chlamydia trachomatis serotypes
45 Types A, 8, and C Chronic infection, cause blindness due to \DC = Africa, Blindness, C hronic infection.
46 follicular conjunctivitis in Africa.
47
Types 0 - K Urethritis/PID, ectopic pregnancy, neonatal 0 - K =everything else.
48
pneu monia (staccato cough) with cosi no phi I ia, 'eonata l disease can be acquired during
49
neonatal conjunctivitis ( 1-2 weeks after birth). passage through infected birth canal.
50
51 Types l 1, l 2, and l 3 l ymphogranuloma venereum-small, painless
52 ulcers on genitals ..... swollen, painful inguinal
53 lymph nodes that ulcerate (buboes). Treat with
54 doxycycl inc.
55
56
FA1 7 p 146.1
. 57
• 58 Chlamydiae Chlamydiae cannot make their own TP. They C hlamys =cloak (intracellular).
. 59 are obligate intracellular organisms that cause C f)sittaci-has an avian reser\'oir (parrots),
. 60 • mucosal infections. 2 forms: causes at~ pica! pneumonia .
8
Lode.
s
Suspe-nd
8
End Bloc:k
Item: S6 of 60 - ,• Mark -<J [:::> "'I ~ · ~
QIO: 3911 ~ P~v•ous N@xt Labl lues No tes Calcula t o r

34 A A
Types L1, L2, and L3 Lymphogranuloma venereum-small, painless
35
ulcers on genitals ..... swollen , painful inguina l
36
37
lymph nodes that ulcerate (buboes). Treat with
38
doxycycline.
39
40 FA17 p 146.1
41
Chlamydiae Chlamydiae cannot make their own t\TP. They Chlamys =cloak (intracellular).
42
are obligate intracellular organisms that cause C fJsittaci-has an avian reservoir (parrots),
43
mucosal infections. 2 forms: causes at) pica I pneumonia.
44

45
Elementary body (small, dense) Lab diagnosis: PCR, nucleic acid amplification
46
is "Enfectious" and Lnters cell \'ia test. Cytoplasmic inclusions (reticulate bodies)
47 E ndocrtosis; transforms into reticulate body. seen on Giemsa or Auorescent antibody-
48 Reticulate body Replicates in cell by fission; stained smear.
49 Reorganizes into elementary bodies. The chlamrdial cell wall lacks classic
50 Chlamydia trachoma tis causes reactive arthritis peptidoglycan (due to reduced muramic acid),
51 (Reiter syndrome), folli cular conjunctivitis rJ, rendering ~-lactam antibiotics ineffective.
52 nongonococcal urethritis, and PID.
53 Chlamydophila pneumoniae and Chlamydophila
54 psittaci cause at ypica I pneumon ia; tra nsmittcd
55 by aerosol.
56
Treatment: azithromycin (fm ored because one-
. 57
time treatment) or doxycycline (+ ceftriaxone
• 58
for possible concomitant gonorrhea).
. 59
. 60

8
Lode.
s
Suspe-nd
8
End Bloc:k
Item: S7 of 60 - ,• Mark -<J [:::> "'I ~ · ~
QIO: 3901 ~ P~v•ous N@xt Labl lues No tes Calcula t o r

34
Pri or to vaccine development, a particular pat hogen was highly prevalent in the United St ates. Culturi ng it f rom a sit e of infection requ ires ~~AI
35
swiping with a calcium alg inat e swab, followed by plat ing ont o potato agar or Regan-Lowe medium.
36
37
What is the organism described?
38
:
39
A. Bacillus anthracis
40
41
B. Bordetella pertussis
42 C. Chlamydia trachomatis
43
0 . Clostridium tetani
44

45
E. Corynebacterium diphtheriae
46 F. Legionella pneumophila
47 G. Neisseria gonorrhoeae
48
49
50
51
52
53
54
55
56
. 57
• 58
. 59
. 60 •
8
Lode.
s
Suspe-nd
8
End Bloc:k
Item: S7 of 60 - ,• Mark -<J [:::> "'I ~ · ~
QIO: 3901 ~ P~v•ous N@xt Labl lues No tes Calcula t o r

34 A A

35
36 The correct answ er is B. 51% chose this.
37 The causative agent of whooping cou gh, the gram- negative rod Bordetella pertussis. may be cultured from an infected pharynx. A calcium
38
alginate swab should be used because t he bacteria do not t ransfer well on cotton. From t he swab, the organisms are grown on either potato
(Bordet-Gengou) agar or Regan-Lowe m edium (which is more commonly used today). Since vaccine development, the incidence of whooping
39 cough in the United States has dropped from severa l hundred-t housand cases a year to several thou sand. Several recent reports, however,
40 suggest that it is still may be a frequent cause of bronchitis and chronic cough in adults.
Bordeteua Pf" .. ot ~ Gn. egao 'e biiiCtena Pharynx &"onchitis Pertussis Bordetel a Cough Bactt:na Lg e C C•UI
41
42 A is not correct . 5°/o chose this.
43 Bacillus anthracis does not require special culture conditions. A vaccine has been developed against t his gram- positive rod, but it is not given
as standard practice.
44
Bac us anthr acas G a pos1b 1e bact:erra Bacillus Vaccine
45
46
C is not correct. 4 °/o chose this.
Chlamydia trachomatis is an obligate intracellular species and requires co-culture with eukaryotic cells. Again, there is no effective vaccine.
47
Chlamydta trachomatrs Chlamydra rnfectron Eukaryote Intracellular Obligate intracellular parasrte Vaccrne Sper:res
48
D is not correct . 10°/o chose this.
49
The causat ive agent of tetanus, Clostridium tetani (as well as all clostri dial species), requ ires anaerobic culture conditions for laboratory
50
growth.
51 Clostridium tetanr Tetanus Anaerobrc orgamsm Clostridium Hypoxia (environmental) Clostridia

52
E is not correct. 22% chose this.
53
Tellurite agar or Loffl er's medium is used fo r culturing Corynebacterium diphtheriae. Immunity against this gram-posit ive rod is conferred by
54 th e combination DPT (diphtheria- pertussis-t et anus) vaccine.
Corynebacterium drphtheriae Gram·posrbve bacteria Tellurite Agar Corynebacterium Vaccrne Hoyle's agar
55
56 F is no t correct . 5°/o chose this.
57 Legionel/a pneumophila is grown on BYCE agar (buffered-charcoal-yeast-extract) supplemented wit h cyst eine and iron. No vaccine has been
• 58 developed against this gram- negat ive rod .
Legtonella pneumoph•la LeQIOneUa Gram-negative bacteria Cysteine Agar Vaccine
. 59
. 60 G is not correct . 3°/o chose this.

8
Lode.
s
Suspe-nd
8
End Bloc:k
Item: S7 of 60 ~ ,• Mark <::J [:::> ""I ~· ~'j
QID: 3901 J.. Previous Next LAbfaiUI~S Notes Calculator

34
35 F is not correct. 5°/o chos e this.
Legionel/a pneumophila is grown on BYCE agar (buffered -charcoal -yeast -extract ) supplement ed with cyst eine and iron. No vaccine has been
36
developed against t his gram -negative rod .
37 Legionella pneumophila Legionella Gram-negative bacteria Cysteine Agar Vaccine

38
G is not correct. 3% chos e this.
39
Neisseria gonorrhoeae is grown on Thayer- Martin med ia, also known as VCN media. It is chocolat e agar supplemented wit h vancomycin,
40 colistin, and nyst at in . Alt hough candidate vaccines against gonococcal disease have been st udied, there is current ly no effect ive means of
41 vaccinat ion .
Nystatin Colistin Neisseria gonorrhoeae vancomycin Chocolate agar Neisseria Gonorrhea Agar Vaccination Thayer-Martin agar
42
43
44 Bottom Line:
45 Whooping cough is caused by Bordetel/a pertussis, which is collected using a calcium alginat e swab rat her t han cotton and is cultured on
46 potat o agar or Regan -Lowe medium.
Pertussis Bordetella pertussis Cough Bordetella Calcium Agar
47
48
49
l@ljl'il·1i•J for year:[2017 • J
50 FIRST AID FA CTS

51
52 FA17 p 123.1
53 Special culture requirements
54 BUG MEDIAUSEDFOR ISOLATI ON MEDIACONTENTS/OTHER
55 H influenzae Chocolate agar Factors V ( AD+) and X (hematin)
56
N gonorrhoeae, Thayer-Marti n agar Selectively favors growth of 1 eisseria by
57
• 58
N meningitidis inhibiting growth of gram EEl organisms
. 59 with Vancomycin, gram 8 organ isms except
• 60 ~
Neisseria with Trimethoprim and Colistin,
8
Lock
s
Suspend
0
End Block
Item: S7 of 60 - ,• Mark -<J [:::> "'I ~ · ~
QIO: 3901 ~ P~v•ous N@xt Labl lues No tes Calcula t o r

34 A A
FA17p123.1
35
Special culture requirements
36
BUG MEDIAUSED FOR ISOLATION MEDIA CONTENTS/OTHER
37
38 H influenzae Chocolate agar l<a ctors V (NAD+) and X (hematin)
39 N gonorrhoeae, Thayer- lartin agar Selective)) favors growth of 1 eisseria by
40 N meningitidis inhibiting growth of gram EEl organisms
41 "ith Vancomycin, gram 8 organisms except
42
'eisseria '' ith Trimethoprim and Colistin,
43
and fungi with '\ystatin
44
Very Tr pically Cultures :-..·eisseria
45
46 Bpertussis Bordet-Gengou agar (Bordct for Bordetefla) Potato extract
47 Regan-Lowe medium Charcoal, blood, and antibiotic
48 C diphtheriae 'Jellurite agar, LofAer medium
49
M tuberculosis Lowcnstein-Jenscn agar
50
51 M pneumoniae Eaton agar Requires cholesterol
52 Lactose-fermen ting MacConkey agar Fermentat ion produces acid, causing colonies to
53 enterics turn pink
54
E coli Eosin- methylene blue (EM B) agar Colonies with green metallic sheen
55
56 Legionella Charcoal reast extract agar buffered with
57 cysteine and iron
• 58
Fungi Sabouraud agar " ,Sab's a fu n guy,.,
.
. 59
. 60 •
8
Lode.
s
Suspe-nd
8
End Bloc:k
Item: 57 of 60 - ,• Mark --<] [::> ""'I ~· 1!';:'1
QIO: 3901 ~ Prev1o u s Next Labf a lu es Note s Calculator
34 • •
FA17 p 128.1
35
36
Bugs with exotoxins
37 BACTERIA TOXIN MECHANISM MANIFESTATION
38 Inhibit protein synthesis
39
Corynebacterium Diphtheria toxina Phat: ngitis '' ith pseudomembranes in throat
40
diphtheriae Inactivate elongation factor and severe lymphadenopathy (bull neck)
41
42 Pseudomonas Exotoxin N (EF-2) Host cell death
43 aeruginosa
44 Shigella spp. Shiga toxin (ST)3 Gl mucosal damage -- d~·sentery; ST also
45 Inactivate 60S ribosome by enhances cytokine release, causing hemolytic-
46
removing adenine from uremic syndrome (HUS)
47
Enterohemorrhagic Shiga-like toxin rR 'A SLT enhances cytokine release, causing II U
48
49
E coli (EHEC) (SLT)3 (prototypically in EHEC serotype 0157:H7).
50 Unlike Shigella, EH EC does not im'adc host
51 cells
52 Increase fluid secretion
53
Enterotoxigenic Heat-labile Overactivates adenylate Watery diarrhea: "labile in the Air (Aclenylate
54
E coli (ETEC) toxin (LT)a cyclase (t cAMP) -- t Cl- cyclase), stable on the G round (Guanylate
55
secretion in gut and 1110 cyclase)"
56
57
efAux
• 58
ll eal-~ tabl e Overactivates guan)date
. 59 toxin (ST) cyclase (t cG ~lP)
• 60 -- l resorption of 1 aCI •
8
L.odt
s
Su~pl'nd
~
End Block
Item: 57 of 60 - ,• Mark --<] [::> ""'I ~· 1!';:'1
~
QIO: 3901 Prev1o u s Next Labf a lues Note s
..
Calculator
. - -

35 Clostridium Botulinum toxina of proteins required for Flaccid paralysis, Aoppy baby; toxin prevents
36 botulinum neurotransm itter release release of stimulatory (ACh) signals at
37 via vesicular fusion neuromuscular junctions - Aaccid paralysis
38
a An AB toxin (aka, two-component toxin [or three for anthrax]} with B enabling binding and triggering uptake (endocytosis)
39
of the acti' e A component. The A components are usually ADP ribosyltransferases; others have enzymatic activities as listed
40
in chart.
41
42 Lyse cell membranes
43
Clostridium Alpha toxin Phospholipase (lecithinase) Degradation of phospholipids - myonecrosis
44
perfringens that degrades tissue and ("gas gangrene") and hemolysis ("double zone"
45
46
cell membranes of hemolysis on blood agar)
47 Streptococcus Streptolysin 0 Protein that degrades cell l.yses RBCs; contributes to ~-h emolysis;
48 pyogenes membrane host antibodies against toxin (ASO) used to
49 diagnose rheumatic fe,·er (do not confuse
50 with immune complexes of poststreptococcal
51 glomerulonephritis)
52
Superantigens causing shock
53
54 Staphylococcus Toxic shock Binds to IHC II and TC R Toxic shock syndrome: feve r, rash, shock; other
55 aureus syndrome toxin outside of antigen binding toxins cause scalded skin syndrome (exfoliative
56 (TSST-1) site to cause overwhelming toxin) and food poisoning (heat-stable
57 release of IL-1, I L-2, enterotoxin)
• 58
Streptococcus Exotoxin A IF, -y, and T l F-a Toxic shock-like smdrome: fever, rash, shock;
. 59
pyogenes - shock scarlet b ·cr
• 60 •
8
L.odt
s
Su~pl'nd
~
End Block
Item: S8 of 60 - ,• Mark -<J [:::> "'I ~ · ~
QIO: 4 929 ~ P~v•ous N@xt Labl lues No tes Calcula t o r

34
A 23-year-old man from Kenya presents wit h night sweats, fevers, oliguria, and a large submandibular mass. Biopsy of t he mass shows an ~~AI
35
aggressive tumor with sheets of lymphocytes staining posit ive for CD20, as well as very high levels of nuclear c-myc wit h int erspersed
36 macrophages as shown in th e image.
37
38
39
40
41
42
43
44

45
46
47
48
49
50
51
What is the genetic makeup of t he causal organism?
52
53 :
A. Circu lar partially double-stranded DNA
54
55 B. Linear double-st randed DNA
56 C. Linear negative-sense single-stranded DNA
57
D. Linear positive-sense segment ed double-st randed RNA
. 58
. 59 E. Linear positive-sense single-stranded RNA
. 60 •
8
Lode.
s
Suspe-nd
8
End Bloc:k
Item: SB of 60 - ,• Mark -<J [:::> "'I ~ · ~
QIO: 4 929 ~ P~v•ous N@xt Labl lues No tes Calcula t o r

34 A A
The correct a ns w e r is B. 67% chose this .
35
This patient has Burkitt's lymphoma, a lymphoma endemic to Africa. Clinically, Burkitt's lymphoma often
36 presents with "B symptoms" (fever, night sweats, weight loss); signs of tumor lysis syndrome, such as
37 oliguria; and solitary jaw masses like t hat shown in the image here. Histopathologically, Burkitt's lymphoma
38
typically assumes a "starry sky" appearance, in which sheets of lymphocytes are interspersed with
occasional macrophages, as seen in the stem image. The cytogenetic abnormality associated with Burkitt's
39 lymphoma is a t(S; 14) translocation in which the oncogene c-myc is placed under the expression of the
40 immunoglobulin heavy-chain enhancer. There is a strong association between Burkitt's lymphoma and
41
Epstein-Barr virus (EBV). In regions where the disease is endemic, children are typically infected with EBV
by the age of 3 years, whereas infection typically occurs during adolescence in other regions. EBV, the
42 cause of infectious mononucleosis, is spread in saliva and respiratory secretions. EBV is a linear double-
43 stranded DNA virus.
Infect:JO ·~ mono,• jeo ~ Tu 10r lys•s syndrome ONA V1rus Epstein-Barr virus Ohguria Oncogene Lymphoma Endem~sm Myc Lymphocyte Macrophage Image courtesy of Mike Blyth
44
Anbbody N•ght sweats Neopl..m V~rus Cytogenetics Weoght loss Fever Lysos Sah"" DNA Chromosomal translocat•on H•stopathology
45 Chromosome abnormality Infection
46
A is not correct. 8°/o chose t h is .
47
One example of a circu lar partially double-stranded DNA virus is hepatit is B virus (HBV). HBV is blood-borne. Although this virus is associated
48 with hepatocellular carcinoma, t here is no association between HBV and Burkitt's lymphoma.
49 DNA viru! Hepatocellular carc~noma Hepat1bs B Hepatitis 8 virus Hepatitis Lymphoma Virus DNA Carc1noma

50 C is not correct. 10% chos e this .


51 An example of a linear negative-sense single-stranded DNA virus is parvovirus B19. Parvovirus B19 causes aplastic crises in sickle cell
52 disease, erythema infect iosum (fifth disease) in children, and hydrops feta lis in fetuses as a resu lt of RBC dest ruction, but th ere is no
association between parvovirus B19 and Burkit t's lymphoma.
53
DNA virus F1fth disease Parvov1rus 819 Hydrops fetalis Sickle- cell disease Parvovirus Sense (molecular biology) Erythema V1rus Lymphoma DNA Red blood cell
54
55
D is not correct . 7°/o chos e t his .
One example of a linear positive-sense segmented double-stranded RNA virus is rotavirus. Rotavirus causes diarrh ea in children, but there is
56
no association between rotavirus and Burkitt's lymphoma.
57 Rotav1rus 01~rrhea V1ru:. RNA v1ru:. Sense (molecular b1ology) RNA Double-stranded RNA v1ruses Pos1t1ve~sense Lymphoma

58
E is not correct. 8°/o chose t h is .
. 59
One example of a linear positive-sense single-stranded RNA virus is hepatitis A virus (HAV). HAVis transmitted by the feca l-oral route. There
. 60 is no association between HAV and Burkitt's lvmohoma .
8
Lode.
s
Suspe-nd
8
End Bloc:k
Item: sa of 60 ~ ,• Mark <::J [:::> ""I ~· ~'I
QID: 4929 J.. Previous Next LAbfaiUI~S Notes Calculator

34 .. Infectious mononucleosis Tumor lysis syndrome DNA virus Epstein-Barr virus Oliguria Oncogene Lymphoma Endemism Myc Lymphocyte Macrophage I mage cowtesy of Mike Blyth
35 Antibody Night sweats Neoplasm Virus Cytogenetics Weight loss Fever Lysis Saliva DNA Chromosomal translocation Histopathology
Chromosome abnormality Infection
36
37 A is not co rre ct . 8°/o chose this.
38 One example of a circu lar partially double-st ra nded DNA virus is hepat it is B virus ( HBV) . HBV is blood-born e. Alt hough t his virus is associat ed
with hepatocellular carcinom a, t here is no association bet ween HBV and Burkit t's lymphoma .
39
DNA virus Hepatocellular carcinoma Hepatitis 8 Hepatitis B virus Hepatitis Lymphoma Virus DNA Carcinoma
40
41
C is not co rrect. 10% chose this .
An example of a linea r negat ive -sense single-stranded DNA virus is parvovirus B19. Parvovirus B19 causes aplastic crises in sickle cell
42
disease, eryt hema infect iosum (fifth disease) in children, and hydrops feta lis in fetuses as a resu lt of RBC dest ruct ion, but t here is no
43 associat ion bet ween parvovirus B19 and Burkit t's lymphoma.
DNA virus Fifth disease Parvovirus 819 Hydrops fetalis Sickle-cell disease Parvovirus Sense (molecular biology) Erythema Virus Lymphoma DNA Red blood cell
44
45 D is not corre ct. 7 °/o chose this.
46 One example of a linear positive-sense segmented double-stranded RNA virus is rotavirus. Rotavirus causes dia rrhea in children, but there is
47 no association bet ween rotavirus and Burkit t's lymphoma .
Rotavirus Diarrhea Virus RNA virus Sense (molecular biology) RNA Double-stranded RNA viruses Positive-sense Lymphoma
48
49 E is not correct. 8 °/o chose this .
50
One example of a linear positive-sense single-stranded RNA virus is hepat itis A virus (HAV) . HAVis t ransm itt ed by t he fecal-oral route. There
is no associat ion between HAV and Burkit t's lymphoma .
51 Fecal-oral route Hepatitis A RNA virus Sense (molecular biology) Hepatitis RNA Virus Hepatitis A virus Lymphoma
52
53
Bottom Line :
54
55
There is a strong association bet ween Burkitt's lymphoma and Epst ein -Barr v irus, which is a linear double-st randed DNA virus.
Burkitt's lymphoma DNA virus Epstein-Barr virus DNA Lymphoma Virus
56
57
58
i@l;fil·1i•J fo r yea r:[2017 • J
. 59 FIRST AID FACTS

• 60

8
Lock
s
Suspend
0
End Block
Item: S9 of 60 - ,• Mark -<J [:::> "'I ~ · ~
QIO: 24 3 1 ~ P~v•ous N@xt Labl lues No tes Calcula t o r

34
A 28-year-old woman with a history of asthma presents wit h a temperature of 38.1 °C (100 .8°F) and complains of shaking chills and pain ~~AI
35
on her right side, which she locates by pointing t o t he area above her right iliac crest. She complains of some pain on urination, urinary
36 frequency, and urgency.
37
38 What is the organism most likely causing her condit ion?
39
:
40 A. A lactose-fermenting, gram- negative rod
41
B. An oxidase-negative, lactose- nonfermenting, gram-negative rod
42
43 C. An oxidase-positive, lactose- nonferment ing, gram-negative rod
44 0 . Gram-negative bacteria capable of catalyzing urea to ammonia
45
E. Novobiocin-resistant, coagulase -negat ive, gram- positive cocci in clusters
46
47
48
49
50
51
52
53
54
55
56
57
58
. 59
. 60 •
8
Lode.
s
Suspe-nd
8
End Bloc:k
Item: 59 of 60 ~ ,• Mark <::J [:::> ""I ~· ~'j
QID: 2431 J.. Previous Next faiUI~S
LAb Notes Calculator

34
35
The correct a nswer is A. 59°/o chose t h is.
36
This pat ient is present ing with t he cl assic symptoms of pyelonephrit is, an ascending infection from t he lower uri nary t ract t hat reaches t he
37 kidneys. This infection is more common in women t han in men and is caused most often by Escherichia coli (a lactose-ferment ing, gram-
38 negative rod) . E. coli can be found in 82% of cases in women and 73% of cases in men.
Pyelonephritis Escherichia coli Gram-negative bacteria Urinary system Kidney Infection
39
40 B is not correct. 11 % chose this .
41 This choice descri bes Proteus mirabilis, which is an uncommon cause of pyelonephrit is. Proteus is more commonly found in cases of infect ed
kidney stones.
42
Pyelonephritis Proteus mirabilis Proteus (bacterium) Kidney stone Proteus Kidney
43
C is not co rrect. 10% chose this .
44
This choice descri bes Pseudomonas species, which are increasingly prevalent in cases of iat rogenic infection (cathet ers, etc). Howeve r~
45
Pseudomonas does not commonly cause communit y acquired pyelonephri t is or uri nary t ract infections.
46 Pyelonephritis Jatrogenesis Pseudomonas Urinary system Urinary tract infection Catheter

47
D is not correct. 12% c hose this.
48
Proteus mirabilis and Ureaplasma urealyticum are urease-posit ive bact eri a t hat can cause uri nary t ract infect ions and pyelonephri t is, but E.
49 coli is the most common cause of pyelonephrit is.
Pyelonephritis Ureaplasma urealyticum Proteus mirabilis Escherichia coli Proteus (bacterium) Urinary tract infection Bacteria Ureaplasma Proteus Urinary system
50
51 E is not correct. 8°/o chose this .
52 This answer choice refers t o Staphylococcus saprophyticus, which is less common ly the cause of pyelonephritis (3% of cases) but more
53 commonly a cause of cystit is. Localized cystit is usually is not accom panied by systemic symptoms such as fever or t achycardia. Symptoms of
cystit is include urinary f requency and dysuria.
54 Dysuria Pyelonephritis Staphylococcus saprophyticus Urinary tract infection Tachycardia Staphylococcus Fever
55
56
57
Bottom Line :
58
E. coli (a lact ose-fe rment ing, gram- negat ive rod) is the most common cause of pyelonephri t is.
Pyelonephritis Escherichia coli Gram-negative bacteria
59
• 60

8
Lock
s
Suspend
0
End Block
Item: 59 of 60 ~ ,• Mark <::J [:::> ""I ~· ~'j
QID: 2431 J.. Previous Next faiUI~S
LAb Notes Calculator

34
FA17 p 570.2
35
Pyelonephritis
36
37 Acute pyelonephritis eutrophils infiltrate renal interstitium r:il. Affects cortex with relative sparing of glomeruli/vessels.
38 Presents with fevers, Rank pain (costovertebra l angle tenderness}, nausea/vomiting, chills.
39 Causes include ascending UTI (E coli is most common), hematogenous spread to kid ney. Presents
40 with WBCs in urine+/- WBC casts. CT wou ld show striated parenchymal enhancement [l).
41 Risk factors include indwell ing urinary catheter, urinary tract obstruction, vesicoureteral reAux,
42 diabetes mellitus, pregnancy.
43
Complications include chronic pyelonephritis, renal papillary necrosis, perinephric abscess,
44
uroseps1s.
45
Treatment: antibiotics.
46
47 Chronic The result of recurrent episodes of acute pyelonephritis. Typically requires predisposition to
48 pyelonephritis infection such as vesicoureteral reAux or chronically obstructing kidney stones.
49 Coarse, asymmetric corticomedullary scarring, blunted calyx. Tubules can contain eosinophilic
50 casts resembling thyroid tissue B1 (thyroidization of kidney).
51 Xanthogranulomatous pyelonephritis- rare; grossly orange nodules that can mimic tumor
52 nodules; characterized by widespread kidney damage due to granulomatous t·issue containing
53
54
55
, .
foamy macrophages.
~

56
57
58
59
• 60

8
Lock
s
Suspend
0
End Block
Item: 59 of 60 ~ ,• Mark <::J [:::> ""I ~· ~'j
QID: 2431 J.. Previous Next LAb faiUI~S Notes Calculator

34
FA17p177.1
35
36 Urinary tract Cystitis presents with dysuria, frequency, urgency, suprapubic pain, and WBCs (but not WBC
37 infections casts) in urine. Primarily caused by ascension of microbes from ureth ra to bladder. Males-
38 infants with congen ital defects, vesicoureteral reAux. Elderly-enlarged prostate. Ascension to
39 kidney results in pyelonephritis, which presents with fever, chills, Aank pain, costovertebral angle
40 tenderness, hematuria, and WBC casts.
41
Ten times more common in women (shorter urethras colonized by fecal flora). Other predisposing
42
factors: obstruction, kidney surgery, catheterization, GU malformation, diabetes, pregnancy.
43
44
45 FA17 p 177.2
46 UTI bugs
47 SPECIES FEATURES COMMENTS
48
Escherichia coli Leading cause of UTI. Colonies show green Diagnostic markers:
49
metallic sheen on EMB agar. Ef> Leukocyte esterase= evidence ofWBC
50
Staphylococcus 2nd leading cause of UTI in sexua lly active activity.
51
saprophyticus women. Ef> litrite test = reduction of urinary nitrates
52
by bacterial species (eg, E coli).
53 Klebsiella pneumoniae 3rd leading cause of UTI. Large mucoid capsule
Ef> Urease test = urease-producing bugs (eg,
54 and viscous colonies.
55
S sa(Jrophyticus, Proteus, Klebsiella).
Serratia marcescens Some strains produce a red pigment; often
56
nosocomial and drug resistant.
57
58 Enterococcus Often nosocomial and drug resistant.
59 Proteus mirabilis Motility causes "swarming" on agar; produces
• 60 nr~"lC'O• ..,('('t"\,.. ; .,t~,.J " ' ;t), c-f.-rn H;f.-.6 c- tl"\n~C'

8
Lock
s
Suspend
0
End Block
Item: 60 of 60 - ,• Mark -<J [:::> "'I ~ · ~
QIO: 1 2 11 ~ P~v•ous N@xt Labl lues No t es Calcula t o r

34
An 8-year-old boy comes to the physician's office complaining of a sore throat. He and his fami ly recent ly immigra ted to t he Unit ed States ~~AI
35
from Cameroon, and his parents do not know if he has had any vaccinations. Currently, he is having difficulty breathing and swallowing. A
36 grayish-white mucous fi lm is noted on t he boy's oropharynx.
37
38 Which of the following best explains this patient's symptoms?
39
:
40 A. Impaired initiation of translation
41
B. Impaired translocation
42
43 C. Inactivated 305 ribosome
44 0. Inactivated 805 ribosome
45
E. Inhibition of acetylcholine release
46
47
48
49
50
51
52
53
54
55
56
57
58
59
. 60 •
8
Lode.
s
Suspe-nd
8
End Bloc:k
Item: 60 of 60 ~ ,• Mark <::J [:::> ""I ~· ~'j
QID: 1211 J.. Previous Next LAbfaiUI~S Notes Calculator

34
35 The correct answer is B. 41% chose this .
36 This boy has diphtheri a. I t is caused by Corynebacterium diphtheriae, a bacterium that produces dipht heria toxin. Diphtheria toxin inhibi ts
37 elongat ion factor EF2-a . EF2-a is an important molecule in prot ein synthesis. It complexes wit h guanosine t ri phosphat e and binds to the
ri bosome, init iat ing t he t ranslocation step of protein synt hesis.
38
Corynebacterium diphtheriae Diphtheria Ribosome Diphtheria toxin Protein biosynthesis Toxin Corynebacterium Protein Elongation factor Bacteria Protein synthesis Molecule
39
40
A is not correct. 33% chose this .
I nit iat ion of t ranslation involves eukaryotic init iation factor 2. Diphtheri a toxin inh ibits elongat ion facto r EF2-a, which is involved in the
41
t ranslocation step of protein synt hesis. I n dipht heria, init iation occurs normally.
42 Diphtheria Protein biosynthesis Protein Diphtheria toxin Elongation factor Protein synthesis Eukaryote Eukaryotic initiation factor Toxin

43
C is not correct. 13% chose this .
44 Ant ibiotics such as tetracycline and aminoglycosides inhibit t he 305 subunit of ri bosomes found in prokaryotes. Dipht heri a t oxin is causing t he
45 patient's sympt oms, not a drug.
Tetracycline Diphtheria Diphtheria toxin Aminoglycoside Antibiotics Ribosome Prokaryote Toxin 305
46
47 D is not correct. 8°/o chose this.
48 This boy has diphtheri a. Dipht heri a t oxin does not inh ibit ri bosomes. I nst ead, it inhibits the t ranslocation of prot eins being synt hesized .
Diphtheria Diphtheria toxin Ribosome Toxin
49
50 E is not correct. 5°/o chose this.
51 I nhibit ion of acet ylcholine release is caused by a toxin released by Clostridium botulinum. Botulism causes flaccid paralysis.
Clostridium botulinum Botulism Acetylcholine Flaccid paralysis Toxin Paralysis Botulinum toxin Clostridium
52
53
54 Bottom Line:
55 Corynebacterium diphtheriae produces diphtheria toxin. Diphtheri a toxin inh ibits elongat ion factor EF2-a, inhibit ing t ranslocation and prot ein
56 synt hesis.
Corynebacterium diphtheriae Diphtheria Diphtheria toxin Protein biosynthesis Protein Corynebacterium Toxin Elongation factor
57
58
59
l@ljl'il·1i•J for year:[2017 • J
60 FIRST AID FA CTS

8
Lock
s
Suspend
0
End Block
Item: 60 of 60 - ,• Mark --<) [::> ""'I ~· 1!';:'1
QIO: 1 2 11 ~ Prev1o u s Next Labf a lu es Notes Calculator
34
FA17 p 135.1
35
36 Corynebacterium Gram$ rod; transmitted via respiratory Cor)•ne = club shaped.
37 diphtheriae droplets. Causes diphtheria 'ia exotoxin Black colonies on C)'Stine-tellurite agar.
38 encoded by ~-prophage. Potent exotoxin ABCDEFG:
39 inhibits protein srnthesis via AOP-ribOS) lation ADP-ribosrlation
40 of EF-2. ~-prophage
41
Symptoms include pseudomembranous Corrnebacterium
42
pharyngitis (grayish-\\ hite membrane ) Diphtheriae
43
with lymphadenopathy. myocarditis, and Elongation Factor 2
44
arrlwthmias. Granules
45 '
Lab diagnosis based on gram $ rods with
46
47
metachromat·ic (blue and reel) granule~ and
48
$ Elek test for toxin.
49
Toxoid vaccine prevents diphtheria.
50
51 FA17p41 .1
52 Protein synthesis
53
Init iation Initiated by GTP hydrolysis; inihation factors Eukaryotes: 40S +60S - 80S (Even).
54
(eukaryotic IF's) help assemble the 40S PrOkaryotes: 30S + 50S - 70S (Odd).
55
56
ribosomal subunit with the initia tor tR A Synthesis occurs from Lterminus to
57
and are released when the mR 'A and the C-terminus.
58
ribosomal 60S subunit assemble with the
\TP-tR ' \ cti,·ation (charging).
59 complex.
GTP-tRNA Gripping and Going places
Elonaation I. Aminoacd-tRI\ hinds to site (cxccnt for •
60 "-----•---'-=- -'
8
L.odt
s
Su~pl'nd
~
End Block
Item: 60 of 60 - ,• Mark --<) [::> ""'I ~· 1!';:'1
QIO: 1 2 11 ~ Prev1o u s Next Labf a lu es Notes Calculator
34
FA1 7 p 41 .1
35
Protein synthesis
36
37 Initiation Initiated by GTP hydrolysis; initial ion factors Eukaryotcs: 40S +60S - 80S (Even).
38 (eukaryotic IF's) help assemble the 40S PrO karyotes: 30S + 50S - 70S (Odd).
39 ribosomal subunit with the initiator tR 'A Synthesis occurs from N-terminus to
40 and arc released when the mR A and the C-tem1inus.
41 ribosomal 60S subunit assemble with the
42 \TP-tR \ cti,·ation (charging).
complex.
43 GTP-tR:--JA G ripping and Going places
44
Elongation 1. minoacyl-tRI'\A binds to site (except for (I ranslocation).
45
initiator methionine)
2. rRt A ("ribozyme") eatal)7eS peptide bond Think of'·going \PE":
46
formation, transfers growing pol) pepI ide to \ site = incoming Aminoacyl-tR A.
47
48 amino acid in site P site = accommodates grow·ing Peptide.
49 3. Ribosome advances 3 nucleotides toward 3' t•: site = holds Empty tR A as it Exits.
50 end of mRr A, moving peptidyl tR A to P
Eukaryotlc
51 site (Iransloeal ion) nbosome
52
Termination Stop codon is recognized by release factor, 3'
53
and completed polypeptide is released from
54 s·
ribosome.
55
56
57
58
59 0
60 •
8
L.odt
s
Su~pl'nd
~
End Block
Item: 60 of 60 - ,• Mark -<J [:::> "'I ~ · ~
QIO: 1 2 11 ~ P~v•ous N@xt Labl lues No t es Calcula t o r

34 A A

FA17p40.1
35
36
tRNA
37 Structure zo
75-90 nucleotides, structurc, dO\ crlcaf form, anticodon end is opposite 3' aminoacyl end. All
38 t!U As, both eukaryotic and prokaryotic, have CCA at 3' end along with a high percentage of
39 chem icalh· modified bases. The amino acid is CO\'a lentlv bound to the 3' end of the tR ' . CG\
' '
40 Can Carry Amino acids.
41 T-arm: contains the T \f'C (ribothymidine, p eudouridine, cytidine) sequence necessary for tR 'A-
42
ribosome binding. T-am1 Tethers tRNA molecule to ribosome.
43
0-arm: contains dihydrouridine residues necessar) for I RI':A recognition by the correct aminoacrl-
44
tRt synthetase. D-arm Detects the tR1\A by aminoacrl-tR synthetase.
45
Acceptor stem: the 5'-CC -3' is the amino acid acceptor site.
46
47 Charging Aminoacyl-tRl A synthetase (I per amino acid; "matchmaker"; uses ATP) scrutinizes amino acid
48 before and after it binds to tRt A. If incorrect, bond is hydrolrzcd. The amino acid-tR bond
49 has energy for formation of peptide bond. A mischarged tR A reads usual codon but inserts
50 wrong amino acid.
51 Aminoacyl-tR A synthetase and binding of charged tR 'A to the codon are responsible for
52 accuracy of amino acid selection.
53
Structure Charging Pairing
54 (codon-anticodon)
(aminoacytation)
55
Amino at~d,
0' Amino acid,
56
57
Acceptor stem{OH - i 3. . 3.
(
(
0 ' . 3.
(
(

58 s· s· s·
Aminoacvl-tRNA
59 syntl e IF2
60 T-arm n-:a.rm 6TO AUO.J.OO

8
Lode.
s
Suspe-nd
8
End Bloc:k
Item: 60 of 60 - ,• Mark --<) [::> ""'I ~- 1!';:'1
QIO: 1 2 11 ~ Prev1o u s Next Labf a lues Notes Calculator
34
tRt A synthetase. D-arm Detects the IR ' by aminoacyl-t R1 A synthetase.
35
Acceptor stem: the 5'-CCA-3' is the am ino acid acceptor site.
36
37
Charging Aminoacyl-tRl A synthetase (l per amino acid; "matchmaker"; uses ATP) scrutinizes amino acid
38 before and after it binds to tR, A. If incorrect, bond is hydrolyzed. The amino acid-tR1 bond
39 has energy for formation of peptide bond. A mischarged tR 'A reads usual codon but inserts
40 wrong amino acid.
41 Aminoacyl-tR1 A synthetase and binding of charged tRI'\ to the codon are responsible for
42 accuracy of amino acid selection.
43
Structure Charging Pairing
44 (codon-anticodon)
(aminoacylatlon)
45
AmulO aod, Am1no acid,
46 0 0
AcceptOf stem{0 H- ~ 3• ' . 3'
c
' . 3'
c
47 c c
48
AminoacyHRNA
49 synthPt .e IF2
50 T·arm
D-arm ATP AMP t PP, (initiation factor)
51
C 'P T
• C 'P T

52 D

53 Variable arm_/
54
55 Anticodon
loop , c..._ ••c Anticodon (5· CAU 3')-[ • • c
56 • "'-Wobble
57 position mRNA ~ ~ ~ • II G M
58
L,-J
Codon
59 (5' AUG 3')
60 •
8
L.odt
s
Su~pl'nd
~
End Block
Item: 61 of - ,• Mark -<J
P~v1ous
[:::> "'I ~ ·· ~
100 ~ N @xt Labl lues N o tes Calcula to r
6

39 A 7-year-old girl has numerous vesicles on her face, part icularly around her mouth, after fa lling and scraping her fa ce on t he ground. Over ~~AI
40
a few days the vesicles t urn int o pustules and crust over, becoming flaky and light yellow.
41
Which of the following statements about the organism most likely respons ible for this girl's infection is correct?
42
43 :
A. Endotoxin is present in the outer membrane of this organism
44

45 B. Sabouraud agar is required to culture this bacterium


46 C. The bacterium is a-hemolytic
47
D. The bacterium is coagulase positive
48
49 E. The bacterium is sensitive to bacit racin
50
51
52
53
54
55
56
57
58
59
60
. 61
• 62
• 63
. 64

8
Lode.
s
Suspe-nd
8
End Bloc:k
Item: 61 of - ,• Mark -<J
P~v1ous
[:::> "'I ~ ·· ~
100 ~ N @xt Labl lues N o tes Calcula to r

The correct answ er is D. 49°/o chose this.


A A
60
61 The most common cause of impetigo is Staphylococcus aureus, followed by Streptococcus pyogenes. S.
' 62 aureus is typically coagulase positive, whereas other Staphylococcus species are coagulase negative.
Non bullous impetigo is characterized by an eruption of vesicles on the face. These vesicles later turn into
' 63
pustules with a characteristic honey-colored crust. Bullous impetigo is characterized by flu id-filled bullae
. 64 that also rupture, leaving a thin brown crust (see image). Bullous impetigo is caused by strains of S. aureus
. 65 that produces exfoliative toxin A, which disrupts cell adhesion in the epidermis .
Streptococc 1 pyage 15 Impetigo stuphylococcus aureus Coagulase Bullous tmpetigo Eptderm- Strertococ-ru .;tapl fococcus Bhste- Cutaneous condition
• 66
ToXIn Cell adheston
• 67
• 68
. 69
. 70

' 71
Image courtesy of Wikimedia
' 72
Commons
' 73
' 74 A is not correct . 7°/o chose thi s.
. 75 The presence of a lipopolysaccharide endot oxin in the cell membrane is a characteristic of gram-negative bacteria, but not Staphylococcus
• 76
aureus or Streptoccus pyogenes. Endot oxin t riggers multiple reactions, including activating macrophages, t he complement pathway, and
possibly coagulation cascades.
· 77 Lipopolysaccharide Cell membrane Gram-negative bacteria Staphylococcus aureus Staphylococcus Macrophage Bacteria Coagulation Btological membrane Membrane
• 78
B is not correct. 5% chose this.
' 79
Sabouraud agar is requ ired to culture f ungi, not Staphylococcus au reus. Other important agars include Lowenstein-Jensen agar fo r
. 80 Mycobacterium tuberculosis, charcoal yeast extract agar with cysteine for Legionella, chocolate agar fo r Haemophilus influenzae, and Bordet-
. 81 Gengou agar for Bordetella pertussis .
Bordetella pertussts Mycobactenum tuberculosis Haemophilus influenzae Chocolate agar Staphylococcus aureu~ Fungus Legtonella Cystetne Sabouraud agar Tuberculosis Agar Pertuss1s Staphylococcu5
' 82
YeMt extr~ct Bordetella Yewt Mycobactenum
' 83
' 84 C is not correct . 10°/o chose thi s.
. 85 Streptococcus pneumonia and the Viridans group Streptococcus are both a-hemolytic, but Staphylococcus aureus and Streptococcus
.. pyogenes are ~-hemolytic .

8
Lode.
s
Suspe-nd
8
End Bloc:k
Item: 61 of ~ ,• Mark <::J [:::> ""I ~· ~'j
100 J.. Previous Next LAb faiUI~S Notes Calculator

60 C is not correct. 10% chos e this.


61 Streptococcus pneumonia and the Viridans group St reptococcus are bot h a- hemolytic, bu t Staphylococcus aureus and Streptococcus
• 62 pyogenes are 13-hemolyt ic .
Streptococcus pyogenes Pneumonia Staphylococcus aureus Streptococcus Streptococcus pneumoniae Staphylococcus Viridans streptococci
• 63
. 64 E is not correct. 29°/o chose this .
. 65 The second most common cause of im petigo is Streptococcus pyogenes, a gram- positive group A 13-hemolytic organism that is bacit racin
sensit ive. Bacitracin is used to different iateS. pyogenes (bacit racin sensit ive) from S. agalactiae (bacit racin resistant ), whereas optochin is
• 66
used t o different iat eS. pneumoniae (optochin sensit ive) from v irid ans grou p st re pt ococci (optochin resist ant) .
• 67 Streptococcus pyogenes Bacitracin Impetigo Gram- positive bacteria Streptococcus pneumoniae Streptococcus Optochin Organism

• 68
. 69
Bottom Line:
. 70
Staphylococcus aureus is a gram-posit ive cocci t hat is a common cause of impet igo. The next most common cause is Streptococcus
• 71 pyogenes, eit her alone or in conj unction with S. aureus. Nonbullous impet igo is characterized by vesicl es t hat become pustules and develop a
· 72 yellow crust.
Streptococcus pyogenes Impetigo Coccus Gram- positive bacteria Staphylococcus aureus Streptococcus Staphylococcus Vesicle (biology and chemistry)
• 73
• 74
. 75
l@);fil ~1hl for year:l 2o17 y
• 76 FIRST AID FACTS

· 77
• 78 FA17 p 131 .3
• 79
Staphylococcus aureus Gram EE>, ~-hemolytic, catalase EE>, coagulase TSST-1 is a superantigen that binds to MHC
• 80
(E) cocci in clusters fJ. Protein A (virulence JJ and T-ccl l receptor, resulting in polyclonal
. 81
• 82 • factor) binds Fc-lgG, inhibiting complement T-cell activation .
• 83
activation and phagocytosis. Common ly Staphylococca l toxic shock syndrome
• 84
colonizes the nares, axilla, and groin. (TSS) presents as fever, vomiting, rash,
. 85 Causes: desquamation, shock, end-orga n failure. TSS
. l nA~mm~ torv cli~P.~~P. - ~kin in fP.rt ion ~ rt>~ult~ in t AST t AI T t hilirnhin A~~ori~tf'rl

8
Lock
s
Suspend
0
End Block
Item: 61 of ~ ,• Mark <::J [:::> ""I ~· ~'j
100 J.. Previous Next faiUI~S
LAb Notes Calculator

60
FA17 p451 .1
61
Skin infections
• 62
• 63
Bacterial infections
. 64 Impetigo Very superficial skin infection. Usually from S aureus or S pyogenes. Highly contagious. Honey-
. 65 colored crusting a.
• 66 Bullous impetigo Ill has bullae and is usually caused by S aure11s .
• 67
Erysipelas In fection involving upper dermis and superficia l lymphatics, usually from S pyogenes. Presents with
• 68
well-defined dema rcation between infected and normal skin B!.
. 69
. 70 Cellulitis Acute, painful, spreading infection of deeper dermis and subcutaneous tissues. Usually from
• 71 S pyogenes or S aureus. Often starts with a break in skin from trauma or another infection 1!].
· 72 Abscess Collection of pus from a walled-off infection within deeper layers of skin 0 . Offending organism is
• 73
almosl· always S aureus.
• 74
. 75
Necrotizing fasciitis Deeper tissue injury, usually from anaerobic bacteria or S pyogenes. Pain may be out of proportion
• 76
to exam find ings. Results in crepitus from methane and C02 production. "Flesh-eating bacteria."
· 77 Causes bullae and a purple color to the skin 1).
• 78 Staphylococcal scalded Exotoxin destroys keratinocyte attachments in stratum granulosum only (vs toxic epidermal
• 79 skin syndrome necrolysis, which destroys epidermal-derma l junction}. C haracterized by fever and genera lized
• 80 erythematous rash with sloughing of the upper layers of the epidermis (!I that heals completely.
. 81
® 1 ikolsky sign. Seen in newborns and chi ldren, adults wil·h renal insufficiency.
• 82
• 83
Viral infections
• 84 Herpes Herpes virus infections (HSVI and HSV2) of skin can occur anywhere from mucosal surfaces to
. 85 norma l skin. T hese include herpes labial is, herpes genita lis, herpetic whitlow CJ (fi nger}.
.
8
Lock
s
Suspend
0
End Block
Item: 61 of - ,• Mark -<J
P~v1ous
[:::> "'I ~ ·· ~
100 ~ N @xt Labl lues N o tes Calcula to r
A A
60
Herpes Herpes virus infections (HS I ancl ii SV2) of skin can occur anywhere from mucosa l surfaces to
61
0 62
normal skin. These include herpes labial is, herpes genita lis, herpetic whitlow CJ (finger).
0
63 Molluscum Umbilicated papules caused by a poxvirus. While frequently seen in children, it may be sexually
0
64 contagiosum lransmilled in adults.
65
Varicella zoster virus Causes ,·aricella (chickenpox) and zoster (shingles). Varicella presents with multiple crops of
0

66
0

lesions in ,·arious stages from vesicles to crusts. Zoster is a reactivation of the virus in dermatomal
67
distribution (unless it is disseminated).
0

0
68
0
69
Hairy leukoplakia Irregular, white, painless plaques on lateral tongue that cannot be scraped off . EB mediated.
0
70 Occurs in HIV-positi\'e patients, organ transplant recipients. Contrast with thrush (scrapable) and
0 71 leukoplakia (precancerous).
0 72 ;;:::::::===:;::;:::=;:==~ ..-
0
73
0
74
0
75
0 76
· 77
• 78

• 79
0
80
0
81
• 82
• 83
• 84
0
85
0 •

8
Lode.
s
Suspe-nd
8
End Bloc:k
Item: 61 of ~ ,• Mark <::J [:::> ""I ~· ~'j
100 J.. Previous Next LAbfaiUI~S Notes Calculator

60
FA17 p 449.1
61
Common skin disorders
• 62
• 63
Acne Multifactorial etiology- t sebum/androgen production, abnormal keratinocyte desquamation,
. 64 Propionibacterium acnes colonization of the pilosebaceous unit (comedones), and inflammation
. 65 (papules/pustules rJ, nodules, cysts). Treatment includes retinoids, benzoyl peroxide, and antibiotics .
• 66 Atopic dermatitis Pruritic eruption, commonly on skin flexures. Often associated with other atopic diseases (asthma,
• 67 (eczema) allergic rhin itis, food allergies); t serum lgE. Usua lly appears on face in infancy (] and then in
• 68
antecubital fossa m in children and adults.
. 69
. 70
Allergic contact Type IV hypersensitivity reaction that follows exposure to al lergen. Lesions occur at site of contact
• 71
dermatitis (eg, nickel m,
poison ivy, neomycin 1]).
· 72 Melanocytic nevus Common mole. Benign, but melanoma can arise in congenital or atypical moles. Intradermal nevi
• 73 are papular 1]. Junctional nevi are flat macules m.
• 74
Pseudofolliculitis Foreign body inflammatory facial skin disorder characterized by firm , hyperpigmented papules and
. 75
barbae pustules that are painful and pruritic. T.ocated on cheeks, jawline, and neck. Commonly occurs as
• 76
a result of shaving ("razor bumps"), primarily affects African-American males.
· 77
• 78 Psoriasis Papules and plaques with silvery scaling Cl, especially on knees and elbows. Acanthosis with
• 79 parakeratotic scaling (nuclei still in stratum corneum), Munro microabscesses. t stratum
• 80 spinosum, l stratu m granulosum. Auspitz sign (arrow in D )-pinpoint bleeding spots from
. 81 exposure of dermal papillae when scales are scraped off. Can be associated with nail pitting and
• 82 psoriatic arthritis.
• 83
Rosacea Inflammatory facial skin disorder characterized br erythematous papules and pustules D, but no
• 84
comedones. May be associated with faci al Rushing in response to external stimuli (cg, alcohol,
. 85
. heat). Phymatous rosacea can cause rhinophyma (bulbous deformation of nose).
8
Lock
s
Suspend
0
End Block
Item: 61 of ~ ,• Mark <::J [:::> ""I ~· ~'j
100 J.. Previous Next faiUI~S
LAb Notes Calculator


60 older persons.
61 Leser-Trelat sign 0 -sudden appearance of multiple seborrheic keratoses, indicating an underlyi ng
• 62
malignancy (eg, G l, lymphoid).
• 63
. 64
Verrucae Warts; caused by HPV. Soft, tan-colored, cauliflower-like papul es ~- Epidermal hrperplasia,
. 65
hyperkeratosis, koilocytosis. Condyloma acuminatum on genitals m.
• 66 Urticaria Hives. Pruritic wheals that form after mast cell degranulation l!]. Characterized by superficial
• 67 dermal edema and lymphatic channel dilation .
• 68
--·~====~~~~~~
. 69
. 70
• 71
· 72
• 73
• 74
. 75
• 76
· 77
• 78
• 79
. 80
. 81
• 82
• 83
• 84
. 85
.
8
Lock
s
Suspend
0
End Block
Item: 62 of - ,• Mark -<] 1:> ""'I ~· 1!';:'1
100 ~ Prev1o u s Next Labf a lu es Note s Calculator

60 •
A man who Is sexually active with other men presents to the clinic complaining of perianal lesions. He re ports 8 sexual partners in the past r~AJ
61 month and uses condoms "most of t he t ime. " The patient's lesions resemble those shown in the image.
• 62

• 63
. 64
. 65
• 66
. 67
• 68
. 69
. 70
• 71
• 72
• 73
• 74
. 75
• 76
• 77
• 78
• 79 What pathogen is most likely responsible for t his pat ient's condit ion?
. 80 :
. 81 A. Chlamydia trachomatis
• 82 B. Haemophilus ducreyi
• 83
C. Herpes simplex virus type 1
• 84
. 85 D . Herpes simplex virus type 2
. •
8
L.odt
s
Su~pl'nd
~
End Block
61
. 62
• 63
. 64
. 65
• 66
• 67
• 68
. 69
. 70
• 71
• 72
• 73
• 74
. 75
• 76
What pathogen is most likely responsible for t his pat ient's condition?
· 77
:
• 78
A. Chlamydia trachomatis
• 79
B. Haemophilus ducreyi
• 80
. 81 C. Herpes simplex virus type 1
• 82 D. Herpes simplex virus type 2
• 83
E. Human papillomavirus type 16
• 84
. 85 F. Human papillomavirus type 6
..
8
Lode.
s
Suspe-nd
8
End Bloc:k
Item: 62 of ~ ,• Mark <::J [:::> ""I ~· ~'j
100 J.. Previous Next LAb faiUI~S Notes Calculator

60 •
The correct answer is F. 53°/o chose this.
61
The image demonst rates condylomata acuminat a, which are mult iple whit e hyperkeratotic pedunculated verrucae clustered in the peri anal
62 reg ion . These genital warts are caused by t he human papillomavirus ( HPV), which is usually t ransmit ted by sexual contact . HPV t ypes 6 and
• 63 11 are common subtypes associat ed wit h wa1ts in the anogenital area . The extent of condylomata in t his case suggests that t he pat ient is
immunocomprom ised .
. 64
Human papillomavirus Genital wart Immunodeficiency Wart Anus Perineum Sexually transmitted infection Peduncle (anatomy) Hyperkeratosis Sex organ
. 65
• 66
A is not correct. 5°/o chos e this .
Chlamydia trachomatis is the pat hogen responsible fo r a series of infectious diseases. Types A-C cause a chronic conj unctivit is that leads t o
• 67
blindness. Types D-K cause uret hrit is, pelv ic inf lammat ory disease, ectopic pregnancy, and neonatal pneumonia or conj unctiv it is v ia vertica l
• 68 t ransm ission at birth (the newborn passes t hrough the infect ed birth canal) . Types Ll -L3 cause lymphog ranuloma venereum, which is an
. 69 acute lym phadenit is. None of t hese presentat ions is consistent with t he pedunculat ed verrucae seen in the image .
Pelvic inflammatory disease Lymphogranuloma venereum Chlamydia trachomatis Ectopic pregnancy Chlam(dia infection Urethritis Conjunctivitis Pneumonia Lymphadenopathy Vagina
. 70
Vertically transmitted infection Pathogen Infection Wart Pelvis Visual impairment Pregnancy Inflammation Infant
• 71
· 72 B is not correct. 5% chose this.
• 73 Haemophilus ducreyi is t he pat hogen responsible for chancroid, which manifest s as a painful gen it al u lce 1~ not white, hyperkeratot ic,
pedunculat ed verrucae .
• 74 Genital ulcer Chancroid Haemophilus ducreyi Ulcer (dermatology) Pathogen Wart Ulcer Peptic ulcer
. 75
C is not correct. 4 °/o chos e this .
• 76
Herpes simplex virus- 1 causes gingivost omat it is, keratoconjunct ivit is, temporal lobe encepha lit is, and herpes labialis. It is not responsible for
· 77 condyloma acum inat a.
• 78 Herpes labialis Encephalitis Herpes simplex Temporal lobe 9(philis Genital wart

• 79
D is not correct. 9°/o chos e this .
. 80 Herpes simplex virus-2 causes genita l herpes and neonatal herpes. The init ial lesions appear as ulcers, rat her t han mult iple pedunculated
. 81 verrucae .
Herpes simplex virus Genital herpes Herpes simplex Wart Neonatal herpes simplex Infant Ulcer (dermatology) Ulcer
• 82
• 83 E is not correct. 24°/o chos e this .
• 84 Human papillomavirus type 16 causes gen it al f lat warts and has been associated wit h cervica l dysplasia and carcinoma. It is not freq uent ly
associat ed with condyloma acuminat a.
. 85
Human papillomavirus Cervical intraepithelial neoplasia Verruca plana Genital wart Dysplasia 9(philis Carcinoma Papillomaviridae Cervix Cervical cancer Sex organ
.
8
Lock
s
Suspend
0
End Block
Item: 62 of ~ ,• Mark <::J [:::> ""I ~· ~'j
100 J.. Previous Next LAbfaiUI~S Notes Calculator

60
A is not co rrect . 5°/o chos e this.
61 Chlamydia trachomatis is the pat hogen responsible for a series of infectious diseases. Types A-C cause a chronic conj unctivit is that leads t o
62 blindness. Types D-K cause ureth rit is, pelv ic inf lammat ory disease, ectopic pregnancy, and neonatal pneumonia or conj unctiv it is v ia vertica l
• 63
t ransm ission at birth (the newborn passes t hrough the infect ed birth canal) . Types Ll -L3 cause lymphog ranuloma venereum, which is an
acute lym phadenit is. None of t hese presentat ions is consistent with t he pedunculat ed verrucae seen in the image .
. 64 Pelvic inflammatory disease Lymphogranuloma venereum Chlamydia trachomatis Ectopic pregnancy Chlam(dia infection Urethritis Conjunctivitis Pneumonia Lymphadenopathy Vagina
. 65 Vertically transmitted infection Pathogen Infection Wart Pelvis Visual impairment Pregnancy Inflammation Infant
• 66
B is not correct. 5% chose t his .
• 67
Haemophilus ducreyi is t he pat hogen responsible for chancroid, which manifest s as a painful gen it al u lce r~ not white, hyperkeratot ic,
• 68 pedunculat ed verrucae.
. 69 Genital ulcer Chancroid Haemophilus ducreyi Ulcer (dermatology) Pathogen Wart Ulcer Peptic ulcer

. 70
C is not correct. 4 °/o chose this .
• 71 Herpes simplex virus- 1 causes gingivost omat it is, keratoconj unct ivit is, temporal lobe encepha lit is, and herpes labialis. It is not responsible for
· 72 condyloma acum inat a.
Herpes labialis Encephalitis Herpes simplex Temporal lobe 9(philis Genital wart
• 73
• 74 D is not correct. 9°/o chose t his .
. 75 Herpes simplex virus-2 causes genita l herpes and neonatal herpes. The init ial lesions appear as ulcers, rat her t han mult iple pedunculated
verrucae .
• 76
Herpes simplex virus Genital herpes Herpes simplex Wart Neonatal herpes simplex Infant Ulcer (dermatology) Ulcer
· 77
• 78
E is not correct. 24 °/o chose this .
Human papillomavirus type 16 causes gen it al f lat warts and has been associated wit h cervica l dysplasia and carcinoma. It is not freq uent ly
• 79
associat ed with condyloma acuminat a.
• 80 Human papillomavirus Cervical intraepithelial neoplasia Verruca plana Genital wart Dysplasia syphilis Carcinoma Papillomaviridae Cervix Cervical cancer Sex organ

. 81
• 82
Bottom Line :
• 83
Human papillomavirus ( HPV) t ypes 6 and 11 are the most common cause of condylomata acuminata. These t wo types rarely t ransform into
• 84
malignancy. HPV types 16 and 18 are more commonly associated wit h cervica l dysplasia.
. 85 Human papillomavirus Cervical intraepithelial neoplasia Genital wart Malignancy Dysplasia Cancer Papillomaviridae Cervix Cervical cancer
.
8
Lock
s
Suspend
0
End Block
Item: 62 of - ,• Mark -<] 1:> ""'I ~· 1!';:'1
100 ~ Prev1o u s Next Labf a lu es Note s Calculator

60 FA17 p 180.1
61 Sexually transmitted infections
62
DISEASE CLINICAL FEATURES ORGANISM
• 63
AIDS Opportunistic infections, Kaposi sarcoma, Ill
. 64
. 65
lymphoma
• 66 Chancroid Painful genital ulcer with exudate, inguinal Haemophilus ducreyi (it's so painful, you "do
. 67 adenopathy en " )
• 68
Chlamydia Urethritis, cen·icitis, epididrmitis, Chlamydia trachomatis (0 - K)
. 69
conjuncti\'itis, reacti\e arthritis, PID
. 70
• 71 Condylomata G enital warts, koilocytes TIP -6and-ll
• 72 acuminata
• 73 Genital herpes Painful penile, vulvar, or cervical vesicles and li S -2, less commonly IISV-1
• 74 ulcers; can cause systemic symptoms such as
. 75
fever, headache, myalgia
• 76
• 77
Gonorrhea Urethritis, cervicitis, PID, prostatitis, eisseria gonorrhoeae
• 78
epididymitis, arthritis, creamy pllrlllelll
• 79
discharge
• 80 Granuloma inguinale Painless, beefy red ulcer that bleeds rce~dily 0 11 Klebsiella (Calymmatobacterium) granulomatis;
. 81 (Donovanosis) contact fJ cytoplasmic Donovan bodies (bipolar staining)
• 82 Not common in US seen on microscopy
• 83
• 84
. 85
. •
8
L.odt
s
Su~pl'nd
~
End Block
Item: 62 of - ,• Mark -<J [:::> "'I ~ · ~
100 ~ P~v1ous N @xt Labl lues N o tes Calcula to r
A A
60
61
Gonorrhea Urethritis, cervicitis, PID, prostatitis, eisseria gonorrhoeae
62 epididymitis, arthritis, cream)' pmulcnl
• 63 discharge
. 64 Granuloma inguinale Painless, beefy red ulcer that bleeds readil) on Klebsiella (Calymmatobacterium) grarw lomatis;
. 65 (Donovanosis) contact fJ cytoplasmic Dono,·an bodies (bipolar staining)
• 66 1 ot common in US seen on m1croscopy
• 67
• 68
. 69
. 70
• 71
• 72
• 73
• 74
. 75
Hepatitis B Jaundice IIBV
• 76 Lymphogranuloma Infection of lymphatics; pa inless genital ulcers, C trachomatis (Ll- L3)
· 77 venereum painful lymphadenopathy (ic, buboes)
• 78
Primary syphilis Painless chancre Treponema pallidum
• 79
• 80
Secondary syphilis Fever, lymphadenopathy, skin rashes,
. 81 condylomata lata
• 82 Tertiary syphilis Gummas, tabes dorsalis, general paresis, aortitis,
• 83 Argyll Robertson pupil
• 84
Trichomoniasis Vaginitis, strawberry cervix, motile in wet prep Trichomorws vagina/is
. 85
..
8
Lode.
s
Suspe-nd
8
End Bloc:k
Item: 63 of - ,• Mark -<] 1:> ""'I ~· 1!';:'1
100 ~ Prev1o u s Next Labf a lu es Note s Calculator

60 •
A 32-year-old woman presents t o the clinic with sore throat, fat igue, and malaise for t he past month. The pa t ient denies any other
61 associated symptoms. She does not have any significant past medical history, other than occasional treat ment fo r t richomoniasis and
62 bacterial vaginosis. Social history is signif icant for occasional alcohol use, no illicit drug use, and mult iple sexual partners. Physical
examination is significant for generalized lym phadenopathy and the nontender lesions seen in t he image. Patient denies any arthritic symptoms .
• 63
. 64
. 65
• 66
. 67
• 68
. 69
. 70
• 71
• 72
• 73
• 74
. 75
• 76 Image cowtesy of CDC/Robett Sumpter
• 77
• 78 If left untreated, which of the fo llowing is a possible sequela of the most likely cause of her symptoms?
• 79 :
. 80 A . AIDS
. 81 B. Aortic insufficiency
• 82
C. Cervical carcinoma
• 83
• 84 D. Fitz- Hugh-Curtis syndrome
. 85 E. Hepatic carcinoma and cirrhosis
. •
8
L.odt
s
Su~pl'nd
~
End Block
Item: 63 of - ,• Mark -<J [:::> "'I ~ · ~
100 ~ P~v1ous N @xt Labl lues N o tes Calcula to r

60 and th e nontender lesions seen in t he image. Pat ient denies any arth rit ic symptoms. •
61
62
. 63
. 64
. 65
• 66
• 67
• 68
. 69
. 70
• 71
• 72
• 73
• 74
. 75
If left untreated, which of the fo llowing is a possible sequela of th e most likely cause of her symptoms?
• 76
:
· 77
A. AIDS
• 78
B. Aort ic Insufficiency
• 79
• 80 C. Cervical carcinoma
. 81
D. Fitz-Hugh-Curtis syndrome
• 82
E. Hepatic carcinoma and cirrhosis
• 83
• 84 F. Pelvic inflammat ory disease
. 85
..
8
Lode.
s
S uspe-nd
8
End Bloc:k
Item: 63 of ~ ,• Mark <::J [:::> ""I ~· ~'j
100 J.. Previous Next LAb faiUI~S Notes Calculator

60 •
61
The correct answer is B. 54% chos e this.
62 This patient most likely has secondary syphilis . The clues are systemic symptoms with t he presence of nonprurit ic, re d, discrete macules
63 t hat progress to papular lesions. These lesions are classically found on t he soles and palms; ot her secondary syphilis signs include
. 64 condylomata lata (present in only 10% of pat ients), and silver-gray mucosal lesions with surrounding erythema . Primary syphilis is a
painless, single papule t hat can often go unnot iced by females because t he primary sit e is on t he labia or cerv ix. I f left unt reat ed, secondary
. 65
syphilis can enter the latent phase; approximat ely 33% of patients will progress f rom t he lat ent phase t o t ertiary syphilis. I n tertiary
• 66 syphilis, t he presentation includes gummatous syphilis, cardiovascular syphilis, or neurosyphilis. Cardiovascular syphilis can occur in 10% of
• 67 untreat ed patients, and characteristically resu lts in an aortic arch or ascending aort ic aneurysm, result ing from dest ruct ion of the vasa
vasorum and subsequent necrosis of t he media layer of t he aorta. Aortic aneurysms t hat include the a01tic root can cause aortic insufficiency
• 68
due to dilat ion of t he aortic valve ring .
. 69 Syphilis Itch Papule Condylomata lata Neurosyphilis Cervix Erythema Aortic insufficiency vasa vasorum Aortic aneurysm Aorta Primary syphilis Aortic valve Cardiovascular syphilis Secondary syphilis
. 70 Aortic arch Gumma (pathology) Genital wart Necrosis Aneurysm Tertiary syphilis Circulatory system Cutaneous condition Mucous membrane

• 71
A is not correct . 8°/o chose this.
· 72 AIDS is caused by HIV infection . While laboratory evidence of HI V infect ion is one component in t he diagnosis of AIDS, other clinical or
• 73 laboratory cri teria are required for t he diagnosis as def ined by t he CDC. These ot her crit eria include t he presence of certain opport unistic
• 74 infections or CD4+ cells below a cert ain t hreshold .
HIV HIV/AIDS CD4 Centers for Disease Control and Prevention Opportunistic infection T cell
. 75
• 76 C is not correct. 10% chos e this .
· 77 Human papillomaviruses ( HPVs) can result in both benign and malignant neoplasms. There are over 100 t ypes of HPV ident ified, and several
t ypes have been associated wit h anogenital and cervical lesions. Types 6 and 11 produce benign exophyt ic warts (condylomat a acum inata),
• 78
whereas types 16, 18, 3 1, 33, and 35 are associat ed with dysplasia and neoplasia.
• 79 Genital wart Human papillomavirus Neoplasm Malignancy Dysplasia Cancer Benign tumor Cervix Cervical cancer Neck Benignity

. 80
D is not correct. 11% chose thi s •
. 81 Fit z-Hugh-Curtis syndrome is another complication of Chlamydia trachomatis or Neisseria gonorrhoeae infectio n t hat can affect both men and
• 82 women. Infect ion of t he liver capsule results in a present at ion of right upper quadrant pain. Patient denies arthrit is (a classic manifestation of
• 83
disseminat ed gonorrhea, alongside the rash) which makes gonorrh oeae infection less likely .
Chlamydia trachomatis Fitz-Hugh-Curtis syndrome Gonorrhea Neisseria gonorrhoeae Chlam(dia infection Arthritis Neisseria Chlamydia (genus) Liver Quadrant (abdomen) Infection
• 84
. 85 E is not correct. 6 °/o chose this •
. Hepatic carcinoma and cirrh osis can be seauelae of hepat it is B C and D infections This pat ient does not manifest symptoms of hepatit is
8
Lock
s
Suspend
0
End Block
Item: 63 of ~ ,• Mark <::J [:::> ""I ~· ~'j
100 J.. Previous Next LAbfaiUI~S Notes Calculator

60
D is not correct. 11% chose this .
61 Fit z-H ugh-Curtis syndrome is another complication of Chlamydia trachomatis or Neisseria gonorrhoeae infection t hat can affect both men and
62 women. I nfect ion of t he liver capsule results in a present at ion of right upper quadra nt pain. Pa tient denies arthrit is (a classic manifestation of
disseminat ed gonorrhea, alongside the rash) which makes gonorrhoeae infection less likely.
63
Chlamydia trachomatis Fitz-Hugh-Curtis syndrome Gonorrhea Neisseria gonorrhoeae Chlam(dia infection Arthritis Neisseria Chlamydia (genus) Liver Quadrant (abdomen) Infection
. 64
. 65 E is not correct. 6°/o chose this .
• 66
Hepatic carcinoma and cirrh osis can be sequelae of hepat it is B, C, and D infections. This pat ient does not man ifest symptoms of hepatit is,
such as right upper quadrant t enderness, pain, liver swelling, ascites, or j aundice. Furthermore, although hepat itis can be present in
• 67 secondary syphilis, it does not typically cause hepat ic carcinoma or cirrhos is.
• 68 Ascites Cirrhosis Jaundice Hepatitis 8 Syphilis Quadrant (abdomen) Hepatitis Liver Sequela Carcinoma

. 69 F is not correct. 11% chose this .


. 70 Pelvic inf lammat ory disease is a possible sequela of Chlamydia trachomatis or Neisseria gonorrhoeae infection of the cervix . Pat ients will have
. 71 signs and symptoms of lower abdominal pain, nausea, vomit ing, fever, vaginal discharge/uterine bleeding, and dyspare unia. The classic sign is
t he "chandelier sign, " or cervical motion t enderness.
· 72
Pelvic inflammatory disease Dyspareunia Chlamydia trachomatis Neisseria gonorrhoeae Cervical motion tenderness Cervix Chlamydia infection Nausea Vomiting Chlamydia (genus) Abdominal pain Fever
• 73
Inflammation Infection
• 74
. 75
• 76 Bottom Line:
· 77 Secondary syphilis can present with widely dist ribut ed nonprurit ic, red, discret e, macules on t he t runk or proximal extremities, but is
classically associat ed wit h being present on t he palms and soles. Ot her common sympt oms and signs include feve r, generalized
• 78
lymphadenopathy, malaise, condylomat a lat a, and silver-gray mucosal lesions .
• 79 Itch Condylomata lata Syphilis Lymphadenopathy Secondary syphilis Malaise Genital wart Cutaneous condition Fever
• 80
. 81
• 82 l@ljl'il·1i•J for year:[2017 • J
FIRS T AID FACTS
• 83
• 84
FA17 p 143.1
. 85
. Syphilis Caused by spirochete Treponema pallidwn.
8
Lock
s
Suspend
0
End Block
Item: 63 of ~ ,• Mark <::J [:::> ""I ~· ~'j
100 J.. Previous Next LAbfaiUI~S Notes Calculator

60
FA17 p 143.1
61
Syphilis Caused by spirochete Treponema pallidwn.
62
63
Primary syphilis Localized disease presenti ng with painless chancre fJ. If available, use dark-field microscopy to
. 64
visualize treponemes in fluid from chancre Ill VORL ED in ~ 80%.
. 65 Secondary syphilis Disseminated disease with constitutional symptoms, macu lopapu lar rash 1!1 (including palms [!]
• 66 and soles), condylomata lata 0 (smooth, moist, painless, wart-like white lesions on genitals),
• 67 lymphadenopathy, patchy hair loss; also con firmable with dark-field microscopy.
• 68 Serologic testing: VORL/RPR (nonspecific), confirm diagnosis with specific test (eg, FTA-ABS).
. 69
Secondary syphilis = Systemic. Latent syphilis (® serology without symptoms) may follow.
. 70
• 71
Tertiary syphilis Gummas I] (chronic granulomas), aortitis (vasa vasorum destruction), neurosyphilis (tabes dorsalis,
· 72
"general paresis"), Argyll Robertson pupil (constricts with accommodation but is not reactive to
• 73
light; also called "prostitute's pupil" since it accommodates but does not react) .
• 74 Signs: broad-based ataxia,® Romberg, Charcot joint, stroke without hypertension .
. 75 For neurosyphilis: test spinal fluid with VORL, ITA-ABS, and PCR.
• 76 Congenital syphilis Presents with facial abnormalities such as rhagades (l inear scars at angle of mouth, black arrow
· 77 in [!1), snufAes (nasal discharge, red arrow in [!1), saddle nose, notched (Hutchinson) teeth Cl,
• 78
mulberry molars, and short maxilla; saber shins; C Vlli deafness.
• 79
To prevent, treat mother early in pregnancy, as placental transmission typically occurs after first
• 80
trimester.
. 81
• 82
• 83
• 84
. 85
.
8
Lock
s
Suspend
0
End Block
Item: 63 of - ,• Mark -<] 1:> ""'I ~· 1!';:'1
100 ~ Prev1o u s Next Labf a lu es Note s Calculator

60
- -
For neurosyphilis: test spinal fluid \\·ith VORL, FTA-ABS, and PCR.
61
Congenital syphilis Presents with fa cial abnormalities such as rh;1gades (linea r scars at angle of mouth, black arrO\\
62
in [!!), snuffles (nasal discharge, red arrO\\ in [!!), saddle nose, notched (Hutchinson) Leeth CJ,
63
. 64
mulberry molars, and short maxilla; saber shins; C 1 Ill deafn ess .
. 65 To prevent. treat mother early in pregnancy, as placental transmission typically occurs after first
• 66 trimester.
. 67
• 68
. 69
. 70
• 71
• 72
• 73
• 74
. 75
• 76
• 77
• 78
• 79
• 80
. 81
• 82
• 83
• 84
. 85
. •

8
L.odt
s
Su~pl'nd
~
End Block
Item: 64 of - ,• Mark -<] 1:> ""'I ~· 1!';:'1
100 ~ Prev1o u s Next Labf a lu es Notes Calculator

60 •
A 13-month-old girl is brought to the pediatri cian because of fever and a sore throat. Her mother confides t hat the child has always been r~AJ
61 healthy and has never needed any immunizat ions. However, over the past several days, the child is less active and refusing to eat. Today
62 her neck appears swollen. Examination reveals a grayish membrane-like material in the posterior pharynx. The pathogen most likely
responsible for this child's illness is normally not virulent unless it undergoes a certain process.
63
. 64
Which of the following organisms can increase its pathogenicity by the same process as the bacteria causing the condition in the question?
. 65
• 66 :
A. Coagulase-negative staphylococci
. 67
• 68 B. Coagulase-positive staphylococci
. 69 C. Gram-negative diplococci
. 70
D. Group A streptococci
• 71
• 72 E. Group B streptococci
• 73
• 74
. 75
• 76
• 77
• 78
• 79
. 80
. 81
• 82
• 83
• 84
. 85
. •

8
L.odt
s
Su~pl'nd
~
End Block
Item: 64 of ~ ,• Mark <:::1 t::> ""I ~· ~'j
100 J.. Previous Next LAbfaiUI~S Notes Calculator

I
60
The correct a nswer is 0. 37°/o chose t his .
61
The quest ion describes infection by Co1ynebacterium diphtheriae . This child is particularl y at risk because she has not received the
62 appropriate immunization (the dipht heria -pertussis-tetanus vaccine). Classically, C. diphtheriae infection produces a sore throat, difficult y
63 swallowing, swollen neck, and feve r. Ot her sequelae include myoca rdit is and peripheral nerve palsies. Physical examination may reveal t he
64
presence of a grayish pseudomembrane in t he pharynx. C. diphtheriae's virulence fact or, a t wo-subunit exotoxin responsible fo r these
symptoms, is deri ved f rom a temperate bacteriophage. Bact eri ophages are v iruses that only infect bacteria. Once a bacteriophage infects a
. 65 host, it may either re plicat e immed iately and lyse t he bact eri um (a lyt ic bact eri ophage), or incorporate its DNA into the bacteria l genome and
• 66 ent er a proviral lat ent st at e (a temperate bacteriophage) . Certain temperate bacteriophages ca rry genes that t ransfo rm t he phenot ype of t he
• 67 host once they become incorporat ed . This is known as lysogenic conversion. Only strains of C. diphtheriae t hat have been lysogenized by a
phage encod ing an exotoxin are capable of causing dipht heria. Other examples of lysogenic conversion include st ra ins of Streptococcus
• 68 pyogenes, t he grou p A st reptococcal species t hat causes sca rlet feve r~ and strains of Clostridium botulinum, the gram -positive bacillus that
. 69 causes botulism .
Bacteriophage Clostridium botulinum Scarlet fever Streptococcus pyogenes Corynebacterium diphtheriae Botulism Myocarditis Diphtheria Virulence factor Lysogenic cycle Lysis Exotoxin Pharynx
. 70
Gram-positive bacteria Phenotype Virulence Streptococcus Lytic cycle Virus Corynebacterium Genome Bacteria Fever Bacillus Clostridium Botulinum toxin Infection vaccine DNA Sore throat Immunization
• 71
Lysogenic conversion Bacillus (shape)
· 72
• 73 A is not correct . 11% chos e this.
• 74 Coagulase- negative staphylococci include Staphylococcus epidermidis and Staphylococcus saprophyticus. Staphylococcus epidermidis is often
. 75
involved in nosocom ial infections of prost het ic devices (joints, heart va lves, and intravenous lines) . S. saprophyticus is a common cause of
uri nary t ract infection in young, sexually active women. I t is resistant to novobiocin, as opposed t o S. epidermidis, wh ich is novobiocin -
• 76 sensit ive. These bacteria do not undergo lysogenic conversion.
· 77 Urinary tract infection Staphylococcus epidermidis Staphylococcus Novobiocin Staphylococcus saprophyticus Hospital-acquired infection Intravenous therapy Urinary system Bacteria Prosthesis Heart valve

• 78 Infection

• 79
B is not correct. 18% chose this .
. 80 The on ly gram- posit ive staphylococca l species that is also coagulase- posit ive is Staphylococcus aureus. Direct S. aureus inoculat ion causes
. 81 pneumonia, ost eomyelit is, acute bacteria l endocardit is, skin infect ion, and bacterem ia/sepsis. I ngestion of preformed exotoxin causes
• 82
gast roent eri t is. S. aureus's exotoxin is also responsible fo r t oxic shock syndrome and scalded skin synd rome. S. aureus does not undergo
lysogenic conversion .
• 83 Osteomyelitis Toxic shock syndrome Gastroenteritis Gram-positive bacteria Endocarditis Staphylococcus aureus Lysogenic cycle Pneumonia Exotoxin Staphylococcus Infective endocarditis Skin infection
• 84 Staphylococcal scalded skin syndrome
. 85
. C is not co rrect. 23% chos e this .
8
Lock
s
Suspend
0
End Block
Item: 64 of ~ ,• Mark <:::1 t::> ""I ~· ~'j
100 J.. Previous Next LAb faiUI~S Notes Calculator

60
C is not co rrect. 23% chose this .
61
Gram-negative diplococci include Neisseria meningitidis and Neisseria gonorrhoeae. N. meningitidis causes meningit is, Wat erhouse-
62 Fri derichsen synd rome, and meningococcemia. N. gonorrhoeae causes urethrit is, cerv icit is, and pelvic inflammatory disease. These organisms
63 are not known to undergo lysogen ic conversion.
Pelvic inflammatory disease Waterhouse-Friderichsen syndrome Neisseria meningitidis Gram-negative bacteria Diplococcus Meningococcemia Neisseria gonorrhoeae Urethritis Meningitis
64
Meningococcal disease Cervicitis Neisseria
. 65
• 66
• 67
• 68
. 69
E is not correct. 11°/o chose this .
Group B streptococci (GBS) include Streptococcus agalactiae. These are gram-posit ive, 13-hemolyt ic, cat alase-negative cocci. GBS are among
t he most common causes of neonat al mening it is, neonata l pneumonia, and neonat al sepsis. The pathogenesis of GBS does not depend on a
t emperate phage t o increase its pathogenicity.
Streptococcus agalactiae Sepsis Meningitis Gram-positive bacteria Pneumonia Coccus Streptococcus Temperateness (virology) Pathogen Neonatal sepsis Bacteriophage Neonatal meningitis Pathogenesis
I
. 70 Infant Catalase
. 71
· 72
Bottom Line :
• 73
The pat hogenesis of certa in bact eria is dependent on t he expression of a temperate bacteriophage exot oxin, a process known as lysogen ic
• 74
conversion . Examples include diphtheria (Corynebacterium diphtheriae), scarl et fever (Streptococcus pyogenes), botulism (Clostridium
. 75 botulinum), and cholera (Vibrio cholerae) .
Clostridium botulinum Bacteriophage Scarlet fever Vibrio cholerae Streptococcus pyogenes Corynebacterium diphtheriae Botulism Diphtheria Exotoxin Lysogenic cycle Cholera Streptococcus Pathogenesis
• 76
Vibrio Corynebacterium Bacteria Botulinum toxin Clostridium Fever
· 77
• 78

• 79
l@ljl'il·1i•J for yea r:[ 2017 • J
• 80 FIRST AID FACTS

. 81
• 82 FA17 p 128 .1
• 83 Bugs with exotoxins
• 84 BACTERIA TOXIN MECHANISM MANIFESTATION
. 85
Inhibit protein synthesis
.
8
Lock
s
Suspend
0
End Block
Item: 64 of - ,• Mark -<] 1:> ""'I ~· 1!';:'1
100 ~ Prev1o u s Next Labf a lu es Notes Calculator

60 • •
FA17 p 128.1
61
Bugs with exotoxins
62
BACTERIA TOXIN MECHANISM MANIFESTATION
63
64
Inhibit protein synthesis
. 65 Corynebacterium Diphtheria toxina Phar) ngitis '' ith pseudomembranes in throat
• 66 diphtheriae Inactivate elongation factor and severe lymphadenopathy (bull neck)
. 67
Pseudomonas Exotoxin N (EF-2) I lost cell death

I
• 68
aeruginosa
. 69
. 70 Shigella spp. Shiga toxin (ST)3 Gl mucosal damage -+ dysentery; ST also
• 71 Inactivate 60S ribosome by enhances cytokine release, causing hemolytic-
• 72 removing adenine from uremic syndrome (HUS)
• 73
Enterohemorrhagic Shiga-like toxin rR A SLT enhances cytokine release, causing I-I US
• 74
E coli (EHEC) (SLT)3 (prototypically in EHEC serotype 0157:1-17).
. 75
Unlike Shigella, F.HEC does not invade host
• 76
cells
• 77
• 78 Increase flUid secretion
• 79 Enterotoxigenic Heat-labile Overactivates adenylate Watery diarrhea: "labile in the Air (Adenylate
. 80 E coli (ETEC) toxin (LT)3 cyclase (t cAMP) -+ f Cl- cyclase), stable on the Ground (Guanylate
. 81
secretion in gut and H20 cyclase)"
• 82
efnu x
• 83
Heat-stable Overactivates guanylate
• 84
toxin (ST) cyclase (f cC ~IP)
. 85
. - l resorption of aCI •
8
L.odt
s
Su~pl'nd
~
End Block
Item: 64 of - ,• Mark -<] 1:> ""'I ~· 1!';:'1
100 ~ Prev1o u s Next Labf a lu es Notes Calculator

60 • Bacillus anthracis Edema toxin 3 Mimics the adenylate Li kely responsible for characteristic edematous •
61 cyclase enzyme (t cAl\ IP) borders of black eschar in cutaneous anthrax
62
Vibrio cholerae C holera toxina Overacti,·ates adenylate Voluminous "rice-water" diarrhea
63
c~ clase (t cA\I P) by
64
permanently activating C,
. 65
- t CI- secretion in gut
• 66
. 67
and l-1 0 efAux
• 68 Inhibit phagocytiC ability
. 69
Bordetella pertussis Pertussis toxin;• Q, eracti,·ates adenylate Whooping cough-child coughs on e:\piration
. 70
cyclase (t c \ l P) by and ..whoops" on inspiration (toxin may

I
• 71
disabling C;. impairing not actually be a cause of cough; can cause
• 72
phagocytosis lo permit "100-day cough" in adults)
• 73
survival of microbe
• 74
. 75 Inhibit release of neurotransmitter
• 76 Clostridium tetani Tetanospasmi na Both are proteases that Spastic paralysis, risus sardonicus, and "lockjaw";
• 77
cleave S ARE (soluble toxin prevents release of inhibitory (CABA and
• 78
SF attachment glycine) neurotransmitters from Renshaw cells
• 79
protein receptor), <1 set in spinal cord
• 80
. 81
Clostridium Botulinum toxin3 of proteins required for Flaccid paralysis, floppy baby; toxin prevents
• 82
botulinum neurotransm itter release release of stimulatory (ACh) signals at
• 83 via vesicular fusion neuromuscular junctions - flaccid paralysis
• 84 :•An B to\ in (aJ..a, two-component toxin [or three for anthrax]) with B enabling binding and triggering uptake (endocytosis)
. 85 of the acti'e A component. The A components are usually ADP ribosyltransferases; others ha,·e enzymatic activities as listed
. •
8
L.odt
s
Su~pl'nd
~
End Block
Item: 64 of - ,• Mark -<] 1:> ""'I ~· 1!';:'1
100 ~ Prev1o u s Next Labf a lu es Notes Calculator

60
FA17 p 126.1
61
Bacterial genetics
62

63
Transformation Competent bacteria are able to bind and import Degraded
uncombined \ _ frRec1pteot DNA

---... ret
64 short pieces of environmental naked bacterial DNA
. 65 chromosomal 01\ (from bacterial cell ~
""
0 Donor DNA

• 66 lysis). The transfer and expression of newly


Naked DNA Recipient cell Transformed cell
. 67 transferred genes is called transformation .
• 68 A feature of many bacteria, especially S
. 69 pneumoniae, II influen::ae type B, and
. 70 1\'eisseria (S Hi~). Any 0 'A can be used.
• 71 Adding deoxyribonuclease to environment
• 72
will degrade naked 01\A in medium -+ no
• 73
transformation seen .
• 74
. 75
Conjugation
• 76 F+ X F' F"' plasmid contains genes required for sex pi lus Single strand
• 77
• 78
• 79
• 80
and conjugation. Bacteria without this plasmid
are termed F-. Sex pilus on F"' bacteri11111
contacts F- bacterium. A single strand
of plasmid DNA is transferred across the
Plasmid ol
Sex pilus

~
-+
transferred

el~ -+
0 0
@ ~
I
f' cell f· cell Pcell f· cell f' cell Pcell
. 81
conjugal bridge ("mating bridge"). 'Jo I ransfcr Cl
• 82
of chromosomal Dr A.
• 83
• 84
Hfr X F- F"' plasmid can become incorporated into
. 85 bacterial chromosomal D A. termed high-
. frPntll'nr,· rpromhin::~tinn fllfr) r·PII Tr:m~fPT •
8
L.odt
s
Su~pl'nd
~
End Block
Item: 64 of - ,• Mark -<J [:::> "'I ~ · ~
100 ~ P~v1ous N @xt Labl lues N o tes Calculato r

60
A

Transduction
.. - .
- -. A

61
62
63
Generalized A "packaging" e\·ent. Lytic phage infects
bacterium, leading to cleavage of bacterial
Lytic
phage )
! Cleavage of
[ 'Bacteria bacterial DNA
Bacterial DNA package
in phage caps~d

DNA. Parts of bacterial chromosomal DNA


64
. 65 may become packaged in phage capsid. Phage

• 66
• 67
• 68
. 69
infects another bacterium, transferring these
genes. ..•••
. . •
. 70
• 71 Release of new phage Infects other Phage's genes
from lysed cell bacteria transferred Cl
• 72
• 73
• 74
Specialized n "excision" e\·ent. Lysogenic phage infects
bacterium; viral DNA incorporates into
lysogeniC
phage
rJ, Viral DNA . Viral DNA .
InCorporates In
[ 'Bacteria bacterial DNA
Phage parttcles
carry bacterial DNA
. 75 bacterial chromosome. When phage D A
• 76 is excised, flanking bacterial genes may be 'YI - 0
· 77 excised with it. DNA is packaged into phage
capsid and can infect another bacteriu m.

- -
• 78

• 79 Genes for the following 5 bacterial toxins are

I
• 80 encoded in a lysogenic phage (ABCD'S): Croup
. 81
A strep eryth rogenic toxin, Botulinum toxin,
• 82 Release of new phage Infects other Genes different from
C holera toxin, Diphtheria toxin, Shiga toxin. from lysed cell bacteria donorandreop~nt Cl
• 83
• 84 Transposition Segment of 0 1A (eg, transposon) that can Plasmid
. 85
..
"jump" (excision and reintegration) from
I ~· ~ ~I ~ (. \~
8
Lode.
s
S uspe-nd
8
End Bloc:k
Item: 64 of - ,• Mark -<] 1:> ""'I ~· 1!';:'1
100 ~ Prev1o u s Next Labf a lu es Notes Calculator

60
Transposition Segment of DNA (eg, transposon) that can Plasmid
61
"jump" (excision and reintegration) from ~
62
one location to another, can transfer genes
63
64
from plasmid to chromosome and 'ice versa. ~ I Integration of genes
Transposons
. 65 \Vhen excision occurs, mar include some
Bactenal -
• 66 Aanking chromosomal 0 1 , which can be DNA -
. 67 incorporated into a plasmid and transferred Tar~ site
• 68 to another bacterium (eg, vanA gene from
. 69 \'ancomycin-resistant Enterococcus to
. 70 S aureus).
• 71
• 72
FA17 p 135.1
• 73
• 74 Corynebacterium Gram$ rod; transmitted via respiratory Coryne = club shaped.
. 75 diphtheriae droplets. Causes diphtheria \'ia exotoxin Black colonies on cystine-tellurite agar.
• 76 encoded by ~-proph age. Potent exotoxin ABC DEFG:
• 77 inhibits protein synthesis via ADP-ribosylation A DP-ribosylation
• 78 of EF-2 . ~-pro phage
• 79 Symptoms include pseudomembranous Corynebacterium
• 80 pharyngitis (grayish-white membrane tl) Diphtheriae
. 81 with lymphadenopathy, myocarditis, and Elongation Factor Z
• 82 Granules
arrhythmias .
• 83
Lab diagnosis based on gram $ rods with
• 84
metachromatic (blue and red) granule~ and
. 85
. $ Elek test for toxin. •
8
L.odt
s
Su~pl'nd
~
End Block
Item: 65 of - ,• Mark -<] 1:> ""'I ~· 1!';:'1
100 ~ Prev1o u s Next Labf a lu es Note s Calculator

60 •
A 38-year-old man comes to t he emergency department complaining of recurrent episodes of int ense fevers and headaches that, accord ing r~AJ
61 to the patient, do not appear to occur in any cyclic pattern. He also mentions that he wakes up with his sheet soaked through some nights
62 and attributes some of his "generalized weakness" to lack of sleep during these episodes. His symptoms began about 1 week after he
returned from a trip to Africa several weeks ago. Physical examination reveals hepatosplenomegaly. Imaging of the brain shows signs of
63
significant cerebral involvement.
64
. 65
Which of the following parasites most likely caused this patient's symptoms?
• 66
:
. 67
A. Plasmodium falciparum
• 68
. 69
B. Plasmodium knowlesi
. 70 C. Plasmodium malariae
• 71 D. Plasmodium ovale
• 72
E. Plasmodium vivax
• 73
• 74
. 75
• 76
• 77
• 78
• 79
. 80
. 81
• 82
• 83
• 84
. 85
. •

8
L.odt
s
Su~pl'nd
~
End Block
Item: 6S of ~ ,• Mark <::J [:::> ""I ~· ~'j
100 J.. Previous Next faiUI~S
LAb Notes Calculator

60 •
61 The correct ans wer is A. 66°/o chose this.
62
This patient has sympt oms that are suggestive of malaria given his recent t ravel hist ory. Four members of the Plasmodium genus of prot ozoa
common ly infect humans. All are t ransmitt ed through t he female Anopheles mosquito. Of these four species, P. falciparum is known for it s
63 cerebral involvement (cerebral malaria), which can lead to coma and deat h. Of not e, P. falciparum is unique among st rains of mala ria because
64 it causes irregu lar, not cyclic, fevers.
Protozoa Malaria Mosquito Anopheles Plasmodium falciparum Plasmodium Genus Coma Species
65
• 66 B is not correct. 6 % chose this .
• 67 Plasmodium knowlesi is a sim ian ma laria parasite that prima ri ly infect s macaques, although it has been reported to infect humans in
• 68
southeast Asia. There are reports of cerebral involvement in monkeys, and of isolat ed fata l human cases .
Plasmodium knowlesi Malaria Simian Plasmodium Parasitism Southeast Asia Plasmodium falciparum Macaque Asia
. 69
. 70 Cis not correct. 14% chos e this .
• 71
Plasmodium malariae infection causes a 72-hour cyclic feve r. It does not cause cerebral ma laria .
Plasmodium malariae Malaria Plasmodium Fever Infection
· 72
• 73
D is not correct. 6°/o chose this .
Plasmodium ovale infection causes a 48-hour cycl ic fever. A unique feat ure of P. vivax and P. ovale organisms is that t hey can fo rm
• 74
hypnozoites that can remain dormant in the liver for long peri ods and cause disease later in t he infect ed ind iv idual's life. However, P. ovale
. 75 does not cause cerebral ma laria .
• 76 Plasmodium ovale Malaria Plasmodium Plasmodium vivax Liver Infection Fever

· 77 E is not correct. 8 °/o chos e this.


• 78 Plasmodium vivax infect ion causes a 48- hour cyclic fever. A unique feat ure of bot h P. vivax and P. ovale organisms is t hat they can form
• 79 hypnozoites that can remain dormant in the liver for long peri ods and cause disease months t o years lat er. However, P. vivax does not cause
cerebral ma laria .
. 80
Plasmodium vivax Malaria Plasmodium Plasmodium ovale Liver Infection Fever
. 81
• 82
• 83 Bottom Line:
• 84 Of t he Plasmodium species t hat cause malari a in humans, on ly P. falciparum has cerebral involvement.
Malaria Plasmodium Plasmodium falciparum Species
. 85
.
8
Lock
s
Suspend
0
End Block
Item: 6S of ~ ,• Mark <::J [:::> ""I ~· ~'j
100 J.. Previous Next faiUI~S
LAb Notes Calculator

60 • •
FA17 p 153.1
61
Protozoa- hematologic infections
62
ORGANISM DISEASE TRANSMISSION DIAGNOSIS TREATMENT
63
64 Plasmodium Malaria -fever, headache, anemia, Anopheles mosquito Blood smear: Chloroquine (for
65 P vivax/ovale splenomegaly trophozoite ring sensitive species),
• 66 P falciparum P vivax/ova/e-48-hr cycle (tertian; form within which blocks
• 67 P malariae incl udes fever on first day and RBC [J, schizont Plasmodium heme
• 68 third day, thus fevers are actually containing polymerase; if
. 69 48 hr apart); dormant form merozoites; red resistant, use
. 70 (hypnozoitc) in liver granules (Schuffner meAoquine or
• 71
P falciparwn-severe; irregular stippling) rn atovaquone/
· 72
fever patterns; parasitized RBCs throughout RBC proguanil
• 73
occlude capillaries in brain cytoplasm seen If life-threatening,
• 74
(cerebral malaria), kidneys, lungs with P vivax/ovctfe use intravenous
. 75
P malariae-72-hr cycle (quartan) quinidine or
• 76
artesunate (lest for
· 77
• 78
G6PD deficiency)
• 79
For P vivaxlovale,
. 80
add primaquine for
. 81 hypnozoitc (test for
• 82 G6PD deficiency)
• 83 Babesia Babesiosis- fever and hemolytic Ixodes tick (sa me as Blood smear: ring Atovaquone
• 84 anemia; predominantly in Borrelia burgdor{eri form (jJ, "Maltese + azithromycin
. 85
northeastern United States; of Lyme disease; cross" (i); PCR
. •

8
Lock
s
Suspend
0
End Block
Item: 66 of - ,• Mark --<) [::> ""'I ~· 1!';:'1
100 ~ Prev1o u s Next Labf a lu es Notes Calculator

60 •
A 7-year-old boy is brought to his physician with a productive cough. He has a history of recurri ng and increasingly severe pneumonia. His
61 fami ly history is significant for an uncle who was diagnosed with infe rtility and another uncle who died in his 20s as a result of pneumonia.
62 He has a temperature of 35 .8°C (101.3°F), respiratory rate of 28/min, pulse of 115/min, and blood pressure of 120/70 mm Hg. On
physical exam, he has decreased breath sounds and positive egophony in the right lower lung field, but otherwise no abnormal findings. Review
63
of medical records reveals a sputum culture positive for Staphylococcus aureus 2 months ago.
64
65
This patient is at risk of developing which of the following?
. 66
:
. 67
A. Abscess formation
• 68
. 69
B. Bronchiectasis
. 70 C. Granuloma formation
• 71 D. Mesothelioma
• 72
E. Pneumocystis jiroveci pneumonia
• 73
• 74
. 75
• 76
• 77
• 78
• 79
. 80
. 81
• 82
• 83
• 84
. 85
. •

8
L.odt
s
Su~pl'nd
~
End Block
Item: 66 of ~ ,• Mark <::J [:::> ""I ~· ~'j
100 J.. Previous Next LAbfaiUI~S Notes Calculator

60 •
The correct a nswer is B. 66% chos e t his .
61 Recurrent pulmonary infect ion in a pat ient with a family history of infert ilit y among male members is highly suggest ive of cystic fibrosis (CF) .
62 Primary ciliary dyskinesia patient s also suffer f rom infe rtility and recurrent pulmonary infect ions, but t hey often also present with situs
inversus (which t his pat ient does not have). Because of an impaired ability to cl ear m ucous secret ions f rom t he airways, pat ients wit h CF are
63
especially prone to bact erial infection . Staphylococcus aureus is the most common cause of pneumonia in CF pat ients under 20 years of
64 age, while Pseudomonas aeruginosa is most common cause for t hose over 20 years of age . I nfection wit h t hese organisms produces
65 obst ruct ion and dilat ion of t he bronchi, resu lt ing in bronchiect asis. Prophylaxis against t hese infections includes ant ipseudomonal ant ibiotics
66
and inhaled acet ylcysteine, which breaks up mucous plugs. Signs and sympt oms of bronchiectasis include halit osis, bloody spu tum, and a
productive cough .
• 67 Bronchiectasis Primary ciliary dyskinesia Pseudomonas aeruginosa Cystic fibrosis Halitosis Bronchus Sputum Pneumonia Acetylcysteine Hemoptysis Cough Antibiotics Staphylococcus Fibrosis Infertility
• 68 Situs inversus Preventive healthcare Infection Pseudomonas Mucous membrane
. 69
A is not correct . 19 % chos e this .
. 70
Abscess format ion in t he lung parenchyma is a serious con dit ion, because it can cause hypoxia, fever, and sepsis. Patients at an increased risk
• 71 for aspirat ion pneumonia are also at an increased risk fo r a lung abscess. Patient s with cyst ic f ibrosis are generally not predisposed to lung
· 72 abscess formation, however. The most common pathogen in lung abscesses is Staphylococcus aureus, alt hough these bact eria tend to be
• 73
polymicrobial, wit h anaerobes often fwther complicating the pneumonia picture .
Cystic fibrosis Aspiration pneumonia Hypoxia (medical) Parenchyma Sepsis Staphylococcus aureus Lung abscess Pneumonia Abscess Pathogen Anaerobic organism Staphylococcus Bacteria Fibrosis Fever
• 74
Lung
. 75
• 76 C is not correct. 7 °/o chose this .
While pat ients wit h cyst ic fibrosis have an increased risk of infect ion, they usually do not have granuloma formation in their lungs. Diseases
· 77
t hat can cause granuloma format ion include t uberculosis, hist oplasmosis, and sarcoidosis.
• 78 Cystic fibrosis Histoplasmosis Sarcoidosis Tuberculosis Granuloma Fibrosis Lung Infection
• 79
D is not correct. 2°/o chose t his .
. 80
Lung cancer is not more common in patient s with cyst ic f ibrosis. Pat ients wit h a history of exposure to beryllium, silicon, asbestos, and other
. 81 nonorganic mat erial are at an elevated ri sk fo r developing lu ng cancers such as mesot helioma . The prognosis fo r patients wit h this disease is
• 82 usually bleak, alt hough t here is a benign variant.
Cystic fibrosis Mesothelioma Lung cancer Asbestos Beryllium Cancer Fibrosis Prognosis Lung Silicon
• 83
• 84 E is not correct. 6 °/o chos e this .
. 85 Pat ients wit h cystic fib rosis usually suffer from bact erial S aureus or P aeruginosa infect ions, not Pneumocystis infection . One is more likely t o
. see Pneumocystis infection in immunocom promised patient s, mainly patients wit h HI V and a CD4 + cell count <200/mm 3 . Signs and

8
Lock
s
Suspend
0
End Block
Item: 66 of ~ ,• Mark <::J [:::> ""I ~· ~'j
100 J.. Previous Next faiUI~S
LAb Notes Calculator

60
E is not correct. 6 °/o chose this.
61
Pat ients wit h cystic fib rosis usually suffer from bact erial 5 aureus or P aeruginosa infect ions, not Pneumocystis infection . One is more likely t o
62 see Pneumocystis infection in immunocom promised patient s, mainly patients wit h HI V and a CD4+ cell count <200/mm 3 . Signs and
63 symptoms include a dry cough, dyspnea, and hazy bilateral hilar infilt rate on x- ray of the chest. Treatment is usually limited t o
t rimet hoprim/sulfamet hoxazole.
64 Trimethoprim/sulfamethoxazole Cystic fibrosis Dyspnea Immunodeficiency Fibrosis Staphylococcus aureus Cough Root of the lung CD4 Pneumocystis pneumonia
65
66
• 67 Bottom Line:
• 68 Pat ients wit h cystic fib rosis are at increased risk for bronchiect asis .
Bronchiectasis Cystic fibrosis Fibrosis
. 69
. 70
. 71
l@ljl'il·1i•J for year:[ 2017 • J
· 72 FIRST AID FA CTS

• 73
• 74 FA17 p 56.3
. 75 Cystic fibrosis
• 76
GENETICS Autosomal recessive; defect in CFTR gene on chromosome 7; commonly a deletion of Phe508 .
· 77
Most common lethal genetic disease in Caucasian population.
• 78

• 79
PATHOPHYSIOLOGY CFTR encodes an ATP-gated CI- cha nnel that secretes CI- in lungs and GI tract, and reabsorbs
• 80
CI- in sweat glands. Most common mutation ..... misfolded protein ..... protein retained in RER
. 81 and not transported to cell membrane, causing l CI- (and H7 0 } secretion; t in tracellular CI-
• 82 results in compensatorr t r a+ reabsorption via epithelial a+ channels ..... t H 20 reabsorption
• 83 ..... abnormally thick mucus secreted into lungs and GI tract. t a+ reabsorption also causes more
• 84 negative transepithelial potential difference.
. 85
DIAGNOSIS t Cl- concentration (> 60 mEq/L} in sweat is diagnostic. Can present with contraction alkalosis
.
8
Lock
s
Suspend
0
End Block
Item: 66 of - ,• Mark --<) [::> ""'I ~· 1!';:'1
100 ~ Prev1o u s Next Labf a lu es Notes Calculator

60 •
FA17 p 56.3
61
Cystic fibrosis
62
63 GENETICS Autosomal recessi,·e; defect in C FT R gene on chromosome 7; commonly a deletion of Phe508.
64 Most common lethal genetic disease in Caucasian population.
65 PATHOPHYSIOlOGY CFTR encodes an AT P-gated CJ- channel that secretes CJ- in lungs and Gl tract, and reabsorbs
66 CJ- in sweat glands. lost common mutation ..... misfolded protein ..... protein retained in RER
. 67
and not transported to cell membrane, causing l CJ- (and 11 20 ) secretion; t intracellular CJ-
• 68
resuJts in compensatory t t\a+ reabsorption via epithelial 'aT channels .... t H 20 reabsorption
. 69
..... abnormally thick mucus secreted into lungs and C ltracl. t 1 a+ reabsorption also causes more
. 70
negati,·e transepitheliaJ potential difference.
• 71
• 72 DIAGNOSIS t CJ- concentration (> 60 mEq/L) in sweat is diagnostic. Can present with contraction alkalosis
• 73 and hypokalemia (ECF effects analogous to a patient taking a loop diuretic) because of ECF
• 74 H 20 / a+ losses and concomitant renal K+/1 1+ wasting. t immunoreactive trypsinogen (newborn
. 75 screening).
• 76
COMPliCATIONS Recurrent pulmonary infections (eg, S aureus [early infancy], P aeruginosa [adolescence]), chronic
• 77
bronchitis and bronchiectasis -+ reticulonodu lar pattern on CXR, opacification of sinuses .
• 78
Pancreatic insufficiency, malabsorption with steatorrhea, fat-soluble vita min deficiencies (A, 0 , 1<~,
• 79
K), biliar)' ci rrhosis, liver disease. leconium ileus in newborns.
. 80
In fertil ity in men (absence of vas deferens, spermatogenesis may be unaffected) and subfertilit) in
. 81
• 82
women (amenorrhea, abnormally thick cervical mucus).
• 83
Nasal polyps, clubbing of nails.
• 84 TREATMENT Multifactorial: chest phrsiotherapy, albuterol, aerosolized dornase alfa (D lAse), and hypertonic
. 85 saline facilitate mucus clearance. Azithromycin used as anti-inAammatory agent. Ibuprofen slows
. I • n .. r · rr . •
8
L.odt
s
Su~pl'nd
~
End Block
Item: 66 of ~ ,• Mark <::J [:::> ""I ~· ~'j
100 J.. Previous Next LAbfaiUI~S Notes Calculator

60 •
61 FA17 p 56.2
62
Autosomal recessive Albinism, autosomal recessive polycystic kidney disease (ARPKD), cystic fibrosis, glycogen
63
diseases storage diseases, hemochromatosis, Ka rtagener syndrome, mucopolysaccharidoses (except
64
Hunter synd rome), phenylketonuria, sickle cell anemia, sphingolipidoses (except Fabry disease),
65
thalassemias, Wilson disease.
66
• 67
FA17 p 645.1
• 68
. 69
Pneumonia
. 70 TYPE TYPICAL ORGANISMS CHARACTERISTICS
• 71 Lobar S pneumoniae most frequently, also Legionella, Intra-alveolar exudate -+ consolidation r.il; may
· 72 Klebsiella rn
iJwolve enti re lobe or lung.
• 73
Bronchopneumonia S pneumoniae, S aureus, H infiuenzae, Acute inflammatory infiltrates B1 from
• 74
Klebsiella bronchioles into ad jacent alveoli; patchy
. 75
distribution involving;;:: 1 lobe (!].
• 76
· 77 Interstitial (atypical) Mycoplasma, Chlamydophila pneumoniae, Diffuse patchy inflammation localized to
• 78 pneumonia Chlamydia psittaci, Legionella, viruses (RSV, interstitial areas at alveolar walls; diffuse
• 79 C tfV, influenza, adenovirus) distribution involving;;:: 1 lobe 1]. Generally
. 80 follows a more indolent course ("walking"
. 81 pneumonia) .
• 82
Cryptogenic Formerly known as bronchiolitis obliterans
• 83
organizing organizing pneumonia (BOOP). Noninfectious
• 84
pneumonia pneumonia characterized by inflammation of
. 85
. bronchioles and surrounding structure. Etiology
8
Lock
s
Suspend
0
End Block
Item: 66 of - ,• Mark --<) [::> ""'I ~· 1!';:'1
100 ~ Prev1o u s Next Labf a lu es Notes Calculator

60 Bronchopneumonia S pneumoniae, S aureus, H influen;:ae, Acute inAammatory i nfiltrates ~ from


61 Klebsiella bronchioles into adjacent alveoli; patchy
62 distribution invoking 2: 1 lobe [!].
63
Interstitial (atypical) Mycoplasma, Chlamydophila pneumoniae, Diffuse patchy inflammation localized to
64
pneumonia Chlamydia psittaci, Legionella, viruses (R V, interstitial areas at ah-eolar walls; diffuse
65
C~ I V, influenza, adenovirus) distribution in\'olving 2: l lobe J]. Cenerallr
66
. 67
follows a more indolent course ('\,·alking''
• 68 pneumonia).
. 69 Cryptogenic Formerly known as bronchiolitis obliterans
. 70 organizing organizing pneumonia {BOOP). 'oninfectious
• 71 pneumonia pneumonia characterized br inflammation of
• 72 bronchioles and surrounding structure. Etiology
• 73
unknown. Secondary organizing pneumonia
• 74
caused by chronic inflammatory diseases (eg,
. 75
rheumatoid arthritis) or medication side effects
• 76
(eg, amiodarone). e sputum and blood cultures,
• 77
• 78
no response to antibiotics.
• 79
• 80
. 81
• 82
• 83
• 84
. 85
. •
8
L.odt
s
Su~pl'nd
~
End Block
Item: 67 of - ,• Mark -<J
P~v1ous
[:::> "'I ~ ·· ~
100 ~ N @xt Labl lues N o tes Calcula to r
6
60
A 34-year-old woman with newly diagnosed HIV is unable to access antiretroviral medications. She inquires about t he natura l course of her ~~AI
61 disease.
62
63 Which of the fol lowing conditi ons is most likely to manifest only when the CD4+ count drops below 50 cells/mm 3 ?
64 :
65 A. Disseminated Mycobacterium avium-intracel/ulare infection
66
B. Herpes simplex virus infection
• 67
• 68
C. Herpes zoster infection
. 69 0 . Oral thrush
. 70
E. Pneumocystis penumonia
• 71
F. Toxoplasmosis brain abscess
' 72
' 73
' 74
. 75
• 76
· 77
• 78

' 79
. 80
. 81

' 82
' 83
' 84
. 85
..
8
Lode.
s
Suspe-nd
8
End Bloc:k
Item: 67 of ~ ,• Mark <::J [:::> ""I ~· ~'j
100 J.. Previous Next LAb faiUI~S Notes Calculator

60 •
The correct answer is A. 56°/o chose this.
61
Disseminated Mycobacterium avium-intracel/ulare (aka MAC) is nat ive to birds and ot her an ima ls but can manifest as a t uberculosis- like
62
disease in humans when t he CD4+ count is <50 cells/mm 3 . Prophylaxis is wit h azit hromycin at 50 cells/mm 3 This is the only answer choice
63 whose development requires a CD4+ count <50 cells/mm 3 and is t herefore t he correct answer.
Azithromycin Mycobacterium avium-intracellulare infection Mycobacterium Preventive healthcare CD4
64
65 B is not correct. 4 % chose this.
66 Herpes simplex infect ion can occur when T lymphocyt e count is <400/mm 3 or earl ier. Although herpes simplex can affect individuals wit h a
67
normal immune system, infection in immunocompromised patients will be more severe, wit h more oral and genital ulcers. Prophylaxis for
herpes simplex is not usually given to patients with HI V.
• 68 T cell Lymphocyte Immunodeficiency Herpes simplex Immune system Ulcer (dermatology) Preventive healthcare Infection Ulcer
. 69
C is not correct. 4 °/o chose this .
. 70
Herpes zost er infect ion can man ifest when t he T lymphocyte count is <400/mm 3 or earl ier. I t is charact eri zed as a painf ul collection of
• 71 vesicles in a dermatomal pattern . Prophylaxis for herpes zost er is not usually given to patients with HI V.
· 72 T cell Shingles Lymphocyte Herpes simplex Vesicle (biology and chemistry) Preventive healthcare Infection

• 73
D is not correct. 6°/o chose this .
• 74 Oral t hrush can occur when t he T lymphocyte count is <400/mm 3 or earl ier. I t manifests with whit e patches and plaques on t he oral m ucosa .
. 75 Prophylaxis fo r oral t hrush is not usually given t o pat ients with HIV.
T cell Lymphocyte Oral candidiasis Oral mucosa Candidiasis Mucous membrane Preventive healthcare
• 76
· 77 E is not correct. 12°/o chose this.
• 78 Pneumocystisjirovecii pneumonia is a common opportu nistic infect ion when CD4+ is <200 cells/mm 3 . The disease commonly man ifests wit h
fever, malaise, dyspnea on exertion, and a nonproductive cough . Physical exam is notable for scat tered rales, and X- ray of the chest may
• 79
reveal diffuse int erstit ial infilt rat es or may be completely normal if t he infect ion is new. Prophy laxis wit h t rimet hoprim-sulfamet hoxazole
. 80 should be sta1ted when CD4+ count drops below 200 cells/mm 3 .
. 81 Opportunistic infection Dyspnea Pneumocystis jirovecii Crackles Trimethoprim/sulfamethoxazole Chest radiograph Pneumonia Pneumocystis pneumonia Malaise Cough Preventive healthcare X-ray Fever CD4

• 82 Physical examination Infection

• 83 F is not correct . 18% chos e this.


• 84 Toxoplasmosis can occur when CD4+ count is < 100 cells/mm 3 in pat ients wit h HIV. I t most commonly manifests with foca l neurolog ic def icits,
. 85 and CT scan will revea l mult iple ring -enhancing lesions. Toxoplasmosis prophylaxis (with TMP-SMX) is st art ed when t he T lymphocyte count is
. < 100/mm 3 Typically t he pat ient is already on TMP-SMX for pnemocyst itis pneumonia prophylaxis, but t he dose is usually increased t o cover

8
Lock
s
Suspend
0
End Block
Item: 67 of ~ ,• Mark <::J [:::> ""I ~· ~'j
100 J.. Previous Next LAb faiUI~S Notes Calculator

60
F is not correct. 18% chose this.
61 Toxoplasmosis can occur when CD4+ count is < 100 cells/mm 3 in pat ients wit h HIV. I t most commonly manifests with foca l neurologic def icits,
62 and CT scan will revea l multiple ring -enhancing lesions. Toxoplasmosis prophylaxis (with TMP-SMX) is st art ed when t he T lymphocyte count is
63 < 100/mm 3 . Typically t he pat ient is already on TMP-SMX for pnemocyst itis pneumonia prophylaxis, but t he dose is usually increased t o cover
t oxoplasma when CD4 drops below < 100 cells/mm 3 .
64 T cell Toxoplasmosis CD4 CT scan Pneumonia Lymphocyte Toxoplasma gondii Trimethoprim/sulfamethoxazole Preventive healthcare Neurology
65
66
Bottom Line:
67
CD4+ counts < 50/mm 3 predispose HIV-posit ive patient s to Mycobacterium avium-intrace/lu/are infect ions. Herpes simplex, herpes zoste r~
• 68
and oral t hrush can occur at a higher t hresholds, wit h CD4+ <400/mm 3 .
. 69 Shingles Oral candidiasis Herpes simplex Mycobacterium avium-intracellulare infection Candidiasis Mycobacterium HIV CD4
. 70
. 71
· 72 141;fil·1i•J
FIRS T AID FACTS
for year:[ 2017
.
•j .
• 73
• 74
FA17 p 173.1
. 75
• 76 Common diseases of As CD4+ cell count ~. risks of reactivation of past infections (eg, T B, HSV, shingles), dissemination
· 77 HIV-positive adults of bacterial infections and fungal infections (eg, coccidioidomycosis}, and non-Hodgkin
• 78 lymphomas t .
• 79
PATHOGEN PRESENTATION FINDINGS
• 80
. 81 CD4+ cell count < 500/mm 3
• 82 Candidaalbicans Oral thrush Scrapable white plaque, pseudohyphae on
• 83 mrcroscopy
• 84
EBV Oral hairy leukoplakia Unscrapable white plaque on lateral tongue
. 85
. Bartonella henselae Bacillary angiomatosis Biopsy with neutrophilic in Aammation
8
Lock
s
Suspend
0
End Block
Item: 67 of ~ ,• Mark <::J [:::> ""I ~· ~'j
100 J.. Previous Next LAb faiUI~S Notes Calculator

60
FA17 p 173.1
61
62 Common diseases of As CD4+ cell count ~, risks of reactivation of past infections (eg, TB, HSV, shingles), dissemination
63 HIV-positive adults of bacterial infections and fungal infections (eg, coccidioidomycosis}, and non-Hodgkin
64 lymphomas t.
65
PATHOGEN PRESENTATION FINDINGS
66
67
CD4+ cell count< 500/mm 3
• 68 Candidaalbicans Oral thrush Scrapable white plaque, pseudohyphae on
. 69 m1croscopy
. 70 EBV Oral hairy leukoplakia Unscrapable white plaque on lateral tongue
• 71
Bartonella henselae Bacillary angiomatosis Biopsy with neutroph ilic in Aammation
· 72
• 73
HHV-8 Kaposi sarcoma Biopsy with lymphocytic inflammation
• 74 HPV Squamous cell carcinoma, commonly of anus
. 75 (men who have sex with men) or cerYix
• 76 (women)
· 77
CD4+ cell count< 200/mm 3
• 78

• 79
Histoplasma Fever, weight loss, fatigue, cough, dyspnea, Oval yeast cells within macrophages
• 80 capsulatum nausea, vom iting, diarrhea
. 81 HIV Dementia
• 82
JC virus (reactivation) Progressive mu ltifoca l leukoencephalopathy onenhancing areas of demyelination on IRl
• 83
• 84
Pneumocystis jirovecii Pneumocrstis pneumonia "Ground-glass" opacities on CXR
. 85 CD4+ cell count< 100/ mm 3
.
8
Lock
s
Suspend
0
End Block
Item: 67 of - ,• Mark --<) [::> ""'I ~· 1!';:'1
100 ~ Prev1o u s Next Labf a lu es Note s Calculator

60 HIV Dementia
61
JC virus (reactivation) Progressive mu ltifocal leukocncephalopathy onenhancing areas of demyelination on ~ I RI
62

63
Pneumocystis jirovecii Pn eumocystis pneumonia "Ground-glass" opacities on CXR
64 CD4+ cell count < 100/ mm3
65
Aspergillus fumigatus Hemoptysis, pleuritic pain Ca\ italian or infiltrates on chest imaging
66
67
Candida albicans Esophagitis White plaques on endoscopr; yeast and
• 68
pseudo h) phae on biopsy
. 69 CMV Retinitis, esophagitis, colitis, pneumonitis, Linear ulcers on endoscopy, cotton-wool spots
. 70 encephalitis on fundoscopy
• 71 Biopsy re,·eals cells with intranuclear (owl eye)
• 72 inclusion bodies
• 73
Cryptococcus Meningitis Encapsulated yeast on India ink stain or
• 74
neoformans capsular antigen E£>
. 75
• 76
Cryptosporidium spp. Chronic, ,,·atery diarrhea Acid-fast oocysts in stool
• 77 EBV B-cell lymphoma (eg, non-Hodgkin lymphoma, C S lymphoma- ring enhancing, may be
• 78 CNS lymphoma) solitary (vs Toxoplasma)
• 79 Mycobacterium onspecific systemic symptoms (fever, night
• 80
avium- intracellulare, sweats, weight loss) or foca l lymphaden itis
. 81
Mycobacterium avium
• 82
complex
• 83
• 84 Toxoplasma gondii Brain abscesses Multiple ring-enhancing lesions on ~IRI
. 85
. •
8
L.odt
s
Su~pl'nd
~
End Block
Item: 68 of - ,• Mark -<] 1:> ""'I ~· 1!';:'1
100 ~ Prev1o u s Next Labf a lu es Note s Calculator

60 • Several days after a bioterro rist attack wi t h an un identif ied pathogen, patients begin presenting to emergency departments with painless
61
skin sores on their face, neck, and arms, like the one shown in th e image. Patients also have significant edema in the areas around the
ulcers. Biopsy of ulcers show necrosis.
62

63
64
65
66
67
• 68
. 69
. 70
• 71
• 72
• 73
• 74
. 75
• 76
• 77
• 78
• 79
Image courtesy of CDC
• 80
. 81 Which of the fo llowing organisms releases a toxin with a similar mechanism of action as one of the toxins produced by the pathogen in this
• 82 attack?
• 83
:
• 84 A. Streptococcus pyogenes
. 85
B. Corynebacterium diphtheriae •
.
8
L.odt
s
Su~pl'nd
~
End Block
61
62

63
64
65
66
67
• 68
. 69
. 70
• 71
• 72
• 73
Image courtesy of CDC
• 74
. 75
Which of the fol lowing organisms releases a toxin with a similar mechanism of action as one of the toxins produced by the pathogen in t his
• 76 att ack?
• 77
:
• 78 A. Streptococcus pyogenes
• 79
B. Corynebacterium diphtheriae
• 80
. 81 C. Enterohemorrhagic E. coli
• 82 D. Mycobacterium tuberculosis
• 83
E. Staphylococcus aureus
• 84
. 85 F. Vibrio cholerae
. •
8
L.odt
s
Su~pl'nd
~
End Block
Item: 68 of ~ ,• Mark <::J [:::> ""I ~· ~'j
100 J.. Previous Next LAbfaiUI~S Notes Calculator

60 • The correct answer is F. 39°/o chose this .


61 There are very few causes of painless cut aneous ulcers, t he most important of which are cutaneous ant hrax and syphilis. Ment ion of
biot errori sm is a helpful clue t o cutaneous ant hrax. The image in t he stem shows a typical ant hrax lesion : an ulcer wit h an associat ed
62
overly ing black escha r. Ant hrax poisoning can also cause severe pu lmona ry man ifestat ions.
63
One of the virulence facto rs of Bacillus anthracis, edema factor, acts by increasing intracellular cyclic adenosine monophosphat e (cAMP) .
64 Cholera toxin released by Vibrio cholerae creat es a similar effect (increasing the level of cAM P) t hrough a slight ly diffe rent sequence of
65 events. The other main virulence facto r of Bacillus anthracis is letha l factor, wh ich acts t o increase TNF-alpha and I L- l.
66 Eschar Vibrio cholerae Bacillus anthracis Cyclic adenosine monophosphate Virulence factor Bioterrorism syphilis Anthrax Cholera toxin Tumor necrosis factor alpha Cholera Ulcer Edema Anthrax toxin

67 Jnterleukin-1 family Bacillus Toxin Virulence Vibrio Lesion Intracellular Peptic ulcer

68
A is not correct . 12% chos e this.
. 69
Streptococcus pyogenes creates exotoxin A which acts in a mechanism sim ilar to TSST-1. It binds t he MHC II and TCR outside of t he antigen
. 70 bind ing sit e to cause overwhelm ing re lease of I L-l , IL- l, I FN-gamma, and TNF-alpha lead ing t o shock .
• 71 Streptococcus pyogenes Tumor necrosis factor alpha Exotoxin MHC class II Jnterleukin-1 family Streptococcus Antigen Pseudomonas exotoxin Interferon-gamma T cell receptor

· 72 Major histocompatibility complex Binding site

• 73 B is not correct. 18% chose this .


• 74 Corynebacterium diphtheriae produces diphtheria toxin, which inactivates elongation fact or- 2 (EF-2) . This is t he same mechanism as exotoxin
. 75 A produced by Pseudomonas aeruginosa. Pseudomonal infection can present cut aneously as ect hyma gangrenosum, wh ich appea rs as
necrot ic ulcers on the skin . However, t his rare presentation tends to occur in immunocom promised indiv iduals and has not been associat ed
• 76
with biot errori sm .
· 77 Corynebacterium diphtheriae Pseudomonas aeruginosa Diphtheria EEF2 Diphtheria toxin Exotoxin Immunodeficiency Pseudomonas exotoxin Corynebacterium Pseudomonas Toxin Ecthyma gangrenosum

• 78 Necrosis Elongation factor Ecthyma

• 79
C is not correct. 11% chos e this .
. 80
Enterohemorrhagic E. coli (EHEC) releases an exot oxin called Shiga-like t oxin, wh ich acts sim ilarl y to t he exotoxin of Shigella. However,
. 81 edema factor f rom Bacillus anthracis actua lly shares t he same mechanism as an heat-labile toxin re leased by ent erotoxigen ic E. coli (ETEC) .
• 82 Bot h increase levels of cAMP by overactivating adenylate cyclase .
Bacillus anthracis Escherichia coli Exotoxin Shigella Shiga-like toxin Adenylyl cyclase Cyclic adenosine monophosphate Verotoxin-producing Escherichia coli Edema Bacillus Anthrax toxin Toxin
• 83
• 84 D is not correct. 8°/o chose this .
. 85 Mycobacterium tuberculosis possesses a variety of v irulence fact ors that cont ribute t o disease pat hogenesis. However, none of t hese are
. considered exot oxins, because they are not produced and released into t he surrounding medium .

8
Lock
s
Suspend
0
End Block
Item: 68 of ~ ,• Mark <::J [:::> ""I ~· ~'j
100 J.. Previous Next LAbfaiUI~S Notes Calculator

60 E is not correct. 12°/o chos e this.


61 Staphylococcus aureus produces alpha toxin, which act s as a lecit hinase (phosphololipase) t hat destroys cell membranes and tissues. It
62
presents wit h "gas gangrene" t issue necrosis and hemolysis. Bacillus anthracis does not produce a t oxin t hat functions as a lecithinase.
Bacillus anthracis Staphylococcus aureus Hemolysis Gangrene Lecithinase Necrosis Staphylococcus Clostridium perfringens alpha toxin Cell membrane Bacillus Staphylococcus aureus alpha toxin Toxin
63
64
65 Bottom Line:
66 Edema factor released by Bacillus anthracis works by increasing int racellular cAMP, which is t he same mechanism of action as cholera t oxin.
Bacillus anthracis Cholera Cholera toxin Cyclic adenosine monophosphate Edema Anthrax toxin Bacillus Toxin Intracellular
67
68
. 69
. 70
i@l;fil·1i•J for year:[2017 • J
FIRST AID FACTS

• 71
· 72 FA17 p 133.4
• 73
Bacillus anthracis Gram®, spore-forming rod that produces anthrax toxin. T he only bacterium with a polypeptide
• 74
capsule (contains o-glutamate). Colonies show a ha lo of projections, sometimes referred to as
. 75
• 76
"medusa head" appearance.
· 77 Cutaneous anthrax Painless papule surrounded by vesicles - ulcer with black eschar (I'J) (painless, necrotic)
• 78 - uncommonly progresses to bacteremia and death .
• 79
• 80
. 81
• 82
• 83
• 84
. 85
.
8
Lock
s
Suspend
0
End Block
Item: 68 of ~ ,• Mark <::J [:::> ""I ~· ~'j
100 J.. Previous Next faiUI~S
LAb Notes Calculator

60 FA17 p 128.1
61 Bugs with exotox ins
62
BACTERIA TOXIN MECHANISM MANIFESTATION
63
Inhibit protein synthesis
64
65 Corynebacterium Diphtheria toxin3 Pharyngitis with pseudomembranes in th roat
66 diphtheriae Inactivate elongation factor and severe lymphadenopathy (bull neck)
67
Pseudomonas Exotoxin A3 (EF-Z) Host cell death
68
aeruginosa
. 69
. 70 Shigella spp. Shiga toxin (ST)3 Gl mucosal damage -+ dysentery; ST also
• 71 Inactivate 60S ribosome by enhances eytokine release, causing hcmolrtic-
· 72 removing adenine from uremic syndrome (HUS)
• 73 Enterohemorrhagic Shiga-like toxin rRI A SLT enhances cytokine release, causing HUS
• 74 E coli (EHEC) (SLT)a (prototypically in EHEC serotype 0157:H7).
. 75
Unlike Shigella, F.HEC does not invade host
• 76
cells
· 77
• 78 Increase fluid secretion
• 79 Enterotoxigenic Heat-labile Overactivates adenylate Watery diarrhea: "labile in the Air (Adenylate
• 80 E coli (ETEC) toxin (LT)3 cyclase (t cAMP) - t CJ- cyclase), stable on the Ground (Guanylate
. 81 secretion in gut and H20 cyclase)"
• 82 efflux
• 83
Heat-stable Overactivates guanylate
• 84
toxin (ST) cyclase (t cGMP)
. 85
. - l resorption of aCI
8
Lock
s
Suspend
0
End Block
Item: 68 of - ,• Mark -<] 1:> ""'I ~· 1!';:'1
100 ~ Prev1o u s Next Labf a lu es Note s Calculator

• Botulinum toxin3
60 Clostridium of proteins required for Flaccid paralysis, floppy baby; toxin prevents
61 botulinum neurotransmitter release release of stimulatory ( Ch) signals at
62
via vesicular fusion neuromuscular junctions -+ flaccid paralysis
63
a An AB to\ in (aka, two-component toxin [or three for anthrax]} with B enabling binding and triggering uptake (endocytosis)
64
65
of the acti' e A component. T he A components are usually ADP ribosyhransferases; others have enz~111atic activities as listed
66 in chart.
67
Lyse cell membranes
68
. 69 Clostridium Alpha toxin Phospholipase (lecithinase) Degradation of phospholipids -+ myonccrosis
. 70 perfringens that degrades tissue and ("gas ga ngrene") and hemolysis ("double 1one"
• 71 cell membranes of hemolysis on blood agar)
• 72 Streptococcus Streptolysin 0 Protein that degrades cell J.yses RBCs; contributes to ~ -h emolys is;
• 73 pyogenes membrane host antibodies against toxin (ASO) used to
• 74
diagnose rheumatic fe,·er (do not confuse
. 75
with immune complexes of poststreptococcal
• 76
glomerulonephritis)
• 77
• 78 Superantigens causing shock
• 79 Staphylococcus Toxic shock Binds to lHC II and TC R Toxic shock syndrome: fever, rash, shock; other
. 80 aureus syndrome toxin outside of antigen binding toxins cause scalded skin syndrome (exfoliative
. 81 (TSS1 ~ 1 ) site to cause overwhelming toxin) and food poisoning (heat-stable
• 82 enterotoxin)
release of I L-1, IL-2,
• 83
Streptococcus Exotoxin A IF1 -y, and T N F-a Toxic shock-like smdrome: fe,·er, rash, shock;
• 84
pyogenes - shock scarlet fc,·er
. 85
. •
8
L.odt
s
Su~pl'nd
~
End Block
Item: 69 of - ,• Mark -<J [:::> "'I ~· ~
100 ~ P~v1ous N @xt Labl lues N o tes Calcula to r

60 A A 5-year-old boy complains of a sore t hroat and a tem perature of 38.1°C (100.6°F). His fam ily recent ly immigrat ed to the United States
61 from Kazakhstan. The physician not es a membrane covering the surface of his posterior pharynx, like t hat shown in t he image.
62
63
64
65
66
67
68
. 69
. 70
• 71
• 72
• 73
• 74
. 75
• 76
· 77
• 78
Image cowtesy of CDC
• 79
• 80
The organisms res ponsible for t his patient's sym ptoms produces a special toxin. A different bact eriu m produces a similar toxin with the same
. 81 mechanism of action as the one in question. What pathognomonic substance is produced by th e other bacteria?
• 82
:
• 83 A. Actin filaments
• 84
B. Cyclic adenosine monophosphate
. 85
.. C Exotoxin A
8
Lode.
s
Suspe-nd
8
End Bloc:k
61
62
63
64
65
66
67
68
. 69
. 70
• 71

' 72
' 73
' 74
. 75 Image courtesy of CDC

' 76
· 77 The organisms responsible for t his pat ient's sympt oms produces a special toxin. A different bacteri um produces a similar toxin with the same
mechanism of action as the one in quest ion . What pathognomonic substance is produced by t he other bacteria?
' 78
:
' 79
A. Actin filaments
• 80
. 81 B. Cyclic adenosine monophosphat e
' 82 C. Exotoxin A
' 83
D. M protein
' 84
. 85 E. Tetanospasmin
..
8
Lode.
s
Suspe-nd
8
End Bloc:k
Item: 69 of ~ ,• Mark <::J [:::> ""I ~· ~'j
100 J.. Previous Next LAb faiUI~S Notes Calculator

60 •
61 The correct a nswer is C. 63°/o chose this .
62 This child has signs and sympt oms of Corynebacterium diphtheriae infect ion (dipht heri a), including sore t hroat , low-grade feve r~ and a
pseudomembrane covering his t hroat (shown in the image) . The bacteria re lease an exot oxin of the A/B type: Subunit B binds the cell and
63 enables subunit A to gain ent ry. Once subunit A enters its ta rget cell, it cata lyzes t he ADP-ribosy lat ion of elongation fact or 2 (EF-2), causing
64 its inactivation and st opping prot ein synthesis, causing t issue necrosis. Pseudomonas aeruginosa possesses a sim ila r exot oxin A and
65 ult imat ely leads to inactive EF-2. It can lead to pneumonia, extern al ot it is, uri nary t ract infect ion, and drug-relat ed or diabet ic ost eomyelit is.
Pseudomonas fo rms pyocyanin, wh ich gives t he cha racterist ic green ish -blue pigment.
66 Urinary tract infection Osteomyelitis Corynebacterium diphtheriae Pseudomonas aeruginosa Pyocyanin Diphtheria Catalysis Exotoxin Pneumonia Pseudomonas exotoxin Protein ADP-ribosylation
67
Pseudomonas EEF2 Bacteria Corynebacterium Necrosis Protein biosynthesis Otitis externa Urinary system Infection Elongation factor Fever Diabetes mellitus
68
A is not correct . 8°/o chos e this .
69
Shigella dysenteriae produces Sh iga toxin, which inhibits prot ein synt hesis by inact ivat ing t he 60S subunit of t he host cell's ri bosome . The
. 70
Shiga t oxin, wh ich is also produced by Escherichia coli 0157:H7 inactivates ribosomes and enhances cytokine re lease, causing hemolytic
• 71 uremic syndrome, wh ich is charact eri zed by acut e renal fa ilure, low platelet count, and hemolytic anemia. Shigella organisms use actin
· 72 f ilaments to propel t hemselves via polymerizat ion.
Shigella dysenteriae Shiga toxin Escherichia coli Cytokine Ribosome Shigella Hemolytic-uremic syndrome Actin Hemolytic anemia Toxin Acute kidney injury Platelet Protein Anemia Protein synthesis
• 73
Thrombocytopenia Protein biosynthesis Hemolysis Kidney
• 74
. 75 B is not correct. 16 % chose this .
• 76 Pertu ssis toxin acts locally t o increase levels of cyclic adenosine monophosphate in phagocyt es and to initiate t he format ion of ion- permeable
pores in cell membranes, leading to cell lysis. This patient 's present at ion is not consist ent wit h Bordete/la pertussis infection.
· 77 Bordetella pertussis Pertussis toxin Cyclic adenosine monophosphate Pertussis Bordetella Toxin Phagocyte Adenosine monophosphate Cell membrane Lysis Infection
• 78
D is not correct. 9°/o chose t his .
• 79
The main virulence facto r fo r Streptococcus pyogenes is t he ant iphagocytic M protein. M prot ein binds to a factor H, a complement cont rol
. 80 protein. This results in destruction of C3 convertase and subsequently prevents opson ization by C3 b. Cardiac muscle cells have an antigen
. 81 t hat is structurally sim ila r t o M protein and an immune response mount ed duri ng a st rept ococcal infect ion can result in cross-react ivity wit h
• 82 t hese muscle cells. A phenomenon called molecular m imicry which is t he basis for rheumatic heart disease. S. pyogenes also produces an
eryt hrogenic exotoxin t hat can cause toxic shock- like synd rome, which includes the t riad of symptoms fever, rash, and shock. Other
• 83
symptoms include hypotension, stupor, and coma .
• 84 Streptococcus pyogenes Molecular mimicry Virulence factor Opsonin Rheumatic fever Exotoxin Hypotension Antigen M protein (Streptococcus) Virulence Protein C3b Immune system Factor H

. 85 Cardiac muscle Streptococcus Cross-reactivity Group A streptococcal infection Rash Fever C3-convertase Cardiovascular disease
.
8
Lock
s
Suspend
0
End Block
Item: 69 of ~ ,• Mark <::J [:::> ""I ~· ~'j
100 J.. Previous Next LAbfaiUI~S Notes Calculator

60 E is not correct. 4 °/o chos e this .


61 Clostridium tetani produces tetanospasmin, an exot oxin t hat cl eaves t he releasing prot ein synaptobrevin necessary for SNARE proteins
62 functioning in exocytosis. This blocks the re lease of t he inhibitory neurot ransm itt er glycine and GABA, causing t rismus ("lockjaw" ) risus
sardonicus (charact eri stic smile), and opist hotonos (arching of t he back) due to m uscle contraction . The patient does not have cl inical
63
symptoms of C. tetani infect ion .
64 Risus sardonicus SVnaptobrevin Clostridium tetani Trismus Tetanospasmin Opisthotonus Exotoxin Exocytosis Glycine Neurotransmitter Gamma-Aminobutyric acid SNARE (protein) Protein Clostridium

65 Muscle Muscle contraction

66
67
Bottom Line :
68
Dipht heria t oxin inact ivat es elongat ion facto r 2, wh ich int erf eres wit h t ranslation and results in pharyngit is and pseudomembrane format ion
69
in the t hroat. Pseudomonas aeruginosa has a sim ila r exot oxin .
. 70 Pseudomonas aeruginosa Diphtheria Diphtheria toxin Pharyngitis Exotoxin Pseudomonas Elongation factor Toxin EEF2

• 71
· 72
• 73 i@l;fil·1i•J for yea r:[2017 • J
FIRST AID FACTS
• 74
. 75
FA17 p 128 .1
• 76
Bugs with exotoxins
· 77
• 78
BACTERIA TOXIN MECHANISM MANIFESTATI ON
• 79 Inhibit protein synthesis
• 80 Corynebacterium Diphtheria toxin3 Pharyngitis with pseuclomembranes in th roat
. 81
diphtheriae Inactivate elongation factor and severe lymphadenopathy (bull neck)
• 82
• 83
Pseudomonas Exotoxin N (EF-2) Host cell death
• 84
aeruginosa
. 85 Shigella spp. Shiga toxin (ST)a GI mucosal damage .... d)'sentery; ST also
. 1 • 1 •

8
Lock
s
Suspend
0
End Block
Item: 69 of - ,• Mark -<J [:::> "'I ~ · ~
100 ~ P~v1ous N @xt Labl lues N o tes Calcula to r
A A
60
FA17 p 139.3
61
62 Pseudomonas Aerobic, motile, gram 8 rod. Non-lactose Treatments include "CA.\ 1PFIRE" drugs:
63 aeruginosa fermenting, oxidase®. Produces p)ocyanin C arbapenems
64 (blue-green pigment rJ); has a grape-! ike odor. .\m inoglycosides
65 Produces endotoxin (fe\'cr, shock), exotoxin \ Jonobactams
66
(inactivates EF-2), phospholipase C (degrades Polymp:ins (eg, pol~myxin B, colistin)
67
cell membranes). and pyocyanin (generates F luoroquinolones (eg, ciprofloxacin,
68
reactive oxygen species). levoAoxacin)
69
PSEUDO~ IO:'\A S is associated with: Th i Rd- and fourth-generation
. 70
Pneumonia, pyocyanin cephalosporins (eg, ceftazidime, cefcpime)
• 71
Sepsis E xtended-spectrum penicillins (eg,
' 72
Ecthyma gangrenosum piperacillin, ticarcillin)
' 73
UTls Aeruginosa-aerobic.
' 74
. 75 D iabetes, drug use Mucoid polysaccharide capsule m<lY contribute
• 76 • Osteomrelitis (eg, puncture wounds) to chronic pneumonia in cystic fibrosis patients
· 77 • \lucoid polysaccharide capsule due to biofilm formation.
• 78 • O titis externa (swimmer's ear) Can cause wound infection in burn victims.
' 79 • Nosocom ial infections (catheters, Cornea l ulcers/keratitis in contact lens wearers/
. 80 equipment) minor eye trauma .
. 81 Exotoxin A Frequently found in water ..... hot tub foll iculitis.
' 82 Skin infections (hot tub folliculitis) Ecthyma gangrenosum-rapidly progressi,·e,
' 83 necrotic cutaneous lesion [lJ caused by
' 84 Pseudomonas bacteremia. Typically seen in
. 85 immunocompromised patients.
..
8
Lode.
s
Suspe-nd
8
End Bloc:k
• Otitis externa (swimmer's ear) Can cause wound infection in burn victims.
61
Nosocomial infections (catheters, Corneal ulcers/keratitis in contact lens wearers/
62
equipment) minor eye trauma.
63
Exotoxin A Frequently found in water ..... hot tub folliculitis.
64
Skin infections (hot tub folliculitis) Ecthyma gangrenosum- rapidly progressi,·e,
65
necrotic cutaneous lesion : caused by
66
67
Pseudomonas bacteremia. Typical!) seen in
68
immunocompromised patients.
69
. 70 FA17 p 135.1
• 71
Corynebacterium Gram Et> rod; transmitted via respiratory Coryne =club shaped.
' 72
diphtheriae droplets. Causes diphtheria \'ia cxoto..:in Black colonies on cystine-tellurite agar.
' 73
encoded by ~-proph age. Potent cxoto:..in ABCDEFG:
' 74
inhibits protein synthesis via AOP-ribosylation ADP-ribosylation
. 75
of EF-2. ~-pro phage
' 76
· 77
Symptoms include pseudomembranous Corynebacterium
' 78
pharyngitis (grayish-white membrane t'.J) Diphtheriae
' 79
with lymphadenopathy, myocarditis, and Elongation Factor 2
• 80 e~rrhy thmias . Granules
. 81 Lab diagnosis based on gram Et> rods with
' 82 metachromatic (blue and red) granules and
' 83 Et> Elek test for toxin.
' 84 To..:oid vaccine prevents diphtheria.
. 85
..
8
Lode.
s
Suspe-nd
8
End Bloc:k
Item: 70 of - ,• Mark -<] 1:> ""'I ~· 1!';:'1
100 ~ Prev1o u s Next Labf a lu es Note s Calculator

60 •
A 5-year-old boy is brought to t he emergency department by his parents, who state that his legs look swollen and his urine has been r~AJ
61 t inged red. His vaccinations are up to dat e and past medical history is significant only for a sore throat about 10 days ago that was treated
62 with penicillin. Investigat ions reveal decreased levels of complement.
63
64 What is the etiology of this patient's condition?
65 :
66 A. Delayed allergic reaction to penicillin
67 B. Type I hypersensitivity reaction
68
C. Type II hypersensitivity reaction
69
. 70 0 . Type III hypersensitivity reaction
• 71 E. Type IV hypersensitivity reaction
• 72
• 73
• 74
. 75
• 76
• 77
• 78
• 79
. 80
. 81
• 82
• 83
• 84
. 85
. •

8
L.odt
s
Su~pl'nd
~
End Block
Item: 70 of ~ ,• Mark <:::1 t::> ""I ~· ~'j
100 J.. Previous Next LAb faiUI~S Notes Calculator

I
60
The correct a nswer is 0. 63°/o chose t his.
61
This patient is likely sufferi ng from poststreptococca l glomerulonephri t is (or postinfectious glomerulonephri t is), which is a nephri t ic syndrome
62 t hat develops approximately 10 days after pharyngit is or skin infection wit h a nephrit ogenic strain of group A 13- hemolyt ic Streptococcus. It is
63 believed t o be a type III hypersensit ivity react ion, as ant igen-ant ibody complexes fo rmed during t he infect ion are deposited in the glomerula r
basement membrane, leading t o complement act ivation. Light m icroscopy revea ls diffuse proliferative glomeruloneph rit is.
64
I mmunofluorescence m icroscopy typically shows coarse granula r deposits of IgG and C3. This nephrit ic syndrome may develop despit e
65 appropriate ant ibiotic t reat ment of the init ial infection. Symptoms include hematu ria, prot einuri a, oliguria, and hypertension. Treat ment is
66 t ypically support ive, as the disease resolves spontaneously in approximat ely 2 weeks. Serum levels of C3 are often decreased and t it ers of
ant ist rept olysin (a streptococcal exoenzyme) ant ibodies are elevated.
67
Oliguria Nephritic syndrome Hematuria Proteinuria Type III hypersensitivity Glomerular basement membrane Hypersensitivity Basement membrane Immunofluorescence Antibiotics Immunoglobulin G
68
Immune complex Pharyngitis Glomerulonephritis Hypertension Anti-streptolysin 0 Streptococcus Nephritis Antibody Glomerulus Nephron Skin infection Glomerulus (kidney) Infection Microscopy
69
70
A is not correct . 4 °/o chos e this.
Delayed allergic reaction t o penicillin usually manifest s with a maculopapular or morbilliform rash aft er 72 hours upon exposure to the drug .
• 71
Renal manifestations are not typical of this t ype of allergic react ion.
· 72 Penicillin Morbilliform Allergy Maculopapular rash Rash
• 73
B is not correct. 4 % chose t his .
• 74
A type I hypersensit ivit y react ion is IgE-mediated as mast cells and basophils re lease vasoact ive mediat ors. Examples include allergies, at opic
. 75 diseases, and anaphylaxis .
• 76 Anaphylaxis Type I hypersensitivity Allergy Mast cell Hypersensitivity Atopic dermatitis Ptopy vasoactive

· 77 C is not co rrect. 21% chos e this .


• 78 A type II hypersensit ivity react ion is ant ibody-mediat ed as IgG binds t o an ant igen on t he surface of target cells, leading to complement
• 79 act ivat ion, ant ibody-dependent cell-mediated cytotoxicity, or phagocytosis. Examples include Goodpasture syndrome, aut oimmune hemolytic
anemia, pemphigus, and hyperacut e t ransplant rej ection .
. 80
Antibody-dependent cell-mediated cytotoxicity Phagocytosis Transplant rejection Type II hypersensitivity Autoimmune hemolytic anemia Hemolytic anemia Immunoglobulin G Antigen Autoimmune disease
. 81
Anemia Goodpasture syndrome Pemphigus Autoimmunity Hypersensitivity Cytotoxicity Hemolysis
• 82
• 83
E is not correct. 8 °/o chos e this .
A type I V hypersensit ivit y react ion is mediat ed by helper T cells and macrophages, and does not involve ant ibodies. Examples include contact
• 84
dermat it is, poison ivy, and acut e and chronic t ransplant reject ions .
. 85 Type IV hypersensitivity Toxicodendron radicans Contact dermatitis Dermatitis T cell Antibody T helper cell Macrophage Hypersensitivity
.
8
Lock
s
Suspend
0
End Block
Item: 70 of ~ ,• Mark <:::1 t::> ""I ~· ~'j
100 J.. Previous Next faiUI~S
LAb Notes Calculator

60
Bottom Line:
61
Type III hypersensit ivit y react ion is characterized by ant igen-ant ibody com plexes that lead to complement activat ion.
62 Type III hypersensitivity Hypersensitivity Immune complex
63
64
65 141;fil·1i•J for year:[ 2o17 •j

I
FIRST AID FA CTS . .
66
67
FA17 p 564.1
68
69 Nephritic syndrome ephrltic syndrome = Inflammatory process. When glomeruli are im·olved, leads to hematuria
70 and RBC casts in urine. Associated with azotemia, oliguria, hypertension (due to salt retention},
. 71 proteinuria .
· 72
Acute LM-glomeruli en larged and hypercellu lar r.J. Most frequently seen in children. Occurs
• 73
poststreptococcal TF- ("starr)' sky") granular appearance - 2-4 weeks after group A streptococcal
• 74
glomerulonephritis ("lumpy-bumpy") I]) due to lgG, lgM, and C3 infection of pharynx or skin. Resolves
. 75
• 76
deposition along GBM and mesangium. spontaneously. Type III hypersensitivity
· 77
EM-subepithelial immune complex (lC} reaction.
• 78
humps. Presents with peripheral and periorbital edema,
• 79
cola-colored urine, hypertension.
• 80 Positi,·e strep titers/serologies, ~ complement
. 81 levels (C3) due to consumption .
• 82 Rapidly progressive LM and IF-crescent moon shape [!1. C rescents Poor prognosis. Rapidly deteriorating renal
• 83
(crescentic) consist of fibrin and plasma proteins (eg, C3b) function (days to weeks).
• 84
glomerulonephritis with glomerular parietal cells, monocytes,
. 85
macro phages.
.
8
Lock
s
Suspend
0
End Block
Item: 70 of - ,• Mark -<] 1:> ""'I ~· 1!';:'1
100 ~ Prev1o u s Next Labf a lu es Note s Calculator

60 • IF- lgA-based IC deposits in mesa ngium. infections (lgA is secreted by mucosal linings) .
61 Renal pathology of llenoch-Schonlein purpura. ot to be confused with Buerger disease
62 (thromboangiitis obliterans).
63
AIport syndrome Ylutation in type IV collagen .... thinning and Eye problems (eg, retinopathy, lens dislocation),
64
splitting of glomerular basement membrane. glomerulonephritis, sensorineural deafness;
65
1ost commonh- X-I inked dominant. " can 't see, can 't pee, can' t hear a bee."
66
67
" Basl..el-\\ ea, e" appearance on EVI.
68 Membrana- Type 1-subendothelial immune complex \1 PC ' is a nephritic srndrome that often
69 proliferative (IC) deposits with granular IF; " tram-track" coprcsents with nephrotic syndrome.
70 glomerulonephritis appearance on PAS stain (!) and I J&E slain Type I may be 2° to hepatitis B or C infection.
• 71 0 due to CB l splitting caused by mcsangial May also be idiopathic.
• 72
• 73
• 74
. 75
ingrowth .
Type 11 - also called dense deposit disease.
Type II is associated with C3 nephritic factor
(lgC antibody that stabilizes C3 convertase
.... persistent complement acti,·ation .... l C 3
levels).
I
• 76
• 77
• 78
• 79
. 80
. 81
• 82
• 83
• 84
. 85 LM = light microscopy; E~l =electron microscopy; IF = immunoAuorescence.
. •
8
L.odt
s
Su~pl'nd
~
End Block
Item: 70 of - ,• Mark -<J [:::> "'I ~ · ~
100 ~ P~v1ous N @xt Labl lues N o tes Calcula to r

60 •
FA17 p 132.4
61
62
Streptococcus Gram El1 cocci in chains r.J. Group A strep J•NES (major criteria for acute rheumatic
pyogenes (group A cause: fe\"er):
63
64
streptococci) • Pyogenic-pharyngitis, cellulitis, impetigo Joints-polya rth ritis
65 ("honey-crusted" lesions), erysipelas • -carditis
66 Toxigenic- scarlet fever, toxic shock- liJ..e '\odules (subcutaneous)
67 syndrome, necrotizing fasciitis Erythema marginatum
68 Immunologic-rheumatic fever, S~den h am chorea
69 glomerulonephritis Pharyngitis can result in rheumatic ''phever"
70 Bacitracin sensitive, P-hemol) tic, p) rrol idon}I and glomerulonephritis.
• 71 arylamidase (PYR) EB. Hyaluronic acid capsule Impetigo usually precedes glomerulonephritis.
• 72
inhibits phagocytosis. ntibodies to VI protein Scarlet fever-blanching, sa ndpaper-like body
• 73
enhance host defenses against S Progenes but rash, strawberry tongue, and circumoral
• 74
can give rise to rheumatic fe\'er. pallor in the setting of group streptococca l
. 75
• 76
· 77
• 78
ASO titer or anti-0 ase B antibodies indicate
recent S pyogenes infection.
pharyngitis (erythrogenic toxin EB).
I
• 79 FA17 p 108.1
• 80
Hypersensitivity types Four types: Anaphylactic and Atopic (type 1), Cytotoxic (antibody mediated, type II), Immune
. 81
complex (type Ill ), Delayed (cell mediated, type IV) (AC lD).
• 82
• 83 Type I Anaphylactic and atopic-free antigen cross- First (type) and Fast (anaphylaxis).
• 84 Allergen'"\ / AIIer9t'n· links lgE on presensitized mast cells and Types I, II , and lll are all antibody mediated.
\ specmc lgE
. 85 basophils, triggering immediate release of Test: skin test or blood test (ELISA) for allergen-
.. < .... L " "II •C I C'

8
Lode.
s
Suspe-nd
8
End Bloc:k
Item: 70 of - ,• Mark -<] 1:> ""'I ~· 1!';:'1
100 ~ Prev1o u s Next Labf a lu es Note s Calculator

60 FA17 p 108.1
61
Hypersensitivity types Four types: Anaphylactic and \ topic (type 1), Cytotoxic (antibody mediated, type II), l mmune
62
complex (tr pe Ill), Delared (cell mediated, type IV) (ACID).
63
64 Type I Anaphylactic and atopic-free antigen cross- First (type) and Fast (anaphylaxis).
65 Allergen· links lgE on prescnsit izcd mast cells and Types I, II , and Ill are all antibody mediated.
speafic lgE basophils, triggering immediate release of Test: skin test or blood test (ELISA) for allergen-
66
/
67 Fe receptor vasoactive amines that act at postcapillar) specific lgE.
lorlgE\
68 venules (ie, histamine). Reaction de,·elops Example:
69 rapidly after antigen exposure because of Anaph) lax is (eg, food, drug, or bee sting
70 preformed antibody. Delayed phase results allergies)

. •
• 71 from mast cells and basophils releasing

• 72 •
•:• •••

• • cytokines that induce cellular inAammation .
• 73
• 74 DegranulatiOil
. 75 Direct Coombs test- detects antibodies
Typ e II Antibodies bind to cell-surface antigens
• 76
.... cellular destruction, inAammation, and attached directly to the RBC surface.
/ NKcell

I
• 77
cellular dysfunction . Ind irect Coombs test- detects presence of

1·~
• 78
unbound antibodies in the serum
• 79
•• Cellular destruction: cell is opsonized (coated) Examples:
••••• •
• 80
. 81
•••• by antibodies, leading to either: • Autoimmune-hemol)tic anemia
• 82 Phagocytosis and/or activation of Immune thrombocytopenic pmpura
fcreceptor . / (
• 83
lor lgG
complement system. Transfusion reactions
• 84
Surface antigen . )
NK cell killing (antibody-dependent cellular Hemolytic disease of the newborn
. 85
cytotoxicity).
. ' ..- •
8
L.odt
s
Su~pl'nd
~
End Block
Item: 70 of - ,• Mark -<] 1:> ""'I ~· 1!';:'1
100 ~ Prev1o u s Next Labf a lu es Note s Calculator

60
Type Ill Immune complex-a ntigen-antibody (lgC) In type III reaction, imagine an immune
61
complexes acti,·ate complement, which attracts complex as 3 things stuck together: antigen-
62
ncutrophils; neutrophils release lysosomal antibody-complement.
63
64
enzvmes. Examples:
65
Can be associated with vasculitis and ~\·stem ic
'
SLE
66 manifestations. Polvarteritis nodosa
67 Poststreptococcal glomerulonephritis
68
69 ... .... Serum sickness-an immune comple~ disease \lost e rum sickness is now caused b) drugs
70
• 71
:~·: ~Enzymes from in which antibodies to foreign proteins are (not serum) acting as haptens. Fe,·cr. urticaria,
neutrophlls produced (takes 5 days). Immune complc\ CS arthralgia, proteinuria, lymphadenopathy
• 72 damage
endothelial cells form and are deposited in membranes, where occur 5-10 days after antigen exposure.
• 73
they fix complement (leads to tissue damage).
• 74
More common than Arthus reaction .
. 75
• 76
• 77
Arthus reaction- a local subacute antibody- Antigen-antibody complexes cause the Arthus
• 78 mediated hypersensitivity reaction . reaction .
• 79 Intradermal injection of antigen into a
• 80 prcsensitized (has circulat ing lgC) individual

I
. 81 leads to immune complex formation in the
• 82 skin. C haracterized by edema, necrosis, and
• 83 acti,·ation of complement.
• 84
Type IV Two mechanisms, each involving T cells: Response does not involve antibodies (vs t)'pes I,
. 85
. II, and Ill ). •
8
L.odt
s
Su~pl'nd
~
End Block
Item: 71 of - ,• Mark -<J
P~v1ous
[:::> "'I ~ ·· ~
100 ~ N @xt Labl lues N o tes Calcula to r
6

54 A 24-year-old woman who is sexually act ive presents t o a medical clinic with complaints of vaginal discharge. On examinat ion, she is found ~~AI
55 to have a copious, yellow-green, frot hy cervical discharge. Mot ile organisms are seen on wet-mount preparati on.
56
57 Which of the following is the appropri at e treatment for this woman's condition?
58 :
59 A. Ciprofloxacin
60 B. Doxycycline
61
C. Metronidazole
62
63 D. Penicillin
64 E. Trimethoprim-sulfamethoxazole
65
66
67
68
69
70
• 71
· 72
• 73
• 74
. 75
0 76
0 77
0 78
0
79
..
8
Lode.
s
Suspe-nd
8
End Bloc:k
Item: 71 of ~ ,• Mark <::J [:::> ""I ~· ~'j
100 J.. Previous Next LAb faiUI~S Notes Calculator


54
55
The correct a nswer is C. 7 3°/o chose this .
56
This pat ient is suffering f rom Trichomonas vagina/is, a mot ile parasit ic organism t hat is best visualized on wet mount showing t richomonads
57 (f lagellated organisms but larger than WBCs) . Trichomonas infection classically presents as a sexually t ransmitt ed infect ion causing yellow-
58 green, malodorous, frot hy vaginal discharge. The appropriate t reatment is oral met ronidazole, which is also used commonly fo r anaerobic
bact eri al infect ions, Giardia and Entamoeba. Adverse effect s of metronidazole include gastrointestinal discomfort and a disulf iram - like effect
59
when combined wit h alcohol.
60 Sexually transmitted infection Metronidazole Trichomonas vaginalis Flagellate Giardia Anaerobic organism vaginal discharge Parasitism Trichomonas Motility Flagellum Trichomonadida Entamoeba Organism
61 Microscope slide Human gastrointestinal tract
62
A is not correct . 6°/o chose this.
63
Ciprofloxacin is a broad-spect rum f luoroquinolone that is not used in t he t reat ment of Trichomonas vagina/is.
64 Quinolone Ciprofloxacin Broad-spectrum antibiotic

65
B is not correct. 9 % chose t his .
66
Doxycycline can be used to treat Chlamydia trachomatis infection, but it is not an appropriat e t reatment fo r Trichomonas vagina/is infection .
67 Chlamydia trachomatis Doxycycline Trichomonas vaginalis Chlamydia infection Chlamydia (genus) Trichomonas Infection

68
D is not correct. 5°/o chose t his.
69 Penicillin is a fi rst - line agent in t he t reat ment of syphilis, a sexually t ransm it ted disease.
70 Sexually transmitted infection Penicillin 9(philis

71
E is not correct. 7°/o chose this .
· 72 Trimet hoprim -sulfa met hoxazole is used commonly in t he t reat ment of urinary t ract infections, not in t he t reatment of Trichomonas vagina/is .
• 73 Trichomonas vaginalis Trimethoprim/sulfamethoxazole Urinary system Urinary tract infection Trichomonas

• 74
. 75
Bottom Line :
• 76
Yellow-g reen, frot hy vaginal discharge wit h mot ile organisms visible on wet mount are indicat ive of Trichomonas, which is t reated wit h
· 77 metronidazole.
• 78 Metronidazole Vaginal discharge Motility Trichomonas Microscope slide

• 79
.. •
8
Lock
s
Suspend
0
End Block
Item: 71 of ~ ,• Mark <::J [:::> ""I ~· ~'j
100 J.. Previous Next faiUI~S
LAb Notes Calculator

• •
54 FA17 p 191 .3
55 Metronidazole
56
MECHANISM Forms toxic free radical metabolites in the
57
bacterial cell that damage Ot A. Bactericidal,
58
59
anti protozoal.
60 CLINI CAL USE Treats Giardia, Entamoeba, Trichomonas, GET GAP on the Metro with metronidazole!
61 Cardn.erella vaginalis, Anaerobes (Bacteroides, Treats anaerobic infection below the diaphragm
62 C diffi.cile). Can be used in place of amoxicillin vs clindamycin (anaerobic infections above
63 in H jJylori "triple therapy" in case of penicill in diaphragm}.
64 allergy.
65
ADVERSE EFFECTS Disulfiram-like reaction (severe Rushing,
66
67
tachycardia, hypotension) with alcohol;
68
headache, metallic taste.
69
70 FA17 p 154.1
71 Protozoa- others
· 72
ORGANISM DISEASE TRANSMISSION DIAGNOSIS TREATM ENT
• 73
• 74
Visceral infections
. 75 Trypanosoma Chagas disease- dilated Reduviid bug Trypomastigote in Benzn idazole
• 76 cruzi cardiomyopathy with ("kissing bug") blood smear fJ or nifurtimox;
· 77 apical atrophy, megacolon, feces, deposited Cruzing in my
• 78 megaesophagus; predominantly in a painless bite Benz, with a fur
• 79 in South America (much Iike a kiss) coat on
.. • •

8
Lock
s
Suspend
0
End Block
Item: 71 of - ,• Mark --<) [::> ""'I ~· 1!';:'1
100 ~ Prev1o u s Next Labf a lu es Notes Calculator

54 FA17 p 154.1
55 Protozoa- others
56 ORGANISM OISEASE TRANSMISSION DIAGNOSIS TREATMENT
57
Visceral infections
58
59 Trypanosoma Chagas disease-dilated Rcduviid bug Trypomastigotc in Ben.r.n idazolc
60 cruzi cardiomyopathy with (" kissing bug") blood smear fl or nifurtimox;
61 apical atrophy, megacolon, feces, deposited Cru.r.ing in m)
62 megaesophagus; predominant!)' in a painless bite Benz, with a fur
63 in South America (much like a li.,.,) coat on
64 Unilateral periorbital swelling
65 (Romatia sign) characteristic of
66 acute stage
67
68
Leishmania Visceral leishmaniasis SandAy Macrophages Amphotericin B,
69
donovani (kala-azar) - spiking fevers, containing sodium
70 hepatosplenomegaly, a mastigotes : stibogluconate
71 pancytopenia
· 72 Cutaneous leishmaniasis-skin
• 73 ulcers
• 74
. 75
0 76
0 77 Sexually transmitted infections
0 78 Trichomonas Vaginitis-foul-smelling, greenish Sexual (cannot exist Trophozoitcs letron idazolc for
1

0
79
..
vagina/is discharge; itching and burning; outside human m
(motile) on wet patient and partner •
8
L.odt
s
Su~pl'nd
~
End Block
Item: 71 of - ,• Mark --<) [::> ""'I ~· 1!';:'1
100 ~ Prev1o u s Next Labf a lu es Notes Calculator

54 apical atrophy, megacolon, feces, deposited C ruzing in my


55 megaesophagus; predominantly in a painless bite Benz, with a fur
56 in South America (much like a li" ) coat on
57 Unilateral periorbital swelling
58 (Romaiia sign) characteristic of
59 acute stage
60
Leishmania Visceral leishmaniasis Sand fly ~ lacrophages Amphotericin B,
61
62 donovani (kala-azar)- spiking fevers, containing sodium
63 hepatosplenomegaly, amastigotes : stibogluconate
64 pancytopenia
65 Cutaneous leishmaniasis-skin
66 ulcers
67
68
69
70
Sexually transmitted infections
71 Trichomonas Vaginitis- foul-smelling, green ish Sexua l (cannot exist Trophozoitcs Metronidazole for
· 72 vagina/is discharge; itching and burning; outside human (motile) [!I on wet patient and partner
• 73 do not confuse with Cardnerella because it ca nnot moun t ; " st rawberry (prophyi<Jxis)
• 74
vagina/is, a gram-variable form cysts) ."
cerv1x
. 75
bacterium associated with
76
0
bacterial vaginosis
0 77
0 78
D
0
79
.. •
8
L.odt
s
Su~pl'nd
~
End Block
Item: 72 of - ,• Mark -<J [:::> "'I ~ · ~
100 ~ P~v1ous N @xt Labl lues N o tes Calcula to r
6

54 A pregnant woman comes to the physician fo r a check-up during her first trimester. She has been exposed, fo r t he first t ime, t o an ~~AI
55 infectious disease. The pat ient 's doct or reassured her t hat this infectious disease is not known to cause any adverse effects in fetuses, and
56 she continues with routine prenat al care. The woman later gives birth to a healthy child.
57
To which of the following pathogens was t he woman most likely exposed?
58
59 :
A. Cytomegalovirus
60
61 B. Epstein-Barr virus
62 C. Herpes simplex virus
63
0 . HIV
64
65 E. Rubella
66 F. Syphilis
67
G. Toxoplasmosis
68
69
70
71
· 72
• 73
• 74
. 75
0 76
0 77
0 78
0
79
..
8
Lode.
s
S uspe-nd
8
End Bloc:k
Item: 72 of - ,• Mark -<J [:::> "'I ~ · ~
100 ~ P~v1ous N @xt Labl lues N o tes Calcula to r

54
6

A is n ot correct. 6°/o chose this . •


55 Congenital cytomegalovirus (CMV) can result in hepatosplenomegaly, jaundice, and brain calcifications.
Cytomegalovirus Hepato5plenomegaly Jaundace Congenital disorder Human bram
56
57 C is n ot correct. 12°/o c h ose thi s.
58 Intrauterine herpes simplex virus (HSV) infection is rare but can result in a variety of congenital defects, spontaneous abortion, and neonatal
59
encephalitis. The vast majority of neonatal HSV infections are acquired in the birth canal and result in a very severe illness that can cause
seizures, high fever, brain damage, and death. Mothers who develop HSV infection during pregnancy must be treated with a regimen of oral
60 acyclovir or valacyclovir.
61 e n.a Vi. o;oCtclo 1r Encephalitis Herpes simplex Mascarriage 'varus \lac]•na ,.bortJon Prevnancy Congerutc.. dasorder PelVIS Epijeptic seizure fever Infant Infection Bnwn

62 D is not correct. 5°/o c h ose this.


63 Congenital HIV results in neonatal AIDS. Maternal highly active antiretroviral therapy treatment dramatically reduces maternal-to-fetal HIV
64 transmission.
Man>gtf'1Mt of HIVIAIDS HIV H!VIAIDS Congenital dtsorder Infant
65
66 E i s n ot correct. 4 °/o chose this .
67 Congenital rubella infection can result in deafness, patent ductus arteriosus, pulmonary artery stenosis, cataracts, and microcephaly. These
defects are associated with maternal infection in the first half of pregnancy.
68
Patent ductus artenosus Macrocephaly Pulmonary artery Ductus arteriosus Congenatal d1sorder Pulmonac 'ltenOSIS Rubella Stenosas Congemtal rubella syndrome Cataract Hearing loss
69 Stenos1s of pulmonary artery Pregnancy Infectaon
70
F is n ot correct. 4°/o chose this.
71
Congenital syphilis can result in cranial nerve VIII deafness, mulberry molars, saber shins, saddle nose, and Hutchinson teeth.
72
Syphilis Congenatal syph1hs Vesbbulocochlear nerve Hutchinson's teeth Saddle nose Congen1tal disorder Molar (tooth) Hearing loss
• 73
G is not correct. 3% chose thi s .
• 74
Congenital toxoplasmosis infection can resu lt in mental retardation and chorioretinitis .
. 75 Toxoplasmosis Chonoret1n1t1s Congemtal d1sorder Intellectual disability Infection
0 76
0 77
Bottom Lin e :
0 78
0
79
The ToRCHeS infections are toxoplasmosis, ru bella, CMV, HIV, HSV, and syphilis. Transplacental t ransmission of EBV is rare, and it is not
.. included in the ToRCHeS infections .

8
Lode.
s
S uspe-nd
8
End Bloc:k
Item: 72 of ~ ,• Mark <:::1 t::> ""I ~· ~'j
100 J.. Previous Next LAbfaiUI~S Notes Calculator

54 FA17 p 178.1
55
ToRCHeS infections Microbes that may pass from mother to fetus. Transmission is transplacental in most cases, or via
56
delivery (especially HSV-2). 1 onspecilic signs common to many ToRCHeS infections include
57
58
hepatosplenomegaly, jaundice, thrombocytopenia, and growth retardation.
59
Other important infectious agents include Streptococcus agalactiae (group B streptococci), E coli,
60 and Listeria monocytogenes- all causes of meningitis in neonates. Parvovirus Bl9 causes hydrops
61 fetalis.
62 AGENT MODES OF MATERNAL TRANSMISSION MATERNAL MANIFESTATIONS NEONATAL MANIFESTIONS
63
Toxoplasma gondii Cat feces or ingestion of Usually asymptomatic; Classic triad: chorioretinitis,
64
undercooked meat lymphadenopathy (rarely) hydrocephalus, and
65
66
intracran ia I calcifications,
67
+1- "blueberry muffin" rash m.
68
69
70
71
72
• 73 Rubella Respiratory droplets Rash, lymphadenopathy, Classic triad: abnormalities
• 74 polyarthritis, polyarthralgia of eye (cataract) and car
. 75 (deafness) and congenital heart
• 76 disease (PDA); ± "blueberry
· 77 muffin" rash. "I (eye) • ruby
• 78 (rubella) earrings."
• 79
.. Cvtomeaalovirus Sexual contact. organ Usuallv asvmntomatic: Hearing loss. seizures. netechial
8
Lock
s
Suspend
0
End Block
Item: 72 of - ,• Mark --<) [::> ""'I ~· 1!';:'1
100 ~
54
Prev1o u s Next Labf a lu es Notes Calculator
. . ... .-
disease (PDA); ± "blueberry
55
muffin" rash. "I (eye) • ruby
56
(rubella) earrings."
57
58 Cytomegalovirus Sexual contact, organ Usually as) mptomatic; Hearing loss, seizures, petechial
59 transplants mononucleosis-like illness rash, ''blueberry muffin" rash,
60 periventricular calcificalions
61
62
63
64
65
66
67 HIV Sexual contact, needlestick Variable presentation depending Recurrent infections, chronic
68 on CD4+ cell count diarrhea
69
Herpes simplex virus-2 Skin or mucous membrane Usually asymptomatic; herpetic Men ingoencephal it is, herpetic
70
71
contact (vesicular) lesions (vesicular) lesions
72 Syphilis Sexual contact C hancre (1°) and disseminated Often results in stillbirth,
• 73 rash (2°) arc the lwo stages hydrops fctalis; if child
• 74 likely to resu lt in fe tal infection survives, presents " ·ith facial
. 75 abnormalities (eg, notched
0 76 teeth, saddle nose, short
77
0
maxilla), saber shins, C ' Il l
78
0

deafness
0
79
.. •
8
L.odt
s
Su~pl'nd
~
End Block
Item: 73 of - ,• Mark -<J [:::> "'I ~ · ~
100 ~ P~v1ous N @xt Labl lues N o tes Calcula to r
6

54 A 36-year-old man comes t o the physician complaining of an aching back, high fever, and vomit ing of dark material. He is obviously ill and ~~AI
55 states that he has felt very poorl y for approximately 1 week. Physical examination shows that t he patient has a t emperature of 39°C
56 (102.2°F) and Icteric sclera. The pat ient recent ly return ed from a trip on safari in Africa. If a liver biopsy were done, it would show the
fo llowing pathology.
57
58
59
60
61
62
63
64
65
66
67
68
69
70
71 What are the names of the eosinophilic globules shown in t his image?
72 :
• 73 A. Councilman bodies
• 74 B. Dahle bodies
. 75
C. Mallory bodies
0 76
0 77 D. Negri bodies
0 78 E. Weibei -Palade bodies
0
79
..
8
Lode.
s
S uspe-nd
8
End Bloc:k
Item: 73 of - ,• Mark -<J [:::> "'I ~ · ~
100 ~ P~v1ous N @xt Labl lues N o tes Calcula to r
A A
54
55 The correct answer is A. 53°/o chose this.
56 The disease described is yellow fever, caused by a member of the Flaviviridae fami ly. It manifests ,...........,.,""""c ::.;~~.:::S!!I~il"l~ !IJ&:~;.~~~11IJ!II
57 with symptoms of jaundice, aching pain, and high fever. Its vector includes the Aedes (most
tropical and subtropical zones) and Haemogogus (South America) mosquitoes. Liver biopsy can
58 reveal Councilman bodies (green circles and blue arrows in image), which are eosinophilic
59 globules believed to be the result of apoptosis of individual hepatocytes. Necrosis of cells in
60 periportal zone 2 is commonly seen in yellow fever.
Ff .fa,e 'tel... rcve.. J*J dtce Apoptosts Counci man body Uver biopsy Biopsy South Amenca LJ~ Hepatoqte Aedes Necros•s
61
vector (ep•d..,.ology) Fever Mosqu•to
62
63
64
65
66 Image courtesy of CDC/ Dr. Williams
67
68
B is not correct . 11 °/o chose this.
69
Dahle bodies (blue arrow in the image) are oval bodies found in the neutrophils of patients wit h
infections, trauma, pregnancy, or cancer.
70 Neutrophtl Cancer Pregnancy

71
72
73
• 74
. 75
0 76
0 77
0 78
Image copyright © 2009 Modabbemia et
0
79 al.; licensee Cases Network Ltd.
..
8
Lode.
s
S uspe-nd
8
End Bloc:k
Item: 73 of - ,• Mark -<J [:::> "'I ~ · ~
100 ~ P~v1ous N @xt Labl lues N o tes Calcula to r
A A
54 C is not co rrect . 2 1 % chose this .
55 Mallory bodies are most commonly found in alcoholic hepat itis and alcoholic cirrhosis, but can be
56 manifest in a number of ot her con dit ions as well. They are eosinophillic inclusions in the
cytoplasm of swollen, necrotic hepatocyt es (indicat ed by the arrow in this image). They have a
57
more irregular and rope- like appearance compared to Councilman's bodies. •
_.,..
' ..
58 Alcoholic hep~1t ~ M llo 1 body C1rrhos1s Cytoplasm Hepatitis Hepatocyte Necros1s Alcohoh~m
59
60
61
62
63
64
65
0 is not correct . 9% chose this.
66
Negri bodies are pathognomonic for t he rabies virus. They are eosinophilic inclusion bodies
67 (indicated by the arrow in this image) found in the cytoplasm of nerve cells of infected
68 individuals.
Negri bod1es PathognomoniC Rab1es Cytoplasm Rabies virus I nclusion bodies Virus Neuron Eos1noph1IIC
69
70
71
72
73
• 74
. 75
0 76
E is not correct . 6 °/o chose t his.
0 77
Weibei-Palade bodies can be seen by elect ron microscopy in vascular endothelial cells (as
0 78 indicated by arrows in these images). They st ore and release von Willebrand factor and p-
0
79 selectin.
..
8
Lode.
s
S uspe-nd
8
End Bloc:k
59 E is not correct. 6°/o chose this.
60 Weibei -Palade bodies can be seen by electron microscopy in vascular endothelial cells (as
61 indicated by arrows in these images). They store and release von Willebrand factor and p-
selectin.
62
P-~etectJn \<On • wnmd factor Endothelium Electron mcroscope Blood vessel
63
64
65
66
67
68
69
Images copyright© 2012 Bai et at.;
70 licensee BioMed Central Ltd.
71
72
73
Bottom line:
• 74 Councilman bodies are eosinophilic globules seen on liver biopsy indicative of t he apoptosis of hepatocyt es characteri st ic of yellow fever.
Negri bodies are eosinophilic inclusion bodies within neurons seen in rabies .
. 75
Yellow fever Liver b1opsy Rab1es Eos~noph1hc Apoptosis Councilman body Negri bodies Biopsy Liver Inclus1on bod1e.. Hepatocyte Fever Neuron
• 76
• 77
• 78 I@ lifi 1 ·11-1 for year:
FI R ST AID FA CTS
2017 •
• 79
..
8
Lode.
s
Suspe-nd
8
End Bloc:k
Item: 73 of - ,• Mark -<J [:::> "'I ~ · ~
100 ~ P~v1ous N @xt Labl lues N o tes Calcula to r
A A
54
55 Ulifil·!ltl fo r year:
FIRST AID FACTS
2017 •
56
57
FA17 p 164.5
58
59 Yellow fever viru s A Aa,·ivirus (also an arbo,oirus) transmitted by Flari =yellow, jaundice.
60 Aedes mosquitoes. Virus has a monkey or
61 human reservoir.
62 Symptoms: high fever, black ,·omitus, and
63 jaundice. \ Jar see Councilman bodies
64 (eosinophilic apoptotic globules) on liver
65
biopsy.
66
67
68 FA17 p 163.1
69 RNA viru ses
70 VIRAL FAMILY ENVELOPE RNA STRUCTURE CAPSID SYMMETRY MEDICAL IMPORTANCE
71
Reoviruses No OS linear Icosahedral Coltivirusa - Colorado tick fever
72
10-12 segments (double) Rotavirus-causc of fata l diarrhea in child ren
73
• 74 Picornaviruses No SS ® linear Icosahedral Poliovirus-polio-Salk/Sabin vaccines- IPV/OPV
. 75 Echovirus-aseptic meningitis
0 76 Rhinovirus-"common cold"
0 77 Coxsackievirus-aseptic meningitis; herpa ngina
0 78 (mouth blisters, b ·er); hand, foot, and mouth
0
79 disease; myocarditis; pericarditis
.. t' \\ / __ ....... -=-- 1 L, __ __ .._ . .._._
8
Lode.
s
S uspe-nd
8
End Bloc:k
Item: 74 of - ,• Mark -<J [:::> "'I ~ · ~
100 ~ P~v1ous N @xt Labl lues N o tes Calcula to r
6

54 A 4-year-old child is brought to t he pediatri cian because of abdominal pain, vomit ing, and diarrh ea containing mucus and blood. The child ~~AI
55 has a fever of 39.4°C ( 103°F) . St ool cult ure and Gram stain reveal the causative organism to be a gram- negative, straight, rod-shaped,
56 nonlactose-fermenting, non-hydrogen-sulf ide-prod ucing bacterium that is extremely virulent.
57
Which is most likely to resu lt from cont inued infect ion with t his organism?
58
59 :
A. Ascending muscle weakness
60
61 B. Headache and rose spots on abdomen
62 C. Pulmonary hemorrhage, mediastinitis, and shock
63
0 . Renal failure, microangiopathic hemolytic anemia, and thrombocytopenia
64
65 E. Subcutaneous nodules, polyarthrit is, chorea, and a heart murmur
66
67
68
69
70
71
72
73
• 74
. 75
0 76
0 77
0 78
0
79
..
8
Lode.
s
Suspe-nd
8
End Bloc:k
Item: 74 of - ,• Mark --<) [::> ""'I ~· 1!';:'1
100 ~ Prev1o u s Next Labf a lu es Notes Calculator

54
• The correct answ er is D. 62°/o chose this.
55 Shigella is an invasive pat hogen producing febrile gast roent eri tis. It is a nonlactose fermenter and does not produce hydrogen sulfide.
Infection usually affects preschool-age children and nursing-home populations. Shigella is extremely virulent, req uiri ng only 10 organisms for
56
infection. Shigella dysenteriae also produces Shiga toxin, which can cause hemolytic uremic syndrome (HUS). The pathogenesis of HUS is
57 thought to involve Shiga toxin causing leukocyte-dependent inflammation by altering endothelial cell-adhesion properties and metabolism,
58 ultimately resu lting in microvascular thrombosis. It has been shown that fibrinolysis is markedly inhbit ed in HUS. Note that S dysenteriae is
not transmitted in the United States.
59
60
61
62
63
64
65
66
67
68
Hydrogen sulf1de Sh1ga tox1n Gastroentent1s Hemolytic-uremic syndrome Sh1gella Pathogen ThrombOSIS Endothelium H\drogen Metabolism Inflammation Toxin Hemolysis Infect1on V1rulence Fever
69
Industrial fermentation Pathogenesis
70
71 A is not correct. 11 °/o chose this.
72 This describes Guillain-Barre synd rome, which is often associat ed with Campylobacter jejuni infection. Alt houg h C j ejuni is a common cause of
dysentery, it does not fit this lab descript ion; it is a gram- negat ive organism with a characteri st ic curved-rod shape. Diagnosis is confirmed
73
with culture, and further testing wil l be posit ive for cat alase and oxidase.
74 Campylobacter Je]unl Catalase Campylobacter Gram-negative bacteria Dysentery Ox1dase Orgamsm Infection
. 75
B is not correct. 12°/o chose this.
76
0
These two symptoms, along with diarrhea and fever~ are characteristic of typhoid fever caused by Salmonella Typhi. On culture, it appears
0 77 motile and produces H2S. In addit ion, S Typhi does not cause dysentery.
0 78 Typhood levOJ 0., entery Ooarrhea Salmonella Mobhty Fever

0
79 C is not correct. 10% chose this .
.. •
8
L.odt
s
Su~pl'nd
~
End Block
Item: 74 of ~ ,• Mark <::J [:::> ""I ~· ~'j
100 J.. Previous Next LAbfaiUI~S Notes Calculator

54 B is not correct. 12% chose this .


55 These two symptoms, along wit h diarrhea and feve r~ are characterist ic of typhoid feve r caused by Salmonella Typhi. On culture, it appears
motile and produces H2 S. I n addit ion, S Typhi does not cause dysentery.
56
Typhoid fever Dysentery Diarrhea Salmonella Motility Fever
57
58
C is not correct. 10% chose this.
This describes inhaled infection with Bacillus anthracis, a gra m -posit ive rod, which is not connected to any of t he other sympt oms this child
59
has.
60 Bacillus anthracis Gram-positive bacteria Bacillus Infection

61
E is not correct. 5 °/o chose this.
62
These symptoms describe rheumatic feve r, which is caused by infect ion wit h Streptococcus pyogenes, a gram-posit ive coccus. S. pyogenes
63 does not cause dysent ery, nor does it fit the laborat ory descri ption given in t he stem (it is a coccus).
Streptococcus pyogenes Rheumatic fever Gram-positive bacteria Streptococcus Coccus Dysentery Infection Fever
64
65
66 Bottom Line:
67
Shigella is an invasive, nonlactose fermenter that does not produce hydrogen sulfide and can cause dysentery wit h feve r. Shigella
68 dysenteriae also produces a t oxin t hat may cause hemolyt ic uremic syndrome, which manifest s with t he typical t riad of anemia,
69 t hrombocyt openia, and acut e renal fa ilure.
Hydrogen sulfide Shigella Thrombocytopenia Hemolytic-uremic syndrome Anemia Acute kidney injury Dysentery Fever Toxin Hemolysis Hydrogen Kidney
70
71
72
i@l;fil·1i•J for year:[2017 • J
73 FIRST AID FACTS

74
. 75 FA17 p 141 .1
• 76
Salmonella vs Shigella Both Salmonella and Shigella are gram 8 rods, non-lactose fennenters, oxidase 8, and can invade
· 77
the Gllracl via M cells of Peyer patches.
• 78
Salmonella typhi Salmonella spp. Shigella
• 79
.. (pxrPnt .~ tvnhil
8
Lock
s
Suspend
0
End Block
Item: 74 of ~ ,• Mark <:::1 t::> ""I ~· ~'j
100 J.. Previous Next LAbfaiUI~S Notes Calculator

• •
54 FA17 p 141 .1
55
Salmonella vs Shigella Both Salmonella and Shigella are gram 8 rods, non-lactose fermenters, oxidase 8, and can invade
56
the Gllracl via I cells of Peyer patches.
57
58 Salmonella typhi Salmonella spp. Shigella
59 (except S typhi)
60 RESERVOIRS Humans only Humans and animals Humans only
61 SPREAD Can disseminate Can disseminate Cell to cell; no hematogenous spread
62
hematogenously hematogenously
63
64
H2SPRODUCTION Yes Yes I o
65 FLAGELLA Yes (salmon swim) Yes (salmon swim) 0
66 VIRULENCE FACTORS Endotoxin; Vi capsule Endotoxin Endoloxi n; Shiga toxin (enterotoxi n)
67
IN FECTIOUS DOSE (ID50) High- large inoculum High Low- ,·ery small inoculum required;
68
69 required because organism resistant to gastric acids
70 inactivated by gastric acids
71 EFFECT OF ANTIBIOTICS ON FECAL Prolongs duration Prolongs duration Shortens duration
72 EXCRETION
73 IMMUNE RESPONSE Primarily monocytes PM1 s in disseminated Primarily PM I in nitration
74
disease
. 75
Gl MANIFESTATIONS Constipation, followed by Diarrhea (possiblr bloody) Bloody diarrhea (bacillary dysentery)
• 76
· 77 diarrhea
• 78 VACCINE Oral vaccine contains )i,·e o ,·accme o vaccme
• 79 attenuated S typhi •
.. 8 s 0
Lock Suspend End Block
Item: 74 of ~ ,• Mark <::J [:::> ""I ~· ~'j
100 J.. Previous Next faiUI~S
LAb Notes Calculator

• •
54 FA17 p 128.1
55
Bugs with exotox ins
56
BACTERIA TOXIN MECHANISM MANIFESTATION
57
Inhibit protein synthesis
58
59 Corynebacterium Diphtheria toxin3 Pharyngitis with pseudomembranes in th roat
60 diphtheriae Inactivate elongation factor and severe lymphadenopathy (bull neck)
61 (EF-2)
Pseudomonas Exotoxin A3 Host cell death
62
aeruginosa
63
64 Shigella spp. Shiga toxin (ST)3 GI mucosal damage -+ dysentery; ST also
65 Inactivate 60S ribosome by enhances cytokine release, causing hcmolrtic-
66 removing adenine from uremic syndrome (HUS)
67 Enterohemorrhagic Shiga-!ike toxin rRNA SLT en hances cytokine release, causing HUS
68 E coli (EHEC) (SLT)a (prototypically in EHEC serotype 0157:H7).
69
Unlike Shigella, F.HEC does not invade host
70
cells
71
72 Increase fluid secretion
73 Enterot oxigenic Heat-labile Overactivates adenylate Watery diarrhea: "labile in the Air (Adenylate
74 E coli (ETEC) toxin (LT)3 cyclase (t cAMP) - t CJ- cyclase), stable on the Ground (Guanylate
. 75 secretion in gut and H20 cyclase)"
• 76 efflux
· 77
Heat-stable Overactivates guanylate
• 78
toxin (ST) cyclase (t cGMP)
• 79
.. • - l resorption of aCI •

8
Lock
s
Suspend
0
End Block
Item: 74 of ~ ,• Mark <::J [:::> ""I ~· ~'j
100 J.. Previous Next LAbfaiUI~S Notes Calculator


54
FA17 p 175.1
55
Bugs causing diarrhea
56
57
Bloody diarrhea
58 Campylobacter Comma- or S-shaped organisms; growth at 42°C
59
E histolytica Protozoan; amebic dysentery; liver abscess
60
61 Enterohemorrhagic 0157:H7; can cause llUS; makes Shiga-like toxin
62 Ecoli
63 Enteroinvasive Ecoli Invades colonic mucosa
64
Salmonella (non- Lactose 8; Aagellar motility; has animal reservoir, especially poultry and eggs
65
typhoidal)
66
67 Shigella Lactose 8; very low 10 ;0; produces Shiga toxin (human reservoir only); bacillary dysentery
68 Yenterocolitica Day care outbreaks, pseudoappendicitis
69
Watery diarrhea
70
71 Cdifficile Pseudomembranous colitis; associated with antibiotics and PPis; occasionally bloody diarrhea
72
C perfringens Also causes gas gangrene
73
74
Enterotoxigenic E coli Tra,·elers' diarrhea; produces heat-labile (LT) and heat-stable (ST) toxins
. 75 Protozoa Giardia, Cryptosporidium
• 76
Vcholerae Comma-shaped organisms; rice-water diarrhea; often from infected seafood
· 77
• 78
Viruses Rota,·irus, norovirus, adenovirus
• 79
.. •
8
Lock
s
Suspend
0
End Block
Item: 75 of - ,• Mark --<) [::> ""'I ~· 1!';:'1
100 ~ Prev1o u s Next Labf a lu es Notes Calculator

54 • A 3-week-old girl is brought to t he emergency depart ment by her mother, who says her daughter has suddenly developed a large, tense
bulge on the top of her head and a fever. The mother also notes that the patient has become more irritable and has not been feeding well.
55 This all occurs in the setting of a recent upper respiratory infection diagnosed 2 days ago. The child was born at 38 weeks ' gestation via
56 spontaneous vaginal delivery without complications. The patient undergoes lumbar puncture, and a Gram st ain of cerebrospinal fluid is shown in
57
the image.
58
59
60
61
62
63
64
65
66

I
67
68
69

/
..
70
71
72 Which of the fol lowing best characterizes t he disease-causing agent in this patient?
73
:
74
A . Gram -negative bacilli, lactose ferm ent er
. 75
B. Gram -negative coccobacilli, grows on chocolate agar with factors V and X
0 76
0 77 C. Gram-positive bacilli, facu ltative int racellula r
0 78 D . Gram-positive cocci, a-hemolytic, optochin -sensitive, bile-soluble
0
79
.. E. Gram-oositive cocci. 8-hemolvtic. bacitracin-resistant •
8
L.odt
s
Su~pl'nd
~
End Block
Item: 75 of - ,• Mark -<J [:::> "'I ~ · ~
100 ~ P~v1ous N @xt Labl lues N o tes Calcula to r
A A
54
The correct a ns wer is E. 65°/o chose this.
55
This patient's presentation is highly suggestive of bacterial meningitis. In the neonate, clinical symptoms Most common bacterial causes of
56 are nonspecific (fever, irritab ility, lethargy, poor feeding) relative to those in older children or adult s (nuchal meningitis by age
57 rig idity) . The most common cause of neonat al meningitis is group B Streptococcus (GBS), specifically -
Age Most lmponant Bacteria
Streptococcus agalactiae, a (3-hemolytic, gram- positive coccus found in chains. In contrast with its group A
58 Group B streptococci
counterparts, S. agalactiae is bacitracin resistant . Early-onset GBS infection usually manifests in the first 24 N90f\a19 Eseill>llttwleoli
59 hours of life. It occurs through vertical transmission from the birth canal during delivery, as 40% of women Usteria
60 are asymptomatic carriers of this bacterium in the gastrointestinal tract and vagina. Prenatal screening for Children Streptoooocus pne1J17>0111ao
GBS colonization has greatly decreased the number of early-onset GBS infection. Late-onset GBS infection
61 Young adults Neisseria men/ngllidis
occurs in the 4-5 weeks of life and is independent of prenatal care. It occurs as a result of community
62 acquired GBS infection and is typically preceded by an upper respiratory infection. Preterm infants are at Aduls SlrepiOCOCCIJ$ pno1J17>0111ao
--
63 increased risk of contracting both early- and late-onset GBS. The table details the most common causes of
bacterial meningitis by age.
64
Streptococcu agar t•at. Gra pos•b 'e bacter•a Meningitis Infant Human gastrointestinal tract Streptococcus wrt•cally transm•tted infection Asymptomatic Gastrointestinal tract Neck sbffness Sactena Vao•na
65
Upper r~ptrato(\j tra,:t tnfecteon Coccus Dysphagia lethargy Respiratory tract infection Fever lrrttabtltty Preterm b1rth Prenatal care Meningism Asymptomatic carrier Pelvis
66
67
A is not correct. 4 °/o chose this.
Escherichia coli is a gram-negat ive, lact ose-fe rmenting bacillus. It is the second leading cause of neonatal meningit is.
68 Eschertchta colt Gntm~negat•ve bactena Men•ng•bs Bacillus (shape) Bacillus Infant Neonatal meningitis
69
B is not correct. 4 °/o chose this .
70
Haemophilus influenzae type B ( Hib) is a small gram-negat ive encapsulated coccobacillus that was once a major cause of serious bacterial
71
infections, including meningit is, sepsis, and epiglottit is. It req uires factors V and X to successfully cult ure on chocolate agar. However, the
72 incidence of H. influenzae meningit is has decreased significant ly in the last 10-15 years as a result of t he wid espread adm inistration of the
73 Hib vaccine. Nevertheless, Hib still causes 5% of cases of meningit is in children ages 6 months t o 6 years.
Ep1glott1t1s Haemoph1lus 1nfluenzae Gram-negative bacteria Coccobacillus Chocolate agar H1b vaccine Sepsis Men1ng1t1s vacc~ne Haemophilus
74
75 C is not correct . 5°/o chose this.
0 76 Listeria monocytogenes is the third most common cause of neonatal meningitis (2%). It is a gram-posit ive bacillus.
Listeria monocytogenes Men1ng1bs Gram-pos1bve bacteria Listeria Bacillus (shape) Bac1llus Inf-.nt
0 77
0 78 0 is not correct . 22% chose this.
0
79
Streptococcus pneumoniae is a gram-positive coccus that is found in chains, but it is not a significant cause of meningitis in neonates.
.. However, in children ages 6 months to 6 years, it is the leading cause of meningitis .

8
Lode.
s
Suspe-nd
8
End Bloc:k
Item: 7S of ~ ,• Mark <::J [:::> ""I ~· ~'j
100 J.. Previous Next LAbfaiUI~S Notes Calculator

54
55
Bottom Line:
56
The most likely cause of meningit is in a neonat e is Streptococcus agalactiae. Gram stain of cere brospinal fluid shows gram- posit ive cocci in
57 chains.
58 Gram staining Cerebrospinal fluid Streptococcus agalactiae Coccus Meningitis Gram-positive bacteria Streptococcus Infant

59
60
61 l@l;fil·1i•l for year:[ 2017
FIRST AID FACTS .
•j .
62
63 FA17 p 176.1
64 Common causes of meningitis
65
NEWBORN (0- 6MOl CHILDREN (6 M0- 6 YR) 6- 60YR 60YR+
66
67
Group B streptococci S pneumoniae S pneumoniae S pneumoniae
68
E coli N meningitidis N meningiticlis (#1 in teens) Gram 8 rods
69
Listeria H infiuenzae type B Enterovi ruses Listeria
70 Enteroviruses HSV
71 Give ceftriaxone and vancomycin empirically (add ampicillin if Listeria is suspected).
72 Viral causes of meningitis: enteroviruses (especially coxsackievirus), llSV-2 {llSV-1 =encephalitis), IIIV, West ile virus (also
73 causes encephalitis), VZV.
74 In HIV: Cryptococcus spp.
75
Note: Incidence of H infiuenzae meningitis has l greatly due to conjugate H infiuenzae vaccinations. Today, cases are usually
• 76
seen in unimmunized children.
· 77
• 78
• 79 FA17 p 133.1
..
8
Lock
s
Suspend
0
End Block
Item: 75 of - ,• Mark --<) [::> ""'I ~· 1!';:'1
100 ~ Prev1o u s Next Labf a lu es Notes Calculator

54
Croup B streptococci S pnewnoniCie S pneumoniCie S pneumoniCie
55
E coli meningitidis meningitic/is (Il l in teens) Cram 8 rods
56
ListeriCI H influenzae type B F:nteroviruses Listeria
57
E nterovi ruses H V
58
59 Give ceftriaxone and vancomycin empirically (add ampicillin if ListeriCI is suspected).
60 Viral causes of meningitis: enteroviruses (especially coxsackie' irus), IISV-2 (IISV-1 =encephalitis), HIV, West ile virus (also
61 causes encephalitis), ZV.
62 In HI V: Cr)'plococcus spp.
63 'ole: Incidence of H influen;:.ae meningitis has l greatly due to conjugate H influen;:Cie vaccinations. Today, cases arc usually
64 seen in unimmunized children.
65
66
67
FA17 p 133.1

68 Streptococcus Cram ED cocci, bacitracin resistant, ~-hemolytic, Croup B for Babies!


69 agalactiae (group 8 colonizes ,·agina; causes pneu lllonia,
70 streptococci) meningitis, and sepsis, mainly in babies.
71 Produces CAMP fac tor, which enlargcs the
72
area of hemolysis formed by S Mtreus. ( otc:
73
CAMP stands for the authors of the test, not
74
cyclic A IP.) Hippurate test ED. PYR 8 ,
75
Screen pregnant women at 35-37 weeks of
0 76
0 77
gestation with rectal + vaginal swabs. Patients
0 78
with ED culture receive intrapartum penicillin
0
79
prophylaxis.
.. •
8
L.odt
s
Su~pl'nd
~
End Block
Item: 76of
100
~ , .1 • Mar k
~
<1
Previous
t>
Next Lab
d\ ues
•• .
Notes
~
calculator

54 A 17-year-old boy visits his physician with complaints of recurrent bouts of dizziness, palpitations, and joint pain. He went ~~AJ
55 on a summer hiking trip in eastern Massachusetts about 6 weeks ago and recalls m inor bug bites but no rashes. The ECG
56
is shown in the image .
57
58
59
60
61
62
63
64
65
66
67 Image courtesy of James Heilman, MD
68
69 What is the most likely diagnosis of this patient's symptoms?
70 :
71 A. Brugada syndrome
72
B. Chagas disease
73
C. Fam ilial second-degree heart block
74
75 D. Hypertrophic card iomyopathy
• 76
E. Lyme disease

a
Lock
s
Suspend
o
End Block
Item: 76 of ~ ,• Mark <::J [:::> ""I ~· ~'j
100 J.. Previous Next LAbfaiUI~S Notes Calculator


54
55 The correct answer is E. 69°/o chose this .
Lyme disease is caused by infection wit h t he spirochet e Borrelia burgdoderi and is t ransmit ted by the bit e of t he
56
Ixodes t ick ( pictured here). Init ially, t he disease manifests wit h const it utional sympt oms such as feve r and malaise, as
57 well as a rash surrounding t he bit e sit e. However, the bit e site often goes unnot iced, and erythema ch ron icum m igrans
58 is not necessari ly present in every case. Early dissem inat ed disease manifests 4-6 weeks after the init ial infection and
is characteri zed by cardiac and neurolog ic abnormalit ies. Cardiac abnormalit ies include myocardit is, arrhythmias (eg,
59
AV hea1t block), and conduct ion disturbances. The ECG in t he stem shows no relationship bet ween t he P waves and
60 QRS complexes, suggest ing t hat t he at ri a and vent ricles are beat ing independent ly (AV dissociation), an exam ple of
61 t hird-degree AV heart block. Lyme arthri t is is a late-st age finding, occu rs in about 60% of patients months to years
l ate 1~ and is associated wit h pain and swelling of large joints, most often in one or bot h knees. Lyme disease is most
62
preva lent in stat es along t he nort heast Atlant ic Coast, but cases have been reported t hroughout the Unit ed Stat es.
63 Spirochaete Lyme disease Borrelia burgdorferi Myocarditis Erythema Heart block Atrium (heart) Malaise Electrocardiography Tick Cardiac arrhythmia Borrelia Ixodes Rash
Image cowtesy of
64 Arthritis Fever Erythema chronicum migrans Ventricle (heart) Constitutional symptoms Infection
CDC/Or. Amanda
65 Loftis, Dr. William
66 Nicholson, Dr. Will
Reeves, and Dr. Chris
67
Paddock
68
69 A is not correct. 7°/o chose this.
70 Brugada synd rome is a conduct ive heart disease t hat usually affects young men and carries an increased risk of sudden ca rdiac death . The
disease has been associated wit h sod ium ion channel abnormalit ies. The t ypical ECG pattern is a right bund le-bra nch block and ST-segment
71
elevation on leads V1 -V 3 .
72 Sodium channel Brugada syndrome Sudden cardiac death Ion channel Electrocardiography Bundle branch block Sodium Cardiovascular disease Ion

73
B is not correct. 9 % chose this.
74
Chronic Chagas disease usually manifests duri ng its earliest phase wit h arrhythmias (eg, heart block and ventri cular tachycardia). Dilat ed
75 cardiomyopat hy, megacolon, and megaesophagus occur later in t he course of t he disease. The disease manifests acutely after the t ransfer of
76 Trypanosoma cruzi (found in the sout hern United States, Mexico, and Central and Sout h Ameri ca) by t he reduviid bug (also ca lled t he kissing
bug). Transmission is associated wit h a ha rdened red area or chagoma . This is fo llowed by feve r, ma laise, lymphadenopat hy, tachyca rdia, and
· 77
men ingoencephalit is t hat resolve wit hin 1 mont h. The patient's ECG t racing (in t he vignet te) shows classic t hird-degree heart block. Chagas
• 78 disease, however, is associated wit h dilat ed cardiomyopat hy and would be a rare diagnosis in the northern Un it ed Stat es.
• 79 Chagas disease Trypanosoma cruzi Megaesophagus Dilated cardiomyopathy Heart block Chagoma Megacolon Ventricular tachycardia Meningoencephalitis Lymphadenopathy Cardiomyopathy Triatominae

.. •
8
Lock
s
Suspend
0
End Block
Item: 76 of ~ ,• Mark <::J [:::> ""I ~· ~'j
100 J.. Previous Next LAb faiUI~S Notes Calculator

54 C is not co rrect. 10% chose this .


55
I n second-degree heart block, some (but not all) atri al impulses fai l to conduct to the ventricular system . This fa ilure in conduct ion leads to
"dropped" beats. There are t wo t ypes of second-degree heart block, type 1 (also known as Mobit z I or Wenckebach), and t ype 2 (also known
56 as Mobitz II) . I n type 1 second-degree AV block, t here is a characteristic progressive lengthening in t he PR int erval leading up to the dropped
57 beat. This ty pe of block is usually benign, and often req uires no treat ment. Type 2 second-degree AV block is a fa ilure furt her down in the
58
conduction syst em (usually below t he AV node), t hat shows int ermit tent dropped beats, but a consistent PR interval. This ty pe of block may
prog ress t o com plet e heart block. Definit ive t reatment is implantat ion of a pacemaker. Though t here is a fami lial version of second-degree
59 block, Lyme disease-re lat ed heart block is still t he most likely answer here given ot her clinical manifest ations. In contrast, in t hird-degree
60 heart block (ie, complete heart block), none of t he at rial impulses are conduct ed, leaving the at ria and vent ricles to depolarize separately at
61 t heir respective intrinsic rat es.
Heart block Second-degree atrioventricular block Atrium (heart) PR interval Third-degree atrioventricular block Atrioventricular node Depolarization Ventricle (heart) Artificial cardiac pacemaker
62
Atrioventricular block Bundle branch block Karel Frederik Wenckebach Benignity
63
64 D is not correct. 5°/o chose this.
65 Hypertrophic cardiomyopathy is t he most common cause of deat h in young at hletes in the Unit ed States. It is characterized by an asymmetri c
hypert rophic, nondilat ed left vent ricle. Hist opat holog ically, t he myocardial archit ecture is disorganized and scarred. The t ypical ECG shows
66 repolarization changes or frank hypertrophy.
67 Hypertrophic cardiomyopathy Ventricle (heart) Cardiomyopathy Hypertrophy Electrocardiography Ventricular hypertrophy Repolarization Ventricular system United States

68
69
Bottom Line :
70
Lyme disease is a cause of hea1t arrhyt hmias in young people.
71 Lyme disease Cardiac arrhythmia

72
73
74 l@);fil ~1hl for yea r:l 2o17 y
FIRST AID FACTS
75
76
FA17 p 284. 1
· 77
• 78
ECG tracings
RHYTHM DESCRIPTION EXAMPLE
• 79
.. ~·
.. . .. ... . ~ . .

8
Lock
s
Suspend
0
End Block
Item: 76 of ~ ,• Mark <::J [:::> ""I ~· ~'j
100 J.. Previous Next faiUI~S
LAb Notes Calculator

54 FA17 p 284. 1
55 ECG tracings
56 RHYTHM DESCRIPTION EXAMPLE

. .
57
Atrial fibrillation Chaotic and erratic baseline with no discrete P waves in between RR, ~ RR2 • RR1 • RR,
58
irregularly spaced QRS complexes. irregularly irregular '
59
heartbeat. Most common risk factors include hypertension and
60
coronary artery disease (CAD). Can lead to thromboembolic
61 Irregular baseUne (absent P waves)
events, particularly stroke.
62
63
Treatment includes anticoagulation, rate control, rhythm control,
64
and/or cardioversion.
65 Atrial flutter A rapid succession of identical, back-to-back atrial depolarization RR, =
66 waves. T he identical appearance accounts for the "sawtooth"
67 appearance of the flutter waves.
68 Treat like atrial fibrillation. Definitive treatment is catheter
69 4l sawtooth pattern
ablation.
70
Ventricular A completely erratic rhythm with no identifiable waves. Fatal
71
72
fibrillation arrhythmia without immediate CPR and defibri llation.
73
74 No discernible rhythm
75
AV block
76
· 77 First degree The PR interval is prolonged (> 200 msec). Benign and
• 78 asymptomatic. l o treatment required .
• 79
..
8
Lock
s
Suspend
0
End Block
Item: 76 of - ,• Mark -<J [:::> "'I ~ · ~
100 ~ P~v1ous N @xt Labl lues N o tes Calcula to r
A A
54
AV block
55
56 First degree The PR interval is prolonged (> 200 msec). Benign and
57 asymptomatic. lo treatment required.
58
59
II
60
61 Second degree
62 Mobitz type I Progressi,·e lengthening of PR inter\'aluntil a beat is "dropped"
63 (Wenckebach) (a P \\'a,·e not followed by a QRS complex). Usually
64
asrmptomatic. Variable RR inten·al with a pattern (regularly
65 0-..J
irregular). Pwave. abselt ORS 11
66
67 Mobitz type II Dropped beats that arc not preceded by a change in the length of
68 the PR interval (as in type 1).
69 May progress to 3rd-degree block. Often treated wil h pacemaker.
70 '--' '--' ._____t
PR1 • PR, P wave, absent ORS 11
71
72 Third degree The atria and ,·entricles beat independently of each other. P waves and QRS complexes not rhythmica lly
73 (complete) associated. Atrial rate> ventricular rale. Usually lrealed with pacemaker. Can be caused by Lyme
74 disease.
75
76
• RR1

• 77
• 78
• 79 PP1 • PP1 • PP1 • PP, II
.
8
Lode.
s
Suspe-nd
8
End Bloc:k
Item: 76 of - ,• Mark -<J [:::> "'I ~ · ~
100 ~ P~v1ous N@xt Labl lu es No tes Calcula t o r

A A
54 FA17 p 142.4
55 Lyme disease Caused by Borrelia burgdorferi, which is A Key Lyme pie to the FACE:
56
transmitted by the Ixodes deer tick (also Facial nerve palsy (typically bilateral)
57
,·ector for Anaplasma spp. and protozoa Arthritis
58
Babesia). l atural rescn·oir is the mouse (and Cardiac block
59
important to tick life cycle). Er} thema migrans
60
Common in northeastern United States. Treatment: doxrcycl ine (lst line); amoxicillin
61
62
Stage !-early localized: erythema migrans and cefuroxime in pregnant women and
63
(typical ''bulls-eye" configuration •s children.
64
pathognomonic but not always present),
65 Au-like symptoms.
66 Stage 2-early disseminated: secondary lesions,
67 carditis, AV block, facial nerve (Bell) palsy,
68 migratory myalgias/transient arthritis.
69 Stage 3- late disseminated: encephalopathies,
70 chronic arthritis.
71
72
73
74 FA17p 144. 1
75
Zoonotic bacteria Zoonosis: infectious disease transmitted between animals and humans.
76
• 77 SPECIES DISEASE TRANSMISSION AND SOURCE
• 78 Anaplasma spp. Anaplasmosis Ixodes ticks (live on deer and mice)
• 79
Bartonella spp. Cat scratch disease, bacillary angiomatosis Cat scratch
.
8
Lode.
s
Suspe-nd
8
End Bloc:k
Item: 76 of - ,• Mark --<) [::> ""'I ~- 1!';:'1
100 ~ Prev1o u s Next Labf a lu es Notes Calculator

• •
54
FA17 p 144.1
55
56 Zoonotic bacteria Zoonosis: infectious disease transmitted between animals and humans.
57 SPECIES DISEASE TRANSMISSIONAND SOURCE
58
Anaplasma spp. Anaplasmosis Ixodes ticks (live on deer and mice)
59
60 Bartonella spp. Cat scratch disease, bacillarr angiomatosis Cat scratch
61 Borrelia burgdorferi L} me disease Ixodes ticks (live on deer and mice)
62
Borrelia recurrentis Relapsing fever Louse (recurrent due to variable surface
63
antigens)
64
65 Brucella spp. Brucellosis/undulant fe\'er Unpasteuri7ed dairy
66 Campy/obacter Bloody diarrhea Feces from infected pets/animals; contaminated
67 meats/foods/hands
68
Chlamydophila psittaci Psittacosis Parrots, other birds
69
70 Coxiella burnetii Q fever Aerosols of cattle/sheep amniotic Auid
71 Ehrlichia chaffeensis Ehrlich iosis Amblyomma (Lone Star tick)
72
Francisella tularensis Tularemia Ticks, rabbits, deer Aies
73
74 Leptospira spp. Leptospirosis Animal urine in water; recreational water use
75 Mycobacterium leprae I .eprosy llumans with lepromatous leprosy; armadillo
76
(rare)
• 77
• 78
Pasteurella multocida Cellulitis, osteomyelitis nimal bite, cats, dogs
• 79 Rickettsia prowazekii Epidemic trphus Human to human \'ia human body louse •
8
L.odt
s
Su~pl'nd
~
End Block
Item: 77 of - ,• Mark -<J [:::> "'I ~ · ~
100 ~ P~v1ous N @xt Labl lues N o tes Calcula to r
6

54 A type of bunyavirus seen in t he United Stat es is characterized clinically by a prodromic phase of nonspecific symptoms including fever, ~~AI
55 myalgia, headache, nausea, and vomiting . This init ial phase is fol lowed by a very rapid transit ion t o a cardiopulmonary phase characterized
56
by pulmonary edema (due to leaky pulmonary capillari es), hypotension, and coagulopat hy.
57
What is a natural reservoir for this pathogen?
58
59 :
A. Bats
60
61 B. Rodents
62 C. The reservoir is unknown
63
0 . Ticks
64
65
66
67
68
69
70
71
72
73
74
75
76
. 77
• 78
• 79
. •
8
Lode.
s
S uspe-nd
8
End Bloc:k
Item: 77 of ~ ,• Mark <:::1 t::> ""I ~· ~'j
100 J.. Previous Next LAbfaiUI~S Notes Calculator


54
55 The correct answer is B. 67% chos e this.
56 The clinical scenari o describes hant avirus cardiopulmonary syndrome. Rodents are the natural reservoir of the hantavirus. Contrasting wit h
57 most ot her bunyavirus, which are t ransmitted by art hropod vectors (mosquit oes, t icks, or sand flies), t ransm ission of hant avirus occurs
t hrough cont act with deer mice feces . I n t he Unit ed St at es ra re cases of huma n disease have been described in rura l areas of the Southwest.
58 The only ot her bunyavirus that might present wit h sim ilar sympt oms is Cri mean-Congo hemorrhagic fever. This virus, however, is not known
59 t o be present in North Ameri ca ( rat her, Africa, the Middle East , and East ern Europe) .
Bunyaviridae Arthropod Hantavirus Peromyscus Crimean-Congo hemorrhagic fever Natural reservoir Viral hemorrhagic fever Feces North America Rodent Mouse Virus Sandfly Middle East Fever Mosquito
60
61 Europe Africa Tick

62 A is not correct . 13% chos e this.


63 It has been hypot hesized t hat f ruit bats are a reservoir for t he filov irus ebola in sub-Saharan Africa. They are not a reservoir for hant avirus.
Filoviridae Hantavirus Sub-Saharan Africa Ebola virus disease Megabat Africa
64
65 C is not correct. 8 °/o chos e this .
66 Rodents are t he known reservoir for hant avirus.
Hantavirus Rodent
67
68 D is not correct. 12% chose this .
69 Ticks are the vect or for a number of viral, bacterial, and protozoal diseases. They are not, however, the rese rvoir or t he vect or for hantavirus.
Hantavirus Vector (epidemiology) Protozoa Tick
70
71
72 Bottom Line:
73 Hant avirus is carri ed by rodents, and presents wit h nonspecific symptoms followed by pulmonary edema, hypotension, and coagulopathy.
74 Coagulopathy Pulmonary edema Hantavirus Hypotension Edema Rodent

75
76
77 141;fil·1i•J
FIRS T AID FACTS
for year:[2017 •
• 78
• 79 FA17 p 163.1
.. •
8
Lock
s
Suspend
0
End Block
Item: 77 of ~ ,• Mark <:::1 t::> ""I ~· ~'j
100 J.. Previous Next LAb faiUI~S Notes Calculator

• •
54 FA17 p 163. 1
55 RNA viruses
56 VIRAL FAMILY ENVELOPE RNA STRUCTURE CAPSID SYMMETRY MEDICAL IMPORTANCE
57
Reoviruses 0 OS linear Icosahedral Coltivirus" - Colorado tick fever
58
10-12 segments (double) Rotavirus-cause offatal diarrhea in children
59
60 Picornaviruses No SS ®linear Icosahedral Poliovirus-polio-Salk/Sabin vaccines-IPV/OPV
61 Echovirus- aseptic meningitis
62 Rhi novirus-"common cold"
63 Coxsackievirus- aseptic meningitis; herpangina
64 (mouth blisters, fever); hand, foot, and mouth
65 disease; myoca rditis; pericarditis
66 IIAV-acute viral hepatitis
67 PERCI I
68 Hepevirus No SS ®linear Icosahedral HEV
69
70
Caliciviruses No SS ® linear Icosahedral 'orovirus-viral gastroenteritis
71 Flaviviruses Yes SS ® linear Icosahedral HCV
72 Yellow fever1
73 Dengue3
74 St. Louis encephalitis 3
75 West Tile virus3 (meningoencephalitis)
76 Zika virus
77
Togaviruses Yes SS ®l inear Icosahedral Rubella
• 78
Western and Eastern equine encephalitis"
• 79
Chikum!llnva ,·irus •
.. •
8
Lock
s
Suspend
0
End Block
Item: 77 of - ,• Mark -<J [:::> "'I ~ · ~
100 ~ P~v1ous N @xt Labl lues N o tes Calcula to r
A A
54 Retroviruses Yes SS ® linear Icosahedral ! lave re,·erse transcriptase
55 2 copies (HTLV), HTLV-T-cellleukemia
56 complex II IV-AIDS
57 and conical
58 (HIV)
59
60
Coronaviruses Yes SS ® linear Helical "Common cold," SARS, \fERS
61 Orthomyxoviruses Yes sse linear Helical lnAuenza virus
62 8 segments
63
64
Paramyxoviruses Yes sse linear Helical PaRaMno,·irus:
'
1 onsegmented ParainAucnza-croup
65
RSV-bronchiolitis in babies; Rx- ribavirin
66
\lcaslcs, \lumps
67
68 Rhabdoviruses Yes sse linear Helical Rabies
69 Filoviruses Yes sse linear Helical Ebola/ 1arburg hemorrhagic fever-often fatal!
70
71
Arenaviruses Yes SS®and 8 f felica l I.CMV- lymphocytic choriomeningitis virus
72
circular Lassa feve r encephalitis-spread by rodents
73 2 segments
74 Bunyaviruses Yes ss e circular Helical Ca Iiforn ia encephal itis 3
75 3 segments S<l11d ny/Rift Valley fevers 3
76 Crimean-Congo hemorrhagic fe,·e~
77 llantavirus- hemorrhagic fever, pneumonia
• 78
• 79
Delta virus Yes ss e circular Uncertain HDV is a "defective'' ,-irus that requires the
. • presence of II BV to rep) icate
8
Lode.
s
S uspe-nd
8
End Bloc:k
Item: 78 of - ,• Mark -<J [:::> "'I ~ · ~
100 ~ P~v1ous N @xt Labl lues N o tes Calcula to r

54 A A 41-year-old man who underwent kidney t ransplantation 4 months ago presents to t he emergency depart ment with a nonproductive ~AI A

cough and shortness of breath. He reports t hat his symptoms started about 3 weeks ago and have been gradually getting worse. He
55
admits to recently losing his health insurance and has been unable to refill some of his medications but is unsure of which ones. The
56 patient's temperature is 38.5°C (101.3°F}, blood pressure is 132/81 mm Hg, pulse is 96 beats/min, respi ratory rate is 30 breaths/min, and
57 oxygen saturation is 91%. His chest CT appears in the image.
58
59
60
61
62

63
64
65
66
67
68
69
70
71
Subsquent bronchioal veolar lavage was diagnost ic, while also demonstrating foamy alveolar cast s. Wh ich of the fo llowing would be most
72
beneficial fo r this patient?
73
:
74 A. Azithromycin
75
B. Fluconazole or ketoconazole
76
77 C. Ganciclovir
. 78 D. Sulfadiazine and pyrimethamine
• 79
. E. Trimethoorim-sulfamethoxa zole
8
Lode.
s
Suspe-nd
8
End Bloc:k
Item: 78 of - ,• Mark -<J [:::> "'I ~ · ~
100 ~ P~v1ous N@xt Labl lu es No tes Calcula t o r

A A
54
The correct answ er is E. 45°/o chose this.
55
Pneumocystis jirovecii, like most funga l infections, does not cause illness in the
56
immunocompetent host. However, in children, t he elderly, or immunocompromised patients, such
57 as those infected with HIV or those who have been taking immunosuppressant medications (eg,
58 after transplantation), P. jirovecii infection can cause symptomatic pneumonia (PJP). The fact
that our patient has missed a few doses of his posttransplant medication (eg, TMP-SMX
59
prophylaxis) puts him at a greater risk of encountering PJP. Infected patients often present with
60 progressively worsening symptoms, including fever, cough, and tachypnea, often with associated
61 hypoxia. Chest radiography of patients with P. jirovecii pneumonia are described as showing
diffuse, bilateral, interstitial infiltrates (X-rays) and diffuse ground glass opacities and
62
pneumatoceles, as shown in the CT. Diagnosis requires visualization of the organism, such as
63 with lung biopsy or more commonly bronchioalveolar lavage (BAL). Foamy alveolar casts on BAL
64 is quite specific for P. jirovecii infection, which is treated with trimethoprim -sulfamethoxazole.
ini'N!thopr•m/•ulfa !tho -ole Tachypnea Hypoxia (medical) Pneumocystis jiroveci1 lmr'""\Uno~uppr~"CIOn Imt unodefictenCl HIV Stops~
65
Pneumonta M.,cos•s Cough Chest radtograph Pneumocysbs pneumonia Fungus Immunocompetence X·ray Lung Pulmonary alveolus Fever
66 Image copyright ©Annals of Thoracic
Preventtve healthcare lnfectton Organ transplantation Organism
67 Medicine
68
A is not correct . 1 3°/o chose this.
69
Azithromycin is commonly used to t reat bact eri al pneumonia. It has good coverage for gram-posit ive cocci such as Pneumococcus, which is
70 th e most common cause of bacterial pneumonia in immunocompetent adults. However, in light of t his pat ient's recent history of kidney
71 transplant (ie, immunocompromise) and t he x- ray findings (pneumococcal pneumonia causes consolidations or effusions, which are not seen
here), this patient's symptoms are likely not caused by a typical bacterial pneumonia.
72
Immunodeficiency AztthrOITJfCtn Pneumoma Bacterial pneumonia Streptococcus pneumoniae Gram~posttlve bacteria Coccus Kidney Immunocompetence X-ray
73
B i s not correct. 14°/o chose this .
74
Fluconazole or ketoconazole is used fo r t he t reat ment of local blastomycosis infections. If the infection prog resses t o a systemic infection, it
75
may cause flu-like symptoms, fevers, chills, productive cough, myalgia, arthralgia, and pleurit ic chest pain . On chest x-ray, blastomycosis may
76 mimic tuberculosis and present with granulomatous nodules.
77 Arthr1tlgu1 Ketoconazole Blastomycosis Fluconazole Myalgia Tuberculosis Chest radiograph Cough Pleunsy Granuloma Influenza-like 1llness Systemic disease X-ray Chest pam Infect1on Nodule (med1c1ne)

78
• 79 C is not correct. 16% chose thi s .
.
8
Lode.
s
Suspe-nd
8
End Bloc:k
Item: 78 of ~ ,• Mark <::J [:::> ""I ~· ~'j
100 J.. Previous Next LAbfaiUI~S Notes Calculator

54 B is not correct. 14% chose this .


55 Fluconazole or ketoconazole is used for t he t reatment of local blast omycosis infect ions. If t he infection progresses t o a syst em ic infect ion, it
56
may cause fl u-like sym ptoms, fevers, chills, product ive cough, myalgia, arthra lgia, and pleurit ic chest pain. On chest x-ray, blastomycosis may
m im ic tuberculosis and present wit h granulomat ous nodules.
57 Arthralgia Ketoconazole Blastomycosis Fluconazole Myalgia Tuberculosis Chest radiograph Cough Pleurisy Granuloma Influenza-like illness Systemic disease X-ray Chest pain Infection Nodule (medicine)
58 Chills
59
C is not co rrect. 16% chose this .
60
Ganciclovier is an ant iv iral medication used to t reat CMV infect ions. CMV is the most common v ira l infect ion in solid organ t ransplant
61 recipients and may present with similar sim ilarly to a PJP pneumonia. However, t he bronchioalveolar washings demonstrat ing foamy alveolar
62 casts is typical for P jirovecii.
Organ transplantation Pneumonia Antiviral drug Viral disease Cytomegalovirus Virus Pulmonary alveolus Pharmaceutical drug Human cytomegalovirus Infection
63
64 D is not correct. 1 2% c hose this.
65 Sulfad iazine and pyri met hamine are used t o t reat toxoplasmosis. Although Toxoplasma gondii infection of the lung can cause pneumonia-like
symptoms, T gondii more commonly presents wit h brain abscesses in immunocompromised pat ients, part icularly t hose infect ed with HI V.
66
Toxoplasma gondii Toxoplasmosis Pyrimethamine Immunodeficiency Sulfadiazine Lung Pneumonia Infection Abscess Brain
67
68
69
Bottom Line :
70 Consider Pneumocystis jirovecii pneumonia in the immunocom promised patient present ing wit h progressively worsening cough, fevers,
mala ise, and shortness of breat h. The t reatment of choice is t rimethoprim -sulfa met hoxazole.
71 Pneumocystis jirovecii Trimethoprim/sulfamethoxazole Immunodeficiency Pneumonia Malaise Dyspnea Cough Pneumocystis pneumonia
72
73
74 i@l;fil·1i•J for yea r:[2017 • J
FIRST AID FACTS
75
76
77
FA17 p 150.1
78 Pneumocystis jirovecii Causes Pneumocystis pneumonia (PCP), a diffuse interstitial pneumonia fi.l. Yeast-l ike
• 79 fungus (originally classifi ed as protozoan). Inhaled. Most infections are asymptomatic.
..
8
Lock
s
Suspend
0
End Block
Item: 78 of - ,• Mark --<) [::> ""'I ~· 1!';:'1
100 ~ Prev1o u s Next Labf a lu es Notes Calculator

54
55
14lif.ii!11•J for year: 201 7 •
FIRST AIO FACTS

56
57 FA17 p 150.1
58
Pneumocystis jirovecii Causes Pneumocystis pneumonia (PCP), a diffuse interstitial pneumonia · _Yeast-like
59
fungus (originally classified as proto7oan). Inhaled. \ lost infections are as) mptomatic.
60
61
Immunosuppression (eg, AJ OS) predisposes to disease. Diffuse, bilateral ground-glass opacities on
62
C ' R/CT IE). Diagnosed by lung biops) or lavage. Disc-shaped yeast seen on methenamine sih er
63
stain of lung tissue 9 .
64
Treatment/prophylaxis: T.\ IP-S \ IX, pent amid inc, dapsone (prophylaxis only), ato,·aquone. Start
65 prophylaxis when CD4+ count drops to < 200 cells/mm3 in HIV patients.
66
67
68
69
70
71
72
73
74
75
76
77
78
• 79 FA17 p 190.1

8
L.odt
s
Su~pl'nd
~
End Block
56
57
58
FA17 p 190.1

59 Sulfonamides Sul fa methoxazole (SI\ IX), sulfisoxazole, PABA+ Pteridine


60 sul fadiazine.
61 D hyurop1er0o ll--0- Sulfonamides,
MECHANISM Inhibit dihydropteroate synthase, thus inhibiting c nt~ dapsone
62
63
folate synthesis. Bacteriostatic (bactericidal ..
Oihydropteroic acid
64
when combined with trimethoprim).
65 CliNICAl USE Gram Et>, gram 8 , 1 ocardia. S~ IX for simple
66 UTI.
67
ADVERSE EFFECTS llypersensitivity reactions, hemolysis if C6PD
68
deficient, nephrotoxicity (tubulointerstitial
69 Dihydrofotic acid
nephritis), photosensitivity, Stevens-Johnson
70
71
syndrome, kern icterus in infants, displace
other drugs from albumi n (eg, warfa rin).
o...1 arofol
reduct •
·~ ]1--0- Trimethoprim,
pyrimethamine
72
73 MECHANISM OF RESISTANCE Altered enzyme (bacterial dihrdropteroate Tetrahydrofolic acid
74
75
synthase), l uptake, or t PAS synthesis.
/!"'-
Purines Thymidine Methionine
76
77
t
DNA. RNA
! ! DNA Protein
78
• 79

. 8
L.odt
s
Su~pl'nd
~
End Block
54 other drugs from albumin (eg, warfarin).
55 MECHANISMOF RESISTANCE ltered enzyme (bacterial dihydropt eroe~ te Tetrahydrofolic acid
56
57
synthase), l uptake, or t PABA synthesis.
/!"-.
Purines Thymidine Methionine
58
59 1
DNA. RNA
1
DNA Protein
60
61
62 FA17 p 190.3
63 Trimethoprim
64
65
MECHANISM Inhibits bacterial dihydrofolate reductase.
66
Bacteriostatic.
67 CLINICAL USE Used in combination \\'ith sulfonamides
68 (trimethoprim-sulfamethoxazole [TM P-
69 SMX]), causing sequential block of folate
70 synthesis. Combination used for UTis,
71 Shigella, Salmonella, Pneumocystis iirovecii
72
pneu monia treatment and prophylaxis,
73
toxoplasmosis prophylaxis.
74
75
ADVERSE EFFECTS Megaloblastic anemie~, lcukopcnie~,
76
granulocytopenia. (Mar alb·iatc with
77 supplemental folinic acid). T:\IP 'I reats
78 l\Iarrow Poork.
'
• 79

8
L.odt
s
Su~pl'nd
~
End Block
Item: 79 of - ,• Mark -<J [:::> "'I ~ · ~
100 ~ P~v1ous N @xt Labl lues N o tes Calculato r
74
75 A hepatit is panel is ord ered for a 27-year-old woman as part of a routine work-up for abdominal pain. Resu lt s of serologic test ing are:
76 I Marker Result
HBoAg Nogativo
77
HBsAg N&gative
78 Anti-HBs Positive
Total anti·HBc Positive
• 79
. 80
. 81 Which of the following is the appropriat e conclusion?
• 82 :

• 83 A . The patient has been exposed to hepati tis B virus and has completely recovered
• 84 B. The patient has been exposed to hepati tis B virus and is in the acute disease phase
. 85
C. The patient has been exposed to hepatit is B virus and is in the window phase
. 86
• 87
D . The patient has been exposed to hepatit is B virus and is now chronically infected
• 88 E. The patien t has been exposed to hepatit is B virus but was never infected
• 89
F. The patient was vaccinated against hepatit is Band has immunity
• 90
• 91
• 92
• 93
• 94

• 95
. 96
. 97

• 98
• 99

• 100 •
8
Lode.
s
S uspe-nd
8
End Bloc:k
Item: 79 of - ,• Mark --<) [::> ""'I ~· 1!';:'1
100 ~ Prev1o u s Next Labf a lu es Notes Calculator
74
75 The correct answer is A. 62°/o chose this.
76 In isolation, the presence of anti-HBs in t he patient's serum could suggest prior exposure or vaccination . However the presence of anti-HBc by
77 definit ion indicates prior exposure.
78 The absence of current viral antigens in t he serum {HBsAg negative, HBeAg negative) signify that t here is no current infection. HBeAg is
79
generally an indicator of infectivity and will be positive during an active infection as well as the early chronic stage which can still be
associated with high infectivity. The HB surface antigen (HBsAg), which is found on the surface of t he hepat itis B virus and as spheres and
. 80
filaments (without virions) in the blood will be present during an active infection, and can persist for more than 6 months causing a chronic
. 81 infection. Since HBsAg is negative in this patient she is not in a state of active or chronic infection .
• 82 Hep..,b• 8 He.- ~bgO!fl Heoo"tlto 8 wus Hepatitis \llrus HBeAg Vaccmabon Serum Iblood Blood pl&.o
. 83
B is not correct. 4°/o chose this.
• 84
The patient has completely recovered and is not in the acute disease phase.
. 85 Acute (....doc oe)
. 86
C is not correct. 9°/o chose this .
• 87
The patient has completely recovered. I n the window phase, HBsAg and Anti- HBs are bot h absent. The presence of anti-HBs means she is not
• 88 in the window phase. The window phase occurs because the antigen and antibody are both below t he limits of detectability.
• 89 HBsAg Antobody Antogen

• 90 D i s not correct. 8 °/o chose this .


. 91 The patient has completely recovered, and th e absence of HBsAg confirms that she is not chronically infect ed .
• 92 HBsAg

• 93 E is not correct. 3°/o chose this.


• 94 The patient has completely recovered, but the presence of anti-HBc indicates t hat she was once infected, th at is, exposed to the virus, not
• 95
just vaccinated with HBsAg .
HBsAg HepatitiS 6 VlrU~ Virus vacc1ne
. 96
. 97 F is not correct. 14°/o chose this .
• 98
Presence of HBsAb, an antibody to Hepat it is B surface antigen, alone, would indicate immunity via vaccinat ion. This pat ient has antibodies
against the HBsAg and against HBcAg, indicating prior infection .
• 99 HBcAg Hepatoto> 8 HB&Ag Antobod Antigen V.Ccmation I mmunity (medocal) Hepatotos Infectoon
• 100 •
8
L.odt
s
Su~pl'nd
~
End Block
Item: 79 of ~ ,• Mark <::J [:::> ""I ~· ~'j
100 J.. Previous Next LAbfaiUI~S Notes Calculator

74
75 Bottom Line:
76 The presence of both anti-HBs and ant i-HBc wit hout hepatit is B ant igens suggests prior infect ion wit h complete recovery.
Hepatitis 8 Hepatitis 8 virus Hepatitis Antigen
77
78
79
. 80
141;fil·1i•J
FIRS T AID FACTS
for year:[ 2017
.
•j .

. 81
• 82 FA17 p 170.1
• 83 Hepatitis serologic markers
• 84
Anti-HAV (lgM) lgM antibody to HAV; best test to detect acute hepatitis A.
. 85
. 86 Anti-HAV (lgG) IgG antibody indicates prior HAV infection and/or prior vaccination; protects against reinfection.
• 87 HBsAg Antigen found on surface of HBV; indicates hepatitis B infection.
• 88
Anti-HBs Antibody to HBsAg; indicates immunity to hepatitis B due to ,·accination or reco,·ery from
• 89
• 90
infection .
. 91 HBcAg Antigen associated with core of HBV.
• 92
Anti-HBc Antibody to HBcAg; lg 1 =acute/recent infection; lgG = prior exposure or chronic infection. lg 1
• 93
anli-IIBe may be the sole EF> marker of infection during window period .
• 94
• 95
HBeAg Secreted br infected hepatocyte into circulation. ot part of mature HBV ,·irion. Ind icates acti,·e
. 96
viral replication and therefore high transmissibility and poorer prognosis.
. 97 Anti-HBe Antibody to HBeAg; indicates low transmissibility.
• 98
• 99
Important Incubation Prodrome• Convalescence
diagnostic period acute disease Early Late
• 100 tests
-~··"-·-·---~----...................:=:::~
8
Lock
s
Suspend
0
End Block
Item: 79 of - ,• Mark --<) [::> ""'I ~· 1!';:'1
100 ~ Prev1o u s Next Labf a lu es Notes Calculator
74
75 Important Incubation Prodrome. Convalescence
diagnostic penod acute disease Early Late
76 tests
HBsAg Anb· Antt·HBs Surface antigen
77 HBsAg
(anb·HBcl HBc lantt·HBc) HBsAg \
DNA
78 polymerase
DNA Anti· HBc
79 Relative polymerase
. 80 concentration
of reactants
HBV particles DNA ~
. 81 Envelope
anttgen (-)
• 82 HBsAg HBeAg- - - . (+)
. 83
""'
• 84 Anti· HBs
Coreanttgen _ /
. 85 Anti·HBe HBcAg
. 86 Level of
detection
• 87
• 88
Months after 0 1 2 3 4 5 6 7 8
exposure Symptoms
• 89
SGPT!ALT)
• 90
. 91
• 92 HBsAg Anti- HBs HBeAg Anti-HBe Anti-HBc
• 93 Acute HBV ,/ ,/ lg 1
• 94
Window ,/ lgM
• 95
Chronic HBV (high infectivity) ,/ ,/ lgC
. 96
. 97 ,/ ,/
Chronic HBV (low infectivity) lgC
• 98
Recovery ,/ ,/ lgC
• 99
Immunized ,/ •
• 100

8
L.odt
s
Su~pl'nd
~
End Block
Item: 79 of ~ ,• Mark <::J [:::> ""I ~· ~'j
100 J.. Previous Next LAb faiUI~S Notes Calculator

74
FA17 p 168.1
75
76 Hepatitis Signs and symptoms of all hepatitis viruses: episodes of fever, jaundice, t ALT and AST. Naked viruses (HAV
77 viruses and HEV) lack an envelope and are not destroyed by the gut: the vowels hit your bowels.
78 HBV D A polymerase has D A- and RNA-dependent activities. Upon entry into nucleus, the polymerase
79 completes the partial clsD 1A. Host R1 A polymerase transcribes mRNA from ,·iral D A to make ,·ira!
. 80
proteins. The DNA polymerase then reverse transcribes viral R A to D1 A, which is the genome of the
. 81
progeny v1rus.
• 82
HCV lacks 3'-5' exonuclease activity - no proofreading ability - variation in antigenic structures of HCV
• 83
envelope proteins. Host antibody production lags behind production of new muta nt strains ofHC V.
• 84
. 85
Virus H~ H~ HCV H~ HEV
. 86 FAMILY R 'A picornm·irus DNA hepadnavirus R A Aavivirus R A dcltavirus R1 A hepevirus
• 87 TRANSMISSION Fecal-oral (shellfish, Parenteral (Blood), Primarily blood Parenteral, sexual, Fecal-oral,
• 88
travelers, day care) sexual (Baby- (IVDU, post- perinatal especially
• 89
making), perinatal transfusion) waterborne
• 90
(Birthing)
. 91
• 92
INCUBATION Short (weeks) Long (months) Long Superinfection Short
• 93 (l lDV after
• 94 HBV) = short
• 95 Coinfcc tion (HDV
. 96 with HBV) = long
. 97 CLINICAL COURSE Asymptomatic Initially li ke serum ~'layprogress to Similar to HBV Fulminant hepatitis
• 98
(usually), Acute sickness {fever, C irrhosis or in E xpectant
• 99
arthralgias, rash); Carcinoma (pregnant) women
• 100 ---·· - ··------ .. _
8
Lock
s
Suspend
0
End Block
Item: 80 of - ,• Mark -<J [:::> "'I ~· ~
100 ~ P~v1ous N @xt Labl lues N o t es Calcula to r
74
A 23-year-old sexually active woman presents t o her physician with a high temperature, purulent cervical discharge, and cervical motion ~~AI
75
tenderness. A Gram stain prepared from a swab of t he discharge is shown in the image.
76
77
78 •
-
. 80
79
.. ..
. 81
• 82
• 83 ..
• 84
. 85
. 86
0 87
0 88 Image courtesy
• 89
• 90 Which of the fol lowing is t rue about the infect ious organ ism?
. 91
:
• 92 A. It demonstrates inclusion bodies under microscopy
0 93
B. It has adherent pili
• 94
0
95
C. It is a dimorphic opportunist
. 96 D. It is a flagellated protozoan
. 97
E. It is pleomorphic and Gram variable
0 98
0 99
. 100 •
8
Lode.
s
Suspe-nd
8
End Bloc:k
Item: 80 of ~ ,• Mark <:::1 t::> ""I ~· ~'j
100 J.. Previous Next LAbfaiUI~S Notes Calculator

74
75
The correct a nswer is B. 53% chose t his .
76
This cl inical pict ure is consist ent wit h pelvic inflammatory disease (PID). Of the bact eria that cause PID, Neisseria gonorrhoeae is most likely
77 t o cause high feve r and purulent discha rge. The Gram st ain of t he gram-negat ive diplococci in the vagina l discharge is diagnost ic of
78 gonococca l PID. N. gonorrhoeae has type 4 pili, which are used for at tachment to host cells and are t he pri mary v irulence factor of this
bacteri a. A complication of PID includes t ubo-ovarian abscess. Add it iona lly, sca rri ng of t he fallopian tubes may cause ectopic pregnancy,
79
infertility, and chronic pain.
80 Pelvic inflammatory disease Gram staining Ectopic pregnancy Neisseria gonorrhoeae Virulence factor Tubo-ovarian abscess Gram-negative bacteria Diplococcus Abscess Vaginal discharge Pus Gonorrhea
. 81 Fallopian tube Bacteria Infertility Pregnancy Neisseria Fever Chronic pain Virulence Pilus Pelvis
• 82
A is not correct . 20% chose this.
• 83
Pelvic inf lammat ory disease caused by Chlamydia trachomatis is a more subacut e t ype of infect ion. Alt hough these intracellular organisms are
• 84 poorly v isua lized by Gram sta ining and light m icroscopy, evidence of infection by Chlamydia can manifest as inclusion bodies seen within the
. 85 cyt oplasm of infect ed cells under microscopy. Polymerase cha in reaction or immunoassay tests are more likely to provide an accurate
. 86
diagnosis of Chlamydia infect ion .
Pelvic inflammatory disease Chlamydia trachomatis Polymerase chain reaction Cytoplasm Gram staining Chlamydia infection Chlamydia (genus) Immunoassay Inclusion bodies Intracellular Microscopy
• 87
Acute (medicine) Inflammation Infection
• 88
• 89 C is not correct. 9°/o chose this .
Candida albicans is anot her cause of vulvovag init is; it is diagnosed by visualizat ion of pseudohyphae on pot assium hydroxide preparation .
• 90
Potassium hydroxide Candida albicans Candida (fungus) Hypha Vaginitis Potassium Pseudohyphae
. 91
• 92
D is not correct. 6°/o chose this .
Trichomonas vagina/is is a motile prot ozoan (seen on a wet mount) t hat causes a fou l-smelling, yellow-green vaginal discharge as a result of
• 93
vu lvovaginal infection . Trichomonas infect ion is also wh iff t est posit ive .
• 94 Trichomonas vaginalis Vaginal discharge Protozoa Trichomonas Motility Microscope slide
• 95
E is not correct. 1 2°/o chose this .
. 96
Gardnerella vagina/is causes bacteria l vag inosis that results in vaginal discharge with a f ishy odor, especially in t he presence of a base (wh iff
. 97 t est) . Gardnerella species are pleomorphic bacilli wit h va riable response to Gram st aining. The cl assic hist ologic pict ure for Gardnerella
• 98 vu lvovaginit is is clue cells, which are vag inal epithelial cells covered wit h adherent offending bacteria .
Gardnerella vaginalis Bacterial vaginosis Gram staining vaginal discharge Epithelium Vaginitis Bacteria Clue cell Bacilli Bacillus (shape)
• 99
• 100 ~

8
Lock
s
Suspend
0
End Block
Item: 80 of - ,• Mark -<] 1:> ""'I ~· 1!';:'1
100 ~ Prev1o u s Next Labf a lu es Note s Calculator
74
75
Bottom Line:
76 Pelvic inflammatory disease (PID) is commonly caused by Neisseria gonorrhoeae or Chlamydia trachomatis. Gram -negative diplococci in a
vaginal discharge point t o gonococcal PID.
77 PelviC tnflar ttOrt dt$el$e Chlamydta trachomatis Diplococcus Gram-negative bactena Nessena gonorrhoeae Chlamyclla infectton vaginal dtscharge Chlamydia (genus) Netssena Gonorrhea InflammatiOn
78
79
80 141it'il;1i•J fo r year:
FJ~ST AIO FACTS
2017 •
. 81
• 82
FA17 p 181 .1
. 83
• 84 Pelvic inflammatory Top bugs- Chlamydia trachomatis (subacute, Salpingitis is a risk factor for ectopic prcgnanC)',
. 85 disease often undiagnosed), Neisseria gonorrhoeae infertilit), chronic peh-ic pain, and ad h e~ions.
. 86 (acute). C trachomatis-most common Can lead to Fitz-Hugh-Curtis syndrome-
• 87 bacterial STI in the United Stales. Cervical infection of the li\·er capsule ancl"violin string"
• 88
• 89
motion tenderness (chandelier sign), purulent· adhesions of peritoneum to liver m.
cervical discharge fJ. PID may include
• 90
salpingitis, endometritis, hydrosalpinx, and
. 91
tuba-ovarian abscess .
• 92
• 93
• 94
• 95 B . !·• ·. ·--~
1 11y I,
. 96
. 97
'il;.·..~ . ~,·
I ,:·~t~· ' I
..
• 98 r I . l..;j
• 99
• 100 ~. •
8
L.odt
s
Su~pl'nd
~
End Block
Item: 80 of - ,• Mark -<J [:::> "'I ~ · ~
100 ~ P~v1ous N @xt Labl lues N o t es Calcula to r
74 A A

75
FA17 p 138.2
76 Neisseria Gram 8 diplococci. Metabolize glucose \leninGococci ferment .\laltose and Glucose.
77 and produce lgA proteases. Contain Gonococci fe rment Glucose.
78
Iipooligosaccharides (LOS) with strong
79
endotoxin acti\'ity. gonorrhoeae is oft en
80
intracellular (within neutrophils)
. 81
• 82 Gonococci Meningococci
• 83
1 o polysaccharide capsule Polysaccharide capsule
• 84
. 85 No maltose metabolized Maltose fermentation
. 86 1 o \'accine due to antigenic \'ariation of pilus Vaccine (type B \'accine not widely a\'ailablc)
• 87 proteins
• 88
Sexually or peri natally transmitted Transmitted via respiratory and oral secretions
• 89
• 90 Causes gonorrhea, septic arthritis, neonatal Causes meningococcemia with petechial
. 91 conjunctivitis (2-5 days after birth), pelvic hemorrhages and gangrene of toes rn.
• 92 inAammatory disease (PI D), and Fit z-llugh- meni ••gil is, Waterhouse-Friderichsen
• 93 Curtis syndrome syndrome (adrenal insufficiency, fever, DIC,
• 94 shock)
• 95
Condoms l sexual transmission, erythromycin Rifampin, ciproAoxacin, or ceftriaxone
. 96
eye ointment prevents neonatal blindness prophylaxis in close contacts
. 97
• 98 Treatment: ceftriaxone + (azithromycin Treatment: ceftriaxone or penicillin G
• 99 or doxycycline) for possible chlamydia!
. 100 coinfection
8
Lode.
s
Suspe-nd
8
End Bloc:k
Item: 80 of - ,• Mark -<] 1:> ""'I ~· 1!';:'1
100 ~ Prev1o u s Next Labf a lu es Note s Calculator
74 • •
FA17 p 154.1
75
76
Protozoa- others
77 ORGANISM OISEASE TRANSMISSION DIAGNOSIS TREATMENT
78 Visceral infections
79
Trypanosoma Chagas disease-dilated Rcduviid bug Trypomastigotc in Benm idazolc
80
cruzi cardiomyopathy with (" kissing bug") blood smear fJ or nifurtimox;
. 81
apical atrophy, megacolon, feces, deposited Cn11:ing in m)
• 82
megaesophagus; predominantly in a painless bite Benz, with a fur
. 83
• 84
in South America (much like a l..i.,.,) coat on
. 85
Unilateral periorbital swelling
. 86 (Romai\a sign) characteristic of
• 87 acute stage
• 88
Leishmania Visceral leishmaniasis SandAy Macrophages Amphotericin B,
• 89
donovani (kala-azar) - spiking fevers, containing sodium
• 90
hepatosplenomegaly, amastigotes : stibogluconate
. 91
pancytopenia
• 92
Cutaneous leishmaniasis-skin
• 93
• 94
ulcers
• 95
. 96
. 97
Sexually transmitted infections
• 98
• 99
Trichomonas Vaginitis-foul-smelling, greenish Sexual (cannot exist Trophozoitcs 'let ron ida zoic for
• 100 vagina/is discharge; itching and burning; outside human m
(motile) on wet patient and partner •
8
L.odt
s
Su~pl'nd
~
End Block
Item: 81 of - ,• Mark -<J
P~v1ous
[:::> "'I ~ ·· ~
100 ~ N @xt Labl lues N o tes Calcula to r
74
75 Infection with Malassezia furfur causes skin hypopigmentation. It is found most frequently in hot, humid, t ropical reg ions. It also is
responsible for causing one of the cut aneous mycoses.
76
77
Infection with Malassezia fudur causes which of the following disorders?
78
:
79
A. Tinea cruris
80
. 81 B. Tinea nigra
• 82 C. Tinea pedis
• 83
0 . Tinea capitis
• 84
. 85
E. Tinea versicolor
. 86
• 87
• 88
• 89
• 90
. 91
• 92
• 93
• 94
• 95
. 96
. 97
• 98
• 99
. 100 •
8
Lode.
s
Suspe-nd
8
End Bloc:k
Item: 81 of - ,• Mark -<J
P~v1ous
[:::> "'I ~ ·· ~
100 ~ N @xt Labl lues N o tes Calcula to r
74 A A

75
The correct answ er is E. 75°/o chose this.
76
Malassezia furfur infecti on is t he cause of t inea versicolor. Symptoms include hypopigmented skin lesions
77 t hat occur in hot and humid conditions. A pot assium hydroxid e (KOH) test will demonst rate the
78 characteristic spores with short mycelium referred t o as t he "spaghetti and meatballs" sign of tinea
versicolor (shown in the image). M. fwfur is trea ted wit h t opical miconazole or selenium sulfide (Selsun),
79
and empiric therapy is usually attempted before a formal work-up is performed.
80 Tmea en- '"'Oio P uJm .-<fro tde Mtconazole Malassezia Mycelium Selenium Selenrum dtcc-ulfide Sfolen tm 5 de T ea Infecbon Spore

81 Hypopogmentlltlon

• 82
• 83
• 84
. 85
I mage courtesy of CDC/Or.
. 86 Lucille K. Georg
• 87
• 88 A is not correct. 6°/o chose thi s .
• 89
Epidermophyton floccosum and Trichophyton rubrum are th e most common causes of t inea cruris, more commonly kn own as "jock itch." Its
incidence in men vastl y out numbers that in women . The patient's sweat provides a cult ure media for the fung us. KOH mounts will revea l
• 90 segmented hyphae and arth rospores.
. 91 Trichophyton rubrum Epidermophyton floccosum Tinea cruris Tinea Epidermophyton Trichophyton Hypha Itch Potasstum hydroxtde Fungus

• 92
B is not correct. 5% chose this .
• 93
The causat ive agent fo r t inea nigra, a superficial infection of the stratum corneum, is Hortaea werneckii. The fu ngus is most often acquired
• 94 through direct contact with soil, wood, and decaying vegetation.
Tinea ntgra Hortaea wernecku Fungus Ttnea Infection
• 95
. 96 C is not correct . 6°/o chose this .
. 97 Trichophyton causes approximately 90% of the dermatophyte hair infections in the Unit ed States. Typically, it is t his genus of fungi that
• 98
causes tinea pedis and t inea unguium. This fungus does not fluoresce, and thus ultraviolet examination should not be relied on for diagnosis .
DermMophyte Onychomycot~o ~ Athlete's foot Fungus Tinea Ultraviolet Trichophyton Fluorescence Genu
• 99
. 100
D is not correct. 8 °/o c hose this .

8
Lode.
s
Suspe-nd
8
End Bloc:k
Item: 81 of ~ ,• Mark <::J [:::> ""I ~· ~'j
100 J.. Previous Next faiUI~S
LAb Notes Calculator

74
75 D is not correct. 8°/o chose this.
76 Microsporum canis is among t he organisms responsible fo r causing t inea capit is and tinea corporis. Members of Trichophyton and
Microsporum bot h can cause t inea capi t is and t inea cor·porus but not t inea versicolor. These diseases are usually acquired f rom domestic cats
77
and dogs. Microscopically, M. canis produces macroconid ia, microconidia, and septat e hyphae .
78 Tinea versicolor Microsporum canis Tinea capitis Conidium Tinea corporis Microsporum Tinea Hypha Trichophyton Septate

79
80
Bottom Line:
81
Ma/assezia furfur causes tinea vers icolor; KOH prepa rat ion shows "spaghetti and meat balls" mycelia .
• 82 Tinea versicolor Malassezia Mycelium Tinea Potassium hydroxide
• 83
• 84
. 85 l@ljl'il·1i•J for year:[2017 • J
FIRS T AID FACTS
. 86
• 87
FA17 p 148.1
• 88
• 89
Cutaneous mycoses
• 90 Tinea Tinea is the cli nica l name g iven to dermatophyte (cutaneous fungal } infections. Dermatophytes
. 91 (dermatophytes) include Microsporum, Trichophyton, and Epidermophyton. Branching septate hyphae visible on
• 92 KOH preparation with blue fungal stain r.J. Associated with pruritus.
• 93
Tinea capitis Occurs on head, scalp. Associated with lymphadenopathy, alopecia, scaling 1]).
• 94
• 95 Tinea corporis Occurs on torso. Characterized by erythematous scaling rings ("ringworm") and central
. 96 clearing 9 . Can be acquired from contact with an infected cat or dog .
. 97 Tinea cruris Occurs in inguinal area 1!]. Often does not show the central clearing seen in tinea corporis.
• 98
Tinea pedis T hree varieties:
• 99
• Interdigital I]; most common
• 100

8
Lock
s
Suspend
0
End Block
Item: 81 of - ,• Mark -<] 1:> ""'I ~· 1!';:'1
100 ~ Prev1o u s Next Labf a lu es Note s Calculator
74
75 Tinea unguium Onychomycosis; occurs on nails.
76
Tinea (pityriasis) Caused by Malassezia spp. (Pityrosporum spp.), a yeast-like fungus (not a dennatophyte despite
77
versicolor being called tinea). Degradation of lipids produces acids that damage mclanoeytes and cause
78
hypopigmented [!1, hyperpigmented, and/or pink patches. Less pruritic than dermatophr tes.
79
Can occur any time of year, but more common in summer (hot, humid weather). "Spaghetti and
80
meatballs" appearance on microscopy
81
• 82
Treatment: selenium sulfide, topical and/or oral antifungal medications.
. 83
• 84
. 85
. 86
• 87
• 88
• 89

• 90
. 91
• 92
• 93
• 94
• 95
. 96
. 97
• 98
• 99
• 100 •
8
L.odt
s
Su~pl'nd
~
End Block
Item: 82 of - ,• Mark -<] 1:> ""'I ~· 1!';:'1
100 ~ Prev1o u s Next Labf a lu es Notes Calculator
74
75
A 78-year-old HIV-positive man presents t o his physician complaining of a new "bruise" on his left forearm t hat is progressively enlarging.
I nspection of the area reveals multiple purple macules and papules that are firm to palpation. The lesion was biopsied and histologic
76 examination revealed a predominance of endothelioid spindle cells often inflltrated wit h capillaries and extravasated erythrocytes.
77
78 The pathology described above is caused by a virus. Which of the diseases below is caused by a virus in t he same taxonomic family?
79
:
80 A. Cervical cancer
81
B. Hepatitis B
• 82
. 83 C. Hepatitis C
• 84 0 . Molluscum contagiosum
. 85
E. Roseola infantum
. 86
• 87
• 88
• 89

• 90
. 91
• 92
• 93
• 94
• 95
. 96
. 97
• 98
• 99
• 100 •

8
L.odt
s
Su~pl'nd
~
End Block
Item: 82 of ~ ,• Mark <::J [:::> ""I ~· ~'j
100 J.. Previous Next LAb faiUI~S Notes Calculator

74
75 The correct a nswer is E. 5 4 °/o chose this .
76
The pat ient is suffering from Kaposi sa rcoma (KS), a malignant prolife rat ion of lymphatic endothelium that often manifest s with mu lt ifoca l
purple, brown, or black cutaneous lesions cha racterized as papu lar, macular, or nodular. The lesions are generally firm t o palpat ion. The
77 hist olog ic pict ure of KS is notable for a predom inance of endot helioid spind le cells, often infilt rat ed with capillari es and ext ravasated
78 eryt hrocytes. KS is caused by the double-stranded DNA human herpesvirus (HHV) 8. KS is considered an AIDS-def ining illness.
79 Of t he answer choices, only roseola infantum is caused by another member of t he human herpesvirus fami ly, HHV-6. Ot her mem bers of t he
80 herpesvirus fami ly are: herpes simplex virus (HSV)-1 and HSV-2 (ora l and gen it al lesions), Epstein-Barr virus (mononucleosis and Burkit t
lymphoma), cytomega lovirus (mononucleosis), and vari cella-zoster virus (ch ickenpox and shingles).
81
Burkitt's lymphoma Epstein-Barr virus Chickenpox Cytomegalovirus Kaposi's sarcoma Infectious mononucleosis Varicella zoster virus Herpes simplex virus Endothelium Herpes simplex Malignancy
82
Herpesviridae Lymphoma Sarcoma Papule Capillary Roseola Red blood cell Human herpesvirus 6 Shingles Virus Histology Cancer Palpation DNA Lymph AIDS-defining clinical condition
• 83
• 84 A is not correct . 14% chose this.
. 85 Cervical cancer is caused by the human papillomavirus (HPV). HPV is a double-st randed, circular DNA virus. Gen it al warts are commonly
. 86
caused by HPV strains 6 and 11. HPV st rains 16 and 18 are associat ed with cervical, penile, and ana l cancer. Kaposi sarcoma is caused by
HHV-8, a herpesvirus. Therefore t his is not t he correct answer.
• 87 Anal cancer DNA virus Human papillomavirus Kaposi's sarcoma Kaposi's sarcoma-associated herpesvirus Cervical cancer Genital wart Sarcoma Virus Herpesviridae Cancer Cervix DNA Papillomaviridae
• 88 Sex organ
• 89
B is not correct. 7% chose t his .
• 90
Hepatit is B is caused by t he hepadnavirus, which is a partia lly double-st randed, circu lar DNA virus particle. Kaposi sarcoma is caused by HHV-
. 91 8, a herpesvirus. Therefore this is not the correct answer.
• 92 DNA virus Kaposi's sarcoma Hepadnaviridae Hepatitis B Kaposi's sarcoma-associated herpesvirus Hepatitis Herpesviridae Virus Sarcoma DNA

• 93
C is not co rrect. 7°/o chose this .
• 94 Hepatit is C is caused by an single-st randed linear RNA f lavivirus. Disease including yellow feve r, Dengue fever, St. Louis encephalit is, Zika
• 95 virus, and West Nile are also caused by f laviv iruses. Kaposi sarcoma is caused by HHV-8, a herpesvirus. Therefore t his is not the correct
answer.
. 96
Flavivirus Yellow fever Kaposi's sarcoma Hepatitis C Saint Louis encephalitis Dengue fever Kaposi's sarcoma-associated herpesvirus Encephalitis Hepatitis Zika virus Herpesviridae Virus RNA Sarcoma Fever
. 97
• 98
D is not correct. 18% c hose this .
Molluscum contag iosum is a cutaneous disease characterized by sma ll, umbilicated papu les. It is caused by a poxvirus, which is a double-
• 99
stranded, linear DNA v irus. Kaposi sarcoma is caused by HHV-8, a herpesvirus. Therefore t his is not t he correct answer.
• 100 ~ DNA virus Kaoosi's sarcoma Poxviridae Molluscum contaoiosum Kaoosi's sarcoma-associated heroesvirus Cutaneous condition Heroesviridae Paoule Sarcoma Virus DNA

8
Lock
s
Suspend
0
End Block
Item: 82 of ~ ,• Mark <::J [:::> ""I ~· ~'j
100 J.. Previous Next LAbfaiUI~S Notes Calculator

74
75 Bottom Line:
76 Kaposi sarcoma is an AIDS -def ining illness caused by HHV-8. Ot her members of t he herpesvirus fam ily are HSV-1 and HSV-2, vari cella-
77
zoster v irus, Epst ein-Barr virus, cytomegalovirus, and HHV-6. It is import ant to know t he disease associat ed with all of t he herpesviruses for
St ep 1 and clinical practice.
78 Epstein-Barr virus Cytomegalovirus Kaposi's sarcoma Varicella zoster virus Herpesviridae Herpes simplex virus Herpes simplex Human herpesvirus 6 Shingles Sarcoma Virus
79
80
81 l@ljl'il·1i•J for year:[ 2017 • J
FIRST AID FACTS
82
• 83
FA17 p 160.2
• 84
. 85 Herpesviruses Enveloped, OS, and linear viruses
. 86
VIRUS ROUTE OF TRANSMISSION CLI NICAL SIGNIFICANCE NOTES
• 87
• 88
Herpes Respiratory Gingivostomatitis, keratoconjunctivitis fJ, Most common cause of sporadic
• 89
simplex secretions, saliva herpes labialis [l), herpetic whitlow on finger, encephalitis, can present as altered
• 90
virus-1 temporal lobe encephalitis, esophagitis, mental status, seizures, and/or
. 91
erythema multifo rme. aphasia.
• 92 Herpes Sexua I contact, t-Ierpes genital i s ~, neonatal herpes. Latent in sacral ganglia. Viral
• 93 simplex perinatal meningitis more common with
• 94 virus-2 HSV-2 than with HSV-1.
• 95
Varicella- Respiratory Varicella-zoster (chickenpox 1!], shingles 0}, Latent in dorsal root or trigeminal
. 96
Zoster virus secretions encephalitis, pneumonia. ganglia; Cl V1 bra nch
. 97
(HHV-3) Most common complication of shingles is post- involvement can cause herpes
• 98
• 99
herpetic neuralgia. zoster ophthalmicus.
• 100 • Epstein-Barr Respiratory Mononucleosis- fever, hepatosplenomegaly, Infects B cells through CD21. •

8
Lock
s
Suspend
0
End Block
Item: 82 of ~ ,• Mark <::J [:::> ""I ~· ~'j
100 J.. Previous Next LAb faiUI~S Notes Calculator

74
75 FA17 p 173.1
76
Common diseases of As CD4+ cell count l, risks of reactivation of past infections (eg, TB, HSV, shingles), dissemination
77
HIV-positive adults of bacterial infections and fungal infections (eg, coccidioidomycosis), and non-llodgkin
78
lymphomas t.
79
80 PATHOGEN PRESENTATION FINDINGS
81 CD4+ cell count < 500/mm 3
82
Candida albicans Oral thrush Scrapable white plaque, pseudohyphae on
• 83
1111 Cf OSCOp y
• 84
. 85 EBV Oral hairy leukoplakia Unscrapable white plaque on lateral tongue
. 86 Bartonella henselae Bacillary angiomatosis Biopsy with neutrophilic inAammation
• 87
HHV-8 Kaposi sarcoma Biopsy with lymphocytic inAammation
• 88
HPV Squamous cell carcinoma, commonly of anus
• 89
• 90
(men who have sex with men) or cen·ix
. 91
(women)
• 92 CD4+ cell count < 200/mm3
• 93
Histoplasma Fever, weight loss, fatigue, cough, dyspnea, Oval yeast cells within macrophages
• 94
capsulatum nausea, vomiting, diarrhea
• 95
. 96
HIV Dementia
. 97 JC virus (reactivation) Progressive multifocal leukoencephalopathy onenhancing areas of demyelination on !RJ
• 98 Pneumocystisjirovecii Pneumocystis pneumonia "Ground-glass" opacities on CXR
• 99
CD4+ cell count < 100/ mm 3
• 100 ~

8
Lock
s
Suspend
0
End Block
Item: 83 of - ,• Mark -<J [:::> "'I ~ · ~
100 ~ P~v1ous N @xt Labl lues N o tes Calcula to r
74
A 45-year-old man who has not seen a primary care doctor in many years presents to a walk-in clinic complaining of 2 weeks of headache, ~~AI
75
increasing fatigue, and fever. He also complains of photophobia. On physical examinati on, t he man is t hin and appears ill . He has visible
76
track marks on his left arm. The physician ord ers a lumbar puncture. The micrograph shown in t he image was obt ained from cerebrospinal
77 fluid that demonstrated lymphocytosis, decreased and increased protein.
78
79
80
81
82
• 83
• 84
. 85
. 86
• 87
• 88
• 89
• 90
Which of the following is t he most appropriate t reat ment ?
. 91 :

• 92
A . Acyclovir
• 93 B. Amphotericin B
• 94
C. Ampicillin
• 95
D. Cefotaxime
. 96
. 97 E. Isoniazid
• 98
• 99
. 100 •
8
Lode.
s
S uspe-nd
8
End Bloc:k
Item: 83 of ~ ,• Mark <::J [:::> ""I ~· ~'j
100 J.. Previous Next LAbfaiUI~S Notes Calculator

74
75
76
The correct a nswer is B. 75% chose t his .
Amphot eri cin B is the treat ment of choice for crypt ococcal mening it is. This drug binds ergost erol, a compound unique to t he fungal cell
77
membrane, and induces pore format ion. This drug causes severe side effects, including feve r, chills, hypotension, nephrotoxicity, and
78 arrhythmias. This is a case of cryptococca l meningitis, which can be diagnosed if encapsulated yeast fo rm s are seen on I ndia ink stain of
79 cerebrospinal fluid (CSF) . Cryptococcus is t he most common opportunist ic cause of meningitis that presents in a subacut e man ner. The CSF
f indings are t ypical of f ungal or mycobact eri al meningitis, showing lymphocytosis. I nt ravenous drug users are at increased risk of acquiri ng
80
HIV and developing opp01t unistic infect ions such as cryptococcal meningit is.
81 Amphotericin 8 Ergosterol Cerebrospinal fluid Cryptococcosis Nephrotoxicity Cell membrane Hypotension Meningitis Intravenous therapy India ink HIV Cryptococcus Opportunistic infection Fungus Yeast
82 Cryptococcus neoformans India Fever Cardiac arrhythmia Mycobacterium Recreational drug use Adverse drug reaction
83
A is not correct . 6°/o chose this .
• 84
This patient 's hist ory suggests t hat he has AIDS . The t rack marks on his arm indicat e int ravenous drug use, which is a risk factor. His
. 85 presentat ion, combined with t he lab find ings, suggests cryptococcal meningitis. The image shows classic encapsulat ed yeast forms. Acyclovir
. 86 is an ant iviral agent t hat is used to t reat herpes simplex virus (HSV) infections, including HSV mening it is .
Aciclovir Cryptococcosis Herpes simplex virus Meningitis HIV/AIDS Herpes simplex Antiviral drug Intravenous therapy Virus Risk factor Yeast Drug injection Cryptococcus Cryptococcus neoformans
• 87
• 88 C is not correct. 7 °/o chose this .
• 89 Ampicillin is an am inopenicillin t hat is often used in newborn s who present wit h meningitis to cover Listeria monocytogenes. Newborns may
acquire t his bacteria in t he birth canal.
• 90
Aminopenicillin Listeria monocytogenes Ampicillin Meningitis Bacteria Listeria vagina
. 91
• 92
D is not correct. 8°/o chose t his .
Cefotaxime is a third-generat ion cepha losporin that is commonly used t o t reat meningit is caused by Neisseria meningitidis. A bact erial
• 93
meningitis such as this would cause findings of increased polymorphonuclear leukocyt es, inst ead of a lym phocytosis, in CSF. Addit ionally, the
• 94 presentat ion would be more acute, wit h neck stiff ness and a pet echial rash .
• 95 Cephalosporin Neisseria meningitidis Cefotaxime Meningitis Petechia Purpura White blood cell Neck stiffness Neisseria Granulocyte Rash Meningism Bacterial meningitis Lymphocytosis

. 96 E is not correct. 4 °/o chose this .


. 97 Isoniazid, along wit h rifampi n, pyrazinam ide, and ethambut ol, is a component of t he t reatment fo r tubercu lous meningit is. Although
• 98 t uberculous meningitis also features t he CSF f inding of lym phocytosis, the photomicrograph is not consist ent wit h Mycobacterium tuberculosis
infection, which is visualized with t he use of an acid-fast st ain.
• 99
Isoniazid Pyrazinamide Ethambutol Mycobacterium tuberculosis Tuberculous meningitis Rifampicin Tuberculosis Meningitis Acid-fast Mycobacterium Cerebrospinal fluid
• 100 ~

8
Lock
s
Suspend
0
End Block
Item: 83 of ~ ,• Mark <::J [:::> ""I ~· ~'j
100 J.. Previous Next LAbfaiUI~S Notes Calculator

74
75 Bottom Line:
76 Cryptococcus is clearly seen as fung i with surrounding capsules on I ndia ink sta ins of CSF (as in the image). Cryptococcal mening it is, an
77 AID S- def ining illness, rarely occurs in immunocompetent patients. It is best t reat ed wit h amphotericin B.
Cryptococcosis India ink Fungus Meningitis HIV/AIDS Cryptococcus India Cryptococcus neoformans Immunocompetence Amphotericin B
78
79
80
81
i@l;fil·1i•J f or yea r:[ 2017 • J
FIRST AID FACTS

82
83 FA17 p 149.1
• 84 Opportunistic fungal infections
. 85
Candida albicans alba = white. Dimorph ic; Forms pseudohyphae and buddi ng yeasts at zooc fJ, germ tubes at
. 86
• 87
37°C rn.
• 88
Systemic or superficial fungal in Feclion. Causes ora l ~ and esophageal lh rush in
• 89
immunocompromised (neonates, steroids, diabetes, AIDS), vulvovaginitis (diabetes, use of
• 90
antibiotics), diaper rash, endocarditis (IV drug users), disseminated candidiasis (especially in
. 91 neutropenic patients), chronic mucocutaneous candidiasis .
• 92 Treatment: oral Auconazolc/topical azoic for vaginal; nystati n, flu conazole, or caspofungin for oral!
• 93 esophageal; fluconazole, caspofungin, or amphotericin B for systemic.
• 94 Aspergillus Septate hyphae that branch at 45° Acute Angle [!]. Produces conidia in radiating chains at end of
• 95
fumigatus conid iophore D .
. 96
Causes invasive aspergillosis in immunocompromised, patients with chronic granulomatous disease.
. 97
Can cause aspergillomas in pre-existing lung cavities, especially after T B infection .
• 98
Some species of Aspergillus produce AAaloxins (associated with hepatocellular carcinoma).
• 99
. 100 Allergic bronchopulmonary aspergillosis (ABPA): hypersensitivity response associated with
8
Lock
s
Suspend
0
End Block
Item: 83 of ~ ,• Mark <::J [:::> ""I ~· ~'j
100 J.. Previous Next LAbfaiUI~S Notes Calculator

* * * *
75
76
FA17 p 176.1
77
Common causes of meningitis
78
NEWBORN (0- 6MOl CHILDREN (6 M0- 6 YR) 6- 60YR 60YR+
79
80
Group B streptococci S pneumoniae S pneumoniae S pneumoniae
81
E coli N meningitidis N meningiticlis (f/ 1 in teens) Gram 8 rods
82 Listeria H influenzae type B Enterovi ruses Listeria
83 Enteroviruses HSV
• 84 Give ceftri axone and vancomycin empirically (add ampicill in if Listeria is suspected).
. 85 Viral causes of meningitis: enteroviruses (especially coxsackievirus), HSV-2 (HSV-1 = encephalitis), HlV, West ile virus (also
. 86
causes encephalitis), VZV.
• 87
In HIV: Cryptococcus spp.
• 88
ote: Incidence of H influenzae meningitis has l greatly due to conjug<ltc H influenzae vaccinations. Today, cases are usually
• 89
seen in unimmunized children .
• 90
. 91
• 92 FA17 p 176.2
• 93 CSF findings in meningitis
• 94
OPENING PRESSURE CEl l TYPE PROTEIN GLUCOSE
• 95
Bacterial t t P II s t l
. 96
. 97 Fungai/TB t t lymphocytes t l
• 98 Viral orma 1/t t lymphocytes Norma lit Normal
• 99
• 100 ~

8
Lock
s
Suspend
0
End Block
Item: 84 of - ,• Mark -<J [:::> "'I ~ · ~
100 ~ P~v1ous N @xt Labl lues N o tes Calculato r
74
A 17-year-old girl is accompanied t o the doctor's office by her sister because of a 6-month history of severe myoclonus characterized by ~~AI
75
frequent jerking movement s in t he arms and legs. For a year prior to the onset of the myoclonus, th e sister report s that t he patient's
76
memory has been worsening, and she has been acti ng "stra ngely." The patient and her sister moved from rura l Guatemala 2 years ago
77 but are not up to date on their routine childhood vaccinations.
78
79 Infection with which of the following agents is most likely responsible for the patient's presentation?
80 :
81 A. JC virus
82 B. Poliovirus
83
C. Rabies virus
• 84
. 85 D. Rubella virus
. 86 E. Rubeola virus
• 87
• 88
• 89
• 90
. 91
• 92
• 93
• 94
• 95
. 96
. 97
• 98
• 99
. 100 •
8
Lode.
s
S uspe-nd
8
End Bloc:k
Item: 84 of ~ ,• Mark <:::1 t::> ""I ~· ~'j
100 J.. Previous Next faiUI~S
LAb Notes Calculator

74
75
The correct a nswer is E. 36°/o chose this .
76 The pat ient's presentat ion fits t hat of the second stage of subacute scl erosing panencephalit is (SSPE), a sequela of infect ion wit h rubeola, or
77 measles virus. SSPE typically present s 7- 10 years after init ial infection wit h the virus; SSPE result s from persistent infection and
78
demyelinat ion of central nervous syst em neu rons. The init ial stage of t he cond it ion presents wit h dementia and personality changes, and lasts
up t o 1 yea r. The second st age of t he condit ion is charact eri zed by severe myoclonus and ty pica lly lasts for 3- 12 months. The t hird and fo urth
79 stages are cha racterized by worsening dement ia and det eri oration of t he aut onomic nervous system, as well as decorticat e rigidity or
80 f laccidity. The cond it ion is typically fata l. The fact t hat the pat ient is from ru ral Guatemala suggests that she may not have received all of her
81
ch ildhood immunizations, including her measles vaccines.
Sequela Subacute sclerosing panencephalitis Autonomic nervous system Measles Central nervous system Myoclonus Demyelinating disease Measles virus Neuron Dementia Virus Nervous system
82
Flaccid paralysis Infection Acute (medicine) Vaccine
83
84 A is not correct . 17% chose this .
. 85
The JC virus causes prog ressive m ult ifoca l leukoencephalopat hy, and t ypically affects only immunosuppressed people through react ivation of
lat ent infect ion wit h the virus. Clinica l presentat ion includes altered ment al st at us, mot or deficits such as hemiparesis or limb at axia, and
. 86 visual symptoms such as hemianopia.
• 87 Progressive multifocal leukoencephalopathy JC virus Hemiparesis Ataxia Immunosuppression Leukoencephalopathy Virus Altered level of consciousness

• 88
B is not correct. 17% chose this .
• 89 Poliovirus infection is charact erized by flaccid paralysis and muscular at rophy due t o t he death of secondary mot or neurons. Deat h can result
• 90 f rom pa ralysis of t he respirat ory m uscles. A postpolio syndrome wit h cont inued weakening of affected m uscl e groups can persist fo r decades
. 91
after t he init ia l infect ion .
Flaccid paralysis Poliovirus Muscle atrophy Paralysis Motor neuron Neuron Atrophy Infection Thoracic diaphragm Muscle
• 92
• 93
C is not correct. 13% chose this .
• 94
Rabies virus infects t he nervous syst em, spreading from the site of init ial bite by an infect ed an imal to the cent ral nervous system t hrough
peri pheral nerves. Mont hs after init ia l infection, patients will develop convulsions, parast hesias, and difficu lty swallowing solids and liqu ids due
• 95 t o spasticity of t he muscles of t he pharynx. Cogn it ive effects can include manic episodes and alt ered ment al statu s, and t he disease
. 96 eventu ally progresses to flaccid paralysis and death .
Central nervous system Spasticity Rabies Pharynx Flaccid paralysis Peripheral nervous system Virus Nervous system Paralysis Dysphagia Paresthesia Altered level of consciousness Convulsion Mania
. 97
Cognition Rabies virus Infection
• 98
• 99 D is not correct. 17% c hose this .
• 100 ~
Rubella, or German measles, does not typically lead t o long-term neurologic sequelae. If the virus is acquired antenata lly, pat ients can

8
Lock
s
Suspend
0
End Block
Item: 84 of - ,• Mark -<] 1:> ""'I ~· 1!';:'1
100 ~ Prev1o u s Next Labf a lu es Note s Calculator
74
75 Dis not correct. 17°/o chose thi s.
76
Rubella, or German measles, does not typically lead t o long-term neurologic sequelae. If the virus is acquired antenata lly, patients can
develop congenital rubella sydrome (CRS). CRS is charact erized by a high degree of feta l mortality and cardiac and ophthalmologic defects.
77 Rare case reports of a subacute sclerosing panencephalitis-like syndrome related to CRS have been described.
78 Me..les Rubella Congen1tal d1sorder Neurology Ophthalmology Virus Congemtal rubella syndrome Sequela

79
80
Botto m line:
81
Subacute sclerosing panencephalitis is a sequela of rubeola, or measles virus.
82 ScQuda Sub 1tt ~cle-., •no pl'nencephal•tis Measles Measles virus Virus Acute mechc.ne1

83
84
. 85 141 :f.;l ·11•i for year : 2017 •
FIRST AID FA CTS
. 86
• 87
FA17 p 166.2
• 88
• 89 Measles (rubeola) A paramyxovirus that causes measles. Usual 3 C's of measles:
• 90 virus presentation im·olves prodroma l fe,er with Cough
. 91 cough, coryza, and conju nctivitis, then Coryza
• 92 eventually Koplik spots (bright red spots with Conju net ivit is
• 93 blue-white center on buccal mucosa C ), Vitamin A supplementation can reduce
• 94 morbidity and mortality from measles,
followed 1-2 days later by a maculopapular
• 95
rash rn that starts at the head/neck and spreads particularly in malnourished children .
. 96
downward. Lymphadenitis with Warthin-
. 97
Finkeldey giant cells (fused lymphocytes) in
• 98
a background of paracortical hyperplasia .
• 99
SSPE (subacute sclerosing panencephalitis,
• 100 I • " t l• o • • ._ ,..,..,...

8
L.odt
s
Su~pl'nd
~
End Block
Item: 84 of ~ ,• Mark <:::1 t::> ""I ~· ~'j
100 J.. Previous Next LAbfaiUI~S Notes Calculator

74 • •
FA17 p 179.1
75
76
Red rashes of childhood
77 AGENT ASSOCIATED SYNDROME/DISEASE CLINICAL PRESENTATION
78 Coxsackievirus type A Hand-foot-mouth disease Oval-shaped vesicles on palms and soles rJ;
79 vesicles and ulcers in oral mucosa
80
Human herpesvirus 6 Roseola (exanthem subitum) Asymptomatic rose-colored macules appear
81
on body after se,·eral days of high fever; can
82
present with febrile seizures; usually affects
83
infants
84
. 85 Measles virus Measles (rubeola) ConAuent rash begi nning at head and
. 86 moving down; preceded by cough, coryza,
• 87 conjuncti,·itis, and blue-white (Koplik) spots
• 88 on buccal mucosa
• 89
Parvovirus B19 Erythema infectiosum (fifth disease) "Slapped cheek" rash on face llJ (can cause
• 90
hydrops feta lis in pregnant women)
. 91
• 92 Rubella virus Rubella (German measles) Pink macules and papules begin at head
• 93 and move down, remain discrete ..... fine
• 94 desquamating truncal rash; postauricular
• 95 lymphadenopathy
. 96
Streptococcus pyogenes Scarlet fever Erythematous, sandpaper-like rash with fever
. 97
and sore throat
• 98
• 99
Varicella-Zoster virus Chickenpox Vesicular rash begins on trunk; spreads to face
• 100 •
m and extremities with lesions of different •

8
Lock
s
Suspend
0
End Block
Item: 84 of - ,• Mark -<] 1:> ""'I ~· 1!';:'1
100 ~ Prev1o u s Next Labf a lu es Note s Calculator
74
75
Measles virus Measles (rubeola) ConAuent rash begi nning at head and
76
moving down; preceded by cough, coryza,
77 conjunctivitis, and blue-white (Koplik) spots
78 on buccal mucosa
79 Parvovirus 819 Erythema infectiosum (fifth disease) "Slapped cheek" rash on face I] (can cause
80 hydrops fetal is in pregnant women)
81
Rubella virus Rubella (German measles) Pink macules and papules begin at head
82
83
and mo,·e down, remain discrete - fi ne
84
desquamating truncal rash; postauricular
. 85 lymphadenopathy
. 86 Streptococcus pyogenes Scarlet fe,·er Erythematous, sandpaper-like rash with fever
• 87 and sore throat
• 88
Varicella-Zoster virus Chickenpox Vesicular rash begins on trunk; spreads to face
• 89
[!] and extremities with lesions of different
• 90
. 91
stages
• 92
• 93
• 94
• 95
. 96
. 97
• 98
• 99
• 100 •
8
L.odt
s
Su~pl'nd
~
End Block
Item: 85 of - ,• Mark -<J [:::> "'I ~ · ~
100 ~ P~v1ous N @xt Labl lues N o tes Calcula to r
74
A 23-year-old man presents for evaluat ion of a lesion on his penis. He does not know how long it has been there, bu t a new sexual partner ~~AI
75
noticed it a few days ago. Examination reveals a single ulcer with a clean, hard base on the ventral surface of his penis; it is not tender to
76
palpation. Moderate, painless bilateral inguinal lymphadenopathy is also appreciated. He denies a recent history of illness. Given this
77 patient's presentation and posit ive laboratory results, prompt treatment is initiated.
78
79 Which of the following is the mechanism of action of the drug that was most likely prescribed?
80 :
81 A. Incorporation into repl icating DNA and early-chain termination
82 B. Inhibit ion of DNA gyrases
83
C. Inhibition of tetrahydrofolate synthesis
84
. 85 D. Inhibition of the 305 ribosomal subunit
. 86 E. Interruption of cell-wall synthesis
0 87
0 88
• 89
• 90
. 91
• 92
0 93
• 94
0
95
. 96
. 97
0 98
0 99
0
100 •
8
Lode.
s
Suspe-nd
8
End Bloc:k
Item: as of ~ ,• Mark <::J [:::> ""I ~· ~'I
100 J.. Previous Next LAbfaiUI~S Notes Calculator

74
75
The correct a nswer is E. 66°/o chose this .
76
This single painless chancre (sim ilar to t hose shown in this image) is charact eri st ic of a pri mary syphilis
77
infection . Syph ilis is a sexua lly t ransmitt ed infect ion caused by t he spirochet e Treponema pa/lidum. The Image cowtesy of CDC/Susan
Lindsley
78 t reatment choice fo r this infect ion is benzathine penicillin G given by a single dose intramuscula r inj ection .
79 Penicillin antibiot ics contain a ~- l actam ri ng that binds t o and inhibits t he bact erial enzyme t ranspept idase,
t hereby arresting cell-wall synthesis.
80 Sexually transmitted infection Spirochaete Treponema pallidum Penicillin syphilis Benzylpenicillin Enzyme Benzathine benzylpenicillin Chancre Intramuscular injection Antibiotics Infection 00-transpeptidase
81
A is not correct . 7°/o chose t his.
82
Ant iv iral medications such as acyclovir are nucleot ide analogs that are t aken up by vira l DNA polymerase and incorporated int o t he growing
83 chain, leading t o earl y t erminat ion. This class of medication is used t o t reat herpes sim plex v irus-2, varicella zoster v irus, and Epst ein-Ba rr
84 virus infections.
varicella zoster virus Epstein-Barr virus Aciclovir DNA polymerase Herpes simplex virus Reverse-transcriptase inhibitor Herpes simplex Antiviral drug Nucleotide Chickenpox Virus DNA Polymerase
85
Pharmaceutical drug varicella vaccine
. 86
• 87 B is not correct. 7% chose t his .
• 88 Fluoroquinolones belong to a class of ant ibiotics t hat work by inhibiting DNA gyrase, result ing in the breakage of bact erial DNA st ruct ure. They
• 89
are useful fo r t reat ing gram-negat ive organisms such as Pseudomonas and Neisseria species .
DNA gyrase Gram-negative bacteria Quinolone Pseudomonas Antibiotics Neisseria DNA Species
• 90
. 91 C is not co rrect. 6°/o chose this .
• 92
The sulfonam ide antibiot ics such as sulfamet hoxazole work by m im icking para-aminobenzoic acid, which bacteria require to synt hesize
t et rahydrofo late and ult imat ely DNA nucleot ides. Sulfonam ides are often used in conjunct ion wit h t ri methopri m t o provide wide gram-negative
• 93 and gram-posit ive coverage.
• 94 Sulfonamide (medicine) Trimethoprim Tetrahydrofolic acid Gram-negative bacteria Sulfonamide Gram-positive bacteria Sulfamethoxazole Antibiotics Bacteria Nucleotide DNA 4-Aminobenzoic acid

• 95 D is not correct. 14% c hose this .


. 96 Am inoglycosides and t et racyclines work by inhibi t ing t he 30S ribosoma l subunit to disrupt bact erial prot ein synthesis. Such antibiotics have
. 97 good activity aga inst most gram-negat ive bact eria, but are ineffective in t reating infections caused by spirochetes such as Treponema
• 98
pal/idum. Tetracyclines can, however, be used to t reat lymphogranu loma venereum (LGV), wh ich is caused by Chlamydia trachomatis serovars
Ll, L2, or L3. LGV occurs infrequently in the Unit ed St at es (few hund red cases/year) . LGV man ifests diffe rent ly f rom syphilis, as a pa inless
• 99 ulcer wit h a "beefy red " appearance, with no associated lymphadenopathy.
• 100 ~
Lymphogranuloma venereum Chlamydia trachomatis Treponema pallidum Syphilis Gram-negative bacteria Lymphadenopathy Tetracycline antibiotics Aminoglycoside Chlamydia infection Antibiotics Protein •

8
Lock
s
Suspend
0
End Block
Item: as of ~ ,• Mark <::J [:::> ""I ~· ~'I
100 J.. Previous Next LAbfaiUI~S Notes Calculator

74
75 Bottom Line:
76
Pri mary syphilis is charact eri zed by painless chancre (s) at t he point of direct cont act. Penicillin, which acts by arrest ing cell-wall synt hesis, is
77 t he t reatment of choice fo r syphilis.
Primary syphilis Penicillin Syphilis
78
79
80
81
l@ljl'il·1i•J for year:[ 2017 • J
FIRST AID FA CTS

82
83 FA17 p 183.2
84 Penicillin G, V Penici ll in C (IV and IM form), penicillin \1 (oral). Prototype ~-lactam antibiotics.
85
MECHANISM 0 -Aia-0 -Aia structural analog. Bind penicill in-binding proteins (transpeptidases).
. 86
• 87
Block transpeptidase cross-li nking of peptidoglycan in cell wall.
• 88
Activate autolytic enzymes .
• 89 CLINICAL USE s
Mostly IISed for gram ® organisms (S pneumoniae, pyogenes, Actinomyces). Also used for gram e
• 90 cocci (mainly N meningitidis) and spirochetes (namely T pallidum). Bactericidal for gram ®cocci,
. 91 gram® rods, gram 8 cocci, and spirochetes. Pen icillinase sensitive .
• 92
• 93
AOVERSE EFFECTS Hypersensitivity reactions, direct Coombs ® hemolytic anemia.
• 94 RESISTANCE Penici ll inase in bacl·eria (a type of~-lacta mase) cleaves ~-l actam ring.
• 95
. 96
FA17 p 143.1
. 97
Syphilis Caused by spirochete Treponema pallidum.
• 98
• 99 Primary syphilis Loca lized disease presenting with painless chancre rJ. If available, use dark-fi eld microscopy to
• 100 visualize treponemes in fluid from chancre [l]. VORL® in ~ 80%.
8
Lock
s
Suspend
0
End Block
Item: BS of - ,• Mark -<) 1:> ""'I ~· 1!';:'1
100 ~ Prev1o u s Next Labf a lu es Note s Calculator
74
75 FA17 p 143.1
76 Syphilis Caused by spirochete Treponema pallidum.
77 Primary syphilis Localized disease presenting" ith painless chancre tl lf available, use dark-field microscopy to
78
visualize treponemes in Auid from chancre .:l]. DRL EEl in ~ 80%.
79
Secondary syphilis Disseminated disease with constitutiona l S) mptoms, maculopapular rash B (including palms [!]
80
81
and soles), condylomata lata (smooth, moist, painless, wart-like white lesions on genitals),
82
lp11phadenopathy, patchy hair loss; also confirmable" ith dark-field microscopy.
83 Serologic testing: VDRURPR (nonspecific), confirm diagnosis with specific test (eg, FTA- BS).
84 Secondary syphilis= S~stemic. Latent syphilis (EEl serolog) without S) mptoms) ma~ folio\\.
85 Tertiary syphilis Gummas (chronic granulomas), aortitis (,·asa ,·asorum destruction), neurosyphilis (tabes dorsalis,
. 86 "general paresis"), Argyll Robertson pupil (constricts with accommodation but is not reacti,·e to
• 87
light; also called "prostitute's pupil" since it accommodates but does not react).
• 88
Signs: broad-based ataxia, EEl Romberg, Charcot joint, stroke without hypertension .
• 89
For neurosyphilis: test spinal Auid with VORL, FTA-ABS, and PCR.
• 90
. 91 Congenital syphilis Presents with facial abnormalities such as rhagades (linear scars at angle of mouth, black arrow
• 92 in nJ), snufnes (nasal discharge, red arrow in n!), saddle nose, notched (Hutchinson) teeth CJ,
• 93 mulberry molars, and short maxi lla; saber shins; C VI II deafness.
• 94 To prevent, treat mother early in pregna ncy, as placental transmission typically occurs after first
• 95 trimester.
. 96
. 97
• 98
• 99
• 100 •
8
L.odt
s
Su~pl'nd
~
End Block
gns: nt, s
75 For neurosyphilis: test spinal Auid with VORL, FT -ABS, and PCR.
76
Congenital syphilis Presents with fa cial abnormalities such as rhagades (linear scars at angle of mouth, black arro\\
77
in ~), snufAes (nasal discharge, red arro'' in ~). saddle nose, notched (Hutchinson) teeth Cl.
78
79
mulberry molars, and short maxilla; saber shins; Ci Ill deafn ess.
80
To prevent, treat mother early in pregnancy, as placental transmission typically occurs after first
81 trimester.
82
83
84
85
. 86
• 87
• 88
• 89

• 90
. 91
• 92
• 93
• 94
• 95
. 96
. 97
• 98
• 99
• 100 •
8
L.odt
s
Su~pl'nd
~
End Block
Item: 86 of - ,• Mark --<) [::> ""'I ~· 1!';:'1
100 ~ Prev1o u s Next Labf a lu es Note s Calculator
74
75
A 16-year-old boy comes to the physician with a fever of 39. 5°C ( 103.1 ° F). He also reports feeling exhausted and complains of a severe r~AJ
headache. Review of systems is posit ive for abdominal pain and nausea, negative for sore throat. The patient says he returned yesterday
76 from a hiking trip to Tennessee with his fam ily. Physical exam reveals red conjunctivae. The physician advises conservat ive management
77 with flu ids and bed rest. Two days later, t he patient is brought back in by his parents. He now appears to be worsening rapidly and the rash
78 shown below also covers his palms and soles.
79
80
81
82
83
84
85
. 86
• 87
• 88
• 89

• 90
. 91
Which of the fol lowing statements regarding his illness is correct ?
• 92
:
• 93
A . Cold antibody agglut ination is a classic finding
• 94
B. He should be treated prompt ly wit h doxycycline
• 95
. 96 C. The causative agent is a facult at ive intracellular bacterium
. 97 D . The causative agent is a virus
• 98
E. The causative agent is spread by t he Ixodes tick
• 99
• 100 •
8
L.odt
s
Su~pl'nd
~
End Block
Item: 86 of ~ ,• Mark <::J [:::> ""I ~· ~'j
100 J.. Previous Next LAbfaiUI~S Notes Calculator

74
The correct answer is B. 57% chos e this .
75
The pat ient presents wit h signs and symptoms of Rocky Mountain spotted fever due to infect ion with Rickettsia rickettsii. He should be
76 promptly t reated (even before conf irmatory laboratory results are ava ilable) wit h a t et racycline (eg, doxycycl ine) or chloramphen icol, as any
77 delay in ant ibiotic therapy can lead t o death. Recent t ravel to an endemic area ( eg, Tennessee, North Carolina, Oklahoma, Arkansas, Missouri )
78
is an important risk facto r because t he disease is t ransmitt ed to humans by ticks . Common clinical find ings include early nonspecif ic
symptoms (fever, malaise, headache, nausea) followed by a rash bet ween t he t hird and fift h days of illness. The disease can quickly progress
79 t o seri ous neurolog ic, pulmonary, and gast roint est inal involvement.
80
Other diseases t hat can manifest wit h a rash of t he palms and soles include Coxsackie A and secondary syphilis infection . Coxsackie A is the
81 causative agent of "hand, foot, and mout h disease" and usually manifests with a sore t hroat , oral ulcers, rash, and a fever. It most commonly
82 affects young children and is spread easily in daycare centers. Syphilis is a sexually t ransmit ted infection caused by Treponema
pal/idum. Pri mary syphilis t ypically presents wit h a painless chancre, wh ich precedes t he secondary phase by weeks t o mont hs. The rash of
83
secondary syphilis can affect the t runk and/or extremit ies. Syphilis is t reated wit h pen icillin.
84 Rocky Mountain spotted fever Sexually transmitted infection Rickettsia rickettsii Doxycycline Tetracycline Chloramphenicol Treponema pallidum Syphilis Penicillin Antibiotics Rickettsia Primary syphilis

85 Chancre Nausea Headache Malaise Rash Spotted fever Sore throat Secondary syphilis Ulcer (dermatology) Fever Infection Mouth ulcer Nonspecific symptoms Risk factor Gastrointestinal tract

86
A is not correct . 4°/o chose this •
• 87
Cold ant ibody agglut ination is used t o detect Mycoplasma infect ion, not rickettsial infection . The Weii-Felix react ion (a serologic f luorescent
• 88 ant ibody screen), however, can detect a ri ckettsial infect ion due to cross- reacting ant igens f rom Proteus vulgaris .
Proteus vulgaris Antibody Mycoplasma Rickettsia Proteus (bacterium) Antigen Infection
• 89
• 90 Cis not correct. 14% chos e thi s .
. 91 Rickettsia rickettsii is an obligate, not facult at ive, int racellular bacterium that replicates freely in the cytoplasm . It is dependent on host
• 92 nicot inamide adenine dinucleotide and acety i-CoA.
Acetyi-CoA Cytoplasm Rickettsia Adenine Bacteria Nicotinamide adenine dinucleotide Intracellular Nicotinamide
• 93
• 94 D is not correct. 9°/o chose this •
• 95
Although many v iral infect ions can cause rash in a young pat ient (eg, measles), a rash t hat begins on the palms and soles suggests Rocky
Mountain spotted fever.
. 96 Rocky Mountain spotted fever Measles Virus Spotted fever Fever Rash Sole (foot)
. 97
E is not correct. 16°/o chose this •
• 98
Rickettsia rickettsii, t he causative bact eri um of Rocky Mountain spotted feve r, is spread by t icks. The main vectors are Ameri can dog t ick
• 99 (Dermacentor variabilis), Rocky Mountain wood t ick (Dermacentor andersoni), and t he brown dog t ick (Rhipicephalus sanguineus) in t he
• 100 ~
United States. Babesiosis and Lyme disease are t ransmit ted by the Ixodes t ick .

8
Lock
s
Suspend
0
End Block
Item: 86 of ~ ,• Mark <::J [:::> ""I ~· ~'j
100 J.. Previous Next LAbfaiUI~S Notes Calculator

74
75 Bottom Line:
76 Rocky Mountain spotted fever is caused by Rickettsia rickettsii and is spread by t icks. The charact eristic rash first appears on t he palms of
77 t he hands and soles of t he feet, then spreads toward the t runk and fa ce.
Rocky Mountain spotted fever Rickettsia rickettsii Rickettsia Spotted fever Fever
78
79
80
81
l@l;fil·1i•l for year:[ 2017
FIRST AID FACTS .
•j .

82
83 FA17 p 145.1
84
Rickettsial diseases Treatment: doxycycline (caution during pregnancy; alternative is chloramphenicol).
85
and vector-borne
86
illnesses
• 87
RASH COMMON
• 88
• 89 Rocky Mountain Rickettsia rickettsii, vector is tick. Despite its Classic triad- headache, fever, rash (vasculitis).
• 90 spotted fever name, disease occu rs primarily in the South Palms and soles rash is seen in Coxsackievirus
. 91 Atlantic states, especially J orth Carolina. A infection (hand, foot, and mouth disease),
• 92 Rash typically starts at wrists · and ankles and Rocky Mountain spotted fever, and 2° Syphilis
• 93 then spreads to trunk, palms, and soles. (you drive CARS using your palms and soles).
• 94
Typhus Endemic (Aeas)- R typhi. Rickettsii on the wRists, Typhus on the Trunk.
• 95
Epidemic (human body lousc)- R prowazekii .
. 96
. 97
Rash starts centrally and spreads out, sparing
• 98
palms and soles.
• 99 RASHRARE
. 100 Ehrlichiosis Ehrlichia. vector is tick. \1onocvtes with MEGA berrv-
8
Lock
s
Suspend
0
End Block
Item: 86 of ~ ,• Mark <::J [:::> ""I ~· ~'j
100 J.. Previous Next faiUI~S
LAb Notes Calculator

74
Typhus Endemic (Aeas)-R typhi. Rickettsii on the wRists, Typhus on the Trunk.
75
Epidemic (human body lousc)-R prowazekii.
76
77
Rash starts centrally and spreads out, sparing
78
palms and soles.
79
RASHRARE

80 Ehrlichiosis Ehrlichia, vector is tick. \itonocytes with MEGA berry-


81 rn
morulae (mulberry-like inclusions) in Monocytes = Ehrlichiosis
82 cytoplasm. Granulocytes = Anaplasmosis
83
Anaplasmosis Anaplasma, vector is tick. GranulOC)ies with
84
morulae 9 in cytoplasm.
85
86 Qfever Coxiella bumetii, no arthropod vector. Spores Q fever is Queer because it has no rash or vector
• 87 in haled as aerosols from cattle/sheep amniotic and its causative organism can survive outside
• 88 Auid. Presents as pneumonia. Common cause in its endospore form. ot in the Rickettsia
• 89 e
of culture endocarditis. genus, but closely related .
• 90 ___;::::,;-;:;;;:-;;;:::;::;-:;:::::;
. 91
• 92
• 93
• 94
• 95
. 96
. 97
• 98
• 99
• 100

8
Lock
s
Suspend
0
End Block
Item: 86 of - ,• Mark --<) [::> ""'I ~· 1!';:'1
100 ~ Prev1o u s Next Labf a lu es Note s Calculator
74
FA17 p 144.1
75
76 Zoonotic bacteria Zoonosis: infectious disease transmitted between animals and humans.
77
SPECIES DISEASE TRANSMISSION AND SOURCE
78
79
Anaplasma spp. Anaplasmosis Ixodes ticks (live on deer and mice)
80 Bartonella spp. Cat scratch disease, bacillary angiomatosis Cat scratch
81
Borrelia burgdorferi Lyme disease Ixodes ticks (live on deer and mice)
82
83
Borrelia recurrentis Relapsing fever Louse (recurrent due to \<Iriable surface
84 antigens)
85 Brucella spp. Brucellosis/undulant fe,·er Unpasteurized dairy
86
Campylobacter Bloody diarrhea Feces from infected pets/animals; contaminated
• 87
meats/foods/hands
• 88
• 89 Chlamydophila psittaci Psittacosis Parrots, other birds
• 90 Coxiella burnetii Q fever Aerosols of cattle/sheep amniotic Auid
. 91
Ehrlichia chaffeensis Ehrlich iosis Amblyomma (Lone Star tick)
• 92
• 93 Francisella tularensis Tularemia Ticks, rabbits, deer Aies
• 94 Leptospira spp. Animal urine in water; recreational water use
Leptospirosis
• 95
. 96
Mycobacterium leprae Leprosy ll umans '' ith lepromatous leprosy; armadillo
. 97
(rare)
• 98 Pasteurella multocida Cellulitis, osteomvclitis

nimal bite, cats, dogs
• 99
Rickettsia prowazekii Epidemic typhus ! Iuman to human via human body louse
• 100 •
8
L.odt
s
Su~pl'nd
~
End Block
Item: 87 of - ,• Mark -<J [:::> "'I ~ · ~
100 ~ P~v1ous N @xt Labl lues N o tes Calcula to r
74
A 22-year-old man comes t o t he physician complaining of difficu lt y walking up stairs because he "can't catch his breath." Physical ~~AI
75
examination reveals moderate pitting edema in his lower extremities and a new 53 gallop. The pat ient's history includes a recent upper
76
respiratory infection treated with supportive care only. The pat ient denies any history of diabet es, hypertension, or dyslipidemia, and all of
77 his family members are healthy.
78
79 Which of the following is the most likely cause of this patient's symptoms?
80 :
81 A. Brugia malayi
82 B. Coxsackie B virus
83
C. Haemophilus influenzae type B
84
85 D. Norovirus
86 E. Streptococcus pneumoniae
' 87
F. Wuchereria bancrofti
' 88
• 89
• 90
. 91
• 92

' 93
• 94

' 95
' 96
. 97

' 98
' 99
' 100 •
8
Lode.
s
Suspe-nd
8
End Bloc:k
Item: 87 of - ,• Mark --<) [::> ""'I ~· 1!';:'1
100 ~ Prev1o u s Next Labf a lu es Notes Calculator
74
75 Th e correct a nsw er is B. 69°/o chose this.
76 Based on the physical exam findings, t he patient is likely experiencing congestive heart failure. Given his
77 young age and lack of risk factors, myocardit is should be high on the differential. I n North America and
Western Europe, the most freque ntly identified viru ses were enteroviruses ( including coxsackievirus) until
78
t he 1990s. However, parvovirus 819 and human herpesvirus 6 are the viruses most frequently found
79 currently causing myocarditis. Coxsackie B, an icosahedral member of the Picornaviridae family, can cause a
80 variety of illnesses, including meningit is, respiratory infections, and epidemic pleurodynia. However, it is
important to note that coxsackievirus is not t he most common infectious cause anymore. Fifty percent of all
81
myocarditis is idiopathic. However, many infectious agents may cause myocarditis, including other viruses
82 (eg, adenovirus), bacteria (eg, Haemophilus influenzae, Mycoplasma pneumoniae, streptococcal infections),
83 and protozoa (Toxoplasma and Trypanosoma cruzi).
84 The image illustrates two foci (arrows) of inflammatory cellular infiltration characteristic of viral myocardit is.
85 Plcorna. ,ru. M,<:opl .... pneumonlae Trypanosoma cruzJ Protozoa MyocarditiS Parvovlru> B19 CO>SGC c u ><..-nophilu• ,nfluenzae I mage copyright © 2015,
86
Macmillan Publishers Limited
Hui"Mn herpesv1rus 6 Adenovtndae Heart failure Congestive heart failure Memngttis Coxsackie B vtrus Enterovtrus Vtrus ParvoVJrus Herpesviridae

87 Mycoplasma Toxoplasma gond11 Streptococcu;..

• 88 Bornholm dt5ease Bacterta Idiopathy Epidemic

• 89
A i s no t correct . 3°/o chose this .
• 90 Brugia malayi is a filarial worm t hat inf ect s t he lymphat ics and subcu taneous tissues. It can cause lym phadenopat hy, lymphatic destruction,
. 91 and subsequent edema . Infect ion with this worm wou ld not , however, explain th e 53 gallop .
Brugta malayi Lymphadenopathy Edema Filariasis Subcutaneous tissue Lymphatic system Filanotdea Lymph
• 92
• 93 C is n ot correct . 7 °/o chose thi s .
• 94 Haemophilus influenzae type B is a rod-shaped gram-negative bacterium t hat is responsible fo r respiratory infections and men ingitis in
• 95
childhood . It is currently recommended t hat all children receive t he Hib vaccine, which is effect ive against t his strain of bacteria. Although it
can be associated with infect ions of t he heart, it is not the most common cause of myocardit is .
. 96 Haemophtlus tnfluenzae Gram-negattve bacteria Hib vaccine Meningitis Myocard1bs Bactena Haemoph1lus va c1ne Bac1llu ($hape)
. 97
D is n ot co rrect . 3°/o chose this .
• 98
Norovirus is part of the Calcivirus family. It is a single-stranded, linear, icosahedral, nonenveloped virus that causes viral gastroenteritis. It is
• 99 not associated with myocarditis. Other vira l infections associa ted with myocarditis include enterovirus, coxsackie B, adenovirus, influenza,
• 100 cytomegalovirus, poliomyeli tis, Epstein- Barr virus, HIV-1, viral hepatitis, mumps, rubeola, varicella-zoster, variola/vaccinia, arbovirus, •
8
L.odt
s
Su~pl'nd
~
End Block
Item: 87 of ~ ,• Mark <::J [:::> ""I ~· ~'j
100 J.. Previous Next LAb faiUI~S Notes Calculator

74
D is not correct. 3°/o chose t his.
75
Norovirus is part of the Calcivirus family. I t is a sing le-stranded, linear, icosahedral, nonenveloped virus t hat causes v iral gast roent erit is. It is
76 not associat ed with myocardit is. Ot her viral infections associat ed with myoca rdit is include ent erovirus, coxsackie B, adenovirus, influenza,
77 cyt omegalovirus, poliomyelit is, Epstein-Barr virus, HIV-1, viral hepat it is, mumps, rubeola, varicella-zost er, variola/vaccinia, arbovirus,
78 respiratory syncytial v irus, herpes simplex v irus, yellow fever v irus, rabies, and parvovirus. The most common viral cause of myoca rdit is being
parvovirus B19 and human herpesvirus 6.
79 Arbovirus Enterovirus Epstein-Barr virus Norovirus Parvovirus 819 Polio,.,elitis Yellow fever Cytomegalovirus Myocarditis Human herpesvirus 6 Adenoviridae Mumps Rabies
80 Human respiratory syncytial virus Herpes simplex virus Gastroenteritis Viral envelope Viral hepatitis Parvovirus varicella zoster virus Herpes simplex Influenza Yellow fever virus HIV-1 Coxsackievirus
81 Herpesviridae Coxsackie A virus Hepatitis Shingles Virus Icosahedron Coxsackie 8 virus Regular icosahedron Fever
82
E is not correct. 1 5°/o chose this.
83
Streptococcus pneumoniae is a gram-posit ive diplococcus and the most common cause of lobar pneumonia worl dwide. Alt hough it can be
84 associat ed with infect ions of the heart, it is ( 1) not t he most common cause of myoca rdit is, and (2) a bact eri um, wh ich wou ld t herefore
85 respond to antibiot ics. Although it is possible that a poststreptococca l glomeru lonephri t is could lead to renal fa ilure and subsequent volume
86 overload, result ing in a clinica l picture of congestive hea1t failure, group A Streptococcus is the type of Streptococcus that may cause
glomerulonephrit is.
87 Streptococcus pneumoniae Myocarditis Diplococcus Heart failure Congestive heart failure Gram-positive bacteria Pneumonia Streptococcus Lobar pneumonia Glomerulonephritis Antibiotics Bacteria
• 88
F is not correct . 3°/o chose this .
• 89
Wuchereria bancrofti is a f ila ria l worm t hat produces symptoms similar to those from Brugia malayi infection. Howeve 1~ of all of t he f ilariae, W.
• 90 bancrofti causes t he overwhelm ing maj ori ty of genital lymphat ic involvement.
. 91 Wuchereria bancrofti 8rugia malayi Filarioidea Filariasis Worm Lymphatic system Sex organ

• 92
• 93
Bottom Line :
• 94
Coxsackie B, an icosahedral member of the Picornaviridae fami ly, can cause a va ri ety of illnesses, including meningit is, respirat ory infect ions,
• 95 and epidemic pleurodyn ia. However, it is import ant to note t hat although coxsackievirus was the most common cause of vira l myocardit is in
. 96 t he 1990s, parvovirus B19 and human herpesvirus 6 have become more common causes .
Picornavirus Parvovirus 819 Coxsackievirus Myocarditis Human herpesvirus 6 Meningitis Coxsackie 8 virus Parvovirus Icosahedron Herpesviridae Regular icosahedron Bornholm disease Virus Epidemic
. 97
• 98
• 99
l@);fil ~1hl for yea r:l 2o17 y
• 100 FIRST AID FACTS

8
Lock
s
Suspend
0
End Block
Item: 87 of - ,• Mark --<) [::> ""'I ~· 1!';:'1
100 ~ Prev1o u s Next Labf a lu es Notes Calculator
74
75 FA17 p 297.1
76 Cardiomyopathies
77
Dilated Most common card iomyopathy (90% of cases). Systolic dysfunction ensues.
78
cardiomyopathy Often idiopathic or familial. Other etiologies Eccentric hypertrophy rJ (sarcomcrcs added in
79
include chronic \ lcohol abuse, wet Beriberi, series).
80
Coxsackie B \'ira! nwocarditis, chronic ABCCCD.
81
82
~
..... '
Cocaine use, Chagas disease, Doxorubicin Takotsubo cardiomyopathy: "broken heart

(
I
'\
toxicity, hemochromatosis, sarcoidosis, srndrome"-ventricular apical ballooning
83 ..'(
84
'< .... .,·- peripartum cardiomyopathy. likel) due to increased sympathetic stimulation
85 Findings: H F, S3, systolic regurgitant murmur, (stressful situations).
86 dilated heart on echocardiogram, balloon
87 appearance of heart on CXR.
• 88 Treatment: 1 a+ restriction, ACE inhibitors,
• 89 ~-blockers, diuretics, digoxin, lCD, heart
• 90 transplant.
. 91
Hypertro phic 60-70% of cases are familial, autosoma l Diastol ic dysfunction ensues.
• 92
cardiomyopathy dom inant (most commonly due to 1111 1tations Marked ventricular concentric hypertrophy
• 93
in genes encoding sarcomeric proteins, such as (sarcomeres added in parallel) 1]), often septa I
• 94
myosin bind ing protein C and ~ -m)OS in heavy predominance. Myofibrillar disa rray and
• 95
. 96
chain). Can be associated with Friedreich fibrosis.
. 97
ata>.ia. Causes S) ncope during cxerci~c and llypert rophic obstructive cardiomyopathy
• 98 mar lead to sudden death in young athletes (subset)-asymmetric septal hypertroph)'
• 99 due to ventricular a rrlwth

111 ia. and systolic anterior motion of mitral valve
• 100 Findings: S4, srstolic murmur. \1ay see mitral ...... outflow obstruction ...... dyspnea, possible •
8
L.odt
s
Su~pl'nd
~
End Block
Item: 87 of ~ ,• Mark <::J [:::> ""I ~· ~'j
100 J.. Previous Next LAb faiUI~S Notes Calculator

74 • •
FA17 p 163. 1
75
76
RNA viruses
77 VIRAL FAMILY ENVELOPE RNA STRUCTURE CAPSID SYMMETRY MEDICAL IMPORTANCE
78 Reoviruses 0 OS linear Icosahedral Coltivirus"- Colorado tick fever
79 10-1 2 segments (double) Rotavirus- cause offa tal diarrhea in children
80
Picornaviruses No SS ® linear Icosahedral Poliovirus-polio-Salk/Sabin vaccines- IPV/OPV
81
Echovirus- aseptic meningitis
82
Rhi novirus-"common cold"
83
Coxsackievirus- aseptic meningitis; herpangina
84
(mouth blisters, fever); hand, foot, and mouth
85
disease; myoca rditis; pericarditis
86
IIAV-acute viral hepatitis
87
PERCI I
• 88
• 89 Hepevirus No SS ® linear Icosahedral HEV
• 90 Caliciviruses No Icosahedral 'orovirus-viral gastroenteritis
SS ® linear
. 91
• 92
Flaviviruses Yes SS ® linear Icosahedral HCV
• 93
Yellow fever1
• 94
Dengue3
• 95
St. Louis encephalitis3
. 96
West Tile virus3 (meningoencephalitis)
. 97
Zika virus
• 98 Togaviruses Yes SS ® linear Icosahedral Rubella
• 99 Western and Eastern equine encephalitis"
• 100 • Chikum!llnva ,·irus •

8
Lock
s
Suspend
0
End Block
Item: 87 of - ,• Mark -<J [:::> "'I ~ · ~
100 ~ P~v1ous N @xt Labl lues N o tes Calcula to r
74 •
complex II IV-AIDS
75
76
and conical
77
(IIIV)
78 Coronaviruses Yes SS <±) linear Helical "Common cold," SARS, ~ JERS
79
80
Orthomyxoviruses Yes ss e linear Helical lnAuenza virus
8 segments
81
82 Paramyxoviruses Yes ss e linear Helical PaRa \h-xo,·irus:

83 1 onsegmented Parain Auenza-croup
84 RS -bronchiolitis in babies; Rx- ribavirin
85 \1easles, \Jumps
86 Rhabdoviruses Yes ss e linear Helical Rabies
87
0 88
Filoviruses Yes ss e linear Helical Ebola/J\ larburg hemorrhagic fe,•er- often fatal!
• 89 Arenaviruses Yes ss <±)and e llelical I.CM - lymphocytic choriomeningit is \'irus
• 90 circular Lassa fever encephalitis-spread by rodents
. 91 2 segments
• 92
93
Bunyaviruses Yes ss e circular Helica l California encephalitisa
Sa nd ny/Ri ft Valley feversa
0

3 segments
• 94
Crimean-Congo hemorrhagic fe,·efl
0
95
l lanlavirus-hemorrhagic fever, pneumonia
. 96
. 97 Delta virus Yes ss e circular Uncertain HOVis a "defective" ,-irus that requires the
0 98 presence of HBV to replicate
0 99
ss. single-stranded; OS, double-stranded;<±>. positi\·e sense; e . negative sense; 3
= arbm·ims, arthropod borne (mosquitoes, ticks).
0
100 •
8
Lode.
s
Suspe-nd
8
End Bloc:k
Item: 88 of - ,• Mark -<J [:::> "'I ~ · ~
100 ~ P~v1ous N @xt Labl lues N o t es Calcula to r
74
A previously healthy 12-year-old boy presents to the emergency department for abdominal pain. Three days after eating in a fast-food ~~AI
75
hamburger restaurant, he developed diffuse abdomi nal pain and bloody diarrhea. He is up to date on all of his vaccinations. He is admitted
76
to the hospital. His stool smear shows many gram -negati ve organisms in the absence of feca l leukocytes. Stool cultu re reveals a gram-
77 negative oxidase-negative bacterial species t hat does not ferment sorbitol.
78
79 Which of the following findings or symptoms are consistent with a condition that can be caused by t he responsible organism?
80 :
81 A. Migratory polyarthritis and elevated C- reactive protein
82 B. New murmur and splinter hemorrhages
83
C. Petechial rash and bilateral hemorrhage into the adrenal gland
84
85 D. Symmetric ascending muscle weakness and hyporeflexia
86 E. Thrombocytopenia and elevated creatinine
87
' 88
• 89
• 90
. 91
• 92

' 93
• 94

' 95
. 96
. 97

' 98
' 99
' 100 •
8
Lode.
s
Suspe-nd
8
End Bloc:k
Item: 88 of ~ ,• Mark <::J [:::> ""I ~· ~'j
100 J.. Previous Next LAb faiUI~S Notes Calculator

74
75 The correct a nswer is E. 62°/o chose this .
76 Escherichia coli 0 157: H7 is an ent erohemorrhagic st ra in t hat has acquired Shiga t oxin and can cause bloody diarrhea, abdominal pain, and (in
some cases) mild feve r. It is often ingest ed in undercooked hamburger. The st ool smear indicates an infect ion and t he presence of a gram -
77
negative oxidase-negat ive bacteria. These f indings, combined wit h the history of ham burger ingestion and bloody diarrhea, make E. coli
78 0 157 : H7 a likely cause. Hemolytic- urem ic syndrome is a life-t hreaten ing condit ion associat ed wit h infection wit h t his organism. It is
79 ch aract erized by a low platelet count (t hrom bocytopenia), anemia, and renal fa ilure ( indicat ed by elevat ed creatinine), often manifested by
uremia.
80
Hemolytic-uremic syndrome Uremia Escherichia coli Thrombocytopenia Gram-negative bacteria Creatinine Diarrhea Platelet Anemia Verotoxin-producing Escherichia coli Toxin Bacteria Abdominal pain Fever
81
Kidney Infection Dysentery
82
83
A is not correct . 9°/o chose t his.
Rh eumatic feve r is characterized by fever, migrat ory polyarthrit is, and cardit is. C-react ive protein is a nonspecif ic inf lammat ory marker. It may
84
follow group A st rept ococcal pharyng it is, which has no connection with bloody diarrhea.
85 Streptococcal pharyngitis C-reactive protein Rheumatic fever Pharyngitis Polyarthritis Diarrhea Carditis Streptococcus Rheumatology Protein Rheumatism

86
B is not correct. 6% chose t his .
87
Fever, a new murmur, Janeway lesions, and nail bed hemorrhages are signs of bacterial endocarditis. Acut e endocardit is is caused by
88 Staphylococcus aureus; subacut e infection can be caused by viridans st re ptococci. However, neit her of t hese is associated wit h bloody
• 89 diarrhea .
Janeway lesion Endocarditis Staphylococcus aureus Viridans streptococci Infective endocarditis Streptococcus Staphylococcus Diarrhea Heart murmur Bleeding Nail (anato,.,) Fever
• 90
. 91 C is not co rrect. 10% chose this .
• 92 Wat erhouse-Frideri chsen syndrome is charact eri zed by high fever, shock, purpura, and adrenal insufficiency classically associated wit h
meningococcemia, although virt ually any bact erial cause of sept ic shock can t rigger t he same findings .
• 93
Waterhouse-Friderichsen syndrome Meningococcemia Adrenal insufficiency Septic shock Meningococcal disease Purpura Fever Adrenal gland
• 94
D is not correct. 13% c hose this .
• 95
Guillain-Barre syndrome is characteri zed by rapidly progressing ascending paralysis. Hyporef lexia is usually observed. It is t hought to follow a
. 96 vari et y of infect ious diseases, such as cytomegalovirus, Epst ein-Barr virus, HI V, and gast roent eri tis caused by Campylobacter j ejuni. Alt hough
. 97 C. jejuni can cause bloody diarrhea, it is distinguished on stool culture by its curved comma -shaped structure and is also oxidase posit ive .
Campylobacter jejuni Cytomegalovirus Epstein-Barr virus Hyporeflexia Gastroenteritis Campylobacter Diarrhea Stool test HIV Infection Paralysis Microbiological culture Virus Oxidase
• 98
• 99
• 100 Bottom Line :
8
Lock
s
Suspend
0
End Block
Item: 88 of ~ ,• Mark <::J [:::> ""I ~· ~'j
100 J.. Previous Next LAbfaiUI~S Notes Calculator

74
75 Bottom Line:
76 E. coli 0157: H7 is often acquired f rom undercooked hamburger and can cause bloody diarrhea . In some cases it can lead to hemolyt ic-
77 uremic syndrome characterized by t hrom bocyt openia, anemia, and uremia.
Hemolytic-uremic syndrome Uremia Escherichia coli Thrombocytopenia Diarrhea Anemia
78
79
80
81
i@l;fil·1i•J for year:[ 2017 • J
FIRST AID FACTS

82
83 FA17 p 140.1
84
Escherichia coli Gram 8 rod. E coli virulence factors: fimbriae- cystitis and pyelonephritis (P-pili); K capsule-
85
pneumonia, neonatal meningitis; LPS endotoxin-septic shock.
86
87 STRAIN TOXIN ANO MECHANISM PRESENTATION
88 EIEC Microbe invades intestinal mucosa and causes Invasive; dysentery. Clinica l manifestat ions
• 89
necrosis and inflammation . similar to Shigella.
• 90
. 91
ETEC Produces heat-labile and heat-stable Travelers' diarrhea (watery).
• 92
cnteroToxins. o inflammation or invasion.
• 93 EPEC o toxin produced. Adheres to apical surface, Diarrhea, usually in children (Pediatrics).
• 94 fl attens vi lli, prevents absorption .
• 95
EHEC 0157:H7 is most common serotype in US. Often Dysentery (toxin alone causes necrosis and
. 96
transmit led via undercooked meal, raw leafy inflammation).
. 97
vegetables. Does not ferment sorbitol (vs other E coli).
• 98
• 99
Shiga-like toxin causes hemolytic-uremic JT emorrhagic, Hamburgers, Hemolytic-u remic
• 100
syndrome: triad of anemia, thrombocytopenia, syndrome.

8
Lock
s
Suspend
0
End Block
Item: 88 of ~ ,• Mark <::J [:::> ""I ~· ~'j
100 J.. Previous Next faiUI~S
LAb Notes Calculator

74 peripheral blood smear), platelet consumption,


75
and ! rena l blood Aow.
76
77
78 FA17 p 174.3

79 Bugs causing food- S aureus and B cereus food poisoning starts quickly and ends quickly.
80 borne illness
MICROORGANISM SOURCE OF INFECTION
81
82 B cereus Reheated rice. "Food poisoning from reheated
83 rice? Be serious!" (B cereus)
84 C botulinum Improperly ca nned foods (toxins), raw honey
85 (spores)
86
87
C per{ringens Reheated meat
88 E coli 0157:1-17 Undercooked meat
• 89
L monOC)Itogenes Deli meats, soft cheeses
• 90
. 91
Salmonella Poultry, meat, and eggs
• 92 S aureus Meats, mayonnaise, custard; preformed toxin
• 93
V fJarahaemol)lticus and V vulnificus:• Contaminated seafood
• 94
aV vulnificus can also cause wound infections from contact with contaminated water or shellfish .
• 95
. 96
. 97 FA17 p 175.1
• 98 Bugs causing diarrhea
• 99
Bloody diarrhea
• 100

8
Lock
s
Suspend
0
End Block
Item: 88 of ~ ,• Mark <::J [:::> ""I ~· ~'j
100 J.. Previous Next LAbfaiUI~S Notes Calculator

74
75 FA17 p 175.1
76 Bugs causing diarrhea
77 Bloody diarrhea
78
Campylobacter Comma- or S-shaped organisms; growth at 42°C
79
80 E histolytica Protozoan; amebic dysentery; liver abscess
81 Enterohemorrhagic 0157:1-17; can cause IIUS; makes Shiga-like toxin
82 Ecoli
83
Enteroinvasive Ecoli Invades colonic mucosa
84
85 Salmonella (non- Lactose 8; Aagellar motility; has animal reservoir, especially poultry and eggs
86 typhoidal)
87
Shigella Lactose 8; very low 10 ;0; produces Shiga toxin (human reservoir only); bacillary dysentery
88
• 89 Yenterocolitica Day care outbreaks, pseudoappendicitis
• 90 Watery diarrhea
. 91
Cdifficile Pseudomembranous colitis; associated with antibiotics and PPis; occasionally bloody diarrhea
• 92
• 93 C perfringens Also causes gas gangrene
• 94 Enterotoxigenic E coli Tra,·elers' diarrhea; produces heat-labile (LT) and heat-stable (ST) toxins
• 95
Protozoa Giardia, Cryptosporidium
. 96
. 97 Vcholerae Comma-shaped organisms; rice-water diarrhea; often from infected seafood
• 98 Viruses Rota,·irus, norovirus, adenovirus
• 99
• 100

8
Lock
s
Suspend
0
End Block
Item: 89 of - ,• Mark -<J [:::> "'I ~ · ~
100 ~ P~v1ous N @xt Labl lues N o tes Calcula to r
74
A 27-year-old man is admit ted to t he hospital after present ing with shortness of breath and a f ever of 38°C (100 .4° F). One day after ~~AI
75
admission, he develops extreme pu lmonary edema and hypot ension and subsequently dies of acute respiratory distress syndrome. His
76
fami ly says that he had recent ly gone hiking and caving in an area known to be heavily populated wit h rod ent s.
77
78 Which of the following is the most likely cause of deat h in this patient?
79
:
80 A. Dengue virus
81
B. Ebola virus
82
83 C. Hantavirus
84 D. Marburg virus
85
E. Rhabdovirus
86
87
88
• 89
• 90
. 91
• 92
• 93
• 94
• 95
. 96
. 97
• 98
• 99
. 100 •
8
Lode.
s
Suspe-nd
8
End Bloc:k
Item: 89 of ~ ,• Mark <::J [:::> ""I ~· ~'j
100 J.. Previous Next LAbfaiUI~S Notes Calculator

74
75 The correct a nswer is C. 7 3 °/o chos e this .
76 Hant avirus pu lmona ry syndrome is a rare viral cause of acut e respiratory dist ress syndrome (ARDS). Hantavirus is a Bunyavirus that has been
found in rodents throughout the Unit ed Stat es. It is thought to be t ransmitted via rodent droppings and sa liva .
77
Bunyaviridae Hantavirus pulmonary syndrome Acute respiratory distress syndrome Hantavirus Rodent Dyspnea Virus
78
79
A is not correct . 5°/o chos e this.
Dengue virus is an Aedes mosquito-t ransmit ted v irus. It is found in t ropical reg ions of Asia and has spread t o Sout h and Cent ral America .
80
Pat ients most commonly present with fever and muscle/j oint pain, hence t he name "break-bone feve r," in addit ion t o headache and ret ro-
81 orbit al pain. Less common ly, they can present wit h a more severe form of t he disease, dengue hemorrhagic feve r, in which patients may
82 experience shock, circulatory collapse, thrombocytopenia, and hemorrhage f rom mucous membranes.
Thrombocytopenia Dengue virus Dengue fever Aedes Central America Virus Headache Mucous membrane Bleeding Viral hemorrhagic fever Asia Tropics Fever
83
84 B is not correct. 5% chose t his .
85 Ebola virus and Marburg v irus are members of the Filovirus genus, wh ich cause hemorrhagic feve r. Bot h are fou nd only in central and
southern Africa. They have an anima l reservoir t hat has not been found . Treat ment is support ive, and symptoms include massive hemorrhage
86
f rom t he mucous membranes accompanied by high fevers.
87 Marburg virus Genus Marburg Filoviridae Virus Ebola virus disease Viral hemorrhagic fever Fever Africa Bleeding

88
D is not correct. 8°/o chos e t his.
89 Ebola virus and Marburg v irus are members of the Filovirus genus, wh ich cause hemorrhagic feve r. Bot h are fou nd only in central and
• 90 southern Africa. They have an anima l reservoir t hat has not been found . Treat ment is support ive, and symptoms include massive hemorrhage
. 91 f rom t he mucous membranes accompanied by high fevers .
Marburg virus Genus Marburg Filoviridae Virus Ebola virus disease Viral hemorrhagic fever Fever Africa Bleeding
• 92
• 93 E is not correct. 9 °/o chos e this .
• 94 Rh abdovirus is t he causative agent of rabies. It is possible t o become infected wit h rab ies f rom a rodent ; however, the incubation period is
much longer (weeks to a year), and the later stages of t he disease are classically acute encephalit is .
• 95 Rodent Rabies Encephalitis Incubation period Rhabdoviridae
. 96
. 97
• 98
Bottom Line :
• 99 Hant avirus pulmona ry syndrome is one form of disease caused by t he rodent- borne hantavirus.
Hantavirus pulmonary syndrome Hantavirus
• 100

8
Lock
s
Suspend
0
End Block
Item: 89 of ~ ,• Mark <::J [:::> ""I ~· ~'j
100 J.. Previous Next LAb faiUI~S Notes Calculator

74 • •
FA17 p 163. 1
75
76
RNA viruses
77 VIRAL FAMILY ENVELOPE RNA STRUCTURE CAPSID SYMMETRY MEDICAL IMPORTANCE
78 Reoviruses 0 OS linear Icosahedral Coltivirus"- Colorado tick fever
79 10-1 2 segments (double) Rotavirus- cause offa tal diarrhea in children
80
Picornaviruses No SS ® linear Icosahedral Poliovirus-polio-Salk/Sabin vaccines- IPV/OPV
81
Echovirus- aseptic meningitis
82
Rhi novirus-"common cold"
83
Coxsackievirus- aseptic meningitis; herpangina
84
(mouth blisters, fever); hand, foot, and mouth
85
disease; myoca rditis; pericarditis
86
IIAV-acute viral hepatitis
87
PERCI I
88
89 Hepevirus No SS ® linear Icosahedral HEV
• 90 Caliciviruses No Icosahedral 'orovirus-viral gastroenteritis
SS ® linear
. 91
• 92
Flaviviruses Yes SS ® linear Icosahedral HCV
• 93
Yellow fever1
• 94
Dengue3
• 95
St. Louis encephalitis3
. 96
West Tile virus3 (meningoencephalitis)
. 97
Zika virus
• 98 Togaviruses Yes SS ® linear Icosahedral Rubella
• 99 Western and Eastern equine encephalitis"
• 100 • Chikum!llnva ,·irus •

8
Lock
s
Suspend
0
End Block
Item: 89 of - ,• Mark -<] 1:> ""'I ~· 1!';:'1
100 ~ Prev1o u s Next Labf a lu es Note s Calculator
74
SS, single-stranded; OS, double-stranded; Ef>, positi,·e sense; 8, negative sense; a= arbmims, arthropod borne (mosquitoes, ticks).
75
76
77 FA17 p 642.1
78
Acute respiratory Diagnosis of exclusion characterized by
79
distress syndrome respiratory failure within I \\ Cck of alveolar
80
insult, bilateral lung opacities, l Pa0z!Fi0 2
81
< 300 (hypoxemia due to f intrapulmonar)
82
83
shunting and diffusion abnormalities),
84
no evidence of 1-1 F/lluid O\erload. \ I an)
85
causes and associations, including sepsis,
86 pancreal itis, pneumonia, aspiral ion, I rauma,
87 shock. Endothelial damage - f aheolar
88 capillary permeability - protein-rich leakage
89 into alveoli - diffuse alveolar damage and
• 90 noncardiogcn ic pu lmonary edema (norma I
. 91 PCWP) fl. Results in formation of intra-
• 92 alveolar hyal ine membranes [l]. Initia l damage
• 93 due to release of neutrophilic substances
• 94 toxic to alveolar wall and pulmonary C<lpillary
• 95
endothelial cells, activation of coagulation
. 96
cascade, and oxygen-derived free radicals.
. 97
Management: mechanica l ventilation with low
• 98
tidal volumes, address underlying cause .
• 99
• 100 •

8
L.odt
s
Su~pl'nd
~
End Block
Item: 90 of - ,• Mark -<J [:::> "'I ~ · ~
100 ~ P~v1ous N @xt Labl lues N o t es Calcula to r
74
A previously healthy 41-year- old man misses several days of work in early February as a resu lt of a vira l ill ness with sympt oms including ~~AI
75
fever, headache, and fatigue. He also experiences a nonproductive cough and a sore throat . By the third day, his sympt oms begin to
76
subside and he is able to return to work. The next week, however, he experiences a rapid relapse. His cough ret urns, but now it is a
77 productive cough with mucopurulent sput um . He also begins t o experience pleuritic chest pain. On visiting a physician, X-ray of the chest is
78 ordered and is shown in the image.
79
80
81
82
83
84
85
86
87
88
89
• 90
. 91
• 92
• 93
• 94
• 95
. 96
Which of the following describes t he st ruct ure of t he viral genome that most likely caused his init ial illness?
. 97
:
• 98
A. Enveloped, double-stranded DNA
• 99
. 100 B. Enveloped, nonsegmented, single-stranded RNA

8
Lode.
s
Suspe-nd
8
End Bloc:k
75
76
77
78
79
80
81
82
83
84
85
86
87
88
89
• 90
. 91
• 92 Which of the fol lowing describes t he struct ure of the viral genome that most likely caused his init ial illness?
• 93 :
• 94 A . Enveloped, double-stranded DNA
• 95
B. Enveloped, nonsegmented, single-st randed RNA
. 96
. 97
C. Enveloped, segmented, single-stranded RNA
• 98 D. Nonenveloped, nonsegmented, single-stranded RNA
• 99
E. Nonenveloped, segmented, double-stranded RNA
• 100 •
8
L.odt
s
Su~pl'nd
~
End Block
Item: 90 of ~ ,• Mark <:::1 t::> ""I ~· ~'j
100 J.. Previous Next LAbfaiUI~S Notes Calculator

74
75 The correct a nswer is C. 55°/o chos e this .
76 The man init ia lly experiences nonspecific viral symptoms, wh ich alone are insuff icient t o make a presumptive diagnosis of t he causative
77 organ ism . It is clea r, however, that his init ial sym ptoms are dist inct from what he experi ences on relapse. He now appea rs to be suffe ri ng
f rom lobar pneumonia, wh ich can be caused by any number of bacterial species. The question t hat must be asked, therefore, is, "What viral
78
illness predisposes to su bsequent bacterial pneumonia in an otherwise healthy individual?" The classic answer is influenza . The inf luenza v irus
79 is an enveloped, sing le-stranded RNA virus with a segment ed genome that permits reassort ment of the genes encoding t he hemagglut inin and
80 neuraminidase proteins, result ing in the phenomenon of ant igen ic shift. Complicat ions of inf luenza include both viral pneumonia (due t o a
spread ing of the illness into t he lower resp iratory t ract ) and bacterial pneumon ia. The latter is t hought to be due largely to t he fact t hat
81
influenza damages t he epit helium of t he upper respiratory t ract , comprom ising its ability to keep t he lower respiratory t ract steri le.
82 Streptococcus pneumoniae, Staphylococcus aureus, and Haemophilus influenzae are t he organisms most commonly seen in bact eri al
83 pneumonia secondary to inf luenza. This chest X-ray shows a consolidation in t he right lower lobe along with a parapneumonic effusion, highly
suggest ive of bact erial pneumonia.
84
Antigenic shift Streptococcus pneumoniae Haemophilus influenzae Hemagglutinin Pneumonia Viral pneumonia Lobar pneumonia Neuraminidase Staphylococcus aureus Chest radiograph Viral neuraminidase
85
Hemagglutinin (influenza) Bacterial pneumonia Reassortment RNA virus Influenza Streptococcus Respiratory tract Orthomyxoviridae Virus Upper respiratory tract Staphylococcus Genome Epithelium RNA
86
Organism
87
88 A is not correct . 8°/o chos e t his.
The pat ient's init ial symptoms mig ht be seen in a person with acute infectious mononucleosis, consistent with an infect ion with Epstein- Barr
89
virus, wh ich is an enveloped, double-st randed DNA v irus. However, the sympt oms of mononucleosis t ypica lly last longer t han 3 days, and
90 bact eri al pneumonia is not a common complicat ion.
. 91 Infectious mononucleosis DNA virus Epstein-Barr virus Pneumonia Virus DNA Infection

• 92 B is not correct. 16% chose this .


• 93 The pat ient's init ial symptoms are consistent with an upper respirat ory infection, wh ich could be caused by coronavirus, which is an
• 94 enveloped, nonsegmented, single-stranded RNA virus. Infect ion wit h a coronavirus would not be expected to lead to bact erial pneumonia in a
healt hy individual.
• 95
Bacterial pneumonia Pneumonia Coronavirus Upper respiratory tract infection RNA virus Respiratory tract infection Virus RNA Infection
. 96
. 97
D is not correct. 13% c hose this .
Anot her cause of upper respiratory infections in adults are rhinoviruses, which are nonenveloped, nonsegment ed, single-st randed RNA
• 98
viruses. Rh inovirus infect ions are t ypically mild and uncomplicated in healthy individua ls, and a seconda ry bact erial pneumonia would be
• 99 atypical.
• 100 Rhinovirus Viral envelope Bacterial pneumonia Pneumonia Upper respiratory tract infection Virus RNA virus RNA
~

8
Lock
s
Suspend
0
End Block
Item: 90 of ~ ,• Mark <:::1 t::> ""I ~· ~'j
100 J.. Previous Next faiUI~S
LAb Notes Calculator

74
75
D is not correct. 13% c hose this.
Anot her cause of upper respiratory infections in adults are rhinoviruses, which are nonenveloped, nonsegment ed, single-st randed RNA
76
viruses. Rh inovirus infect ions are t ypically mild and uncomplicated in healthy individuals, and a secondary bact erial pneumonia would be
77 atypical.
78 Rhinovirus Viral envelope Bacterial pneumonia Pneumonia Upper respiratory tract infection Virus RNA virus RNA

79 E is not correct. 8°/o chose this .


80 Nonenveloped, segmented, double-st randed RNA viruses include reovirus; a frequent ly encount ered member of t he reovirus fami ly is the
81 rotavirus. Neit her t he rotavirus in part icular nor t he reoviral fa m ily in general are a cause of respirat ory illness in adu lts.
Rotavirus Viral envelope Reoviridae Double-stranded RNA viruses RNA virus Virus RNA
82
83
84 Bottom Line :
85 The inf luenza virus is an enveloped, segment ed, single-st ra nded RNA virus t hat predisposes an otherwise hea lt hy patient to secondary
86 bact eri al pneumonia.
Bacterial pneumonia Pneumonia Influenza RNA virus RNA Ortho,.,xoviridae Virus
87
88
89
141;fil·1i•J for yea r:[2017
FIRST AID FA CTS .
•j .
90
. 91
• 92 FA17 p 165.1
• 93
Influenza viruses Orthomyxoviruses. Enveloped, 8 ssRNA Reformulated vaccine ("the Au shot") contains
• 94
viruses with 8-segment genome. Contain viral strains most likely to appear during the Au
• 95
hemagglutinin (binds sialic acid and promotes season, due to the virus' rapid genetic change.
. 96
viral entry) and neuramin idase (promotes Killed viral vaccine is most frequent])' used.
. 97
progeny virion release) antigens. Patients at Live attenuated vaccine contains temperature-
• 98
risk for fata l bacterial superinfection, most sensitive mutant that replicates in the nose but
• 99
• 100
commonly S aureus, S pneumoniae, and not in the lung; administered intranasall)'.

8
Lock
s
Suspend
0
End Block
Item: 90 of - ,• Mark -<J [:::> "'I ~ · ~
100 ~ P~v1ous N @xt Labl lues N o t es Calcula to r


75 Influenza viruses Orthomyxoviruses. Enveloped, 8 ssR A Reformulated vaccine ("the Au shot") contains
76 viral strains most likely to appear dming the An
viruses with 8-segment genome. Contain
77
hemagglutinin (binds sialic acid and promotes season, due to the virus' rapid genetic change.
78
viral entry) and neuraminidase (promotes Killed viral \'accine is most frequently used.
79
progeny \'irion release) antigens. Patients at Live attenuated vaccine contains temperature-
80
risk for fatal bacterial superinfection, most sensitive mutant that replicates in the nose but
81
commonly S aureus, S pneumoniae, and not in the lung; administered intranasally.
82
83
I 1 influen::ae.
84 Genetic Causes pandemics. Reassortment of \'ira I Sudden shift is more deadly than gradual drift.
85 shift/antigenic shift genome segments, such as when segments of
86
•• human Au A virus reassort with swine Au
87 ••
••• \ ' lrtiS .
88 •
••••
89 Reassortment !
90 ••
• • 13
. 91
• 92 Genetic drift/ Causes epidem ics. Minor (antigenic drifl)
• 93
antigenic drift changes based on random mutation in
• 94
hemagglutinin or neura minidase genes.
• 95 ••• •
. 96 Random • •
mutations-- !
. 97
• 98
•.
·~
~ ~ 0
• 99
. 100 •
8
Lode.
s
Suspe-nd
8
End Bloc:k
Item: 90 of ~ ,• Mark <:::1 t::> ""I ~· ~'j
100 J.. Previous Next LAb faiUI~S Notes Calculator

74 • •
FA17 p 163. 1
75
76
RNA viruses
77
VIRAL FAMILY ENVELOPE RNA STRUCTURE CAPSID SYMMETRY MEDICAL IMPORTANCE
78 Reoviruses 0 OS linear Icosahedral Coltivirus"- Colorado tick fever
79 10- 12 segments (double) Rotavirus-cause of fata l diarrhea in children
80
Picornaviruses No SS ® linear Icosahedral Poliovirus-polio-Salk/Sabin vaccines- IPV/OPV
81
Echovirus-aseptic men ingitis
82
Rhi novirus-"common cold"
83
Coxsackievirus-aseptic meningitis; herpangina
84
(mouth blisters, fever); hand, foot, and mouth
85
disease; myoca rditis; pericarditis
86
IIAV- acute viral hepatitis
87
PERC ! I
88
89
Hepevirus No SS ® linear Icosahedral HEV
90 Caliciviruses No SS ® linear Icosahedral Norovirus- viral gastroenteritis
. 91
Flaviviruses Yes SS ® linear Icosahedral HCV
• 92
• 93
Yellow fever1
• 94
Dengue3
• 95
St. Louis encephalitis3
. 96
West ile virus3 (meningoencephalitis)
. 97
Zika virus
• 98 Togaviruses Yes SS ® linear Icosahedral Rubella
• 99 Western and Eastern equine encephalitis3
• 100 • Ch ikungunya virus •

8
Lock
s
Suspend
0
End Block
Item: 90 of - ,• Mark -<J [:::> "'I ~ · ~
100 ~ P~v1ous N @xt Labl lues N o t es Calcula to r
74 A
Retroviruses Yes SS ® li near Icosahedral Have re,·erse transcriptase A

75
2 copies (IITLV), IITLV-T-cellleukemia
76
complex HIV-AIDS
77
and conical
78
(HIV)
79
80 Coronaviruses Yes SS ®linear Helical "Common cold," SARS, ~rERS
81 Orthomyxoviruses Yes ss e linear llelical lnAuen1.a virus
82
8 segments
83
84
Paramyxoviruses Yes ss e linear llelical PaRa \ h• xovirus:
85 1 onsegmented ParainAuenza-croup
86
RSV-bronchiolitis in babies; Rx-riba,·irin
87
\leasles, \lumps
88 Rhabdoviruses Yes ss e linear Helical Rabies
89
90
Filoviruses Yes ss e linear llelica l Ebola/ larburg hemorrhagic fever-often fa tal!
. 91 Arenaviruses Yes ss ®a nd e Helica l LCMV- lymphocytic choriomeningitis virus
• 92 circular Lass<~ fever encephalitis-spread by rodents
• 93 2 segments
• 94
• 95
Bunyaviruses Yes ss e circular Helica l California encephalitis 3
3 segments SandAy/Rirt aile)' fe\"ers 3
. 96
Crimean-Congo hemorrhagic fe,·e~
. 97
I lanta\'irus-hemorrhagic fever, pneumonia
• 98
• 99
Delta virus Yes ss e circular Uncertain HOVis a "defective'' ,-irus that requires the
. 100
presence of H BV to rep Iicate

8
Lode.
s
Suspe-nd
8
End Bloc:k
Item: 91 of - ,• Mark -<] 1:> ""'I ~· 1!';:'1
100 ~ Prev1o u s Next Labf a lu es Note s Calculator
74
75
• A 58-year-old alcoholic is admit ted t o t he hospi tal after presenting with a 2-week history of worsening sore t hroat , hoarseness, and a
productive cough. Blood serum t ests reveal an elevated y-glutamyltransferase level. The patien t does not appear t o be in acute respiratory
(.AA] •
76 distress, but X-ray of the chest is taken and shown here. Culture on MacConkey agar reveals mucoid lactose-fermenting colonies. The
77 physician decides to treat the patient with cefuroxime.
~~~----------
78
79
80
81
82
83
84
85
86
87
88
89
90 Image copyright \t) 2008 BenvoiShltiif':z~,-;eJta/.
. 91
• 92 Which of the fol lowing organisms is t his pat ient most likely to have?
• 93 :
• 94 A. Chlamydia pneumoniae
• 95
B. Klebsiella pneumoniae
. 96
C. Legionella species
. 97
• 98 D. Mycoplasma pneumoniae
• 99 E. Streptococcus pneumoniae
• 100 •
8
L.odt
s
Su~pl'nd
~
End Block
Item: 91 of - ,• Mark -<] 1:> ""'I ~· 1!';:'1
100 ~ Prev1o u s Next Labf a lu es Note s Calculator
74
75 The correct answ er is B. 85°/o chose this.
76 Klebsiella pneumoniae is a frequ ent cause of nosocomial pneumonia (diagnosed with X- ray showing Most common ceu-Hs or pneumon41 in patleftts
77 segmental infiltrate, like that in t he vignette image) and is also associated with community-acquired W1lh predtSpoiJng behavlota l fK tOB
pneumonia in alcohol abusers. Chronic alcoholics, like t he patien t in this question, often have an elevated Prwdispoaing MOlt knpof11nt C I UMt of
78 FtctCW' Pneumonl•
serum y-glutamyltransferase level. Spu t um smear ty pically shows gram-negative encapsulated rods, and Mouth anMf'Otllll . • 0 .
-
79 culture on MacConkey agar grows mucoid colonies. Resist ance is common in Klebsiella species, so ~ Al\iOI ... (Ofttn mb:tcl
lnlechonl
80 treatment is dictated by the sensit ivity of the bacteria causing an individual patient's infection. Carbapenem
81
82
83
84
antibiotics are often required for resistant strains.
Cwbc.pe 1et t lef't. •el•l pneurnonr!'e Graf'P"''-negative bacteria HacConke'!' agar Sputum Convnumty·acqUired pueumon•a Pneumon•a

Mosp•tb·ac'-lu e pnem''10n•a Ho~p•t?l-acqUired •nfection Bacteria AlcohoiAsm Sputum culture serum (blood) Infection

A is not correct. 3°/o chose this.


I(Jebsu~lla Antibiotics

-- ---
lnnvtnous~utl St..,1c cccmr ~

Chlamydia pneumoniae is a cause of atypical pneumonia and is characterized by gradual onset of symptoms, sore throat, and hoarseness. The
85
infection is typically mildly symptomatic but can be life-threatening in older individuals, especially if coexisting chronic obst ructive pulmonary
86 disease or heart fa ilure is present . First -line t reatments are doxycycline (tetracyclines) or macrolides (erythromycin), but fluoroquinolones are
87 also effective.
Erythromycin Doxycycline ttyp1cttl pneumon1a Chronic obstructive pulmonary d1sease Chlamydoph1la pneumon•ae Chlamydia 1nfect1on Pneumonia Quinolone Macrolide Sore throat Heart fa1lure Infect1on
88
89 C is not correct. 4 °/o chose this.
90 Legionella species are common causes of community-acquired and nosocomial pneumonia in immunocompetent patient s. Infections are
associat ed wi t h a contaminated water source such as an air condit ioner. First- line t reatment is erythromycin with or wit hout rifam pin, bu t
91
doxycycline ca n also be used .
• 92 Erythromycin Doxycycline Legionella Rifampicin Pneumonia Hospital-acquired pneumonia A1r cond1t10n1ng Hosp1tal-acquired infect1on Immunocompetence

• 93
D is not correct . 5°/o chose this .
• 94
Mycoplasma pneumoniae occurs in young adult s and is seasonal, typi cally occurri ng in summer or fall. Diagnosis is made by pol ym erase chain
• 95 reaction analysis of sputum or by culture, and the preferred t rea tm ent is eith er doxycycline or eryth romycin .
Erythromycin Mycoplasma pneumon1ae Doxycycline Polymerase chain reaction Sputum Mycoplasma
. 96
. 97 E is not correct. 3°/o chose this .
• 98 Although Streptococcus pneumoniae is t he most common cause of bacterial pneumonia in t his age group, S. pneumoniae does not grow on
• 99 MacConkey agar.
streptococcu pneurnon1-e Bacter1a& pneumonia Pneumonia streptococcus MacConkey agar Agar
• 100 •
8
L.odt
s
Su~pl'nd
~
End Block
Item: 91 of - ,• Mark -<J
P~v1ous
[:::> "'I ~ ·· ~
100 ~ N @xt Labl lu es N o tes Calcula t o r
74 •
75 Botto m Li ne :
76 Klebsiella pneumoniae usually causes a productive cough and forms mucoid colonies on MacConkey agar. Since resistance is common,
77 treatment is guided by suscept ibility testing .
Klebs1ella pna~mon~ae M,c; C on~ agar Klebs1ella Agar Cough Mucus
78
79
80
14hfil·1i•J for year : 2017 •
81 FIRST AID FAC'"'S

82
83 FA17 p 140.2
84
Klebsiella 5 A'"' of KlebsielL-l

..... .
Cram 8 rod; intestinal flora that causes lobar
85

.. -· . ..,.... ...
- ~;;;p;::=' pneumonia in alcoholics and diabetics when Aspiration pneumonia
86
aspirated. Very mucoid colonies · caused by Abscess in lungs and liver
87
,· abundant polysaccharide capsu les. Dark red Alcoholics
88
..
89

•• •• "currant jelly" sputum (blood/mucus).


Also cause of nosocomial UTls.
Di- \ -betics
"Curr-A-nt jelly" sputum
90
91
• 92
• 93
• .· ~ *
• 94
• 95 FA17p1 38.1
. 96
lactose-fermenting Fermentation of lactose ..... pink colonies Lactme is key.
. 97
enteric bacteria on MacConkcy agar. Examples include Test with ~ l acCon KEE'S agar.
• 98
• 99
Klebsiella, E coli, Enterobacter, and E\ rB agar-lactose fermenters grow as pmple/
. 100
Serratia (weak fermenter). E coli produces black colonies. E coli grows colonies with a

8
Lode.
s
Suspe-nd
8
End Bloc:k
Item: 91 of - ,• Mark -<] 1:> ""'I ~· 1!';:'1
100 ~ Prev1o u s Next Labf a lu es Note s Calculator
74
FA17 p 185.1
75
Cephalosporins (generations 1-V)
76
77 MECHANISM ~-lactam drugs that inhibit cell wall synthesis Organisms typically not covered by 1st-4th
78 but are less susceptible to penicillinases. generation cephalosporins are LA:\ IE:
79 Bactericida I. Listeria, .\typicals (Chlamydia, Mycoplasma ),
80 ~t RSA, and Enterococci.
81
CLINICALUSE 1st generation (cefazolin, cephalexin)-gram @ lsi generation- PEcK.
82
cocci, Proteus mirabilis. E coli, Klebsiella
83
84
pneumoniae. Cefazolin used prior to surger) to
85
pre,·ent S aureus wound infections.
86 2nd generation (cefaclor, cefo\ilin, Fake fox fur.
87 cefuroxime)- gram (±) cocci, H influenzae, 2nd generation- HENS PEcK
88 Enterobacter aerogenes, 'Jeisseria ~pp., Serratia
89 marcescens, Proteus mirabilis, E coli, Klebsiella
90 pneumoniae.
91 3rd generation (ceftriaxone, cefotaxime, Can cross blood-brain barrier.
• 92 cefpodoxim e, ceftazidime)-scrious gram e Ceftriaxone-meningitis, gonorrhea,
• 93 infections resistant to other ~-lactams. disseminated Lyme disease.
• 94
Cefl azid ime- Pseudomonas.
• 95
. 96
4th generation (cefepime)- gram organisms, e
with t activity against Pseudomonas and gram
. 97
@organisms.
• 98
• 99 5th generation (ceftaroline)-broad gram @ and
• 100 gram 8 organism CO\'e rage; unlike 1st-4th •
8
L.odt
s
Su~pl'nd
~
End Block
Item: 92 of - ,• Mark -<J [:::> "'I ~ · ~
100 ~ P~v1ous N @xt Labl lues N o tes Calcula to r
74
A man presents to the emergency department ( ED) with drooling and prominent salivary glands. He is agitated and confused and appears ~~AI
75
manic. He is unable to provide an adequate history. His wife, who enters the ED shortly after his arrival, states t hat he has not been
76
himself lately. She denies any recent chemical exposures. She does reca ll, however, that he recently removed a bat from their closet and
77 during the encounter the bat became entangled in her husband's hair.
78
79 How does this patient's infection cause disease in the central nervous system?
80 :
81 A. Acute change in central nervous system osmolality
82 B. Axonal transport and direct infection
83
C. Coagulopathy causing intracranial hemorrhage
84
85 D. Hematogenous spread and direct infect ion
86 E. Metabolic abnormalities
87
88
89
90
91
• 92
• 93
• 94
• 95
. 96
. 97
• 98
• 99
. 100 •
8
Lode.
s
S uspe-nd
8
End Bloc:k
Item: 92 of - ,• Mark -<] 1:> ""'I ~· 1!';:'1
100 ~ Prev1o u s Next Labf a lu es Note s Calculator
74
The correct answer is B. 7 5°/o chose this.
75
This patient has rab ies, as evidenced by his altered mental status and enlarged salivary glands wit h profuse
76 salivation. Rabies is a sing le-stranded RNA virus that infects a host's central nervous system via retrograde
77 axonal transport. On histology, Negri bodies are eosinophilic inclusion bodies seen in the cytoplasm of
neurons from patients infected wit h rabies (blue circles). For all patients, postexposure prophylaxis begins
78
with thorough cleaning of the wound with soap and water. Previously unvaccinated patients should undergo
79 injection of rabies immunoglobulin directly around t he bite wound before receiving four rounds of t he
80 intramuscular rabies vaccine (day 0, 3, 7, 14, usually into the deltoid, but at a site distant to the bite).
Previously vaccinated patients should not be administered immunoglobulin at the injection cite; they should
81
receive two rounds of the rabies vaccine (day 0 and 3).
82 Negr-- bod•es Ceo ·~ ne tO 1 !1: \tern H•stolog Cytoplasm Rabies Antibod Rab•es tac.cme AA:»II a,.,, c transport Post~e".(posure prophytax~s Virus RNA virus
83 Sa11vary gland Vac:cme Sa11va Nenous system RNA lnclus•on bodies Neuron Prevenbve healthcar-e AJtered level of consc•ousness Eos•nophilic
84 Intramuscular •nJectJOn
85
86 Image courtesy CDC/Or.
87 Makonnen Fekadu
88
A i s not correct. 5°/o chose this .
89
Acute central nervous syst em osmolality change can alter mental status as a resu lt of herniation or neuronal deat h. This is commonly seen
90
when correcting severe hyponat remia or hypernat rem ia.
91 Hyponatremia Central nervous system Hypernatremia Osmolality Nervous system

92
C is not correct . 4 °/o chose thi s .
• 93
Whereas many infections can cause disseminat ed intravascular coagulation (DIC) and predispose t o intracranial hemorrhage, rabies is not
• 94 kn own to cause DIC. Think of DIC in bacterial septicemia, pancreat it is, and bacterial meningit is.
• 95
Disseminated Intravascular coagulation Intracranial hemorrhage Sepsis Rabies Memng1bs PancreatitiS Coagulation Bleed1ng Blood vessel Bacterial meningitis

. 96 D is not correct . 13°/o c hose thi s .


. 97 Although many infections spread hematogenously, the rabies virus is initially transported from the peripheral t o t he central nervous system
• 98 via peripheral nerves .
Central nervous s~ 'Stem Rabt~ Rab1es virus Virus Peripheral nervous system Nervous system
• 99
• 100 E i s not correct. 3% chose this . •
8
L.odt
s
Su~pl'nd
~
End Block
Item: 92 of - ,• Mark -<] 1:> ""'I ~· 1!';:'1
100 ~ Prev1o u s Next Labf a lu es Note s Calculator
74
E is not correct. 3°/o chose this.
75
Whereas metabolic abnormalities are a common cause of alt ered mental status (especially sodium changes), this is not a feature of rabies.
76 Rab1es Altered level of consciousness Sodtum Metabolism
77
78
Bottom Line:
79
Rabies spreads from peripheral to central nervous system via retrograde axonal transport.
80 Centra; neo rou tf'f"'"' Rabteo AlcDplasmc transport Nervous system
81
82
83 14);fi1·1hi for year : 2011 •
FllltST A ID FA C'"'S
84
85
FA17 p 167.1
86
87 Rabies virus Bullet-shaped virus · . 1\egri bodies Infection more commonly from bat, raccoon, and
88 (cytoplasmic inclusions Ill) commonly skunk bites than from dog bites in the United
89 found in Purkinje cells of cerebellum and States; aerosol transmission (eg, bat caves) also
90 in hippoca mpal neurons. Rabies has long possible.
91 incubation period (weeks to months) before
92
symptom onset. Postexposure prophylaxis
• 93
is wound cleaning plus immunization with
• 94
killed vaccine and rabies immunoglobulin .
• 95
Example of passi,·c-aetivc immunity.
. 96
Tra,·els to the C1 S by migrating in a retrograde
. 97
• 98
fashion (,;a dynein motors) up ner\'e axons
• 99
after binding to Ch receptors .
• 100
Progression of disease: fever, malaise •
8
L.odt
s
Su~pl'nd
~
End Block
75 is wound cleaning plus immunization with
76 killed vaccine and rabies immunoglobulin.
77 Example of passive-active immunity.
78
Tra,·els to the CNS by migrating in a retrograde
79
fashion (\·ia dynein motors) up nene axons
80
after binding to ACh receptors.
81
Progression of disease: fever, malaise
82
_... agitation, photophobia, hydrophobia,
83
hypersalivation _... paralysis, coma _... death.
84
85
86
87
FA17 p 158.3
88
Viral vaccines
89
90
Live attenuated \t1 'IR, Yellow fe,·er, Rota virus, In nuenza "Music and LYRICSS are best enjoyed Lhc."
91 vaccines (intranasal), Chickenpox (VZV), Smallpox, MMR = measles, mumps, rubella; live
92 Sabin polio virus. attenuated vaccine that can be given to
• 93 HIV ® patients who do not show signs of
• 94 immunodeficiency.
• 95 Sal K= Killed.
Killed Rabies, l nAuenza (injected), Salk Polio, and
. 96
HAV vaccines. Killed/inactivated \'accines Rl P Always.
. 97
induce only humoral immunity but are stable.
• 98
• 99 Subunit liB (antigen= liBsAg), liP (types 6, 11, 16,
• 100 and 18). •
8
L.odt
s
Su~pl'nd
~
End Block
Item: 93 of - ,• Mark -<J [:::> "'I ~ · ~
100 ~ P~v1ous N @xt Labl lues N o tes Calcula to r
74
A 6-year-old girl is broug ht t o her doctor by her mot her because t he daughter has pain in her right leg. The girl 's t emperat ure is 39. 7°C ~~AI
75
( 103.5° F). During gait t esting, she is reluctant t o stand because of the pain in her leg. A radiograph of her leg shows a small lytic lesion in
76
the distal metaphysis of t he ri ght t ibia wit h periost eal reaction. Blood cultures indicate the presence of a bacteri al infection.
77
78 What is the most likely appearance of t he responsible bacteria when stained with a Gram stain and visualized by a light microscope?
79
:
80 A. Gram-negative bacilli in clusters
81
B. Gram-negative cocci in pairs
82
83 C. Gram-positive cocci that are catalase+ and coagulase+
84 0 . Gram-positive cocci that are catalase+ and coagulase-
85
E. Gram-positive cocci that are catalase-
86
87
88
89
90
91
92
• 93
• 94
• 95
. 96
. 97
• 98
• 99
. 100 •
8
Lode.
s
S uspe-nd
8
End Bloc:k
Item: 93 of ~ ,• Mark <::J [:::> ""I ~· ~'j
100 J.. Previous Next LAb faiUI~S Notes Calculator

74
75
The correct answer is C. 80°/o chos e this.
This is a typical presentation of osteomyelit is in a child. The most common cause of osteomyelit is in ot herwise healt hy patient s is
76
Staphylococcus aureus, which accounts for >50% of cases. I n patients wit h ost eomyelit is, about 50% have posit ive blood cultures. S. aureus
77 is a gram-posit ive organ ism t hat is t ypically seen in clust ers. In add it ion to causing osteomyelitis, the organism causes t oxic shock syndrome,
78 staphylococcal scalded skin synd rome, fo lliculit is, food poison ing, and sepsis. S. aureus is cat alase+ and coagulase+ .
Staphylococcal scalded skin syndrome Osteomyelitis Toxic shock syndrome Staphylococcus aureus Sepsis Gram-positive bacteria Folliculitis Staphylococcus Foodborne illness Blood culture
79
80 A is not correct . 5°/o chos e this.
81 Gram -negative bacilli inclu de Escherichia coli, Citrobacter, and Enterobacter. A gram-negat ive bact eri um, Salmonella, common ly causes
ost eomyelit is in patients with sickle cell disease, but it is otherwise not a common cause of t he disease .
82 Osteomyelitis Gram-negative bacteria Escherichia coli Sickle-cell disease Citrobacter Salmonella Enterobacter Bacillus (shape) Bacteria Bacilli
83
B is not correct. 4% chose this.
84
Neisseria species are gram- negative bact eri a t hat are typically found in pairs. N. meningitides commonly causes meningit is, pneumon ia, and
85 sept ic arthrit is. N. gonorrhoeae infect ion typically manifests as cervicit is, urethrit is, proctit is, or conj unctivit is.
86 Gram-negative bacteria Septic arthritis Neisseria gonorrhoeae Urethritis Conjunctivitis Cervicitis Meningitis Pneumonia Proctitis Neisseria Neisseria meningitidis Bacteria Arthritis Species

87
D is not correct. 7°/o chose this.
88 A Gram sta in conta ining gram-posit ive cocci t hat are catalase+ but coagulase- means t hat the bact eria is a st aphylococcal speci es ot her t han
89 Staphylococcus aureus (epidermidis or saprophyticus) . Both of t hese are not known for being causes of osteomyelit is in healthy pat ients
90
without prosthet ics. Staphylococcus epidermidis is known t o cause osteomyelit is in t hose wit h prosthet ics.
Gram staining Osteomyelitis Staphylococcus epidermidis Coccus Coagulase Staphylococcus aureus Gram-positive bacteria Staphylococcus Bacteria Prosthesis Species
91
92 E is not correct. 4 °/o chos e this.
93
Streptococcus pneumoniae is a gram- posit ive organism t hat is t ypically found in pairs. It is a lead ing cause of lobar pneumon ia, acut e
sinusit is, and otit is med ia. I t is known to cause ost eomyelit is, but it does so much less commonly than does Staphylococcus aureus. The
• 94 pneumococcus bacterium is cat alase- .
• 95 Osteomyelitis Streptococcus pneumoniae Otitis media Gram-positive bacteria Sinusitis Staphylococcus aureus Lobar pneumonia Pneumonia Streptococcus Staphylococcus Bacteria Otitis Organism

. 96
. 97
Bottom Line:
• 98
Ost eomyelit is in otherwise healthy pat ients is most oft en caused by S. aureus .
• 99 Osteomyelitis Staphylococcus aureus

• 100

8
Lock
s
Suspend
0
End Block
Item: 93 of - ,• Mark -<] 1:> ""'I ~· 1!';:'1
100 ~ Prev1o u s Next Labf a lu es Notes Calculator
74
75 Ul:fi1·1i•J for yea r : 201 7 •
FIRST AIO f'ACTS
76
77
FA17 p 176.4
78
79 Osteomyelitis RISK FACTOR ASSOCIATED INFECTION
80 Assume if no other information is a\ailablc S aureus (most common o,·erall)
81
Sexually active 1 eisseria gonorrhoeae (rare), septic arthritis more
82
common
83
84 Sickle cell disease Salmonella and S aureus
85 Prosthetic joint replacement S aureus and S epidermidis
86
Vertebral invokement S aureus, lycobacterium tuberculosis (Poll
87
disease)
88
89 Cat and dog bites Pasteurella nwltocida
90 IV drug abuse Pseudomonas, Candida, S aureus are most
91 common
92
93
Elevated C-reactive protein (CRP) and eryth rocyte sedimentation rate common but nonspecifi c.
• 94
M Rl is best fo r detecting acute infection and detail ing anatomic involvement rJ. Radiographs are
• 95 insensiti\·e early but can be useru J in chronic OSteomyelitis I].
. 96
. 97
FA17 p 131 .3
• 98
• 99 Staphylococcus au reus Cram Ef), ~-hemolytic, catalase Ef>, coagulase TSST-l is a superantigen that binds to MHC
• 100 ~
Ef) cocci in clusters . Protein A (\ irulcncc II and T-cell receptor, resulting in polyclonal •
8
L.odt
s
Su~pl'nd
~
End Block
Item: 93 of - ,• Mark -<J [:::> "'I ~ · ~
100 ~ P~v1ous N @xt Labl lues N o tes Calcula to r
74 A A

75
76 FA17 p 131 .3
77
Staphylococcus aureus Gram (f), P-hemolytic, catalase (f), coagulase TSST-1 is a superantigen that binds to Ml IC
78
(f) cocci in clusters r.J. Protein A ('irulence II and T-cell receptor, resulting in polyclonal
79
factor) binds Fc-lgG, inhibiting complement T-cell acti\'ation.
80
acti,·ation and phagoc) tosis. Commonly Staphrlococcal toxic shock syndrome
81
colonizes the nares, axilla, and groin. (TSS) presents as fe\'er, \'Omiting, rash,
82
Causes: desquamation, shock, end-organ failure. T
83
84
lnAammatory disease-skin infections, results in t AST, t ALT, t bilirubin. Associated
85
organ abscesses, pneumonia (often after "ith prolonged use of \'aginaltampons or nasa l
86 inAuenza virus infection), endocarditis, packing.
87 septic arthritis, and osteomyel it is. Compa re '' ith Streptococcus pyogenes T S (a
88 loxin-mediated disease-toxic shock toxic shock-like syndrome associated with
89 syndrome (TSST-1), scalded skin syndrome painful skin infection).
90 (exfoliative toxin), rapid-onset food S aureus food poisoning due to ingestion of
91 poisoning (enterotoxins). preformed toxin - short incubation period
92 MRSA (methicilli n-resistantS aureus) (2-6 hr) followed by nonbloody diarrhea
93 infection- important cause of scriOltS <tnd emesis. Enterotoxin is heat stable - not
• 94 nosocomial and community-acquired destroyed by cooking.
• 95 infections; resistant to methicillin and Bad staph (cutreus) make coagulase and toxins.
. 96
nafcillin because of altered penicillin- Forms fibrin clot around self - abscess.
. 97
binding protein .
• 98
• 99
. 100
FA17 p 130.1

8
Lode.
s
S uspe-nd
8
End Bloc:k
Item: 93 of
wo - ,•
~ ~
rk <J 1:> •""'I ~·
y
1!';:'1
Prev1o u s Next Lab V alues Note s Calculator

74
FA17 p 130.1
75
76 Gram-positive lab algorithm
77
Gram<±> (purple/blue)
78
79
80
Branching
81 Bacim Cocci
filaments
82
83
AerobiC Anaerob4c Anaeroblcllacultallve
I
Aerob4c
-::::::1
Anaerob4c
84
85
Ustem
86 NOC4rd~a Actmomyces
Bicillus aostridium
87 (weakly aod lasU (not acid lasU
Corynebacterium Propiombacterium
88 Catalase
89 I...------<8)-- - ~'---------®~--------)
90 streptococcus Staphylococcus
91
T T
92 Hemolysis Coagulase
93 j_
• 94
(Partial (Complete
• 95 a hemolysis•
green)
hemolysis.
clear)
(No hemolySis.
grows in bile)
. 96 Saureus
Novobiocin
. 97
Optochin sensitivity Bacitracin sensitivity Growth in 6.5% NaCI sensitivity
• 98
and bile solubility
• 99
• 100 •
8
Lock
s
Su~pl'nd
~
End -Block
Item: 93 of - ,• Mark -<J [:::> "'I ~ · ~
100 ~ P~v1ous N @xt Labl lues N o tes Calcula to r
74 A A

75 Listt~
Clostridium Nocardta Actinomyces
Bacillus
76 (weakly acid fa sO (not acid fast)
Corynebacterium Propionibacterium
77 Catalase
78 I.------~e}--­ l f'D
~--------~~r------~1
79
streptococcus Staphylococcus
80
81 Hemolysis Coagulase
82
83
(Partial (Complete
(X hemolysis. (No hemotygs.
84 hemolygs.
green) dear) grows in bile)
85 SaumJS
Novobiocin
86
Optochin sensitivity Bacitracin sensitivity Growth in 6.5% NaCI sensitivity
87
and bile solubility
88
89
90 Group B Group A Ssaprophyticus Sepidtrmidis
91 Sagalactiae Spyogenes
92
93 -& 1
• 94 Vindans streptococci Group D
(no capsule) Spneumoniae Nonenterococcus (enterococcus)
• 95
Smutans (encapsulated) SboVIS Efaecium
. 96 Smitts Efaecatis
. 97
Important tests are 1n bold. Important pathogens are in bold italics.
• 98 Note: Enterococcus IS either a- or y-hemolytic.
• 99
. 100 •
8
Lode.
s
S uspe-nd
8
End Bloc:k
Item: 94 of - ,• Mark -<J [:::> "'I ~ · ~
100 ~ P~v1ous N @xt Labl lues N o tes Calculato r
74
A 4-year old boy from a rural community in Californ ia presents with a 1-day history of nausea, fever, and abdom inal cramps. His mother ~~AI
75
also says that the patient has had several loose bloody stools since his symptoms started. The patient has not t raveled recently, and his
76
mother does not recall him eating any undercooked foods. No one else in the household is sick. When asked about pets, the mother
77 mentions that the patient's older brother bought a pet turtle about a week ago and both her sons have been playing with it almost nonstop.
78
79 In terms of diagnosis and treatment of this condition, which of the following is the most important considerat ion?
80 :
81 A. Hematogenous spread is likely with nontyphoidal salmonellosis
82 B. Laboratory studies will show Salmonella species as a lactose-fermenting, gram-negative, nonmotile rod
83
C. Nontyphoidal salmonellosis has decreased in incidence in the United States
84
85 D. Symptoms of nontyphoidal Salmonella species infection can be prolonged with antibiotic treatment
86 E. There is no animal reservoir for Salmonella species
87
88
89
90
91
92
93
• 94
• 95
. 96
. 97
• 98
• 99
. 100 •
8
Lode.
s
S uspe-nd
8
End Bloc:k
Item: 94 of ~ ,• Mark <:::1 t::> ""I ~· ~'j
100 J.. Previous Next faiUI~S
LAb Notes Calculator

74
The correct ans wer is 0. 51°/o chose this.
75
I n cases of gastroenterit is due to infect ion wit h Salmonella typhimurium or S. enteritidis, t he sym ptoms are prolonged by ant ibiot ic t reatment.
76
St ud ies have shown t hat t reatment wit h ant ibiotics has increased rates of relapse and prolonged carri age of t he organ ism in the
77 gast roint estinal t ract. Dehydration due to diarrhea should be t reat ed wit h f luid and electrolyte re placement. Pat ients wit h evidence of S. typhi
78 bacteremia or foca l infect ion should, however, be t reat ed with antibiotics.
Salmonella enterica subsp. enterica Bacteremia Gastroenteritis Electrolyte Diarrhea Antibiotics Human gastrointestinal tract Salmonella Gastrointestinal tract Dehydration Organism Infection
79
80 A is not correct . 18% chos e this .
81
Hemat ogenous spread of these Salmonella species does occur in up to 5% of pat ients. Of note, S. typhi is more likely t o cause systemic
illness t han nontyphoidal spp., but it does so by first gaining access to t he lymphatics and then t he bloodstream. In general, infants, t he
82 elderly, and immunosuppressed individua ls are more likely to experience disseminat ion of a Salmonella infection. This, however, is st ill an
83 uncommon presentat ion.
Salmonella Immunosuppression Lymphatic system Systemic disease Bacteremia Species Circulatory system Infection
84
85 B is not correct. 12% chose this .
86 Ninety-nine percent of Salmonella species are non-lactose-fe rment ing, gram- negative, and mot ile rods.
Gram-negative bacteria Motility Salmonella
87
88 Cis not correct. 14% chos e this .
89 Nonty phoidal sa lmonellosis refers t o infections caused by most seroty pes of Salmonella ot her t han Typhi, Paratyphi A, Paratyphi B, and
Paratyph i C. I t most commonly manifests as gastroenterit is. Nont yphoidal sa lmonellosis is t he lead ing cause of bacterial diarrh ea worldwide,
90
and it has doubled in incidence in the Unit ed Stat es in the last 20 years.
91 Salmonellosis Gastroenteritis Diarrhea Serotype Salmonella United States
92
E is not correct. 5 °/o chos e this.
93
Although Salmonella typhi uses only humans as a reservoir, S. typhimurium and S. enteritidis can be carri ed by hens (most common ly ) and
94 by rept ile pet s such as iguanas.
• 95 Reptile Salmonella Iguana

. 96
. 97
Bottom Line:
• 98
Treatment of Salmonella- mediat ed gast roent eri t is wit h antibiotics in the absence of bact eremia can bot h prolong the sympt omat ic course of
• 99 t he disease and increase t he risk of relapse .
• 100 Bacteremia Gastroenteritis Salmonella Antibiotics

8
Lock
s
Suspend
0
End Block
Item: 94 of - ,• Mark --<] [::> ""'I ~· 1!';:'1
100 ~ Prev1o u s Next Labf a lu es Notes Calculator
74 • •
75 FA17 p 141 .1
76 Salmonella vs Shigella Both Salmonella and Shigella are gram 8 rods, non-lactose fermenters, oxidase 8, and can invade
77 the CI tract via \I cells of Peyer p<ltchcs.
78
Salmonella typhi Salmonella spp. Shigella
79
(exceptS typhi)
80
81
RESERVOIRS Humans onlr Humans and animals Humans onh-
82 SPREAD Can disseminate Can dissemi nate Cell to cell; no hematogenous spread
83 hematogenously hematogenous!)
84
H2SPRODUCTION Yes Yes 1'\o
85
86
FLAGEI A Yes (\almon swim) Yes (\almon \\\ im) 'o
87 VIRULENCE FACTORS Endotoxin; Vi capsule Endotoxin Endotoxin; Shiga toxin (enterotoxin)
88
INFECTIOUS DOSE IIDsol ll igh-large inoculum lligh Low-\·ery small inoculum required;
89
required because organism resistant to gastric acids
90
91
inactivated by gastric acids
92 EFFECT Of ANTIBIOTICS ONFECAL Prolongs duration Prolongs duration Shortens duration
93
EXCRETION
94 IMMUNE RESPONSE Primarily monocrtes PM s in disseminated Primarily PM l in filtrati on
• 95 disease
. 96
Gl MANIFESTATIONS Constipation, followed by Diarrhea (possibly bloody) Bloody diarrhea (bacillary dysentery)
. 97
diarrhea
• 98
• 99 VACCINE Oral \'accine contains li\'e 'o \'accine No vaccine
• 100 attenuated S typhi •
8
L.odt
s
Su~pl'nd
~
End Block
Item: 95 of - ,• Mark -<J [:::> "'I ~ · ~
100 ~ P~v1ous N @xt Labl lues N o tes Calcula to r
74
A 43-year-old man who is HIV posit ive presents t o emergency department complaining of vision problems fo r t he past 2 days. He has not ~~AI
75
been seeing a physician regularly, and his CD4 cell count is 24/mm 3 . Neurologic examination reveals problems wit h speech, memory, and
76
coordination. He is admitted to the hospi tal but his symptoms rapid ly worsen, and 3 weeks after admission the patient dies. During the
77 course of his hospital stay, a 3D FLAIR MRI revealed nonenchancing areas of demyelination.
78
79 What is the most likely cause of death in this patient ?
80 :
81 A. Cryptococcus neoformans
82 B. Herpes simplex virus
83
C. JC virus
84
85 D. Pneumocystis jirovecii
86 E. Toxoplasma gondii
87
88
89
90
91
92
93
94
• 95
. 96
. 97
• 98
• 99
. 100 •
8
Lode.
s
Suspe-nd
8
End Bloc:k
Item: 95 of - ,• Mark -<J [:::> "'I ~ · ~
100 ~ P~v1ous N @xt Labl lues N o tes Calcula to r
74
The correct answ er is c. 67°/o chose this.
A A

75
This patient presented with progressive mult ifocalleukoencepha lopathy (PML), which is caused by t he JC
76 virus in patients with AIDS. PML is a react ivat ion of a dormant virus to which the patient has previously
77 been exposed. Initial findings include neurologic deficits of speech, memory, and coordination. Vision
78 problems are also common. The disease causes a very rapid decline in neurologic function, resulting in
coma and death. The 3-week course for this patient is not uncommon. The disease causes multiple areas of
79 demyelination throughout the white matter of the central nervous system (similar to that indicated by the
80 arrows in this image). FLAIR MRI demonstrates subcortical white matter signal hyperintensity. There is no
81 specific treatment for PML, but some patients have shown some clinical improvement with the initiation of
highly active antiretroviral therapy (HAART).
82 Progress <e leul oencephalopath JC V1rus Central nen.ous system Management of HJVJJ(IOS •lh1te 11\btter- OemyeunMJng d•sease
83 Leukoencephalopathy HIV/AIOS Magnetic resonance m-.ag.ng Virus Nervous system Coma Neurology
84
85
86
87
88
89
Image copyright© 2013 Lind~
90 and von Heijne
91
92 A is not correct . 10 °/o chose thi s.
93 Cryptococcus neoformans is a common cause of meningit is is patients wit h HIV/AIDS . The classic meningit is t riad of fever, headache, and
nuchal rigid ity are usually present. Abnormalit ies on cerebrospinal fl uid examination would also be present.
94 Cryptococcus neoformans Cerebrospinal flUid Meningitis Neck stiffness Headache Men1ng1sm Fever Cryptococcus Neck
95
B is not co rrect. 6 % chose this .
. 96
Herpes simplex virus can cause temporal lobe encephalit is in patients with HIV/AIDS. It is also seen in t he general population but at a lower
. 97
frequency. Rapid onset of fever and foca l neurologic deficits are the most common presenting features. Deficit s often stem from damage to
• 98 the temporal lobe and can include memory problems, personality changes, and potentially seizures. The rapidity of the onset and a high fever
• 99 help to differentiate HSV encephalitis from PML.
Her-pes <~~'1mple: 11 u E1 C'l'$hahb. Temporal lobe Herpes s•mplex Virus Fever Epilepbc setzure Neurology
. 100 •
8
Lode.
s
Suspe-nd
8
End Bloc:k
Item: 9S of ~ ,• Mark <::J [:::> ""I ~· ~'j
100 J.. Previous Next LAb faiUI~S Notes Calculator

74
D is not correct. 7°/o chos e this.
75
Pneumocystis jirovecii (formerly carinii) is a common cause of pneumonia in pat ients wit h HIV whose CD4 + cell counts are <200/mm 3 • On X-
76
ray of the chest, the classic pict ure is one of "ground glass, " alt hough other rad iologic feat ures are also common.
77 Pneumocystis jirovecii Chest radiograph Pneumonia HIV Pneumocystis pneumonia X-ray Radiology CD4

78
E is not correct. 10°/o chos e this.
79 Toxoplasmosis is t he most common cause of encephalit is in pat ients with HI V/AIDS and is seen mostly in pat ients wit h a CD4+ cell count
80 < 100/mm 3 . The most common manifestation of t oxoplasmosis is seizures and headache, although ot her focal neurologic def icits may be seen.
81 The classic radiologic pictu re is one or more ri ng-enhancing lesions wit h surrounding edema. It would be unusual fo r a Toxoplasma infection
t o progress from clinical present at ion to deat h in 3 weeks.
82 Toxoplasmosis Encephalitis Headache Toxoplasma gondii Edema Epileptic seizure HIV/AIDS CD4 Neurology Infection Radiology
83
84
85
Bottom Line:
86 JC v irus is an infect ion of HI V patients t hat results in PML, in which m ult iple neurolog ic problems lead to a quick deat h.
JC virus Virus HIV Infection Progressive multifocal leukoencephalopathy Neurology
87
88
89
l@ljl'il·1i•J for year:[2017 • J
90 FIRS T AID FACTS

91
92 FA17 p 494.1
93 Other demyelinating and dysmyelinating diseases
94 Acute disseminated Multifocal in Aammation and demyelination after infection or vaccination. Presents with rapidly
95 (postinfectious) progressive multifocal neurologic symptoms, altered mental status.
. 96
encephalomyelitis
. 97
• 98
Charcot-Marie-Tooth Also known as hereditary motor and sensory neuropathy (HMSN). Group of progressive hereditary
• 99 disease nen·e disorders related to the defective production of proteins invoh-ed in the structure and
• 100 function of peripheral nerves or the myelin sheath. Typically autosomal dominant inheritance
8
Lock
s
Suspend
0
End Block
Item: 95 of - ,• Mark -<] 1:> ""'I ~· 1!';:'1
100 ~ Prev1o u s Next Labf a lu es Note s Calculator
74
75
Acute disseminated Multifocal inAammation and demyelination after infection or vaccination. Presents with rapidly
76
(postinfectious) progressive multifocal neurologic symptoms, altered mental status.
77 encephalomyelitis
78 Charcot-Marie-Tooth Also known as hereditary motor and sensory neuropathy ( H~ lSN). Group of progressive hereditary
79 disease nen·e disorders related to the defective production of proteins invoked in the structure and
80 function of peripheral nen·es or the myelin sheath. Typically autosomal dominant inheritance
81 pattern and associated with foot deform it ics (cg, pes cants, hammer toe), lower extremity
82
weakness (eg, foot drop) and sensory deficits.
83
84
Krabbe disease utosomal recessive lysosomal storage disease due to deficiency of galactocerebrosidase. Buildup
85
of galactocerebroside and psychosine destroys myelin sheath. Findings: peripheral neuropathy,
86 de,elopmental delay, optic atrophy, globoid cells.
87 Metachromatic Autosomal recessi,·e lysosomal storage disease, most commonly due to arylsulfatase A deficiency.
88 leukodystrophy Buildup of sulfatidcs -- impaired production and destruction of myelin sheath. Findings: central
89 and peripheral demyelination with ataxia, dementia.
90
Progressive multifocal Demyel ination of Cl S fJ clue to destruction of oligodenclrocytes. Seen in 2-4% of AIDS patients
91
92
leukoencephalopathy (reactivation of latent JC virus infection). Rapidly progressive, usually fatal. t risk associated with
93
natal iwmab, rituximab.
94
95
. 96
. 97
• 98
• 99
• 100 •
8
L.odt
s
Su~pl'nd
~
End Block
Item: 9S of ~ ,• Mark <::J [:::> ""I ~· ~'j
100 J.. Previous Next faiUI~S
LAb Notes Calculator

74
FA17 p 500.1
75
76
Spinal cord lesions
77 AREA AFFECTED DISEASE CHARACTERISTICS
78 Poliomyelitis and Werdnig-Hoffmann Congenital degeneration of anterior horns of spinal
79 disease cord. LM lesions only. "Floppy baby" with marked
80
81
82
•• hypotonia and tongue fasciculations. Infa ntile type
has median age of death of 7 months. Autosoma I
recessive inheritance.
83 Poliomyelitis ..... asymmetric weakness.
84 Werdnig-Hoffmann disease ..... symmetric weakness.
85
Amyotrophic lateral sclerosis Combined UMN and LMt defi cits with no sensory
86

•• • •
or bowel/bladder deficits (clue to loss of cortical and
87
spinal cord motor neurons, respectively).
88
Can be caused by defect in superoxicle clismutase 1.
89
Commonly presents with asymmetric limb weakness
90
(hands/feet), fascieulations, eventual atrophy. Fatal.
91
Commonly known as Lou Gehrig disease.
92
Treatment: riluzole.
93
Poslerior spinal arteries Complete occlusion of anterior Spares dorsal columns and Lissauer tract; mid-
94
./"-... spinal artery thoracic ASA territory is watershed area, as artery
95
. 96 of Adamkiewicz supplies ASA below - T8. Presents
. 97 with U I deficit below the lesion (corticospinal
• 98 tract), LM N deficit at the level of the lesion (anterior
• 99 horn), and loss of pain and temperature sensation
• 100
below the lesion (spinothalamic tract).
8
Lock
s
Suspend
0
End Block
Item: 95 of - ,• Mark -<] 1:> ""'I ~· 1!';:'1
100 ~ Prev1o u s Next Labf a lu es Note s Calculator
74 • •
75
Tabes dorsalis Caused by 3° syph iIis. Results from degeneration
76
(demyelination) of dorsal columns and roots
77 - progressive sensory ataxia (impaired
78 proprioception - poor coordination).
79
Associated with Charcot joints, shooting pain, Arg)•ll
80
Robertson pupils.
81
E'am "ill demonstrate absence of DTRs and
82 Et> Romberg sign.
83 Syringomyelia S) rinx expands and damages anterior white
84 commissure of spinothalamic tract (2nd-order

• neurons) - bilateral loss of pain and temperature


85
86 sensation in cape-like distribution; seen with Chiari
87 I malformation; can expand and affect other tracts.
88
Vitamin 812 deficiency Subacute combined degeneration (SCD)-
89
demyelination of Spinocerebellar tracts, lateral

' '
90
Corticospinal tracts, and Dorsal columns. Ataxic
91
gail, pares!hesia, impaired position/vibrat ion sense.
92
93
94
Cauda equina syndrome Unilateral symptoms including radicular pai11, absent
95
knee and ankle reAex, loss of bladder and anal
. 96
sphincter control. Can cause saddle anesthesia.
. 97 Treatment: emergent surgery and steroids.
• 98
Due to compression of spinal roots from L2 <mel
• 99
below, often caused by intravertebral disk herniation
• 100
or tumors. •
8
L.odt
s
Su~pl'nd
~
End Block
Item: 9S of ~ ,• Mark <::J [:::> ""I ~· ~'j
100 J.. Previous Next LAb faiUI~S Notes Calculator

74
75 FA17 p 173.1
76
Common diseases of As CD4+ cell count ~, risks of reactivation of past infections (eg, TB, HSV, shingles), dissemination
77
HIV-positive adults of bacterial infections and fungal infections (eg, coccidioidomycosis}, and non-Hodgkin
78
lymphomas t.
79
80 PATHOGEN PRESENTATION FINDINGS
81 CD4+ cell count< 500/mm 3
82
Candidaalbicans Oral thrush Scrapable white plaque, pseudohyphae on
83
m1croscopy
84
85
EBV Oral hairy leukoplakia Unscrapable white plaque on lateral tongue
86 Bartonella henselae Bacillary angiomatosis Biopsy with neutroph ilic in Aammation
87 HHV-8 Kaposi sarcoma Biopsy with lymphocytic inflammation
88
HPV Squamous cell carcinoma, commonly of anus
89
(men who have sex with men) or cerYix
90
(women)
91
92 CD4+ cell count< 200/mm 3
93 Histoplasma Fever, weight loss, fatigue, cough, dyspnea, Oval yeast cells within macrophages
94 capsulatum nausea, vom iting, diarrhea
95
HIV Dementia
. 96
. 97 JC virus (reactivation) Progressive mu ltifoca lleukoencephalopathy onenhancing areas of demyelination on IRl
• 98 Pneumocystis jirovecii Pneumocrstis pneumonia "Ground-glass" opacities on CXR
• 99
CD4+ cell count< 100/ mm 3
• 100 ~

8
Lock
s
Suspend
0
End Block
Item: 95 of - ,• Mark -<J [:::> "'I ~ · ~
100 ~ P~v1ous N @xt Labl lues N o tes Calcula to r
74 A
CD4+ cell count < 200/mm 3 A

75
76
Histoplasma Fever, weight loss, fatigue, cough, dyspnea, Oval yeast cells within macrophages
77 capsulatum nausea, ,·omiting, diarrhea
78 HIV Dementia
79
JC virus (reactivation) Progressi\'e multifocal leukoencephalopat h) onenhancing areas of demyelination on \I Rl
80
81
Pneumocystis jirovecii Pneumocystis pneumonia "Ground-glass" opacities on CXR
82 CD4+ cell count < 100/mm 3
83 Aspergillus fumigatus Hemoptysis, pleuritic pain Cm itation or infiltrates on chest imaging
84
Candida albicans Esophagitis White plaques on endoscopy; yeast and
85
pseudohyphae on biopsy
86
87 CMV Retinitis, esophagitis, colitis, pneumonitis, Linear ulcers on endoscopy, cotton-wool spots
88 encepha Iit is on fundoscopy
89 Biopsy reveals cells with intranuclear (owl eye)
90 inclusion bodies
91 Cryptococcus Meningitis Encapsulated yeast on India ink stain or
92 neoformans capsular antigen<±'!
93
Cryptosporidium spp. Chronic, watery diarrhea Acid-fast oocysts in stool
94
95 EBV B-celllymphoma (eg, non-Hodgkin lymphoma, C S lymphoma- ring enhancing, may be
. 96 CNS lymphoma) solitary (,·s Toxoplasma )
. 97 Mycobacterium onspecific systemic symptoms (fever, night
• 98 avium- intracellulare, sweats, weight loss) or focal lymphadenitis
• 99
Mycobacterium avium
. 100

8
Lode.
s
Suspe-nd
8
End Bloc:k
Item: 96 of - ,• Mark -<J [:::> "'I ~ · ~
100 ~ P~v1ous N @xt Labl lues N o tes Calcula to r
74
75 A 34-year-old man presents to the emergency department due t o diff iculty breathing, dry cough, and low-grade fevers for t he past 2-3
weeks. Initially, he noted t hat recent ly he becomes more winded t han usual when climbing stairs. Present ly, he is unable to speak full
76
sentences without feel ing short of breath. He reports increasing fat igue and weight loss in recent mont hs. He denies smoking or alcohol or
77 recreational drug use but admits to having had multiple sexual partners in the past 5 years wit hout using protect ion. He has not seen a doctor
78 for 9 years. Vital signs are significant fo r a fever of 38.1 oc (100.6° F), heart rate of 96/min, respirat ory rate of 22/min, and oxygen saturation of
86% on room air. There are bilat eral ra les at the lung bases as well as right axillary and left inguinal lymphadenopathy. X-ray of the chest is
79 shown.
80
81
82
83
84
85
86
87
88
89
90
91
92
93
94
I mage cowtesy of Radiopaedia/ Dr. Ayush Goel and Dr. Behrang Amini
95
. 96
. 97 What is the mechanism of act ion of the drug considered t he t reat ment of choice for t his pat ient 's pulmonary symptoms?
• 98 :
• 99
A. Inhibition of folate synthesis
. 100 • B. Inhibition of peptidoglycan cross-linking

8
Lode.
s
Suspe-nd
8
End Bloc:k
Item: 96 of - ,• Mark -<J [:::> "'I ~· ~
100 ~ P~v1ous N @xt Labl lues N o tes Calcula to r
A s own. A

75
76
77
78
79
80
81
82
83
84
85
86
87
88
89
Image courtesy of Radiopaedia/Dr. Ayush Goel and Dr. Behrang Amini
90
91
What is the mechanism of act ion of t he drug considered the trea tment of choice for this pa t ient's pulmonary sym pt oms?
92
:
93
A. Inhibition of folate synthesis
94
95
B. Inhibition of peptidoglycan cross- linking
. 96 C. Inhibition of protein synt hesis by blocking attachment of the aminoacyl-transfer RNA
. 97
D. Inhibition of protein synthesis by blocking t ranslocation
• 98
• 99
E. Inhibition of topoisomerase II and IV
. 100 •
8
Lode.
s
Suspe-nd
8
End Bloc:k
Item: 96 of - ,• Mark -<J [:::> "'I ~ · ~
100 ~ P~v1ous N @xt Labl lues N o tes Calcula to r
74 A A

75 The correct a ns w e r is A. 3 8 °/o chos e this.


76 This patient presents with Pneumocystis jirovecii (PCP) pneumonia in the setting of HIV/AIDS (the
77 combination of multiple sexual partners without using protection on a USMLE-style question should raise
78
your suspicion of HIV). Chest X-rays (CXR) are appropriate in the init ial workup for patients with suspected
PCP. Commonly described findings of PCP pneumonia on CXR include diffuse fine reticular opacities and
79 bilateral upper lobe pneumatoceles. A classic "ground-glass appearance" describes PCP pneumonia on high-
80 resolution CT like that shown here. It is important to note that CXR may be normal in up to 40% of patients
81
with PCP pneumonia. PCP pneumonia is an AIDS -defining illness and is typically seen in patients with CD4
counts <200 cells/ mm 3 . The standard treatment for this illness is trimethoprim -sulfamethoxazole (TMP-
82 SMX), a combination antibiotic that acts synergistically to inhibit folate synthesis by inhibiting dihydrofolate
83 reductase and dihydropteroate synthetase, respectively. If the patient has a sulfa-drug allergy, the
treatment of choice would be pentamidine instead of TMP-SMX. Last, lactate dehydrogenase is a highly
84
sensitive (yet nonspecific) marker of PCP infection. Image copyright © Annals of
85 Penta,... dme Pneu,.-,oc t•5JirO ect Tnmethopnm/sulfamethoxazole Oihydrofolate reductase Lactate dehydrooen•e Pneumoc,sb.. pneumonia Antibiotics
Thoracic Medicine
86 Folic actd C04 HIV PneuMOnta Oth dropteroate synthase HIV/AIOS Chest radiograph Allergy Lacttc actd X·ray AIOS defimng chmcal condttion Infection
4

87
B is not correct . 14°/o chos e this .
88
I mipenem is a member of the carbapenem class of antibiotics and inhibits peptidoglycan cross- linking by binding to penicillin-binding protein.
89 Unlike penicillin, however~ it is resistant to 13-lactamase. It is effective against gram-negat ive organisms and against Pseudomonas aeruginosa.
90 P aeruginosa is a common cause of nosocomia l pneumonia, especially in intubated patient s. It is also a common cause of pneumonia in those
91
with cystic fibrosis. Ce ftazidime, a t hird-generat ion cephalosporin, can also be used fo r antipseudomonal activity.
Cephalosporin Carbapenem Pemctlhn btndtng proteins Imipenem Cystic fibrosis Ceftazidtme Pseudomonas aerugtnosa Peptidoglycan Penicillin Gram-negative bacteria Hospital-acquired pneumonia
92
Pseudomonas Pneumon1a Anbb1obcs Hospital-acquired infection Protein Tracheal intubat1on Fibrosis
93
94
C is not correct . 19% chos e this.
The tetracycl ine antibiotics include doxycycline, demeclocycline, and m inocycline. These drugs act by binding t o t he 30S-ri bosomal subunit
95
and preventing attachment of t he aminoacyl-t ransfer RNA. Tetracyclines can be used to treat infection wit h Mycoplasma pneumoniae but are
96 not indicated for infection with Pneumocystis jirovecii.
. 97 MycopiMma pneumon1ae M1nocychne Doxycycline Tetracycline Pneumocyst1s j1rovecll Tetracycline ant1b10t1CS Oemeclocychne Mycoplasma Anbb1obcs RNA Infection

• 98 0 is not correct . 20% chose this.


• 99 The macrolide antibiotics include clarithromycin, erythromycin, and azithromycin. These drugs act by binding to the 23S-ribosomal RNA of the
. 100 50S-ribosomal subunit, thereby inhibiting protein synthesis by blocking translocation. Macrolides are first-line medications for the treatment of

8
Lode.
s
Suspe-nd
8
End Bloc:k
Item: 96 of ~ ,• Mark <::J [:::> ""I ~· ~'j
100 J.. Previous Next LAb faiUI~S Notes Calculator

74
75 C is not correct. 19% chose this .
76
The tetracycline ant ibiot ics include doxycycline, demeclocycline, and m inocycline. These drugs act by binding to t he 305-ri bosomal subunit
and prevent ing attachment of t he aminoacyl-t ransfer RNA. Tet racyclines can be used to t reat infection wit h Mycoplasma pneumoniae but are
77 not ind icated for infect ion wit h Pneumocystis jirovecii.
78 Mycoplasma pneumoniae Minocycline Doxycycline Tetracycline Pneumocystis jirovecii Tetracycline antibiotics Oemeclocycline Mycoplasma Antibiotics RNA Infection

79
D is not correct. 20% c hose this.
80 The macrolide antibiotics include clarit hromycin, erythromycin, and azit hromycin. These drugs act by binding t o t he 235-ribosomal RNA of t he
81 50S-ri bosomal subunit, t hereby inhibiting protein synt hesis by blocking t ranslocation . Macrolides are f irst-line medications fo r t he t reat ment of
82
pneumonia caused by Streptococcus pneumoniae and Mycoplasma pneumoniae. M. pneumoniae is one of t he most common causes of at ypical
pneumonia.
83 Erythromycin Macrolide Mycoplasma pneumoniae Azithromycin Clarithro,.,cin Streptococcus pneumoniae Atypical pneumonia Pneumonia Mycoplasma Antibiotics Streptococcus Protein biosynthesis Protein
84 Protein synthesis RNA Protein synthesis inhibitor
85
E is not correct. 9°/o chose this .
86
Fluoroqu inolones include moxif loxacin, ciprofloxacin, and levofloxacin. These drugs act by inhibit ing topoisomerase II and IV and are
87 considered bactericidal. Fluoroquinolones can be used for UTis (Eschericia coli, Staphylococcus saprophyticus), respirat ory infect ions
88 (Haemophilus influenzae, Mycoplasma pneumoniae ), bacterial GI infections, as well as ost eomyelit is. Adverse side effects of quinolones
89
include GI irrit at ion, headaches, dizziness, photosensit ivit y, t endinit is, and t endon rupture. Its use in pregnancy and children is relatively
cont raindicated due t o risk of cart ilage damage.
90 Osteomyelitis Moxifloxacin Ciprofloxacin Tendinitis Photosensitivity Type II topoisomerase Quinolone Bactericide Topoisomerase Contraindication Pregnancy Tendon Adverse drug reaction Dizziness
91 Urinary tract infection
92
93
Bottom Line :
94
The yeast-like fungus Pneumocystis jirovecii is t he most common pneumonia -causing infection in HI V pat ients wit h CD4 counts < 200
95
cells/mm 3 , and t rimetho prim -sulfamethoxazole (TMP-SMX) is t he t reat ment of choice for t his opport unistic infection. I t can be dist inguished
96 f rom community-acquired pneumonia, because it typically causes bilateral lung infilt rat es wit h a "ground-g lass" appearance on chest
. 97 radiogra ph . It is important to keep P, jirovecii in your differential diagnosis fo r pneumonia, especially in pat ients wit h risk factors fo r
• 98
HIV/AIDS, as it is often indicat ive of underl ying immunocompromise .
Immunodeficiency Opportunistic infection Pneumocystis jirovecii Trimethoprim/sulfamethoxazole Chest radiograph CD4 Community-acquired pneumonia Pneumocystis pneumonia Radiography Pneumonia HIV
• 99 Differential diagnosis HIV/AIDS Fungus Lung Infection
• 100

8
Lock
s
Suspend
0
End Block
Item: 96 of ~ ,• Mark <::J [:::> ""I ~· ~'j
100 J.. Previous Next LAb faiUI~S Notes Calculator

74
75 FA17 p 173.1
76
Common diseases of As CD4+ cell count ~, risks of reactivation of past infections (eg, TB, HSV, shingles), dissemination
77
HIV-positive adults of bacterial infections and fungal infections (eg, coccidioidomycosis}, and non-Hodgkin
78
lymphomas t.
79
80 PATHOGEN PRESENTATION FINDINGS
81 CD4+ cell count< 500/mm 3
82
Candidaalbicans Oral thrush Scrapable white plaque, pseudohyphae on
83
m1croscopy
84
85 EBV Oral hairy leukoplakia Unscrapable white plaque on lateral tongue
86 Bartonella henselae Bacillary angiomatosis Biopsy with neutroph ilic in Aammation
87 HHV-8 Kaposi sarcoma Biopsy with lymphocytic inflammation
88
HPV Squamous cell carcinoma, commonly of anus
89
(men who have sex with men) or cerYix
90
91
(women)
92 CD4+ cell count< 200/mm 3
93 Histoplasma Fever, weight loss, fatigue, cough, dyspnea, Oval yeast cells within macrophages
94 capsulatum nausea, vom iting, diarrhea
95
HIV Dementia
96
. 97 JC virus (reactivation) Progressive mu ltifoca lleukoencephalopathy onenhancing areas of demyelination on IRl
• 98 Pneumocystis jirovecii Pneumocrstis pneumonia "Ground-glass" opacities on CXR
• 99
CD4+ cell count< 100/ mm 3
• 100 ~

8
Lock
s
Suspend
0
End Block
Item: 96 of - ,• Mark -<J [:::> "'I ~ · ~
100 ~ P~v1ous N @xt Labl lues N o tes Calcula to r
74 A A

75
76
FA17 p 150.1
77 Pneumocystis jirovecii Causes Pneumocrstis pneumonia (PCP), a diffuse interstitial pneumonia fJ. Yeast-like
78 fungus (originally classified as prolo.wan). Inhaled. \lost infections are as) mptomat ic.
79
Immunosuppression (eg, AIDS) predisposes to disease. Diffuse, bilateral ground-glass opacities on
80
C R/CT [l). Diagnosed b) lung biops)' or lavage. Disc-shaped yeast seen on methenamine siher
81
stain of lung tissue 9.
82
Treatment/prophylaxis: T.VI P-S~ IX, pentamidine, dapsone (prophylaxis only), ato\'aquonc. Start
83
prophylaxis when CD4+ count drops to< 200 cells/mm3 in HIV patients.
84
85
86
87
88
89
90
91
92
93
94
95
96
. 97
FA17 p 190.3
• 98
Trimethoprim
• 99
. 100 MECHANISM Inhibits bacterial dihrdrofolate reductase.
8
Lode.
s
Suspe-nd
8
End Bloc:k
81
82
83 FA17 p 190.3
84 Trimethoprim
85 MECHANISM Inhibits bacterial dihrdrofolate reductase.
86 Bacteriostatic.
87
CLINICAL USE Used in combination with sulfonamidcs
88
89
(trimethoprim-sulfamethoxazole [TMP-
90
S IX]), causing sequential block of folate
91
synthesis. Combination used for UTls,
92 Shigella, Salmonella, P11eumocystis jirovecii
93 pneumonia treatment and prophylaxis,
94 toxoplasmosis prophylaxis.
95 ADVERSE EFFECTS Megaloblastic anemia, leukopenia,
96 granulocytopenia. (May alb·iatc with
. 97
supplemental folinic acid). T~ IP '1reats
' 98
!\Iarrow Poork
' 99 '
' 100 •

8
L.odt
s
Su~pl'nd
~
End Block
Item: 97 of - ,• Mark -<J [:::> "'I ~ · ~
100 ~ P~v1ous N @xt Labl lues N o tes Calcula to r
74
A 32-year-old janitor presents wit h a disease of infect ious origin. He reports feeling progressively worse over t he course of the last month ~~AI
75
or so, but delayed seeking medical care because he has been working a second shift in anot her school district. He is a single parent of two
76
children. When he does have t ime off, he either spends t ime in his garden or he takes his kids to a local park where t hey like t o feed the
77 pigeons and ducks. The organism responsible for his symptoms is revea led to be a yeast with a thick gelat inous capsule.
78
79 What clinical findings would be most likely in this patient?
80 :
81 A. Diffuse abdominal pain with rebound tenderness
82 B. Dull bone pain that worsens on movement
83
C. Fever, fatigue, cough, and pleuritic chest pain
84
85 D. Ulcerative cutaneous papules
86 E. Urinary frequency and urgency
87
88
89
90
91
92
93
94
95
96
. 97
• 98
• 99
. 100 •
8
Lode.
s
Suspe-nd
8
End Bloc:k
Item: 97 of ~ ,• Mark <::J [:::> ""I ~· ~'j
100 J.. Previous Next LAb faiUI~S Notes Calculator

74
75 The correct a nswer is C. 77°/o chose this .
76 The organism responsible fo r the patient's symptoms is Clyptococcus neoformans. Board review guides often em phasize that t his species is a
causat ive agent of fungal meningit is in immunocompromised patients (with Blastomyces, Coccidioides, and Histoplasma as causes of fungal
77
pneumonia) . St udents should not e, however, t hat these are oversimplif icat ions. All t he syst emic mycoses can cause infection in a nu mber of
78 sit es. Although meningoencephalit is is t he most common clinical manifestation of cryptococcal disease, it is not t he only one. I n fact, the
79 init ial site of inoculat ion wit h Clyptococcus is the lung; meningit is can ensue only after t he yeasts have disseminat ed from this site. Pat ients
present more oft en wit h crypt ococcal meningitis, in large pa1t because many lung infections are asymptomat ic. When an individual does
80
experience symptoms from cryptococcal pneumonia (t he second most common clinical manifestation), t he symptoms can include feve r,
81 fatigue, cough, and pleurit ic chest pain. Ot her less common find ings may include hemoptysis, auscultatory rales, or a pleura l rub.
82 Hemoptysis Cryptococcus neoformans Cryptococcosis Immunodeficiency Mycosis Meningitis Crackles Pneumonia Pleurisy Histoplasma Fungus Asymptomatic Cryptococcus Blastomyces dermatitidis Cough

83 Fever Organism Meningoencephalitis Lung Pleural cavity Coccidioides Chest pain Infection Yeast

84 A is not correct . 4 °/o chose this.


85 Perit onit is is a rare manifestation of the disease. The most f requent manifestation of cryptococcosis is meningoenceph alitis, and cryptococcal
86 infections are more often seen in immunocompromised patients. I n this pat ient 's case, good quest ions to ask him would be about HIV status
and lifet ime sexual part ners, as well as surgical hist ory (organ transplants) and use of glucocorticoids. However, keep in m ind t hat
87
immunocompet ent people can st ill develop cryptococcal infect ions, so a lack of immunocompromised history does not rule out crypt ococcal
88 infection .
Cryptococcosis Peritonitis Meningoencephalitis Immunodeficiency HIV Glucocorticoid Cryptococcus neoformans Immunocompetence
89
90 B is not correct. 5% chose t his .
91 These symptoms are indicat ive of bony involvement. Ost eolyt ic bone lesions are anot her less common manifest ation of disseminated
92 cryptococcosis. Clinical present at ion can vary (acut e or chronic, and generally nonspecific symptoms first ); however, it is very unlikely that a
previously healt hy patient wit h no hist ory of immunocompromised st atus would present init ially wit h bony involvement.
93 Cryptococcosis Immunodeficiency Nonspecific symptoms Bone Osteolysis
94
D is not correct. 11% c hose this.
95
Cutaneous lesions appear in 10%- 15% of cases of disseminat ed cryptococcosis. They are often seen as a sign of part icularl y severe disease.
96 Isolat ed cry ptococcal skin lesions without disseminated infection are rare in immunocompet ent individuals, but can be seen in pat ients wit h
97 HIV.
Cryptococcosis Immunocompetence Cryptococcus Cryptococcus neoformans
• 98
• 99 E is not correct. 3°/o chose this .
• 100 ~
These are symptoms of prost at it is. Cryptococcal prost at it is is occasionally seen, albeit less frequently, in cases of disseminated disease .

8
Lock
s
Suspend
0
End Block
Item: 97 of ~ ,• Mark <::J [:::> ""I ~· ~'j
100 J.. Previous Next faiUI~S
LAb Notes Calculator

74
75 E is not correct. 3°/o chose this .
76
These are symptoms of prost at it is. Cryptococca l prost at it is is occasiona lly seen, albeit less frequently, in cases of disseminated disease.
However, in the case of this patient , it is very unlikely that he would first present with symptoms of prost atit is. Cry ptococcal infections often
77 man ifest init ia lly with subacut e and nonspecific symptoms. An infection with Cryptococcus should be suspect ed in pat ients who present wit h
78 fever, headache, and other signs or sym ptoms linked t o t he cent ral nervous syst em, part icula lrly patient s with a known immunocompromised
79
state (although immunocompetent patient s can also develop cryptococcal diseases) .
Central nervous system Prostatitis Immunodeficiency Immunocompetence Headache Cryptococcus neoformans Nervous system Fever
80
81
82 Bottom Line :
83 Cryptococcus neoformans, visualized via I nd ia ink stain and ca rried by pigeons, most commonly causes meningoencepha lit is, fo llowed by
84
cry ptococcal pneumonia.
Cryptococcus neoformans India ink India Pneumonia Cryptococcus
85
86
87
l@l;fil·1i•l for yea r:[2017
FIRST AID FACTS .
•j .
88
89
90 FA17 p 149.1
91 Opportunistic fungal infections
92 Candida albicans alba = white. Dimorphic; Forms pseudohyphae and budding yeasts at 20°C r.J, germ tubes at
93 37°C rn.
94
Systemic or superficia I fungaI in Feel ion. Causes ora I [!I and esophageal thrush in
95
immunocompromised (neonates, steroids, diabetes, AIDS), vulvovaginitis (diabetes, use of
96
antibiotics), diaper rash, endocarditis (IV drug users), disseminated candidiasis (especially in
97
neutropenic patients), chronic mucocutaneous candidiasis .
• 98
Treatment: oral Auconazolc/topical azoic for vaginal; nystatin, flu conazole, or caspofungin for oral/
• 99
• 100
esophageal; fluconazole, caspofungin, or amphotericin B for systemic.

8
Lock
s
Suspend
0
End Block
Item: 97 of - ,• Mark --<) [::> ""'I ~· 1!';:'1
100 ~ Prev1o u s Next Labf a lu es Notes Calculator
74
75 Aspergillus Septate hyphae that branch at 45° Acute Angle (!]. Produces conidia in radiating chains at end of
76 fumigatus conidiophore [l
77 Causes invasive aspergillosis in immunocompromised, patients with chronic granulomatous disease.
78 Can cause aspergillomas in pre-existing lung ca' ities, especially after TB infection.
79 Some species of Aspergillus produce \ fl ato,ins {associated with hepatocellular carcinoma).
80
Allergic bronchopulmonary aspergillosis { BP ): hypersensitivity response associated with
81
82
asthma and cystic fibrosis; ma} cause bronchiectasis and eosinophilia.
83 Cryptococcus 5-lO jlm with narrow budding. Ilea' ily encapsulated )Cast. ~ot dimorphic.
84 neoformans Found in soil, pigeon droppings. Acquired through inhalation with hematogenous dissemination
85 to meninges. Culture on Sabouraud agar. II ighlighted "ith India ink (clear halo 0 ) and
86 mucicarmine (red inner capsule ). Latex agglutination test detects polysaccharide capsular
87 antigen and is more specific.
88 Causes cryptococcosis, cryptococcalmcningitis, cryptococcal encephalitis ("soap bubble" lesions
89 in brain}, primarily in immunocompromised.
90 Treatment: amphotericin B + flucytosi ne followed by fluconazole for cryptococcalmeningitis.
91
92
Mucor and Rhizopus Irregular, broad, nonseptate hyphae bra nching at wide angles f!l.
93
spp. lucormycosis. Causes disease mostly in ketoacidotic diabetic and/or neutropenic patients (cg,
94 leukemia). Fungi proliferate in blood vessel walls, penetrate cribriform plate, and enter bra in.
95 Rhinocerebral, fronta l lobe abscess; cavernous sinus thrombosis. llcadache, faciaJ pain, black
96 necrotic eschar on fa ce; may have cranial nerve invokcmcnt.
97 Treatment: surgical debridement, amphotericin B.
• 98
• 99
• 100 •
8
L.odt
s
Su~pl'nd
~
End Block
Item: 97 of - ,• Mark --<) [::> ""'I ~· 1!';:'1
100 ~ Prev1o u s Next Labf a lu es Notes Calculator
74
75 FA17 p 122.1
76 Stains
77
Gram stain First-line lab test in bacterial identification. Bacteria with thick peptidoglycan layer retain crystal
78
violet dye (gram<±>); bacteria with thin peptidoglycan layer turn red or pink (gram 8 ) with
79
80
counterstain.
81
These bugs do not Gram stain well ('I hese Little \l icrobes ~lay Unfortunately Lack Real Color
82 But Are Everp\ here).
83 Treponema. Leptospira Too thin to be visualized.
84
.\tycobacteria Cell wall has high lipid content.
85
86 :\1ycoplasma, Ureaplasma No cell wall.
87 Legionella, Rickettsia, Chlamydia, Bartonella, Primarily intracellular; also, Chlamydia lack
88 Anaplasma, Ehrlichia classic peptidoglycan because of l muramic
89 acid.
90
Giemsa stain Chlamydia, Borrelia, Rickettsia, Certain Bugs Really Try my Patience.
91
92
Trypanosomes fl. Plasmodium
93 Periodic acid- Schiff Stains glycogen, mucopolysaccha rides; used PaSs the sugar.
94 stain to diagnose Whipple disease (Tropherynw
95 whipplei I}])
96
Ziehi-Neelsen stain Acid-fast bacteria (cg, J\lrcobacteria Current standard of care is auram ine-
97
(carbolfuchsin) Nocardia; stains mycolic acid in cell wall); rhodamine stain for screening (ine:-.:pensi\·e,
• 98
protozoa (eg, Cryptosporidium oocysts) more sensitive but less specific).
• 99
• 100 India ink stain Cryptococcus neoformans • ; mucicarmine •
8
L.odt
s
Su~pl'nd
~
End Block
Item: 97 of - ,• Mark --<) [::> ""'I ~· 1!';:'1
100 ~ Prev1o u s Next Labf a lu es Notes Calculator
74
acid.
75
76
Giemsa stain Chlamydia, Borrelia, Rickettsia, Certain Bugs Really Try my Patience.
77
Trypanosomes rJ, Plasmodium
78 Periodic acid- Schiff Stains glycogen, mucopolysaccharides; used PaSs the sugar.
79 stain to diagnose Whipple disease (Tropher)•llw
80 whipplei III)
81
Ziehi-Neelsen stain cid-fast bacteria (eg, Mycobacteria Current standard of care is auraminc-
82
(carbol fuchsin) 'ocardia; stains mycolic acid in cell wall); rhodamine stain for screening (inexpensi,·e,
83
protozoa (eg, Crrptosporidium oocysts) more sensitive but less specific).
84
85 India ink stain Cryptococcus neofomwns • ; mucicarmine
86 can also be used to stain thick pol) saccharide
87 capsule red
88
Silver stain Fungi (eg, Coccidioides 11 P11eumocystis
89
jirovecii), Legionel/a, Helicobacter pylori
90
91
Fluorescent antibody Used to identify ma ny bacteria and viruses. Example is F'TA-ABS for syphilis.
92 stain
93
94
95
96
97
• 98
• 99
• 100 •
8
L.odt
s
Su~pl'nd
~
End Block
Item: 98 of - ,• Mark -<J [:::> "'I ~· ~
100 ~ P~v1ous N @xt Labl lues N o tes Calcula to r
74
A 2-week-old boy is brought to t he pediatrician because of a puru lent, yellow discharge oozing from his eyes (similar to t hat shown in the ~~AI
75
image). He was born in Cent ral America and his mother does not know if he received eye drops fo llowing delivery. On physical examination
76
both eyelids are swollen and the conjunct ivae appear inflamed. Microscopically, conjunctival scrapings reveal basophilic int racytop lasmic
77 inclusion bodies.
78
79
80
81
82
83
84
85
86
87
88
89
90
91
Which of the fol lowing organisms is most likely responsible for t his infant's conj unctivitis?
92
:
93
A. Chlamydia trachomatis
94
95
B. Chlamydophila pneumoniae
96 C. Chlamydophila psittaci
97
D. Neisseria gonorrhoeae
. 98
• 99
E. Ureaplasma urealyticum
. 100 •
8
Lode.
s
Suspe-nd
8
End Bloc:k
Item: 98 of ~ ,• Mark <::J [:::> ""I ~· ~'j
100 J.. Previous Next LAbfaiUI~S Notes Calculator

74
75
76
The correct ans wer is A. 77°/o chose this.
Swollen eyelids, conjunct ival inflammat ion, and yellow, purulent discharge (sim ilar to that I Type of ConjunctiVitis Timing of Discharge Quality of Discharge
77
in the v ignette image) are symptoms of conjunct ivit is. Chlamydia trachomatis infect ion- Chemical conjunctivffis
Chlamydia conjunctivitis
Wtth1n th~ first 24 hours
A few days 2 weeks
Clear
Copiousamounts of d ear
78 t he most common sexually t ransmit ted disease in the Un it ed Stat es- can cause discharge. Can also
man,tes.l w1th pneumoma.
79 conjunct ivitis in newborns who are colonized while passing through an infected birth cana l. Gonococcal t-2 days after Oelr-tery Purulenl
It t ends t o develop a few days t o 2 weeks after bi1th . Basophilic intracytoplasmic inclusions contunctlvltls
80
are found within epithelial cells from the conj unctival surface and reflect the bact eri a in its
81 reticulat e/init ial body phase. Serotypes D through K can lead to urogen it al infect ions as well as conjunct ivit is in t he newborn . There is no
82 prophylaxis for ch lamydia! conjunct iv it is, t he prophylatic drops given in this quest ion are for t reating Neisseria gonorrhoeae infect ion.
Sexually transmitted infection Chlamydia trachomatis Neisseria gonorrhoeae Conjunctivitis Chlamydia infection Pus Vagina Conjunctiva Epithelium Inflammation Neisseria Bacteria Genitourinary system
83
Pelvis Preventive healthcare Serotype
84
85 B is not corre ct. 4 % chose this.
86 Chlamydophila pneumoniae is another cause of atypical pneumonia worldwide, but not a common cause of ocular infections. It is t ransm itted
person-to -person .
87 Atypical pneumonia Pneumonia Chlamydophila
88
C is not correct. 4 °/o chose this .
89
Chlamydophila psittaci causes psittacosis. I nfected ind ividua ls experi ence symptoms sim ilar t o atypical pneumonia, but not a common cause
90 of ocu lar infect ions. The organism is spread through poult ry and bird contact.
91 Psittacosis Pneumonia Atypical pneumonia Chlamydophila

92 D is not correct. 14% chose this .


93 Neisseria gonorrhoeae infection causes gonorrhea and is anot her cause of neonatal conjunct ivit is acquired during birt h. However, it ty pica lly
94 appears 1- 2 days after delivery and would not be associat ed with cytoplasmic inclusions. Erythromycin eye drops adm inistered at birth also
95 protect against Neisseria conj unctivit is. Ot her prophylact ic t reatments include silver nit rate 1% aqueous solution or tetracycline 1%
opht halm ic ointment.
96 Erythromycin Silver nitrate Gonorrhea Tetracycline Neisseria gonorrhoeae Conjunctivitis Neonatal conjunctivitis Neisseria Nitrate Infant Preventive healthcare Infection Eye drop Aqueous solution
97
E is not correct. 1 °/o chos e this.
98
Ureap/asma urealyticumcan cause nongonococcal uret hrit is. This organism converts urea into ammon ia and carbon dioxide .
• 99 Ureaplasma urealyticum Ammonia Carbon dioxide Urea Non-gonococcal urethritis Ureaplasma Urethritis
• 100 ~

8
Lock Suspend
s 0
End Block
Item: 98 of ~ ,• Mark <::J [:::> ""I ~· ~'j
100 J.. Previous Next LAbfaiUI~S Notes Calculator

74
75 Bottom Line:
76 I n t he Unit ed St at es, neonat es receive prophylactic erythromycin (macrolide) eyed rops to prevent Neisseria gonorrhoeae infections. It is
77 important to note t hat there is no prophylactic t reat ment for Chlamydia trachomatis conjunct ivit is. When assessing conj unctivit is in the
78
neonate, it is important to look at t he t ime table of when signs and symptoms began. For the USM LE, you must know t he t iming and qualit y
of t he discharge to determine t he cause if t he infection.
79 Erythromycin Macrolide Chlamydia trachomatis Neisseria gonorrhoeae Conjunctivitis Chlamydia infection Chlamydia (genus) Infant Preventive healthcare Neisseria Eye drop Infection United States
80
81
82 l@l;fil·1i•l for year:[ 2017
FIRST AID FACTS .
•j .
83
84
FA17 p 146.1
85
86 Chlamydiae Chlamydiae cannot make their own ATP. They Chlamys = cloak (intracellular).
87 are obligate inlracellular organ isms that cause C fJsittaci- has an avian reservoir (parrols),
88 mucosal infections. 2 forms: causes atypical pneumon ia.
89 Elementary body (small, dense) Lab diagnosis: PCR, nucleic acid amplification
90 is "E nfectious" and Enters cell via test. Cytoplasmic inclusions (reticulate bodies)
91 Endocytosis; transforms into reticulate body. seen on Giemsa or Auorescent antibody-
92 • Reticulate body Replicates in cell by fission; stained smear.
93
Reorganizes into elementary bodies. The chlamydia] cell wallh1cks classic
94
Chlamydia trachomatis causes reacti,·e arthritis peptidoglycan (due to reduced muramic acid),
95
(Reiter syndrome}, folli cular conjunctivitis r.J, rendering P-lactam antibiotics ineffective.
96
nongonococcal urethritis, and PTD.
97
Chlamydophi/a fmeumoniae and Chlamydophila
98
• 99
f;sittaci cause atypical pneumon ia; transmitted
. 100 by aerosol.

8
Lock
s
Suspend
0
End Block
Endocytosis; transforms into reticulate body. seen on Ciemsa or Auorescent antibody-
75
• Reticulate body Replicates in cell by fission; stained smear.
76
Reorganizes into elementary bodies. The chlamydia! cell wall lacks classic
77
78
Chlamydia trachomatis causes reactive arthritis peptidoglycan (due to reduced muramic acid),
79
(Reiter syndrome), follicular conjunctivitis fJ, rendering P-lactam antibiotics ineffective.
80
nongonococcal urethritis, and PID.
81 Chlamydophila pneumoniCie and Ch/amydophila
82 psittaci cause atypical pneumonia; transmitted
83 by aerosol.
84 Treatment: azithromycin (favored because one-
85 time treatment) or doxycycline (+ ceftria\one
86 for possible concomitant gonorrhea).
87
88
FA17 p 146.2
89
Chlamydia trachomatis serotypes
90
91 Types A, 8, and C Chronic infection, cause bli ndness due to ABC = Africa, Blindness, Chron ic infection.
92 follicular conjunctivitis in Africa.
93 Types D- K Urethritis/PID, ectopic pregnancy, neonatal 0-K = everything else.
94
pneu monia (staccato cough) with eosinophi lia, eonatal disease can be acquired during
95
neonatal conjunctivitis (1-2 weeks after birth). passage through infected birth canal.
96
97
Types l 1, l 2, and l 3 Lymphogranuloma venereum-small, painless
98 ulcers on genitals - swollen, painful inguinal
• 99 lymph nodes that ulcerate (buboes). Treat with
. 100 • doxycrcline.
8
Lode.
s
Suspe-nd
8
End Bloc:k
Item: 99 of - ,• Mark -<J [:::> "'I ~ · ~
100 ~ P~v1ous N @xt Labl lues N o tes Calcula to r
74
An outbreak of severe pneumonia and high feve r occurs among seven elderly smokers in a nursing home. Laboratory f indings from one of ~~AI
75
the affected individuals are as fol lows:
76
77 WBC count: 22,000/mm 3
78 Hematocrit: 42%
Hemoglobin: 14.2 g/dL
79 Platelet count: 100,000/mm 3
80 Sodium: 125 mEq/L
81 Potassium: 4.0 mEq/L
Chloride: 99 mEq/L
82
Bicarbonate: 28 mEq/L
83 Magnesium: 1.5 mEq/L
84 Phosphate: 1.8 mg/dL
Blood urea nitrogen: 20 mg/dL
85
Creatinine: 1. 7 mg/dL
86
87 The typical sputum Gram stain shows large neutrophils but no organisms.
88
89
What is the next step to aid in the diagnosis?
90 :

91
A . Culture the organism on buffered charcoal yeast ext ract agar
92 B. Culture the organism on Lowenst ein-Jensen agar
93
C. Stain with Congo red and then place under polarized light
94
D. Stain with India ink
95
96 E. Use Ziehi-Neelsen stain
97
98
. 99
. 100 •
8
Lode.
s
Suspe-nd
8
End Bloc:k
Item: 99 of ~ ,• Mark <::J [:::> ""I ~· ~'j
100 J.. Previous Next LAbfaiUI~S Notes Calculator

74
75
76 The correct ans wer is A. 59°/o chose this.
77
The abnormal labs in this patient (throm bocytopenia, elevated creat inine and BUN, hyponat rem ia, hypophosphatemia) are classic for
Legionel/a pneumonia. Ant ibodies aga inst Legionel/a can cross- react wit h host ADAMTS13, the enzyme responsible fo r von Willebrand factor
78 deg radat ion. Wit h reduced act ivit y of ADAMTS13, more clots are able to fo rm and t hrombot ic t hrombocytopenia pupura may develop.
79 I ncreased int ravascular hemolysis leads t o elevat ed BUN and creat inine. Kidney function may also be comprom ised, reducing t he ability to
80 reabsorb sod ium and phosphat e. The single most important test fo r diagnosing legionnaire's disease is isolation of t he organisms using a
buffered charcoal yeast extract agar, which contains L-cyst eine, iron, vancomycin, and dyes t o prevent t he overgrowth of compet ing
81 organ isms and t o sta in t he organ ism of int erest.
82 Von Willebrand factor Hyponatremia Vancomycin Thrombocytopenia Legionella Hemolysis Legionnaires' disease Enzyme Pneumonia Creatinine Yeast Antibody Kidney Cysteine Phosphate

83 Blood urea nitrogen Sodium Hemolytic anemia Renal function ADAMTS13

84 B is not correct. 14% chose this .


85 Lowenstein-Jensen agar is used to cult ure Mycobacterium tuberculosis, an acid-fast organ ism t hat causes t uberculosis. Symptoms of
86 t uberculosis include recu rre nt feve rs, night sweats, weight loss, chronic cough, and hemoptysis.
Hemoptysis Mycobacterium tuberculosis Tuberculosis Acid-fast Mycobacterium Cough Night sweats Weight loss Agar Organism
87
88 C is not correct. 7 °/o chos e this .
89 Congo red stains amyloid bu t m ust be visua lized wit h polarized light. If amyloid is present , t his kind of stain produces apple-green bi-
refri ngence under polarized light. Amyloid is not a cause of pneumonia.
90
Amyloid Congo red
91
92 D is not correct. 8°/o chos e this.
I ndia ink is used to stain Clyptococcus neoformans, a fu ngus t hat causes opportunistic infect ions, such as meningoencephalitis. Diagnosis is
93
made by pe1form ing a lumbar puncture and analyzing t he cerebrospinal fluid v ia an I ndia ink sta in. The st ain will reveal t he yeast cells wit h a
94 surround ing halo, which represents t he polysaccharide capsule. This t est is posit ive in only SO% of cases; a more sensit ive test is t he
95 cryptococcal ant igen test.
Polysaccharide Cryptococcus neoformans Lumbar puncture Cerebrospinal fluid India ink India Meningoencephalitis Antigen Opportunistic infection Cryptococcus Fungus Yeast EUSA
96
97 E is not correct. 12°/o chos e this.
98 Ziehi-Neelsen stain is used to st ain acid-fast bacteria such as Mycobacterium tuberculosis. This clinical scena rio is not typical of pneumonia
caused by M. tuberculosis .
99
Ziehi-Neelsen stain Mycobacterium tuberculosis Tuberculosis Pneumonia Acid-fast Mycobacterium Bacteria
• 100 ~

8
Lock
s
Suspend
0
End Block
Item: 99 of ~ ,• Mark <::J [:::> ""I ~· ~'j
100 J.. Previous Next LAbfaiUI~S Notes Calculator

74
75
Bottom Line:
76
When Legionel/a pneumophila infect ion is suspect ed, t he appropriate culture is bu ffered charcoal yeast extract agar wit h iron and cysteine.
77 Typical gram st ain shows large neut rophils but no organisms.
78 Legionella pneumophila Legionella Buffered charcoal yeast extract agar Cysteine Yeast extract Gram staining Neutrophil Yeast Agar I nfection

79
80
81 i@l;fil·1i•J for year:[ 2017 • J
FIRST AID FACTS
82
83
FA17 p 123.1
84
Special culture requirements
85
BUG MEDIA USED FOR ISOLATION MEDIACONTENTS/OTHER
86
87
H influenzae Chocolate agar Factors V (1 AD+) and X (hematin)
88 N gonorrhoeae, Thayer-Martin agar Selectively favors growth of eisseria by
89 N meningitidis in hibiting growth of gram Etl organisms
90 with Vancomycin, gram 8 organisms except
91 Neisseria with Trimethoprim and Colistin,
92 and fungi with ;\'ystatin
93
Very Typically C ultures Neisseria
94
95
Bpertussis Bordet-Cengou agar (Bordet for Bordetella) Potato extract
96
Regan-Lowe medium C harcoal, blood, and antibiotic
97 Cdiphtheriae Tellurite agar, LofAer medium
98
M tuberculosis Lowenstein-Jensen agar
99
. 100 M pneumoniae Eaton agar Requires cholesterol
8
Lock
s
Suspend
0
End Block
Item: 99 of - ,• Mark -<J [:::> "'I ~ · ~
100 ~ P~v1ous N @xt Labl lues N o tes Calcula to r
74 A A
M pneumoniae Eaton agar Requires cholesterol
75
76 Lactose-fermenting MacConker agar Fermentation produces acid, causing colonies to
77 enterics turn pink
78 Ecoli Eosin-methylene blue (EMB) agar Colonies with green metallic sheen
79
Legionel/a Charcoal yeast extract agar buffered "ith
80
81
C)Steine and iron
82 Fungi Sabouraud agar "Sab's a fun guy!"
83
84
FA17 p 139.2
85
86 Legionella Gram 8 rod. Gram stains poorly-use sih er Think of a French legionnaire (soldier) with
87 pneumophila stain. Grow on ch<ucoal yeast extract his )ilvcr helmet, sitting around a campfire
88 medium with iron and cysteine. Detected by (charcoal) with his iron dagger- he is no sissy
89 presence of antigen in urine. Labs may show (cysteine).
90 hyponatremia. Aerosol transmission from
91 cm·ironmental water source habitat (cg, air
92
conditioning systems, hot water tanks). No
93
person-to-person transm ission.
94
Treatment: macrolide or quinolone.
95
Legionnaires' disease-severe pneumonia
96
(often unilateral and lobar ), fever, Gl and
97
98
C 'S symptoms. Common in smokers and in
99
chronic lung disease.
. 100 Pontiac fever-mild Au-likeS) ndromc.
8
Lode.
s
Suspe-nd
8
End Bloc:k
Item: 99 of - ,• Mark -<] 1:> ""'I ~· 1!';:'1
100 ~ Prev1o u s Next Labf a lu es Note s Calculator
74
FA17 p 405.1
75
76 Platelet disorders Defects in platelet plug formation - t bleeding time (BT).
77 Platelet abnormalities - microhemorrhagc: mucous membrane bleeding, epista-.:is, pelechi<le,
78 purpura, t bleeding time, possibly decreased platelet count (PC).
79
80
DISORDER PC BT MECHANISMAND COMMENTS

81 Bernard-Soulier -ll f Defect in platelet plug formation. Large platelets.


82 syndrome l Cplb - defect in platelet-to-\ WF adhesion.
83 Glanzmann f Defect in platelet integrin a 11 ~ (Cpll b/l lla) - defect in platelet-to-platelet
84
thrombasthenia aggregation, and lherefore platelet plug formation.
85
Labs: blood smear sho"s no platelet clumping.
86
87
Hemolytic-uremic f Characterized b) thrombocytopenia, microangiopathic hemolytic anemia, and
88 syndrome acute renal failure.
89 Typica l I IUS is seen in children, accompanied by diarrhea and commonly
90 caused br enterohcmorrhagic E coli (EHEC) (eg, Ol57:H7). HUS in adults
91 does not present with diarrhea; F:HEC infection not required.
92 Same spectrum as TTP, with a similar clinica l presentation and same initial
93 l reatmenl of plasmapheresis.
94
Immune f Anti-Cpllb/l ll a antibodies - splenic macrophage consumption of
95
thrombocytopenia platelet-antibodr complex. ~ l ay be 1° (idiopathic) or 2° to autoimmune
96
disorder, viral illness, ma lignancy, or drug reaction.
97
Labs: f megakaryocytes on bone marrow biopsy.
98
99
Treatment: steroids, I IC, splenectomy (for refractory ITP).
• 100 Thrombotic f Inhibition or deficicnc' of AD.\ YIT 13 (,o\VF metalloprotease) •
8
L.odt
s
Su~pl'nd
~
End Block
Item: 99 of - ,• Mark -<] 1:> ""'I ~· 1!';:'1
100 ~ Prev1o u s Next Labf a lu es Note s Calculator
74
75 Glanzmann f Defect in platelet integrin a 11 J31 (Cpllb/llla) - defect in platelet-to-platelet
76 thrombasthenia aggregation, and therefore platelet plug formation.
77 Labs: blood smear shows no platelet clumping.
78
Hemolytic-uremic f Characterized b) thrombocytopenia, microangiopathic hemolytic anemia, and
79
syndrome acute renal failure.
80
Typical I IUS is seen in children, accompanied by diarrhea and common I)
81
caused by enterohemorrhagic E coli (EHEC) (eg, 01 )7:H7). HUS in adults
82
does not present with diarrhea; EHEC infection not required.
83
84
Same spectrum as TTP, with a similar clinical presentation and same initial
85
treatment of plasmapheresis.
86 Immune f Anti-Gpllb/l lla antibodies - splenic macrophage consumption of
87 thrombocytopenia platelet-antibod) complex. ~ l ay be )0 (idiopathic) or 2° to autoimmune
88 disorder, ,·ira I illness, malignanC)', or drug reaction.
89 Labs: f mega karyocyles on bone marrow biopsy.
90 Treatment: steroids, IVIC, splenectomy (for refractory ITP).
91
92
Thrombotic f Inhibition or deficiency of ADA ITS l3 (vWF metalloprotease)
93
thrombocytopenic - l dcgrade~ tion of vWI" multimcrs.
94 purpura Pathogenesis: f large vWF multimcrs - f platelet adhesion - f platelet
95 aggregation and thrombosis.
96 Labs: schistocytes, f LDI I, normal coagulation parameters.
97 Symptoms: pentad of neurologic and renal symptoms, feve r, thrombocytopenia,
98 and microangiopathic hemol) tic anemia.
99 Treatment: plasmapheresis, steroids.
• 100 •
8
L.odt
s
Su~pl'nd
~
End Block
Item: 100 of - ,• Mark -<J [:::> "'I ~ · ~
100 ~ P~v1ous N @xt Labl lues N o t es Calcula to r
74
75 A you ng woman presents to t he doct or's office with a rash on her t run k, as shown in the image. Several days ago she had a sore t hroat
accompanied by a fever of 38. go C ( 102° F). On physical examinat ion t he tonsils are eryth ematous, t here are small petechiae and
76
punctuate red macules on th e hard and soft palat es, and her tongue appears bright red . There are eryt hemat ous pa tches on t he t runk, in
77 the axillae, and on the ear lobes.
78
79
80
81
82
83
84
85
86
87
88
89
90
91
92
93
94
95
96
97
98
Which of the fol lowing is the most likely diagnosis?
99 :

. 100
A. Erythema multiforme

8
Lode.
s
Suspe-nd
8
End Bloc:k
75
76
77
78
79
80
81
82
83
84
85
86
87
88
89
90
91
92 Which of the fol lowing is the most likely diagnosis?
93 :
94 A . Erythema mult ifo rm e
95 B. Rheumatic fever
96
C. Scalded skin syndrome
97
98 D. Scarlet fever
99 E. Toxic shock syndrome
• 100 •
8
L.odt
s
Su~pl'nd
~
End Block
Item: 100 of - ,• Mark -<J [:::> "'I ~ · ~
100 ~ P~v1ous N @xt Labl lues N o t es Calcula to r
74 A A

75
The correct answ er is D. 64°/o chose this.
76
Scarlet fever (aka, scarlatina) is caused by Streptococcus pyogenes (GAS) strains producing erythrog enic
toxins. These toxi ns stim ulate lymphocyte blastogenesis, pot ent iat e endotoxi n-induced shock, and induce
77 fever. Some strains produce pyrogenic exot oxins that are superantigens, stimulating a large portion of T
78 lymphocytes, leading t o an increased production of cyt okines and an inflammato1y response. The rash of
79
scarlet fever is made up of many 1- 2 mm papular lesions, making it rough in texture. Papules are
erythematous, but blanch wit h pressure. Lesions first appear on t he neck, around the ears, or in the axilla,
80 and then spread to the trunk. The rash spares t he fa ce, but facial flushing with circumoral pallor, st rawberry
81 tongue (shown in this image), and petechiae on the soft and hard palates are all features of the disease.
Strep•ococc I r f() lellle5 Sc.-~et fever A- ja Petechia llfiTIPhOcyte Super-antigen Feo.e- Streptococc OS E ~ef"!"'" Cyto •e T cet Inflammation Exotoxin Rash
82
&Jddong Papule Strawberry tongue Tooon
83
84
85
86
87
88
89 Photo courtesy of Martin
90 Kronawitter, Kellberg
91
A is not correct . 11 °/o chose this.
92
Eryt hema mult ifo rme is a hypersensitivity reaction t hat can be t riggered by a variety of st imuli, including bacteria, viruses, and chemical
93 products. The rash of erythema multiforme is rapidly progressive and consists of symm et rically dist ri but ed cutaneous and/or mucocutaneous
94 lesions. Classically, t he lesions are ta rget shaped, t hough t here is vari at ion in th eir appearance.
Erythema mult1forme Erythema Hypersens1bv1ty Mucocutaneous zone Bacteria Virus Rash
95
96 B is not correct. 1 5°/o chose this.
97 Rheumatic fever can fo llow an unt reated group A Streptococcus (GAS) pha1yngitis infection and manifests wit h fever, myocarditis, arthritis,
98
chorea, subcutaneous nodules, and eryt hema marginatum (an erythematous, macular or papular rash on the trunk and extremit ies that does
not hurt or itch.) Rheumat aic fever is caused by antibody cross-reactivity called type 2 hypersensitivity (also known as molecular m imicry).
99 Infection with GAS leads to production of antibodies against the bacterial cell wall. However, t he antibodies may also be reactive against the
100 host's myocardium, joints, and ot her connecti ve tissues, resulting in the symptoms of rheumatic fever.

8
Lode.
s
Suspe-nd
8
End Bloc:k
Item: 100 of ~ ,• Mark <::J [:::> ""I ~· ~'j
100 J.. Previous Next LAb faiUI~S Notes Calculator

74
C is not correct. 6 °/o chose this .
75
Scalded skin synd rome is caused by a strain of Staphylococcus aureus t hat produces exfoliat in toxin. It prima ri ly affects neonates and young
76 children, and st arts as a local infect ion before causing diffuse erythema, followed by widespread desquamation.
77 Exfoliatin Desquamation Staphylococcus aureus Staphylococcus Erythema Staphylococcal scalded skin syndrome Toxin Infant

78 E is not correct. 4 °/o chos e this.


79 Toxic shock syndrome is caused by Staphylococcus aureus releasing TSST- 1 toxin, which is a superantigen. Toxic shock syndrome causes
80 eryt hroderma (widespread erythema of the skin), followed by desquamat ion 1- 2 weeks later. Pa t ients are acutely ill, wit h high feve r and
hypotension .
81 Superantigen Toxic shock syndrome Staphylococcus aureus Desquamation Erythema Hypotension Staphylococcus Toxic shock syndrome toxin Toxin Toxicity Fever
82
83
84
Bottom Line:
85
Scarl et fever is caused by Streptococcus pyogenes (g roup A streptococci). It is mediated by a pyrogenic extoxin, and symptoms include high
fever and a diffuse, eryt hematous, rough -t extured rash.
86 Streptococcus pyogenes Scarlet fever Streptococcus Erythema Fever Rash Pyrogen (fever)
87
88
89 i@l;fil·1i•J for year:[2017 • J
FIRST AID FACTS
90
91
FA17 p 132.4
92
93 Streptococcus Gram EB cocci in chains r.:.l. Group A strep J•NES (major criteria for acute rheumatic
94 pyogenes (group A cause: fe\'er):
95 streptococci) • Pyogenic- pharyngitis, cellulitis, impetigo Joints- polyarthritis
96 ("honey-crusted" lesions), erysipelas • - carditis
97 • Toxigenic-scarlet fever, toxic shock-like Nodules (subcutaneous)
98 E rythema marginatum
syndrome, necrotizing fasciitis
99
• Immunologic- rheumatic fever, Sydenham chorea
100 - • - - - ____ J __ - - •- _! t..! -

8
Lock
s
Suspend
0
End Block
Item: 100 of ~ ,• Mark <::J [:::> ""I ~· ~'j
100 J.. Previous Next faiUI~S
LAb Notes Calculator

75
76
l@l;fil·1i•l for year:[2017
FIRST AID FACTS .
•j .
77
78
79 FA17 p 132.4
80 Streptococcus Gram EB cocci in chains r.J_Group A strep J•NES (major criteria for acute rheumatic
81 pyogenes (group A cause: fever):
82
streptococci) • Pyogenic-pharyngitis, cellulitis, impetigo Joints- polyarthritis
83
("honey-crusted" lesions), erysipelas • -carditis
84
• Toxigenic-scarlet fever, toxic shock-like Nodules (subcutaneous)
85
syndrome, necrotizing fasciitis E rythema marginatum
86
• Immunologic- rheumatic fever, Sydenham chorea
87
88
glomerulonephritis Pharyngitis can resu lt in rheumatic "phever"
89 Bacitracin sensitive, ~-hemolytic, pyrrolidonyl and glomeruloneph ritis.
90 arylamidase (PYR) EB. Hyaluronic acid capsule Impetigo usually precedes glomerulonephritis.
91 inhibits phagocytosis. Antibodies toM protein Scarlet fever- blanchi ng, sandpaper-like body
92 enhance host defenses aga inst S pyogenes but rash, strawberry tongue, and circumoral
93 can give rise to rheumatic fever. pallor in the setting of group A streptococcal
94 ASO titer or anti-ONase B antibodies indicate pharyngitis (erythrogenic toxin EB).
95 recent S pyogenes infection.
96
97
98 FA17 p 179.1
99 Red rashes of childhood
100 AGENT ASSOCIATED SYNDROME/DISEASE CLINICAL PRESENTATION

8
Lock
s
Suspend
0
End Block
Item: 100 of ~ ,• Mark <::J [:::> ""I ~· ~'j
100 J.. Previous Next LAbfaiUI~S Notes Calculator

74 • •
FA17 p 179.1
75
76
Red rashes of childhood
77 AGENT ASSOCIATED SYNDROME/DISEASE CLINICAL PRESENTATION
78 Coxsackievirus type A Hand-foot-mouth disease Oval-shaped vesicles on palms and soles rJ;
79 vesicles and ulcers in oral mucosa
80
Human herpesvirus 6 Roseola (exanthem subitum) Asymptomatic rose-colored macules appear
81
on body after se,·eral days of high fever; can
82
present with febrile seizures; usually affects
83
infa nts
84
85 Measles virus Measles (rubeola) ConAuent rash beginning at head and
86 moving down; preceded by cough, coryza,
87 conjuncti,·itis, and blue-white (Koplik) spots
88 on buccal mucosa
89
Parvovirus B19 Erythema infectiosum (fifth disease) "Slapped cheek" rash on face llJ (can cause
90
hydrops fetalis in pregnant women)
91
92 Rubella virus Rubella (German measles) Pin k macules and papules begin at head
93 and move down, remain discrete ..... fine
94 desquamating truncal rash; postauricular
95 lymphadenopathy
96 Streptococcus pyogenes Scarlet fever Erythematous, sandpaper-like rash with feve r
97
and sore throat
98
99
Varicella-Zoster virus Chickenpox Vesicular rash begins on trunk; spreads to face
100 •
m and extremities with lesions of different •

8
Lock
s
Suspend
0
End Block
Item: 100 of - ,• Mark -<] 1:> ""'I ~· 1!';:'1
100 ~ Prev1o u s Next Labf a lu es Note s Calculator
74
Measles virus Measles (rubeola) ConAuent rash begi nning at head and
75
moving down; preceded by cough, coryza,
76
conjunctivitis, and blue-white (Koplik) spots
77
78
on buccal mucosa
79 Parvovirus 819 Errthema infectiosum (fifth disease) "Slapped cheek" rash on face IE] (can cause
80 hydrops fetal is in pregnant women)
81
Rubella virus Rubella (German measles) Pink macules and papules begin at head
82
and mo,·e down, remain discrete - fine
83
desquamating truncal rash; postauricular
84
lymphadenopathy
85
86 Streptococcus pyogenes Scarlet fe,·er Erythematous, sandpaper-like rash with fever
87 and sore throat
88 Varicella-Zoster virus Vesicular rash begins on trunk; spreads to face
Chickenpox
89
90
m and extremities with lesions of different
stages
91
92
93
94
95
96
97
98
99
100 •
8
L.odt
s
Su~pl'nd
~
End Block

You might also like